You are on page 1of 846

Algebra

With Sessionwise Theory & Exercises


Algebra
With Sessionwise Theory & Exercises

Dr. SK Goyal
ARIHANT PRAKASHAN (Series), MEERUT
All Rights Reserved

© AUTHOR
No part of this publication may be re-produced, stored in a retrieval system or
by any means, electronic mechanical, photocopying, recording, scanning, web or
otherwise without the written permission of the publisher. Arihant has obtained
all the information in this book from the sources believed to be reliable and true.
However, Arihant or its editors or authors or illustrators don’t take any responsibility
for the absolute accuracy of any information published, and the damages or loss
suffered thereupon.
All disputes subject to Meerut (UP) jurisdiction only.

Administrative & Production Offices


Regd. Office
‘Ramchhaya’ 4577/15, Agarwal Road, Darya Ganj, New Delhi -110002
Tele: 011- 47630600, 43518550
Head Office
Kalindi, TP Nagar, Meerut (UP) - 250002 Tel: 0121-7156203, 7156204
Sales & Support Offices
Agra, Ahmedabad, Bengaluru, Bareilly, Chennai, Delhi, Guwahati,
Hyderabad, Jaipur, Jhansi, Kolkata, Lucknow, Nagpur & Pune.

ISBN : 978-93-25298-63-7

PO No : TXT-XX-XXXXXXX-X-XX
Published by Arihant Publications (India) Ltd.
For further information about the books published by Arihant, log on to
www.arihantbooks.com or e-mail at info@arihantbooks.com
Follow us on
PREFACE

‘‘THE ALGEBRAIC SUM OF ALL THE TRANSFORMATIONS OCCURRING IN A CYCLICAL


PROCESS CAN ONLY BE POSITIVE, OR, AS AN EXTREME CASE EQUAL TO NOTHING’’
MEANS IF YOU CONTINUOUSLY PUT YOUR EFFORTS ON AN ASPECT YOU HAVE VERY
GOOD CHANCE OF POSITIVE OUTCOME i.e. SUCCESS

It is a matter of great pride and honour for me to have received such an overwhelming response to the
previous editions of this book from the readers. In a way, this has inspired me to revise this book
thoroughly as per the changed pattern of JEE Main & Advanced. I have tried to make the contents more
relevant as per the needs of students, many topics have been re-written, a lot of new problems of new
types have been added in etcetc. All possible efforts are made to remove all the printing errors that had
crept in previous editions. The book is now in such a shape that the students would feel at ease while
going through the problems, which will in turn clear their concepts too.
A Summary of changes that have been made in Revised & Enlarged Edition
— Theory has been completely updated so as to accommodate all the changes made in JEE Syllabus & Pattern in
recent years.
— The most important point about this new edition is, now the whole text matter of each chapter has been
divided into small sessions with exercise in each session. In this way the reader will be able to go through the
whole chapter in a systematic way.
— Just after completion of theory, Solved Examples of all JEE types have been given, providing the students a
complete understanding of all the formats of JEE questions & the level of difficulty of questions generally
asked in JEE.
— Along with exercises given with each session, a complete cumulative exercises have been given at the end of
each chapter so as to give the students complete practice for JEE along with the assessment of knowledge
that they have gained with the study of the chapter.
— Last 13 Years questions asked in JEE Main & Adv, IIT-JEE & AIEEE have been covered in all the chapters.
However I have made the best efforts and put my all Algebra teaching experience in revising this book.
Still I am looking forward to get the valuable suggestions and criticism from my own fraternity i.e. the
fraternity of JEE teachers.
I would also like to motivate the students to send their suggestions or the changes that they want to be
incorporated in this book.
All the suggestions given by you all will be kept in prime focus at the time of next revision of the book.

Dr. SK Goyal
CONTENTS
1. COMPLEX NUMBERS 1-102
LEARNING PART Session 4
Session 1 — nth Root of Unity
— Integral Powers of Iota (i) — Vector Representation of Complex Numbers
— Switch System Theory — Geometrical Representation of Algebraic
Session 2 Operation on Complex Numbers
— Rotation Theorem (Coni Method)
— Definition of Complex Number
— Shifting the Origin in Case of Complex
— Conjugate Complex Numbers
Numbers
— Representation of a Complex Number in
— Inverse Points
Various Forms
— Dot and Cross Product
Session 3
— Use of Complex Numbers in Coordinate
— amp (z)– amp (–z)=± p, According as amp (z)
Geometry
is Positive or Negative
— Square Root of a Complex Number PRACTICE PART
— Solution of Complex Equations — JEE Type Examples
— De-Moivre’s Theorem — Chapter Exercises
— Cube Roots of Unity

2. THEORY OF EQUATIONS 103-206


LEARNING PART Session 4
Session 1 — Equations of Higher Degree
— Polynomial in One Variable — Rational Algebraic Inequalities
— Identity — Roots of Equation with the
— Linear Equation Help of Graphs
— Quadratic Equations Session 5
— Standard Quadratic Equation — Irrational Equations
Session 2 — Irrational Inequations
— Transformation of Quadratic Equations — Exponential Equations
— Condition for Common Roots — Exponential Inequations
— Logarithmic Equations
Session 3
— Logarithmic Inequations
— Quadratic Expression
— Wavy Curve Method PRACTICE PART
— Condition for Resolution into Linear Factors — JEE Type Examples
— Location of Roots (Interval in which Roots Lie) — Chapter Exercises
3. SEQUENCES AND SERIES 207-312
LEARNING PART Session 5
Session 1 — Mean
— Sequence Session 6
— Series — Arithmetico-Geometric
— Progression Series (AGS)
Session 2 — Sigma (S) Notation

— Arithmetic Progression — Natural Numbers


Session 7
Session 3
— Application to Problems of Maxima and
— Geometric Sequence or Geometric
Minima
Progression
Session 4 PRACTICE PART
— Harmonic Sequence or Harmonic Progression — JEE Type Examples
— Chapter Exercises

4. LOGARITHMS AND THEIR PROPERTIES 313-358


LEARNING PART Session 3
Session 1 — Properties of Monotonocity of Logarithm
— Definition — Graphs of Logarithmic Functions
— Characteristic and Mantissa
PRACTICE PART
Session 2 — JEE Type Examples
— Principle Properties of Logarithm — Chapter Exercises

5. PERMUTATIONS AND COMBINATIONS 359-436


LEARNING PART Session 5
Session 1 — Combinations from Identical Objects
— Fundamental Principle of Counting Session 6
— Factorial Notation — Arrangement in Groups
Session 2 — Multinomial Theorem
— Divisibility Test — Multiplying Synthetically
— Principle of Inclusion and Exclusion Session 7
— Permutation — Rank in a Dictionary
Session 3 — Gap Method
— Number of Permutations Under Certain [when particular objects are never together]
Conditions
— Circular Permutations
PRACTICE PART
— JEE Type Examples
— Restricted Circular Permutations
— Chapter Exercises
Session 4
— Combination
— Restricted Combinations
6. BINOMIAL THEOREM 437-518
LEARNING PART Session 4
Session 1 — Use of Complex Numbers in Binomial Theorem
— Binomial Theorem for Positive Integral Index — Multinomial Theorem
— Pascal’s Triangle — Use of Differentiation
— Use of Integration
Session 2
— When Each Term is Summation Contains the
— General Term
Product of Two Binomial Coefficients or
— Middle Terms
Square of Binomial Coefficients
— Greatest Term
— Binomial Inside Binomial
— Trinomial Expansion
— Sum of the Series
Session 3
— Two Important Theorems PRACTICE PART
— Divisibility Problems — JEE Type Examples
— Chapter Exercises

7. DETERMINANTS 519-604
LEARNING PART — System of Linear Equations
Session 1 — Cramer’s Rule
— Definition of Determinants — Nature of Solutions of System of Linear
— Expansion of Determinant Equations
— Sarrus Rule for Expansion — System of Homogeneous Linear Equations
— Window Rule for Expansion Session 4
Session 2 — Differentiation of Determinant
— Minors and Cofactors — Integration of a Determinant
— Use of Determinants in Coordinate Geometry — Walli’s Formula
— Properties of Determinants — Use of S in Determinant
Session 3 PRACTICE PART
— Examples on Largest Value of a — JEE Type Examples
Third Order Determinant — Chapter Exercises
— Multiplication of Two Determinants of
the Same Order

8. MATRICES 605-690
LEARNING PART Session 2
Session 1 — Transpose of a Matrix
— Definition — Symmetric Matrix
— Types of Matrices — Orthogonal Matrix
— Difference Between a Matrix and a — Complex Conjugate (or Conjugate) of a Matrix
Determinant — Hermitian Matrix
— Equal Matrices — Unitary Matrix
— Operations of Matrices — Determinant of a Matrix
— Various Kinds of Matrices — Singular and Non-Singular Matrices
Session 3 Session 4
— Adjoint of a Matrix — Solutions of Linear Simultaneous Equations
— Inverse of a Matrix Using Matrix Method
— Elementary Row Operations
PRACTICE PART
— Equivalent Matrices
— JEE Type Examples
— Matrix Polynomial
— Chapter Exercises
— Use of Mathematical Induction

9. PROBABILITY 691-760
LEARNING PART Session 4
Session 1 — Binomial Theorem on Probability
— Some Basic Definitions — Poisson Distribution
— Mathematical or Priori or Classical Definition — Expectation
of Probability
— Multinomial Theorem
— Odds in Favours and Odds Against the Event
— Uncountable Uniform Spaces
Session 2
— Some Important Symbols PRACTICE PART
— Conditional Probability — JEE Type Examples
Session 3 — Chapter Exercises
— Total Probability Theorem
— Baye’s Theorem or Inverse Probability

10. MATHEMATICAL INDUCTION 761-784


LEARNING PART PRACTICE PART
— Introduction — JEE Type Examples
— Statement — Chapter Exercises
— Mathematical Statement

11. SETS, RELATIONS AND FUNCTIONS 785-836


LEARNING PART Session 3
Session 1 — Definition of Function
— Definition of Sets — Domain, Codomain and Range
— Representation of a Set — Composition of Mapping
— Different Types of Sets — Equivalence Classes
— Laws and Theorems — Partition of Set
— Venn Diagrams (Euler-Venn Diagrams) — Congruences
Session 2
PRACTICE PART
— Ordered Pair
— JEE Type Examples
— Definition of Relation
— Chapter Exercises
— Ordered Relation
— Composition of Two Relations
SYLLABUS
numbers. Relation between AM and GM Sum upto n
JEE MAIN terms of special series: ∑ n, ∑ n2, ∑n3. Arithmetico -
Unit I Sets, Relations and Functions Geometric progression.
Sets and their representation, Union, intersection and Unit VIII Probability
complement of sets and their algebraic properties, Power Probability of an event, addition and multiplication
set, Relation, Types of relations, equivalence relations, theorems of probability, Baye’s theorem, probability
functions, one-one, into and onto functions, composition distribution of a random variate, Bernoulli and Binomial
of functions. distribution.
Unit II Complex Numbers
Complex numbers as ordered pairs of reals,
Representation of complex numbers in the form a+ib and JEE ADVANCED
their representation in a plane, Argand diagram, algebra
of complex numbers, modulus and argument (or Algebra
amplitude) of a complex number, square root of a Algebra of complex numbers, addition, multiplication,
complex number, triangle inequality. conjugation, polar representation, properties of modulus
and principal argument, triangle inequality, cube roots of
Unit III Matrices and Determinants unity, geometric interpretations.
Matrices, algebra of matrices, types of matrices,
determinants and matrices of order two and three. Quadratic equations with real coefficients, relations
Properties of determinants, evaluation of deter-minants, between roots and coefficients, formation of quadratic
area of triangles using determinants. Adjoint and equations with given roots, symmetric functions of roots.
evaluation of inverse of a square matrix using Arithmetic, geometric and harmonic progressions,
determinants and elementary transformations, Test of arithmetic, geometric and harmonic means, sums of finite
consistency and solution of simultaneous linear arithmetic and geometric progressions, infinite geometric
equations in two or three variables using determinants series, sums of squares and cubes of the first n natural
and matrices. numbers.
Unit IV Permutations and Combinations Logarithms and their Properties
Fundamental principle of counting, permutation as an Permutations and combinations, Binomial theorem for a
arrangement and combination as selection, Meaning of positive integral index, properties of binomial
P(n,r) and C (n,r), simple applications. coefficients.
Unit V Mathematical Induction Matrices as a rectangular array of real numbers, equality
Principle of Mathematical Induction and its simple of matrices, addition, multiplication by a scalar and
applications. product of matrices, transpose of a matrix, determinant of
a square matrix of order up to three, inverse of a square
Unit VI Binomial Theorem and its matrix of order up to three, properties of these matrix
Simple Applications operations, diagonal, symmetric and skew-symmetric
Binomial theorem for a positive integral index, general matrices and their properties, solutions of simultaneous
term and middle term, properties of Binomial coefficients linear equations in two or three variables.
and simple applications.
Addition and multiplication rules of probability,
Unit VII Sequences and Series conditional probability, independence of events,
Arithmetic and Geometric progressions, insertion of computation of probability of events using permutations
arithmetic, geometric means between two given and combinations.
CHAPTER

01
Complex Numbers
Learning Part
Session 1
● Integral Powers of Iota (i)

● Switch System Theory

Session 2
● Definition of Complex Number

● Conjugate Complex Numbers

● Representation of a Complex Number in Various Forms

Session 3
● amp ( z ) - amp ( - z ) = ± p , According as amp ( z ) is Positive or Negative

● Square Root of a Complex Number

● Solution of Complex Equations

● De-Moivre’s Theorem

● Cube Roots of Unity

Session 4
● nth Root of Unity

● Vector Representation of Complex Numbers

● Geometrical Representation of Algebraic Operation on Complex Numbers

● Rotation Theorem (Coni Method)

● Shifting the Origin in Case of Complex Numbers

● Inverse Points

● Dot and Cross Product

● Use of Complex Numbers in Coordinate Geometry

Practice Part
● JEE Type Examples
● Chapter Exercises

Arihant on Your Mobile !


Exercises with the #L
symbol can be practised on your mobile. See inside cover page to activate for free.
2 Textbook of Algebra

The square of any real number, whether positive, negative Remark


or zero, is always non-negative i.e. x 2 ³ 0 for all x Î R. - a = i a ,where a is positive quantity. Keeping this result in
mind, the following computation is correct
Therefore, there will be no real value of x , which when
- a - b = i a × i b = i 2 ab = - ab
squared, will give a negative number.
where, a and b are positive real numbers.
Thus, the equation x 2 + 1 = 0 is not satisfied for any real But the computation, - a - b = ( - a)( - b) = |a||b| is wrong.
value of x. ‘Euler’ was the first Mathematician to Because the property, a b = ab is valid only when atleast one
introduce the symbol i (read ‘Iota’) for the square root of of a and b is non-negative.
If a and b are both negative, then a b = - a b .
- 1 with the property i 2 = - 1. The theory of complex
number was later on developed by Gauss and Hamilton. y Example 1. Is the following computation correct?
According to Hamilton, ‘‘Imaginary number is that If not, give the correct computation.
number whose square is a negative number ’’. Hence, the
-2 -3 = ( -2) ( -3) = 6
equation x 2 + 1 = 0
Sol. No,
Þ x2 = -1 If a and b are both negative real numbers, then a b = - ab
or x = ± -1 Here, a = - 2 and b = - 3.
(in the sense of arithmetic, -1 has no meaning). \ -2 - 3 = - ( - 2) ( - 3) = - 6
Symbolically, -1 is denoted by i (the first letter of the
y Example 2. A student writes the formula
word ‘Imaginary ’).
ab = a b . Then, he substitutes a = - 1 and b = - 1
\ Solutions of x 2 + 1 = 0 are x = ± i. and finds 1 = - 1. Explain, where he is wrong.
Also, i is the unit of complex number, since i is present in Sol. Since, a and b are both negative, therefore ab ¹ a b .
every complex number. Generally, if a is positive quantity, Infact a and b are both negative, then we have a b = - ab .
then
y Example 3. Explain the fallacy
-a ´ -a = ( -1) ´ a ´ ( -1) ´ a
- 1 = i ´ i = -1 ´ -1 = ( -1) ´ ( -1) = 1 = 1.
= -1 ´ a ´ -1 ´ a
Sol. If a and b are both negative, then
=i a ´i a a b = - |a | |b |
= i2a = - a \ -1 ´ -1 = - | -1| | -1| = - 1

Session 1
Integral Powers of Iota (i ), Switch System Theory
Integral Powers of Iota ( i ) Þ n = 4q + r
(i) If the index of i is whole number, then When, 0 £ r £ 3
\ i n = i 4 q + r = (i 4 ) q (i ) r = (1) q × (i ) r = i r
i 0 = 1, i 1 = i , i 2 = ( -1 ) 2 = - 1,
In general, i 4n = 1, i 4n + 1 = i, i 4n + 2 = - 1,
i 3 = i × i 2 = - i, i 4 = (i 2 ) 2 = ( -1) 2 = 1
n
i 4n + 3 = - i for any whole number n.
To find the value of i (n > 4 ) First divide n by 4.
(ii) If the index of i is a negative integer, then
Let q be the quotient and r be the remainder.
1 i i 1
i.e. 4 ) n (q i -1 = = = = - i, i -2 = = - 1,
i i 2 -1 i 2
- 4q
1 i 1 1
r i -3 = = = i, i -4 = = = 1, etc.
3 4
i i i4 1
Chap 01 Complex Numbers 3

2
y Example 4. Evaluate. æ1 + i ö æ 1 + i 2 + 2i ö
Sol. Q a 2 = ç ÷ =ç ÷
(i) i 1998 è 2 ø è 2 ø

(ii) i - 9999 æ 1 - 1 + 2i ö
=ç ÷ =i
è 2 ø
(iii) ( - -1 ) 4n +3 , n Î N
\ a1929 = a × a1928 = a × (a 2 )964 = a (i )964
Sol. (i) 1998 leaves remainder 2, when it is divided by 4.
i.e. 4 ) 1998 (499 = a (i )4 ´ 241 = a × (i 4 )241 = a
1996
2
y Example 7. Dividing f (z ) by z - i , where i = -1, we
\ i 1998 = i 2 = - 1 obtain the remainder i and dividing it by z + i , we get
the remainder 1 + i. Find the remainder upon the
Aliter
division of f (z ) by z 2 + 1.
i 2000
1
i 1998 = = = -1 Sol. z - i = 0 Þ z = i
i2 -1
(ii) 9999 leaves remainder 3, when it is divided by 4. Remainder, when f (z ) is divided by (z - i ) = i
i.e. 4 ) 9999 (2499 i.e. f (i ) = i K (i)
9996 and remainder, when f (z ) is divided by (z + 1) = 1 + i
3 i.e. f ( - i ) = 1 + i [Qz + i = 0 Þ z = - i ] K (ii)
1 1 i i Since, z 2 + 1 is a quadratic expression, therefore remainder
\ i - 9999 = 9999 = 3 = 4 = = i
i i i 1 when f (z ) is divided by z 2 + 1, will be in general a linear
Aliter expression. Let g (z ) be the quotient and az + b (where a
1 i i and b are complex numbers) be the remainder, when f (z ) is
i - 9999 = 9999
=
=i 10000
= divided by z 2 + 1.
i i 1
(iii) 4n + 3 leaves remainder 3, when it is divided by 4. Then, f (z ) = (z 2 + 1) g ( z ) + az + b K (iii)
i.e., 4 ) 4n + 3 (n \ 2
f (i ) = (i + 1) g (i ) + ai + b = ai + b
4n or ai + b = i [from Eq. (i)] K (iv)
3 and f ( - i ) = (i 2 + 1) g ( - i ) - ai + b = - ai + b
\ i 4n + 3 = i 3 = - i or - ai + b = 1 + i [from Eq. (ii)] …(v)
Now, ( - -1 )4n + 3 = ( - i )4n + 3 = - (i )4n + 3 From Eqs. (iv) and (v), we get
= - (- i) 1 i
b = + i and a =
=i 2 2
Aliter ( - -1 )4n + 3 = ( - i )4n + 3 = - i 4n + 3 Hence, required remainder = az + b
1 1
= - (i 4 )n × i 3 = iz + + i
2 2
= - (1)n ( - i ) = i

y Example 5. Find the value of 1 + i 2 + i 4 + i 6 + ... + i 2n , The Sum of Four Consecutive


where i = -1 and n Î N . Powers of i (Iota) is Zero
Sol. Q 1 + i 2 + i 4 + i 6 + ... + i 2 n = 1 - 1 + 1 - 1 + ... + ( - 1)n If n Î I and i = - 1, then
Case I If n is odd, then i n + i n + 1 + i n + 2 + i n + 3 = i n (1 + i + i 2 + i 3 )
1 + i 2 + i 4 + i 6 + ... + i 2 n = 1 - 1 + 1 - 1 + ... + 1 - 1 = 0 = i n (1 + i - 1 - i ) = 0
Case II If n is even, then
1 + i 2 + i 4 + i 6 + ... + i 2 n = 1 - 1 + 1 - 1 + ... + 1 = 1 Remark
m m- p + 1

1+ i
1. å f( r ) = å f ( r + p - 1)
y Example 6. If a = , where i = - 1, then find the r =p r =1

2 m m+ p + 1

value of a 1929
.
2. å f( r ) = å f ( r - p - 1)
r =-p r =1
4 Textbook of Algebra

13
y Example 8. Find the value of å (i n + i n + 1 )
Switch System Theory
n =1
( where ,i = - 1 )
(Finding Digit in the Unit’s Place)
13 13 13
Sol. Q å (i n + i n + 1 ) = å i n + å i n + 1 = ( i + 0) + ( i 2 + 0) We can determine the digit in the unit’s place in
n =1 n =1 n =1 a b , where a, b Î N . If last digit of a are 0, 1, 5 and 6, then
é 13 n 13 ù digits in the unit’s place of a b are 0, 1, 5 and 6
êQ å i = 0 and å i
n +1
=0 ú
=i -1 n=2 respectively, for all b Î N .
ê n=2 ú
êë(three sets of four consecutive powers of i )úû
100 Powers of 2
y Example 9. Find the value of å in !
2 1 , 2 2 , 2 3 , 2 4 , 2 5 , 2 6 , 2 7 , 2 8 , 2 9 , ... the digits in unit’s place
n=0
( where , i = - 1). of different powers of 2 are as follows :
Sol. n! is divisible by 4, " n ³ 4. 2, 4, 8, 6, 2, 4, 8, 6, 2,... (period being 4)
100 97 ­ ­ ­ ­ ­ ­ ­ ­ ­
\ å in ! = å i (n + 3 )! 1 2 3 0 1 2 3 0 1 ... (switch number)
n=4 n =1

= i 0 + i 0 + i 0 + ... 97 times = 97 …(i)


(The remainder when b is divided by 4, can be 1 or 2 or 3 or 0).
100 3 100
\ åi n!
= åi n!
+ åi n! Then, press the switch number and then we get the digit
n=0 n=0 n=4 in unit’s place of a b (just above the switch number) i.e.
= i 0 ! + i 1! + i 2 ! + i 3 ! + 97 [from Eq. (i)] ‘press the number and get the answer’.
1 1 2 6
= i + i + i + i + 97 = i + i - 1 - 1 + 97
y Example 12. What is the digit in the unit’s place of
= 95 + 2i
4n + 7 ( 5172)11327 ?
y Example 10. Find the value of å ir Sol. Here, last digit of a is 2.
r =1 The remainder when 11327 is divided by 4, is 3. Then,
( where ,i = - 1 ). press switch number 3 and then we get 8.
4n + 7 4n + 7 4n + 4 Hence, the digit in the unit’s place of (5172)11327 is 8.
Sol. å ir = i1 + i 2 + i 3 + å ir = i - 1 - i + å ir +3

r =1 r =4 r =1
= - 1 + 0 [(n + 1) sets of four consecutive powers of i ] Powers of 3
= -1 3 1 , 3 2 , 3 3 , 3 4 , 3 5 , 3 6 , 3 7 , 3 8 , ... the digits in unit’s place of
y Example 11. Show that the polynomial different powers of 3 are as follows:
x 4 p + x 4q + 1 + x 4r + 2 + x 4 s + 3 is divisible by 3, 9, 7, 1, 3, 9, 7, 1, ... (period being 4)
x 3 + x 2 + x + 1, where p , q, r , s Î N . ­ ­ ­ ­ ­ ­ ­ ­
Sol. Let f ( x ) = x 4 p + x 4q + 1 + x 4r +2
+ x 4s + 3 1 2 3 0 1 2 3 0 ... (switch number)
and x 3 + x 2 + x + 1 = ( x 2 + 1) ( x + 1)
The remainder when b is divided by 4, can be 1 or 2 or 3
= ( x + i ) ( x - i ) ( x + 1), or 0. Now, press the switch number and get the unit’s
where i = -1 place digit ( just above).
Now, f (i ) = i 4 p + i 4q + 1 + i 4r +2
+ i 4s + 3 = 1 + i + i 2 + i 3 = 0
[sum of four consecutive powers of i is zero]
y Example 13. What is the digit in the unit’s place
f ( - i ) = ( - i )4 p + ( - i )4q + 1 + ( - i )4r + 2 + ( - i )4s + 3
of
= 1 + ( -i )1 + ( -i )2 + ( - i )3 = 1 - i - 1 + i = 0
(143) 86 ?
and f ( - 1) = ( - 1)4 p + ( - 1)4q + 1 + ( - 1)4r +2
+ ( - 1)4s + 3 Sol. Here, last digit of a is 3.
=1-1+1-1=0 The remainder when 86 is divided by 4, is 2.
Then, press switch number 2 and then we get 9.
Hence, by division theorem, f ( x ) is divisible by
x 3 + x 2 + x + 1. Hence, the digit in the unit’s place of (143)86 is 9.
Chap 01 Complex Numbers 5

Powers of 4 Powers of 8
4 1 , 4 2 , 4 3 , 4 4 , 4 5 ,... the digits in unit’s place of different 8 1 , 8 2 , 8 3 , 8 4 , 8 5 , 8 6 , 8 7 , 8 8 ,... the digits in unit’s place of
powers of 4 are as follows: different powers of 8 are as follows:
4, 6, 4, 6, 4, ... (period being 2) 8, 4, 2, 6, 8, 4, 2, 6, ... (period being 4)
­ ­ ­ ­ ­
­ ­ ­ ­ ­ ­ ­ ­
1 0 1 0 1 ... (switch number)
1 2 3 0 1 2 3 0 ... (switch number)
The remainder when b is divided by 2, can be 1 or 0. Now, The remainder when b is divided by 4, can be 1 or 2 or 3
press the switch number and get the unit’s place digit or 0.
( just above the switch number). Now, press the switch number and get the unit’s place
y Example 14. What is the digit in unit’s place of digit (just above the switch number).
(1354 ) 22222 ? y Example 16. What is the digit in the unit’s place of
Sol. Here, last digit of a is 4. (1008 ) 786 ?
The remainder when 22222 is divided by 2, is 0. Then, Sol. Here, last digit of a is 8.
press switch number 0 and then we get 6. The remainder when 786 is divided by 4, is 2. Then, press
Hence, the digit in the unit’s place of (1354 )22222 is 6. switch number 2 and then we get 4.
Hence, the digit in the unit’s place of (1008)786 is 4.

Powers of 7
Powers of 9
7 1 , 7 2 , 7 3 , 7 4 , 7 5 , 7 6 , 7 7 , 7 8 , ... the digits in unit’s place of
9 1 , 9 2 , 9 3 , 9 4 , 9 5 ,... the digits in unit’s place of different
different powers of 7 are as follows:
powers of 9 are as follows:
7, 9, 3, 1, 7, 9, 3, 1, ... (period being 4)
9, 1, 9, 1, 9, ... ( period being 2)
­ ­ ­ ­ ­ ­ ­ ­
­ ­ ­ ­ ­
1 2 3 0 1 2 3 0 ... (switch number)
1 0 1 0 1 ... (switch number)
(The remainder when b is divided by 4, can be 1 or 2 or 3 The remainder when b is divided by 2, can be 1 or 0.
or 0). Now, press the switch number and get the unit’s Now, press the switch number and get the unit’s place
place digit ( just above). digit (just above the switch number).
y Example 15. What is the digit in the unit’s place of y Example 17. What is the digit in the unit’s place of
(13057 ) 941120579 ? (2419 )111213 ?
Sol. Here, last digit of a is 7. Sol. Here, last digit of a is 9.
The remainder when 941120579 is divided by 4, is 3. Then, The remainder when 111213 is divided by 2, is 1. Then,
press switch number 3 and then we get 3. press switch number 1 and then we get 9.
Hence, the digit in the unit’s place of (13057 )941120579 is 3. Hence, the digit in the unit’s place of (2419 )111213 is 9.
6 Textbook of Algebra

#L Exercise for Session 1


1 If (1 + i )2 n + (1 - i )2 n = - 2n + 1 (where, i = - 1) for all those n, which are
(a) even (b) odd
(c) multiple of 3 (d) None of these

2 If i = - 1, the number of values of i n + i - n for different n Î I is


(a) 1 (b) 2
(c) 3 (d) 4

3 If a > 0 and b < 0, then a b is equal to (where, i = -1)


(a) - a × b (b) a × b i

(c) a × b (d) None of these

4 Consider the following statements.


S1 : - 6 = 2i ´ 3i = ( - 4) ´ ( - 9) ( where, i = - 1) S2 : ( - 4) ´ ( - 9) = ( - 4) ´ ( - 9)

S3 : ( - 4) ´ ( - 9) = 36 S4 : 36 = 6
Of these statements, the incorrect one is
(a) S1 only (b) S 2 only
(c) S 3 only (d) None of these
50

5 The value of S
n=0
i ( 2n + 1) ! (where, i = - 1) is

(a) i (b) 47 - i
(c) 48 + i (d) 0
1003

6 The value of S
r =-3
i r ( where i = - 1) is

(a) 1 (b) - 1
(c) i (d) - i

7 The digit in the unit’s place of (153)98 is


(a) 1 (b) 3
(c) 7 (d) 9

8 The digit in the unit’s place of (141414)12121 is


(a) 4 (b) 6
(c) 3 (d) 1
Session 2
Definition of Complex Number, Conjugate Complex
Numbers, Representation of a Complex Number in
Various Forms
Definition of Complex Number Algebraic Operations on
A number of the form a + ib, where a, b Î R and i = - 1, is Complex Numbers
called a complex number. It is denoted by z i.e. z = a + ib. Let two complex numbers be z 1 = a + ib and z 2 = c + id ,
A complex number may also be defined as an ordered pair where a, b, c , d Î R and i = - 1.
of real numbers; and may be denoted by the symbol (a, b). 1. Addition z 1 + z 2 = (a + ib ) + (c + id )
If we write z = (a, b ), then a is called the real part and b is
= (a + c ) + i (b + d )
the imaginary part of the complex number z and may be
denoted by Re (z ) and Im (z), respectively i.e., a = Re (z ) 2. Subtraction z 1 - z 2 = (a + ib ) - (c + id )
and b = Im (z ). = (a - c ) + i (b - d )
Two complex numbers are said to be equal, if and only if 3. Multiplication z 1 × z 2 = (a + ib ) × (c + id )
their real parts and imaginary parts are separately equal. = ac + iad + ibc + i 2 bd
Thus, a + ib = c + id
= ac + i (ad + bc ) - bd
Û a = c and b = d
= (ac - bd ) + i (ad + bc )
where, a, b, c , d Î R and i = - 1.
z (a + ib ) (c - id )
4. Division 1 = ×
i.e. z1 = z2 z 2 (c + id ) (c - id )
Û Re (z 1 ) = Re (z 2 ) and Im (z 1 ) = Im (z 2 )
[multiplying numerator and denominator by c - id
Important Properties of Complex Numbers where atleast one of c and d is non-zero]
1. The complex numbers do not possess the property of order, ac - iad + ibc - i 2 bd ac + i (bc - ad ) + bd
= =
i.e., ( a + ib) > or < ( c + id ) is not defined. For example,
(c ) 2 - (id ) 2 c 2 - i 2d 2
9 + 6 i > 3 + 2i makes no sense.
2. A real number a can be written as a + i × 0. Therefore, every (ac + bd ) + i (bc - ad ) (ac + bd ) (bc - ad )
= = +i
real number can be considered as a complex number, whose 2 2 2 2
c +d (c + d ) (c 2 + d 2 )
imaginary part is zero. Thus, the set of real numbers (R) is a
proper subset of the complex numbers ( C ) i.e. R Ì C. Hence,
the complex number system is N Ì W Ì I Ì Q Ì R Ì C Remark
3. A complex number z is said to be purely real, if Im ( z ) = 0; and 1+ i 1- i
= i and = - i, where i = -1.
is said to be purely imaginary, if Re ( z ) = 0. The complex 1- i 1+ i
number 0 = 0 + i × 0 is both purely real and purely imaginary.
4. In real number system, a2 + b2 = 0 Þ a = 0 = b. Properties of Algebraic Operations
But if z 1 and z 2 are complex numbers, then z 12 + z 22 = 0 on Complex Numbers
does not imply z1 = z2 = 0.
For example, z 1 = 1 + i and z 2 = 1 - i
Let z 1 , z 2 and z 3 be any three complex numbers.
Here, z 1 ¹ 0, z 2 ¹ 0
Then, their algebraic operations satisfy the following
But z 12 + z 22 = ( 1 + i ) 2 + ( 1 - i ) 2 = 1 + i 2 + 2i + 1 + i 2 - 2i
properties :
= 2 + 2i 2 = 2 - 2 = 0 Properties of Addition of Complex Numbers
However, if product of two complex numbers is zero, then
(i) Closure law z 1 + z 2 is a complex number.
atleast one of them must be zero, same as in case of real
numbers. (ii) Commutative law z 1 + z 2 = z 2 + z 1
If z 1z 2 = 0, then z 1 = 0, z 2 ¹ 0 or z 1 ¹ 0, z 2 = 0 (iii) Associative law (z 1 + z 2 ) + z 3 = z 1 + (z 2 + z 3 )
or z 1 = 0, z 2 = 0
8 Textbook of Algebra

(iv) Additive identity z + 0 = z = 0 + z , then 0 is called


the additive identity.
Properties of Conjugate
(v) Additive inverse - z is called the additive inverse of Complex Numbers
z, i.e. z + ( - z ) = 0. Let z, z 1 and z 2 be complex numbers. Then,
Properties of Multiplication (i) (z ) = z
of Complex Numbers (ii) z +z = 2 Re (z )
(i) Closure law z 1 × z 2 is a complex number. (iii) z -z = 2 Im (z )
(ii) Commutative law z 1 × z 2 = z 2 × z 1 (iv) z +z = 0 Þ z = - z Þ z is purely imaginary.
(iii) Associative law (z 1 × z 2 ) z 3 = z 1 (z 2 × z 3 ) (v) z -z = 0 Þ z = z Þ z is purely real.
(iv) Multiplicative identity z × 1 = z = 1 × z , then 1 is (vi) z 1 ± z 2 = z 1 ± z 2 Ingeneral,
called the multiplicative identity.
z 1 ± z 2 ± z 3 ± ... ± z n = z 1 ± z 2 ± z 3 ± ... ± z n
(v) Multiplicative inverse If z is a non-zero complex
1 (vii) z 1 × z 2 = z 1 × z 2
number, then is called the multiplicative inverse
z In general, z 1 × z 2 × z 3 ... z n = z 1 × z 2 × z 3 ... z n
1 1
of z i.e. z. = 1 = × z
z z æz ö z
(viii) ç 1 ÷ = 1 , z 2 ¹ 0
(vi) Multiplication is distributive with respect to èz2 ø z2
addition z 1 (z 2 + z 3 ) = z 1 z 2 + z 1 z 3
(ix) z n = (z ) n
(x) z 1 z 2 + z 1 z 2 = 2 Re (z 1 z 2 ) = 2 Re (z 1 z 2 )
Conjugate Complex Numbers (xi) If z = f (z 1 , z 2 ), then z =f (z 1 , z 2 )
The complex numbers z = (a, b ) = a + ib and
z = (a, - b ) = a - ib, where a and b are real numbers, x-3 y -3
y Example 18. If + = i , where x , y ÎR and
i = -1 and b ¹ 0, are said to be complex conjugate of each 3+ i 3-i
other (here, the complex conjugate is obtained by just i = - 1, find the values of x and y.
changing the sign of i). x -3 y -3
Sol. Q + =i
Note that, sum = (a + ib ) + (a - ib ) = 2a, which is real. 3+i 3 -i
And product = (a + ib ) (a - ib ) = a 2 - (ib ) 2 Þ ( x - 3) ( 3 - i ) + ( y - 3 ) ( 3 + i ) = i ( 3 + i ) ( 3 - i )
Þ ( 3x - xi - 9 + 3i ) + ( 3y + yi - 9 - 3i ) = 10i
= a 2 - i 2 b 2 = a 2 - ( -1) b 2
Þ (3x + 3y - 18) + i (y - x ) = 10i
= a 2 + b 2 , which is real. On comparing real and imaginary parts, we get
Geometrically, z is the mirror image of z along real axis on 3x + 3y - 18 = 0
argand plane. Þ x +y = 6 …(i)
and y - x = 10 ...(ii)
Remark On solving Eqs. (i) and (ii), we get
Let z = - a - ib, a > 0, b > 0 = ( - a, - b) (III quadrant ) x = - 2, y = 8
Imaginary axis
P(z) y Example 19. If (a + ib ) 5 = p + iq, where i = - 1,
b prove that (b + ia ) 5 = q + ip .
θ a
Real
O θ axis Sol. Q (a + ib ) 5 = p + iq
b
\ (a + ib ) 5 = p + iq Þ (a - ib ) 5 = ( p - iq )
Q(z)
Þ ( - i 2a - ib ) 5 = ( -i 2 p - iq ) [Qi 2 = - 1]
Then, z = - a + ib = ( - a, b) ( II quadrant). Now,
(i) If z lies in I quadrant, then z lies in IV quadrant and
Þ ( - i )5 (b + ia ) 5 = ( - i ) (q + ip )
vice-versa. Þ ( - i ) (b + ia ) 5 = ( - i ) (q + ip )
(ii) If z lies in II quadrant, then z lies in III quadrant and
vice-versa. \ (b + ia ) 5 = (q + ip )
Chap 01 Complex Numbers 9

y Example 20. Find the least positive integral value of y Example 23. Find real values of x and y for which
n
æ 1-i ö the complex numbers - 3 + i x 2 y and x 2 + y + 4i ,
n, for which ç ÷ , where i = -1, is purely
è 1+ i ø where i = - 1, are conjugate to each other.
imaginary with positive imaginary part. Sol. Given, - 3 + ix 2y = x 2
+ y + 4i
n n n 2 2
æ1 - i ö æ1 - i 1 - i ö æ 1 + i 2 - 2i ö æ 1 - 1 - 2i ö n
Þ - 3 - ix y = x + y + 4i
Sol. ç ÷ =ç ´ ÷ =ç ÷ =ç ÷
è1 +i ø è1+ i 1 -i ø è 2 ø è 2 ø On comparing real and imaginary parts, we get
= ( - i )n = Imaginary x 2 +y = -3 …(i)
2
Þ n = 1, 3, 5, ... for positive imaginary part n = 3. and -x y=4 …(ii)
4
y Example 21. If the multiplicative inverse of a From Eq. (ii), we get x 2 = -
y
complex number is ( 3 + 4i ) / 19, where i = - 1, find 4 é 4 ù
complex number. Then, - + y = - 3 ê putting x 2 = - in Eq. (i)ú
y ë y û
Sol. Let z be the complex number.
æ 3 + 4i ö y 2 + 3y - 4 = 0 Þ (y + 4 ) (y - 1) = 0
Then, z × ç ÷ =1 \ y = - 4, 1
è 19 ø
For y = - 4 , x2 = 1 Þ x = ± 1
19 ( 3 - 4i )
or z= ´ For y = 1, x 2 = - 4 [impossible]
( 3 + 4i ) ( 3 - 4i )
\ x = ± 1, y = - 4
19 ( 3 - 4i )
= = ( 3 - 4i )
19 y Example 24. If x = - 5 + 2 - 4, find the value of
3 + 2 i sin q x 4 + 9 x 3 + 35x 2 - x + 4.
y Example 22. Find real q, such that ,
1 - 2 i sin q Sol. Since, x = - 5 + 2 - 4 Þ x + 5 = 4i
where i = -1, is Þ ( x + 5)2 = ( 4i )2 Þ x 2 + 10x + 25 = - 16
(i) purely real. (ii) purely imaginary. \ x 2 + 10x + 41 = 0 …(i)
3 + 2i sin q Now,
Sol. Let z =
1 - 2i sin q x 2 + 10x + 41 x 4
+ 9 x 3 + 35x 2
-x+4 x 2
-x+4
4 3 2
On multiplying numerator and denominator by conjugate x + 10x + 41x
of denominator, - - -
- x 3 - 6x 2 - x + 4
(3 + 2i sin q ) (1 + 2i sin q ) (3 - 4 sin 2 q ) + 8i sin q
z= = 3 2
(1 - 2i sin q ) (1 + 2i sin q ) (1 + 4 sin 2 q ) -x - 10x - 41x
+ + +
(3 - 4 sin 2 q ) (8 sin q ) 4x 2
+ 40x + 4
= 2
+i 2
(1 + 4 sin q ) (1 + 4 sin q ) 4x 2
+ 40x + 164
(i) For purely real, Im(z ) = 0 - - -
- 160
8 sin q
Þ = 0 or sin q = 0 \ x 4 + 9x 3
+ 35x 2
-x+4
1 + 4 sin 2 q
= ( x 2 + 10x + 41) ( x 2 - x + 4 ) - 160
\ q = n p, n Î I
= 0 - 160 = - 160 [from Eq. (i)]
(ii) For purely imaginary, Re (z ) = 0
(3 - 4 sin 2 q ) y Example 25. Let z be a complex number satisfying
Þ = 0 or 3 - 4 sin 2 q = 0
(1 + 4 sin 2 q) the equation z 2 - ( 3 + i ) z + l + 2 i = 0, where l ÎR and
3 æ 3ö pö
2 2 i = -1. Suppose the equation has a real root, find the
2 æ
or sin q = = ç ÷ = çsin ÷ non-real root.
4 è 2 ø è 3ø
Sol. Let a be the real root. Then,
p
\ q = np ± , n Î I a 2 - (3 + i ) a + l + 2i = 0
3
10 Textbook of Algebra

Þ (a 2 - 3 a + l ) + i (2 - a ) = 0 Argument of z will be q, p - q , p + q and 2p - q


On comparing real and imaginary parts, we get according as the point z lies in I, II, III and IV
y
a 2 - 3a + l = 0 …(i) quadrants respectively, where q = tan - 1 .
Þ 2-a =0 …(ii) x
From Eq. (ii), a = 2
y Example 26. Find the arguments of z 1 = 5 + 5i ,
Let other root be b.
Then, a + b = 3 + i Þ 2+ b = 3 + i z 2 = - 4 + 4 i , z 3 = - 3 - 3 i and z 4 = 2 - 2i ,
\ b =1+i where i = - 1.
Hence, the non-real root is 1 + i. Sol. Since, z 1, z 2 , z 3 and z 4 lies in I, II, III and IV quadrants
respectively. The arguments are given by
5
arg (z1 ) = tan - 1 = tan - 1 1 = p / 4
Representation of a Complex 5

Number in Various Forms arg (z 2 ) = p - tan - 1


4
-4
p 3p
= p - tan -1 1 = p - =
4 4
-3 p 5p
Cartesian Form arg (z 3 ) = p + tan - 1
-3
= p + tan -1 1 = p + =
4 4
(Geometrical Representation) -2
and arg (z 4 ) = 2p - tan - 1
Every complex number z = x + iy , where x , y Î R and 2
i = - 1, can be represented by a point in the cartesian p 7p
= 2p - tan -1 1 = 2p - =
plane known as complex plane (Argand plane) by the 4 4
ordered pair ( x , y ).
Principal Value of the Argument
The value q of the argument which satisfies the inequality
Modulus and Argument of a -p < q £ p is called the principal value of the argument.
Complex Number If z = x + iy = ( x , y ), " x , y Î R and i = - 1, then
Let z = x + iy = ( x , y ) for all x , y Î R and i = - 1. æy ö
arg(z ) = tan - 1 ç ÷ always gives the principal value. It
èxø
Imaginary axis
P (x, y) depends on the quadrant in which the point ( x , y ) lies.

Y Imaginary
r
y axis

θ P (x, y)
Real axis
O x
y
The length OP is called modulus of the complex number z
denoted by z , θ
X′ X
i.e. OP = r = z = ( x 2 + y 2 ) O x
Real axis
Y′
and if ( x , y ) ¹ (0, 0 ), then q is called the argument or
amplitude of z, (i) ( x , y ) Î first quadrant x > 0, y > 0.
æy ö æy ö
i.e. q = tan - 1 ç ÷ [angle made by OP with positive X-axis] The principal value of arg (z ) = q = tan - 1 ç ÷
èxø èxø
or arg (z ) = tan - 1 (y / x ) It is an acute angle and positive.
(ii) ( x , y ) Î second quadrant x < 0, y > 0.
Also, argument of a complex number is not unique, since
The principal value of arg (z ) = q
if q is a value of the argument, so also is 2np + q, where
n Î I . But usually, we take only that value for which æ y ö
= p - tan - 1 çç ÷
÷
0 £ q < 2 p. Any two arguments of a complex number differ è x ø
by 2np.
Chap 01 Complex Numbers 11

Y
Imaginary or tan - 1 1, p - tan - 1 1, - p + tan - 1 1, - tan - 1 1
(x, y) axis p p p p p 3p 3p p
or ,p- ,-p+ , - or , , - ,-
4 4 4 4 4 4 4 4
y Hence, the principal values of the arguments of z1, z 2 , z 3
θ
p 3p 3p p
X′ x X and z 4 are , , - , - , respectively.
O 4 4 4 4
Real axis

Y′ Remark
1. Unless otherwise stated, amp z implies principal value of the
It is an obtuse angle and positive. argument.
(iii) ( x , y ) Î third quadrant x < 0, y < 0. 2. Argument of the complex number 0 is not defined.
æy ö 3. If z1 = z2 Û z1 = z2 and arg ( z1 ) = arg ( z2 ).
The principal value of arg (z ) = q = - p + tan -1 ç ÷ 4. If arg ( z ) = p /2 or - p /2, z is purely imaginary.
èxø
Y 5. If arg ( z ) = 0 or p, z is purely real.
Imaginary
axis y Example 28. Find the argument and the principal
value of the argument of the complex number
X′
x O
X 2+ i
θ Real axis z= , where i = - 1.
y 4i + (1 + i ) 2
2+i 2+i 2+i 1 1
Sol. Since, z = = = = - i
(x, y) 4i + ( 1 + i ) 2 4i + 1 + i 2 + 2i 6i 6 3
Y′
\ z lies in IV quadrant.
It is an obtuse angle and negative.
1
(iv) ( x , y ) Î fourth quadrant x > 0, y < 0. -
-1 3 = tan - 1 2
Here, q = tan
The principal value of arg (z ) = q 1
æ y ö 6
= - tan - 1 çç ÷
÷ \ arg (z ) = 2p - q = 2p - tan -1 2
è x ø
Hence, principal value of arg (z ) = - q = - tan -1 2.
Y
Imaginary
axis Properties of Modulus
x (i) z ³ 0 Þ z = 0, iff z = 0 and z > 0, iff z ¹ 0
X′ X
O θ Real axis (ii) - z £ Re (z ) £ z and - z £ Im (z ) £ z
y
(iii) z = z = - z = - z
2
(x, y) (iv) zz = z
Y′
(v) z 1 z 2 = z 1 z 2
It is an acute angle and negative. In general, z 1 z 2 z 3 ... z n = z 1 z 2 z 3 ... z n
y Example 27. Find the principal values of the z z1
(vi) 1 = (z 2 ¹ 0 )
arguments of z 1 = 2 + 2i , z 2 = - 3 + 3i , z 3 = - 4 - 4i z2 z2
and z 4 = 5 - 5i , where i = -1.
(vii) z 1 ± z 2 £ z 1 + z 2
Sol. Since, z 1, z 2 , z 3 and z 4 lies in I, II, III and IV quadrants In general, z 1 ± z 2 ± z 3 ± ... ± z n £ z 1 + z 2
respectively. The principal values of the arguments are
+ z 3 + ... + z n
given by
æ 3 ö æ- 4ö (viii) z 1 ± z 2 ³ z 1 - z 2
æ2ö
tan -1 ç ÷ , p - tan -1 ç ÷ , - p + tan -1 ç ÷,
è2ø è -3 ø è- 4ø n
(ix) z n = z
æ -5 ö
- tan -1 ç ÷ (x) z1 - z2 £ z1 + z2 £ z1 + z2
è 5 ø
12 Textbook of Algebra

Thus, z 1 | + | z 2 is the greatest possible value of é 1ù


\ sin q i Î ê0, ú ,
z 1 + z 2 and | z 1 | - | z 2 | is the least possible value of ë 2û
z1 + z2 . 1
i.e. 0 £ sinq i £
2 2 2 2
(xi) z 1 ± z 2 = (z 1 ± z 2 ) (z 1 ± z 2 ) = z 1 + z2
\ Inequality Eq. (i) becomes,
± (z 1 z 2 + z 1 z 2 ) 1 4 1 3 1 2 1 1
2 2 2£ z + z + z + z +
or z 1 + z2 ± 2 Re (z 1 z 2 ) 2 2 2 2 2
4 3 2
Þ 3£ z + z + z + z
(xii) z 1 z 2 + z 1 z 2 = 2 z 1 z 2 cos(q 1 - q 2 ), where
q 1 = arg (z 1 ) and q 2 = arg (z 2 ) Þ 3£ z + z + z
2 3
+ z
4
< z + z
2

2 2 z 3 4
(xiii) | z 1 + z 2 |2 = z 1 + z 2 Û 1 is purely imaginary. + z + z +... + ¥
z2 2 3 4
2 2 2 2 Þ 3< z + z + z + z + ... + ¥
(xiv) z 1 + z 2 + z1 - z2 = 2 {z1 + z2 }
z
2 2 2 2 Þ 3< [here, | z | < 1]
(xv) az 1 - bz 2 + bz 1 + az 2 2
= (a + b ) ( z 1 2
+ z 2 ), 1- z
where a, b Î R Þ 3- 3 z < z Þ 3 < 4 z
3
(xvi) Unimodular i.e., unit modulus \ z >
4
If z is unimodular, then z = 1. In case of unimodular, 3
Hence, < z <1 [Q | z | < 1]
let z = cos q + i sin q, q Î R and i = -1. 4

Remark y Example 30. If z - 2 + i £ 2, find the greatest


1. If f ( z ) is unimodular, then f ( z ) = 1and let
f ( z ) = cos q + i sin q, q ÎR and i = -1. and least values of | z | , where i = -1.
z Sol. Given that, z - 2 + i £ 2 …(i)
2. is always a unimodular complex number, if z ¹ 0.
z
Q z-2+i ³ z - 2-i [by property (x)]
(xvii) The multiplicative inverse of a non-zero complex
number z is same as its reciprocal and is given by \ z-2+i ³ z - 5 …(ii)
1 z z From Eqs. (i) and (ii), we get
= = .
z zz 2
z z - 5 £ z-2+i £2

\ z - 5 £2
y Example 29. If q i Î [0, p / 6], i = 1 , 2, 3, 4, 5 and
sin q 1 z 4 + sin q 2 z 3 + sin q 3 z 2 + sin q 4 z Þ -2£ z - 5 £2
3 Þ 5 -2£ z £ 5 +2
+ sin q 5 = 2, show that < z < 1.
4 Hence, greatest value of z is 5 + 2 and least value of z
Sol. Given that,
is 5 - 2.
sin q 1 z 4 + sin q 2 z 3 + sin q 3 z 2 + sin q 4 z + sin q 5 = 2
or 2 = sin q 1 z 4 + sin q 2 z 3 + sin q 3 z 2 + sin q 4 z + sin q 5
y Example 31. If z is any complex number such
4 3 2
2 £ sin q 1 z + sin q 2 z + sin q 3 z that z + 4 £ 3, find the greatest value of z + 1 .
+ sin q 4 z + sin q 5 [by property (vii)]
Sol. Q z + 1 = (z + 4 ) - 3
Þ 2 £ sin q 1 z 4 + sin q 2 z 3 + sin q 3 z 2
= ( z + 4 ) + ( - 3) £ z + 4 + - 3
+ sin q 4 z + sin q 5 [by property (v)]
4 3 2 = z + 4 +3
Þ 2 £ sin q 1 z + sin q 2 z + sin q 3 z
£3+3=6 [Q z + 4 £ 3 ]
+ sin q 4 z + sin q 5 [by property (ix)] …(i)
But given, q i Î[0, p / 6] \ z +1 £6
Hence, the greatest value of z + 1 is 6.
Chap 01 Complex Numbers 13

2
y Example 32. If z 1 = 1, z 2 = 2, z 3 = 3 and Þ z1 - 2z 2
2
= 2 - z1z 2
9z 1z 2 + 4z 3z 1 + z 2 z 3 = 6, find the value of Þ (z 1 - 2z 2 ) ( z1 - 2 z 2 ) = ( 2 - z1z 2 ) ( 2 - z1z 2 )
z1 +z 2 +z 3 .
[by property (iv)]
2
Sol. Q z1 = 1 Þ z1 =1 Þ ( z1 - 2z 2 ) ( z1 - 2z 2 ) = ( 2 - z1z 2 ) ( 2 - z1z 2 )
1 Þ z1z1 - 2z1z 2 - 2z 2 z1 + 4z 2 z 2
Þ z1 z1 = 1 Þ = z1
z1 = 4 - 2z1z 2 - 2z1z 2 + z1z1z 2 z 2
2 2 2 2 2
z2 = 2 Þ z2 = 4 Þ z2 z2 = 4 Þ z1 + 4 z2 = 4 + z1 z2
2 2 2 2
Þ
4
= z 2 and z 3 = 3 Þ | z 3 | 2 = 9 Þ z1 - z1 × z2 + 4 z2 - 4 =0
z2
çæ z1 - 4 ÷öø æçè1 - z 2 ö÷ø = 0
2 2
Þ è
9
Þ z3z3 =9 Þ =z3
z3 But z2 ¹ 1 [given]
2
and given 9z 1z 2 + 4z 3z 1 + z 2 z 3 = 6 \ z1 =4
Hence, z1 = 2
9 4 1
Þ z1 z 2 z 3 + + =6
z3 z 2 z1
Þ z1 z2 z3 z 3 + z 2 + z1 = 6 Properties of Arguments
é 1 4 9 ù (i) arg (z 1 z 2 ) = arg (z 1 ) + arg (z 2 ) + 2kp, k Î I
êQ z = z 1, z = z 2 and z = z 3 ú In general, arg (z 1 z 2 z 3 ... z n )
ë 1 2 3 û
Þ 1 × 2 × 3 z1 + z 2 + z 3 = 6 = arg (z 1 ) + arg (z 2 ) + arg (z 3 ) +... + arg (z n ) + 2kp,
k Î I.
\ z1 + z 2 + z 3 = 1 [Q | z | = | z | ]
æz ö
(ii) arg ç 1 ÷ = arg (z 1 ) - arg (z 2 ) + 2kp, k Î I
èz2 ø
y Example 33. Prove that
1 1 æz ö
z 1 + z 2 = (z 1 + z 2 ) + z 1 z 2 + (z 1 + z 2 ) - z 1 z 2 . (iii) arg ç ÷ = 2 arg (z ) + 2kp, k Î I
2 2 èz ø

Sol. RHS =
1 1
(z1 + z 2 ) + z1z 2 + (z1 + z 2 ) - z1z 2 (iv) arg (z n ) = n. arg (z ) + 2kp, k Î I , where proper value
2 2 of k must be chosen, so that RHS lies in ( -p, p ].
z1 + z 2 + 2 z1z 2 z + z 2 - 2 z1z 2 æz ö æz ö
= + 1 (v) If arg ç 2 ÷ = q, then arg ç 1 ÷ = 2np - q, where n Î I .
2 2 èz1 ø èz2 ø
1 2 2
(vi) arg (z ) = - arg (z)
= { z 1 + z2 + z 1 - z2 }
2
1 2 2 17 p 7p
= . 2 { z1 + z 2 } [ by property (xiv)] y Example 35. If arg (z 1 ) = and arg (z 2 ) = , find
2 18 18
= z1 + z 2 = LHS the principal argument of z 1z 2 and (z 1 / z 2 ).
Sol. arg (z1z 2 ) = arg (z1 ) + arg (z 2 ) + 2kp
y Example 34. z 1 and z 2 are two complex numbers, 17 p 7 p
= + + 2kp
z 1 - 2z 2 18 18
such that is unimodular, while z 2 is not 4p
2 - z 1 ×z 2 = + 2kp
3
unimodular. Find | z 1 |. 4p 2p
z1 - 2z 2 = - 2p = - [for k = - 1]
Sol. Here, =1 3 3
2 - z1z 2 æ z1 ö
and arg ç ÷ = arg (z1 ) - arg (z 2 ) + 2kp
z1 - 2z 2 èz2 ø
Þ =1 [by property (vi)] 17 p 7 p 10p
2 - z1z 2 = - + 2kp = + 2kp
18 18 18
Þ z1 - 2z 2 = 2 - z1z 2 5p 5p
= +0= [for k = 0]
9 9
14 Textbook of Algebra

y Example 36. If z 1 and z 2 are conjugate to each (b) Trigonometric or Polar or


other, find the principal argument of ( - z 1z 2 ).
Modulus Argument Form of a
Sol. Qz1 and z 2 are conjugate to each other i.e., z 2 = z1, there-
fore, z1z 2 = z1z1 = z1
2 Complex Number
2
Let z = x + iy , where x , y Î R and i = -1, z is represented
\arg ( - z1 z 2 ) = arg ( - z1 ) = arg [negative real number] by P ( x , y ) in the argand plane.
=p
Y
y Example 37. Let z be any non-zero complex

Imaginary axis
number, then find the value of arg (z ) + arg (z ). P (x, y)

Sol. arg (z ) + arg (z ) = arg (zz )


2 y
= arg ( z ) = arg [positive real number]
θ
=0 O x X
M Real axis

(a) Mixed Properties of Modulus By geometrical representation,


OP = ( x 2 + y 2 ) = z
and Arguments
(i) z 1 + z 2 = z 1 + z 2 Û arg (z 1 ) = arg (z 2 ) ÐPOM = q = arg (z )
(ii) z 1 + z 2 = z 1 - z 2 Û arg (z 1 ) - arg (z 2 ) = p In DOPM, x = OP cos ( Ð POM ) = z cos (arg z )
Proof (i) Let arg (z 1 ) = q and arg (z 2 ) = f and y = OP sin ( Ð POM ) = z sin (arg z )
Q z = x + iy
\ z1 + z2 = z1 + z2
\ z = z (cos (arg z ) + i sin (arg z ) )
On squaring both sides, we get
2 2 2
or z = r (cos q + i sin q )
z1 + z2 = z1 + z2 + 2 z1 z2 z = r (cos q - i sin q )
Þ z1
2
+ z2
2
+ 2 z 1 z 2 cos (q - f) where, r = z and q = principal value of arg (z ).
2 2
= z1 + z2 + 2 z1 z2 Remark
1. cos q + i sin q is also written as CiS q.
Þ cos (q - f) = 1 2. Remember
\ q - f = 0 or q = f 1 = cos 0 + i sin 0 Þ - 1 = cos p + i sin p
\ arg (z 1 ) = arg (z 2 ) p p p p
i = cos + i sin Þ - i = cos - i sin
2 2 2 2
(ii) Q | z 1 + z 2 | = | z 1 | - | z 2 |
On squaring both sides, we get 1 i 3
2 2 2
y Example 38. Write the polar form of - -
z1 + z2 = z1 + z2 - 2 z1 z2 2 2
2 2
(where, i = - 1).
Þ z1 + z2 + 2 z1 z 2 cos (q - f)
1 i 3 æ 1 3ö
2 2 Sol. Let z = - - . Since, ç - , - ÷ lies in III quadrant.
= z1 + z2 - 2 z1 z2 2 2 è 2 2 ø
- 3 /2
Þ cos (q - f) = - 1 \ Principal value of arg (z ) = - p + tan - 1
- 1/2
\ q - f = p or arg (z 1 ) - arg (z 2 ) = p -1 p 2p
= - p + tan 3 = -p + =-
3 3
Remark 2 2
1. z1 - z2 = z1 + z2 Û arg ( z1 ) = arg ( z2 ) æ 1ö æ 3ö æ1 3ö
and z = ç - ÷ + ç - ÷ = ç + ÷ = 1 =1
2. z1 - z2 = z1 - z2 Û arg ( z1 ) - arg ( z2 ) = p è 2ø è 2 ø è4 4ø
p -
3. z1 - z2 = z1 + z2 Û arg ( z1 ) - arg ( z2 ) = ± , z1 z2 \ Polar form of z = z [cos (arg z ) + i sin(arg z )]
2
z
and 1 are purely imaginary. 1 i 3 é æ 2p ö æ 2p ö ù
z2 i.e. - - = ê cos ç - ÷ + i sin ç - ÷
2 2 ë è 3 ø è 3 ø úû
Chap 01 Complex Numbers 15

z =1
(c) Euler’s Form Sol. Given,
\ z =eiq …(i)
If q Î R and i = - 1, then e i q = cos q + i sin q is known as
Þ arg (z ) = q …(ii)
Euler’s identity. æ arg (z ) ö
1 + i tan ç ÷
Now, e - i q = cos q - i sin q è 2 ø 1 + i tan (q / 2)
RHS = = [from Eq. (ii)]
Let z = ei q æ arg (z ) ö 1 - i tan (q / 2)
1 - i tan ç ÷
è 2 ø
\ z = 1 and arg (z ) = q
iq -i q cos q / 2 + i sin q / 2 e i q/ 2
Also, e +e = 2 cos q and e i q - e - i q 2i sin q = = - i q/ 2
cos q / 2 - i sin q / 2 e
and if q, f Î R and i = - 1, then = e i q = z = LHS [from Eq. (i)]
æq+ fö
iç ÷
è 2 øæ q - fö æ æ a - ib ö ö 2ab
(i) e iq + e if = e × 2 cos ç ÷ y Example 41. Prove that tan ç i ln ç ÷÷ = 2
è 2 ø
è è a + ib ø ø a - b 2
æ q - fö
\ e iq + e if = 2 cos ç ÷ ( where a, b ÎR + and i = - 1 ).
è 2 ø
a - ib a - ib
æ q + fö Sol. Q = =1 [Q | z | = | z | ]
and arg (e i q + e if ) = ç ÷ a + ib a + ib
è 2 ø
æq+ fö a - ib
iç ÷
iq if æ q - fö
è 2 ø Let = e iq …(i)
(ii) e -e =e
× 2i sin ç ÷ a + ib
è 2 ø
By componendo and dividendo , we get
æ q - fö
\ e iq - e if = 2 sin ç ÷ ( a - ib ) - (a + ib ) e i q - 1 b
è 2 ø = - i = i tan (q / 2)
(a - ib ) + (a + ib ) e i q + 1 a
q+f p
and arg (e iq - e if ) = + [Qi = e ip / 2 ] æq ö b
2 2 or tan ç ÷ = - …(ii)
è2ø a
Remark
1. e iq + 1 = e iq / 2× 2cos (q/ 2) (Remember) æ æ a - ib ö ö
\ LHS = tan çi ln ç ÷÷
2. e iq
-1= e iq / 2
× 2i sin (q/ 2) (Remember) è è a + ib ø ø
= tan (i ln (e i q ))
iq
e -1 [from Eq. (i)]
3. iq
= i tan ( q / 2) (Remember)
e +1 = tan (i × i q ) = - tan q
4. If z = r e iq ; z = r , then arg ( z ) = q, z = r e- iq 2 tan q / 2
=-
5. If z - z0 = 1, then z - z0 = e iq 1 - tan 2 q / 2
2 ( - b / a)
y Example 39. Given that z - 1 = 1, where z is a point =- [from Eq. (ii)]
1 - ( - b / a )2
z -2
on the argand plane, show that = i tan (arg z ) , =
2ab
= RHS
z a - b2
2
where i = -1.
Sol. Given, z - 1 = 1 Applications of Euler’s Form
\ z - 1 = e i q Þ z = e i q + 1 = e i q / 2 × 2 cos (q / 2)
If x , y , q Î R and i = -1, then
\ arg (z ) = q / 2 …(i)
let z = x + iy [cartesian form]
z - 2 1 + e iq - 2 e iq - 1
LHS = = = iq = i tan (q / 2) = z (cos q + i sin q ) [polar form]
z 1 + e iq e +1
= i tan (arg z ) = RHS [from Eq. (i)] = z e iq [Euler’s form]

y Example 40. Let z be a non-real complex number (i) Product of Two Complex Numbers
æ arg (z ) ö Let two complex numbers be
1 + i tan ç ÷
è 2 ø z 1 = | z 1 | e iq1 and z 2 = | z 2 | e iq2 ,
lying on z = 1, prove that z =
æ arg (z ) ö
1 - i tan ç ÷ where q 1 , q 2 Î R and i = - 1
( where, i = - 1 ). è 2 ø
16 Textbook of Algebra

\ z 1 × z 2 = z 1 e iq1 × z 2 e iq2 = z 1 z 2 e i ( q1 + q2 ) = - w (cos (arg w ) - i sin (arg w ))


= - w (cos ( - arg w ) + i sin ( - arg w ))
= z 1 z 2 (cos (q 1 + q 2 ) + i sin (q 1 + q 2 )) = - w (cos (arg w ) + i sin (arg w )) = - w
Thus, z1 z2 = z1 z2
and arg (z 1 z 2 ) = q 1 + q 2 = arg (z 1 ) + arg (z 2 ) y Example 44. Express ( 1 + i ) - i , (where, i = -1) in the
form A + iB.
(ii) Division of Two Complex Numbers
Sol. Let A + iB = ( 1 + i )- i
Let two complex numbers be
On taking logarithm both sides, we get
z 1 = z 1 e iq1 and z 2 = z 2 e iq2 ,
loge ( A + iB ) = - i loge (1 + i )
where q 1 , q 2 Î R and i = - 1 æ æ 1 i öö
iq1 = - i loge ç 2 ç + ÷÷
z1 z1 e z 1 i(q - q ) è è 2 2 øø
\ = = e 1 2
z2 i q z2 æ æ p p öö
z2 e 2
= - i loge ç 2 ç cos + i sin ÷ ÷
è è 4 4 øø
z1
= (cos (q 1 - q 2 ) + i sin (q 1 - q 2 )) = - i loge ( 2 e i p / 4 ) = - i (loge 2 + loge e i p / 4)
z2 ipö p
æ1 i
= - i ç loge 2 + ÷ = - loge 2 +
z1 z1 è2 4 ø 2 4
Thus, = , (z 2 ¹ 0 ) i p
z2 z2 - log e 2 +
p/ 4 2 - 1/ 2
\ A + iB = e 2 4 =e × e i log e
æz ö =e p/ 4
× (cos ( loge 2- 1/ 2 ) + i sin ( loge 2- 1/ 2 ))
and arg ç 1 ÷ = q 1 - q 2 = arg (z 1 ) - arg (z 2 )
èz2 ø æ æ 1 öö æ æ 1 öö
p/ 4 p/ 4
=e × cos ç loge ç ÷ ÷ + i e sin ç loge ç ÷ ÷
è è 2 øø è è 2 øø
(iii) Logarithm of a Complex Number
iq
log e (z ) = log e ( z e ) = log e z + log e (e iq ) y Example 45. If sin ( log e i i ) = a + ib , where i = - 1,
= log e z + iq = log e z + i arg (z ) find a and b, hence and find cos ( log e i i ).
So, the general value of log e (z ) Sol. a + ib = sin ( loge i i ) = sin ( i loge i )
= log e (z ) + 2npi ( -p < arg z < p ). = sin ( i ( loge i + i arg i ))
= sin ( i ( loge 1 + (i p / 2)))
y Example 42. If m and x are two real numbers and = sin ( i ( 0 + (i p / 2))) = sin ( - p / 2) = - 1
m
2 m i cot -1 x æ xi + 1 ö \ a = -1,b = 0
i = - 1 , prove that e ç ÷ = 1.
è xi - 1 ø Now, cos( loge i i ) = 1 - sin 2 ( loge i i )
-1
Sol. Let cot x = q , then cot q = x
= 1 - ( - 1)2 = (1 - 1) = 0
m m
-1 æ xi + 1 ö 2 m i q æ i cot q + 1 ö Aliter
\LHS = e 2 m i cot x
ç ÷ =e ç ÷
è xi - 1 ø è i cot q - 1 ø Q i i = ( e ip / 2 ) i = e - p / 2
æ i (cot q - i ) ö
m m
æ p ö
2 m i q æ cos q - i sin q ö \ sin (loge i i ) = sin (loge e - p / 2 ) = sin ç - loge e ÷
= e2 m iq ç ÷ =e ç ÷ è 2 ø
è i (cot q + i ) ø è cos q + i sin q ø
m = sin ( - p / 2) = - 1 = a + ib [given]
2miq æe - i q ö 2 mi q \ a = - 1, b = 0
=e × ç iq ÷ =e × (e - 2 i q )m
èe ø and cos ( loge i i ) = cos ( loge e - p / 2 )
=e 2 mi q
× e -2 m i q = e 0 = 1 = RHS æ p ö æ pö
= cos ç - loge e ÷ = cos ç - ÷ = 0
è 2 ø è 2ø
y Example 43. If z and w are two non-zero complex
numbers such that z = w and arg (z ) + arg (w ) = p, y Example 46. Find the general value of log 2 ( 5i ),
prove that z = - w . where i = - 1.
Sol. Let arg (w ) = q , then arg (z ) = p - q loge 5i 1
Sol. log 2 5i = = { loge 5i + i arg ( 5i ) + 2npi }
\ z = z (cos (arg z ) + i sin (arg z )) loge 2 loge 2
= z (cos ( p - q ) + i sin ( p - q )) 1 ip
= { loge 5 + + 2npi } , n Î I
= z ( - cos q + i sin q ) = - z (cos q - i sin q ) loge 2 2
Chap 01 Complex Numbers 17

#L Exercise for Session 2


1 - ix
1 If = a - ib and a 2 + b 2 = 1, where a, b Î R and i = -1, then x is equal to
1 + ix
2a 2b 2a 2b
(a) 2 2
(b) 2 2
(c) 2 2
(d)
(1 + a ) + b (1 + a ) + b (1 + b ) + a (1 + b ) 2 + a 2
n
æ1+ i ö 2æ -1 1 ö
2 The least positive integer n for which ç ÷ = çsec + sin- 1 x ÷ ( where, x ¹ 0 , - 1 £ x £ 1and i = - 1), is
è 1- i ø pè x ø
(a) 2 (b) 4 (c) 6 (d) 8
3 6 6
If z = (3 + 4i ) + (3 - 4i ) , where i = - 1, then Im(z ) equals to
(a) - 6 (b) 0 (c) 6 (d) None of these
æx yö
4 If ( x + iy ) 1/ 3
= a + ib , where i = - 1, then ç + ÷ is equal to
èa b ø
(a) 4 a 2b 2 (b) 4 (a 2 - b 2 ) (c) 4 a 2 - b 2 (d) a 2 + b 2
3
5 If = a + ib , where i = - 1 and a 2 + b 2 = l a - 3, the value of l is
2 + cos q + i sin q
(a) 3 (b) 4 (c) 5 (d) 6
z -1
6 If is purely imaginary, then z is equal to
z +1
1
(a) (b) 1 (c) 2 (d) 2
2

7 The complex numbers sin x + i cos 2x and cos x - i sin 2x , where i = - 1, are conjugate to each other, for

(c) x = æçn + ö÷,n ÎI


1
(a) x = np,n Î I (b) x = 0 (d) no value of x
è 2ø
b- a
8 If a and b are two different complex numbers with | b | = 1, then is equal to
1- ab
1
(a) 0 (b) (c) 1 (d) 2
2

9 If x = 3 + 4i ( where, i = - 1), the value of x 4 - 12x 3


+ 70x 2
- 204 x + 225, is
(a) - 45 (b) 0 (c) 35 (d) 15

10 If z1 - 1 £ 1, z 2 - 2 £ 2, z 3 - 3 £ 3, the greatest value of z1 + z 2 + z 3 is


(a) 6 (b) 12 (c) 17 (d) 23
æ 8p ö 8p
11 The principal value of arg (z ), where z = ç1 + cos
è
÷ + i sin
ø
( where, i = - 1) is given by
5 5
p 4p p 4p
(a) - (b) - (c) (d)
5 5 5 5

12 If z1 = 2, z 2 = 3, z 3 = 4 and z1 + z 2 + z 3 = 5, then 4z 2 z 3 + 9z 3 z1 + 16 z1 z 2 is
(a) 24 (b) 60 (c) 120 (d) 240

13 If z - i £ 5 and z1 = 5 + 3i ( where, i = - 1), the greatest and least values of iz + z 1 are


(a) 7 and 3 (b) 9 and 1 (c) 10 and 0 (d) None of these
æz ö æz ö
14 If z1, z 2 and z 3, z4 are two pairs of conjugate complex numbers, then arg ç 1 ÷ + arg ç 2 ÷ equals to
è z4 ø èz3 ø
p 3p
(a) 0 (b) (c) p (d)
2 2
Session 3
amp(z) — amp (—z) = ± π; According as amp (z) is Positive or
Negative, Square Root of a Complex Number, Solution of
Complex Equations, De-Moivre’s Theorem, Cube Roots of Unity

amp ( z ) - amp (- z ) = ± p , From Eq. (ii), we get


z1 = z 2 (cos ( p + arg (z 2 )) + i sin ( p + arg (z 2 )))
According as amp ( z) is Positive [from Eq. (i) and z1 = z 2 ]
or Negative = z 2 ( - cos (arg z 2 ) - i sin (arg z 2 )) = - z 2
[from Eq. (iii)]
Case I amp (z ) is positive.
\ z1 + z 2 = 0
If amp (z ) = q , we have
Y y Example 48.Let z and w be two non-zero complex
numbers, such that z = w and
P z
amp (z ) + amp (w ) = p, then find the relation between
r z and w .
θ Sol. Given, amp (z ) + amp (w ) = p
X
O
r
– ( π – θ)
Þ amp (z ) - amp (w ) = p
Here, z =w = w [given | z | = | w | ]
P′ –z
and amp (z ) > 0
Then, z +w =0
amp ( - z ) = - ( ÐP ¢ OX ) = - ( p - q )
\ amp (z ) - amp ( -z ) = p
Case II amp (z ) is negative.
[here, OP = OP ¢]
Square Root of a Complex Number
Let z = x + iy ,
If amp (z ) = - q
where x , y Î R and i = - 1.
We have, amp ( -z ) = ÐP ¢OX = p - q
\ amp (z ) - amp ( -z ) = - p [here, OP = OP ¢] Suppose ( x + iy ) = a + ib …(i)
Y On squaring both sides, we get
( x + iy ) = (a 2 - b 2 ) + 2iab
P' –z

π–θ On comparing the real and imaginary parts, we get


r
O –θ
X
a2 - b2 = x …(ii)
r
P z and 2ab = y …(iii)
2 2 2 2 2 2 2
\ a +b = (a - b ) + 4a b = (x + y 2 )
2

a2 + b2 = z …(iv)
y Example 47. If z 1 = z 2 and arg (z 1 / z 2 ) = p, then
From Eqs. (ii) and (iv), we get
find the value of z 1 + z 2 .
æz ö æ z +xö æ z -xö
Sol. Q arg ç 1 ÷ = p a = ± çç ÷, b = ±
÷
ç
ç 2 ÷
÷
èz2 ø è 2 ø è ø
Þ arg (z1 ) - arg (z 2 ) = p …(i)
æ z + Re (z ) ö æ z - Re (z ) ö
Q z1 = z1 (cos ( arg z1 ) + i sin (arg z1 )) …(ii) or a = ± çç ÷ , b=± ç ÷
2 ÷ ç 2 ÷
and z 2 = z 2 (cos (arg z 2 ) + i sin (arg z 2 )) …(iii) è ø è ø
Chap 01 Complex Numbers 19

Now, from Eq. (i), the required square roots, ìïæ 3 ö 2 æ i ö 2 3 i üï


ì æ z + Re ( z ) = íç ÷ + ç ÷ + 2 × × ý
z - Re ( z ) ö ïîè 2 ø è 2ø 2 2 ïþ
ï± ç +i ÷ , if Im ( z ) > 0
ïï çè 2 2 ÷
ø 2
i.e. z = í æ3 +iö æ3 +i ö
ï æç z + Re ( z ) z - Re ( z ) ö
÷ , if Im ( z ) < 0
= ç ÷ = ±ç ÷
ï± ç -i è 2 ø è 2 ø
2 2 ÷
ïî è ø
(ii) Let z = - 5 + 12i
Aliter \ | z | = 13, Re (z ) = - 5, Im (z ) = 12 > 0
If ( x + iy ), where x , y Î R and i = - 1, then
æ z + Re (z ) z - Re (z ) ö
(i) If y is not even, then multiply and divide in y by 2, Q z =±ç +i ÷
ç 2 2 ÷
then ( x + iy ) convert in è ø
æ y 2 ö÷ æ æ 13 - 5 ö æ 13 + 5 ö ö
x +y - 1 = çx + 2 - . \ ( - 5 + 12i ) = ± ç ç ÷ +i ç ÷÷
ç
è 4 ÷ø è è 2 ø è 2 øø
= ± (2 + 3i )
y2
(ii) Factorise: - say a , b (a < b). Aliter
4
( - 5 + 12i ) = ( - 5 + 12 - 1 )
Take that possible factor which satisfy
x = (ai ) 2 + b2 , if x > 0 or x = a 2 + (ib)2 , if x < 0 = ( -5 +2 ( - 36 )
(iii) Finally, write x + iy = (ai)2 + b2 + 2iab = ( -5 +2 ( - 9 ´ 4) )
2 2
or a + (ib) + 2iab = (- 9 + 4 + 2 ( - 9 ´ 4 ))
and take their square root.
= (3i ) 2 + 2 2 + 2 × 3i × 2
ì± (ai + b) ì± (b - ia )
(iv) ( x + iy ) = í and ( x - iy ) = í
î or ± (a + i b) îor ± (a -ib) = ( 2 + 3i ) 2 = ± (2 + 3i )
(iii) Let z = - 8 - 15i
Remark
1+ iö \ z = 17, Re (z ) = - 8, Im (z ) = - 15 < 0
1. The square root of i is ± æç ÷, where i = -1.
è 2 ø æ æ 17 - 8 ö æ 17 + 8 ö ö
1- iö \ ( - 8 - 15i ) = ± ç ç ÷ -i ç ÷÷
2. The square root of ( - i ) is æç ÷. è è 2 ø è 2 øø
è 2 ø
æ 3 - 5i ö
y Example 49. Find the square roots of the following =±ç ÷
è 2 ø
(i) 4 + 3i (ii) - 5 + 12i
Aliter ( - 8 - 15i ) = ( - 8 - 15 - 1 )
(iii) - 8 - 15i (iv) 7 - 24i (where, i = - 1)
Sol. (i) Let z = 4 + 3i æ æ 225 ö ö æ æ 25 9 ö ö
\ | z | = 5, Re (z ) = 4, Im (z ) = 3 > 0 = ç- 8 - 2 ç- ÷ ÷ = ç- 8 - 2 ç- ´ ÷÷
è è 4 øø è è 2 2ø ø
æ z + Re (z ) z - Re (z ) ö
z =±ç +i ÷
æ 9 25 æ 25 9 ö ö
Q
ç 2 2 ÷ = ç - - 2 ç- ´ ÷÷
è ø
è2 2 è 2 2ø ø
æ æ5 + 4ö æ 5 - 4 ö ö æ3 +iö
\ ( 4 + 3i ) = ± ç ç ÷ +i ç ÷÷ =±ç ÷
è è 2 ø è 2 øø è 2 ø 2 2
Aliter æ 3 ö æ 5i ö 3 5i
= ç ÷ + ç ÷ -2× ×
æ 9ö è 2ø è 2ø 2 2
( 4 + 3i ) = 4 + 3 - 1 = 4 + 2 ç - ÷
è 4ø 2
æ 3 - 5i ö æ 3 - 5i ö
9 1 æ 9ö = ç ÷ =±ç ÷
= - + 2 ç- ÷ è 2 ø è 2 ø
2 2 è 4ø
20 Textbook of Algebra

(iv) Let z = 7 - 24i æ x 2 + x + 1ö æ x 2


- x + 1ö
ç ÷-ç ÷
\ z = 25, Re (z ) = 7, Im (z ) = - 24 < 0 è 2 ø è 2 ø
=
æ z + Re (z ) z - Re (z ) ö éæ x 2
+ x + 1ö æx 2
- x + 1öù
Q z =±ç -i ÷ + 2 êç ÷´-ç ÷ú
ç 2 2 ÷ êë è 2 ø è 2 ø úû
è ø
æ æ 25 + 7 ö æ 25 - 7 ö ö
\ (7 - 24i ) = ± ç ç ÷ -i ç ÷÷ ìæ 2
ö æ ö
2 ü
è è 2 ø è 2 øø ï ç æç x + x + 1 ö÷ ÷ + çi æç x - x + 1 ö÷ ÷
2 2
ï
ïï çè è 2 ø ÷ø çè è 2 ø ÷ø ïï
= ± ( 4 - 3i ) = í ý
Aliter ï æ x 2 + x +1ö æ x 2 - x + 1ö ï
ï+ 2 ç ÷ ×i ç ÷ ï
(7 - 24 i ) = (7 - 24 -1 ) = 7 - 2 ( - 144 ) ïî è 2 ø è 2 ø ïþ
2
= 7 - 2 (16 ´ - 9 ) æ æ x 2 + x +1ö æ x 2 - x +1ö ö
= ç ç ÷ +i ç ÷÷
ç è 2 ø è 2 ø ÷ø
è
= 16 - 9 - 2 (16 ´ - 9 )
æ æx 2
+ x + 1ö æx 2
- x + 1 ö ö÷
= ( 4 ) 2 + (3i ) 2 - 2 × 4 × 3i =±ç ç ÷ +i ç ÷
ç è 2 ø è 2 ø ÷ø
è
= ( 4 - 3i ) 2 = ± ( 4 - 3i )

y Example 50. Find the square root of Solution of Complex Equations


x + ( - x4 - x 2
- 1).
Putting z = x + iy , where x , y Î R and i = - 1 in the given
Sol. Let z = x + (- x 4 - x 2
- 1) equation and equating the real and imaginary parts, we
= x + i (x 4 + x 2
+ 1) [Q -1 = i ]
get x and y, then required solution is z = x + iy .

\ z = x 2
+ (x 4 + x 2
+ 1) y Example 51. Solve the equation z 2 + z = 0.
= ( x 4 + 2x 2
+ 1) = ( x 2
+ 1) 2 Sol. Let z = x + iy , where x , y Î R and i = - 1 …(i)
2 2 2 2
\ z = ( x 2 + 1) Þ z = ( x + iy ) = x - y + 2ixy
2 2
Re (z ) = x and z = (x + y )
4 2
Im (z ) = ( x + x + 1) > 0 Then, given equation reduces to
2
æ z + Re (z ) z - Re (z ) ö x - y 2 + 2ixy + ( x 2
+ y 2) = 0
Q z = ± çç +i ÷
÷
è 2 2 ø On comparing the real and imaginary parts, we get
2
x - y 2 + (x 2
+ y 2) = 0 …(ii)
4 2
\ x + (- x - x - 1)
and 2xy = 0 …(iii)
æ æx 2
+1+ xö æx 2
+ 1 - x ö ö÷ From Eq. (iii), let x = 0 and from Eq. (ii),
=±ç ç ÷ +i ç ÷
ç è 2 ø è 2 ø ÷ø - y2 + y2 = 0
è
2
Aliter Þ - y + y =0
æ- x - x 4 2
- 1ö \ y = 0, 1
x + (- x 4 - x 2
- 1) = x + 2 ç ÷
è 4 ø Þ y = 0, ± 1
From Eq. (iii), let y = 0 and from Eq. (ii),
æ - ( x 2 + x + 1) ( x 2 - x + 1) ö 2 2
= x +2 ç ÷ x + x =0
è 4 ø 2
Þ x + x =0
éæ x 2
+ x + 1ö æx 2
- x + 1öù Þ x
2
+ x =0 Þ x =0
= x + 2 êç ÷´-ç ÷ú
êë è 2 ø è 2 ø úû \ x + iy are 0 + 0 ×i, 0 + i, 0 - i
i.e. z = 0, i , - i are the solutions of the given equation.
Chap 01 Complex Numbers 21

y Example 52. Find the number of solutions of the


2
equation z 2 + z = 0.
De-Moivre’s Theorem
2 Statements
Sol. Q z2 + z = 0 or z 2 + z z = 0
(i) If q 1 , q 2 , q 3 , ..., q n Î R and i = - 1, then
Þ z (z + z ) = 0
(cos q 1 + i sin q 1 ) (cos q 2 + i sin q 2 )
\ z =0 …(i) (cos q 3 + i sin q 3 )... (cos q n + i sin q n )
and z + z = 0 Þ 2 Re (z ) = 0 = cos (q 1 + q 2 + q 3 + ...+ q n )
\ Re (z ) = 0 + i sin (q 1 + q 2 + q 3 + ... + q n )
If z = x + iy [Q x = Re (z )]
(ii) If q Î R, n Î I (set of integers) and i = - 1, then
= 0 + iy , y Î R
and i = -1 …(ii) (cos q + i sin q ) n = cos nq + i sin nq
On combining Eqs. (i) and (ii), then we can say that the (iii) If q Î R, n Î Q (set of rational numbers)
given equation has infinite solutions. and i = - 1, then cos n q + i sin n q is one of the values
y Example 53. Find all complex numbers satisfying of (cos q + i sin q ) n .
2
the equation 2 z + z 2 - 5 + i 3 = 0, where i = - 1. Proof
Sol. Let z = x + iy , where x , y Î R and i = - 1 (i) By Euler’s formula, e iq = cos q + i sin q
Þ z 2 = ( x + iy )2 = x 2 - y 2 + 2ixy LHS = (cos q 1 + i sin q 1 ) (cos q 2 + i sin q 2 )
and z = (x 2 + y 2 ) (cos q 3 + i sin q 3 ) ... (cos q n + i sin q n )
Then, given equation reduces to = e iq1 × e iq2 × e iq3 ... e iqn = e i ( q1 + q2 + q3 + ... + qn )
2 ( x 2 + y 2 ) + x 2 - y 2 + 2ixy - 5 + i 3 = 0 = cos (q 1 + q 2 + q 3 + ... + q n )
Þ 2 2
(3x + y - 5) + i (2xy + 3 ) = 0 = 0 + i × 0 + i sin (q 1 + q 2 + q 3 + ... + q n ) = RHS
On comparing the real and imaginary parts, we get (ii) If q 1 = q 2 = q 3 = ... = q n = q, then from the above
3x 2 + y 2 - 5 = 0 …(i) result (i), (cos q + i sin q ) (cos q + i sin q )
and 2xy + 3 = 0 …(ii) (cos q + i sin q ) ... upto n factors
On substituting the value of x from Eq. (ii) in Eq. (i), we get = cos (q + q + q + ... upton times)
æ 3ö
2 + i sin (q + q + q + ... upto n times)
2
3 ç- ÷ +y -5=0
è 2y ø i.e., (cos q + i sin q ) n = cos nq + i sin nq
9
Þ + y2 = 5 p
4y 2 (iii) Let n = , where p, q Î I and q ¹ 0, from above result (ii),
q
or 4y 4 - 20y 2 + 9 = 0 q
æ æp ö æ p öö
Þ 2 2
(2y - 9 ) (2y - 1) = 0 we have ç cos ç q ÷ + i sin ç q ÷ ÷
è èq ø è q øø
9 1 3 1
\ y 2 = , y 2 = or y = ± ,y = ± ææ p ö ö ææ p ö ö
2 2 2 2 = cos ç ç q ÷ q ÷ + i sin ç ç q ÷ q ÷ = cos p q + i sin p q
or y=-
3 3
, ,-
1 1
,
èè q ø ø èè q ø ø
2 2 2 2 æ pq ö æ pq ö
From Eq. (ii), we get Þ cos ç ÷ + i sin ç ÷ is one of the values of
è q ø è q ø
1 1 3 3
x= ,- , ,- (cos p q + i sin p q ) 1 /q
6 6 2 2
\ z = x + iy æ pq ö æ pq ö
Þ cos ç ÷ + i sin ç ÷ is one of the values of
1 3i 1 3i 3 i 3 i è q ø è q ø
= - ,- + , - ,- +
6 2 6 2 2 2 2 2 [(cos q + i sin q ) p ]1 /q
are the solutions of the given equation.
22 Textbook of Algebra

æ pq ö æ pq ö To Find the Roots of (a + ib ) p /q , where a , b Î R ;


Þ cos ç ÷ + i sin ç ÷ is one of the values of
è q ø è q ø p , q Î I , q ¹ 0 and i = - 1
/q
(cos q + i sin q ) p Let a + ib = r (cos q + i sin q ) [polar form]
\ (a + ib )p / q = {r (cos (2np + q )
Other Forms of De-Moivre’s Theorem + i sin (2np + q) )} p /q
,n ÎI
1. (cos q - i sin q) n = cos n q - i sin n q, " n Î I
p /q p /q
Proof (cos q - i sin q) n = (cos ( - q) + i sin ( - q)) n =r (cos (2np + q ) + i sin (2np + q ))
= cos ( - nq) + i sin ( - nq) = cos nq - i sin nq /qæ æp ö æp öö
2. (sin q + i cos q) n = ( i ) n (cos nq - i sin n q), " n Î I
=rp ç cos ç (2np + q )÷ + i sin ç (2np + q )÷ ÷ ,
è èq ø èq øø
Proof (sin q + i cos q) n = ( i (cos q - i sin q)) n where, n = 0, 1, 2, 3, ..., q - 1
= i n (cos q - i sin q) n = ( i ) n (cos nq - i sin n q)
[from remark (1)] y Example 56. Find all roots of x 5 - 1 = 0.
3. (sin q - i cos q) n = ( - i ) n (cos nq + i sin nq), " n Î I
Proof (sin q - i cos q) n = ( - i (cos q + i sin q)) n
Sol. Q x 5 - 1 = 0 Þ x 5 = 1
= ( - i ) n (cos q + i sin q) n \ x = (1)1 / 5 = (cos 0 + i sin 0)1 / 5 ,
= ( - i ) n (cos nq + i sin nq)
n
4. (cos q + i sin f) ¹ cos nq + i sin nf, " n Î I
where i = -1
[here, q ¹ f\ De-Moivre’s theorem is not applicable] = [cos (2np + 0) + i sin (2np + 0)]1/ 5
1
5. = (cos q + i sin q) - 1
cos q + i sin q æ 2np ö æ 2n p ö
= cos ç ÷ + i sin ç ÷,
è 5 ø è 5 ø
= cos ( - q) + i sin ( - q) = cos q - i sin q
where, n = 0, 1, 2, 3, 4
p p
y Example 54. If z r = cos æç r ö÷ + i sin æç r ö÷ , where
\ Roots are
è3 ø è3 ø æ 2p ö æ 2p ö æ 4p ö
1, cos ç ÷ + i sin ç ÷, cos ç ÷ + i sin ç ÷,
æ 4p ö
i = - 1, prove that z 1 z 2 z 3 ... upto infinity = i. è 5 ø è 5 ø è 5 ø è 5 ø
æpö æpö æ 6p ö æ 6p ö æ 8p ö æ 8p ö
Sol. We have, zr = cos ç r ÷ + i sin ç r÷ cos ç ÷ + i sin ç ÷, cos ç ÷ + i sin ç ÷
è3 ø è3 ø è 5 ø è 5 ø è 5 ø è 5 ø
æp p p ö æ 6p ö æ 6p ö
\ z1 z 2 z 3 ... ¥ = cos ç + 2 + 3 + ... + ¥ ÷ Now, cos ç ÷ + i sin ç ÷
è3 3 3 ø è 5 ø è 5 ø
æp p p ö æ 4p ö æ 4p ö
+ i sin ç + 2 + 3 +... + ¥ ÷ = cos ç2p - ÷ + i sin ç2p - ÷
è3 3 3 ø è 5 ø è 5 ø
æ p ö æ p ö æ 4p ö æ 4p ö
ç ÷ ç ÷ æpö æpö = cos ç ÷ - i sin ç ÷
= cos ç 3 ÷ + i sin ç 3 ÷ = cos ç ÷ + i sin ç ÷ è 5 ø è 5 ø
1
ç1 - ÷ ç1 - ÷1 è 2 ø è2ø
è æ 8p ö æ 8p ö
3ø è 3ø and cos ç ÷ + i sin ç ÷
è 5 ø è 5 ø
= 0 + i ×1 = i
æ 2p ö æ 2p ö
4 = cos ç2p - ÷ + i sin ç2p - ÷
(cos q + i sin q ) è 5 ø è 5 ø
y Example 55. Express in a + ib
(sin q + i cos q ) 5 æ 2p ö æ 2p ö
= cos ç ÷ - i sin ç ÷
è 5 ø è 5 ø
form, where i = - 1.
Sol. Q (sin q + i cos q )5 = (i )5 (cos q - i sin q ) 5 æ 2p ö æ 2p ö
Hence, roots are 1, cos ç ÷ ± i sin ç ÷
è 5 ø è 5 ø
= i (cos q + i sin q )- 5
æ 4p ö æ 4p ö
(cos q + i sin q )4 (cos q + i sin q )4 and cos ç ÷ ± i sin ç ÷.
\ = è 5 ø è 5 ø
(sin q + i cos q ) 5 i (cos q + i sin q )- 5
(cos q + i sin q )9 Remark
=
i Five roots are 1, z1, z2, z1, z2 (one real, two complex and two
cos 9 q + i sin 9 q conjugate of complex roots).
= = - i cos 9 q + sin 9 q
i
= sin 9 q - i cos 9 q
Chap 01 Complex Numbers 23

y Example 57. Find all roots of the equation


x 6 - x 5 + x 4 - x 3 + x 2 - x + 1 = 0.
Properties of Cube Roots of Unity
(i) 1 + w + w2 = 0 and w 3 = 1
Sol. Q 1- x + x2 - x3 + x4 - x5 + x6 =0
[1 - ( - x )7 ] (ii) To find the value of wn (n > 3 ).
Þ 1× = 0, 1 + x ¹ 0
1 - (- x ) First divide n by 3. Let q be the quotient and r be the
or 1 + x 7 = 0, x ¹ - 1 or x 7 = - 1 remainder. 3 ) n (q
\ x = ( - 1)1 / 7 = (cos p + i sin p)1 / 7 , i = -1 - 3q
r
= [cos (2n + 1) p + i sin (2n + 1) p ]1 / 7
i.e. n = 3q + r , where 0 £ r £ 2
æ (2n + 1)p ö æ (2n + 1)p ö
= cos ç ÷ + i sin ç ÷
è 7 ø è 7 ø \ wn = w 3q + r = ( w 3 ) q × wr = wr
for n = 0, 1, 2, 4, 5, 6. In general, w 3n = 1 , w 3n + 1 = w , w 3n + 2 = w 2
Remark ì3, when n is a multiple of 3
(iii) 1 + wr + w2r = í
QFor n = 3, x = - 1 but here x ¹ - 1 î0, when n is not a multiple of 3
\ n¹ 3
(iv) Cube roots of - 1 are - 1, - w and - w2 .
(v) a + b w + c w2 = 0 Þ a = b = c , if a, b, c Î R.
Cube Roots of Unity (vi) If a, b, c are non-zero numbers such that
Let z = (1) 1/ 3
Þ z =1 Þ z -1=0 3 3 a + b + c = 0 = a 2 + b 2 + c 2 , then a : b : c = 1 : w : w2 .
Þ (z - 1) (z + z + 1) = 0 Þ z - 1 = 0 or z 2 + z + 1 = 0
2
(vii) A complex number a + ib (where i = -1), for which
- 1 ± (1 - 4 )
-1 ± i 3 a : b = 1 : 3 or 3 : 1 can always be expressed in
\ z = 1 or z = =
2 2 terms of w or w2 .
- 1 + i 3 - 1 -i 3
Therefore, z = 1 , , , where i = -1. For example,
2 2 (a) 1 + i 3 = - ( - 1 - i 3 ) [Q |1 : 3 | = 1 : 3 ]
If second root is represented by w (omega), third root will
be w2 . æ-1-i 3ö 2
= -2 ç ÷ = -2w
\ Cube roots of unity are 1, w, w2 and w, w2 are called è 2 ø
non-real complex cube roots of unity. i ( 3 + i ) (- 1 + i 3 )
(b) 3 +i = =
i i
Remark æ - 1 + i 3 ö æ2ö
1. w = w2, ( w) 2 = w 2. w = ± w2, w2 = ± w =ç ÷ç ÷ [Q | 3 : 1| = 3 : 1]
2
è 2 ø èiø
3. w = w =1
2w
= = -2i w
Aliter i
Let z = (1) 1 / 3 = (cos 0 + i sin 0 ) 1 / 3 , i = - 1 (viii) The cube roots of unity when represented on complex
plane lie on vertices of an equilateral triangle
= [cos (2 np + 0 ) + i sin (2 np + 0 )]1 / 3 inscribed in a unit circle, having centre at origin. One
æ 2 np ö æ 2 np ö vertex being on positive real axis.
= cos ç ÷ + i sin ç ÷ , where, n = 0, 1, 2
è 3 ø è 3 ø Y
i
Therefore, roots are 2π/3
ω
æ 2p ö æ 2p ö æ 4p ö æ 4p ö 2π/3
1, cos ç ÷ + i sin ç ÷ , cos ç ÷ + i sin ç ÷
è 3 ø è 3 ø è 3 ø è 3 ø X′ X
–1 1
2pi / 3 4pi / 3 O
or 1, e ,e
ω2 2π/3
If second root is represented by w, then third root will be w2 –i
or if third root is represented by w, then second root will be w2 . Y′
24 Textbook of Algebra

Important Relations in Terms y Example 59. If a , b and g are the roots of


of Cube Root of Unity x 3 - 3x 2 + 3x + 7 = 0, find the value of
(i) a 2+ ab + b 2 = (a - bw ) (a - bw2 ) a - 1 b - 1 g -1
+ + .
(ii) a 2- ab + b 2 = (a + bw ) (a + bw2 ) b -1 g -1 a -1
(iii) a 3+ b 3 = (a + b ) (a + bw ) (a + bw2 ) Sol. We have, x 3 - 3x 2 + 3x + 7 = 0
(iv) a 3 - b 3 = (a - b ) (a - bw ) (a - bw2 ) Þ ( x - 1 )3 + 8 = 0
(v) a 2 + b 2 + c 2 - ab - bc - ca
Þ ( x - 1) 3 + 2 3 = 0
= (a + bw + cw2 ) (a + bw2 + cw )
(vi) a 3 + b 3 + c 3 - 3abc Þ ( x - 1 + 2) ( x - 1 + 2 w) ( x - 1 + 2 w2 ) = 0

= (a + b + c ) (a + bw + cw2 ) (a + bw2 + cw ) Þ ( x + 1) ( x - 1 + 2 w) ( x - 1 + 2 w2 ) = 0

y Example 58. If w is a non-real complex cube root of \ x = - 1, 1 - 2 w, 1 - 2 w2


unity, find the values of the following. Þ a = - 1, b = 1 - 2 w, g = 1 - 2 w2
1999
(i) w a - 1 b -1 g - 1 -2 - 2 w - 2 w2
- 998 Then, + + = + +
(ii) w b - 1 g - 1 a - 1 - 2 w - 2 w2 -2
3n + 2
æ - 1+ i 3ö 1 1
(iii) ç ÷ , n Î N and i = - 1 = + + w2 = w2 + w2 + w2 = 3 w2
è 2 ø w w

(iv) (1 + w) (1 + w 2 ) (1 + w 4 ) (1 + w 8 )... upto 2n factors 3+i


y Example 60. If z = , where i = - 1, find the
2
æ a + bw + gw 2 + dw 2 ö
(v) ç ÷ , where a, b, g , d ÎR value of (z 101 + i 103 )105 .
2
è b + aw + gw + dw ø æi 3 + i 2 ö
3 +i 1
Sol. Q z= = ç ÷ [Qi 2 = - 1]
(vi) 1 × (2 - w) (2 - w 2 ) + 2 × ( 3 - w) ( 3 - w 2 ) + 3 × 2 i è 2 ø
(4 - w)(4 - w 2 ) + K + K + (n - 1) ×(n - w)(n - w 2 ) æ- 1 + i 3ö
= -i ç ÷ = -iw
Sol. (i) w 1999
=w 3 ´ 666 + 1
=w è 2 ø
1 w \ z 101 = ( - iw)101 = - i 101 × w101 = - i w2 and i 103 = i 3 = - i
(ii) w- 998 = = =w
w998 w999 Then, z 101 + i 103 = - i w2 - i = - i ( w2 + 1)
3n + 2
æ - 1+i 3 ö
(iii) ç ÷ = w3n + 2 = w3n × w2 = ( w3 )n × w2 = - i ( - w) = i w
è 2 ø
Hence, (z 101 + i 103 )105 = (iw)105 = i 105 × w105 = i × 1 = i
= (1)n × w2 = w2
50
(iv) (1 + w) (1 + w2 ) (1 + w4 ) (1 + w8 ) ... upto 2n factors æ3 i 3ö
y Example 61. If ç + ÷ = 3 25 ( x - iy ), where
= (1 + w) (1 + w2 ) (1 + w) (1 + w2 ) ... upto 2n factors è2 2 ø
= ( - w2 ) ( - w) ( - w2 ) (- w) ... upto 2n factors x , y ÎR and i = - 1, find the ordered pair of ( x , y ).
= ( w3 ) ( w3 )... upto n factors = 1 × 1× 1 × ...upto n factors 3 i 3 æ 3 +iö 3 æi 3 + i 2 ö
Sol. Q + = 3ç ÷ = ç ÷
= (1)n = 1 2 2 è 2 ø i è 2 ø

æ a + b w + g w2 + d w2 ö w (a + b w + g w2 + d w2 ) æ- 1 + i 3ö
(v) ç ÷= = -i 3 ç ÷ = -i 3 w
è b + a w2 + g w + d w ø ( b w + a w3 + g w2 + d w2 ) è 2 ø
50
w (a + b w + g w2 + d w2 ) æ3 i 3 ö
= =w \ç + ÷ = ( - i 3 w) 50 = i 50 × 325 × w50
( b w + a + g w2 + d w2 ) è2 2 ø

(vi) S (n - 1) (n - w) (n - w2 ) = S (n 3 - 1) = S n3 - S 1 æ- 1 - i 3ö
= - 1 × 325 × w2 = - 325 × ç ÷
2 è 2 ø
ì n ( n + 1) ü
=í ý -n
î 2 þ
Chap 01 Complex Numbers 25

æ1 i 3 ö Then, f ( - w) = 0 and f ( - w2 ) = 0
= 325 ç + 25
÷ = 3 ( x - iy ) [given]
è 2 2 ø Þ - 7 w3 - aw + b = 0 and - 7 w6 - aw2 + b = 0
1 3 or - 7 - aw + b = 0
\ x = ,y = -
2 2 and - 7 - aw2 + b = 0
æ1 3ö On adding, we get
Þ Ordered pair is ç , - ÷.
è2 2 ø - 14 - a ( w + w2 ) + 2b = 0
or - 14 + a + 2b = 0 or a + 2b = 14 …(i)
y Example 62. If the polynomial 7 x 3 + ax + b is
and on subtracting, we get
divisible by x 2 - x + 1, find the value of 2a + b . - a ( w - w2 ) = 0
3
Sol. Let f (x ) = 7x + ax + b Þ a=0 [Q w - w2 ¹ 0]
and x 2 - x + 1 = ( x + w) ( x + w2 ) From Eq. (i), we get b = 7
Q f ( x ) is divisible by x - x + 1 2 \ 2a + b = 7

#L Exercise for Session 3


1 The real part of (1 - i )- i , where i = - 1 is

(a) e - p / 4 cos æç loge 2ö÷ (b) - e - p / 4 sin æç loge 2ö÷


1 1
è2 ø è2 ø

cos æç loge 2ö÷ (d) e - p / 4 sin æç loge 2ö÷


p/4 1 1
(c) e
è2 ø è2 ø
- iq
2 The amplitude of e e , where q ÎR and i = - 1 is
(a) sinq (b) - sinq
(c) e cos q (d) e sinq

3 If z = i loge (2 - 3 ), where i = - 1, then the cos z is equal to


(a) i (b) 2i (c) 1 (d) 2
i
4 If z = i i , where i = - 1, then z is equal to
(a) 1 (b) e - p / 2 (c) e - p (d) e p

5 ( - 8 - 6i ) is equal to (where, i = -1)


(a) 1 ± 3i (b) ± (1 - 3i ) (c) ± (1 + 3 i ) (d) ± (3 - i )

(5 + 12i ) + (5 - 12i )
6 is equal to (where, i = - 1)
(5 + 12i ) - (5 - 12i )
3 3 3 3
(a) - i (b) i (c) - i (d) -
2 4 4 2

7 If 0 < amp (z ) < p, then amp (z ) - amp ( - z ) is equal to


(a) 0 (b) 2 amp (z ) (c) p (d) - p

8 If z1 = z 2 and amp (z1) + amp (z 2 ) = 0, then


(a) z1 = z 2 (b) z1 = z 2 (c) z1 + z 2 = 0 (d) z1 = z 2

9 The solution of the equation z - z = 1 + 2i , where i = - 1, is


3 3 3 3
(a) 2 - i (b) + 2i (c) - 2i (d) - 2 + i
2 2 2 2
26 Textbook of Algebra

10 The number of solutions of the equation z 2 + z = 0, is


(a) 1 (b) 2
(c) 3 (d) 4
æ ra ö æ ra ö
11 If z r = cos ç 2 ÷ + i sin ç 2 ÷, where r = 1, 2, 3, ..., n and i = - 1, then lim z1z 2 z 3 K z n is equal to
èn ø èn ø n®¥

(a) e ia (b) e - ia / 2
ia / 2
(c) e (d) 3 e ia
n
æ 1 + sin q + i cos q ö
12 If q ÎR and i = - 1, then ç ÷ is equal to
è 1 + sin q - i cos q ø
np np np np
(a) cos æç - nqö÷ + i sin æç - nqö÷ (b) cos æç + nqö÷ + i sin æç + nqö÷
è 2 ø è 2 ø è 2 ø è 2 ø
np np æ p ö æ p ö
(c) sin æç - nqö÷ + i cos æç - nqö÷ (d) cos çn æç + 2qö÷ ÷ + i sinçn æç + 2 qö÷ ÷
è 2 ø è 2 ø è è2 øø è è2 øø

13 If i z 4 + 1 = 0, where i = - 1, then z can take the value


1+ i p p
(a) (b) cos æç ö÷ + i sin æç ö÷
2 è 8ø è 8ø
1
(c) (d) i
4i
14 If w ( ¹ 1) is a cube root of unity, then (1 - w + w2 ) (1 - w2 + w4 ) (1 - w4 + w8 ) ... upto 2n factors, is
(a) 2n (b) 22 n
(c) 0 (d) 1
x a+ yb+z g
15 If a, b and g are the cube roots of p ( p < 0), then for any x , y and z , is equal to
xb+ y g +z a
1 1
(a) (- 1 -i 3 ), i = -1 (b) (1 + i 3 ), i = -1
2 2
1
(c) (1 - i 3 ), i = -1 (d) None of these
2
Session 4
nth Root of Unity, Vector Representation of Complex Numbers,
Geometrical Representation of Algebraic Operation on Complex Numbers,
Rotation Theorem (Coni Method), Shifting the Origin in Case of Complex
Numbers, Inverse Points, Dot and Cross Product, Use of Complex Numbers
in Coordinate Geometry

nth Root of Unity Remark


1 × a × a2 × a3 ... an - 1 = ( - 1) n - 1 is the basic concept to be
Let x be the nth root of unity, then
understood.
x = (1) 1 /n = (cos 0 + i sin 0 ) 1 /n
(c) If a is an imaginary nth root of unity, then other roots
= (cos (2kp + 0 ) + i sin (2kp + 0 ) 1 /n are given by a 2 , a 3 , a 4 , ... , a n .
[where k is an integer]
1 /n
= (cos 2kp + i sin 2kp) Imaginary
axis
æ 2kp ö æ 2kp ö
\ x = cos ç ÷ + i sin ç ÷ Y
è n ø è n ø
i
where, k = 0, 1, 2, 3, ..., n - 1 A2(α2)
r
2p 2p A1(α)
Let a = cos + i sin , the n, nth roots of unity are θ
n n θ A0
X′ X Real axis
a k (k = 0, 1, 2, 3, ... , n - 1) i.e, the n, nth roots of unity are –1 θ 1

1, a, a 2 , a 3 , ... , a n - 1 which are in GP with common ratio An-1(αn-1)

= e 2pi / n . –i

(a) Sum of n, nth roots of unity Y′

1 × (1 - a n ) (d) Q 1 + a + a 2 + ... + a n - 1 = 0
1 + a + a 2 + a 3 + ... + a n - 1 =
(1 - a ) n -1

=
1 - (cos 2 p + i sin 2 p) Þ S ak = 0
k=0
1-a
n -1 n -1
æ 2 pk ö æ 2 pk ö
=
1 - (1 + 0 )
=0 or S cos ç ÷ + i S sin ç ÷ =0
1-a k=0 è n ø k = 0 è n ø
n -1
æ 2 pk ö
Remark Þ S
k=0
cos ç
è
÷ =0
ø
1 + a + a2 + a3 + ... + an - 1 = 0 is the basic concept to be n
understood. n -1

(b) Product of n , nth roots of unity and S sin æçè 2pk ö÷ø = 0
k=0 n
1 ´ a ´ a 2 ´ a 3 ´ ... ´ a n - 1 = a 1 + 2 + 3 + ... + (n - 1 )
(n - 1 ) n These roots are located at the vertices of a regular
(n - 1 ) n
æ 2p 2p ö 2 plane polygon of n sides inscribed in a unit circle
=a 2 = ç cos + i sin ÷ having centre at origin, one vertex being on positive
è n n ø
real axis.
= cos (n - 1) p + i sin (n - 1) p
n -1 n -1
(e) x n - 1 = ( x - 1) ( x - a ) ( x - a 2 ) ... ( x - a n - 1 ).
= (cos p + i sin p) = ( - 1)
28 Textbook of Algebra

Important Benefits ì
iq
niq
2 × 2i sin æ nq ö ï
ü
1. If 1, a1, a2, a3, ... , an - 1 are the n, nth root of unity, then
ï e × e ç ÷
ìe iq { (e iq ) n - 1} ü ï è 2 øï
( 1) p + ( a1 ) p + ( a2 ) p + ... + ( an - 1 ) p = Im í ý = Im í ý
iq
ì0, if p is not an integral multiple of n î e -1 þ ï e iq /2 × 2i sin æç q ö÷ ï

în, if p is an integral multiple of n ï è2ø ï
î þ
ì0 , if nis even
2. ( 1 + a1 ) ( 1 + a2 ) ... ( 1 + an - 1 ) = í ì æ nq ö ü æ nq ö
î1, if nis odd ïsin çè 2 ÷ø æç n + 1 ö÷iq ï sin çè 2 ÷ø éæ n + 1ö ù
ï è ø ï
3. ( 1 - a1 ) ( 1 - a2 ) ... ( 1 - an - 1 ) = n = Im í ×e 2 ý= × sin ê ç ÷ qú
( n - 2) / 2 ï sin æç q ö÷ ï sin æç q ö÷ ëè 2 ø û
æ z 2 - 2z cos 2rp + 1ö,
4. z n - 1 = ( z - 1) ( z + 1) P
r =1
ç
è
÷
ø
ïî è2ø þï è2ø
n
if ‘n’ is even.
( n - 2) / 2 Remark
5. z n + 1 = P
æ 2 æ ( 2r + 1) pö ö 2p
ç z - 2z cos ç ÷ + 1÷, if n is even. For q = , we get
r =0 è è n ø ø
n
( n - 3) / 2
æ 2 æ ( 2r + 1) p ö + 1ö, 2p 4p 6p ( 2n - 2) p ö
6. z n + 1 = ( z + 1) P ç z - 2z cos ç ÷ ÷ 1. 1 + cos æç ö÷ + cos æç ö÷ + cos æç ö÷ + ... + cos æç ÷ =0
r =0 è è n ø ø è nø è nø è nø è n ø
if ‘n’ is odd. æ 2p ö æ 4p ö æ 6p ö æ ( 2n - 2) p ö
2. sin ç ÷ + sin ç ÷ + sin ç ÷ + ... + sin ç ÷ =0
è nø è nø è nø è n ø

The Sum of the Following y Example 63. If 1, w , w 2 , ..., wn - 1 are n, nth roots of
Series Should be Remembered unity, find the value of (9 - w) (9 - w 2 ) ...(9 - wn -1 ).
(i) cos q + cos 2 q + cos 3 q + ... + cos nq Sol. Let x = (1)1/n Þ xn - 1 = 0
æ nq ö has n roots 1, w, w2 , ... , wn - 1
sin ç ÷
è 2 ø éæ n + 1ö ù \ x n - 1 = ( x - 1) ( x - w) ( x - w2 ) K ( x - wn - 1 )
= . cos ê ç ÷ qú
æqö
sin ç ÷ ëè 2 ø û On putting x = 9 in both sides, we get
è2ø 9n - 1
= (9 - w) (9 - w2 ) (9 - w3 ) ... (9 - wn - 1 )
(ii) sin q + sin 2 q + sin 3 q + ... + sin nq 9 -1
æ nq ö (9 - w) (9 - w2 ) ... (9 - wn - 1 ) =
9n - 1
sin ç ÷ or
è 2 ø éæ n + 1ö ù 8
= × sin ê ç ÷ qú
æqö
sin ç ÷ ëè 2 ø û Remark
è2ø x n -1
= ( x - w) ( x - w2 ) ... ( x - wn - 1 )
Proof x -1
xn -1
(i) cos q + cos 2 q + cos 3 q + ... + cos n q \ lim = lim ( x - w) ( x - w2 ) ... ( x - wn - 1 )
x ®1 x -1 x ®1
= Re {e iq + e 2iq + e 3i q + ... + e niq }, where i = -1 Þ n = ( 1 - w) ( 1 - w2 ) ... ( 1 - wn - 1 )
ì iq niq /2 æ nq ö ü
ï e ×e × 2i sin ç ÷ ï æ 2p ö æ 2p ö
iq
ìe { (e ) - 1} ü iq n
ï è 2 øï y Example 64. If a = cos ç ÷ + i sin ç ÷ , where
= Re í ý = Re í ý è 7 ø è 7 ø
iq iq /2
î e -1 þ ï e × 2i sin (q /2 ) ï i = -1, find the quadratic equation whose roots
ïî ïþ
are a = a + a 2 + a 4 and b = a 3 + a 5 + a 6 .
ì æ nq ö ü æ nq ö æ 2p ö æ 2p ö
ïsin çè 2 ÷ø æç n + 1 ö÷iq ï sin çè 2 ÷ø éæ n + 1ö ù
Sol. Q a = cos ç ÷ + i sin ç ÷
ï è ø ï è7 ø è7 ø
= Re í ×e 2 ý= × cos ê ç ÷ qú
q
ï sin æç ö÷ ï sin æç ö÷q ëè 2 ø û \ a 7 = cos 2p + i sin 2p = 1 + 0 = 1
ïî è2ø ïþ è2ø or a = (1)1 / 7
(ii) sin q + sin 2 q + sin 3 q + ... + sin nq \ 1 , a , a 2 , a 3 , a 4 , a 5 , a 6 are 7, 7 th roots of unity.
= Im {e iq + e 2iq + e 3iq + ... + e niq }, where i = -1 \ 1 + a + a2 + a3 + a4 + a5 + a6 = 0 …(i)
2 4 3 5 6
Þ (a + a + a ) + (a + a + a ) = - 1 or a + b = - 1
Chap 01 Complex Numbers 29

and ab = (a + a 2 + a 4 ) (a 3 + a 5 + a 6 ) y Example 67. If n ³ 3 and 1, a 1 , a 2 , a 3 , ..., a n -1 are


4 6 7 5 7 8 7 9 10
= a + a + a +a + a + a + a + a + a the n, nth roots of unity, then find the value of
= a4 + a6 + 1 + a5 + 1 + a + 1 + a2 + a3 [Qa 7 = 1] S S ai a j .
= (1 + a + a 2 + a 3 + a 4 + a 5 + a 6 ) + 2 1 £ i < j £ n -1

=0+2 [from Eq. (i)] Sol. Let x = (1)1/n


=2
\ x n = 1 or x n - 1 = 0
Therefore, the required equation is
\ 1 + a 1 + a 2 + a 3 + ... + a n - 1 = 0
x 2 - (a + b ) x + ab = 0 or x 2 + x + 2 = 0
or a 1 + a 2 + a 3 + ... + a n - 1 = - 1
y Example 65. Find the value of On squaring both sides, we get
2 2 2
10 a 1 + a 2 + a 3 + ... + a 2n - 1 + 2 (a 1a 2 + a 1a 3
é æ 2pk ö æ 2pk ö ù
S ê
k =1 ë
sin ç ÷ - i cos ç
è 11 ø
÷ , where i = - 1.
è 11 ø úû + ... + a 1a n - 1 + a 2 a 3 + ... + a 2 a n - 1
10
+ ... + a n - 2 a n - 1 ) = 1
Sol. S ésin æçè 2pk ö÷ø - i cos æçè 2pk ö÷ø ùú
k =1 ê
or 12 + (a 1 )2 + (a 2 )2 + (a 3 )2 + ... + (a n - 1 )2
ë 11 11 û
10
+2 SS a i a j = 1 + 12
1£i < j £n -1
é æ 2p k ö æ 2p k ö ù
= -i S
k =1 ê cos çè 11 ÷ø + i sin çè 11 ÷ø ú 0+2 SS ai a j = 2
ë û
1£i < j £n -1
ìï 10 é æ 2p k ö æ 2p k ö ù üï [here, p is not a multiple of n]
= -iíS ê cos çè 11 ÷ø + i sin çè 11 ÷ø ú - 1ý
îï
k =0 ë û ïþ \ SS ai a j = 1
1£i < j £n -1
= - i ( 0 - 1) [sum of 11, 11th roots of unity] Aliter
=i
Q x n - 1 = ( x - 1) ( x - a 1 ) ( x - a 2 ) ... ( x - a n -1 )
y Example 66. If a 0 , a 1 , a 2 , ..., a n - 1 are the n, nth
n -1 On comparing the coefficient of x n - 2 both sides, we get
ai
roots of the unity, then find the value of S . 0= SS a i a j + a 1 + a 2 + ... + a n - 1
i =0 2 - a i 0 £i < j £n -1

Sol. Let x = (1)1/n Þ x n = 1 \ xn - 1 = 0 0= SS ai a j - 1


n 1£i < j £n -1
or x - 1 = ( x - a 0 ) ( x - a 1 ) ( x - a 2 ) ... ( x - a n - 1 ) [ Q1 + a 1 + a 2 + ... + a n - 1 = 0]
n -1
\ SS ai a j = 1
= P (x - ai )
i =0
1£i < j £n -1

On taking logarithm both sides, we get


n -1 Vector Representation of
loge ( x n - 1) = S loge ( x - a i )
i =0 Complex Numbers
On differentiating both sides w.r.t. x, we get If P is the point ( x , y ) on the argand plane corresponding
n -1
n -1
æ 1 ö to the complex number z = x + iy , where x , y Î R and
nx
= S ç ÷ i = - 1.
x -1n i =0 è x - ai ø
Y
On putting x = 2, we get P (x, y)
n -1
n (2)n - 1
= S 1
…(i)
2n - 1 i =0 (2 - a i ) y
n -1 n -1
θ
ai æ 2 ö
Now, S = S ç- 1 + ÷ O x M
X
i =0 (2 - a i ) i = 0 è 2 - ai ø
n -1 n -1 ¾® ¾®
2 × n × 2n - 1 OP = x $i + y $j Þ OP = ( x 2 + y 2 ) = z
=- S 1 + 2 i S= 0 1
= - (n ) + n [from Eq. (i)]
Then,
¾®
i =0 (2 - a i ) 2 -1
n
and arg (z ) = direction of the vector OP = tan - 1 (y / x ) = q
n ×2 n ¾®
= -n + n
= n Therefore, complex number z can also be represented by OP.
2 -1 2 -1
30 Textbook of Algebra

Geometrical Representation (c) Product


z 1 = r1 (cos q 1 + i sin q 1 ) = r1 e i q1
of Algebraic Operation on Let
\ z 1 = r1 and arg (z 1 ) = q 1
Complex Numbers and z 2 = r2 (cos q 2 + i sin q 2 ) = r2 e iq2
(a) Sum \ z 2 = r2 and arg (z 2 ) = q 2
Let the complex numbers z 1 = x 1 + iy 1 =( x 1 , y 1 ) and Then, z 1 z 2 = r1 r2 (cos q 1 + i sin q 1 ) (cos q 2 + i sin q 2 )
z 2 = x 2 + iy 2 = ( x 2 , y 2 ) be represented by the points P and = r1 r2 {cos (q 1 + q 2 ) + i sin (q 1 + q 2 )}
Q on the argand plane. \ z 1 z 2 = r1 r2 and arg (z 1 z 2 ) = q 1 + q 2
Y
Y R(z1z2)
R (z1 + z2)
Q (z2)

r1 r2
Q(z2)
P (z1)
X P(z1)
r2
O θ1 r1
θ2 θ1
Complete the parallelogram OPRQ. Then, the mid-points O A
X
of PQ and OR are the same. The mid-point of
æ x + x2 y1 + y2 ö Let P and Q represent the complex numbers z 1 and z 2 ,
PQ = ç 1 , ÷. respectively.
è 2 2 ø
\ OP = r1 , OQ = r2
Hence, R = ( x 1 + x 2 , y 1 + y 2 )
ÐPOX = q 1 and ÐQOX = q 2
Therefore, complex number z can also be represented by
¾® Take a point A on the real axis OX, such that OA = 1 unit.
OR = ( x 1 + x 2 ) + i (y 1 + y 2 ) = ( x 1 + iy 1 ) + ( x 2 + iy 2 ) Complete the ÐOPA
= z 1 + z 2 = (x 1 , y 1 ) + (x 2 , y 2 ) Now, taking OQ as the base, construct a DOQR similar to
In vector notation, we have OR OP
DOPA, so that =
¾® ¾® ¾® ¾® ¾® OQ OA
z 1 + z 2 = OP + OQ = OP + PR = OR
i.e. OR = OP × OQ = r1 r2 [since, OA = 1 unit]
and ÐROX = ÐROQ + ÐQOX = q 1 + q 2
(b) Difference
Hence, R is the point representing product of complex
We first represent - z 2 by Q ¢, so that QQ ¢ is bisected at O. numbers z 1 and z 2 .
Complete the parallelogram OPRQ ¢. Then, the point R
represents the difference z 1 - z 2 . Remark
1. Multiplication by i
Y p p
Since, z = r (cos q + i sin q) and i = æçcos + i sin ö÷
Q(z2) è 2 2ø
é æ p ö æ p ö ù
\ iz = r êcos ç + q÷ + i sin ç + q÷ ú
P(z1) ë è2 ø è2 øû
O
X′ X Hence, multiplication of z with i, then vector for z rotates a
right angle in the positive sense.
R(z1 – z2) 2. Thus, to multiply a vector by ( - 1) is to turn it through two
Q ′( –z2) right angles.
Y′ 3. Thus, to multiply a vector by (cos q + i sin q) is to turn it
through the angle q in the positive sense.
¾® ¾®
We see that ORPQ is a parallelogram, so that OR = QP
We have in vectorial notation, (d) Division
¾® ¾® ¾® ¾®
z 1 - z 2 = OP - OQ = OP + QO Let z 1 = r1 (cos q 1 + i sin q 1 ) = r1 e iq1
¾® ¾® ¾® ¾®
\ z 1 = r1 and arg (z 1 ) = q 1
= OP + PR = OR = QP and z 2 = r2 (cos q 2 + i sin q 2 ) = r2 e iq2
Chap 01 Complex Numbers 31

\ | z 2 | = r2 and arg (z 2 ) = q 2 ¾® ¾®
Then, we have AC = z 3 - z 1 and AB = z 2 - z 1
z r (cos q 1 + i sin q 1 )
Then, 1 = 1 × [z 2 ¹ 0, r2 ¹ 0 ] ¾®
z 2 r2 (cos q 2 + i sin q 2 ) and let arg AC = arg (z 3 - z 1 ) = q
z 1 r1 ¾®
= [cos (q 1 - q 2 ) + i sin(q 1 - q 2 )] and arg AB = arg (z 2 - z 1 ) = f
z 2 r2
z1 r æz ö Let ÐCAB = a
\ = 1 , arg ç 1 ÷ = q 1 - q 2 ¾® ¾®
z2 r2 èz2 ø ÐCAB = a = q - f = arg AC - arg AB
Let P and Q represent the complex numbers z 1 and z 2 , = arg (z 3 - z 1 ) - arg (z 2 - z 1 )
respectively.
\ OP = r1 , OQ = r2 , ÐPOX = q 1 and ÐQOX = q 2 æz - z1 ö
= arg ç 3 ÷
Let OS be new position of OP, take a point A on the real è z2 - z1 ø
axis OX, such that OA = 1 unit and through A draw a line or angle between AC and AB
making with OA an angle equal to the ÐOQP and meeting æ affix of C - affix of A ö
OS in R. = arg ç ÷
è affix of B - affix of A ø
Then, R represented by (z 1 /z 2 ).
Y Q(z2) For any complex number z, we have
i (arg z )
z= z e
r2 é æ z - z1 ö ù
i ê arg ç 3 ÷ú
æz - z1 ö z 3 - z1 ë è z 2 - z1 ø û
r1 P(z1) Similarly, ç 3 ÷= e
θ2 è z2 - z1 ø z2 - z1
θ1 A
θ1 – θ2 X
O
z 3 - z1 z 3 - z 1 i ( Ð CAB ) AC i a
or = e = e
R z2 - z1 z2 - z1 AB
S
Now, in similar DOPQ and DOAR. Remark
OR OP r 1. Here, only principal values of the arguments are considered.
= Þ OR = 1 æ z - z2 ö
OA OQ r2 2. arg ç 1 ÷ = q, if AB coincides with CD, then
è z3 - z4 ø
since OA = 1 and ÐAOR = ÐPOR - ÐPOX = q 2 - q 1 æ z - z2 ö z1 - z2
arg ç 1 ÷ = 0 or p, so that is real. It follows that
Hence, the vectorial angle of R is - (q 2 - q 1 ) i.e., q 1 - q 2 . è z3 - z4 ø z3 - z4
z - z2
if 1 is real, then the points A, B, C, D are collinear.
Remark z3 - z4
If q1 and q2 are the principal values of z1 and z2 , then q1 + q2 and D
q1 - q2 are not necessarily the principal value of arg ( z1z2 ) and
arg ( z1 / z2 ). P(z1)
S(z4) θ
A
Rotation Theorem (Coni Method)
B
R(z3)

Let z 1 , z 2 and z 3 be the affixes of three points A, B and C Q(z2)


respectively taken on argand plane. C

Y 3. If AB is perpendicular to CD, then


C(z3)
æ z - z2 ö p z - z2
B(z2)
arg ç 1 ÷ = ± , so 1 is purely imaginary.
α è z3 - z4 ø 2 z3 - z4

A(z1) 4. It follows that, if z1 - z2 = ± k ( z3 - z4 ), where k is purely


α imaginary number, then AB and CD are perpendicular to
each other.
φ θ
X
O
32 Textbook of Algebra

y Example 68. Complex numbers z 1 , z 2 and z 1 - z 2 a ip / 3


From Coni method, = e …(i)
z 3 - z2 a
z 3 are the vertices A, B, C respectively of an isosceles
p
right angled triangle with right angle at C. Show and ÐBAC =
3
that (z 1 - z 2 ) 2 = 2 (z 1 - z 3 ) (z 3 - z 2 ). z 3 - z 1 a ip / 3
From Coni method, = e …(ii)
Sol. Since, ÐACB = 90° and AC = BC , then by Coni method z 2 - z1 a
z1 - z 3 AC ip/ 2
= e =i A(z1)
z 2 - z 3 BC
B(z2)
π/3
π/4
a a

π/3 π/3
π/4 B(z2) a C(z3)
A(z1) C(z3)
z1 - z 2 z 3 - z1
Þ z1 - z 3 = i ( z 2 - z 3 ) From Eqs. (i) and (ii), we get =
z 3 - z 2 z 2 - z1
On squaring both sides, we get Þ (z1 - z 2 )(z 2 - z1 ) = (z 3 - z1 )(z 3 - z 2 )
( z 1 - z 3 ) 2 = -( z 2 - z 3 ) 2
2 2 2 2
Þ z 12 + z 22 + z 23 = z1z 2 + z 2 z 3 + z 3z1
Þ z1 + z 3 - 2z1z 3 = -(z 2 + z 3 - 2z 2z 3 )
2 2
Þ z1 + z 2 - 2z1z 2 = 2(z1z 3 - z1z 2 - z 3 + z 2z 3 )
2 Remark
Triangle with vertices z1, z2, z3, then
Therefore, (z1 - z 2 )2 = 2(z1 - z 3 ) (z 3 - z 2 )
(i) ( z1 - z2 ) 2 + ( z2 - z3 ) 2 + ( z3 - z1 ) 2 = 0
1
Aliter CA = CB = BA (ii) ( z1 - z2 ) 2 = ( z2 - z3 )( z3 - z1 )
2
1
B(z2) (iii) å( z1 - z2 )( z2 - z3 ) = 0 (iv) å =0
( z1 - z2 )
π/4

Complex Number as a Rotating Arrow


π/4 in the Argand Plane
A(z1) C(z3) Let z = r (cos q + i sin q ) = re iq …(i)
Q ÐBAC = ( p / 4 ) be a complex number representing a point P in the argand
z 2 - z1 BA (i p / 4 ) plane.
\ = e Y Q(ze if)
z 3 - z1 CA
z1 - z2 (i p/ 4 ) r
or = 2e …(i) P(z)
z1 - z 3 φ
r
θ
X′ X
and Ð CBA = (p / 4) O
z 3 - z 2 CB (i p/ 4) z - z2 1 (i p/ 4)
\ = e or 3 = e …(ii)
z 1 - z 2 AB z1 - z2 2 Y′
On dividing Eq. (i) by Eq. (ii), we get Then, OP = z = r and ÐPOX = q
( z1 - z 2 ) 2 = 2 ( z1 - z 3 ) ( z 3 - z 2 ) Now, consider complex number z 1 = ze if
or z 1 = re iq × e if = re i ( q + f ) [from Eq. (i)]
y Example 69. Complex numbers z 1 , z 2 , z 3 are the
vertices of A, B, C respectively of an equilateral Clearly, the complex number z 1 represents a point Q in the
triangle. Show that argand plane, when OQ = r and ÐQOX = q + f
¾®
z 12 + z 22 + z 32 = z 1z 2 + z 2z 3 + z 3z 1 . Clearly, multiplication of z with e if rotates the vector OP
Sol. Let AB = BC = CA = a through angle f in anti-clockwise sense. Similarly,
¾®
p multiplication of z with e - if will rotate the vector OP in
Q ÐABC =
3 clockwise sense.
Chap 01 Complex Numbers 33

Remark
1. If z1, z2 and z3 are the affixes of the C(z3)
Shifting the Origin in Case
three points A, B and C, such that
AC = AB and ÐCAB = q. Therefore,
of Complex Numbers
¾® ¾®
AB = z2 - z1 , AC = z3 - z1.
B(z2) Let O be the origin and P be a point with affix z 0 . Let a
θ
¾® point Q has affix z with respect to the coordinate system
Then, AC will be obtained by rotating A(z1) passing through O. When origin is shifted to the point P
¾®
AB through an angle q in anti- (z 0 ), then the new affix Z of the point Q with respect to
clockwise sense and therefore, new origin P is given by Z = z - z 0 .
¾® ¾®
AC = AB e i q i.e., to shift the origin at z 0 , we should replace z by Z + z 0 .
z 3 - z1
or ( z 3 - z1 ) = ( z2 - z1 ) e iq or = e iq y
z2 - z1 Y
Q
2. If A, B and C are three points in argand plane, such that
AC = AB and ÐCAB = q, then use the rotation about Ato find
e iq , but if AC ¹ AB, then use Coni method. X
P(z0)

y Example 70. Let z 1 and z 2 be roots of the equation


z 2 + pz + q = 0, where the coefficients p and q may be O M
x

complex numbers. Let A and B represent z 1 and z 2 in


the complex plane. If Ð AOB = a ¹ 0 and OA = OB,
where O is the origin, prove that p 2 = 4 q cos 2 (a / 2). y Example 71. If z 1 , z 2 and z 3 are the vertices of an
¾® ¾®
equilateral triangle with z 0 as its circumcentre, then
Sol. Clearly, OB is obtained by rotating OA through angle a.
¾® ¾® changing origin to z 0 , show that Z 1 2+ Z 2 2 + Z 3 2 = 0,
\ OB = OA e i a
where Z 1 , Z 2 , Z 3 are new complex numbers of the
Þ z 2 = z1 e i a
vertices.
z2
Þ =eia …(i) Sol. In an equilateral triangle, the circumcentre and the
z1
centroid are the same point.
B(z2)
z1 + z 2 + z 3
So, z0 =
3
A(z1) \ z1 + z 2 + z 3 = 3z 0 …(i)
α To shift the origin at z 0 , we have to replace z1, z 2 , z 3 and z 0
O by Z 1 + z 0 , Z 2 + z 0 , Z 3 + z 0 and 0 + z 0 .
z2 Then, Eq. (i) becomes
or + 1 = ( e i a + 1)
z1 ( Z 1 + z 0 ) + ( Z 2 + z 0 ) + ( Z 3 + z 0 ) = 3( 0 + z 0 )
( z1 + z 2 ) Þ Z1 + Z 2 + Z 3 = 0
Þ = e i a / 2 × 2 cos (a / 2)
z1 On squaring, we get
2 2 2
On squaring both sides, we get Z 1 + Z 2 + Z 3 + 2 (Z 1Z 2 + Z 2Z 3 + Z 3Z 1 ) = 0 …(ii)
( z1 + z 2 ) 2 But triangle with vertices Z 1 , Z 2 and Z 3 is equilateral, then
= e i a × ( 4 cos 2 a / 2) 2 2 2
z1
2 Z 1 + Z 2 + Z 3 = Z 1Z 2 + Z 2 Z 3 + Z 3Z 1 …(iii)
( z1 + z 2 ) 2
z2 From Eqs. (ii) and (iii), we get
Þ = × ( 4 cos 2 a / 2) [from Eq. (i)] 2 2 2
z12 z1 3 (Z 1 + Z 2 + Z 3 ) = 0
2 2 2
(z1 + z 2 )2 = 4 z1z 2 cos 2 (a / 2) Therefore, Z1 + Z 2 + Z 3 = 0
( - p )2 = 4 q cos 2 (a / 2)
éQz1 and z 2 are the roots of z 2 + pz + q = 0ù
ê ú
Inverse Points
ë\z1 + z 2 = - p and z1 z 2 = q û (a) Inverse points with respect to a line Two points P
or p 2 = 4 q cos 2 (a / 2) and Q are said to be the inverse points with respect to
the line RS. If Q is the image of P in RS, i.e., if the line
RS is the right bisector of PQ.
34 Textbook of Algebra

y Example 72. Show that z 1 , z 2 are the inverse points y Example 73. Show that inverse of a point a with
with respect to the line z a + a z = b , if z 1 a + a z 2 = b . respect to the circle z - c = R (a and c are complex
R2
Sol. Let RS be the line represented by the equation, numbers, centre c and radius R) is the point c + .
z a +az =b …(i) a -c
Let P and Q are the inverse points with respect to the line RS. Sol. Let a¢ be the inverse point of a with respect to the circle
The point Q is the reflection (inverse) of the point P in the z - c = R, then by definition,
line RS, if the line RS is the right bisector of PQ. Take any
point z in the line RS, then lines joining z to P and z to Q are
equal. c a′
P a |z – c | = R

The points c , a, a ¢ are collinear.


R S We have, arg (a ¢ - c ) = arg(a - c )
-
= - arg ( a - c ) [Q arg z = - arg z ]
Þ arg (a ¢ - c ) + arg ( a - c ) = 0
Q Þ arg {(a ¢ - c ) ( a - c )} = 0
2 2 \ (a ¢ - c ) ( a - c ) is purely real and positive.
i.e., z - z1 = z - z 2 or z - z1 = z - z2
By definition, a ¢ - c a - c = R 2 [QCP × CQ = r 2 ]
i.e., ( z - z1 ) ( z - z1 ) = ( z - z 2 ) ( z - z 2 )
Þ z ( z 2 - z1 ) + z ( z 2 - z1 ) + ( z1z1 - z 2 z 2 ) = 0 …(ii) Þ a¢ - c a - c = R2 [Q | z | = | z | ]
Hence, Eqs. (i) and (ii) are identical, therefore, comparing Þ (a ¢ - c ) ( a - c ) = R 2
coefficients, we get
Þ (a ¢ - c ) ( a - c ) = R 2
P z1
[Q(a ¢ - c )( a - c ) is purely real and positive]
R2 R2
Þ a¢ - c = Þ a¢ = c +
a -c a -c
R S
A(z)

Dot and Cross Product


Q z2 Let z 1 = x 1 + iy 1 º ( x 1 , y 1 ) and z 2 = x 2 + iy 2 º ( x 2 , y 2 ),
a a -b where x 1 , y 1 , x 2 , y 2 Î R and i = - 1, be two complex
= =
z 2 - z1 z 2 - z1 z1z1 - z 2 z 2 numbers.
z1a az 2
If ÐPOQ = q, then from Coni method,
So that, =
z1( z 2 - z1 ) z 2 (z 2 - z1 ) z2 - 0 z 2 iq
= e Q(z2)
-b z a + az 2 - b z1 - 0 z1
= = 1
z1z1 - z 2 z 2 0 z 2 iq
z2z1
[by ratio and proportion rule] Þ = e θ
P(z1)
z1z1 z1
z1a + az 2 - b = 0 or z1 a + az 2 = b
O
(b) Inverse points with respect to a circle If C is the z2z1 z2
Þ = e iq
centre of the circle and P , Q are the inverse points z1
2 z1
with respect to the circle, then three points C , P , Q are
collinear and also CP × CQ = r 2 , where r is the z 2 z 1 = | z 1 || z 2 | e iq
radius of the circle. z 2 z 1 = z 1 z 2 (cos q + i sin q )
Þ Re (z 2 z 1 ) = z 1 z 2 cos q …(i)
and Im (z 2 z 1 ) = z 1 z 2 sin q …(ii)
Q The dot product z 1 and z 2 is defined by,
C P
z 1 ×z 2 = z 1 z 2 cos q
= Re (z 1 z 2 ) = x 1 x 2 + y 1 y 2 [from Eq. (i)]
Chap 01 Complex Numbers 35

and cross product of z 1 and z 2 is defined by Remark


z 1 ´ z 2 = z 1 z 2 sin q 1. The distance of a point z from origin, z - 0 = z
2. Three points A( z1 ), B ( z2 ) and C ( z3 ) are collinear, then
= Im (z 1 z 2 ) = x 1 y 2 - x 2 y 1 [from Eq. (ii)] AB + BC = AC
Hence, z 1 ×z 2 = x 1 x 2 + y 1 y 2 = Re (z 1 z 2 ) C(z3)
and z 1 ´ z 2 = x 1 y 2 - x 2 y 1 = Im (z 1 z 2 )

Results for Dot and Cross B(z2)

Products of Complex Number A(z1)


1. If z1 and z2 are perpendicular, then z1 × z2 = 0 i.e. z1 - z2 + z2 - z3 = z1 - z3 .
2. If z1 and z2 are parallel, then z1 ´ z2 = 0
3. Projection of z1 on z2 = ( z1× z2 ) / z2 y Example 75. Show that the points representing the
4. Projection of z2 on z1 = ( z1× z2 ) / z1 complex numbers ( 3 + 2i ), (2 - i ) and - 7i, where
5. Area of triangle, if two sides represented by z 1 and z2 is
1
i = - 1, are collinear.
z1 ´ z2 . Sol. Let z 1 = 3 + 2i , z 2 = 2 - i and z 3 = - 7i .
2
6. Area of a parallelogram having sides z 1 and z2 is z1 ´ z2 . Then, z 1 - z 2 = 1 + 3i = 10, z 2 - z 3 = 2 + 6i
7. Area of parallelogram, if diagonals represented by z1 and z2 is
1 = 40 = 2 10
z1 ´ z2 .
2 and z1 - z 3 = 3 + 9i = 90 = 3 10
y Example 74. If z 1 = 2 + 5i , z 2 = 3 - i , where i = -1, find \ z1 - z 2 + z 2 - z 3 = z1 - z 3
(i) z 1 × z 2 (ii) z 1 ´ z 2 Hence, the points (3 + 2i ), (2 - i ) and - 7i are collinear.
(iii) z 2 × z 1 (iv) z 2 ´ z 1
(v) acute angle between z 1 and z 2 . (b) Equation of the
(vi) projection of z 1 on z 2 . Perpendicular Bisector
Sol. (i) z1×z 2 = x 1x 2 + y1y 2 = (2) (3) + (5) ( - 1) = 1
If P (z 1 ) and Q (z 2 ) are two fixed points and R (z ) is
(ii) z1 ´ z 2 = x 1y 2 - x 2 y1 = (2) ( - 1) - (3) (5) = - 17
moving point, such that it is always at equal distance from
(iii) z 2 ×z1 = x 1 x 2 + y1y 2 = (2) (3) + (5) ( - 1) = 1 P (z 1 ) and Q (z 2 ).
(iv) z 2 ´ z1 = x 2 y1 - x 1y 2 = (3) (5) - (2) ( - 1) = 17 P(z1)

(v) Let angle between z1 and z 2 be q, then


R (z)
z1 × z 2 = z1 z 2 cosq
Þ 1 = ( 4 + 25) (9 + 1) cosq Q(z2)

1 æ 1 ö i.e. PR = QR
\ cosq = \ q = cos - 1 ç ÷
290 è 290 ø or z - z1 = z - z2
z ×z 1 1
(vi) Projection of z1 on z 2 = 1 2 = = or z (z 1 - z 2 ) + z (z 1 - z 2 ) = z 1 z 1 - z 2 z 2
z2 ( 9 + 1) 10 2 2
or z (z 1 - z 2 ) + z (z 1 - z 2 ) = z 1 - z2

Use of Complex Numbers in Hence, z lies on the perpendicular bisectors of z 1 and z 2 .

Coordinate Geometry y Example 76. Find the perpendicular bisector of 3 + 4i


and - 5 + 6i, where i = - 1.
(a) Distance Formula Sol. Let z1 = 3 + 4i and z 2 = - 5 + 6i
The distance between two points P (z 1 ) and Q (z 2 ) is given by If z is moving point, such that it is always equal distance
Q(z2) from z1 and z 2 .
i.e. z - z1 = z - z 2
2
or z ( z1 - z 2 ) + z ( z1 - z 2 ) = z1 2 - z 2
Þ z (( 3 - 4i ) - ( - 5 - 6i )) + z (( 3 + 4i ) - ( - 5 + 6i )) = 25 - 61
P(z1) Hence, ( 8 + 2i ) z + ( 8 - 2i ) z + 36 = 0
PQ = z 2 - z 1 = affix of Q - affix of P which is required perpendicular bisector.
36 Textbook of Algebra

z1 + z 2 + z 3 2 ×0 + 1 × z
(c) Section Formula Þ
3
=
2+1
If R (z ) divides the joining of P (z 1 ) and Q (z 2 ) in the ratio Þ z1 + z 2 + z 3 = z
m 1 : m 2 (m 1 , m 2 > 0 ). Therefore, z1 + z 2 + z 3 - z = 0
Q(z2)
(m2) y Example 78. Let z 1 , z 2 and z 3 be three complex
(m 1 ) numbers and a, b , c ÎR, such that a + b + c = 0 and
R(z)
az 1 + bz 2 + cz 3 = 0, then show that z 1 , z 2 and z 3 are
P(z1) collinear.
Sol. Given, a+b+c =0 …(i)
(i) If R (z ) divides the segment PQ internally in the ratio and az1 + bz 2 + cz 3 = 0 …(ii)
m z + m2 z 1 Þ az1 + bz 2 - (a + b ) z 3 = 0 [from Eq. (i)]
of m 1 : m 2 , then z = 1 2
m1 + m2 az + bz 2
or z3 = 1
(ii) If R (z ) divides the segment PQ externally in the ratio a+b
m z - m2 z 1 It follows that z 3 divides the line segment joining z1 and z 2
of m 1 : m 2 , then z = 1 2 internally in the ratio b : a. (If a, b are of same sign and
m1 - m2
opposite sign, then externally.)
Hence, z1,z 2 and z 3 are collinear.
m2 R(z)
m1
(d) Area of Triangle
Q(z2)
If z 1 , z 2 and z 3 are the affixes of the vertices of a triangle,
P(z1) z1 z1 1
1
Remark then its area = | z 2 z 2 1 |
4
z1 + z2 z3 z3 1
1. If R ( z ) is the mid-point of PQ, then affix of R is .
2 A(z1)
2. If z1 , z2 and z3 are affixes of the vertices of a triangle, then
z + z2 + z 3
affix of its centroid is 1 .
3
3. In acute angle triangle, orthocentre ( O), nine point centre ( N ),
OG 2
centroid ( G) and circumcentre ( C) are collinear and = ,
GC 1
ON 1 B(z2) C(z3)
= .
NG 1
4. If z1, z2, z 3 and z4 are the affixes of the vertices of a Remark
parallelogram taken in order, then z1 + z 3 = z2 + z4 . The area of the triangle with vertices z, wz and z + w z is
3 2
z ,
4
y Example 77. If z 1 , z 2 and z 3 are the affixes of the where w is the cube root of unity.
vertices of a triangle having its circumcentre at the y Example 79. Show that the area of the triangle on
origin. If z is the affix of its orthocentre, prove that
the argand plane formed by the complex numbers z, iz
z 1 + z 2 + z 3 - z = 0. 1 2
Sol. We know that orthocentre O, centroid G and circumcentre and z + iz is z , where i = - 1.
2
C of a triangle are collinear, such that G divides OC in the
z1 + z 2 + z 3 z z 1
ratio 2 : 1. Since, affix of G is and C is the 1
3 Sol. Required area = | iz iz 1|
4
origin. Therefore, by section formula, we get z + iz z + iz 1
C
1 z z 1
1
2 G = | iz iz 1|
4
z + iz z + iz 1
O
Chap 01 Complex Numbers 37

z z 1 Proof Equation of the straight line joining points having


1
= | iz -iz 1 | affixes z 1 and z 2 is
4
z + iz z - iz 1 z = tz 1 + (1 - t ) z 2 , where t Î R ~ {0 }
On applying R 3 ® R 3 - ( R 1 + R 2 ), we get Þ z - z 2 = t (z 1 - z 2 ) …(i)
z z 1 and z - z 2 = t (z 1 - z 2 )
1 1
Area = | iz -iz 1 | = ( - 1) ( - izz - izz ) or z - z 2 = t (z 1 - z 2 ) …(ii)
4 4
0 0 -1
From Eqs. (i) and (ii), we get
=
1
2izz =
1
zz =
1 2 z - z2 z1 - z2 z - z2 z - z2
4 2
i
2
z = Þ =
z - z2 z1 - z2 z1 - z2 z1 - z2
Aliter
z - z2 z - z2 0
We have, iz = z (cos( p / 2) + i sin( p / 2)) = ze (ip / 2 ) iz is the
vector obtained by rotating vector z in anti-clockwise Þ z1 - z2 z1 - z2 0 =0
direction through ( p / 2). Therefore, OA ^ AB, z2 z2 1
Imaginary axis
Y Now, applying R 1 ® R 1 + R 3 and R 2 ® R 2 + R 3 , we get
B z z 1
z1 z1 1 = 0
iz z2 z2 1
z + iz
π/2
A
or z (z 1 - z 2 ) - z (z 1 - z 2 ) + z 1 z 2 - z 1 z 2 = 0
z Aliter
X Real axis
O
Let P (z ) be an arbitrary point on the line, which pass
through A (z 1 ) and B (z 2 ).
1 1
Now, area of DOAB = OA ´ AB = z iz \ Ð BAP = 0 or p
2 2
1 1 æ z - z1 ö
= z i z = z
2
\ arg ç ÷ = 0 or p [by rotation theorem]
2 2 èz2 - z1 ø
z - z1
Þ is purely real.
(e) Equation of a Straight Line z2 - z1
æ z - z1 ö æ z - z1 ö z - z1 z - z1
(i) Parametric form \ ç ÷ =ç ÷ Þ =
èz2 - z1 ø èz2 - z1 ø z2 - z1 z2 - z1
Equation of the straight line joining the points having
affixes z 1 and z 2 is A(z1)
z = tz 1 + (1 - t ) z 2 , where t Î R ~ {0 } B(z2)
Proof P(z)
tz + (1 - t ) z 2
Q z = tz 1 + (1 - t ) z 2 = 1
t + (1 - t )
z (z 1 - z 2 ) - z (z 1 - z 2 ) + z 1 z 2 - z 1 z 2 = 0
Hence, z divides the line joining z 1 and z 2 in the ratio
z z 1
1 - t : t . Thus, the points z 1 , z 2 , z are collinear.
or z1 z1 1 = 0
(ii) Non-parametric form z2 z2 1
Equation of the straight line joining the points having
affixes z 1 and z 2 is Remark
z z 1 z1 z1 1
z1 z1 1 = 0 If z1, z 2 and z 3 are collinear, z2 z2 1 =0
z3 z3 1
z2 z2 1
or z (z 1 - z 2 ) - z (z 1 - z 2 ) + z 1 z 2 - z 1 z 2 = 0
or å z1( z2 - z3 ) = 0.
38 Textbook of Algebra

(iii) General form The general equation of a straight (i) If the lines are perpendicular, then
line is of the form a z + a z + b = 0, where a is a (z3)
complex number and b is a real number.
Sol. The equation of a straight line passing through points
having affixes z1 and z 2 is given by (z1) (z2)
z (z1 - z 2 ) - z (z1 - z 2 ) + z1z 2 - z1z 2 = 0 …(i)
On multiplying Eq. (i) by i (where, i = -1), we get
(z4)
zi ( z1 - z 2 ) - z i ( z1 - z 2 ) + i ( z1z 2 - z1z 2 ) = 0
Þ z { - i ( z1 - z 2 )} + z {i ( z1 - z 2 )} + i ( z1z 2 - z1z 2 ) = 0
(z 1 - z 2 ) (z 3 - z 4 ) ip / 2
Þ z { - i ( z1 - z 2 )} + z { - i ( z1 - z 2 )} + {i ( 2i Im (z1z 2 ))} = 0 = e
z1 - z2 z3 -z4
Þ z { - i ( z1 - z 2 )} + z { - i ( z1 - z 2 )} + { ( - 2 Im (z1z 2 )} = 0
(z 1 - z 2 ) 2 (z 3 - z 4 ) 2
Þ z a + z a + b = 0, Þ = e ip
2 2
where, a = - i (z1 - z 2 ), b = - 2 Im(z1z 2 ) z1 - z2 z3 -z4
Hence, the general equation of a straight line is of the form
(z 1 - z 2 ) 2 (z 3 - z 4 ) 2
a z + a z + b = 0, Þ = e ip
where a is complex number and b is a real number. (z 1 - z 2 ) (z 1 - z 2 ) (z 3 - z 4 ) (z 3 - z 4 )
(iv) Slope of the line a z + a z + b = 0 (z 1 - z 2 ) (z 3 - z 4 )
Þ = ( - 1)
Let A (z 1 ) and B (z 2 ) be two points on the line (z 1 - z 2 ) (z 3 - z 4 )
a z + a z + b = 0, then Þ a1 = - a2 [from Eq. (i)]
a z1 + a z1 + b = 0 \ a1 + a2 = 0
and a z2 + a z2 + b = 0 (ii) If the lines are parallel, then
\ a (z 1 - z 2 ) + a (z 1 - z 2 ) = 0 z1 - z2 z -z4 0
= 3 e
z1 - z2 a z1 - z2 z3 -z4
Þ =- [Remember]
z1 - z2 a
(z 1 - z 2 ) 2 (z 3 - z 4 ) 2
a coefficient of z Þ =
Complex slope of AB = - =- z1 - z2
2
z3 -z4
2
a coefficient of z
Thus, the complex slope of the line a z + a z + b = 0 is (z 1 - z 2 ) 2 (z 3 - z 4 ) 2
Þ =
a (z 1 - z 2 ) (z 1 - z 2 ) (z 3 - z 4 ) (z 3 - z 4 )
- .
a (z 1 - z 2 ) (z 3 - z 4 )
Þ =
(z 1 - z 2 ) (z 3 - z 4 )
Remark
The real slope of the line a z + az + b = 0 is Þ a1 = a2
Re ( a) Re (coefficient of z )
- , i.e. - .
Im ( a) Im (coefficient of z ) Remark
1. The equation of a line parallel to the line a z + az + b = 0 is
Important Theorem a z + az + l = 0, where l ÎR.
2. The equation of a line perpendicular to the line
If a 1 and a 2 are the complex slopes of two lines on the a z + a z + b = 0 is a z - a z + i l = 0
argand plane, then prove that the lines are where, l ÎR and i = - 1
(i) perpendicular, if a 1 + a 2 = 0.
.

(v) Length of perpendicular from a given point on a


(ii) parallel, if a 1 = a 2 . given line
Proof Let z 1 and z 2 be the affixes of two points on one The length of perpendicular from a point P (z 1 ) to the
line with complex slope a 1 and z 3 and z 4 be the affixes of line
two points another line with complex slope a 2 . Then,
az 1 + a z 1 + b
z - z2 z 3 -z4 a z + a z + b = 0 is given by .
a1 = 1 and a 2 = …(i) 2 a
z1 - z2 z 3 -z4
Chap 01 Complex Numbers 39

Proof Let PM be perpendicular from P on the line a - a¢ æ b ö


Þ + ç- ÷ = 0
a z + a z + b = 0 and let the affix of M be z 2 , then a - a¢ è b ø
P(z1) Þ b (a - a ¢ ) - b ( a - a ¢ ) = 0 ...(ii)
On subtracting Eq. (ii) from Eq. (i), we get
a ¢ b + ab + c = 0
Aliter
M(z2) Equation of perpendicular bisector of PQ is
PM = z 1 - z 2 z ( a ¢ - a ) + z (a ¢ - a ) - a ¢ a ¢ + aa = 0 …(i)
and given line zb + z b + c = 0 …(ii)
a z +a z +b =0
Since, Eqs. (i) and (ii) are identical, we have
and M(z 2 ) lies on a z + a z + b = 0, then
a ¢ - a a ¢ - a aa - a ¢ a ¢
a z2 + a z2 + b = 0 …(i) = = =k [say]
b b c
Since, PM perpendicular to the line (a z + a z + b = 0 ). Q a ¢ - a = b k , a ¢ - a = bk
z1 - z2 æ a ö aa - a ¢ a ¢ = ck
Therefore, + ç- ÷ = 0 and
z1 - z2 è a ø ì æa¢ - a ö æ a ¢ - a öü
Now, a ¢ b + ab = ía ¢ ç ÷+a ç ÷ý
Þ a z1 - a z2 - a z1 + a z2 = 0 î è k ø è k øþ
Þ a z1 + a z1 + b = 2 a z1 + a z2 - a z2 + b 1
= {a ¢ a ¢ - a a } =
1
( - ck ) = - c
= 2 a z 1 - az 2 + (az 2 + b ) k k
Hence, a ¢b + ab + c = 0
= 2 a z1 - a z2 - a z2 [Qa z 2 + b = - a z 2 ]
= 2 a (z 1 - z 2 )
or a z1 + a z1 + b = 2 a z1 - z2 (f) Circle
= 2 a z1 - z2 [ Q|z | = |z | ] The equation of a circle whose centre is at point affix z 0
and radius r, is z - z 0 = r .
= 2 a PM
a z1 + a z1 + b P(z)
\ PM = r
2 a
C(z0)
y Example 80. Show that the point a ¢ is the reflection
of the point a in the line z b + z b + c = 0, if
a ¢b + ab + c = 0. Remark
Sol. Since, a¢ is the reflection of point a through the line. 1. If the centre of the circle is at origin and radius r, then its
So, the mid-point of PQ equation is z = r .
P a 2. z - z0 < r represents interior of a circle z - z0 = r and
z - z0 > r represent exterior of the circle z - z0 = r.
3. r < z - z0 < R, this region is known as annulus.
B A(z)
(i) General Equation of a Circle
Q a' The general equation of the circle is
a + a¢ z z + a z + a z + b = 0,
i.e., lies on z b + z b + c = 0
2 where a is a complex number and b Î R, having centre at ( -a )
æ a + a¢ ö æ a + a¢ ö 2
or bç ÷ +bç ÷ +c =0 and radius = a - b.
è 2 ø è 2 ø
Þ b (a + a ¢ ) + b (a + a ¢ ) + 2c = 0 …(i) Proof The equation of circle having centre at z 0 and
Since, PQ ^ AB. Therefore, radius r is
Complex slope of PQ + Complex slope of AB = 0 z -z0 =r
40 Textbook of Algebra

2 From Eqs. (i) and (ii), we get


Þ z -z0 =r2
(z - z 1 ) (z 2 - z 3 )
Þ (z - z 0 ) (z - z 0 ) = r 2 = Real …(iii)
(z - z 2 ) (z 1 - z 3 )
Þ zz - zz 0 - z 0 z + z 0 z 0 = r 2
2 Remark
Þ zz + ( - z 0 ) z + ( - z 0 ) z + z 0 -r2 =0 ( z4 - z1 ) ( z2 - z3 )
If four points z1, z2, z 3, z4 are concyclic, then =
Þ zz + az + az + b = 0 ( z4 - z2 ) ( z1 - z3 )
2 real [replacing z by z4 in Eq. (iii)]
where, a = - z 0 and b = z 0 -r2
é ( z - z 3 ) ( z4 - z1 ) ù
or arg ê 2 ú = p, 0.
Þ zz + a z + a z + b = 0 êë ( z1 - z 3 ) ( z4 - z2 ) úû
where, b Î R represents a circle having centre at ( - a ) and
radius = z0
2
- b = | a |2 - b . (iii) Equation of Circle in Diametric Form
If end points of diameter represented by A (z 1 ) and B (z 2 )
Remark and P (z ) is any point on circle.
Rule to find the centre and radius of a circle whose equation is \ Ð APB = 90°
given
1. Make the coefficient of z z equal to 1 and right hand side
\ Complex slope of PA + Complex slope of PB = 0
equal to zero. P(z)
2. The centre of circle will be = ( - a) = ( - coefficient of z ).
90°
3. Radius = (|a|2 - constant term) B(z2)
ter
Di ame
y Example 81. Find the centre and radius of the circle A(z1)
2 z z + ( 3 - i ) z + ( 3 + i ) z - 7 = 0, where i = -1.
Sol. The given equation can be written as æz - z1 ö æz - z2 ö
æ3 + i ö Þ ç ÷ +ç ÷ =0
æ3 + i ö 7 èz - z1 ø èz - z2 ø
zz + ç ÷z +ç ÷z - =0
è 2 ø è 2 ø 2
Hence, ( z - z 1 ) ( z - z 2 ) + ( z - z 2 ) ( z - z 1 ) = 0
æ3 + i ö
So, it represent a circle with centre at - ç ÷ and radius which is required equation of circle in diametric form.
è 2 ø
æ æ3 + i ö 2
7ö æ9 1 7ö (iv) Other Forms of Circle
= ç -ç + ÷ = ç + + ÷= 6
ç è 2 ÷ø ÷
2ø è 4 4 2ø (a) Equation of all circles which are orthogonal to
è
z - z 1 = r1 and z - z 2 = r2 .
(ii) Equation of Circle Through Three Let the circle be z - a = r cut given circles
Non-Collinear Points orthogonally.
2 2
Let A (z 1 ), B (z 2 ), C (z 3 ) be three points on the circle and \ r 2 + r1 = a - z 1 …(i)
P (z ) be any point on the circle, then 2 2
and r 2 + r2 = a - z 2 …(ii)
P(z)
On solving,
2 2 2 2
θ
C(z3)
r2 - r1 = a (z 1 - z 2 ) + a (z 1 - z 2 ) + z 2 - z1
θ and let a = a + ib , i = - 1, a, b Î R
A(z1)
z - z1
B(z2) (b) Apollonius circle =k ¹1
z - z2
Ð ACB = Ð APB It is the circle with join of z 3 and z 4 as a diameter,
Using Coni method, z + kz 2 z - kz 2
where z 3 = 1 , z4 = 1
z 2 - z 3 BC iq 1+k 1-k
in DACB, = e …(i)
z 1 - z 3 CA for k = 1, the circle reduces to the straight line which
z 2 - z BP iq is perpendicular bisector of the line segment from z 1
in DAPB, = e …(ii) to z 2 .
z 1 - z AP
Chap 01 Complex Numbers 41

æz - z1 ö z + z + 2a
(c) Circular arc arg ç ÷ =a z -a =
èz - z2 ø 2
This is an arc of a circle in which the chord joining z 1 1 2
or z z - 4a (z + z ) = {z + (z ) 2 }
and z 2 subtends angle a at any point on the arc. 2
p where, a Î R (focus), directrix is z + z + 2a = 0.
If a = ± , then locus of zis a circle with the join of
2
z 1 and z 2 as diameter. If a = 0 or p, then locus is a (h) Equation of Ellipse
straight line through the points z 1 and z 2 .
2 2 For ellipse
(d) The equation z - z 1 + z - z2 = k, will represent a Imaginary axis
1 2 P(z)
circle, if k ³ z 1 - z 2 .
2
y Example 82. Find all circles which are orthogonal
Real axis
to z = 1 and z - 1 = 4 . S′(z2) C S(z1)

Sol. Let z -a = k …(i)


(where, a = a + ib and a, b, k Î R and i = - 1) be a circle
which cuts the circles
SP + S ¢ P = 2a
z =1 …(ii)
Þ z - z1 + z - z 2 = 2a
and z -1 = 4 …(iii)
Orthogonally, then using the property that the sum of where, 2a > z 1 - z 2 [since, eccentricity < 1]
squares of their radii is equal to square of distance between Then, point z describes an ellipse having foci at z 1 and z 2
centres. Thus, the circle (i) will cut the circles (ii) and (iii) and a Î R + .
orthogonally, if
k 2 + 1 = a -0
2
= aa (i) Equation of Hyperbola
2 2
and k + 16 = a - 1 = ( a - 1) ( a - 1) For hyperbola
= aa - ( a + a ) + 1 Imaginary axis

\ 1 - ( a + a ) - 15 = 0 Þ a + a = -14
\ 2a = - 14 Þ a = - 7 P(z)
Þ a = a + ib = - 7 + ib
2 Real axis
Also, k2 = a - 1 = ( - 7 )2 + b 2 - 1 = b 2 + 48 S′(z2) C S(z1)

Þ k = (b 2 + 48)
Therefore, required family of circles is given by
z + 7 - ib = ( 48 + b 2 ).
SP - S ¢ P = 2a Þ z - z 1 - z - z 2 = 2a
where, 2a < z 1 - z 2 [since, eccentricity > 1]
(g) Equation of Parabola
Then, point z describes a hyperbola having foci at z 1 and
Now, for parabola z 2 and a Î R + .
Imaginary axis

M
P(z)
Examples on Geometry
y Example 83. Let z 1 = 10 + 6i , z 2 = 4 + 6i , where
Real axis
A (O) S (a + i . 0)
z + z + 2a = 0

N
i = - 1. If z is a complex number, such that the
argument of (z - z 1 ) / (z - z 2 ) is p / 4, then prove that
z - 7 - 9i = 3 2.
æ z - z1 ö p
Sol. Q arg ç ÷=
SP = PM èz - z2 ø 4
42 Textbook of Algebra

It is clear that z , z1, z 2 are non-collinear points. Always a y Example 85. In the argand plane, the vector
circle passes through z , z1 and z 2 . Let z 0 be the centre of the
z = 4 - 3i , where i = - 1, is turned in the clockwise
circle.
A(z) sense through 180° and stretched three times. Then,
find the complex number represented by the new
π/4 vector.
O(z0) Sol. Q z = 4 - 3i Þ z = ( 4 ) 2 + ( - 3) 2 = 5
r r
π/2 Let z1 be the new vector obtained by rotating z in the
(z2)B C(z1) clockwise sense through 180°, therefore
z1 = z e - ip = - z = - ( 4 - 3i ) = - 4 + 3i .
4 3
On applying rotation theorem in DBOC, The unit vector in the direction of z1 is -
+ i.
5 5
z1 - z 0 OC (ip / 2 )
= e =i [QOC = OB ] æ 4 3 ö
z 2 - z 0 OB Therefore, required vector = 3 z ç - + i ÷
è 5 5 ø
Þ ( z1 - z 0 ) = i ( z 2 - z 0 )
æ 4 3 ö
Þ 10 + 6i - z 0 = i ( 4 + 6i - z 0 ) = 15 ç - + i ÷ = - 12 + 9i
è 5 5 ø
Þ 16 + 2i = (1 - i ) z 0
(16 + 2i ) (1 + i ) Aliter
or z0 = × Imaginary axis
(1 - i ) (1 + i )
z1 3
16 + 16i + 2i + 2i 2
=
2 4 Real axis
14 + 18i –4 O
= = 7 + 9i
2
and radius, r = OC = z 0 - z1 = 7 + 9i - 10 - 6i –3 z
= - 3 + 3i
= (9 + 9 ) = 3 2 Here, z1 = - 4 + 3i
Hence,3 z1 = - 12 + 9i
Hence, required equation is
z - z0 = r y Example 86.ABCD is a rhombus. Its diagonals AC
Þ z - 7 - 9i = 3 2 and BD intersect at the point M and satisfy BD = 2AC .
If the points D and M represents the complex numbers
y Example 84.If z - 2 + i £ 2, where i = - 1, then 1 + i and 2 - i, where i = - 1, respectively, find A.
find the greatest and least value of z . Sol. Let A ºz
Sol. Q Radius = 2 units Q BD = 2AC or DM = 2 AM
C B
Y

M
O
X (2 – i)
A 2
C(2, –1) π/2
2 A(z)
D(1 + i)
B
Now, in DDMA,
i.e., AC = CB = 2 units Applying Coni method, we have

\ Least value of z = OA = OC - AC = 5 - 2 z - (2 - i ) AM ip / 2 1
= e = i
(1 + i ) - (2 - i ) DM 2
and greatest value of z = OB = OC + CB = 5 + 2
i i 3
Þ z -2+i = ( - 1 + 2i ) = - - 1 or z = 1 - i
Hence, greatest value of z is 5 + 2 and least value of z 2 2 2
is 5 - 2. 3 i
\ A º 1 - i or 3 -
2 2
[if positions of A and C interchange]
Chap 01 Complex Numbers 43

b y Example 87. Find the maximum and minimum


If z ± = a , then the greatest and least values of | z |
z 1
values of z satisfying z + = 2.
a + (a 2 + 4 | b |) - a + (a 2 + 4 | b | ) z
are and ,
2 2 Sol. Here, b = 1 and a = 2
respectively.
2 + (4 + 4)
b b \ Maximum and minimum values of z are
Proof z± ³ z - 2
z z
- 2 + (4 + 4)
|b | and i.e., 1 + 2 and - 1 + 2, respectively.
Þ a³ z - 2
|z |
4
or -a£ z -
|b |
£a
y Example 88. If z + = 2, find the maximum and
|z | z
|b | minimum values of z .
Now, z - £a
|z | Sol. Here, b = 4 and a = 2.

Þ |z |2 - a |z | - |b | £ 0 \ Maximum and minimum values of z are


2 + ( 4 + 16) - 2 + ( 4 + 16)
a - (a 2 + 4 | b| ) a + (a 2 + 4 | b | ) and
\ £ |z | £ 2 2
2 2 i.e. 1 + 5 and - 1 + 5, respectively.
a + (a 2 + 4 | b| )
or 0 £ |z | £ …(i) y Example 89. If z ³ 3, then determine the least
2
|b | 1
and |z | - ³ - a Þ |z |2 + a |z | - |b | ³ 0 value of z + .
|z | z
- a + (a 2 + 4 | b | ) 1 1 1
\ |z | ³ …(ii) Sol. Q z+ ³ z - = z - …(i)
2 z z z
From Eqs. (i) and (ii), we get 1 1 1 1
Q z ³3 Þ £ or - ³-
- a + (a 2 + 4 | b | ) a + (a 2 + 4 | b | ) z 3 z 3
£ |z | £
2 2 1 1 8 1 8
\ z - ³3- = Þ z - ³
z 3 3 z 3
a + (a 2 + 4 | b | )
Hence, the greatest value of | z | is
2 1 8
or z - ³ …(ii)
- a + (a 2 + 4 | b | ) z 3
and the least value of | z | is .
2 From Eqs. (i) and (ii), we get
1 1 8
Corollary For b = 1, z ± =a z+ ³
z z 3
- a + (a 2 + 4 ) a + (a 2 + 4 ) 1 8
Then, £ z £ \ Least value of z + is .
2 2 z 3
44 Textbook of Algebra

#L Exercise for Session 4


4
1 If z1, z 2, z 3 and z4 are the roots of the equation z 4 = 1, the value of S z i 3 is
i =1

(a) 0 (b) 1 (c) i , i = -1 (d) 1 + i , i = -1

2 If z1, z 2, z 3, ... , z n are n, nth roots of unity, then for k = 1, 2, 3, ... , n


(a) z k = k z k + 1 (b) z k + 1 = k z k

(c) z k + 1 = z k + z k - 1 (d) z k = z k + 1

3 If 1, a1, a 2, a 3, ... , a n - 1 are n, nth roots of unity, then (1 - a1) (1 - a 2 ) (1 - a 3 ) ... (1 - a n - 1) equals to
(a) 0 (b) 1 (c) n (d) n 2
6
æ æ 2 pk ö æ 2 pk ö ö
4 The value of å çsin çè
è 7 ø
÷ - i cos ç ÷ , where i = - 1, is
è 7 ø ÷ø
k =1

(a) - 1 (b) 0 (c) - i (d) i

5 If a ¹ 1 is any nth root of unity, then S = 1 + 3 a + 5 a 2 + ... upto n terms is equal to


2n 2n n n
(a) (b) - (c) (d) -
1- a 1- a 1- a 1- a

6 If a and b are real numbers between 0 and 1, such that the points z1 = a + i ,z 2 = 1+ bi and z 3 = 0 form an
equilateral triangle, then
(a) a = b = 2 + 3 (b) a = b = 2 - 3
(c) a = 2 - 3, b = 2 + 3 (d) None of these

7 If z = 2, the points representing the complex numbers - 1 + 5z will lie on


(a) a circle (b) a straight line (c) a parabola (d) an ellipse

8 If z - 2 / z - 3 = 2 represents a circle, then its radius is equal to


1 3 2
(a) 1 (b) (c) (d)
3 4 3

9 If centre of a regular hexagon is at origin and one of the vertex on argand diagram is 1 + 2i , where i = - 1, its
perimeter is
(a) 2 5 (b) 6 2 (c) 4 5 (d) 6 5

10 If z is a complex number in the argand plane, the equation z - 2 + z + 2 = 8 represents


(a) a parabola (b) an ellipse (c) a hyperbola (d) a circle

11 If z - 2 - 3i + z + 2 - 6i = 4, where i = - 1, then locus of P (z ) is


(a) an ellipse (b) f
(c) line segment of points 2 + 3i and - 2 + 6i (d) None of these

12 Locus of the point z satisfying the equation iz - 1 + z - 1 = 2, is (where, i = - 1)


(a) a straight line (b) a circle (c) an ellipse (d) a pair of straight lines

13 If z, iz and z + iz are the vertices of a triangle whose area is 2 units, the value of z is
(a) 1 (b) 2 (c) 4 (d) 8
4
14 If z - = 2, the greatest value of z is
z
(a) 5 - 1 (b) 3 + 1 (c) 5 + 1 (d) 3 - 1
Shortcuts and Important Results to Remember
1 z1 - z2 £ z1 + z2 £ z1 + z2 æ z - z1 ö
12 If arg ç ÷ = a (fixed), then the locus of z is a segment
è z - z2 ø
Thus, z1 + z2 is the greatest possible value of
z1 + z2 and z1 - z2 is the least possible value of of circle. P(z)
z1 + z2 . P(z)
α
b α
If z ± = a, then the greatest and least values of z are
z
a + (a2 + 4| b|) - a + (a2 + 4| b|) A(z1)
and , respectively.
2 2
B(z2)
z1 + ( z12 2
- z2 ) + z2 - ( z12 2
- z2 ) æ z - z1 ö
If arg ç ÷ = ± p / 2 , the locus of z is a circle with z1
è z - z2 ø
= z1 + z2 + z1 - z2
and z2 as the vertices of diameter.
z1 + z2 = z1 + z2 Û arg ( z1 ) = arg ( z2 )
æ z - z1 ö
14 If arg ç ÷ = 0 or p, the locus of z is a straight line
i.e. z1 and z2 are parallel. è z - z2 ø
passing through z1 and z2 .
z1 + z2 = z1 - z2 Û arg ( z1 ) - arg ( z2 ) = p
15 If three complex numbers are in AP, they lie on a straight
6 z1 + z2 = z1 - z2 Û arg ( z1 ) - arg ( z2 ) = ± p / 2 line in the complex plane.
16 If three points z1, z2 , z3 connected by relation
7 If z1 = z2 and arg ( z1 ) + arg ( z2 ) = 0, then z1 and z2 are a z1 + b z2 + c z3 = 0, where a + b + c = 0, the three points
conjugate complex numbers of each other. are collinear.
2 2 17 If z1, z2 , z3 are vertices of a triangle, its centroid
The equation z - z1 + z - z2 = k, k Î R will
1
represent a circle with centre at ( z1 + z2 ) and radius is
2 z0 =
z1 + z2 + z 3
, circumcentre z1 =
å| z1|2 ( z2 - z3 ) ,
1 2 1 2
3 å z1( z2 - z3 )
2 k - z1 - z2 provided k³ z1 - z2 .
2 2
orthocentre z =
å z1( z2 - z3 ) + å| z1|2 ( z2 - z3 )
9 Area of triangle whose vertices are z, iz and z + iz, å ( z1 z2 - z1 z2 )
1 z1 z1 1
where i = - 1, is | z|2 .
2 1
and its area = | z2 z2 1 |.
4
10 Area of triangle whose vertices are z, w z and z + w z is z3 z3 1
3 2
z , where w is cube root of unity. 18 If| z1| = n1,| z2 | = n2 ,| z 3| = n 3 , ...,| z m | = n m,
4

11 arg ( z ) - arg (- z ) = p or - p according as arg ( z ) is n12 n22 n 23 n2


then + + + ... + m = | z1 + z2 + z3 + ... + zm|.
positive or negative. z1 z2 z3 zm
JEE Type Solved Examples :
Single Option Correct Type Questions
n This section contains 10 multiple choice examples. a b c
Sol. (a) We have, + + =1+i
Each example has four choices (a), (b), (c) and (d) out of a1 b1 c 1
which ONLY ONE is correct.
On squaring both sides, we get
a2 b2 c 2 æ ab bc ca ö
l Ex. 1 If z1 and z 2 be the n th root of unity which subtend + 2 + 2 + 2ç + + ÷
2
a1 b1 c 1 è a1b1 b1c 1 c 1a1 ø
right angled at the origin. Then, n must be of the form
(a) 4k + 1 (b) 4k + 2 (c) 4k + 3 (d) 4k = 1 + i 2 + 2i
Sol. (d) The nth roots of unity is given by a2 b2 c2 abc æ c 1 a1 b1 ö
Þ + + +2 ç + + ÷
æ 2r p ö æ 2r p ö 2r pi /n a12 b12 c 12 a1b1c 1 è c a bø
cos ç ÷ + i sin ç ÷ =e ,
è n ø è n ø = 1 - 1 + 2i
where r = 0, 1, 2, K, (n - 1) a 2
b 2
c 2

2 pr 1 i / n 2 pr 2 i / n
Þ + + + 0 = 2i
So, let z1 = e and z 2 = e , where 0 £ r1, r 2 < n , a12 b12 c 12
r1 ¹ r 2 . a2 b2 c2
\ + + = 2i
It is given that the line segment joining the points having a12 b12 c 12
affixes z1 and z 2 , subtends a right angled at the origin.
Therefore, l Ex. 4 Let z and w be complex numbers. If Re( z ) = | z - 2|,
æz ö p p
arg ç 1 ÷ = ± Re( w ) = | w -2 | and arg ( z - w ) = , the value of Im ( z + w ) is
èz2 ø 2 3
2pr1 2pr 2 p 1 2 4
Þ - =± (a) (b) (c) 3 (d)
n n 2 3 3 3
Þ n = ± 4(r1 - r 2 ) Sol. (d) Let z = x + iy , x, y Î R and i = -1
\ n = 4k , where k = ± (r1 - r 2 ) Q Re(z ) = | z - 2|
Þ x = ( x - 2) 2 + y 2
z -1
l Ex. 2 If | z | =1 and w = (where z ¹ -1), then Re( w ) is Þ y 2 = 4( x - 1)
z +1
\ z = 1 + t 2 + 2ti , parametric form and let w = p + iq
1
(a) 0 (b) - Similarly, w = 1 + s 2 + 2si
| z + 1|2
\ z - w = (t 2 - s 2 ) + 2i (t - s )
½ z ½ ½× 1 2
(c)½ (d) p
½z + 1½ | z + 1|
2
| z + 1|2 Þ arg(z - w) = [given]
3
Sol. (a) We have, | z | = 1.
æ 2 ö p 2
Let z = e i q , where q Î R and i = -1. \ tan -1 ç ÷= Þ = 3
èt + s ø 3 t +s
z -1 eiq -1 æq ö
Then, w= = iq = i tan ç ÷ Þ (t + s ) =
2
z +1 e +1 è2ø
3
\ Re( w) = 0 Now, z + w = 2 + t 2 + s 2 + 2i (t + s )
4
l Ex. 3 If a, b, c, a 1 , b1 and c 1 are non-zero complex \ Im(z + w) = 2(t + s ) =
3
a b c a b c
numbers satisfying + + = 1 + i and 1 + 1 + 1 = 0,
a1 b1 c 1 a b c æ z - 3ö p
a 2
b 2
c 2
l Ex. 5 The mirror image of the curve arg ç ÷= ,
where i = -1, the value of + + is è z -i ø 6
2 2
a 1 b1 c 12 i = -1 in the real axis, is
(a) 2i (b) 2 + 2i (c) 2 (d) None of these
Chap 01 Complex Numbers 47

æz + 3ö p æz - 3ö p 1 æ 1ö
(a) arg ç ÷= (b) arg ç ÷= \ a = z 2017 + = - ç w + ÷ = - ( w + w2 ) = 1
èz +iø 6 èz +iø 6 z 2017 è wø
n n-4 n-4
æz +iö p æz +iö p and 22 = 24× 2 = 162 has last digit 6.
(c) arg ç ÷= (d) arg ç ÷=
èz + 3ø 6 èz - 3ø 6 \ b=6-1=5
Sol. (d) Q The image of z in the real axis is z. Hence, a + b 2 = 12 + 52 = 26
2

The image is given by


æz - 3ö p l Ex. 8 If w is complex cube root of unity and a, b, c are
arg ç ÷=
èz - i ø 6 such that 1 + 1 + 1 = 2 w 2 and
æz - 3ö p a +w b+w c +w
Þ - arg ç ÷= [Q arg(z ) = - arg(z )]
èz + i ø 6 1 1 1
+ + = 2 w, then the value of
2 2
æz + i ö p é æ z1 ö æz2 öù a +w b+w c + w2
Þ arg ç ÷= êQ arg çè z ÷ø = - arg çè z ÷ø ú
èz - 3ø 6 ë 1 û
1 1 1
2 + + is equal to
a +1 b +1 c +1
æz +iö p (a) -2 (b) -1 (c) 1 (d) 2
l Ex. 6 The mirror image of the curve arg ç ÷= , 1 1 1 2
è z - 1ø 4 Sol. (d) Q + + 2
= 2w =
a+w b+w c +w w
i = -1 in the line x - y = 0, is
1 1 1 2
æz + iö p æ z + 1ö p and 2
+ 2
+ 2
= 2w =
(a) arg ç ÷= (b) arg ç ÷= a+w b+w c +w w2
è z + 1ø 4 èz - iø 4
It is clear that, w and w2 are the roots of the equation
æz - iö p æz + iö p 1 1 1 2
(c) arg ç ÷= (d) arg ç ÷= + + = …(i)
è z + 1ø 4 è z - 1ø 4 a+x b+x c +x x
Sol. (c) Q The image of z in the line y = x is iz. Þ x å (b + x )(c + x ) = 2(a + x )(b + x )(c + x )
\ The image of the given curve is Þ x 3 - (ab + bc + ca )x - 2abc = 0
æiz + i ö p
arg ç ÷= Q Coefficient of x 2 = 0, the sum of roots = 0
èi z - 1ø 4
Þ a + w + w2 = 0 Þ a - 1 = 0
æz + 1ö p
Þ arg ç ÷= \ a =1
èz + i ø 4
\ Third root is 1.
æz + 1ö p From Eq. (i), we get
Þ - arg ç ÷= [Q arg( z ) = - arg(z )]
èz - i ø 4 1 1 1
+ + =2
æz - i ö p é æz ö æz öù a+1 b+1 c +1
Þ arg ç ÷= êQ arg ç 1 ÷ = - arg ç 2 ÷ ú
èz + 1ø 4 ë èz2 ø è z1 ø û
l Ex. 9 If a, b and c are distinct integers and w( ¹1) is a
1 1 cube root of unity, then the minimum value of
l Ex. 7 If z + = 1 and a = z 2017 + and b is the last
z z 2017 | a + bw + cw 2 | + | a + bw 2 + c w |, is
n
digit of the number 2 2 -1,, when the integer n >1, the value (a) 2 3 (b) 3 (c) 4 2 (d) 2
2
of a 2 + b 2 is Sol. (a) Let z = a + bw + c w . Then,
(a) 23 (b) 24 (c) 26 (d) 27 | z | 2 = zz = (a + bw + cw2 )(a + b w + c w2 )
1
Sol. (c) Qz + = 1 Þ z2 - z + 1 = 0 = (a + bw + cw2 )(a + bw2 + cw)
z
= a 2 + b 2 + c 2 - ab - bc - ca
-( - 1 ) ± ( 1 - 4 )
\ z= = - w, - w2 1
2 = [(a - b )2 + (b - c )2 + (c - a )2 ]
2
[w is cube root of unity]
2017 2017 éQ a ¹ b ¹ c ù
and z = ( - w) = - w, 1 ê
2
Þ |z | ³ ´ 6 = 3 \ | a - b | ³ 1, | b - c | ³ 1ú
z 2017 = ( - w2 )2017 = - w2 2 ê ú
êë and | a - c | ³ 2 úû
48 Textbook of Algebra

\ |z | ³ 3 Sol. (b) Q | 3 - i (z - 1)| = | - i (z - 1 - 3i )| = | -i || z - 1 - 3i |


\ | a + bw + cw2 | + | a + bw2 + cw| = | z - 1 - 3i | …(i)
2 2
= | a + bw + cw | + | a + b w + c w| and | z - 1 - 3i | = |(z - 2i ) + ( -1 - i )| £ | z - 2i | + | -1 - i |
= | a + bw + cw2 | + | a + bw + cw2 | [Q | z1 + z 2 | £ | z1 | + | z 2 | ]
= 2| a + bw + cw2 | = 2| z | ³ 2 3 \ | z - 1 - 3i | £ | z - 2i | + 2
Hence, the minimum value of | a + bw + cw2 | + | a + bw2 + cw|
Þ | z - 1 - 3i | £ 2 + 2 [Q | z - 2i | £ 2 ]
is 2 3.
Þ | z - 1 - 3i | £ 2 2
l Ex. 10 If | z - 2i | £ 2, where i = -1, then the maximum From Eq. (i), we get
value of | 3 - i ( z - 1) | , is | 3 - i (z - 1)| £ 2 2
(a) 2 (b) 2 2 (c) 2 + 2 (d) 3 + 2 2 Hence, the maximum value of | 3 - i (z - 1)| is 2 2.

JEE Type Solved Examples :


More than One Correct Option Type Questions
n This section contains 5 multiple choice examples. Each Q (z 2 - z 3 ) = (1 + i )(z1 - z 3 )
example has four choices (a), (b), (c) and (d) out of which B(z2)
more than one may be correct.

l Ex. 11 If z 1 = a + ib and z 2 = c + id are complex numbers


such that | z 1 | = | z 2 | = 1 and Re( z 1 z 2 ) = 0, where i = -1,
then the complex numbers w1 = a + ic and w 2 = b + id
π/2
satisfy A(z1)
C(z3)
(a) | w1| = 1 (b) | w2 | = 1
(c) Re( w1 w2 ) = 0 (d) None of these Þ ( z 2 - z1 ) = i ( z1 - z 3 )
Þ ( z 2 - z1 ) = - i ( z 3 - z1 )
Sol. (a, b, c)
or ( z 3 - z1 ) = i ( z 2 - z1 )
Q | z1| = 1, | z 2 | = 1 Þ z1 = CiS q, z 2 = CiS f
z 3 - z1
Re(z1z 2 ) = Re(CiS (q - f )) = 0 [given] Þ = e i p/ 2
z 2 - z1
p p p
Þ cos(q - f ) = 0 Þ q - f = Þ f=q - AC is obtained by rotating AB about A through
2 2 2
anti-clockwise.
and a = cosq, b = sinq, c = cos f, d = sin f p
\ w1 = a + ic = cos q + i cos f = cos q + i sin q \ AB = AC , ÐCAB =
2
é pù
êQ f = q - 2 ú
Hence, z1, z 2 , z 3 form isosceles right angled triangle.
ë û
and w2 = b + id = sin q + i sin f = sin q - i cos q l Ex. 13 If z satisfies the inequality | z - 1 | < | z + 1 |, then
é pù
êQ f = q - 2 ú one has
ë û
(a) | z - 2 - i | < | z + 2 - i |, i = -1
\ | w1| = 1, | w2 | = 1
p
and Re ( w1 w2 ) = Re (cos q + i sin q )(sin q + i cos q ) = 0 (b) |arg( z + i )| < , i = -1
2
l Ex. 12 The complex numbers z 1 , z 2 , z 3 satisfying (c) Re( z ) < 0
( z 2 - z 3 ) = (1 + i )( z 1 - z 3 ), where i = -1, are vertices of a (d) Im(i z ) > 0, i = -1

triangle which is Sol. (a, b, d)


(a) equilateral (b) isosceles On putting z = x + iy in the given relation, we have
(c) right angled (d) scalene ( x - 1) 2 + y 2 < ( x + 1) 2 + y 2
Sol. (b,c) Þ x >0
Chap 01 Complex Numbers 49

i.e. Re(z ) > 0 …(i) Case II When y = - x , then 2xy = ( x - 1)2 + y 2 reduces to
and on putting z = x + iy in alternate (a), then - 2x 2 = ( x - 1) 2 + x 2
( x - 2) 2 + ( y - 1) 2 < ( x + 2) 2 + ( y - 1) 2 Þ ( x - 1) 2 + 3x 2
= 0 which is not possible. …(ii)
Þ x >0 [from Eq. (i)] 1+i
which is true. From Eqs. (i) and (ii), we get z =
2
\Real part of (z + i ) = x > 0, i.e., no real and no purely imaginary roots
p p
then arg(z + i ) lies between - and and |z | =
1
<1
2 2
p 2
and hence |arg (z + i )| <
2 l Ex. 15 Let z 1 and z 2 be two complex numbers represented
and Im(i z ) = Im(i ( x - iy )) = Im(y + ix )
= x >0 [from Eq. (i)] by points on circles | z | =1 and | z | = 2 respectively, then
which is true. (a) max.| 2z 1 + z 2 | = 4 (b) min.| z 1 - z 2 | = 1
½ 1½ ½ 2½
(c)½z 2 + ½ £ 3 (d)½z 1 + ½ £ 2
l Ex. 14 The equation z 2 - i| z - 1 | 2 = 0, where i = -1, has ½ z 1½ ½ z2½
(a) no real root Sol. (a, b, c, d)
(b) no purely imaginary root Q | z1| = 1 and | z 2 | = 2
(c) all roots inside | z | = 1 \ |2z1 + z 2 | £ | 2z1| + | z 2 | = 2| z1| + | z 2 |
(d) atleast two roots Þ | 2z1 + z 2 | £ 2| z1| + | z 2 | = 2 + 2 = 4
Sol. (a, b, c) \ | 2z1 + z 2 | £ 4
On putting z = x + iy , we have Þ max.|2z1 + z 2 | = 4 [alternate (a)]
( x + iy )2 - i | x + iy - 1| 2 = 0 and | z1 - z 2 | ³ || z1| - | z 2 || = |1 - 2| = 1
2 Þ | z1 - z 2 | ³ 1
Þ x - y 2 + 2ixy - i (( x - 1)2 + y 2 ) = 0
\ min.| z1 - z 2 | = 1 [alternate (b)]
On comparing real and imaginary parts, we get
½ 1½ ½ 1½
½z 2 + ½ £ | z 2 | +½ ½ = | z 2 | + 1 1
x 2 - y 2 = 0 and 2xy = ( x - 1)2 + y 2 =2+ =3
½ z1½ ½z1½ | z1| 1
Case I When y = x , then 2xy = ( x - 1)2 + y 2 reduces to ½
\ ½z 2 + 1½ ½£ 3 [alternate (c)]
2x 2 = ( x - 1) 2 + x 2 ½ z1½
Þ 0 = - 2x + 1 ½
and ½z1 + 2 ½ ½ £ | z1| +½
½ 2½½ = | z1| + 2 = 1 + 2 = 2
1
\ x = =y ½ z 2½ ½z 2½ |z 2 | 2
2
1+i ½
½z1 + 2 ½½£ 2
\ [alternate (d)]
Þ z = x + iy = …(i) ½ z 2½
2

JEE Type Solved Examples :


Passage Based Questions
n
This section contains 2 solved passages based upon each (b) (ab + ab ) (bc + bc ) + (ca - ca )2 = 0
of the passage 3 multiple choice examples have to be
answered. Each of these examples has four choices (a), (b), (c) (ab - ab ) (bc - bc ) + (ca + ca )2 = 0
(c) and (d) out of which ONLY ONE is correct. (d) (ab + ab ) (bc - bc ) + (ca - ca )2 = 0
Passage I (Ex. Nos. 16 to 18) Sol. (b) Q az 2 + bz + c = 0 …(i)
2
Consider a quadratic equation az + bz + c = 0, where a, b and c \ az 2 + bz + c = 0
are complex numbers. Þ a (z )2 + bz + c = 0
16. The condition that the equation has one purely For purely imaginary root,
imaginary root, is z = -z
(a) (ab - ab ) (bc + bc ) + (ca - ca )2 = 0 Then, az 2 - bz + c = 0 …(ii)
50 Textbook of Algebra

From Eqs. (i) and (ii), we get Passage II (Ex. Nos. 19 to 21)
z2 z 1
= = Let P be a point denoting a complex number z on the complex
bc + bc ca - ca - ab - ab plane.
bc + bc ca - ca i.e. z = Re (z ) + i Im (z ), where i = -1
Þ z= =
ca - ca - ab - ab If Re (z ) = x and Im (z ) = y , then z = x + iy
2
\ (ab + ab ) (bc + bc ) + (ca - ca ) = 0 19. If P moves such that
17. The condition that the equation has one purely real |Re ( z )| + | Im( z )| = a (a Î R + )
root, is The locus of P is
(a) (ab + ab ) (bc - bc ) = (ca + ca )2 (a) a parallelogram which is not a rhombus
(b) a rhombus which is not a square
(b) (ab - ab ) (bc + bc ) = (ca + ca )2
(c) a rectangle which is not a square
(c) (ab - ab ) (bc - bc ) = (ca - ca )2 (d) a square
(d) (ab - ab ) (bc - bc ) = (ca + ca )2 Sol. (d) Q |Re( z )| + |Im( z )| = a
Sol. (c) Q az 2 + bz + c = 0 …(i) Þ | x | + |y | = a

Þ az 2 + bz + c = 0 Y
a
Þ a (z )2 + bz + c = 0
For purely real root, z = z X′ X
–a O a
2
Then, az + bz + c = 0 …(ii)
From Eqs. (i) and (ii), we get –a
2 Y′
z z 1
= =
bc - bc ca - ca ab - ab \ Locus of P is a square.

bc - bc ca - ca 20. The area of the circle inscribed in the region denoted


Þ z= = by |Re ( z )| + |Im ( z )| =10 equals to
ca - ca ab - ab
(a) 50 p sq units (b) 100 p sq units
Þ (ab - ab ) (bc - bc ) = (ca - ca )2 (c) 55 sq units (d) 110 sq units
18. The condition that the equation has two purely Sol. (a) From above, a = 10
imaginary roots, is Diameter of circle = Distance between sides of square
a b c a b c = Length of side of square = a 2 = 10 2
(a) = = (b) - = =
a b c a b c or 2r = 10 2
a b c a b c
(c) = = - (d) = - = Þ r =5 2
a b c a b c
\ Area of a circle = pr 2 = 50p sq units
Sol. (d) Q az 2 + bz + c = 0 …(i)
21. Number of integral solutions satisfying the inequality
\ az 2 + bz + c = 0
|Re ( z )| + |Im ( z )| < 21, is
Þ a ( z )2 + bz + c = 0 (a) 841 (b) 839
Since, both roots are purely imaginary. (c) 840 (d) 842
Sol. (c) Q | x | + | y | < 21 Þ 0 £ | x | + | y | £ 20
\ z = -z
Then, 2
az - bz + c = 0 …(ii) If x > 0, y > 0, 0 £ x + y £ 20
Hence, Eqs. (i) and (ii) are identical. 21 × 20
Number of solutions = 21C 2 = = 210
a b c 2
\ =- =
a b c \ Total integral solutions = 4 ´ 210 = 840
Chap 01 Complex Numbers 51

JEE Type Solved Examples :


Single Integer Answer Type Questions
n This section contains 2 examples. The answer to each Y
example is a single digit integer ranging from 0 to 9
(both inclusive). B

l Ex. 22 If z 1 , z 2 Î C , z 12 + z 22 Î R , z 1 ( z 12 - 3 z 22 ) = 2 and X′ X
A′ S1 O S2 A
z 2 (3 z 12 - z 22 ) = 11, the value of z 12 + z 22 is
Sol. (5) We have, z1 (z12 - 3z 22 ) = 2 …(i) B′
Y′
and z 2 (3z12 - z 22 ) = 11 …(ii)
multiplying Eq. (ii) by i ( -1 ) and then adding in Eq. (i), we Also, | z - ( 3 + 5i )| + | z - ( 5 + 11i )| = 4 5
get represents an ellipse.
z13 - 3z1z 22 + i (3z12z 2 - z 32 ) = 2 + 11i \ |(3 + 5i ) - (5 + 11i )| = 4 + 36 = 40 < 4 5

Þ (z1 + iz 2 )3 = 2 + 11i …(iii) with foci S1(3, 5) and S 2 (5, 11).


Distance between foci = S1S 2 = 40 = 2 10 = Diameter of
Again, multiplying Eq. (ii) by (- i) and then adding in Eq. (i),
we get circle
2
z13 - 3z1z 22 - i (3z1 z 2 - z 23 ) = 2 - 11i i.e., 2ae = 2 10
Þ ae = 10 and 2a = 4 5 Þ a = 2 5
Þ (z1 - iz 2 )3 = 2 - 11i …(iv)
ae 1
Now, on multiplying Eqs. (iii) and (iv), we get \ e= =
a 2
(z12 + z 22 )3 = 4 + 121 = 125 = 53
2 æ 1ö
\ z12 + z 22 = 5 Now, b = a (1 - e ) = 2 5 ç1 - ÷ = 10 = Radius of circle
è 2ø
l Ex. 23 The number of solutions of the equations \ Centre of the ellipse = Mid-point of S1 and S 2
| z - ( 4 + 8i )| = 10 and | z - (3 + 5i )| + | z - ( 5 + 11i )| = 4 5, 3 + 5i + 5 + 11i
= = 4 + 8i i.e., ( 4, 8)
where i = - 1. 2
which coincides with the centre of the circle and length of
Sol. (2) Here, | z - ( 4 + 8i )| = 10 minor-axis is equal to the radius of the circle. Hence, there
represents a circle with centre (4, 8) and radius 10. are only (2) two solutions of the given equations.

JEE Type Solved Examples :


Matching Type Questions
n
This section contains 2 examples. Examples 24 and 25 have three statements (A, B and C) given in Column I and four
statements (p, q, r and s) in Column II. Any given statement in Column I can have correct matching with one or more
statement(s) given in Column II.

l Ex. 24
Column I Column II
(A) If l and m are the greatest and least values of | z - 1 | , if | z + 2 + i | £ 1, where i = -1, then (p) l + m = rational
(B) If l and m are the greatest and least values of | z - 2 | , if | z + i | £ 1, where i = -1, then (q) l + m = irrational

(C) If l and m are the greatest and least values of | z + 2i | , if 1 £ | z - 1| £ 3, where i = -1, then (r) l - m = rational
(s) l - m = irrational
52 Textbook of Algebra

Sol. (A) ® (q, r); (B) ® (q, r); (C) ® (p, s) From the figure, the greatest value of | z - 2| = | w |
(A) Q |z + 2 + i | £ 1 = | w - 0| = OB
Þ |(z - 1) + (3 + i )| £ 1 = OP + PB = 5 + 1
Þ | w + (3 + i )| £ 1 where, w = z - 1 \ l = 5 +1
From the figure, the greatest value of and the least value of | z - 2| = | w |
| z - 1| = | w| = | w - 0| = OB = OP + PB = 10 + 1 = | w - 0| = OA = OP - AP = 5 - 1
\ l = 10 + 1 \ m= 5 -1
Y Þ l + m = ( 5 + 1) + ( 5 - 1) = 2 5 = irrational
and l - m = ( 5 + 1) - ( 5 - 1) = 2 = rational
B (–3, –1) Aliter
P Q |z + i | £ 1
1 A Þ |(z - 2) + (2 + i )| £ 1
X′ X Þ | w + (2 + i )| £ 1
O
where, w= z - 2 …(i)
and the least value of | z - 1| = | w| Q | w + (2 + i )| ³ || w| - | 2 + i ||
= | w - 0 | = OP - AP = 10 - 1 or | w + (2 + i )| ³ || w| - 5 | …(ii)
\ m = 10 - 1 From Eqs. (i) and (ii), we get
|| w| - 5| £ | w + 2 + i | £ 1
Þ l + m = ( 10 + 1) + ( 10 - 1) = 2 10 = irrational
Þ || w| - 5| £ 1
and l - m = ( 10 + 1) - ( 10 - 1) = 2 = rational
Aliter or - 1 £ | w| - 5 £ 1 or 5 - 1 £ | w| £ 5 + 1
Q |z + 2 + i | £ 1 \ l = 5 + 1 and m = 5 - 1
Þ |(z - 1) + (3 + i )| £ 1 Þ l + m = 2 5 = irrational
Þ | w + (3 + i )| £ 1 …(i) and l - m = 2 = rational
where, w = z - 1 (C) Q 1 £ | z - 1| £ 3
\ | w + (3 + i )| ³ || w| - | 3 + i || Þ 1 £ |(z + 2i ) - (1 + 2i )| £ 3
or | w + (3 + i )| ³ || w| - 10 | …(ii) Þ 1 £ | w - (1 + 2i )| £ 3 …(i)
From Eqs. (i) and (ii), we get where, w = z + 2i
|| w| - 10 | £ | w + 3 + i | £ 1
Y
Þ || w| - 10 | £ 1 A
or - 1 £ | w| - 10 £ 1
or 10 - 1 £ | w| £ 10 + 1
\ l = 10 + 1 and m = 10 - 1 P
(1, 2)
Þ l + m = 2 10 = irrational
O
and l - m = 2 = rational X
(B) Q |z + i | £ 1 B
Þ |(z - 2) + ( 2 + i )| £ 1
Þ | w + (2 + i )| £ 1 From the figure, the greatest value of | z + 2i | = | w|
where, w= z - 2 = | w - 0| = OA = OP + PA = 5 + 3
Y \ l =3+ 5
and the least value of | z + 2i | = | w|
(–2, –1)
= | w - 0| = OB = PB - OP = 3 - 5
B
P \ m=3- 5
1 A Þ l + m = (3 + 5 ) + (3 - 5 ) = 6 = rational
X
O and l - m = (3 + 5 ) - (3 - 5 ) = 2 5 = irrational
Chap 01 Complex Numbers 53

l Ex. 25 = 3 - 3i, 1 + i, - 1 - i, - 3 + 3i
p p 3p 3p
Column I Column II \ Principal values of arg (z ) = - , , - ,
4 4 4 4
(A) If (3 - 4 i ) + (- 3 - 4 i ) = z, the principal (p) 0 æ 13 + 5 13 - 5 ö
value of arg (z) can be (where i = -1) (B) (5 + 12i ) = ± ç +i ÷
è 2 2 ø
(B) If (5 + 12i ) + (- 5 + 12i ) = z, the principal (q) p
± = ± (3 + 2i )
value of arg (z) can be (where i = -1) 4
æ 13 - 5 13 + 5 ö
p ( - 5 + 12i ) = ± ç +i ÷
(C) If (-15 + 8i ) + (- 15 - 8i ) = z, the principal (r) ± è 2 2 ø
2
value of arg (z) can be (where i = -1)
= ± (2 + 3i )
(s) 3p z = (5 + 12i ) + ( - 5 + 12i )
± Q
4
= ± (3 + 2i ) ± (2 + 3i )
Sol. (A) ® (q, s); (B) ® (q, s); (C) ® (p, r) = 5 + 5i , 1 - i , - 1 + i , - 5 - 5i
æ | z | + Re (z ) | z | - Re(z ) ö p p 3p 3p
Q z =±ç +i ÷, Im (z ) > 0 \ Principal values of arg (z ) = , - , , -
è 2 2 ø 4 4 4 4
æ 17 - 15 17 + 15 ö
æ | z | + Re(z ) | z | - Re (z ) ö (C) - 15 + 8i = ± ç +i ÷
=±ç -i ÷, Im (z ) < 0 è 2 2 ø
è 2 2 ø
= ± (1 + 4i )
æ 5+3 5 - 3ö
(A) (3 - 4i ) = ± ç -i ÷ = ± (2 - i ) and - 15 - 8i = - 15 + 8i = ± (1 + 4i )
è 2 2 ø
= ± (1 - 4i )
æ 5-3 5 + 3ö
( - 3 - 4i ) = ± ç -i ÷ = ± (1 - 2i ) Q z = ( - 15 + 8i ) + ( - 15 - 8i )
è 2 2 ø
= ± (1 + 4i ) ± (1 - 4i ) = 2, 8i , - 8i , - 2
Q z = ( 3 - 4i ) + ( - 3 - 4i ) p p
\ Principal values of arg (z ) = 0, , - , p.
\ z = ± (2 - i ) ± (1 - 2i ) 2 2

JEE Type Solved Examples :


Statement I and II Type Questions
n
Directions Example numbers 26 and 27 are Sol. (c) Statement-1
Assertion-Reason type examples. Each of these examples AB = | z1 - z 2 | = | 4i | = 4,
contains two statements:
BC = | z 2 - z 3 | = | 4 | = 4,
Statement-1 (Assertion) and Statement-2 (Reason)
Each of these examples also has four alternative choices, CD = | z 3 - z 4 | = | - 4i | = 4
only one of which is the correct answer. You have to select DA = | z 4 - z1| = | - 4 | = 4
the correct choice as given below.
AC = | z1 - z 3 | = | 4 + 4i | = 4 2
(a) Statement-1 is true, Statement-2 is true; Statement-2
is a correct explanation for Statement-1 and BD = | z 2 - z 4 | = | 4 - 4i | = 4 2
(b) Statement-1 is true, Statement-2 is true; Statement-2
is not a correct explanation for Statement-1 Y
(c) Statement-1 is true, Statement-2 is false
(d) Statement-1 is false, Statement-2 is true z4 = –2 + 2i z1 = 2 + 2i
D A
l Ex. 26 Consider four complex numbers z 1 = 2 + 2i,
z 2 = 2 - 2i, z 3 = - 2 - 2i and z 4 = - 2 + 2i, where i = -1. X′ X
O
Statement-1 z 1 , z 2 , z 3 and z 4 constitute the vertices of a
square on the complex plane because C B
Statement-2 The non-zero complex numbers z , z , - z , - z z3 = –2 – 2i z2 = 2 – 2i
always constitute the vertices of a square. Y′
54 Textbook of Algebra

It is clear that, AB = BC = CD = DA and AC = BD Þ (| z1 | + | z 2 | )2 = | z1| 2 + | z 2 | 2 + 2 | z1| | z 2 | cos (q 1 - q 2 )


Hence, z1, z 2 , z 3 and z 4 are the vertices of a square. [from Eq. (i)]
\ Statement-1 is true. 2 2
Þ | z1 | + | z 2 | + 2 | z1 | | z 2 |
Statement-2 If z = a + ib
If a ¹ b = | z1 | 2 + | z 2 | 2 + 2 | z1 | | z 2 | cos (q 1 - q 2 )
Then, AB = | z - z | = |(a + ib ) - (a - ib )| = 2| b | \ cos (q 1 - q 2 ) = 1
BC = | z - ( - z )| = | z + z | = | a - ib + a + ib | = 2 | a |
\ AB ¹ BC z1 z2 O
Statement-2 is false. Þ q1 - q 2 = 0
or amp (z1 ) - amp (z 2 ) = 0
l Ex. 27 Consider z 1 and z 2 are two complex numbers \ Statement-1 is true.
such that | z 1 + z 2 | = | z 1 | + | z 2 | Statement-2 Since, z1, z 2 and O (origin) are collinear,
Statement-1 amp ( z 1 ) - amp ( z 2 ) = 0 then
Statement-2 The complex numbers z 1 and z 2 are collinear æ O - z1 ö
amp ç ÷ =0
with origin. èO - z2 ø
Sol. (b) Statement-1 æz ö
Þ amp ç 1 ÷ = 0
Q | z1 + z 2 | = | z1| + | z 2 | …(i) èz2 ø
If amp (z1 ) = q 1 and amp (z 2 ) = q 2 , then Þ amp (z1 ) - amp (z 2 ) = 0
2 2 2
| z1 + z 2 | = | z1 | + | z 2 | + 2 | z1 | | z 2 | cos (q 1 - q 2 ) \ Statement-2 is true, which is not a correct explanation of
Statement-1.

Subjective Type Examples


n
In this section, there are 24 subjective solved examples. Hence, the least value of z1 + z 2 is 19 and the greatest
value of | z1 + z 2 | is 31.
l Ex. 28 If z - i Re ( z ) = z - Im ( z ) , then prove that z
lies on the bisectors of the quadrants, where i = -1. l Ex. 30 Let S denotes the real part of the complex number
Sol. Let z = x + iy , where x , y Î R and i = -1 5 + 2i 20 + 5i
z= + + 3i, where i = -1, K denotes the sum
2 - 5i 7 + 6i
\ Re (z ) = x and Im (z ) = y
Then, z - i Re (z ) = z - Im (z ) of the imaginary parts of the roots of the equation
Þ x + iy - ix = x + iy - y z 2 - 8 (1 - i ) z + 63 - 16i = 0 and G denotes the value of
2012
Þ x - i ( x - y ) = ( x - y ) + iy
S i r , where i = -1, find the value of S - K + G.
Þ x 2
+ ( x - y )2 = ( x - y )2 + y 2 r =4

Þ x 2
+ ( x - y )2 = ( x - y )2 + y 2 Sol. For S,
(5 + 2i ) (20 + 5i )
Þ x 2
= y 2 or y = ± x z= + + 3i
(2 - 5i ) (7 + 6i )
Hence, z lies on the bisectors of the quadrants.
(5 + 2i ) (2 + 5i ) (20 + 5i ) (7 - 6i )
= + + 3i
l Ex. 29 Find the greatest and the least values of z 1 + z 2 , 29 85
0 + 29i 170 - 85i
if z 1 = 24 + 7i and z 2 = 6, where i = -1. = + + 3i
29 85
Sol. Q z1 = 24 + 7i = i + 2 - i + 3i = 2 + 3i
\ z1 = (24 )2 + (7 )2 = 25 \ Re (z ) = 2 \ S =2
For K,
z1 - z 2 £ z1 + z 2 £ z1 + z 2
Put z = x + iy in the given equation, then
Þ 25 - 6 £ z1 + z 2 £ 25 + 6 ( x + iy )2 - 8 (1 - i ) ( x + iy ) + 63 - 16i = 0
or 19 £ z1 + z 2 £ 31
Chap 01 Complex Numbers 55

On comparing the real and imaginary parts, we get 1


l Ex. 32 If arg ( z 1/ 3 ) = arg ( z 2 + z z 1/ 3
), find the value
x 2 - y 2 - 8 ( x + y ) + 63 = 0 …(i) 2
of z .
and xy + 4 ( x - y ) = 8 …(ii)
1
On substituting the value of x from Eq. (ii) in Eq. (i), we get Sol. We have, arg (z 1 / 3 ) = arg (z 2 + z z 1 / 3 )
2
y 4 + 16y 3 + ... = 0
Þ 2 arg (z 1/ 3 ) = arg (z 2 + z z 1/ 3 )
\ K = - 16
2012 2009 Þ arg (z 2 / 3 ) = arg (z 2 + z z 1/ 3 ) [by property]
For G, G= S r
i = S i r +3
=i 1+ 3
+ 0 =1 Þ 2
arg (z + z z 1/ 3
) - arg (z 2/3
)=0
r =4 r =1

\ S - K + G = 2 - ( - 16) + 1 = 19 æz + z z 2 1/ 3 ö
Þ arg ç ÷ =0 [by property]
è z2/ 3 ø
l Ex. 31 If z - 1 = 1, where z is a point on the argand
æ z ö
z -2 Þ arg çz 4 / 3 + 1/ 3 ÷ = 0
plane, show that = i tan (arg z ), where i = -1 . è z ø
z
z
Sol. Given, z -1 =1 Þ z -1
2
=1 Þ z 4/3 + 1/ 3
is purely real.
z
Þ ( z - 1) ( z - 1) = 1 Þ z z - z - z = 0 æ z ö
Þ Im çz 4 / 3 + 1/ 3 ÷ = 0
z è z ø
Þ (z + z ) = z z Þ +1=z
z
z æ 4/3 z ö æ z ö
Þ _ =z -1 …(i) çz + 1/ 3 ÷ - çz 4 / 3 + 1/ 3 ÷
z è z ø è z ø =0
æ Im (z ) ö Þ
Now, RHS = i tan (arg z ) = i ç ÷ 2i
è Re (z ) ø —
ì z -z ü 4/3 z 4/3 (z )
ïï ïï Þ z + = (z ) +
æ z -z ö z 1/ 3 (z )1 / 3
= i í 2i ý = i ç ÷
z +z è i (z + z ) ø
ï ï (z ) (z )1 / 3 z (z )1 / 3
ïî 2 ïþ Þ z4/ 3 + 2/3
= (z )4 / 3 + 2/3
z z z
-1
z -z z ( z - 1) - 1 z - 2
= = = = [from Eq. (i)] [Qz 1 / 3 (z )1 / 3 = (z z )1 / 3 = z
2/3
]
z +z z
+ 1 ( z - 1) + 1 z
z 1
Þ z 4 / 3 - (z )4 / 3 - ((z )4 / 3 - (z )4 / 3 ) = 0
= LHS 2/3
z
Aliter
We have, z - 1 = 1 i.e.(z - 1) is unimodular, so we can take é 1 ù
Þ {z 4 / 3 - (z )4 / 3 } ê1 - ú =0
z - 1 = cos q + i sin q êë z
2 /3
úû
\ z - 2 = - 1 + cos q + i sin q 2/3
q q q \ z =1 [Qz ¹ z ]
= - 2 sin 2 + 2i sin cos
2 2 2 Therefore, z =1
2 2 q q q
= 2i sin + 2i sin cos
2 2 2 l Ex. 33 C is the complex numbers f : C ® R is defined by
q æ q qö f ( z ) = z 3 - z + 2 . Find the maximum value of f ( z ), if
or z - 2 = 2i sin ç cos + i sin ÷ …(i)
2 è 2 2ø z =1.
and z = 1 + cos q + i sin q Sol. Q z =1
q q q
= 2 cos 2 + 2i sin cos \ z = e iq
2 2 2
q æ q qö \ f ( e iq ) = e 3iq
- e iq + 2 = e 2 iq
( e iq - e - iq ) + 2
z = 2 cos ç cos + i sin ÷ …(ii)
2 è 2 2ø 2iq
= e × 2i sin q + 2
From Eqs. (i) and (ii), we get
z -2 q = (cos 2q + i sin 2q ) × 2i sin q + 2
= i tan
z 2 = (2 - 2 sin 2q sin q ) + 2i sin q cos 2q
z -2 = 2 (1 - sin 2q sin q ) + i sin q cos 2q
Therefore, = i tan (arg z ) [Q arg (z ) = q /2 from Eq. (ii)]
z = 2 (1 - sin 2q sin q )2 + (sin q cos 2q )2
56 Textbook of Algebra

= 2 (1 + sin 2 q - 2 sin q sin 2q ) II. Aliter


Here, OA = OB …(i)
= 2 1 + sin 2 q (1 - 4 cos q )
From Rotation theorem,
= 2 1 + (1 - cos q ) (1 + cos q ) (1 - 4 cos q ) z1 - 0 OA 2 pi / 3
= e
1 é 1ù z 2 - 0 OB
For maximum value, cosq = - êQ cos q ¹ - 1, 1, 4 ú
2 ë û z1 æ 2p 2p ö
Þ = ç cos + i sin ÷ [from Eq. (i)]
z2 è 3 3 ø
æ3ö æ1ö
\ Maximum value of f (z ) = 2× 1 + ç ÷ ç ÷ (3) = 13 1 i 3
è2ø è2ø Þ
z1
=- +
z2 2 2
l Ex. 34 Prove that the complex numbers z 1 and z 2 and æ z1 1 ö i 3
2p Þ ç + ÷= …(ii)
the origin form an isosceles triangle with vertical angle , if èz2 2ø 2
2 2 3
z 1 + z 2 + z 1 z 2 = 0. On squaring both sides in Eq. (ii), we get
2 2 2
Sol. Given, z1 + z 2 + z1z 2 = 0 Þ
z1
+
1 z1
+ =-
3
2
Þ 2
( z 1 - wz 2 ) ( z 1 - w z 2 ) = 0 z2 4 z2 4
2
Þ z1 = wz 2 or z1 = w z 2 2 z1 z1
Þ 2
+ +1=0
In the first case, z1 = wz 2 Þ z1 = w z 2 Þ z1 = z 2 z2 z2
2 2
Hence, two sides equal Þ z1 + z1z 2 + z 2 = 0
2 2
amp (z1 ) = amp (w) z 2 = amp (w) + amp (z 2 ) \ z1 + z 2 + z1z 2 = 0
2p
Þ amp (z1 ) = + amp (z 2 )
3 l Ex. 35 If a = e 2 pi / 7 , where i = -1 and
2p
Þ amp (z1 ) - amp (z 2 ) = 20
3 f (x ) = A 0 + å A k x k , then find the value of
2p k =1
So, the angle between two sides is .
3 f ( x ) + f (ax ) + f (a 2 x ) + ... + f (a 6 x ) independent of a.
Similarly, the other case Sol. Q a = e 2 pi / 7
I. Aliter
2 2 \ a 7 = e 2 pi = cos 2p + i sin 2p = 1 + 0 = 1 or a = (1)1/ 7
Given, z1 + z 2 + z1z 2 = 0
\ 1, a , a 2 , a 3 , a 4 , a 5 , a 6 are the seven, 7th roots of unity.
Þ (z1 - wz 2 ) (z1 - w2z 2 ) = 0
20 20
Þ z1 = wz 2 or z1 = w2z 2 Q f (x ) = A0 + å Ak x k = å Ak x k
k =1 k =0
In the first case, z1 = wz 2
é 2 pi ù
Now, f ( x ) + f (ax ) + f (a 2 x ) + ... + f (a 6 x )
Þ ( z 1 - 0) = ( z 2 - 0) e 2 pi / 3 êQ w = e 3 ú 20
êë úû
¾® ¾®
= å Ak x k [( (1)k + (a )k + (a 2 )k + ... + (a 6 )k ]
k =0
Þ OA = OB e 2 pi / 3
¾® ¾® = A 0 x 0 (7 ) + A 7 x 7 (7 ) + A14 x 14 (7 )
2p
i.e., OA is obtained by rotating OB through angle of . = 7 ( A 0 + A 7 x 7 + A14 x 14 )
3
O é k k 2 k 6 k
ê Q(1) + (a ) + (a ) + ... + (a )
2π/3 ë
ì 7, k is multiple of 7 ù
=í ú
î 0, k is not multiple of 7 û
B(z2) A(z1)

2p
Þ OA = OB and ÐAOB = l Ex. 36 Show that all the roots of the equation
3
a 1 z 3 + a 2 z 2 + a 3 z + a 4 = 3, (where a i £ 1, i = 1, 2, 3, 4) lie
Thus, triangle formed by z1, z 2 and origin is isosceles with
2p outside the circle with centre at origin and radius 2 / 3.
vertical angle .
3 Sol. Given that, a1z 3 + a 2 z 2 + a 3z + a 4 = 3
Chap 01 Complex Numbers 57

We have, 3 = a1z 3 + a 2 z 2 + a 3z + a 4 On squaring both sides, we get


2
3 £ a1z 3 + a 2 z 2 + a 3z + a 4 (z 3 - z 2 )2 a æ a aö
= - 4 sin 2 ç cos + i sin ÷
( z 2 - z1 ) 2 2 è 2 2ø
Þ 3 £ a1 z 3 + a2 z 2 + a3 z + a4 a
3 2
= - 4 sin 2 (cos a + i sin a )
Þ 3 £ a1 z + a2 z + a3 z + a4 2
3 2 [from De-Moivre’s theorem]
Þ 3£ z + z + z +1 [Q | ai | £ 1]
(z 3 - z 2 )2 a æ z - z1 ö
Þ 3£1+ z + z
2
+ z
3
< 1+ z + z
2
+ z
3
+ ... = - 4 sin 2 ç 3 ÷ [from Eq. (ii)]
( z 2 - z1 ) 2
2 è z 2 - z1 ø
2 3
Þ 3<1+ z + z + z + ... Therefore, (z 2 - z 3 )2 = 4 sin 2 (a / 2) (z 3 - z1 ) (z1 - z 2 )
1 1
= Þ 3< Aliter
1 - |z | 1- z æp a ö
Q ÐABC = ç - ÷
è2 2ø
1 2
Þ 1- z < Þ - z <0 From Coni method, we have
3 3
æp aö
Þ z > 2 / 3 or z - 0 > 2 / 3 z1 - z 2 AB i çè 2 - ÷

Hence, all the roots lie in the exterior of circle, = e …(i)
z 3 - z 2 BC
z - 0 = 2 / 3.
æp a ö
and ÐACB = ç - ÷
l Ex. 37 If A, B and C represent the complex numbers è2 2ø
z 1 , z 2 and z 3 respectively on the complex plane and the From Coni method, we have
æp aö
1 z 2 - z 3 BC i çè 2 - 2 ÷ø
angles at B and C are each equal to ( p - a ), then prove = e …(ii)
2 z1 - z 3 AC
a
that ( z 2 - z 3 ) = 4 ( z 3 - z 1 ) ( z 1 - z 2 ) sin 2 æç ö÷ .
2 On dividing Eq. (ii) by Eq. (i), we get
è2ø (z 2 - z 3 )2 ( BC )2 æ BC ö
2
= =ç ÷ [Q AB = AC ]
Sol. It is given that, (z 3 - z1 ) (z1 - z 2 ) AB × AC è AB ø
2
p a æ ö
Ð ABC = ÐACB = - ç ÷
2 2 sin a
=ç ÷ [using sine rule]
Þ ÐA = a ç æp a ö÷
ç sin ç - ÷
\ AC = AB …(i) è è 2 2 ø ÷ø
So, D ABC is an isosceles triangle. 2
Considering rotation of AB about A through angle a, we get æ æa ö æa ö ö
ç 2 sin ç ÷ cos ç ÷ ÷
è 2 ø è 2 ø÷ æa ö
A(z1) =ç = 4 sin 2 ç ÷
ç æa ö ÷ è2ø
ç cos ç ÷ ÷
a è è2ø ø
Therefore, (z 2 - z 3 )2 = 4 (z 3 - z1 ) (z1 - z 2 ) sin 2 a / 2

p– a p– a l Ex. 38 If z 1 and z 2 are two complex numbers such that


2 2 2 2
B(z2) C(z3) z1 - z 2 iz
= 1, then prove that 1 = k , where k is a real
z 3 - z1 z1 + z 2 z2
Þ =eia
z 2 - z1 number. Find the angle between the lines from the origin to
the points z 1 + z 2 and z 1 - z 2 in terms of k.
z 3 - z1
Þ = cos a + i sin a …(ii) z1 - z 2
z 2 - z1 Sol. (i) Given, =1
z1 + z 2
z 3 - z1
- 1 = cos a - 1 + i sin a z1
z 2 - z1 -1
z2
z3 - z2 a a a Þ =1
Þ = - 2 sin 2 + 2i sin cos z1
+1
z 2 - z1 2 2 2 z2
z3 - z2 a æ a aö
or = 2i sin ç cos + i sin ÷ …(iii) Þ
z1 z
-1 = 1 +1 …(i)
z 2 - z1 2 è 2 2ø z2 z2
58 Textbook of Algebra

On squaring Eq. (i) both sides, we have (ii) Now, let the angle between OB and OA be q, then from
z1
2
æz ö z
2
æz ö Coni method,
+ 1 - 2 Re ç 1 ÷ = 1 + 1 + 2 Re ç 1 ÷ z1 + z 2 - 0 OB iq
z2 èz2 ø z2 èz2 ø = e
z1 - z 2 - 0 OA
æz ö
Þ 4 Re ç 1 ÷ = 0 z 1 + z 2 iq
èz2 ø = e
z1 z1 - z 2
Þ is purely imaginary number
z2 æ z1 + z 2 ö iq
Þ ç ÷ =e [from Eq. (i)]
z z è z1 - z 2 ø
Þ 1 can be written as i 1 = k , where k is a real number.
z2 z2
æ z1 ö
(ii) Let q be the angle between z1 - z 2 and z1 + z 2 , then ç + 1÷ - ki + 1
z ÷ = e iq Þ
Þ ç 2 = e iq [from Eq. (ii)]
Q(z1 + z2) ç 1 - 1÷
z - ki - 1
ç ÷
è z2 ø
B(z1 + z2)

θ
O P(z1 – z2)

æ z1 ö θ
æ z1 + z 2 ö ç + 1÷ æ - ik + 1 ö
O A(z1 – z2)
z
q = arg ç ÷ = arg ç ÷ = arg ç
2
÷ - 1 + ki
è z1 - z 2 ø ç z1 - 1 ÷ è - ik - 1 ø Þ = e iq
ç ÷ 1 + ki
è z2 ø
æ k 2 - 1 + 2ik ö ( - 1 + ki ) (1 - ki )
æ -1 + ik ö Þ = e iq
= arg ç ÷ = arg ç ÷ (1 + ki ) (1 - ki )
è 1 + ik ø è k 2 +1 ø
( k 2 - 1) 2ki
-1 æ 2k ö Þ + = e iq
Therefore, q = tan ç 2 ÷ ( k 2 + 1) 1 + k2
èk - 1ø
k2 - 1
z1 - z 2 \ Re (e iq ) = cos q =
Aliter (i) Given, =1 …(i) k2 + 1
z1 + z 2
2k
z1 - z 2 cos a + i sin a and Im(e iq ) = sin q =
Let = k2 + 1
z1 + z 2 1
2k
(z1 - z 2 ) + (z1 + z 2 ) 1 + cos a + i sin a \ tan q = 2
Þ = [ by k -1
(z1 + z 2 ) - (z1 - z 2 ) 1 - cos a - i sina
æ 2k ö
componendo and dividendo] Therefore, q = tan - 1 ç 2 ÷
èk -1ø
æa ö æa ö æa ö
2 cos 2 ç ÷ + 2i sin ç ÷ cos ç ÷
z1 è2ø è2ø è2ø
Þ = l Ex. 39 If z = x + iy is a complex number with rationals x
z2 2 æa ö æa ö æa ö
2 sin ç ÷ - 2i sin ç ÷ cos ç ÷
è2ø è2ø è2ø and y and z =1, then show that z 2 n - 1 is a rational
æa ö æ æa ö æa öö number for every n Î N .
2 cos ç ÷ ç cos ç ÷ + i sin ç ÷ ÷
è2ø è è2ø è 2 øø
Þ = Sol. Since, z = 1, where z is unimodular
æa ö æ æa ö æa öö \ z = cos q + i sin q
-2i sin ç ÷ ç cos ç ÷ + i sin ç ÷ ÷
è2ø è è2ø è 2 øø As x and y are rational, cos q , sinq are rationals
æa ö æ 1ö
zn - z - n
n
cot ç ÷ \ z 2 n - 1 = z n çz n - n ÷ = z
z1 è2ø è z ø
Þ =-
z2 i = 1 2i sin nq
iz1 æa ö = 2 sin nq
Þ = - cot ç ÷ = k ( say) = real
z2 è2ø Since, sinnq is rational, therefore z 2 n - 1 is a rational
iz1
Hence, =k ...(ii) number.
z2
Chap 01 Complex Numbers 59

l Ex. 40 If a is a complex number such that a =1, then Therefore, z lies on the right bisector of the segment
connecting the points 0 + i × 0 and - 1 + 0 × i. Thus,
find the value of a, so that equation az 2 + z + 1 = 0 has one Re (z ) = - 1 / 2. Hence, roots are collinear and will have
purely imaginary root. their real parts equal to - 1 / 2. Hence, sum of the real parts
æ 1 ö
Sol. We have, az 2 + z + 1 = 0 …(i) of roots is ç - (n - 1)÷.
è 2 ø
On taking conjugate both sides, we get
Aliter
az 2 + z + 1 = 0
Q z n = (z + 1)n
_
n
Þ a(z )2 + z + 1 = 0 æz + 1ö z +1
Þ ç ÷ = 1 or = (1)1 / n
Þ a ( -z ) 2 + ( -z ) + 1 = 0 è z ø z
[since, z is purely imaginary, z = - z ] = (cos 0 + i sin 0)1 /n
or a z2 - z + 1 = 0 …(ii) = (cos 2r p + i sin 2r p)1/ n
Eliminating z from Eqs. (i) and (ii) by cross-multiplication 1 æ 2rp ö æ 2r p ö 2 r pi / n
rule, we get Þ 1+ = cos ç ÷ + i sin ç ÷=e
z è n ø è n ø
( a - a )2 + 2 (a + a ) = 0 r pi
1 æ pr ö
On dividing each by 4, we get or = (e 2r pi / n - 1) = e n × 2i sin ç ÷
2
z èn ø
æa - aö æa + a ö pr i
ç ÷ +ç ÷ =0 æ1ö 1 -
è 2 ø è 2 ø or z =-ç ÷i× ×e n
2
è2ø æ pr ö
æa - a ö æa + a ö sin ç ÷
Þ -ç ÷ +ç ÷ =0 èn ø
è 2i ø è 2 ø
æ pr pr ö
or - ( Im (a ))2 + Re (a ) = 0 …(iii) ç cos - i sin ÷
æi ö è n n ø
=-ç ÷×
Given, a =1 è2ø pr
sin
Let a = cos a + i sin a n
\ Re (a ) = cos a , Im (a ) = sin a 1
\ Re (z ) = - [ here r ¹ 0]
Then, from Eq. (iii), we get 2
- sin 2 a + cos a = 0 or cos 2 a + cos a - 1 = 0 where, r = 1, 2, 3, ... , n - 1
1 1 1
-1± 1+ 4 Sum of real parts of z = - - - - ... - (n - 1) times
\ cosa = 2 2 2
2
1
5 -1 = - (n - 1).
Only feasible value of cos a = 2
2
Hence, a = cos a + i sin a l Ex. 42 Prove that the angle between the line a z + a z = 0
æ 5 -1ö
where, a = cos - 1 ç ÷ and its reflection in the real axis is
è 2 ø ì 2 Re (a ) Im (a ) ü
q = tan -1 í

.
2
l Ex. 41 If n Î N >1, find the sum of real parts of the roots î {Im (a )} - {Re (a )} þ
Sol. Let z = x + iy , then equation a z + a z = 0 can be written as
of the equation z n = ( z + 1) n .
( a + a) x + i ( a - a) y = 0
Sol. The equation z n = (z + 1)n will have exactly n - 1 roots. æa + a ö æa - a ö
We have, Þ ç ÷x+ç ÷y =0
è 2 ø è 2i ø
n n
æz + 1ö æz + 1ö Þ {Re (a )} x + {Im (a )} y = 0
ç ÷ =1 Þ ç ÷ = 1
è z ø è z ø {Re (a )}
\ Slope of the given line (m ) = -
z +1 {Im (a )}
Þ =1 {Re (a )} {Re (a )}
|z | Then, tan (180°- f ) = - Þ tan f =
{Im (a )} {Im (a )}
Þ z +1 = z
Þ z - ( - 1) = z - 0 Hence, angle between the given line and its reflection in
real axis
60 Textbook of Algebra

æ3 4ö
\ z P = 20 (cos q + i sin q ) = 20 ç + i ÷

Imaginary axis
è5 5ø
\ z P = 12 + 16i
Inc
ide ra y Similarly, zQ = - 12 + 16i
nt c t ed
az+ ra fle From the figure, E is the point with least modulus and D is
az= y Re
0φ φ the point with maximum modulus.
Real axis ¾® ¾® ¾®
O φ Hence, z E = OE = OC - EC = 25i - 15i = 10i
¾® ¾® ¾®
and z D = OD = OC + CD = 25i + 15i = 40i
ì 2 tan f ü
= 2f = tan -1 {tan 2f } = tan -1 í 2 ý
î 1 - tan f þ
l Ex. 44 Two different non-parallel lines meet the circle
ì | z | = r in the points a , b and c , d , respectively. Prove that these
{Re (a )} ü
ï 2 ï a -1 + b -1 - c -1 - d -1
ï {Im(a )} ï ì 2 Re (a ) Im (a ) ü lines meet in the point z given by z = ,
= tan -1 í 2ý
= tan -1 í 2 2 ý a -1b -1 - c -1d -1
ï 1 - {Re (a )} ï î {Im (a )} - {Re(a )} þ
ïî where a, b, c, d are complex constants.
{Im(a )} 2 ïþ
Sol. Let two non-parallel straight lines PQ , RS meet the circle
| z | = r in the points a, b and c , d , then
l Ex. 43 Among the complex numbers z which satisfies | a | = r , | b | = r , | c | = r and | d | = r
| z - 25i | £ 15, find the complex numbers z having
P a
(i) least positive argument. b S
(ii) maximum positive argument.
z
(iii) least modulus. d Q
R c
(iv) maximum modulus.
Sol. The complex numbers z satisfying the condition or |a |2 = |b |2 = |c |2 = |d |2 = r 2
| z - 25i | £ 15 …(i) Þ aa =bb =c c =d d =r2
r2 r2 r2 r2
D 40i \ a= ,b = ,c = and d =
Tangent from a b c d
origin For line PQ, points a, b and z are collinear, then
C 25i
z z 1
θ
Q P a a 1 =0
E b b 1
φ
θ Þ z ( a - b ) - z (a - b ) + (ab - ab ) = 0
O N
ær r2ö2 æ ar 2 br 2 ö
Þ z ç - ÷ - z (a - b ) + ç - ÷ =0
are represented by the points inside and on the circle of èa bø è b a ø
radius 15 and centre at the point C (0, 25). On dividing both sides by (b - a ), we get
The complex numbers having least positive argument and r2 r2
maximum positive arguments in this region are the points z +z - (a + b ) = 0
ab ab
of contact of tangents drawn from origin to the circle.
z z (a + b )
Here, q = Least positive argument Þ + 2 - =0 …(i)
ab r ab
and f = Maximum positive argument
Similarly, for line RS, we get
\ In DOCP , OP = (OC )2 - (CP )2 = (25)2 - (15)2 = 20 z z (c + d )
+ 2 - =0 …(ii)
OP 20 4 cd r cd
and sin q = = =
OC 25 5 On subtracting Eq. (ii) from Eq. (i), we get
4 æ4ö æ1 1 ö (a + b ) (c + d )
\ tan q = Þ q = tan -1 ç ÷ zç - ÷- + =0
3 è3ø è ab cd ø ab cd
Thus, complex number at P has modulus 20 and argument Þ z (a -1b -1 - c -1d -1 ) = a -1 + b -1 - c -1 - d -1
æ4ö
q = tan -1 ç ÷ a -1 + b -1 - c -1 - d -1
è3ø Therefore, z=
a -1b -1 - c -1d -1
Chap 01 Complex Numbers 61

l Ex. 45 If n is an odd integer but not a multiple of 3, then Sol. (i) Since, | z - 1 | + | z + 1 | = 4
prove that xy ( x + y ) ( x 2 + y 2 + xy ) is a factor of i.e., (distance of z from the point 1 + 0 × i ) + (distance
(x + y )n - x n - y n . of z from the point - 1 + 0 × i ) = 4 (constant)
i.e., The sum of the distances of z from two fixed
Sol. We have, xy ( x + y ) ( x 2 + y 2 + xy ) = xy ( x + y )
points 1 + 0 ×i and - 1 + 0 × i is constant, which is the
( x - w y ) ( x - w2y ) definition of an ellipse.
and let f ( x , y ) = ( x + y )n - x n - y n …(i) Therefore, locus of z satisfying the given condition
On putting x = 0 in Eq. (i), we get will be an ellipse with foci at 1 + 0 ×i and - 1 + 0 × i and
f (0, y ) = y n - 0 - y n = 0 centre at origin.
(ii) Given that,
\ x - 0 is a factor of Eq. (i). p
On putting y = 0 in Eq. (i), we get arg(z + i ) - arg(z - i ) =
2
f ( x , 0) = x n - x n = 0 æz + i ö p
\ y - 0 is a factor of Eq. (i).
or arg ç ÷= …(i)
èz - i ø 2
On putting x = - y in Eq. (i), we get
Let the points A and B have affixes i and - i and the
f ( - y , y ) = ( - y + y )n - ( - y )n - y n point P has affix z. Then, Eq. (i) can be written as
= 0 - ( - y )n - y n = y n - y n = 0 [because n is odd] p é æz + i öù
ÐBPA = êQ ÐBPA = arg ç ÷ú
\ x + y is a factor of Eq. (i). 2 ë èz - i øû
On putting x = wy in Eq. (i), we get
Thus, locus of P (z ) is such that the angle subtended at
f ( wy , y ) = ( wy + y )n - ( wy )n - y n p
P by the line joining points A and B is . This is the
= y n [( w + 1)n - wn - 1] 2
definition of a circle with diameter AB.
= y n [( - w2 )n - wn - 1)] [Q1 + w + w2 = 0]
Y
= - y n { w2 n + wn + 1} [because n is odd]
A(i)
Since, n is odd but not a multiple of 3, then n = 3k + 1 or
P(z)
n = 3k + 2 , where k is an integer. π/2
\ w2 n + wn + 1 = 0 [in both cases] …(ii)
X
\ f ( wy , y ) = 0 O
\ x - wy is a factor of Eq. (i).
On putting x = w2y in Eq. (i), we get
B(–i)
f ( w2y , y ) = ( w2y + y )n - ( w2y )n - y n
= y n {( w2 + 1)n - w2 n - 1} Therefore, locus of point z is a circle with diameter AB
n n 2n and centre at origin with radius 1.
= y {( - w) - w - 1}
(iii) We have, 1 < | z - 2 - 3i | < 4 represents a circle with
= - y n { wn + w2 n + 1} [because, n is odd]
centre at ( 2, 3) and radius r Î(1, 4 ).
=0 [from Eq. (ii)]
\ x - w2y is a factor of Eq. (i).
Combining all the factors, we get
1
( x - 0) (y - 0) ( x + y ) ( x - wy ) ( x - w2y ) C 4
2 2
Therefore, xy ( x + y ) ( x + xy + y ) is a factor of (2, 3)

f ( x , y ) = ( x + y )n - x n - y n .
l Ex. 46 Interpret the following equations geometrically on
the Argand plane. Since, | z - 2 - 3i | > 1 represents the region in the
plane outside the circle.
p
(i) | z - 1 | + | z + 1 | = 4 (ii) arg ( z + i ) - arg ( z - i ) = \ | z - 2 - 3i | = 1 …(i)
2
p p and | z - 2 - 3i | < 4 represents the region inside circle.
(iii) 1 < | z - 2 - 3i | < 4 (iv) < arg ( z ) <
4 3 \ | z - 2 - 3i | = 4 …(ii)
ì | z - 1| + 4 ü Hence, 1 < | z - 2 - 3i | < 4 represent the angular space
(v) log cos p/ 3 í ý>1
î 3 | z - 1 | - 2þ between the two circles (i) and (ii).
62 Textbook of Algebra

p p On multiplying Eqs. (i) and (ii), we get


(iv) We have, < arg (z ) <
4 3 ( z1 - z 2 ) ( z 2 - z 3 ) = ( z1 - z 3 ) 2
æy ö
Let z = x + iy Þ arg (z ) = tan -1 ç ÷ 2 2 2
èx ø or z1 + z 2 + z 3 = z1z 2 + z 2 z 3 + z 3z1
Y Hence, the triangle whose vertices are z1, z 2 and z 3 is
equilateral.
Similarly, it can be shown that the triangle whose vertices
√3x

are z1 ¢ , z 2 ¢ and z 3 ¢ is also equilateral.


y=

x
l Ex. 48 Show that the triangle whose vertices are
y= z 1 , z 2 , z 3 and z 1 ¢ , z 2 ¢ , z 3 ¢ are directly similar, if
z1 z1¢ 1
π/3 π/4
O X
z 2 z 2 ¢ 1 = 0.
z3 z3 ¢ 1
The given inequality can be written as
p Sol. Let A , B, C be the points of affix z1, z 2 , z 3 and A ¢ , B ¢ , C ¢ be
æy ö p
< tan -1 ç ÷ < the points of affix z1 ¢ , z 2 ¢ , z 3 ¢.
4 èx ø 3
Since, the triangles ABC and A ¢ B ¢ C ¢ are similar, if
p y p ¾® ¾¾®
Þ tan < < tan BC = l B ¢ C ¢
4 x 3
i.e., (z 3 - z 2 ) = l (z 3 ¢ - z 2 ¢ ) …(i)
y
Þ 1< < 3 ¾® ¾¾®
x and CA = l C ¢ A ¢
Þ x <y < 3 x i.e., ( z1 - z 3 ) = l ( z1 ¢ - z 3 ¢ ) …(ii)
This inequality represents the region between the lines A ′ (z1′ )
y = x and y = 3x
A(z1)
ì |z - 1 | + 4 ü
(v) We have, log cos p/ 3 í ý>1
î 3 | z - 1 | - 2þ
ì |z - 1 | + 4 ü α α
or log1 / 2 í ý>1
î 3 | z - 1 | - 2þ B(z2) C(z3) B ′ (z2′) C ′ (z3′ )
|z - 1 | + 4 1 On dividing Eq. (i) by Eq. (ii), we get
Þ <
3 |z - 1 | - 2 2 z3 - z2 z3 ¢ - z2 ¢
=
or 2 |z - 1 | + 8 < 3 |z - 1 | - 2 z1 - z 3 z1 ¢ - z 3 ¢
Þ | z - 1 | > 10 Þ z 3 ( z1 ¢ - z 3 ¢ ) - z 2 ( z1 ¢ - z 3 ¢ )
Hence, the inequality represents exterior of a circle of = z1 ( z 3 ¢ - z 2 ¢ ) - z 3 ( z 3 ¢ - z 2 ¢ )
radius 10 with centre at (1, 0). Þ z1(z 2 ¢ - z 3 ¢ ) - z 2 (z1 ¢ - z 3 ¢ ) + z 3 (z1 ¢ - z 2 ¢ ) = 0
z1 z1 ¢ 1
l Ex. 47 Show that the triangles whose vertices are Hence, z2 z2 ¢ 1 = 0
z 1 , z 2 , z 3 and z 1¢, z 2 ¢ , z 3 ¢ are equilateral, if
z3 z3 ¢ 1
(z 1 - z 2 ) (z 1 ¢ - z 2 ¢ ) = (z 2 - z 3 ) (z 2 ¢ - z 3 ¢ )
Aliter
= (z 3 - z 1 ) (z 3 ¢ - z 1 ¢ )
Since, DABC and DA ¢ B ¢ C ¢ are similar.
Sol. From the first two relations, we have
AB BC
z1 - z 2 z - z3 If = and ÐABC = ÐA ¢ B ¢C ¢ = a [say]
= 2 A ¢B ¢ B ¢C ¢
z 2 ¢ - z 3 ¢ z1 ¢ - z 2 ¢
Then, from Coni method in D ABC and D A ¢ B ¢C ¢, we have
( z1 - z 2 ) + ( z 2 - z 3 ) z - z3
= = 1 z1 - z 2 AB ia
( z 2 ¢ - z 3 ¢ ) + ( z1 ¢ - z 2 ¢ ) z1 ¢ - z 3 ¢ = e …(i)
z 3 - z 2 BC
z1 - z 2 z - z3
\ = 1 …(i) z 1 ¢ - z 2 ¢ A ¢ B ¢ ia
z 2 ¢ - z 3 ¢ z1¢ - z 3¢ and = e …(ii)
z 3 ¢ - z 2 ¢ B ¢C ¢
Also, from the last two relations
( z 2 - z 3 ) ( z 2 ¢ - z 3 ¢ ) = ( z 3 - z1 ) ( z 3 ¢ - z1 ¢ ) …(ii)
Chap 01 Complex Numbers 63

AB BC AB A ¢ B ¢ Þ (a1z1 + a 3z 3 ) = - (a 2z 2 + a 4z 4 ) …(ii)
Since, = \ =
A ¢ B ¢ B ¢C ¢ BC B ¢ C ¢ On dividing Eq. (ii) by Eq. (i), we get
z - z 2 z1 ¢ - z 2 ¢ a1z1 + a 3z 3 a 2 z 2 + a 4 z 4
From Eqs. (i) and (ii), we get 1 = ' = …(iii)
z3 - z2 z3 ¢ - z2 ¢ a1 + a 3 a2 + a4
z1 z1 ¢ 1 Eq. (iii) implies that point O divides PR in the ratio a 3 : a1
On simplifying as in Ist method, we get z 2 z 2 ¢ 1 = 0 and O divides QS in the ratio a 4 : a 2 .
z3 z3 ¢ 1 Let OR = a1k , OP = a 3k , OQ = a 4l , OS = a 2 l
Now, in DOPQ,
l Ex. 49 If w is the nth root of unity and z 1 , z 2 are any two ( PQ )2 = (OP )2 + (OQ )2 - 2 (OP ) (OQ ) cos q
complex numbers, then prove that
n -1 Þ | z1 - z 2 | 2 = a 32k 2 + a 42l 2 - 2a 3a 4lk cos q
2 2 2
å z1 + w z 2 k
= n { z1 + z2 }, where n Î N . \ a1a 2 | z1 - z 2 | 2 = a1a 2 a 23k 2 + a1a 2 a 42l 2
k =0
-2a1a 2 a 3a 4lk cos q
Sol. If 1, w, w2 , w3 , ..., wn - 1 are the n, nth roots of unity, then
2
n -1 n -1 Similarly, a 3a 4 | z 3 - z 4 | = a 3a 4a12k 2
+ a 3a 4a 22l 2
S k
w = 0 and S ( w) = 0k
…(i) - 2a1a 2 a 3a 4lk cos q
k =0 k =0
n -1
From given condition, a1a 2 | z1 - z 2 | = a 3a 4 | z 3 - z 4 | 2
2

S
2
LHS = z1 + wk z 2 \ a1a 2 a 32k 2 + a1a 2 a 42l 2 = a 3a 4a12k 2 + a 3a 4a 22 l 2
k =0
n -1 Þ k 2a 3a1 (a 2 a 3 - a1a 4 ) = l 2a 2 a 4 (a 2 a 3 - a1a 4 )
= S
k =0
k
( z1 + w z 2 ) ( z1 + ( w ) z 2 ) k
Þ (a1k ) (a 3k ) = (a 2 l ) (a 4 l )
n -1
Þ OP × OR = OQ × OS
= å {z1z1 + z1z 2 ( w)k + z1z 2 wk + z 2 z 2 ( wk ) ( w)k }
k =0 So, P , Q , R and S are concyclic.
n -1 n -1 n -1 n -1

= S |z1|2 + k S= 0 z1z 2 ( w)k + S z1z 2 wk + S | z 2 |2


l Ex. 51 If a and b are the roots of
k =0 k =0 k =0 1
n -1 n -1 n -1 n -1
z + = 2 (cos q + i sin q ), where 0 < q < p and i = -1 ,
z
= | z1| 2 S 1 + z1z 2 k S= 0 ( w)k
k =0
+ z1z 2 S ( w)k + |z 2 |2 k S= 0 1
k =0
show that | a - i | = | b - i |.
1
Sol. Since, z + = 2 (cos q + i sin q )
= n | z1| 2 + 0 + 0 + n | z | 2 [from Eq. (i)] z
= n {| z1 | 2 + | z 2 | 2 } = RHS 1
\ z+ = 2e iq Þ z 2 - 2e iqz + 1 = 0
z
4 4 2e iq ± ( 4e 2 i q - 4 )
l Ex. 50 Let S a i = 0 and S a j z j = 0, then prove that Þ z=
i =1 j =1 2
iq 2 iq
z 1 , z 2 , z 3 and z 4 are concyclic, if Þ z = e ± (e - 1) Þ z = e i q ± e i q × 2i sin q
a 1a 2 | z 1 - z 2 | 2 = a 3 a 4 | z 3 - z 4 | 2 . Þ z - i = e i q - i ± e i q × 2i sin q
4
Sol. Q S ai = 0 = (e i q - e i p/ 2 ) ± e i (q + p/ 2 ) × 2sin q
i =1 P(z1) æ q pö æ q pö
iç + ÷ ç - ÷ q /2+ p /4
è 2 4ø
=e × 2i sin è 2 4ø
± e i( )
× 2sin q

θ ì æq p ö ü
= e i (q / 2 + p/ 4 )í2i sin ç - ÷ ± 2sin q ý
Q(z2)
O î è 2 4 ø þ
θ S(z4)
æq p ö
\ | z - i | = 1 × 4 sin 2 ç - ÷ + 2 sin q
è2 4ø
R(z3) æ p öö
æ
\ a1 + a 2 + a 3 + a 4 = 0 = 2 ç1 - cos çq - ÷ ÷ + 2 sin q
è è 2 øø
Þ (a1 + a 3 ) = - (a 2 + a 4 ) …(i)
4
= 2 - 2sin q + 2sin q = 2
and S aj z j = 0
j =1
Þ | a - i | = | b - i | = 2 [here, a ,b are two values of z - i ]
\ a1z1 + a 2 z 2 + a 3z 3 + a 4 z 4 = 0
#L Complex Numbers Exercise 1 :
Single Option Correct Type Questions
n This section contains 30 multiple choice questions. 8. Let a and b be two fixed non-zero complex numbers and
Each question has four choices (a), (b), (c) and (d) out of z is a variable complex number. If the lines
which ONLY ONE is correct a z + a z + 1 = 0 and b z + b z − 1 = 0 are mutually
1. If cos (1 − i ) = a + ib, where a, b ∈ R and i = − 1, then perpendicular, then
1 1 1 1 (a) ab + ab = 0 (b) ab − ab = 0
(a) a = e −  cos 1, b = e +  sin 1
2 e 2 e (c) ab − ab = 0 (d) ab + ab = 0
1 1 1 1
8π   8π 
9. If α = cos 
(b) a = e +  cos 1, b = e −  sin 1
2 e 2 e  + i sin   , where i = − 1, then
 11   11 
1 1 1 1
(c) a = e +  cos 1, b = e +  sin 1 Re (α + α 2 + α 3 + α 4 + α 5 ) is
2 e 2 e
1 1 1 1 1 1
(d) a = (a) (b) − (c) 0 (d) None of these
e −  cos 1, b = e −  sin 1 2 2
2 e 2 e
10. The set of points in an Argand diagram which satisfy both
2. Number of roots of the equation z 10 − z 5 − 992 = 0, where π
real parts are negative, is | z | ≤ 4 and 0 ≤ arg (z ) ≤ , is
(a) 3 (b) 4 (c) 5 (d) 6 3
(a) a circle and a line (b) a radius of a circle
3. If z and z represent adjacent vertices of a regular (c) a sector of a circle (d) an infinite part line
polygon of n sides with centre at origin and if
Im (z ) 11. If f ( x ) = g ( x 3 ) + xh( x 3 ) is divisible by x 2 + x + 1, then
= 2 − 1, the value of n is equal to (a) g( x ) is divisible by ( x − 1 ) but not h( x )
Re (z )
(b) h( x ) is divisible by ( x − 1 ) but not g( x )
(a) 2 (b) 4 (c) 6 (d) 8
r
(c) both g ( x ) and h ( x ) are divisible by ( x − 1 )
4. If Π e ipθ = 1, where Π denotes the continued product
p =1
(d) None of the above
12. If the points represented by complex numbers
and i = − 1, the most general value of θ is z 1 = a + ib, z 2 = c + id and z 1 − z 2 are collinear, where
2nπ 2nπ i = − 1, then
(a) , n ∈I (b) , n ∈I
r (r − 1 ) r (r + 1 ) (a) ad + bc = 0 (b) ad − bc = 0
4nπ 4nπ (c) ab + cd = 0 (d) ab − cd = 0
(c) , n ∈I (d) , n ∈I
r (r − 1 ) r (r + 1 )
13. Let C denotes the set of complex numbers and R is the
(where, n is an integer)
set of real numbers. If the function f : C → R is defined
5. If (3 + i ) (z + z ) − (2 + i ) (z − z ) + 14 i = 0, where i = − 1, by f (z ) = | z |, then
then z z is equal to (a) f is injective but not surjective
(a) 10 (b) 8 (b) f is surjective but not injective
(c) − 9 (d) – 10 (c) f is neither injective nor surjective
(d) f is both injective and surjective
6. The centre of a square ABCD is at z = 0, A is z 1 . Then,
the centroid of ∆ABC is 14. Let α and β be two distinct complex numbers, such that
z1 | α | = | β |. If real part of α is positive and imaginary part
(a) z1 (cos π ± i sin π ) (b) (cos π ± i sin π )
3 of β is negative, then the complex number
 π π z1  π π (α + β ) / (α − β ) may be
(c) z1  cos ± i sin  (d)  cos ± i sin 
 2 2 3  2 2 (a) zero (b) real and negative
(where, i = − 1) (c) real and positive (d) purely imaginary
15. The complex number z, satisfies the condition
3 −i
7. If z = , where i = − 1, then (i 101 + z 101 )103 equals  z − 25  = 24. The maximum distance from the origin of
2
 z
to
(a) iz (b) z
coordinates to the point z, is
(a) 25 (b) 30
(c) z (d) None of these
(c) 32 (d) None of these
Chap 01 Complex Numbers 65

16. The points A, B and C represent the complex numbers  z 


z1 − 
z 1 , z 2 , (1 − i ) z 1 + iz 2 respectively, on the complex plane | z | π z 
23. If arg  = and − z 1 = 3, then | z 1 | equals to
 z  2 | z | 
(where, i = − 1 ). The ∆ABC, is  
 |z | 
(a) isosceles but not right angled
(b) right angled but not isosceles (a) 3 (b) 2 2 (c) 10 (d) 26
π
24. If | z − 2 − i | = | z |
sin  − arg z 
(c) isosceles and right angled
, where i = − 1, then
(d) None of the above
 4 
17. The system of equations | z + 1 − i | = 2 and | z | = 3 has locus of z, is
(where, i = − 1) (a) a pair of straight lines (b) circle
(c) parabola (d) ellipse
(a) no solution (b) one solution
(c) two solutions (d) None of these 25. If 1, z 1 , z 2 , z 3 , …, z n − 1 are the n, nth roots of unity, then
n −1
18. Dividing f (z ) by z − i, we obtain the remainder 1 − i 1
and dividing it by z + i, we get the remainder 1 + i.
the value of ∑ (3 − z r )
, is
r =1
Then, the remainder upon the division of f (z ) by z 2 + 1, n ⋅ 3n − 1 1 n ⋅ 3n − 1
(a) n + (b) −1
is 3 −1 2 3n − 1
(a) i + z (b) 1 + z
n ⋅ 3n − 1
(c) 1 − z (d) None of these (c) +1 (d) None of these
3n − 1
19. The centre of the circle represented by | z + 1 | = 2 | z − 1 |
26. If z = (3 + 7i ) ( λ + iµ ), when λ , µ ∈ I ~ { 0 } and i = − 1,
on the complex plane, is
5 is purely imaginary then minimum value of | z | 2 is
(a) 0 (b)
3 (a) 0 (b) 58
1 3364
(c) (d) None of these (c) (d) 3364
3 3

20. If x = 9 1 / 3 ⋅ 9 1 / 9 ⋅ 9 1 / 27 … ∞, y = 4 1 / 3 ⋅ 4 − 1 / 9 ⋅ 4 1 / 27 … ∞ and 27. Given, z = f ( x ) + ig ( x ), where i = −1 and


∞ f , g : (0, 1) → (0, 1) are real-valued functions, which of the
z= ∑ (1 + i ) − r , where i = − 1, then arg ( x + yz ) is following hold good?
r =1 1  1  1  1 
(a) z = +i   (b) z = +i  
equal to 1 − ix  1 + ix  1 + ix  1 − ix 
 2
(a) 0 (b) − tan −1   1  1  1  1 
 3  (c) z = +i   (d) z = +i  
1 + ix  1 + ix  1 − ix  1 − ix 
 2   2
(c) − tan −1   (d) π − tan −1   28. If z 3 + (3 + 2i ) z + ( − 1 + ia ) = 0, where i = −1, has one
 3  3 
real root, the value of a lies in the interval (a ∈ R )
21. If centre of a regular hexagon is at origin and one of the (a) ( − 2, − 1 ) (b) ( − 1, 0 )
vertices on Argand diagram is 1 + 2i, where i = − 1, (c) ( 0, 1 ) (d) (1, 2 )
then its perimeter is 29. If m and n are the smallest positive integers satisfying
m n
(a) 2 5 (b) 4 5  π  π
(c) 6 5 (d) 8 5 the relation 2 CiS  =  4 CiS  , where i = − 1,
 6  4
 n  (m + n ) equals to
22. Let | z r − r | ≤ r , ∀ r = 1, 2, 3, …, n, then ∑ z r is less than (a) 60 (b) 72 (c) 96 (d) 120
r = 1 
30. Number of imaginary complex numbers satisfying the
(a) n (b) 2n
n(n + 1 ) equation, z 2 = z ⋅ 21 − | z | is
(c) n(n + 1 ) (d)
2 (a) 0 (b) 1 (c) 2 (d) 3
66 Textbook of Algebra

#L Complex Numbers Exercise 2 :


More than One Option Correct Type Questions
n This section contains 15 multiple choice questions. z1 − z 4
(a) is purely real
Each question has four choices (a), (b), (c) and (d) out of z2 − z3
which MORE THAN ONE may be correct. z − z3
(b) 1 is purely imaginary
z +1 z2 − z4
31. If is a purely imaginary number (where i = − 1),
z +i (c) | z1 − z 3 | ≠ | z 2 − z 4 |
then z lies on a  z − z4  z − z4 
(a) straight line (d) amp  1  ≠ amp  2 
z2 − z4  z3 − z4 
(b) circle
(c) circle with radius =
1 38. If | z − 3 | = min {| z − 1 |, | z − 5 | }, then Re(z ) is equal to
2 (a) 2 (b) 2.5 (c) 3.5 (d) 4
(d) circle passing through the origin
π 2π
39. If arg (z + a ) = and arg (z − a ) = (a ∈ R + ), then
32. If z satisfies | z − 1 | < | z + 3 |, then ω = 2 z + 3 − i (where, 6 3
i = − 1) satisfies (a) | z | = a (b) | z | = 2a
π π
(a) | ω − 5 − i | < | ω + 3 + i | (b) | ω − 5 | < | ω + 3 | (c) arg (z ) = (d) arg (z ) =
π 3 2
(c) Im (iω ) > 1 (d) | arg (ω − 1 )| <
2 40. If z = x + iy, where i = −1, then the equation
33. If the complex number is (1 + ri ) = λ (1 + i ), when  (2z − i )

3
 = m represents a circle, then m can be
i = − 1, for some real λ, the value of r can be  (z + i ) 
π 3π 1
(a) cos (b) cosec (a) (b) 1 (c) 2 (d) ∈(3, 2 3 )
5 2 2
π π
(c) cot (d) tan 41. Equation of tangent drawn to circle | z | = r at the point
12 12
A (z 0 ), is
34. If z ∈ C , which of the following relation(s) represents a z z
circle on an Argand diagram? (a) Re   = 1 (b) Im   = 1
z0 z0
(a) | z − 1 | + | z + 1 | = 3 (b) | z − 3 | = 2
z 
(c) | z − 2 + i | =
7
(d) (z − 3 + i ) ( z − 3 − i ) = 5 (c) Im  0  = 1 (d) z z 0 + z 0 z = 2 r 2
z
3
(where, i = −1 ) 42. z 1 and z 2 are the roots of the equation z 2 − az + b = 0,
35. If 1, z 1 , z 2 , z 3 , …, z n − 1 be the n, nth roots of unity and ω where | z 1 | = | z 2 | = 1 and a, b are non-zero complex
numbers, then
be a non-real complex cube root of unity, then
n −1 (a) | a | ≤ 1 (b) | a | ≤ 2
∏ (ω − z r ) can be equal to (c) arg (a ) = arg (b 2 ) (d) arg (a 2 ) = arg (b )
r =1
(a) 1 + ω (b) − 1 43. If α is a complex constant, such that αz 2 + z + α = 0 has
(c) 0 (d) 1 a real root, then
(a) α + α = 1
36. If z is a complex number which simultaneously satisfies
(b) α + α = 0
the equations
(c) α + α = −1
3 | z − 12 | = 5 | z − 8i | and | z − 4 | = | z − 8 |, where (d) the absolute value of real root is 1
i = − 1 , then Im (z ) can be
44. If the equation z 3 + (3 + i ) z 2 − 3z − (m + i ) = 0, where
(a) 8 (b) 17
(c) 7 (d) 15 i = −1 and m ∈ R, has atleast one real root, value of m is
(a) 1 (b) 2 (c) 3 (d) 5
37. If P (z 1 ), Q(z 2 ), R(z 3 ) and S(z 4 ) are four complex
numbers representing the vertices of a rhombus taken in 45. If z + (3 + 2i ) z + ( − 1 + ia ) = 0, where i = − 1, has one
3

order on the complex plane, which one of the following real root, the value of a lies in the interval (a ∈ R )
is hold good? (a) ( − 2, 1 ) (b) ( − 1, 0 ) (c) ( 0, 1 ) (d) (− 2, 3)
Chap 01 Complex Numbers 67

Complex Numbers Exercise 3 :


Passage Based Questions
n This section contains 4 passages. Based upon each of Passage III
the passage 3 multiple choice questions have to be (Q. Nos. 52 to 54)
answered. Each of these questions has four choices (a),
(b), (c) and (d) out of which ONLY ONE is correct. For any two complex numbers z1 and z 2 ,
| z | − | z 2 |
| z1 − z 2 | ≥  1
Passage I  | z 2 | − | z1 |
(Q. Nos. 46 to 48)
and equality holds iff origin z 1 and z 2 are collinear
π , if arg ( z ) < 0 and z 1 , z 2 lie on the same side of the origin.
arg ( z ) + arg ( − z ) =  , where
 − π , if arg ( z ) > 0 52. If z −  = 2 and sum of greatest and least values of | z |
1
− π < arg ( z ) ≤ π.  z
is λ, then λ 2 , is
46. If arg (z ) > 0, then arg ( − z ) − arg (z ) is equal to (a) 2 (b) 4 (c) 6 (d) 8
π
(a) − π (b) −
53. If z +  = 4 and sum of greatest and least values of | z |
2
2
π  z
(c) (d) π
2 is λ, then λ 2 , is
(a) 12 (b) 18 (c) 24 (d) 30
47. Let z 1 and z 2 be two non-zero complex numbers, such
54. If z −  = 6 and sum of greatest and least values of | z | is
3
that | z 1 | = | z 2 | and arg (z 1 z 2 ) = π, then z 1 is equal to
(a) z 2 (b) z 2  z
(c) − z 2 (d) − z 2 2λ, then λ 2 , is
(a) 12 (b) 18 (c) 24 (d) 30
 z1 
48. If arg ( 4 z 1 ) − arg (5 z 2 ) = π, then is equal to
z2  Passage IV
(a) 1 (b) 1.25 (Q. Nos. 55 to 57)
(c) 1.50 (d) 2.50
Consider the two complex numbers z and w, such that
Passage II z −1
w= = a + ib, where a, b ∈ R and i = − 1.
(Q. Nos. 49 to 51) z+2
Sum of four consecutive powers of i (iota) is zero.
55. If z = C iS θ, which of the following does hold good?
i.e., i n + i n + 1 + i n + 2 + i n + 3 = 0, ∀ n ∈ I . 9b
(a) sin θ =
25 1 − 4a
49. If Σ i n ! = a + ib, where i = −1, then a − b , is 1 − 5a
n =1 (b) cos θ =
1 + 4a
(a) prime number
(b) even number (c) (1 + 5a ) 2 + (3b ) 2 = (1 − 4a ) 2
(c) composite number (d) All of these
(d) perfect number b
95 50 56. Which of the following is the value of − , whenever it
Σ Σ
a
50. If ir + i r ! = a + ib, where i = −1, the unit place exists?
r=−2 r=0
 θ 1  θ
digit of a 2011
+b2012
, is (a) 3 tan   (b) tan  
 2 3  2
(a) 2 (b) 3
1 θ
(c) 5 (d) 6 (c) − cot θ (d) 3 cot
3 2
100 101
51. If Σ i r ! + rΠ= 1 i r
r=4
= a + ib, where i = −1, then a + 75b , is 57. Which of the following equals to | z | ?
(a) | w | (b) (a + 1 ) 2 + b 2
(a) 11 (b) 22
(c) 33 (d) 44 (c) a + (b + 2 )
2 2
(d) (a + 1 ) 2 + (b + 1 ) 2
68 Textbook of Algebra

#L Complex Numbers Exercise 4 :


Single Integer Answer Type Questions
n This section contains 10 questions. The answer to π 1 − π i π −i 
each question is a single digit integer, ranging from 0 63. If z = (1 + i ) 4  +  , where i = −1,
to 9 (both inclusive).
4  π +i 1 + π i
 |z | 
58. The number of values of z (real or complex) then   equals to
 amp (z )
simultaneously satisfying the system of equations
1 + z + z 2 + z 3 + … + z 17 = 0
64. Suppose A is a complex number and n ∈ N , such that
A n = ( A + 1)n = 1, then the least value of n is
and 1 + z + z 2 + z 3 + … + z 13 = 0 is
65. Let z r ; r = 1, 2, 3, …, 50 be the roots of the equation
59. Number of complex numbers z satisfying z 3 = z is 50 50
1
60. Let z = 9 + ai, where i = − 1 and a be non-zero real. ∑ ( z )r = 0. If ∑ (z = − 5λ, then λ equals to
r=0 r =1 r − 1)
p
If Im (z ) = Im (z ), sum of the digits of a is
2 3 2 32  10  2qπ 2qπ  
66. If P = Σ (3p + 2)  Σ sin − i cos  , where
61. Number of complex numbers z, such that | z | = 1 p =1 q = 1  11 11  

i = − 1 and if (1 + i ) P = n (n !), n ∈ N , then the value of n is
and +  = 1 is
z z
z z 67. The least positive integer n for which
1 + x 2 
n
62. If x = a + ib, where a, b ∈ R and i = − 1 and x = 3 + 4i, 2 1 + i  2 −1
  = sin   , where x > 0 and i = −1 is
x 3 = 2 + 11i, the value of (a + b ) is 1 − i  π  2x 

#L Complex Numbers Exercise 5 :


Matching Type Questions
n
This section contains 4 questions. Questions 68 and 69 have three statements (A, B and C) given in Column I and
four statements (p, q, r and s) in Column II and questions 70 and 71 have four statements (A, B, C and D) given in
Column I and five statements (p, q, r, s and t) in Column II. Any given statement in Column I can have correct
matching with one or more statement(s) given in Column II.

68. Column I Column II

If z −  = 2 and if greatest and least values of | z | are G and L respectively, then G − L , is
1
(A) (p) natural number
 z

Ifz +  = 4 and if greatest and least values of | z | are G and L respectively, then G − L , is
2
(B) (q) prime number
 z

If z −  = 6 and if greatest and least values of | z | are G and L respectively, then G − L , is
3
(C) (r) composite number
 z
(s) perfect number

69.
Column I Column II

(A) If (6 + 8 i ) + (− 6 + 8 i ) = z1 , z2 , z3 , z4 (where i = − 1), then | z1 |2 + | z2 |2 | + | z3 |2 + | z4 |2 is divisible by (p) 7

(B) If (5 − 12i ) + (− 5 − 12i ) = z1 , z2 , z3 , z4 (where i = − 1), then | z1 |2 + | z2 |2 + | z3 |2 + | z4 |2 is divisible by (q) 8

(C) If (8 + 15 i ) + (− 8 − 15 i ) = z1 , z2 , z3 , z4(where i = − 1), then | z1 |2 + | z2 |2 + | z3 |2 + | z4 |2 is divisible by (r) 13


(s) 17
Chap 01 Complex Numbers 69

70. 71. Column I Column II


Column I Column
II (A)
If z −  = 5 and maximum and
6 (p) λµ + µ λ = 8
 z
(A) If λ and µ are the unit’s place digits of (p) 2
minimum values of | z | are λ and µ
(143)861 and (5273)1358 respectively,
respectively, then
then λ + µ is divisible by
(B)
If z −  = 6 and maximum and
7 (q) λµ − µ λ = 7
(B) If λ and µ are the unit’s place digits (q) 3
 z
of (212)7820 and (1322)1594
minimum values of | z | are λ and µ
respectively, then λ + µ is divisible
respectively, then
by
(C)
If z −  = 7 and maximum and
8 (r) λµ + µ λ = 7
(C) If λ and µ are the unit’s place digits of (r) 4
 z
(136)786 and (7138)13491 respectively,
minimum values of | z | are λ and µ
then λ + µ is divisible by
respectively, then
(s) 5
(s) λµ − µ λ = 6
(t) 6
(t) λµ + µ λ = 9

#L Complex Numbers Exercise 6 :


Statement I and II Type Questions
n
Directions (Q. Nos. 72 to 78) are Assertion-Reason 75. Statement-1 Locus of z satisfying the equation
type questions. Each of these questions contains two | z − 1 | + | z − 8 | = 5 is an ellipse.
statements:
Statement-1 (Assertion) and Statement-2 (Reason) Statement-2 Sum of focal distances of any point on
Each of these questions also has four alternative ellipse is constant for an ellipse.
choices, only one of which is the correct answer. You 76. Let z 1 , z 2 and z 3 be three complex numbers in AP.
have to select the correct choice as given below.
(a) Statement-1 is true, Statement-2 is true; Statement-2 Statement-1 Points representing z 1 , z 2 and z 3 are
is a correct explanation for Statement-1 collinear.
(b) Statement-1 is true, Statement-2 is true; Statement-2 Statement-2 Three numbers a, b and c are in AP, if
is not a correct explanation for Statement-1 b − a = c − b.
(c) Statement1 is true, Statement-2 is false
77. Statement-1 If the principal argument of a complex
(d) Statement-1 is false, Statement-2 is true
number z is θ, the principal argument of z 2 is 2θ.
72. Statement-1 3 + 7i > 2 + 4i, where i = − 1.
Statement-2 arg (z 2 ) = 2 arg (z )
Statement-2 3 > 2 and 7 > 4
78. Consider the curves on the Argand plane as
73. Statement-1 (cos θ + i sin φ) 3 = cos 3 θ + i sin 3 φ, π
i = −1 C 1 : arg (z ) = ,
2 4
 π π 3π
Statement-2  cos + i sin  = i C 2 : arg (z ) =
 4 4 4
74. Statement-1 Let z 1 , z 2 and z 3 be three complex and C 3 : arg (z − 5 − 5i ) = π, where i = −1.
numbers, such that | 3z 1 + 1 | = | 3z 2 + 1 | = | 3z 3 + 1 | and Statement-1 Area of the region bounded by the curves
1 + z 1 + z 2 + z 3 = 0, then z 1 , z 2 , z 3 will represent vertices 25
of an equilateral triangle on the complex plane. C 1 , C 2 and C 3 is .
2
Statement-2 z 1 , z 2 and z 3 represent vertices of an Statement–2 The boundaries of C 1 , C 2 and C 3 constitute
equilateral triangle, if a right isosceles triangle.
z 12 + z 22 + z 32 + z 1z 2 + z 2 z 3 + z 3 z 1 = 0.
70 Textbook of Algebra

Complex Numbers Exercise 7 :


Subjective Type Questions
n In this section, there are 24 subjective questions. 88. Show that if a and b are real, then the principal value of
79. If z 1 , z 2 and z 3 are three complex numbers, then prove arg (a ) is 0 or π, according as a is positive or negative and
π π
that z 1 Im ( z 2 z 3 ) + z 2 Im ( z 3 z 1 ) + z 3 Im (z 1 z 2 ) = 0. that of b is or − , according as b is positive or negative.
2 2
80. The roots z 1 , z 2 and z 3 of the equation
89. Two different non-parallel lines meet the circle | z | = r .
x 3 + 3ax 2 + 3bx + c = 0 in which a, b and c are complex
One of them at points a and b and the other which is
numbers, correspond to the points A, B, C on the tangent to the circle at c. Show that the point of
Gaussian plane. Find the centroid of the ∆ ABC and 2c −1 − a −1 − b −1
show that it will be equilateral, if a 2 = b. intersection of two lines is .
c − 2 − a −1b −1
81. If 1, α 1 , α 2 , α 3 and α 4 are the roots of x
− 1 = 0, then 5
90. A, B and C are the points representing the complex
prove that
numbers z 1 , z 2 and z 3 respectively, on the complex
ω − α1 ω − α 2 ω − α 3 ω − α 4 plane and the circumcentre of ∆ABC lies at the origin. If
⋅ ⋅ ⋅ = ω, where ω is
ω 2 − α1 ω 2 − α 2 ω 2 − α 3 ω 2 − α 4 the altitude of the triangle through the vertex A meets
a non-real complex root of unity. the circumcircle again at P, prove that P represents the
 z z 
82. If z 1 and z 2 both satisfy the relation z + z = 2 z − 1 and complex number  − 2 3  .
 z1 
π
arg (z 1 − z 2 ) = , find the imaginary part of (z 1 + z 2 ). 91. If | z | ≤ 1 and | ω | ≤ 1, show that
4
83. If ax + cy + bz = X , cx + by + az = Y , bx + ay + cz = Z, | z − ω | 2 ≤ (| z | − | ω | ) 2 + {arg (z ) − arg (ω )} 2 .
show that 92. Let z, z 0 be two complex numbers. It is given that | z | = 1
(i) (a 2 + b 2 + c 2 − bc − ca − ab ) ( x 2 + y 2 and the numbers z, z 0 , z z 0 , 1 and 0 are represented in an
Argand diagram by the points P, P0 , Q, A and the origin,
+ z − yz − zx − xy ) = X
2 2
+Y 2
+ Z − YZ − ZX − XY
2
respectively. Show that ∆POP0 and ∆AOQ are
(ii) (a 3 + b 3 + c 3 − 3abc ) ( x 3 + y 3 + z 3 − 3xyz ) congruent. Hence, or otherwise, prove that
= X 3 + Y 3 + Z 3 − 3XYZ . | z − z 0 | = | z z 0 − 1 |.
93. Suppose the points z 1 , z 2 , …, z n (z i ≠ 0) all lie on one
84. For every real number c ≥ 0, find all complex numbers z
side of a line drawn through the origin of the complex
which satisfy the equation | z | 2 − 2iz + 2c (1 + i ) = 0,
planes. Prove that the same is true of the points
where i = −1 . 1 1 1
, , …, . Moreover, show that
85. Find the equations of two lines making an angle of 45° z1 z 2 zn
with the line (2 − i ) z + (2 + i ) z + 3 = 0, where i = −1 1 1 1
z 1 + z 2 + … + z n ≠ 0 and + +…+ ≠ 0.
and passing through ( − 1, 4 ). z1 z 2 zn

86. For n ≥ 2, show that 94. If a, b and c are complex numbers and z satisfies
2  2 2
 1 + i   1 + i    1 + i   az 2 + bz + c = 0, prove that | a | | b | = a (b ) 2 c and
1 + 
  2   1 +    1 +  
    2     2   | a | = | c | ⇔ | z | = 1.
 
 2n 
95. Let z 1 , z 2 and z 3 be three non-zero complex numbers
1 + i    1 
… 1 +   = (1 + i ) 1 − n  , where i = −1. | z1 | | z 2 | | z 3 |
  2    22  and z 1 ≠ z 2 . If | z 2 | | z 3 | | z 1 | = 0, prove that
 
| z 3 | | z1 | | z 2 |
87. Find the point of intersection of the curves
arg (z − 3i ) = 3π / 4 and arg (2z + 1 − 2i ) = π , where
(i) z 1 , z 2 , z 3 lie on a circle with the centre at origin.
4 2
z  z − z1 
i = − 1. (ii) arg  3  = arg  3  .
z 2  z 2 − z1 
Chap 01 Complex Numbers 71

96. Prove that, if z 1 and z 2 are two complex numbers and π 3π 5π 1


(iii) sin sin sin =
 1 14 14 14 8
c > 0, then | z 1 + z 2 | 2 ≤ (1 + c ) | z 1 | 2 + 1 +  | z 2 | 2 . π 3π 5π 1
 c (iv) tan tan tan =
14 14 14 7
97. Find the circumcentre of the triangle whose vertices are
given by the complex numbers z 1 , z 2 and z 3 . Also, show that
98. Find the orthocentre of the triangle whose vertices are  π
(1 + y ) 7 + (1 − y ) 7 = 14 y 2 + tan 2 
given by the complex numbers z 1 , z 2 and z 3 .  14 
 2 2 3π   2 2 5π 
99. Prove that the roots of the equation y + tan  y + tan 
π 3π 5π  14   14 
8x 3 − 4 x 2 − 4 x + 1 = 0 are cos , cos and cos .
7 7 7 and then deduce that
Hence, obtain the equations whose roots are  π  3π   5π 
tan 2   + tan 2   + tan 2   = 5
π 3π 5π  14   14   14 
(i) sec 2 , sec 2 , sec 2
7 7 7
101. If the complex number z is to satisfy
2 π 2 3π 2 5π
(ii) tan , tan , tan | z | = 3, | z − {a (1 + i ) − i }| ≤ 3 and| z + 2a − (a + 1) i | > 3, where
7 7 7 i = − 1 simultaneously for atleast one z, then find all
π 3π 5π
(iii) Evaluate sec + sec + sec a ∈ R.
7 7 7
100. Solve the equation z 7 + 1 = 0 and deduce that 102. Write equations whose roots are equal to numbers
π 2π 3π nπ
π 3π 5π 1 (i) sin 2 , sin 2 , sin 2 , ..., sin 2 .
(i) cos cos cos =− 2n + 1 2n + 1 2n + 1 2n + 1
7 7 7 8
π 3π 5π π 2π 3π nπ
(ii) cos cos cos =
7 (ii) cot 2 , cot 2 , cot 2 , ..., cot 2 .
14 14 14 8 2n + 1 2n + 1 2n + 1 2n + 1

#L Complex Numbers Exercise 8 :


Questions Asked in Previous 13 Years’ Exams
n
This section contains questions asked in IIT-JEE, π
(a) | z − 1 | > 2; | arg (z − 1 )| <
AIEEE, JEE Main & JEE Advanced from year 2005 4
to year 2017. π
(b) | z − 1 | > 2; | arg (z − 1 )| <
2
103. If ω is a cube root of unity but not equal to 1, then π
(c) | z + 1 | > 2; | arg (z + 1 )| <
minimum value of | a + bω + cω 2 |, (where a, b and c are 4
integers but not all equal), is π
(d) | z + 1 | > 2; | arg (z + 1 )| <
[IIT-JEE 2005, 3M] 3
3
(a) 0 (b) (c) 1 (d) 2 105. If one of the vertices of the square circumscribing the
2 circle | z − 1 | = 2 is 2 + 3i, where i = − 1. Find the
104. PQ and PR are two infinite rays. QAR is an arc. Point other vertices of the square. [IIT-JEE 2005, 4M]
lying in the shaded region excluding the boundary
106. If z 1 and z 2 are two non-zero complex numbers, such
satisfies [IIT-JEE 2005, 3M]
that | z 1 + z 2 | = | z 1 | + | z 2 |, then arg (z 1 ) − arg (z 2 ) is
Y
(–1+√2, √2i) equal to [AIEEE 2005, 3M]
Q (a) − π (b) − π / 2
0) (1,0) (c) π / 2 (d) 0
(–1,
P A
X′
O
X 107. If 1, ω, ω 2 are the cube roots of unity, then the roots of
the equation ( x − 1) 3 + 8 = 0 are [AIEEE 2005, 3M]
R (a) − 1, 1 + 2 ω, 1 + 2 ω 2
(b) − 1, 1 − 2 ω, 1 − 2 ω 2
(–1+√2, –√2 i )
Y′ (c) − 1, − 1, − 1 (d) None of these
72 Textbook of Algebra

108. If ω =
z
and | ω | = 1, where i = − 1, then z lies on 115. The number of elements in the set A ∩ B ∩ C , is
1
z− i (a) 0 (b) 1
3 [AIEEE 2005, 3M] (c) 2 (d) ∞
(a) a straight line (b) a parabola 116. Let z be any point in A ∩ B ∩ C . Then,
(c) an ellipse (d) a circle
| z + 1 − i | 2 + | z − 5 − i | 2 lies between
109. If ω = α + iβ, where β ≠ 0, i = − 1 and z ≠ 1, satisfies the
(a) 25 and 29 (b) 30 and 34
 ω − ωz 
condition that   is purely real, the set of values (c) 35 and 39 (d) 40 and 44
 1 −z 
117. Let z be any point in A ∩ B ∩ C and ω be any point
of z is [IIT-JEE 2006, 3M]
satisfying | ω − 2 − i | < 3. Then, | z | − | ω | + 3 lies between
(a) {z : | z | = 1 } (b) {z : z = z }
(a) − 6 and 3 (b) − 3 and 6
(c) {z : z ≠ 1 } (d) {z : | z | = 1, z ≠ 1 }
(c) − 6 and 6 (d) − 3 and 9
2kπ 2kπ 
110. The value of Σ sin
10
+ i cos  (where i = − 1) 118. A particle P starts from the point z 0 = 1 + 2i, i = −1. It
 k =1 11 11 
moves first horizontally away from origin by 5 units and
is [AIEEE 2006, 3M]
then vertically away from origin by 3 units to reach a
(a) i (b) 1
point z 1 . From z 1 , the particle moves 2 units in the
(c) − 1 (d) − i
direction of the vector i$ + $j and then it moves through
111. If z 2 + z + 1 = 0, where z is a complex number, the value of
π
2 2 2 2 an angle in anti-clockwise direction on a circle with
 1  2 1  3 1  6 1
z +  + z + 2  + z + 3  + … + z + 6  2
 z  z   z   z  centre at origin, to reach a point z 2 , then the point z 2 is
is [AIEEE 2006, 6M] given by [IIT-JEE 2008, 3M]
(a) 18 (b) 54 (a) 6 + 7i (b) − 7 + 6i
(c) 6 (d) 12 (c) 7 + 6i (d) − 6 + 7i
112. A man walks a distance of 3 units from the origin 1
119. If the conjugate of a complex numbers is , where
towards the North-East (N 45° E) direction. From there, i −1
he walks a distance of 4 units towards the North-West i = − 1. Then, the complex number is [AIEEE 2008, 3M]
(N 45° W) direction to reach a point P. Then, the position
−1 1
of P in the Argand plane, is [IIT-JEE 2007, 3M] (a) (b)
i −1 i +1
(a) 3 e iπ / 4 + 4i (b) (3 − 4i ) e iπ / 4
−1 1
(c) ( 4 + 3i ) e iπ / 4 (d) (3 + 4i ) e iπ / 4 (c) (d)
i +1 i −1
(where i = −1)
z 120. Let z = x + iy be a complex number, where x and y are
113. If | z | = 1 and z ≠ ± 1, then all the values of lie on integers and i = − 1. Then, the area of the rectangle
1−z2
[IIT-JEE 2007, 3M] whose vertices are the roots of the equation
(a) a line not passing through the origin z z 3 + z z 3 = 350, is [IIT-JEE 2009, 3M]
(b) | z | = 2 (a) 48 (b) 32
(c) the X -axis (c) 40 (d) 80
(d) the Y -axis 121. Let z = cos θ + i sin θ, where i = − 1. Then the value of
114. If | z + 4 | ≤ 3, the maximum value of | z + 1 | is 15

[AIEEE 2007, 3M]


Σ Im (z 2 m − 1 ) at θ = 2° is
m =1 [IIT-JEE 2009, 3M]
(a) 4 (b) 10 1 1
(c) 6 (d) 0 (a) (b)
sin 2 ° 3 sin 2 °
Passage (Q. Nos. 115 to 117) (c)
1
(d)
1
Let A , B and C be three sets of complex numbers as defined 2 sin 2 ° 4 sin 2 °
A = {z : Im ( z ) ≥ 1}
122. If  z −  = 2, the maximum value of | z | is equal to
below: 4
B = {z : | z − 2 − i | = 3}  z [AIEEE 2009, 4M]
C = {z : Re ((1 − i ) z ) = 2}, where i = − 1 (a) 2 + 2 (b) 3 + 1
[IIT-JEE 2008, 4+4+4M] (c) 5 + 1 (d) 2
Chap 01 Complex Numbers 73

123. Let z 1 and z 2 be two distinct complex numbers and 128. If z is any complex number satisfying | z − 3 − 2i | ≤ 2 ,
z = (1 − t ) z 1 + iz 2 , for some real number t with 0 < t < 1 where i = − 1 , then the minimum value of | 2z − 6 + 5i |,
and i = − 1 . If arg (w ) denotes the principal argument is [IIT-JEE 2011, 4M]
of a non-zero complex number w, then [IIT-JEE 2010, 3M] 129. The set
(a) | z − z1| + | z − z 2| = | z1 − z 2 |   2iz  
(b) arg (z − z1 ) = arg (z − z 2 )  Re   : z is a complex number | z | = 1, z ≠ ± 1 is
z − z1 z − z1   1 − z 2  
(c) =0
z 2 − z1 z 2 − z1 [IIT-JEE 2011, 2M]
(d) arg (z − z1 ) = arg (z 2 − z1 ) (a) ( − ∞, − 1 ] ∩ [1, ∞ ) (b) ( − ∞, 0 ) ∪ ( 0, ∞ )
2π 2π (c) ( − ∞, − 1 ) ∪ (1, ∞ ) (d) [2, ∞ )
124. Let ω be the complex number cos + i sin , where
3 3  1 
130. The maximum value of arg   for | z | = 1, z ≠ 1, is
i = − 1, then the number of distinct complex numbers z 1 − z 
z +1 ω ω2 given by [IIT-JEE 2011, 2M]
π π π 2π
satisfying ω z + ω2 1 = 0, is equal to (a) (b) (c) (d)
6 3 2 3
ω2 1 z +ω
[IIT-JEE 2010, 3M] 131. Let w = e iπ / 3 , where i = − 1 and a, b, c , x , y and z be
(a) 0 (b) 1
non-zero complex numbers such that
(c) 2 (d) 3
a +b +c = x
125. Match the statements in Column I with those in a + bw + cw 2 = y
Column II.
a + bw 2 + cw = z.
[Note Here, z takes values in the complex plane and Im
(z ) and Re (z ) denote respectively, the imaginary part | x |2 + | y |2 + | z |2
The value of , is
and the real part of z.] [IIT- JEE 2010, 8M] | a |2 + | b |2 + | c |2 [IIT-JEE 2011, 4M]

Column I Column II 132. Let α and β be real and z be a complex number. If


(A) The set of points z satisfying (p) an ellipse with z 2 + α z + β = 0 has two distinct roots on the line
| z − i | z || = | z + i | z ||, where eccentricity 4/5
Re (z ) = 1, then it is necessary that [AIEEE 2011, 4M]
i = − 1, is contained in or equal to
(a) β ∈ ( − 1, 0 ) (b) | β | = 1
(B) The set of points z satisfying (q) the set of points z (c) β ∈ (1, ∞ ) (d) β ∈( 0, 1 )
| z + 4 | + | z − 4 | = 10 is contained in satisfying
or equal to Im (z) = 0
133. If ω ( ≠ 1) is a cube root of unity and (1 + ω ) 7 = A + Bω,
1 (r) the set of points z then ( A, B ) equals to [AIEEE 2011, 4M]
(C) If | w | = 2, the set of points z = w − (a) (1, 1) (b) (1, 0)
w satisfying
is contained in or equal to |Im (z)| ≤ 1 (c) (−1, 1) (d) (0, 1)
1 (s) the set of points 134. Let z be a complex number such that the imaginary part
(D) If | w | = 1, the set of points z = w +
w satisfying of z is non-zero and a = z 2 + z + 1 is real. Then, a cannot
is contained in or equal to |Re (z) | ≤ 2
take the value [IIT-JEE 2012, 3M]
(t) the set of points z 1 1 3
satisfying | z | ≤ 3 (a) −1 (b) (c) (d)
3 2 4
126. If α and β are the roots of the equation x 2 − x + 1 = 0, z2
135. If z ≠ 1 and is real, the point represented by the
α 2009 + β 2009 is equal to [AIEEE 2010, 4M] z −1
(a) – 1 (b) 1 complex number z lies [AIEEE 2012, 4M]
(c) 2 (d) – 2 (a) on a circle with centre at the origin
127. The number of complex numbers z, such that (b) either on the real axis or on a circle not passing through
the origin
| z − 1 | = | z + 1 | = | z − i |, where i = − 1, equals to
(c) on the imaginary axis
[AIEEE 2010, 4M]
(d) either on the real axis or on a circle passing through the
(a) 1 (b) 2
origin
(c) ∞ (d) 0
74 Textbook of Algebra

136. If z is a complex number of unit modulus and argument 5


(a) is strictly greater than
1 + z 2
θ, then arg   equals to 5
1 + z  [JEE Main 2013, 4M]
(b) is equal to
2
π 3 5
(a) −θ (b) θ (c) is strictly greater than but less than
2 2 2
(c) π − θ (d) −θ (d) lies in the interval (1, 2)

137. Let complex numbers α and


1  2k π   2k π 
lie on circles 142. Let z k = cos   + i sin   ; k = 1, 2,K, 9. Then,
α  10   10 
( x − x 0 ) 2 + (y − y 0 ) 2 = r 2 and match the column.
( x − x 0 ) 2 + (y − y 0 ) 2 = 4 r 2 , respectively. If
Column I Column II
z 0 = x 0 + i y 0 satisfies the equation 2 | z 0 | 2 = r 2 + 2 ,
(A) For each zk there exists a z j such that (1) True
then | α | equals to [JEE Advanced 2013, 2M] zk ⋅ z j = 1
1 1 1 1
(a) (b) (c) (d)
2 2 7 3 (B) There exists a k ∈{1, 2, K , 9} such that (2) False
z1 ⋅ z = zk has no solution z in the set of
3 +i complex numbers
138. Let w = and P = {w n : n = 1, 2, 3, K }. Further,
2 (C) |1 − z1 ||1 − z2 | K |1 − z9 | (3) 1
 1   1  equals to
H 1 = z ∈ C : Re(z ) >  and H 2 = z ∈ C : Re(z ) <  −   , 10
 2   2 
 2kπ 
9
where C is the set of all complex numbers. If (D) 1− ∑ cos  10  equals to (4) 2
z 1 ∈ P ∩ H 1 , z 2 ∈ P ∩ H 2 and O represents the origin, k=1

then ∠z 1O z 2 equals to
[JEE Advanced 2013, 3M] [JEE Advanced 2014, 3M]
π π Codes
(a) (b)
2 6 A B C D A B C D
2π 5π (a) 1 2 4 3 (b) 2 1 3 4
(c) (d)
3 6 (c) 1 2 3 4 (d) 2 1 4 3

Passage (Q. Nos. 139 to 140) 143. A complex number z is said to be unimodular if | z | = 1.
Let S = S 1 ∩ S 2 ∩ S 3 , where Suppose z 1 and z 2 are complex numbers such that
z 1 − 2z 2
S 1 = {z ∈ C : | z | < 4}, is unimodular and z 2 is not unimodular. Then
2 − z 1z 2
  z − 1 + 3i 
S 2 =  z ∈ C : Im   > 0 the point z 1 lies on a [JEE Main 2015, 4M]
  1 − 3i   (a) circle of radius z
(b) circle of radius 2
and S 3 = {z ∈ C : Re z > 0}. [JEE Advanced 2013, 3+3M]
(c) straight line parallel to X -axis
139. min | 1 − 3i − z | equals to (d) straight line parallel to Y -axis
z ∈S
2− 3 2+ 3 144. Let ω ≠ 1 be a complex cube root of unity.
(a) (b)
2 2 If (3 − 3 ω + 2 ω 2 ) 4n + 3 + (2 + 3 ω − 3 ω 2 ) 4n + 3
3− 3 3+ 3 +( −3 + 2 ω + 3 ω 2 ) 4n + 3 = 0, then possible value(s) of n is
(c) (d)
2 2 (are) [JEE Advanced 2015, 2M]
140. Area of S equals to (a) 1 (b) 2
10 π 20 π (c) 3 (d) 4
(a) (b)
kπ   kπ 
3
16 π
3
32 π
145. For any integer k, let α k = cos   + i sin   , where
(c) (d) 7 7
12
3 3
141. If z is a complex number such that | z | ≥ 2, then the
∑ | αk +1 − αk |
k =1
i = −1. The value of the expression
  1  3
minimum value ofz +   , is
  2  ∑ | α 4k −1 − α 4k − 2 |
[JEE Main 2014, 4M] k =1
is [JEE Advanced 2015, 4M]
Chap 01 Complex Numbers 75

2 + 3i sin θ 1 1 
146. A value of θ for which is purely imaginary, is (a) the circle with radius and centre  , 0 for a > 0, b ≠ 0
 2a 
1 − 2i sin θ 2a
[JEE Main 2016, 4M]
 3
1  1 
π −1 (b) the circle with radius − and centre  − , 0 for
(a) (b) sin   2a  2a 
6  4
a < 0, b ≠ 0
 1  π
(c) sin −1   (d) (c) the X -axis for a ≠ 0, b = 0
 3 3
(d) the Y -axis for a = 0, b ≠ 0
147. Let a, b ∈ R and a 2 + b 2 ≠ 0. 148. Let ω be a complex number such that 2ω + 1 = z, when
 1  1 1 1
Suppose S = z ∈ C : z = , t ∈ R, t ≠ 0 , where
 a + ibt  z = −3 if 1 − ω − 1 ω 2 = 3k, then k is equal to
2

i = −1. If z = x + iy and z ∈ S, then ( x , y ) lies on 1 ω2 ω7 [JEE Main 2017, 4M]


[JEE Advanced 2016 4M] (a) 1 (b) − z (c) z (d) − 1

Answers
Exercise for Session 1 71. A → (r); B → (p, s); C → (q, t)
72. (d) 73. (d) 74. (c) 75. (d) 76. (a) 77. (d)
1. (d) 2. (c) 3. (b) 4. (b) 5. (c) 6. (b)
78. (d)
7. (d) 8. (a)
82. 2

Exercise for Session 2 84. z = c + i (−1 ± (1 − c2 − 2c)) for 0 ≤ c ≤ 2 − 1 and no solution for
1. (b) 2. (b) 3. (b) 4. (b) 5. (b) 6. (b) c> 2 −1
7. (d) 8. (c) 9. (b) 10. (b) 11. (a) 12. (c) 85. (1 − 3i ) z + (1 + 3i ) z − 22 = 0 and (3 + i ) z + (3 − i ) z + 14 = 0
13. (c) 14. (a)
87. No solution 97.
∑ | z1 |2 ( z2 − z3 )
Exercise for Session 3 ∑ z1 ( z2 − z3 )
1. (a) 2. (b) 3. (d) 4. (a) 5. (b) 6. (a) 98.
∑ z12 ( z 2 − z 3 ) + ∑ | z1 | 2 ( z2 − z3 )
7. (c) 8. (b) 9. (c) 10. (d) 11. (c) 12. (a) ∑( z1 z2 − z2 z1 )
13. (b) 14. (b) 15. (a)
99. (i) x3 − 24x2 + 80x − 64 = 0
Exercise for Session 4 (ii) x3 − 21x2 + 35x − 7 = 0
1. (a) 2. (d) 3. (c) 4. (d) 5. (b) 6. (b) (iii) 4
7. (a) 8. (d) 9. (d) 10. (b) 11. (b) 12. (a) -
– –3 –5
13. (b) 14. (c) 100. Roots of z 7 + 1 = 0 are −1, α , α3, α 5, α , α , α , where
π π
α = cos + i sin
Chapter Exercises 7 7
1. (b) 2. (c) 3. (d) 4. (d) 5. (a) 6. (d)  1 − 71 −1 − 4 11   −1 + 4 11 1 + 71 
7. (b) 8. (d) 9. (b) 10. (c) 11. (c) 12. (b) 101. a ∈  ,  ∪ , 
 2 5   5 2 
13. (c) 14. (d) 15. (a) 16. (c) 17. (a) 18. (c)
2n+ 1
19. (b) 20. (b) 21. (c) 22. (c) 23. (c) 24. (c) 102. (i) C 1 (1 − x)n − 2n+ 1C 3 (1 − x)n− 1 x + ... + (−1)n xn = 0
2n+ 1
25. (d) 26. (d) 27. (b) 28. (b) 29. (b) 30. (c) (ii) C 1xn − 2n+ 1 C 3xn− 1 + 2n+ 1 C 5xn− 2 −... = 0
31. (b,c,d) 32. (b,c,d) 33. (b,c,d) 34. (b,c,d) 35. (a,c,d) 36. (a,b) 103. (c) 104. (c) 105. (1 − 3 ) + i, −i 3, ( 3 + 1) − i 106. (d)
37. (a,b,c) 38. (a,d) 39. (a,c) 40. (a,b,d) 41. (a,d)
107. (b) 108. (a) 109. (d) 110. (d) 111. (d) 112. (d)
42. (b,d) 43. (a,c,d) 44. (a,d) 45. (a,b,d)
113. (d) 114. (c) 115. (b) 116. (c) 117. (d) 118. (d)
46. (a) 47. (d) 48. (b) 49. (a) 50. (c) 51. (b)
119. (c) 120. (a) 121. (d) 122. (c) 123. (a, c, d) 124. (b)
52. (d) 53. (c) 54. (a) 55. (c) 56. (d) 57. (b) 125. A → (q, r); B → (p); C → (p, s); D → (q, r, s, t) 126. (b) 127. (a)
58. (1) 59. (5) 60. (9) 61. (8) 62. (3) 63. (4) 128. (5) 129. (a) 130. (c) 131. (3) 132. (c) 133. (a)
64. (6) 65. (5) 66. (4) 67. (4) 134. (d) 135. (d) 136. (b) 137. (c) 138. (c) 139. (c)
68. A → (p, q); B → (p, r); C → (p, r, s) 140. (b) 141. (d) 142. (c) 143. (a) 144. (a, b, d)
69. A → (q); B → (q, r); C → (q, s) 145. (4) 146. (c) 147. (a,c,d) 148. (b)
70. A → (p, q, r, t); B → (p, s); C → (p, r)
Solutions
r
4. We have, Π e ipθ = 1
p =1

⇒ e i θ ⋅ e 2i θ ⋅ e 3i θ … e ri θ = 1
 r (r + 1) 
iθ  
⇒ e iθ(1 + 2 + 3 + … + r ) = 1 ⇒ e  2 
=1
1. We have, r (r + 1 )  r (r + 1 ) 
or cos  θ  + i sin  θ = 1 + i ⋅ 0
a + ib = cos (1 − i ) = cos 1 cos i + sin 1 sin i  2   2 
= cos1 cosh 1 + sin 1 i sinh 1 On comparing, we get
[Q cos i = cosh 1, sin i ⋅ 1 = i sinh 1 ]
r (r + 1 )  r (r + 1 ) 
 e + e −1   e − e −1  cos  θ  = 1 and sin  θ = 0
= cos 1   + i sin 1    2   2 
 2   2  r (r + 1 ) r (r + 1 )
⇒ θ = 2m π and θ = m1π
1 1 1 1 2 2
= e +  cos 1 + i ⋅ e −  sin 1
2 e 2 e ⇒ θ=
4m π
and θ =
2m1π
1 1 r (r + 1 ) r (r + 1 )
∴ a = e +  cos1
2 e where, m, m1 ∈ I
1 1 4nπ
and b = e −  sin 1 Hence, θ = , n ∈I.
2 e r (r + 1 )

2. Given that, z 10 − z 5 − 992 = 0 5. Let z = x + iy , then


(3 + i ) (z + z ) − (2 + i ) (z − z ) + 14i = 0 reduces to
Let t =z 5
(3 + i ) 2 x − (2 + i ) (2iy ) + 14i = 0
⇒ t 2 − t − 992 = 0 ⇒ 6 x + 2y + i (2 x − 4y + 14 ) = 0
1 ± 1 + 3968 1 ± 63 On comparing real and imaginary parts, we get
⇒ t= = = 32, − 31
2 2 6 x + 2y = 0
⇒ 3x + y = 0 …(i)
∴ z 5 = 32
and 2 x − 4y + 14 = 0
and z 5 = − 31 ⇒ x − 2y + 7 = 0 …(ii)
But the real part is negative, therefore z 5 = 32 does not hold. On solving Eqs. (i) and (ii), we get
∴ Number of solutions is 5. x = − 1 and y = 3
3. From Coni method, ∴ z = − 1 + 3i
O ∴ zz = | z | 2 = | − 1 + 3i | 2 = ( −1 ) 2 + (3 ) 2 = 10
6. Since, affix of A is z1.

n C B

π
-
An(z) A1(z) O π/2
z −0 z
= e 2π i /n or = e 2π i /n … (i)
z −0 z D A(z1)
Im(z ) → → → →
But given = 2 −1
Re(z ) ∴ OA = z1 and OB and OC are obtained by rotating OA
z −z z  π → →
− 1 through and π. Therefore, OB = iz1 and OC = -z1.
2i 1 z 2
⇒ = 2 −1 ⇒   = 2 −1 z + iz1 + ( −z1 )
z +z i z + 1 Hence, centroid of ∆ABC = 1
2 z  3
e 2 π i /n
− 1 i z1  π π
⇒  2 π i /n  = i ( 2 − 1) = z1 =  cos + i sin 
e + 1
[from Eq. (i)] 3 3  2 2
 π If A, B and C are taken in clockwise, then centroid of ∆ABC
⇒ i tan   = i ( 2 − 1 ) 1  π π
 n = z1  cos − i sin 
 π  π 3  2 2
⇒ tan   = tan   z1  π π
 n  8 ∴ Centroid of ∆ABC =  cos ± i sin 
∴ n =8 3  2 2
Chap 01 Complex Numbers 77

3 −i  −1 − i 3  z1 z1 1
7. Given that, z = = i  = iω 2
2  2  ⇒ z2 z2 1 =0
∴ z 101 = (iω 2 )101 = i 101 ω 202 = iω z1 − z 2 z1 − z 2 1
Now, i 101 + z 101 = i + iω = i ( − ω 2 ) z1 z1 1

∴ +z = −i ω
= − i ω = iω = z
101 101 103 103 206 3 2 2 M
(i )
a Applying R3 → R3 − R1 + R2, then z 2 z2 1 =0
8. The complex slope of the line az + a z + 1 = 0 is α = −
a M
b
and the complex slope of the line bz + b z − 1 = 0 is β = − 0… 0 …1
b Expand w.r.t. R3, then
Since, both lines are mutually perpendicular, then z1z 2 − z1 z 2 = 0
∴ α+β=0 ⇒ z1z 2 − (z1z 2 ) = 0
a b
⇒ − − =0 ⇒ Im (z1 z 2 ) = 0
a b
⇒ Im ((a + ib ) (c + id )) = 0
⇒ ab + a b = 0
⇒ Im ((a + ib ) (c − id )) = 0
 8π   8π 
9. We have, α = cos   + i sin   ⇒ bc − ad = 0 ⇒ ad − bc = 0
 11   11 
Now, Re (α + α 2 + α 3 + α 4 + α 5 ) 13. Let z = a + ib

α + α2 + α3 + α4 + α5 + α + α2 + α3 + α4 + α 5 ∴ f (a + ib ) = (a 2 + b 2 )
=
2 ⇒ f (z ) = f (z ) = f ( −z ) = f ( −z ) = (a 2 + b 2 )
− 1 + (1 + α + α 2 + α 3 + α 4 + α 5 + α + α 2 + α 3 + α 4 + α 5
= ∴ f is not injective (i.e., it is many-one).
2
−1 + 0 but | z | > 0 i. e. f (z ) > 0 ⇒ f (z ) ∈ R + (Range)
= [sum of 11, 11th roots of unity]
2 ⇒ R+ ⊂ R
1
=− ∴ f is not surjective (i.e., into).
2
Hence, f is neither injective nor surjective.
10. |z | ≤ 4 …(i)
14. Let α = re iθ , β = re iφ [Q| α | = | β |, given]
π
and 0 ≤ arg(z ) ≤ …(ii)  π π
3 where, θ ∈  − ,  and φ ∈ ( − π , 0 )
 2 2
P(z) θ+ φ
i
 2 

 θ − φ
e ⋅ 2 cos  
π/3 α + β re iθ + reiφ  2 
∴ = =
Real axis α − β re iθ − reiφ θ+ φ
O i
 2 

 θ − φ
e ⋅ 2i sin  
 2 
 θ − φ
= − i cot   = Purely imaginary
which implies the set of points in an argand plane, is a sector  2 
of a circle. 25 25 25 25
15. We have, | z | = z + − ≤ z+ +
11. Since, x 2 + x + 1 = ( x − ω ) ( x − ω 2 ), where ω is the cube root z z z −z
of unity and f ( x ) = g ( x 3 ) + x h ( x 3 ) is divisible by x 2 + x + 1. ⇒ | z | ≤ 24 +
25
Therefore, ω and ω 2 are the roots of f ( x ) = 0. |z |
⇒ f (ω ) = 0 and f (ω 2 ) = 0 ⇒ | z | 2 − 24 | z | − 25 ≤ 0 ⇒ (| z | − 25 ) (| z | + 1 ) ≤ 0
∴ | z | − 25 ≤ 0 [Q | z | + 1 > 0 ]
⇒ g (ω 3 ) + ω h (ω 3 ) = 0
⇒ | z | ≤ 25 or | z − 0 | ≤ 25
and g ((ω 2 ) 3 ) + ω 2h (ω 2 ) 3 = 0 Hence, the maximum distance from the origin of coordinates
⇒ g (1 ) + ω h (1 ) = 0 to the point z is 25.
and g (1 ) + ω 2h (1 ) = 0 16. Q A ≡ z1, B ≡ z 2, C ≡ (1 − i ) z1 + iz 2
⇒ g (1 ) = h (1 ) = 0 ∴ AB = | z1 − z 2 |
Hence, g ( x ) and h ( x ) both are divisible by ( x − 1 ). BC = | z 2 − (1 − i ) z1 − iz 2| = | (1 − i ) (z 2 − z1 )|
12. Since, z1, z 2 and z1 − z 2 are collinear. = 2 | z1 − z 2 |
z1 z1 1 and CA = | (1 − i ) z1 + iz 2 − z1| = | − i (z1 − z 2)|
= | − i || z1 − z 2 | = | z1 − z 2 |
∴ z2 z2 1 =0
It is clear that, AB = CA and ( AB ) 2 + (CA ) 2 = ( BC ) 2
z1 − z 2 z1 − z 2 1
∴ ∆ ABC is isosceles and right angled.
78 Textbook of Algebra

17. Centre and radius of circle | z | = 3 …(i) y = 41 /3 ⋅ 4 − 1 /9 ⋅ 41 /27…∞ = 41 /3 − 1 /9 + 1 /27 … ∞


are C1 ≡ 0, r1 = 3 1 /3

and centre and radius of circle = 41 + 1 /3 = 41 /4 = 2



|z + 1 −i | = 2 …(ii) 1 1 1
and C 2 = − 1 + i , r2 = 2
and z = ∑ (1 + i )− r = (1 + i ) + (1 + i )2 + (1 + i )3 + …∞
r =1

Q | C1C 2 | = | − 1 + i | = 2 1
and | C1C 2 | < r1 − r2 (1 + i ) 1
= = = −i
1 i
1−
(1 + i )
|z|=3
Now, x + yz = 3 − i 2
 2
O′ |z+1–i|=√2 ∴ arg ( x + yz ) = arg (3 − i 2 ) = − tan −1  
 3 
O
21. Q A1 ≡ 1 + 2i
A2 A1(1+2i )

π/3
A3 A6
Hence, circle (ii) completely inside circle (i) O
∴ Number of solutions = 0
18. We have, f (z ) = g(z ) (z 2 + 1) + h(z )
A4 A5
where, degree of h (z ) < degree of (z 2 + 1 )
∴ A2 = (1 + 2i ) e iπ /3
⇒ h(z ) = az + b ; a, b ∈ C
1 i 3 1 i 3
∴ f (z ) = g (z ) (z 2 + 1 ) + az + b ; a, b ∈ C = (1 + 2i )  + = + +i − 3
2 2  2 2
⇒ f (z ) = g (z ) (z − i ) (z + i ) + az + b ; a, b ∈ C …(i)
Now, f (i ) = 1 − i 1   3 
[given] =  − 3 + i  + 1
⇒ ai + b = 1 − i [from Eq. (i)] …(ii) 2   2 
and f ( −i ) = 1 + i [given] 1   3 
∴ | A1A2 | = 1 + 2i −  − 3 − i  + 1
⇒ a ( −i ) + b = 1 + i [from Eq. (i)] …(iii) 2   2 
On solving Eqs. (ii) and (iii) for a and b, we get
1  3
a = − 1 and b = 1 = + 3 + i 1 − 
2  2 
∴ Required remainder, h (z ) = az + b = − z + 1 = 1 − z
2
19. We have, | z + 1| = 2 | z − 1| 1 
2
 3
=  + 3 + 1 −  = 5
Put z = x + iy , we get 2   2 
( x + 1 ) 2 + y 2 = 4 [( x − 1 ) 2 + y 2 ] ∴ Perimeter = 6 | A1A2 | = 6 5
⇒ 3 x + 3y − 10 x + 3 = 0
2 2
22. We have,
10 n n n
⇒ x 2 + y2 − x + 1 = 0
∑ zr ∑ (zr − r ) + r ∑ (| zr − r | + | r | )
…(i) = ≤
3
r =1 r =1 r =1
On comparing Eq. (i) with the standard equation
n n n n
x 2 + y 2 + 2 gx + 2 fy + c = 0 = ∑ | zr − r | + ∑ | r | ≤ ∑ r + ∑ | r |
10 5 r =1 r =1 r =1 r =1
⇒ g=− = − and f = 0
6 3 n(n + 1 ) n(n + 1 )
= + = n(n + 1 )
5  2 2
∴ Required centre of circle ≡ ( − g, − f ) ≡  , 0 n
3  ∴ ∑ zr ≤ n (n + 1 )
5 5 r =1
i.e. + 0⋅i =
3 3  z 
 z1 − 
20. Q x = 91 /3 ⋅ 91 /9 ⋅ 91 /27 …∞ | z| π z
23. We have, arg  = and − z1 = 3
1 /3  z  2 |z |
 
= 91 /3 + 1 /9 + 1 /27 + … ∞ = 91 − 1/3 = 91 /2 = 3  |z | 
Chap 01 Complex Numbers 79

which implies the following diagram λ 7


⇒ =
Y µ 3
z Q λ, µ ∈ I − { 0}
|z| For minimum value λ = 7, µ = 3
3
1
90° ∴ | z | 2 = | (3 + 7i ) ( λ + iµ ) | 2
z1
X′
O
X = | 3 + 7i | 2 | λ + iµ| 2 = 58 ( λ2 + µ 2 )
= 58 (7 2 + 3 2 ) = (58 ) 2 = 3364
27. We have,
z = f ( x ) + i g( x )
Y′ where, i = −1 and f , g : ( 0,1 ) → ( 0,1 ) are real-valued
z functions.
⇒ − z1 = 3 ⇒ | z1| = 9 + 1 = 10
|z | 1  1 
(a) z = + i 
24. Let z = x + iy = r (cos θ + i sin θ ) 1 − ix  1 + ix 
∴ | z | = r , arg (z ) = θ 1 + ix x+i 1+x (1 + x )
= + = +i
π  1 + x2 1 + x2 1 + x2 1 + x2
Given, | z − 2 − i | = | z | sin  − arg (z )
4  1+x 1+x
⇒ f (x ) = and g( x ) =
π  1+x 2
1 + x2
⇒ | x + iy − 2 − i | = r sin  − θ
4  But for x = 0.5, f ( 0.5 ) > 1 and g( 0.5 ) > 1, which is out of
range.
1
⇒ | ( x − 2 ) + i (y − 1 ) | = r (cos θ − sin θ ) Hence, (a) is not a correct option.
2
1  1 
1 (b) z = + i 
⇒ ( x − 2 ) 2 + (y − 1 ) 2 = x −y 1 + ix  1 − ix 
2
On squaring both sides, we get 1 − ix (i − x )  1 − x   1−x 
= + =  + i 
2 ( x 2 + y 2 − 4 x − 2y + 5 ) = x 2 + y 2 − 2 xy 1 + x2 1 + x2 1 + x 2 1 + x 2
⇒ ( x + y ) 2 = 2( 4 x + 2y − 5 ) 1−x 1−x
⇒ f (x ) = and g( x ) =
1+ x2 1+ x2
which is a parabola.
25. Since, 1, z1, z 2, z 3, …, zn − 1 are the n, nth roots of unity. Clearly, f ( x ), g( x ) ∈ ( 0, 1 ), if x ∈( 0,1 )
Hence, (b) is the correct option.
∴ (z n − 1 ) = (z − 1 ) (z − z1 ) (z − z 2 ) (z − z 3 ) … (z − zn − 1 ) 1 − ix i (1 − ix ) (1 + x ) i (1 − x )
(c) z = + = +
n −1 1 + x2 1 + x 2 (1 + x 2 ) (1 + x 2 )
= (z − 1 ) ∏ (z − zr )
r =1 Hence, (c) is not a correct option.
Taking log on both sides, we get 1  1  1 + ix i (1 + ix )
n −1
(d) z = + i = +
1 − ix  1 − ix  1 + x 2 (1 + x 2 )
loge (z n − 1 ) = loge (z − 1 ) + ∑ loge (z − zr )
(1 − x ) i (1 + x )
r =1 = +
(1 + x 2 ) (1 + x 2 )
On differentiating both sides w.r.t.z, we get
n −1 Hence, (d) is not a correct option.
nz n − 1 1 1
− =∑ 28. Let z = α be a real roots of equation.
(z − 1 )
n
(z − 1 ) r =1
(z − zr )
z 3 + (3 + 2i )z + ( −1 + ia ) = 0

Putting z = 3, we get ⇒ α 3 + (3 + 2i ) α + ( −1 + ia ) = 0
n −1 ⇒ (α 3 + 3 α − 1 ) + i (a + 2 α ) = 0
1 n ⋅ 3n − 1 1
∑ = n
(3 − zr ) (3 − 1 ) 2
− On comparing the real and imaginary parts, we get
r =1
α 3 + 3 α − 1 = 0 and a + 2 α = 0
26. We have, a
z = (3 + 7i ) ( λ + iµ ) ⇒ α=−
2
= (3 λ − 7µ ) + i (7 λ + 3 µ ) a 3
3a
Since, z is purely imaginary. ⇒ − − −1 = 0
8 2
∴ 3 λ − 7µ = 0
⇒ a 3 + 12a + 8 = 0
80 Textbook of Algebra

Let f (a ) = a 3 + 12a + 8 which is a circle and passing through the origin


⇒ f ( −1 ) < 0 and f ( 0 ) > 0 1+i
2
1+i 1
and radius = −0= =
∴ a ∈ ( −1, 0 ) 2 2 2
π π π
29. CiS = cos + i sin 32. Given, |z − 1 | < |z + 3 |
6 6 6
⇒ |z − 1 |2 < |z + 3 |2
 3 + i  1  −1 + i 3  ω
=  =   = = − iω ⇒ | z | 2 + 1 − 2 Re (z ) < | z | 2 + 9 + 2 Re (3z )
 2  i  2  i
m ⇒ 2 Re ( 4z ) > − 8
 π
∴ 2 CiS  = ( −2iω )m = (( −2iω ) 3 )m /3 = (8i )m /3 ⇒ Re ( 4z ) > − 4
 6 4z + 4z
n n ⇒ >−4
 π   π π 2
and  4 CiS  =  4  cos + i sin   = (2 2 (1 + i )n
 4   4 4 ∴ z + z > −2
and ω = 2z + 3 − i
= (8(1 + i ) 2 )n /2 = (16i )n /2
∴ ω + ω = 2z + 3 − i + 2z + 3 + i
Thus, (8i )m / 3 = (16i )n / 2 = 2(z + z ) + 6 > − 4 + 6
which is satisfy only when m = 48 and n = 24 ⇒ ω + ω >2
∴ m + n = 72 Option (a) | ω − 5 − i | < | ω + 3 + i |
30. We have, z 2 = z ⋅ 21 − | z| ⇒ | 2z + 3 − i − 5 − i | < | 2z + 3 − i + 3 + i |
Taking modulus on both sides, we get ⇒ | 2z − 2 − 2i | < | 2z + 6 |
| z | 2 = | z | ⋅ 21 − | z| ⇒ |z −1 −i | <|z + 3|
which is false.
⇒ | z | (| z | − 21 − | z | ) = 0 …(i)
1 − | z|
Option (b) | ω − 5 | < | ω + 3 |
and arg (z ) = arg (z ⋅ 2
2
)
⇒ | 2z + 3 − i − 5 | < | 2z + 3 − i + 3 |
⇒ 2 arg (z ) = arg (z ) = − arg (z ) ⇒ | 2z − 2 − i | < | 2z + 6 − i |
⇒ 3 arg (z ) = 0 i i
⇒ z −1 − < z + 3 −
∴ arg (z ) = 0 2 2
Then, y =0 [Q z = x + iy ] ⇒ |z −1| <|z + 3|
From Eq. (i), | z | = 0 ⇒ x = 0 [Q y = 0 ] which is true.
One solution is z = 0 + i ⋅ 0 = 0. Option (c) Im (iω ) > 1
Also, from Eq. (i), iω − iω
⇒ >1
–x
y=2
Y 2i
iω + i ω
/2 ⇒ >1
(0, 1) y = |x| 2i
⇒ ω + ω >2
X′ o X which is true.
π
Option (d) | arg (ω − 1 ) | <
2
π
⇒ | arg (2z + 3 − i − 1 ) | <
Y′ 2
| z | = 21 − | z| ⇒ | x | = 21 − x π
⇒ | arg (2z + 2 − i ) | <
|x| 2
⇒ = 2 − x = y (say)
2 −1  Im(2z + 2 − i )  π
⇒ tan   <
Hence, total number of solutions = 2  Re(2z + 2 − i )  2
z +1
31. Q is a purely imaginary number. ∴ Re(2z + 2 − i ) > 0
z +i (2z + 2 − i ) + (2z + 2 + i )
 z + 1  z + 1 z +1  z + 1 ⇒ >0
∴   =−  ⇒ =−  2
z + i z + i z −i z + i ⇒ z +z +2>0
⇒ (z + 1 ) (z + i ) + (z − i ) (z + 1 ) = 0 ⇒ z + z > −2
⇒ 2zz + z (1 + i ) + z (1 − i ) = 0 which is true.
1 + i 1 − i 33. ∴ (1 + ri ) 3 = λ (1 + i )
⇒ zz +  z + z =0
 2   2  ⇒ 1 + (ri ) 3 + 3(1 ) 2ri + 3(1 ) (ri ) 2 = λ (1 + i )
Chap 01 Complex Numbers 81

⇒ 1 − r 3i + 3ri − 3r 2 = λ + i λ ⇒ 2( x 2 + y 2 ) + 27 x − 50y + 38 = 0 …(i)


On comparing real and imaginary parts, we get and | z − 4 | = | z − 8 | ⇒| z − 4 | = | z − 8 |
2 2

1 − 3r 2 = λ ⇒ | z | 2 + 16 − 2 Re ( 4z ) = | z | 2 + 64 − 2 Re(8z )
and − r 3 + 3r = λ ⇒ 8 Re (z ) = 48
∴ Re (z ) = 6
Then, − r + 3r = 1 − 3r
3 2
⇒ x =6 …(ii)
⇒ r 3 − 3r 2 − 3r + 1 = 0
From Eqs. (i) and (ii), we get
⇒ (r 3 + 1 ) − 3r (r + 1 ) = 0 2 (36 + y 2 ) + 162 − 50y + 38 = 0
⇒ (r + 1 ) (r 2 − r + 1 − 3r ) = 0 ⇒ y 2 − 25y + 136 = 0
⇒ (r + 1 ) (r − 4r + 1 ) = 0
2
⇒ (y − 17 ) (y − 8 ) = 0
∴ r = − 1, 2 ± 3 ⇒ y = 17,8
∴ Im (z ) = 17, 8
3π π π
⇒ r = cosec , tan , cot 37.
2 12 12 Q(z2)
P(z1)
34. Option (a) | z − 1 | + | z + 1 | = 3
Here, | 1 − ( −1 ) | < 3 90°
i. e. 2 < 3, which is an ellipse.
Option (b) | z − 3 | = 2
It is a circle with centre 3 and radius 2.
7
Option (c) | z − 2 + i | = (z3)R
S(z4)
3
7
It is a circle with centre (2 − i ) and radius . Option (a)Q PS || QR
3
z − z4 
Option (d) (z − 3 + i ) (z − 3 − i ) = 5 ∴ arg  1  =0
z2 − z3
⇒ (z − 3 + i ) (z − 3 + i ) = 5
z1 − z 4
⇒ | z − 3 + i |2 = 5 ⇒ is purely real.
z2 − z3
⇒ |z −3 + i | = 5 Option (b) Q Diagonals of rhombus are perpendicular.
It is a circle with centre at (3 − i ) and radius 5.  z − z3  π
Then, arg  1  =
35. Since, 1, z1, z 2, z 3, …, zn − 1 are the n, nth roots of unity. z2 − z4  2
Therefore, z1 − z 3
⇒ is purely imaginary.
z − 1 = (z − 1 ) (z − z1 ) (z − z 2 ) … (z − zn − 1 )
n z2 − z4
Option (c) Q PR ≠ QS
zn − 1
⇒ = (z − z1 ) (z − z 2 ) … (z − zn − 1 ) ∴ | z1 − z 3 | ≠ | z 2 − z 4 |
z −1
n −1
Option (d) Q ∠QSP = ∠RSQ
= ∏ (z − zr )  z − z4  z − z4 
r =1 ∴ amp  2  = amp  3 
 z1 − z4  z2 − z4 
Now, putting z = ω, we get
n −1  z − z4  z2 − z4 
ωn − 1 ⇒ − amp  1  = − amp  
∏ (ω − zr ) = z2 − z4  z3 − z4 
r =1 ω −1
 0, if n = 3r, r ∈ Z  z − z4  z − z4 
⇒ amp  1  = amp  2 
 z2 − z4  z3 − z4 
=  1, if n = 3r + 1 , r ∈ Z
1 + ω, if n = 3r + 2, r ∈ Z 38. Q | z − 3 | = min {| z − 1 |, | z − 5 | }

36. Q 3 | z − 12 | = 5 | z − 8i | Case I If | z − 3 | = | z − 1 |
∴ 9 | z − 12 | 2 = 25 | z − 8i | 2 On squaring both sides, we get
| z − 3 |2 = | z − 1 |2
⇒ 9 (z − 12 ) (z − 12 ) = 25(z − 8i ) (z + 8i )
⇒ | z | 2 + 9 − 2 Re (3z ) = | z | 2 + 1 − 2 Re (z )
⇒ 9 (zz − 12(z + z ) + 144 ) = 25(zz + 8i (z − z ) + 64 )
⇒ 16zz + 108 (z + z ) + 200 (z − z ) i + 304 = 0 ⇒ 4 Re (z ) = 8
⇒ 16( x 2 + y 2 ) + 216 x − 400y + 304 = 0 ⇒ Re (z ) = 2
82 Textbook of Algebra

Case II If | z − 3 | = | z − 5 | z z 
+ 
On squaring both sides, we get z0 z0
⇒ =1
| z − 3 |2 = | z − 5 |2 2
⇒ | z | 2 + 9 − 2 Re (3z ) = | z | 2 + 25 − 2 Re (5z ) z
∴ Re   = 1
⇒ 4 Re (z ) = 16 ⇒ Re (z ) = 4 z0
39. C(z) 42. Q z1 + z 2 = a, z1z 2 = b
and given | z1| = | z 2 | = 1
π/6 π/3
a Let z1 = ei θ and z 2 = ei φ
2π/3
π/6 π/3 π/3 Q | a | = | z 1 + z 2 | ≤ | z 1| + | z 2 | = 1 + 1 = 2
A(– a,0) a O a B(a,0) ∴ |a | ≤2
θ+φ
Also, arg (a ) = arg (z1 + z 2 ) = arg (ei θ + ei φ ) =
2
and arg (b ) = arg (z1z 2 ) = arg (ei θ + φ ) ) = θ + φ
From figure, it is clear that z lies on the point of intersection of ∴ 2 arg (a ) = arg (b ) ⇒ arg (a 2 ) = arg (b )
the rays from A and B.
43. Q αz2 + z + α = 0 …(i)
Q ∠ACB = 90 ° and OBC is an equilateral triangle.
Hence, OC = a Then, αz + z + α = 0
2

⇒ | z − 0 | = a or | z | = a ⇒ α (z ) 2 + z + α = 0
π
and arg (z ) = arg (z − 0 ) = ⇒ αz 2 + z + α = 0 [Q z = z ] …(ii)
3
On subtracting Eq. (ii) from Eq. (i), we get
2z − i
40. Q =m (α − α ) z 2 − (α − α ) = 0
z +i
⇒ α − α = 0 and z 2 = 1
z − i/2 m
⇒ = ∴ α = α and z = ± 1
z +i 2
Put z = ± 1 in Eq. (i), we get
m
For circle, ≠1 α+α =±1
2
and absolute value of real root = 1
⇒ m ≠ 2 and m > 0
i.e., |z | =| ± 1| =1
41. ∴ A(z 0 ) lie on | z | = r
44. Let z = α be a real root of equation
⇒ | z 0| = r ⇒ | z 0| 2 = r 2 ⇒ z 0 z 0 = r 2
z 3 + (3 + i ) z 2 − 3z − (m + i ) = 0
⇒ α 3 + (3 + i ) α 2 − 3 α − (m + i ) = 0
A(z0)
r ⇒ (α + 3 α 2 − 3 α − m ) + i (α 2 − 1 ) = 0
3

O On comparing real and imaginary parts, we get


α 3 + 3α 2 − 3α − m = 0
P(z)
and α2 − 1 = 0 ⇒ α = ± 1

Let P (z ) be any point on tangent, then For α = 1, we get


π 1 + 3 −3 −m = 0 ⇒ m =1
∴ ∠PAO =
2 For α = − 1, we get
Complex slope of AP + Complex slope of OA = 0 −1 + 3 + 3 −m = 0 ⇒ m =5
z − z0 z0 − 0 45. Let z = α be a real root of equation
⇒ + =0
z − z 0 z0 − 0
z 3 + (3 + 2i ) z + ( −1 + ia ) = 0
⇒ z z 0 + z 0 z = 2z 0 z 0
⇒ α 3 + (3 + 2i ) α + ( − 1 + ia ) = 0
⇒ zz 0 + z 0z = 2r 2
⇒ (α 3 + 3 α − 1 ) + i (a + 2α ) = 0
⇒ zz 0 = zz 0
zz 0 z 0 z On comparing real and imaginary parts, we get
Also, + 2 =2 α 3 + 3α − 1 = 0
r2 r
zz 0 z 0z and a + 2α = 0
⇒ + =2
z 0z 0 z 0z 0
Chap 01 Complex Numbers 83

⇒ α=−
a 98  50 
2 = ∑ i r−3
+ 

i 0!
+ i 1!
+ i 2!
+ i 3!
+ ∑ i r ! 
r =1  r=4 
a 3 3a
⇒ − − − 1 = 0 ⇒ a 3 + 12a + 8 = 0
8 2  47 
= (i −2 + i −1 + 0 ) + i 1 + i 1 + i 2 + i 6 + ∑ i (r + 3)! 
Let f (a ) = a 3 + 12a + 8  
 r =1 
∴ f ( −1 ) < 0, f ( 0 ) > 0, f ( −2 ) < 0
= ( − 1 − i ) + (i + i − 1 − 1
f (1 ) > 0 and f (3 ) > 0
⇒ a ∈ ( −2, 1 ) or a ∈ ( −1, 0 ) or a ∈ ( −2,3 ) + (i 0 + i 0 + i 0 + … 47 times))
Sol. (Q. Nos. 46 to 48) = ( − 1 − i ) + (2i − 2 + 47 )
46. Q arg (z ) > 0 = 44 + i = a + ib [given]
∴ arg (z ) + arg ( −z ) = − π ∴ a = 44, b = 1
⇒ − arg (z ) + arg ( −z ) = − π Unit place digit of a 2011 = ( 44 ) 2011
⇒ arg ( −z ) − arg (z ) = − π = ( 44 ) (( 44 ) 2 )1005 = ( 44 ) (1936 )1005
47. Q arg (z1z 2) = π = (Unit place of 44)
⇒ arg (z1 ) + arg (z 2 ) = π × (Unit place digit of (1936 )1005)
⇒ arg (z1 ) − arg (z 2 ) = π = Unit place of ( 4 × 6 ) = 4
Given, | z 1| = | z 2 | and unit place digit of b 2012 = (1 ) 2012 = 1
∴ | z 1| = | z 2 | = | z 2 | Hence, the unit place digit of a 2011 + b 2012 = 4 + 1 = 5.
Then, z1 + z 2 = 0 100 101
⇒ z1 = − z 2 51. Q ∑ i r ! + ∏ i r
r=4 r =1
48. arg ( 4z1 ) − arg (5z 2 ) = π 97
is possible only when | 4z1| = | 5z 2 | = ∑ i (r + 3)! + i 1 ⋅ i 2 ⋅ i 3 … i 101
r =1
z1 5
⇒ = = 1 . 25 = (i 0 + i 0 + i 0 + … 97 times) + i 1 + 2 + 3 + … + 101
z2 4
= 97 + i 5151 = 97 + i 3 = 97 − i
and also 4z1 + 5z 2 = 0
∴ a = 97 and b = − 1
z1 5
⇒ =− Hence, a + 75b = 97 − 75 = 22
z2 4
Sol. (Q. Nos. 52 to 54)
z1 5
∴ = = 1 . 25 If z ±
a
= b, where a, b > 0
z2 4 z
Sol. (Q. Nos. 49 to 51) a a
∴ z± ≤|z | +
49. Q n ! is divisible by 4, ∀ n ≥ 4. z |z |
25 22 a
⇒ b ≤|z | +
∴ ∑ i n ! = ∑ i (n + 3)! |z |
n=4 n =1
⇒ | z |2 − b | z | + a ≥ 0
= i 0 + i 0 + i 0 + … (22 times) = 22 …(i)
b − b 2 − 4a
25 25 ∴ |z | ≤
∴ ∑ i n ! = i 1! + i 2! + i 3! + ∑ in! 2
n =1 n=4 b + b 2 − 4a
and |z | ≥ …(i)
= i + i + i + 22
2 6
[from Eq. (i)] 2
= i − 1 − 1 + 22 = 20 + i a a
Also, z± ≥ |z | −
z |z |
∴ a = 20, b = 1
∴ a − b = 20 − 1 = 19 ⇒
a
b ≥ |z | −
which is a prime number. |z |
a
95 50
⇒ −b ≤|z | − ≤b
50. ∴ ∑ ir + ∑ ir ! |z |
r = −2 r=0
⇒ −b | z | ≤ | z | 2 − a ≤ b | z |
84 Textbook of Algebra

Case I −b | z | ≤ | z | 2 − a On squaring and adding Eqs. (i) and (ii), we get


⇒ | z |2 + b | z | − a ≥ 0 (1 − a ) 2 + b 2 = (2a + 1 ) 2 + (2b ) 2

− b − b 2 + 4a ⇒ 3a 2 + 3b 2 + 6a = 0
∴ |z | ≤
2 ⇒ a 2 + b 2 + 2a = 0
− b + b 2 − 4a From option (c),
and |z | ≥
2 (1 + 5a ) 2 + (3b ) 2 = (1 − 4a ) 2
Case II | z | 2 − a ≤ b | z | ⇒ 9a 2 + 9b 2 + 18a = 0
⇒ | z |2 − b | z | − a ≤ 0 ∴ a 2 + b 2 + 2a = 0

b − b 2 + 4a b + b 2 + 4a 56. From Eq. (i), we get


∴ ≤|z | ≤  1 − tan 2 θ / 2   2 tan θ / 2 
2 2 (1 − a )   +b  = 2a + 1
From Case I and Case II, we get  1 + tan 2 θ / 2  1 + tan 2 θ / 2
− b + b 2 + 4a b + b 2 + 4a θ θ
≤|z | ≤ …(ii) ⇒ (1 − a ) − (1 − a ) tan 2 + 2b tan
2 2 2 2
θ
From Eqs. (i) and (ii), we get = (2a + 1 ) + (2a + 1 ) tan 2
2
− b + b 2 + 4a b + (b 2 + 4a ) θ θ
≤|z | ≤ ⇒ (2 + a ) tan 2 − 2b tan + 3a = 0
2 2 2 2
b + b 2 + 4a θ 2b ± 4b 2 − 12a (2 + a )
∴ The greatest value of | z | is ∴ tan =
2 2 2(2 + a )
− b + b 2 + 4a
and the least value of | z | is . 2b ± 4b 2 − 12( −b 2 )
2 = [Q a 2 + b 2 + 2a = 0 ]
− 2b 2 / a
52. Here, a = 1 and b = 2
(2b ± 4b ) a 6ba − 2ab 3a a
λ = Sum of the greatest and least values of | z | = = or =− or
− 2b 2
− 2b 2
− 2b 2
b b
= b 2 + 4a = 4 + 4 = 8 θ b b b θ θ
∴ cot = − or or − = 3 cot or − cot
∴ λ2 = 8 2 3a a a 2 2
53. Here, a = 2 and b = 4 57. ∴ a 2 + b 2 + 2a = 0 ⇒ (a + 1) 2 + b 2 = 1
λ = Sum of the greatest and least value of | z |. Now, | z | = 1 = (a + 1 ) 2 + b 2

= b 2 + 4a = 16 + 8 = 24 58. Q 1 + z + z 2 + z 3 + … + z 17 = 0

∴ λ2 = 24 1 ⋅ (1 − z 18 )
∴ =0
54. Here, a = 3 and b = 6 (1 − z )

λ = Sum of the greatest and least value of | z | ⇒ 1 − z 18 = 0, 1 − z ≠ 0

= b 2 + 4a = 36 + 12 = 48 = 4 3 ∴ z 18 = 1, z ≠ 1 …(i)

⇒ λ =2 3 and 1 + z + z + z + … + z
2 3 13
=0

∴ λ = 12
2 1 ⋅ (1 − z )
14
∴ =0
(1 − z )
Sol. (Q. Nos. 55 to 57)
z −1 ⇒ 1 − z 14 = 0, 1 − z ≠ 0
Q W = = a + ib
z +2 ∴ z 14 = 1, z ≠ 1 …(ii)

55. Q z = CiS θ = e From Eqs. (i) and (ii), we get

e −1 z 14 ⋅ z 4 = 1 ⇒ 1 ⋅ z 4 = 1
∴ = a + ib
e iθ + 2 ∴ z4 = 1
⇒(cosθ + i sin θ − 1 ) = (a + ib ) (cosθ + i sin θ + 2 ) Then, z = 1, − 1, i , − i
On comparing real and imaginary parts, we get Q z ≠1
cosθ − 1 = a cosθ + 2a − b sin θ Q z = − 1, i , − i
⇒ (1 − a ) cosθ + b sin θ = 2a + 1 …(i) Hence, only z = − 1 satisfy both Eqs. (i) and (ii).
and sin θ = a sin θ + b cosθ + 2b ∴ Number of values of z is 1.
(1 − a ) sin θ − b cosθ = 2b …(ii)
Chap 01 Complex Numbers 85

59. We have, z3 = z …(i) 3 1


For x = ,y = ± [from Eq. (i)]
⇒ |z | =|z | =|z |
3 2 2
3 1
⇒ | z | (| z | 2 − 1 ) = 0 For x = − ,y = ± [from Eq. (i)]
2 2
⇒ | z | = 0 and | z | 2 = 1
1 i 3 1 i 3 3 i 3 i
Now, | z | 2 = 1 ∴ Solutions are ± ,− ± , ± ,− ±
2 2 2 2 2 2 2 2
1 Hence, number of solutions is 8.
⇒ zz = 1 ⇒ z =
z 62. We have, x = a + ib
On putting this value in Eq. (i), we get
⇒ x 2 = (a 2 − b 2 ) + 2iab = 3 + 4i [given]
1
z3 =
z ∴ a − b = 3 and ab = 2
2 2
…(i)
⇒ z4 = 1 …(ii) and x = x ⋅ x = (a + ib ) [(a − b ) + 2iab ]
3 2 2 2

Clearly, Eq. (ii) has 4 solutions. = (a 3 − ab 2 − 2ab 2 ) + i [2a 2b + b(a 2 − b 2 )]


Therefore, the required number of solutions is 5. = (a 3 − 3ab 2 ) + i (3a 2b − b 3 ) = 2 + 11i [given]
60. We have, z = 9 + ai
∴ a − 3ab = 2
3 2

⇒ z 2 = (81 − a 2 ) + 18ai
and 3a 2b − b 3 = 11 …(ii)
z 3 = (729 − 27a 2 ) + (243a − a 3 ) i From Eq. (i), we get
According to the question, we have a 2 + b 2 = (a 2 − b 2 ) 2 + 4a 2 b 2 = 5
Im (z 2 ) = Im (z 3 )
Then, 2a 2 = 8, 2b 2 = 2
⇒ 18a = 243a − a 3
⇒ a (a − 225 ) = 0
2
∴ a 2 = 4, b 2 = 1
⇒ a = 0 or a = 225
2
⇒ a = 2, b = 1
But a≠0 and a = − 2, b = − 1 [Q ab = 2 ]
∴ a 2 = 225 Finally, a = 2, b = 1 satisfies Eq. (ii).
∴ The sum of digits of a 2 = 2 + 2 + 5 = 9 Hence, a + b =2 + 1 =3
63. Q (1 + i ) 4 = [(1 + i ) 2 ]2
61. Let z = x + iy
= (1 + i 2 + 2i ) 2 = (1 − 1 + 2i ) 2
Q |z | =1 …( i )
∴ x2 + y 2 = 1 = 4i 2 = − 4 …(i)
z z 1− πi π −i
and + =1 and +
z z π +i 1+ πi
x + iy x − iy (1 − π i ) ( π − i ) ( π − i ) (1 − π i )
⇒ + =1 = +
x − iy x + iy π+1 1+π

( x + iy ) 2 + ( x − iy ) 2 π − i − πi − π + π − πi − i − π
⇒ =1 =
x2 + y 2 π +1
− 2 πi − 2i
2 (x 2 − y 2 ) = = − 2i …(ii)
⇒ =1 [from Eq. (i)] π+1
1
π 1 − π i π −i 
1 Given, z = (1 + i ) 4  + 
⇒ x −y = ± 2 2
…(ii) 4  π +i 1 + π i
2
π
From Eqs. (i) and (ii), we get =
( − 4 ) ( −2i ) = 2πi [from Eqs. (i) and (ii)]
4
1 1 3
2x 2 = 1 ± = ,  |z |  2π
2 2 2 Now,   = =4
 amp (z) π / 2
1 3 1 3
⇒ x2 = , ⇒ x = ± , ±
4 4 2 2 64. Q An = 1
1
For x = , y = ±
3
[from Eq. (i)] ⇒ A = (1 )1/n = e 2π r i /n , r = 0, 1, 2, …, n − 1
2 2 ∴ A = 1, e 2 π i /n , e 4 π i /n , e 6 πi /n , …, e 2 π (n − 1) i /n
1 3
For x = − , y = ± [from Eq. (i)] and ( A + 1 )n = 1 ⇒ A + 1 = (1 )1/n = e 2 π pi /n
2 2
86 Textbook of Algebra

 πp   10  2q π 2q π  
⇒ A = e 2 π pi /n − 1 = e p π i /n ⋅ 2i sin   , = − i  ∑  cos + i sin  − 1
 n q = 0 11 11  
p = 0, 1, 2, …, n − 1 = − i {(sum of 11, 11th roots of unity) − 1}
 π  2π 
∴ A = 0, e π i /n ⋅ 2i sin   , e 2 π i /n 2i sin   ,..., = − i (0 − 1) = i
 n n p
32  10  2qπ 2qπ  
π i (n − 1)/n  π (n − 1 ) Q P = ∑ (3 p + 2 )  ∑ sin − i cos 
e ⋅ 2i sin   p =1  q =1  11 11  
 n 
For n = 6, 32
= ∑ (3 p + 2 ) (i ) p
e 4 π i /n = e 4 π i / 6 = e 2 π i / 3 p =1

2π 2π 1 i 3 32 32
= cos + i sin =− + = 3 ∑ p(i ) p + 2 ∑ (i ) p
3 3 2 2 p =1 p =1
π i /n  π πi /6  π
and e ⋅ 2i sin   = e ⋅ 2i sin   32
 n  6 = 3 ∑ p(i ) p + 0 = 3S (say)
p =1
 π π
=  cos + i sin  ⋅ i 32
 6 6 where, S = ∑ p(i ) p
p =1
 3 i 1 i 3
= + i =− +
 2 2 2 2 S = 1 ⋅ i + 2 ⋅ i 2 + 3 ⋅ i 3 + … + 31 ⋅ i 31 + 32 ⋅ i 32
Hence, the least value of n is 6. iS = 1 ⋅ i 2 + 2 ⋅ i 3 + … + 31 ⋅ i 32 + 32i 33
65. Given, z1, z 2, z 3, … , z 50 are the roots of the equation (1 − i ) S = (i + i 2 + i 3 + … + i 32 ) − 32i 33
50
= ( 0 ) − 32i
∑ (z )r = 0, then
r=0 32i ⋅ (1 + i )
∴ S =−
50 50 (1 − i ) ⋅ (1 + i )
∑ (z ) r
= (z − z1 ) (z − z 2 ) (z − z 3 ) … (z − z 50 ) = Π (z − zr )
r =1 = − 16 (i − 1 ) = 16 (1 − i )
r=0

Taking log on both sides on base e, we get ∴ P = 3S = 48 (1 − i )


Given, (1 + i ) P = n(n !) ⇒ (1 + i ) ⋅ 48 (1 − i ) = n(n !)
 50  50
loge  ∑ (z )r  = ∑ loge (z − zr ) ⇒ 96 = n(n !) ⇒ 4( 4 !) = n(n !)
 
r = 0  r =1
∴ n=4
On differentiating both sides w.r.t. z, we get
1+i (1 + i ) 2 1 + i 2 + 2i
50 67. Q = = =i
1 − i (1 − i ) (1 + i )
∑ r (z )r − 1 50
2
1 n
−1 1 + x 
r=0
∑ 1 + i 2
= 2
50
(z − zr ) Given,   = sin  
1 − i π
∑ (z )r r =1  2x 
r=0
2 1 + x2
On putting z = 1 in both sides, we get ⇒ in = sin − 1  
π  2x 
50

∑r 50 ⇒
1 + x2 π n
sin − 1   = (i )
r=0 1
50
= ∑ (1 − zr )
 2x  2
∑ 1 r =1

1 + x2 π 
= sin  (i )n  …(i)
r=0
2x 2 
(1 + 2 + 3 + … + 50 ) 1 50
⇒ =−∑ Now, AM ≥ GM
51 r =1
(zr − 1 ) 1
x+
x ≥1 ⇒ x + 1 ≥1
2
= − ( − 5λ ) [given]
50 2 2x
× 51  π n
⇒ 2 = 5λ ⇒ sin  (i )  ≥ 1 [Q −1 ≤ sin θ ≤ 1]
51 2 
⇒ λ =5 π 
∴ sin  (i )n  = 1
10
 2q π 2q π  2 
66. Q ∑ sin − i cos 
q =1 11 11  ⇒ n = 4, 8, 12, 16, …

10 2q π 2q π  ∴ Least positive integer, n = 4
= − i ∑  cos + i sin 
q = 1 11 11 
Chap 01 Complex Numbers 87

68. (A) → ( p,q ), (B) → ( p,r ),(C) → ( p,r ,s )  13 + 5 13 − 5 


(B) 5 − 12i = ±  −i  = ± (3 − 2i )
a  2 2 
If z ± = b, where a > 0 and b > 0, then
z  13 − 5 13 + 5 
and − 5 − 12i = ±  −i  = ± (2 − 3i )
− b + b 2 + 4a b + b 2 + 4a  2 2 
≤ |z | ≤
2 2 ∴ z = 5 − 12i + −5 − 12i = ± (3 − 2i ) ± (2 − 3i )
(A) Here, a = 1 and b = 2
= 5 − 5i , − 1 − i , − 5 + 5i , 1 + i
Then, − 1 + 2 ≤ | z | ≤ 1 + 2
∴ z1 = 5 − 5i , z 2 = − 1 − i ,
∴ G =1 + 2 z 3 = − 5 + 5i and z 4 = 1 + i
and L = −1 + 2
∴ | z1| 2 + | z 2 | 2 + | z 3| 2 + | z 4 | 2 = 50 + 2 + 50 + 2
⇒ G − L =2 [natural number and prime number]
= 104 = 8 × 13
(B) Here, a = 2 and b = 4
 17 + 8 17 − 8 
Then, − 2 + 6 ≤ | z | ≤ 2 + 6 (C) 8 + 15i = ±  +i 
 2 2 
∴ G =2 + 6
 5 3i  1
and L = −2 + 6 =± + =± (5 + 3i )
 2 2  2
⇒ G − L = 4 [natural number and composite number]
 17 − 8 17 + 8
(C) Here, a = 3 and b = 6 and − 8 − 15i = ±  −i 
 2 2 
Then, − 3 + 2 3 ≤ | z | ≤ 3 + 2 3
 3 5  1
∴ G =3 + 2 3 =± − i = ± (3 − 5i )
 2 2  2
and L = −3 + 2 3
∴ z = 8 + 15i + − 8 − 15i
⇒ G − L =6
1 1
[natural number, composite number and perfect number] =± (5 + 3i ) ± (3 − 5i )
2 2
69. (A) → (q), B → (q, r), C → (q, s)
1 1
We know that, z= (8 − 2i ), ( − 2 − 8i ),
2 2
 | z | − Re (z ) | z | − Re (z )  1 1
z =± +i  ( − 8 + 2i ), (2 + 8i )
 2 2  2 2
 | z | + Re (z ) | z | − Re (z )  ∴ z1 = 2 ( 4 − i ), z 2 = 2 ( − 1 − 4i )
If Im(z ) > 0 = ±  −i 
 2 2  z 3 = 2 ( − 4 + i ) and z 4 = 2 (1 + 4i )
If Im (z ) < 0
∴ | z1| + | z 2 | 2 + | z 3| 2 + | z 4 | 2 = 34 + 34 + 34 + 34
2
 10 + 6 10 − 6  = 136 = 17 × 8
(A) 6 + 8i = ±  +i 
 2 2 
70. (A) → (p, q, r, t);(B) → (p,s);(C) → (p, r)
= ± (2 2 + i 2 ) (A) Here, the last digit of 143 is 3. The remainder when 861 is
= ± 2 (2 + i ) divided by 4 is 1. Then, press switch number 1 and we get
3. Hence, the digit in the units place of (143 ) 861 is 3.
 10 − 6 10 + 6 
and − 6 + 8i = ±  +i  ∴ λ =3
 2 2 
Next, the last digit of 5273 is 3. The remainder when 1358
= ± ( 2 + i 2 2 ) = ± 2 (1 + 2i ) is divided by 4 is 2. Then, press switch number 2 and we
∴ z = 6 + 8i + − 6 + 8i get 9. Hence, the digit in the units place of (5273 )1358 is 9.
∴ µ =9
= ± 2 (2 + i ) ± 2 (1 + 2i )
Hence, λ + µ = 3 + 9 = 12
= 3 2 (1 + i ), 2 (1 − i ), − 3 2 (1 + i ), 2 ( − 1 + i ) which is divisible by 2, 3, 4 and 6.
∴ z1 = 3 2 (1 + i ), z 2 = 2 (1 − i ), (B) Here, the last digit of 212 is 2. The remainder when 7820
is divided by 4 is 0. Then, press switch number 0 and we
z 3 = − 3 2 (1 + i ) get 6. Hence, the digit in the unit’s place of (212 ) 7820 is 6.
and z 4 = 2 (− 1 + i ) ∴ λ =6
∴ | z 1 | + | z 2 | + | z 3| + | z 4 |
2 2 2 2 Next, the last digit of 1322 is 2. The remainder when 1594
is divided by 4 is 2. Then, press switch number 2 and we
= 36 + 4 + 36 + 4 = 80 which is divisible by 8. get 4.
88 Textbook of Algebra

Hence, the digit in the unit’s place of (1322 )1594 is 4. 1 + z 1 + z2 + z3


⇒ =0
∴ µ=4 3
z 1+ z2 + z3 1
Hence, λ + µ = 6 + 4 = 10, which is divisible by 2 and 5. ⇒ =−
3 3
(C) Here, the last digit of 136 is 6. Therefore, the unit’s place
 1 
of (136 ) 786 is 6. ∴ Centroid of the triangle is  − , 0 .
 3 
∴ λ =6
Next, the last digit of 7138 is 8. The remainder when So, the circumcentre and centroid of the triangle coincide.
Hence, required triangle is an equilateral triangle.
13491 is divided by 4 is 3. Then, press switch number 3
and we get 2. Hence, unit’s place of (7138 )13491 is 2. Therefore, Statement-1 is true. Also, z1, z 2 and z 3 represent
vertices of an equilateral triangle, if
∴ µ =2
z12 + z 22 + z 32 − (z1z 2 + z 2 z 3 + z 3z1 ) = 0.
Hence, λ + µ =6 + 2 =8
Therefore, Statement-2 is false.
which is divisible by 2 and 4.
75. We have,
71. (A) → (r); ( B) → (p,s); ( C) → (q, t)
|z − 1| + |z − 8| = 5 …(i)
a Here, z1 = 1, z 2 = 8 and 2a = 5
If z − = b, where a > 0 and b > 0, then
z Now, | z1 − z 2 | = | 1 − 8 | = | − 7 | = 7
− b + b 2 + 4a b + b 2 + 4a ∴ 2a = 5 < 7
≤ |z | ≤
2 2 Therefore, locus of Eq. (i) does not represent an ellipse. Hence,
Statement-1 is false. Statement-2 is true by the property of
b + b 2 + 4a − b + b 2 + 4a ellipse.
∴ λ= and µ =
2 2 76. Since, z1, z 2 and z 3 are in AP.
(A) Here, a = 6 and b = 5
∴ 2z 2 = z1 + z 3
∴ λ = 6 and µ = 1
z + z3
⇒ λµ + µ λ = 61 + 1 6 = 7 ⇒ z2 = 1
2
and λµ − µ λ = 61 − 1 6 = 5 It is clear that, z 2 is the mid-point of z1 and z 3.
(B) Here, a = 7 and b = 6 ∴ z1, z 2 and z 3 are collinear.
∴ λ = 7 and µ = 1 Statement-1 is true, Statement-2 is true; Statement-2 is a
µ λ
∴ λ + µ = 71 + 1 7 = 8 correct explanation of Statement-1.
and λµ − µ λ = 71 − 1 7 = 6 77. Principal argument of a complex number depend upon
quadrant and principal argument lies in ( − π , π ].
(C) Here, a = 8 and b = 7 Hence, Statement-1 is always not true and Statement-2 is
∴ λ = 8 and µ = 1 obviously true.
⇒ λµ + µ λ = 81 + 1 8 = 9 π
78. We have, C1 : arg (z ) =
and λµ − µ λ = 81 − 1 8 = 7 4
−1 y  π
72. Statement-1 is false because 3 + 7i > 2 + 4i is meaningless in ⇒ tan   = [ let z = x + iy ]
 x 4
the set of complex number as set of complex number does not
hold ordering. But Statement-2 is true. y π
⇒ = tan = 1
73. Statement-1 is false as x 4
⇒ y =x
(cos θ + i sin φ )n ≠ cos nθ + i sin nφ
2 ∴ C1 : y = x …(i)
 π π π π
Now,  cos + i sin  = cos + i sin 3π
 4 4 2 2 C 2 : arg(z ) =
4
=i [by De-Moivre’s theorem] −1 y  3π
⇒ tan   = [ let z = x + iy ]
∴ Statement-2 is true.  x 4
74. We have, y 3π
⇒ = tan = −1
| 3z1 + 1 | = | 3z 2 + 1 | = | 3z 3 + 1 | x 4
 1  ⇒ y =−x
∴ z1, z 2 and z 3 are equidistant from  − , 0 and circumcentre
 3  ∴ C2 : y = − x …(ii)
 1  and C 3 : arg (z − 5 − 5i ) = π
of triangle is  − , 0 .
 3   y − 5
⇒ tan − 1   =π [ let z = x + iy ]
Also, 1 + z1 + z 2 + z 3 = 0  x − 5
Chap 01 Complex Numbers 89

y −5 On putting x = ω in Eq. (i), we get


⇒ = tan π = 0 ⇒ y = 5
x −5 ω5 − 1
= (ω − α 1 ) (ω − α 2 ) (ω − α 3 ) (ω − α 4 )
∴ C3 : y = 5 …(iii) ω −1
We get the following figure. ω2 − 1
⇒ = (ω − α 1 ) (ω − α 2 )(ω − α 3 ) (ω − α 4 ) ...(ii)
Y ω −1

x
=
and putting x = ω 2 in Eq. (i), we get

:y
1
C
B C3 : y=5 ω10 − 1
= (ω 2 − α 1 ) (ω 2 − α 2 ) (ω 2 − α 3 )(ω 2 − α 4 )
(–5, 5) A(5, 5) ω2 − 1
C2 : y= – x ω −1
X ⇒ = (ω 2 − α 1 ) (ω 2 − α 2 ) (ω 2 − α 3 )(ω 2 − α 4 ) ...(iii)
O(0, 0) ω 2 −1
On dividing Eq. (ii) by Eq. (iii), we get
ω − α1 ω − α 2 ω − α 3 ω − α 4 (ω 2 − 1 ) 2
⋅ 2 ⋅ 2 ⋅ 2 =
∴ Area of the region bounded by C1, C 2 and C 3 ω − α1 ω − α 2 ω − α 3 ω − α 4
2
(ω − 1 ) 2
1 5−0 5−0 ω 4 + 1 − 2ω 2 ω + 1 − 2 ω 2
= = 25 = =
2 −5 − 0 5 − 0 ω 2 + 1 − 2ω ω 2 + 1 − 2 ω
∴ Statement-1 is false. − ω 2 − 2ω 2 − 3ω 2
= = =ω
Now, OA = 5 2, OB = 5 2 and AB = 10 − ω − 2ω − 3ω
(OA ) 2 + (OB ) 2 = ( AB ) 2 and OA = OB z +z
Q 82. Let z = x + iy , then =x
2
Therefore, the boundary of C1, C 2 and C 3 constitutes right
∴ From given relation, we get
isosceles triangle.
Hence, Statement-2 is true. ⇒ x = x + iy − 1
⇒ x = ( x − 1 ) + iy
z 2 z 3 − (z 2 z 3 ) 1
79. Since, Im ( z 2 z 3 ) = = {z 2 z 3 − z 2 z 3 } ⇒ x 2 = (x − 1)2 + y 2 ⇒ 2x = 1 + y 2
2i 2i
1 If z1 = x1 + iy1 and z 2 = x 2 + iy 2
z1 Im ( z 2 z 3 ) = {z1z 2 z 3 − z1z 2 z 3} ...(i)
2i Then, 2 x1 = 1 + y12 ...(i)
1
Similarly, z 2 Im ( z 3 z1 ) = {z 2 z 3 z1 − z 2 z1 z 3} ...(ii) and 2 x 2 = 1 + y 22 ...(ii)
2i On subtracting Eq. (ii) from Eq. (i), we get
1
and z 3 Im ( z1 z 2 ) = {z 3 z1 z 2 − z 3z1z 2} ...(iii) 2 ( x1 − x 2 ) = y12 − y 22
2i
2 ( x1 − x 2 ) = (y1 + y 2 ) (y1 − y 2 ) ...(iii)
On adding Eqs. (i), (ii) and (iii), we get
But, given that arg (z1 − z 2 ) = π / 4
z1 Im ( z 2 z 3 ) + z 2 Im ( z 3 z1 ) + z 3 Im ( z1 z 2 ) = 0
y − y 2 π y − y2
Therefore, this is proved. Then, tan − 1  1  = ⇒ 1 =1
 1
x − x 2 4 x1 − x 2
80. Since, z1, z 2 and z 3 are the roots of
∴ y1 − y 2 = x1 − x 2 ...(iv)
x 3 + 3ax 2 + 3bx + c = 0,
From Eqs. (iii) and (iv), we get
we get z1 + z 2 + z 3 = − 3a
y1 + y 2 = 2 [Q y1 − y 2 ≠ 0]
z1 + z 2 + z 3
⇒ = −a ∴ Im (z1 + z 2 ) = 2
3
Hence, the imaginary part (z1 + z 2 ) is 2.
and z1z 2 + z 2 z 3 + z 3z1 = 3b
83. (i) LHS = (a 2 + b 2 + c 2 − bc − ca − ab )
Hence, the centroid of the ∆ABC is the point of affix ( − a ).
( x 2 + y 2 + z 2 − yz − zx − xy )
Now, the triangle will be equilateral, if
= (a + bω + c ω 2 ) (a + bω 2 + cω )
z12 + z 22 + z 32 = z1z 2 + z 2 z 3 + z 3z1
( x + yω + zω 2 ) ( x + yω 2 + zω )
⇒ (z1 + z 2 + z 3 ) 2 = 3(z1z 2 + z 2 z 3 + z 3z1 )
= {(a + bω + cω 2 ) ( x + yω + zω 2 )}
⇒ ( − 3a ) = 3(3b )
2
{(a + bω 2 + cω ) ( x + yω 2 + zω )}
Therefore, the condition is a = b. 2
= {ax + cy + bz + ω (bx + ay + cz )
81. Q x 5 − 1 = 0 has roots 1, α 1, α 2, α 3, α 4 . + ω 2 (cx + by + az )} × {ax + cy + bz + ω 2
(bx + ay + cz ) + ω (cx + by + az )}
∴ (x 5 −1) = (x − 1) (x − α1 ) (x − α 2 ) (x − α 3 ) (x − α 4 ) = ( X + ωΖ + ω 2 Y ) ( X + ω 2 Z + ω Y )
x 5 −1 = RHS
⇒ = (x − α1 ) (x − α 2 ) (x − α 3 ) (x − α 4 ) ...(i)
x −1
90 Textbook of Algebra

(ii) LHS = (a 3+ b 3 + c 3 − 3abc ) ( x 3+ y 3 + z 3 − 3 xyz ) Using Eqs. (i) and (ii), then equations of lines are
= (a + b + c ) (a 2 + b 2 + c 2 − ab − bc − ca ) × z + z 3 (z − z )
( x + y + z )( x 2+ y 2 + z 2 − xy − yz − zx ) + − 11 = 0
2 2i
= (a + b + c ) ( x + y + z ) 3 (z + z ) (z − z )
(a 2 + b 2 + c 2 − ab − bc − ca ) × and − +7=0
2 2i
( x + y + z − xy − yz − zx ) [using (i) part]
2 2 2
i.e., (1 − 3i ) z + (1 + 3i ) z − 22 = 0
= (ax + ay + az + bx + by + bz + cx + cy + cz ) and (3 + i ) z + (3 − i ) z + 14 = 0
( X 2 + Y 2 + Z 2 − YZ − ZX − XY ) 1+i
86. Putting = x in LHS, we get
= {(ax + cy + bz ) + (cx + by + az ) + (bx + ay + cz )} 2
( X 2 + Y 2 + Z 2 − YZ − ZX − XY ) 2
LHS = (1 + x ) (1 + x 2 ) (1 + x 2 ) … (1 + x 2 )
n

= ( X + Y + Z ) ( X + Y + Z − YZ − ZX − XY )
2 2 2 2 n
(1 − x ) (1 + x ) (1 + x 2 ) (1 + x 2 ) … (1 + x 2 )
=X 3
+ Y + Z − 3 XYZ = RHS
3 3 =
(1 − x )
84. Let z = x + iy 2 n
(1 − x 2 ) (1 + x 2 ) (1 + x 2 ) … (1 + x 2 )
∴ |z |2 = x 2 + y 2 =
(1 − x )
∴ x 2 + y 2 − 2i ( x + iy ) + 2c (1 + i ) = 0 2 2 n
(1 − x 2 ) (1 + x 2 ) … (1 + x 2 )
=
( x + y + 2y + 2c ) + i ( − 2 x + 2c ) = 0
2 2
(1 − x )
On comparing the real and imaginary parts, we get n n n
(1 − x 2 ) (1 + x 2 ) 1 − ( x 2 ) 2
x 2 + y 2 + 2y + 2c = 0 …(i) = =
(1 − x ) (1 − x )
and − 2 x + 2c = 0 …(ii) 2n
i
From Eqs. (i) and (ii), we get 1− 
 2  1 + i
y 2 + 2y + c 2 + 2c = 0 =
1 + i Q x = 2 
−2 ± 4 − 4(c 2 + 2c ) 1− 
⇒ y = = − 1 ± (1 − c 2 − 2c )  2 
2 1
1 − n (1 )
Q x and y are real.
22 (1 + i )  1 
∴ 1 − c 2 − 2c ≥ 0 or c 2 + 2c + 1 ≤ 2 = ⋅ = (1 + i ) 1 − n  = RHS
 1 − i  (1 + i )  22 
 
(c + 1 ) 2 ≤ ( 2 ) 2 ⇒ − 2 − 1 ≤ c ≤ 2 − 1  2 
∴ 0 ≤c ≤ 2 −1 [Q given c ≥ 0] 87. Since, arg (z − 3i ) = 3π / 4 is a ray which is start from 3i and
makes an angle 3 π / 4 with positive real axis as shown in the
Hence, the solution is z = x + iy = c + i ( − 1 ± 1 − c 2 − 2c ) figure.
for 0 ≤ c ≤ 2 − 1 x+y=3 Y
and z = x + iy ≡ no solution for c > 2 − 1
85. Let z = x + iy (0, 3) 3
z +z y=x+ –
2
∴ Re (z ) = x = …(i) 2
2
z −z 3/2
and Im (z ) = y = …(ii)
2i 1
The equation (2 − i ) z + (2 + i ) z + 3 = 0 can be written as X′ X
1/2 O
2(z + z ) − i (z − z ) + 3 = 0
or 4 x + 2y + 3 = 0 Y′
∴ Slope of the given line, m = − 2 ∴ Equation of ray in cartesian form is
Let slope of the required line be m1, then y − 3 = tan (3 π / 4 ) ( x − 0 )
m −m   ⇒ 1 =  m1 + 2 ⇒ ± 1 = m1 + 2 or y − 3 = − x or x + y = 3
tan 45 ° =  1
1 + m1m   1 − 2m1  1 − 2m1 and arg (2z + 1 − 2i ) = π / 4
1   1 
∴ m1 = − , 3 ⇒ arg 2 z + − i   = π / 4
3   2 
∴ Equation of straight lines through ( − 1, 4 ) and having slopes  1 
or arg(2 ) + arg z + − i  = π / 4
1 1
− and 3 are y − 4 = − ( x + 1 ) and y − 4 = 3 ( x + 1 )  2 
3 3  1 
⇒ x + 3y − 11 = 0 and 3 x − y + 7 = 0 or 0 + arg z + − i  = π / 4
 2 
Chap 01 Complex Numbers 91

  1  On subtracting Eq. (i) from Eq. (ii), we get


or arg z −  − + i   = π / 4
  2  z (c −2 − a −1b −1 ) = 2c −1 − a −1 − b −1
1
which is a ray that start from point − + i and makes an angle 2c −1 − a −1 − b −1
2 Hence, z=
π / 4 with positive real axis as shown in the figure. c −2 − a −1b −1
∴Equation of ray in cartesian form is which is a required point.
y − 1 = 1 [ x − ( − 1 / 2 )] ⇒ y = x + 3 / 2 90. Q AD ⊥ BC
From the figure, it is clear that the system of equations has no ∴ AP is also perpendicular to BC.
solution. A(z1)
88. Let a = r cos α and 0 = r sin α …(i)
So that, a 2 + 02 = r 2
∴ r = |a | O
Then, a = | a | cos α [from Eq. (i)]
C(z3)
∴ cos α = ± 1 (z2)B D
Then, cos α = 1 or − 1 according as a is + ve or – ve and
sin α = 0. P(z)
Hence, α = 0 or π according as a is + ve and – ve.  z −z  π
Then, arg  1  =
Again, let 0 = r1 cos β or b = r1 sin β …(ii) z3 − z2 2
So that, 0 2 + b 2 = r12  z −z 
∴ Re  1  =0
∴ r1 = | b | z3 − z2
From Eq. (ii), we get b = | b | sin β z1 − z z −z
+ 1
∴ sin β = ± 1 z3 − z2 z 3 − z 2
⇒ =0
Then, sin β = 1 or − 1 according as b is + ve or – ve and cosβ = 0. 2
π π z1 − z z1 − z
Hence, β = or − according as b is + ve or −ve. ⇒ + =0 …(i)
2 2 z3 − z2 z 3 − z 2
89. Let two non-parallel straight lines PQ, RS meet the circle But O is the circumcentre of ∆ABC, then
| z | = r in the points a, b and c. OP = OA = OB = OC
P a | z | = | z1 | = | z 2 | = | z 3 |
b S On squaring the above relation, we get
O
z | z | 2 = | z 1| 2 = | z 2 | 2 = | z 3 | 2
Q
⇒ zz = z1z1 = z 2z 2 = z 3z 3
c z z
R
From first two relations 1 = …(ii)
z z1
Then, | a | = r , | b | = r and | c | = r or | a | 2 = | b | 2 = | c | 2 = r 2 z z
From first and third relation 2 = …(iii)
∴ a a = b b = c c = r 2, z z2
r2 r2 r2 z z
then a = , b = and c = and from first and fourth relation 3 = …(iv)
a b c z z3
1 z1
z z −1
z1 − z
Points a, b and z are collinear, then a a 1 =0 From Eq. (i), we get + z =0 …(v)
z3 − z2 z3 − z2
b b 1
z z
∴ z ( a − b ) − z (a − b ) + ab − a b = 0 From Eqs. (ii), (iii), (iv) and (v), we get
r 2 r 2 r 2a r 2b z
⇒ z  −  − z (a − b ) + − =0 −1
a b b a z1 − z z1
+ =0
z3 − z2 z

z
On dividing both sides by r 2 (b − a ), we get
z3 z2
z z
+ 2 = a −1 + b −1 …(i)  z1 − z  z2z3   z1 − z 
ab r ⇒  1 + =0 Q z − z ≠ 0 
z3 − z2  zz1   3 2 
For RS, replace a = b = c in Eq. (i), then
z z z 2z3 z z
+ = 2c −1 …(ii) ⇒ 1+ =0 ⇒z =− 2 3
c2 r2 z z1 z1
92 Textbook of Algebra

91. From the figure, 92. Given, OA = 1 and | z | = 1


α = ( arg (z ) − arg (ω )) …(i)
2
Y P0(z0)
α α
and for every α, sin ≤  2
…(ii)
2  2

A(z)
P(z)
Imaginary axis

A(1)
|z| |z – ω | O
X

B(ω)
α Q(zz0)
|ω |
O Real axis ∴ = |z − 0 | = |z | = 1
OP
∴ = OA
OP
In ∆OAB, from cosine rule = |z 0 − 0 | = |z 0 |
OP0
( AB ) 2 = (OA ) 2 + (OB ) 2 − 2OA ⋅ OB cos α and = | zz 0 − 0 | = | zz 0| = | z | | z 0| = 1 | z 0| = | z 0|
OQ
⇒ | z − ω | 2 = | z | 2 + | ω | 2 − 2 | z | | ω | cos α ∴ = OQ
OP0
⇒ | z − ω | 2 = (| z | − | ω | ) 2 + 2 | z | | ω | (1 − cos α )  z − 0 z  z z 
Also, ∠P0OP = arg  0  = arg  0  = arg  0 
 z − 0   z  zz 
α
⇒ | z − ω | 2 = (| z | − | ω | ) 2 + 4 | z | | ω |sin 2
2 z z  z z 
= arg  20  = arg  0  = − arg (z z 0 )
2  |z |   1 
α
⇒ | z − ω | 2 ≤ (| z | − | ω | ) 2 + 4 | z | | ω |   [from Eq. (ii)]
 2  1 
= − arg (z z 0 ) = arg  
⇒ | z − ω | ≤ (| z | − | ω | ) + α
2 2 2
[Q | z | ≤ 1, | ω | ≤ 1] z z0
| z − ω | ≤ (| z | − | ω | ) + ( arg (z ) − arg (ω )) 2
2 2
[from Eq. (i)]  1−0 
= arg   = ∠AOQ
 z z 0 − 0
I. Aliter
Let z = r (cosθ + i sin θ ) and ω = r1 (cos θ1 + i sin θ1 ) , Thus, the triangles POP0 and AOQ are congruent.
then | z | = r and | ω | = r1 ∴ PP0 = AQ
Also, arg (z ) = θ and arg (ω ) = θ1 | z − z 0| = | z z 0 − 1|
and r ≤ 1 and r1 ≤ 1 [Q given | z | ≤ 1, | ω | ≤ 1 ] 93. Let the equation of line passing through the origin be
We have, z − ω = (r cos θ − r1 cos θ1 ) + i (r sin θ − r1 sin θ1 ) a z + az = 0 …(i)
∴ | z − ω | 2 = (r cos θ − r1 cos θ1 ) 2 + (r sin θ − r1 sin θ1 ) 2 According to the question, z1, z 2, …, zn all lie on one side of
⇒ | z − ω | 2 = r 2 + r12 − 2rr1 cos (θ − θ1 ) line (i)
∴ a zi + azi > 0 or < 0 for all i = 1, 2, 3, …, n …(ii)
= (r − r1 ) 2 + 2rr1 − 2rr1 cos (θ − θ1 ) n n
= (r − r1 ) + 2rr1 (1 − cos (θ − θ1 ))
2 ⇒ a ∑ zi + a ∑ zi > 0 or < 0 …(iii)
i =1 i =1
 θ − θ1   n
= (r − r1 ) 2 + 4rr1 sin 2   ⇒
n
∑ zi ≠ 0
n
 2  If ∑ zi = 0, then ∑ zi = 0,
i =1  i =1 i =1
2
 θ − θ1  n n 
≤ (r − r1 ) + 4rr1 2
 [Q|sin θ | ≤ | θ | ] hence a ∑ zi + a ∑ zi = 0 
 2  i =1 i =1 
= (r − r1 ) + rr1(θ − θ1 )
2 2
From Eq. (ii), we get
≤ (r − r1 ) 2 + (θ − θ1 ) 2 [Qr , r1 ≤ 1] a zi + azi > 0 or < 0 for all i = 1, 2, 3, …, n
⇒ | z − ω | 2 ≤ (| z | − | ω | ) 2 + ( arg z − arg ω ) 2 a zi zi azi zi
⇒ + > 0 or < 0
zi zi
II. Aliter
a a
Let z = r cos θ ⇒ | zi | 2  +  > 0 or < 0
and ω = r1 cos θ1  i
z z i
a a
∴ r 2 + r12 − 2rr1 cos (θ − θ1 ) ≤ r 2 + r12 − 2rr1 + (θ − θ1 ) 2 ⇒ + > 0 or < 0 for all i = 1, 2, 3, …, n
zi zi
 θ − θ1   θ − θ1 
2
Q r , r1 ≤ 1 
⇒ rr1 sin 2   ≤   2
1 1 1
 2   2  2
≤ ⇒ , , …, lie on one side of the line a z + az = 0
 and sin x x  z1 z 2 zn
Chap 01 Complex Numbers 93

n 1 n 1
or a ∑ +a ∑ > 0 or < 0 z2
i = 1 zi i = 1 zi z1
r
r
n 1  n 1 n 1 o
Therefore, ∑ ≠ 0 If ∑ = 0, then ∑ =0
i = 1 zi  i = 1 zi i = 1 zi r
n 1 n 1 
⇒ a ∑ +a ∑ = 0 z3
i = 1 zi i = 1 zi  From the given condition,
94. Given, | a | | b | = ab 2c ; | a | = | c |; az 2 + bz + c = 0, then we r1 r2 r3
have to prove that | z | = 1 r2 r3 r1 = 0
On squaring, we get r3 r1 r2
| a | 2 | b | 2 = a b 2c and | a | 2 = | c | 2
⇒ r13 + r23 + r33 − 3r1r2 r3 = 0
⇒ a a b b = a b 2c and a a =c c
1
⇒ a b =b c and a a = c c …(i) ⇒ (r1 + r2 + r3 ) {(r1 − r2 ) 2 + (r2 − r3 ) 2 + (r3 − r1 ) 2 } = 0
2
If z1 and z 2 are the roots of az 2 + bz + c = 0 Since, r1 + r2 + r3 ≠ 0,
Then, z1 and z 2 are the roots of a ( z ) 2 + b z + c = 0 …(A) then (r1 − r2 ) 2 + (r2 − r3 ) 2 + (r3 − r1 ) 2 = 0
b c
∴ z1 + z 2 = − , z1z 2 = It is possible only when
a a
 …(ii) r1 − r2 = r2 − r3 = r3 − r1 = 0
b c 
and z1 + z 2 = − , z1z 2 = ∴ r1 = r2 = r3
a a 
1 1 z1 + z 2 − b / a b b and | z1 | = | z 2 | = | z 3 | = r [say]
Q + = = = − = − = z1 + z 2 Hence, z1, z 2, z 3 lie on a circle with the centre at the origin.
z1 z 2 z1 z 2 c /a c a
[from Eqs. (i) and (ii)]
1 1 z1 + z 2 −b / a (ii) Again, in ∆ oz 2 z 3 by Coni method
and + = =
z1 z 2 z1 z 2 c /a  z − 0 z 
arg  3  = ∠z 2 oz 3 ⇒ arg  3  = ∠z 2 oz 3 …(i)
b a bc b  2
z − 0  z2
=− =− = − = z1 + z 2
c ca a a In ∆ z 2z1z 3 by Coni method
[from Eqs. (i) and (ii)]  z − z1  1
1 1 arg  3  = ∠z 2 z1 z 3 = ∠z 2 oz 3 [property of circle]
Now, it is clear that z1 = and z 2 =  z 2 − z1  2
z1 z2
1 z 
=
arg  3  [from Eq. (i)]
Then, | z1 | 2 = 1 and | z 2 | 2 = 1 2  z1 
Hence, |z | = 1 z3  z 3 − z1 
∴ arg   = 2 arg  
Conversely For az 2 + bz + c = 0, we have to prove  z1   z 2 − z1 
| z | = 1 ⇒| a | | b | = a b 2c 2
z   z − z1 
and |a | = |c | Hence, arg  3  = arg  3 
 z1   z 2 − z1 
1
| z | = 1 ⇒ | z | 2 = 1 ⇒ z z = 1 ⇒z =
z 96. We know that,
From Eq. (A), we get Re (z1z 2 ) ≤ | z1z 2 |
 1
2
 1 ∴ | z1 | + | z 2 | + 2 Re (z1 z 2 ) ≤ | z1 | 2 + | z 2 | 2 + 2 | z1 z 2|
2 2
a   +b   + c = 0 or cz + b z + a = 0
2
z z ⇒ | z1 + z 2 | 2 ≤ | z1 | 2 + | z 2 | 2 + 2 | z1 | | z 2 | …(i)
Also, az + bz + c = 0, on comparing
2
Also, AM ≥ GM
2
c b a  1 
= = ( c | z1 | ) 2 +  | z | 1 /2
a b c  c 2   1 
∴ ≥  c ⋅| z1 | 2 ⋅ | z 2 | 2  [Qc > 0]
∴ a a = c c and a b = b cs 2  c 
⇒ | a | = | c | and | a | | b | = a b 2c 1
⇒ c | z 1| 2 + | z 2 | 2 ≥ 2 | z 1 | | z 2 |
95. (i) Let z1 = r1 (cos α + i sin α ), c
1
z 2 = r2 (cos β + i sin β ) and z 3 = r3 (cos γ + i sin γ ) ∴ | z1 | + | z 2 | 2 + 2 | z1 | | z 2 | ≤ | z1 | 2 + | z 2 | 2 + c | z1 | 2 + | z 2 | 2
2
c
∴ | z1 | = r1, | z 2 | = r2, | z 3 | = r3
⇒ | z1| 2 + | z 2 | 2 + 2 | z1 | | z 2 | ≤ (1 + c ) | z1 | 2 + (1 + c −1 ) (| z 2 | 2 )
and arg (z1 ) = α , arg (z 2 ) = β, arg (z 3 ) = γ
…(ii)
94 Textbook of Algebra

From Eqs. (i) and (ii), we get 98. Let z be the complex number corresponding to the orthocentre
| z1 + z 2 | 2 ≤ (1 + c ) | z1 | 2 + (1 + c −1) | z 2 | 2 O, since AD ⊥ BC , we get
A(z 1)
Aliter
Here, (1 + c ) | z1 | 2 + (1 + c −1 ) | z 2 | 2 − | z1 + z 2 | 2
F
 1 E
= (1 + c ) z1z1 + 1 +  z 2 z 2 − (z1 + z 2 ) ( z1 + z 2 )
 c O(z)

 1
= (1 + c ) z1z1 + 1 +  z 2 z 2 − z1z1 − z1z 2 − z 2 z1 − z 2 z 2
 c
1 B(z 2 ) D C(z 3 )
= c z1 z1 + z 2 z 2 − z1z 2 − z 2 z1
c
1  z − z1  π
= {c 2z1z1 + z 2 z 2 − cz1z 2 − cz 2 z1 } arg   =
c z2 − z3 2
1 z − z1
= {cz1 (cz1 − z 2 ) − z 2 (cz1 − z 2 )} i. e. is purely imaginary.
c z2 − z3
1 1
= (cz1 − z 2 ) (cz1 − z 2 ) = (cz1 − z 2 ) (cz1 − z 2 )  z − z1  z − z1 z − z1
c c i. e. Re   = 0 or + =0 ...(i)
1  2
z − z 3 z 2 − z 3 z 2 − z3
= | cz1 − z 2 | ≥ 0 as c > 0
2
c z − z2 z − z2
Similarly, + =0 [Q BE ⊥ CA ] ...(ii)
 1 z 3 − z1 z 3 − z1
∴ (1 + c ) | z1| 2 + 1 +  | z 2 | 2 − | z1 + z 2 | 2 ≥ 0
 c
From Eq. (i), we get
 1
Hence, | z1 + z 2 | 2 ≤ (1 + c ) | z1| 2 + 1 +  | z 2 | 2 (z − z 2 ) ( z 2 − z 3 )
 c z = z1 − ...(iii)
(z 2 − z 3 )
97. If z be the complex number corresponding to the circumcentre
O, then we have From Eq. (ii), we get
OA = OB = OC (z − z 2 ) ( z 3 − z1 )
z = z2 − ...(iv)
A(z 1) (z 3 − z1 )
Eliminating z from Eqs. (iii) and (iv), we get
( z − z1 ) (z − z 2 ) ( z 3 − z1 )
z1 − z 2 = (z 2 − z 3 ) −
(z) (z 2 − z 3 ) (z 3 − z1 )
O
or (z − z1 ) ( z 2 − z 3 ) ( z 3 − z1 ) − (z − z 2 ) (z 3 − z1 ) (z 2 − z 3 )
C(z 3)
B(z 2) = ( z1 − z 2 ) (z 2 − z 3 ) (z 3 − z1 )
⇒ | z − z1 | = | z − z 2 | = | z − z 3 | or z {( z 2 − z 3 ) (z 3 − z1 ) − (z 3 − z1 ) (z 2 − z 3 )}
⇒ | z − z1 | 2 = | z − z 2 | 2 = | z − z 3 | 2 = ( z1 − z 2 ) (z 2 − z 3 ) (z 3 − z1 ) + z1 (z 2 − z 3 ) ( z 3 − z1 )
− z 2 ( z 3 − z1 ) (z 2 − z 3 )
⇒ (z − z1 ) ( z − z1 ) = (z − z 2 ) ( z − z 2 )
⇒ z [ z 2 z 3 − z 2 z1 − z 3z 3 + z 3z1 − z 3z 2 + z 3z 3 + z1z 2 − z1 z 3 ]
= (z − z 3 ) ( z − z 3 ) …(i)
From first two members of Eq. (i), we get = ( z1 − z 2 ) {z 2 z 3 − z 2z1 − z 32 + z 3z1 }

z (z 2 − z1 ) = z1 (z − z1 ) − z 2 (z − z 2 ) …(ii) + ( z 2 − z 3 ) (z 3z1 − z12 ) + ( z 3 − z1 ) (z 2 z 3 − z 22 )


and from last two members of Eq. (i), we get = − {z12 ( z 2 − z 3 ) + z 22 ( z 3 − z1 ) + z 32 ( z1 − z 2 )}
z (z 3 − z 2 ) = z 2 (z − z 2 ) − z 3 (z − z 3 ) …(iii) + {z1z 2 z 3 − z 2 z1z1 + z 3z1z1 + z 2 z1z 3
Eliminating z from Eqs. (ii) and (iii), we get − z1z 3z 3 + z 2z 3z 3 − z1z 2z 3 } − z ∑ (z1z 2 − z 2 z1 )
(z 2 − z1 ) [ z 2 (z − z 2 ) − z 3 (z − z 3 )] = (z 3 − z 2 ) = − ∑ z12 ( z 2 − z 3 ) − Σ z1z1 (z 2 − z 3 )
[ z1 (z − z1 ) − z 2 (z − z 2 )] 2
∑ z12 (z 2 − z 3 ) + ∑ z1 (z 2 − z 3 )
Hence, z =
or z [z 2 (z 2 − z1 ) − z 3 (z 2 − z1 ) − z1 (z 3 − z 2 ) + z 2 (z 3 − z 2 )] ∑ (z1z 2 − z 2 z1 )
= z 2 z 2 (z 2 − z1 ) − z 3z 3 (z 2 − z1 ) − z1z1(z 3 − z 2 ) + z 2 z 2 (z 3 − z 2 ) 1
99. Let θ = (2n + 1) π , where n = 0, 1, 2, 3, ... , 6
or z ∑ z1 (z 2 − z 3 ) = ∑ z1 z1 (z 2 − z 3 ) 7
∴ 7θ = (2n + 1 ) π or 4 θ = (2n + 1 ) π − 3 θ
∑ | z1 | 2 (z 2 − z 3 ) or cos 4 θ = − cos3 θ
or z=
∑ z1 (z 2 − z 3 ) or 2 cos2 2 θ − 1 = − ( 4 cos3 θ − 3 cos θ )
Chap 01 Complex Numbers 95

or 2 (2 cos2 θ − 1 ) 2 − 1 = − ( 4 cos3 θ − 3 cos θ ) (z 7 + 1 )


⇒ = (z − α ) (z − α ) (z − α 3 ) (z − α 3 ) (z − α 5 ) (z − α 5 )
or 8 cos4 θ + 4 cos3 θ − 8 cos2 θ − 3 cos θ + 1 = 0 (z + 1 )
Now, if cos θ = x, then we have ⇒ z6 − z5 + z4 − z3 + z2 − z + 1
8x 4 + 4x 3 − 8x 2 − 3x + 1 = 0  π  3π 
= z 2 + 1 − 2z cos  z 2 + 1 − 2z cos 
or ( x + 1 ) (8 x − 4 x − 4 x + 1 ) = 0
3 2  7   7
 2 5π 
x+1≠0 [Q θ ≠ π] z + 1 − 2z cos  ...(A)
 7
∴ 8x 3 − 4x 2 − 4x + 1 = 0 ...(i) Dividing by z 3 on both sides, we get
Hence, the roots of this equation are  3 1  2 1  1
z + 3  − z + 2  + z +  − 1
π 3π 5π  z   z   z
cos , cos , cos .
7 7 7  1 π  1 3π   1 5π 
9π 5π 11 π = z + − 2 cos  z + − 2 cos  z + − 2 cos 
[ since cos = cos , cos  z 7  z 7 z 7
7 7 7 1
3π 13π π On putting z + = 2 x, we get
= cos , cos = cos and Eq. (i) is cubic] z
7 7 7
1 1 (8 x 3 − 6 x ) − ( 4 x 2 − 2 ) + 2 x − 1
(i) On putting 2 = y or x = in Eq. (i), then Eq. (i) becomes
x y  π  3π   5π 
= 8  x − cos   x − cos   x − cos 

8
− −
4 4
+1=0  7  7  7
y y y y  π
2 or 8 x 3 − 4 x 2 − 4 x + 1 = 8  x − cos 
 4
2
 4  2   7
⇒ 1 −  =  1 −   3π   5π 
 y 
 y  y  x − cos   x − cos  …(i)
 7  7
16 8 16  4 4
or 1+ 2 − = 1 + 2 − 
y y y  y y So, 8 x 3 − 4 x 2 − 4 x + 1 = 0 and this equation has roots
π 3π 5π
or y 3 − 24y 2 + 80y − 64 = 0 ...(ii) cos , cos , cos
7 7 7
1 1
where y = 2 = = sec 2 θ π 3π 5π Constant term
x cos2 θ ∴ cos cos cos =−
7 7 7 Coefficient of x 3
Thus, the roots of x 3 − 24 x 2 + 80 x − 61 = 0
π 3π 5π π 3π 5π 1
are sec 2 , sec 2 , sec 2 cos cos cos =− [ proved (i) part]
7 7 7 7 7 7 8
(ii) Again, putting y = 1 + z i. e. z = y − 1 On putting x = 1 in Eq. (i), we get
 π  3π   5π 
= sec 2 θ − 1 = tan 2 θ, Eq. (ii) reduces to 1 = 8 1 − cos  1 − cos  1 − cos 
 7  7  7
(1 + z ) 3 − 24 (1 + z ) 2 + 80 (1 + z ) − 64 = 0
 π 3π 5π 
z 3 − 21z 2 + 35z − 7 = 0 or 1 = 8 8 sin 2 sin 2 sin 2 
or …(iii)  14 14 14 
π 3π 5π Since, sinθ > 0 for 0 < θ < π / 2 , we get
Hence, tan 2 , tan 2 , tan 2 are the roots of
7 7 7 π 3π 5π 1
x 3 − 21 x 2 + 35 x − 7 = 0 ∴ sin sin sin = ...(ii) [ proved (iii) part]
14 14 14 8
1
(iii) Putting x = in Eq. (i), then Eq. (i) reduces to Again, putting x = − 1 in Eq. (i), we get
u
u 3 − 4u 2 − 4u + 8 = 0 whose roots are  π  3π   5π 
− 7 = − 8 1 + cos  1 + cos  1 + cos 
π 3π 5π  7  7 7
sec , sec , sec .
7 7 7  π 3π 5π 
7 = 8 8 cos2 cos2 cos2 
Therefore, sum of the roots is  14 14 14 
π 3π 5π
sec + sec + sec =4 Since, cosθ > 0 for 0 < θ < π / 2 , we get
7 7 7
100. Let roots of z 7 + 1 = 0 are − 1, α , α 3, α 5, α , α 3, α 5, π 3π 5π 7
cos cos cos = ...(iii) [ proved (ii) part]
π π 14 14 14 8
where α = cos + i sin
7 7 On dividing Eq. (ii) by Eq. (iii), we get
∴ (z 7 + 1 ) = (z + 1 ) (z − α ) (z − α ) (z − α 3 ) π 3π 5π 1
tan tan tan = [ proved (iv) part]
(z − α ) (z − α ) (z − α )
3 5 5 14 14 14 7
96 Textbook of Algebra

(1 + y ) Again, circles (i) and (iii) should not cut or touch, then distance
On putting z = in Eq. (A), we get
(1 − y ) between their centres > sum of their radii
π 3π 5π
2 6 cos2 cos2 cos2 ( − 2a − 0 ) 2 + (a + 1 − 0 ) 2 > 3 + 3
(1 + y ) 7 + (1 − y ) 7 14 14 14
=
2 (1 − y ) 6 (1 − y ) 6 or 5a 2 + 2a + 1 > 6
 2 2 π 2 2 3π   2 2 5π  ⇒ 5a 2 + 2a + 1 > 36
y + tan  y + tan  y + tan 
 14  14   14  or 5a 2 + 2a − 35 > 0
7  2 2 π 2a
∴ (1 + y ) 7 + (1 − y ) 7 = 2 7 ⋅ y + tan  ⇒ a2 + −7 > 0
64  14 5
 2 2 3π   2 2 5π   − 1 − 4 11   − 1 + 4 11 
y + tan  y + tan  or a −  a −  >0
 14   14 
Using result (ii), we get  5   5 
 π
(1 + y ) 7 + (1 − y ) 7 = 14 y 2 + tan 2   − 1 − 4 11   − 1 + 4 11 
 14 ∴ a ∈  − ∞,  ∪ , ∞ ...(v)
 2 2 3π   2 2 5π 
 5   5 
y + tan  y + tan 
 14   14  Hence, the common values of a satisfying Eqs. (iv) and (v) are
4
Equating the coefficient of y on both sides, we get  1 − 71 − 1 − 4 11   − 1 + 4 11 1 + 71 
a ∈ ,  ∪ , 
 π 3π 5π   2 5   5 2 
7
C 4 + 7C 4 = 14 tan 2 + tan 2 + tan 2 
 14 14 14  102. (i) From De-moivre’s theorem, we know that
π 3π 5π sin (2n + 1 ) α = 2n + 1
C1 (1 − sin 2 α )n
Therefore, tan 2 + tan 2 + tan 2 =5
14 14 14
sin α − 2n + 1C 3 (1 − sin 2 α )n − 1 sin 3 α
101. Equation z = 3 represents boundary of a circle and equation
+ ... + ( − 1 )n sin 2n + 1 α
z − {a (1 + i ) − i } ≤ 3 represents the interior and the
It follows that the numbers
boundary of a circle and equation z + 2a − (a + 1 ) i > 3
represents the exterior of a circle. Then, any point which π 2π nπ
sin , sin , ..., sin
satisfies all the three conditions will lie on first circle, on or 2n + 1 2n + 1 2n + 1
inside the second circle and outside the third circle. are the roots of the equation.
2n + 1
C1(1 − x 2 )n x − 2n + 1 C 3(1 − x 2 )n − 1 x 3 + ... + ( − 1 )n x 2n + 1
= 0 of the (2n + 1 ) th degree
C B
II Consequently, the numbers
A π 2π nπ
sin 2 , sin 2 , ... , sin 2 are the roots of the
III 2n + 1 2n + 1 2n + 1
I equation
2n + 1
For the existence of such a point first two circles must cut or C1 (1 − x )n − 2n + 1C 3 (1 − x )n − 1 x + ... + ( − 1 )n xn = 0 of
atleast touch each other and first and third circles must not the nth degree
intersect each other. The arcABC of first circle lying inside the (ii) From De-moivre’s theorem, we know that
second but outside the third circle, represents all such possible sin(2n + 1 ) α = 2n + 1C1(cosα ) 2n sin α
points.
2n + 1
Let z = x + iy , then equation of circles are − C 3(cosα ) 2n − 2 sin 3 α + ... + ( −1 )n sin 2n + 1 α
x2 + y 2 = 9 ...(i) or sin (2n + 1 ) α = sin 2n + 1 α
( x − a ) + (y − a + 1 ) = 9
2 2
...(ii) { 2n + 1C1 cot 2n α − 2n + 1 C 3 cot 2n − 2 α + 2n +1C 5 cot 2n − 4 α − ...}
and ( x + 2a ) + (y − a − 1 ) = 9
2 2
...(iii) π 2π 3π nπ
It follows that α = , , , ... ,
Circles (i) and (ii) should cut or touch, then distance between 2n + 1 2n + 1 2n + 1 2n + 1
their centres ≤ sum of their radii
Therefore, equality holds
⇒ (a − 0 ) 2 + (a − 1 − 0 ) 2 ≤ 3 + 3 2n +1
C1 cot 2n α − 2n +1C 3 cot 2n − 2 α + 2n +1C 5 cot 2n − 4 α−... = 0
⇒ a + (a − 1 ) ≤ 36
2 2
It follows that the numbers
⇒ 2a 2 − 2a − 35 ≤ 0 π 2π nπ
cot 2 , cot 2 , ... , cot 2 are the roots of the
2n + 1 2n + 1 2n + 1
35  1 + 71   1 − 71 
⇒ a2 − a − ≤ 0 or a −  a −  ≤0 equation
2  2   2 
2n + 1 2n + 1
C1xn − C 3x n − 1 + 2n + 1
C 5xn − 2 − ... = 0
1 − 71 1 + 71
∴ ≤a≤ ...(iv)
2 2 of the nth degree.
Chap 01 Complex Numbers 97

103. Let y = | a + bω + cω 2 |. For y to be minimum, y 2 must be


z i
minimum. 108. = 1 ⇒ |z | = z −
i 3
∴ y 2 = | a + bω + cω 2 | 2 = (a + bω + cω 2 ) (a + bω + cω 2 ) z−
3
= (a + bω + cω 2 ) (a + b ω + c ω 2 )
Clearly, locus of z is perpendicular bisector of line joining
y = (a + bω + cω 2 ) (a + bω 2 + cω )3
2
i
points having complex number 0 + i 0 and 0 + .
= (a 2 + b 2 + c 2 − ab − bc − ca ) 3
Hence, z lies on a straight line.
1
= [(a − b ) 2 + (b − c ) 2 + (c − a ) 2 ]  ω − ωz 
2 109. Given,   is purely real ⇒ z ≠ 1
 1 −z 
Since, a, b and c are not equal at a time, so minimum value of
y 2 occurs when any two are same and third is differ by 1.  ω − ω z   ω − ω z  ω − ωz
∴   = =
⇒ Minimum of y = 1 (as a, b, c are integers)  1 −z   1 −z  1 −z
π ⇒ ( ω − ω z ) ( 1 − z ) = (1 − z ) ( ω − ω z )
104. Equation of ray PQ is arg (z + 1) =
4 ⇒ ( zz − 1 ) ( ω − ω) = 0
π ⇒ (| z | 2 − 1 ) (2iβ ) = 0 [Q ω = α + iβ ]
Equation of ray PR is arg (z + 1 ) = −
4 ∴ |z |2 − 1 = 0
π π π
Shaded region is − < arg (z + 1 ) < ⇒ | arg (z + 1 )| < ⇒ | z | = 1 and z ≠ 1 [Q β ≠ 0 ]
4 4 4 10
2kπ   2kπ 
∴ | PQ | = ( 2 ) 2 + ( 2 ) 2 = 2 110. ∑ sin   + i cos  
k =1
 11   11 
So, arc QAR is of a circle of radius 2 units with centre at −2k π i
 2kπ   2kπ  
10 10
P ( − 1, 0 ). All the points in the shaded region are exterior to  11
= i ∑ cos   − i sin   =i ∑ e
this circle | z + 1 | = 2. k =1 
 11   11   k =1
π
i.e. | z + 1 | > 2 and | arg (z + 1 )| < .  10 −2k π i 
4  11 
= i  ∑e − 1 = i ( 0 − 1 ) [Qsum of 11, 11th roots of unity = 0]
zB − 1 k = 0 
105. In ∆AOB from Coni method, = e iπ / 2 = i  
zA − 1
= −i
B(zB) A(2 + √3i ) = zA 111. Q z 2+ z + 1 = 0
π/2 √2 ∴ z = ω, ω 2
√2
(1) 1 1
√2 √2 Q z+ = ω + = ω + ω2 = − 1
z ω
D (zD) 1 1
(zC)C ⇒ z 2 + 2 = ω2 + 2 = ω2 + ω = − 1
z ω
1 1
z B − 1 = (z A − 1 ) i ∴ z 3 + 3 = ω3 + 3 = 1 + 1 = 2
∴ z B = 1 + (2 + 3i − 1 ) i = 1 + (1 + i 3 ) i z ω
=1 + i − 3 =1 − 3 + i 1 1 1
z 4 + 4 = ω4 + 4 = ω + = − 1
z ω ω
zC = 2 − z A = 2 − (2 + 3i ) = − 3 i
1 1
and z D = 2 − z B = 2 − (1 − 3 + i ) = 1 + 3 − i z 5 + 5 = ω5 + 5 = ω2 + ω = − 1
z ω
Hence, other vertices are (1 − 3 ) + i , − 3i , (1 + 3 ) − i. 1 1
and z 6 + 6 = ω6 + 6 = 2
106. Let z1 = r1 (cos θ1 + i sin θ1 ) and z 2 = r2 (cos θ 2 + i sin θ 2 ) z ω
∴ Required sum = ( −1 ) 2+ ( −1 ) 2 + (2 ) 2 + ( −1 ) 2 + ( −1 ) 2 + (2 ) 2 = 12
∴| z1 + z 2 | = [(r1 cos θ1 + r2 cos θ 2 ) 2 + (r1 sin θ1 + r2 sin θ 2 ) 2 ]1/2
112. Let OA = 3, so that the complex number associated with A is
= [| r12 + r22 + 2r1r2 cos ( θ1 − θ 2 )]1/2 = [(r1 + r2 ) 2 ]1/2
3e iπ /4 . If z is the complex number associated with P, then
∴ | z1 + z 2 | = | z1 | + | z 2 |
N (North)
Therefore, cos ( θ1 − θ 2 ) = 1
z
⇒ θ1 − θ 2 = 0
⇒ θ1 = θ 2 4

Thus, arg ( z1 ) – arg ( z 2 ) = 0 π/2 A 3ei π/4


107. ( x − 1) = − 8 ⇒ x − 1 = ( − 8)
3 1/ 3
3
⇒ x − 1 = − 2, − 2ω, − 2ω 2 π/4
W E
O (East)
⇒ x = − 1, 1 − 2ω, 1 − 2ω 2
S
98 Textbook of Algebra

z − 3e iπ /4 4 − π /2 4i 120. Q z z (z 2 + z 2 ) = 350
= e =−
0 − 3e iπ /4 3 3 Put z = x + iy
iπ / 4 iπ / 4 iπ / 4 ⇒ ( x 2 + y 2 ) ⋅ 2( x 2 − y 2 ) = 350
⇒ 3z − 9e = 12 ie ⇒ z = (3 + 4i ) e
113. Let z = cos θ + i sin θ ⇒ ( x 2 + y 2 ) ( x 2 − y 2 ) = 175 = 25 × 7
z cos θ + i sin θ ⇒ x 2 + y 2 = 25, x 2 − y 2 = 7
⇒ =
1 − z 2 1 − (cos 2 θ + i sin 2 θ ) ⇒ x 2 = 16 , y 2 = 9
cos θ + i sin θ ∴ x = ± 4, y = ± 3; x , y ∈ I
=
2 sin 2 θ − 2 i sin θ cos θ Area of rectangle = 8 × 6 = 48 sq units
15 15 15
cos θ + i sin θ i
= =
− 2 i sin θ (cos θ + i sin θ ) 2 sin θ
121. ∑ Im (z 2m − 1 ) = ∑ Im [e ( 2m − 1) iθ ] = ∑ sin (2m − 1) θ
m =1 m =1 m =1
z 15
2 sin (2m − 1 ) θ sin θ
lies on the imaginary axis i.e. x = 0 or on Y-axis.
Hence,
1 − z2 = ∑ 2 sin θ
m =1
Aliter
z z 1 1 1
15
cos (2m − 2 ) θ − cos 2mθ
Let E =
1 −z 2
=
zz − z 2
=
z − z
=−
z − z
=−
 z − z
= ∑ 2 sin θ
  2i m =1
 2i 
i cos 0 ° − cos 30 θ 1 − cos 60 °
= which is imaginary. = = (Qθ = 2 ° )
2 Im | z | 2 sin θ 2 sin 2 °
114. | z + 4 | ≤ 3 1
1−
2 = 1
⇒z lies inside or on the circle of radius 3 and centre at ( − 4, 0 ). =
2 sin 2 ° 4 sin 2 °
∴ Maximum value of | z + 1 | is 6.
Y 4 4 4
122. z − ≥ |z | − ⇒ 2 ≥ |z | −
z |z | |z |
4
⇒ − 2 ≤ |z | − ≤ 2 ⇒ − 2 |z | ≤ |z |2 − 4 ≤ 2 |z |
X′ O
X |z |
(–4, 0) A (0, 0)
(–1,0) ⇒ |z |2 + 2 |z | − 4 ≥ 0
and 12 − 2 |z | − 4 ≤ 0
Y′
⇒ (| z | + 1 ) 2 ≥ 5 and (| z | − 1 ) 2 ≤ 5
115. Let A = set of points on and above the line y = 1 in the argand − 5 ≤ | z | − 1 ≤ 5 and | z | + 1 ≥ 5
plane.
⇒ 5 − 1 ≤ |z | ≤ 5 + 1
B = set of points on the circle ( x − 2 ) 2 + (y − 1 ) 2 = 3 2
123. As z = (1 − t ) z1 + tz 2
C = Re (1 − i ) z = Re [(1 − i ) ( x + iy )] = x + y
z
⇒ x+y = 2
z1 t 1–t z2
Hence, ( A ∩ B ∩ C ) has only one point of intersection.
⇒ z1, z and z 2 are collinear.
116. The points ( − 1 + i ) and (5 + i ) are the extremities of diameter
Thus, options (a) and (d) are correct.
of the given circle.
z − z1 z − z1
Hence, | z + 1 − i | 2 + | z − 5 − i | 2 = 36 Also, =
z 2 − z1 z 2 − z1
117. Q | z − w | ≤ | z | − | w | Hence, option (c) is correct.
and | z − w | = distance between z and w 2π 2π 1 3
124. ω = cos + i sin =− +i
z is fixed, hence distance between z and w would be maximum 3 3 2 2
for diametrically opposite points. ω is one of the cube root of unity.
⇒ | z − w | < 6 ⇒ || z | − | w || < 6
z +1 ω ω2
⇒ − 6 < |z | − |w | < 6 ⇒ − 3 < |z | − |w | + 3 < 9
∴ ω z + ω2 1 =0
118. Q z 0 = 1 + 2i
ω 2
1 z +ω
∴ z1 = 6 + 5i ⇒ z 2 = − 6 + 7i
Applying R 1 → R 1 + R 2 + R 3, we get
119. Put ( − i ) in place of i.
z z z
−1
Hence, ω z + ω2 1 =0 [Q 1 + ω + ω 2 = 0 ]
i +1
ω2 1 z +ω
Chap 01 Complex Numbers 99

Now, applying C 2 → C 2 − C1, C 3 → C 3 − C1, we get (D) Let w = cos θ + i sin θ, then
z 0 0 1
z = x + iy = w +
ω z + ω2 − ω 1 −ω =0 w
⇒ x + iy = 2 cos θ
ω2 1 −w2 z + ω − ω2
∴ x = 2 cos θ and y = 0
⇒ z [(z + ω 2 − ω ) (z + ω − ω 2 ) − (1 − ω ) (1 − ω 2 )] = 0
126. Q x2 − x + 1 = 0
⇒ z [z 2 − (ω 2 − ω ) 2 − (1 − ω 2 − ω + ω 3 )] = 0
⇒ z [z − (ω 4 + ω 2 − 2ω 3 ) − 1 + ω 2 + ω − ω 3 ] = 0
2 1 ± (1 − 4 ) 1 ± i 3
∴ x= =
⇒ z3 = 0 2 2
1+i 3 1 −i 3
∴ z =0 = and
2 2
125. | z − i | z || = | z + i | z ||
∴ x = − ω 2, − ω
(A) Putting z = x + iy , we get y x 2 + y 2 = 0
∴ α = − ω 2, β = − ω
i. e. Im(z ) = 0
4 ⇒ α 2009 + β 2009 = − ω 4018 − ω 2009
(B) 2ae = 8, 2a = 10 ⇒ 10e = 8 ⇒e =
5 = − ω − ω 2 = − (ω + ω 2 )
(0, 3) = − (− 1) = 1
127. | z − 1 | = | z + 1 | = | z − i |
(0,0)
(–5, 0) (5, 0) ⇒ |z − 1|2 = |z + 1|2 = |z − i |2
⇒ (z − 1 ) ( z − 1 ) = (z + 1 ) ( z + 1 ) = (z − i ) ( z + i )
(0, – 3) ⇒ z z − z − z + 1 = z z + z + z + 1 = z z + iz − i z + 1
⇒ − z − z = z + z = i (z − z )
 16
∴ b 2 = 25 1 −  = 9 From first two relations,
 25
2 (z + z ) = 0 ⇒ Re(z ) = 0 …(i)
x2 y 2
⇒ + =1 From last two relations,
25 9 z + z = i (z − z ) ⇒ 2 Re (z ) = − 2 Im (z )
1
(C) z = 2 (cos θ + i sin θ ) − From Eq. (i), Im(z ) = 0
2 (cos θ + i sin θ ) ∴ z = Re(z ) + i Im(z ) = 0 + i ⋅ 0 = 0
1
= 2 (cos θ + i sin θ ) − (cos θ − i sin θ ) Hence, number of solutions is one.
2
128. We have, | z − 3 − 2i | ≤ 2
3 5
z = cos θ + i sin θ ⇒ | 2z − 6 − 4i | ≤ 4 …(i)
2 2
Now, | 2z − 6 − 4i | = |(2z − 6 + 5i ) − 9i |
≥ || 2z − 6 + 5i | − 9 | …(ii)
(0, 0)
From Eqs. (i) and (ii), we get
3 3, 0
– ,0
2 2 | 2z − 6 + 5i | − 9 | ≤ 4
⇒ − 4 ≤ | 2z − 6 + 5i | − 9 ≤ 4
⇒ 5 ≤ | 2z − 6 + 5i | ≤ 13
Hence, the minimum value of | 2z − 6 + 5i | is 5.
Let z = x + iy , then 129. Q | z | = 1 ∴ z = e iθ
3 5
x = cos θ and y = sin θ  2iz   2ie iθ   2i 
2 2 ∴ Re  2
= Re  2 iθ
 = Re  − iθ iθ 
2 2 1 − z   1 − e   e − e 
 2x   2y 
⇒   +   =1  2i   1 
3 5 = Re   = Re  − 
 − 2i sin θ   sin θ 
4 x 2 4y 2
⇒ + =1 1
9 25 =− = − cosec θ
sin θ
x2 y2
⇒ + =1 Q cosec θ ≤ − 1 ⇒ cosec θ ≥ 1
9 / 4 25 / 4
⇒ − cosec θ ≥ 1 ⇒ − cosec θ ≤ − 1

9 25
= (1 − e 2 ) ⇒ − cosec θ ∈ ( − ∞, − 1 ] ∩ [1, ∞ )
4 4  2iz 
9 16 4 ∴ Re  2
∈ ( − ∞, −1 ] ∩ [1, ∞ )
∴ e2 = 1 − = ⇒ e= 1 − z 
25 25 5
100 Textbook of Algebra

130. Q | z | = 1. Let z = e i θ 134. Given, z 2 + z + 1 = a ⇒ z 2 + z + 1 − a = 0



∴ z −1 =e −1=e iθ / 2
⋅ 2i sin (θ / 2 ) − 1 ± ( 4a − 3 )
∴ z=
1 1 ie − iθ /2 2
⇒ = iθ / 2
=−
z − 1 2ie ⋅ sin (θ / 2 ) 2 sin (θ / 2 ) Hence, a≠
3
[for a = 3 / 4, z will be purely real]
4
1 i ⋅ e − i θ /2  1   π θ
⇒ = ∴ arg   = −  135. Let z = x + iy , then
1 − z 2 sin (θ / 2 )  1 − z   2 2
 1  π θ z2 ( x + iy ) 2 ( x 2 − y 2 + 2ix )
⇒ = =
arg   = − z − 1 ( x + iy − 1 ) ( x − 1 + iy )
1 − z 2 2
 1  π ( x 2 − y 2 + 2ixy ) ( x − 1 − iy )
∴ Maximum value of arg   = =
1 − z 2 ( x − 1 + iy ) ( x − 1 − iy )

131. Q | x | 2 = x x = (a + b + c ) (a + b + c ) ( x − 1 ) ( x 2 − y 2 ) + 2 xy 2 + i [ 2 xy ( x − 1 ) − y ( x 2 − y 2 )]
=
= (a + b + c ) ( a + b + c ) (x − 1)2 + y 2
= | a | 2 + | b | 2 + | c | 2 + ab + a b + bc + b c + ca + ca …(i)  z2 
Now, Im   =0
| y | 2 = y y = (a + bω + cω 2 ) (a + b ω + c ω 2 )  z − 1
= (a + bω + cω 2 ) ( a + b ω + c ω 2 ) ⇒ 2 xy ( x − 1 ) − y ( x 2 − y 2 ) = 0
⇒ y (2 x 2 − 2 x − x 2 + y 2 ) = 0
= (a + bω + cω ) ( a + b ω + c ω )
2 2
⇒ y (x 2 + y 2 − 2x ) = 0
= | a | 2 + | b | 2 + | c | 2 + ab ω 2 + a bω
+ bc ω 2+ b cω + ca ω 2+ caω …(ii) ⇒ y = 0 or x 2 + y 2 − 2 x = 0
Hence, z lies on the real axis or on a circle passing through the
and| z | 2 = zz = (a + bω 2 + cω ) (a + bω 2 + cω ) origin.
= (a + bω 2+ cω ) (a + b ω 2 + c ω )
136. Given, | z | = 1 and arg (z ) = θ …(i)
= (a + bω 2 + cω ) (a + b ω + c ω 2 ) ⇒ |z | = 1 ⇒ z z = 1
2

= | a | 2 + | b | 2 + | c | 2 + ab ω + a bω 2 1
⇒ z = …(ii)
z
+ bcw + b cω 2 + ca ω + caω 2 …(iii) 1 + z  1+z 
∴ arg   = arg   [from Eq. (ii)]
On adding Eqs. (i), (ii) and (iii), we get 1 + z   1 + 1 /z 
| x | 2 + | y | 2 + | z | 2 = 3 (| a | 2 + | b | 2 + | c | 2 ) = arg (z ) = θ [from Eq. (i)]
+ 0 + 0 + 0 + 0 + 0 + 0 (Q 1 + ω + ω 2 = 0 ) Aliter I
Given, | z | = 1 and arg (z ) = θ
| x |2 + |y |2 + |z |2
∴ =3 ⇒ z = eiθ
|a |2 + |b |2 + |c |2  1 + eiθ 
1 + z
z +z ∴ arg   = arg  − iθ
 = arg (e i θ ) = arg (z ) = θ
132. Q Re(z ) = 1 ∴ =1 ⇒ z + z =2 1 + z  1 + e 
2
Aliter II Given, | z | = 1 and arg (z ) = θ
Since, α , β ∈ R
Let z = ω (cube root of unity)
∴ The complex roots are conjugate to each other, if z1, z 2 are
two distinct roots, then z1 = z 2 or z1 = z 2 1 + z  1 + ω 1+ω
∴ arg   = arg   = arg  2
(Q ω = ω 2 )
∴ Product of the roots = z1z 2 = β 1 + z  1 + ω 1 + ω 
⇒ z1z1 = β  − ω2 
= arg   (Q 1 + ω + ω 2 = 0 )
∴ β = | z1 | = [ Re(z1 )] + Im | z1 |
2 2 2  −ω 
= arg (ω ) = arg (z ) = θ
= 1 + Im | z1 | 2 > 1
1
[Q roots are distinct∴ Im (z1 ) ≠ 0] 137. z 0 = 2α −
α
∴ β > 1 or β ∈ (1, ∞ )
Q 2 |z 0 |2 = r 2 + 2
133. Q (1 + ω ) = ( − ω ) = − ω
7 2 7 14
= −ω =1 + ω
2
2 2 2
1 1 1
Given, (1 + ω ) = A + Bω ⇒ 1 + ω = A + Bω
7 ∴ 2 2α − = r 2 + 2 ⇒ 2 2α − = α− +2
α α α
On comparing, we get A = 1, B = 1 1 1 1
⇒ 7 |α |2 + − 8 = 0 ⇒ | α | 2 = 1 or ⇒ | α | = 1 or
∴ ( A, B ) = (1, 1 ) |α | 2
7 7
Chap 01 Complex Numbers 101

138. (B) z1 ⋅ z = zk is clearly incorrect.


z 10 − 1
ei5π/6 e iπ/6 lim
z →1 z − 1
(C) Expression = =1
10
–1 1
9
 2kπ 
(D) 1 + Σzk = 0 ⇒ 1 + ∑ cos   =0
e–i 5π/6 e– i π/6 k =1  10 
∴ Expression = 2
1/2 1/2
z1 − 2z 2
143. Q =1
3+i 2 − z1z 2
ω= = e iπ /6, P = e inπ /6
2
As z1 ∈ P ∩ H1 ⇒ z1 = 1, e iπ /6, e − iπ /6 ⇒ | z1 − 2z 2 | 2 = | 2 − z1z 2 | 2

As z 2 ∈ P ∩ H 2 ⇒ z 2 = − 1, ei 5π /6, e − i 5π /6 ⇒ (z1 − 2z 2 ) (z1 − 2z 2 ) = (2 − z1z 2 ) (2 − z1z 2 )


∠z1Oz 2 = 2 π / 3, where z1 = e iπ /6, z 2 = e i 5π /6 ⇒ (z1 − 2z 2 ) (z1 − 2z 2 ) = (2 − z1z 2 ) (2 − z1z 2 )
Sol. (Q. Nos. 139-140) ⇒ z1z1 − 2z1z 2 − 2z1z 2 + 4z 2z 2 = 4 − 2z1z 2 − 2z1z 2 + z1z1z 2z 2
Let z = x + iy , S1 : x + y < 16
2 2
⇒ | z 1| 2 + 4 | z 2 | 2 + 4 + | z 1 | 2 | z 2 | 2
( x − 1 ) + i (y + 3 )  ⇒ ( | z1 | 2 − 4 ) (1 − | z 2 | 2 ) = 0
Now, Im  >0
 1 − 3i  Q | z2 | ≠ 1
⇒ S 2 : 3x + y > 0 ⇒ S 3 : x > 0 ∴ | z1 | 2 = 4 or | z1 | = 2
⇒ Point z1 lies on circle of radius 2.
√3x + y = 0 (0, 4)
144. Let a = 3, b = −3, c = 2, then
S (a + bω + cω 2 ) 4n + 3 + (c + aω + bω 2 ) 4n + 3 + (b + cω + aω 2 ) 4n + 3
=0
0 60° (4, 0) ⇒(a + bω + cω 2 ) 4n + 3
P
(1, –3)   c + aω + bω 2 ) 4n + 3  4n + 3  b + cω + aω 2 ) 4n + 3 
1 +   +  =0
  a + bω + cω  a + bω + cω 
2 2
 
139. min | 1 − 3i − z | = min | z − 1 + 3i | ⇒ (a + bω + cω 2 ) 4n + 3(1 + ω 4n + 3 + (ω 2 ) 4n + 3 ) = 0
= perpendicular distance of the point (1, − 3 ) from the straight ⇒ 4n + 3 should be an integer other than multiple of 3.
3 −3 3 − 3
line 3 x + y = 0 = = ∴ n = 1, 2, 4, 5
2 2
kπ kπ
145. Q α k = cos  + i sin   = e iπk /7
20 π
140. Area of S =   π × 4 2 +   π × 4 2 =  7  7
1 1
 4  6 3 ∴ α k + 1 − α k = e iπ (k + 1) /7− e iπk /7 = e iπk /7(e iπ /7− 1 )
141. Since, | z | ≥ 2 is the region lying on or outside circle centered at
 π
( 0, 0 ) and radius 2. Therefore, | z + (1 / 2 )| is the distance of z = e iπk /7⋅ e iπ /14⋅ 2 i sin  
 14
from ( − 1 / 2, 0 ), which lies inside the circle.
Hence, minimum value of | z + (1 / 2 )|  π
⇒ | α k + 1 − α k | = 2 sin  
 14
= distance of ( − 1 / 2, 0 ) from ( − 2, 0 )
 π  π
12

 1 
2
∑ |αk + 1 − αk | = 12 × 2 sin  14 = 24 sin  14
=  − + 2 + | 0 − 0 | 2 = 3 / 2 k =1
 2 
Aliter and α 4k − 1 − α 4k − 2 = e iπ ( 4k − 1) /7− e iπ ( 4k − 2 ) /7= e iπ ( 4k − 2) /7(e iπ /7− 1 )
1 1
Q | z + (1 / 2 )| ≥ | z | − ≥ 2− [Q | z | ≥ 2 ]  π
2 2 = e iπ ( 4k − 2) /7⋅e iπ /14 ⋅ 2i sin  
 14
∴ | z + (1 / 2 )| ≥ 3 / 2  π
⇒ | α 4k − 1 − α 4k − 2| = 2 sin  
142. Clearly, zk10 = 1, ∀ k, where zk ≠ 1  14

 π  π
3
(A) zk ⋅ z j = e i ( 2 π /10) (k + j)
= 1, if (k + j ) is multiple of 10 ∴ ∑ | α 4k −1 − α 4k − 2| = 3 × 2 sin  14 = 6 sin  14
i.e. possible for each k. k =1
102 Textbook of Algebra

12 x
x2 + y 2 − =0
∑ |αk + 1 − αk | or
a
k =1
Hence, =4 1 
3 ∴ Locus of z is a circle with centre  , 0
 2a 
∑ | α 4k − 1 − α 4k − 2| 1
k =1
and radius = , a > 0.
2 + 3i sin θ 2a
146. Let z =
1 − 2i sin θ Also for b = 0, a ≠ 0, we get y = 0.
Q z is purely imaginary ∴ locus is X -axis and for a = 0, b ≠ 0, we get x = 0
∴ locus is Y -axis.
∴ z = −z
 2 + 3i sin θ   2 + 3i sin θ  1 1 1 1 1 1
⇒   =− 
 1 − 2i sin θ   1 − 2i sin θ  148. Let ∆ = 1 −ω 2 − 1 ω 2 = 1 ω ω2
1 ω2 ω7 1 ω2 ω
 2 − 3i sin θ   2 + 3i sin θ 
⇒   =− 
 1 + 2i sin θ   1 − 2i sin θ  (Q1 + ω + ω = 0 and ω = 1)
2 3

⇒ (2 − 3i sin θ ) (1 − 2i sin θ ) + (1 + 2i sin θ ) (2 + 3i sin θ ) = 0 Applying C1 → C1 + C 2 + C 3, then we get


1
⇒ 4 − 12 sin 2 θ = 0 or sin 2 θ = 3 L 1 L 1
3
M
 1 
∴ θ = sin −1   ∆= 0 ω ω2 (Q 1 + ω + ω 2 = 0)
 3
M
1
147. Q x + iy = 0 ω2 ω
a + ibt
a − ibt = 3 (ω 2 − ω 4 )
⇒ x + iy = 2
a + b 2t 2 = 3 (− 1 − ω − ω ) (Q ω 3 = 1 and 1 + ω + ω 2 = 0)
a bt
⇒ x= 2 , y =− 2 = − 3 (1 + 2ω )
(a + b 2t 2 ) (a + b 2t 2 )
1 x = − 3z = 3k (given) (Q 1 + 2ω = z)
or x2 + y 2 = 2 = ∴ k = −z
a + b 2t 2 a
CHAPTER

02
Theory of Equations
Learning Part
Session 1
● Polynomial in One Variable ● Identity

● Linear Equation ● Quadratic Equations

● Standard Quadratic Equation

Session 2
● Transformation of Quadratic Equations ● Condition for Common Roots

Session 3
● Quadratic Expression

● Wavy Curve Method

● Condition for Resolution into Linear Factors

● Location of Roots (Interval in which Roots Lie)

Session 4
● Equations of Higher Degree

● Rational Algebraic Inequalities

● Roots of Equation with the Help of Graphs

Session 5
● Irrational Equations

● Irrational Inequations

● Exponential Equations

● Exponential Inequations

● Logarithmic Equations

● Logarithmic Inequations

Practice Part
● JEE Type Examples
● Chapter Exercises

Arihant on Your Mobile !


Exercises with the #L
symbol can be practised on your mobile. See inside cover page to activate for free.
Session 1
Polynomial in One Variable, Identity, Linear Equation,
Quadratic Equations, Standard Quadratic Equation
Polynomial in One Variable Thus, every polynomial is a rational expression but a
rational expression may or may not be a polynomial.
An algebraic expression containing many terms of the
For example,
form cx n , n being a non-negative integer is called a
2
polynomial, (i) x 2 - 7 x + 8 (ii)
x -3
i.e., f (x ) = a 0 × x n + a1 × x n - 1 + a2 × x n - 2 3 2
x - 6 x + 11x - 6 3 x2 +3
+ ... + a n - 1 × x + a n , (iii) (iv) x + or
( x - 4) x x
where x is a variable, a 0 , a 1 , a 2 , ..., a n are constants and
a 0 ¹ 0.
4. Degree of Polynomial
1. Real Polynomial The highest power of variable (x ) present in the
polynomial is called the degree of the polynomial.
Let a 0 , a 1 , a 2 , ..., a n be real numbers and x is a real variable.
For example, f ( x ) = a 0 × x n + a 1 × x n - 1 + a 2 × x n - 2
Then,
+ ... + a n - 1 × x + a n is a polynomial in x of degree n.
f ( x ) = a 0 × x n + a 1 × x n - 1 + a 2 × x n - 2 + ... + a n - 1 × x + a n
Remark
is called a real polynomial of real variable (x) with real A polynomial of degree one is generally called a linear
coefficients. polynomial. Polynomials of degree 2, 3, 4 and 5 are known as
quadratic, cubic, biquadratic and pentic polynomials,
For example, 5 x 3 - 3 x 2 + 7 x - 4, x 2 - 3 x + 1, etc., are real
respectively.
polynomials.
5. Polynomial Equation
2. Complex Polynomial If f (x ) is a polynomial, real or complex, then f (x ) = 0 is
Let a 0 , a 1 , a 2 , ..., a n are complex numbers and x is a called a polynomial equation.
varying complex number.
(i) A polynomial equation has atleast one root.
Then f ( x ) = a 0 × x n + a 1 × x n - 1 + a 2 × x n - 2 +... + a n - 1 × x + a n (ii) A polynomial equation of degree n has n roots.
is called a complex polynomial or a polynomial of complex
variable with complex coefficients. Remarks
3 2 2
For example, x - 7ix + (3 - 2i ) x + 13, 3 x - (2 + 3i ) x + 5i, 1. A polynomial equation of degree one is called a linear
equation i.e. ax + b = 0, where a, b Î C, set of all complex
etc. (where i = -1) are complex polynomials. numbers and a ¹ 0.
2. A polynomial equation of degree two is called a quadratic
equation i.e., ax 2 + bx + c, where a, b, c Î C and a ¹ 0.
3. Rational Expression 3. A polynomial equation of degree three is called a cubic
or Rational Function equation i.e., ax 3 + bx 2 + cx + d = 0, where a, b, c, d Î C and
P(x ) a ¹ 0.
An expression of the form , where P ( x ) and Q ( x ) 4. A polynomial equation of degree four is called a biquadratic
Q(x ) equation i.e., ax4 + bx 3 + cx 2 + dx + e = 0, where
are polynomials in x, is called a rational expression. As a a, b, c, d, e Î C and a ¹ 0.
particular case when Q ( x ) is a non-zero constant, P ( x ) 5. A polynomial equation of degree five is called a pentic
Q(x ) equation i.e., ax5 + bx4 + cx 3 + dx 2 + ex + f = 0, where
reduces to a polynomial. a, b, c, d, e, f Î C and a ¹ 0.
Chap 02 Theory of Equations 105

6. Roots of an Equation y Example 1. If equation


The values of the variable for which an equation is ( l2 - 5l + 6 ) x 2 + ( l2 - 3l + 2)x + ( l2 - 4 ) = 0 is
satisfied are called the roots of the equation. satisfied by more than two values of x , find the
If x = a is a root of the equation f ( x ) = 0, then f (a ) = 0. parameter l.
Sol. If an equation of degree two is satisfied by more than two
Remark values of unknown, then it must be an identity. Then, we
The real roots of an equation f ( x ) = 0 are the values of x, where must have
the curve y = f ( x ) crosses X-axis. l2 - 5l + 6 = 0, l2 - 3l + 2 = 0, l2 - 4 = 0
Þ l = 2 , 3 and l = 2 , 1 and l = 2 , - 2
7. Solution Set Common value of l which satisfies each condition is l = 2.
The set of all roots of an equation in a given domain is
called the solution set of the equation. y Example 2. Show that
For example, The roots of the equation (x + b ) (x + c ) (x + c ) (x + a) (x + a) (x + b )
+ + =1
x 3 - 2 x 2 - 5 x + 6 = 0 are 1, - 2, 3, the solution set is (b - a ) (c - a ) (c - b ) (a - b ) (a - c ) (b - c )
{1, - 2, 3 }. is an identity.
Remark Sol. Given relation is
Solve or solving an equation means finding its solution set or ( x + b) ( x + c ) ( x + c ) ( x + a) ( x + a) ( x + b)
+ + =1 …(i)
obtaining all its roots. (b - a ) (c - a ) (c - b ) (a - b ) (a - c ) (b - c )
(b - a ) (c - a )
When x = - a, then LHS of Eq. (i) = =1
(b - a ) (c - a )
Identity = RHS of Eq. (i)
If two expressions are equal for all values of x, then the When x = - b, then LHS of Eq. (i)
statement of equality between the two expressions is (c - b ) (a - b )
called an identity. = = 1 = RHS of Eq. (i)
(c - b ) (a - b )
For example, ( x + 1) 2 = x 2 + 2 x + 1 is an identity in x. (a - c ) (b - c )
and when x = - c , then LHS of Eq. (i) = =1
or (a - c ) (b - c )
If f ( x ) = 0 is satisfied by every value of x in the domain of = RHS of Eq. (i).
f ( x ), then it is called an identity. Thus, highest power of x occurring in relation of Eq. (i) is 2
For example, f ( x ) = ( x + 1) 2 - ( x 2 + 2 x + 1) = 0 is an and this relation is satisfied by three distinct values of
x ( = - a, - b, - c ). Therefore, it cannot be an equation and
identity in the domain C, as it is satisfied by every hence it is an identity.
complex number.
or y Example 3. Show that x 2 - 3| x | + 2 = 0 is an
If f ( x ) = a 0 × x n + a 1 × x n - 1 + a 2 × x n - 2 equation.
+ ... + a n - 1 × x + a n = 0 have more than n distinct roots, it Sol. Put x = 0 in x 2 - 3 | x | + 2 = 0
is an identity, then Þ 02 - 3 | 0| + 2 = 2 ¹ 0
a 0 = a 1 = a 2 = ... = a n - 1 = a n = 0
Since, the relation x 2 - 3 | x | + 2 = 0 is not satisfied by x = 0.
For example, If ax 2 + bx + c = 0 is satisfied by more than Hence, it is an equation.
two values of x, then a = b = c = 0.
or
In an identity in x coefficients of similar powers of x on Linear Equation
the two sides are equal.
An equation of the form
For example, If ax 4 + bx 3 + cx 2 + dx + e
ax + b = 0 …(i)
= 5 x 4 - 3 x 3 + 4 x 2 - 7 x - 9 be an identity in x, then where a, b Î R and a ¹ 0, is a linear equation.
a = 5, b = - 3, c = 4, d = - 7, e = - 9. Eq. (i) has an unique root equal to - .
b
Thus, an identity in x satisfied by all values of x, where as a
an equation in x is satisfied by some particular values of x .
106 Textbook of Algebra

x ( 3x - 1) x A root of the quadratic Eq. (i) is a complex number a, such


y Example 4. Solve the equation + =1-
2 6 2 that aa 2 + ba + c = 0. Recall that D = b 2 - 4ac is the
x ( 3x - 1) x discriminant of the Eq. (i) and its roots are given by the
Sol. We have, + =1-
2 6 2 following formula.
x x x 1
or + + =1+ -b ± D
2 2 2 6 x= [Shridharacharya method]
3x 7 2a
or =
2 6
or x=
7
9
Nature of Roots
1. If a, b, c Î R and a ¹ 0, then
y Example 5. Solve the equation (a - 3)x + 5 = a + 2 .
(i) If D < 0, then Eq. (i) has non-real complex roots.
Sol. Case I For a ¹ 3 , this equation is linear, then (ii) If D > 0, then Eq. (i) has real and distinct roots,
( a - 3) x = ( a - 3) namely
( a - 3)
\ x= =1 -b + D -b - D
( a - 3) x1 = , x2 = and then
Case II For a = 3, 2a 2a
0× x + 5 = 3 + 2 ax 2 + bx + c = a( x - x 1 ) ( x - x 2 ). …(ii)
Þ 5=5
(iii) If D = 0, then Eq. (i) has real and equal roots, then
Therefore, any real number is its solution.
b
x1 = x2 = - and then
2a
Quadratic Equations ax 2 + bx + c = a( x - x 1 ) 2 .
An equation in which the highest power of the unknown …(iii)
quantity is 2, is called a quadratic equation. To represent the quadratic ax 2 + bx + c in form
Quadratic equations are of two types :
Eqs. (ii) or (iii), is to expand it into linear factors.
1. Purely Quadratic Equation (iv) If D ³ 0, then Eq. (i) has real roots.
A quadratic equation in which the term containing the (v) If D 1 and D 2 be the discriminants of two
first degree of the unknown quantity is absent, is called a quadratic equations, then
purely quadratic equation. (a) If D 1 + D 2 ³ 0, then
i.e., ax 2 + c = 0, l atleast one of D 1 and D 2 ³ 0.
where a, c Î C and a ¹ 0. l if D 1 < 0, then D 2 > 0 and if D 1 > 0, then
D 2 < 0.
2. Adfected Quadratic Equation (b) If D 1 + D 2 < 0, then
A quadratic equation in which it contains the terms of l atleast one of D 1 and D 2 < 0.
first as well as second degrees of the unknown quantity, is l If D 1 < 0, then D 2 > 0 and if D 1 > 0, then
called an adfected (or complete) quadratic equation. D 2 < 0.
i.e., ax 2 + bx + c = 0, 2. If a, b, c Î Q and D is a perfect square of a rational
where a, b, c Î C and a ¹ 0, b ¹ 0. number, the roots are rational and in case it is not a
perfect square, the roots are irrational.
3. If a, b, c Î R and p + iq is one root of Eq. (i) (q ¹ 0 ),
Standard Quadratic Equation then the other must be the conjugate ( p - iq ) and
An equation of the form vice-versa (where, p, q Î R and i = -1).
ax 2 + bx + c = 0 …(i) 4. If a, b, c Î Q and p + q is one root of Eq. (i), then the
where a, b, c Î C and a ¹ 0, is called a standard quadratic other must be the conjugate p - q and vice-versa
equation. (where, p is a rational and q is a surd).
The numbers a, b, c are called the coefficients of this
5. If a = 1 and b, c Î I and the roots of Eq. (i) are rational
equation.
numbers, these roots must be integers.
Chap 02 Theory of Equations 107

6. If a + b + c = 0 and a, b, c are rational, 1 is a root of the y Example 8. Show that if p , q, r and s are real numbers
Eq. (i) and roots of the Eq. (i) are rational. and pr = 2(q + s ), then atleast one of the equations
1 x 2 + px + q = 0 and x 2 + rx + s = 0 has real roots.
7. a 2 + b 2 + c 2 - ab - bc - ca =
2 Sol. Let D1 and D 2 be the discriminants of the given equations
{(a - b ) 2 + (b - c ) 2 + (c - a ) 2 } x 2 + px + q = 0 and x 2 + rx + s = 0, respectively.
= - {(a - b ) (b - c ) + (b - c ) (c - a ) + (c - a ) (a - b )} Now, D1 + D 2 = p 2 - 4q + r 2 - 4s = p 2 + r 2 - 4(q + s )
= p 2 + r 2 - 2pr [given, pr = 2(q + s )]
y Example 6. Find all values of the parameter a for 2
= (p - r ) ³ 0 [Q p and q are real]
which the quadratic equation
or D1 + D 2 ³ 0
(a + 1) x 2 + 2 (a + 1) x + a - 2 = 0 Hence, atleast one of the equations x 2 + px + q = 0 and
(i) has two distinct roots. x 2 + rx + s = 0 has real roots.
(ii) has no roots. y Example 9. If a , b are the roots of the equation
(iii) has two equal roots.
( x - a ) ( x - b ) = c , c ¹ 0. Find the roots of the
Sol. By the hypothesis, this equation is quadratic and therefore
a ¹ - 1 and the discriminant of this equation, equation ( x - a ) ( x - b ) + c = 0.
D = 4 ( a + 1) 2 - 4 ( a + 1) ( a - 2) Sol. Since, a , b are the roots of
= 4 ( a + 1) ( a + 1 - a + 2) ( x - a) ( x - b) = c
or ( x - a ) ( x - b ) - c = 0,
= 12(a + 1)
Then ( x - a) ( x - b) - c = ( x - a ) ( x - b)
(i) For a > ( -1), then D > 0, this equation has two distinct
roots. Þ ( x - a ) ( x - b) + c = ( x - a) ( x - b)
Hence, roots of ( x - a ) ( x - b ) + c = 0 are a, b.
(ii) For a < ( - 1), then D < 0, this equation has no roots.
(iii) This equation cannot have two equal roots. Since, y Example 10. Find all roots of the equation
D = 0 only for a = - 1 and this contradicts the
hypothesis. x 4 + 2x 3 - 16 x 2 - 22x + 7 = 0, if one root is 2 + 3.
Sol. All coefficients are real, irrational roots will occur in
y Example 7. Solve for x, conjugate pairs.
2 2
( 5 + 2 6 ) x - 3 + ( 5 - 2 6 ) x - 3 = 10. Hence, another root is 2 - 3.
Sol. Q (5 + 2 6 ) (5 - 2 6 ) = 1 \ Product of these roots = ( x - 2 - 3 ) ( x - 2 + 3 )

\ (5 - 2 6 ) =
1 = ( x - 2)2 - 3 = x 2 - 4 x + 1.
(5 + 2 6 ) On dividing x 4 + 2x 3 - 16x 2 - 22x + 7 by x 2 - 4 x + 1, then
2 2
-3 -3
\ (5 + 2 6 )x + (5 - 2 6 )x = 10 the other quadratic factor is x 2 + 6x + 7.
2
x -3 Then, the given equation reduce in the form
2
- 3 æ 1 ö
reduces to (5 + 2 6 )x +ç ÷ = 10 ( x 2 - 4 x + 1) ( x 2 + 6x + 7 ) = 0
è5 + 2 6 ø
\ x 2 + 6x + 7 = 0
2
-3 1
Put (5 + 2 6 )x = t , then t + = 10 - 6 ± 36 - 28
t Then, x= = -3 ± 2
2
Þ t 2 - 10t + 1 = 0
Hence, the other roots are 2 - 3, - 3 ± 2.
10 ± (100 - 4 )
or t = = (5 ± 2 6 )
2
Þ (5 + 2 6 )x
2
-3
= (5 ± 2 6 ) = (5 + 2 6 )± 1
Relation between Roots
\ x 2
-3= ±1
and Coefficients
2
1. Relation between roots and coefficients of
Þ x - 3 = 1 or x 2 - 3 = - 1 quadratic equation If roots of the equation
Þ x 2 = 4 or x 2 = 2 ax 2 + bx + c = 0 (a ¹ 0 ) be real and distinct and a < b,
Hence, x = ± 2, ± 2 -b + D -b - D
then a = ,b = .
2a 2a
108 Textbook of Algebra

(i) Sum of roots (vii) a 5 + b5 = (a 2 + b2 ) (a 3 + b 3 ) - a 2 b2 (a + b)


b Coefficient of x
=S =a +b = - =- . æ b 2 - 2ac ö æ (b 3 - 3abc ) ö c 2 æ bö
a Coefficient of x 2 =ç ÷ ç- ÷- 2 ç- ÷
è a2 ø è a3 ø a è aø
(ii) Product of roots
c Constant term - (b 5 - 5ab 3 c + 5a 2 bc 2 )
= P = ab = = . = .
a Coefficient of x 2 a5
(iii) Difference of roots (viii) a 5 - b5 = (a 2 + b2 ) (a 3 - b 3 ) + a 2 b2 (a - b)
D Discriminant
= D ¢ =a -b = = .
a Coefficient of x 2 æ b 2 - 2ac ö æ D ( D + 3ac ) ö æ c ö 2 æ D ö
=ç ÷ç ÷ +ç ÷ ç ÷
2. Formation of an equation with given roots è a2 ø è a3 ø èa ø è a ø
A quadratic equation whose roots are a and b, is
D (b 4 - 3acb 2 + 3a 2 c 2 )
given by ( x - a ) ( x - b) = 0 or x 2 - (a + b) x + ab = 0 = .
a5
i.e. x 2 - (Sum of roots) x + Product of roots = 0
\ x 2 - Sx + P = 0. y Example 11. If one root of the equation
x 2 - ix - (1 + i ) = 0, (i = -1 ) is 1 + i, find the other root.
3. Symmetric function of roots A function of a and
b is said to be symmetric function, if it remains Sol. All coefficients of the given equation are not real, then
other root ¹ 1 - i.
unchanged, when a and b are interchanged.
Let other root be a, then sum of roots = i
For example, a 3 + 3 a 2 b + 3 ab2 + b 3 is a symmetric i.e. 1 + i + a = i Þ a = ( - 1)
function of a and b, whereas a 3 - b 3 + 5 ab is not a Hence, the other root is ( -1).
symmetric function of a and b. In order to find the
y Example 12. If one root of the equation
value of a symmetric function in terms of a + b, ab
9+ 5
and a - b and also in terms of a, b and c. x 2 - 5 x - 19 = 0 is , then find the other root.
2
(i) a 2 + b2 = (a + b) 2 - 2 ab
2
Sol. All coefficients of the given equation are not rational,
2
æ bö æ c ö b - 2ac 9- 5
= ç- ÷ - 2 ç ÷ = . then other root ¹ .
è aø èa ø a2 2
Let other root be a, sum of roots = 5
(ii) a 2 - b2 = (a + b) (a - b)
9+ 5 -9 + 5
æ bö æ D ö b D Þ +a = 5 Þ a =
= ç- ÷ ç ÷ =- 2 . 2 2
è aø è a ø a -9 + 5
Hence, other root is .
(iii) a 3 + b 3 = (a + b) 3 - 3 ab (a + b) 2
3
æ bö æc ö æ bö æ b 3 - 3abc ö y Example 13. If the difference between the
= ç- ÷ - 3 ç ÷ ç- ÷ = - ç ÷.
è aø èa ø è a ø è a3 ø corresponding roots of the equations x 2 + ax + b = 0
and x 2 + bx + a = 0 (a ¹ b ) is the same, find the value
(iv) a 3 - b 3 = (a - b) 3 + 3 ab (a - b)
3 of a + b .
æ Dö æc ö æ D ö D ( D + 3ac ) Sol. Let a , b be the roots of x 2 + ax + b = 0 and g , d be the
=ç ÷ +3ç ÷ ç ÷= .
è a ø è a ø è a ø a3 roots of x 2 + bx + a = 0, then given
(v) a 4 + b 4 = (a 2 + b2 ) 2 - 2 a 2 b2 a -b = g - d
2 é
æ b 2 - 2ac ö 2
b 4 + 2a 2 c 2 - 4acb 2
2
a - 4b b 2 - 4a Dù
æc ö Þ = êQa - b = a ú
=ç ÷ -2ç ÷ = . 1 1
è a2 ø èa ø a4 ë û
Þ a 2 - 4b = b 2 - 4a
(vi) a 4 - b 4 = (a 2 + b2 ) (a 2 - b2 )
Þ (a 2 - b 2 ) + 4 (a - b ) = 0 Þ (a - b ) (a + b + 4 ) = 0
2
b D (b - 2ac ) a-b¹0
=- . Q
a4 \ a + b + 4 = 0 or a + b = - 4.
Chap 02 Theory of Equations 109

y Example 14. If a + b + c = 0 and a, b , c are rational. y Example 16. If a is a root of 4 x 2 + 2x - 1 = 0.


Prove that the roots of the equation Prove that 4 a 3 - 3 a is the other root.
(b + c - a ) x 2 + (c + a - b ) x + (a + b - c ) = 0 Sol. Let other root is b,
2 1 1
are rational. then a + b = - = - or b = - - a …(i)
4 2 2
Sol. Given equation is and so 4 a 2 + 2a - 1 = 0, because a is a root of
(b + c - a ) x 2 + (c + a - b ) x + (a + b - c ) = 0 …(i) 4 x 2 + 2x - 1 = 0.
Q (b + c - a ) + (c + a - b ) + (a + b - c ) = a + b + c = 0
Now, b = 4 a 3 - 3 a = a ( 4 a 2 - 3)
\ x = 1 is a root of Eq. (i), let other root of Eq. (i) is a, then
= a ( 1 - 2 a - 3) [Q 4 a 2 + 2 a - 1 = 0]
a+b-c
Product of roots = = -2 a 2 - 2 a
b+c -a
1
-c - c = - (4 a 2 ) - 2 a
Þ 1´a = [Qa + b + c = 0] 2
-a - a 1
= - (1 - 2 a ) - 2 a [Q 4 a 2 + 2 a - 1 = 0]
c 2
\ a= [rational]
a 1
= - -a =b [from Eq. (i)]
Hence, both roots of Eq. (i) are rational. 2
3
Aliter Hence, 4 a - 3 a is the other root.
Let b + c - a = A , c + a - b = B, a + b - c = C
Then, A + B+C =0 [Qa + b + c = 0] …(ii) y Example 17. If a , b are the roots of the equation
Now, Eq. (i) becomes l ( x 2 - x ) + x + 5 = 0. If l 1 and l 2 are two values of l
Ax 2 + Bx + C = 0 …(iii) a b 4
for which the roots a , b are related by + = , find
Discriminant of Eq. (iii), b a 5
D = B 2 - 4 AC l1 l 2
the value of + .
= ( - C - A )2 - 4 AC [Q A + B + C = 0] l 2 l1
2 Sol. The given equation can be written as
= (C + A ) - 4 AC
l x 2 - ( l - 1) x + 5 = 0
= (C - A )2 = (2a - 2c )2
Qa , b are the roots of this equation.
= 4 (a - c )2 = A perfect square l -1 5
\ a +b = and ab =
Hence, roots of Eq. (i) are rational. l l
a b 4
y Example 15. If the roots of equation But, given + =
b a 5
a (b - c ) x 2 + b (c - a ) x + c (a - b ) = 0
a 2 + b2 4
be equal, prove that a, b , c are in HP. Þ =
ab 5
Sol. Given equation is ( l - 1) 2 10
2 -
a (b - c ) x + b (c - a ) x + c (a - b ) = 0 …(i) 2
(a + b ) - 2ab 4 l2 l =4
Þ = Þ
Here, coefficient of x 2 + coefficient of x + constant term = 0 ab 5 5 5
l
i.e., a (b - c ) + b (c - a ) + c (a - b ) = 0
( l - 1)2 - 10l 4
Then, 1 is a root of Eq. (i). Þ = Þ l2 - 12l + 1 = 4 l
Since, its roots are equal. 5l 5
Therefore, its other root will be also equal to 1. Þ l2 - 16l + 1 = 0
c (a - b ) It is a quadratic in l, let roots be l1 and l 2 , then
Then, product of roots = 1 ´ 1 =
a (b - c ) l1 + l 2 = 16 and l1l 2 = 1
Þ ab - ac = ca - bc l1 l 2 l21 + l22 ( l1 + l 2 )2 - 2l1l 2
\ + = =
2ac l 2 l1 l1l 2 l1l 2
\ b=
a+c (16)2 - 2(1)
= = 254
Hence, a, b and c are in HP. 1
110 Textbook of Algebra

y Example 18. If a , b are the roots of the equation = ( a + b ) (a - b )2 {(a + b )2 - ab }


x 2 - px + q = 0, find the quadratic equation the roots = p ( p 2 - 4q ) ( p 2 - q )
of which are (a 2 - b 2 ) (a 3 - b 3 ) and a 3b 2 + a 2 b 3 . and a 3b 2 + a 2 b 3 = a 2 b 2 (a + b )= pq 2
Sol. Since, a , b are the roots of x 2 - px + q = 0. S = Sum of roots = p ( p 2 - 4q ) ( p 2 - q ) + pq 2
\ a + b = p , ab = q = p ( p 4 - 5p 2q + 5q 2 )
Þ a - b = ( p 2 - 4q ) P = Product of roots = p 2q 2 ( p 2 - 4q ) ( p 2 - q )

Now, (a 2 - b 2 ) (a 3 - b 3 ) \ Required equation is x 2 - Sx + P = 0


i.e. x 2 - p ( p 4 - 5p 2q + 5q 2 ) x + p 2q 2 ( p 2 - 4q ) ( p 2 - q ) = 0
= (a + b ) (a - b ) (a - b ) (a 2 + ab + b 2 )

#L Exercise for Session 1


1. If (a 2 - 1)x 2 + (a - 1) x + a 2 - 4a + 3 = 0 be an identity in x , then the value of a is/are
(a) -1 (b) 1 (c) 3 (d) -1, 1, 3

2. 2
The roots of the equation x + 2 3x + 3 = 0 are
(a) real and unequal (b) rational and equal
(c) irrational and equal (d) irrational and unequal

3. Ifa, b , c Î Q, then roots of the equation (b + c - 2a ) x 2 + (c + a - 2b ) x + (a + b - 2c ) = 0 are


(a) rational (b) non-real (c) irrational (d) equal

4. 2 2
If P ( x ) = ax + bx + c and Q( x ) = - ax + dx + c, where ac ¹ 0, then P ( x ) Q ( x ) = 0 has atleast
(a) four real roots (b) two real roots
(c) four imaginary roots (d) None of these

5. If roots of the equation (q - r ) x 2 + (r - p ) x + ( p - q ) = 0 are equal, then p, q , r are in


(a) AP (b) GP (c) HP (d) AGP

6. If one root of the quadratic equation ix 2 - 2 (i + 1) x + (2 - i ) = 0,i = -1 is 2 - i , the other root is


(a) - i (b) i (c) 2 + i (d 2 - i

7. If the difference of the roots of x 2 - lx + 8 = 0 be 2, the value of l is


(a) ± 2 (b) ± 4 (c) ± 6 (d) ± 8

8. If 3p 2 = 5p + 2 and 3q 2 = 5q + 2 where p ¹ q , pq is equal to


2 2 3 3
(a) (b) - (c) (d) -
3 3 2 2

9. If a, b are the roots of the quadratic equation x 2 + bx - c = 0, the equation whose roots are b and c, is
(a) x 2 + ax - b = 0 (b) x 2 - [(a + b ) + ab ] x - ab (a + b ) = 0
(c) x 2 + [(a + b ) + ab ] x + ab (a + b ) = 0 (d) x 2 + [(a + b ) + ab ] x - ab (a + b ) = 0

10. Let p, q Î {1, 2, 3, 4}. The number of equations of the form px 2 + qx + 1 = 0 having real roots, is
(a) 15 (b) 9 (c) 8 (d) 7

11. 2
If a and b are the roots of the equation ax + bx + c = 0 (a ¹ 0, a, b , c being different), then
(1 + a + a 2 ) (1 + b + b 2 ) is equal to
(a) zero (b) positive (c) negative (d) None of these
Session 2
Transformation of Quadratic Equations, Condition
for Common Roots
Transformation of y Example 19. If a , b be the roots of the equation
Quadratic Equations x 2 - px + q = 0, then find the equation whose roots are
q q
Let a, b be the roots of the equation ax 2 + bx + c = 0, then and ×
p -a p -b
the equation
q q
(i) whose roots are a + k, b + k, is Sol. Let =x Þ a=p-
p -a x
a (x - k )2 + b (x - k ) + c = 0 [replace x by ( x - k )] q
So, we replacing x by p - in the given equation, we get
(ii) whose roots are a - k, b - k, is x
2
a ( x + k ) 2 + b ( x + k ) + c = 0 [replace x by ( x + k )] æ qö æ qö
çp - ÷ - p çp - ÷ + q = 0
è xø è xø
(iii) whose roots are ak, bk, is 2
q 2pq pq
é æ x öù Þ p2 + 2 - - p2 + +q =0
ax 2 + kbx + k 2 c = 0 ê replace x by çè k ÷ø ú x x x
a b ë û pq q 2
(iv) whose roots are , , is Þ q- + 2 =0
k k x x
ak 2 x 2 + bkx + c = 0 [replace x by xk] or qx 2 - pqx + q 2 = 0 or x 2 - px + q = 0
q q
(v) whose roots are - a, - b , is is the required equation whose roots are and ×
p -a p -b
ax 2 - bx + c = 0 [replace x by (- x )]
1 1 y Example 20. If a and b are the roots of
(vi) whose roots are , , is
a b ax 2 + bx + c = 0, then find the roots of the equation
é æ 1 öù
cx 2 + bx + a = 0 ê replace x by çè x ÷ø ú ax 2 - bx ( x - 1) + c ( x - 1) 2 = 0.
ë û
1 1 Sol. Q ax 2 - bx ( x - 1) + c ( x - 1)2 = 0 …(i)
(vii) whose roots are - , - , is 2
a b æ x ö æ x ö
Þ aç ÷ - bç ÷ +c =0
é æ 1 öù è x - 1ø è x - 1ø
cx 2 - bx + a = 0 ê replace x by çè - x ÷ø ú 2
ë û æ x ö æ x ö
k k or aç ÷ + bç ÷ +c =0
(viii) whose roots are , , is è1 - x ø è1 - x ø
a b
Now, a , b are the roots of ax 2 + bx + c = 0.
é æ k öù
cx 2 + kbx + k 2 a = 0 ê replace x by çè x ÷ø ú Then, a=
x
and b =
x
ë û 1- x 1- x
(ix) whose roots are pa + q, pb + q, is a b
2 Þ x= and x =
æ x -q ö æx -qö é æ x - q öù a +1 b +1
aç ÷ +b ç ÷ + c = 0 ê replace x by ç ÷ú a b
è p ø è p ø ë è p øû Hence, , are the roots of the Eq. (i).
a +1 b +1
(x) whose roots are a n , bn , n Î N , is
a ( x 1 /n ) 2 + b ( x 1 /n ) + c = 0 [ replace x by ( x 1 /n )] y Example 21. If a , b be the roots of the equation
3 3
(xi) whose roots are a 1 /n , b1 /n , n Î N is æ1 - a ö æ1 - b ö
3x 2 + 2x + 1 = 0, then find value of ç ÷ +ç ÷ .
è 1+ a ø è 1+ b ø
a (x n )2 + b (x n ) + c = 0 [ replace x by ( x n )] 1-a 1- x
Sol. Let =x Þ a=
1+a 1+ x
112 Textbook of Algebra

1- x y Example 22. For what values of m, the


So, replacing x by in the given equation, we get
1+ x
2 equation x 2 + 2 (m - 1) x + m + 5 = 0 has (m ÎR )
æ1 - x ö æ1 - x ö 2
3 ç ÷ + 2ç ÷ + 1 = 0 Þ x - 2x + 3 = 0 …(i) (i) roots are equal in magnitude but opposite in
è1 + x ø è1 + x ø sign?
1-a 1-b
It is clear that and are the roots of Eq. (i). (ii) roots are reciprocals to each other?
1+a 1+b
(iii) roots are opposite in sign?
æ1 - a ö æ1 - b ö (iv) both roots are positive?
\ ç ÷+ç ÷ =2 …(ii)
è1 + a ø è1 + b ø (v) both roots are negative?
æ1 - a ö æ1 - b ö (vi) atleast one root is positive?
and ç ÷ç ÷ =3 …(iii)
(vii) atleast one root is negative?
è1 + a ø è1 + b ø

æ1 - a ö
3 3
æ1 - b ö æ1 - a 1 - bö
3 Sol. Here, a = 1, b = 2(m - 1) and c = m + 5
\ ç ÷ +ç ÷ =ç + ÷ -3
è1 + a ø è1 + b ø è 1 + a 1 + b ø \ D = b 2 - 4ac = 4 (m - 1)2 - 4 (m + 5)
= 4 (m 2 - 3m - 4 )
æ1 - a ö æ1 - b ö æ1 - a 1 - b ö 3
ç ÷ç ÷ç + ÷ = 2 - 3 × 3 × 2 = 8 - 18 = - 10
è1 + a ø è1 + b ø è1 + a 1 + b ø \ D = 4 (m - 4 ) (m + 1) and here a = 1 > 0
(i) b = 0 and D > 0
Þ 2(m - 1) = 0 and 4 (m - 4 ) (m + 1) > 0
Roots Under Special Cases Þ m = 1 and m Î( - ¥, - 1) È ( 4, ¥ )
Consider the quadratic equation ax 2 + bx + c = 0 …(i) \ m Îf [null set]
where a, b, c Î R and a ¹ 0. Then, the following hold good : (ii) a = c and D ³ 0
(i) If roots of Eq. (i) are equal in magnitude but opposite in Þ 1 = m + 5 and 4 (m - 4 ) (m + 1) ³ 0
sign, then sum of roots is zero as well as D > 0, i.e. b = 0 Þ m = - 4 and m Î (-¥, - 1] È [ 4, ¥ )
and D > 0.
\ m = -4
(ii) If roots of Eq. (i) are reciprocal to each other, then product (iii) a > 0, c < 0 and D > 0
of roots is 1 as well as D ³ 0 i.e., a = c and D ³ 0.
Þ 1 > 0, m + 5 < 0 and 4 (m - 4 ) (m + 1) > 0
(iii) If roots of Eq. (i) are of opposite signs, then product of
Þ m < - 5 and m Î( -¥, - 1) È ( 4, ¥ )
roots < 0 as well as D > 0 i.e., a > 0, c < 0 and D > 0 or
a < 0, c > 0 and D > 0. \ m Î (- ¥, - 5)
(iv) If both roots of Eq. (i) are positive, then sum and product of (iv) a > 0, b < 0, c > 0 and D ³ 0
roots > 0 as well as D ³ 0 i.e., a > 0, b < 0, c > 0 and D ³ 0 or Þ 1 > 0, 2(m - 1) < 0, m + 5 > 0
a < 0, b > 0, c < 0 and D ³ 0. and 4 ( m - 4 ) ( m + 1) ³ 0
(v) If both roots of Eq. (i) are negative, then sum of roots < 0, Þ m < 1, m > - 5 and m Î (- ¥, - 1] È [ 4, ¥ )
product of roots > 0 as well as D ³ 0 i.e., a > 0, b > 0, c > 0 Þ m Î (- 5, - 1]
and D ³ 0 or a < 0, b < 0, c < 0 and D ³ 0.
(v) a > 0, b > 0, c > 0 and D ³ 0
(vi) If atleast one root of Eq. (i) is positive, then either one root
Þ 1 > 0, 2(m - 1) > 0, m + 5 > 0
is positive or both roots are positive i.e., point (iii) È (iv).
and 4 (m - 4 ) (m + 1) ³ 0
(vii) If atleast one root of Eq. (i) is negative, then either one root
is negative or both roots are negative i.e., point (iii) È (v). Þ m > 1, m > - 5 and m Î (-¥, - 1] È [ 4, ¥ )
(viii) If greater root in magnitude of Eq. (i) is positive, then \ m Î[ 4, ¥ )
sign of b = sign of c ¹ sign of a. (vi) Either one root is positive or both roots are
(ix) If greater root in magnitude of Eq. (i) is negative, then positive
sign of a = sign of b ¹ sign of c. i.e., (c) È (d)
(x) If both roots of Eq. (i) are zero, then b = c = 0. Þ m Î (-¥, - 5) È ( - 5, - 1]
æ bö (vii) Either one root is negative or both roots are
(xi) If roots of Eq. (i) are 0 and ç - ÷ , then c = 0.
è aø negative
c i.e., (c) È (e)
(xii) If roots of Eq. (i) are 1 and , then a + b + c = 0.
a Þ m Î (-¥, - 5) È [ 4, ¥ )
Chap 02 Theory of Equations 113

Condition for Common Roots y Example 23. Find the value of l, so that the
equations x 2 - x - 12 = 0 and lx 2 + 10x + 3 = 0 may
1. Only One Root is Common have one root in common. Also, find the common root.
Sol. Q x 2 - x - 12 = 0
Consider two quadratic equations
Þ ( x - 4 ) ( x + 3) = 0
ax 2 + bx + c = 0 and a ¢ x 2 + b ¢ x + c ¢ = 0
\ x = 4, - 3
[where a, a ¢ ¹ 0 and ab ¢ - a ¢ b ¹ 0] If x = 4 is a common root, then
Let a be a common root, then l( 4 )2 + 10( 4 ) + 3 = 0
a a 2 + ba + c = 0 and a ¢ a 2 + b ¢ a + c ¢ = 0. 43
\ l=-
On solving these two equations by cross-multiplication, 16
we have and if x = - 3 is a common root, then
a2 a 1 l( -3)2 + 10( -3) + 3 = 0
= =
bc ¢ - b ¢ c ca ¢ - c ¢ a ab ¢ - a ¢ b \ l =3
43
From first two relations, we get Hence, for l = - , common root is x = 4
16
bc ¢ - b ¢ c
a= …(i) and for l = 3, common root is x = - 3.
ca ¢ - c ¢ a
and from last two relations, we get y Example 24. If equations ax 2 + bx + c = 0, (where
ca ¢ - c ¢ a a, b , c ÎR and a ¹ 0) and x 2 + 2x + 3 = 0 have a
a= …(ii)
ab ¢ - a ¢ b common root, then show that a :b : c = 1 : 2 : 3.
From Eqs. (i) and (ii), we get Sol. Given equations are
bc ¢ - b ¢ c ca ¢ - c ¢ a
= ax 2 + bx + c = 0 …(i)
ca ¢ - c ¢ a ab ¢ - a ¢ b 2
and x + 2x + 3 = 0 …(ii)
Þ (ab ¢ - a ¢ b ) (bc ¢ - b ¢ c ) = (ca ¢ - c ¢ a ) 2
Clearly, roots of Eq. (ii) are imaginary, since Eqs. (i) and (ii)
2
a b b c c a have a common root. Therefore, common root must be
or ´ = [remember] imaginary and hence both roots will be common.
a¢ b¢ b¢ c¢ c¢ a¢
Therefore, Eqs. (i) and (ii) are identical.
This is the required condition for one root of two
a b c
quadratic equations to be common. \ = = or a : b : c = 1 : 2 : 3
1 2 3

2. Both Roots are Common y Example 25. If a, b , c are in GP, show that the
Let a, b be the common roots of the equations equations ax 2 + 2bx + c = 0 and dx 2 + 2ex + f = 0
ax 2 + bx + c = 0 and a ¢ x 2 + b ¢ x + c ¢ = 0, then a b c
have a common root, if , , are in HP.
b b¢ a b d e f
a +b = - = - Þ = …(iii) Sol. Given equations are
a a¢ a¢ b¢
ax 2 + 2bx + c = 0 …(i)
c c¢ a c
and ab = = Þ = …(iv) and 2
dx + 2ex + f = 0 …(ii)
a a¢ a¢ c¢
a b c Since, a, b, c are in GP.
From Eqs. (iii) and (iv), we get = =
a¢ b¢ c¢ \ b 2 = ac or b = ac
This is the required condition for both roots of two
From Eq. (i), ax 2 + 2 ac x + c = 0
quadratic equations to be identical.
c
or ( a x + c )2 = 0 or x = -
Remark a
To find the common root between the two equations, make the Q Given Eqs. (i) and (ii) have a common root.
same coefficient of x 2 in both equations and then subtract of the
two equations. c
Hence, x = - also satisfied Eq. (ii), then
a
114 Textbook of Algebra

æc ö c d f 2e
d ç ÷ - 2e + f =0 or + =
èa ø a a c b
d 2e f d e f
Þ - + =0 \ , , are in AP.
a ac c a b c

d 2e f a b c
or - + =0 [Qb = ac ] Hence, , , are in HP.
a b c d e f

#L Exercise for Session 2


1. If a and b are the roots of the equation 2x 2 - 3x + 4 = 0, then the equation whose roots are a 2 and b 2, is
(a) 4x 2 + 7x + 16 = 0 (b) 4x 2 + 7x + 6 = 0 (c) 4x 2 + 7x + 1 = 0 (d) 4x 2 - 7x + 16 = 0

æ 1 1 ö
2. If a, b are the roots of x 2 - 3x + 1 = 0, then the equation whose roots are ç , ÷ , is
è a - 2 b - 2ø
(a) x 2 + x - 1 = 0 (b) x 2 + x + 1 = 0 (c) x 2 - x - 1 = 0 (d) None of these

3. The equation formed by decreasing each root of ax 2 + bx + c = 0 by 1 is 2x 2 + 8x + 2 = 0, then


(a) a = - b (b) b = - c (c) c = - a (d) b = a + c
2
x - bx m - 1
4. If the roots of equation = are equal but opposite in sign, then the value of m will be
ax - c m +1
a -b b -a a+b b +a
(a) (b) (c) (d)
a+b a+b a -b b -a

5. If x 2 + px + q = 0 is the quadratic equation whose roots are a - 2 and b - 2, where a and b are the roots of
x 2 - 3x + 1 = 0, then
(a) p = 1, q = 5 (b) p = 1, q = - 5 (c) p = - 1, q = 1 (d) None of these

6. 2
If both roots of the equation x - (m - 3) x + m = 0 (m Î R ) are positive, then
(a) m Î (3, ¥) (b) m Î(- ¥, 1] (c) m Î [9, ¥) (d) m Î(1, 3)

7. 2
If the equation (1 + m ) x - 2 (1 + 3m ) x + (1 + 8m ) = 0, where m Î R ~ {-1}, has atleast one root is negative, then

(b) m Îæç - , ¥ö÷ (c) m Î æç -1, -


1 1ö
(a) m Î (- ¥, - 1) ÷ (d) m ÎR
è 8 ø è 8ø

8. If both the roots of l (6x 2 + 3) + rx + 2x 2 - 1 = 0 and 6l (2x 2 + 1) + px + 4x 2 - 2 = 0 are common, then 2r - p is


equal to
(a) -1 (b) 0 (c) 1 (d) 2
a3 + b 3 + c 3
9. 2 2
If ax + bx + c = 0 and bx + cx + a = 0 have a common root a ¹ 0, then is equal to
abc
(a) 1 (b) 2 (c) 3 (d) None of these
10. If a ( p + q )2 + 2bpq + c = 0 and a ( p + r )2 + 2bpr + c = 0, then qr is equal to
c a a b
(a) p 2 + (b) p 2 + (c) p 2 + (d) p 2 +
a c b a
Session 3
Quadratic Expression, Wavy Curve Method, Condition
for Resolution into Linear Factors, Location of Roots

Quadratic Expression 4. Intersection with axes


2
An expression of the form ax + bx + c , where a, b, c Î R (i) Intersection with X-axis
and a ¹ 0 is called a quadratic expression in x. So, in For X-axis, y = 0.
general quadratic expression is represented by -b ± D
2 2
\ ax 2 + bx + c = 0 Þ x =
f ( x ) = ax + bx + c or y = ax + bx + c . 2a
For D > 0, parabola cuts X-axis in two real and
Graph of a Quadratic Expression distinct points
a < 0, D > 0
We have, y = ax 2 + bx + c = f ( x ), [a ¹ 0 ]
éæ bö
2
D ù X-axis
Þ y = a êç x + ÷ - ú
êë è 2a ø 4a 2 úû X-axis
a > 0, D > 0
2
æ Dö æ bö
or çy + ÷ = a ç x + ÷ For D = 0, parabola touches X-axis in one point
è 4a ø è 2a ø
b
D b i.e., x = - .
Now, let y + = Y and x + =X 2a
4a 2a a<0 , D = 0
2
\ Y = aX X-axis
2 Y
Þ X =
a X-axis
1. The shape of the curve y = f (x ) is parabolic. a >0 , D =0
b
2. The axis of parabola is X = 0 or x + =0 For D < 0, parabola does not cut X-axis i.e.,
2a
imaginary values of x.
b
or x = - i.e. parallel to Y-axis. a < 0, D < 0
2a X-axis
1
3. (i) If > 0 Þ a > 0, the parabola open upwards.
a
X-axis
a > 0, D < 0

(ii) Intersection with Y-axis


1 For Y-axis, x = 0.
(ii) If < 0 Þ a < 0, the parabola open downwards.
a \ y =c
5. Greatest and least values of f(x)
1
Vertex of the parabola X 2 = Y is
a
X = 0, Y = 0
116 Textbook of Algebra

b D 3. a > 0 and D = 0. So, f ( x ) ³ 0 for all x Î R,


Þ x+ = 0, y + =0
2a 4a i.e. f ( x ) is positive for all real values of x except at
b D vertex, where f ( x ) = 0.
or x = - ,y = -
2a 4a
æ b Dö
Hence, vertex of y = ax 2 + bx + c is ç - , - ÷ .
è 2a 4a ø a>0

Vertex X-axis

a>0 4. a < 0 and D = 0. So, f ( x ) £ 0 for all x Î R,


a<0
i.e. f ( x ) is negative for all real values of x except at
Vertex vertex, where f ( x ) = 0.
b X-axis
For a > 0, f ( x ) has least value at x = - .
2a a<0
æ bö D
This least value is given by f ç - ÷ = -
è 2a ø 4a
D
or y least = - .
4a 5. a > 0 and D > 0.
æ D ö Let f ( x ) = 0 have two real roots a and b (a < b), then
\ Range of y = ax 2 + bx + c is ç - , ¥÷ .
è 4a ø f ( x ) > 0 for all x Î( - ¥, a ) È (b, ¥) and f ( x ) < 0 for
b
For a < 0, f ( x ) has greatest value at x = - . all x Î(a, b).
2a
æ bö D
This greatest value is given by f ç - ÷ = -
è 2a ø 4a
D
or y greatest = - a>0
4a
æ Dö a b
X-axis
\ Range of y = ax 2 + bx + c is ç - ¥, - ÷ .
è 4a ø
6. a < 0 and D > 0
Sign of Quadratic Expression Let f ( x ) = 0 have two real roots a and b (a < b),
then f ( x ) < 0 for all x Î ( - ¥, a ) È (b, ¥)
Let f ( x ) = ax 2 + bx + c or y = ax 2 + bx + c ,
and f ( x ) > 0 for all x Î(a, b).
where a, b, c Î R and a ¹ 0, for some values of x , f ( x ) may
a b
be positive, negative or zero. This gives the following X-axis
cases : a<0
1. a > 0 and D < 0.
So, f ( x ) > 0 for all x Î R,
i.e. f ( x ) is positive for all real values of x.
Wavy Curve Method
a>0
(Generalised Method of Intervals)
Wave Curve Method is used for solving inequalities of the
X-axis form
2. a < 0 and D < 0. So, f ( x ) < 0 for all x Î R, ( x - a 1 ) k 1 ( x - a 2 ) k 2 K( x - a m ) k m
f (x ) = >0
i.e. f ( x ) is negative for all real values of x. ( x - b 1 ) p 1 ( x - b 2 ) p 2 K( x - b n ) p n
X-axis
( < 0, ³ 0 or £ 0),
a<0 where, k 1 , k 2 , K, k m , p 1, p 2 , ..., p n are natural numbers and
such that a i ¹ b j , where i = 1, 2, K , m and j = 1, 2, K, n.
Chap 02 Theory of Equations 117

We use the following methods: Important Results


k1 k2 km
1. Solve ( x - a 1 ) ( x - a 2 ) K(x - am ) = 0 and 1. The point where denominator is zero or function approaches
infinity, will never be included in the answer.
( x - b1 )p1 ( x - a 2 )p 2 K ( x - bn )pn = 0, then we get 2. For x 2 < a2 or|x| < a Û - a < x < a
x = a1, a 2 , K , am , b1, b 2 , K , bn [critical points] i.e., x Î( - a, a)
2. Assume a 1 < a 2 < K < a m < b 1 < b 2 < K < b n 3. For 0 < x 2 < a2 or 0 < |x| < a

Plot them on the real line. Arrange inked (black) Û - a < x < a ~ {0}
circles (·) and un-inked (white) circles ( ), such o i.e., x Î( - a, a) ~{0}
that 4. For x 2 ³ a2 or|x| ³ a Û x £ - a or x ³ a

a1 a2 K am b1 b2 K bn i.e., x Î( - ¥, - a] È [ a, ¥)
5. For x 2 > a2 or|x| > a Û x < - a or x > a
If f ( x ) > 0 oo o oo o
... ...
i.e., x Î( - ¥, - a) È ( a, ¥)
f (x ) < 0 oo o oo o
... ...
6. For a2 £ x 2 £ b2 or a £ |x| £ b
f ( x ) ³ 0 · · ... · oo o ...
Û a £ x £ b or - b £ x £ - a
f ( x ) £ 0 · · ... · oo o ... i.e., x Î[ - b, - a] È [ a, b]
3. Obviously, b n is the greatest root. If in all brackets 7. For a2 < x 2 £ b2 or a < |x| £ b
before x positive sign and expression has also Û a < x £ b or - b £ x < - a
positive sign, then wave start from right to left, i.e., x Î[ - b, - a) È ( a, b]
beginning above the number line, i.e.
8. For a2 £ x 2 < b2 or a £ |x| < b
( x - a 1 ) k 1 ( x - a 2 ) k 2 K( x - a m ) k m Û a £ x < b or - b < x £ - a
+ , then
( x - b 1 ) p 1 ( x - b 2 ) p 2 K( x - b n ) p n i.e., x Î( - b, - a] È [ a, b)
9. For a2 < x 2 < b2 or a < |x| < b

+
Û a < x < b or - b < x < - a
i.e., x Î( - b, - a) È ( a, b)
bn 10. For ( x - a)( x - b) < 0 and a < b, then a < x < b
i.e., x Î( a, b)
and if in all brackets before x positive sign and 11. If ( x - a) ( x - b) £ 0 and a < b,
expression has negative sign, then wave start from then a £ x £ b, x Î[ a, b]
right to left, beginning below the number line, i.e. 12. If ( x - a)( x - b) > 0 and a < b, then x < a or x > b
( x - a1 )k 1( x - a 2 )k 2 K( x - am )k m i.e., x Î( - ¥, a) È ( b, ¥)
- , then
( x - b1 )p 1 ( x - b 2 )p 2 K( x - bn )p n 13. If ( x - a) ( x - b) ³ 0 and a < b,
then x £ a or x ³ b
bn
i.e., x Î( - ¥, a] È [ b, ¥)

y Example 26. Solve the inequality
( x + 3)( 3x - 2) 5 (7 - x ) 3 ( 5x + 8 ) 2 ³ 0.
4. If roots occur even times, then sign remain same Sol. We have, ( x + 3)(3x - 2)5 (7 - x )3 (5x + 8)2 ³ 0
from right to left side of the roots and if roots Þ - ( x + 3)(3x - 2)5 ( x - 7 )3 (5x + 8)2 ³ 0
occur odd times, then sign will change from right to Þ ( x + 3) ( 3x - 2) 5 ( x - 7 ) 3 ( 5x + 8) 2 £ 0
left through the roots of
[take before x , + ve sign in all brackets]
x = a 1 , a 2 , K, a m , b 1 , b 2 , K, b n .
+ +
5. The solution of f ( x ) > 0 or f ( x ) ³ 0 is the union –3 +
7
of all intervals in which we have put the plus sign – –8 2 –
5 3
and the solution of f ( x ) < 0 or f ( x ) £ 0 is the
union of all intervals in which we have put the æ 8ö 2
The critical points are ( - 3), ç - ÷, , 7.
minus sign. è 5ø 3
é 2 ù ì 8ü
Hence, x Î ( - ¥, - 3] È ê , 7 ú È í - ý.
ë 3 û î 5þ
118 Textbook of Algebra

+ + +
y Example 27. Solve the inequality – –1
971
–2 –2 – –1
æ 1ö 3 2
( x - 2)10000 ( x + 1) 253 ç x - ÷ ( x + 8 ) 4 æ 2 1ö
è 2ø Hence, x Î ( - 2, - 1) È ç - , - ÷.
500
³0 è 3 2ø
x ( x - 3) 75 ( x + 2) 93
æ 1ö
971 y Example 30. For x ÎR, prove that the given
( x - 2)10000 ( x + 1)253 ç x - ÷ ( x + 8)4
è 2ø x 2 + 34 x - 71
Sol. We have, ³0 expression 2 cannot lie between 5 and 9.
500 75
x ( x - 3) ( x + 2)93 x + 2x - 7
1 x 2 + 34 x - 71
The critical points are ( - 8), ( - 2), ( - 1), 0, , 2, 3. Sol. Let =y
2 x 2 + 2x - 7
[Q x ¹ - 2, 0, 3 ] +
+
+ + + + 5 – 9
2 +
1 3
–8 –2 – –1 0 – – Þ x 2 (y - 1) + (2y - 34 )x + 71 - 7y = 0
2

æ 1ù For real values of x , discriminant ³ 0


Hence, x Î ( - ¥, - 8] È [ - 8, - 2) È [ - 1, 0) È ç0, ú È (3, ¥ )
è 2û \ (2y - 34 )2 - 4(y - 1)(71 - 7y ) ³ 0

æ 1ù Þ 8y 2 - 112y + 360 ³ 0
or x Î ( - ¥, - 2) È [ - 1, 0) È ç0, ú È ( 3, ¥ )
è 2û
Þ y 2 - 14y + 45 ³ 0
( x - 3)( x + 2)( x + 6 ) Þ (y - 9 )(y - 5) ³ 0
y Example 28. Let f ( x ) = .
( x + 1)( x - 5) Þ y Î ( - ¥, 5] È [9, ¥ )
Find intervals, where f ( x ) is positive or negative. Hence, y can never lie between 5 and 9.
( x - 3)( x + 2)( x + 6) y Example 31. For what values of the parameter k in
Sol. We have, f (x ) =
( x + 1)( x - 5) ½ x 2 + kx + 1 ½
The critical points are ( - 6), ( - 2), ( - 1), 3, 5 the inequality ½ 2 ½< 3, satisfied for all real
½ x + x+1 ½
+ + + values of x ?
– –6 –2 – –1 3 – 5 ½ x 2 + kx + 1 ½
Sol. We have, ½ ½< 3
2
½ x + x +1½
For f ( x ) > 0, " x Î ( - 6, - 2) È ( - 1, 3) È (5, ¥ )
x 2 + kx + 1
For f ( x ) < 0, " x Î ( - ¥, - 6) È ( - 2, - 1) È (3, 5) Þ -3< 2 <3
x + x +1
2
y Example 29. Find the set of all x for which æ 1ö 3
Since, x 2 + x + 1 = çx + ÷ + > 0
2x
>
1
. è 2ø 4
( 2x 2 + 5x + 2) ( x + 1) \ - 3( x 2 + x + 1) < x 2 + kx + 1 < 3( x 2 + x + 1)
2x 1 \ 4 x 2 + ( k + 3) x + 4 > 0
Sol. We have, > …(i)
( 2x + 5x + 2) ( x + 1)
2
and 2x 2 - ( k - 3) x + 2 > 0 …(ii)
2x 1 4 > 0 and 2 > 0
Þ - >0 Q
( x + 2)(2x + 1) ( x + 1) The inequality (i) will be valid, if
( 2x 2 + 2x ) - ( 2x 2 + 5x + 2) (k + 3)2 - 4 × 4 × 4 < 0 Þ (k + 3)2 < 64
Þ >0
( x + 2)( x + 1)(2x + 1) or -8<k +3<8
( 3x + 2) or - 11 < k < 5 …(iii)
Þ - >0 and the inequality (ii) will be valid, if
( x + 2)( x + 1)(2x + 1)
(k - 3)2 - 4 × 2 × 2 < 0 or (k - 3)2 < 16
( 3x + 2)
or <0 or - 4 <k -3< 4
( x + 2)( x + 1)(2x + 1) or -1<k <7 ...(iv)
æ 2ö æ 1ö The conditions (iii) and (iv) will hold simultaneously, if
The critical points are ( - 2), ( -1), ç - ÷, ç - ÷.
è 3ø è 2ø -1<k <5
Chap 02 Theory of Equations 119

Condition for Resolution y Example 34. Find the linear factors of


x 2 - 5xy + 4 y 2 + x + 2y - 2.
into Linear Factors Sol. Given expression is
The quadratic function x 2 - 5xy + 4y 2 + x + 2y - 2 …(i)
f ( x , y ) = ax 2 + 2hxy + by 2 + 2 gx + 2 fy + c Its corresponding equation is
may be resolved into two linear factors, iff x 2 - 5xy + 4y 2 + x + 2y - 2 = 0
D = abc + 2 fgh - af 2 - bg 2 - ch 2 = 0 or x 2 - x (5y - 1) + 4y 2 + 2y - 2 = 0
(5y - 1) ± (5y - 1)2 - 4 × 1 × ( 4y 2 + 2y - 2)
½a h g½ \ x=
2
i.e., ½h b f ½= 0
½ ½ (5y - 1) ± (9y 2 - 18y + 9 )
=
½g f c½ 2
(5y - 1) ± (3y - 3)2
y Example 32. Find the value of m for which the =
2
expression 12x 2 - 10xy + 2y 2 + 11x - 5y + m can be (5y - 1) ± (3y - 3)
= = 4y - 2, y + 1
resolved into two rational linear factors. 2
Sol. Comparing the given expression with \ The required linear factors are ( x - 4y + 2) and ( x - y - 1).
ax 2 + 2hxy + by 2 + 2gx + 2 fy + c , we have

a = 12, h = - 5, b = 2, g =
11
2
æ 5ö
, f = ç - ÷, c = m
è 2ø
Location of Roots
The given expression will have two linear factors, if and (Interval in which Roots Lie)
only if
Let f ( x ) = ax 2 + bx + c , a, b, c Î R, a ¹ 0 and a, b be the
abc + 2 fgh - af 2 - bg 2 - ch 2 = 0
2
roots of f ( x ) = 0. Suppose k, k 1 , k 2 Î R and k 1 < k 2 . Then,
æ 5 ö æ 11 ö æ 5ö the following hold good :
or (12)(2)(m ) + 2 ç - ÷ ç ÷ ( - 5) - (12) ç - ÷
è ø
2 2 è ø è 2ø
1. Conditions for Number k
2
æ 11 ö
- (2) ç ÷ - (m )( - 5)2 = 0
è2ø
275 121
(If both the roots of f ( x ) = 0 are less than k)
Þ 24 m + - 75 - - 25 m = 0 or m = 2
2 2 – b ,– D
a>0 2 a 4a
y Example 33. If the expression f (k) β k
– b α
ax 2 + by 2 + cz 2 + 2ayz + 2bzx + 2cxy can be resolved 2a
X-axis
X-axis – b
into two rational factors, prove that α β k 2a f (k )
a 3 + b 3 + c 3 = 3abc .
– b ,– D a<0
Sol. Given expression is 2a 4a

ax 2 + by 2 + cz 2 + 2ayz + 2bzx + 2cxy …(i)


(i) D ³ 0 (roots may be equal)
é æx ö 2
æy ö
2
æy ö æx ö æ x ö æy öù
2
= z êa ç ÷ + b ç ÷ + c + 2a ç ÷ + 2b ç ÷ + 2c ç ÷ ç ÷ ú (ii) af (k ) > 0
è
êë z ø è z ø è z ø è z ø è z ø è z øú
û b
= z 2 [aX 2 + bY 2 + c + 2aY + 2bX + 2cXY ] (iii) k > - , where a £ b.
2a
x y
where, = X and =Y
z z y Example 35. Find the values of m, for which both
Expression (i) will have two rational linear factors in x , y roots of equation x 2 - mx + 1 = 0 are less than unity.
and z, if expression
Sol. Let f ( x ) = x 2 - mx + 1, as both roots of f ( x ) = 0 are less
2 2
aX + bY + 2cXY + 2bX + 2aY + c will have two linear b
than 1, we can take D ³ 0, af (1) > 0 and - < 1.
factors, if 2a
+
abc + 2abc - aa 2 - bb 2 - cc 2 = 0 +
–2 – 2
or a 3 + b 3 + c 3 = 3abc
120 Textbook of Algebra

(i) Consider D ³ 0 ( - m )2 - 4 × 1 × 1 ³ 0 æ 11 ö
Þ çm - ÷ ( m - 1) > 0
è 9ø
Þ (m + 2)(m - 2) ³ 0
Þ m Î ( - ¥, - 2] È [2, ¥ ) …(i) æ 11 ö
Þ m Î ( - ¥, 1) È ç , ¥ ÷ …(ii)
(ii) Consider af (1) > 0 1(1 - m + 1) > 0 è9 ø

Þ m -2<0 Þ m <2 æ b ö
(iii) Consider ç - > 3÷
è 2a ø
Þ m Î ( - ¥, 2) …(ii)
æ b ö 6m
(iii) Consider ç - < 1÷ >3
è 2a ø 2
Þ m >1
m
< 1 Þm < 2 Þ m Î (1, ¥ ) …(iii)
2
Hence, the values of m satisfying Eqs. (i), (ii) and (iii)
Þ m Î ( - ¥, 2) …(iii)
æ 11 ö
Hence, the values of m satisfying Eqs. (i), (ii) and (iii) at the same time are m Î ç , ¥ ÷.
è9 ø
at the same time are m Î ( - ¥, - 2].

2. Conditions for a Number k 3. Conditions for a Number k


If both the roots of f ( x ) = 0 are greater than k If k lies between the roots of f ( x ) = 0
– b ,– D
– b ,– D 2a 4a
a>0 2a 4 a a>0
f(k) β
α
f (k ) k α β
– b X-axis X-axis
2a X-axis α
X-axis – b f (k ) β
k α β
f (k) 2a a<0

– b ,– D
– b ,– D a<0 2a 4a
2a 4 a

(i) D ³ 0 (roots may be equal) (i) D > 0 (ii) af (k ) < 0, where a < b
(ii) af (k ) > 0 y Example 37. Find all values of p, so that 6 lies
b between roots of the equation x 2 + 2(p - 3)x + 9 = 0.
(iii) k < - , where a £ b.
2a
Sol. Let f ( x ) = x 2 + 2( p - 3)x + 9, as 6 lies between the roots
y Example 36. For what values of m ÎR , both roots of of f ( x ) = 0, we can take D > 0 and af (6) < 0
the equation x 2 - 6mx + 9m 2 - 2m - 2 = 0 exceed 3? (i) Consider D > 0
+ +
Sol. Let f ( x ) = x 2 - 6mx + 9m 2 - 2 m + 2 –
0 6
As both roots of f ( x ) = 0 are greater than 3, we can take
2
b {2 ( p - 3)} - 4 × 1 × 9 > 0
D ³ 0, af (3) > 0 and- > 3.
2a
Þ ( p - 3) 2 - 9 > 0
(i) Consider D ³ 0
Þ p ( p - 6) > 0
( - 6m )2 - 4 × 1(9m 2 - 2m + 2) ³ 0 Þ 8m - 8 ³ 0 Þ p Î ( - ¥, 0) È (6, ¥ ) …(i)
\ m ³ 1 or m Î [1, ¥ ) …(i) (ii) Consider a f (6 ) < 0
1 × {36 + 12( p - 3) + 9 } < 0
(ii) Consider a f (3 ) ³ 0 3
Þ 12p + 9 < 0 Þ p + < 0
1 × (9 - 18m + 9m 2 - 2m + 2) > 0 4
æ 3ö
+ + Þ p Î ç - ¥, - ÷ …(ii)
1 – 11/9 è 4ø
Hence, the values of p satisfying Eqs. (i) and (ii) at the
Þ 9m 2 - 20m + 11 > 0
æ 3ö
same time are p Î ç - ¥, - ÷.
Þ (9m - 11)(m - 1) > 0 è 4ø
Chap 02 Theory of Equations 121

4. Conditions for Numbers k1 and k2 y Example 39. Find all values of a for which the
equation 4 x 2 - 2x + a = 0 has two roots lie in the
If exactly one root of f ( x ) = 0 lies in the interval (k 1 , k 2 )
interval ( - 1, 1).
k1 α f(k2) β Sol. Let f ( x ) = 4 x 2 - 2x + a as both roots of the equation,
a>0 k2 X-axis f ( x ) = 0 are lie between ( - 1, 1), we can take D ³ 0,
f(k1) 1
f (k 2 ) f(k1) af ( - 1) > 0, af (1) > 0 and - 1 < < 1.
a<0 4
k1 α k2 β X-axis
(i) Consider D ³ 0
1
(i) D > 0 ( - 2) 2 - 4 × 4 × a ³ 0 Þ a £ …(i)
4
(ii) f (k 1 ) f (k 2 ) < 0, where a < b.
(ii) Consider a f ( - 1 ) > 0
y Example 38. Find the values of m, for which exactly 4( 4 + 2 + a ) > 0
one root of the equation x 2 - 2mx + m 2 - 1 = 0 lies in Þ a > - 6 Þ a Î ( - 6, ¥ ) …(ii)
the interval ( - 2, 4 ). (iii) Consider a f (1 ) > 0
Sol. Let f ( x ) = x 2 - 2mx + m 2 - 1, as exactly one root of 4 ( 4 - 2 + a) > 0 Þ a > - 2
f ( x ) = 0 lies in the interval ( -2, 4 ), we can take D > 0 and Þ a Î ( - 2, ¥ ) …(iii)
f ( - 2) f ( 4 ) < 0.
Hence, the values of a satisfying Eqs. (i), (ii) and (iii) at
(i) Consider D > 0 æ 1ù
( - 2 m )2 - 4 × 1(m 2 - 1) > 0 Þ 4 > 0 the same time are a Î ç - 2, ú.
è 4û
\ m ÎR …(i)
(ii) Consider f ( - 2 ) f ( 4 ) < 0
6. Conditions for Numbers k1 and k2
( 4 + 4m + m 2 - 1) (16 - 8m + m 2 - 1) < 0
(If k 1 and k 2 lie between the roots of f ( x ) = 0)
Þ (m 2 + 4m + 3) (m 2 - 8m + 15) < 0
Þ (m + 1)(m + 3)(m - 3)(m - 5) < 0
Þ (m + 3)(m + 1)(m - 3)(m - 5) < 0 a>0 f(k2)
f(k1 )
+ + + k1 k2 α β
α X-axis X-axis
–3 – –1 3 – 5 β k1 k2
f(k1) f (k2)
\ m Î ( - 3 , - 1) È (3, 5) …(ii) a<0

Hence, the values of m satisfying Eqs. (i) and (ii) at the


same time are m Î ( - 3, - 1) È (3, 5).
(i) D > 0
5. Conditions for Numbers k1 and k2 (ii) af (k 1 ) < 0
(If both roots f ( x ) = 0 are confined between k 1 and k 2 ) (iii) af (k 2 ) < 0, where a < b.


b ,– D y Example 40. Find the values of a for which one
a>0 2a 4a
root of equation (a - 5)x 2 - 2ax + a - 4 = 0 is smaller
f (k1) f (k2) k1 a b k2
X-axis than 1 and the other greater than 2.
X-axis
k1 a b k2
f (k1) f(k2 ) Sol. The given equation can be written as

– b ,– D a<0 æ 2a ö æa - 4 ö
2 a 4a x2 - ç ÷x + ç ÷ = 0, a ¹ 5.
èa - 5ø èa - 5ø
(i) D ³ 0 (roots may be equal) æ 2a ö æa - 4 ö
Now, let f ( x ) = x 2 - ç ÷x + ç ÷
(ii) af (k 1 ) > 0 èa - 5ø èa - 5ø
(iii) af (k 2 ) > 0 As 1 and 2 lie between the roots of f ( x ) = 0, we can take
b D > 0, 1 × f (1) < 0 and 1 × f (2) < 0.
(iv) k 1 < - < k 2 , where a £ b and k 1 < k 2 .
2a
122 Textbook of Algebra

(i) Consider D > 0 i.e., (m - 1) (m - 9 ) ³ 0


æ æ 2a ö ö
2 \ m Î ( - ¥, 1] È [9, ¥ ) …(i)
æa - 4 ö
ç- ç ÷ ÷ - 4 ×1 × ç ÷ >0 f (2 ) > 0
è èa - 5øø èa - 5ø
æ 20 ö i.e., 4 - 2( m - 3) + m > 0
36 ça - ÷
è 9ø Þ m < 10
Þ >0 [Qa ¹ 5]
( a - 5) 2 \ m Î ( - ¥, 10) …(ii)
20 and x-coordinate of vertex < 2
or a> …(i)
9 ( m - 3)
i.e., <2 Þ m <7
(ii) Consider 1 × f (1 ) < 0 2
æ 2a ö æ a - 4 ö 9 \ m Î ( - ¥, 7 ) …(iii)
12 - ç ÷+ç ÷ <0Þ > 0 or a > 5 …(ii) On combining Eqs. (i), (ii) and (iii), we get
èa - 5ø è a - 5 ø ( a - 5)
m Î ( - ¥, 1]
(iii) Consider 1 × f (2 ) < 0
(ii) Both the roots are greater than 2
4a æa - 4ö
4- +ç ÷ <0 D³0
(a - 5 ) è a - 5 ø
( 4a - 20 - 4a + a - 4 ) (a - 24 )
Þ <0 Þ <0
(a - 5 ) (a - 5 ) f(2)
or 5 < a < 24 …(iii)
X-axis
Hence, the values of a satisfying Eqs. (i), (ii) and (iii) at the 2 α β
same time are a Î(5, 24 ).

y Example 41. Let x 2 - (m - 3)x + m = 0 (m ÎR ) be a i.e. (m - 1)(m - 9 ) ³ 0


quadratic equation. Find the value of m for which \ m Î ( - ¥, 1] Î [9, ¥ ) …(i)
(i) both the roots are smaller than 2. f (2 ) > 0
(ii) both the roots are greater than 2. i.e. 4 - 2( m - 3) + m > 0
Þ m < 10
(iii) one root is smaller than 2 and the other root is
greater than 2. \ m Î ( - ¥, 10) …(ii)
and x-coordinate of vertex > 2
(iv) exactly one root lies in the interval (1, 2).
( m - 3)
(v) both the roots lie in the interval (1, 2). i.e., >2 Þ m >7
2
(vi) one root is greater than 2 and the other root is \ m Î ( 7, ¥ ) …(iii)
smaller than 1.
On combining Eqs. (i), (ii) and (iii), we get
(vii) atleast one root lie in the interval (1, 2).
m Î[9, 10)
(viii) atleast one root is greater than 2.
(iii) One root is smaller than 2 and the other root is
Sol. Let f ( x ) = x 2 - ( m - 3) x + m greater than 2
Here, a = 1, b = - (m - 3), c = m D > 0
and D = b 2 - 4ac = (m - 3)2 - 4 m
= m 2 - 10m + 9 = (m - 1)(m - 9 )
b ( m - 3) 2
and x-coordinate of vertex = - = a X-axis
2a 2 f(2) b
(i) Both the roots are smaller than 2
D³0
i.e., (m - 1)(m - 9 ) > 0
\ m Î ( - ¥, 1) È (9, ¥ ) …(i)
f(2) f (2 ) < 0

X-axis i.e. 4 - 2(m - 3) + m < 0


a b 2
\ m > 10
Chap 02 Theory of Equations 123

\ m Î (10, ¥ ) …(ii) 1 < x -coordinate of vertex < 2


On combining Eqs. (i) and (ii), we get ( m - 3)
i.e., 1< <2
m Î (10, ¥ ). 2
Þ 2 < m - 3 < 4 or 5 < m < 7
(iv) Exactly one root lies in the interval (1, 2)
\ m Î(5, 7 ) …(iv)
D > 0
On combining Eqs. (i), (ii), (iii) and (iv), we get
m Îf
f (1)
(vi) One root is greater than 2 and the other root is
2
X-axis smaller than 1 D > 0
1a f (2) b

i.e., (m - 1)(m - 9 ) > 0


\ m Î ( - ¥, 1) È (9, ¥ ) …(i) 1 2
X-axis
f (1 ) f (2 ) < 0 α f(2) β
f(1)
( 1 - ( m - 3) + m ) ( 4 - 2( m - 3) + m ) < 0
Þ 4 ( - m + 10) < 0 i.e., (m - 1)(m - 9 ) > 0
Þ m - 10 > 0 Þ m > 10 \ m Î ( - ¥, 1) È (9, ¥ ) …(i)
\ m Î (10, ¥ ) …(ii) f (1 ) < 0
On combining Eqs. (i) and (ii), we get i.e., 4 < 0, which is not possible.
m Î (10, ¥ ) Thus, no such ‘m’ exists.
(v) Both the roots lie in the interval (1, 2) (vii) At least one root lie in the interval (1, 2)
D³ 0 Case I Exactly one root lies in (1, 2)
m Î (10, ¥ ) [from (iv) part]
Case II Both roots lie in the interval (1, 2).
f (1) f (2)
m Îf [from (v) part]
X-axis Hence, at least one root lie in the interval (1, 2)
1 α β2
m Î (10, ¥ ) È f or m Î (10, ¥ )
(viii) Atleast one root is greater than 2
i.e., (m - 1)(m - 9 ) ³ 0
\ m Î ( - ¥, 1] È [9, ¥ ) …(i)
Case I One root is smaller than 2 and the other root
is greater than 2.
f (1 ) > 0
i.e., (1 - (m - 3) + m ) > 0 Þ 4 > 0 Then, m Î (10, ¥ ) [from (iii) part]
\ m ÎR …(ii) Case II Both the roots are greater than 2, then
m Î[9, 10).
f (2 ) > 0
Hence, atleast one root is greater than 2.
i.e., 4 - 2(m - 3) + m > 0 Þ m < 10
\ m Î (10, ¥ ) È [9, 10) or m Î [9, 10) È (10, ¥ )
\ m Î ( - ¥, 10) …(iii)
124 Textbook of Algebra

#L Exercise for Session 3


x 2 + 14x + 9
1. If x is real, the maximum and minimum values of expression will be
x 2 + 2x + 3
(a) 4, - 5 (b) 5, - 4 (c) - 4, 5 (d) - 4, - 5
x +2
2. If x is real, the expression takes all values in the interval
(2x 2 + 3x + 6)

(a) æç ,

(b) é -

(c) æç - , ö÷
1 1 1 1
÷ êë 13 ,
(d) None of these
è 13 3ø 3 úû è 3 13 ø

3. If x be real, then the minimum value of x 2 - 8x + 17, is


(a) - 1 (b) 0 (c) 1 (d) 2
æ 1ö
4. If the expression çmx - 1 + ÷ is non-negative for all positive real x , the minimum value of m must be
è xø
1
(a) - (b) 0
2
1 1
(c) (d)
4 2
mx 2 + 3x + 4
5. If the inequality < 5 is satisfied for all x Î R then
x 2 + 2x + 2
(a) 1 < m < 5 (b) - 1 < m < 5
71
(c) 1 < m < 6 (d) m <
24
( x 2 - 1)
6. The largest negative integer which satisfies > 0, is
( x - 2)( x - 3)
(a) - 4 (b) - 3
(c) - 2 (d) - 1

7. If the expression 2x 2 + mxy + 3y 2 - 5y - 2 can be resolved into two rational factors, the value of | m | is
(a) 3 (b) 5
(c) 7 (d) 9

8. If c > 0 and 4a + c < 2b , then ax 2 - bx + c = 0 has a root in the interval


(a) (0, 2) (b) (2, 4)
(c) (0, 1) (d) (- 2, 0)

9. If the roots of the equation x 2 - 2ax + a 2 + a - 3 = 0 are less than 3 then


(a) a < 2 (b) 2 £ a £ 3
(c) 3 < a £ 4 (d) a > 4

10. The set of values of a for which the inequation x 2 + ax + a 2 + 6a < 0 is satisfied for all x Î(1, 2) lies in the
interval
(a) (1, 2) (b) [1, 2]
(c) [- 7, 4] (d) None of these
Session 4
Equations of Higher Degree, Rational Algebraic
Inequalities, Roots of Equation with the Help of Graphs,

Equations of Higher Degree and abg = ( - 1) 3


d
=-
d
a a
The equation a 0 x n + a 1 x n - 1 + a 2 x n - 2
or ax 3 + bx 2 + cx + d = a( x - a )( x - b)( x - g )
+ K+ a n - 1 x + a n = 0,
where a 0 , a 1 , a 2 , ... , a n - 1 , a n are constants but a 0 ¹ 0, is a = a( x 3 - S a × x 2 + S ab × x - abg )]
polynomial equation of degree n. It has n and only n roots.
(ii) For n = 4, if a, b, g, d are the roots of the equation
Let a 1 , a 2 , a 3 , K, a n - 1 , a n be n roots, then
ax 4 + bx 3 + cx 2 + dx + e = 0, where a, b, c , d , e are
a
l S a 1 = a 1 + a 2 + a 3 + K + a n - 1 + a n = (- 1 )1 1 constants and a ¹ 0, then
a0
b b
[sum of all roots] S a = a + b + g + d = ( - 1) 1 = - ,
l S a1 a 2 = a1a 2 + a1a 3 + K + a1an + a 2 a 3 + a a
K + a 2 an +K + an - 1 an c c
S ab = (a + b)( g + d) + ab + gd = ( - 1) 2 = ,
2 a2
a a
= ( - 1) [sum of products taken two at a time] d d
a0 S abg = ab( g + d) + g d (a + b) = ( - 1) 3 = -
a a
a
l S a 1 a 2 a 3 = (- 1 ) 3 3 e e
a0 and abgd = ( - 1) 4 =
a a
[sum of products taken three at a time]
or ax + bx + cx 2 + dx + e = a( x - a )
4 3
a
l a 1 a 2 a 3 K a n = (- 1 )n n [ product of all roots] ( x - b)( x - g )( x - d)
a0
ap = a( x 4 - S a × x 3 + S ab × x 2 - S abg × x + abgd)
In general, Sa 1 a 2 a 3 K a p = ( - 1) p
a0 y Example 42. Find the conditions, if roots of the
Remark equation x 3 - px 2 + qx - r = 0 are in
1. A polynomial equation of degree n has n roots (real or
imaginary). (i) AP (ii) GP
2. If all the coefficients, i.e., a0, a1, a2, K, an are real, then the
imaginary roots occur in pairs, i.e. number of imaginary roots (iii) HP
is always even. Sol. (i) Let roots of the given equation are
3. If the degree of a polynomial equation is odd, then atleast one A - D , A , A + D , then
of the roots will be real.
p
4. ( x - a1 )( x - a2 )( x - a3 ) K ( x - an ) A-D+A+A+D=p Þ A=
3
= x n + ( - 1)1 S a1 × x n - 1 + ( - 1) 2 S a1 a2 × x n - 2
Now, A is the roots of the given equation, then it must
+ K + ( - 1) n a1 a2 a3 K an
be satisfy
In Particular A 3 - pA 2 + qA - r = 0
(i) For n = 3, if a, b, g are the roots of the equation æp ö
3
æp ö
2
æp ö
Þ ç ÷ - pç ÷ + qç ÷ - r = 0
ax 3 + bx 2 + cx + d = 0, where a, b, c , d are constants è3ø è3ø è3ø
b b
and a ¹ 0, then Sa = a + b + g = ( - 1) 1 = - , Þ p 3 - 3p 3 + 9qp - 27r = 0
a a
or 2p 3 - 9 pq + 27r = 0,
c c
Sab = ab + bg + g a = ( - 1) 2 = which is the required condition.
a a
126 Textbook of Algebra

A Product of roots = (a - b ) ×a × (a + b ) = 6
(ii) Let roots of the given equation are , A , AR , then
R Þ ( 2 - b ) 2( 2 + b ) = 6 Þ 4 - b 2 = 3
A æ rö \ b = ±1
× A × AR = ( - 1)3 × ç - ÷ = r
R è 1ø \ Roots of Eqs. (i) are 1, 2, 3 or 3, 2, 1.
1 1 1 1
Þ A3 = r Hence, roots of the given equation are 1, , or , , 1.
2 3 3 2
1
Þ A =r3 y Example 44. If a , b, g are the roots of the equation
Now, A is the roots of the given equation, then x 3 - px 2 + qx - r = 0, find
A 3 - pA 2 + qA - r = 0
(i) åa 2 . (ii) åa 2 b. (iii) åa 3 .
Þ r - p (r )2 / 3 = q (r )1/ 3 - r = 0
Sol. Since, a , b, g are the roots of x 3 - px 2 + qx - r = 0.
or p (r )2 / 3 = q (r )1/ 3
\ å a = p , å a b = q and abg = r
or p 3r 2 = q 3r
(i) Q å a × å a = p × p
or p 3r = q 3
Þ (a + b + g )(a + b + g ) = p 2
which is the required condition.
Þ a 2 + b 2 + g 2 + 2 (ab + bg + ga ) = p 2
(iii) Given equation is
or å a 2 + 2 å ab = p 2
x 3 - px 2 + qx - r = 0 …(i)
1 or å a 2 = p 2 - 2q
On replacing x by in Eq. (i), then
x (ii) Q å a × å ab = p × q
æ1ö
3
æ1ö æ1ö
2 Þ (a + b + g ) × (ab + bg + ga ) = pq
ç ÷ - pç ÷ + qç ÷ - r = 0
èx ø èx ø èx ø Þ a 2 b + abg + a 2 g + b 2 a + b 2 g + abg
Þ rx 3 - qx 2 + px - 1 = 0 …(ii) + g 2 b + g 2 a = pq
Now, roots of Eq. (ii) are in AP. Þ (a 2 b + a 2 a + b 2 g + b 2 g + g 2 a + g 2 b )
Let roots of Eq. (ii) are A - P , A , A + P , then + 3abg = pq
q q or å a 2 b + 3r = pq
A-P+A+A+P= or A =
r 3r or å a 2 b = pq - 3r
Q A is a root of Eq. (ii), then (iii) Q å a 2 × å a = ( p 2 - 2q ) × p [from result (i)]
rA 3 - qA 2 + pA - 1 = 0 2 2 2 3
3 2 Þ (a + b + g )(a + b + g ) = p - 2pq
æq ö æq ö æq ö
Þ r ç ÷ - qç ÷ + pç ÷ - 1 = 0 Þ a 3 + b 3 + g 3 + (a 2 b + a 2 g + b 2 a + b 2 g
è 3r ø è 3r ø è 3r ø
Þ q 3 - 3q 3 + 9 pqr - 27r 2 = 0 + g 2 a + g 2 b ) = p 3 - 2pq
Þ 2q 3 - 9 pqr + 27r 2 = 0, Þ å a 3 + å a 2 b = p 3 - 2pq
which is the required condition. Þ å a 3 + pq - 3r = p 3 - 2pq [from result (ii)]
3 3
or å a = p - 3pq + 3r
y Example 43. Solve 6 x 3 - 11x 2 + 6 x - 1 = 0, if roots of
the equation are in HP. y Example 45. If a , b, g are the roots of the cubic
1 equation x 3 + qx + r = 0 , then find the equation whose
Sol. Put x = in the given equation, then
y
roots are (a - b ) 2 , (b - g ) 2 , ( g - a ) 2 .
6 11 6
3
- 2
+ -1=0 Sol. Qa , b, g are the roots of the cubic equation
y y y
x 3 + qx + r = 0 …(i)
Þ y 3 - 6y 2 + 11y - 6 = 0 …(i)
Then, å a = 0, å ab = q, abg = - r …(ii)
Now, roots of Eq. (i) are in AP.
If y is a root of the required equation, then
Let the roots be a - b, a , a + b.
Then, sum of roots = a - b + a + a + b = 6 y = (a - b )2 = (a + b )2 - 4 ab
Þ 3a = 6 4 abg
= (a + b + g - g )2 -
\ a =2 g
Chap 02 Theory of Equations 127

4r
= (0 - g )2 + [from Eq. (ii)] or
g
4r
If l is a root of the equation f ( x ) = 0, then f ( x ) is
Þ y = g2 + exactly divisible by ( x - l) and conversely, if f ( x ) is
g
exactly divisible by ( x - l), then l is a root of the
[replacing g by x which is a root of Eq. (i)]
equation f (x ) = 0 and the remainder obtained is f ( l).
4r
\ y = x2 +
x y Example 47. If x 2 + ax + 1 is a factor of
or x 3 - yx + 4r = 0 …(iii) ax 3 + bx + c , find the conditions.
The required equation is obtained by eliminating x between Sol. Qax 3 + bx + c = ( x 2 + ax + 1)Q (x )
Eqs. (i) and (iii).
Let Q ( x ) = Ax + B,
Now, subtracting Eq. (iii) from Eq. (i), we get
(q + y ) x - 3r = 0 then ax 3 + bx + c = ( x 2 + ax + 1)( Ax + B )
3r On comparing coefficients of x 3 , x 2 , x and constant on
or x=
q +y both sides, we get
On substituting the value of x in Eq. (i), we get a = A, …(i)
3 0 = B + aA, …(ii)
æ 3r ö æ 3r ö
ç ÷ + qç ÷ +r =0 b = aB + A , …(iii)
èq + y ø èq + y ø
and c =B …(iv)
Thus, y 3 + 6qy 2 + 9q 2y + ( 4q 3 + 27r 2 ) = 0 From Eqs. (i) and (iv), we get
which is the required equation. A = a and B = c
From Eqs. (ii) and (iii), a 2 + c = 0 and b = ac + a are the
Remark required conditions.
å( a - b ) 2 = - 6q, Õ( a - b ) 2 = - ( 4 q3 + 27r 2 )
y Example 48. A certain polynomial f ( x ), x ÎR, when
Some Results on Roots of a divided by x - a, x - b and x - c leaves remainders a, b
and c, respectively. Then, find the remainder when f ( x )
Polynomial Equation is divided by ( x - a )( x - b )( x - c ), where a, b , c are
1. Remainder Theorem If a polynomial f ( x ) is distinct.
divided by a linear function x - l, then the remainder Sol. By Remainder theorem f (a ) = a, f (b ) = b and f (c ) = c
is f ( l), Let the quotient be Q ( x ) and remainder is R( x ).
i.e. Dividend = Divisor ´ Quotient + Remainder \ f ( x ) = ( x - a )( x - b )( x - c )Q ( x ) + R( x )
Let Q ( x ) be the quotient and R be the remainder, thus \ f (a ) = 0 + R(a ) Þ R(a ) = a
f ( x ) = ( x - l) Q ( x ) + R f (b ) = 0 + R(b ) Þ R(b ) = b and f (c ) = 0 + R(c )
Þ R(c ) = c
Þ f ( l) = ( l - l) Q ( l) + R = 0 + R = R
So, the equation R( x ) - x = 0 has three roots a, b andc . But
y Example 46. If the expression 2x 3 + 3px 2 - 4 x + p its degree is atmost two. So, R( x ) - x must be zero
polynomial (or identity).
has a remainder of 5 when divided by x + 2, find the Hence, R( x ) = x .
value of p.
3. Every equation of an odd degree has atleast one real
Sol. Let f ( x ) = 2x 3 + 3px 2 - 4 x + p
root, whose sign is opposite to that of its last term,
Q f ( x ) = ( x + 2) Q ( x ) + 5 provided that the coefficient of the first term is
Þ f ( - 2) = 5 positive.
Þ 2( - 2)3 + 3p ( - 2)2 - 4( - 2) + p = 5 or 13p = 13 4. Every equation of an even degree has atleast two real
\ p =1 roots, one positive and one negative, whose last term
2. Factor Theorem Factor theorem is a special case of is negative, provided that the coefficient of the first
Remainder theorem. term is positive.
Let f (x ) = ( x - l) Q ( x ) + R = ( x - l) Q (x ) + f ( l) 5. If an equation has no odd powers of x, then all roots
If f ( l) = 0, f (x ) = ( x - l) Q (x ), therefore f (x ) is of the equation are complex provided all the
exactly divisible by x - l. coefficients of the equation have positive sign.
128 Textbook of Algebra

A (λ , f (λ))
6. If x = a is root repeated m times in f ( x ) = 0
( f ( x ) = 0 is an nth degree equation in x), then + B (µ , f (µ))
f ( x ) = ( x - a ) m g(x )
f (λ) +
where,g( x )isa polynomialofdegree(n - m ) and theroot + f (µ)
P (α, 0) (,β, 0) (γ, 0) (δ,0)
x = a isrepeated(m - 1) timein f ¢ ( x ) = 0,(m - 2 ) times – Q R – S (µ ,0)
X-axis
(λ, 0)
in f ¢¢( x ) = 0,…,(m - (m - 1)) timesin f m - 1 (x ) = 0.
7. Let f (x ) = 0 be a polynomial equation and l, m are (v)
two real numbers. +
+
Then, f (x ) = 0 will have atleast one real root or an (,λ0) (β, 0) (δ, 0) (µ, 0)
– (α, 0) – X-axis
odd number of roots between l and m, if f ( l) and (γ ,0)

f(λ) f(µ)
f (m ) are of opposite signs.
But if f ( l) and f (m ) are of same signs, then either A (λ , f (λ)) B ( µ , f ( µ))
(vi)
f ( x ) = 0 has no real roots or an even number of roots
between l and m. (a) In figure (i), (ii) and (iii), f ( l) and f (m ) have
opposite signs and equationf ( x ) = 0, has one,
three, five roots between l and m, respectively.
Illustration by Graphs (b) In figure (iv), (v) and (vi), f ( l) and f (m ) have
Since, f (x ) be a polynomial in x, then graph of y = f (x ) same signs and equation f ( x ) = 0, has no, four
will be continuous in every interval. and four roots between l and m, respectively.
A (λ ,f ( λ))
y Example 49. If a, b , c are real numbers, a ¹ 0. If a is
root of a 2 x 2 + bx + c = 0 , b is a root of
f (λ)
+ a 2 x 2 - bx - c = 0 and 0 < a < b, show that the
(µ,0)
equation a 2 x 2 + 2bx + 2c = 0 has a root g that always
(λ, 0) P (α, 0)
X-axis
satisfies a < g < b.
Sol. Since, a is a root of a 2 x 2 + bx + c = 0.
– f (µ)
Then, a 2 a 2 + ba + c = 0 …(i)
2 2
and b is a root of a x - bx - c = 0,
B (µ , f ( µ))
2 2
(i) then a b - bb - c = 0 …(ii)
A (λ ,f ( λ))
2 2
Let f ( x ) = a x + 2bx + 2c
\ f (a ) = a 2 a 2 + 2ba + 2c = a 2 a 2 - 2a 2 a 2
f (λ) +
[from Eq. (i)]
+ (µ, 0) = -a a 2 2
X-axis
(λ, 0) P (α, 0) Q R (γ, 0)
– (β,0) Þ f (a ) < 0 and f ( b ) = a 2 b 2 + 2b b + 2c
f ( µ)
– = a 2 b 2 + 2a 2 b 2 [from Eq. (ii)]
B [µ, f (µ)] = 3a b 2 2
(ii)

B (µ, f (µ)) Þ f (b) > 0


+
+
( λ ,0) Q R S + f(µ) Since, f (a ) and f ( b ) are of opposite signs, then it is clear
X-axis

f (λ) (α,0) (β, 0) – (γ, 0) (δ,0) – T (µ, 0) that a root g of the equation f ( x ) = 0 lies between a and b.
(ν, 0)
A (λ , f (λ)) (iii)
Hence, a < g <b [Qa < b]
B( µ , f (µ))
y Example 50. If a < b < c < d , then show that
f (λ) A [λ , f (λ)] +
f (µ) ( x - a )( x - c ) + 3 ( x - b )( x - d ) = 0 has real and distinct
X-axis roots.
(λ, 0) ( µ, 0)
(iv) Sol. Let f ( x ) = ( x - a )( x - c ) + 3( x - b )( x - d )
Chap 02 Theory of Equations 129

Then, f (a ) = 0 + 3(a - b )(a - d ) > 0 [Qa - b < 0, a - d < 0] 9. Rolle’s Theorem If f ( x ) is continuous function in
and f (b ) = (b - a )(b - c ) + 0 < 0 [Qb - a > 0, b - c < 0] the interval [a, b ] and differentiable in interval (a, b )
Thus, one root will lie between a and b. and f (a ) = f (b ), then equation f ¢ ( x ) = 0 will have
and f (c ) = 0 + 3(c - b )(c - d ) < 0 [Qc - b > 0, c - d < 0] atleast one root between a and b. Since, every
and f (d ) = (d - a )(d - c ) + 0 > 0 [Qd - a >0, d - c >0] polynomial f ( x ) is always continuous and
Thus, one root will lie between c and d. Hence, roots of differentiable in every interval. Therefore, Rolle’s
equation are real and distinct. theorem is always applicable to polynomial function
8. Let f ( x ) = 0 be a polynomial equation then in every interval [a, b ] if f (a ) = f (b ).
(a) the number of positive roots of a polynomial y Example 51. If 2a + 3b + 6c = 0 ; a, b , c ÎR, then show
equation f (x ) = 0 (arranged in decreasing order
of the degree) cannot exceed the number of that the equation ax 2 + bx + c = 0 has atleast one root
changes of signs in f ( x ) = 0 as we move from left between 0 and 1.
to right. Sol. Given, 2a + 3b + 6c = 0
For example, Consider the equation a b
2 x 3 - x 2 - x + 1 = 0. Þ + +c =0 …(i)
3 2
The number of changes of signs from left to right Let f ¢ ( x ) = ax 2 + bx + c ,
is 2 (+ to -, then - to +). Then, number of positive ax 3 bx 2
roots cannot exceed 2. Then, f (x) = + + cx + d
3 2
(b) The number of negative roots of a polynomial a b
equation f ( x ) = 0 cannot exceed the number of Now, f (0) = d and f (1) = + + c + d
3 2
changes of signs in f (-x ). =0+d [from Eq. (i)]
For example, Consider the equation Since, f ( x ) is a polynomial of three degree, then f ( x ) is
5x 4 + 3x 3 - 2x 2 + 5x - 8 = 0 continuous and differentiable everywhere and f (0) = f (1),
Let f ( x ) = 4 x 4 + 3 x 3 - 2 x 2 + 5 x - 8 then by Rolle’s theorem f ¢ ( x ) = 0 i.e., ax 2 + bx + c = 0 has
\ f ( - x ) = 5x 4 - 3x 3 - 2x 2 - 5x - 8 atleast one real root between 0 and 1.
The number of changes of signs from left to right
is (+ to -). Then number of negative roots cannot Reciprocal Equation of the Standard
exceed 1.
Form can be Reduced to an Equation
(c) If equation f ( x ) = 0 have atmost r positive roots
and atmost t negative roots, then equation of Half Its Dimensions
f ( x ) = 0 will have atmost (r + t ) real roots, i.e. it Let the equation be
will have atleast n - (r + t ) imaginary roots, ax 2m + bx 2m - 1 + cx 2m - 2 + K + kx m + K + cx 2 + bx + a = 0
where n is the degree of polynomial.
For example, Consider the equation On dividing by x m , then
c
5x 6 - 8x 3 + 3x 5 + 5x 2 + 8 = 0 ax m + bx m - 1 + cx m - 2 + K + k + K +
m -2
x
The given equation can be written as b a
5x 6 + 3x 5 - 8x 3 + 5x 2 + 8 = 0 + + =0
m -1
x xm
On rearranging the terms, we have
Let f ( x ) = 5 x 6 + 3 x 5 - 8 x 3 + 5 x 2 + 8
æ 1 ö æ 1 ö
Here, f ( x ) has two changes in signs. a çx m + + bçx m -1 + +c
è m ÷
ø è m -1 ÷
ø
So, f ( x ) has atmost two positive real roots x x
and f ( - x ) = 5 x 6 - 3 x 5 + 8 x 3 + 5 x 2 + 8 æ m -2 1 ö
çx + ÷ +K + k = 0
Here, f ( - x ) has two changes in signs. è x -2 ø
m

So, f ( x ) has atmost two negative real roots. 1 æ 1 öæ 1ö


Now, x p + 1 + = çx p + ÷ çx + ÷
and x = 0 cannot be root of f ( x ) = 0. x p +1 è x øè
p xø
Hence, f ( x ) = 0 has atmost four real roots, æ 1 ö
- çx p -1 + ÷
therefore atleast two imaginary roots. è x -1 ø
p
130 Textbook of Algebra

1 y Example 53. Solve the equation


Hence, writing z for x + and given to p succession the
x (12x - 1)(6 x - 1)(4 x - 1)( 3x - 1) = 5.
values 1, 2, 3,K, we obtain Sol. The given equation can be written as
1 æ 1 öæ 1öæ 1 öæ 1ö 5
x2 + =z2 -2 çx - ÷çx - ÷çx - ÷çx - ÷ = ...(i)
x 2 è 12 ø è 6 ø è 4 ø è 3 ø 12 × 6 × 4 × 3
3 1 1 1 1 1 1 1 1 1
x + = z (z 2 - 2 ) - z = z 3 - 3z Since, < < < and - = -
3 12 6 4 3 6 12 3 4
x
4 1 We can introduced a new variable,
x + = z (z 3 - 3z ) - (z 2 - 2 ) = z 4 - 4z 2 + 2
x 4 1 éæ 1ö æ 1ö æ 1ö æ 1öù
y = êçx - ÷ + çx - ÷ + çx - ÷ + çx - ÷
1 4ë è 12 ø è 6 ø è 4ø è 3 ø úû
and so on and generally x m + is of m dimensions in 5
xm y=x-
z and therefore the equation in z is of m dimensions. 24
5
On substituting x = y + in Eq. (i), we get
y Example 52. Solve the equation 24
2x 4 + x 3 - 11x 2 + x + 2 = 0. æ 3 öæ 1 öæ
çy + ÷ çy + ÷ çy
1 öæ
- ÷ çy - ÷ =
3ö 5
è 24 ø è 24 ø è 24 ø è 24 ø 12 × 6× 4 × 3
Sol. Since, x = 0 is not a solution of the given equation.
é 2 æ 1 ö2 ù é 2 æ 3 ö2 ù 5
On dividing by x 2 in both sides of the given equation, we Þ êy - ç ÷ ú êy - ç ÷ ú =
get êë è 24 ø úû êë è 24 ø úû 12 × 6 × 4 ×3
æ 1 ö æ 1ö
2 ç x 2 + 2 ÷ + ç x + ÷ - 11 = 0 …(i) Hence, we find that
è x ø è xø 49
1 y2 = 2
Put x + = y in Eq. (i), then Eq. (i) reduce in the form 24
x 7 7
2(y 2 - 2) + y - 11 = 0 i.e. y1 = and y 2 = -
24 24
Þ 2y 2 + y - 15 = 0
Hence, the corresponding roots of the original equation are
5 1 1
\ y1 = - 3 and y 2 = - and .
2 12 2
Consequently, the original equation is equivalent to the
collection of equations Type II An equation of the form
ì 1 ( x - a )( x - b )( x - c )( x - d ) = Ax 2
ïx + x = - 3
í , where, ab = cd can be reduced to a collection of two
ïx + 1 = 5 ab
î x 2 quadratic equations by a change of variable y = x + .
x
-3- 5 -3+ 5 1
we find that, x 1 = , x2 = , x3 = , x4 = 2
2 2 2 y Example 54. Solve the equation
( x + 2)( x + 3)( x + 8 )( x + 12) = 4 x 2 .
Equations which can be Reduced Sol. Since, ( - 2)( - 12) = ( - 3)( - 8), so we can write given equa-
to Linear, Quadratic and Biquadratic tion as
( x + 2)( x + 12)( x + 3)( x + 8) = 4 x 2
Equations
Þ ( x 2 + 14 x + 24 )( x 2 + 11x + 24 ) = 4 x 2 …(i)
Type I An equation of the form
Now, x = 0 is not a root of given equation.
( x - a )( x - b )( x - c )( x - d ) = A
On dividing by x 2 in both sides of Eq. (i), we get
where, a < b < c < d , b - a = d - c , can be solved by a
æ 24 öæ 24 ö
change of variable. çx + + 14 ÷ ç x + + 11÷ = 4 …(ii)
è x øè x ø
(x - a ) + (x - b ) + (x - c ) + (x - d )
i.e. y= Put x +
24
= y , then Eq. (ii) can be reduced in the form
4 x
(a + b + c + d ) (y + 14 )(y + 11) = 4 or y 2 + 25y + 150 = 0
y =x -
4 \ y1 = - 15 and y 2 = - 10
Chap 02 Theory of Equations 131

Thus, the original equation is equivalent to the collection of ìP ( x ) > 0, Q ( x ) > 0


P(x ) ï
> 0 Þ {P ( x ) Q ( x ) > 0 Þ í
equations
(1) or
é 24 Q(x )
ê x + x = - 15, ïP ( x ) < 0, Q ( x ) < 0
î
ê
ê x + 24 = - 10, ìP ( x ) > 0, Q ( x ) < 0
ë P(x ) ï
< 0 Þ {P ( x ) Q ( x ) < 0 Þ í
x
(2) or
é x 2 + 15x + 24 = 0 Q(x )
i.e. ê 2 ïP ( x ) < 0, Q ( x ) > 0
î
êë x + 10x + 24 = 0
On solving these collection, we get ìP ( x ) ³ 0, Q ( x ) > 0
P(x ) ìP ( x ) Q ( x ) ³ 0 ï
- 15 - 129 - 15 + 129 (3) ³ 0 Þí Þí or
x1 = , x2 = , x 3 = - 6, x 4 = - 4 Q(x ) î Q(x ) ¹ 0 ïP ( x ) £ 0, Q ( x ) < 0
2 2 î
Type III An equation of the form ( x - a ) 4 + ( x - b ) 4 = A ìP ( x ) ³ 0, Q ( x ) < 0
P(x ) ìP ( x ) Q ( x ) £ 0 ï
can also be solved by a change of variable, i.e. making a (4) £ 0 Þí Þí or
(x - a ) + (x - b ) Q(x ) î Q(x ) ¹ 0 ïP ( x ) £ 0, Q ( x ) > 0
substitution y = . î
2

y Example 55. Solve the equation y Example 56. Find all values of a for which the set
(6 - x ) 4 + (8 - x ) 4 = 16. of all solutions of the system
ì x 2 + ax - 2
Sol. After a change of variable, ï <2
(6 - x ) + (8 - x ) ï x2 - x +1
y= í 2
ï x + ax - 2 > - 3
2
\ y = 7 - x or x = 7 - y
ïî x 2 - x + 1
Now, put x = 7 - y in given equation, we get
(y - 1)4 + (y + 1)4 = 16 is the entire number line.
Sol. The system is equivalent to
Þ y 4 + 6y 2 - 7 = 0
ì x 2 - ( a + 2) x + 4
Þ (y 2 + 7 )(y 2 - 1) = 0 ï >0
ï x2 - x + 1
í 2
y2 + 7 ¹ 0 ï 4 x + ( a - 3) x + 1 > 0
[y gives imaginary values] ïî x2 - x + 1
\ y2 - 1 = 0 æ 1ö
2
3
Since, x 2 - x + 1 = ç x - ÷ + > 0, this system is
Then, y1 = - 1 and y 2 = 1 è 2ø 4
Thus, x 1 = 8 and x 2 = 6 are the roots of the given equation. ì x - ( a + 2) x + 4 > 0
2
equivalent to í 2
î 4 x + ( a - 3) x + 1 > 0
Rational Algebraic Inequalities Hence, the discriminants of the both equations of this
system are negative.
Consider the following types of rational algebraic ì(a + 2)2 - 16 < 0
i.e., í Þ (a + 6)(a - 2) < 0
inequalities 2
î(a - 3) - 16 < 0
P(x ) P(x )
> 0, < 0, +

+
Q(x ) Q(x ) –6 2

P(x ) P(x ) i.e., x Î( - 6, 2) …(i)


³ 0, £0
Q(x ) Q(x ) Þ (a + 1)(a - 7 ) < 0
+ +
If P ( x ) and Q ( x ) can be resolved in linear factors, then use –1 – 7
Wavy curve method, otherwise we use the following
statements for solving inequalities of this kind. i.e. x Î ( -1, 7 ) …(ii)
Hence, from Eqs. (i) and (ii), we get
x Î ( -1, 2)
132 Textbook of Algebra

Equations Containing The first system of collection Eq. (i) is equivalent to the
system of collection
Absolute Values ì 2x - 4 - 2x = 4, if 2x ³ 4
í
By definition, | x | = x , if x ³ 0 | x | = - x , if x < 0 î - 2x + 4 - 2x = 4, if 2x < 4
ì - 4 = 4, if x ³ 2
y Example 57. Solve the equation x 2 - 5| x | + 6 = 0. Þ í
î - 4 x = 0, if x < 2
Sol. The given equation is equivalent to the collection of
systems The first system is failed and second system gives x = 0.
ì x 2 - 5x + 6 = 0, if x ³ 0 ì( x - 2)( x - 3) = 0, if x ³ 0 Hence, x = 0 is unique solution of the given equation.
í 2 Þ í
î x + 5x + 6 = 0, if x < 0 î( x + 2)( x + 3) = 0, if x < 0
Hence, the solutions of the given equation are Important Forms Containing
x 1 = 2, x 2 = 3, x 3 = - 2, x 4 = - 3 Absolute Values
y Example 58. Solve the equation Form 1 The equation of the form
| f ( x ) + g ( x )| = | f ( x )| + | g ( x )|
½ x 2 - 8 x + 12 ½ x 2 - 8 x + 12
½ 2 ½= - 2 . is equivalent of the system
½ x - 10x + 21½ x - 10x + 21
f ( x ) g ( x ) ³ 0.
Sol. This equation has the form | f ( x )| = - f ( x )
x 2 - 8x + 12
y Example 60. Solve the equation
when, f ( x ) = ½ 2
x 2 - 10x + 21 ½ x ½ ½+ | x | = x .
such an equation is equivalent to the collection of systems ½ x - 1½ | x - 1|
ì f ( x ) = - f ( x ), if f ( x ) ³ 0 Sol. Let f ( x ) =
x
and g ( x ) = x ,
í x -1
î f ( x ) = f ( x ), if f ( x ) < 0
The first system is equivalent to f ( x ) = 0 and the second x x2
Then, f (x ) + g(x ) = +x=
system is equivalent to f ( x ) < 0 the combining both x -1 x -1
systems, we get
\ The given equation can be reduced in the form
f (x ) £ 0
x 2 - 8x + 12 | f ( x )| + | g ( x )| = | f ( x ) + g ( x )|
\ £0
x 2 - 10x + 21 Hence, f (x )× g(x ) ³ 0
( x - 2)( x - 6) x2
Þ £0 Þ ³0
( x - 3)( x - 7 ) x -1

+
+ + +
– – – 0 – 1
2 3 6 7

From Wavy curve method, x Î (1, ¥ ) È {0}.


Hence, by Wavy curve method,
x Î [2, 3) È [6, 7 )
Form 2 The equation of the form
y Example 59. Solve the equation | f 1 ( x )| + | f 2 ( x )| + K + | f n ( x )| = g ( x ) …(i)
| x - | 4 - x || - 2x = 4.
where, f 1 ( x ), f 2 ( x ), K, f n ( x ), g( x ) are functions of x and
Sol. This equation is equivalent to the collection of systems
g( x ) may be constant.
ì| x - ( 4 - x )| - 2x = 4, if 4 - x ³ 0
í Equations of this form solved by the method of
î| x + ( 4 - x )| - 2x = 4, if 4 - x < 0 intervals. We first find all critical points of
ì| 2x - 4 | - 2x = 4, if x £ 4 f 1 ( x ), f 2 ( x ), ..., f n ( x ), if coefficient of x is positive, then
Þ í …(i)
î 4 - 2 x = 4 , if x>4 graph start with positive sign (+) and if coefficient of x is
The second system of this collection negative, then graph start with negative sign (-). Then,
gives x =0 using the definition of the absolute value, we pass from
but x>4 Eq. (i) to a collection of systems which do not contain the
Hence, second system has no solution. absolute value symbols.
Chap 02 Theory of Equations 133

x -1
y Example 61. Solve the equation For £ 0, if x ³ 0
x +1
| x - 1| + | 7 - x | + 2| x - 2| = 4.
+ +
Sol. Here, critical points are 1, 2, 7 using the method of inter- 1
–1 –
vals, we find intervals when the expressions x - 1, 7 - x
and x - 2 are of constant signs. \ 0£ x £1 …(i)
i.e. x < 1, 1 < x < 2, 2 < x < 7, x > 7 x +1
For £ 0, if x < 0
+
x -1
+ +
(x – 1)
– 1 + +
(7 – x) + + + –1 – 1
7 –
\ -1£ x <0 …(ii)
( x – 2) + + Hence, from Eqs. (i) and (ii), the solution of the given
– – 2 equation is x Î [ - 1, 1].
Thus, the given equation is equivalent to the collection of Aliter
four systems, ½1 - | x | ½
½ ½³ 1 Þ ½½ 1 ½ ½³ 1
é ìx < 1 é ìx < 1 ½ 1 + | x | ½ 2 ½ 1 + | x |½ 2
ê í - ( x - 1) + (7 - x ) - 2( x - 2) = 4 ê íx = 2 1 1
ê î êî Þ ³ Þ 1 + | x | £ 2 or | x | £ 1
ê ì1 £ x < 2 ê ì1 £ x < 2 1 + |x | 2
ê íî( x - 1) + (7 - x ) - 2( x - 2) = 4 êíx = 3 \ - 1 £ x £ 1 or x Þ [ -1, 1]
ê Þ êî
ê ìí2 £ x < 7 êìí2 £ x < 7
ê î( x - 1) + (7 - x ) + 2( x - 2) = 4
ê ìx ³ 7
êî x = 1
êì x ³ 7
Equations Involving Greatest Integer,
ê í êí Least Integer and Fractional Part
êë î( x - 1) - (7 - x ) + 2(x - 2) = 4 êëî x = 4
From the collection of four systems, the given equation has 1. Greatest Integer
no solution. [x] denotes the greatest integer less than or equal to x i.e.,
[ x ] £ x . It is also known as floor of x.
Inequations Containing Thus, [3.5779] = 3, [0.89] = 0, [3 ] = 3
Absolute Values [ - 8.7285] = - 9
| x | < a Þ - a < x < a (a > 0 )
By definition, [ - 0.6] = - 1
|x | £a Þ-a £ x £a [ - 7] = - 7
| x | > a Þ x < - a and x > a
In general, if n is an integer and x is any real number
and | x | ³ a Þ x £ - a and x ³ a. between n and n + 1.
½ |x| ½ 1 n £ x < n + 1, then [ x ] = n
y Example 62. Solve the inequation ½1 - ½³ . i.e.
½ 1 + | x |½ 2
Sol. The given inequation is equivalent to the collection of Properties of Greatest Integer
systems (i) [ x ± n ] = [ x ] ± n, n Î I
ì½ x ½ ì 1 (ii) [ - x ] = - [ x ], x Î I
½ ½ ³ 1 , if x ³ 0 1
ï 1- ïï | 1 + x | ³ 2 , if x ³ 0
ï½ 1 + x½ 2 (iii) [ - x ] = - 1 - [ x ], x Ï I
í Þ í
1 1
ï½½1 + x ½ ½ ³ 1 , if x < 0 ï ³ , if x < 0 (iv) [ x ] - [ - x ] = 2n, if x = n, n Î I
ïî½ 1 - x½ 2 ï
î |1 - x | 2
(v) [ x ] - [ - x ] = 2n + 1, if x = n + { x }, n Î I and 0 < { x } < 1
ì 1 1 ì1- x
ïï 1 + x ³ 2 , if x ³ 0 ïï 1 + x ³ 0, if x ³ 0 (vi) [ x ] ³ n Þ x ³ n, n Î I
Þ í Þ í (vii) [ x ] > n Þ x ³ n + 1, n Î I
1 1 1+ x
ï ³ , if x < 0 ï ³ 0, if x < 0
ïî 1 - x 2 ïî 1 - x (viii)[ x ] £ n Þ x < n + 1, n Î I
ìx -1 (ix) [ x ] < n Þ x < n, n Î I
ïï x + 1 £ 0, if x ³ 0
Þ íx +1 (x) n2 £ [ x ] £ n1 Þ n2 £ x < n1 + 1, n1 , n2 Î I
ï £ 0, if x < 0 (xi) [ x + y ] ³ [ x ] + [y ]
ïî x - 1
134 Textbook of Algebra

é[ x ]ù é x ù
(xii) ê ú = ê ú , n Î N 2. Least Integer
ë n û ënû
( x ) or éx ù denotes the least integer greater than or equal
é n + 1ù é n + 2 ù é n + 4 ù é n + 8 ù
(xiii) ê ú +ê ú +ê ú +ê ú + K = n, n Î N to x i.e., ( x ) ³ x or éx ù ³ x . It is also known as ceilling
ë 2 û ë 4 û ë 8 û ë 16 û
of x.
é 1ù é 2ù é n - 1ù
(xiv)[ x ] + ê x + ú + ê x + ú + K + ê x + = [nx ], Thus, (3.578 ) = 4 , (0.87 ) = 1,
ë nû ë nû ë n úû
n ÎN ( 4) = 4
Graph of y = [x ]
Y
é- 8.239ù = - 8, é- 0.7ù = 0
3 In general, if n is an integer and x is any real number
2
between n and n + 1
i.e., n < x £ n + 1, then ( x ) = n + 1
1
[x] = n
X′ –3 –2 –1 X x=[x] = n+1
1 2 3 4
–1

–2 n n+1
x
–3
Y′
Relation between Greatest Integer and Least Integer
Remark ì[ x ], x ÎI
(x ) = í
Domain and Range of [ x ] are R and I, respectively. î [ x ] + 1, x Ï I
y Example 63. If [ x ] denotes the integral part of x for i.e. If x Î I , then x = [ x ] = ( x ) . [remember]
real x, then find the value of
Remark
é 1ù é 1 1 ù é1 1 ù é1 3 ù
êë 4 úû + êë 4 + 200úû + êë 4 + 100úû + êë 4 + 200úû
If ( x ) = n, then ( n - 1) < x £ n

é 1 199 ù Graph of y = ( x ) = éxù


+ K+ ê + .
ë 4 200úû Y

Sol. The given expression can be written as sin (–1)


é1ù é1 1 ù é1 2 ù é1 3 ù 3
êë 4 úû + êë 4 + 200 úû + êë 4 + 200 úû + êë 4 + 200 úû
2
é 1 199 ù
+K+ ê +
ë 4 200 úû
1
–2 –1
é 1ù X′ X
= ê200 × ú = [50] = 50 [from property (xiv)] 0 1 2 3
ë 4û

Y′
y Example 64. Let [a ] denotes the larger integer not
exceeding the real number a. If x and y satisfy the
Remark
equations y = 2 [ x ] + 3 and y = 3 [ x - 2] Domain and Range of ( x ) are R and [ x ] + 1, respectively.
simulaneously, determine [ x + y ].
Sol. We have, y = 2[ x ] + 3 = 3 [ x - 2] …(i) y Example 65. If [ x ] and ( x ) are the integral part of
Þ 2[ x ] + 3 = 3([ x ] - 2) [from property (i)] x and nearest integer to x, then solve ( x )[ x ] = 1.
Þ 2[ x ] + 3 = 3[ x ] - 6 Sol. Case I If x Î I , then x = [ x ] = ( x )
Þ [x ] = 9 \ Given equation convert in x 2 = 1.
From Eq. (i), y = 2 ´ 9 + 3 = 21
\ x = ( ± 1)
\ [ x + y ] = [ x + 21] = [ x ] + 21 = 9 + 21 = 30
Case II If x Ï I , then ( x ) = [ x ] + 1
Hence, the value of [ x + y ] is 30.
Chap 02 Theory of Equations 135

\Given equation convert in y Example 67. If { x } and [ x ] represent fractional and


([ x ] + 1 )[ x ] = 1 Þ [ x ]2 + [ x ] - 1 = 0 integral part of x respectively, find the value of
-1± 5
or [x ] = [impossible] 2000
{x + r }
2 [x ] + å 2000
.
Then, final answer is x = ± 1. r =1
2000
{x + r } 2000
{x }
y Example 66. Find the solution set of Sol. [ x ] + å 2000
= [x ] + å
2000
[from property (i)]
( x ) 2 + ( x + 1) 2 = 25, where ( x ) is the least integer r =1 r =1

greater than or equal to x . { x } 2000 {x }


= [x ] + å
2000 r = 1
1 = [x ] +
2000
´ 2000 = [ x ] + { x } = x
Sol. Case I If x Î I , then x = ( x ) = [ x ]
Then, ( x )2 + ( x + 1)2 = 25 reduces to
2
y Example 68. If { x } and [ x ] represent fractional
x 2 + x + 1 = 25 Þ 2x 2 + 2x - 24 = 0
and integral part of x respectively, then solve the
Þ x 2 + x - 12 = 0 Þ ( x + 4 )( x - 3) = 0
equation x - 1 = ( x - [ x ])( x - { x }).
\ x = - 4, 3 …(i)
Case II If x Ï I , then ( x ) = [ x ] + 1 Sol. Q x = [ x ] + { x }, 0 £ { x } < 1
Then, ( x )2 + ( x + 1)2 = 25 reduces to Thus, given equation reduces to
{[ x ] + 1} 2 + {[ x + 1] + 1} 2 = 25 [ x ] + { x } - 1 = { x }[ x ]
Þ {[ x ] + 1} 2 + {[ x ] + 2} 2 = 25 Þ { x }[ x ] - [ x ] - { x } + 1 = 0
Þ 2[ x ]2 + 6[ x ] - 20 = 0 Þ ([ x ] - 1)({ x } - 1) = 0
Þ 2
[ x ] + 3[ x ] - 10 = 0 Now, {x } - 1 ¹ 0 [Q0 £ { x } < 1]
Þ {[ x ] + 5}{[ x ] - 2} = 0 \ [x ] - 1 = 0
\ [ x ] = - 5 and [ x ] = 2 Þ [x ] = 1
Þ x Î [ - 5, - 4 ) È [2, 3) \ x Î[1, 2 )
Q x Ï I,
\ x Î ( - 5, - 4 ) È (2, 3) …(ii) Problem Solving Cycle
On combining Eqs. (i) and (ii), we get If a problem has x , | x |,[ x ], ( x ), { x }, then first solve | x |,
x Î ( - 5, - 4 ] È (2, 3] then problem convert in x ,[ x ], ( x ), { x }.

3. Fractional Part x, | x |, [x], (x), {x}

{ x } denotes the fractional part of x, i.e.0 £ { x } < 1. x = [x] + {x} x, [x], (x), {x}
Thus, {2 × 7} = 0.7, {5 } = 0, { - 3.72} = 0.28
If x is a real number, then x = [ x ] + { x }
i.e., x = n + f , where n Î I and 0 £ f < 1
Properties of Fractional Part of x
[x], {x}
(i) { x ± n } = { x }, n Î I (ii) If 0 £ x < 1, then { x } = x x, [x], {x}
Graph of y = {x }
Y
1 ì[ x ], x ÎI
Secondly, solve ( x ) = í
î[ x ] + 1 , x Ï I
X′ X Then, problem convert in x ,[ x ], { x }.
–3 –2 –1 0 1 2 3 4
Y′ Now, put x = [ x ] + { x }
Remark Then, problem convert in [ x ] and { x }. …(i)
1. For proper fraction 0 < {x } < 1. Since, 0 £ { x } < 1, then we get [ x ]
2. Domain and range of { x } are R and [ 0, 1), respectively.
From Eq. (i), we get {x}
3. {- 5238
. } = {- 5 - 0.238} = {- 5 - 1 + 1 - 0.238}
= { - 6 + 0.762} = {6.762} = 0.762 Hence, final solution is x = [ x ] + { x }.
136 Textbook of Algebra

y Example 69. Let { x } and [ x ] denotes the fractional y Example 71. Solve the equation
and integral parts of a real number x, respectively. ( x ) 2 = [ x ] 2 + 2x
Solve 4{ x } = x + [ x ]. where, [ x ] and ( x ) are integers just less than or equal
Sol. Q x = [x ] + {x } …(i) to x and just greater than or equal to x, respectively.
Then, given equation reduces to Sol. Case I If x Î I then
4 {x } = [x ] + {x } + [x ] x = [x ] = ( x )
2 The given equation reduces to
Þ {x } = [x ] …(ii)
3 x 2 = x 2 + 2x
2 3 Þ 2x = 0 or x = 0 …(i)
Q 0 £ { x } < 1 Þ 0 £ [ x ] < 1 or 0 £ [ x ] <
3 2 Case II If x Ï I , then ( x ) = [ x ] + 1
\ [ x ] = 0, 1 The given equation reduces to
2
From Eq. (ii), { x } = 0, ([ x ] + 1)2 = [ x ]2 + 2x
3 1
2 5 Þ 1 = 2( x - [ x ]) or { x } =
From Eq. (i), x = 0, 1 + i.e., x = 0, 2
3 3 1 1
\ x = [x ] += n + ,n ÎI …(ii)
2 2
y Example 70. Let { x } and [ x ] denotes the fractional 1
Hence, the solution of the original equation is x = 0, n + ,
and integral part of a real number ( x ), respectively. n Î I. 2
Solve | 2x - 1| = 3[ x ] + 2{ x }.
1 y Example 72. Solve the system of equations in x , y
Sol. Case I 2x - 1 ³ 0 or x ³
2 and z satisfying the following equations:
Then, given equation convert to x + [ y ] + {z } = 3 × 1
2x - 1 = 3 [ x ] + 2{ x } …(i)
{ x } + y + [z ] = 4 × 3
Q x = [x ] + {x } …(ii)
From Eqs. (i) and (ii), we get [ x ] + {y } + z = 5 × 4
2([ x ] + { x }) - 1 = 3[ x ] + 2 { x } where, [ × ] and { ×} denotes the greatest integer and frac-
\ [x ] = - 1 tional parts, respectively.
\ -1£ x <0 Sol. Q[ x ] + { x } = x , [y ] + {y } = y and [z ] + {z } = z ,
é 1ù
No solution êëQ x ³ 2 úû On adding all the three equations, we get
1
Case II 2x - 1 < 0 or x < 2( x + y + z ) = 128 .
2
Þ x + y + z = 6.4 …(i)
Then, given equation reduces to
Now, adding first two equations, we get
1 - 2x = 3[ x ] + 2 { x } …(iii)
x + y + z + [y ] + { x } = 7.4
Q x = [x ] + {x } …(iv)
Þ 6.4 + [y ] + { x } = 7.4 [from Eq. (i)]
From Eqs. (iii) and (iv), we get
Þ [y ] + { x } = 1
1 - 2([ x ] + { x }) = 3[ x ] + 2 { x }
\ [y ] = 1 and { x } = 0 …(ii)
Þ 1 - 5[ x ] = 4 { x }
On adding last two equations, we get
1 - 5[ x ]
\ {x } = …(v) x + y + z + {y } + [z ] = 9.7
4
Now, 0 £ {x } < 1 {y } + [z ] = 3.3 [from Eq. (ii)]
1 - 5[ x ] \ [z ] = 3 and {y } = 0.3 …(iii)
Þ 0£ <1
4 On adding first and last equations, we get
Þ 0 £ 1 - 5[ x ] < 4 x + y + z + [ x ] + {z } = 8.5
Þ 0 ³ - 1 + 5[ x ] > - 4 Þ [ x ] + {z } = 2.1 [from Eq. (i)]
3 1 \ [ x ] = 2, {z } = 0.1 …(iv)
Þ 1 ³ 5[ x ] > - 3 or - < [ x ] £
5 5 From Eqs. (i), (ii) and (iii), we get
\ [x ] = 0
1 x = [x ] + {x } = 2 + 0 = 2
From Eq. (v), { x } = y = [y ] + {y } = 1 + 0.3 = 1.3
4
1 1 and z = [z ] + {z } = 3 + 0.1 = 3.1
\ x =0+ =
4 4
Chap 02 Theory of Equations 137

\ 1< x <2
Roots of Equation with We have, f ( x ) = x 3 - 3 and g ( x ) = 1
the Help of Graphs or x3-3=1 Þ x3= 4
Here, we will discuss some examples to find the roots of \ x = ( 4 )1 / 3
equations with the help of graphs. Hence, x = 41 / 3 is the solution of the equation x 3 - [ x ] = 3.
Important Graphs Aliter
Q x = [ x ] + f , 0 £ f < 1,
1. y = ax 3 + bx 2 + cx + d
Then, given equation reduces to
x 3 - (x - f ) = 3 Þ x 3 - x = 3 - f
Hence, it follows that
2< x3 - x £3
Þ 2 < x ( x + 1) ( x - 1) £ 3
a>0 a<0
Further for x ³ 2, we have x ( x + 1) ( x - 1) ³ 6 > 3
2. x = ay 3 + by 2 + cy + d For x < - 1, we have x ( x + 1) ( x - 1) < 0 < 2
For x = - 1, we have x ( x + 1) ( x - 1) = 0 < 2
For -1 < x £ 0, we have x ( x + 1) ( x - 1) £ - x < 1
and for 0 < x £ 1, we have x ( x + 1) ( x - 1) < x < x 3 £ 1
Therefore, x must be 1 < x < 2
\ [x ] = 1
a>0 a<0
Now, the original equation can be written as
3. y = ax 4 + bx 3 + c x 2 + dx + e x 3- 1 = 3 Þ x 3= 4
Hence, x = 41 / 3 is the solution of the given equation.

y Example 74. Solve the equation x 3 - 3x - a = 0 for


different values of a.
Sol. We have, x 3 - 3x - a = 0 Þ x 3 - 3x = a
a>0 a<0
Let f ( x ) = x 3 - 3x and g ( x ) = a
y Example 73. Solve the equation x 3 - [ x ] = 3, where Q f ¢( x ) = 0
[ x ] denotes the greatest integer less than or equal to x . Þ 3x 2 - 3 = 0
Sol. We have, x 3 - [ x ] = 3 Þ x = - 1, 1
3 f ¢ ¢ ( x ) = 6x
Þ x - 3 = [x ]
Y
Let f ( x ) = x 3 - 3 and g ( x ) = [ x ].
2
It is clear from the graphs, the point of intersection of
two curves y = f ( x ) and y = g ( x ) lies between (1, 0) and y=a
(2 , 0).
Y y = f (x)
1
X' X
2 – 3 –1 0 3
y = g(x)
1

X¢ X
–2 –1 1 2 3
–1 –2
y = x3 – 3x
Y'
–2
\ f ¢ ¢ ( -1) = - 6 < 0 and f ¢ ¢ (1) = 6 > 0
–3
\ f ( x ) local maximum at x = ( -1) and local minimum at
x = 1 and f ( -1) = 2 and f (1) = - 2 and y = g ( x ) = a is a
Y¢ straight line parallel to X -axis.
138 Textbook of Algebra

Following cases arise y Example 76. Find all values of the parameter k for
Case I When a > 2, which all the roots of the equation
In this case y = f ( x ) and y = g ( x ) intersects at only one x 4 + 4 x 3 - 8 x 2 + k = 0 are real.
point, so x 3 - 3x - a = 0 has only one real root.
Sol. We have, x 4 + 4 x 3 - 8x 2 + k = 0
Case II When a = 2, Y
In this case y = f ( x ) and y = g ( x ) intersects at two points, –4 1
X¢ 0 X
so x 3 - 3x - a = 0 has three real roots, two are equal and
one different.
y = g(x)
Case III When -2 < a < 2,
In this case y = f ( x ) and y = g ( x ) intersects at three points, –3
so x 3 - 3x - a = 0 has three distinct real roots.
Case IV When a = - 2,
In this case y = f ( x ) and y = g ( x ) touch at one point and y = f(x)
intersect at other point, so x 3- 3x - a = 0 has three real
roots, two are equal and one different. 128
Case V When a < - 2,

In this case y = f ( x ) and y = g ( x ) intersects at only one
4 3 2
point, so x 3 - 3x - a = 0 has only one real root. Þ x + 4 x - 8x = - k
Let f ( x ) = x 4 + 4 x 3 - 8x 2 and g ( x ) = - k
y Example 75. Show that the equation Q f ¢( x ) = 0
x 3 + 2x 2 + x + 5 = 0 has only one real root, such that Þ 4 x 3 + 12x 2 - 16x = 0 Þ x = - 4, 0, 1
[a ] = - 3, where [ x ] denotes the integral point of x . and f ¢ ¢ ( x ) = 12x 2 + 24 x - 16
3 2
Sol. We have, x + 2x + x + 5 = 0 \ f ¢ ¢ ( -4 ) = 80, f ¢ ¢ (0) = - 16, f ¢ ¢ (1) = 20
Þ x 3 + 2x 2 + x = - 5 \ f ( x ) has local minimum at x = - 4 and x = 1 and local
Let f ( x ) = x 3 + 2x 2 + x and g ( x ) = - 5 maximum at x = 0
and f ( -4 ) = - 128, f (0) = 0, f (1) = - 3.
Q f ¢ ( x ) = 0 Þ 3x 2 + 4 x + 1 = 0
1 Following cases arise
Þ x = - 1, - and f ¢¢( x ) = 6x + 4
3 Case I When – k > 0 i.e., k < 0
æ 1ö
\ f ¢ ¢ ( - 1) = - 2 < 0 and f ¢ ¢ ç - ÷ = - 2 + 4 = 2 > 0 In this case y = x 4 + 4 x 3 - 8x 2 and y = ( -k ) intersect at
è 3ø
two points, so x 4 + 4 x 3 - 8x 2 + k = 0 has two real roots.
\ f ( x ) local maximum at x = - 1 and local minimum at
1 Case II When -k = 0 and -k = - 3, i.e. k = 0, 3
x=- In this case y = x 4 + 4 x 3 - 8x 2 and y = - k intersect at four
3
æ 1ö 4 points, so x 4 + 4 x 3 - 8x 2 + k = 0 has two distinct real roots
and f ( -1) = 0, f ç - ÷ = -
è 3ø 27 and two equal roots.
Y Case III When - 3 < - k < 0, i.e. 0 < k < 3
In this case y = x 4 + 4 x 3 - 8x 2 and y = - k intersect at four

1 3 2
y = f(x) = x + 2x + x distinct points, so x 4 + 4 x 3 - 8x 2 + k = 0 has four distinct
–1 3
X¢ X real roots.
0
Case IV When -128 < - k < - 3, i.e. 3 < k < 128
In this case y = x 4 + 4 x 3 - 8x 2 and y = - k intersect at two
distinct points, so x 4 + 4 x 3 - 8x 2 + k = 0 has two distinct
real roots.
Case V When -k = - 128 i.e., k = 128
y =g(x) = – 5 In this casey = x 4 + 4 x 3 - 8x 2 andy = - k touch at one
Y′ point, so x 4 + 4 x 3 - 8x 2 + k = 0 has two real and equal roots.
and f ( -2) = - 2 and f ( -3) = - 12 Case VI When -k < - 128, i.e. k > 128
Therefore, x must lie between ( -3) and ( -2). In this case y = x 4 + 4 x 3 - 8x 2 and y = - k do not
i.e. -3 < a < - 2 Þ [a ] = - 3 intersect, so there is no real root.
Chap 02 Theory of Equations 139

y Example 77. Let -1 £ p £ 1, show that the equation We observe that, the line y = g ( x ) = p , where -1 £ p £ 1
é1 ù
é1 ù intersect the curve y = f ( x ) exactly at point a Îê , 1ú.
4 x 3 - 3x - p = 0 has a unique root in the interval ê , 1ú ë2 û
ë2 û
Hence, 4 x 3 - 3x - p = 0 has exactly one root in the interval
and identify it.
é1 ù
Sol. We have, 4 x 3 - 3x - p = 0 êë 2 , 1úû .
Þ 4 x 3 - 3x = p Now, we have to find the value of root a.
Let f ( x ) = 4 x 3 - 3x and g ( x ) = p Let a = cos q , then 4 cos 3 q - 3 cos q - p = 0
1
\ f ¢( x ) = 0 Þ cos3q = p Þ 3q = cos -1( p ) or q = cos -1( p )
3
Þ 12x 2 - 3 = 0 ì1 ü
\ a = cos q = cos í cos -1( p )ý
1 1 î3 þ
Þ x = - , - and f ¢ ¢ ( x ) = 24 x
2 2 Aliter
æ 1ö æ1ö Let f( x ) = 4 x 3 - 3x - p
\ f ¢ ¢ ç - ÷ = - 12 < 0 and f ¢ ¢ ç ÷ = 12 > 0
è 2ø è2ø æ 1ö æ 1ö
\ f¢ ( x ) = 12x 2 - 3 = 12 ç x + ÷ ç x - ÷
æ 1ö è 2 ø è 2ø
\ f ( x ) has local maximum at ç x = - ÷ and local minimum
è 2ø
+ +
æ 1ö
at ç x = ÷. –1 – 1
è 2ø 2 2
æ 1ö 4 3 æ1ö 4 3 é1 ù
Also, f ç - ÷ = - + = 1 and f ç ÷ = - = - 1 Clearly, f¢ ( x ) > 0 for x Î ê , 1ú.
è 2ø 8 2 è2ø 8 2 ë2 û
é1 ù
Y Hence, f( x ) can have atmost one root in ê , 1ú.
ë2 û
1 æ1ö
y = f(x) Also, f ç ÷ = - 1 - p and f (1) = 1 - p
è2ø
y = g(x) æ1ö
\ f ç ÷ f ( 1) = - ( 1 - p 2 ) = ( p 2 - 1) £ 0 [Q - 1 £ p £ 1]
1 è2ø
2 é1 ù
X′ X Since, f( x ) being a polynomial, continuous on ê , 1ú and
–1 0
– 3 3 ë2 û
2 2 2 æ1ö
f ç ÷ f (1) £ 0. Therefore, by intermediate value theorem
è2ø
é1 ù
f( x ) has atleast one root in ê , 1ú.
ë2 û
–1
é1 ù
Hence, f( x ) has exactly one root in ê , 1ú .
Y′ ë2 û
140 Textbook of Algebra

#L Exercise for Session 4


æ1+ aö
1. If a, b, g are the roots of x 3 - x 2 - 1 = 0, the value of å ç ÷ , is equal to
è 1- a ø
(a) - 7 (b) - 6
(c) - 5 (d) - 4

2. If r , s, t are the roots of the equation 8x 3+ 1001x + 2008 = 0. The value of


(r + s ) 3 + (s + t ) 3 + (t + r ) 3 is
(a) 751 (b) 752
(c) 753 (d) 754

3. If a, b, g, d are the roots of equation x 4 + 4x 3 - 6x 2 + 7x - 9 = 0, the value of Õ (1 + a 2 ) is


(a) 9 (b) 11
(c) 13 (d) 15

4. If a, b , c, d are four consecutive terms of an increasing AP, the roots of the equation
( x - a ) ( x - c ) + 2 ( x - b ) ( x - d ) = 0 are
(a) non-real complex (b) real and equal
(c) integers (d) real and distinct

5. If x 2 + px + 1 is a factor of the expression ax 3 + bx + c then


(a) a 2 - c 2 = ab (b) a 2 + c 2 = - ab
(c) a 2 - c 2 = - ab (d) None of these

6. The number of real roots of the equation x 2 - 3 | x | + 2 = 0 is


(a) 1 (b) 2
(c) 3 (d) 4

7. Let a ¹ 0 and p ( x ) be a polynomial of degree greater than 2, if p ( x ) leaves remainder a and ( -a ) when divided
respectively by x + a and x - a, the remainder when p ( x ) is divided by x 2 - a 2, is
(a) 2x (b) -2x
(c) x (d) -x

8. The product of all the solutions of the equation ( x - 2)2 - 3 | x - 2 | + 2 = 0 is


(a) 2 (b) -4
(c) 0 (d) None of these
é x ù é x ù é x ù 31
9. If 0 < x < 1000 and ê ú + ê ú + ê ú = x , where [ x ] is the greatest integer less than or equal to x, the
ë 2 û ë 3 û ë 5 û 30
number of possible values of x is
(a) 32 (b) 33
(c) 34 (d) None of these

10. If [ x ] is the greatest integer less than or equal to x and ( x ) be the least integer greater than or equal to x and
[ x ]2 + ( x )2 > 25 then x belongs to
(a) [3, 4] (b) (- ¥, - 4]
(c) [4, ¥) (d) (- ¥, - 4] È [4, ¥)
Session 5
Irrational Equations, Irrational Inequations, Exponential
Equations, Exponential Inequations, Logarithmic Equations,
Logarithmic Inequations

Irrational Equations This equation is defined for 2x + 7 ³ 0


Here, we consider equations of the type which contain the ìï 7
x³-
and x + 4 ³0 Þ í 2
unknown under the radical sign and the value under the
ïî x ³ - 4
radical sign is known as radicand. 7
\ x³-
l If roots are all even (i.e. x , 4 x , 6 x ,..., etc) of an equation 2
7
are arithmetic. In other words, if the radicand is negative For x ³ - , the left hand side of the original equation
2
( i.e. x < 0 ), then the root is imaginary, if the radicand is
is positive, but right hand side is zero. Therefore, the
zero, then the root is also zero and if the radicand is equation has no roots.
positive, then the value of the root is also positive.
(ii) We have, (x - 4) = - 5
l If roots are all odd ( i.e. 3 x , 5 x , 7 x ,... etc) of an equation,
The equation is defined for x - 4 ³ 0
then it is defined for all real values of the radicand. If the
radicand is negative, then the root is negative, if the \ x³4
radicand is zero, then the root is zero and if the radicand For x ³ 4, the left hand side of the original equation is
is positive, then the root is positive. positive, but right hand side is negative.
Therefore, the equation has no roots.
(6 - x ) - x - 8 = 2
Some Standard Formulae to (iii) We have,
The equation is defined for
Solve Irrational Equations 6 - x ³ 0 and x - 8 ³ 0
If f and g be functions of x , k Î N . Then, ìx £ 6
\ í
1. 2k
f 2k
g = 2k fg , f ³ 0, g ³ 0 îx ³ 8
Consequently, there is no x for which both expressions
2. 2k f / 2k g = 2k ( f /g ), f ³ 0, g > 0 would have sense. Therefore, the equation has no roots.
(iv) We have, ( -2 - x ) = 5 ( x - 7 )
2k
3. | f | 2k g = ( f 2k g ), g ³ 0 This equation is defined for
4. 2k ( f /g ) = 2k | f | / 2k | g |, fg ³ 0, g ¹ 0 -2 - x ³ 0 Þ x £ - 2
For x £ - 2 the left hand side is positive, but right
5. 2k fg = 2k | f | 2k g , fg ³ 0 hand side is negative.
Therefore, the equation has no roots.
y Example 78. Prove that the following equations has (v) We have, x + ( x + 16) = 3
no solutions.
The equation is defined for
(i) (2 x + 7) + (x + 4) = 0 (ii) (x - 4) = - 5 ìx ³0
x ³ 0 and x + 16 ³ 0 Þ í
(iii) (6 - x ) - (x - 8) = 2 (iv) -2 - x = (x - 7)
5
î x ³ - 16
15 Hence, x ³0
(v) x + (x + 16) = 3 (vi) 7 x + 8 -x + 3 = 98
x For x ³ 0 the left hand side ³ 4, but right hand side is
3. Therefore, the equation has no roots.
(vii) (x - 3) - x + 9 = (x - 1)
15
Sol. (i) We have, ( 2x + 7 ) + ( x + 4 ) = 0 (vi) We have, 7 x + 8 - x + 3 = 98
x
142 Textbook of Algebra

For x < 0, the expression 7 x is meaningless, Sol. We have, 3 ( x + 3) - x - 2 = 7


For x > 0, the expression 8 -x is meaningless Þ 3 ( x + 3) = 7 + ( x - 2)
15 On squaring both sides of the equation, we obtain
and for x = 0, the expression 3 is meaningless.
x 9 x + 27 = 49 + x - 2 + 14 x - 2
Consequently, the left hand side of the original Þ 8x - 20 = 14 ( x - 2)
equation is meaningless for any x Î R. Therefore, the
equation has no roots. ( 4 x - 10) = 7 x - 2
Again, squaring both sides, we obtain
(vii) We have, ( x - 3) - ( x + 9 ) = x - 1
16x 2 + 100 - 80x = 49 x - 98
This equation is defined for
Þ 16x 2 - 129 x + 198 = 0
ìx - 3 ³ 0 ì x ³3
ï ï æ 33 ö
Þ ( x - 6) ç x - ÷ = 0
íx + 9 ³ 0 Þ íx ³ - 9 è 16 ø
ïx - 1 ³ 0 ï x ³1
î î x 1 = 6 and x 2 =
33
Hence, x ³3 16
33
For x ³ 3, x - 3 < x + 9 i.e. ( x - 3) - ( x + 9 ) < 0 Hence, x 1 = 6 satisfies the original equation, but x 2 =
16
Hence, for x ³ 3, the left hand side of the original does not satisfy the original equation.
equation is negative and right hand side is positive. 33
\ x2 = is the extraneous root.
Therefore, the equation has no roots. 16
Form 2 An equation in the form
Some Standard Forms to 2n
f ( x ) = g ( x ), n Î N
Solve Irrational Equations ì g (x ) ³ 0
is equivalent to the system í 2n
Form 1 An equation of the form î f (x ) = g (x )
2n
f ( x ) = g 2n ( x ), n Î N is equivalent to f ( x ) = g ( x ). y Example 81. Solve the equation
Then, find the roots of this equation. If root of this
equation satisfies the original equation, then its root of the
(6 - 4 x - x 2 ) = x + 4.
original equation, otherwise, we say that this root is its Sol. We have, (6 - 4 x - x 2 ) = x + 4
extraneous root. This equation is equivalent to the system
Remark ì x + 4 ³0
í 2 2
Squaring an Equation May Give Extraneous Roots î6 - 4 x - x = ( x + 4 )
Squaring should be avoided as for as possible. If squaring is ìx ³ -4
necessary, then the roots found after squaring must be checked Þ í 2
whether they satisfy the original equation or not. If some values î x + 6x + 5 = 0
of x which do not satisfy the original equation. These values of x On solving the equation x 2 + 6x + 5 = 0
are called extraneous roots and are rejected.
We find that, x 1 = ( -1) and x 2 = ( -5) only x 1 = ( -1) satisfies
y Example 79. Solve the equation x = x - 2 . the condition x ³ - 4.
Consequently, the number -1 is the only solution of the
Sol. We have, x = x -2 given equation.
On squaring both sides, we obtain
Form 3 An equation in the form
x = ( x - 2) 2 3 f (x ) + 3 g (x ) = h (x ) …(i)
Þ x 2 - 5x + 4 = 0 Þ ( x - 1) ( x - 4 ) = 0
where f ( x ), g( x ) are the functions of x , but h( x ) is a
\ x 1 = 1 and x 2 = 4
function of x or constant, can be solved as follows cubing
Hence, x 1 = 4 satisfies the original equation, but x 2 = 1 does
not satisfy the original equation. both sides of the equation, we obtain
\ x 2 = 1 is the extraneous root. f ( x ) + g( x ) + 3 3 f ( x ) g( x ) ( 3 f ( x ) + 3 g( x )) = h 3 ( x )

y Example 80. Solve the equation Þ f ( x ) + g( x ) + 3 3 f ( x ) g( x ) (h( x )) = h 3 ( x )


3 ( x + 3) - ( x - 2) = 7. [from Eq. (i)]
Chap 02 Theory of Equations 143

We find its roots and then substituting, then into the We get, u = 4, v = 3
original equation, we choose those which are the roots of \ 2
2x + 5x - 2 = 4
the original equation. \ 2x 2 + 5x - 18 = 0
y Example 82. Solve the equation \ x 1 = 2 and x 2 = - 9 / 2
Both roots satisfies the original equation.
3 (2x - 1) + 3 ( x - 1) = 1. Hence, x 1 = 2 and x 2 = - 9 / 2 are the roots of the original
Sol. We have, 3 ( 2x - 1) + 3 ( x - 1) = 1 …(i) equation.
Cubing both sides of Eq. (i), we obtain
2x - 1 + x - 1 + 3 × 3 ( 2x - 1) ( x - 1)
Irrational Inequations
( 3 (2x - 1) + 3 ( x - 1)) = 1
We consider, here inequations which contain the
Þ 3x - 2 + 3 × 3 ( 2x 2 - 3x + 1) ( 1) = 1 [from Eq. (i)] unknown under the radical sign.
Þ 3 × 3 ( 2x 2 - 3x + 1) = 3 - 3x
Þ 3 ( 2x 2 - 3x + 1) = ( 1 - x )
Some Standard Forms
Again cubing both sides, we obtain
to Solve Irrational Inequations
2x 2 - 3x + 1 = ( 1 - x ) 3 Form 1 An inequation of the form
Þ ( 2x - 1) ( x - 1) = ( 1 - x ) 3
2n f ( x ) < 2n g ( x ), n Î N
Þ ( 2x - 1) ( x - 1) = - ( x - 1) 3 ì f (x ) ³ 0
is equivalent to the system í
Þ ( x - 1) {2x - 1 + ( x - 1) } = 0 2
îg ( x ) > f ( x )
Þ ( x - 1) ( x 2 ) = 0 and inequation of the form 2n + 1 f ( x ) < 2n + 1 g ( x ), n Î N
\ x 1 = 0 and x 2 = 1 is equivalent to the inequation f ( x ) < g ( x ).
Q x 1 = 0 is not satisfies the Eq. (i), then x 1 = 0 is an
extraneous root of the Eq. (i), thus x 2 = 1 is the only root of y Example 84. Solve the inequation
the original equation.
é 3 7 ù 6
Form 4 An equation of the form 5
ê x + 1 + x + 2ú < 5 x - 1 .
n a - f ( x ) + n b + f ( x ) = g ( x ). ë û
Sol. The given inequation is equivalent to
Let u = n a - f ( x ), n = n b + f ( x ) 3 7 6
+ <
Then, the given equation reduces to the solution of the x +1 x +2 x -1
system of algebraic equations. 4 x 2 - 15x - 25
Þ <0
ì u + v = g(x ) ( x + 1) ( x + 2) ( x - 1)
í n n
îu + v = a + b ( x + 5 / 4 ) ( x - 5)
Þ <0
( x + 1) ( x + 2) ( x - 1)
y Example 83. Solve the equation
From Wavy Curve Method :
2 2
(2x + 5x - 2) - 2x + 5x - 9 = 1 . + + +
–2 –5 –1 –
Sol. Let u = ( 2x 2 + 5x - 2) –
4
– 1 5

and v = ( 2x 2 + 5x - 9 ) æ 5 ö
x Î ( -¥, - 2) È ç - , 1÷ È (1, 5)
\ 2
u = 2x + 5x - 22 è 4 ø
and v 2 = 2x 2 + 5x - 9 Form 2 An inequation of the form
Then, the given equation reduces to the solution of the 2n f ( x ) < g ( x ), n Î N .
system of algebraic equations.
u -v =1 ì f (x ) ³ 0
u2 - v 2 = 7 ï
is equivalent to the system í g ( x ) > 0
Þ (u + v ) (u - v ) = 7 ï f ( x ) < g 2n ( x ),
Þ u +v =7 [Qu - v = 1] î
144 Textbook of Algebra

and inequation of the form 2n + 1 f ( x ) < g ( x ), n Î N Exponential Equations


2n + 1
is equivalent to the inequation f ( x ) < g ( x ). If we have an equation of the form a x = b (a > 0 ), then
y Example 85. Solve the inequation ( x + 14 ) < ( x + 2). (i) x Î f, if b £ 0
(ii) x = log a b, if b > 0, a ¹ 1
Sol. We have, ( x + 14 ) < ( x + 2)
(iii) x Î f, if a = 1, b ¹ 1
This inequation is equivalent to the system
(iv) x Î R, if a = 1, b = 1 (since, 1 x = 1 Þ 1 = 1, x Î R )
ì x + 14 ³ 0 ì x ³ - 14
ï ï
í x +2>0 Þí x > -2 y Example 87. Solve the equation
ï x + 14 < ( x + 2)2 ï x 2 + 3x - 10 > 0 2
î î (6 - x ) ( 3 x - 7.2 x + 3.9 - 9 3 ) = 0.
ì x ³ - 14 ì x ³ - 14 Sol. We have,
ï ï
Þ í x > -2 Þí x > -2 ( 6 - x ) ( 3x
2
- 7.2x + 3.9
- 9 3) = 0
ï( x + 5) ( x - 2) > 0 ï x < - 5 and x > 2
î î This equation is defined for
On combining all three inequation of the system, we get 6 - x ³ 0 i.e., x £ 6 …(i)
x > 2, i.e. x Î (2 , ¥ ) This equation is equivalent to the collection of equations
2
- 7.2 x + 3. 9
Form 3 An inequation of the form 6 - x = 0 and 3x -9 3 =0
x 2 - 7. 2 x + 3. 9
2n f ( x ) > g( x ), n Î N \ x 1 = 6 and 3 = 32 . 5
is equivalent to the collection of two systems of then x 2 - 7.2 x + 3.9 = 2.5
inequations x 2 - 7.2 x + 1.4 = 0
ì g (x ) ³ 0 ìg ( x ) < 0 We find that, x2 =
1
and x 3 = 7
i.e. í and í
2n 5
îf (x ) > g (x ) î f (x ) ³ 0
Hence, solution of the original equation are
and inequation of the form 2 n + 1 f ( x ) > g ( x ), n Î N [which satisfies Eq. (i)]
is equivalent to the inequation f ( x ) > g 2 n + 1 ( x ). 1
x 1 = 6, x 2 = .
5
y Example 86. Solve the inequation
Some Standard Forms to
( - x 2 + 4 x - 3) > 6 - 2x .
Solve Exponential Equations
Sol. We have, ( - x 2 + 4 x - 3) > 6 - 2x Form 1 An equation in the form a f ( x ) = 1 , a > 0 , a ¹ 1
This inequation is equivalent to the collection of two is equivalent to the equation f ( x ) = 0
systems, of inequations
2
ì 6 - 2x ³ 0 ì 6 - 2x < 0 + 3x + 2
i.e. í 2 y Example 88. Solve the equation 5 x = 1.
2 and í 2
î - x + 4 x - 3 > ( 6 - 2 x ) î- x + 4 x - 3 ³ 0 Sol. This equation is equivalent to
ì x £3 ì x >3 x 2 + 3x + 2 = 0
Þ í and í
î( x - 3) (5x - 13) < 0 î( x - 1) ( x - 3) £ 0 Þ ( x + 1) ( x + 2) = 0
ìï x £ 3 \x 1 = - 1, x 2 = - 2 consequently, this equation has two
ì x >3
Þ í 13 and í roots x 1 = - 1 and x 2 = - 2 .
< x <3 î1 £ x < 3
ïî 5 Form 2 An equation in the form
The second system has no solution and the first system has
æ 13 ö
f (a x ) = 0
solution in the interval ç < x < 3÷.
è5 ø is equivalent to the equation f (t ) = 0, where t = a x .
æ 13 ö
Hence, x Îç , 3÷ is the set of solution of the original If t 1 , t 2 , t 3 , ..., t k are the roots of f (t ) = 0, then
è5 ø
inequation. a x = t 1 , a x = t 2 , a x = t 3 , ..., a x = t k
Chap 02 Theory of Equations 145

y Example 89. Solve the equation 5 2x - 24 × 5 x - 25 = 0. Let t = 2x ,


2
Then, 30t - 135t + 60 = 0
Sol. Let 5x = t , then the given equation can reduce in the form
Þ 6t 2 - 27t + 12 = 0
t 2 - 24t - 25 = 0
Þ 6t 2 - 24t - 3t + 12 = 0
Þ (t - 25) (t + 1) = 0 Þ t ¹ - 1,
Þ (t - 4 ) (6t - 3) = 0
\ t = 25, 1
then 5 = 25 = 52 , then x = 2
x Then, t 1 = 4 and t 2 =
2
Hence, x 1 = 2 is only one root of the original equation. Thus, given equation is equivalent to
Form 3 An equation of the form 1
2x = 4 and 2x =
aa f ( x ) + bb f (x)
+ gc f ( x ) = 0, 2
Then, x 1 = 2 and x 2 = - 1
where a, b, g Î R and a, b, g ¹ 0 and the bases satisfy the Hence, roots of the original equation are x 1 = 2 and
condition b 2 = ac is equivalent to the equation x 2 = - 1.
at 2 + bt + g = 0, where t = (a / b ) f ( x ) Form 5 An equation of the form a f ( x ) + b f ( x ) = c ,
If roots of this equation are t 1 and t 2 , then where a, b, c Î R and a, b, c satisfies the condition
f (x) f (x) a 2 + b 2 = c , then solution of this equation is f ( x ) = 2 and
(a / b ) = t 1 and (a / b ) = t2
no other solution of this equation.
y Example 90. Solve the equation
y Example 92. Solve the equation 3 x - 4 + 5 x - 4 = 34.
64 × 9 x - 84 × 12 x + 27 × 16 x = 0.
Sol. Here, 32 + 52 = 34, then given equation has a solution
Sol. Here, 9 ´ 16 = (12)2 .
x - 4 = 2.
Then, we divide its both sides by 12x and obtain \ x 1 = 6 is a root of the original equation.
x x
æ3ö æ4ö Form 6 An equation of the form { f ( x )} g ( x ) is
Þ 64 × ç ÷ - 84 + 27 × ç ÷ = 0 …(i)
è4ø è3ø
x equivalent to the equation
æ3ö
Let ç ÷ = t, then Eq. (i) reduce in the form { f ( x )} g ( x ) = 10 g ( x ) log f ( x ) ,
è4ø
64t 2 - 84t + 27 = 0 where f ( x ) > 0.
3 9
\ t1 = and t 2 = y Example 93. Solve the equation 5 x x 8 x -1 = 500.
4 16
x 1 x 2 Sol. We have, 5x x 8x - 1 = 53 × 22
æ3ö æ3ö æ3ö æ3ö
then, ç ÷ = ç ÷ and ç ÷ = ç ÷ æ x - 1ö
è4ø è4ø è4ø è4ø ç
è x ø
÷
Þ 5x × 8 = 53 × 22
\ x 1 = 1 and x 2 = 2 3x - 3
Hence, roots of the original equation are x 1 = 1 and x 2 = 2. Þ 5x × 2 x = 53 × 22
Form 4 An equation in the form æ x - 3ö
ç ÷
è x ø
a×a f (x )
+b×b f (x )
+ c = 0, Þ 5x - 3 × 2 =1
1 / x (x - 3 )
where a, b, c Î R and a, b, c ¹ 0 and ab = 1(a and b are Þ (52
. ) =1
inverse positive numbers) is equivalent to the equation is equivalent to the equation
2 f (x) - 3 ) log ( 5× 21 / x )
at + ct + b = 0, where t = a . 10(x =1
1/ x
If roots of this equation are t 1 and t 2, then a f (x)
= t 1 and Þ ( x - 3) log (5 × 2 )=0
af (x) = t2. Thus, original equation is equivalent to the collection of
equations
y Example 91. Solve the equation x - 3 = 0, log (5 × 21/ x ) = 0
æ1ö
15 × 2 x +1 + 15 × 2 2 - x = 135. \ x 1 = 3 , 5 × 21/ x = 1 Þ 21/ x = ç ÷
è5ø
Sol. This equation rewrite in the form \ x 2 = - log 5 2
60 Hence, roots of the original equation are x 1 = 3 and
30.2x + x = 135 x 2 = - log 5 2.
2
146 Textbook of Algebra

Exponential Inequations where a, b, g Î R and a, b, g ¹ 0 and the bases satisfy the


condition b 2 = ac is equivalent to the inequation
When we solve exponential inequation
at 2 + bt + g ³ 0 or at 2 + bt + g £ 0,
a f ( x ) > b (a > 0 ), we have
(i) x Î D f , if b £ 0 where t = (a / b ) f ( x ) .
(ii) If b > 0, then we have f ( x ) > log a b , if a > 1 Form 3 An inequation of the form
and f ( x ) < log a b, if 0 < a < 1 for a = 1, then b < 1. aa f ( x ) + bb f ( x ) + g ³ 0

Remark or aa f ( x ) + bb f ( x ) + g £ 0
The inequation af ( x ) £ b has no solution for b £ 0, a > 0, a ¹ 1. where a, b, g Î R and a, b, g ¹ 0 and ab = 1(a and b are
inverse (+ve) numbers) is equivalent to the inequation
1/ x
x +2 æ 1ö a t 2 + bt + g ³ 0 or a t 2 + bt + g £ 0
y Example 94. Solve the inequation 3 >ç ÷ .
è9ø
+2
where t = af (x)
Sol. We have, 3x > (3-2 )1 / x Þ 3x +2
> 3-2 / x
Form 4 If an inequation of the exponential form reduces
+ to the solution of homogeneous algebraic inequation, i.e.
0

a 0 f n ( x ) + a 1 f n - 1 ( x ) g( x ) + a 2 f n - 2 ( x ) g 2 ( x ) + K

Here, base 3 > 1 + a n - 1 f ( x ) g n - 1 ( x ) + a n g n ( x ) ³ 0,


2
2 x + 2x + 2 where a 0 , a 1 , a 2 , ,..., a n are constants (a 0 ¹ 0 ) and f ( x )
Þ x +2> - Þ >0
x x and g( x ) are functions of x .
( x + 1) 2 + 1
Þ >0 Þ x >0
x y Example 96. Solve the inequation
2 2 2
\ x Î (0, ¥ ) 2 2x -10x + 3 + 6 x - 5x +1 ³ 3 2x -10x + 3 .
Sol. The given inequation is equivalent to
Some Standard Forms to Solve 2
- 5x ) 2
- 5x 2
- 5x 2
- 5x )
8 × 22 ( x + 6 × 2x × 3x - 27 × 32 (x ³0
Exponential Inequations Let 2
2
x - 5x
= f ( x ) and 3
2
x - 5x
= g(x ) ,
Form 1 An inequation of the form 2 2
then 8 × f ( x ) + 6 f ( x ) × g ( x ) - 27 g ( x ) ³ 0
f (a x ) ³ 0 or f (a x ) £ 0
On dividing in each by g 2 ( x ) [Q g ( x ) > 0]
is equivalent to the system of collection 2
æ f (x )ö æ f (x )ö
ìt > 0, where t = a x
Then, 8 ç ÷ + 6ç ÷ - 27 ³ 0
è g(x ) ø è g(x ) ø
í
î f (t ) ³ 0 or f (t ) £ 0 and let
f (x )
=t [Qt > 0]
g(x )
y Example 95. Solve the inequation then 8t 2 + 6t - 27 ³ 0
4 x +1 - 16 x < 2 log 4 8. æ 3ö
Þ çt - ÷ ( t + 9 / 4 ) ³ 0
Sol. Let 4 x = t , then given inequation reduce in the form è 2ø
3 Þ t ³ 3 / 2 and t £ - 9 / 4
4t - t 2 > 2 × The second inequation has no root. [Qt > 0]
2
Þ t 2 - 4t + 3 < 0 Þ ( t - 1) ( t - 3) < 0 From the first inequation, t > 3 / 2
x 2 - 5x -1
Þ 1<t <3 [Qt > 0] æ2ö æ2ö é 2 ù
ç ÷ ³ç ÷ êëQ 3 < 1úû
Þ 1 < 4x < 3 è3ø è3ø
Þ 0 < x < log 4 3 Þ x 2 - 5x £ - 1 Þ x 2 - 5x + 1 £ 0
\ x Î(0, log 4 3)
5 - 21 5 + 21
Form 2 An inequation of the form \ £x£
2 2
aa f ( x ) + bb f ( x ) + gc f ( x ) ³ 0 é 5 - 21 5 + 21 ù
Hence, x Î ê , .
or aa f ( x ) + bb f (x)
+ gc f (x)
£0 ë 2 2 úû
Chap 02 Theory of Equations 147

Logarithmic Equations y Example 99. Solve the equation log (log 5 x ) 5 = 2 .

If we have an equation of the form Sol. We have, log (log 5 x ) 5 = 2

log a f ( x ) = b , (a > 0 ), a ¹ 1 Base of logarithm > 0 and ¹ 1 .


is equivalent to the equation \ log 5 x > 0 and log 5 x ¹ 1
f ( x ) = a b ( f ( x ) > 0 ). Þ x > 1 and x ¹ 5

y Example 97. Solve the equation \ The original equation is equivalent to


log 3 ( 5 + 4 log 3 ( x - 1)) = 2 .
log 5 x = 51 / 2 = 5
Sol. We have, log 3 (5 + 4 log 3 ( x - 1)) = 2
5
is equivalent to the equation (here, base ¹ 1, > 0). \ x1 = 5
\ 5 + 4 log 3 ( x - 1) = 32 Hence, 5 5
is the only root of the original equation.
1
Þ log 3 ( x - 1) = 1 Þ x - 1 = 3 Form 2 Equations of the form
\ x=4 (i) f (log a x ) = 0, a > 0, a ¹ 1 and
Hence, x 1 = 4 is the solution of the original equation.
(ii) g (log x A ) = 0, A > 0

Some Standard Formulae to Solve Then, Eq. (i) is equivalent to


f (t ) = 0, where t = log a x
Logarithmic Equations If t 1 , t 2 , t 3 , ..., t k are the roots of f (t ) = 0, then
f and g are some functions and a > 0, a ¹ 1, then, if
log a x = t 1 , log a x = t 2 , ..., log a x = t k
f > 0, g > 0, we have
and Eq. (ii) is equivalent to f (y ) = 0, where y = log x A.
(i) log a ( fg ) = log a f + log a g
If y 1 , y 2 , y 3 ,K, y k are the roots of f (y ) = 0, then
(ii) log a ( f / g ) = log a f - log a g
a log x A = y 1 , log x A = y 2 , ..., log x A = y k
(iii) log a f 2 a = 2 a log a | f | (iv) log ab f a = log a f
b y Example 100. Solve the equation
loga g loga f loga f
(v) f =g (vi) a =f 1 - 2 (log x 2 ) 2
= 1.
y Example 98. Solve the equation log x - 2 (log x ) 2
2 x log 4 3 + 3log 4 x = 27. Sol. The given equation can rewrite in the form
1 - 2 (2 log x )2
Sol. The domain of the admissible values of the equation is =1
x > 0. The given equation is equivalent to log x - 2 (log x )2
2.3log 4 x + 3log 4 x = 27 [from above result (v)] 1 - 8 (log x )2
Þ -1=0
Þ 3.3log 4 x = 27 log x - 2 (log x )2
Þ 3log 4 x = 9 Let log x = t ,
Þ 3log 4 x = 32 1 - 8t 2
1 - 8t 2 - t + 2t 2
then -1=0 Þ =0
Þ log 4 x = 2 t - 2t 2
t - 2t 2
Þ x 1 = 4 2 = 16 is its only root.
1 - t - 6t 2 (1 + 2t ) (1 - 3t )
Þ =0 Þ =0
(t - 2t 2 ) t (1 - 2t )
Some Standard Forms to
Solve Logarithmic Equations ì 1 ì 1
ït = - 2 ï log x = - 2 ì x = 10-1 / 2
Form 1 An equation of the form log x a = b, a > 0 has Þ í Þí Þ í 1
1 1 î x 2 = 10
1/ 3
ït = ï log x =
(i) Only root x = a 1 /b , if a ¹ 1 and b = 0. î 3 î 3
(ii) Any positive root different from unity, ifa = 1andb = 0. Hence, x 1 =
1
and x 2 = 3 10 are the roots of the original
(iii) No roots, if a = 1, b ¹ 0. 10
equation.
(iv) No roots, if a ¹ 1, b = 0.
148 Textbook of Algebra

y Example 101. Solve the equation ì x <1


ïé x ù é x ù
log 3x 10 - 6 log 2x 10 + 11 log x 10 - 6 = 0. Þ í æ1ö - 3 æ1ö + 1 = 0
êç ÷ ú êç ÷ ú
ïê è 2 ø úû êë è 2 ø úû
Sol. Put log x 10 = t in the given equation, we get îë
ì x <1
t 3 - 6t 2 + 11t - 6 = 0 Þ (t - 1) (t - 2) (t - 3) = 0, ï x x ì x <1
Þ íæ 1 ö æ1ö Þí
ìt = 1 ï çè 2 ÷ø = 3, çè 2 ÷ø + 1 ¹ 0 î x = ( - log 2 3)
ï î
then ít = 2
ït = 3 Hence, x 1 = - log 2 3 is the root of the original equation.
î
It follows that Example 103. Solve the equation log æ 2 + x ö 7 = log æ 2 ö 7.
ç ÷ ç ÷
è 10 ø è x + 1ø
ì log x 10 = 1 ì x = 10 ì x = 10
ï ï 2 ï Sol. The given equation is equivalent to
í log x 10 = 2 Þ í x = 10 Þ í x = 10 [Q x > 0 and ¹ 1]
ï log x 10 = 3 ï x 3 = 10 ï x = 3 10 ì 2
î î î ï x +1 >0
[Q x > 0 and ¹ 1] ï ìx + 1 > 0
\ x 1 = 10, x 2 = 10 and x 3 = 10 are the roots of the
3 ï 2 ï
í ¹ 1 Þ í x ¹1
original equation. ï x + 1 ï x = - 6, 3
ï2 + x = 2 î
Form 3 Equations of the form ïî 10 x +1
(i) log a f ( x ) = log a g ( x ), a > 0 , a ¹ 1 is equivalent to
\ x 1 = 3 is root of the original equation.
two ways.
Form 4 Equations of the form
ì g (x ) > 0
Method I í (i) log f ( x ) g ( x ) = log f ( x ) h ( x ) is equivalent to two
îf (x ) = g (x ) ways.
ì f (x ) > 0 ì g( x ) > 0 ì h( x ) > 0
Method II í
îf (x ) = g (x ) ï f (x ) > 0 ï f (x ) > 0
ï ï
Method I í Method II í
(ii) log f ( x ) A = log g ( x ) A , A > 0 is equivalent to two ï f (x ) ¹ 1 ï f (x ) ¹ 1
ways. ïîg( x ) = h( x ) ïîg( x ) = h( x )
ì g (x ) > 0
ï (ii) log g ( x ) f ( x ) = log h ( x ) f ( x ) is equivalent to two
Method I í g ( x ) ¹ 1 ways.
ïf (x ) = g (x )
î ì f (x ) > 0
ì f (x ) > 0 ï g( x ) > 0
ï
ï Method I í
Method II í f ( x ) ¹ 1 ï g( x ) ¹ 1
ïf (x ) = g (x ) ïîg( x ) = h( x )
î
ì f (x ) > 0
y Example 102. Solve the equation ï h( x ) > 0
é æ 1öx ù éæ 1 ö x ù ï
Method II í
log 1 / 3 ê2 ç ÷ - 1ú = log 1 / 3 ê ç ÷ - 4ú . ï h( x ) ¹ 1
è ø è ø
ë 2 û ë 4 û ïîg( x ) = h( x )
Sol. The given equation is equivalent to
ì æ1ö
x y Example 104. Solve the equation
ï 2ç ÷ - 1 > 0 log ( x 2 - 1) ( x 3 + 6 ) = log ( x 2 - 1) (2x 2 + 5x ).
ï è2ø
í x x
ï 2æ1ö - 1 = æ 1 ö - 4 Sol. This equation is equivalent to the system
ïî çè 2 ÷ø ç ÷
è4ø ì 5
ì 2x 2 + 5x > 0
ìæ 1 ö x 1 ï 2
ï x < - 2 and x > 0
ï x -1>0 ï
ïç ÷ > í Þ í x < - 1 and x > 1
ïè 2 ø 2 2
x -1¹1
Þ í 2x x ï ï x¹± 2
ïæ 1 ö - 2 æ 1 ö - 3 = 0 ïî x 3 + 6 = 2x 2 + 5x ï x = - 2, 1, 3
ïî çè 2 ÷ø ç ÷
è2ø
î
Hence, x 1 = 3 is only root of the original equation.
Chap 02 Theory of Equations 149

y Example 105. Solve the equation y Example 107. Solve the equation
log ( x 3 + 6 ) ( x 2 - 1) = log ( 2 x 2 +5x ) ( x 2 - 1). 2 log 2x = log (7 x - 2 - 2x 2 ).
Sol. This equation is equivalent to Sol. This equation is equivalent to the system
ì x 2 -1>0 ì 2x > 0
ï í 2
ï 2x 2 + 5x > 0 î( 2 x ) = 7 x - 2 - 2x 2
í 2
ï 2x + 5x ¹ 1 ì x >0
ïî x 3 + 6 = 2x 2 + 5x Þ í 2
î6 x - 7x + 2 = 0
ì x < - 1 and x > 1 ì x >0
ï 5 Þ í
ïï x < - 2 and x > 0 î( x - 1 / 2 ) ( x - 2 / 3) = 0
Þ í
- 5 ± 33 ìx = 1 / 2
ï x¹ Þ í
ï 4 îx = 2 / 3
ïî x = - 2, 1, 3
Hence, x 1 = 1 / 2 and x 2 = 2 / 3 are the roots of the original
Hence, x 1 = 3 is only root of the original equation.
equation.
Form 5 An equation of the form
Form 7 An equation of the form
log h ( x ) (log g ( x ) f ( x )) = 0 is equivalent to the system
(2m + 1 ) log a f ( x ) = log a g ( x ), a > 0 , a ¹ 1 , m Î N
ì h( x ) > 0
ï h( x ) ¹ 1 ì g( x ) > 0
ïï is equivalent to the system í 2m + 1
.
îf (x ) = g (x )
í g( x ) > 0
ï g( x ) ¹ 1
ï y Example 108. Solve the equation
ïî f ( x ) = g( x ) log ( 3x 2 + x - 2) = 3 log ( 3x - 2).
y Example 106. Solve the equation Sol. This equation is equivalent to the system
log x 2 - 6 x + 8 [log 2x 2 - 2x + 8 ( x 2 + 5x )] = 0. ì 3x 2 + x - 2 > 0
í 2 3
Sol. This equation is equivalent to the system î3x + x - 2 = (3x - 2)
ì 2 ì ( x - 2 / 3) ( x - 2) > 0
ï x - 6x + 8 > 0 Þ í
ï x 2 - 6x + 8 ¹ 1 î( x - 2 / 3) (9 x 2 - 13x + 3) = 0
ï
í 2x 2 - 2x - 8 > 0 ì x < 2 / 3 and x > 2
ï ï
2 Þ í 2 13 ± 61
ï 2x - 2x - 8 ¹ 1 x = ,x =
ïî x + 5x = 2x 2 - 2x - 8
2 îï 3 18
Solve the equations of this system 13 - 61
Original equation has the only root x 1 = ×
ì x < 2 and x > 4
18
ï
ï x ¹3± 2 Form 8 An equation of the form
ïï 1 - 17 1 + 17 log a f ( x ) + log a g ( x ) = log a m ( x ), a > 0 , a ¹ 1
Þ íx < and x >
ï 2 2 is equivalent to the system
ï 1 ± 19
x¹ ì f (x ) > 0
ï 2 ï
ïî x = - 1, 8 í g( x ) > 0
x = - 1, does not satisfy the third relation of this system. ï f ( x ) g ( x ) = m( x )
î
Hence, x 1 = 8 is only root of the original equation.
Form 6 An equation of the form y Example 109. Solve the equation
2m log a f ( x ) = log a g ( x ), a > 0 , a ¹ 1 , m Î N is 2 log 3 x + log 3 ( x 2 - 3) = log 3 0.5 + 5log 5 (log 3 8 )
equivalent to the system
Sol. This equation can be written as
ì f (x ) > 0 log 3 x 2 + log 3 ( x 2 - 3) = log 3 0.5 + log 3 8
í 2m
î f ( x ) = g( x )
log 3 x 2 + log 3 ( x 2 - 3) = log 3 ( 4 )
150 Textbook of Algebra

This is equivalent to the system


ì
Logarithmic Inequations
x2 > 0 ì x < 0 and x > 0
ï 2 ï When we solve logarithmic inequations
í x -3>0 Þ í x < - 3 and x > 3
ï x ( x 2 - 3) = 4 ï ( x 2 - 4 ) ( x 2 + 1) = 0 ìlog f ( x ) > log a g ( x )
î
2
î (i) í a
îa > 1
Þ x 2 - 4 = 0 \ x = ± 2, but x > 0
ì g (x ) > 0
Consequently, x 1 = 2 is only root of the original equation. ï
Þ í a >1
Form 9 An equation of the form ïf (x ) > g (x )
î
log a f ( x ) - log a g ( x ) = log a h ( x ) - log a t ( x ), a > 0, a ¹ 1
ìlog f ( x ) > log a g ( x )
is equivalent to the equation (ii) í a
log a f ( x ) + log a t ( x ) = log a g( x ) + log a h( x ), î 0 <a <1
which is equivalent to the system ì f (x ) > 0
ï
ì f (x ) > 0 Þ í 0 <a <1
ï ïf (x ) < g (x )
t(x ) > 0 î
ïï
í g( x ) > 0
ï
y Example 111. Solve the inequation
h( x ) > 0 log 2x + 3 x 2 < log 2x + 3 (2x + 3).
ï
ïî f ( x ) × t ( x ) = g( x ) × h( x )
Sol. This inequation is equivalent to the collection of the
systems
y Example 110. Solve the equation
éì 2x + 3 > 1 éì x > -1
æ 3p ö êí( x - 3) ( x + 1) < 0
ç sin ÷ êï x 2 < 2x + 3
êî
log 2 ( 3 - x ) - log 2 ç 4 = 1 + log ( x + 7 ). êïí Þ êì
÷ 2 3
êï0 < 2x + 3 < 1 ï - < x < -1
ç 5-x ÷ 2 êï x 2 > 2x + 3 êí 2
è ø ëî ëêïî( x - 3) ( x + 1) > 0
Sol. This equation is equivalent to é ì x > -1
ê í -1 < x < 3 Þ -1 < x < 3
æ 3p ö
ç sin ÷ 1 ê î
4 ÷ + log 2 + log ( x + 7 ) Þ
log 2 (3 - x ) = log 2 ç 2 2 êìï - 3 < x < - 1 3
ç5- x ÷ 2 êí 2 Þ- < x < -1
è ø êëïî x < - 1 and x > 3 2
æ 1 ö Hence, the solution of the original inequation is
Þ log 2 (3 - x ) = log 2 ç ÷ + log 2 2 + log 2 ( x + 7 )
è 2( 5 - x ) ø æ 3 ö
x Î ç - , - 1÷ È ( -1, 3).
which is equivalent to the system è 2 ø
ì
ï
3- x >0 Canonical Logarithmic Inequalities
ï
ï 1 ìlog x > 0 ìx > 1
ï >0 1. í a Þ í
í 2 (5 - x )
ï x +7 >0 î a >1 îa > 1
ï ìlog x > 0 ì0 < x < 1
ï(3 - x ) = 2 ( x + 7 ) 2. í a Þ í
ïî 2 (5 - x ) î 0 <a <1 î0 < a < 1
ì x <3 ìlog x < 0 ì0 < x < 1
ï x <5 3. í a Þ í
ï
Þ í î a >1 î a >1
ï x > -7
ïî( x - 1) ( x - 8) = 0 ìlog x < 0 ì x >1
4. í a Þ í
Hence, x 1 = 1 is only root of the original equation. î 0 <a <1 î0 < a < 1
Chap 02 Theory of Equations 151

Some Standard Forms to Solve Û


ì x < 6 - 6 and x > 6 + 6 and 2 < x < 10
í
î 0< x <5
Logarithmic Inequations
Therefore, the system has solution 2 < x < 6 - 6
Form 1 Inequations of the form
combining both systems, then solution of the original
Forms Collection of systems inequations is
(a) logg (x) f (x ) > 0 ì f (x ) > 1, ì 0 < f (x ) < 1 x Î (2, 6 - 6 ) È (10, ¥ ).
Û í í
î g (x ) > 1, î 0 < g (x ) < 1 Form 2 Inequations of the form
logg (x) f (x ) ³ 0 ì f (x ) ³ 1, ì 0 < f (x ) £ 1
(b) Û í í Forms Collection of systems
î g (x ) > 1, î 0 < g (x ) < 1
(a) log f ( x ) f ( x ) > log f ( x ) g( x ) Û ì f ( x ) > g( x ),
(c) log g ( x ) f ( x ) < 0 Û ì f ( x ) > 1, ì0 < f ( x ) < 1 ï
í í í g( x ) > 0,
î0 < g( x ) < 1, î g( x ) > 1 ï f( x ) > 1,
î
(d) log g ( x ) f ( x ) £ 0 Û ì f ( x ) ³ 1, ì0 < f ( x ) £ 1 ì f ( x ) < g( x )
í í ï
î0 < g( x ) < 1, î g( x ) > 1 í f (x ) > 0
ï 0 < f( x ) < 1
y Example 112. Solve the inequation î
æ 2x ö
(b) log f ( x ) f ( x ) ³ log f ( x ) g( x ) Û ì f ( x ) ³ g( x ),
log æ x 2 -12x + 30 ö ç log 2 ÷ > 0.
ç ÷è 5ø ï
ç
è 10 ÷
ø í g( x ) > 0,
ï f( x ) > 1,
Sol. This inequation is equivalent to the collection of two î
systems ì f ( x ) £ g( x )
ì x 2 - 12x + 30 ï
> 1, í f (x ) > 0
ïï ï 0 < f( x ) < 1
10
í î
ï log 2 æç 2x ö÷ > 1,
ïî è 5 ø (c) log f ( x ) f ( x ) < log f ( x ) g( x ) Û ì f ( x ) < g( x ),
ï
ì x 2 - 12x + 30 í f ( x ) > 0,
ïï0 < <1 ï f( x ) > 1,
10 î
í
ï 0 < log 2 æç ö÷ < 1
2x
ì f ( x ) > g( x )
ïî è 5 ø ï
í g( x ) > 0
On solving the first system, we have ï 0 < f( x ) < 1
î
ì x 2 - 12x + 20 > 0
ï
Þ í 2x (d) log f ( x ) f ( x ) £ log f ( x ) g( x ) Û ì f ( x ) £ g( x ),
ïî >2 ï
5 í f ( x ) > 0,
ì( x - 10) ( x - 2) > 0 ï f( x ) > 1,
Û í î
î x >5
ì f ( x ) ³ g( x )
ì x < 2 and x > 10 ï
Û í í g( x ) > 0
î x >5 ï 0 < f( x ) < 1
î
Therefore, the system has solution x > 10.
On solving the second system, we have y Example 113. Solve the inequation
ì0 < x 2 - 12x + 30 < 10 log ( x - 3) (2 ( x 2 - 10x + 24 )) ³ log ( x - 3) ( x 2 - 9 ).
ï
Þ í 2x
ïî 1< <2 Sol. This inequation is equivalent to the collection of systems
5
ì x 2 - 12x + 30 > 0 and x 2 - 12x + 20 < 0 ì2( x 2 - 10x + 24 ) ³ x 2 - 9,
Û í ï
5/2< x <5 í x 2 - 9 > 0,
î
ï x - 3 > 1,
î
152 Textbook of Algebra

ì2( x 2 - 10x + 24 ) £ x 2 - 9 On solving the second system, we have


ï 2
í 2( x - 10x + 24 ) > 0
ï 0< x -3<1
î
On solving the first system, we have
ì x 2 - 20x + 57 ³ 0, 3 10 – 43 4 6 10 + 43
ï
í( x + 3) ( x - 3) > 0, ì x - 20x + 57 £ 0,
2

ï x > 4, ï
î í( x - 6) ( x - 4 ) > 0,
ï 3 < x < 4,
ì x Î ( - ¥, 10 - 43 ] È [10 + 43, ¥ ) î
ï ì x Î [10 - 43, 10 + 43 ]
Û í x Î ( - ¥, - 3) È (3, ¥ )
ï ï
x Î ( 4, ¥ ) Û í x Î ( - ¥, 4 ) È (6, ¥ )
î
ï x Î (3, 4 )
Therefore, the system has solution î
Therefore, the system has solution
10 - 43 £ x < 4,
–3 3 10 – 43 4 10 + 43 i.e., x Î [10 - 43, 4 )
On combining the both systems, the solution of the original
x ³ 10 + 43
inequation is
i.e. x Î [10 + 43, ¥ ) x Î [10 - 43, 4 ) È [10 + 43, ¥ ).

#L Exercise for Session 5


1. The equation ( x + 1) - ( x - 1) = (4x - 1) has
(a) no solution (b) one solution (c) two solutions (d) more than two solutions

2. The number of real solutions of ( x 2 - 4x + 3) + ( x 2 - 9) = (4x 2 - 14x + 6) is


(a) one (b) two (c) three (d) None of these

3. The number of real solutions of (3x 2 - 7x - 30) - (2x 2 - 7x - 5) = x - 5 is


(a) one (b) two (c) three (d) None of these

4. 2
The number of integral values of x satisfying ( - x + 10x - 16) < x - 2 is
(a) 0 (b) 1 (c) 2 (d) 3
x
æ9ö
5. The number of real solutions of the equation ç ÷ = - 3 + x - x 2 is
è 10 ø
(a) 2 (b) 1 (c) 0 (d) None of these

6. The set of all x satisfying 3 2x x


- 3 - 6 > 0 is given by
(a) 0 < x < 1 (b) x > 1 (c) x > 3-2 (d) None of these

7. The number of real solutions of the equation 2x / 2


+ ( 2 + 1)x = (3 + 2 2 )x / 2
is
(a) one (b) two (c) four (d) infinite
2 2
-3 -3
8. The sum of the values of x satisfying the equation (31 + 8 15 )x + 1 = (32 + 8 15 )x is
(a) 3 (b) 0 (c) 2 (d) None of these

9. The number of real solutions of the equation log0. 5 x = | x | is


(a) 0 (b) 1 (c) 2 (d) None of these

10. The inequality ( x - 1)ln (2 - x ) < 0 holds, if x satisfies


(a) 1 < x < 2 (b) x > 0 (c) 0 < x < 1 (d) None of these
Shortcuts and Important Results to Remember
1 ‘0’ is neither positive nor negative even integer, ‘2’ is the 2 a2c = ab2 + bc 2 , i.e. ab2 , bc 2 , ca2 are in AP
only even prime number and all other prime numbers are 2a b c c a b
or = + i.e. , , are in AP.
odd, ‘1’ (i.e. unity) is neither a composite nor a prime b c a a b c
number and 1, -1are two units in the set of integers.
13 Given, y = ax 2 + bx + c
2 (i) If a > 0, b > 0 and a < b Þ a2 < b2
4ac - b2
(ii) If a < 0, b < 0 and a < b Þ a2 > b2 (i) If a > 0, ymin =
4a
(iii) If a1, a2 , a3 , ..., an Î R 4ac - b2
(ii) If a < 0, ymax =
and a12 + a22 + a32 +K+ an2 =0 4a
Þ a1 = a2 = a3 = K = an = 0 14 If a, b are the roots of ax 2 + bx + c = 0 and S n = a n + b n ,
1 then aS n + 1 + bS n + c S n - 1 = 0.
3 (i) Max (a, b) = (| a + b| + | a - b|)
2 15 If D1 and D2 are discriminants of two quadratics P( x ) = 0
1 and Q( x ) = 0, then
(ii) Min (a, b) = (| a + b| - | a - b|)
2
(i) If D1D2 < 0, then the equation P( x )× Q( x ) = 0 will have
4 If the equation f ( x ) = 0 has two real roots a and b , then two real roots.
f ¢ ( x ) = 0 will have a real root lying between a and b.
(ii) If D1D2 > 0, then the equation P( x )× Q( x ) = 0 has either
5 If two quadratic equations P( x ) = 0 and Q ( x ) = 0 have an four real roots or no real root.
irrational common root, both roots will be common.
(iii) If D1D2 = 0, then the equation P( x )× Q( x ) = 0 will have
6 In the equation ax 2 + bx + c = 0 [a, b, c Î R ], if (a) two equal roots and two distinct roots such that
c D1 > 0 and D2 = 0 or D1 = 0 and D2 > 0.
a + b + c = 0, the roots are 1, and if a - b + c = 0, the
a
c (b) only one real solution such that
roots are -1and . D1 < 0 and D2 = 0 or D1 = 0 and D2 < 0.
a
1 + (4a + 1)
7 The condition that the roots of ax 2 + bx + c = 0 may be in 16 If a > 0 and x = a + a + a + K+ ¥ , then x = .
2
the ratio p : q, is
pq b2 = ac ( p + q )2 (here, a : b = p : q ) 17 If a1, a2 , a3 , ..., an are positive real numbers, then least
p q b 2 æ1 1 1 1ö
i.e., + =± value of (a1 + a2 + a3 + K + an ) ç + + +K+ ÷
q p ac è a1 a2 a3 an ø
(i) If one root of ax 2 + bx + c = 0 is n times that of the is n 2 .
other, then nb2 = ac (n + 1)2 , here a : b = n : 1. æ 1 1 1ö
(i) Least value of (a + b + c ) ç + + ÷ = 32 = 9
èa b cø
(ii) If one root of ax 2 + bx + c = 0 is double of the other
(ii) Least value of
here n = 2, then 2 b2 = 9ac .
æ 1 1 1 1ö
8 If one root of ax 2 + bx + c = 0 is nth power of the other, (a + b + c + d ) ç + + + ÷ = 42 = 16
èa b c d ø
1 1

then (anc )n + 1 + (ac n )n + 1 = - b. a c e


18 Law of Proportions If = = = K, then each of
b d f
9 If one root of ax 2 + bx + c = 0 is square of the other, then these ratios is also equal to
a2c + ac 2 + b3 = 3abc . a+c +e +K
(i)
10 If the ratio of the roots of the equation ax 2 + bx + c = 0 is b+d + f +K
equal to the ratio of the roots of Ax 2 + Bx + C = 0 and æ pan + qc n + re n + K ö
1/ n

b 2
B 2 (ii) ç n ÷ (where, p, q, r , ..., n Î R )
a ¹ 0, A ¹ 0, then = . è pb + qd n + rf n + K ø
ac AC
ac n (ace ... )
11 If sum of the roots is equal to sum of their squares then (iii) =
bd n (bdf K )
2 ac = ab + b2 .
12 If sum of roots of ax 2 + bx + c = 0 is equal to the sum of 19 Lagrange’s Mean Value Theorem Let f ( x ) be a function
defined on [a, b] such that
their reciprocals, then

contd...
(i) f ( x ) is continuous on [a, b] and Let g( x ) = ( x - a ) be a linear monic polynomial a Î R.
(ii) f ( x ) is derivable on (a, b), then c Î(a, b) such that When g( x )| f ( x ); we can find quotient and remainder as
f (b) - f (a) follows :
f ¢ (c ) =
b-a
a a0 a1 a2 … an
20 Lagrange’s Identity If a1, a2 , a3 , b1, b2 , b3 Î R, then 0 a a0 b1a a bn - 1
(a12 + a22 + a32 ) (b12 + b22 + b32 ) - (a1b1 + a2 b2 + a3 b3 )2 a1 a2 an + ab n - 1 = 0
= (a1b2 - a2 b1 )2 + (a2 b3 - a3 b2 )2 + (a3 b1 - a1b3 )2 a0 + a a0 + b1a

or (a12 + a22 + a32 ) (b12 + b22 + b32 ) - (a1b1 2


+ a2 b2 + a3 b3 ) = b0 = b1 = b2

2 2 2
a1 a2 a2 a3 a3 a1
= + + \ f ( x ) = ( x - a ) (b0 x n - 1 + b1 x n-2
+ b2 x n-3
+ K + bn - 1 )
b1 b2 b2 b3 b3 b1 3 2
e.g. Find all roots of x - 6 x + 11x - 6 = 0.
Remark
Q( x - 1) is a factor of x 3 - 6 x 2 + 11x - 6, then
If (a12 + a22 + a32 ) (b12 + b22 + b32 ) £ (a1b1 + a2 b2 + a3 b3 )2 ,
a1 a2 a3 x=1 1 -6 11 -6
then = = .
b1 b2 b3
0 1 -5 6
21 Horner’s Method of Synthetic, Division When, we
1 -5 6 0
divide a polynomial of degree ³ 1 by a linear monic
polynomial, the quotient and remainder can be found by
this method. Consider \ x 3 - 6 x 2 + 11x - 6 = ( x - 1) ( x 2 - 5 x + 6)
f ( x ) = a0 x n + a1 x n - 1 + a2 x n - 2 + K + an = ( x - 1) ( x - 2 ) ( x - 3)
where a0 ¹ 0 and a0 , a1, a2 , ..., an Î R. Hence, roots of x - 6 x 2 + 11x - 6 = 0 are 1, 2 and 3.
3
JEE Type Solved Examples :
Single Option Correct Type Questions
x
This section contains 10 multiple choice examples. Ex. 3 Let f ( x ) = ò ( 2 - t 2 ) dt, the real roots of the
n
l
Each example has four choices (a), (b), (c) and (d) out of 1
which ONLY ONE is correct. equation x 2 - f ¢ ( x ) = 0 are
1
l Ex. 1 If a and b (a < b), are the roots of the equation (a) ±1 (b) ±
2
2
x + bx + c = 0, where c < 0 < b, then 1
(c) ± (d) 0 and 1
(a) 0 < a < b(b) a < 0 < b < |a | 2
(c) a < b < 0(d) a < 0 < |a | < b x

Sol. (b) Q a + b = - b, ab = c …(i)


Sol. (a) We have, f ( x ) = ò1 (2 - t 2 ) dt

Q c < 0 Þ ab < 0 Þ f ¢ ( x ) = (2 - x 2 )
Let a < 0, b > 0 \ x 2 - f ¢( x ) = 0
\ | a | = - a and a < 0 < b [Q a < b] …(ii) Þ x 2 - (2 - x 2 ) = 0 Þ x 4 + x 2 - 2 = 0
From Eq. (i), we get - | a | + b < 0
Þ x 2 = 1, - 2
Þ b < |a | …(iii)
Þ x = ±1 [only for real value of x ]
From Eqs. (ii) and (iii), we get
a < 0 < b < |a |
l Ex. 4 If x 2 + 3 x + 5 = 0 and ax 2 + bx + c = 0 have a
l Ex. 2 Let a, b be the roots of the equation x 2 - x + p = 0 common root and a, b, c Î N , the minimum value of a + b + c
is
and g, d be the roots of the equation x 2 - 4 x + q = 0. If
(a) 3 (b) 9
a, b, g and d are in GP, the integral values of p and q respec-
(c) 6 (d) 12
tively, are
Sol. (b) Q Roots of the equation x 2 + 3x + 5 = 0 are non-real.
(a) -2, - 32 (b) -2, 3
Thus, given equations will have two common roots.
(c) -6, 3 (d) -6, - 32 a b c
Sol. (a) Let r be the common ratio of the GP, then Þ = = =l [say]
1 3 5
b = ar , g = ar 2 and d = ar 3 \ a + b + c = 9l
\ a + b = 1 Þ a + ar = 1 Thus, minimum value of a + b + c = 9 [Qa, b, c Î N ]
or a(1 + r ) = 1 …(i)
and ab = p Þ a (ar ) = p l Ex. 5 If x 1 , x 2 , x 3 , K, x n are the roots of the equation
or a 2r = p …(ii) n
x + ax + b = 0, the value of
2 3
and g + d = 4 Þ ar + ar = 4 ( x 1 - x 2 )( x 1 - x 3 )( x 1 - x 4 ) K ( x 1 - x n ) is
or 2
ar (1 + r ) = 4 …(iii) (a) nx 1 + b
and gd = q (b) n ( x 1 )n - 1
Þ (ar 2 )(ar 3 ) = q (c) n ( x 1 )n - 1 + a
or a 2r 5 = q …(iv) (d) n ( x 1 )n - 1 + b
On dividing Eq. (iii) by Eq. (i), we get Sol. (c) Q x n + ax + b = ( x - x 1 )( x - x 2 )( x - x 3 )K( x - x n )
r 2 = 4 Þ r = - 2, 2 x n + ax + b
Þ ( x - x 2 )( x - x 3 )K( x - x n ) =
If we take r = 2, then a is not integer, so we take r = - 2. x - x1
On substituting r = - 2 in Eq. (i), we get a = - 1 On taking lim both sides, we get
x ®x 1
x n + ax + b é0 ù
Now, from Eqs. (ii) and (iv), we get ( x 1 - x 2 )( x 1 - x 3 )K( x 1 - x n ) = lim êë 0 form úû
p = a 2r = ( -1)2 ( -2) = - 2
x ® x1 x - x1
n -1
and q = a 2r 5 = ( -1)2 ( -2)5 = - 32 nx +a
= lim = n ( x 1 )n - 1 + a
x ® x1 1
Hence, ( p , q ) = ( -2, - 32)
156 Textbook of Algebra

Ex. 6 If a, b are the roots of the equation ax 2 + bx + c = 0 1 1


l
Sol. (d) Let = u and = v, then
and An = a n + bn , then a An + 2 + bAn + 1 + cAn is equal to
x (1 - x 2 )
35
u +v = and u 2 + v 2 = u 2v 2
(a) 0 (b) 1 (c) a + b + c (d) abc 12 2
b c æ 35 ö
Sol. (a) Qa + b = - and ab = Þ (u + v )2 = ç ÷
a a è 12 ø
n+2 n+2 2
\ An + 2 = a +b æ 35 ö
Þ u 2 + v 2 + 2uv = ç ÷
è 12 ø
= (a + b )(a n + 1 + bn + 1 ) - abn + 1 - ba n + 1 2
2 2 æ 35 ö
= (a + b )(a n + 1 + bn + 1 ) - ab(a n + bn ) Þ u v + 2uv = ç ÷ [Qu 2 + v 2 = u 2v 2 ]
è 12 ø
b c 2
= - An + 1 - An æ 35 ö
a a Þ u 2v 2 + 2uv - ç ÷ = 0
è 12 ø
Þ a An + 2 + b An + 1 + c An = 0
æ 49 ö æ 25 ö
Þ çuv + ÷ çuv - ÷ = 0
è 12 ø è 12 ø
l Ex. 7 If x and y are positive integers such that 49 25
Þ uv = - , uv =
xy + x + y = 71, x 2 y + xy 2 = 880, then x 2 + y 2 is equal to 12 12
49
(a) 125 (b) 137 (c) 146 (d) 152 Case I If uv = - , then
Sol. (c) Q xy + x + y = 71 Þ xy + ( x + y ) = 71 12
1 1 49
and x 2y + xy 2 = 880 Þ xy ( x + y ) = 880 × =- [here x < 0]
x (1 - x ) 2 12
Þ xy and ( x + y ) are the roots of the quadratic equation.
t 2 - 71t + 880 = 0 (12)2
Þ x4 - x2 + =0
Þ (t - 55)(t - 16) = 0 ( 49 )2
Þ t = 55, 16 (5 + 73 )
Þ x=-
\ x + y = 16 and xy = 55 14
25
Case II If uv = , then
So, x 2 + y 2 = ( x + y )2 - 2xy = (16)2 - 110 = 146 12
1 1 25
× = [here x > 0]
l Ex. 8 If a, b are the roots of the equation x 2 - 3 x + 5 = 0 x (1 - x 2 ) 12
and g, d are the roots of the equation x 2 + 5 x - 3 = 0, then (12)2
Þ x4 - x2 + =0
the equation whose roots are a g + b d and ad + bg, is (25)2
(a) x 2 - 15x - 158 = 0 (b) x 2 + 15x - 158 = 0 æ 2 9 ö æ 2 16 ö 3 4
Þ çx - ÷ çx - ÷ = 0 Þ x = ,
(c) x 2 - 15x + 158 = 0 (d) x 2 + 15x + 158 = 0 è 25 ø è 25 ø 5 5
Sol. (d) Q a + b = 3, ab = 5, g + d = ( - 5), g d = ( - 3) On combining both cases,
Sum of roots = (ag + bd ) + (ad + bg ) (5 + 73 ) 3 4
x=- , ,
= (a + b ) ( g + d ) = 3 ´ ( -5) = ( -15) 14 5 5
Product of roots = (ag + bd ) (ad + bg ) Hence, number of roots = 3
= a 2 gd + abg 2 + bad 2 + b 2 gd
= gd (a 2 + b 2 ) + ab( g 2 + d 2 )
l Ex. 10 The sum of the roots of the equation
33 x - 2
= - 3(a 2 + b 2 ) + 5( g 2 + d 2 ) 2 + 2 11x + 2 = 2 22 x + 1 + 1 is
= - 3[(a + b )2 - 2ab ] + 5[( g + d )2 - 2gd ] 1 2 3 4
(a) (b) (c) (d)
11 11 11 11
= - 3[9 - 10] + 5[25 + 6] = 158
Sol. (b) Let 211x = t , given equation reduces to
\ Required equation is x 2 + 15x + 158 = 0.
t3
+ 4t = 2t 2 + 1
l Ex. 9 The number of roots of the equation 4
1 1 35 Þ t 3 - 8t 2 + 16t - 4 = 0 Þ t 1 × t 2 × t 3 = 4
+ = is
x 2
(1 - x ) 12 Þ 211x 1 × 211x 2 × 211x 3 = 4 Þ 211(x 1 + x 2 + x 3 ) = 22
(a) 0 (b) 1 Þ 11( x 1 + x 2 + x 3 ) = 2
2
(c) 2 (d) 3 \ x1 + x 2 + x 3 =
11
Chap 02 Theory of Equations 157

JEE Type Solved Examples :


More than One Correct Option Type Questions
n This section contains 5 multiple choice examples. Each l Ex. 14 If cos 4 q + p, sin 4 q + p are the roots of the equa-
example has four choices (a), (b), (c) and (d) out of which
tion x 2 + a ( 2 x + 1) = 0 and cos 2 q + q, sin 2 q + q are the
more than one may be correct.
roots of the equation x 2 + 4 x + 2 = 0 then a is equal to
l Ex. 11 For the equation 2 x 2 - 6 2 x - 1 = 0 (a) -2 (b) -1 (c) 1 (d) 2
(a) roots are rational
Sol. (b,d)
(b) roots are irrational
Q cos 4 q - sin 4 q = cos 2q
(c) if one root is ( p + q ), the other is ( - p + q )
Þ cos 4 q - sin 4 q = cos 2 q - sin 2 q
(d) if one root is ( p + q ), the other is ( p - q )
Þ (cos 4 q + p ) - (sin 4 q + p ) = (cos 2 q + q ) - (sin 2 q + q )
Sol. (b,c) As the coefficients are not rational, irrational roots
need not appear in conjugate pair. 4a 2 - 4a 16 - 8 é Dù
Þ = êQa - b = a ú
1 1 1 ë û
Here, a + b = 3 2 and ab = -
2 Þ 2 2
4a - 4a = 8 or a - a - 2 = 0
Let a = p + q , then prove that other root b = - p + q .
or (a - 2)(a + 1) = 0 or a = 2, - 1
l Ex. 12 Given that a, g are roots of the equation l Ex. 15 If a, b, g are the roots of x 3 - x 2 + ax + b = 0 and
Ax 2 - 4 x + 1 = 0 and b, d the roots of the equation
b, g, d are the roots of x 3 - 4 x 2 + mx + n = 0. If a, b, g and d
Bx 2 - 6 x + 1 = 0, such that a, b, g and d are in HP then
are in AP with common difference d then
(a) A = 3 (b) A = 4 (c) B = 2
(d) B = 8
(a) a = m (b) a = m - 5
1 1 1 1
Sol. (a,d) Since, a , b, g and d are in HP, hence , , and (c) n = b - a - 2 (d) b = m + n - 3
a b g d
Sol. (b,c,d)
are in AP and they may be taken as a - 3d , a - d , a + d
1 Qa,b, g , d are in AP with common difference d, then
and a + 3d . Replacing x by , we get the equation whose
x b = a + d, g = a + 2d and d = a + 3d ...(i)
roots are a - 3d , a + d is x 2 - 4 x + A = 0 and equation Given, a,b, g are the roots of x 3 - x 2 + ax + b = 0, then
whose roots are a - d , a + 3d is x 2 - 6x + B = 0, then a + b + g =1 ...(ii)
(a - 3d ) + (a + d ) = 4 Þ 2(a - d ) = 4 ab + bg + ga = a ...(iii)
and (a - d ) + (a + 3d ) = 6 Þ 2(a + d ) = 6 abg = -b ...(iv)
\
5
a = and d =
1 Also, b, g , d are the roots of x 3 - 4 x 2 + mx + n = 0, then
2 2 b+g+d= 4 ...(v)
æ5 3ö æ5 1ö
Now, A = (a - 3d )(a + d ) = ç - ÷ ç + ÷ = 3 bg + gd + db = m ...(vi)
è2 2ø è2 2ø
bgd = -n ...(vii)
æ5 1ö æ5 3ö
and B = (a - d )(a + 3d ) = ç - ÷ ç + ÷ = 8 From Eqs. (i) and (ii), we get
è2 2ø è2 2ø
3a + 3d = 1 ...(viii)
l 2
Ex. 13 If | ax + bx + c | £ 1 for all x in [0, 1], then and from Eqs. (i) and (v), we get
3a + 6d = 4 ...(ix)
(a) |a | £ 8 (b) |b | > 8
From Eqs. (viii) and (ix), we get
(c) |c | £ 1 (d) |a | + |b | + |c | £ 17 2
1 d = 1, a = -
Sol. (a,c,d) On putting x = 0, 1 and , we get 3
2 Now, from Eq. (i), we get
|c | £ 1 …(i)
1 4 7
|a + b + c | £ 1 …(ii) b = , g = and d =
3 3 3
and | a + 2b + 4c | £ 4 …(iii) From Eqs. (iii), (iv), (vi) and (vii), we get
From Eqs. (i), (ii) and (iii), we get 2 8 13 28
a = - , b = ,m = ,n = -
| b | £ 8 and | a | £ 8 3 27 3 27
\ a = m - 5, n = b - a - 2 and b = m + n - 3
Þ | a | + | b | + | c | £ 17
158 Textbook of Algebra

JEE Type Solved Examples :


Passage Based Questions
n This section contains 2 solved passages based upon each 19. (b - c ) 2 + (c - a ) 2 + (d - b ) 2 is equal to
of the passage 3 multiple choice examples have to be
(a) a - d (b) (a - d )2 (c) a 2 - d 2 (d) (a + d )2
answered. Each of these examples has four choices (a), (b),
(c) and (d) out of which ONLY ONE is correct. Sol. (b) Let b = ar , c = ar 2 and d = ar 3
Passage I Now, (b - c )2 + (c - a )2 + (d - b )2
(Ex. Nos. 16 to 18) = (ar - ar 2 )2 + (ar 2 - a )2 + (ar 3 - ar )2
If G and L are the greatest and least values of the expression = a 2r 2 (1 - r )2 + a 2 (r 2 - 1)2 + a 2r 2 (r 2 - 1)2
x2 - x + 1
, x Î R respectively, then = a 2 (1 - r )2 {r 2 + (r + 1)2 + r 2 (r + 1)2 }
x2 + x + 1
= a 2 (1 - r )2 (r 4 + 2r 3 + 3r 2 + 2r + 1)
5 5
16. The least value of G + L is = a 2 (1 - r )2 (1 + r + r 2 )2 = a 2 (1 - r 3 )2
(a) 0 (b) 2 (c) 16 (d) 32 = (a - ar 3 )2 = (a - d )2
2
x - x +1
Sol. (b) Let y= 20. (u + v + w ) is equal to
x2 + x + 1
1 1
Þ x 2y + xy + y = x 2 - x + 1 (a) 2 (b) (c) 20 (d)
2 20
Þ ( y - 1) x 2 + ( y + 1) x + y - 1 = 0 [Q x Î R ]
2 2
Sol. (a) Now, u + 2v + 3w = 6 …(i)
\ ( y + 1) - 4 × ( y - 1) ( y - 1) ³ 0 [Qb - 4ac ³ 0] 4 u + 5v + 6w = 12 …(ii)
Þ (y + 1)2 - (2y - 2)2 ³ 0 and 6u + 9v = 4 …(iii)
Þ (3y - 1) (y - 3) £ 0 From Eqs. (i) and (ii), we get
1 1
\ £ y £ 3 Þ G = 3 and L = \ GL = 1 2u + v = 0 …(iv)
3 3 Solving Eqs. (iii) and (iv), we get
G 5 + L5 1/ 5 1/ 5
³ (GL ) = (1) =1 1 2
2 u = - ,v =
5
G +L 5 3 3
Þ ³ 1 or G 5 + L5 ³ 2 5
2 Now, from Eq. (i), we get w =
\ Minimum value of G 5 + L5 is 2. 3
1 2 5
17. G and L are the roots of the equation \ v +u +w = - + + =2
3 3 3
(a) 3 x 2 - 10x + 3 = 0 (b) 4 x 2 - 17 x + 4 = 0
21. If roots of f ( x ) = 0 be a, b, the roots of g ( x ) = 0 will be
(c) x 2 - 7 x + 10 = 0 (d) x 2 - 5x + 6 = 0
1 1 1 1
Sol. (a) Equation whose roots are G and L, is (a) a , b (b) - a , - b (c) , (d) - ,-
a b a b
x 2 - (G + L ) x + GL = 0
æ1 1 1 ö
10 Sol. (c) Now, f ( x ) = ç + + ÷ x 2 + [(b - c )2
Þ x 2 - x + 1 = 0 or 3x 2 - 10x + 3 = 0 èu v w ø
3
+ (c - a )2 + (d - b )2 ] x + u + v + w = 0
18. If L < l < G and l Î N , the sum of all values of l is
9 2
(a) 2 (b) 3 (c) 4 (d) 5 Þ f (x ) = - x + (a - d )2 x + 2 = 0
1 10
Sol. (b) Q L < l < G Þ < l <3 \ l = 1, 2
3 Þ f ( x ) = - 9 x 2 + 10(a - d )2 x + 20 = 0 …(v)
Sum of values of l = 1 + 2 = 3 Given, roots of f ( x ) = 0 are a and b.
1
Passage II Now, replace x by in Eq. (v), then
(Ex. Nos. 19 to 21) x
-9 10(a - d )2
Let a, b, c and d are real numbers in GP. Suppose u, v, w satisfy + + 20 = 0
x2 x
the system of equations u + 2v + 3w = 6, 4u + 5v + 6w = 12 and Þ 2 2
20x + 10(a - d ) x - 9 = 0
6u + 9v = 4. Further, consider the expressions
g(x ) = 0
æ1 1 1ö
f ( x ) = ç + + ÷ x 2 + [( b - c ) 2 + ( c - a ) 2 + ( d - b ) 2 ] \
1 1
Roots of g ( x ) = 0 are , .
è u v wø
a b
x + u + v + w = 0 and g (x ) = 20x 2 + 10 ( a - d ) 2 x - 9 = 0.
Chap 02 Theory of Equations 159

JEE Type Solved Examples :


Single Integer Answer Type Questions
n This section contains 2 examples. The answer to each l Ex. 23 If a root of the equation
example is a single digit integer ranging from 0 to 9 n 2 sin 2 x - 2 sin x - ( 2n + 1) = 0 lies in [0, p / 2 ], the
(both inclusive). minimum positive integer value of n is
Sol. (3) Q n 2 sin 2 x - 2sin x - (2n + 1) = 0
l Ex. 22 If the roots of the equation 10 x 3 - cx 2
2 ± 4 + 4n 2 (2n + 1)
- 54 x - 27 = 0 are in harmonic progression, the value of c is Þ sin x =
2n 2
Sol. (9) Given, roots of the equation [by Shridharacharya method]
10x 3 - cx 2 - 54 x - 27 = 0 are in HP. …(i)
1 ± (2n 3 + n 2 + 1)
1 =
Now, replacing x by in Eq. (i), we get n2
x
Q 0 £ sin x £ 1 [Q x Î [0, p/2]]
27 x 3 + 54 x 2 + cx - 10 = 0 …(ii)
1 + (2n 3 + n 2 + 1)
Hence, the roots of Eq. (ii) are in AP. Þ 0£ £1
Let a - d , a and a + d are the roots of Eq. (ii). n2
54 Þ 0 £ 1 + (2n 3 + n 2 + 1) £ n 2
Then, a -d + a + a +d = -
27 Þ (2n 3 + n 2 + 1) £ (n 2 - 1) [Qn > 1]
2
Þ a=- …(iii) On squaring both sides, we get
3
Since, a is a root of Eq. (ii), then 2n 3 + n 2 + 1 £ n 4 - 2n 2 + 1
27a 3 + 54a 2 + ca - 10 = 0 Þ n 4 - 2n 3 - 3n 2 ³ 0
æ 8ö æ4ö æ 2ö Þ n 2 - 2n - 3 ³ 0 Þ (n - 3) (n + 1) ³ 0
Þ 27 ç - ÷ + 54 ç ÷ + c ç - ÷ - 10 = 0 [from Eq. (iii)]
è 27 ø è9 ø è 3ø Þ n ³3
2c \ n = 3, 4, 5, K
Þ 6- = 0 or c = 9
3 Hence, the minimum positive integer value of n is 3.

JEE Type Solved Examples :


Matching Type Questions
n
This section contains 2 examples. Examples 24 and 25 Sol. (A) ® (p); (B) ® (p, q, r, s); (C) ® (p, q, s)
have three statements (A, B and C) given in Column I and x 2 - 2x + 9
four statements (p, q, r and s) in Column II. Any given (A) y= Þ x 2y + 2xy + 9y = x 2 - 2x + 9
x 2 + 2x + 9
statement in Column I can have correct matching with
one or more statement(s) given in Column II. Þ ( y - 1) x 2 + 2x ( y + 1) + 9 ( y - 1) = 0
Q x ÎR
l Ex. 24 Column I contains rational algebraic expressions
\ 4 ( y + 1) 2 - 4 × 9 × ( y - 1) 2 ³ 0
and Column II contains possible integers which lie in their
range. Match the entries of Column I with one or more Þ (y + 1)2 - (3y - 3)2 ³ 0
entries of the elements of Column II. Þ ( 4y - 2) ( -2y + 4 ) ³ 0
Þ (2y - 1) (y - 2) £ 0
Column I Column II 1
2 \ £ y £ 2 Þ y = 1, 2 (p)
x - 2x + 9 (p) 1 2
(A) y= , x ÎR
x 2 + 2x + 9 x 2 - 3x - 2
(B) Q y= Þ 2xy - 3y = x 2 - 3x - 2
x 2 - 3x - 2 (q) 3 2x - 3
(B) y= , x ÎR
2x - 3 Þ x 2 - x (3 + 2y ) + 3y - 2 = 0 Q x Î R
2
2x - 2x + 4 (r) -4 \ (3 + 2y )2 - 4 × 1 × (3y - 2) ³ 0
(C) y= , x ÎR
x2 - 4 x + 3 Þ 4y 2 + 17 ³ 0
(s) -9 \ y Î R (p, q, r, s)
160 Textbook of Algebra

2x 2 - 2x + 4 Sol. (A) ® ( r, s , t ); ( B) ® ( p , q , r ); ( C) ® ( r , s , t )
(C)Q y=
x 2 - 4x + 3 (A) Let f ( x ) = ax 2 + bx + c
Þ x 2y - 4 xy + 3y = 2x 2 - 2x + 4 Then, f ( 1) = a + b + c = - c [Qa + b + 2c = 0]
and f ( 0) = c
Þ x 2 (y - 2) + 2x (1 - 2y ) + 3y - 4 = 0
\ f ( 0) f ( 1) = - c 2 < 0 [Qc ¹ 0]
Q x ÎR
\ 4 (1 - 2y )2 - 4 (y - 2) (3y - 4 ) ³ 0 \Equation f ( x ) = 0 has a root in (0, 1).
Þ ( 4y 2 - 4y + 1) - (3y 2 - 10y + 8) ³ 0 \ f ( x ) has a root in (0, 2) as well as in ( -1, 1) (r)
Þ y 2 + 6y - 7 ³ 0 (B) Let f ¢( x ) = ax 2 + bx + c
Þ ( y + 7 ) ( y - 1) ³ 0 ax 3 bx 2
\ y £ - 7 or y ³ 1(p,q,s) \ f (x ) = + + cx + d
3 2
\ f ( 0) = d
l Ex. 25 Entries of Column I are to be matched with one a b æ 2a - 3b + 6c ö
or more entries of Column II. and f ( - 1) = - + + c + d = - ç ÷ +d
3 2 è 6 ø
Column I Column II = 0+d = d [Q 2a - 3b + 6c = 0]
(A) If a + b + 2c = 0 but c ¹ 0, then atleast one root in Hence, f (0) = f ( -1)
ax 2 + bx + c = 0 has (p) (-2, 0) Hence, f ¢ ( x ) = 0 has atleast one root in ( -1,0) (q)
(B) If a, b, c Î R such that atleast one root in \ f ( x ) = 0 has a root in ( -2,0) (p) as well as (-1,1) (r)
2a - 3 b + 6 c = 0, then equation has (q) (-1, 0) (C) Let f ( x ) = ò(1 + cos 8 x )(ax 2 + bx + c )dx
(C) Let a, b, c be non-zero real numbers atleast one root in
Given, f (1) - f (0) = f (2) - f (0)
such that (r) (-1, 1)
1 Þ f ( 1) = f ( 2)
ò0 (12+ cos x ) (ax + bx + c) dx
8 2
(s) atleast one root in
(0, 1)
Þ f ¢ ( x ) = 0 has atleast one root in (0,1).
= ò (1 + cos8 x ) (ax 2 + bx + c) dx , Þ (1 + cos 8 x )(ax 2 + bx + c ) = 0 has atleast one root in (0,1).
0
2
the equation ax + bx + c = 0 has Þ ax 2 + bx + c = 0 has atleast one root in (0, 1) (s)
(t) atleast one root in \ ax 2 + bx + c = 0 has a root in (0, 2) (t) as well as in
(0, 2) (-1, 1)(r)

JEE Type Solved Examples :


Statement I and II Type Questions
n
Directions Example numbers 26 and 27 are If D > 0 and is a perfect square, then roots are real, distinct
Assertion-Reason type examples. Each of these examples and rational.
contains two statements: But, here 2 3 ÏQ
Statement-1 (Assertion) and Statement-2 (Reason)
Each of these examples also has four alternative choices, \ Roots are not rational.
only one of which is the correct answer. You have to select 2 3 ± (12 + 184 )
the correct choice as given below. Here, roots are
2
(a) Statement-1 is true, Statement-2 is true; Statement-2
is a correct explanation for Statement-1 i.e. 3 ± 7. [irrational]
(b) Statement-1 is true, Statement-2 is true; Statement-2 But D = 12 + 184 = 196 = (14 )2
is not a correct explanation for Statement-1
\ Statement-1 is false and Statement-2 is true.
(c) Statement-1 is true, Statement-2 is false
(d) Statement-1 is false, Statement-2 is true
l Ex. 27 Statement 1 The equation a x + b x + c x - d x = 0
l Ex. 26 Statement 1 Roots of x 2 - 2 3 x - 46 = 0 are has only one real root, if a > b > c > d .
rational. Statement 2 If f (x ) is either strictly increasing or decreas-
Statement 2 Discriminant of x 2 - 2 3 x - 46 = 0 is a ing function, then f (x ) = 0 has only one real root.
perfect square. Sol. (c) Q a x + b x + c x - d x = 0
2
Sol. (d) In ax + bx + c = 0, a, b, c Î Q
Þ ax + bx + c x = d x
[here Q is the set of rational number]
Chap 02 Theory of Equations 161

æa ö
x
æb ö æc ö
x x \ f ( x ) is increasing function and lim f ( x ) = - 1
Let f ( x ) = ç ÷ + ç ÷ + ç ÷ - 1 x ®- ¥
èd ø èd ø èd ø Þ f ( x ) has only one real root.
x x x But Statement-2 is false.
æa ö æa ö æb ö æb ö æc ö æc ö
\ f ¢( x ) = ç ÷ ln ç ÷ + ç ÷ ln ç ÷ + ç ÷ ln ç ÷ > 0 For example, f ( x ) = e x
is increasing but f ( x ) = 0 has no
èd ø èd ø èd ø èd ø èd ø èd ø
and f (0) = 2 solution.

Subjective Type Examples


n In this section, there are 24 subjective solved examples. On substituting p = 18 in the given equation, we obtain
2
x + 18x + 45 = 0
l Ex. 28 If a, b are roots of the equation Þ ( x + 3) ( x + 15) = 0
2
x - p ( x + 1) - c = 0, show that (a + 1) (b + 1) = 1 - c . Hence, Þ x = - 3, 5
a 2 + 2 a +1 b2 + 2 b +1 and substituting p = - 18 in the given equation, we obtain
prove that + = 1. 2
a 2 + 2 a + c b2 + 2 b + c x - 18x + 45 = 0
( x - 3) ( x - 15) = 0
Sol. Since, a and b are the roots of the equation,
Þ x = 3, 15
x 2 - px - p - c = 0 Hence, the roots of the given equation are ( - 3), ( -15), 3
\ a +b = p and 15.
and ab = - p - c
Now, ( a + 1) ( b + 1) = ab + a + b + 1
l Ex. 30 If the roots of the equation ax 2 + bx + c = 0 (a ¹ 0 )
= - p -c + p + 1 = 1-c be a and b and those of the equation Ax 2 + Bx + C = 0
Hence, ( a + 1) ( b + 1) = 1 - c …(i) ( A ¹ 0 ) be a + k and b + k. Prove that
2
b 2 - 4ac æ a ö
2
a + 2a + 1 2
b + 2b + 1 =ç ÷ .
Second Part LHS = 2
+ B 2 - 4 AC è A ø
a + 2a + c b 2 + 2b + c
( a + 1) 2 ( b + 1) 2 Sol. Qa - b = ( a + k ) - ( b + k )
= +
2
( a + 1) - ( 1 - c ) 2
( b + 1) - ( 1 - c ) b 2 - 4ac ( B 2 - 4 AC ) é Dù
Þ = êQa -b =
( a + 1) 2 a A ë a úû
= 2
( a + 1) - ( a + 1) ( b + 1) æ b 2 - 4ac ö æ a ö
Þ ç 2 ÷ =ç ÷
( b + 1) 2
è B - 4 AC ø è A ø
+ 2
[from Eq. (i)]
( b + 1) - ( a + 1) ( b + 1) On squaring both sides, then we get
a +1 b +1 a -b b 2 - 4acæaö
2
= + = = 1 = RHS =ç ÷
a -b b -a a -b B - 4 AC è A ø
2

Hence, RHS = LHS


l Ex. 31 Let a , b and c be real numbers such that
l Ex. 29 Solve the equation x 2 + px + 45 = 0. It is given
a + 2b + c = 4. Find the maximum value of (ab + bc + ca ).
that the squared difference of its roots is equal to 144.
Sol. Given, a + 2b + c = 4
Sol. Let a , b be the roots of the equation x 2 + px + 45 = 0 and
Þ a = 4 - 2b - c
given that
Let y = ab + bc + ca = a (b + c ) + bc
(a - b )2 = 144
= ( 4 - 2b - c ) (b + c ) + bc
é Dù = - 2b 2 + 4b - 2bc + 4c - c 2
Þ p 2 - 4 × 1× 45 = 144 êQ a - b = a ú
ë û Þ 2b 2 + 2(c - 2)b - 4c + c 2 + y = 0
2
Þ p = 324 Since, b Î R, so
\ p = ( ± 18) 4 ( c - 2) 2 - 4 ´ 2 ´ ( - 4c + c 2 + y ) ³ 0
162 Textbook of Algebra

Þ (c - 2)2 + 8c - 2c 2 - 2y ³ 0 Sol. Given equations are


Þ 2
c - 4c + 2y - 4 £ 0 x 2 + ax + b = 0 …(i)
2
Since, c Î R, so 16 - 4 (2y - 4 ) ³ 0 Þ y £ 4 x + cx + d = 0 …(ii)
Hence, maximum value of ab + bc + ca is 4. 2
x + ex + f = 0 …(iii)
Aliter Let a , b be the roots of Eq. (i), b, g be the roots of Eq. (ii)
Q AM ³ GM and g , d be the roots of Eq. (iii), then
(a + b ) + (b + c )
Þ ³ (a + b ) (b + c ) a + b = - a, ab = b …(iv)
2
b + g = - c , bg = d …(v)
Þ 2 ³ (ab + bc + ca + b 2 ) [Qa + 2b + c = 4 ]
g + a = - e , ga = f …(vi)
2
Þ ab + bc + ca £ 4 - b \ LHS = (a + c + e )2 = ( - a - b - b - g - g - a )2
\ Maximum value of (ab + bc + ca) is 4. [from Eqs. (iv), (v) and (vi)]
l Ex. 32 Find a quadratic equation whose roots x 1 and x 2 = 4 (a + b + g )2 …(vii)
satisfy the condition RHS = 4(ac + ce + ea - b - d - f )
x 12 + x 22 = 5, 3 ( x 15 + x 25 ) = 11 ( x 13 + x 23 ) (assume that x 1 , x 2 = 4 {(a + b ) ( b + g ) + ( b + g ) ( g + a ) + ( g + a )
(a + b ) - ab - bg - ga )}
are real).
[from Eqs. (iv), (v) and (vi)]
Sol. We have, 3( x 15 + x 25 ) = 11( x 13 + x 23 )
= 4 (a 2 + b 2 + g 2 + 2ab + 2bg + 2ga )
x 15 + x 25 11
Þ = = 4 (a + b + g )2 …(viii)
x 13 + x 23 3
From Eqs. (vii) and (viii), then we get
( x 12 + x 22 ) ( x 13 + x 23 ) - x 12 x 22 ( x 1 + x 2 ) 11
Þ = (a + c + e )2 = 4 (ac + ce + ea - b - d - f )
( x 13 + x 23 ) 3
x 12 x 22 ( x 1 + x 2 ) 11 l Ex. 34 If a, b are the roots of the equation
Þ ( x 12 + x 22 ) - = 2
( x1 + x 2 ) ( x 12 + x 22 - x 1x 2 ) 3 x + px + q = 0 and g, d are the roots of the equation
2
[Q x 1 + x 2 = 5]
2 x 2 + rx + s = 0, evaluate (a - g ) (a - d) ( b - g ) ( b - d) in
terms of p, q, r and s . Deduce the condition that the equa-
x 12 x 22 11
Þ 5- = tions have a common root.
5 - x 1x 2 3
Sol. Qa , b are the roots of the equation
4 x 12 x 22
Þ = x 2
+ px + q = 0
3 5 - x 1x 2
\ a + b = - p , ab = q …(i)
Þ 3x 12 x 22 + 4 x 1x 2 - 20 = 0
and g , d are the roots of the equation x 2 + rx + s = 0
Þ 3x 12 x 22 + 10x 1x 2 - 6x 1x 2 - 20 = 0
\ g + d = - r, g d = s …(ii)
Þ ( x 1x 2 - 2) (3x 1x 2 + 10) = 0
Now, (a - g ) (a - d ) ( b - g ) ( b - d )
æ 10 ö
\ x 1x 2 = 2, ç - ÷ = [a 2 - a ( g + d ) + g d] [ b 2 - b ( g + d ) + g d ]
è 3ø
We have, ( x 1 + x 2 )2 = x 12 + x 22 + 2x 1x 2 = 5 + 2x 1x 2 = (a 2 + r a + s ) ( b 2 + r b + s ) [from Eq. (ii)]

\ 2
( x1 + x 2 ) = 5 + 4 = 9 [if x 1x 2 = 2] = a b + r ab (a + b ) + r ab + s (a + b 2 )
2 2 2 2

\ x1 + x 2 = ± 3 + sr (a + b ) + s 2
æ 10 ö 5é 10 ù = a 2 b 2 + r ab (a + b ) + r 2 ab + s [(a + b )2 - 2ab ]
and ( x 1 + x 2 )2 = 5 + 2 ç - ÷ = - ê if x 1x 2 = - ú
è 3 ø 3ë 3û
+ sr (a + b ) + s 2
which is not possible, since x 1, x 2 are real.
= q 2 - pqr + r 2q + s ( p 2 - 2q ) + sr ( - p ) + s 2
Thus, required quadratic equations are x 2 ± 3x + 2 = 0.
= (q - s )2 - rpq + r 2q + sp 2 - prs
l Ex. 33 If each pair of the three equations = (q - s )2 - rq ( p - r ) + sp ( p - r )
x 2 + ax + b = 0, x 2 + cx + d = 0 and x 2 + ex + f = 0 has = (q - s )2 + ( p - r ) (sp - rq ) …(iii)
exactly one root in common, then show that For a common root (let a = g or b = d ),
(a + c + e ) 2 = 4 (ac + ce + ea - b - d - f ). then (a - g ) (a - d ) ( b - g ) ( b - d ) = 0 …(iv)
Chap 02 Theory of Equations 163

From Eqs. (iii) and (iv), we get l Ex. 37 Solve for ‘ x ’


(q - s )2 + ( p - r ) (sp - rq ) = 0 1 ! + 2 ! + 3 ! + K + ( x - 1) ! + x ! = k 2 and k Î I .
Þ (q - s )2 = ( p - r ) (rq - sp ), which is the required Sol. For x < 4 , the given equation has the only solutions
condition . x = 1, k = ± 1 and x = 3, k = ± 3. Now, let us prove that
there are no solutions for x ³ 4. The expressions
l Ex. 35 Find all integral values of a for which the 1! + 2! + 3! + 4 ! = 33 ü
quadratic Expresion ( x - a ) ( x - 10 ) + 1 can be factored as a
1! + 2! + 3! + 4 ! + 5! = 153 ïï
product ( x + a ) ( x + b) of two factors and a, b ÎI. ý ends with the digit
1! + 2! + 3! + 4 ! + 5! + 6! = 873 ï
Sol. We have, ( x - a ) ( x - 10) + 1 = ( x + a ) ( x + b ) 1! + 2! + 3! + 4 ! + 5! + 6! + 7 ! = 5913ïþ
On putting x = - a in both sides, we get 3.
( - a - a ) ( - a - 10) + 1 = 0 Now, for x ³ 4 the last digit of the sum 1! + 2! + K + x ! is
\ (a + a ) (a + 10) = - 1 equal to 3 and therefore, this sum cannot be equal to a
square of a whole number k (because a square of a whole
a + a and a + 10 are integers. [Qa, a Î I ] number cannot end with 3).
\ a + a = - 1 and a + 10 = 1
or a + a = 1 and a + 10 = - 1
l Ex. 38 Find the real roots of the equation
(i) If a + 10 = 1
\ a = - 9, then a = 8 x + 2 x + 2 x + K + 2 x + 2 3x = x
Similarly, b = - 9 1444444424444444 3
n radical signs
Here, ( x - 8) ( x - 10) + 1 = ( x - 9 )2
Sol. Rewrite the given equation
(ii) If a + 10 = - 1
\ a = - 11, then a = 12 x + 2 x + 2 x + K + 2 x + 2 x + 2x = x …(i)
Similarly, b = 12 On replacing the last letter x on the LHS of Eq. (i) by the
Here, ( x - 12) ( x - 10) + 1 = ( x - 11)2 value of x expressed by Eq. (i), we get
Hence, a = 8, 12 x = x + 2 x + 2 x + K + x + 2x
14444442444444 3
2 n radical signs
l Ex. 36 Solve the equation Further, let us replace the last letter x by the same
expression again and again yields.
x + 3 - 4 ( x - 1) + x + 8 - 6 ( x - 1) = 1.
\ x = x + 2 x + 2 x + K + 2 x + 2x
Sol. Let ( x - 1) = t 144444424444443
3n radical signs
We have, x = t 2 + 1, t ³ 0
The given equation reduce in the form = x + 2 x + 2 x + K + 2 x + 2x = …
2 2 144444424444443
(t + 4 - 4t ) + (t + 9 - 6t ) = 1 4 n radical signs

Þ | t - 2| + | t - 3| = 1 We can write,

+ x = x +2 x +2 x +K
– – 3

= lim x + 2 x + 2 x + K + 2 x + 2x
+ + N ®¥
1 44444444244444444 3
– 2 N radical signs
If follows that
\ 2£t £3 x = x +2 x +2 x +K
2
Þ 4 £t £9
= x + 2 ( x + 2 x + K ) = ( x + 2x )
Þ 4 £ x -1£9
Hence, x 2 = x + 2x
Þ 5 £ x £ 10
Þ x 2 - 3x = 0
\ Solution of the original equation is x Î[ 5, 10].
\ x = 0, 3
164 Textbook of Algebra

Ex. 39 Solve the inequation, ( x 2


+ x + 1) x < 1. 1
l
Þ x =1+ Þ x2 - x - 1 = 0
x
Sol. Taking logarithm both sides on base 10,
1± 5
then x log ( x 2
+ x + 1) < 0 \ x=
2
which is equivalent to the collection of systems 1+ 5 1- 5
éì x > 0, éì x > 0, \ x1 = , x2 =
2 2
êí 2 êí 2
êî log ( x + x + 1) < 0, Þ êî x + x + 1 < 1, satisfy the given equation and this equation has no other
êì x < 0, êì x < 0, roots.
êí êí
2
êëî log ( x + x + 1) > 0, êëî x 2 + x + 1 > 1, l Ex. 41 Solve the system of equations
éì x > 0, éì x > 0, ì| x - 1 | + | y - 2 | = 1
êí x ( x + 1) < 0, êí - 1 < x < 0 í .
Þ êî Þ ê
î î y = 2 - | x -1|
êì x < 0, êì x < 0, Sol. On substituting | x - 1 | = 2 - y from second equation in
êí êí
ëî x ( x + 1) > 0 ëî x > 0 and x < ( - 1) first equation of this system, we get
2 - y + |y - 2| = 1
ì x Î f,
Þ í Now, consider the following cases
î x < ( - 1) If y ³ 2,
Consequently, the interval x Î ( - ¥, - 1) is the set of all then 2-y +y -2=1 Þ 0=1
solutions of the original inequation.
No value of y for y ³ 2.
? Remark If y < 2,
3
When the inequation is in power, then it is better to take log. then 2 - y + 2 - y = 1 Û y = , which is true.
2
l Ex. 40 Solve the equation From the second equation of this system,
1
1+ =x 3
= 2 - | x - 1|
1
1+ 2
1 1 1
1+ Þ | x - 1| = Þ x - 1 = ±
1+ O 2 2
1 1 1 3
1+ Þ x =1± Þx = ,
x 2 2 2
Consequently, the set of all solutions of the original system
When in expression on left hand side the sign of a fraction is 1 3 3
repeated n times. is the set of pairs ( x , y ), where x = , and y = .
2 2 2
Sol. Given equation is
1+
1
=x
l Ex. 42 Let a , b, c be real and ax 2 + bx + c = 0 has two real
1 roots a and b, where a < -1 and b >1, then show that
1+
1 c b
1+ 1+ + < 0.
1+ O
a a
1
1+ Sol. Since, a < - 1 and b > 1
x
Let us replace x on the LHS of the given equation by the a + l = - 1 and b = 1 + m [ l, m > 0 ]
expression of x . This result in an equation of the same c b
Now, 1 + + = 1 + ab + | a + b |
form, which however involves 2n fraction lines. Continuing a a
this process on the basis of this transformation, we can
= 1 + ( - 1 - l ) (1 + m ) + | - 1 - l + 1 + m |
write
= 1 - 1 - m - l - lm + | m - l |
1
x = 1 + lim 1 + = - m - l - lm + m - l [if m > l]
n ®¥ 1
1+ and = - m - l - lm + l - m [if l > m]
1
1+ c b
1+ O \ 1+ + = - 2l - lm or - 2m - lm
a a
1
1+ [n fractions] c b
x On both cases, 1 + + <0 [Q l, m > 0]
a a
Chap 02 Theory of Equations 165

Aliter It follows that


85
ax 2 + bx + c = 0, a ¹ 0
Q
åbi = - 2 and å å bi b j = 3
b c i =1 1 £ i < j £ 85
x2 + x + =0 2
a a 85 æ 85 ö
2
Let f ( x ) = x + x +
b c
–1 1
Then, åbi2 = çç åbi ÷÷ - 2 å å bi b j
a a a i =1 èi = 1 ø 1 £ i < j £ 85
b
f ( -1) < 0 and f (1 ) < 0 = 4 - 6 = -2 < 0
b c b c Thus, the bi ’s is not all real and then a i ’s are not all real.
Þ 1 - + < 0 and 1 + + < 0
a a a a
b c
l Ex. 45 Solve the equation
Then, 1 + + <0
a a 2 |x +1| - 2 x = | 2 x - 1| + 1.
æ3 - x ö æ 3-xö Sol. Find the critical points :
l Ex. 43 Solve the equation x ç ÷ çx + ÷ = 2.
è x +1ø è x +1ø
Sol. Hence, x + 1 ¹ 0
æ3 - x ö 3- x
and let xç ÷ = u and x + =v
è x + 1ø x +1 –1 0
\ uv = 2 …(i) x
x + 1 = 0, 2 - 1 = 0
æ3 - x ö æ3 - x ö \ x = - 1, x = 0
and u +v = xç ÷+x+ç ÷
è x + 1ø è x + 1ø Now, consider the following cases :
æ3 - x ö x < -1
= ( x + 1) ç ÷ + x =3- x + x =3 2- (x + 1) - 2x = - (2x - 1) + 1
è x + 1ø
\ u + v = 3 and uv = 2 Þ 2- (x + 1) = 2
Then, u = 2, v = 1 or u = 1, v = 2 \ - ( x + 1) = 1
\ x = -2 …(i)
Given equation is equivalent to the collection
-1 £ x < 0
ì æ3 - x ö ì æ3 - x ö
÷ =2 ÷ =1 2 x + 1 - 2 x = - ( 2 x - 1) + 1
ïï x ç ïï x ç
è x + 1ø è x + 1ø Þ 2 x +1 = 2
\ í or í
ïx + 3 - x = 1 ïx + 3 - x = 2 \ x +1=1
ïî x +1 ïî x +1 \ x =0
ìx 2 - x + 2 = 0 ì x 2 - 2x + 1 = 0 x ¹0 [Q - 1 £ x < 0]
Þ í 2 or í 2 x³ 0
îx - x + 2 = 0 î x - 2x + 1 = 0 2 x +1 - 2 x = 2 x - 1 + 1
ìæ 1ö
2
7 Þ 2 x + 1 = 2 ×2 x
ì x2 - x + 2 = 0 ïç x - ÷ + ¹ 0
Þ í 2 Þ íè 2 ø 4 Þ 2 x +1 = 2 x +1
î x - 2x + 1 = 0 ï ( x - 1) 2 = 0 which is true for x ³ 0. …(ii)
î
\ ( x - 1) 2 = 0 Now, combining all cases, we have the final solution as
Þ x = 1 is a unique solution of the original equation. x Î [0, ¥ ) È { - 2}

l Ex. 44 Show that for any real numbers a 3 , a 4 , a 5 , ..., a 85 ,


l Ex. 46 Solve the inequation
the roots of the equation - | y | + x - ( x 2 + y 2 - 1) ³ 1.
a 85 x 85 + a 84 x 84 + K + a 3 x 3 + 3 x 2 + 2 x + 1 = 0 are not real. Sol. We have, - | y | + x - ( x 2 + y 2 - 1) ³ 1
85 84
Sol. Let P ( x ) = a 85 x + a 84 x Þ x - | y | ³ 1 + ( x 2 + y 2 - 1)
+K + a 3 x 3 + 3 x 2 + 2 x + 1 = 0 …(i) if x ³ | y |,
Since, P(0) = 1, then 0 is not a root of Eq. (i). then squaring both sides,
Let a 1, a 2 , a 3 , ..., a 85 be the complex roots of Eq. (i).
æ 1 ö x 2 + y 2 - 2x | y | ³ 1 + x 2 + y 2 - 1 + 2 ( x 2 + y 2 - 1)
Then, the bi ç let ÷ the complex roots of the polynomial
è ai ø Þ - x | y | ³ ( x 2 + y 2 - 1) …(i)
85 84 83 82
Q (y ) = y + 2y + 3y + a 3y + K + a 85 Since, x ³ |y | ³ 0 …(ii)
166 Textbook of Algebra

Then, LHS of Eq. (i) is non-positive and RHS of Eq. (ii) is and -a2 £ t £ 0
non-negative. Therefore, the system is satisfied only, when
both sides are zero. But t >0 [from Eq. (ii)]
\ The inequality Eq. (i) is equivalent to the system. - a 2 + ( a 4 + 8)
\ t ³
ì x |y | = 0 2
í 2 2
îx + y - 1 = 0 - a 2 + ( a 4 + 8)
\ ax ³
The Eq.(i) gives x = 0 or y = 0. If x = 0, then we find y = ± 1 2
from Eq. (ii) but x ³ | y | which is impossible.
For 0 < a < 1,
If y = 0, then from Eq. (ii), we find
æ - a 2 + ( a 4 + 8) ö
x2 = 1 x £ loga ç ÷
ç 2 ÷
è ø
\ x = 1, - 1
Taking x =1 [Q x ³ | y | ] é æ - a 2 + ( a 4 + 8) ö ù
\ x Îê - ¥, loga ç ÷ú
\ The pair (1, 0) satisfies the given inequation. Hence, (1, 0) êë ç 2 ÷ú
è øû
is the solution of the original inequation.
æ - a 2 + ( a 4 + 8) ö
l Ex. 47 If a 1 , a 2 , a 3 , ..., a n (n ³ 2 ) are real and and for a > 1, x ³ loga ç ÷
ç 2 ÷
è ø
(n - 1) a 12 - 2na 2 < 0, prove that atleast two roots of the
æ æ - a 2 + ( a 4 + 8) ö ö
equation x n + a 1 x n - 1 + a 2 x n - 2 + K + a n = 0 are \ x Îç loga ç ÷, ¥ ÷
ç ç 2 ÷ ÷
imaginary. è è ø ø
Sol. Let a 1, a 2 , a 3 , ..., a n are the roots of the given equation.
l Ex. 49 Solve the inequation
Then, å a 1 = a 1 + a 2 + a 3 + K + a n = - a1
and å a 1 a 2 = a 1 a 2 + a 1 a 3 + K + a n - 1 a n = a 2 log | x | ( ( 9 - x 2 ) - x - 1) ³ 1.
Now, (n - 1)a12 - 2na 2 = (n - 1) ( å a 1 )2 - 2n å a 1 a 2 Sol. We rewrite the given inequation in the form,

= n {( å a 1 )2 - 2 å a 1a 2 } - ( å a 1 )2 log| x | ( (9 - x 2 ) - x - 1) ³ log| x | (| x | )
= n å a 12 - ( å a1 )2 This inequation is equivalent to the collection of systems.

ïì (9 - x ) - x - 1 ³ | x |, if | x | > 1
2
= å å (a i - a j )2
1£i < j £n í
ïî (9 - x 2 ) - x - 1 £ | x if
|, 0 < | x | < 1
But given that (n - 1)a12 - 2na 2 < 0
éì For x > 1 éì For x > 1
Þ å × å (a i - a j ) < 0 2 êï 2 êï 2
1£i < j £n êïí (9 - x ) - x - 1 ³ x êïí (9 - x ) ³ 2x + 1
êï For x < - 1 êï For x < - 1
which is true only, when atleast two roots are imaginary. êï 2 êï
( 9 - x ) - x - 1 ³ - x (9 - x 2 ) ³ 1
and êî Þ êî
êì For 0 < x < 1 êì For 0 < x < 1
l Ex. 48 Solve the inequation | a 2 x + a x + 2 - 1 | ³ 1 for all
êï ( 9 - x 2 ) - x - 1 £ x êï ( 9 - x 2 ) £ 2x + 1
values of a (a > 0, a ¹ 1). êïí êïí
êï For - 1 < x < 0 êï For - 1 < x < 0
Sol. Using a x = t , êï ( 9 - x 2 ) - x - 1 £ - x êï
ëî ëî (9 - x 2 ) £ 1
the given inequation can be written in the form
é ì For x > 1
| t 2 + a 2t - 1| ³ 1 …(i) ê ï 2 2
ê ï - ( 11 + 1) £ x £ ( 11 - 1)
Q a > 0 and a ¹ 1, then a x > 0 ê í 5 5
ï For x < - 1
ê
\ t >0 …(ii) ê ïî -2 2 £ x £ 2 2
Þ êì
Inequation (i) write in the forms, For 0 < x < 1
êï 2 2
t 2 + a 2t - 1 ³ 1 and t 2 + a 2t - 1 £ - 1 êï x £ - ( 11 + 1) and x ³ ( 11 - 1)
êí 5 5
-a2 - a4 + 8 - a 2 + ( a 4 + 8) êï For - 1 < x < 0
\ t £ ,t ³ êïî
2 2 ë x £ - 2 2 and x ³ 2 2
Chap 02 Theory of Equations 167

é ì x Îf Sol. Putting t = 3x in the original equation, then we obtain


ê í- 2 2 £ x < - 1
ê î
Þ êìï 2 ( 11 - 1) £ x < 1
êí 5 T-axis T-axis
0
êëïî x Îf 0

Hence, the original inequation consists of the intervals


(i) (ii)
2
-2 2 £ x < - 1 and ( 11 - 1) £ x < 1 .
5
é2 ö
Hence, x Î [ -2 2, - 1) È ê ( 11 - 1), 1÷ 0
ë5 ø T-axis T-axis
0

l Ex. 50 Find all values of ‘a’ for which the equation (iv)
(iii)
4 - a2 x - a + 3 = 0 has atleast one solution.
x
at 2 + 4 (a - 1)t + a > 1
Sol. Putting 2x = t > 0, then the original equation reduced in
Þ at 2 + 4 (a - 1)t + (a - 1) > 0 [t > 0, Q 3x > 0]
the form
t 2 - at - a + 3 = 0 …(i) This is possible in two cases. First the parabola
f (t ) = at 2 + 4(a - 1)t + (a - 1) opens upwards, with its
that the quadratic Eq. (i) should have atleast one positive
vertex (turning point) lying in the non-positive part of the
root (t > 0), then
T -axis, as shown in the following four figures.
Discriminant, D = ( -a )2 - 4 × 1 × ( -a + 3) ³ 0 \ a > 0 and sum of roots £ 0
Þ a 2 + 4a - 12 ³ 0 4( a - 1)
Þ - £ 0 and f (0) ³ 0
Þ ( a + 6) ( a - 2) ³ 0 2a
\ a > 0, a - 1 ³ 0 and a - 1 ³ 0
+ + Hence, a³1
–6 – 2

\ a Î ( -¥, - 6] È [2, ¥ )
If roots of Eq. (i) are t 1 and t 2 , then T-axis
0
ì t1 + t 2 = a
í
ît 1t 2 = 3 - a
For a Î ( - ¥, - 6] Second the parabola f (t ) opens upward, with its vertex
t 1 + t 2 < 0 and t 1t 2 > 0. Therefore, both roots are negative lying in positive direction of t, then
and consequently, the original equation has no solutions. 4( a - 1)
a > 0, - > 0 and D £ 0
For a Î[2, ¥ ) 2a
>
t 1 + t 2 > 0 and t 1t 2 < 0, consequently, atleast one of the Þ a > 0,(a - 1) < 0
roots t 1 or t 2 , is greater than zero. and 16(a - 1)2 - 4(a - 1)a £ 0
Thus, for a Î [2, ¥ ), the given equation has atleast one Þ a > 0,a < 1
solution. and 4(a - 1)(3a - 4 ) £ 0
4
l Ex. 51 Find all the values of the parameter a for which Þ a > 0,a < 1 and 1 £ a £
3
the inequality a 9 x + 4 (a - 1) 3 x + a > 1, is satisfied for all These inequalities cannot have simultaneously.
real values of x . Hence, a ³ 1 from Eq. (i).
#L Theory of Equations Exercise 1 :
Single Option Correct Type Questions
n This section contains 30 multiple choice questions. 8. If the roots of the quadratic equation
Each question has four choices (a), (b), (c) and (d) out of ( 4 p - p 2 - 5)x 2 - (2p - 1) x + 3p = 0 lie on either side of
which ONLY ONE is correct
unity, the number of integral values of p is
1. If a, b, c are real and a ¹ b, the roots of the equation (a) 1 (b) 2 (c) 3 (d) 4
2
2 (a - b ) x - 11 (a + b + c ) x - 3 (a - b ) = 0 are 9. Solution set of the equation
2 2
+x +6
(a) real and equal (b) real and unequal 32 x - 2×3x + 3 2 (x + 6 ) = 0 is
(c) purely imaginary (d) None of these
(a) { -3, 2 } (b) {6, - 1 } (c) { -2, 3 } (d) {1, - 6 }
2. The graph of a quadratic polynomial y = ax 2
10. Consider two quadratic expressions f ( x ) = ax 2 + bx + c
+ bx + c ; a, b, c Î R is as shown.
and g ( x ) = ax 2 + px + q (a, b, c , p , q Î R, b ¹ p ) such that
Y
their discriminants are equal. If f ( x ) = g ( x ) has a root
X
O x = a, then
(a) a will be AM of the roots of f ( x ) = 0 and g( x ) = 0
(b) a will be AM of the roots of f ( x ) = 0
(c) a will be AM of the roots of f ( x ) = 0 or g( x ) = 0
(d) a will be AM of the roots of g( x ) = 0

Which one of the following is not correct? 11. If x 1 and x 2 are the arithmetic and harmonic means of
(a) b 2 - 4ac < 0
c
(b) < 0 the roots of the equation ax 2 + bx + c = 0, the quadratic
a equation whose roots are x 1 and x 2 , is
(c) c is negative
æ bö (a) abx 2 + (b 2 + ac ) x + bc = 0
(d) Abscissa corresponding to the vertex is ç - ÷
è 2a ø
(b) 2abx 2 + (b 2 + 4ac ) x + 2bc = 0
3. There is only one real value of ‘a’ for which the (c) 2abx 2 + (b 2 + ac ) x + bc = 0
quadratic equation ax 2 + (a + 3) x + a - 3 = 0 has two (d) None of the above
positive integral solutions. The product of these two 12. f ( x ) is a cubic polynomial x 3 + ax 2 + bx + c such that
solutions is
f ( x ) = 0 has three distinct integral roots and f ( g ( x )) = 0
(a) 9 (b) 8 (c) 6 (d) 12
does not have real roots, where g ( x ) = x 2 + 2x - 5, the
4. If for all real values of a one root of the equation minimum value of a + b + c is
x 2 - 3ax + f (a ) = 0 is double of the other, f ( x ) is equal to (a) 504 (b) 532 (c) 719 (d) 764
(a) 2x (b) x 2 (c) 2 x 2 (d) 2 x
13. The value of the positive integer n for which the
n
5. A quadratic equation the product of whose roots x 1 and
quadratic equation å ( x + k - 1) ( x + k ) = 10n has
x 2 is equal to 4 and satisfying the relation k =1
x1 x2
+ = 2, is solutions a and a + 1 for some a, is
x1 - 1 x 2 - 1 (a) 7 (b) 11 (c) 17 (d) 25
(a) x 2 - 2 x + 4 = 0 (b) x 2 - 4 x + 4 = 0 2
14. If one root of the equation x - lx + 12 = 0 is even
(c) x 2 + 2 x + 4 = 0 (d) x 2 + 4 x + 4 = 0
prime, while x 2 + lx + m = 0 has equal roots, then m is
6. If both roots of the quadratic equation (a) 8 (b) 16 (c) 24 (d) 32
x 2 - 2ax + a 2 - 1 = 0 lie in ( - 2, 2), which one of the 15. Number of real roots of the equation
following can be [a ] ? (where [×] denotes the greatest x + x - (1 - x ) = 1 is
integer function)
(a) -1 (b) 1 (c) 2 (d) 3 (a) 0 (b) 1 (c) 2 (d) 3

7. If ( - 2, 7 ) is the highest point on the graph of 16. The value of 7 + 7 - 7 + 7 - K upto ¥ is


2
y = - 2x - 4ax + l, then l equals (a) 5 (b) 4
1 (c) 3 (d) 2
(a) 31 (b) 11 (c) -1 (d) -
3
Chap 02 Theory of Equations 169

17. For any real x, the expression 2 (k - x ) [x + x 2 + k 2 ] 24. The roots of the equation
2 2
cannot exceed - 15 - 15
(a + b ) x + (a - b ) x = 2a,
2 2
(a) k (b) 2k
(c) 3k 2 (d) None of these where a 2 - b = 1, are
2 (a) ± 2, ± 3 (b) ± 4, ± 14
x - 2x + 4
18. Given that, for all x Î R, the expression 2
lies (c) ± 3, ± 5 (d) ± 6, ± 20
x + 2x + 4
1 25. The number of pairs ( x , y ) which will satisfy the
between and 3, the values between which the
3 equation
9 ×32x + 6×3x + 4 x 2
- xy + y 2
= 4 ( x + y - 4 ), is
expression lies, are
9 ×32x - 6×3x + 4 (a) 1 (b) 2
3 (c) 4 (d) None of these
(a) -3 and 1 (b) and 2
2
26. The number of positive integral solutions of
(c) -1 and 1 (d) 0 and 2
x 4 - y 4 = 3789108 is
19. Let a , b, g be the roots of the equation (a) 0 (b) 1 (c) 2 (d) 4
( x - a ) ( x - b ) ( x - c ) = d , d ¹ 0, the roots of the equation 3
27. The value of ‘a’ for which the equation x + ax + 1 = 0
( x - a ) ( x - b ) ( x - g ) + d = 0 are 4 2
(a) a, b, d (b) b, c, d and x + ax + 1 = 0, have a common root, is
(c) a, b, c (d) a + d , b + d , c + d (a) a = 2 (b) a = - 2
(c) a = 0 (d) None of these
20. If one root of the equation ix 2 - 2 (1 + i ) x + 2 - i = 0 is
(3 - i ), where i = -1, the other root is 28. The necessary and sufficient condition for the equation
(a) 3 + i (b) 3 + -1
(1 - a 2 ) x 2 + 2ax - 1 = 0 to have roots lying in the
(c) -1 + i (d) -1 - i interval (0, 1), is
(a) a > 0 (b) a < 0
21. The number of solutions of |[x ] - 2x | = 4, where [x ] (c) a > 2 (d) None of these
denotes the greatest integer £ x is 29. Solution set of x - 1 - | x | < 0, is
(a) infinite (b) 4 (c) 3 (d) 2
é -1 + 5 ö
22. If x 2
+ x + 1 is a factor of ax 3
+ bx 2
+ cx + d , the real (a) ê -1, ÷ (b) [ -1, 1 ]
ë 2 ø
3 2
root of ax + bx + cx + d = 0 is
é -1 + 5 ù æ -1 + 5 ö
d d a (c) ê -1, ú (d) ç -1, ÷
(a) - (b) (c) (d) None of these ë 2 û è 2 ø
a a d
2
23. The value of x which satisfy the equation 30. If the quadratic equations ax + 2cx + b = 0 and
2
ax + 2bx + c = 0 (b ¹ c ) have a common root, a + 4b + 4c ,
(5x 2 - 8x + 3) - (5x 2 - 9 x + 4 ) = (2x 2 - 2x )
is equal to
- (2x 2 - 3x + 1), is (a) -2 (b) -1
(a) 3 (b) 2 (c) 0 (d) 1
(c) 1 (d) 0

#L Theory of Equations Exercise 2 :


More than One Correct Option Type Questions
n
This section contains 15 multiple choice questions. 32. If A, G and H are the arithmetic mean, geometric mean
Each question has four choices (a), (b), (c) and (d) out of
and harmonic mean between unequal positive integers.
which MORE THAN ONE may be correct.
Then, the equation Ax 2 - | G | x - H = 0 has
31. If 0 < a < b < c and the roots a , b of the equation (a) both roots are fractions
ax 2 + bx + c = 0 are non-real complex numbers, then (b) atleast one root which is negative fraction
(a) | a | = | b | (b) | a | > 1 (c) exactly one positive root
(c) | b | < 1 (d) None of these (d) atleast one root which is an integer
170 Textbook of Algebra

33. The adjoining graph of y = ax 2 + bx + c shows that 40. For which of the following graphs of the quadratic
expression f ( x ) = ax 2 + bx + c , the product ofabc is
Y
negative
Y Y

(a) (b)
X¢ X
(a,0) X X
O
Y¢ (b, 0) O O

(a) a < 0 Y
Y
(b) b 2 < 4ac
X
(c) c > 0 O
(d) a and b are of opposite signs (c) (d)

X
34. If the equation ax 2 + bx + c = 0 (a > 0) has two roots a O
and b such that a < - 2 and b > 2, then
(a) b 2 - 4ac > 0 (b) c < 0 41. If a, b Î R and ax 2
+ bx + 6 = 0, a ¹ 0 does not have two
(c) a + | b | + c < 0 (d) 4a + 2 | b | + c < 0 distinct real roots, the
35. If b 2 ³ 4ac for the equation ax 4 + bx 2 + c = 0, then all (a) minimum possible value of 3a + b is -2
(b) minimum possible value of 3a + b is 2
the roots of the equation will be real, if (c) minimum possible value of 6a + b is -1
(a) b > 0, a < 0, c > 0 (b) b < 0, a > 0, c > 0 (d) minimum possible value of 6a + b is 1
(c) b > 0, a > 0, c > 0 (d) b > 0, a < 0, c < 0
3
3 2
42. If x + 3x 2 - 9 x + l is of the form ( x - a ) 2 ( x - b ), then
36. If roots of the equation x + bx + cx - 1 = 0 from an
l is equal to
increasing GP, then (a) 27 (b) -27
(a) b + c = 0 (c) 5 (d) -5
(b) b Î ( -¥, - 3 ) 2
(c) one of the roots is 1 43. If ax + (b - c ) x + a - b - c = 0 has unequal real roots
(d) one root is smaller than one and one root is more than one for all c Î R, then
2 (a) b < 0 < a (b) a < 0 < b
37. Let f ( x ) = ax + bx + c , where a, b, c Î R, a ¹ 0. Suppose (c) b < a < 0 (d) b > a > 0
| f ( x )| £ 1, " x Î[0, 1], then
(a) | a | £ 8 (b) | b | £ 8
44. If the equation whose roots are the squares of the roots
(c) | c | £ 1 (d) |a | + |b | + |c | £ 17 of the cubic x 3 - ax 2 + bx - 1 = 0 is identical with the
given cubic equation, then
38. cos a is a root of the equation 25x 2 + 5x - 12 = 0,
(a) a = b = 0
-1 < x < 0, the value of sin 2a is (b) a = 0, b = 3
24 12
(a) (b) - (c) a = b = 3
25 25
24 20 (d) a, b are roots of x 2 + x + 2 = 0
(c) - (d)
25 25 45. If the equation ax 2 + bx + c = 0 (a > 0) has two real roots
39. If a, b, c Î R (a ¹ 0) and a + 2b + 4c = 0, then equation a and b such that a < - 2 and b > 2, which of the
2 following statements is/are true?
ax + bx + c = 0 has
(a) 4a - 2 | b | + c < 0
(a) atleast one positive root
(b) 9a - 3 | b | + c < 0
(b) atleast one non-integral root
(c) both integral roots (c) a - | b | + c < 0
(d) no irrational root (d) c < 0, b 2 - 4ac > 0
Chap 02 Theory of Equations 171

#L Theory of Equations Exercise 3 :


Passage Based Questions
n This section contains 6 passages. Based upon each of 52. y = f ( x ) is given by
the passage 3 multiple choice questions have to be x2
answered. Each of these questions has four choices (a), (a) y = x 2 - 8 (b) y = -2 2
2 2
(b), (c) and (d) out of which ONLY ONE is correct.
x2
(c) y = x 2 - 4 (d) y = - 2
Passage I 2
(Q. Nos. 46 to 48) 53. Minimum value of y = f ( x ) is
If G and L are the greatest and least values of the (a) -4 2 (b) -2 2
2x 2 - 3x + 2 (c) 0 (d) 2 2
expression , x Î R respectively. l
2x 2 + 3x + 2 54. Number of integral value of l for which lies between
2
46. The least value of G 100 + L100 is the roots of f ( x ) = 0, is
(a) 9 (b) 10 (c) 11 (d) 12
(a) 2100 (b) 3100 (c) 7100 (d) None of these
47. G and L are the roots of the equation Passage III
2
(a) 5 x - 26 x + 5 = 0 2
(b) 7 x - 50 x + 7 = 0 (Q. Nos. 55 to 57)
(c) 9 x 2 - 82 x + 9 = 0 (d) 11 x 2 - 122 x + 11 = 0 Let f (x ) = x 2 + bx + c and g (x ) = x 2 + b1 x + c1 .
48. If L2 < l < G 2 , l Î N , the sum of all values of l is Let the real roots of f ( x ) = 0 be a, b and real roots of
(a) 1035 (b) 1081 (c) 1225 (d) 1176 g ( x ) = 0 be a + k , b + k for same constant k. The least value
1 7
Passage II of f ( x ) is - and least value of g (x ) occurs at x = .
(Q. Nos. 49 to 51) 4 2

If roots of the equation x 4 - 12x 3 + cx 2 + dx + 81 = 0 are 55. The value of b1 is


positive. (a) -8 (b) -7 (c) -6 (d) 5

49. The value of c is 56. The least value of g ( x ) is


(a) -27 (b) 27 (c) -54 (d) 54 1 1 1
(a) -1 (b) - (c) - (d) -
50. The value of d is 2 3 4
(a) -27 (b) -54 (c) -81 (d) -108
57. The roots of f ( x ) = 0 are
51. Root of the equation 2cx + d = 0, is (a) 3, 4 (b) -3, 4
1 1 (c) - 3, - 4 (d) 3, - 4
(a) -1 (b) - (c) 1 (d)
2 2
Passage IV
Passage II (Q. Nos. 58 to 60)
(Q. Nos. 52 to 54) 2
If ax - bx + c = 0 have two distinct roots lying in the
In the given figure vertices of DABC lie on interval (0, 1); a, b, c Î N .
y = f ( x ) = ax 2 + bx + c. The DABC is right angled isosceles
triangle whose hypotenuse AC = 4 2 units. 58. The least value of a is
(a) 3 (b) 4
Y (c) 5 (d) 6
y = f(x) = ax2 + bx + c
59. The least value of b is
(a) 5 (b) 6
O (c) 7 (d) 8
X
A 90° C
60. The least value of log 5 abc is
B
(a) 1 (b) 2
(c) 3 (d) 4
172 Textbook of Algebra

Passage V Passage VI
(Q. Nos. 61 to 63) (Q. Nos. 64 to 66)
3 2
If 2x + ax + bx + 4 = 0 (a and b are positive real If a, b, g, d are the roots of the equation
x 4 + A x 3 + B x 2 + Cx + D = 0 such that ab = gd = k and
numbers) has three real roots.
A, B , C , D are the roots of x 4 - 2 x 3 + 4 x 2 + 6x - 21 = 0
61. The minimum value of a 3 is such that A + B = 0.
(a) 108 (b) 216 C
(c) 432 (d) 864 64. The value of is
A
62. The minimum value of b 3 is k k
(a) - (b) -k (c) (d) k
(a) 432 (b) 864 2 2
(c) 1728 (d) None of these 65. The value of (a + b ) ( g + d ) in terms of B and k is
(a) B - 2k (b) B - k (c) B + k (d) B + 2k
63. The minimum value of (a + b ) 3 is
(a) 1728 (b) 3456 66. The correct statement is
(a) C 2 = AD (b) C 2 = A 2D (c) C 2 = AD 2 (d) C 2 = ( AD ) 2
(c) 6912 (d) 864

#L Theory of Equations Exercise 4 :


Single Integer Answer Type Questions
2
n This section contains 10 questions. The answer to x - 3x + c
each question is a single digit integer, ranging from 0 72. If the maximum and minimum values of y = 2
x + 3x + c
to 9 (both inclusive).
1
67. The sum of all the real roots of the equation are 7 and respectively, the value of c is
7
| x - 2 | 2 + | x - 2 | - 2 = 0 is
68. The harmonic mean of the roots of the equation 73. Number of solutions of the equation
(5 + 2 ) x 2 - ( 4 + 5 ) x + 8 + 2 5 = 0 is x 2 - ( x - 1) 2 + ( x - 2) 2 = 5 is
69. If product of the real roots of the equation,
74. If a and b are the complex roots of the equation
x 2 - ax + 30 = 2 ( x 2 - ax + 45), a > 0, (1 + i ) x 2
+ (1 - i ) x - 2i = 0, where i = -1, the value of
is l and minimum value of sum of roots of the equation | a - b | is2

is m. The value of (m) (where ( × ) denotes the least integer


function) is 6 75. If a , b be the roots of the equation
æ 1ö æ 6 1 ö
çx + ÷ - çx + 6 ÷ - 2 4 x 2 - 16x + c = 0, c Î R such that 1 < a < 2 and 2 < b < 3,
è x ø è x ø
70. The minimum value of 3
is then the number of integral values of c, are
æ 1ö 3 1
çx + ÷ + x + 3
(for x > 0) è xø x 76. Let r , s and t be the roots of the equation
71. Let a, b, c , d are distinct real numbers and a, b are the 8x 3 + 1001x + 2008 = 0 and if
roots of the quadratic equation x 2 - 2cx - 5d = 0. If c and 99 l = (r + s ) 3 + (s + t ) 3 + (t + r ) 3 , the value of [l ] is
d are the roots of the quadratic equation (where [ × ] denotes the greatest integer function)
x 2 - 2ax - 5b = 0, the sum of the digits of numerical
values of a + b + c + d is
Chap 02 Theory of Equations 173

#L Theory of Equations Exercise 5 :


Matching Type Questions
n This section contains 4 questions. Questions 78 and 80 have three statements (A, B and C) given in Column I and
four statements (p, q, r and s) in Column II and questions 77 and 79 have three statements (A, B and C) given in
Column I and five statements (p, q, r, s and t) in Column II. Any given statement in Column I can have correct
matching with one or more statement(s) given in Column II.

77. Column I contains rational algebraic expressions and 79. Column I contains rational algebraic expressions and
Column II contains possible integers which lie in their Column II contains possible integers of a.
range. Match the entries of Column I with one or more
entries of the elements of Column II. Column I Column II

Column I Column II (A) ax 2 + 3x - 4 (p) 0


y= , x Î R and y Î R
3x - 4 x 2 + a
(A) x 2 - 2x + 4
y= , x ÎR (p) -2
x 2 + 2x + 4 (B) ax 2 + x - 2 1
y= , x Î R and y Î R (q)
2
2x + 4 x + 1 a + x - 2x 2
(B) y= , x ÎR (q) -1
x2 + 4 x + 2
(C) x 2 + 2x + a 3
2
y= , x Î R and y Î R (r)
x - 3x + 4 x 2 + 4 x + 3a
(C) y= , x ÎR (r) 2
x-3
(s) 5
(s) 3
(t) 7
(t) 8
80.
78.
Column I Column II
Column I Column II

(A) If a, b, c, d are four non-zero real (p) a + b + c = 0


(A) The equation x 3 - 6x 2 + 9x + l = 0 have (p) 0
numbers such that exactly one root is (1, 3), then |[ l + 1]| is
(d + a - b)2 + (d + b - c)2 = 0 and (where [ × ] denotes the greatest integer
function)
the roots of the equation
a (b - c)x 2 + b (c - a) x + c (a - b) = 0 (B) x 2 - lx - 2 (q) 1
are real and equal, then If - 3 < < 2, " x Î R , then
x2 + x + 1
(B) If the equation ax 2 + bx + c = 0 (q) a, b, c are in AP |[ l ]| is (where [ × ] denotes the greatest
and x 3 - 3x 2 + 3x - 1 = 0 have a integer function)
common real root, then
(C) If x 2 + lx + 1 = 0 and (r) 2
(C) Let a, b, c be positive real numbers (r) a, b, c are in GP (b - c) x 2 + (c - a) x + (a - b) = 0 have
such that the expression both the roots common, then |[ l - 1]|,
bx 2 + ( (a + c)2 + 4 b2 ) x + (a + c) (where [ × ] denotes the greatest integer
function)
is non-negative, " x Î R, then

(s) a, b, c are in HP (s) 3


174 Textbook of Algebra

#L Theory of Equations Exercise 6 :


Statement I and II Type Questions
n Directions (Q. Nos. 81 to 87) are Assertion-Reason 83. Statement-1 In the equation ax 2 + 3x + 5 = 0, if one
type questions. Each of these questions contains two root is reciprocal of the other, then a is equal to 5.
statements:
Statement-1 (Assertion) and Statement-2 (Reason) Statement-2 Product of the roots is 1.
Each of these questions also has four alternative 84. Statement-1 If one root of Ax 3
+ Bx 2 + Cx + D = 0,
choices, only one of which is the correct answer. You
have to select the correct choice as given below. A ¹ 0, is the arithmetic mean of the other two roots, then
(a) Statement-1 is true, Statement-2 is true; Statement-2 the relation 2B 3 + k 1 ABC + k 2 A 2 D = 0 holds good and
is a correct explanation for Statement-1 then (k 2 - k 1 ) is a perfect square.
(b) Statement-1 is true, Statement-2 is true; Statement-2 Statement-2 If a, b, c are in AP, then b is the arithmetic
is not a correct explanation for Statement-1
mean of a andc .
(c) Statement1 is true, Statement-2 is false
(d) Statement-1 is false, Statement-2 is true 85. Statement-1 If x , y, z be real variables satisfying
2 x + y + z = 6 and xy + yz + zx = 8, the range of variables
81. Statement-1 If the equation ( 4 p - 3) x
x , y and z are identical.
+ ( 4q - 3) x + r = 0 is satisfied by x = a, x = b and x = c
3 Statement-2 x + y + z = 6 and xy + yz + zx = 8 remains
(where a, b, c are distinct), then p = q = and r = 0.
4 same, if x , y, z interchange their positions.
3
Statement-2 If the quadratic equation 86. Statement-1 ax + bx + c = 0, where a, b, c Î R cannot
ax 2 + bx + c = 0 has three distinct roots, then a, b and c have 3 non-negative real roots.
are must be zero.
Statement-2 Sum of roots is equal to zero.
82. Statement-1 The equation
87. Statement-1 The quadratic polynomial
x 2 + (2m + 1) x + (2n + 1) = 0, where m, n Î I , cannot have
y = ax 2 + bx + c (a ¹ 0 and a, b, c Î R ) is symmetric about
any rational roots.
the line 2ax + b = 0.
Statement-2 The quantity (2m + 1) 2 - 4 (2n + 1), where
Statement-2 Parabola is symmetric about its axis of
m, n Î I , can never be perfect square. symmetry.

#L Theory of Equations Exercise 7 :


Subjective Type Questions
n
In this section, there are 24 subjective questions. 89. For what values of m, then equation
88. For what values of m, the equation 2x 2 - 2 (2m + 1) x + m (m + 1) = 0 has (m Î R )
(1 + m ) x 2 - 2 (1 + 3m ) x + (1 + 8m ) = 0 has (m Î R ) (i) both roots are smaller tha 2?
(ii) both roots are greater than 2?
(i) both roots are imaginary?
(iii) both roots lie in the interval (2, 3)?
(ii) both roots are equal?
(iv) exactly one root lie in the interval (2, 3)?
(iii) both roots are real and distinct?
(v) one root is smaller than 1 and the other root is
(iv) both roots are positive? greater than 1?
(v) both roots are negative? (vi) one root is greater than 3 and the other root is
(vi) roots are opposite in sign? smaller than 2?
(vii) roots are equal in magnitude but opposite in sign? (vii) atleast one root lies in the interval (2, 3)?
(viii) atleast one root is positive? (viii) atleast one root is greater than 2?
(ix) atleast one root is negative? (ix) atleast one root is smaller than 2?
(x) roots are in the ratio 2 : 3? (x) roots a and b, such that both 2 and 3 lie between a
and b?
Chap 02 Theory of Equations 175

90. If r is the ratio of the roots of the equation 101. Find all values of a for which the inequation
2 2
(r + 1) b 2 2
ax 2 + bx + c = 0, show that = . 4 x + 2 (2a + 1) 2 x + 4a 2 - 3 > 0 is satisfied for any x .
r ac
1 1 1 æ| x + 4 | - | x | ö
91. If the roots of the equation + = are equal 102. Solve the inequation log x 2 + 2 x - 3 ç ÷ > 0.
x + p x +q r è x -1 ø
in magnitude but opposite in sign, show that p + q = 2r
æ p2+q2 ö 103. Solve the system | x 2 - 2x | + y = 1, x 2 + | y | = 1.
and that the product of the roots is equal to ç - ÷.
è 2 ø 104. If a , b, g are the roots of the cubic x 3 - px 2 + qx - r = 0.
Find the equations whose roots are
92. If one root of the quadratic equation ax 2 + bx + c = 0 is
1 1 1
equal to the nth power of the other, then show that (i) bg + , ga + , ab +
1 1 a b g
n n +1 n n +1
(ac ) + (a c ) + b = 0. (ii) ( b + g - a ), ( g + a - b ), (a + b - g )
93. If a , b are the roots of the equation ax 2
+ bx + c = 0 and Also, find the value of ( b + g - a ) ( g + a - b ) (a + b - g ).
2
g , d those of equation lx + mx + n = 0, then find the 105. If A 1 , A 2 , A 3 , ..., A n , a1 , a 2 , a 3 , ..., an , a, b, c Î R, show
equation whose roots are ag + bd and ad + bg . that the roots of the equation
94. Show that the roots of the equation A 12 A 22 A 32 A n2
2 2 2 2
+ + +K+
(a - bc ) x + 2 (b - ac ) x + c - ab = 0 x - a1 x - a 2 x - a 3 x - an
are equal, if either b = 0 or a 3 + b 3 + c 3 - 3abc = 0. 2 2
= ab + c x + ac are real.
95. If the equation x 2 - px + q = 0 and x 2 - ax + b = 0 have 106. For what values of the parameter a the equation
a common root and the other root of the second x 4 + 2ax 3 + x 2 + 2ax + 1 = 0 has atleast two distinct
equation is the reciprocal of the other root of the first, negative roots?
then prove that (q - b ) 2 = bq ( p - a ) 2 .
107. If [x ] is the integral part of a real number x. Then solve
96. If the equation x 2 - 2px + q = 0 has two equal roots, [2x ] - [x + 1] = 2x .
then the equation (1 + y ) x 2 - 2 ( p + y ) x + (q + y ) = 0 108. Prove that for any value of a, the inequation (a 2 + 3)
will have its roots real and distinct only, when y is x 2 + (a + 2) x - 6 < 0 is true for atleast one negative x .
negative and p is not unity.
log x (x + 3 )2
109. How many real solutions of the equation
97. Solve the equation x = 16. 6x 2 - 77 [x ] + 147 = 0, where [x ] is the integral part of x ?
98. Solve the equation
101 110. If a , b are the roots of the equation x 2 - 2x - a 2 + 1 = 0
x 2 - 2 x +1 x 2 - 2 x -1
(2 + 3 ) + (2 - 3 ) = . and g , d are the roots of the equation
10 (2 - 3 )
æ x ö
2 x 2 - 2 (a + 1) x + a (a - 1) = 0, such that a , b Î ( g , d ), find
99. Solve the equation x 2 + ç ÷ = 8. the value of ‘a’.
è x - 1ø
100. Solve the equation 111. If the equation x 4 + px 3 + qx 2 + rx + 5 = 0 has four
( x + 8) + 2 ( x + 7 ) + ( x + 1) - ( x + 7 ) = 4. positive real roots, find the minimum value of pr.

#L Theory of Equations Exercise 8 :


Questions Asked in Previous 13 Years' Exam
n
This section contains questions asked in IIT-JEE, 113. If S is a set of P( x ) is polynomial of degree £ 2 such that
AIEEE, JEE Main & JEE Advanced from year 2005 P(0) = 0, P(1) = 1, P ¢( x ) > 0, "x Î(0, 1), then [IIT-JEE 2005, 3M]
to year 2017.
(a) S = 0
112. If a , b are the roots of ax 2 + bx + c = 0 and a + b , (b) S = ax + (1 - a ) x 2, " a Î ( 0, ¥ )
a 2 + b 2 , a 3 + b 3 are in GP, where D = b 2 - 4ac , then (c) S = ax + (1 - a ) x 2, " a Î R
[IIT-JEE 2005, 3M] (d) S = ax + (1 - a ) x 2, " a Î ( 0, 2 )
(a) D ¹ 0 (b) bD = 0 (c) cb ¹ 0 (d) cD = 0
176 Textbook of Algebra

114. If the roots of x 2 - bx + c = 0 are two consecutive 122. If the difference between the roots of the equation
2
integers, then b - 4c is [AIEEE 2005, 3M]
x 2 + ax + 1 = 0 is less than 5, the set of possible values
(a) 1 (b) 2 of a, is [AIEEE 2007, 3M]
(c) 3 (d) 4 (a) ( - 3, 3 ) (b) ( -3, ¥ )
(c) (3, ¥ ) (d) ( -¥, - 3 )
115. If the equation an x n + an - 1 x n - 1 + ... + a1 x = 0, a1 ¹ 0,
123. Let a, b, c , p , q be real numbers. Suppose a , b are roots of
n ³ 2, has a positive root x = a, then the equation 1
nan x n - 1 + (n - 1) an - 1 x n - 2 + K + a1 = 0 has a positive the equation x 2 + 2px + q = 0 and a , are the roots of
b
root, which is [AIEEE 2005, 3M]
the equation ax 2 + 2bx + c = 0, where b 2 Ï {-1, 0, 1}.
(a) greater than or equal to a
(b) equal to a Statement-1 ( p 2 - q ) (b 2 - ac ) ³ 0 and
(c) greater than a
Statement-2 b ¹ pa or c ¹ qa [IIT-JEE 2008, 3M]
(d) smaller than a
(a) Statement-1 is true, Statement-2, is true; Statement-2 is a
116. If both the roots of the quadratic equation correct explanation for Statement-1
x 2 - 2kx + k 2 + k - 5 = 0 (b) Statement-1 is true, Statement-2 is true; Statement-2 is
are less than 5, k lies in the interval [AIEEE 2005, 3M] not a correct explanation for Statement-1
(a) ( - ¥, 4 ) (b) [ 4, 5 ] (c) Statement-1 is true, Statement-2 is false
(c) (5, 6 ) (d) (6, ¥ ) (d) Statement-1 is false, Statement-2 is true
117. Let a and b be the roots of equation x 2 - 10cx - 11d = 0 124. The quadratic equation x 2 - 6x + a = 0 and
and those of x 2 - 10ax - 11b = 0 are c and d, the value of x 2 - cx + 6 = 0 have one root in common. The other
a + b + c + d , when a ¹ b ¹ c ¹ d , is IIT-JEE 2006, 6M] roots of the first and second equations are integers in
the ratio 4 : 3. The common root is [AIEEE 2008, 3M]
118. Let a, b, c be the sides of a triangle. No two of them are (a) 4 (b) 3 (c) 2 (d) 1
equal and l Î R . If the roots of the equation
x 2 + 2 (a + b + c ) x + 3 l (ab + bc + ca ) = 0 are real, then
125. How many real solutions does the equation
[IIT-JEE 2006, 3M]
x 7 + 14 x 5 + 16x 3 + 30x - 560 = 0 have? [AIEEE 2008, 3M]
4 5 (a) 1 (b) 3 (c) 5 (d) 7
(a) l < (b) l <
3 3 126. Suppose the cubic x 3 - px + q = 0 has three distinct real
æ1 5ö æ 4 5ö
(c) l Î ç , ÷ (d) l Î ç , ÷ roots, where p > 0 and q < 0. Which one of the following
è3 3ø è 3 3ø
holds? [AIEEE 2008, 3M]
119. All the values of m for which both roots of the equation æ pö p
x 2 - 2mx + m 2 - 1 = 0 are greater than - 2 but less than (a) The cubic has minima at ç - ÷ and maxima at
è 3ø 3
4, lie in the interval [AIEEE 2006, 3M] p æ pö
(a) - 2 < m < 0 (b) m > 3 (b) The cubic has minima at both and ç - ÷
3 è 3ø
(c) - 1 < m < 3 (d) 1 < m < 4
p æ pö
120. If the roots of the quadratic equation x 2 + px + q = 0 are (c) The cubic has maxima at both and ç - ÷
3 è 3ø
tan 30° and tan 15°, respectively, the value of 2 + q - p is æ
p pö
(a) 2 (b) 3 [AIEEE 2006, 3M] (d) The cubic has minima at and maxima at ç - ÷
3 è 3ø
(c) 0 (d) 1
121. Let a, b be the roots of the equation x 2 - px + r = 0 and 127. The smallest value of k , for which both roots of the
a equation x 2 - 8kx + 16 (k 2 - k + 1) = 0 are real, distinct
, 2 b be the roots of the equation x 2 - qx + r = 0. The
2 and have value at least 4 , is [IIT-JEE 2009, 4M]
value of r is [IIT-JEE 2007, 3M] (a) 6 (b) 4 (c) 2 (d) 0
2
2
(a) ( p - q ) (2q - p ) (b)
2
(q - p ) (2 p - q ) 128. If the roots of the equation bx + cx + a = 0 be
9 9 imaginary, then for all real values of x, the expression
2 2
(c) (q - 2 p ) (2q - p ) (d) (2 p - q ) (2q - p ) 3b 2 x 2 + 6bcx + 2c 2 , is [AIEEE 2009, 4M]
9 9
(a) less than ( - 4ab ) (b) greater than 4ab
(c) less than 4ab (d) greater than ( - 4ab )
Chap 02 Theory of Equations 177

129. Let p and q be real numbers such that p ¹ 0, p 3 ¹ - q . If 137. The equation e sin x
- e - sin x - 4 = 0 has [AIEEE 2012, 4M]
a and b are non-zero complex numbers satisfying (a) exactly one real root
a + b = - p and a 3 + b 3 = q, a quadratic equation (b) exactly four real roots
a b (c) infinite number of real roots
having and as its roots, is [IIT-JEE 2010, 3M] (d) no real roots
b a
(a) ( p 3 + q ) x 2 - ( p 3 + 2q ) x + ( p 3 + q ) = 0 138. If the equations x 2 + 2x + 3 = 0 and ax 2 + bx + c = 0,
(b) ( p 3 + q ) x 2 - ( p 3 - 2q ) x + ( p 3 + q ) = 0 a, b, c Î R have a common root, then a : b : c is
[JEE Main 2013, 4M]
(c) ( p 3 - q ) x 2 - (5 p 3 - 2q ) x + ( p 3 - q ) = 0 (a) 3 : 2 : 1 (b) 1 : 3 : 2 (c) 3 : 1 : 2 (d) 1 : 2 : 3
(d) ( p 3 - q ) x 2 - (5 p 3 + 2q ) x + ( p 3 - q ) = 0 139. If a Î R and the equation
130. Consider the polynomial f ( x ) = 1 + 2x + 3x 2
+ 4 x 3 . Let -3 ( x - [x ]) 2 + 2 ( x - [x ]) + a 2 = 0 (where [ × ] denotes
s be the sum of all distinct real roots of f ( x ) and let the greatest integer function) has no integral solution,
t = | s |, real number s lies in the interval [IIT-JEE 2010, 3M] then all possible values of a lie in the interval
[JEE Main 2014, 4M]
æ 1 ö æ 3ö æ 3 1ö æ 1ö
(a) ç - , 0 ÷ (b) ç - 11, ÷ (c) ç - , - ÷ (d) ç 0, ÷ (a) ( -2, - 1 )
è 4 ø è 4ø è 4 2ø è 4ø
(b) ( -¥, - 2 ) È (2, ¥ )
131. Let a and b be the roots of x 2 - 6x - 2 = 0, with a > b. If (c) ( -1, 0 ) È ( 0, 1 )
(d) (1,2)
a10 - 2a 8
an = a n - b n for n ³ 1, the value of is 140. Let a, b be the roots of the equation px 2 + qx + r = 0,
2a 9
[IIT-JEE 2011, 3 and JEE Main 2015,4M] 1 1
p ¹ 0. If p , q , r are in AP and + = 4 , the value of
(a) 1 (b) 2 (c) 3 (d) 4 a b
132. A value of b for which the equations | a - b |, is [JEE Main 2014, 4M]
2 2 34 2 13
x + bx - 1 = 0 x + x + b = 0 (a) (b)
9 9
have one root in common, is [IIT-JEE 2011, 3M] 61 2 17
(a) - 2 (b) - i 3, i = -1 (c) (d)
9 9
(c) i 5, i = -1 (d) 2
141. Let a Î R and let f : R ® R be given by
133. The number of distinct real roots of f ( x ) = x 5 - 5x + a. Then, [JEE Advanced 2014, 3M]
4 3 2
x - 4x + 12x + x - 1 = 0 is [IIT-JEE 2011, 4M] (a) f ( x ) has three real roots, if a > 4
(b) f ( x ) has only one real root, if a > 4
134. Let for a ¹ a1 ¹ 0, f ( x ) = ax 2 + bx + c , (c) f ( x ) has three real roots, if a < - 4
g ( x ) = a1 x 2 + b1 x + c 1 and p ( x ) = f ( x ) - g ( x ). If p ( x ) = 0 (d) f ( x ) has three real roots, if -4 < a < 4
only for x = ( - 1) and p ( - 2) = 2, the value of p (2) is 142. The quadratic equation p ( x ) = 0 with real coefficients
[AIEEE 2011, 4M]
(a) 18 (b) 3 (c) 9 (d) 6 has purely imaginary roots. Then, p ( p ( x )) = 0 has
[JEE Advanced 2014, 3M]
135. Sachin and Rahul attempted to solve a quadratic
(a) only purely imaginary roots
equation. Sachin made a mistake in writing down the
(b) all real roots
constant term and ended up in roots ( 4, 3). Rahul made a (c) two real and two purely imaginary roots
mistake in writing down coefficient of x to get roots (d) neither real nor purely imaginary roots
(3, 2). The correct roots of equation are [AIEEE 2011, 4M]
(a) - 4, - 3 (b) 6, 1 (c) 4, 3 (d) - 6, - 1 143. Let S be the set of all non-zero real numbers a such that
the quadratic equation ax 2 - x + a = 0 has two distinct
136. Let a(a ) and b(a ) be the roots of the equation
real roots x 1 and x 2 satisfying the inequality
( 3 (1 + a ) - 1)x 2 + ( (1 + a ) - 1)x + ( 6 (1 + a ) - 1) = 0, | x 1 - x 2 | < 1.
where a > - 1, then lim a (a ) and lim b(a ), are Which of the following intervals is (are) a subset(s) of S ?
a ® 0+ a ® 0+
[JEE Advanced 2015, 4M]
[IIT-JEE 2012, 3M]
æ 5ö æ 1ö æ 1 1 ö æ 1 ö
(a) ç - ÷ and 1 (b) ç - ÷ and ( -1 ) (a) ç - , - ÷ (b) ç - , 0 ÷
è 2ø è 2ø è 2 5ø è 5 ø
æ 1 ö æ 1 1ö
æ 7ö æ 9ö (c) ç 0, ÷ (d) ç , ÷
(c) ç - ÷ and 2 (d) ç - ÷ and 3 è 5ø è 5 2ø
è 2ø è 2ø
178 Textbook of Algebra

144. The sum of all real values of x satisfying the equation (a) 2 (sec q - tan q) (b) 2 sec q
2 (c) - 2 tan q (d) 0
+ 4 x - 60
( x 2 - 5x + 5) x = 1 is [JEE Main 2016, 4M]
146. If for a positive integer n, the quadratic equation
(a) 6 (b) 5 (c) 3 (d) -4
p p x ( x + 1) + ( x + 1) ( x + 2) K + ( x + n - 1) ( x + n ) = 10n has
145. Let - < q < - . Suppose a 1 and b 1 are the roots of two consecutive integral solutions, then n is equal to
6 12
equation x 2 - 2x sec q + 1 = 0 and a 2 and b 2 are the roots [JEE Main 2017, 4M]
of the equation x 2 + 2x tan q - 1 = 0. If a 1 > b 1 and (a) 11 (b) 12
a 2 > b 2 , then a 1 + b 2 equals [JEE Advanced 2016, 3M] (c) 9 (d) 10

Answers
Exercise for Session 1 æ 7 - 33 ö æ 7 + 33 ö
89. (i) m Î ç -¥ , ÷ (ii) m Î ç , ¥ ÷ (iii) m Î f
1.(b) 2. (c) 3. (a) 4. (b) 5. (a) 6. (a) è 2 ø è 2 ø
7.(c) 8. (b) 9. (c) 10. (d) 11. (b)
æ 7 - 33 11 - 73 ö æ 7 + 33 11 + 73 ö
(iv) m Îç , ÷ Èç , ÷
Exercise for Session 2 è 2 2 ø è 2 2 ø
1.(a) 2. (c) 3. (b) 4. (a) 5.(d) 6. (c) æ 7 - 33 7 + 33 ö
7. (c) 8. (b) 9. (c) 10. (a) (v) m Î (0, 3) (vi) m Î ç , ÷
è 2 2 ø
Exercise for Session 3 æ 7 - 33 11 - 73 ö æ 7 + 33 11 + 73 ö
(vii) m Î ç , ÷Èç , ÷
1.(a) 2. (b) 3. (c) 4. (c) 5.(d) 6. (c) è 2 2 ø è 2 2 ø
7.(c) 8. (a) 9. (a) 10. (d) æ 7 - 33 7 + 33 ö æ 7 + 33 ö
(viii) m Î ç , ÷ Èç , ¥÷
Exercise for Session 4 è 2 2 ø è 2 ø
2. (c) 3. (c) 4. (d) 5.(a) 6. (d) æ 7 - 33 ö æ 7 - 33 7 + 33 ö
1.(c) (ix) m Î ç -¥ , ÷ Èç , ÷
7.(d) 8. (c) 9. (b) 10. (d) è 2 ø è 2 2 ø
Exercise for Session 5 æ 11 - 73 7 + 33 ö
(x) m Î ç , ÷
1.(a) 2. (a) 3. (b) 4. (c) 5.(c) 6. (b) è 2 2 ø
7.(a) 8. (b) 9. (b) 10. (d) 93. a2l 2x 2 - ablmx + (b2 - 2ac) ln + (m2 - 2ln) ac = 0
Chapter Exercises 97. x Î f
1.(b) 2. (b) 3. (b) 4. (c) 5. (a) 6. (a) 98. x1 = 1 + 1 + log2 + = 1 - 1 + log2 +
3 10 , x2 3 10
7.(c) 8. (b) 9. (c) 10. (a) 11. (b) 12. (c)
13.(b) 14. (b) 15. (b) 16. (c) 17. (b) 18. (b) 99. x1 = 2, x2 = - 1 + 3 and x3 = - 1 - 3
19.(c) 20. (d) 21. (b) 22. (a) 23. (c) 24. (b)
100. x1 = 2
25.(a) 26. (a) 27. (b) 28. (c) 29. (a) 30. (c)
æ 3 ö
31.(a,b) 32.(b,c) 33.(a,d) 34.(a,b,c,d) 35. (b,d) 36. (a,b,c,d) 101. a Î (-¥ , - 1) È ç , ¥÷
37.(a, b,c,d) 38. (a,c) 39. (a,b) 40. (a,b,c,d) 41. (a,c) 42. (b,c) è 2 ø
43.(c,d) 44.(a,c,d) 45.(a,c,d) 102. x Î (-1 - 5 , - 3) È ( 5 - 1, 5)
46.(d) 47. (b) 48. (d) 49.(d) 50.(d) 51. (c)
52.(b) 53. (b) 54. (c) 55. (b) 56. (d) 57. (a) æ1 - 5 1 - 5 ö
103. The pairs (0, 1), (1, 0), ç , ÷ are solutions of the
58.(c) 59. (a) 60. (b) 61. (c) 62. (b) 63. (c) è 2 2 ø
64.(d) 65. (a) 66. (b) 67. (4) 68. (4) 69. (9) original system of equations.
70. (6) 71. (3) 72. (4) 73. (2) 74. (5) 75.(3) 104. (i) ry3 - q (r + 1) y2 + p (r + 1)2 y - (r + 1)3 = 0
76.(7) 77. (A) ® (r,s), (B) ® (p,q,r,s,t), (C) ® (p,q,t)
(ii) y3 - py2 + (4 q - p2 ) y + (8r - 4 pq + p3 ) = 0 and
78. (A) ® (q,r,s), (B) ® (p), (C) ® (q)
4 pq - p3 - 8r
79. (A) ® (q,r,s,t), (B) ® (q,r), (C) ® (p,q)
æ3 ö
80.(A) ® (p,q,r,s), (B) ® (p,q), (C) ® (s) 81.(d) 82. (a) 106. a Î ç , ¥ ÷ 107. x1 = - 1, x2 = - 1/ 2 109. Four
è4 ø
83. (a) 84. (a) 85. (a) 86. (d) 87.(a)
110. a Î æç - , 1ö÷
1
111. 80 112. (d) 113. (d) 114. (a)
88. (i) m Î (0, 3) (ii) m = 0, 3 è 4 ø
(iii) m Î (-¥ , 0) È (3, ¥ ) (iv) m Î (-¥ , - 1) È [ 3, ¥ ) 115. (d) 116. (a) 117. 1210 118.(a) 119. (c) 120. (b)
121. (d) 122. (a) 123. (b) 124. (c) 125. (a) 126. (d)
(v) m Î f (vi) m Î (-1, - 1/ 8)
127. (c) 128. (d) 129. (b) 130. (c) 131. (c) 132. (b)
(vii) m = - 1/ 3 (viii) m Î (-¥ , - 1) È (-1, - 1/ 8) È [ 3, ¥ ) 133.(2) 134. (a) 135. (b) 136. (b) 137. (d) 138. (d)
81 ± 6625 139. (c) 140. (b) 141.(b,d) 142. (d) 143. (a, d)
(ix) m Î (-1, - 1/ 8) (x) m =
32 144. (c) 145. (c) 146. (a)
5. Q x1x 2 = 4 ...(i)

Solutions and



or
x1
+
x2
x1 − 1 x 2 − 1
=2

2 x1x 2 − x1 − x 2 = 2 ( x1x 2 − x1 − x 2 + 1 )
8 − x1 − x 2 = 2 ( 4 − x1 − x 2 + 1 )
x1 + x 2 = 2
[from Eq. (i)]
…(ii)
1. We have, From Eqs. (i) and (ii), required equation is
2 (a − b ) x 2 − 11 (a + b + c ) x − 3 (a − b ) = 0 x 2 − ( x1 + x 2 ) x + x1x 2 = 0
∴ D = { − 11 (a + b + c )} 2 − 4 ⋅ 2 (a − b ) ⋅ ( −3 ) (a − b ) or x 2 − 2x + 4 = 0
= 121 (a + b + c ) 2 + 24 (a − b ) 2 > 0 6. Let f ( x ) = x 2 − 2ax + a 2 − 1
Therefore, the roots are real and unequal. Now, four cases arise:
2. Here, a < 0 Case I D ≥ 0
Cut-off Y -axis, x = 0
⇒ y =c < 0 [from graph]
∴ c<0
x -coordinate of vertex > 0
X
b –2 α β 2
⇒ − >0
2a
b ⇒ ( − 2a ) 2 − 4 ⋅ 1 (a 2 − 1 ) ≥ 0
⇒ <0
a ⇒ 4≥0
But a<0 ∴ a ∈R
∴ b>0 Case II f ( − 2 ) > 0
and y-coordinate of vertex < 0 ⇒ 4 + 4a + a 2 − 1 > 0
D D
⇒ − <0 ⇒ >0 ⇒ a 2 + 4a + 3 > 0
4a 4a
∴ D<0 [Q a < 0 ] ⇒ (a + 1 ) (a + 3 ) > 0
i.e. b 2 − 4ac < 0 ∴ a ∈ ( − ∞, − 3 ) ∪ ( − 1, ∞ )
c Case III f (2 ) > 0
∴ >0 [Qc < 0 , a < 0 ]
a ⇒ 4 − 4a + a 2 − 1 > 0
(a + 3 ) ⇒ a 2 − 4a + 3 > 0
3. Sum of the roots = − =I+ [let]
a ⇒ (a − 1 ) (a − 3 ) > 0
 3 
∴ a = − +  …(i) ∴ a ∈ ( − ∞, 1 ) ∪ (3, ∞ )
 I + 1 Case IV − 2 < x-coordinate of vertex < 2
a −3 ⇒ − 2 < 2a < 2
Product of the roots = αβ = =I+ +2 …(ii)
a ∴ a ∈ ( − 1, 1 )
and D = (a + 3 ) 2 − 4a (a + 3 ) Combining all cases, we get a ∈ ( − 1, 1 )
9 Hence, [a ] = − 1, 0
= + {( I + − 2 ) 2 − 12 } [from Eq. (i)]
(I + 1)2  − 4a 
7. We have, −   = −2
D must be perfect square, then I +
=6  2 (− 2)
From Eq. (ii), ⇒ a =2
+ ∴ y = − 2x 2 − 8x + λ
Product of the roots = I + 2 =6 + 2 =8 …(i)
4. Let α be one root of Since, Eq. (i) passes through points ( − 2, 7 )
x 2 − 3ax + f (a ) = 0 ∴ 7 = − 2 (− 2)2 − 8 (− 2) + λ
⇒ α + 2 α = 3a ⇒ 3 α = 3a ⇒ 7 = − 8 + 16 + λ
⇒ α =a …(i) ∴ λ = −1
and α ⋅ 2 α = f (a ) 8. Since, the coefficient of n 2 = ( 4p − p 2 − 5) < 0
⇒ f (a ) = 2 α 2= 2a 2 [using Eq. (i)] Therefore, the graph is open downward.
⇒ f (x ) = 2x 2 According to the question, 1 must lie between the roots.
180 Textbook of Algebra

Hence, f (1) > 0 Q g( x ) = ( x + 1 ) 2 − 6 ≥ − 6


⇒ 4p − p − 5 − 2p + 1 + 3p > 0
2
∴ α 3 ≤ − 7, α 2 ≤ − 8, α 1 ≤ − 9
⇒ − p + 5p − 4 > 0
2
∴ a + b + c ≥ 719
⇒ p 2 − 5p + 4 < 0 ∴Minimum value of a + b + c is 719.
Q α1 + α 2 + α 3 = − a
⇒ ( p − 4) ( p − 1) < 0
⇒ − a ≤ − 24
⇒ 1<p<4
⇒ a ≥ 24
∴ p = 2, 3
α 1α 2 + α 2 α 3 + α 3α 1 = b
Hence, number of integral values of p is 2.
2 2 ⇒ b ≥ 191
+ x+ 6
9. We have, 3 2 x − 2 ⋅ 3 x + 3 2 ( x + 6) = 0 and α 1α 2 α 3 = − c
2
⇒ (3 x − 3 x + 6 ) 2 = 0 ⇒ − c ≤ − 504
2 ⇒ c ≥ 504
⇒ 3x − 3x + 6 = 0
2
∴ a + b + c ≥ 719
⇒ 3x = 3x + 6 ⇒ x 2 = x + 6 Hence, minimum value of a + b + c is 719.
⇒ x2 − x − 6 = 0 n
13. Q ∑ ( x + k − 1) ( x + k ) = 10n
⇒ (x − 3) (x + 2) = 0 k =1
∴ x = { − 2, 3 } n
10. Given, b 2 − 4ac = p 2 − 4aq …(i) ⇒ ∑ x 2 + x (2k − 1) + (k − 1)k = 10n
k =1
and f ( x ) = g( x )
⇒ nx 2 + x (1 + 3 + 5 +…+ (2n − 1 ))
⇒ ax 2 + bx + c = ax 2 + px + q
⇒ (b − p ) x = q − c + ( 0 + 1 ⋅ 2 + 2 ⋅ 3 + 3 ⋅ 4 +…+ (n − 1 )n ) = 10n
q −c
∴ x= =α [given] …(ii) n
⇒ nx 2 + x ⋅ (1 + 2n − 1 )
b−p 2
From Eq. (i), we get  n (n + 1 ) (2n + 1 ) n(n + 1 )
+ −  = 10n
(b + p ) (b − p ) + 4a (q − c ) = 0  6 2 
⇒ (b + p ) (b − p ) + 4aα (b − p ) = 0 [from Eq. (ii)] n (n 2 − 1 )
(b + p ) ⇒ nx 2 + n 2x + = 10n
or α=− [Qb ≠ p] 3
4a
(n 2 − 31 )
 b  p ⇒ x 2 + nx + =0 [dividing by n]
−  + −  3
 a  a 
= D
4 Q (α + 1 ) − α =
Sum of the roots of ( f (x ) = 0)  1
+ Sum of the roots of (g (x ) = 0) 1= D
= 
4 ⇒ D =1
= AM of the roots of f ( x ) = 0 (n − 31 )
2
⇒ n2 − 4 ⋅1 ⋅ =1
and g( x ) = 0 3
11. Let α and β be the roots of ax 2 + bx + c = 0. ⇒ 3n 2 − 4n 2 + 124 = 3
α+β b ⇒ n 2 = 121
∴ x1 = =−
2 2a ∴ n = 11
c
2⋅ 14. Since, 2 is only even prime.
2αβ a 2c
and x2 = = =− Therefore, we have
α +β −b b
2 2 + λ ⋅ 2 + 12 = 0
a
∴The required equation is ⇒ λ =8
 b   2c   2bc ∴ x 2 + λx + µ = 0
x 2 −  −  +  −   x + =0
 2a   b   2ab ⇒ x 2 + 8x + µ = 0 …(i)
i.e. 2abx + (b + 4ac ) x + 2bc = 0
2 2
But Eq. (i) has equal roots.
12. Let α 1, α 2 and α 3 be the roots of f ( x ) = 0, such that ∴ D=0
α1 < α 2 < α 3 ⇒ 8 − 4 ⋅1 ⋅µ = 0
2

and g( x ) can take all values from [ − 6, ∞ ). ⇒ µ = 16


Chap 02 Theory of Equations 181

15. We have, x + x − (1 − x ) = 1 9 ⋅ 3 2 x + 6 ⋅ 3 x + 4 (3 x + 1 ) 2 + 2 ⋅ 3 x + 1 + 4
Let y = =
9 ⋅ 3 2 x − 6 ⋅ 3 x + 4 (3 x + 1 ) 2 − 2 ⋅ 3 x + 1 + 4
⇒ x − 1 − x =1 − x
t 2 + 2t + 4
On squaring both sides, we get = , where t = 3 x + 1
t 2 − 2t + 4
x − 1 − x =1 + x −2 x
⇒ (y − 1 ) t 2 − 2 (y + 1 ) t + 4 (y − 1 ) = 0
⇒ − 1 − x =1−2 x By the given condition, for every t ∈ R,
1
Again, squaring on both sides, we get <y <3 …(i)
3
1 − x = 1 + 4x − 4 x
But t = 3x + 1 > 0
4 x = 5x
We have, product of the roots = 4 > 0, which is true.
⇒ x =
4 2 (y + 1 )
[on squaring both sides] And sum of the roots = >0
5 (y − 1 )
⇒ x=
16 y +1
⇒ >0
25 y −1
Hence, the number of real solutions is 1. ∴ y ∈ ( − ∞, − 1 ) ∪ (1, ∞ ) …(ii)
From Eqs. (i) and (ii), we get
16. Let x = 7 + 7 − 7 + 7 −…∞
1 <y <3
⇒ x= 7+ 7−x [on squaring both sides] 19. Since α , β and γ are the roots of

⇒ x2 − 7 = 7 − x (x − a ) (x − b ) (x − c ) = d
⇒ (x − a ) (x − b ) (x − c ) − d = (x − α ) (x − β) (x − γ )
⇒ (x 2 − 7)2 = 7 − x [again, squaring on both sides] ⇒ (x − α ) (x − β) (x − γ ) + d = (x − a ) (x − b ) (x − c )
⇒ x − 14 x + x + 42 = 0
4 2 ⇒ a, b and c are the roots of
(x − α ) (x − β) (x − γ ) + d = 0
⇒ ( x − 3 ) ( x + 3 x 2 − 5 x − 14 ) = 0
3
20. Since, all the coefficients of given equation are not real.
⇒ (x − 3) (x + 2) (x 2 + x − 7) = 0 Therefore, other root ≠ 3 + i.
− 1 ± 29 Let other root be α.
⇒ x = 3, − 2,
2 2 (1 + i )
∴ x =3 [Q x > 7] Then, sum of the roots =
i
17. Let y = 2 (k − x ) ( x + ( x + k ) 2 2 2 (1 + i )
⇒ α + 3 −i =
i
⇒ y − 2 (k − x ) x = 2 (k − x ) ( x 2 + k 2 ) ⇒ α + 3 − i = 2 − 2i
On squaring both sides, we get ∴ α = −1 −i
⇒ y 2 + 4 (k − x ) 2 x 2 − 4 xy (k − x ) = 4 (k − x ) 2 ( x 2 + k 2 ) 21. We have, |[ x ] − 2 x | = 4
⇒ y 2 − 4 xy (k − x ) = 4 (k − x ) 2k 2 ⇒ |[ x ] − 2 ([ x ] + { x })| = 4
⇒ |[ x ] + 2 { x }| = 4
⇒ 4 (k − y ) x − 4(2k − ky ) x − y + 4k = 0
2 2 3 2 4
which is possible only when
Since, x is real.
2 { x } = 0, 1
∴ D≥0
1
⇒ 16 (2k 3 − ky ) 2 − 4 ⋅ 4 (k 2 − y ) ( 4k 4 − y 2 ) ≥ 0 If { x } = 0, then [ x ] = ± 4 and then x = − 4, 4 and if { x } = ,
2
[using, b 2 − 4ac ≥ 0] then
⇒ 4k 6 + k 2y 2 − 4k 4y − ( − k 2y 2 + 4k 6 + y 3 − 4yk 4 ) ≥ 0 [x ] + 1 = ± 4
⇒ [ x ] = 3, − 5
⇒ 2k 2y 2 − y 3 ≥ 0
1 1
⇒ y 2 (y − 2k 2 ) ≤ 0 ∴ x = 3 + and − 5 +
2 2
∴ y ≤ 2k 2 7 9 9 7
⇒ x = , − ⇒ x = − 4, − , , 4
1 x 2 − 2x + 4 2 2 2 2
18. We have, < < 3, ∀ x ∈ R
3 x 2 + 2x + 4 22. We know that, x 2 + x + 1 is a factor of ax 3 + bx 2 + cx + d .
1 x 2 + 2x + 4 Hence, roots of x 2 + x + 1 = 0 are also roots of
< < 3, ∀ x ∈ R
3 x 2 − 2x + 4 ax 3 + bx 2 + cx + d = 0. Since, ω and ω 2
182 Textbook of Algebra

 1 3i  26. Since, 3789108 is an even integer. Therefore, x 4 − y 4 is also an


 where ω = − +  are two complex roots of x + x + 1 = 0.
2
 2 2 even integer. So, either both x and y are even integers or both
Therefore, ω and ω 2 are two complex roots of of them are odd integers.
ax 3 + bx 2 + cx + d = 0. Now, x 4 − y 4 = (x − y ) (x + y ) (x 2 + y 2 )
We know that, a cubic equation has atleast one real root. Let ⇒ x − y , x + y , x 2 + y 2 must be even integers.
real root be α. Then, Therefore, ( x − y ) ( x + y ) ( x 2 + y 2 ) must be divisible by 8. But
d d
α ⋅ ω ⋅ ω2 = − ⇒α=− 3789108 is not divisible by 8. Hence, the given equation has no
a a solution.
23. We have, (5 x 2 − 8 x + 3 ) − (5 x 2 − 9 x + 4 ) ∴ Number of solutions = 0
27. We have, x 3 + ax + 1 = 0
= (2 x 2 − 2 x ) − (2 x 2 − 3 x + 1 )
or x 4 + ax 2 + x = 0 …(i)
⇒ (5 x − 3 ) ( x − 1 ) − (5 x − 4 ) ( x − 1 )
and x + ax + 1 = 0
4 2
…(ii)
= 2 x ( x − 1 ) − (2 x − 1 ) ( x − 1 ) From Eqs. (i) and (ii), we get
⇒ x − 1 ( 5x − 3 − 5x − 4 ) = x − 1 ( 2x − 2x − 1 ) x −1 = 0
⇒ x =1
⇒ x −1 = 0 which is a common root.
⇒ x =1 ∴ 1 +a + 1 = 0
24. We have, (a + b ) (a − b ) = a 2 − b = 1 [given] ⇒ a = −2
∴ (a + b ) x 2 − 15
+ (a − b ) x 2 − 15
= 2a
28. Q(1 − a 2 ) x 2 + 2ax − 1 = 0

x 2 − 15 1 1 − a2 ≠ 0
⇒ (a + b ) + = 2a
2
(a + b ) x − 15  2a   1 
x2 +  x −   =0
2 1 − a 
2
1 − a2
Let y = (a + b ) x − 15
1  2a   1 
⇒ y + = 2a ⇒ y 2 − 2ay + 1 = 0 Let f (x ) = x 2 +  x −  
y 1 − a2  (1 − a 2 )

2a ±
4a 2 − 4
⇒ y = = a ± a2 − 1
2
∴ y = a ± b = (a + b ) ± 1 [Qa 2 − b = 1]
2
− 15
⇒ (a + b ) x = (a + b ) ± 1 0 α β1
X

∴ x 2 − 15 = ± 1 The following cases arise:


⇒ x 2 = 15 ± 1 ⇒ x 2 = 16, 14 Case I D ≥ 0
2
⇒ x = ± 4, ± 14  2a   −1 
  − 4 ⋅1 ⋅   ≥0
25. We have, x 2 − xy + y 2 = 4 ( x + y − 4 ) 1 − a2 1 − a2
⇒ x 2 − x (y + 4 ) + y 2 − 4y + 16 = 0 4a 2 4
⇒ + ≥0
Q x ∈R (1 − a )
2 2
(1 − a 2 )
∴ ( − (y + 4 )) 2 − 4 ⋅ 1 ⋅ (y 2 − 4y + 16 ) ≥ 0 4a 2 + 4 − 4a 2
⇒ ≥0
[using, b − 4ac ≥ 0 ]
2 (1 − a 2 ) 2
⇒ y 2 + 8y + 16 − 4y 2 + 16y − 64 ≥ 0 4
⇒ ≥0 [always true]
⇒ 3y 2 − 24y + 48 ≤ 0 (1 − a 2 ) 2

⇒ y 2 − 8y + 16 ≤ 0 ⇒ (y − 4 ) 2 ≤ 0 Case II f ( 0 ) > 0
−1 1
∴ (y − 4 ) 2 = 0 ⇒ >0 ⇒ <0
(1 − a 2 ) 1 − a2
∴ y =4
⇒ 1 − a2 < 0
Then, x 2 − 4 x + 16 = 4( x + 4 − 4 )
∴ a ∈ ( − ∞, − 1 ) ∪ (1, ∞ )
x 2 − 8 x + 16 = 0
Case III f (1 ) > 0
(x − 4)2 = 0 2a 1
⇒ 1+ − >0
x=4 (1 − a 2 ) (1 − a 2 )
Number of pairs is 1 i.e., ( 4, 4 ).
Chap 02 Theory of Equations 183

1 − a 2 + 2a − 1 a 2 − 2a  − 1 + 5
⇒ >0 ⇒ <0 ∴ x ∈ 0, 
(1 − a )
2
1 − a2  2 

+ + + Case II If x < 0, i.e., −1 ≤ x < 0


–1 – 0 1 – 2 x − (1 + x ) < 0
⇒ x < 1+ x [always true]
a (a − 2 ) x ∈ [ − 1, 0 )
⇒ >0
(a + 1 ) (a − 1 ) Combining both cases, we get
∴ a ∈ ( − ∞, − 1 ) ∪ ( 0, 1 ) ∪ (2, ∞ )  −1 + 5
x ∈ − 1, 
Case IV 0 < x -coordinate of vertex < 1  2 
2a a
⇒ 0<− <1 ⇒ 0 < 2 <1 30. We have, (a ⋅ 2b − 2c ⋅ a ) (2c ⋅ c − b ⋅ 2b ) = (ba − ca ) 2
2 (1 − a 2 ) a −1
⇒ 2a (b − c ) ⋅ 2 (c 2 − b 2 ) = a 2(b − c ) 2
a a
⇒ 0< and 1 − 2 >0 ⇒ 4a (c − b ) (c + b ) = a 2(b − c ) [Qb ≠ c ]
(a + 1 ) (a − 1 ) a −1
a ⇒ 4a (c + b ) = − a 2
⇒ >0
(a + 1 ) (a − 1 ) ⇒ a + 4b + 4c = 0
 b c
+ + 31. 0 < a < b < c, α + β =  −  and αβ =
 a a
– –1 0 – 1
For non-real complex roots,
b 2 − 4ac < 0
⇒ a ∈ ( − 1, 0 ) ∪ (1, ∞ )
 1 + 5  1 − 5 b 2 4c
a −  a −  ⇒ − <0
 2   2  a2 a
and >0 ⇒ (α + β ) 2 − 4 αβ < 0
(a + 1 ) (a − 1 )
⇒ (α − β ) 2 < 0
+ + +
Q 0 <a <b <c
–1 – 1– √5 1 – 1+√5
2 2 ∴ Roots are conjugate, then | α | = | β|
c
But αβ =
1 − 5  1 + 5  a
and a ∈ ( − ∞, − 1 ) ∪  , 1 ∪  , ∞
 2   2  c  c 
| αβ | = >1 Q a < c, ∴ > 1
a  a 
1 − 5  1 + 5 
∴ a ∈ , 0 ∪  , ∞ ⇒ | α| | β| > 1
 2   2 
⇒ | α | 2 > 1 or | α | > 1
Combining all cases, we get
a >2 32. Given equation is
29. We have, x − 1 − |x| < 0 …(i) Ax 2 − | G | x − H = 0 …(i)
which is defined only when ∴ Discriminant = ( − | G | ) − 4 A ( − H )
2

1 − |x | ≥ 0 = G 2 + 4 AH
⇒ |x | ≤ 1 = G 2 + 4G 2 [QG 2 = AH ]
⇒ x ∈ [ − 1, 1 ]
= 5G 2 > 0
Now, from Eq. (i), we get
x < 1 − |x| ∴ Roots of Eq. (i) are real and distinct.
a+b 2ab
Case I If x ≥ 0, i.e., 0 ≤ x ≤ 1 Q A= > 0, G = ab > 0, H = >0
2 a+b
x − (1 − | x | ) < 0
[Qa and b are two unequal positive integers]
⇒ x < (1 − x ) Let α and β be the roots of Eq. (i). Then,
On squaring both sides, we get | G|
α+β= >0
x2 + x − 1 < 0 A
H
−1 − 5 −1 + 5 and αβ = − <0
⇒ <x< A
2 2
D G 5
But x≥0 and α −β = = >0
A A
184 Textbook of Algebra

|G | + G 5 The given equation will have four real roots, i.e. Eq. (i) has two
∴ α= >0 non-negative roots.
2A
b
|G | − G 5 Then, − ≥0
and β= <0 a
2A
af ( 0 ) ≥ 0
Exactly one positive root and atleast one root which is
negative fraction. and b 2 − 4ac ≥ 0 [given]
33. It is clear from graph that the equation y = ax 2 + bx + c = 0 ⇒
b
≤0
has two real and distinct roots. Therefore, a
b 2 − 4ac > 0 …(i) ac ≥ 0
Q Parabola open downwards. ⇒ a > 0, b < 0, c > 0
∴ a<0 or a < 0, b > 0, c < 0
a
andy = ax 2 + bx + c cuts-off Y -axis at, x = 0. 36. Let the roots be , a and ar , where a > 0, r > 1
r
∴ y =c < 0 ∴ Product of the roots = 1
⇒ c<0 a
and x-coordinate of vertex > 0 ⇒ ⋅ a ⋅ ar = 1
r
b b
⇒ − >0 ⇒ <0 ⇒ a3 = 1
2a a
∴ a =1 [one root is 1]
⇒ b>0 [Qa < 0]
1
It is clear that a and b are of opposite signs. Now, roots are , 1 and r . Then,
r
34. Let y = ax 2 + bx + c 1
+ 1 + r = −b
r
1
a>0 ⇒ + r = −b −1 …(i)
r
1
–2 2 Q r + >2
α β r
⇒ −b −1 >2
Consider the following cases: ⇒ b < −3 [from Eq. (i)]
Case I D > 0 or b ∈ ( − ∞, − 3 )
⇒ b 2 − 4ac > 0 1 1
Also, ⋅1 + 1 ⋅r + r ⋅ = c
Case II af ( − 2 ) < 0 r r
⇒ a ( 4a − 2b + c ) < 0 1
⇒ + r + 1 =c = −b [from Eq. (i)]
⇒ 4a − 2b + c < 0 r
Case III af (2 ) > 0 ∴ b +c = 0
⇒ a ( 4a + 2b + c ) > 0 1
⇒ 4a + 2 b + c > 0 Now, first root = < 1 [Q one root is smaller than one]
r
Combining Case II and Case III, we get
Second root = 1
4a + 2| b| + c < 0
Third root = r > 1 [Q one root is greater than one]
Also, at x = 0, y <0 ⇒c<0
37. We have, f ( x ) = ax 2 + bx + c
Also, since for − 2 < x < 2,
a, b , c ∈ R [Q a ≠ 0 ]
y <0 1
On putting x = 0, 1, , we get
⇒ ax 2 + bx + c < 0 2
For x = 1, a+b+c<0 …(i) |c | ≤ 1
and for x = − 1, a −b + c < 0 …(ii) |a + b + c | ≤ 1
1 1
Combining Eqs. (i) and (ii), we get and a + b + c ≤1
4 2
a + | b| + c < 0
⇒ −1 ≤ c ≤ 1,
35. Put x 2 = y .
−1 ≤a + b + c ≤1
Then, the given equation can be written as
and − 4 ≤ a + 2b + 4c ≤ 4
f (y ) = ay 2 + by + c = 0 …(i) ⇒ − 4 ≤ 4a + 4b + 4c ≤ 4
and − 4 ≤ − a − 2b − 4c ≤ 4
Chap 02 Theory of Equations 185

On adding, we get b
Option (d) a < 0, c < 0, − <0
− 8 ≤ 3a + 2b ≤ 8 2a
Also, − 8 ≤ a + 2b ≤ 8 or a < 0, c < 0, b < 0
∴ − 16 ≤ 2a ≤ 16 ∴ abc < 0
⇒ | a| ≤ 8 41. Here, D ≤ 0
Q −1 ≤ − c ≤ 1, − 8 ≤ − a ≤ 8 and f ( x ) ≥ 0, ∀ x ∈ R
We get, − 16 ≤ 2b ≤ 16 ∴ f (3 ) ≥ 0
⇒ |b | ≤ 8 ⇒ 9a + 3b + 6 ≥ 0
or 3a + b ≥ − 2
∴ | a| + | b| + | c | ≤ 17
⇒ Minimum value of 3a + b is − 2.
− 5 ± 25 + 1200 −5 ± 35 30 − 40
38. Q x = = = , and f (6 ) ≥ 0
50 50 50 50 ⇒ 36a + 6b + 6 ≥ 0
3 −4 ⇒ 6a + b ≥ − 1
or cos α = ,
5 5 ⇒ Minimum value of 6a + b is −1.
But −1 < x < 0 42. Since, f ( x ) = x 3 + 3x 2 − 9x + λ = ( x − α ) 2( x − β)
4 ∴ α is a double root.
∴ cos α = − [ lies in II and III quadrants]
5 ∴ f ′( x ) = 0 has also one root α.
3 i.e. 3 x 2 + 6 x − 9 = 0 has one root α.
∴ sin α = [ lies in II quadrant]
5
∴ x 2 + 2 x − 3 = 0 or ( x + 3 ) ( x − 1 ) = 0
3
∴ sin α = − [lies in III quadrant] has the root α which can either −3 or 1.
5
If α = 1, then f (1 ) = 0 gives λ − 5 = 0 ⇒ λ = 5.
24
∴ sin 2α = 2 ⋅ sin α ⋅ cos α = − If α = − 3, then f ( − 3 ) = 0 gives
25
− 27 + 27 + 27 + λ = 0
[ lies in II quadrant] ⇒ λ = − 27
24 43. We have, D = (b − c ) 2 − 4a (a − b − c ) > 0
∴ sin 2α = 2 ⋅ sin α ⋅ cosα = [lies in III quadrant]
25
⇒ b 2 + c 2 − 2bc − 4a 2 + 4ab + 4ac > 0
39. Qa + 2b + 4c = 0
⇒ c 2 + ( 4a − 2b ) c − 4a 2 + 4ab + b 2 > 0, ∀c ∈ R
2
 1  1 Since, c ∈ R, so we have
∴ a   + b  + c = 0
 2  2
( 4a − 2b ) 2 − 4 ( − 4a 2 + 4ab + b 2 ) < 0
1
It is clear that one root is . ⇒ 4a 2 − 4ab + b 2 + 4a 2 − 4ab − b 2 < 0
2
1 b ⇒ a (a − b ) < 0
Let other root be α. Then, α + =− If a > 0, then a − b < 0
2 a
1 b i.e. 0 <a <b
⇒ α=− − or b >a > 0
2 a
which depends upon a and b. If a < 0, then a − b > 0
i.e. 0 >a >b
40. Q Cut-off Y -axis, put x = 0, i.e. f ( 0) = c
or b <a < 0
b x 3 − ax 2 + bx − 1 = 0
Option (a) a < 0, c < 0, − <0 44. We have, …(i)
2a
Then, α 2 + β 2 + γ 2 = (α + β + γ ) 2 − 2 (αβ + βγ + γα )
or a < 0, c < 0, b < 0
∴ abc < 0 = a 2 − 2b
b α 2 β 2 + β 2 γ 2 + γ 2 α 2 = (αβ + βγ + γα ) 2
Option (b) a < 0, c > 0, − >0
2a − 2 αβγ (α + β + γ ) = b 2 − 2a
or a < 0, c > 0, b > 0 and α 2 β2 γ 2 = 1
∴ abc < 0
Therefore, the equation whose roots are α 2, β 2 and γ 2, is
b
Option (c) a > 0, c > 0, − >0 x 3 − (a 2 − 2b ) x 2 + (b 2 − 2a ) x − 1 = 0 …(ii)
2a
or a > 0, c > 0, b < 0 Since, Eqs. (i) and (ii) are indentical, therefore
∴ abc < 0 a 2 − 2b = a and b 2 − 2a = b
186 Textbook of Algebra

Eliminating b, we have ⇒ (7y − 1 ) (y − 7 ) ≤ 0


(a 2 − a ) 2 a2 − a ∴
1
≤y ≤7
− 2a =
4 2 7
1
⇒ a {a (a − 1 ) 2 − 8 − 2 (a − 1 )} = 0 ∴ G = 7 and L =
7
⇒ a (a − 2a − a − 6 ) = 0
3 2
∴ GL = 1
⇒ a (a − 3 ) (a 2 + a + 2 ) = 0 G100 + L100 G100 + L100
Now, ≥ (GL )100 ⇒ ≥1
⇒ a = 0 or a = 3 or a 2 + a + 2 = 0 2 2
⇒ b = 0 or b = 3 ⇒ G100 + L100 ≥ 2
or b2 + b + 2 = 0 46. Least value of G100 + L100 is 2.
∴ a =b = 0 47. The quadratic equation having roots G and L, is
or a =b =3 x 2 − (G + L ) x + GL = 0
or a and b are roots of x 2 + x + 2 = 0. 50
⇒ x2 − x + 1 = 0
45. Here, D > 0 7
⇒ 7 x 2 − 50 x + 7 = 0
a > 0
48. We have, L2 < λ < G 2
–2 2
X 2
α β  1
⇒   < λ <7
2
 7
1
⇒ < λ < 49
b 2 − 4ac > 0 49
or b 2 > 4ac …(i) ⇒ λ = 1, 2, 3,…, 48 as λ ∈ N
and f (0) < 0 48 × 49
∴ Sum of all values of λ = 1 + 2 + 3 +…+ 48 = = 1176
⇒ c<0 …(ii) 2
f (1 ) < 0 Solutions (Q. Nos. 49 to 51)
⇒ a+b+c<0 …(iii) Let roots be α , β, γ, δ > 0.
f (− 1) < 0 ∴ α + β + γ + δ = 12
⇒ a −b + c < 0 …(iv) (α + β ) ( γ + δ ) + αβ + γδ = c
f (2 ) < 0 αβ ( γ + δ ) + γδ (α + β ) = − d
⇒ 4a + 2b + c < 0 …(v) α β γδ = 81
f (− 2) < 0 α+β+γ+δ
Q AM = =3
⇒ 4a − 2b + c < 0 …(vi) 4
From Eqs. (i) and (ii), we get and GM = (α β γ δ )1 /4 = (81 )1 /4 = 3
c < 0, b2 − 4ac > 0 ∴ AM = GM
From Eqs. (iii) and (iv), we get ⇒ α =β = γ =δ =3
a − |b | + c < 0 49. c = (α + β) ( γ + δ ) + αβ + γδ
and from Eqs. (v) and (vi), we get = (3 + 3 ) (3 + 3 ) + 3 ⋅ 3 + 3 ⋅ 3 = 36 + 18 = 54
4a − 2 | b | + c < 0 50. Qαβ ( γ + δ ) + γδ (α + β) = − d
Solutions (Q. Nos. 46 to 48) ∴ d = − {3 ⋅ 3 ⋅ (3 + 3 ) + 3 ⋅ 3 ⋅ (3 + 3 )} = − 108
2x 2 − 3x + 2 d ( − 108 )
Let y = 2 51. Required root = − = − =1
2x + 3x + 2 2c 2 × 54
⇒ 2 x 2y + 3 xy + 2y = 2 x 2 − 3 x + 2 Solutions (Q. Nos. 52 to 54)
⇒ 2 (y − 1 ) x 2 + 3 (y + 1 ) x + 2 (y − 1 ) = 0 Given that, AC = 4 2 units
As x ∈ R AC
∴ AB = BC = = 4 units
∴ D≥0 2
⇒ 9 (y + 1 ) − 4 ⋅ 2 (y − 1 ) ⋅ 2 (y − 1 ) ≥ 0
2
and OB = ( BC ) 2 − (OC ) 2
⇒ 9 (y + 1 ) 2 − 16 (y − 1 ) 2 ≥ 0  AC 
= ( 4 ) 2 − (2 2 ) 2 Q OC =
⇒ (3y + 3 ) 2 − ( 4y − 4 ) 2 ≥ 0  2 
⇒ (7y − 1 ) (7 − y ) ≥ 0 = 2 2 units
Chap 02 Theory of Equations 187

∴ Vertices are A ≡ ( − 2 2, 0 ), Solutions (Q. Nos. 58 to 60)


B ≡ ( 0, −2 2 ) Let f ( x ) = ax 2 − bx + c has two distinct roots α and β. Then,
f ( x ) = a ( x − α ) ( x − β ). Since, f ( 0 ) and f (1 ) are of same sign.
and C ≡ (2 2, 0)
Therefore, c (a − b + c ) > 0
52. Since, y = f ( x ) = ax 2 + bx + c passes through A, B and C, then ⇒ c (a − b + c ) ≥ 1
0 = 8a − 2 2b + c − 2 2 = c ∴ a αβ (1 − α ) (1 − β ) ≥ 1
2

and 0 = 8a + 2 2b + c 1 1
2
 1
But α (1 − α ) = −  − α ≤
We get, b = 0, a =
1
and c = − 2 2 4 2  4
2 2
a2
x2 ∴ a 2 αβ (1 − α ) (1 − β ) <
∴ y = f (x ) = −2 2 16
2 2 a2
⇒ >1 ⇒ a > 4 [Q α ≠ β]
x2 16
53. Minimum value of y = − 2 2 is at x = 0.
2 2 ⇒ a ≥ 5 as a ∈ I
∴ (y ) min = − 2 2 Also, b 2 − 4ac ≥ 0
54. f ( x ) = 0 ⇒ b 2 ≥ 4ac ≥ 20
x2 ⇒ b ≥5
⇒ −2 2 = 0 ⇒ x = ± 2 2
2 2 Next, a ≥ 5, b ≥ 5, we get c ≥ 1
λ ∴ abc ≥ 25
Given, −2 2 < <2 2
2 ∴ log 5 abc ≥ log 5 25 = 2
or −4 2<λ<4 2 58. Least value of a is 5.
∴Initial values of λ are 59. Least value of b is 5.
−5, − 4, − 3, − 2, − 1, 0, 1, 2, 3, 4, 5. 60. Least value of logb abc is 2.
∴Number of integral values is 11.
Solutions. (Q. Nos. 61 to 63)
Solutions. (Q. Nos. 55 to 57) Let α , β and γ be the roots of 2 x 3 + ax 2 + bx + 4 = 0.
We have, (α − β ) = (α + k ) − ( β + k ) a
∴ α+β+γ=−
b 2 − 4c b 2 − 4c1 2
⇒ = 1
1 1 b
αβ + βγ + γα = and αβγ = − 2
⇒ b − 4c = b1 − 4c1
2 2
…(i) 2
1 (b 2 − 4c ) 1 61. Q AM ≥ GM
Given, least value of f ( x ) = − − =− (− α ) + (− β) + (− γ )
4 4 ×1 4 ∴ ≥ {( − α ) ( − β ) ( − γ )}1 /3
3
⇒ b 2 − 4c = 1
a
∴ b 2 − 4c = 1 = b12 − 4c1 [from Eq. (i)] …(ii) 2 ≥ (2 )1/ 3

7 3
Also, given least value of g( x ) occurs at x = .
2 ∴ a ≥ 6 (2 )1/ 3 …(i)
b1 7
∴ − = or a 3 ≥ 432
2 ×1 2
Hence, minimum value of a 3 is 432.
∴ b1 = − 7
62. Q AM ≥ GM
55. b 1 = − 7
(− α ) (− β) + (−β) (− γ ) + (− γ ) (− α )
b12 − 4c1 1 ∴
56. Least value of g ( x ) = − =− [from Eq. (ii)] 3
4 ×1 4
≥ {( − α ) ( − β ) ( − β ) ( − γ ) ( − γ ) ( − α )}1 / 3
57. Q g( x ) = 0 b/ 2
⇒ ≥ ( 4 )1/ 3
∴ x 2 + b1x + c1 = 0 3
− b1 ± b12 − 4c1 ⇒ b ≥ 6 ( 4 )1/ 3 …(ii)
⇒ x=
2 or b 3 ≥ 864
7±1 Hence, minimum value of b 3 is 864.
= = 3, 4
2 63. From Eqs. (i) and (ii), we get
∴ Roots of g( x ) = 0 are 3, 4. ab ≥ 6 (2 )1 / 3 ⋅ 6( 4 )1 / 3
188 Textbook of Algebra

⇒ ab ≥ 36 × 2 68. We have,
a+b a+b (5 + 2 ) x 2 − ( 4 + 5 ) x + 8 + 2 5 = 0
Q ≥ ab ≥ 6 2 ⇒ ≥6 2
2 2
4+ 5
∴ a + b ≥ 12 2 ∴Sum of the roots =
5+ 2
or (a + b ) 3 ≥ 3456 2
8+2 5
Hence, minimum value of (a + b ) 3 is 3456 2. and product of the roots =
5+ 2
Solutions (Q. Nos. 64 to 66) ∴The harmonic mean of the roots
∴ α+β+γ+δ=−A …(i)
2 × Product of the roots 2 × (8 + 2 5 )
(α + β ) ( γ + δ ) + αβ + γδ = B …(ii) = = =4
αβ ( γ + δ ) + γδ (α + β ) = − C …(iii)
Sum of the roots (4 + 5 )
and αβγδ = D …(iv) 69. Let x 2 − ax + 30 = y
C αβ ( γ + δ ) + γδ (α + β ) ∴ y = 2 y + 15
64. Q = …(i)
A α+β+γ+δ
k ( γ + δ ) + k (α + β ) ⇒ y − 4y − 60 = 0
2

= [Qαβ = γδ =k]
α+β+γ+δ ⇒ (y − 10 ) (y + 6 ) = 0
=k …(v) ∴ y = 10, − 6
65. From Eq. (ii), we get ⇒ y = 10, y ≠ − 6 [Qy > 0]
(α + β ) ( γ + δ ) = B − (αβ + γδ ) = B − 2k [Qαβ = γδ = k] Now, x − ax + 30 = 10
2

66. From Eq. (iv), we get


⇒ x 2 − ax + 20 = 0
αβγδ = D
⇒ k ⋅k = D [Qαβ = γδ = k] Given, αβ = λ = 20
2 α+β
C ∴ ≥ αβ = 20
⇒   =D [from Eq. (v)] 2
 A
⇒ α + β ≥ 2 20
∴ C 2 = A 2D
67. The given equation is | x − 2 | 2 + | x − 2 | − 2 = 0. or µ=4 5

There are two cases: ∴ Minimum value of µ is 4 5.


Case I If x ≥ 2, then ( x − 2 ) 2 + x − 2 − 2 = 0 i.e., µ = 4 5 = 8.9 ⇒ (µ ) = 9
6
⇒ x − 3x = 0
2
 1  1
70. Q N r =  x +  −  x 6 +  −2
 x  x6
⇒ x (x − 3) = 0
2
⇒ x = 0, 3  1   1 
6 3
 1 1 
=  x +  −  x 3 + 3  =   x +  +  x 3 + 3  
Here, 0 is not possible.  x   x    x   x 
∴ x =3
 1
3
1 
Case II If x < 2, then   x +  −  x 3 + 3 
(x − 2)2 − x + 2 − 2 = 0  x  x 

⇒ x 2 − 5x + 4 = 0   1
= Dr ⋅ 3  x +  
⇒ (x − 1) (x − 4) = 0   x
⇒ x = 1, 4 Nr  1
∴ = 3 x +  ≥ 6
Here, 4 is not possible. Dr  x 
∴ x =1 Nr
Hence, minimum value of is 6.
∴The sum of roots = 1 + 3 = 4 Dr
71. a + b = 2c …(i)
Aliter ab = − 5d …(ii)
Let | x − 2 | = y . c + d = 2a …(iii)
Then, we get y2 + y −2 = 0 cd = − 5b …(iv)
⇒ (y − 1 ) (y + 2 ) = 0 ⇒ y = 1, − 2 From Eqs. (i) and (iii), we get
But − 2 is not possible. a + b + c + d = 2 (a + c )
Hence, | x − 2| = 1 ⇒ x = 1, 3 ∴ a + c =b +d …(v)
∴ Sum of the roots = 1 + 3 = 4
Chap 02 Theory of Equations 189

From Eqs. (i) and (iii), we get 74. Q (1 + i ) x 2 + (1 − i ) x − 2i = 0


b − d = 3 (c − a ) …(vi) (1 − i ) 2i
⇒ x2 + x− =0
Also, a is a root of x 2 − 2cx − 5d = 0 (1 + i ) (1 + i )
∴ a 2 − 2ac − 5d = 0 …(vii) ⇒ x 2 − ix − (1 + i ) = 0
And c is a root of ∴ α + β = i , and αβ = − (1 + i )
c 2 − 2ac − 5b = 0 …(viii) ∴ α − β = (α + β ) 2 − 4 αβ = i 2 + 4 (1 + i ) = (3 + 4i )
From Eqs. (vii) and (viii), we get
| α − β| = 9 + 16 = 5
a 2 − c 2 − 5 (d − b ) = 0
∴ | α − β| = 5
2
⇒ (a + c ) (a − c ) + 5 (b − d ) = 0
⇒ (a + c ) (a − c ) + 15 (c − a ) = 0 [from Eq. (vi)] 75. Q 4x − 16x + c = 0
2

⇒ (a − c ) (a + c − 15 ) = 0 ⇒ x 2 − 4x +
c
=0
∴ a + c = 15, a − c ≠ 0 4
c
From Eq. (v), we get b + d = 15 Let f (x ) = x 2 − 4x +
4
∴ a + b + c + d = a + c + b + d = 15 + 15 = 30
Then, the following cases arises:
⇒ Sum of digits of a + b + c + d = 3 + 0 = 3
x 2 − 3x + c
72. Q y =
x 2 + 3x + c
⇒ x 2(y − 1 ) + 3 x (y + 1 ) + c (y − 1 ) = 0 2
1 α β 3
Q x ∈R
∴ 9 (y + 1 ) 2 − 4c (y − 1 ) 2 ≥ 0 Case I D>0
⇒ 16 − c > 0
( 2 cy − 2 c ) 2 − (3y + 3 ) 2 ≤ 0
∴ c < 16
⇒ {( 2 c + 3 ) y − (2 c − 3 )} {(2 c − 3 )y − (2 c + 3 )} ≤ 0 Case II f (1 ) > 0
2 c −3 2 c +3 c
or ≤y ≤ ⇒ 1−4+ >0
2 c +3 2 c −3 4
c
2 c +3 ⇒ >3
But given, =7 4
2 c −3 ∴ c > 12
⇒ 2 c + 3 = 14 c − 21 Case III f (2 ) < 0
c
or 12 c = 24 or c =2 ⇒ 4 −8 + < 0
4
∴ c=4 c
⇒ <4
73. We have, x − (x − 1) + (x − 2)2 = 5
2 2 4
∴ c < 16
⇒ | x | − | x − 1| + | x − 2| = 5
Case IV f (3 ) > 0
Case I If x < 0, then c
⇒ 9 − 12 + > 0
− x + (x − 1) − (x − 2) = 5 4
x =1 − 5 c
⇒ >3
4
Case II If 0 ≤ x < 1, then
⇒ c > 12
x + (x − 1) − (x − 2) = 5
⇒ x = 5 − 1, which is not possible. Combining all cases, we get
Case III If 1 ≤ x < 2, then 12 < c < 16
x − (x − 1) − (x − 2) = 5 Thus, integral values of c are 13, 14 and 15.
⇒ x = 3 − 5, which is not possible. Hence, number of integral values of c is 3.
76. We have, r +s +t=0 …(i)
Case IV If x > 2, then
x − (x − 1) + (x − 2) = 5 1001
rs + st + tr = …(ii)
8
⇒ x =1 + 5 2008
and rst = − = − 251 …(iii)
Hence, number of solutions is 2. 8
190 Textbook of Algebra

Now, (r + s ) 3 + (s + t ) 3 + (t + r ) 3 = ( − t ) 3 + ( − r ) 3 + ( − s ) 3 Q a (b − c ) + b (c − a ) + c (a − b ) = 0
[Qr + s + t = 0] ∴ x = 1 is a root of
= − (t 3 + r 3 + s 3 ) = − 3 rst [Qr + s + t = 0] a (b − c ) x 2 + b (c − a ) x + c (a − b ) = 0 …(ii)
= − 3 ( − 251 ) = 753 Given, roots [Eq. (ii)] are equal.
c (a − b )
Now, 99 λ = (r + s ) 3 + (s + t ) 3 + (t + r ) 3 = 753 ∴ 1 ×1 =
a (b − c )
753
∴ λ= = 7.6 ⇒ a (b − c ) = c (a − b )
99
2ac
∴ [λ ] = 7 or b=
a+c
77. A → (r,s); B → (p, q, r,s, t); C → (p, q, t)
∴ a, b and c are in HP. …(iii)
x 2 − 2x + 4
(A) We have, y = From Eqs. (i) and (ii), we get
x 2 + 2x + 4
a =b =c
⇒ x 2 (y − 1 ) + 2 (y + 1 ) x + 4 (y − 1 ) = 0 ∴ a,b and c are in AP, GP and HP.
As x ∈ R , we get (B)Q x 3 − 3x 2 + 3x − 1 = 0
D≥0 ⇒ (x − 1)3 = 0
⇒ 4 (y + 1 ) 2 − 16 (y − 1 ) 2 ≥ 0
∴ x = 1, 1, 1
⇒ 3y 2 − 10y + 3 ≤ 0 ⇒ Common root, x = 1
⇒ (y − 3 ) (3y − 1 ) ≤ 0 ∴ a (1 ) 2 + b (1 ) + c = 0
1 ⇒ a+b+c=0
⇒ ≤y ≤3
3
(C) Given, bx + ( (a + c ) 2 + 4b 2 ) x + (a + c ) ≥ 0
2
2x 2 + 4x + 1
(B) We have, y = 2 ∴ D≤0
x + 4x + 2
⇒ (a + c ) 2 + 4b 2 − 4b (a + c ) ≤ 0
⇒ x 2(y − 2 ) + 4(y − 1 ) x + 2y − 1 = 0
⇒ (a + c − 2b ) 2 ≤ 0
As x ∈ R , we get
D≥0 or (a + c − 2b ) 2 = 0
⇒ 16 (y − 1 ) 2 − 4 (y − 2 ) (2y − 1 ) ≥ 0 ∴ a + c = 2b
Hence a, b and c are in AP.
⇒ 4 (y − 1 ) − (y − 2 ) (2y − 1 ) ≥ 0
2
79. A → (q,r,s,t); B → (q,r); C → (p,q)
⇒ 2y 2 − 3y + 2 ≥ 0
ax 2 + 3 x − 4
3 (A) We have, y =
⇒ y2 − y + 1 ≥ 0 3x − 4x 2 + a
2
2 ⇒ x 2 (a + 4y ) + 3 (1 − y ) x − (ay + 4 ) = 0
 3 7
⇒ y −  + ≥0
 4 16 As x ∈R , we get
∴ y ∈R D≥0
x − 3x + 4
2 ⇒ 9 (1 − y ) 2 + 4 (a + 4y ) (ay + 4 ) ≥ 0
(C) We have, y =
x −3 ⇒ ( 9 + 16a ) y 2 + ( 4a 2 + 46 )y + (9 + 16a ) ≥ 0, ∀ y ∈ R
⇒ x 2 − (3 + y ) x + 3y + 4 = 0 ⇒ If 9 + 16a > 0, then D ≤ 0
As x ∈ R , we get Now, D≤0
D ≥ 0 ⇒ (3 + y ) 2 − 4 (3y + 4 ) ≥ 0 ⇒ ( 4a 2 + 46 ) 2 − 4 ( 9 + 16a ) 2 ≤ 0

⇒ y 2 − 6y − 7 ≥ 0 ⇒ (y + 1 ) (y − 7 ) ≥ 0 ⇒ 4 [( 2a 2 + 23 ) 2 − ( 9 + 16a ) 2 ] ≤ 0

⇒ y ∈ ( − ∞, − 1 ] ∪ [7, ∞ ) ⇒ [(2 a 2 + 23 ) + ( 9 + 16a )] [( 2a 2 + 23 ) − ( 9 + 16a )] ≤ 0


78. A → (q,r,s); B → (p); C → (q) ⇒ ( 2a 2 + 16a + 32 ) ( 2a 2 − 16a + 14 ) ≤ 0
(A)Q(d + a − b ) 2 + (d + b − c ) 2 = 0 ⇒ 4 (a + 4 ) 2 (a 2 − 8a + 7 ) ≤ 0
which is possible only when ⇒ a 2 − 8a + 7 ≤ 0
d + a − b = 0, d + b − c = 0 ⇒ (a − 1 ) (a − 7 ) ≤ 0
⇒ b −a =c −b ⇒ 1 ≤a ≤7
⇒ 2b = a + c ∴ 9 + 16a > 0 and 1 ≤ a ≤ 7
∴ a, b and c are in AP. …(i) ⇒ 1 ≤a ≤7
Chap 02 Theory of Equations 191

ax 2 + x − 2 f (1 ) > 0
(B) We have, y =
a + x − 2x 2 ⇒ 1 −6 + 9 + λ > 0
⇒ x 2 (a + 2y ) + x (1 − y ) − (2 + ay ) = 0 ⇒ λ>−4 …(ii)
and f (3 ) < 0
As x ∈R , we get
⇒ 27 − 54 + 27 + λ < 0
D≥0
⇒ λ<0 …(iii)
⇒ ( 1 − y ) 2 + 4 ( 2 + ay ) (a + 2y ) ≥ 0
From Eqs. (i), (ii) and (iii), we get
⇒ ( 1 + 8a ) y 2 + ( 4a 2 + 14 ) y + (1 + 8a ) ≥ 0 −4<λ<0
⇒ If 1 + 8a > 0, then D ≤ 0 ⇒ −3 < λ + 1 < 1
⇒ ( 4a 2 + 14 ) 2 − 4 (1 + 8a ) 2 ≤ 0 ∴ [ λ + 1 ] = − 3, − 2, − 1, 0
⇒ 4 [(2a 2 + 7 ) 2 − (1 + 8a ) 2 ] ≤ 0 ∴ |[ λ + 1 ]| = 3, 2, 1, 0
(B)Q x 2 + x + 1 > 0, ∀ x ∈ R
⇒ [(2a 2 + 7 ) + (1 + 8a )] [(2a 2 + 7 ) − (1 + 8a )] ≤ 0
x 2 − λx − 2
⇒ (2a 2 + 8a + 8 ) (2a 2 − 8a + 6 ) ≤ 0 Given, −3 < <2
x2 + x + 1
⇒ 4 (a + 2 ) 2 (a 2 − 4a + 3 ) ≤ 0
⇒ − 3 x 2 − 3 x − 3 < x 2 − λx − 2 < 2 x 2 + 2 x + 2
⇒ a 2 − 4a + 3 ≤ 0
⇒ 4x 2 − (λ − 3) x + 1 > 0
⇒ (a − 1 ) (a − 3 ) ≤ 0
⇒ 1 ≤a ≤3 and x 2 + (λ + 2)x + 4 > 0
Thus, 1 + 8a > 0 and 1 ≤ a ≤ 3 ∴ (λ − 3)2 − 4 ⋅ 4 ⋅ 1 < 0
⇒ 1 ≤a ≤3 and (λ + 2)2 − 4 ⋅ 1 ⋅ 4 < 0
x 2 + 2x + a
(C) We have, y = 2 ⇒ (λ − 3)2 − 42 < 0
x + 4 x + 3a
and (λ + 2)2 − 42 < 0
⇒ x 2(y − 1 ) + 2 (2y − 1 ) x + a (3y − 1 ) = 0
⇒ − 4 < λ −3 < 4
As x ∈ R , we get and −4<λ+2<4
D≥0 or − 1 < λ <7
⇒ 4 (2y − 1 ) 2 − 4 (y − 1 ) a (3y − 1 ) ≥ 0 and −6 < λ <2
⇒ ( 4 − 3a ) y 2 − ( 4 − 4a )y + (1 − a ) ≥ 0 We get, −1 < λ <2
⇒ If 4 − 3a > 0, then D ≤ 0 ∴ [ λ ] = − 1, 0, 1
⇒ ( 4 − 4a ) 2 − 4 ( 4 − 3a ) (1 − a ) ≤ 0 ⇒ |[ λ ]| = 0, 1

⇒ 4 (2 − 2a ) 2 − 4 ( 4 − 3a ) (1 − a ) ≤ 0 (C)Q (b − c ) + (c − a ) + (a − b ) = 0
∴ x = 1 is a root of
⇒ 4 + 4a − 8a − ( 4 − 7a + 3a ) ≤ 0
2 2
(b − c ) x 2 + (c − a ) x + (a − b ) = 0
⇒ a2 − a ≤ 0
Also, x = 1 satisfies
⇒ a (a − 1 ) ≤ 0 x2 + λ x + 1 = 0
⇒ 0 ≤a ≤1
⇒ 1+λ+1=0
80. A → (p,q,r,s);B → (p,q); C → (s) ∴ λ = −2
(A) Let y = f ( x ) = x 3 − 6 x 2 + 9 x + λ Now, λ −1 = −3
[ λ − 1] = − 3
f ′ ( x ) = 3 x 2 − 12 x + 9 = 0
⇒ |[ λ − 1 ]| = 3
∴ x = 1, 3 81. If quadratic equation ax 2 + bx + c = 0 is satisfied by more than
f ′′ ( x ) = 6 x − 12 two values of x, then it must be an identity.
f ′′(1 ) < 0 and f ′′ (3 ) > 0 Therefore, a = b = c = 0
∴ Statement-2 is true.
But in Statement-1,
4 p − 3 = 4q − 3 = r = 0
3
0 3 Then, p =q = ,r = 0
1 4
λ
which is false.
Since, at one value of p or q or r, all coefficients at a time ≠ 0.
Also, f (0) < 0 ⇒ λ < 0 …(i) ∴ Statement-1 is false.
192 Textbook of Algebra

82. We have, x 2 + (2m + 1 ) x + (2n + 1 ) = 0 …(i)  4 4 


and z ∈ 2− ,2 +
 3 
m, n ∈ I 3
∴ D = b 2 − 4ac Since, Eqs. (i) and (ii) remains same, if x, y , z interchange their
positions.
= ( 2m + 1 ) 2 − 4 ( 2n + 1 )
Hence, both statements are true and Statement-2 is a correct
is never be a perfect square. explanation of Statement-1.
Therefore, the roots of Eq. (i) can never be integers. Hence, the 86. Let y = ax 3 + bx + c
roots of Eq. (i) cannot have any rational root as a = 1, b, c ∈ I .
dy
Hence, both statements are true and Statements –2 is a correct ∴ = 3ax 2 + b
explanation of Statement-1. dx
dy
83. Let α be one root of equation ax 2 + 3x + 5 = 0. Therefore, For maximum or minimum = 0, we get
dx
1 5
α⋅ = b
α a x=± −
3a
5
⇒ 1= dy
a Case I If a > 0, b > 0, then >0
dx
⇒ a =5
In this case, function is increasing, so it has exactly one root
Hence, both the statements are true and Statement-2 is the
dy
correct explanation of Statement-1. Case II If a < 0, b < 0, then <0
dx
84. Let roots of Ax 3 + Bx 2 + Cx + D = 0 …(i)
In this case, function is decreasing, so it has exactly one root.
are α − β, α , α + β (in AP).
Case III a > 0, b < 0 or a < 0, b > 0, then y = ax 3 + bx + c is
B
Then, (α − β) + α + (α + β) = − maximum at one point and minimum at other point.
A
B Hence, all roots can never be non-negative.
⇒ α=− , which is a root of Eq. (i). ∴Statement-1 is false. But
3A
Then, Aα 3 + Bα 2 + Cα + D = 0 Coefficient of x 2
Sum of roots = − =0
3 2 Coefficient of x 3
 B  B  B
⇒ A −  + B −  + C −  +D=0 i.e., Statement-2 is true.
 3A   3A   3A 
87. Statement-2 is obviously true.
B3 B3 BC y = ax 2 + bx + c
⇒ − 2
+ − +D=0 But
27 A 9A 2 3A
 b c
⇒ 2 B 3 − 9 ABC + 27 A 2D = 0 y = a x2 + x + 
 a a
Now, comparing with 2 B 3 + k1ABC + k2 A 2D = 0, we get  b
2
D 
= a  x +  − 2  [where, D = b 2 − 4ac]
k1 = − 9, k2 = 27  2a  4a 

∴ k2 − k1 = 27 − ( − 9 ) = 36 = 6 2 2
 b 1  D
Hence, both statements are true and Statement-2 is a correct ⇒  x +  = y + 
 2a  a  4a 
explanation of Statement-1.
b D
85. Q x, y , z ∈ R Let x+ = X and y + = Y.
2a 4a
x + y + z =6 …(i) 1
and xy + yz + zx = 8 …(ii) ∴ X2 = Y
a
⇒ xy + ( x + y ) {6 − ( x + y )} = 8 [from Eq. (i)] b
⇒ xy + 6 x + 6y − ( x 2 + 2 xy + y 2 ) = 8 Equation of axis, X = 0 i.e. x + =0
2a
or y 2 + (x − 6) y + x 2 − 6x + 8 = 0 or 2ax + b = 0
∴ ( x − 6 ) 2 − 4 ⋅ 1 ⋅ ( x 2 − 6 x + 8 ) ≥ 0, ∀ y ∈ R Hence, y = ax + bx + c is symmetric about the line
2

2ax + b = 0.
⇒ − 3 x 2 + 12 x + 4 ≥ 0 or 3 x 2 − 12 x − 4 ≤ 0
∴ Both statements are true and Statement-2 is a correct
4 4
or 2− ≤ x ≤2 + explanation of Statement-1.
3 3
88. Q(1 + m ) x 2 − 2 (1 + 3m ) x + (1 + 8m ) = 0
 4 4 
or x∈ 2− ,2 + ∴ D = 4 (1 + 3m ) 2 − 4 (1 + m ) (1 + 8m ) = 4m (m − 3 )
 3 3 
(i) Both roots are imaginary.
 4 4 
Similarly, y ∈ 2− ,2 + ∴ D<0
 3 3 
⇒ 4m (m − 3 ) < 0
Chap 02 Theory of Equations 193

⇒ 0 <m <3 Combining all cases, we get


or m ∈( 0, 3 )  1
m ∈  − 1, − 
(ii) Both roots are equal.  8
∴ D=0 (vii) Roots are equal in magnitude but opposite in sign, then
⇒ 4m (m − 3 ) = 0 Consider the following cases:
⇒ m = 0, 3 Case I Sum of the roots = 0
(iii) Both roots are real and distinct. 2 (1 + 3m )
⇒ =0
∴ D>0 (1 + m )
⇒ 4m (m − 3 ) > 0 1
⇒ m = − ,m ≠1
⇒ m < 0 or m > 3 3
∴ m ∈ ( − ∞, 0 ) ∪ (3, ∞ ) Case II D > 0 ⇒ 4m(m − 3 ) > 0
(iv) Both roots are positive. ⇒ m ∈ ( − ∞, 0 ) ∪ ( 3, ∞ )
Case I Sum of the roots > 0 Combining all cases, we get
2 (1 + 3m ) 1
⇒ >0 m=−
(1 + m ) 3
 1  (viii) Atleast one root is positive, then either one root is positive
⇒ m ∈ ( − ∞, − 1 ) ∪  − , ∞ or both roots are positive.
 3 
i.e. (d ) ∪ ( f )
Case II Product of the roots > 0
(1 + 8m )  1
or m ∈ ( − ∞, − 1 ) ∪  − 1, −  ∪ [3, ∞ )
⇒ >0  8
(1 + m )
(ix) Atleast one root is negative, then either one root is
 1 
m ∈ ( − ∞, − 1 ) ∪  − , ∞ negative or both roots are negative.
 8 
 1
D≥0 i.e. (e ) ∪ ( f ) or m ∈  − 1, − 
Case III  8
⇒ 4m (m − 3 ) ≥ 0
(x) Let roots are 2α are 3α. Then,
m ∈ ( − ∞, 0 ] ∪ [ 3, ∞ )
Consider the following cases:
Combining all Cases, we get 2 (1 + 3m )
m ∈ ( − ∞, − 1 ) ∪ [ 3, ∞ ) Case I Sum of the roots = 2 α + 3 α =
(1 + m )
(v) Both roots are negative.
2 (1 + 3m )
Consider the following cases: ⇒ α=
5 (1 + m )
2 (1 + 3m )
Case I Sum of the roots < 0 ⇒ <0 (1 + 8m )
(1 + m ) Case II Product of the roots = 2 α ⋅ 3 α =
(1 + m )
 1
(1 + 8m )
⇒ m ∈  − 1, − 
 3 ⇒ 6α 2 =
(1 + m )
(1 + 8m )
Case II Product of the roots > 0 ⇒ >0 From Eqs. (i) and (ii), we get
(1 + m ) 2
 1  2 (1 + 3m )  (1 + 8m )
⇒ m ∈ ( − ∞, 1 ) ∪  − , ∞ 6  =
 8   5 (1 + m )  (1 + m )
Case III D ≥ 0 ⇒ 24 (1 + 3m ) 2 = 25 (1 + 8m ) (1 + m )
4m (m − 3 ) ≥ 0 ⇒ m ∈ ( − ∞, 0 ] ∪ [3, ∞ ) ⇒ 24 ( 9m 2 + 6m + 1 ) = 25 ( 8m 2 + 9m + 1 )
Combining all cases, we get
16m 2 − 81m − 1 = 0
m ∈φ
(vi) Roots are opposite in sign, then 81 ± ( − 81 ) 2 + 64
or m=
Case I Consider the following cases: 32
Product of the roots < 0 81 ± 6625
⇒ m=
(1 + 8m ) 32
⇒ <0
(1 + m ) 89. Q2x 2 − 2 (2m + 1) x + m (m + 1) = 0 [Q m ∈ R ]
 1 ∴ D = [ − 2 (2m + 1 )] − 8m (m + 1 )
2
[ D = b 2 − 4ac ]
m ∈  − 1, − 
 8
= 4 {(2m + 1 ) 2 − 2m (m + 1 )}
Case II D > 0 ⇒ 4m (m − 3 ) > 0
⇒ m ∈ ( − ∞, 0 ) ∪ (3, ∞ ) = 4 (2m 2 + 2m + 1 )
194 Textbook of Algebra

 1  1
2
1 Combining all cases, we get
= 8  m 2 + m +  = 8  m +  +  > 0
 2  2  4  7 + 33 
 m ∈ , ∞
 2 
or D > 0, ∀ m ∈ R …(i)
(iii) Both roots lie in the interval (2, 3).
b 2 (2m + 1 )  1
x -coordinate of vertex = − = = m +  …(ii) Consider the following cases:
2a 4  2
and let
1
f ( x ) = x 2 − (2m + 1 ) x + m (m + 1 ) …(iii)
2 f(2) f(3)
(i) Both roots are smaller than 2.
X
2 α β 3

Case I D ≥ 0
∴ m ∈R [from Eq. (i)]
X Case II f (2 ) > 0
α β 2  7 − 33   7 + 33 
∴ m ∈  − ∞,  ∪ , ∞ [from part (a)]
Consider the following cases:  2   2 
Case I D ≥ 0 Case III f (3 ) > 0
∴ m ∈R [from Eq. (i)] 1
Case II x -coordinate of vertex < 2. ⇒ 9 − 3 (2m + 1 ) + m (m + 1 ) > 0
2
1
⇒ m + <2 [from Eq. (ii)] or m 2 − 11m + 12 > 0
2
 11 − 73   11 + 73 
or m<
3 ∴ m ∈  − ∞,  ∪ , ∞
2  2   2 
Case III f (2 ) > 0 Case IV 2 < x -coordinate of vertex < 3
1 1
⇒ 4 − (2m + 1 ) 2 + m (m + 1 ) > 0 ⇒ 2 <m + <3
2 2
⇒ m 2 − 7m + 4 > 0 3 5  3 5
or < m < or m ∈  , 
2 2  2 2
 7 − 33   7 + 33 
∴ m ∈  − ∞,  ∪ , ∞ Combining all cases, we get
 2   2 
m ∈φ
Combining all cases, we get (iv) Exactly one root lie in the interval (2,3) .
 7 − 33  Consider the following cases:
m ∈  − ∞, 
 2  Case I D > 0
∴ m ∈R [from Eq. (i)]
(ii) Both roots are greater than 2.
Consider the following cases:

3
f(2) X
2 α β
X
2 α β
Case II f (2 ) f (3 ) < 0
Case I D ≥ 0  1 
 4 − 2 ( 2m + 1 ) + m ( m + 1 )
∴ m ∈R [from Eq. (i)]  2 
Case II x -coordinate of vertex > 2  1 
9 − 3 ( 2m + 1 ) + m ( m + 1 ) < 0

1
m + >2 [from Eq. (ii)]  2 
2
⇒ ( m 2 − 7m + 4 ) ( m 2 − 11m + 12 ) < 0
3
∴ m>  7 − 33   7 + 33 
2 ⇒ m −  m − 
Case III f (2 ) > 0  2   2 
 7 − 33   33   11 − 73   11 + 73 
m ∈  − ∞,  ∪ 7 + , ∞ [from part (a)] m −  m −  <0
 2   2   2   2 
Chap 02 Theory of Equations 195

7 + √33  11 − 73 11 + 73 
∴ m ∈ , 
+ + 2 +  2 2 
7 – √33 – 7 – √73 – 11 + √73
Combining all cases, we get
2 2 2
 7 − 33 7 + 33 
m ∈ , 
 7 − 33 11 − 73   7 + 33 11 + 73   2 2 
∴ m ∈ ,  ∪ , 
 2 2   2 2  (vii) Atleast one root lies in the interval (2, 3).
Combining all cases, we get i.e. (d ) ∪ (c )
 7 − 33 11 − 73   7 + 33 11 + 73   7 − 33 11 − 73   7 + 33 11 + 73 
m ∈ ,  ∪ ,  ∴ m ∈ ,  ∪ , 
 2 2   2 2   2 2   2 2 
(v) One root is smaller than 1 and the other root is greater (viii) Atleast one root is greater than 2.
than 1.
i.e. (Exactly one root is greater than 2) ∪ (Both roots are
Consider the following cases: greater than 2)

1 X 2
α β X
α β
Case I D > 0
∴ m ∈R [from Eq. (i)] or(Exactly one root is greater than 2) ∪ (b ) …(I)
Case II f (1 ) < 0 Consider the following cases:
1 Case I D > 0
⇒ 1 − (2m + 1 ) + m (m + 1 ) < 0 [from Eq. (iii)]
2 ∴ m ∈R [from Eq. (i)]
⇒ m 2 − 3m < 0 Case II f (2 ) < 0
⇒ m (m − 3 ) < 0 ⇒ m 2 − 7m + 4 < 0
∴ m ∈( 0, 3 )
 7 − 33 7 + 33 
Combining both cases, we get ∴ m ∈ , 
m ∈( 0, 3 )  2 2 
(vi) One root is greater than 3 and the other root is smaller Combining both cases, we get
than 2.  7 − 33 7 + 33 
Consider the following cases: m ∈ ,  …(II)
 2 2 
Finally from Eqs. (I) and (II), we get
 7 − 33 7 + 33   7 + 33 
m ∈ ,  ∪ , ∞
 2 2   2 
2 3 X
α β (ix) Atleast one root is smaller than 2.
i.e. (Exactly one root is smaller than 2) ∪(Both roots are
smaller than 2)
Case I D > 0
or (h) (II) ∪ (a)
∴ m ∈R [from Eq. (i)]
 7 − 33   7 − 33 7 + 33 
Case II f (2 ) < 0 We get, m ∈  − ∞,  ∪ , 
⇒ m 2 − 7m + 4 < 0  2   2 2 

7 − 33 7 + 33 (x) Both 2 and 3 lie between α and β.


∴ <m < Consider the following cases:
2 2
 7 − 33 7 + 33  Case I D > 0
∴ m ∈ ,  ∴ m ∈R [from Eq. (i)]
 2 2 
Case III f (3 ) < 0
⇒ m 2 − 11m + 12 < 0
11 − 73 11 + 73 2 3
∴ <m < α β
X
2 2
196 Textbook of Algebra

Case II f (2 ) < 0 1
cn + 1
⇒ m − 7m + 4 < 0
2 ⇒ α= 
a
 7 − 33 7 + 33  ∴ From Eq. (i), we get
∴ m ∈ , 
 2 2  1 n
cn + 1 cn + 1 b
Case III f (3 ) < 0   +  =−
a a a
⇒ m 2 − 11m + 12 < 0
1 1 1 n
− +1 − +1
 11 − 73 11 + 73  ⇒ (c )n + 1 ⋅ a n+1
+ (c n )n + 1 ⋅ a n+1
+b=0
∴ m ∈ , 
 2 2  1 n 1 1

Combining all cases, we get ⇒ c n + 1 ⋅ a n + 1 + (cn )n + 1 ⋅ a n + 1 + b = 0


 11 − 73 7 + 33  1 1
m ∈ , 
 2 2  ⇒ (anc )n + 1 + (c na )n + 1 + b = 0
α 93. We have, α+β=−
b
90. Q =r
β a
c m n
α+β r +1 αβ = ⇒ γ+δ=− and γδ =
⇒ = a l l
α −β r −1
Now, sum of the roots
[using componendo and dividendo method]
= (αγ + βδ ) + (αδ + βγ ) = (α + β ) γ + (α + β ) δ
− b /a r + 1
⇒ = ⇒ b (1 − r ) = (1 + r ) D = (α + β ) ( γ + δ )
D r −1
 b   m  mb
a = −  −  =
 a  l  al
On squaring both sides, we get
and product of the roots
⇒ b 2(1 − r ) 2 = (1 + r ) 2 (b 2 − 4ac ) = (αγ + βδ ) (αδ + βγ )
(1 + r ) 2 b 2 = (α 2 + β 2 ) γδ + αβ ( γ 2 + δ 2 )
or (1 + r ) 2 ⋅ 4ac = b 2( 4r ) or =
r ac = {(α + β ) 2 − 2αβ } γδ + αβ {( γ + δ ) 2 − 2 γδ }
1 1 1
91. We have, + =  b  2 2c  n c  m  2 2n 
x+p x+q r =  −  −  +  −  − 
  a  l a  l 
(x + q ) + (x + p ) 1  a l 
⇒ =
x 2 + ( p + q ) x + pq r b − 2ac  n c m − 2nl  (b 2 − 2ac ) ln + ( m 2 − 2nl ) ac
2 2
= 2  +  2 =
⇒ x 2 + ( p + q − 2r ) x + pq − ( p + q ) r = 0  a l a  l  a 2l 2
Now, since the roots are equal in magnitudes, but opposite in ∴ Required equation is
sign. Therefore,  mb  (b 2 − 2ac ) ln + (m 2 − 2nl ) ac
Sum of the roots = 0 x2 −   x + =0
 al  a 2l 2
⇒ p + q − 2r = 0
⇒ a 2l 2x 2 − mbalx + (b 2 − 2ac ) ln + (m 2 − 2nl ) ac = 0
⇒ p + q = 2r …(i)
and product of the roots = pq − ( p + q ) r 94. Since, the roots are equal.
∴ D=0
 p + q
= pq − ( p + q )   [from Eq. (i)] ⇒ 4 (b 2 − ac ) 2 − 4 (a 2 − bc ) (c 2 − ab ) = 0
 2 
⇒ (b 2 − ac ) 2 − (a 2 − bc ) (c 2 − ab ) = 0
2 pq − p 2 − q 2 − 2 pq
= ⇒ b (a 3 + b 3 + c 3 − 3abc ) = 0
2
p2 + q2 ⇒ b = 0 or a 3 + b 3 + c 3 − 3abc = 0
=−
2 95. Let α and β be the roots of x 2 − px + q = 0. Then,
92. Let α be one root of the equation ax 2 + bx + c = 0. α+β=p …(i)
Then, other root be α . n αβ = q …(ii)
b 1
∴ α + αn = − …(i) And α and be the roots of x 2 − ax + b = 0. Then,
a β
c 1
and α ⋅ αn = α + =a …(iii)
a β
⇒ αn + 1 =
c α
=b ... (iv)
a β
Chap 02 Theory of Equations 197

Now, LHS = (q − b ) 2 ∴ λ = 10,


1
2 10
 α
= αβ −  [from Eqs. (ii) and (iv)] ⇒ (2 + 3 ) x
2
− 2x
= 10, 10 − 1
 β
2 2 ⇒ x 2 − 2 x = log 2 + 10, − log 2 + 10
21   1  3 3
= α β −  = α 2 (α + β ) − α +  
 β   β  ⇒ ( x − 1 ) 2 = 1 + log 2 + 3
10, 1 − log 2 + 3
10
= α ( p − a ) [from Eqs. (i) and (iii)]
2 2
∴ ( x − 1 ) = 1 + log 2 +
2
10
3
α
= αβ ⋅ ( p − a ) 2 [Q ( x − 1 ) 2 ≠ 1 − log 2 + 10 ]
β 3

= bq ( p − a ) 2 [from Eqs. (ii) and (iv)] ⇒ x = 1 ± (1 + log 2 + 3


10 )
= RHS ⇒ x 1 = 1 + (1 + log 2 + 3
10 )
96. Since, roots of x 2 − 2px + q = 0 are equal.
x 2 = 1 − (1 + log 2 + 3
10 )
∴ D=0
2
i.e., ( −2 p ) 2 − 4q = 0 or p 2 = q …(i)  x 
99. We have, x2+   =8
Now, (1 + y ) x 2 − 2 ( p + y ) x + (q + y ) = 0  x − 1
2
∴ Discriminant = 4 ( p + y ) 2 − 4 (1 + y ) (q + y )  x  x
⇒ x +  − 2⋅ x ⋅ =8
= 4 ( p 2 + 2 py + y 2 − q − y − qy − y 2 )  x − 1 (x − 1)
2
= 4 [(2 p − q − 1 ) y + p 2 − q ]  x2   x2 
⇒   −2   −8 = 0 …(i)
= 4 [(2 p − p 2 − 1 ) y + 0 ] [from Eq. (i)]  x − 1  x − 1
= − 4 ( p − 1 ) 2y x2
Let y = . Then, Eq. (i) reduces to
>0 [Qy < 0 and p ≠ 1] x −1
Hence, roots of (1 + y ) x 2 − 2 ( p + y ) x + (q + y ) = 0 are real y 2 − 2y − 8 = 0
and distinct. ⇒ (y − 4 ) (y + 2 ) = 0
2
97. x log x ( x + 3) = 16 …(i) ∴ y = 4, − 2
Equation is defined, when x2
If y = 4, then 4=
x > 0, x ≠ 1, x ≠ − 3, x −1
Then, (x + 3)2 = 42 [by property] or x 2 − 4x + 4 = 0
⇒ x+3=±4
or (x − 2)2 = 0
∴ x = 1 and x = − 7
But x ≠ 1, x ≠ − 7 or x =2
i.e. no solution. ∴ x1 = 2
∴ x ∈φ x2
and if y = − 2, then −2 =
98. Q(2 + 3 ) x
2
− 2x + 1
+ (2 − 3 ) x
2
− 2x − 1
=
101 x −1
10 (2 − 3 ) or x 2 + 2x − 2 = 0
x2 − 2x
⇒ (2 + 3 ) ⋅ (2 + 3 ) (2 − 3 ) − 2 ± (4 + 8)
101 ∴ x=
x2 − 2x − 1 2
+ (2 − 3 ) ⋅ (2 − 3 ) =
10 ⇒ x = −1 ± 3
x2 − 2x x2 − 2x 101
⇒ (2 + 3 ) + (2 − 3 ) = ∴ x 2 = − 1 + 3, x 3 = − 1 − 3
10
or (2 + 3 ) x2 − 2x
+
1
=
101
…(i) 100. We have, x + 8 + 2 ( x + 7 ) + ( x + 1 ) − ( x + 7 ) = 4 …(i)
2
(2 + 3 ) x − 2 x 10
Let (x + 7) = λ …(ii)
 1 
Q 2 − 3 = 2 + 3  or x = λ2 − 7
  Then, Eq. (i) reduces to
2
− 2x
Let (2 + 3 ) x = λ, then Eq. (i) reduces to
( λ2 − 7 + 8 + 2 λ ) + ( λ2 − 7 + 1 − λ ) = 4
1 101
λ+ = ⇒ ( λ + 1 ) + ( λ2 − λ − 6 ) = 4
λ 10
⇒ 10 λ2 − 101 λ + 10 = 0 or ( λ2 − λ − 6 ) = 3 − λ
or ( λ − 10 ) (10 λ − 1 ) = 0
198 Textbook of Algebra

On squaring both sides, we get Case I If 0 < x 2 + 2 x − 3 < 1


λ2 − λ − 6 = 9 + λ2 − 6 λ ⇒ 4 < x 2 + 2x + 1 < 5
⇒ 5 λ = 15 ⇒ 4 < (x + 1)2 < 5
∴ λ =3
⇒ − 5 < ( x + 1 ) < − 2 or 2 < x + 1 < 5
⇒ (x + 7) = 3 [from Eq. (ii)]
⇒ − 5 − 1 < x < − 3 or 1 < x < 5 − 1
or x + 7 =9
∴ x =2 ∴ x ∈ ( − 5 − 1, − 3 ) ∪ (1, 5 − 1 ) …(ii)
and x = 2 satisfies Eq. (i). | x + 4| − | x |
Then, <1
Hence, x 1 = 2 (x − 1)
2 2
4 x + 2 (2a + 1 ) 2 x + 4a 2 − 3 > 0 − (x + 4) + x
101. We have, …(i) Now, x < − 4, then <1
(x − 1)
x2
Putting t = 2 in the Eq. (i), we get
4
⇒ 1+ >0
t 2 + 2 (2a + 1 ) t + 4a 2 − 3 > 0 x −1
Let f (t ) = t 2 + 2 (2a + 1 ) t + 4a 2 − 3
2
[Q t > 0,∴2 x > 0] (x + 3)
⇒ >0
Q f (t ) > 0 (x − 1)
∴ x ∈ ( − ∞, − 3 ) ∪ (1, ∞ )
⇒ x ∈ ( − ∞, − 4 ) [Q x < − 4] …(iii)
x+4+x
− 4 ≤ x < 0, then −1 < 0
(x − 1)
(x + 5)
⇒ <0
T-axis (x − 1)
Consider the following cases: ∴ x ∈ ( − 5, 1 )
Case I Sum of the roots > 0 ⇒ x ∈ [ − 4, 0 ) [Q − 4 ≤ x < 0] …(iv)
( 2a + 1 ) (x + 4) − x
−2 >0 and x ≥ 0, then <1
1 (x − 1)
 1 ⇒ 1−
4
>0
∴ a ∈  − ∞, − 
 2 x −1
Case II Product of the roots > 0 (x − 5)
⇒ >0
4a 2 − 3 (x − 1)
⇒ >0
1 ∴ x ∈ ( − ∞, 1 ) ∪ ( 5, ∞ )
3 ⇒ x ∈ [ 0, 1 ) ∪ ( 5, ∞ ) [Q x ≥ 0] …(v)
or a2 >
4 From Eqs. (iii), (iv) and (v), we get
 3  3  x ∈ ( − ∞, 1 ) ∪ ( 5, ∞ ) …(vi)
or a ∈  − ∞, −  ∪  , ∞
 2   2  Now, common values in Eqs. (ii) and (iv) is
x ∈ ( − 5 − 1, − 3 ) …(vii)
Case III D<0
⇒ 4 ( 2a + 1 ) 2 − 4 ⋅ 1 ⋅ ( 4a 2 − 3 ) < 0 Case II If x 2 + 2x − 3 > 1
⇒ 4a + 4 < 0 ⇒ x 2 + 2x + 1 > 5 ⇒ (x + 1)2 > 5
∴ a < −1 ⇒ x+1<− 5
or a ∈ ( − ∞, − 1 ) or x+1> 5
Combining all cases, we get
∴ x ∈ ( − ∞, − 1 − 5 ) ∪ ( 5 − 1, ∞ ) …(viii)
 3 
a ∈ ( − ∞, − 1 ) ∪  , ∞ | x + 4| − | x |
 2  Then, >1
(x − 1)
 | x + 4| − | x | −4
102. We have, log x 2 + 2 x − 3   >0 Now, x < − 4, then >1
 x −1  x −1
The given inequation is valid for 4
| x + 4| − | x | ⇒ 1+ <0
>0 x −1
(x − 1) x+3
⇒ <0
and x 2 + 2 x − 3 > 0, ≠ 1 …(i) x −1
Now, consider the following cases: ∴ x ∈ ( − 3, 1 )
Chap 02 Theory of Equations 199

which is false. [Qx < − 4] ⇒ x2 − x − 1 = 0


2x + 4 ∴ x =1
− 4 ≤ x < 0, then −1 > 0
(x − 1) 1± 5
∴ x=
(x + 5) 2
⇒ >0
(x − 1) 1± 5
∴ x= fail
∴ x ∈ ( − ∞, − 5 ) ∪ (1, ∞ ) 2
which is false. [Q− 4 ≤ x < 0 ] 1− 5
⇒ x= [Q x < 0]
4 2
and x ≥ 0, then >1
x −1 1− 5 1− 5
⇒ x= , 1, then y = ,0
4 2 2
⇒ 1− <0 1 − 5 1 − 5
x −1 ∴ Solutions are  ,  and (1, 0).
x −5  2 2 
⇒ <0
x −1 1 − 5 1 − 5
Hence, all pairs ( 0, 1 ), (1, 0 ) and  ,  are solutions
∴ x ∈(1, 5 ) …(ix)  2 2 
which is false. [Qx ≥ 0] of the original system of equations.
Now, common values in Eq. (viii) and (ix) is 104. Given, α , β and γ are the roots of the cubic equation
∴ x ∈ ( 5 − 1, 5 ) …(x) x 3 − px 2 + qx − r = 0 …(i)
Combining Eqs. (viii) and (x), we get ∴ α + β + γ = p, αβ + βγ + γα = q , αβγ = r
x ∈ ( − 5 − 1, − 3 ) ∪ ( 5 − 1, 5 ) 1
(i) Let y = βγ +
103. Let y ≥ 0, then | y | = y α
αβγ + 1 r + 1
and then given system reduces to ⇒ y = =
α α
| x 2 − 2x | + y = 1 …(i) r +1
∴ α=
and x + y =1
2
…(ii) y
From Eqs. (i) and (ii), we get From Eq. (i), we get
x 2 = | x 2 − 2x | α 3 − pα 2 + qα − r = 0
⇒ x 2 = | x | | x − 2| (r + 1 ) 3 p (r + 1 ) 2 q (r + 1 )
⇒ − + −r = 0
y3 y2 y
Now, x < 0, 0 ≤ x < 2, x ≥ 2
x 2 = x ( x − 2 ), x 2 = − x ( x − 2 ) or ry 3 − q(r + 1 )y 2 + p (r + 1 ) 2y − (r + 1 ) 3 = 0

x 2 = x (x − 2) (ii) Let y = β + γ − α = (α + β + γ ) − 2α = p − 2 α
p −y
∴ x=0 ∴ α=
2
⇒ x (x + x − 2) = 0
From Eq. (i), we get
∴ x=0 α 3 − pα 2 + qα − r = 0
fail ∴ x = 0, 1 fail
(p − y ) 3 p (p − y )2 q (p − y )
⇒ x = 0, 1, then y = 1, 0 ⇒ − + −r = 0
∴Solutions are (0, 1) and (1, 0). 8 4 2
If y < 0 then | y | = − y and then given system reduces to or y 3 − py 2 + ( 4q − p 2 )y + ( 8r − 4 pq + p 3 ) = 0
| x 2 − 2x | + y = 1 …(iii) Also product of roots = − (8r − 4 pq + p 3 )
and x2 − y = 1 …(iv) 105. Assume α + iβ is a complex root of the given equation, then
From Eqs. (iii) and (iv), we get conjugate of this root, i.e. α − iβ is also root of this equation.
| x 2 − 2x | + x 2 = 2 On putting x = α + iβ and x = α − iβ in the given equation, we
get
⇒ | x | | x − 2| + x 2 = 2
2
A12 A22 A3 An2
Now, x < 0, 0 ≤ x < 2, x ≥ 2 + + +… +
α + iβ − a1 α + iβ − a 2 α + iβ − a 3 α + iβ − an
x (x − 2) + x 2 = 2
− x (x − 2) + x 2 = 2 = ab 2 + c 2(α + iβ ) + ac …(i)
x (x −2 ) + x = 2
2
A12 A22 A32 An2
and + + +… +
⇒ 2x 2 − 2x − 2 = 0 ⇒ 2x = 2 α − iβ − a1 α − iβ − a 2 α − iβ − a 3 α − iβ − an
⇒ x2 − x − 1 = 0 = ab + c (α − iβ ) + ac
2 2
…(ii)
200 Textbook of Algebra

On subtracting Eq. (i) from Eq. (ii), we get ∴ (a + (a 2 + 1 ) ) 2 − 4 > 0


 A12 A22 A32
2iβ  + + ⇒ (a + (a 2 + 1 ) + 2 ) (a + (a 2 + 1 ) − 2 ) > 0
( α − a1 ) + β (α − a2 ) + β (α − a 3 )2 + β2
2 2 2 2

a + (a 2 + 1 ) + 2 > 0

Q
An2
+…+ + c2 = 0
(α − an ) + β
2 2
 ∴ a + (a 2 + 1 ) − 2 > 0
The expression in bracket ≠ 0 ⇒ (a 2 + 1 ) > 2 − a
∴ 2iβ = 0 ⇒ β = 0
a ≥ 2
Hence, all roots of the given equation are real. 
or a + 1 > (2 − a ) , if a < 2
2 2
106. Given equation is
x 4 + 2ax 3 + x 2 + 2ax + 1 = 0 …(i) a ≥ 2

⇒ or a > 3 , if a < 2
On dividing by x 2, we get 
2a 1 4
x 2 + 2ax + 1 + + 2 =0
a ≥ 2
x x 
 2 1  1 ⇒ or 3 < a < 2
⇒  x + 2  + 2a x +  +1=0  4
 x   x
 1
2
 1 3 3 
⇒  x +  − 2 + 2a  x +  + 1 = 0 Hence, < a < ∞ or a ∈  , ∞
 4 
x  x 4

1
2 107. We have, [2 x ] − [ x + 1 ] = 2 x
  1
or  x +  + 2a x +  − 1 = 0 LHS = Integer
 x  x Since,
1 ∴ RHS = 2x = Integer
or y 2 + 2ay − 1 = 0, where y = x + ⇒ [2 x ] = 2 x
x
Now, − [ x + 1] = 0
− 2a ± ( 4a 2 + 4 )
∴ y = = − a ± (a 2 + 1 ) ⇒ [ x + 1] = 0
2
or 0 ≤ x + 1 <1
Taking ‘+’ sign, we get
or −1 ≤ x < 0
y = − a + (a 2 + 1 ) or − 2 ≤ 2x < 0
⇒ x+
1
= − a + (a 2 + 1 ) ∴ 2 x = − 2, − 1
x 1
or x = − 1, −
or x 2 + (a − (a 2 + 1 ) ) x + 1 = 0 …(ii) 2
1
Taking ‘−’ sign, we get y = − a − (a 2 + 1 ) or x 1 = − 1, x 2 = −
2
1 108. We have, (a 2 + 3 ) x 2 + (a + 2 ) x − 6 < 0
⇒ x+ = − a − (a 2 + 1 )
x
or x 2 + (a + (a 2 + 1 ) ) x + 1 = 0 …(iii)
Let α , β be the roots of Eq. (ii) and γ, δ be the roots of Eq. (iii).
Then, α + β = (a 2 + 1 ) − a
X
and αβ = 1
and γ + δ = − (a 2 + 1 ) − a Let f ( x ) = (a 2 + 3 ) x 2 + (a + 2 ) x − 6
and γδ =1 Q (a 2 + 3 ) > 0 and f ( x ) < 0
Clearly, α + β > 0 and αβ > 0 ∴ D>0
∴Either α , β will be imaginary or both real and positive ⇒(a + 2 ) + 24 (a + 3 ) > 0 is true for all a ∈ R .
2 2
according to the Eq. (i) has atleast two distinct negative roots.
Therefore, both γ and δ must be negative. Therefore, 109. We have, 6 x 2 − 77[ x ] + 147 = 0
(i) γδ > 0, which is true as γ δ = 1. 6 x 2 + 147
⇒ = [x ]
(ii) γ+δ<0 77
⇒ − (a + (a 2 + 1 ) ) < 0 ⇒ (0.078) x 2 = [x ] − 1.9
⇒ a + (a + 1 ) > 0, which is true for all a.
2 Q (0.078 ) x 2 > 0 ⇒ x 2 > 0

∴ a ∈R ∴ [ x ] − 1.9 > 0
(iii) D>0 or [ x ] > 1.9
Chap 02 Theory of Equations 201

∴ [ x ] = 2, 3, 4, 5,…
If [ x ] = 2, i. e. 2 ≤ x < 3
2 − 1 .9
Then, x2 = = 1. 28
0.078 α β
X
γ δ
∴ x = 1.13 [fail]
If [ x ] = 3, i. e. 3 ≤ x < 4
Let f ( x ) = x 2 − 2(a + 1 ) x + a (a − 1 ), thus the following
3 − 1.9
Then, x2 = = 14.1 conditions hold good:
0.078
Consider the following cases:
∴ x = 3.75 [true]
Case I D>0
If [ x ] = 4, i.e. 4 ≤ x < 5
4 − 1.9 ⇒ 4 (a + 1 ) 2 − 4a (a − 1 ) > 0
Then, x2 = = 26.9
0.078 ⇒ 3a + 1 > 0
∴ x = 5.18 [fail] 1
∴ a>−
If [ x ] = 5, i.e. 5 ≤ x < 6 3
5 − 1.9 Case II f (α ) < 0
Then, x2 = = 39.7
0.078 ⇒ f (1 + a ) < 0
∴ x = 6.3 [fail] ⇒ (1 + a ) 2 − 2 (1 + a ) (1 + a ) + a (a − 1 ) < 0
If [ x ] = 6, i. e. 6 ≤ x < 7 ⇒ − (1 + a ) 2 + a (a − 1 ) < 0
6 − 1.9 4.1
Then, x2 = = = 52.56 ⇒ − 3a − 1 < 0
0.078 0.078 1
∴ x = 7.25 [fail] ⇒ a>−
3
If [ x ] = 7, i. e. 7 ≤ x < 8 Case III f (s ) = 0
7 − 1.9 5.1
Then, x2 = = = 65.38 ⇒ f (1 − a ) < 0
0.078 0.078
⇒ (1 − a ) 2 − 2 (a + 1 ) (1 − a ) + a (a − 1 ) < 0
∴ x = 8.08 [fail]
⇒ ( 4a + 1 ) (a − 1 ) < 0
If [ x ] = 8, i. e. 8 ≤ x < 9
1
8 − 1.9 6.1 ∴ − <a <1
Then, x2 = = = 78.2 4
0.078 0.078
Combining all cases we get
 1 
∴ x = 8.8 [true] a ∈  − , 1
 4 
If [ x ] = 9, i. e. 9 ≤ x < 10
9 − 1.9 7.1 111. pr = ( − p ) ( − r )
Then, x2 = = = 91.03
0.078 0.078 = ( α + β + γ + δ ) ( αβγ + αβδ + γδα + γδβ )
∴ x = 9.5 [true] = α 2 βγ + α 2 βδ + α 2 γδ + αβγδ + β 2 γα
If [ x ] = 10, i. e.10 ≤ x < 11
+ β 2 αδ + αβγδ + β 2 γδ + γ 2 αβ + αβγδ
10 − 1.9 8.1
Then, x2 = = = 103.8
0.078 0.078 + γ 2 δα + γ 2δβ + αβγδ + αβδ 2 + γαδ 2 + γβδ 2
∴ x = 10.2 [true] Q AM ≥ GM
If [ x ] = 11, i.e. 11 ≤ x < 12 pr
⇒ ≥ (α 16 β16 γ16 δ16 )1/6 = α β γδ = 5
11 − 1.9 16
Then, x2 =
0.078 pr
⇒ ≥5
9.1 16
= = 116.7
0.078 or pr ≥ 80
∴ x = 10.8 [fail] ∴ Minimum value of pr is 80.
Other values are fail. 112. (α 2 + β 2 ) 2 = (α + β ) (α 3 + β 3 )
Hence, number of solutions is four.
⇒ {(α + β ) 2 − 2 αβ } 2 = (α + β ) {(α + β ) 3 − 3 αβ (α + β )}
110. Since, the given equation is 2
 b 2 2c   b  − b
3
3bc 
x 2 − 2x − a 2 + 1 = 0 ⇒  2 −  = −   3 + 2 
a a  
a  a a 
⇒ (x − 1)2 = a 2
2
∴ x − 1 ≠ a or x = 1 ± a  b 2 − 2ac   − b   − b + 3abc 
3
⇒   =    
⇒ α = 1 + a and β = 1 − a  a2   a   a3 
202 Textbook of Algebra

⇒ 4a 2c 2 = acb 2 Combining all cases, we get


⇒ ac (b − 4ac ) = 0
2 k ∈ ( −∞,4 )
117. We have, a + b = 10c, ab = − 11d
As a≠0
⇒ c∆ = 0 and c + d = 10a, cd = − 11b
113. Let P ( x ) = bx 2 + ax + c ∴ a + b + c + d = 10 (a + c )
and abcd = 121 bd
As P (0) = 0
⇒ b + d = 9 (a + c )
⇒ c=0
and ac = 121
As P (1 ) = 1
Next, a − 10 ac − 11d = 0
2
⇒ a + b =1
P ( x ) = ax + (1 − a ) x 2 and c 2 − 10ac − 11b = 0
Now, P ′ ( x ) = a + 2 (1 − a ) x ⇒ a 2 + c 2 − 20 ac − 11 (b + d ) = 0
As P ′ ( x ) > 0 for x ∈( 0, 1 ) ⇒ (a + c ) 2 − 22 × 121 − 99 (a + c ) = 0
Only option (d) satisfies above condition. ⇒ a + c = 121 or − 22
114. Let the roots are α and α + 1, where α ∈ I . If a + c = − 22 ⇒a = c , rejecting these values, we have
Then, sum of the roots = 2 α + 1 = b a + c = 121
Product of the roots = α (α + 1 ) = c ∴ a + b + c + d = 10 (a + c ) = 1210
118. D≥0
Now, b 2 − 4c = ( 2α + 1 ) 2 − 4α (α + 1 ) 4 (a + b + c ) 2 − 12 λ (ab + bc + ca ) ≥ 0
= 4α + 1 + 4α − 4α − 4α = 1
2 2
(a 2 + b 2 + c 2 ) − (3 λ − 2 ) (ab + bc + ca ) ≥ 0
∴ b − 4c = 1
2
(a 2 + b 2 + c 2 )
n− 1 ∴ (3 λ − 2 ) ≤
115. Let f ( x ) = an x + an − 1 x
n
+…+ a1 x, (ab + bc + ca )
f ( 0 ) = 0;f (α ) = 0 Since, |a − b | < c
⇒ f ′( x ) = 0 has atleast one root between ( 0, α ). ⇒ a 2 + b 2 − 2ab < c 2 (i)
i.e. Equation |b − c | < a
nan xn − 1 + (n − 1 ) an − 1xn − 2 +…+ a1 = 0 ⇒ b 2 + c 2 − 2bc < a 2 …(ii)
has a positive root smaller than α. | c − a| < b
116. Let f ( x ) = x 2 − 2kx + k 2 + k − 5 ⇒ c 2 + a 2 − 2ca < b 2 …(iii)
Consider the following cases: From Eqs. (i), (ii) and (iii), we get
a2 + b2 + c2
<2 …(iv)
ab + bc + ca
f(5) From Eqs. (i) and (iv), we get
4
α X 3λ − 2 < 2 ⇒ λ <
P S 3
119. Q x 2 − 2mx + m 2 − 1 = 0
Case I D≥0 ⇒ (x − m )2 = 1
⇒ 4k − 4 .1(k + k − 5 ) ≥ 0
2 2
∴ x − m = ± 1 or x = m − 1, m + 1
⇒ −4(k − 5 ) ≥ 0 According to the question,
⇒ k −5 ≤ 0 m − 1 > − 2, m + 1 > − 2
⇒ k ≤ 5 or k ∈ ( −∞, 5 ] ⇒ m > − 1, m > − 3
Case II x-Coordinate of vertex x < 5 Then, m > −1 …(i)
2k and m − 1 < 4, m + 1 < 4
⇒ <5
2 ⇒ m < 5, m < 3 and m < 3 …(ii)
⇒ k < 5 or k ∈ ( −∞,5 ) From Eqs. (i) and (ii), we get − 1 < m < 3
Case III f (5 ) > 0 120. x 2 + px + q = 0
⇒ 25 − 10k + k 2 + k − 5 > 0 Sum of the roots = tan 30 ° + tan 15 ° = − p
Product of the roots = tan 30 °⋅ tan 15 ° = q
⇒ k − 9k + 20 > 0
2
tan 30 ° + tan 15 °
⇒ (k − 4 )(k − 5 ) > 0 or k ∈ ( −∞,4 ) ∪ (5, ∞ ) tan 45 ° = tan (30 ° + 15 ° ) =
1 − tan 30 °⋅ tan 15 °
Chap 02 Theory of Equations 203

−p 125. Let f ( x ) = x 7 + 14x 5 + 16x 3 + 30x − 560


⇒ 1= ⇒ − p =1 −q
1 −q
∴ f ′( x ) = 7 x 6 + 70 x 4 + 48 x 2 + 30 > 0, ∀ x ∈ R
⇒ q− p =1
∴ 2 + q − p =3 ⇒ f ( x ) is an increasing function, for all x ∈ R
121. The equation x 2 − px + r = 0 has roots (α , β) and the equation Hence, number of real solutions is 1.
α  126. Let f ( x ) = x 3 − px + q
x 2 − qx + r has roots  , 2β .
2 
∴ f ′( x ) = 3 x 2 − p
α
⇒ r = αβ and α + β = p and + 2β = q ⇒ f ′′( x ) = 6 x
2
2q − p 2 (2 p − q )
⇒ β= and α = p
3 3 √3
2
⇒ αβ = r = (2q − p ) (2 p − q ) –
p
9 √3
122. α + β = −a
| α − β | < 5 ⇒ (α − β ) 2 < 5 For maxima or minima, f ′( x ) = 0
⇒ a − 4 < 5 ⇒ a ∈ ( − 3, 3 )
2
∴ x=±
p
3
123. Suppose roots are imaginary, then β = α
 p  p
1
=α ⇒ f ′′   =6   > 0
and  3  3
β

⇒ β=−
1  p p
[not possible] and f ′′  −  = −6 <0
β  3 3
⇒ Roots are real ⇒( p 2 − q ) (b 2 − ac ) ≥ 0 p
Hence, given cubic minima at x = and maxima at
⇒ Statement −1 is true. 3
2b 1 p
− =α + x=− .
a β 3
α c 127. Let f ( x ) = x 2 − 8kx + 16 (k 2 − k + 1)
and = , α + β = − 2 p, αβ = q
β a
If β = 1, then α = q
⇒ c = qa [not possible]
− 2b
Also, α+1=
a
X
− 2b 4
⇒ − 2p =
a ∴ D>0
⇒ b = ap [not possible] ⇒ 64k 2 − 4 ⋅ 16 (k 2 − k + 1 ) > 0
⇒ Statement −2 is true but it is not the correct explanation of ⇒ k >1 …(i)
Statement-1. −b 8k
124. Let α,4β be roots of x 2 − 6x + a = 0 and α , 3 β be the roots of ⇒ >4 ⇒ >4
2a 2
x 2 − cx + 6 = 0. ⇒ k >1 …(ii)
Then, α + 4 β = 6 and 4αβ = a …(i) and f (4) ≥ 0
α + 3 β = c and 3α β = 6 …(ii) ⇒ 16 − 32k + 16 (k 2 − k + 1 ) ≥ 0
From Eqs. (i) and (ii), we get ⇒ k 2 − 3k + 2 ≥ 0
a = 8, αβ = 2
⇒ (k − 1 ) (k − 2 ) ≥ 0
Now, first equation becomes ⇒ k ≤ 1 or k ≥ 2 …(iii)
x 2 − 6x + 8 = 0 From Eqs. (i), (ii) and (iii), we get
⇒ x = 2, 4 k ≥2
If α = 2, 4 β = 4, then 3 β = 3 kmin = 2
3 128. Since, roots of bx 2 + cx + a = 0 are imaginary.
If α = 4, 4 β = 2, then 3 β = [non-integer]
2 ∴ c 2− 4ab < 0
∴Common root is x = 2. ⇒ − c 2 > − 4ab …(i)
204 Textbook of Algebra

Let f ( x ) = 3b 2x 2 + 6bcx + 2c 2 132. Let α be the common root.


Then, α 2 + bα − 1 = 0 and α 2 + α + b = 0
Since, 3b > 0
2
2
1 b b −1 −1 1
and D = (6bc ) 2 − 4 (3b 2 ) (2c 2 ) = 12b 2c 2 ⇒ × =
1 1 1 b b 1
D 12b 2c 2
∴ Minimum value of f (x ) = − =− = −c 2 > −4ab ⇒ (1 − b ) (b + 1 ) = ( − 1 − b ) 2
2
4a 4 (3b 2 )
⇒ b 3 + 3b = 0
α β α 2 + β 2 (α + β ) 2 − 2 αβ
129. + = = …(i) ∴ b = 0, i 3, − i 3 , where i = − 1.
β α αβ αβ
and given, α 3 + β 3 = q, α + β = − p 133. Let f ( x ) = x 4 − 4 x 3 + 12x 2 + x − 1
⇒ (α + β ) 3 − 3 αβ (α + β ) = q ∴ f ′( x ) = 4 x 3 − 12 x 2 + 24 x + 1
⇒ − p 3 + 3 pαβ = q ⇒ f ′′( x ) = 12 x 2 − 24 x + 24
q + p3 = 12 ( x 2 − 2 x + 2 )
or αβ =
3p = 12 [( x − 1 ) 2 + 1 ] > 0
∴ From Eq. (i), we get
2 (q + p 3 ) i.e. f ′′( x ) has no real roots.
p2 − Hence, f ( x ) has maximum two distinct real roots, where
α β 3p p 3 − 2q
+ = = f ( 0 ) = − 1.
β α (q + p )
3
(q + p 3 )
134. Given, p ( x ) = f ( x ) − g ( x )
3p
⇒ p( x ) = (a − a1 ) x 2 + (b − b1 ) x + (c − c1 )
α β
and product of the roots = ⋅ = 1 It is clear that p( x ) = 0 has both equal roots − 1, then
β α
(b − b1 )
 p 3 − 2q  −1 −1 = −
∴ Required equation is x 2 −   x+1=0 (a − a1 )
 q + p3 
c − c1
and − 1 × −1 =
or (q + p 3 ) x 2 − ( p 3 − 2q ) x + (q + p 3 ) = 0 a − a1
130. Since, f ′ ( x ) = 12x 2 + 6x + 2 ⇒ b − b1 = 2 (a − a1 ) and c − c1 = (a − a1 ) …(i)
Here, D = 6 − 4 ⋅ 12 ⋅ 2 = 36 − 96 = − 60 < 0
2 Also given, p( −2 ) = 2
∴ f ′( x ) > 0, ∀ x ∈ R ⇒ 4 (a − a1 ) − 2 (b − b1 ) + (c − c1 ) = 2 …(ii)
⇒ Only one real root for f ( x ) = 0 From Eqs. (i) and (ii), we get
Also, f ( 0 ) = 1, f ( − 1 ) = − 2 4 (a − a1 ) − 4 (a − a1 ) + (a − a1 ) = 2
⇒ Root must lie in ( − 1, 0 ). ∴ (a − a1 ) = 2 …(iii)
 1 1 ⇒ b − b1 = 4 and c − c1 = 2 [from Eq. (i)] …(iv)
Taking average of 0 and ( − 1 ), f  −  = Now, p(2 ) = 4 (a − a1 ) + 2 (b − b1 ) + (c − c1 )
 2 4
= 8 + 8 + 2 = 18 [from Eqs. (iii) and (iv)]
 1
⇒ Root must lie in  − 1, −  . 135. Let the quadratic equation be
 2
ax 2 + bx + c = 0
 3 1
Similarly, f  −  = −
 4 2 Sachin made a mistake in writing down constant term.
∴ Sum of the roots is correct.
 3 1
⇒ Root must lie in  − , −  . i.e. α + β =7
 4 2
Rahul made a mistake in writing down coefficient of x .
131. Qα 2 − 6 α − 2 = 0 ⇒α 2 − 2 = 6 α …(i)
∴ Product of the roots is correct.
and β − 6β − 2 = 0 ⇒ β − 2 = 6β
2 2
…(ii) i.e. αβ = 6
a10 − 2a 8 ( α 10
− β ) − 2 (α − β )
10 8 8
⇒ Correct quadratic equation is
∴ =
2a 9 2 (α 9 − β9 ) x 2 − (α + β ) x + αβ = 0
α 8( α 2 − 2 ) − β 8 ( β 2 − 2 ) ⇒ x 2 − 7x + 6 = 0
=
2 (α 9 − β9 ) ⇒ ( x − 6 )( x − 1 ) = 0 ⇒ x = 6,1
α ⋅ 6α − β ⋅ 6β
8 8 Hence, correct roots are 1 and 6.
= [from Eqs. (i) and (ii)]
2 (α 9 − β9 )
6 (α 9 − β9 )
136. Let a + 1 = h 6
= =3
2 (α 9 − β9 ) ∴ (h 2 − 1 ) x 2 + (h 3 − 1 ) x + (h − 1 ) = 0
Chap 02 Theory of Equations 205

 h 2 − 1 2  h 3 − 1 −
q
⇒  x +   x+1=0
 h −1  h −1 ⇒
p
=4
r
As a → 0, then h → 1
p
 h 2 − 1 2  h 3 − 1
lim   x + lim   x+1=0 ⇒ q = − 4r ... (i)
h → 1 h − 1  h → 1 h − 1 
Also, given p, q, r are in AP.
⇒ 2x 2 + 3x + 1 = 0 ∴ 2q = p + r
⇒ 2x 2 + 2x + x + 1 = 0 ⇒ p = − 9r [from Eq. (i)] …(ii)
⇒ (2 x + 1 ) ( x + 1 ) = 0 D Q for ax 2 + bx + c = 0, α − β = D 
Now, | α − β | =
∴ x = − 1 and x = −
1 | a|  a 
2
137. Let e sin x = t …(i) (q 2 − 4 pr )
=
Then, the given equation can be written as | p|
1 (16r 2 + 36r 2
t − − 4 = 0 ⇒ t 2 − 4t − 1 = 0 = =
52 | r |
[from Eqs. (i) and (ii)]
t 9 |r | 9 |r |
4 ± (16 + 4 )
∴ t= 2 13
2 =
9
⇒ e sin x = (2 + 5 ) [Qe sin x > 0,∴taking + ve sign]
141. f ( x ) = x 5 − 5x and g( x ) = − a
⇒ sin x = loge ( 2 + 5 ) …(ii)
∴ f ′( x ) = 5 x 4 − 5
Q (2 + 5 ) > e [Qe = 2.71828… ]
⇒ loge ( 2 + 5 ) > 1 …(iii) 4
From Eqs. (ii) and (iii), we get
sin x > 1 [which is impossible]
Hence, no real root exists. –1 1
138. Given equations are
–4
ax 2 + bx + c = 0 …(i)
and x 2 + 2x + 3 = 0 …(ii) = 5 (x 2 + 1) (x − 1) (x + 1)
Clearly, roots of Eq. (ii) are imaginary, since Eqs. (i) and (ii) Clearly, f ( x ) = g( x ) has one real root, if a > 4 and three real
have a common root, therefore common root must be roots, if | a| < 4.
imaginary and hence both roots will be common. Therefore,
Eqs. (i) and (ii) are identical. 142. Since, b = 0 for p ( x ) = ax 2 + bx + c, as roots are pure
a b c imaginary.
∴ = = or a : b : c = 1 : 2 : 3
1 2 3 (− c ± i c )
⇒x = ± , which are clearly neither pure real nor
139. Q x − [ x ] = { x } [fractional part of x ] a
pure imaginary, as c ≠ 0.
For no integral solution, { x } ≠ 0
∴ a≠ 0 …(i) 143. Qαx 2 − x + α = 0 has distinct real roots.
The given equation can be written as ∴ D>0
3 { x } 2 − 2{ x } − a 2 = 0  1 1
⇒ 1 − 4α 2 > 0 ⇒ α ∈  − ,  ...(i)
 2 2
2 ± ( 4 + 12a ) 1 + (1 + 3a )
2 2
⇒ {x } = = [Q 0 < { x } < 1 ] Also, | x1 − x 2 | < 1 ⇒ | x1 − x 2 | 2 < 1
6 3
D 1 − 4α 2 1
1 + (1 + 3a 2 ) ⇒ <1 ⇒ < 1 ⇒α 2 >
⇒ 0< < 1 ⇒ (1 + 3a 2 ) < 2 a 2
α2 5
3
⇒ a2 < 1 ⇒ − 1 < a < 1  1   1 
…(ii) ⇒ α ∈  −∞,−  ∪  , ∞ ...(ii)
 5  5 
From Eqs. (i) and (ii), we get
a ∈ ( − 1, 0 ) ∪ ( 0, 1 ) From Eqs. (i) and (ii), we get
1 1 α+β  1 1   1 1
140. Q + = 4 ⇒ =4 S =  − ,−  ∪  , 
α β αβ  2 5   5 2
206 Textbook of Algebra

2
+ 4 x − 60 Q α 1, β1 are roots of x 2 − 2 x sec θ + 1 = 0 and α 1 > β1
144. ( x 2 − 5x + 5) x =1
Case I ∴ α 1 = sec θ − tan θ and β1 = sec θ + tan θ
x 2 − 5 x + 5 = 1 and x 2 + 4 x − 60 can be any real number ⇒ α 2, β 2 are roots of x 2 + 2 x tan θ − 1 = 0
⇒ x = 1, 4 and α 2 > β2
Case II
∴ α 2 = − tan θ + sec θ
x 2 − 5 x + 5 = − 1 and x 2 + 4 x − 60 has to be an even number
and β 2 = − tan θ − sec θ
⇒ x = 2, 3
For x = 3, x 2 + 4 x − 60 is odd, ∴ x ≠ 3 Hence, α 1 + β 2 = − 2 tan θ

Hence, x =2 146. Q x ( x + 1) + ( x + 1) ( x + 2) + .... + ( x + n − 1) ( x + n ) = 10n


Case III x 2 − 5 x + 5 can be any real number and
⇒ nx 2 + x (1 + 3 + 5 + K + (2n − 1 )) + (1 . 2 + 2 . 3
x 2 + 4 x − 60 = 0
+ ...+ (n − 1 ) . n ) = 10n
⇒ x = − 10, 6 1
⇒ Sum of all values of x = 1 + 4 + 2 − 10 + 6 = 3 or nx + n x + (n − 1 ) n (n + 1 ) = 10n
2 2
3
145. Q x 2 − 2x sec θ a + 1 = 0 ⇒ x = sec θ ± tan θ
or 3 x 2 + 3nx + (n 2 − 1 ) = 30 (Q n ≠ 0)
π π
and − <θ < − or 3 x + 3nx + (n − 31 ) = 0
2 2
6 12
 π  π |α −β| =1
⇒ sec  −  > sec θ > sec  −  Q
 6  12
or (α − β ) 2 = 1
 π  π
or sec   > sec θ > sec   D
 6  12 or =1
a2
 π  π
and tan  −  < tan θ < tan  −  or D = a2
 6  12
 π  π or 9n 2 − 12 . (n 2 − 31 ) = 9
⇒ − tan   < tan θ < − tan  
 6  12
or n 2 = 121
 π  π
or tan   > − tan θ > tan   ∴ n = 11
 6  12
CHAPTER

03
Sequences
and Series
Learning Part
Session 1
● Sequence

● Series

● Progression

Session 2
● Arithmetic Progression

Session 3
● Geometric Sequence or Geometric Progression

Session 4
● Harmonic Sequence or Harmonic Progression

Session 5
● Mean

Session 6
● Arithmetico-Geometric Series (AGS)

● Sigma ( S) Notation

● Natural Numbers

Session 7
● Application to Problems of Maxima and Minima

Practice Part
● JEE Type Examples
● Chapter Exercises

Arihant on Your Mobile !


Exercises with the #L
symbol can be practised on your mobile. See inside cover page to activate for free.
208 Textbook of Algebra

The word “Sequence” in Mathematics has same meaning as in ordinary English. A collection of objects listed in a
sequence means it has identified first member, second member, third member and so on. The most common examples
are depreciate values of certain commodity like car, machinery and amount deposits in the bank for a number of years.

Session 1
Sequence, Series, Progression
Sequence Recursive Formula
A succession of numbers arranged in a definite order or A formula to determine the other terms of the sequence in
arrangement according to some well-defined law is called terms of its preceding terms is known as recursive
a sequence. formula.
Or For example,
A sequence is a function of natural numbers (N) with If T1 = 1 and Tn + 1 = 6 Tn , n Î N .
codomain is the set of real numbers (R) [complex numbers Then, T2 = 6 T1 = 6 × 1 = 6
(C )]. If range is subset of real numbers (complex numbers), T 3 = 6 T2 = 6 × 6 = 36
it is called a real sequence (complex sequence). T 4 = 6 T 3 = 6 × 36 = 216 ...
Or Then, sequence is 1, 6, 36, 216,...
A mapping f : N ® C, then f (n ) = t n , n Î N is called a
sequence to be denoted it by
{ f (1), f (2 ), f (3 ), ... } = {t 1 , t 2 , t 3 , ... } = {t n }. Types of Sequences
The nth term of a sequence is denoted by There are two types of sequences
Tn , t n , a n , a(n ), u n , etc.
1. Finite Sequence
Remark A sequence is said to be finite sequence, if it has finite
The sequence a1, a2, a3, ... is generally written as {an}. number of terms. A finite sequence is described by
For example , a 1 , a 2 , a 3 , ... , a n or T1 , T2 , T 3 , ..., Tn , where n Î N.
(i) 1, 3, 5, 7, ... is a sequence, because each term (except For example
first) is obtained by adding 2 to the previous term and (i) 3, 5, 7, 9, …, 37
Tn = 2n - 1, n Î N . (ii) 2, 6, 18, 54, …, 4374
Or
If T1 = 1, Tn + 1 = Tn + 2 , n ³ 1 2. Infinite Sequence
(ii) 1, 2, 3, 5, 8, 13, ... is a sequence, because each term A sequence is said to be an infinite sequence, if it has
(except first two) is obtained by taking the sum of infinite number of terms. An infinite sequence is described
preceding two terms. by a 1 , a 2 , a 3 , ... or T1 , T2 , T 3 ,K
Or For example,
If T1 = 1, T2 = 2, Tn + 2 = Tn + Tn + 1 , n ³ 1 1 1 1
(i) 1, , , , …
(iii) 2, 3, 5, 7, 11, 13, 17, 19, ... is a sequence. 3 9 27
Here, we cannot express Tn , n Î N by an algebraic 1 1 1 1
(ii) 1, , , , , ...
formula. 2 4 8 16
Chap 03 Sequences and Series 209

Series Or
A sequence is said to be progression, if its terms increases
In a sequence, the sum of the directed terms is called a (respectively decreases) numerically.
series.
For example, The following sequences are progression :
For example, If 1, 4, 7, 10, 13, 16,... is a sequence, then its 1 1 1 1
sum i.e., 1 + 4 + 7 + 10 + 13 + 16 +K is a series. (i) 1, 3, 5, 7, ... (ii) , , , , ...
2 6 18 54
In general, if T1 , T2 , T 3 , ..., Tn ,... denote a sequence, then the
1 1 1
symbolic expression T1 + T2 + T 3 + ... + Tn + ... is called a (iii) 1, - , , - , K (iv) 1, 8, 27, 256, ...
series associated with the given sequence. 3 9 27
1
Each member of the series is called its term. (v) 8, - 4, 2, - 1, , K
2
In a seriesT1 + T2 + T 3 + ... + Tr + ..., the sum of first n terms
is denoted by S n . Thus, The sequences (iii) and (v) are progressions, because
n
1 1 1
S n = T1 + T2 + T 3 + ... + Tn = å Tr = å Tn | 1| > - > > - > ...
r =1 3 9 27
If S n denotes the sum of n terms of a sequence. 1 1 1
i.e. 1> > > >K
Then, S n - S n - 1 = (T1 + T2 + T 3 + ... + Tn ) 3 9 27
- (T1 + T2 + ... + Tn - 1 ) = Tn 1
Thus, Tn = S n - S n - 1 and | 8 | > | - 4 | > | 2 | > | - 1| > > ...
2
1
Types of Series i.e. 8 > 4 >2 >1>
2
> ...
There are two types of series
Remark
1. Finite Series All the definitions and formulae are valid for complex numbers
A series having finite number of terms is called a finite in the theory of progressions but it should be assumed (if not
otherwise stated) that the terms of the progressions are real
series. numbers.
For example,
(i) 3 + 5 + 7 + 9 + ... + 21 n
y Example 1. If f : N ® R, where f (n ) = an = ,
(ii) 2 + 6 + 18 + 54 + ... + 4374 (2n + 1) 2
write the sequence in ordered pair form.
2. Infinite Series n
Sol. Here, an =
A series having an infinite number of terms is called an ( 2n + 1) 2
infinite series. On putting n = 1, 2, 3, 4, ... successively, we get
For example, 1 1 2 2
a1 = = , a2 = =
1 1 1 ( 2 × 1 + 1) 2 9 (2 × 2 + 1)2 25
(i) 1 + + + + ...
3 9 27 3 3 4 4
a3 = = , a4 = =
1 1 1 ( 2 × 3 + 1) 2 49 (2 × 4 + 1)2 81
(ii) 1 + + + + ...
2 4 8 M M M
1 2 3 4
Hence, we obtain the sequence , , , , ...
9 25 49 81
Progression Now, the sequence in ordered pair form is
If the terms of a sequence can be described by an explicit ìæ 1 ö æ 2 ö æ 3 ö æ 4 ö ü
í ç1, ÷, ç2, ÷, ç3, ÷, ç 4, ÷, ...ý
formula, then the sequence is called a progression. î è 9 ø è 25 ø è 49 ø è 81 ø þ
210 Textbook of Algebra

y Example 2. The Fibonacci sequence is defined by y Example 4.


an + 1 n
a1 = 1 = a 2 , an = an - 1 + an - 2 , n > 2. Find
an
for (i) Write å(r 2 + 2) in expanded form.
n = 1, 2, 3, 4, 5. r =1
1 2 3 4 n
Sol. Q a1 = 1 = a 2 (ii) Write the series + + + +K + in sigma
3 4 5 6 n +2
\ a3 = a 2 + a1 = 1 + 1 = 2,
a4 = a3 + a2 = 2 + 1 = 3 form.
a5 = a4 + a3 = 3 + 2 = 5 Sol. (i) On putting r = 1, 2, 3, 4, K, n in (r 2 + 2),
and a6 = a5 + a4 = 5 + 3 = 8
we get 3, 6, 11, 18, ... , (n 2 + 2)
a2 a 2 a 3 a 5 a 8
\ = 1, 3 = = 2 , 4 = , 5 = and 6 = n
a1 a2 1 a3 2 a4 3 a5 5 Hence, å(r 2 + 2) = 3 + 6 + 11 + 18 + K + (n 2 + 2)
r =1
y Example 3. If the sum of n terms of a series is
r
2n 2 + 5n for all values of n, find its 7th term. (ii) The r th term of series = .
r +2
Sol. Given, Sn = 2n 2 + 5n Hence, the given series can be written as
Þ Sn - 1 = 2 (n - 1)2 + 5 (n - 1) = 2n 2 + n - 3
1 2 3 4 n n
æ r ö
\ Tn = Sn - Sn - 1 = (2n 2 + 5n ) - (2n 2 + n - 3) = 4n + 3 + + + + ... + = å ç ÷
3 4 5 6 n + 2 r = 1 èr + 2ø
Hence, T 7 = 4 ´ 7 + 3 = 31

#L Exercise for Session 1


1 First term of a sequence is 1 and the (n + 1) th term is obtained by adding (n + 1) to the nth term for all natural
numbers n, the 6th term of the sequence is
(a) 7 (b) 13
(c) 21 (d) 27

2. The first three terms of a sequence are 3, 3, 6 and each term after the second is the sum of two terms
preceding it, the 8th term of the sequence is
(a) 15 (b) 24
(c) 39 (d) 63
6
æ np ö
3. If a n = sin ç ÷ , the value of å a n2 is
è 6 ø n =1

(a) 2 (b) 3
(c) 4 (d) 7

4. If for a sequence {a n}, Sn = 2n 2 + 9n, where Sn is the sum of n terms, the value of a 20 is
(a) 65 (b) 75
(c) 87 (d) 97
5
5. If a1 = 2, a 2 = 3 + a1 and a n = 2 a n - 1 + 5 for n > 1, the value of å a r is
r =2

(a) 130 (b) 160


(c) 190 (d) 220
Session 2
Arithmetic Progression (AP)

Types of Progression Algorithm to determine whether a sequence is


Progressions are various types but in this chapter we will an AP or not
studying only three special types of progressions which Step I Obtain t n (the nth term of the sequence).
are following : Step II Replace n by n - 1 in t n to get t n - 1 .
1. Arithmetic Progression (AP) Step III Calculate t n - t n - 1 .
2. Geometric Progression (GP) If t n - t n - 1 is independent of n, the given sequence is an
3. Harmonic Progression (HP) AP otherwise it is not an AP.

y Example 5.
Arithmetic Progression (AP) (i) 1 , 3, 5, 7, ... (ii) p , p + e p , p + 2e p , K
(iii) a , a - b , a - 2b , a - 3b , K
An arithmetic progression is a sequence in which the
Sol. (i) Here, 2nd term – 1st term = 3rd term – 2nd term = ...
difference between any term and its just preceding term
Þ 3 - 1 = 5 - 3 = ... = 2, which is a common
(i.e., term before it) is constant throughout. This constant
difference.
is called the common difference (abbreviated as CD) and is
(ii) Here, 2nd term – 1st term = 3rd term – 2nd term = ...
generally denoted by ‘d’.
Or Þ ( p + e p ) - p = ( p + 2e p ) - ( p + e p ) = ...
An arithmetic progression is a sequence whose terms = e p, which is a common difference.
increase or decrease by a fixed number. This fixed number (iii) Here, 2nd term – 1st term = 3rd term – 2nd term = ...
is called the common difference of the AP. Þ (a - b ) - a = (a - 2b ) - (a - b ) = ...
A finite or infinite sequence {t 1 , t 2 , t 3 , K, t n } = - b, which is a common difference.
or {t 1 , t 2 , t 3 , ... } is said to be an arithmetic progression y Example 6. Show that the sequence < t n > defined by
(AP), if t k - t k - 1 = d , a constant independent of k, for
t n = 5n + 4 is an AP, also find its common difference.
k = 2, 3, 4, ..., n or k = 2, 3, 4,K as the case may be :
Sol. We have, t n = 5n + 4
The constant d is called the common difference of the AP.
On replacing n by (n - 1), we get
i.e. d = t 2 - t 1 = t 3 - t 2 = ... = t n - t n - 1 t n - 1 = 5 ( n - 1) + 4
Remarks Þ t n - 1 = 5n - 1
1. If a be the first term and d be the common difference, then \ t n - t n - 1 = (5n + 4 ) - (5n - 1) = 5
AP can be written as Clearly, t n - t n - 1 is independent of n and is equal to 5. So,
a, a + d, a + 2d,... , a + (n - 1) d,... ," n Î N. the given sequence is an AP with common difference 5.
2. If we add the common difference to any term of AP, we get
the next following term and if we subtract it from any term,
y Example 7. Show that the sequence < t n > defined
we get the preceding term. by t n = 3n 2 + 2 is not an AP.
3. The common difference of an AP may be positive, zero,
Sol. We have, t n = 3n 2 + 2
negative or imaginary.
4. Constant AP common difference of an AP is equal to zero. On replacing n by (n - 1), we get
5. Increasing AP common difference of an AP is greater than t n - 1 = 3 ( n - 1) 2 + 2
zero. Þ t n - 1 = 3n 2 - 6n + 5
6. Decreasing AP common difference of an AP is less than \ t n - t n - 1 = (3n 2 + 2) - (3n 2 - 6n + 5)
zero. = 6n - 3
7. Imaginary AP common difference of an AP is imaginary. Clearly, t n - t n - 1 is not independent of n and therefore it is
not constant. So, the given sequence is not an AP.
212 Textbook of Algebra

Remark y Example 8. Find first negative term of the sequence


If the nth term of a sequence is an expression of first degree in n. 1 1 3
For example, tn = An + B, where A, B are constants, then that 20, 19 , 18 , 17 , ...
4 2 4
sequence will be in AP for tn - tn - 1 = ( An + B) - [ A ( n - 1) + B]
= A [ n - ( n - 1)] = A = constant = Common difference or Sol. The given sequence is an AP in which first term, a = 20
coefficient of n in tn Students are advised to consider the above 3
point as a behaviour of standard result. and common difference, d = - . Let the nth term of the
4
given AP be the first negative term. Then,
General Term of an AP t n < 0 Þ a + ( n - 1) d < 0
æ 3ö
Let ‘a’ be the first term, ‘d’ be the common difference and ‘ Þ 20 + (n - 1) ç - ÷ < 0
è 4ø
l ’ be the last term of an AP having ‘n’ terms, where n Î N .
Þ 80 - 3n + 3 < 0
Then, AP can be written as a, a + d , a + 2d , ..., l - 2d , l - d , l 83 2
Þ n> or n > 27
(i) nth Term of an AP from Beginning 3 3
Þ n = 28
1st term from beginning = t 1 = a = a + (1 - 1) d Thus, 28th term of the given sequence is the first negative
2nd term from beginning = t 2 = a + d = a + (2 - 1) d term.
3rd term from beginning = t 3 = a + 2d = a + (3 - 1) d 1
M M M M
y Example 9. If the mth term of an AP is and the
n
n th term from beginning = t n = a + (n - 1) d , " n Î N 1
nth term is , then find mnth term of an AP.
Hence, n th term of an AP from beginning m
= t n = a + (n - 1) d = l [last term] Sol. If A and B are constants, then r th term of AP is
t r = Ar + B
(ii) nth Term of an AP from End 1 1
Given, tm = Þ Am + B = …(i)
1st term from end = t ¢1 = l = l - (1 - 1) d n n
1 1
2nd term from end = t ¢2 = l - d = l - (2 - 1) d and tn = Þ An + B = …(ii)
m m
3rd term from end = t ¢ 3 = l - 2d = l - (3 - 1) d 1
From Eqs. (i) and (ii), we get A = and B = 0
M M M M mn
¢
nth term from end = t n = l - (n - 1) d , " n Î N 1
mn th term = t mn = Amn + B = × mn + 0 = 1
Hence, n th term of an AP from end mn
t ¢n = l - (n - 1) d = a [first term] Hence, mn th term of the given AP is 1.
Now, it is clear that y Example 10. If | x - 1 |, 3 and | x - 3 | are first three
t n + t ¢n = a + (n - 1) d + l - (n - 1) d = a + l terms of an increasing AP, then find the 6th term of on AP.
or t n + t ¢n = a + l Sol. Case I For x < 1,
i.e. In a finite AP, the sum of the terms equidistant from | x - 1 | = - ( x - 1)
the beginning and end is always same and is equal to the and | x - 3 | = - ( x - 3)
sum of first and last term. \1 - x, 3 and 3 - x are in AP.
Þ 6=1- x +3- x
Remark
Þ x = -1
1. nth term is also called the general term.
Then, first three terms are 2, 3, 4,
2. If last term of AP is tn and common difference be d, then
terms of AP from end are tn, tn - d, tn - 2d, ... which is an increasing AP.
3. If in a sequence, the terms an alternatively positive and \ 6th term is 7. [Qd = 1]
negative, then it cannot be an AP. Case II For 1 < x < 3,
l-a | x - 1| = x -1
4. Common difference of AP = , where, a = first term of AP,
n+ 1
and | x - 3 | = - ( x - 3) = 3 - x
l = last term of AP and n = number of terms of AP.
5. If tn, tn + 1, tn + 2 are three consecutive terms of an AP, then
\x - 1, 3 and 3 - x are in AP.
2 tn + 1 = tn + tn + 2 . In particular, if a, b and c are in AP, then Þ 6= x -1+3- x
2b = a + c . Þ 6=2 [impossible]
Chap 03 Sequences and Series 213

Case III For x > 3, | x - 1 | = x - 1 and | x - 3 | = x - 3 announced the answer almost at once. The teacher
\ x - 1, 3 and x - 3 are in AP. overawed at this asked Gauss to explain how he got this
Þ 6= x -1+ x -3 Þ x =5 answer. Gauss explained that he had added these numbers
Then, first three terms are 4, 3, 2, which is a decreasing AP. in pairs as follows
(1 + 100 ), (2 + 99 ), (3 + 98 ) , K
y Example 11. In the sequence 1, 2, 2, 3, 3, 3, 4, 4, 4,
4, ..., where n consecutive terms have the value n, find 100
There are = 50 pairs. The answer can be obtained by
the 150th term of the sequence. 2
Sol. Let the 150th term = n multiplying 101 by 50 to get 5050.
Then, 1 + 2 + 3 + ... + (n - 1) < 150 < 1 + 2 + 3 + ... + n
Þ
( n - 1) n
< 150 <
n ( n + 1) Sum of n Terms of an AP
2 2 Let ‘a’ be the first term, ‘d’ be the common difference, ‘l ’
Þ n (n - 1) < 300 < n (n + 1) be the last term of an AP having n terms and S n be the
Taking first two members
sum of n terms, then
n (n - 1) < 300 Þ n 2 - n - 300 < 0
2 S n = a + (a + d ) + (a + 2d ) + K + (l - 2d ) + (l - d ) + l …(i)
æ 1ö 1
Þ çn - ÷ < 300 + Reversing the right hand terms
è 2ø 4
S n = l + (l - d ) + (l - 2d ) + ... + (a + 2d ) + (a + d ) + a …(ii)
1 1201
Þ 0<n < + On adding Eqs. (i) and (ii), we get
2 2
Þ 0 < n < 17.8 …(i) 2 S n = (a + l ) + (a + l ) + (a + l ) + ...
and taking last two members, + (a + l ) + (a + l ) + (a + l )
n (n + 1) > 300 = (a + l ) + (a + l ) + ... upto n terms = n (a + l )
2
æ 1ö 1 n
Þ ç n + ÷ > 300 + \ S n = (a + l ) …(iii)
è 2ø 4
2
1 1201
\ n>- + Now, if we substitute the value of l viz., l = a + (n - 1) d , in
2 2
this formula, we get
Þ n > 16. 8 …(ii)
n n
From Eqs. (i) and (ii), we get S n = [a + a + (n - 1) d ] = [2a + (n - 1)d ]
16.8 < n < 17.8 2 2
Þ n = 17 n
\ S n = [2a + (n - 1) d ]
2
y Example 12. If a1 , a 2 , a 3 , a 4 and a 5 are in AP with
5 If we substitute the value of a viz.,
common difference ¹ 0, find the value of å ai when l = a + (n - 1) d
a 3 = 2. i =1
or a = l - (n - 1) d in Eq. (iii), then
Sol. Qa1, a 2 , a 3 , a 4 and a 5 are in AP, we have n
S n = [ 2l - (n - 1) d ]
a1 + a 5 = a 2 + a 4 = a 3 + a 3 [Qt n + t ¢n = a + l ] 2
a1 + a 5 = a 2 + a 4 = 4 [Qa 3 = 2]
If we substitute the value of a + l viz.,
a1 + a 2 + a 3 + a 4 + a 5 = 4 + 2 + 4 = 10
5 t n + t ¢ n = a + l in Eq. (iii), then
Þ åai = 10 n
i =1 S n = (t n + t ¢ n )
2

Sum of a Stated Number of Corollary I Sum of first n natural numbers


i.e. 1 + 2 + 3 + 4 + ... + n
Terms of an Arithmetic Series Here, a = 1 and d = 1
More than 200 yr ago, a class of German School Children
n
was asked to find the sum of all integers from 1 to 100 \ S = [2 × 1 + (n - 1) × 1]
inclusive. One boy in the class, an eight year old named 2
Carl Fredrick Gauss (1777-1855) who later established n (n + 1)
=
his reputation as one of the greatest Mathematicians 2
214 Textbook of Algebra

Corollary II Sum of first n odd natural numbers n


Sum of first n terms = [2a + (n - 1) d ]
i.e., 1 + 3 + 5 + ... 2
Here, a =1 dn 2 æ dö
= + ç a - ÷ n = An 2 + Bn,
and d =2 2 è 2ø
n d d
\ S = [2 × 1 + (n - 1) × 2 ] = n 2 where, A = ,B =a -
2 2 2
Corollary III If sum of first n terms is S n , then sum of next Hence, S n = An 2 + Bn, where A and B are constants
m terms is S m + n - S n .
independent of n.
Hence, the converse is true.
Important Results with Proof Corollary Q S n = An 2 + Bn
1. If S n , t n and d are sum of n terms, nth term
\ t n = A (2n - 1) + B
and common difference of an AP respectively,
then t n = A (replacing n 2 by 2n - 1) + coefficient of n
d = tn - tn -1 [n ³ 2] and d = 2A
t n = Sn - Sn - 1 [n ³ 2] i.e. d =2 [coefficient of n 2 ]
d = Sn - 2 Sn - 1 + Sn - 2 [n ³ 3]
Proof Sn tn d
Q Sn = t1 + t2 + t 3 +K + tn - 1 + tn 1. 5n2 + 3n 5 (2n - 1) + 3 = 10n - 2 10
Þ Sn = Sn - 1 + tn
2. - 7n2 + 2n - 7 (2n - 1) + 2 - 14
\ tn = Sn - Sn - 1 = - 14 n + 9
but d = tn - tn -1 - 9 (2n - 1) - 4 - 18
3. - 9n2 - 4 n
= (S n - S n - 1 ) - (S n - 1 - S n - 2 ) = - 18n + 5
\ d = Sn - 2 Sn - 1 + Sn - 2 4. 4 n2 - n 4 (2n - 1) - 1 = 8n - 5 8
2. A sequence is an AP if and only if the sum of
its n terms is of the form An 2 + Bn, where A 3. If S n = an 2 + bn + c , where S n denotes the sum
and B are constants independent of n. of n terms of a series, then whole series is not
In this case, the nth term and common difference of an AP. It is AP from the second term onwards.
the AP are A (2n - 1) + B and 2A, respectively. Proof As S n = an 2 + bn + c for n ³ 1, we get
Proof As S n = An 2 + Bn S n - 1 = a (n - 1) 2 + b (n - 1) + c for n ³ 2
\ S n - 1 = A (n - 1) 2 + B (n - 1) Now, tn = Sn - Sn - 1
\ tn = Sn - Sn - 1 Þ t n = a (2n - 1) + b, n ³ 2
2 2
= ( An + Bn ) - [ A (n - 1) + B (n - 1)] \ t n - 1 = a [2 (n - 1) - 1] + b, n ³ 3
Þ t n - 1 = a (2n - 3 ) + b, n ³ 3
= A [n 2 - (n - 1) 2 ] + B
\ t n - t n - 1 = 2a = constant, n ³ 3
t n = A (2n - 1) + B
\ t 3 - t 2 = t 4 - t 3 = t 5 - t 4 = ...
Þ t n - 1 = A [2 (n - 1) - 1] + B
But t 2 - t 1 = (S 2 - S 1 ) - S 1 = S 2 - 2 S 1
= A (2n - 3 ) + B
= ( 4a + 2b + c ) - 2 (a + b + c )
Now, t n - t n - 1 = [ A (2n - 1) + B ] - [ A (2n - 3 ) + B ]
= (2a - c ) [QS 1 = t 1 ]
= 2A [a constant]
\ t2 - t1 ¹ t 3 - t2
Hence, the sequence is an AP.
Hence, the whole series is not an AP. It is AP from
Conversely, consider an AP with first term a and
the second term onwards.
common difference d.
Chap 03 Sequences and Series 215

Ratio of Sums is Given y Example 14. The sums of n terms of two arithmetic
progressions are in the ratio (7n + 1) :(4n + 17 ). Find the
1. If ratio of the sums of m and n terms of an AP ratio of their nth terms and also common differences.
is given by
Sol. Given, Sn : Sn¢ = (7n + 1) : ( 4n + 17 )
Sm Am 2 + Bm
= Here, A = 7, B = 1, C = 4 and D = 17
Sn An 2 + Bn
tn 7 (2n - 1) + 1 14n - 6
where A, B are constants and A ¹ 0. \ = =
t ¢n 4 (2n - 1) + 17 8n + 13
2
\ S m = ( Am + Bm ) k , d A 7
and = =
2
S n = ( An + Bn ) k d¢ C 4

Þ t m = S m - S m - 1 = [ A (2m - 1) + B ]k Hence, t n : t n¢ = (14n - 6) : (8n + 13) and d : d ¢ = 7 : 4

t n = S n - S n - 1 = [ A (2n - 1) + B ] k y Example 15. The sums of n terms of two AP’s are in


t m A (2m - 1) + B the ratio ( 3n - 13) :( 5n + 21). Find the ratio of their 24th
\ = terms.
tn A (2n - 1) + B
Sol. Given, Sn : Sn¢ = (3n - 13) : (5n + 21)
y Example 13. The ratio of sums of m and n terms of Here, A = 3, B = - 13, C = 5 and D = 21
an AP is m 2 : n 2 . The ratio of the m th and nth terms is
t 24 3 (2 ´ 24 - 1) - 13 128 1
\ = = =
(a) (2m + 1) : (2n - 1) (b) m : n t ¢24 5 (2 ´ 24 - 1) + 21 256 2
(c) (2m - 1) : (2n - 1) (d) None of these \ t 24 : t ¢24 = 1 : 2
Sm m 2
Sol. (c) Here, = 2 [QA = 1, B = 0] y Example 16. How many terms of the series
Sn n
1 2
t m ( 2m - 1) 20 + 19 + 18 + ... must be taken to make 300?
\ = 3 3
tn ( 2n - 1)
Þ t m : t n = ( 2m - 1) : ( 2n - 1) Explain the double answer.
Sol. Here, given series is an AP with first term a = 20 and the
2. If ratio of the sums of n terms of two AP’s is
2
given by common difference, d = - .
3
Sn An + B Let the sum of n terms of the series be 300.
=
S ¢n Cn + D Then,
n
Sn = {2a + (n - 1) d }
2
where, A, B, C , D are constants and A, C ¹ 0
nì æ 2 öü
\ S n = n ( An + B ) k, S ¢n = n (Cn + D ) k Þ 300 = í2 ´ 20 + (n - 1) ç - ÷ý
2î è 3 øþ
Þ t n = [ A (2n - 1) + B ] k , t n¢ = [C (2n - 1) + D ] k n
Þ 300 = {60 - n + 1}
Þ d = t n - t n - 1 = 2 A , d ¢ = t n¢ - t n¢ -1 = 2C 3
t n A (2n - 1) + B d A Þ 2
n - 61n + 900 = 0
\ = and =
t n¢ C (2n - 1) + D d¢ C Þ (n - 25) (n - 36) = 0
Note If A = 0, C = 0 Þ n = 25 or n = 36
S B t B d 0 \ Sum of 25 terms = Sum of 36 terms = 300
Then, n = Þ n = and = = not defined
S n¢ D t ¢n D d¢ 0 Explanation of double answer
Here, the common difference is negative, therefore terms
Remark æ -2 ö
tn an + b go on diminishing and t 31 = 20 + (31 - 1) ç ÷ = 0 i.e., 31st
If = è 3 ø
tn¢ cn + d
term becomes zero. All terms after 31st term are negative.
where, a, b, c, d are constants and a, c ¹ 0, then
These negative terms (t 32 , t 33 , t 34 , t 35 , t 36 ) when added to
n + 1ö
a æç ÷+ b positive terms (t 26 , t 27 , t 28 , t 29 , t 30 ), they cancel out each
Sn è 2 ø
= other i.e., sum of terms from 26th to 36th terms is zero.
S ¢n n + 1ö
c æç ÷+ d Hence, the sum of 25 terms as well as that of 36 terms is
è 2 ø
300.
216 Textbook of Algebra

y Example 17. Find the arithmetic progression but given


consisting of 10 terms, if the sum of the terms t 1 + t 5 + t 10 + t 15 + t 20 + t 24 = 225
occupying the even places is equal to 15 and the sum Þ (t 1 + t 24 ) + (t 5 + t 20 ) + (t 10 + t 15 ) = 225
1 Þ 3 (t 1 + t 24 ) = 225
of those occupying the odd places is equal to 12 .
2 Þ t 1 + t 24 = 75
Sol. Let the successive terms of an AP be t 1, t 2 , t 3 , ..., t 9 , t 10 . 24
\ S 24 = (t 1 + t 24 ) = 12 ´ 75 = 900
By hypothesis, 2
t 2 + t 4 + t 6 + t 8 + t 10 = 15
y Example 20. If (1 + 3 + 5 + ... + p ) + (1 + 3 + 5 + ... + q )
5
Þ (t 2 + t 10 ) = 15 = (1 + 3 + 5 + ... + r ), where each set of parentheses
2
Þ t 2 + t 10 = 6 contains the sum of consecutive odd integers as
Þ ( a + d ) + ( a + 9d ) = 6
shown, then find the smallest possible value of
p + q + r (where, p > 6).
Þ 2a + 10d = 6 …(i)
1 Sol. We know that, 1 + 3 + 5 + ... + (2n - 1) = n 2
and t 1 + t 3 + t 5 + t 7 + t 9 = 12
2 Thus, the given equation can be written as
2 2 2
5 25 æ p + 1ö æq + 1ö ær + 1ö
Þ (t 1 + t 9 ) = ç ÷ +ç ÷ =ç ÷
2 2 è 2 ø è 2 ø è 2 ø
Þ t1 + t 9 = 5
Þ ( p + 1) 2 + ( q + 1) 2 = ( r + 1) 2
Þ a + a + 8d = 5
Therefore, ( p + 1, q + 1, r + 1) form a Pythagorean triplet as
Þ 2a + 8d = 5 …(ii)
p > 6Þp + 1 > 7
1 1
From Eqs. (i) and (ii), we get d = and a = The first Pythagorean triplet containing a number > 7 is
2 2 (6, 8, 10).
1 1 1
Hence, the AP is , 1, 1 , 2, 2 , K Þ p + 1 = 8, q + 1 = 6, r + 1 = 10
2 2 2 Þ p + q + r = 21

y Example 18. If N, the set of natural numbers is


partitioned into groups S 1 = {1}, S 2 = {2, 3}, Properties of Arithmetic Progression
1. If a 1 , a 2 , a 3 , ... are in AP with common difference d,
S 3 = {4, 5, 6}, ..., find the sum of the numbers in S 50 .
then a 1 ± k, a 2 ± k, a 3 ± k, ... are also in AP with
Sol. The number of terms in the groups are 1, 2, 3, ... common difference d.
Q The number of terms in the 50th group = 50 2. If a 1 , a 2 , a 3 , ... are in AP with common difference d,
\ The last term of 1st group = 1 a a a
The last term of 2nd group = 3 = 1 + 2 then a 1 k, a 2 k, a 3 k, ... and 1 , 2 , 3 , ... are also in AP
k k k
The last term of 3rd group = 6 = 1 + 2 + 3 d
M M M M M M (k ¹ 0 ) with common differences kd and ,
k
The last term of 49th group = 1 + 2 + 3 + ... + 49 respectively.
\ First term of 50th group = 1 + (1 + 2 + 3 + ... + 49 )
49
3. If a 1 , a 2 , a 3 , .. and b 1 , b 2 , b 3 , .. . are two AP’s with
=1+ (1 + 49 ) = 1226 common differences d 1 and d 2 , respectively. Then,
2
a 1 ± b 1 , a 2 ± b 2 , a 3 ± b 3 , ... are also in AP with
50
\ S 50 = {2 ´ 1226 + (50 - 1) ´ 1} common difference (d 1 ± d 2 ).
2
= 25 ´ 2501 = 62525 4. If a 1 , a 2 , a 3 , ... and b 1 , b 2 , b 3 , ... are two AP’s with
common differences d 1 and d 2 respectively, then
y Example 19. Find the sum of first 24 terms of on AP a a a
a 1 b 1 , a 2 b 2 , a 3 b 3 , ... and 1 , 2 , 3 ,K are not in AP.
t 1 , t 2 , t 3 , ... , if it is known that b1 b2 b 3
t 1 + t 5 + t 10 + t 15 + t 20 + t 24 = 225. 5. If a 1 , a 2 , a 3 , ..., a n are in AP, then
Sol. We know that, in an AP the sums of the terms equidistant
from the beginning and end is always same and is equal
ar - k + ar + k
ar = , " k, 0 £ k £ n - r
to the sum of first and last term. 2
Then, t 1 + t 24 = t 5 + t 20 = t 10 + t 15
Chap 03 Sequences and Series 217

6. If three numbers in AP whose sum is given are to be From Eq. (iii),


taken as a - b, a, a + b and if five numbers in AP n n
{2 × 1 + (n - 1) × 1}, {2 × 2 + (n - 1) × 3},
whose sum is given, are to be taken as a - 2b, a - b, 2 2
a, a + b, a + 2b, etc. n
{2 × 3 + (n - 1) × 5}, …,
2
In general, If (2 r + 1) numbers in AP whose sum is n
given, are to be taken as (r Î N ). {2p + (n - 1) (2p - 1)} are also in AP
2
a - r b, a - (r - 1) b, ..., a - b, a , a + b, ... , n
a + (r - 1) b, a + rb [multiplying to each term]
2
i.e. S1, S 2 , S 3 , ..., S p are in AP.
Remark
p
1. Sum of three numbers = 3a \ S1 + S 2 + S 3 + ... + S p = {S1 + S p }
Sum of five numbers = 5a
2
p ìn n ü
M M M M M = í [2 × 1 + (n - 1) × 1] + [2 × p + (n - 1) (2p - 1)]ý
Sum of ( 2r + 1) numbers = ( 2r + 1) a 2 î2 2 þ
2. From given conditions, find two equations in a and b and np
then solve them. Now, the numbers in AP can be obtained.
= {2 + (n - 1) + 2p + (n - 1) (2p - 1)}
4
7. If four numbers in AP whose sum is given, are to be np 1
= (2np + 2) = np (np + 1)
taken as 4 2
a - 3b, a - b, a + b, a + 3b and if six numbers in AP, Aliter
whose sum is given are to be taken as a - 5b, a - 3b, n ( n + 1)
Here, S1 = 1 + 2 + 3 + ... upto n terms =
a - b, a + b, a + 3b, a + 5 b, etc. 2
In general If 2 r numbers in AP whose sum is given, n
S 2 = 2 + 5 + 8 + ... upto n terms = [ 2 × 2 + (n - 1)3]
are to be taken as (r Î N ). 2
a - (2 r - 1) b, a - (2 r - 3 ) b,..., a - 3 b, a - b, n (3n + 1)
=
a + b, a + 3 b, ..., a + (2 r - 3 ) b, a + (2 r - 1) b 2
n (5n + 1)
Similarly, S 3 = 3 + 8 + 13 + ... upto n terms = , etc.
Remark 2
1. Sum of four numbers = 4 a Now, S1 + S 2 + S 3 + ... + S p
Sum of six numbers = 6 a n ( n + 1) n ( 3n + 1) n ( 5n + 1)
= + + + ... upto p terms
M M M M M 2 2 2
Sum of 2r numbers = 2ra n
= [(n + 3n + 5n + ... upto p terms)
2. From given conditions, find two conditions in a and b and 2
then solve them. Now, the numbers in AP can be obtained. + (1 + 1 + 1 + ... upto p terms)]
n ép ù
= ê (2n + ( p - 1) 2n ) + p ú
y Example 21. If S 1 , S 2 , S 3 , ..., S p are the sums of 2 ë2 û
n terms of p AP’s whose first terms are 1, 2, 3, ..., p and =
np 1
[n + n ( p - 1) + 1] = np (np + 1)
common differences are 1, 2, 3, ..., (2p - 1) respectively, 2 2
1
show that S 1 + S 2 + S 3 + .... + S p = np (np + 1). y Example 22. Let a and b be roots of the equation
2
Sol. Q1, 2, 3, ..., p are in AP. x 2 - 2x + A = 0 and let g and d be the roots of the
Then, 2×1, 2×2, 2×3, K, 2p are also in AP. …(i) equation x 2 - 18 x + B = 0. If a < b < g < d are in
[multiplying 2 to each term] arithmetic progression, then find the values of A and B.
and 1, 3, 5, ..., (2p - 1) are in AP. Sol. Qa , b, g , d are in AP.
Then, (n - 1) × 1, (n - 1) ×3, (n - 1)×5, ..., (n - 1) (2p - 1) are Let b = a + d, g = a + 2d, d = a + 3d, d > 0
also in AP. …(ii) [here, sum of a , b, g , d is not given]
[multiplying (n - 1) to each term]
Given, a + b = 2, ab = A
From Eqs. (i) and (ii), we get
2×1 + (n - 1) ×1, 2×2 + (n - 1) ×3, 2×3 + (n - 1) ×5, K, Þ 2a + d = 2, ab = A …(i)

2p + (n - 1) (2p - 1) are also in AP. …(iii) and g + d = 18, gd = B


[adding corresponding terms of Eqs. (i) and (ii)] Þ 2a + 5d = 18, gd = B …(ii)
218 Textbook of Algebra

From Eqs. (i) and (ii), we get Þ b2 - D 2 < b2


d = 4, a = - 1 Þ b (b 2 - D 2 ) < b 3 Þ 4 < b 3
\ b = 3, g = 7, d = 11
\ b > ( 4 )1/ 3 or b > (2)2 / 3
Þ A = ab = ( - 1) (3) = - 3
and B = gd = (7 )(11) = 77 Hence, the minimum value of b is (2)2 / 3 .

y Example 23. The digits of a positive integer having y Example 25. If a, b , c , d are distinct integers form an
three digits are in AP and their sum is 15. The number increasing AP such that d = a 2 + b 2 + c 2 , then find the
obtained by reversing the digits is 594 less than the
value of a + b + c + d .
original number. Find the number.
Sol. Here, sum of numbers i.e., a + b + c + d is not given.
Sol. Let the digit in the unit’s place be a - d , digit in the ten’s
place be a and the digit in the hundred’s place be a + d . Let b = a + D , c = a + 2D , d = a + 3D , " D Î N
Sum of digits = a - d + a + a + d = 15 [given] According to hypothesis,
Þ 3a = 15 a + 3D = a 2 + ( a + D ) 2 + ( a + 2D ) 2
\ a=5 …(i) Þ 5D 2 + 3 (2a - 1) D + 3a 2 - a = 0 …(i)
\ Original number = (a - d ) + 10a + 100 (a + d ) 2 2
- 3 (2a - 1) ± 9 (2a - 1) - 20 (3a - a )
= 111a + 99d = 555 + 99d \ D=
and number formed by reversing the digits 10
= (a + d ) + 10a + 100 (a - d ) - 3 (2a - 1) ± ( - 24a 2 - 16a + 9 )
=
= 111a - 99d = 555 - 99d 10
Given, (555 + 99d ) - (555 - 99d ) = 594 Þ 198d = 594 Now, - 24a 2 - 16a + 9 ³ 0
\ d =3 Þ 24a 2 + 16a - 9 £ 0
Hence, original number = 555 + 99 ´ 3 = 852 1 70 1 70
Þ - - £a£ - +
3 3 3 12
y Example 24. If three positive real numbers are in AP Þ a = - 1, 0 [ Qa Î I ]
such that abc = 4, then find the minimum value of b. 3
When a = 0 from Eq. (i), D = 0, (not possible Q D Î N ) and
Sol. Qa, b, c are in AP. 5
Let a = A - D , b = A , c = A + D for a = - 1
4
Then, a = b - D, c = b + D From Eq. (i), D = 1,
Now, abc = 4 5
\ D =1 [Q D Î N ]
(b - D ) b (b + D ) = 4 \ a = - 1, b = 0, c = 1, d = 2
Þ b (b 2 - D 2 ) = 4 Then, a + b + c + d = - 1 + 0 + 1 + 2 = 2
Chap 03 Sequences and Series 219

#L Exercise for Session 2


1. If nth term of the series 25 + 29 + 33 + 37 + ... and 3 + 4 + 6 + 9 + 13 + K are equal, then n equals
(a) 11 (b) 12 (c) 13 (d) 14
1 7 1 20
2. The rth term of the series 2 + 1 + 1 + + ... is
2 13 9 23
20 20 20
(a) (b) (c) 20 (5r + 3) (d)
5r + 3 5r - 3 5r 2 + 3

3. In a certain AP, 5 times the 5th term is equal to 8 times the 8th term, its 13th term is
(a) 0 (b) -1 (c) - 12 (d) -13

4. If the 9th term of an AP is zero, the ratio of its 29th and 19th terms is
(a) 1 : 2 (b) 2 : 1 (c) 1 : 3 (d) 3 : 1

5. If the p th, q th and rth terms of an AP are a, b and c respectively, the value of a (q - r ) + b (r - p ) + c ( p - q ) is
1
(a) 1 (b) -1 (c) 0 (d)
2
6. The 6th term of an AP is equal to 2, the value of the common difference of the AP which makes the product
a1a4a 5 least is given by
8 5 2 1
(a) (b) (c) (d)
5 4 3 3
7. The sum of first 2n terms of an AP is a and the sum of next n terms is b, its common difference is
a - 2b 2b - a a - 2b 2b - a
(a) (b) (c) (d)
3n 2 3n 2 3n 3n

8. The sum of three numbers in AP is - 3 and their product is 8, then sum of squares of the numbers is
(a) 9 (b) 10 (c) 12 (d) 21
S
9. Let Sn denote the sum of n terms of an AP, if S2n = 3Sn, then the ratio 3n is equal to
Sn
(a) 9 (b) 6 (c) 16 (d) 12

10. The sum of the products of the ten numbers ± 1, ± 2, ± 3, ± 4, ± 5 taking two at a time, is
(a) - 65 (b) 165 (c) - 55 (d) 95

11. If a1, a 2, a 3, ..., a n are in AP, where a i > 0 for all i , the value of
1 1 1
+ + ... + is
a1 + a 2 a2 + a3 an - 1 + an
1 1 n (n - 1)
(a) (b) (c) (d)
a1 + an a1 - an a1 - an a1 + an
Session 3
Geometric Sequence or Geometric Progression (GP)
Geometric Sequence or 5. The common ratio of GP may be positive, negative or

Geometric Progression (GP) imaginary.


6. If common ratio of GP is equal to unity, then GP is known as
A geometric progression is a sequence, if the ratio of any Constant GP.
term and its just preceding term is constant throughout. 7. If common ratio of GP is imaginary or real, then GP is known
This constant quantity is called the common ratio and is as Imaginary GP.
generally denoted by ‘ r’. 8. Increasing and Decreasing GP
For a GP to be increasing or decreasing r > 0. If r < 0, terms
Or of GP are alternatively positive and negative and so neither
A geometric progression (GP) is a sequence of numbers, increasing nor decreasing.
whose first term is non-zero and each of the term is a a>0 a>0 a<0 a<0
obtained by multiplying its just preceding term by a r 0 < r <1 r >1 r >1 0 < r <1
constant quantity. This constant quantity is called Result Decreasing Increasing Decreasing Increasing
common ratio of the GP.
Let t 1 , t 2 , t 3 , ..., t n ; t 1 , t 2 , t 3 , ... be respectively a finite or an y Example 26.
infinite sequence. Assume that none of t n¢ ’s is 0 and that 1 1
tk (i) 1, 2, 4, 8, 16, … (ii) 9, 3, 1, , , …
= r , a constant (i.e., independent of k). 3 9
tk -1 1
(iii) - 2 , - 6 , - 18,K (iv) - 8, - 4, - 2 , - 1, - , ...
For k = 2, 3, 4, ..., n or k = 2, 3, 4, ... as the case may be. We 2
then call {t k } nk = 1 or {t k } k¥= 1 as the case may be a (v) 5 , - 10, 20, K (vi) 5, 5, 5, 5,...
(vii) 1, 1 + i , 2i, - 2 + 2i , ...; i = -1
geometric progression (GP). The constant ratio r is called
Sol. (i) Here, a = 1
the common ratio (CR) of the GP.
2 4 8 16
t t t and r = = = = = ... = 2 i.e. a = 1 and r = 2
i.e., r = 2 = 3 = ... = n 1 2 4 8
t1 t2 tn -1
Increasing GP (a > 0, r > 1)
If t 1 = a is the first term of a GP, then (ii) Here, a = 9
1 1
t 2 = ar , t 3 = t 2 r = ar 2 , t 4 = t 3 r = ar 3 ,K ,
3 1 3 9 1 1
t n = t n - 1 r = ar n - 1 and r = = = = = ... = i.e. a = 9, r =
9 3 1 1 3 3
It follows that, given that first term a and the common ratio 3
r, the GP can be rewritten as Decreasing GP (a > 0, 0 < r < 1)
(iii) Here, a = - 2
a, ar , ar 2 , ..., ar n - 1 (standard GP) or a, ar , ar 2 ,..., ar n - 1 ,…
-6 -18
(standard GP) and r = = = ... = 3
-2 -6
according as it is finite or infinite. i.e. a = - 2, r = 3
Decreasing GP (a < 0, r > 1)
Important Results
(iv) Here, a = - 8
1. In a GP, neither a = 0 nor r = 0.
1
2. In a GP, no term can be equal to ‘0’. -
-4 -2 -1 1
3. If in a GP, the terms are alternatively positive and negative, and r = = = = 2 = .... =
then its common ratio is always negative. -8 -4 -2 -1 2
1
4. If we multiply the common ratio with any term of GP, we get i.e. a = - 8, r =
the next following term and if we divide any term by the 2
common ratio, we get the preceding term. Increasing GP (a < 0, 0 < r < 1)
Chap 03 Sequences and Series 221

(v) Here, a = 5
-10 20
General Term of a GP
and r = = = ... = - 2 i.e., a = 5, r = - 2
5 -10 Let ‘a’ be the first term, ‘r’ be the common ratio and ‘l ’ be
Neither increasing nor decreasing (r < 0) the last term of a GP having ‘n’ terms. Then, GP can be
(vi) Here, a = 5 l l
written as a, ar , ar 2 , ..., , , l
5 5 5
and r = = = = ... = 1 i.e., a = 5, r = 1
r2 r
5 5 5
Constant GP (r = 1) (i) nth Term of a GP from Beginning
(vii) Here, a = 1 1st term from beginning = t 1 = a = ar 1 - 1
1+i 2i - 2 + 2i
and r = = = = ... 2nd term from beginning = t 2 = ar = ar 2 - 1
1 1+i 2i
3rd term from beginning = t 3 = ar 2 = ar 3 - 1
2i (1 - i ) (- 1 + i) i
= (1 + i ) = = = ...
(1 + i ) (1 - i ) i2 M M M M M
n -1
= ( 1 + i ) = ( i + 1) = ( 1 + i ) = K nth term from beginning = t n = ar ,"nÎN
i.e., a = 1, r = 1 + i Hence, n th term of a GP from beginning
Imaginary GP (r = imaginary) t n = ar n -1 = l [last term]

y Example 27. Show that the sequence < t n > defined (ii) n th Term of a GP from End
2 2n - 1 l
by t n = for all values of n Î N is a GP. Also, find 1st term from end = t ¢ 1 = l =
3 1 -1
r
its common ratio. l l
22n - 1 2nd term from end = t ¢ 2 = =
Sol. We have, t n = r r -1 2
3
l l
On replacing n by n - 1, we get 3rd term from end = t ¢ 3 = =
2 3 -1
22n - 1 r r
2n - 3
2 tn
= 2n3- 3 = 22 = 4
tn - 1 = Þ
M M M M
3 tn - 1 2 l
nth term from end = t ¢ n = ,"n ÎN
3 n -1
r
tn
Clearly, is independent of n and is equal to 4. So, the Hence, nth term of a GP from end
tn - 1
l
given sequence is a GP with common ratio 4. = t¢n = =a [first term]
n -1
r
y Example 28. Show that the sequence < t n > l
Now, it is clear that t k ´ t ¢k = ar k - 1 ´ =a ´l
defined by t n = 2 × 3n + 1 is not a GP. r k -1

Sol. We have, t n = 2 × 3n + 1 or t k ´ t ¢k = a ´ l, " 1 £ k £ n


On replacing n by (n - 1) in t n , we get i.e. in a finite GP of n terms, the product of the k th
t n - 1 = 2 × 3n - 1 + 1 term from the beginning and the k th term form the
end is independent of k and equals the product of the
(2 × 3n + 3) first and last terms.
Þ tn - 1 =
3
tn ( 2 × 3n + 1) 3 ( 2 × 3n + 1) Remark
\ = n
=
tn - 1 ( 2 × 3 + 3) ( 2 × 3n + 3) 1. nth term is also called the general term.
3 2. If last term of GP be tn and CR is r, then terms of GP from
tn t t
end are tn, n , n2 , K
Clearly, is not independent of n and is therefore not r r
tn - 1
3. If in a GP, the terms are alternatively positive and negative,
constant. So, the given sequence is not a GP.
then its common ratio is always negative.
222 Textbook of Algebra

4. If a and l represent first and last term of a GP respectively, 1


1
We get, sin q = - 1,
2
then common ratio of GP = r = æç ÷ ön - 1
l
è aø
1
\ sinq = [sinq = - 1 is not possible]
2
5. If tn , tn + 1, tn + 2 are three consecutive terms of a GP, then
1
tn + 1 tn + 2
= Þ tn2 + 1 = tn tn + 2 then first term = a = sinq = and common ratio
tn tn + 1 2
æ1 ö
b c
In particular, if a, b, c are in GP, then = Þ b2 = ac
2 ç + 1÷
è2 ø
a b =r = =3 2
b2 c 2 æ ö
1
On squaring, 2 = 2 ç ÷
a b è2ø
Hence, a2, b2, c 2 are also in GP. 1
\ t 5 = ar = (3 2 )4 = 162
4
2
y Example 29. If first term of a GP is a, third term is b Hence, (c) is the correct answer.
and (n + 1)th term is c . The (2n + 1)th term of a GP is
b bc c2 y Example 32. The 1025th term in the sequence 1, 22,
(a) c (b) (c) abc (d) 4444, 88888888, ... is
a a a
(a) 2 9 (b) 2 10
Sol. Let common ratio = r
b (c) 2 11 (d) 2 12
\ b = ar 2 Þ r = Sol. The number of digits in each term of the sequence are 1,
a
c 2, 4, 8, .... which are in GP. Let 1025th term is 2n .
Also, c = ar n n
Þ r = Then,
a
2 1 + 2 + 4 + 8 + ... + 2n - 1 < 1025 £ 1 + 2 + 4 + 8 + ... + 2n
æc ö c2
\ t 2 n + 1 = ar 2n = a (r n )2 = a ç ÷ = (2 - 1) (1 + 2 + 22 + 23 + K + 2n - 1 )
èa ø a Þ < 1025
( 2 - 1)
Hence, (d) is the correct answer.
(2 - 1) (1 + 2 + 22 + 23 + ... + 2n )
y Example 30. The (m + n )th and (m - n )th terms of a £
( 2 - 1)
GP are p and q, respectively. Then, the m th term of Þ 2n - 1 < 1025 £ 2n + 1 - 1 Þ 2n < 1026 £ 2n + 1 …(i)
the GP is n +1
m or 2 ³ 1026 > 1024
æ q ö 2n Þ 2 n +1
> 210 Þ n + 1 > 10
(a) p ç ÷ (b) pq
èpø \ n > 9 \ n = 10
p [which is always satisfy Eq. (i)]
(c) (d) None of these Hence, (b) is the correct answer.
q
Sol. Let a be the first term and r be the common ratio, then y Example 33. If a, b , c are real numbers such that
t m + n = p Þ ar m + n - 1 = p …(i) 3 (a 2 + b 2 + c 2 + 1) = 2 (a + b + c + ab + bc + ca ), then
t m - n = q Þ ar m - n - 1 = q …(ii) a, b , c are in
From Eqs. (i) and (ii), we get (a) AP only (b) GP only
ar m + n - 1 ´ ar m - n - 1 = p ´ q (c) GP and AP (d) None of these
Þ a 2 r 2 m - 2 = pq Þ ar m - 1 = pq Sol. Given, 3 (a 2 + b 2 + c 2 + 1) = 2 (a + b + c + ab + bc + ca )
Þ t m = pq Þ 2 (a 2 + b 2 + c 2 - ab - bc - ca ) +
Hence, (b) is the correct answer. (a 2 + b 2 + c 2 - 2a - 2b - 2c + 3)
Þ {(a - b )2 + (b - c )2 + (c - a )2 } +
y Example 31. If sin q , 2 (sin q + 1), 6 sin q + 6 are in
{(a - 1)2 + (b - 1)2 + (c - 1)2 } = 0
GP, then the fifth term is
Þ a - b = b - c = c - a = 0 and a - 1 = b - 1 = c - 1 = 0
(a) 81 (b) 81 2 (c) 162 (d) 162 2
Þ a=b=c =1
Sol. [ 2 (sin q + 1)]2 = sin q (6 sin q + 6) Þ a, b, c are in GP and AP.
Þ [(sin q + 1) 2 (sin q + 1) - 6sin q ] = 0 Hence, (c) is the correct answer.
Chap 03 Sequences and Series 223

If r = 1, the above formulae cannot be used. But, then the


Sum of a Stated Number of GP reduces to a, a, a, ...
Terms of a Geometric Series \ S n = a + a + a +K n times = na
The game of chess was invented by Grand Vizier Sissa Sum to Infinity of a GP, when the Numerical Value
Ben Dhair for the Indian king Shirham. Pleased with the of the Common Ratio is Less than Unity, i.e. It is a
game, the king asked the Vizier what he would like as Proper Fraction
reward. The Vizier asked for one grain of wheat to be
If a be the first term, r be the common ratio of a GP, then
placed on the first square of the chess, two grains on the
second, four grains on the third and so on (each time a (1 - r n ) a ar n
Sn = = -
doubling the number of grains). The king was surprised of (1 - r ) (1 - r ) (1 - r )
the request and told the vizier that he was fool to ask for
so little. Let - 1 < r < 1 i.e. | r | < 1, then lim r n ® 0
n®¥
The inventor of chess was no fool. He told the king ‘‘What Let S ¥ denote the sum to infinity of the GP, then
I have asked for is more wheat that you have in the entire a
S¥ = ,
kingdom, in fact it is more than there is in the whole (1 - r )
world’’ He was right. There are 64 squares on a chess where - 1 < r < 1
board and on the nth square he was asking for 2 n - 1 grains,
if you add the numbers
Recurring Decimal
i.e., S = 1 + 2 + 2 2 + 2 3 + .... + 2 62 + 2 63 …(i)
Recurring decimal is a very good example of an infinite
On multiplying both sides by 2, then geometric series and its value can be obtained by means of
2S = 2 + 2 2 + 2 3 + 2 4 + .... + 2 63 + 2 64 …(ii) infinite geometric series as follows
··
0 . 3 2 7 = 0.327272727... to infinity
On subtracting Eq. (i) from Eq. (ii), we get = 0.3 + 0.027 + 0.00027 + 0.0000027 + K upto infinity
64
S = 2 - 1 = 18, 446, 744, 073, 709, 551, 615 grains i.e., 3 27 27 27
= + + + + ... upto infinity
represent more wheat that has been produced on the Earth. 10 10 3
10 5
10 7
3 27 æ 1 1 ö
= + ç 1 + 2 + 4 + K upto infinity ÷
Sum of n Terms of a GP 10 10 è 10
3
10 ø
Let a be the first term, r be the common ratio, l be the last æ ö
term of a GP having n terms and S n the sum of n terms, ç ÷
3 27 ç 1 ÷
then = +
l l 10 10 3 ç 1 ÷
S n = a + ar + ar 2 + ... + + +l …(i) ç1 - 2 ÷
r 2 r è 10 ø
On multiplying both sides by r (the common ratio) 3 27 297 + 27
= + =
l 10 990 990
r S n = ar + ar 2 + ar 3 + K + + l + lr …(ii)
r 324
= [rational number]
On subtracting Eq. (ii) from Eq. (i), we have 990
S n - r S n = a - lr or S n (1 - r ) = a - lr Aliter (Best method)
a - lr Let P denotes the figure which do not recur and suppose
\ Sn = , when r < 1
1 -r them p in number, Q denotes the recurring period
lr - a consisting of q figures. Let R denotes the value of the
Sn = , when r > 1 recurring decimal.
r -1
Then, R = 0 × PQQQ ...
Now, l = t n = ar n - 1
\ 10 p ´ R = P × QQQ ...
Then, above formula can be written as and 10 p + q ´ R = PQ × QQQ ...
a (1 - r n ) a (r n - 1 ) PQ - P
Sn = when r < 1, S n = , \ Therefore, by subtraction R = .
(1 - r ) (r - 1 ) (10 p +q
- 10 p )
when r > 1
224 Textbook of Algebra

Corollary I If R = 0 × QQQ ... a


= {(101 - 1) + (102 - 1) + (103 - 1) + (104 - 1) + ...
Q 9
Then, R = (when Q denote the recurring period upto n terms}
q
10 - 1 a
= {(10 + 102 + 103 + K upto n terms)
consisting of q figures) 9
- (1 + 1 + 1 + ...n times)}
·
3 1 a ì 10 (10n - 1) ü a ì 10 n ü
For example, If R = 0. 3, then R = = = í - n ý = í (10 - 1) - n ý
1
10 - 1 3 9 î 10 - 1 þ 9 î 9 þ
[Remember]
Corollary II The value of recurring decimal is always
rational number. In Particular
1 ì10 ü
· · (i) For a = 1, 1 + 11 + 111 + ... = í (10 n - 1) - ný
y Example 34. Find the value of 0. 32 58. 9î9 þ
·· 2 ì10 ü
Sol. Let R = 0.3258 (ii) For a = 2, 2 + 22 + 222 + ... = í (10 n - 1) - ný
9î9 þ
Þ R = 0.3258585858 ... …(i)
Here, number of figures which are not recurring is 2 and 3 ì10 ü
(iii) For a = 3, 3 + 33 + 333 + ... = í (10 n - 1) - ný
number of figures which are recurring is also 2. 9î9 þ
Then, 100R = 32.58585858 ... …(ii) 4 ì10 n ü
and 10000R = 3258.58585858... …(iii) (iv) For a = 4, 4 + 44 + 444 + ... = í (10 - 1) - ný
9î9 þ
On subtracting Eq. (ii) from Eq. (iii), we get
5 ì10 ü
9900R = 3226 (v) For a = 5, 5 + 55 + 555 + ... = í (10 n - 1) - ný
9î9 þ
3226
\ R= 6 ì10 ü
9900 (vi) For a = 6, 6 + 66 + 666 + ... = í (10 n - 1) - ný
1613 9î9 þ
Hence, R=
4950 7 ì10 ü
(vii) For a = 7, 7 + 77 + 777 + ... = í (10 n - 1) - ný
9î9 þ
Shortcut Methods for 8 ì10 ü
(viii) For a = 8, 8 + 88 + 888 + ... = í (10 n - 1) - ný
Recurring Decimals 9î9 þ
9 ì10 ü
1. The numerator of the vulgar fraction is obtained by (ix) For a = 9, 9 + 99 + 999 + ... = í (10 n - 1) - ný
subtracting the non-recurring figure from the given 9î9 þ
figure.
y Example 36. Find the sum upto n terms of the series
2. The denominator consists of as many 9’s as there are 0.b + 0.bb + 0.bbb + 0.bbbb + ..., " b Î N and 1 £ b £ 9.
recurring figure and as many zero as there are
non-recurring figure. Sol. Let S = 0.b + 0.bb + 0.bbb + 0.bbbb + ... upto n terms

For example, = b (01


. + 011
. + 0111. + 01111
. + K upto n terms)
· · b
3654 - 36 3618 = (09 . + 099
. + 0999 . + 09999
. + .... upto n terms)
(i) 0. 36 5 4 = = 9
9900 9900 b
= {(1 - 01
. ) + (1 - 001
. ) + (1 - 0001. ) + (1 - 00001 . ) + ... upto
·· ··
327 - 3 1314 9 n terms}
(ii) 1 . 3 2 7 = 1 + 0. 3 2 7 = 1 + = b
990 990 = {(1 + 1 + 1 + 1 + ... upto n times)
9
·
3 -0 1 - (01. + 001
. + 0001
. + 00001
. + ... upto n terms)}
(iii) 0. 3 = = bì
9 3 æ1 1 1 1 öü
= ín - ç + 2 + 3 + 4 + ... upto n terms ÷ý
9î è 10 10 10 10 øþ
y Example 35. Find the sum upto n terms of the ì 1 æ 1 öü
n

series a + aa + aaa + aaaa + K, " a Î N and 1 £ a £ 9. ï ç1 - æç ö÷ ÷ ï


bï 10 çè è 10 ø ÷ø ï b ìï 1é æ 1 ö ùüï
n

Sol. Let S = a + aa + aaa + aaaa + ... upto n terms = ín - ý = ín - ê 1 - ç ÷ úý


9ï 1 è 10 ø úï
1- ï 9 îï 9 ê
ë ûþ
= a (1 + 11 + 111 + 1111 + ... upto n terms) ï 10 ï
a î þ
= (9 + 99 + 999 + 9999 + K upto n terms) [Remember]
9
Chap 03 Sequences and Series 225

In Particular 249
\ S 50 = {2 ´ 249 + ( 249 - 1) ´ 1}
(i) For b = 1, 2
1 ìï 1 é æ 1 ö ù üï
n
= 248 ( 250 + 249 - 1)
0.1 + 0.11 + 0.111 + ... = ín - ê1 - ç ÷ ú ý
9ï 9 ê è 10 ø ú ï = 248 [249 ( 2 + 1) - 1] = 248 (3 × 249 - 1)
î ë ûþ
(ii) For b = 2, 1 1 1
y Example 38. If S n = 1 + + 2 + ... + n - 1 , then
2 ìï 1 é æ 1 ö ù üï
n
2 2 2
0.2 + 0.22 + 0.222 + ... = ín - ê1 - ç ÷ ú ý
9ï 9 ê è 10 ø ú ï calculate the least value of n such that
î ë ûþ 1
2 - Sn < .
(iii) For b = 3, 100
3 ìï 1 é æ 1 ö ù üï
n
é æ1ö ù
n
0.3 + 0.33 + 0.333 + ... = ín - ê1 - ç ÷ ú ý 1 × ê1 - ç ÷ ú
9ï 9 ê è 10 ø ú ï 1 1 1 ê è2ø ú
î ë ûþ Sol. Given, Sn = 1 + + + ... + n - 1 = ë û
For b = 4, 2 22 2 æ 1ö
(iv) ç1 - ÷
è 2ø
4 ìï 1 é æ 1 ö ù üï
n
0.4 + 0.44 + 0.444 + ... = ín - ê1 - ç ÷ ú ý Þ Sn = 2 -
1
9ï 9 ê è 10 ø ú ï n -1
î ë ûþ 2
For b = 5, 1 1 é 1 ù
(v) Þ 2 - Sn = n- 1
< êëQ2 - Sn < 100 úû
5 ìï 1 é æ 1 ö ù üï
n 2 100
0.5 + 0.55 + 0.555 + ... = ín - ê1 - ç ÷ ú ý Þ 2n - 1 > 100 > 26
9ï 9 ê è 10 ø ú ï
î ë ûþ
Þ 2n - 1 > 26
(vi) For b = 6,
\ n -1>6 Þ n >7
6 ìï 1 é æ 1 ö ù üï
n
0.6 + 0.66 + 0.666 + ... = ín - ê1 - ç ÷ ú ý Hence, the least value of n is 8.
9ï 9 ê è 10 ø ú ï
î ë ûþ
y Example 39. If x = 1 + a + a 2 + a 3 + ... + ¥ and
(vii) For b = 7,
y = 1 + b + b 2 + b 3 + ... + ¥ show that
7 ìï 1 é æ 1 ö ù üï
n
0.7 + 0.77 + 0.777 + ... = ín - ê1 - ç ÷ ú ý xy
9ï 9 ê è 10 ø ú ï 1 + ab + a 2b 2 + a 3 b 3 + ... + ¥ = , where
î ë ûþ x+ y -1
(viii) For b = 8, 0 < a < 1 and 0 < b < 1.
8 ìï 1 é æ 1 ö ù üï
n
1
0.8 + 0.88 + 0.888 + ... = ín - ê1 - ç ÷ ú ý Sol. Given, x = 1 + a + a 2 + a 3 + ... + ¥ =
9ï 9 ê è 10 ø ú ï 1-a
î ë ûþ
Þ x - ax = 1
(ix) For b = 9, æ x - 1ö
9 ïì 1 é æ 1 ö ù ïü \ a=ç ÷ …(i)
n
0.9 + 0.99 + 0.999 + ... = ín - ê1 - ç ÷ ú ý è x ø
9ï 9 ê è 10 ø ú ï y = 1 + b + b 2 + b 3 + ... + ¥
î ë ûþ and
æy - 1ö
y Example 37. If N, the set of natural numbers is Similarly, b=ç ÷ …(ii)
è y ø
partitioned into groups S 1 = {1}, S 2 = {2, 3},
S 3 = {4, 5, 6, 7}, S 4 = {8, 9, 10, 11, 12, 13, 14, 15}, ..., then find Since, 0 < a < 1, 0 < b < 1
the sum of the numbers in S 50 . \ 0 < ab < 1
1
Sol. The number of terms in the groups are 1, 2, 22 , 23 , ... Now, 1 + ab + a 2b 2 + a 3b 3 + ... + ¥ =
1 - ab
\ The number of terms in the 50th group = 250 - 1 = 249 1
0 1-1 = [from Eqs. (i) and (ii)]
Q The first term of 1st group = 1 = 2 = 2 æ x - 1ö æy - 1ö
1- ç ÷ç ÷
The first term of 2nd group = 2 = 21 = 22 - 1 è x øè y ø

The first term of 3rd group = 4 = 22 = 23 - 1 xy


=
xy - xy + x + y - 1
M M M M M
xy
The first term of 50th group = 250 - 1 = 249 Hence, 1 + ab + a 2b 2 + a 3b 3 + ... + ¥ =
x +y -1
226 Textbook of Algebra

Properties of Geometric Progression Remark


1. Product of three numbers = a3
1. If a 1 , a 2 , a 3 , ... are in GP with common ratio r, then Product of five numbers = a5
a a a
a 1 k, a 2 k, a 3 k, ... and 1 , 2 , 3 , ... are also in GP M M M M M
k k k
Product of ( 2m + 1) numbers = a2 m + 1
(k ¹ 0 ) with common ratio r.
2. From given conditions, find two equations in a and r and
2. If a 1 , a 2 , a 3 , ... are in GP with common ratio r, then then solve them. Now, the numbers in GP can be obtained.
a 1 ± k, a 2 ± k, a 3 ± k, .... are not in GP (k ¹ 0 ). 7. If four numbers in GP whose product is given are to
3. If a 1 , a 2 , a 3 , ... are in GP with common ratio r, then a a
be taken as , , ar , ar 3 and if six numbers in GP
1 1 1 r3 r
(i) , , , ... are also in GP with common
a1 a2 a 3 whose product is given are to be taken as
a a a
1 , , , ar , ar 3 , ar 5 , etc.
ratio . r5 r 3 r
r
(ii) a 1n , a 2n , a n3 , ... are also in GP with common ratio In general If (2m ) numbers in GP whose product is
given are to be taken as (m Î N )
r n and n Î Q .
a a a a
, ,..., , , ar , ar 3 , ... , ar 2m - 3 , ar 2m - 1
(iii) log a 1 , log a 2 , log a 3 , ... are in AP (a i > 0, " i ) r 2m - 1
r 2m - 3
r3 r
In this case, the converse also holds good.
4. If a 1 , a 2 , a 3 , ... and b 1 , b 2 , b 3 , ... are two GP’s with Remark
common ratios r1 and r2 , respectively. Then, 1. Product of four numbers = a4
a a a Product of six numbers = a6
(i) a 1 b 1 , a 2 b 2 , a 3 b 3 , ... and 1 , 2 , 3 ,K are also in
b1 b2 b 3 M M M M M
r Product of ( 2m) numbers = a2 m
GP with common ratios r1 r2 and 1 ,
r2 2. From given conditions, find two equations in a and r and
respectively. then solve them. Now, the numbers in GP can be obtained.
(ii) a 1 ± b 1 , a 2 ± b 2 , a 3 ± b 3 , ... are not in GP.
y Example 40. If S 1 , S 2 , S 3 , ..., S p are the sum of
5. If a 1 , a 2 , a 3 , ..., a n - 2 , a n - 1 , a n are in GP. Then,
infinite geometric series whose first terms are 1, 2, 3,
(i) a 1 a n = a 2 a n - 1 = a 3 a n - 2 = K 1 1 1 1
..., p and whose common ratios are , , , ...,
(ii) a r = a r - k a r + k , " k, 0 £ k £ n - r 2 3 4 p+1
a2 a 3 a 4 a respectively, prove that
(iii) = = = ... = n p (p + 3)
a1 a2 a 3 an - 1 S 1 + S 2 + S 3 + ... + S p = .
2
Þ a 22 = a 3 a 1 , a 23 = a 2 a 4 , ... p
Sol. Q Sp = = ( p + 1)
Also, a 2 = a 1r , a 3 = a 1r , 2 1
1-
p +1
a 4 = a 1 r 3 , ..., a n = a 1 r n - 1
\ S1 = 2, S 2 = 3, S 3 = 4, ...
where, r is the common ratio of GP. \ LHS = S1 + S 2 + S 3 + ... + S p
6. If three numbers in GP whose product is given are p
= 2 + 3 + 4 + ... + ( p + 1) = ( 2 + p + 1)
a 2
to be taken as , a, ar and if five numbers in GP p ( p + 3)
r = = RHS
whose product is given are to be taken as 2
a a
, , a, ar , ar 2 , etc. y Example 41. Let x 1 and x 2 be the roots of the
2 r
r equation x 2 - 3x + A = 0 and let x 3 and x 4 be the
In general If (2m + 1) numbers in GP whose roots of the equation x 2 - 12x + B = 0. It is known
product is given are to be taken as (m Î N ) that the numbers x 1 , x 2 , x 3 , x 4 (in that order) form
a a a an increasing GP. Find A and B.
, , ..., , a, ar , ..., ar m - 1 , ar m
m m -1 r
r r
Chap 03 Sequences and Series 227

Sol. Q x 1, x 2 , x 3 , x 4 are in GP. æ1 ö æ1 + r 2 + r ö


Þ a 2 ç + r + 1÷ = 156 Þ 36 ç ÷ = 156
Let x 2 = x 1r , x 3 = x 1r 2 , x 4 = x 1r 3 èr ø è r ø
[here, product of x 1, x 2 , x 3 , x 4 are not given] æ1 + r + r 2 ö 2
Þ 3ç ÷ = 13 Þ 3r - 10r + 3 = 0
Given, x 1 + x 2 = 3, x 1x 2 = A è r ø
Þ x 1(1 + r ) = 3, x 12 r = A …(i) 1
Þ ( 3r - 1) (r - 3) = 0 Þ r = or r = 3
and x 3 + x 4 = 12, x 3 x 4 = B 3
Putting the values of a and r , the required numbers are 18,
Þ x 1r 2 (1 + r ) = 12, x 12 r 5 = B …(ii)
6, 2 or 2, 6, 18. Hence, the sum of numbers is 26.
From Eqs. (i) and (ii),
r2 = 4 Þ r =2 [for increasing GP] y Example 44. Find a three-digit number whose
From Eq. (i), x 1 = 1 consecutive digits form a GP. If we subtract 792 from
Now, A = x 12 r = 12 × 2 = 2 [from Eq. (i)] this number, we get a number consisting of the same
digits written in the reverse order. Now, if we increase
and B= x 12 r 5 2 5
= 1 × 2 = 32 [from Eq. (ii)]
the second digit of the required number by 2, then the
resulting digits will form an AP.
y Example 42. Suppose a, b , c are in AP and a 2 , b 2 , c 2
3 Sol. Let the three digits be a, ar , ar 2 , then according to hypoth-
are in GP, if a > b > c and a + b + c = , then find the
values of a and c. 2 esis
100a + 10ar + ar 2 - 792 = 100ar 2 + 10ar + a
Sol. Since, a, b, c are in AP and sum of a, b, c is given.
Let a = b - D, c = b + D [ D < 0] [Qa > b > c ] Þ 99a (1 - r 2 ) = 792
3 Þ a (1 + r ) (1 - r ) = 8 …(i)
and given a + b + c =
2 and a, ar + 2, ar 2 are in AP.
3 Then, 2 (ar + 2) = a + ar 2
Þ b- D +b+b+ D =
2
1 Þ a (r 2 - 2r + 1) = 4 Þ a (r - 1)2 = 4 …(ii)
\ b=
2 On dividing Eq. (i) by Eq. (ii), we get
1 1 r +1 1
Then, a = - D and c = + D = -2 Þ r =
2 2 r -1 3
Also, given a 2 , b 2 , c 2 are in GP, then (b 2 )2 = a 2c 2 From Eq. (ii), a = 9
1 1 Thus, digits are 9, 3, 1 and so the required number is 931.
Þ ± b 2 = ac Þ ± = - D 2
4 4
1 1 1
Þ 2
D = ± =
4 4 2
[Q D ¹ 0] Examples on Application of
\ D=±
1
Þ D=-
1
[Q D < 0] Progression in Geometrical Figures
2 2
Hence, a = +
1 1
and c = -
1 1 y Example 45. A square is drawn by joining the
2 2 2 2 mid-points of the sides of a given square. A third
square is drawn inside the second square in the
y Example 43. If the continued product of three
same way and this process continues indefinitely. If
numbers in GP is 216 and the sum of their products in
a side of the first square is 16 cm, then determine
pairs is 156, then find the sum of three numbers.
the sum of the areas of all the squares.
Sol. Here, product of numbers in GP is given.
Sol. Let a be the side length of square, then
a
\ Let the three numbers be , a, ar . G
r D C
a
Then, × a × ar = 216 K O J
r T S
Þ a 3 = 216 H P N F
\ a=6 Q R
Sum of the products in pairs = 156 L M I
a a
Þ × a + a × ar + ar × = 156 A
E B
r r
AB = BC = CD = DA = a
228 Textbook of Algebra

Q E, F , G , H are the mid-points of AB, BC , CD and DA, æ a a a ö


P = Sum of perimeters = 3 ça + + + + K÷
respectively. è 2 4 8 ø
a æ ö
\ EF = FG = GH = HE =
2 ç a ÷
=3ç ÷ = 6a = 6 ´ 24 = 144 cm [Qa = 24 cm ]
and I , J , K , L are the mid-points of EF , FG , GH and HE, 1
ç1 - ÷
respectively. è 2ø
a
\ IJ = JK = KL = LI = y Example 47. Let S 1 , S 2 , ... be squares such that for
2
a each n ³ 1, the length of a side of S n equals the
Similarly, MN = NO = OP = PM = and
2 2 length of a diagonal of S n + 1 . If the length of a side
a of S 1 is 10 cm and the area of S n less than 1 sq cm.
QR = RS = ST = TQ = , ...
4 Then, find the value of n.
S = Sum of areas Sol. We have, length of a side of
= ABCD + EFGH + IJKL + MNOP + QRST + ... Sn = length of diagonal of Sn + 1
æ a ö
2
æa ö æ a ö
2 2 Þ Length of a side of Sn = 2 (length of a side of Sn + 1)
= a2 + ç ÷ + ç ÷ + ç ÷ + ...
è 2ø è2ø è2 2 ø Length of a side of Sn + 1 1
Þ = , for all n ³ 1
æ 1 1 1 ö Length of a side of Sn 2
= a 2 ç1 + + + + ...÷
è 2 4 8 ø Þ Sides of S1, S 2 , S 3 , ... form a GP with common ratio
æ ö 1
and first term 10.
2
ç 1 ÷ 2 2 2
=a ç ÷ = 2a = 2 (16) [Qa = 16 cm]
1 n -1
ç1 - ÷ æ 1 ö 10
è 2ø \ Side of Sn = 10 ç ÷ = (n - 1)
è 2ø
= 512 sq cm 2 2
100
y Example 46. One side of an equilateral triangle is 24 Þ Area of Sn = ( Side)2 =
2n - 1
cm. The mid-points of its sides are joined to form Now, given area of Sn < 1
another triangle whose mid-points, in turn, are joined 100
to form still another triangle. This process continues, Þ n -1
< 1 Þ 2n - 1 > 100 > 26
2
indefinitely. Find the sum of the perimeters of all the
Þ 2n - 1 > 26 Þ n - 1 > 6
triangles.
Sol. Let a be the side length of equilateral triangle, then \ n > 7 or n ³ 8
AB = BC = CA = a
y Example 48. The line x + y = 1 meets X-axis at A and
A
Y-axis at B, P is the mid-point of AB, P1 is the foot of
perpendicular from P to OA, M1 is that of P1 from OP;
P2 is that of M1 from OA, M 2 is that of P2 from OP; P3
H
F E is that of M 2 from OA and so on. If Pn denotes the nth
M L foot of the perpendicular on OA, then find OPn .
I K J Y

B
B C
D
P
Q D , E, F are the mid-points of BC , CA and AB, respectively. M1
a M2
\ EF = FD = DE =
2 O P3 P2 P 1 A
X
and H , I , J are the mid-points of EF , FD and DE,
respectively. Sol. We have,

\ IJ = JH = HI =
a (OMn - 1 )2 = (OPn )2 + ( Pn Mn - 1 )2
4
= (OPn )2 + (OPn )2 = 2 (OPn )2 = 2 a n2 [say]
a
Similarly, KL = ML = KM = , ... Also, (OPn - 1 )2 = (OMn - 1 )2 + ( Pn - 1Mn - 1 )2
8
Chap 03 Sequences and Series 229

1 1 1
Þ a n2 - 1 = 2 a n2 + a n2 - 1 Þ a n2 = a n2 - 1 in GP with a = 1, r = . Let time taken by the insect in
2 4 2
1 covering 3 mm be n seconds.
Þ an = an - 1
2 1 1
1 1 1 \1 + + + ... + n terms = 3
Þ OPn = a n = a n - 1 = 2 a n - 2 = ... = n 2 4
2 2 2 é æ1ö ù
n

æ1ö
n
1 × ê1 - ç ÷ ú
\ OPn = ç ÷ êë è2ø ú
è2ø Þ û =3
1
1-
2
Use of GP in Solving æ1ö 3
n

Practical Problems Þ 1- ç ÷ =
è2ø 2
n
In this part, we will see how the formulae relating to GP æ1ö 1
Þ ç ÷ =-
can be made use of in solving practical problems. è2ø 2
y Example 49. Dipesh writes letters to four of his Þ 2n = - 2
friends. He asks each of them to copy the letter and which is impossible because 2n > 0
mail to four different persons with the request that \Our supposition is wrong.
they continue the chain similarly. Assuming that the \ There is no n Î N , for which the insect could never 3 mm
chain is not broken and that it costs 25 paise to mail in n seconds.
one letter, find the total money spent on postage till Hence, it will never to able to cover 3 mm.
the 8th set of letters is mailed.
Sol. Number of letters in the 1st set = 4 (These are letters sent Remark
by Dipesh) The maximum distance that the insect could cover is 2 mm.
1 1 1
Number of letters in the 2nd set = 4 + 4 + 4 + 4 = 16 i.e., 1 + + + ... = =2
2 4 1
Number of letters in the 3rd set 1-
2
= 4 + 4 + 4 + ... + 16 terms = 64
M M M M y Example 51. The pollution in a normal atmosphere is
The number of letters sent in the 1st set, 2nd set, 3rd set, ... less than 0.01%. Due to leakage of a gas from a
are respectively 4, 16, 64, ... which is a GP with a = 4, factory, the pollution is increased to 20%. If every day
16 64 80% of the pollution is neutralised, in how many days
r = = =4
4 16 the atmosphere will be normal?
\Total number of letters in all the first 8 sets Sol. Let the pollution on 1st day = 20
4 ( 4 8 - 1)
= = 87380 The pollution on 2nd day = 20 ´ 20% = 20 (0.20)
4 -1
The pollution on 3rd day = 20 (0.20)2
25
\ Total money spent on letters = 87380 ´ = ` 21845 M M M M
100
Let in n days the atmosphere will be normal
y Example 50. An insect starts from a point and \ 20 (0.20)n - 1 < 001
.
travels in a straight path 1 mm in the first second and
n -1
half of the distance covered in the previous second in æ2ö 1
Þ ç ÷ <
the succeeding second. In how much time would it è 10 ø 2000
reach a point 3 mm away from its starting point. Taking logarithm on base 10, we get
Sol. Distance covered by the insect in the 1st second = 1 mm
1 1 (n - 1) (log 2 - log 10) < log 1 - log 2000
Distance covered by it in the 2nd second = 1 ´ = mm
2 2 Þ (n - 1) (0.3010 - 1) < 0 - (0.3010 + 3)
1 1 1
Distance covered by it in the 3rd second = ´ = mm Þ n -1>
3.3010
2 2 4 0.6990
M M M M
The distance covered by the insect in 1st second, 2nd Þ n > 5722
.
1 1 Hence, the atmosphere will be normal in 6 days.
second, 3rd second, ... are respectively 1, , , ..., which are
2 4
230 Textbook of Algebra

#L Exercise for Session 3


1. The fourth, seventh and the last term of a GP are 10, 80 and 2560, respectively. The first term and number of
terms in GP are
4 4 5 5
(a) , 12 (b) , 10 (c) , 12 (d) , 10
5 5 4 4
2. If the first and the nth terms of a GP are a and b respectively and if P is the product of the first n terms, then P 2
is equal to
(a) ab (b) (ab )n /2
(c) (ab )n (d) None of these

3. If a1, a 2, a 3, (a1 > 0) are three successive terms of a GP with common ratio r, the value of r for which
a 3 > 4a 2 - 3a1 holds is given by
(a) 1 < r < 3 (b) - 3 < r < - 1 (c) r < 1 or r > 3 (d) None of these
4. If x , 2x + 2, 3x + 3 are in GP, the fourth term is
(a) 27 (b) - 27 (c) 13.5 (d) - 13. 5

5. In a sequence of 21 terms the first 11 terms are in AP with common difference 2 and the last 11 terms are in
GP with common ratio 2, if the middle term of the AP is equal to the middle term of GP, the middle term of the
entire sequence is
10 10 32 32
(a) - (b) (c) - (d)
31 31 31 31

6. Three distinct numbers x , y , z form a GP in that order and the numbers 7x + 5y , 7y + 5z , 7z + 5x form an AP
in that order. The common ratio of GP is
(a) - 4 (b) -2 (c) 10 (d) 18

7. The sum to n terms of the series 11 + 103 + 1005 + ... is


1 n 1 n 10 10
(a) (10 - 1) + n 2 (b) (10 - 1) + 2n (c) (10n - 1) + n 2 (d) (10n - 1) + 2n
9 9 9 9
8. In an increasing GP, the sum of the first and last term is 66, the product of the second and the last but one is
128 and the sum of the sum of the terms is 126, then the number of terms in the series is
(a) 6 (b) 8 (c) 10 (d) 12
S (S - S2 )
9. If S1, S2, S3 be respectively the sum of n, 2n and 3n terms of a GP, then 1 3 is equal to
(S2 - S1)2
(a) 1 (b) 2 (c) 3 (d) 4

10. If | a | < 1and | b | < 1, then the sum of the series 1 + (1 + a )b + (1 + a + a 2 ) b 2 + (1 + a + a 2 + a 3 ) b 3 + ... is
1 1 1 1
(a) (b) (c) (d)
(1 - a ) (1 - b ) (1 - a ) (1 - ab ) (1 - b ) (1 - ab ) (1 - a ) (1 - b ) (1 - ab )

11. If the sides of a triangle are in GP and its larger angle is twice the smallest, then the common ratio r satisfies
the inequality
(a) 0 < r < 2 (b) 1 < r < 2 (c) 1 < r < 2 (d) r > 2

12. 3 2 2
If ax + bx + cx + d is divisible by ax + c, then a, b , c, d are in
(a) AP (b) GP (c) HP (d) None of these

13. If (r )n denotes the number r r r ... (n digits), where r = 1, 2, 3, ..., 9 and a = (6)n, b = (8)n, c = (4)2n, then
(a) a 2 + b + c = 0 (b) a 2 + b - c = 0 (c) a 2 + b - 2c = 0 (d) a 2 + b - 9c = 0
· ·
14. 0.4 2 7 represents the rational number
47 47 47 49
(a) (b) (c) (d)
99 110 999 99

15. If the product of three numbers in GP be 216 and their sum is 19, then the numbers are
(a) 4, 6, 9 (b) 4, 7, 8 (c) 3, 7, 9 (d) None of these
Session 4
Harmonic Sequence or Harmonic Progression (HP)

Harmonic Sequence or Þ
4a - 5
= 2a
Harmonic Progression (HP) Þ
5-2
4a - 5 = 6a or 2a = - 5
A Harmonic Progression (HP) is a sequence, if the 5
\ a=-
reciprocals of its terms are in Arithmetic Progression (AP) 2
1 1 1
i.e., t 1 , t 2 , t 3 , ... is HP if and only if , , , ... is an AP. (i) n th Term of HP from Beginning
t1 t2 t 3
For example, The sequence Let a be the first term, d be the common difference of an
1 1 1
(i) , , , ...
5 10
(ii) 2, , ,K AP. Then, nth term of an AP from beginning = a + (n - 1) d
2 5 8 2 3 Hence, the nth term of HP from beginning
1 1 1 1
(iii) , , , ... are HP’s. = , " n ÎN
a a + d a + 2d a + (n - 1) d

Remark (ii) n th Term of HP from End


1. No term of HP can be zero.
Let l be the last term, d be the common difference of an
2. The most general or standard HP is
AP. Then,
1 1 1 1
, , , , ....
a a + d a + 2d a + 3d nth term of an AP from end = l - (n - 1) d
1
Hence, the nth term of HP from end = ,"n ÎN
y Example 52. If a, b , c are in HP, then show that l - (n - 1) d
a -b a
= . Remark
b -c c
1 1
1. +
Sol. Since, a, b, c are in HP, therefore nth term of HP from beginning nth term of HP from end
1 1
1 1 1 = a+ l = +
, , are in AP first term of HP last term of HP
a b c
2. There is no general formula for the sum of any number of
1 1 1 1
i.e. - = - quantities in HP are generally solved by inverting the terms
b a c b and making use of the corresponding AP.
a -b b -c a-b a
or = or = 1 1 1 1
ab bc b-c c y Example 54. If + + + = 0, then prove
a c a -b c -b
Remark that a, b , c are in HP, unless b = a + c .
A HP may also be defined as a series in which every three 1 1 1 1
I - II I Sol. We have, + + + =0
consecutive terms (say I, II, III) satisfy = this relation.
II - III III a c a-b c -b
æ1 1 ö æ1 1 ö
y Example 53. Find the first term of a HP whose Þ ç + ÷+ç + ÷ =0
èa c - b ø èc a - b ø
5 1
second term is and the third term is .
4 2 (c - b + a ) (a - b + c )
5 1 Þ + =0
Sol. Let a be the first term. Then, a, , are in HP. a (c - b ) c (a - b )
4 2
5 é 1 1 ù
a- Þ (a + c - b ) ê +
4 =a ú =0
Then, [from above note] ë a (c - b ) c (a - b ) û
5 1 1
-
4 2 2 Þ (a + c - b ) [ 2ac - b (a + c )] = 0
232 Textbook of Algebra

If a + c - b ¹ 0, then 2ac - b (a + c ) = 0 12 12 24
Þ + = 25 Þ = 25
2ac 1 - 12d 1 + 12d 1 - 144d 2
or b=
a+c 24 1
Þ 1 - 144d 2 = or d 2 =
Therefore, a, b,c are in HP and if 2ac - b (a + c ) ¹ 0, then 25 25 ´ 144
a + c - b = 0 i.e., b = a + c . 1
\ d=±
60
y Example 55. If a1 , a 2 , a 3 , ..., an are in HP, then prove 1 1 1 1 1 1
\ a - d , a, a + d are , , or , , ×
that a1a 2 + a 2 a 3 + a 3a 4 + ... + an - 1an = (n - 1) a1an 15 12 10 10 12 15
Sol. Given, a1, a 2 , a 3 , ..., an are in HP. Hence, three numbers in HP are 15, 12, 10 or 10, 12, 15.
1 1 1 1
\ , , , ..., are in AP. y Example 57. If p th, qth and r th terms of a HP be
a1 a 2 a 3 an
respectively a, b and c, then prove that
Let D be the common difference of the AP, then (q - r ) bc + (r - p ) ca + (p - q ) ab = 0.
1 1 1 1 1 1 1 1
- = - = - = ... = - =D Sol. Let A and D be the first term and common difference of
a 2 a1 a 3 a 2 a 4 a 3 an an - 1 the corresponding AP. Now, a, b, c are respectively the p
a1 - a 2 a 2 - a 3 a 3 - a 4 an - 1 - an th, q th and r th terms of HP.
Þ = = = ... = =D 1 1 1
a1a 2 a2 a3 a 3a 4 an - 1 an \ , , will be respectively the p th, q th and r th terms of
a b c
a1 - a 2 a - a3 a - a4 the corresponding AP.
Þ a1a 2 = , a2 a3 = 2 , a 3a 4 = 3 ,
D D D 1
an - 1 - an Þ = A + ( p - 1) D …(i)
..., an - 1an = a
D
1
On adding all such expressions, we get = A + ( q - 1) D …(ii)
b
a - an a1an æ 1 1ö
a1a 2 + a 2 a 3 + a 3a 4 + ... + an - 1an = 1 = ç - ÷ 1
D D è an a1 ø = A + ( r - 1) D …(iii)
c
é1
a1an 1ù On subtracting Eq. (iii) from Eq. (ii), we get
= ê a + (n - 1) D - a ú = (n - 1) a1an
D ë 1 1û 1 1 (c - b ) (b - c )
- = (q - r ) D Þ bc (q - r ) = =-
Hence, a1a 2 + a 2 a 3 + a 3a 4 + ... + an - 1an = (n - 1) a1an b c D D
So, LHS = (q - r ) bc + (r - p ) ca + ( p - q ) ab
Remark 1
In particular case, =- {b - c + c - a + a - b } = 0 = RHS
D
1. when n = 4 a1a2 + a2 a3 + a3a4 = 3a1a4
2. when n = 6 a1a2 + a2 a3 + a3a4 + a4 a5 + a5 a6 = 5 a1a6
Theorem Relating to the Three Series
y Example 56. The sum of three numbers in HP is 37
1 If a, b, c are three consecutive terms of a series, then
and the sum of their reciprocals is . Find the a -b a
4 if = , then a, b, c are in AP.
numbers. b -c a
1 1 1 a -b a a -b a
Sol. Three numbers in HP can be taken as , , . if = , then a, b, c are in GP and if = , then
a-d a a+d b -c b b -c c
1 1 1 a, b, c are in HP.
Then, + + = 37 …(i)
a-d a a+d

and a-d +a+a+d =


1
4
Mixed Examples on AP, GP and HP
\ a=
1 y Example 58. If a, b , c are in AP and a 2 , b 2 , c 2 be in
12 a
HP. Then, prove that - , b , c are in GP or else
12 12 2
From Eq. (i), + 12 + = 37 a =b =c.
1 - 12d 1 + 12d
Sol. Given, a, b, c are in AP.
Chap 03 Sequences and Series 233

a+c b, c , d are in GP,


\ b= …(i)
2 \ c 2 = bd …(ii)
2 2 2
and a , b , c are in HP. and c , d , e are in HP.
2 2 2ce
\ b2 =
2a c
…(ii) \ d= …(iii)
c +e
a2 + c 2
Now, substituting the values of b and d in Eq. (ii), then
From Eq. (ii) b 2 {(a + c )2 - 2ac } = 2a 2c 2
æ a + c ö æ 2ce ö
Þ b 2 {(2b )2 - 2ac } = 2a 2c 2 [from Eq. (i)] c2 = ç ÷ç ÷
è 2 ø èc + e ø
4 2 2 2
Þ 2b - acb - a c = 0
Þ c (c + e ) = e (a + c )
Þ (2b 2 + ac ) (b 2 - ac ) = 0 Þ c 2 = ae …(iv)
Þ 2b 2 + ac = 0 or b 2 - ac = 0 Given, a = 2, e = 18
2 1 2 a
If 2b + ac = 0, then b = - ac or - , b, c are in GP From Eq. (iv), c 2 = (2) (18) = 36
2 2 \ c = ±6
2
and if b - ac = 0 Þ a, b, c are in GP. 2±6
From Eq. (i), b = = 4, - 2
But given, a, b, c are in AP. 2
Which is possible only when a = b = c c 2 36 36 36
and from Eq. (ii), d= = = or
b b 4 -2
y Example 59. If a, b , c are in HP, b , c , d are in GP \ d = 9 or - 18
ab 2 Hence, c = 6, b = 4, d = 9 or c = - 6, b = - 2, d = - 18
and c , d , e are in AP, then show that e = .
(2a - b ) 2
y Example 61. If three positive numbers a, b and c are
Sol. Given, a, b, c are in HP.
in AP, GP and HP as well, then find their values.
2ac ab
\ b= or c = …(i) Sol. Since a, b, c are in AP, GP and HP as well
a+c 2a - b
a+c
Given, b, c , d are in GP. \ b= …(i)
2
\ c 2 = bd …(ii) 2
b = ac …(ii)
and given, c , d , e are in AP. 2ac
c +e and b= …(iii)
\ d= a+c
2
From Eqs. (i) and (ii), we have
Þ e = 2d - c 2
æa + c ö
æ 2c 2 ö ç ÷ = ac
e=ç - c÷ [from Eq. (ii)] …(iii) è 2 ø
è b ø
or (a + c )2 = 4ac
2
2 æ ab ö æ ab ö (a + c )2 - 4ac = 0
From Eqs. (i) and (iii), e = ç ÷ -ç ÷ or
b è 2a - b ø è 2a - b ø
or (a - c )2 = 0
ab
= {2a - (2a - b )} \ a=c …(iv)
(2a - b )2
a+a
On putting c = a in Eq. (i), we get b = =a …(v)
ab 2 2
=
(2a - b )2 From Eqs. (iv) and (v), a = b = c , thus the three numbers will
be equal.
y Example 60. If a, b , c , d and e be five real numbers
such that a, b , c are in AP; b , c , d are in GP; c , d , e are in Remark
1. If three positive numbers are in any two of AP, GP and HP,
HP. If a = 2 and e = 18, then find all possible values of
then it will be also in third.
b , c and d. 2. Thus, if three positive numbers are in any two of AP, GP and
Sol. Given, a, b, c are in AP, HP, then they will be in the third progression and the
a+c numbers will be equal.
\ b= …(i)
2
234 Textbook of Algebra

#L Exercise for Session 4


1. If a, b , c are in AP and b , c, d be in HP, then
(a) ab = cd (b) ad = bc (c) ac = bd (d) abcd = 1
a 1 2
2. If a, b , c are in AP, then , , are in
bc c b
(a) AP (b) GP (c) HP (d) None of these
3. If a, b , c are in AP and a, b , d are in GP, then a, a - b , d - c will be in
(a) AP (b) GP (c) HP (d) None of these

4. If x , 1, z are in AP and x , 2, z are in GP, then x , 4, z will be in


(a) AP (b) GP (c) HP (d) None of these

5. If a, b , c are in GP, a - b , c - a, b - c are in HP, then a + 4b + c is equal to


(a) 0 (b) 1 (c) - 1 (d) None of these

6. If (m + 1)th, (n + 1)th and (r + 1)th terms of an AP are in GP and m , n, r are in HP, then the value of the ratio of
the common difference to the first term of the AP is
2 2 n n
(a) - (b) (c) - (d)
n n 2 2

7. If a, b , c are in AP and a 2, b 2, c 2 are in HP, then

(a) a = b = c (b) 2b = 3a + c (c) b 2 = æ ac ö


ç ÷ (d) None of these
è 8ø

a b c
8. If a, b , c are in HP, then , , are in
b +c c+a a+b
(a) AP (b) GP (c) HP (d) None of these
x +y y +z
9. If , y, are in HP, then x , y , z are in
2 2
(a) AP (b) GP (c) HP (d) None of these
a+b b +c 1
10. If ,b, are in AP, then a, , c are in
1 - ab 1 - bc b
(a) AP (b) GP (c) HP (d) None of these
Session 5
Mean

Mean Let two numbers be a and b and A 1 , A 2 , A 3 , K, A n are n


AM’s between them.
Arithmetic Mean Then, a, A 1 , A 2 , A 3 , ..., A n , b will be in AP.
If three terms are in AP, then the middle term is called the \ Sum of n AM’s between a and b
Arithmetic Mean (or shortly written as AM) between the = A1 + A 2 + A 3 + ... + An
other two, so if a, b, c are in AP, then b is the AM of a n
and c. = ( A1 + An ) [Q A1, A 2 , A 3 , ..., An are in AP]
2
n n
(i) Single AM of n Positive Numbers = (a + d + a + nd ) = [ 2a + (n + 1) d ]
2 2
Let n positive numbers be a 1 , a 2 , a 3 , ..., a n and A be the n
AM of these numbers, then = (2a + b - a ) [from Eq. (i)]
2
a + a 2 + a 3 + ... + a n æa + b ö
A= 1 =n ç ÷=n [AM between a and b]
n è 2 ø
In particular Let a and b be two given numbers and A be [Remember]
the AM between them, then a, A, b are in AP. Aliter A1 + A2 + A 3 + ... + An
a +b = (a + A 1 + A2 + A 3 + ... + A n + b ) - (a + b )
\ A=
2 (n + 2 ) æa +b ö
= (a + b ) - (a + b ) = n ç ÷
Remark 2 è 2 ø
2a + 3b + 5c
1. AM of 2a, 3b, 5c is . =n [AM of a and b]
3
a + a2 + a3 + ... + an - 1 + 2an Aliter
2. AM of a1, a2, a3, ..., an - 1, 2an is 1 .
n [This method is applicable only when n is even]
(ii) Insert n-Arithmetic Mean Between A1 + A2 + A 3 + ... + An - 2 + An - 1 + An
Two Numbers = ( A1 + An ) + ( A2 + An - 1 ) + ( A 3 + An - 2 ) + ...
n
Let a and b be two given numbers and A 1 , A 2 , A 3 , ..., A n upto terms
2
are AM’s between them.
n
Then, a, A 1 , A 2 , A 3 , ..., A n , b will be in AP. = (a + b ) + (a + b ) + (a + b ) + ... upto times
2
Now, b = (n + 2 ) th term = a + (n + 2 - 1) d [QTn + T ¢n = a + l ]
æb -a ö n æa +b ö
\ d =ç ÷ ` = (a + b ) = n ç ÷ =n [AM of a and b]
è n + 1ø 2 è 2 ø

[Remember] [where, d = common difference] …(i) Corollary II The sum ofmAM’s between any two
numbers is to the sum of n AM’s between them asm : n .
\ A1 = a + d , A2 = a + 2d , ...., An = a + nd
Let two numbers be a and b.
æb -a ö æb -a ö
Þ A1 = a + ç ÷ , A2 = a + 2 ç ÷ , ... , An \ Sum of m AM’s between a and b =m [AM of a and b]
è n + 1ø è n + 1ø
…(i)
æb -a ö Similarly, sum of n AM’s between a and b = n
=a +n ç ÷
è n + 1ø [AM of a and b] …(ii)
Corollary I The sum of n AM’s between two given Sum of m AM’s m ( AM of a and b ) m
\ = =
quantities is equal to n times the AM between them. Sum of n AM’s n ( AM of a and b ) n
236 Textbook of Algebra

y Example 62. If a, b , c are in AP and p is the AM According to the example,


between a and b and q is the AM between b and c , A8 3
=
then show that b is the AM between p and q. An - 2 5
Sol. Qa, b, c are in AP. Þ 5 (3 + 8d ) = 3 [3 + (n - 2)d ] Þ 6 = d (3n - 46)
\ 2b = a + c …(i) Þ 6 = (3n - 46)
51
[from Eq. (i)]
Q p is the AM between a and b. ( n + 1)
a+b Þ 6n + 6 = 153n - 2346 Þ 147n = 2352
\ p= …(ii)
2 \ n = 16
Qq is the AM between b and c .
b+c y Example 65. If 11 AM’s are inserted between 28 and
\ q= …(iii)
2 10, then find the three middle terms in the series.
On adding Eqs. (ii) and (iii), then Sol. Let A1, A 2 , A 3 , ..., A11 be 11 AM’s between 28 and 10.
a + b b + c a + c + 2b 2b + 2b If d be the common difference, then
p +q = + = = [using Eq. (i)]
2 2 2 2 10 - 28 3
d= =-
p +q 12 2
\ p + q = 2b or b =
2 Total means = 11 (odd)
Hence, b is the AM between p and q. æ 11 + 1 ö
\ Middle mean = ç ÷th = 6th = A 6
è 2 ø
an + 1 + b n + 1
y Example 63. Find n, so that (a ¹ b ) be Then, three middle terms are A 5 , A 6 and A 7 .
an + b n 15 41
\ A 5 = 28 + 5d = 28 - =
the AM between a and b. 2 2
an + 1 + bn + 1 a+b A 6 = 28 + 6d = 28 - 9 = 19
Sol. Q =
n
a +b n
2 21 35
and A 7 = 28 + 7d = 28 - =
2 2
é æ a ön + 1 ù
bn + 1 ê ç ÷ + 1ú
êë è b ø úû b é æ a ö ù y Example 66. If a, b , c are in AP, then show that
Þ = ê ç ÷ + 1ú
é n ù 2 ë è b ø û 2
æa ö
bn ê ç ÷ + 1ú a 2 (b + c ) + b 2 (c + a ) + c 2 (a + b ) = (a + b + c ) 3 .
è b ø 9
êë úû
Sol. Qa, b, c are in AP.
é æ a ön + 1 ù é æ a ön ù æa ö
Þ 2 êç ÷ + 1ú = ê ç ÷ + 1ú ç + 1÷ a+c
è b ø è b ø è b ø \ b= i.e., 2b = a + c …(i)
êë úû êë úû 2
a LHS = a 2 (b + c ) + b 2 (c + a ) + c 2 (a + b )
Let =l
b = (a 2b + a 2c ) + b 2 ( 2b ) + (c 2a + c 2b )
\ 2 ln + 1 + 2 = ( ln + 1 ) ( l + 1 ) = b (a 2 + c 2 ) + ac (a + c ) + 2b 3
n +1 n +1 n
Þ 2l +2= l + l + l +1 = b [(a + c )2 - 2ac ] + ac ( 2b ) + 2b 3
n +1 n n
Þ l - l - l +1=0 Þ ( l - 1) ( l - 1) = 0
= b (a + c )2 + 2b 3 = b ( 2b )2 + 2b 3 = 6b 3
l -1¹0 [Qa ¹ b ]
2 2
\ ln - 1 = 0 Þ ln = 1 = l0 RHS = (a + b + c )3 = (2b + b )3
9 9
Þ n =0 2
= ´ 27b 3 = 6b 3
9
y Example 64. There are n AM’s between 3 and 54 Hence, LHS = RHS
such that 8th mean is to (n - 2) th mean as 3 to 5.
Find n.
Sol. Let A1, A 2 , A 3 , ..., An be n AM’s between 3 and 54. Geometric Mean
If d be the common difference, then If three terms are in GP, then the middle term is called the
54 - 3 51 Geometric Mean (or shortly written as GM) between the
d= = …(i)
n +1 n +1 other two, so if a, b, c are in GP, then b is the GM of a
and c.
Chap 03 Sequences and Series 237

n (n + 1 )
(i) Single GM of n Positive Numbers n (n + 1 ) é 1 ù 2
n êæ b ö n + 1 ú
Let n positive numbers be a 1 , a 2 , a 3 , ..., a n and G be the = an × r 2 =a × ç ÷ [from Eq. (i)]
GM of these numbers, then G = (a 1 a 2 a 3 ... a n ) 1 /n êè a ø ú
êë úû
n
In particular Let a and b be two numbers and G be the
æb ö 2
GM between them, then a, G , b are in GP. = a ç ÷ = a n /2 b n /2 = ( ab ) n
n
èa ø
Hence, G = ab ; a > 0, b > 0
= [GM of a and b] n [Remember]
Remark Aliter [This method is applicable only when n is even]
1. If a < 0, b < 0, then G = - ab
G 1 G 2 G 3 KG n - 2 G n -1 G n = (G 1 G n ) (G 2 G n - 1 )
2. If a < 0, b > 0 or a > 0, b < 0, then GM between a and b does
not exist.
n
(G 3 G n - 2 ) ... factors
2
Example
n
(i) The GM between 4 and 9 is given by = (ab ) (ab ) (ab ) ... factors [QTn ´ Tn¢ = a ´ l ]
2
G = 4 ´9 =6
= (ab ) n /2 = ( ab ) n = [GM of a and b] n
(ii) The GM between - 4 and - 9 is given by
G = -4 ´ - 9 = - 6 y Example 67. If a be one AM and G1 and G 2 be two
geometric means between b and c , then prove that
(iii) The GM between - 4 and 9 or 4 and - 9 does not
G13 + G 23 = 2abc .
exist.
Sol. Given, a = AM between b and c
i.e. (- 4) ´ 9 = -1 36 = 6i
b+c
Þ a= Þ 2a = b + c …(i)
and 4 ´ (-9) = -1 36 = 6i 2
Again, b, G1, G 2 , c are in GP.
(ii) Insert n-Geometric Mean Between G1 G 2 c G2 G2
\ = = Þ b = 1 ,c = 2
Two Numbers b G1 G 2 G2 G1
Let a and b be two given numbers and G 1 , G 2 , G 3 , K, G n and G1G 2 = bc …(ii)
are n GM’s between them. From Eqs. (i) and (ii),
Then, a, G 1 , G 2 , G 3 , ..., G n , b will be in GP. G 2 G 2 G 3 + G 23 G13 + G 23
2a = 1 + 2 = 1 = [QG1G 2 = bc ]
Now, b = (n + 2 ) th term = ar n + 2 - 1 G 2 G1 G1G 2 bc
1 Þ G13 + G 23 = 2abc
æb ö n +1
\ r =ç ÷ [where r = common ratio] [Remember] y Example 68. If one geometric mean G and two
èa ø
arithmetic means p and q be inserted between two
…(i)
quantities, then show that G 2 = (2p - q ) (2q - p ).
\ G 1 = ar , G 2 = ar 2 , ..., G n = ar n
Sol. Let the two quantities be a and b, then
1 2 n
G 2 = ab …(i)
æb ö n +1 æb ö n +1 æb ö n +1
Þ G1 = a ç ÷ , G2 = a ç ÷ , ..., G n = a ç ÷ Again, a, p , q , b are in AP.
èa ø èa ø èa ø
\ p -a=q - p =b-q
Corollary The product of n geometric means between Þ a = 2p - q
a and b is equal to the nth power of the geometric b = 2q - p …(ii)
mean between a and b. From Eqs. (i) and (ii), we get
Let two numbers be a and b and G 1 , G 2 , G 3 , ..., G n are n G 2 = (2p - q ) (2q - p )
GM’s between them.
an + 1 + b n + 1
Then, a, G 1 , G 2 , G 3 , ..., G n , b will be in GP. y Example 69. Find n, so that (a ¹ b ) be
\ Product of n GM’s between a and b an + b n
the GM between a and b.
= G 1 G 2 G 3 K G n = (ar ) (ar 2 ) (ar 3 ) ...(ar n )
an + 1 + bn + 1
=a 1 + 1 + 1 + ... + 1
×r 1 + 2 + 3 + ... + n Sol. Q = ab
an + bn
238 Textbook of Algebra

é æ a ön + 1 ù n +1 An Important Theorem
bn + 1 ê ç ÷ + 1ú æa ö
ç ÷ +1 1
êë è b ø úû a èb ø æa ö 2 Let a and b be two real, positive and unequal numbers and
Þ =b Þ =ç ÷ A, G are arithmetic and geometric means between them,
é n ù b æa ö
n èb ø
n æa ö then
b ê ç ÷ + 1ú ç ÷ +1
êë è b ø úû èb ø (i) a and b are the roots of the equation
Let
a
=l x 2 - 2 Ax + G 2 = 0 [Remember]
b
1 1 1 (ii) a and b are given by A ± ( A + G ) ( A - G )
ln + 1 + 1 n +1
n+
Þ = l2 Þ l +1= l 2 + l2 [Remember]
ln + 1
1 1 1
(iii) A >G [Remember]
n+
Þ l 2 ( l2 - 1) - ( l2 -1) = 0 Proof Q A is the AM between a and b, then
1 1 a +b
n+ A= Þ a + b = 2A …(i)
Þ ( l2 - 1) ( l 2 - 1) = 0 2
1
and G is the GM between a and b, then
Þ l2 -1¹0 [Qa ¹ b ]
1
G = ab Þ ab = G 2 …(ii)
n+
\ l 2 -1=0 \a and b are the roots of the equation, then
n+
1
x 2 - (sum of roots) x + product of roots = 0
Þ l 2 = 1 = l0
1 1 Þ x 2 - (a + b ) x + ab = 0
Þ n + = 0 or n = -
2 2 i.e. x 2 - 2 Ax + G 2 = 0 is the required equation.
1 2A ± ( - 2A) 2 - 4 × 1 × G 2
y Example 70. Insert five geometric means between Þ x= = A ± ( A2 - G 2 )
3 2×1
and 9 and verify that their product is the fifth power of \ x = A ± (A + G ) (A - G )
1
the geometric mean between and 9. Now, for real, positive and unequal numbers of a and b,
3
1 (A + G ) (A - G ) > 0 Þ (A - G ) > 0
Sol. Let G1, G 2 , G 3 , G 4 , G 5 be 5 GM’s between and 9.
3 \ A >G
1
Then, , G1, G 2 , G 3 , G 4 , G 5 , 9 are in GP. Remark
3
1/ 6 1. If a and b are real and positive numbers, then A ³ G
æ ö 2. If a1, a2, a3, ..., an are n positive numbers, then AM ³ GM i.e.,
ç9 ÷ 1
Here, r = common ratio = ç ÷ = 3 2 = 3 a1 + a2 + a3 + ... + an
³ ( a1a2 a3 ... an )1/ n
1
ç ÷ n
è3ø
1 1
\ G1 = ar = × 3 =
3 3 3. (i) If a > 0, b > 0 or a < 0, b < 0 and l1 > 0, l 2 > 0, then
1 a b
2
G 2 = ar = × 3 = 1 l1 + l 2 ³ 2 l1l 2
3 b a
a 1
3 1 if = x > 0 and l1 = l 2 = 1, then x + ³ 2
G 3 = ar = × 3 3 = 3 b x
3
(ii) If a > 0, b < 0 or a < 0, b > 0 and l1 > 0, l 2 > 0, then
4 1 a b
G 4 = ar = × 9 = 3 l1 + l 2 £ - 2 l1l 2
3 b a
1 a 1
5
G 5 = ar = × 9 3 = 3 3 if = x < 0 and l1 > 0, l 2 > 0 then, x + £ - 2
3 b x
Now, Product = G1 ´ G 2 ´ G 3 ´ G 4 ´ G 5
5 5
y Example 71. AM between two numbers whose sum is
1 æ 1 ö 100 is to the GM as 5 : 4, find the numbers.
= ´ 1 ´ 3 ´ 3 ´ 3 3 = 9 3 = ( 3) 2 = ç ´ 9÷
3 è 3 ø Sol. Let the numbers be a and b.
é 1 ù Then, a + b = 100
= ê GM of and 9 5ú
ë 3 û or 2A = 100
Chap 03 Sequences and Series 239

Þ A = 50 …(i) (ii) Insert n-Harmonic Mean Between


A 5 50 5
and given, = Þ = [from Eq. (i)] Two Numbers
G 4 G 4
\ G = 40 …(ii)
Let a and b be two given numbers and H 1 , H 2 , H 3 ,K, H n
are n HM’s between them.
From important theorem a, b = A ± ( A + G ) ( A - G )
Then, a, H 1 , H 2 , H 3 , ..., H n , b will be in HP, if D be the
= 50 ± (50 + 40) (50 - 40) common difference of the corresponding AP.
= 50 ± 30 = 80, 20 \b = (n + 2 ) th term of HP.
\ a = 80, b = 20 1
=
or a = 20, b = 80 ( n + 2 )th term of corresponding AP
y Example 72. If a1 , a 2 , K, an are positive real numbers 1
=
1
whose product is a fixed number c , then find the + (n + 2 - 1) D
minimum value of a1 + a 2 + ... + an - 1 + 3an . a
Sol. Q AM ³ GM 1 1
-
a1 + a 2 + ... + an - 1 + 3an Þ D= b a [Remember]
\ ³ (a1a 2 ...an - 13an )1/n = (3c )1/n (n + 1)
n
Þ a1 + a 2 + ... + an - 1 + 3an ³ n (3c )1/n
1 1 1 1 1 1
\ = + D, = + 2 D , …, = + nD
H1 a H2 a Hn a
Hence, the minimum value of a1 + a 2 + ... + an - 1 + 3an is
1 1 (a - b ) 1 1 2 (a - b ) 1
n (3c )1/n . Þ = + , = + , ...,
H 1 a ab (n + 1) H 2 a ab (n + 1) Hn
1 n (a - b )
Harmonic Mean = +
a ab (n + 1)
If three terms are in HP, then the middle term is called the
Corollary The sum of reciprocals of n harmonic
Harmonic Mean (or shortly written as HM) between the
means between two given numbers is n times the
other two, so if a, b, c are in HP, then b is the HM of a and c.
reciprocal of single HM between them.
(i) Single HM of n Positive Numbers Let two numbers be a and b and H 1 , H 2 , H 3 , ..., H n are n
Let n positive numbers be a 1 , a 2 , a 3 , ..., a n and H be the HM’s between them. Then, a, H 1 , H 2 , H 3 , ..., H n , b will be
HM of these numbers, then in HP.
n 1 1 1 1 næ 1 1 ö
H= \ + + + ... + = ç + ÷
æ 1 1 1 1ö H1 H2 H 3 Hn 2 è H1 Hn ø
ç + + + ... + ÷
é n ù
è a1 a2 a 3 an ø
êëQS n = 2 (a + l )úû
n æ1 1 ö n æ 1 1ö
In particular Let a and b be two given numbers and H be = ç + D + - D÷ = ç + ÷
the HM between them a, H , b are in HP. 2 èa b ø 2 èa b ø
2 2ab n n
Hence, H = i.e., H = = =
1 1 (a + b ) æ ö [HM of a and b ]
+ ç 2 ÷
a b ç ÷
ç 1 + 1÷
Remark èa bø
3 3abc
HM of a, b, c is or .
1 1 1
+ + ab + bc + ca Aliter [This method is applicable only when n is even]
a b c 1 1 1 1 1 1
a +b + + + ... + + +
Caution The AM between two numbers a and b is . H1 H2 H 3 Hn -2 Hn -1 Hn
2
It does not follow that HM between the same numbers is æ 1 1 ö æ 1 1 ö
1 1 =ç + ÷ +ç + ÷
+ è H1 Hn ø è H2 Hn -1 ø
2 a b 2ab
. The HM is the reciprocals of i.e., .
a +b 2 (a + b ) æ 1 1 ö n
+ç + ÷ + ... upto terms
è H 3 Hn -2 ø 2
240 Textbook of Algebra

æ1 1 ö æ1 1 ö and given h1, h 2 , h 3 , ..., h10 are in HP.


= ç + D + - D ÷ + ç + 2D + - 2D ÷ If D be common difference of corresponding AP.
èa b ø èa b ø
1 1 1 1
æ1 1 ö n - -
+ ç + 3 D + - 3 D ÷ + ...upto terms h10 h1 3 2 1
Then, D= = =-
èa b ø 2 9 9 54
æ 1 1ö æ 1 1ö æ 1 1ö n 1 1 1 6 1 1 7 18
= ç + ÷ + ç + ÷ + ç + ÷ + ... upto terms \ = + 6D = - = - = Þ h7 =
èa b ø èa b ø èa b ø 2 h 7 h1 2 54 2 9 18 7
7 18
n æ 1 1ö n n Hence, a 4 × h 7 = ´ =6
= ç + ÷= = 3 7
2 èa b ø æ ö ( HM of a and b )
ç 2 ÷ an + 1 + b n + 1
ç ÷ y Example 75. Find n, so that (a ¹ b ) be
ç 1 + 1÷ an + b n
èa bø
HM between a and b.
y Example 73. If H be the harmonic mean between x an + 1 + bn + 1 2ab
Sol. Q =
H+x H+y n
a +b n
a+b
and y, then show that + =2
H-x H-y é æ a ön + 1 ù
bn + 1 ê ç ÷ + 1ú b 2 é2 æç a ö÷ ù
2xy
êë è b ø úû ê èb øú
Sol. We have, H = ë û
x +y Þ =
é æa ö
n ù æ a ö
H 2y H 2x bn ê ç ÷ + 1ú b ç + 1÷
\ = and = è b ø èb ø
x x +y y x +y êë úû
n +1
By componendo and dividendo, we have æa ö æa ö
ç ÷ +1 2ç ÷
H + x 2y + x + y x + 3y èb ø èb ø
= = Þ =
H - x 2y - x - y y-x æa ö
n
æ a ö
ç ÷ +1 ç ÷ +1
H + y 2x + x + y 3x + y èb ø è b ø
and = = a
H - y 2x - x - y x -y Let =l
b
H + x H + y x + 3y 3x + y
\ + = + ln +1 + 1 2l
H - x H -y y-x x -y Then, =
n
l +1 l +1
x + 3y - 3x - y 2 (y - x )
= = =2
y-x (y - x ) Þ ( l + 1) ( ln + 1 + 1) = 2 l ( ln + 1)
H + x H +y Þ ln + 2 + l + ln + 1 + 1 = 2 ln + 1 + 2l
Aliter + =2
H - x H -y
Þ ln + 2 - ln + 1 - l + 1 = 0
æH + x ö æ H +yö 2x -2y
Þç - 1÷ = ç1 - ÷ Þ = Þ ln + 1 ( l - 1) - 1 ( l - 1) = 0
èH - x ø è H -yø H - x H -y
Þ ( l - 1)( ln + 1 - 1) = 0
i.e. Hx - xy = - Hy + xy Þ H ( x + y ) = 2xy
Þ l -1¹0 [Qa ¹ b ]
2xy
i.e. H = \ ln + 1 - 1 = 0
(x + y )
which is true as, x , H , y are in HP. Hence, the required Þ ln + 1 = 1 = l 0
result. Þ n + 1 = 0 or n = - 1

y Example 74. If a1 , a 2 , a 3 , ..., a10 be in AP and y Example 76. Insert 6 harmonic means
h1 , h 2 , h 3 , ..., h10 be in HP. If a1 = h1 = 2 and 6
a10 = h10 = 3, then find the value of a 4 h 7 . between 3 and .
23
Sol. Q a1, a 2 , a 3 , ..., a10 are in AP. 6
Sol. Let H 1, H 2 , H 3 , H 4 , H 5 , H 6 be 6 HM’s between 3 and .
If d be the common difference, then 23
a - a1 3 - 2 1 6
d = 10 = = Then, 3, H 1, H 2 , H 3 , H 4 , H 5 , H 6 ,
are in HP.
9 9 9 23
3 1 7 1 1 1 1 1 1 1 23
\ a 4 = a1 + 3d = 2 + = 2 + = …(i) Þ , , , , , , , are in AP.
9 3 3 3 H1 H 2 H 3 H 4 H 5 H 6 6
Chap 03 Sequences and Series 241

Let common difference of this AP be D. Remark


23 1 If a1, a2, a3, K, an are n positive numbers, then AM ³ GM ³ HM i.e.,
-
(23 - 2) 21 1 a1 + a2 + ... + an n
\ D= 6 3 = = = ³ (a1 a2 ...an )1 / n ³
7 7 ´6 7 ´6 2 n æ1 1 1ö
ç + + ... + ÷
1 1 1 1 5 è a1 a2 an ø
\ = +D= + =
H1 3 3 2 6 Sign of equality (AH = GM = HM) holds when numbers are equal
i.e., a1 = a2 = ... = an .
6 1
Þ H1 ==1
5 5 Important Theorem 2
1 1 1 4 3
= + 2D = + 1 = Þ H2 = If A, G , H are arithmetic, geometric and harmonic means
H2 3 3 3 4 of three given numbers a, b and c, then the equation
1 1 1 3 11 6 having a, b, c as its roots is
= + 3D = + = Þ H3 =
H3 3 3 2 6 11 3G 3
x 3 - 3Ax 2 + x - G3 = 0 [Remember]
1 1 1 7 3 H
= + 4D = + 2 = Þ H 4 =
H4 3 3 3 7 a +b +c
Proof QA = AM of a, b, c =
1 1 1 5 17 6 3
= + 5D = + = Þ H5 =
H5 3 3 2 6 17 i.e., a + b + c = 3 A…(i)
1 1 1 10 3 G = GM of a, b, c = (abc ) 1 / 3
and = + 6D = + 3 = Þ H6 =
H6 3 3 3 10 i.e. abc = G 3 …(ii)
1 3 6 3 6 3 and H = HM of a, b, c
\ HM’s are 1 , , , , , ×
5 4 11 7 17 10
3 3abc 3G 3
= = =
Important Theorem 1 1 1 1 ab + bc + ca ab + bc + ca
+ +
Let a and b be two real, positive and unequal numbers and a b c
A, G and H are arithmetic, geometric and harmonic means [from Eq. (ii)]
3
respectively between them, then 3G
i.e. ab + bc + ca = …(iii)
(i) A, G , H form a GP i.e., G 2 = AH [Remember] H
\a, b, c are the roots of the equation
(ii) A > G > H [Remember]
x 3 - (a + b + c ) x 2 + (ab + bc + ca ) x - abc = 0
Proof
a +b 2ab 3G 3
(i) Q A= , G = ab and H = i.e., x 3 - 3 Ax 2 +
x -G3 =0
2 a +b H
æ a + b ö æ 2ab ö 2 [from Eqs. (i), (ii) and (iii)]
Now, AH = ç ÷ç ÷ = ab = G
è 2 ø èa +b ø Geometrical Proof of A > G > H
Therefore, G 2 = AH i.e. A, G , H are in GP. Let OA = a unit and OB = b unit and AB be a diameter of
semi-circle. Draw tangent OT to the circle and TM
Remark perpendicular to AB.
The result AH = G2 will be true for n numbers, if they are in GP.
T
(ii) Q A > G [from important theorem of GM] …(i) Y

A
or >1
G
X
G é A G 2 ù O A M C B
Þ >1 êëQ G = H Þ G = AH úû
H Let C be the centre of the semi-circle.
Þ G >H …(ii) OA + OB (OC - AC ) + (OC + CB )
Q =
From Eqs. (i) and (ii), we get 2 2
2 OC
A >G > H = = OC [Q AC = CB = radius of circle]
2
242 Textbook of Algebra

a +b Sol. Let the two numbers be a and b.


\ OC = [i.e. OC = arithmetic mean]
2 1
Given, G= H …(i)
a +b n
Þ A=
2 Now, G 2 = AH
Now, from geometry H2
Þ = AH [from Eq. (i)]
(OT ) 2 = OA ´ OB = ab = G 2 n2
\ OT = G , the geometric mean H
\ A= …(ii)
Now, from similar DOCT and DOMT, we have n2
Now, from important theorem of GM
OM OT (OT ) 2 ab 2ab
= or OM = = = H æH2 H2 ö
OT OC OC a +b a +b a, b = A ± ( A 2 - G 2 ) = ± ç 4 - 2÷
n2 èn n ø
2
\ OM = H , the harmonic mean H
= [1 ± (1 - n 2 )]
Also, it is clear from the figure, that n2
OC > OT > OM i.e. A > G > H H
[1 + (1 - n 2 )]
a n2
y Example 77. If A x = G y = H z , where A, G, H are AM, \ =
b H [1 - (1 - n 2 )]
GM and HM between two given quantities, then prove n2
that x , y , z are in HP. \ a : b = 1 + (1 - n 2 ) : 1 - (1 - n 2 )
x y z
Sol. Let A = G = H = k
Then, A = k 1/ x , G = k 1/ y , H = k 1/ z y Example 80. If three positive unequal quantities
a, b , c be in HP, then prove that a n + c n > 2b n , n Î N
Q G 2 = AH Þ (k 1/ y )2 = k 1/ x × k 1/ z
2 1 1 1 1 1 Sol. Q G > H
+ 1/ z
Þ k 2 / y = k 1/ x Þ = + Þ , , are in AP.
y x z x y z \ ac > b
n n n
Hence, x , y , z are in HP.
Þ (ac ) 2 > bn or a 2 c 2 > bn …(i)
y Example 78. The harmonic mean of two numbers is n n n n

4, their arithmetic mean A and geometric mean G Also, (a 2 - c 2 )2 > 0 Þ an + c n - 2a 2 c 2 > 0


satisfy the relation 2A + G 2 = 27. Find the numbers. n n
Þ an + c n > 2 a 2 c 2 > 2bn [from Eq. (i)]
Sol. Let the numbers be a and b. n n n
Given, H =4 \ a + c > 2b
Q G 2 = AH = 4 A …(i)
y Example 81.
2
and given 2A + G = 27 (i) If a , b , c , d be four distinct positive quantities in AP,
Þ 2A + 4 A = 27 [from Eq. (i)] then
\ A=
9 (a) bc > ad
2 (b) c -1d -1 + a -1b -1 > 2 (b -1d -1 + a -1c -1 - a -1d -1 )
9
From Eq. (i), G 2 = 4 ´ = 18 (ii) If a , b , c , d be four distinct positive quantities in GP,
2
then
Now, from important theorem of GM
(a) a + d > b + c
9 æ 81 ö
a, b = A ± ( A 2 - G 2 ) = ± ç - 18÷ (b) c -1d -1 + a -1b -1 > 2 (b -1d -1 + a -1c -1 - a -1d -1 )
2 è4 ø
9 3
= ± = 6, 3 or 3, 6 (iii) If a , b , c , d be four distinct positive quantities in HP,
2 2 then
1 (a) a + d > b + c (b) ad > bc
y Example 79. If the geometric mean is times the
n Sol. (i) Q a, b, c , d are in AP.
harmonic mean between two numbers, then show that (a) Applying AM > GM
the ratio of the two numbers is For first three members, b > ac
1 + (1 - n 2 ) : 1 - (1 - n 2 ). Þ b 2 > ac …(i)
Chap 03 Sequences and Series 243

and for last three members, c > bd Þ ab + bc > 2ac …(vii)


Þ 2
c > bd 2bd
…(ii) For last three members, c >
b+d
From Eqs. (i) and (ii), we get
Þ bc + cd > 2bd …(viii)
b 2c 2 > (ac ) (bd )
From Eqs. (vii) and (viii), we get
Hence, bc > ad
ab + bc + bc + cd > 2ac + 2bd
(b) Applying AM > HM
or ab + cd > 2 (ac + bd - bc )
For first three members,
2ac Dividing in each term by abcd, we get
b>
a+c c -1d -1 + a -1b -1 > 2 (b -1d -1 + a -1c -1 - a -1d -1 )
Þ ab + bc > 2ac …(iii) (iii) Qa, b, c , d are in HP.
2bd (a) Applying AM > HM
For last three members, c >
b+d For first three members,
bc + cd > 2bd …(iv) a+c
>b
From Eqs. (iii) and (iv), we get 2
ab + bc + bc + cd > 2ac + 2bd Þ a + c > 2b …(ix)
or ab + cd > 2 (ac + bd - bc ) b+d
For last three members, >c
Dividing in each term by abcd, we get 2
c -1d -1 + a -1b -1 > 2 (b -1d -1 + a -1c -1 - a -1d -1 ) Þ b + d > 2c …(x)
(ii) Q a, b, c , d are in GP. From Eqs. (ix) and (x), we get
(a) Applying AM > GM a + c + b + d > 2b + 2c
For first three members, or a+d >b+c
a+c (b) Applying GM > HM
>b
2 For first three members, ac > b
Þ a + c > 2b …(v)
Þ ac > b 2 …(xi)
b+d
For last three members, >c For last three members,
2
Þ b + d > 2c …(vi) bd > c
From Eqs. (v) and (vi), we get Þ bd > c 2 …(xii)
a + c + b + d > 2b + 2c or a + d > b + c From Eqs. (xi) and (xii), we get
(b) Applying GM > HM (ac ) (bd ) > b 2c 2
2ac
For first three members, b > or ad > bc
a+c
244 Textbook of Algebra

#L Exercise for Session 5


1. If the AM of two positive numbers a and b (a > b ) is twice of their GM, then a : b is
(a) 2 + 3 :2 - 3 (b) 7 + 4 3 : 7 - 4 3
(c) 2 : 7 + 4 3 (d) 2 : 3

2. If A1, A2; G1, G2 and H1, H2 are two arithmetic, geometric and harmonic means, respectively between two
quantities a and b , then which of the following is not the value of ab is?
(a) AH
1 2 (b) A2 H1
(c)G1G2 (d) None of these

3. The GM between - 9 and - 16, is


(a) 12 (b) - 12
(c) - 13 (d) None of these

4. Let n Î N, n > 25. If A, G and H denote the arithmetic mean, geometric mean and harmonic mean of 25 and n.
Then, the least value of n for which A, G, H Î {25, 26, ..., n}, is
(a) 49 (b) 81
(c) 169 (d) 225
6
5. If 9 harmonic means be inserted between 2 and 3, then the value of A + + 5 (where A is any of the AM’s and
H
H is the corresponding HM), is
(a) 8 (b) 9
(c) 10 (d) None of these
H1 + a Hn + b
6. If H1, H2, ..., Hn be n harmonic means between a and b , then + is
H1 - a Hn - b
(a) n (b) n + 1
(c) 2n (d) 2n - 2

7. The AM of two given positive numbers is 2. If the larger number is increased by 1, the GM of the numbers
becomes equal to the AM to the given numbers. Then, the HM of the given numbers is
3 2
(a) (b)
2 3
1
(c) (d) 2
2

8. If a, a1, a 2, a 3, ..., a 2n, b are in AP and a, b1, b 2, b 3, K , b 2n, b are in GP and h is the HM of a and b , then
a1 + a 2n a 2 + a 2n - 1 an + an + 1
+ + ... + is equal to
b1b 2n b 2 b 2n - 1 bn bn + 1
2n
(a) (b) 2nh
h
n
(c) nh (d)
h
Session 6
Arithmetico-Geometric Series (AGS), Sigma (Σ)
Notation, Natural Numbers
Arithmetico-Geometric =a +
dr (1 - r n - 1 )
- [a + (n - 1) d ]r n
Series (AGS) (1 - r )

dr (1 - r n - 1 ) [a + (n - 1 ) d]r n
Definition \ Sn =
a
(1 - r )
+ -
(1 - r )
(1 - r ) 2
A series formed by multiplying the corresponding terms
of an AP and a GP is called Arithmetico - Geometric …(iii)
Series (or shortly written as AGS)
Remark
For example, 1 + 4 + 7 + 10 + ... is an AP and
The above result (iii) is not used as standard formula in any
1 + x + x 2 + x 3 + ... is a GP. examination. You should follow all steps as shown above.
Multiplying together the corresponding terms of these To Deduce the Sum upto Infinity from the Sum upto
series, we get n Terms of an Arithmetico - Geometric Series, when
1 + 4 x + 7 x 2 + 10 x 3 + ... which is an |r | < 1
Arithmetico-Geometric Series. From Eq. (iii), we have
Again, a + (a + d ) + (a + 2d ) + ... + [a + (n - 1)d ] is a typical a dr dr n [a + (n - 1) d ]r n
Sn = + - -
AP 1 - r (1 - r ) 2 (1 - r ) 2 (1 - r )
and 1 + r + r 2 + ... + r n - 1 is a typical GP. If | r | < 1, when n ® ¥, r n ® 0
Multiplying together the corresponding terms of these dr n [a + (n - 1) d ]r n
series, we get and and both ® 0
(1 - r ) 2 (1 - r )
a + (a + d ) r + (a + 2d ) r 2 + ... + [a + (n - 1)d ]r n - 1
a dr
which is called a standard Arithmetico-Geometric series. \ S¥ = +
(1 - r ) (1 - r ) 2

Sum of n Terms of an Independent method Let S ¥ denotes the sum to


infinity, then
Arithmetico-Geometric Series S ¥ = a + (a + d ) r + (a + 2d ) r 2 + (a + 3d ) r 3
Let the series be a + (a + d ) r + (a + 2d ) r 2 + ... + ... upto ¥ …(iv)
+ [a + (n - 1)d ]r n - 1
Let S n denotes the sum to n terms, then Multiplying both sides of Eq. (iv) by r, we get
S n = a + (a + d ) r + (a + 2d ) r 2 + ... + [a + (n - 2 ) d ] r n - 2 rS ¥ = ar + (a + d ) r 2 + (a + 2d ) r 3 + ... upto ¥ …(v)
n -1
+ [a + (n - 1) d ]r …(i) Subtracting Eq. (v) from Eq. (iv), we get
Multiplying both sides of Eq. (i) by r, we get (1 - r ) S ¥ = a + (dr + dr 2 + dr 3 + ... upto ¥)
rS n = ar + (a + d ) r 2 + (a + 2d ) r 3 + ... dr
+ [a + (n - 2 ) d ]r n - 1 + [a + (n - 1) d ]r n …(ii) =a +
(1 - r )
Subtracting Eq. (ii) from Eq. (i), we get a dr
(1 - r ) S n = a + (dr + dr 2 + K + dr n - 1 ) - [a + (n - 1) d ]r n \ S¥ = +
(1 - r ) (1 - r ) 2
246 Textbook of Algebra

ìï n - 1ü n
y Example 82. Find the sum of the series 3 æ1ö ï æ1ö
4 7 10 =1+ í 1 - ç ÷ ý - ( 3 n - 2 ) ç ÷
1+ + 2 + 3 +K 4 ïî è5ø ïþ è5ø
5 5 5 n - 1ù n -1
5 15 é æ1ö ( 3n - 2) æ 1 ö
(i) to n terms. (ii) to infinity. \ Sn = + ê1 - ç ÷ ú- ç ÷
4 16 ê è5ø úû 4 è5ø
Sol. The given series can be written as ë
2 3 n -1
æ1ö æ1ö æ1ö 35 (12n + 7 ) æ 1 ö
1 + 4 ç ÷ + 7 ç ÷ + 10 ç ÷ + ... = - ç ÷
è5ø è5ø è5ø 16 16 è5ø
2 3
The series is an Arithmetico-Geometric series, since each æ1ö æ1ö æ1ö
term is formed by multiplying corresponding terms of the (ii) S ¥ = 1 + 4 ç ÷ + 7 ç ÷ + 10 ç ÷ + ... upto ¥ …(iii)
è5ø è5ø è5ø
series 1, 4, 7, ... which are in AP and
1
1 1 Multiplying both sides of Eq. (i) by , we get
1, , 2 , ... which are in GP. 5
5 5 2 3
2 ù 1 æ1ö æ1ö æ1ö
é 1 æ1ö S ¥ = ç ÷ + 4 ç ÷ + 7 ç ÷ + ... upto ¥ …(iv)
\ Tn = [n th term of 1, 4, 7, ...] ên th term of 1, , ç ÷ , ...ú 5 è5ø è5ø è5ø
ë 5 è5ø úû
Subtracting Eq. (iv) from Eq. (iii), we get
n -1 n -1
æ1ö æ1ö æ 1ö éæ1ö æ1ö2 æ1ö3 ù
= [1 + (n - 1) 3] ´ 1 × ç ÷ = ( 3n - 2) ç ÷ - = +
è5ø è5ø ç 1 ÷ ¥
S 1 3 ê ç ÷ + ç ÷ + ç ÷ + ...upto ¥ ú
è 5ø êë è 5 ø è 5 ø è 5 ø úû
n-2
æ1ö æ 1 ö
\ Tn -1 = (3n - 5) ç ÷
è5ø ç ÷ 3
= 1+3ç 5 ÷ = 1 +
(i) Let sum of n terms of the series is denoted by Sn . 1 4
ç1 - ÷
2 è 5ø
æ1ö æ1ö
Then, Sn = 1 + 4 ç ÷ + 7 ç ÷ + ... 4 7
è5ø è5ø Þ S¥ =
n-2 n -1 5 4
æ1ö æ1ö
+ ( 3n - 5) ç ÷ + ( 3n - 2) ç ÷ …(i) 35
è5ø è5ø \ S¥ =
16
1
Multiplying both the sides of Eq. (i) by , we get
5 y Example 83. If the sum to infinity of the series
2 3 n -1 35
\
1 1 æ1ö æ1ö æ1ö
Sn = + 4 ç ÷ + 7 ç ÷ + ... + (3n - 5) ç ÷ 1 + 4 x + 7 x 2 + 10x 3 + ... is , find x .
5 5 è5ø è5ø è5ø 16
n Sol. Let S ¥ = 1 + 4 x + 7 x 2 + 10x 3 + ... upto ¥ …(i)
æ1ö
+ (3n - 2) ç ÷ …(ii) Multiplying both sides of Eq. (i) by x we get
è5ø
x S ¥ = x + 4 x 2 + 7 x 3 + 10x 4 + ... upto ¥ …(ii)
Subtracting Eq. (ii) from Eq. (i), we get
é1 æ1ö2 æ1ö3 n - 1ù Subtracting Eq. (ii) from Eq. (i), we get
æ 1ö æ1ö
ç1 - ÷ Sn = 1 + 3 ê + ç ÷ + ç ÷ + ... + ç ÷ ú (1 - x ) S ¥ = 1 + 3x + 3x 2 + 3x 3 + ... upto ¥
è 5ø êë 5 è 5 ø è5ø è5ø úû
n æ x ö ( 1 + 2x )
æ1ö = 1 + 3 ( x + x 2 + x 3 + ... upto ¥ ) = 1 + 3 ç ÷=
- ( 3n - 2) ç ÷ è 1 - x ø (1 - x )
è5ø
éæ1ö æ1ö2 æ1ö3 ù ( 1 + 2x ) 35 é 35 ù
4 \ S¥ = = êëQS ¥ = 16 úû
or Sn = 1 + 3 ê ç ÷ + ç ÷ + ç ÷ +... + (n - 1) terms ú (1 - x )2 16
5 è5ø è5ø è5ø
ëê úû
n Þ 16 + 32x = 35 - 70x + 35x 2
æ1ö
- ( 3n - 2) ç ÷ Þ 35x 2 - 102x + 19 = 0
è5ø
ì1 é n - 1ùü Þ (7 x - 19 ) (5x - 1) = 0
æ1ö
ï ê1 - ç ÷ úï 19
ï 5 êë è5ø n x¹
úû ï æ1ö 7
= 1 + 3í ý - ( 3 n - 2 ) ç ÷
1 è5ø [Q for infinity series common ratio - 1 < x < 1]
ï 1- ï
ï 5 ï 1
î þ Hence, x=
5
Chap 03 Sequences and Series 247

n n n
y Example 84. Find the sum of the series 2. å (Tr ± Tr¢ ) = åTr ± åTr¢
1 + 2 2 x + 3 2 x 2 + 4 2 x 3 + ... up to ¥, | x | < 1. r =1 r =1 r =1
2 2 2 2
Sol. Here, the numbers 1 , 2 , 3 , 4 , ... i.e. 1, 4, 9, 16, K are not [sigma operator is distributive over
in AP but 1, 4 - 1 = 3, 9 - 4 = 5, 16 - 9 = 7, K are in AP. addition and subtraction]
Let S ¥ = 1 + 22 x + 32 x 2 + 4 2 x 3 + ... upto ¥
¢ ¹ æç T ö÷ æ n ¢ö
n n

= 1 + 4 x + 9 x 2 + 16x 3 + ... upto ¥ …(i)


3. å T r T r çå r÷
ç åT r ÷
ç ÷
r =1 èr = 1 ø èr = 1 ø
Multiplying both sides of Eq. (i) by x, we get
[sigma operator is not distributive over multiplication]
xS ¥ = x + 4 x 2 + 9 x 3 + 16x 4 + ... upto ¥ …(ii)
æ n ö
Subtracting Eq. (ii) from Eq. (i), we get ç åT r ÷
æT ö ç ÷
n
èr = 1 ø
(1 - x ) S ¥ = 1 + 3x + 5x 2 + 7 x 3 + ... upto ¥ …(iii) 4. å çç r ÷
¢÷
¹
æ n ¢ö
r = 1 è Tr ø
Again, multiplying both sides of Eq. (iii) by x, we get ç åT r ÷
ç ÷
x (1 - x ) S ¥ = x + 3x 2 + 5x 3 + 7 x 4 + ... upto ¥ …(iv) èr = 1 ø
Subtracting Eq. (iv) from Eq. (iii), we get [sigma operator is not distributive over division]
2 3
(1 - x ) (1 - x ) S ¥ = 1 + 2x + 2x + 2x + ... upto ¥ n n

= 1 + 2 ( x + x 2 + x 3 + ... upto ¥)
5. å aTr = a åT r [where a is constant]
r =1 r =1
æ x ö (1 + x ) n n æ n öæ n ö
=1+2ç ÷=
è 1 - x ø (1 - x ) 6. å åT i T j = ç åT i ÷ ç åT j ÷
ç ÷ç ÷
(1 + x ) j =1 i =1 èi = 1 ø è j = 1 ø
\ S¥ =
(1 - x )3 [where i and j are independent]

Sigma (S) Notation Examples on Sigma Notation


m
S is a letter of greek alphabets and it is called ‘sigma’. The (i) å a = a + a + a + K upto m times = am
symbol sigma ( S ) represents the sum of similar terms. i =1
Usually sum of n terms of any series is represented by
placing S the nth term of the series. But if we have to find
(ii) å a = a + a + a + ... upto n times = an
i.e. å 5 = 5n, å 3 = 3n
the sum of k terms of a series whose nth term is u n , this 5 5 5
k
(iii) å (i 2 - 3i ) = å i 2 - 3 å i
will be represented by åu n . i =1 i =1 i =1
n =1
2 2 2 2 2
n=9 9 = (1 + 2 + 3 + 4 + 5 ) - 3 (1 + 2 + 3 + 4 + 5 )
For example, å n, i.e. å n only means the sum of n similar = 55 - 45 = 10
n =1 1 3
æ r +1 ö æ 1+1 ö æ 2 +1 ö æ 3 +1 ö
terms when n varies from 1 to 9. (iv) å çè 2 r + 4 ÷ø = çè 2 × 1 + 4 ÷ø + çè 2 × 2 + 4 ÷ø + çè 2 × 3 + 4 ÷ø
9 r =1
Thus, å n = 1 + 2 + 3 + 4 + 5 + 6 + 7 + 8 + 9 2 3 4 40 + 45 + 48 133 13
1
= + + = = =1
6 8 10 120 120 120
Remark
Shortly S is written in place of å.
n Important Theorems on
1 å (Sigma) Operator
n
Properties of Sigma Notation Theorem 1 å f ( r + 1) - f ( r ) = f (n + 1) - f (1)
n r =1
1. åT r = T1 + T2 + T 3 + ... + Tn , when Tn is the Theorem 2
n
r =1
general term of the series. å f (r + 2) - f (r ) = f (n + 2) + f (n + 1) - f (2) - f (1)
r =1
248 Textbook of Algebra

n n
Proof (Theorem 1) å f (r + 1) - f (r ) Taking å on both sides, we get
r =1 r=1
= [ f (2 ) - f (1)] + [ f (3 ) - f (2 )] n n n n

+ [ f ( 4 ) - f (3 )] + ... + [ f (n + 1) - f (n )] å r 3 - (r - 1) 3 = 3 å r 2 - 3 å r + å 1
r =1 r =1 r =1 r =1
= f (n + 1) - f (1)
Þ n 3 - 0 3 = 3 å n2 - 3 å n + n …(i)
Proof (Theorem 2)
n n [from important Theorem 1]
å f (r + 2) - f (r ) = å[ f (r + 2) - f (r + 1)] Substituting the value of å n in Eq. (i), we get
r =1 r =1
+ [ f (r + 1) - f (r )] 3 × n (n + 1)
Þ n 3 = 3 å n2 - +n
n n 2
= å f (r + 2) - f (r + 1) + å f (r + 1) - f (r ) 3n (n + 1) n
r =1 r =1 Þ 3å n 2 = n 3 + - n = (2n 2 + 3n + 1)
2 2
= [ f (n + 2 ) - f (2 )] + [ f (n + 1) - f (1)] [from Theorem 1] n (n + 1) (2n + 1)
= f (n + 2 ) + f (n + 1) - f (2 ) - f (1) =
2
Remark n (n + 1) (2n + 1)
n k k Þ å n2 = 6
[Remember]
1. å f ( r + k ) - f ( r ) = å f ( n + m) - å f ( m), " k Î N
r =1 m=1 m=1 Independent Proof We know that,
n
2. åf (2r + 1) - f (2r - 1) = f (2n + 1) - f (1) (2 r + 1) 3 - (2 r - 1) 3 = 24 r 2 + 2
r =1 n

3.
n
å f (2r ) - f (2r - 2) = f (2n) - f (0 )
Taking å on both sides, we get
r =1 r =1
n n
å(2 r + 1) 3 - (2 r - 1) 3 = å(24 r 2 + 2)
Natural Numbers r =1
n n
r =1

The positive integers 1, 2, 3, ... are called natural numbers. Þ (2n + 1) 3 - 1 3 = 24 å r 2 + 2 å 1


These form an AP with first term and common difference, r =1 r =1
each equal to unity. [from points to consider-2]
(i) Sum of the First n Natural Numbers Þ (2n + 1) - 1 = 24 å n 2 + 2n
3

n (n + 1) Þ (2n + 1) 3 - (2n + 1) = 24 å n 2
1 + 2 + 3 + ... + n = = Sn
2 Þ (2n + 1) [(2n + 1) 2 - 1] = 24 å n 2
n (n + 1)
Þ Sn = [Remember] Þ (2n + 1) (2n + 1 + 1) (2n + 1 - 1) = 24 å n 2
2
n (n + 1) (2n + 1)
Þ å n2 = [Remember]
(ii) Sum of the First n Odd Natural 6
Numbers
n (iv) Sum of the Cubes of the First n
1 + 3 + 5 + ... upto n terms = [2 × 1 + (n - 1) × 2 ] = n 2 Natural Numbers
2
2
Þ å (2n - 1) = n 2
[Remember] 2 ìn (n + 1) ü
1 3 + 2 3 + 3 3 + ... + n 3 = å n 3 = (å n) = í ý
î 2 þ
(iii) Sum of the Squares of the First n Proof We know that,
Natural Numbers
r 4 - (r - 1) 4 = 4 r 3 - 6r 2 + 4 r - 1
n (n + 1) (2n + 1)
12 + 2 2 + 3 2 + ... + n 2 = å n 2 = n

3 3 2
6 Taking å on both sides, we get
Proof We know that, r - (r - 1) = 3r - 3r + 1 r =1
Chap 03 Sequences and Series 249

n n n n n
Substituting the values of å n, å n 2 , å n 3 in Eq. (i), we get
å r 4 - (r - 1) 4 = 4 å r 3 - 6 å r 2 + 4 å r - å1
r =1 r =1 r =1 r =1 r =1 10 n 2 (n + 1) 2
Þ n5 = 5 å n 4 -
Þ n 4 - 0 4 = 4 å n 3 - 6 å n2 + 4 å n - n …(i) 4
10 n (n + 1) (2n + 1) 5n (n + 1)
[from important theorem 1] + - +n
2 6 2
Substituting the values of ån and ån in Eq. (i), we get
ì 5n (n + 1) 2 5 (n + 1) (2n + 1)
6 n (n + 1) (2n + 1) 4n (n + 1) \ 5 å n 4 = n ín 4 + -
Þ n 4 = 4 ån 3 - + -n 2 3
6 2 î
Þ 4 å n 3 = n 4 + n (n + 1) (2n + 1) - 2n (n + 1) + n 5 (n + 1) ü
+ - 1ý
2 þ
= n [n 3 + (n + 1) (2n + 1) - 2 (n + 1) + 1]
n
= n (n 3 + 2n 2 + n ) {6n 4 + 15n (n 2 + 2n + 1) - 10 (2n 2 + 3n + 1)
=
6
= n 2 (n + 1) 2 + 15n + 15 - 6 }
n
ìn (n + 1) ü
3 2
2
Þ å n 4 = (6n 4 + 15n 3 + 10n 2 - 1)
\ ån = í ý = (ån) [Remember] 30
î 2 þ n (n + 1) (2n + 1) (3n 2 + 3n - 1)
=
Independent Proof We know that, 30
r 2 (r + 1) 2 - r 2 (r - 1) 2 = 4 r 3
n
Remark
If nth term of a sequence is given by Tn = an3 + bn2 + cn + d,
Taking å on both sides, we get
where a, b, c, d are constants.
r =1
Then, sum of n terms, Sn = STn = a Sn3 + b Sn2 + c Sn + d S1
n n
å r 2 (r + 1) 2 - r 2 (r - 1) 2 = 4 å r 3 This can be evaluated using the above results.
r =1 r =1
y Example 85. Find the sum to n terms of the series
Þ n (n + 1) - 1 × 0 = 4 å n 3
2 2 2 2
1 2 + 3 2 + 5 2 + K upto n terms.
[from important Theorem 1] Sol. Let Tn be the nth term of this series, then
2
ìn (n + 1) ü Tn = [1 + (n - 1)2]2 = (2n - 1)2 = 4n 2 - 4n + 1
Þ ån 3 = í ý = (ån)
2
[Remember]
î 2 þ \ Sum of n terms Sn = STn = 4 Sn 2 - 4 Sn + S1
3 3 3 3 2
Corollary 1 + 2 + 3 + K + n = (1 + 2 + 3 + ... + n ) 4n (n + 1) (2n + 1) 4n (n + 1)
= - +n
6 2
(v) Sum of the Powers Four of the n
= ( 4n 2 + 6n + 2 - 6n - 6 + 3)
First n Natural Numbers 3
1 4 + 2 4 + 3 4 + ... + n 4 = å n 4 n ( 4n 2 - 1)
=
3
n (n + 1) (2n + 1) (3n 2 + 3n - 1)
=
30 y Example 86. Find the sum to n terms of the series
Proof We know that, 1 × 2 2 + 2 × 3 2 + 3 × 4 2 + ... .
r 5 - (r - 1) 5 = 5r 4 - 10r 3 + 10r 2 - 5r + 1 Sol. Let Tn be the nth term of this series, then
n Tn = (n th term of 1, 2, 3, …) (nth term of 22 , 32 , 4 2 , ... )
Taking å on both sides, we get
= n (n + 1)2 = n 3 + 2n 2 + n
r =1
n n n n n n \ Sum of n terms Sn = STn
år 5
- (r - 1) = 5 å r - 10
5 4
år 3
+ 10 å r - 5 å r +
2
å1 2 = S n 3 + 2 S n 2 + Sn
r =1 r =1 r =1 r =1 r =1 r =1 2
ì n ( n + 1) ü ì n (n + 1) (2n + 1)ü n (n + 1)
5 5 4 3 2 =í ý + 2í ý+
Þ n - 0 = 5 å n - 10 å n + 10 å n - 5 å n + n …(i) î 2 þ î 6 þ 2
[from important Theorem 1]
250 Textbook of Algebra

n (n + 1) ì n (n + 1) 2(2n + 1) ü y Example 89. Show that


= í + + 1ý
2 î 2 3 þ 1 × 2 2 + 2 × 3 2 + ... + n × (n + 1) 2 3n + 5
n ( n + 1) = .
= (3n 2 + 3n + 8n + 4 + 6) 2 2 2
1 × 2 + 2 × 3 + ... + n × (n + 1) 3n + 1
12
n (n + 1) (3n 2 + 11n + 10) n (n + 1) (n + 2) (3n + 5) Sol. Let Tn and Tn ¢ be the nth terms of the series in numerator
= =
12 12 and denominator of LHS. Then,
\ Tn = n (n + 1)2 and Tn ¢ = n 2 (n + 1)
y Example 87. Find the sum of n terms of the series
STn Sn (n + 1)2 S(n 3 + 2n 2 + n )
whose nth terms is (i) n (n - 1) (n + 1) (ii) n 2 + 3n . \ LHS = = =
STn¢ Sn (n + 1)
2
S(n 3 + n 2 )
Sol. (i) We have, Tn = n (n - 1) (n + 1) = n 3 - n
Sn 3 + 2 S n 2 + S n
\ Sum of n terms Sn = STn = Sn 3 - Sn =
Sn 3 + Sn 2
2
ì n ( n + 1) ü ì n ( n + 1) ü ì n ( n + 1) ü
2
ì n (n + 1) (2n + 1)ü ì n (n + 1)ü
=í ý -í ý
î 2 þ î 2 þ í ý + 2í ý+í ý

2 þ î 6 þ î 2 þ
n ( n + 1) ì n ( n + 1) ü 2
= í - 1ý ì n ( n + 1) ü ì n (n + 1) (2n + 1)ü
2 î 2 þ í ý +í ý
î 2 þ î 6 þ
n ( n + 1) ( n - 1) ( n + 2)
= n (n + 1) ì n (n + 1) 2(2n + 1) ü
4 í + + 1ý
=
2 î 2 3 þ
(ii) We have, Tn = n 2 + 3n n (n + 1) ì n (n + 1) (2n + 1)ü
í + ý
\ Sum of n terms Sn = STn = Sn 2 + S3n 2 î 2 3 þ
= Sn 2 + (31 + 32 + 33 + ... + 3n ) 1
(3n 2 + 3n + 8n + 4 + 6)
n (n + 1) (2n + 1) 3 (3n - 1) =6
= + 1
6 ( 3 - 1) (3n 2 + 3n + 4n + 2)
6
n (n + 1) (2n + 1) 3 n (3n 2 + 11n + 10) (3n + 5) (n + 2) (3n + 5)
= + ( 3 - 1) = = = = RHS
6 2 (3n 2 + 7n + 2) (3n + 1) (n + 2) (3n + 1)
y Example 88. Find the sum of the series
3 3
y Example 90. Find the sum of the series
13 1 + 2 13 + 2 3 + 3 3 1 × 2 × 3 + 2 × 3 × 4 + 3 × 4 × 5 + ... upto n terms.
+ + + ... upto n terms.
1 1+ 3 1+ 3+ 5 Sol. Here, Tn = {nth term of 1, 2, 3, ...}
Sol. Let Tn be the nth term of the given series. Then, ´ {nth term of 2, 3, 4, ...} ´ {nth term of 3, 4, 5, ...}
2
ì n ( n + 1) ü \ Tn = n (n + 1) (n + 2) = n 3 + 3n 2 + 2n
í ý \ Sn = Sum of n terms of the series
(13 + 23 + 33 + ... + n 3 )
=î þ
2
Tn =
(1 + 3 + 5 + ... + (2n - 1)) n (1 + 2n - 1) = STn = Sn 3 + 3Sn 2 + 2Sn
2 2
ì n ( n + 1) ü ì n (n + 1) (2n + 1)ü ì n ( n + 1) ü
( n + 1) 2 1 2 =í ý + 3í ý + 2í ý
= = (n + 2n + 1) î 2 þ î 6 þ î 2 þ
4 4
Let Sn denotes the sum of n terms of the given series. Then, n ( n + 1) ì n ( n + 1) ü
= í + (2n + 1) + 2ý
1 2 î 2 þ
Sn = STn = S(n 2 + 2n + 1) n ( n + 1) 2
4 = ( n + n + 4n + 2 + 4 )
1 2 4
= ( Sn + 2Sn + S1) n ( n + 1) ( n + 2) ( n + 3)
4 =
1 ì n (n + 1) (2n + 1) 2n (n + 1) ü 4
= í + + ný
4î 6 2 þ y Example 91. Find sum to n terms of the series
n 1 + (2 + 3) + (4 + 5 + 6 ) + ... .
= {2n 2 + 3n + 1 + 6n + 6 + 6}
24
Sol. Now, number of terms in first bracket is 1, in the second
n (2n 2 + 9n + 13) bracket is 2, in the third bracket is 3, etc. Therefore, the
Hence, Sn =
24 number of terms in the nth bracket will be n.
Chap 03 Sequences and Series 251

Let the sum of the given series of n terms = S y Example 93. Find the nth term and sum of n terms
n (n +1) of the series, 1 + 5 + 12 + 22 + 35 + ... .
\Number of terms in S = 1 + 2 + 3 + ... + n =
2 Sol. The sequence of differences between successive terms is 4,
Also, the first term of S is 1 and common difference is also 1. 7, 10, 13,... . Clearly, it is an AP with common difference
3. So, let the nth term of the given series be Tn and sum
ì n ( n + 1) ü
í ý of n terms be Sn .
é æ n ( n + 1) ö ù
S=î
2 þ
\ ê2 × 1 + çè - 1÷ × 1ú Then, Sn = 1 + 5 + 12 + 22 + 35 + ... + Tn - 1 + Tn …(i)
2 ë 2 ø û
Sn = 1 + 5 + 12 + 22 + ... + Tn - 1 + Tn …(ii)
n ( n + 1)
= ( 4 + n 2 + n - 2) Subtracting Eq. (ii) from Eq. (i), we get
8 0 = 1 + 4 + 7 + 10 + 13 + ... + (Tn - Tn - 1 ) - Tn
n ( n + 1) ( n 2 + n + 2) Þ Tn = 1 + 4 + 7 + 10 + 13 + ... n terms
=
8 n 1
= {2 × 1 + (n - 1) 3} = (3n 2 - n )
2 2
y Example 92. Find the sum of the series 3 2 1
Hence, Tn = n - n
1 × n + 2 × (n - 1) + 3 × (n - 2 ) + 4 × (n - 3 ) + ... + (n - 1) × 2 + n × 1 2 2
3 1
also, find the coefficient of x n - 1 in the expansion of \ Sum of n terms Sn = STn = Sn 2 - Sn
2 2
(1 + 2 x + 3 x 2 + ... + nx n - 1 ) 2 . 3 æ n (n + 1) (2n + 1) ö 1 æ n (n + 1) ö
= ç ÷- ç ÷
2è 6 ø 2è 2 ø
Sol. The r th term of the given series is
n ( n + 1)
Tr = r × (n - r + 1) = (n + 1) r - r 2 = (2n + 1 - 1)
4
\Sum of the series 1 1
= n 2 ( n + 1) = ( n 3 + n 2 )
n n n 2 2
Sn = å T r = ( n + 1 ) å r - å r 2 = ( n + 1 ) Sn - Sn 2
r =1 r =1 r =1 y Example 94. Find the nth term and sum of n terms
n (n + 1) n (n + 1) (2n + 1) of the series, 1 + 3 + 7 + 15 + 31 + ... .
= ( n + 1) -
2 6 Sol. The sequence of differences between successive terms is 2,
n ( n + 1) n ( n + 1) ( n + 2) 4, 8, 16, ... . Clearly, it is a GP with common ratio 2. So, let
= (3n + 3 - 2n - 1) =
6 6 the nth term and sum of the series upto n terms of the
Now, series be Tn and Sn , respectively. Then,
(1 + 2x + 3x 2 + ... + nx n - 1 )2 = (1 + 2x + 3x 2 + ... + nx n - 1 ) Sn = 1 + 3 + 7 + 15 + 31 + ... + Tn - 1 + Tn …(i)
´ (1 + 2x + 3x 2 + ... + nx n - 1 ) Sn = 1 + 3 + 7 + 15 + ... + Tn - 1 + Tn …(ii)
\Coefficient of x n - 1 in (1 + 2x + 3x 2 + ... + nx n - 1 )2 Subtracting Eq. (ii) from Eq. (i), we get
0 = 1 + 2 + 4 + 8 + 16 + ... + (Tn - Tn - 1 ) - Tn
= 1 × n + 2 × (n - 1) + 3 × (n - 2) + ... + n × 1 Þ Tn = 1 + 2 + 4 + 8 + 16 + ... upto n terms
n ( n + 1) ( n + 2) 1 × (2n - 1)
= Sn = =
6 2-1
Hence, Tn = (2n - 1)
Method of Differences \ Sum of n terms Sn = STn = S(2n - 1) = S2n - S1
If the differences of the successive terms of a series are in = (2 + 22 + 23 + ... + 2n ) - n
AP or GP, we can find the nth term of the series by the 2 × (2n - 1)
following steps. = - n = 2n + 1 - 2 - n
( 2 - 1)
Step I Denote the nth term and the sum of the series
upto n terms of the series by Tn and S n , y Example 95. Find the nth term of the series
respectively. 1 + 4 + 10 + 20 + 35 + ...
Step II Rewrite the given series with each term shifted
by one place to the right. Sol. The sequence of first consecutive differences is 3, 6, 10,
Step III Then, subtract the second expression of S n from 15, ... and second consecutive differences is 3, 4, 5, ... .
the first expression to obtain Tn . Clearly, it is an AP with common difference 1. So, let the
nth term and sum of the series upto n terms of the series
be Tn and Sn , respectively.
252 Textbook of Algebra

Then,
Sn = 1 + 4 + 10 + 20 + 35 + ... + Tn - 1 + Tn …(i)
Method of Differences (Shortcut)
Sn = 1 + 4 + 10 + 20 + ... + Tn - 1 + Tn …(ii) to find nth term of a Series
Subtracting Eq. (ii) from Eq. (i), we get The nth term of the series can be written directly on the
0 = 1 + 3 + 6 + 10 + 15 + ... + (Tn - Tn - 1 ) - Tn basis of successively differences, we use the following steps
to find the nth termTn of the given sequence.
Þ Tn = 1 + 3 + 6 + 10 + 15 + ... upto n terms
Step I If the first consecutive differences of the given
or Tn = 1 + 3 + 6 + 10 + 15 + ... + t n - 1 + t n …(iii)
sequence are in AP, then take
Tn = 1 + 3 + 6 + 10 + ... + t n - 1 + t n …(iv)
Tn = a (n - 1) (n - 2 ) + b (n - 1) + c , where a, b, c
Now, subtracting Eq. (iv) from Eq. (iii), we get
are constants. Determine a, b, c by putting
0 = 1 + 2 + 3 + 4 + 5 + ... + (t n - t n - 1 ) - t n
n = 1, 2, 3 and putting the values of T1 , T2 , T 3 .
or t n = 1 + 2 + 3 + 4 + 5 + ... upto n terms
Step II If the first consecutive differences of the given
n ( n + 1)
= Sn = sequence are in GP, then take
2
1 Tn = ar n - 1 + bn + c , where a, b, c are constants
\ T n = St n = ( Sn 2 + Sn )
2 and r is the common ratio of GP. Determine
1 æ n (n + 1) (2n + 1) n (n + 1) ö a, b, c by putting n = 1, 2, 3 and putting the values
= ç + ÷
2è 6 2 ø of T1 , T2 , T 3 .
1 n ( n + 1) 1 Step III If the differences of the differences computed in
= × (2n + 1 + 3) = n (n + 1) (n + 2)
2 6 6 Step I are in AP, then take
Tn = a (n - 1) (n - 2 ) (n - 3 ) + b (n - 1) (n - 2 )
y Example 96. Find the nth term of the series + c (n - 1) + d , where a, b, c , d are
1 + 5 + 18 + 58 + 179 + ... constants.Determine by putting n = 1, 2, 3, 4 and
Sol. The sequence of first consecutive differences is 4, 13, 40, putting the values of T1 , T2 , T 3 , T 4 .
121, ... and second consecutive differences is 9, 27, 81, ... . Step IV If the differences of the differences computed in
Clearly, it is a GP with common ratio 3. So, let the nth Step I are in GP with common ratio r, then take
term and sum of the series upto n terms of the series be
Tn = ar n - 1 + bn 2 + cn + d , where a, b, c , d are
Tn and Sn , respectively. Then,
Sn = 1 + 5 + 18 + 58 + 179 + ... + Tn - 1 + Tn …(i) constants. Determine by putting n = 1, 2, 3, 4 and
Sn = 1 + 5 + 18 + 58 + ... + Tn - 1 + Tn …(ii) putting the values of T1 , T2 , T 3 , T 4 .
Subtracting Eq. (ii) from Eq. (i), we get y Example 97. Find the nth term and sum of n terms of
0 = 1 + 4 + 13 + 40 + 121 + ... + (Tn - Tn - 1 ) - Tn the series 2 + 4 + 7 + 11 + 16 + ... .
Þ Tn = 1 + 4 + 13 + 40 + 121 + ... upto n terms Sol. The sequence of first consecutive differences is 2, 3, 4, 5,
or Tn = 1 + 4 + 13 + 40 + 121 + ... + t n - 1 + t n …(iii) ... . Clearly, it is an AP.
Then, nth term of the given series be
Tn = 1 + 4 + 13 + 40 + ... + t n - 1 + t n …(iv)
Tn = a ( n - 1 ) ( n - 2 ) + b ( n - 1 ) + c …(i)
Now, subtracting Eq. (iv) from Eq. (iii), we get
Putting n = 1, 2, 3, we get
0 = 1 + 3 + 9 + 27 + 81 + ... + (t n - t n - 1 ) - t n
2 = c Þ 4 = b + c Þ 7 = 2a + 2b + c
or t n = 1 + 3 + 9 + 27 + 81 + ... upto n terms 1
After solving, we get a = , b = 2, c = 2
1 × (3n - 1) 1 n 2
= = ( 3 - 1)
( 3 - 1) 2 Putting the values of a, b, c in Eq. (i), we get
1 1 1
\ T n = St n = ( S3n - S1) Tn = ( n - 1 ) ( n - 2 ) + 2 ( n - 1 ) + 2 = ( n 2 + n + 2 )
2 2 2
1 2
1 Hence, sum of series Sn = STn = ( Sn + Sn + 2S1)
= {(3 + 32 + 33 + ... + 3n ) - n } 2
2
1 æ n (n + 1) (2n + 1) n (n + 1) ö
1 ì 3 (3n - 1) ü = ç + + 2n ÷
= í - ný 2è 6 2 ø
2 î ( 3 - 1) þ 1
= n (n 2 + 3n + 8)
3 n 1 6
= ( 3 - 1) - n
4 2
Chap 03 Sequences and Series 253

y Example 98. Find the nth term and sum of n terms Putting n = 1, 2, 3, 4, we get
of the series 5 + 7 + 13 + 31 + 85 + ... . 2=a+b+c +d …(ii)
5 = 3a + 4b + 2c + d …(iii)
Sol. The sequence of first consecutive differences is 2, 6, 18,
54, ... . Clearly, it is a GP with common ratio 3. Then, nth 12 = 9a + 9b + 3c + d …(iv)
term of the given series be 31 = 27a + 16b + 4c + d …(v)
Tn = a (3)n - 1 + bn + c …(i) After, solving these equations, we get
Putting n = 1, 2, 3, we get a = 1, b = 0, c = 1, d = 0
5=a+b+c …(ii) Putting the values of a, b, c , d in Eq. (i), we get
7 = 3a + 2b + c …(iii) Tn = 3n - 1 + n
13 = 9a + 3b + c …(iv)
Solving these equations, we get
a = 1, b = 0, c = 4 Method of Differences
Putting the values of a, b, c in Eq. (i), we get (Maha Shortcut)
Tn = 3n - 1 + 4
To find t 1 + t 2 + t 3 + ... + t n - 1 + t n
Hence, sum of the series
Let S n = t 1 + t 2 + t 3 + ... + t n - 1 + t n
Sn = STn = S(3n - 1 + 4 ) = S(3n - 1 ) + 4 S1
Then, Dt 1 , Dt 2 , Dt 3 ,..., Dt n - 1 [1st order differences]
= (1 + 3 + 32 + ... + 3n - 1 ) + 4n 2 2 2 2
D t 1 , D t 2 , D t 3 ,..., D t n - 1 [2nd order differences]
(3n - 1) 1
= 1× + 4n = (3n + 8n - 1) M M M
( 3 - 1) 2
n -1 n -1 n -1
\ tn = C 0 t1 + C 1 Dt 1 + C 2 D2 t 1 +...
y Example 99. Find the nth term of the series n -1
1 + 2 + 5 + 12 + 25 + 46 + ... . + C r - 1 Dr - 1 t 1

Sol. The sequence of first consecutive differences is 1, 3, 7, 13, and S n = n C 1 t 1 + n C 2 Dt 1 + n C 3 D2 t 1 + ... + n C r Dr t 1


21, ... . The sequence of the second consecutive differences
is 2, 4, 6, 8, ... . Clearly, it is an AP. Then, nth term of the where, Dt 1 = t 2 - t 1 , Dt 2 = t 3 - t 2 , etc.
given series be D2t 1 = Dt 2 - Dt 1 , D3 t 1 = D2t 2 - D2t 1 , etc.
Tn = a ( n - 1 ) ( n - 2 ) ( n - 3 ) + b ( n - 1 ) ( n - 2 )
+ c (n - 1) + d …(i) y Example 101. Find the n th term and sum to n terms
Putting n = 1, 2, 3, 4, we get of the series 12 + 40 + 90 + 168 + 280 + 432 + ... .
1=d …(ii) Sol. Let Sn = 12 + 40 + 90 + 168 + 280 + 432 + ..., then
2=c +d …(iii) 1st order differences are 28, 50, 78, 112, 152, ...
5 = 2b + 2c + d …(iv) (i.e. Dt 1, Dt 2 , Dt 3 , ...)
12 = 6a + 6b + 3c + d …(v) and 2nd order differences are
After, solving these equations, we get 22, 28, 34, 40, ... (i.e. D2t 1, D2t 2 , D2t 3 , ...)
1
a = , b = 1, c = 1, d = 1 and 3rd order differences are
3
6, 6, 6, 6,... (i.e. D3t 1, D3t 2 , D3t 3 , ...)
Putting the values of a, b, c , d in Eq. (i), we get
1 and 4th order differences are
Tn = (n 3 - 6n 2 + 11n - 6) + (n 2 - 3n + 2) + (n - 1) + 1 0, 0, 0, 0, ... (i.e. D4t 1, D4t 2 , D4t 3 , ...)
3
1 n
= (n 3 - 3n 2 + 5n ) = (n 2 - 3n + 5)
3 3 \ t n = 12 × n - 1C 0 + 28 × n - 1C 1 + 22 . n - 1C 2 + 6 × n - 1C 3
22(n - 1) (n - 2)
y Example 100. Find the nth term of the series = 12 + 28 (n - 1) +
2
2 + 5 + 12 + 31 + 86 + ... . 6 ( n - 1) ( n - 2) ( n - 3)
+
Sol. The sequence of first consecutive differences is 3, 7, 19, 1 ×2 ×3
55, ... . The sequence of the second consecutive differences = n 3 + 5n 2 + 6n
is 4, 12, 36, ... . Clearly, it is a GP with common ratio 3.
and Sn = 12 × n C 1 + 28 × n C 2 + 22 × n C 3 + 6 × n C 4
Then, nth term of the given series be
Tn = a (3)n - 1 + bn 2 + cn + d …(i)
254 Textbook of Algebra

28n (n - 1) 22n (n - 1) (n - 2) Corollary II


= 12n + +
2 1 ×2 ×3 1
6× n ( n - 1) ( n - 2) ( n - 3) (i) 1 × 2 + 2 × 3 + ... + n (n + 1) = {n (n + 1)
+ 3
n 1 ×2 ×3 × 4 n (n + 1) (n + 2 )
2
= (n + 1) (3n + 23n + 46) (n + 2 ) - 0 × 1 × 2 } =
12 3
(ii) 1 × 3 × 5 × 7 + 3 × 5 × 7 × 9 + ... + (2n - 1) × (2n + 1) (2n + 5 )
Vn Method 1
×(2n + 3 ) ×
= {(2n - 1) (2n + 1) (2n + 3 ) (2n + 5 ) (2n + 7 )
To find the sum of the series of the forms 11 - ( -1) × 1 × 3 × 5 × 7 }
I. a 1 a 2 ... a r + a 2 a 3 ... a r + 1 + ... + a n a n + 1 ... a n + r - 1
1
1 1 1 = {(2n - 1) (2n + 1) (2n + 3 ) (2n + 5 ) (2n + 7 ) + 105 }
II. + + ... + 11
a 1 a 2 ... a r a 2 a 3 ... a r + 1 a n a n + 1 ... a n + r - 1
Solution of form II Let d be the common difference of
where, a 1 , a 2 , a 3 , ..., a n , ... are in AP. AP, then a n = a 1 + (n - 1) d
Solution of form I Let S n be the sum and Tn be the nth Let sum of the series and nth term are denoted by S n and
term of the series, then Tn , respectively. Then,
S n = a 1 a 2 ... a r + a 2 a 3 ... a r + 1 + ... 1 1 1
Sn = + + ... +
+ a n a n + 1 + ... + a n + r - 1 a 1 a 2 ... a r a 2 a 3 ... a r + 1 a n a n + 1 ... a n + r - 1
\ Tn = a n a n + 1 a n + 2 ... a n + r - 2 a n + r - 1 …(i) 1
\ Tn = …(i)
Let Vn = a n a n + 1 a n + 2 ... a n + r - 2 a n + r - 1 a n + r a n a n + 1a n + 2 ... a n + r - 2 a n + r - 1
[taking one extra factor in Tn for Vn ]
1
\ Vn - 1 = a n - 1 a n a n + 1 ... a n + r - 3 a n + r - 2 a n + r - 1 Let Vn = …(ii)
an + 1 an + 2 ... a n + r - 2 a n + r - 1
Þ Vn - Vn - 1 = a n a n + 1 a n + 2 ... a n + r - 1 (a n + r - a n - 1 )
= Tn (a n + r - a n - 1 ) [from Eq. (i)] …(ii) [leaving first factor from denominator of Tn ]
Let d be the common difference of AP, then 1
So, Vn - 1 =
a n = a 1 + (n - 1) d a n a n + 1 ... a n + r - 3 a n + r - 2
Then, from Eq. (ii) 1
Þ Vn - Vn - 1 =
Vn - Vn - 1 = Tn [{a 1 + (n + r - 1) d } an + 1 an + 2 ... a n + r - 2 a n + r - 1
- {a 1 + (n - 2 ) d }] = (r + 1) d Tn 1
1 -
Þ Tn = (Vn - Vn - 1 ) a n a n + 1 ... a n + r - 3 a n + r - 2
(r + 1) d
an - an + r - 1
n
1 n =
\ S n = ST n = å T n = å ( Vn - Vn - 1 ) anan + 1 an + 2 - an + r - 2 an + r - 1
n =1 (r + 1) d n =1
= Tn (a n - a n + r - 1 ) [from Eq. (i)]
1
= (Vn - V 0 ) = Tn [{a 1 + (n - 1) d } - {a 1 + (n + r - 2 ) d }]
(r + 1) d
= d (1 - r ) Tn
[from important Theorem 1 of S]
1 (Vn - Vn - 1 )
= (a n a n + 1 ... a n + r - a 0 a 1 ... a r ) \ Tn =
(r + 1) (a 2 - a 1 ) d (1 - r )
n (Vn - Vn - 1 ) 1
Corollary I If a 1 , a 2 , a 3 , ..., a n , ... are in AP, then
1
\ S n = ST n = å d (1 - r )
=
d (1 - r )
(Vn - V 0 )
n =1
(i) For r = 2, a 1 a 2 + a 2 a 3 + ... + a n a n + 1 =
3 (a 2 - a 1 ) [from important Theorem 1 of S]
(a n a n + 1 a n + 2 - a 0 a 1 a 2 ) ì
1 1
(ii) For r = 3, = í
1 (a 2 - a 1 ) (1 - r ) îa n + 1 a n + 2 ... a n + r - 2 a n + r - 1
a 1 a 2 a 3 + a 2 a 3 a 4 + ... + a n a n + 1 a n + 2 =
4 (a 2 - a 1 ) 1 ü
- ý
(a n a n + 1 a n + 2 a n + 3 - a 0 a 1 a 2 a 3 ) a 1 a 2 ... a r - 2 a r - 1 þ
Chap 03 Sequences and Series 255

1 Tn = (3n - 2) (3n + 1) (3n + 4 ) (3n + 7 ) …(i)


Hence, the sum of n terms is S n =
(r - 1) (a 2 - a 1 ) \Vn = (3n - 2) (3n + 1) (3n + 4 ) (3n + 7 ) (3n + 10)
ì 1 1 ü Vn - 1 = (3n - 5) (3n - 2) (3n + 1) (3n + 4 ) (3n + 7 )
í - ý
îa 1 a 2 ... a r - 1 a n + 1 a n + 2 ... a n + r - 1 þ Þ Vn = ( 3n + 10) Tn [from Eq. (i)]
Corollary I If a 1 , a 2 , a 3 , ..., a n , ... are in AP, then and Vn - 1 = ( 3n - 5) Tn
(i) For r = 2, \ Vn - Vn - 1 = 15 Tn
1 1 1 1 1
+ + ... + = \ Tn = ( Vn - Vn - 1 ))
a 1a 2 a 2a 3 a n a n + 1 (a 2 - a 1 ) 15
ì1 1 ü 1 n + 1 - a1 ö
æ a n
1
í - ý= ç ÷ \ Sn = STn = å 15 (Vn - Vn - 1 )
îa 1 a n + 1 þ d è a 1 a n + 1 ø n =1

1 æ a 1 + nd - a 1 ö n =
1
(Vn - V0 )
= ç ÷=
d è a 1a n + 1 ø a 1 a n + 1 15
[from important Theorem 1 of S]
1 1 1
(ii) For r = 3, + + ... + 1
a 1a 2a 3 a 2a 3a 4 an an + 1 an + 2 = {(3n - 2) (3n + 1) (3n + 4 ) (3n + 7 ) (3n + 10)
15
1 ì 1 1 ü - ( -2)(1)( 4 )(7 )(10)}
= í - ý
2 (a 2 - a 1 ) îa 1 a 2 a n + 1 a n + 2 þ 1
= {(3n - 2)(3n + 1)(3n + 4 )(3n + 7 )(3n + 10) + 560}
15
(iii) For r = 4,
Shortcut Method
1 1 1
+ + ... + 1
a 1a 2a 3a 4 a 2a 3a 4a 5 a n a n + 1a n + 2a n + 3 Sn =
(last factor of III term - first factor of I term)
1 ì 1 1 ü (Taking one extra factor in Tn in last
= í - ý
3 (a 2 - a 1 ) îa 1 a 2 a 3 a n + 1 a n + 2 a n + 3 þ - Taking one extra factor in I term in start)
1
= {(3n - 2)(3n + 1)(3n + 4 )(3n + 7 )(3n + 10)
Corollary II (16 - 1)
1 1 1 1 n - ( -2) × 1 × 4 × 7 × 10}
(i) + + + ... + = 1
1× 2 2 × 3 3 × 4 n (n + 1) n + 1 = {(3n - 2)(3n + 1)(3n + 4 )(3n + 7 )(3n + 10) + 560}
15
1 1 1 1
(ii) + + + ... +
1× 2 × 3 2 × 3 × 4 3 × 4 × 5 n (n + 1) (n + 2 ) y Example 103. Find the sum to n terms of the series
1ì 1 1 ü 1 1 1 1 1
= í - ý= -
+ + + ... .
2 î1 × 2 (n + 1) (n + 2 ) þ 4 2 (n + 1) (n + 2 ) 1 × 3 × 5 × 7 × 9 3 × 5 × 7 × 9 × 11 5 × 7 × 9 × 11 × 13
Also, find the sum to infinity terms.
1 1
(iii) + Sol. Let Tn be the nth term of the given series.
1× 3 × 5 × 7 3 × 5 × 7 × 9
1
1 Then, Tn = …(i)
+ ... + (2n - 1)(2n + 1)(2n + 3)(2n + 5)(2n + 7 )
(2n - 1) (2n + 1) (2n + 3 ) (2n + 5 )
1
\ Vn =
(2n + 1)(2n + 3)(2n + 5)(2n + 7 )
1ì 1 1 ü
= í - ý [leaving first factor from denominator of Tn ]
6 î1 × 3 × 5 (2n + 1) (2n + 3 ) (2n + 5 ) þ
1
1 1 Vn - 1 =
= - (2n - 1)(2n + 1)(2n + 3)(2n + 5)
90 6 (2n + 1) (2n + 3 ) (2n + 5 )
1
Þ Vn - Vn - 1 =
y Example 102. Find the sum upto n terms of the (2n + 1)(2n + 3)(2n + 5)(2n + 7 )
series 1 × 4 × 7 × 10 + 4 × 7 × 10 × 13 + 7 × 10 × 13 × 16 + ... 1
-
Sol. Let Tn be the nth term of the given series. (2n - 1)(2n + 1)(2n + 3)(2n + 5)
(2n - 1) - (2n + 7 )
\Tn = (n th term of 1, 4, 7, ...(nth term of 4, 7, 10, ...) =
(2n - 1)(2n + 1)(2n + 3)(2n + 5)(2n + 7 )
(nth term of 7, 10, 13, ...) (nth term of 10, 13, 16, ...)
= - 8 Tn [from Eq. (i)]
256 Textbook of Algebra

1
\ Tn = -
8
( Vn - Vn - 1 ) Maha Shortcut Method
n
1 n 1 1
\ Sn = STn = å Tn = - å ( Vn - Vn - 1 ) = - ( Vn - V0 ) Taking outside the bracket
n =1 8n = 1 8 8
æ 1 1 1 ö
[from Important Theorem 1 of S] ç i.e. = = = ...÷ and in bracket leaving last
1 è 9 - 1 11 - 3 13 - 5 ø
= (V0 - Vn )
8 factor of denominator of first term - leaving first factor of
1ì 1 1 ü denominator of last term
= í - ý
8 î 1 × 3 × 5 × 7 (2n + 1)(2n + 3)(2n + 5)(2n + 7 )þ 1æ 1 1 ö
i.e., S n = ç - ÷
1 1 8 è 1 × 3 × 5 × 7 (2n + 1) (2n + 3 ) (2n + 5 ) (2n + 7 ) ø
= -
840 8 (2n + 1) (2n + 3) (2n + 5) (2n + 7 )
1æ 1 ö 1
1 1 1 1 \ S¥ = ç - 0÷ =
and S ¥ = - = - 0= 8 è1× 3 × 5 × 7 ø 840
840 ¥ 840 840
Shortcut Method n
n (n + 1) (n + 2) (n + 3)
1
+
1
+
1
+ ... y Example 104. If å Tr =
12
,
1 × 3 × 5 × 7 × 9 3 × 5 × 7 × 9 × 11 5 × 7 × 9 × 11 × 13 r =1
1
+ …(i) where Tr denotes the rth term of the series. Find
(2n - 1) (2n + 1) (2n + 3) (2n + 5) (2n + 7 ) n
1
Now, in each term in denominator lim å .
r = 1 Tr
n ®¥
9 - 1 = 11 - 3 = 13 - 5 = ... = (2n + 7 ) - (2n - 1) = 8
Then, Eq. (i) can be written as n n -1

1ì 9 - 1 11 - 3 13 - 5 Sol. We have, Tn = å Tr - å Tr
= í + + + ... r =1 r =1
8 î 1 × 3 × 5 × 7 × 9 3 × 5 × 7 × 9 × 11 5 × 7 × 9 × 11 × 13
n ( n + 1) ( n + 2) ( n + 3) ( n - 1) n ( n + 1) ( n + 2)
(2n + 7 ) - (2n - 1) ü = -
+ ý 12 12
(2n - 1) (2n + 1) (2n + 3) (2n + 5) (2n + 7 )þ n ( n + 1) ( n + 2)
= [(n + 3) - (n - 1)]
1ì 1 1 1 1 12
= í - + -
8 î 1 × 3 × 5 × 7 3 × 5 × 7 × 9 3 × 5 × 7 × 9 5 × 7 × 9 × 11 n ( n + 1) ( n + 2) 1 3
= =
1 1 3 Tn n ( n + 1 ) ( n + 2 )
+ - + ...
5 × 7 × 9 × 11 7 × 9 × 11 × 13 n
1 n
3
1 \ lim
n ®¥
åT = lim
n ®¥
å r ( r + 1) ( r + 2)
+ r =1 r r =1
(2n - 1) (2n + 1) (2n + 3) (2n + 5) n
1
-
1 ü
ý
= 3 lim
n ®¥
å r ( r + 1) ( r + 2)
(2n + 1) (2n + 3) (2n + 5) (2n + 7 )þ r =1

1ì 1 1 ü æ 1 1 1 1 ö
= - = 3 lim ç + + + ... + ÷
í ý n ® ¥ è 1 ×2 ×3 2 ×3 × 4 3 × 4 ×5 n ( n + 1) ( n + 2) ø
8 î 1 × 3 × 5 × 7 (2n + 1) (2n + 3) (2n + 5) (2n + 7 )þ
[middle terms are cancelled out] Maha Shortcut Method
1 1 1æ 1 1 ö
= - = Sn [say] = 3 lim ç - ÷
840 8 (2n + 1) (2n + 3) (2n + 5) (2n + 7 ) n ® ¥ 2 è 1 ×2 ( n + 1) ( n + 2) ø
1 1
\ Sum to infinity terms = S ¥ = -0= 3 æ1 ö 3
840 840 = ç - 0÷ =
2 è2 ø 4
Chap 03 Sequences and Series 257

#L Exercise for Session 6


1 3 7 15
1. The sum of the first n terms of the series + + + + ... is
2 4 8 16
(a) 2n - n - 1 (b) 1 - 2-n (c) n + 2- n - 1 (d) 2n - 1

2. 21/ 4 × 41/ 8 × 81/ 16 × 161/ 32 ... is equal to


3 5
(a) 1 (b) (c) 2 (d)
2 2

3. 1 + 3 + 7 + 15 + 31 + ... upto n terms equals


(a) 2n + 1 - n (b) 2n + 1 - n - 2 (c) 2n - n - 2 (d) None of these

4. 99th term of the series 2 + 7 + 14 + 23 + 34 + ... is


(a) 9998 (b) 9999 (c) 10000 (d) 100000

5. The sum of the series 1× 2 × 3 + 2 × 3 × 4 + 3 × 4 × 5 + ... upto n terms is


(a) n (n + 1) (n + 2) (b) (n + 1) (n + 2) (n + 3)
1 1
(c) n (n + 1) (n + 2) (n + 3) (d) (n + 1) (n + 2) (n + 3)
4 4
1 1 1 1
6. + + + ... + equals
1× 2 2 × 3 3 × 4 n (n + 1)
1 n
(a) (b)
n (n + 1) n+ 1
2n 2
(c) (d)
n+ 1 n (n + 1)
3 5 7
7. Sum of the n terms of the series + + 2 + ... is
12 12 + 22 1 + 22 + 33
2n 4n
(a) (b)
n+ 1 n+ 1
6n 9n
(c) (d)
n+ 1 n+ 1
1 1 1 1 1
8. If t n = (n + 2) (n + 3) for n = 1, 2, 3, ... , then + + + ... + equals
4 t1 t 2 t 3 t 2003

4006 4003
(a) (b)
3006 3007
4006 4006
(c) (d)
3008 3009
1 1 1
9. The value of + + + ... upto ¥ is
(1 + a ) (2 + a ) (2 + a ) (3 + a ) (3 + a ) (4 + a )
(where, a is constant)
1 2
(a) (b)
1+ a 1+ a
(c) ¥ (d) None of these
n
10. If f ( x ) is a function satisfying f ( x + y ) = f ( x ) f ( y ) for all x , y ÎN such that f (1) = 3 and å f ( x ) = 120. Then, the
x =1
value of n is
(a) 4 (b) 5 (c) 6 (d) None of these
Session 7
Application to Problems of Maxima
and Minima (Without Calculus)

Application to Problems \ yz = zx = xy = 4

of Maxima and Minima Hence, greatest value of (yz ) (zx ) ( xy ) is 4 × 4 × 4


i.e., greatest value of x 2y 2z 2 is 64.
(Without Calculus) Hence, greatest value of xyz is 8.
Suppose that a 1 , a 2 , a 3 , ..., a n are n positive variables and k
y Example 106. Find the greatest value of x 3y 4 , if
is constant, then
(i) If a 1 + a 2 + a 3 + . . . . . + a n = k (constant), the
2x + 3y = 7 and x ³ 0, y ³ 0.
value of a 1 a 2 a 3 . . . a n is greatest when Sol. To find the greatest value of x 3y 4 or
a 1 = a 2 = a 3 = . . . . = a n , so that the greatest ( x )( x )( x )(y )(y )(y )(y )
n Here, x repeats 3 times and y repeats 4 times.
ækö
value of a 1 a 2 a 3 . . . a n is ç ÷ . Given, 2x + 3y = 7,
èn ø
then multiplying and dividing coefficients of x and y by 3
Proof Q AM ³ GM and 4, respectively.
a 1 + a 2 + a 3 + ... + a n æ 2x ö æ 3y ö
\ ³ (a 1 a 2 a 3 ... a n ) 1 /n Rewrite 3ç ÷ + 4 ç ÷ = 7
è 3 ø è4ø
n
k æ 2x ö æ 2x ö æ 2x ö æ 3y ö æ 3y ö æ 3y ö æ 3y ö
Þ ³ (a 1 a 2 a 3 ... a n ) 1 /n or ç ÷ + ç ÷ + ç ÷ + ç ÷ + ç ÷ + ç ÷ + ç ÷ = 7
è 3 ø è 3 ø è 3 ø è4ø è4ø è4ø è4ø
n 1 2 3 4 5 6 7
n
æk ö Here, k = 7 and n = 7
or (a 1 a 2 a 3 ... a n ) £ ç ÷
ènø Hence, greatest value of
Here, a 1 = a 2 = a 3 = ... = a n 7
n æ 2x ö æ 2x ö æ 2x ö æ 3y ö æ 3y ö æ 3y ö æ 3y ö æ 7 ö
æk ö ç ÷ ç ÷ ç ÷ ç ÷ ç ÷ ç ÷ ç ÷ is ç ÷ .
\ Greatest value of a 1 a 2 a 3 ... a n is ç ÷ . è 3 ø è 3 ø è 3 ø è 4 ø è 4 ø è 4 ø è 4 ø è7 ø
ènø
23 × 34
or greatest value of x 3y 4 is 1.
y Example 105. Find the greatest value of xyz for 33 × 4 4
positive values of x , y , z subject to the condition 32
Thus, greatest value of x 3y 4 is .
yz + zx + xy = 12. 3
Sol. Given, yz + zx + xy = 12 (constant), the value of (ii) If a 1 a 2 a 3 . . . a n = k (constant), the value of
(yz ) (zx ) ( xy ) is greatest when yz = zx = xy a 1 + a 2 + a 3 + . . . + a n is least when
Here, n = 3 and k = 12 a 1 = a 2 = a 3 = . . . = a n , so that the least of
3
æ 12 ö a 1 + a 2 + a 3 + . . . + a n is n (k ) 1 /n .
Hence, greatest value of (yz )(zx )( xy ) is ç ÷ i.e. 64.
è3ø
Proof Q AM ³ GM
\ Greatest value of x 2y 2z 2 is 64. a + a 2 + a 3 + ... + a n
\ 1 ³ (a 1 a 2 a 3 ... a n ) 1 /n = (k )1 /n
Thus, greatest value of xyz is 8. n
Aliter a 1 + a 2 + a 3 + ... + a n
Þ ³ (k ) 1 /n
Given yz + zx + xy = 12, the greatest value of (yz )(zx )( xy ) n
is greatest when
or a 1 + a 2 + a 3 + ... + a n ³ n (k ) 1 /n
yz = zx = xy = c [say]
Since, yz + zx + xy = 12
Here, a 1 = a 2 = a 3 = ... = a n
\ c + c + c = 12 \ Least value of a 1 + a 2 + a 3 + ... + a n is n (k ) 1 /n
Þ 3c = 12 or c = 4
Chap 03 Sequences and Series 259

y Example 107. Find the least value of 3x + 4 y for y Example 109. If a, b , c be positive real numbers,
positive values of x and y, subject to the condition a b c 3
x 2 y 3 = 6. prove that + + ³ .
b + c c + a a+b 2
Sol. Given, x 2y 3 = 6
Sol. Arithmetic mean of ( -1) th powers
or ( x ) ( x ) (y ) (y ) (y ) = 6
³ ( - 1) th power of arithmetic mean
Here, x repeats 2 times and y repeats 3 times -1 -1 -1
æ b+c ö æ c +a ö æ a+b ö
æ 3x ö æ 4y ö ç ÷ +ç ÷ +ç ÷
\ 3x + 4y = 2 ç ÷ + 3 ç ÷ èa + b + c ø èa + b + c ø èa + b + c ø
è 2 ø è 3 ø
3
æ 3x ö æ 3x ö æ 4y ö æ 4y ö æ 4y ö -1
=ç ÷+ç ÷+ç ÷+ç ÷+ç ÷ æ b+c c +a a+b ö
è 2 ø è 2 ø è 3 ø è 3 ø è 3 ø ç + + ÷
a +b+c a+b+c a+b+c
1 2 3 4 5 ³ç ÷
ç 3 ÷
multiplying and dividing coefficient of x and y by 2 and 3 ç ÷
è ø
respectively and write x 2y 3 = 6 a +b +c a +b + c a + b + c
+ + -1
æ 3x ö æ 3x ö æ 4y ö æ 4y ö æ 4y ö 3
2
43 b+c c +a a+b æ2ö
Þç ÷ ç ÷ ç ÷ ç ÷ ç ÷ = 2 ´ 3 ´ 6 = 32 Þ ³ç ÷
è 2 ø è 2 ø è 3 ø è 3 ø è 3 ø 2 3 3 è3ø
a b c 9
Here, n = 5 and k = 32 Þ +1+ +1+ +1³
b+c c +a a+b 2
3x 3x 4y 4y 4y
Hence, least value of + + + + a b c 9
2 2 3 3 3 Þ + + ³ -3
b+c c +a a+b 2
= 5 (32)1/ 5 = 10 a b c 3
or + + ³
i.e. least value of 3x + 4y = 10 b+c c +a a+b 2

y Example 108. Find the minimum value of y Example 110. If a and b are positive and a + b = 1,
bcx + cay + abz , when xyz = abc . æ 1ö
2
æ 1ö
2
25
Sol. To find the minimum value of show that ç a + ÷ + çb + ÷ > .
è aø è bø 2
bcx + cay + abz , Sol. Since, AM of 2nd powers > 2nd power of AM
write, xyz = abc æ 1ö
2
æ 1ö
2 2
ça + ÷ + çb + ÷ æ 1 1ö
or (bcx ) (cay ) (abz ) = a 3b 3c 3 = k [constant] ç a + + b + ÷
è aø è bø a b
\ >ç ÷
Here, n =3 2 ç 2 ÷
Hence, minimum value of bcx + cay + abz = n (k )1/n è ø
1 -1 -1 2 1 -1 -1 2
= 3 (a 3b 3c 3 )1/ 3 = 3abc = (a + b + a + b ) = (1 + a + b ) [Qa + b = 1]
4 4
2 2
æ 1ö æ 1ö 1 -1 -1 2
\ ça + ÷ + çb + ÷ > ( 1 + a + b ) …(i)
An Important Result è aø è bø 2
-1 -1
a -1 + b -1 æ a + b ö æ1ö
If a i > 0, i = 1, 2, 3, ..., n which are not identical, then Again, >ç ÷ = ç ÷ =2
2 è 2 ø è2ø
m
a 1m + a 2m + ... + a nm æ a 1 + a 2 + ... + a n ö a -1 + b -1
(i) >ç ÷ ; If m < 0 or >2
n è n ø 2
or m > 1 Þ a -1 + b -1 > 4
a 1m + a 2m + ... + a nm æ a + a 2 + ... + a n ö
m
\ (1 + a -1 + b -1 ) > 5 or (1 + a -1 + b -1 )2 > 25
(ii) <ç 1 ÷ ; 1 25
n è n ø Þ ( 1 + a -1 + b -1 ) 2 > …(ii)
2 2
If 0 < m < 1
From Eqs. (i) and (ii), we get
2 2
Remark æ 1ö æ 1ö 25
ça + ÷ + çb + ÷ >
If a1 = a2 = .... = an , then use equal sign in inequalities. è aø è bø 2
260 Textbook of Algebra

#L Exercise for Session 7


1. The minimum value of 4x + 42 - x , x Î R is
(a) 0 (b) 2
(c) 4 (d) 8

2. If 0 < q < p, then the minimum value of sin3 q + cosec3 q + 2, is


(a) 0 (b) 2
(c) 4 (d) 8
a b c d
3. If a, b , c and d are four real numbers of the same sign, then the value of + + + lies in the interval
b c d a
(a) [2, ¥) (b) [3, ¥)
(c) (4, ¥) (d) [4, ¥)
p
4. If 0 < x < , then the minimum value of 2 (sin x + cos x + cosec 2x )3 is
2
(a) 27 (b) 13.5
(c) 6.75 (d) 0

5. If a + b + c = 3 and a > 0, b > 0, c > 0, then the greatest value of a 2b 3c 2 is


34 × 210 310 × 24
(a) 7
(b)
7 77
3 × 212
2
3 × 22
12
(c) (d)
77 77
a a a
6. If x + y + z = a and the minimum value of + + is 81l , then the value of l is
1 x y z
(a) (b) 1
2
1
(c) (d) 2
4
1
7. a, b , c are three positive numbers and abc 2 has the greatest value , then
64
1 1 1
(a) a = b = ,c = (b) a = b = c =
2 4 3
1 1 1
(c) a = b = , c = (d) a = b = c =
4 2 4
Shortcuts and Important Results to Remember
1 If Tn = An + B, i.e. nth term of an AP is a linear expression 18 If a, b, c are in GP, then a + b, 2 b, b + c are in HP.
in n, where A, B are constants, then coefficient of n i.e., A 19 If a, b, c are in AP, then l a , l b, lc are in GP, where
is the common difference. l > 0, l ¹ 1.
2 If S n = Cn 2 + Dn is the sum of n terms of an AP, where C 20 If - 1 < r < 1, then GP is said to be convergent, if r < - 1or
and D are constants, then common difference of AP is 2C r > 1, then GP is said to be divergent and if r = -1, then
i.e., 2 times the coefficient of n 2 . series is oscillating.
3 (i) d = Tn - Tn - 1 [n ³ 2 ] (ii) Tn = S n - S n - 1 [n ³ 2 ] 21 If a, b, c , d are in GP, then
(iii) d = S n - 2 S n - 1 + S n - 2 [n ³ 3] (a ± b)n , (b ± c )n , (c ± d )n are in GP, " n Î I
4 If for two different AP’s 22 If a, b, c are in AP as well as in GP, then a = b = c .
2
Sn An + Bn An + B 23 The equations a1 x + a2 y = a3 , a4 x + a5 y = a6 has a unique
= or
¢ Cn 2 + Cn Cn + D solution, if a1, a2 , a3 , a4 , a5 , a6 are in AP and common
Sn
difference ¹ 0.
Tn A (2 n - 1) + B
Then, = 24 For n positive quantities a1, a2 , a3 , ..., an
¢ C (2 n - 1) + D
Tn
AM ³ GM ³ HM
5 If for two different AP’s
æ n + 1ö sign of equality (AM = GM = HM) holds when quantities
Aç ÷+B
Tn An + B Sn è 2 ø are equal
= , then =
Tn¢ Cn + D S n¢ C æç n + 1ö÷ + D i.e. a1 = a2 = a3 = ... = an .
è 2 ø
25 For two positive numbers a and b (AM) (HM)
6 If Tp = q and Tq = p, then Tp + q = 0, Tr = p + q - r = (GM) 2 , the result will be true for n numbers, if they are in
7 If pTp = qTq of an AP, then Tp + q = 0 GP.
8 If S p = Sq for an AP, then S p + q = 0 26 If odd numbers of (say 2 n + 1) AM’s, GM’s and HM’s be
9 If S p = q and Sq = p of an AP, then S p + q = - ( p + q ) inserted between two numbers, then their middle means
1/( p - q )
[i.e., (n + 1) th mean] are in GP.
é Pn - q ù
10 If Tp = P and Tq = Q for a GP, then Tn = ê n - p ú 27 If a2 , b2 , c 2 are in AP.
ëQ û 1 1 1
11 If Tm + n = p, Tm - n = q for a GP, then Þ , , are in AP.
m / 2n
b+c c + a a+ b
æq ö
Tm = pq , Tn = p ç ÷ 28 Coefficient of x n - 1 and x n - 2 in
è pø
( x - a1 ) ( x - a2 ) ( x - a3 ) ... ( x - an )
12 If Tm = n, Tn = m for a HP, then
are - (a1 + a2 + a3 + ... + an ) and S a1a2 , respectively
mn mn
Tm + n = , Tmn = 1,Tp = (S a1 )2 - S a12
(m + n ) p where, S a1a2 = .
2
13 If Tp = qr , Tq = pr for a HP, then Tr = pq
29 1 + 3 + 5 + ... upto n terms = n 2
14 No term of HP can be zero and there is no formula to find
n (n + 1) (n + 2 )
S n for HP. 30 2 + 6 + 12 + 20 + K upto n terms =
a-b a a a 3
15 a, b, c are in AP, GP or HP as = or or .
b-c a b c n (n 2 + 2 )
31 1 + 3 + 7 + 13 + ... upto n terms =
3
16 If A, G, H be AM, GM and HM between a and b, then
n (n + 1)2 (n + 2 )
ì A, when n = 0 32 1 + 5 + 14 + 30 + K upto n terms =
an + 1 + bn + 1 ï 1 12
= íG, when n = -
an + bn ï 2 33 If a1, a2 , a3 , ..., an are the non-zero terms of a non-constant
î H, when n = - 1 AP, then
17 If A and G are the AM and GM between two numbers 1 1 1 1 (n - 1)
+ + + ... + =
a, b, then a, b are given by A ± ( A + G ) ( A - G ) a1a2 a2 a3 a3 a4 an - 1 an a1an
JEE Type Solved Examples :
Single Option Correct Type Questions
n This section contains 10 multiple choice examples. (1 - r 101 ) 1 125
Each example has four choices (a), (b), (c) and (d) out of = 100
= 100
´ [from Eq. (i)]
a1r (1 - r ) a1r a1
which ONLY ONE is correct.
125 125 125 1
= = = =
l (a1r ) 50 2
(a 51 ) 2
(25) 2
5
l Ex. 1 If b - c , 2b - l, b - a are in HP, then a - ,
2
l l l Ex. 3 If x =111 ...1 (20 digits), y = 333 ... 3 (10 digits) and
b- , c - are is
2 2 x -y 2
z = 222 ... 2 (10 digits), then equals
(a) AP (b) GP z
(c) HP (d) None of these 1
2(b - c )(b - a ) (a) (b) 1 (c) 2 (d) 4
Sol. (b) (2b - l ) = 2
(b - c ) + (b - a ) 1 1
2
Sol. (b) Q x = (999...9 ) = (1020 - 1),
Þ (2b - l ) = (2b - (a + c )) = 2 [b - (a + c )b + ac ] 9 9
2 1 1
Þ 2b - 2bl + l(a + c ) - 2ac = 0 y = (999...9 ) = (1010 - 1)
3 3
l
Þ b 2 - bl + (a + c ) - ac = 0 2 2
2 and z = (999...9 ) = (1010 - 1) ]
2
9 9
2
æ lö l l 1 20 1
Þ çb - ÷ - + (a + c ) - ac = 0 (10 - 1) - (1010 - 1)2
è 2 ø 4 2 x -y2 9 9
\ =
2 z 2 10
æ lö l2 l (10 - 1)
Þ çb - ÷ = - (a + c ) + ac 9
è 2ø 4 2
1010 + 1 - (1010 - 1)
æ lö
2
æ löæ lö = =1
Þ çb - ÷ = ça - ÷ çc - ÷ 2
è 2 ø è 2 ø è 2ø
l Ex. 4 Consider the sequence 1, 2, 2, 3, 3, 3, …, where n
l l l
Hence, a - , b - , c - are in GP.
2 2 2 occurs n times. The number that occurs as 2011th terms is
(a) 61 (b) 62
l Ex. 2 Let a 1 , a 2 , a 3 , ..., a 10 are in GP with a 51 = 25 and (c) 63 (d) 64
101 101
æ1ö Sol. (c) The last 4 occurs as 1 + 2 + 3 + 4 = 10th term. The last n
å a i = 125, then the value of å çè a i ÷ø equals æ n ( n + 1) ö
th
i =1 i =1 occurs as ç ÷ term, the last 62 occurs as
1 1 1 è 2 ø
(a)5 (b) (c) (d) th
5 25 125 æ 62 ´ 63 ö
ç ÷ = 1953 rd term and the last 63 occurs as
Sol. (b) Let 1st term be a and common ratio be r , then è 2 ø
101
1 th
å a = 125 æ 63 ´ 64 ö
ç ÷ = 2016 th term.
i =1 i è 2 ø
Þ (a1 + a1r + a1r 2 + ...+ a1r 100 ) = 125 \ 63 occurs from 1954th term to 2016th term.
a1(1 - r 101 ) Hence, (2011)th term is 63.
Þ = 125 [let 0 < r < 1] …(i)
(1 - r )
117
é æ 1 ö101 ù 1
1
ê ç ÷ - 1ú
l Ex. 5 Let S = å , when[ × ] denotes the greatest
101 ai êë è r ø úû 2[ r ] + 1
r =1
1 1 1 1 1
\ åa = + +
ai ai r ai r 2
+...+ 100 =
ai r æ1 ö
p
integer function and if S = , when p and q are co-primes,
i =1 i
ç - 1÷ q
èr ø
é 1 ù the value of p + q is
ê here r > 1ú (a) 20 (b) 76 (c) 19 (d) 69
ë û
Chap 3 Sequences and Series 263

117
1 Ex. 9 Let l be the greatest integer for which
å 2[
l
Sol. (b) Q S=
r =1 r ]+ 1 5 p 2 - 16, 2 pl, l2 are distinct consecutive terms of an AP,
=
3
+
5
+
7
+ ... +
19
+
18 where p ÎR . If the common difference of the AP is
2 ×1 + 1 2 ×2 + 1 2 ×3 + 1 2 × 9 + 1 2 × 10 + 1 æmö
ç ÷ , m, n Î N and m, n are relative prime, the value of m + n
18
=9+ =9+ =
6 69 ènø
21 7 7 is
\ p = 69 and q = 7 Þ p + q = 69 + 7 = 76 (a) 133 (b) 138 (c) 143 (d) 148
2 2
Sol. (c) Q5p - 16, 2pl, l are in AP, then
l Ex. 6 If a, b, c are non-zero real numbers, then the mini-
mum value of the expression 4 pl = 5p 2 - 16 + l2
(a 8 + 4a 4 + 1)(b 4 + 3b 2 + 1)(c 2 + 2c + 2 ) Þ 5p 2 - 4 pl + l2 - 16 = 0 …(i)
equals
a 4b 2 \ B - 4 AC ³ 0 [Q p Î R ]
(a) 12 (b) 24 (c) 30 (d) 60 Þ 16l2 - 4 × 5 × ( l2 - 16) ³ 0
(a 8 + 4a 4 + 1)(b 4 + 3b 2 + 1)(c 2 + 2c + 2)
Sol. (c) Let P = Þ - l2 + 80 ³ 0 or l2 ³ 80
a 4b 2
æ 1 öæ 1ö Þ - 80 £ l £ 80
= ça 4 + 4 + 4 ÷ çb 2 + 3 + 2 ÷ {(c + 1)2 + 1)} \ l =8 [greatest integer]
è a ø è b ø
1 1 From Eq. (i), 5p 2 - 32p + 48 = 0
Q = a4 + 4 + ³ 6, b 2 + 3 + ³ 5 and (c + 1)2 + 1 ³ 1
a 4
b2 Þ ( p - 4 )(5p - 12) = 0
é 1 ù
êQx + x ³ 2 for x > 0ú
12
\ p = 4, p =
ë û 5
\ P ³ 6 × 5 × 1 = 30 Þ P ³ 30
12
Hence, the required minimum value is 30. Þ p= ,p ¹4
5
[for p = 4 all terms are equal]
l Ex. 7 If the sum of m consecutive odd integers is m 4 , then
the first integer is Now, common difference = l2 - 2pl
12 æ 3 ö 128 m
(a) m 3 + m + 1 (b) m 3 + m - 1 = 64 - 16 ´ = 64 ç1 - ÷ = = [given]
5 è 5ø 5 n
(c) m 3 - m - 1 (d) m 3 - m + 1
\ m = 128 and n = 5
Sol. (d) Let 2a + 1, 2a + 3, 2a + 5, ... be the AP, then
4
m = (2a + 1) + (2a + 3) + (2a + 5) + ... upto m terms Hence, m + n = 143
m
={2(2a + 1) + (m - 1) ×2} = m(2a + 1 + m - 1) l Ex. 10 If 2l, l and [ l2 - 14 ], l Î R - {0 } and [.] denotes
2
the greatest integer function are the first three terms of a GP
Þ m 3 = (2a + 1) + m - 1 in order, then the 51th term of the sequence,
\ 2a + 1 = m 3 - m + 1 1, 3 l, 6 l, 10 l, . . . is
¥
( 4r + 5 ) 5 -r (a) 5104 (b) 5304
l Ex. 8 The value of å is (c) 5504 (d) 5704
r = 1 r (5r + 5 )
Sol. (b) Q2l, l, [ l2 - 14 ] are in GP, then
1 2 1 2
(a) (b) (c) (d)
5 5 25 125 l2 = 2l [ l2 - 14 ]
¥
( 4r + 5)5-r n
æ (5r + 5) - r ö 1 l
Sol. (a) å = lim å ç ÷× r Þ = [ l2 - 14 ]
r =1 r (5r + 5) r = 1 è r (5r + 5) ø 5
n ®¥ 2
n
æ1 1 ö1 \ l must be an even integer
= lim
n ®¥
å çè r - 5r + 5 ÷ø 5r Hence, l=4
r =1
n æ 1 ö Now, required sequence 1,12, 24, 40, ...
1
= lim
n ®¥
å çè r ×5-r -
(r + 1)5r +1
÷
ø or 1, 4(1 + 2), 4(1 + 2 + 3), 4(1 + 2 + 3 + 4 ), ...
r =1
\ 51th term = 4(1 + 2 + 3 +... + 51)
næ1 1 ö 1 1
= lim å ç - ÷ = -0 = 51
n ®¥
r = 1 è 5 (n + 1)5
n +1
ø 5 5 = 4 × (1 + 51) = 4 × 51 × 26 = 5304
2
264 Textbook of Algebra

JEE Type Solved Examples :


More than One Correct Option Type Questions
n This section contains 5 multiple choice examples. Each 3 1 1 1 l l
Þ Sn = + Sn + Sn - - n +n 2 - n +n 1
example has four choices (a), (b), (c) and (d) out of which 4 4 2 4 2 2
more than one may be correct. 1 1 ln ln l l
Þ Sn = - n + 2 - n + 1 Þ Sn = 2 - n n+ 1 - n n- 1 < 2
l Ex. 11 The first three terms of a sequence are 3, - 1, -1. 4 2 2 2 2 2
The next terms are
5 5 l Ex. 14 If Sr = r + r + r + ... ¥ , r > 0 then which the
(a) 2 (b) 3 (c) - (d) -
27 9 following is/are correct.
Sol. (b, d) The given sequence is not an AP or GP or HP. It is (a) Sr ,S 6 , S12 , S 20 are in AP
an AGP, 3, (3 + d )r ,(3 + 2d )r 2 ,... (b) S 4 ,S 9 , S16 are irrational
Þ (3 + d )r = -1, (3 + 2d )r 2 = -1 (c) ( 2S 4 - 1 )2 ,( 2S 5 - 1 )2 ( 2S 6 - 1 )2 are in AP
Eliminating r, we get (3 + d )2 = - (3 + 2d ) (d) S 2 ,S12 , S 56 are in GP
Þ 2
d + 8d + 12 = 0 Þ d = -2, - 6, Sol. (a, b, c, d)

1 QSr = r + r + + ... ¥ = r + Sr
then r = -1,
3 Þ
2
Sr - Sr - r = 0
5
\ Next term is (3 + 3d )r 3 = 3, - 1 + ( 1 + 4r )
9 \ Sr = [Qr > 0]
2
l Ex. 12 There are two numbers a and b whose product is Alternate (a) S 2 ,S 6 , S12 , S 20 i.e., 2, 3, 4, 5 are in AP.
192 and the quotient of AM by HM of their greatest common 1 + 17 1 + 37 1 + 65
Alternate (b) S 4 ,S 9 , S16 i.e., , , are
2 2 2
divisor and least common multiple is 169 . The smaller of a irrationals.
48
and b is Alternate (c)(2S 4 - 1 )2 , (2S 5 - 1 )2 , (2S 6 - 1 )2 i.e., 17, 21, 25 are in AP
(a) 2 (b) 4 (c) 6 (d) 12 Alternate (d) S 2 , S12 , S 56 i.e., 2, 4, 8 are in GP.
Sol. (b, d) If G = GED of a and b, L = LCM of a and b, we
have GL = ab = 192
1 1 1
…(i) l Ex. 15 If , , are in AP and a, b, -2c are in GP, where
AM æ G + L ö æ G + L ö 169
a b c
of G and L is ç ÷ç ÷= a, b, c are non-zero, then
HM è 2 ø è 2GL ø 48
(a) a 3 + b 3 + c 3 = 3abc (b) -2a,b , - 2c are in AP
1692 169
Þ (G + L ) = GL = ´ 192 = 132 × 4 2 (c) -2a,b , - 2c are in GP (d) a 2 ,b 2 , 4c 2 are in GP
12 12
Þ G + L = 52 but GL = 192 Sol. (a, b, d)
Þ G = 4, L = 48 Þ a = 4, b = 48 or a = 12, b = 16 Q
1 1 1
, , are in AP Þ a, b, c are in HP
a b c
1 1 2 3 5 ln 2ab
l Ex. 13 Consider a series + 2 + 3 + 4 + 5 +...+ n . \ b= …(i)
2 2 2 2 2 2 a +c
If Sn denotes its sum to n terms, then Sn cannot be and a, b, - 2c are in GP, then b 2 = -2ac …(ii)
(a) 2 (b) 3 (c) 4 (d) 5 From Eqs. (i) and (ii), we get
Sol. (a, b, c, d)
-b 2
1 1 2 3 5 l b= Þ a +b +c = 0 [Qb ¹ 0]
Q Sn = + 2 + 3 + 4 + 5 +... + nn a +c
2 2 2 2 2 2
\ a 3 + b 3 + c 3 = 3abc and a, b, -2c are in GP
3 1 æ1 1 2 3 5 l ö
= + ç + 2 + 3 + 4 + 5 +... + nn ÷ Þ a 2 , b 2 , 4c 2 are also in GP and a + b + c = 0
4 4 è2 2 2 2 2 2 ø
1 æ1 1 2 l ö 1 l l Þ 2b = -2a - 2c
+ ç + 2 + 3 + ... + nn ÷ - - n +n 2 - n n+ 1 \ -2a,b, - 2c are in AP.
2 è2 2 2 2 ø 4 2 2
Chap 3 Sequences and Series 265

JEE Type Solved Examples :


Passage Based Questions
n This section contains 3 solved passages based upon each Aliter By inspection, first common term to both the series
of the passage 3 multiple choice examples have to be is 23, second common term is 51, third common term is 79
answered. Each of these examples has four choices (a), (b), and so on. These numbers form an AP 23, 51, 79, ...
(c) and (d) out of which ONLY ONE is correct. Since, T14 = 23 + 13 (28) = 387 < 407
Passage I and T15 = 23 + 14 (28) = 415 > 407
(Ex. Nos. 16 to 18) Hence, number of common terms = 14
Consider a sequence whose sum to n terms is given by the 20. The 10th common term between the series 3 + 7 + 11 + ...
quadratic function S n = 3n 2 + 5n. and 1 + 6 + 11 + ... is
(a) 189 (b) 191 (c) 211 (d) 213
16. The nature of the given series is
(a) AP (b) GP (c) HP (d) AGP Sol. (b) Series 3 + 7 + 11 + ... has common difference = 4 and
2 series 1 + 6 + 11 + ... has common difference = 5
Sol. (a) Q Sn = 3n + 5n
Hence, the series with common terms has common
\ Tn = Sn - Sn - 1 difference LCM of 4 and 5 which is 20.
= (3n 2 + 5n ) - [3(n - 1)2 + 5(n - 1)] The first common terms is 11.
= 3(2n - 1) + 5 = 6n + 2 Hence, the series is 11 + 31 + 51 + 71 + ...
The nth term is a linear function in n. Hence, sequence \ t 10 = 11 + (10 - 1) (20) = 191
must be an AP.
Aliter t n for 3 + 7 + 11 + ... = 3 + (n - 1) ( 4 ) = 4n - 1
17. For the given sequence, the number 5456 is the and t m for 1 + 6 + 11 + ... = 1 + (m - 1) (5) = 5m - 4
(a) 153 th term (b) 932 th term For a common term, 4n - 1 = 5m - 4 i.e., 4n = 5m - 3
(c) 707 th term (d) 909 th term For m = 3, n = 3 gives the first common term i.e., 11.
Sol. (d) Given, Tn = 5456 For m = 7, n = 8 gives the second common term i.e., 31.
Þ 6n + 2 = 5456 Þ 6n = 5454 For m = 11, n = 13 gives the third common term i.e., 51.
\ n = 909 Hence, the common term series is 11 + 31 + 51 + ...
\ The number 5456 is the 909 th term. \ t 10 = 11 + (10 - 1) 20 = 191
18. Sum of the squares of the first 3 terms of the given
21. The value of largest term common to the sequences 1,
series is
11, 21, 31, ... upto 100 terms and 31, 36, 41, 46, ... upto
(a) 1100 (b) 660 (c) 799 (d) 1000
100 terms, is
Sol. (b) T12 + T 22 + T 32 = 82 + 14 2 + 202 = 64 + 196 + 400 = 660
(a) 281 (b) 381 (c) 471 (d) 521
Passage II Sol. (d) Sequence 1, 11, 21, 31, ... has common difference = 10
and sequence 31, 36, 41, 46, ... has common difference = 5.
(Ex. Nos. 19 to 21)
Hence, the sequence with common terms has common
Let r be the number of identical terms in the two AP’s. difference LCM of 10 and 5 which is 10.
Form the sequence of identical terms, it will be an AP, then The first common term is 31.
the rth term of this AP make t r £ the smaller of the last Hence, the sequence is 31, 41, 51, 61, 71, ... …(i)
term of the two AP’s. Now, t 100 of first sequence = 1 + (100 - 1) 10 = 991
19. The number of terms common to two AP’s 3, 7, 11, …, and t 100 of second sequence = 31 + (100 - 1) 5 = 526
407 and 2, 9, 16, …, 709 is Value of largest common term < 526
(a) 14 (b) 21 (c) 28 (d) 35 \t n of Eq. (i) is 31 + (n - 1) 10 = 10n + 21
Sol. (a) Sequence 3, 7, 11, ... , 407 has common difference = 4
t 50 = 10 ´ 50 + 21 = 521
and sequence 2, 9, 16, ... , 709 has common difference = 7.
is the value of largest common term.
Hence, the sequence with common terms has common
difference LCM of 4 and 7 which is 28. Aliter Let mth term of the first sequence be equal to the
nth term of the second sequence, then
The first common term is 23.
Hence, the sequence is 23, 51, 79, ... , 387 which has 14 terms. 1 + (m - 1) 10 = 31 + (n - 1) 5
266 Textbook of Algebra

Þ 10m - 9 = 5n + 26 Þ 10m - 35 = 5n Þ
x
+
y
+
z
³ 06
.
Þ 2m - 7 = n £ 100 Þ 2m £ 107 2- x 2-y 2-z
1 x y z
Þ m £ 53 Thus, minimum value of + + is 0.6.
2 2- x 2-y 2-z
\ Largest value of m = 53 n

\ Value of largest term = 1 + (53 - 1) 10 = 521


23. If å a i2 = l, " a i ³ 0 and if greatest and least values of
i =1
2
Passage III æ n ö
ç å a i ÷ are l1 and l 2 respectively, then ( l1 - l 2 ) is
(Ex. Nos. 22 to 24) ç ÷
èi = 1 ø
We are giving the concept of arithmetic mean of mth
(a) n l (b) (n - 1) l
power. Let a1 , a 2 , a 3 , ..., a n be n positive real numbers (not
(c) (n + 2) l (d) (n + 1) l
all equal) and m be a real number. Then,
m
Sol. (b) Q AM of 2nd powers ³ 2nd power of AM
a1m + am2 + am3 + ... + anm æ a1 + a 2 + a 3 + ... + an ö
>ç ÷ , a12 + a 22 + a 32 + ... + an2 æ a1 + a 2 + a 3 + ... + an ö
2
n è n ø \ ³ç ÷
n è n ø
if m Î R ~ [0, 1]
2
However, if m Î(0, 1), then æ n ö
ç å ai ÷ 2
a1m + am2 + am3 + ... + anm æ a1 + a 2 + a 3 + ... + an ö
m
l çi = 1 ÷ æ n ö
<ç Þ ³ç \ çç å ai ÷÷ £ n l …(i)
n è n
÷
ø n n ÷ èi = 1 ø
ç ÷
ç ÷
Obviously, if m = {0, 1}, then è ø
a1m + am2 + am3 + ... + anm æ a1 + a 2 + a 3 + ... + an ö
m Also, (a1 + a 2 + a 3 + ... + an )2 = a12 + a 22 + a 32 +
=ç ÷ . ... + an2 + 2 å a1 a 2
n è n ø
= l + 2 å a1a 2 ³ l
22. If x > 0, y > 0, z > 0 and x + y + z =1, the minimum 2
æ n ö
x y z \ ç å ai ÷ ³ l …(ii)
value of + + , is ç ÷
2 - x 2 -y 2 - z èi = 1 ø
(a) 0.2 (b) 0.4 From Eqs. (i) and (ii), we get
(c) 0.6 (d) 0.8 2
æ n ö
Sol. (c) Since, AM of ( -1) th powers ³ ( -1) th powers of AM l £ çç å ai ÷÷ £ n l
-1 èi = 1 ø
( 2 - x ) -1 + ( 2 - y ) -1 + ( 2 - z ) -1 æ 2 - x + 2 - y + 2 - z ö
\ ³ç ÷ \ l1 = n l and l 2 = l
3 è 3 ø
-1 -1 Then, l1 - l 2 = ( n - 1 ) l
é6 - (x + y + z )ù æ6 - 1ö 3
=ê ú =ç ÷ = [Q x + y + z = 1] 24. If sum of the mth powers of first n odd numbers is l,
ë 3 û è 3 ø 5
" m > 1, then
( 2 - x ) -1 + ( 2 - y ) -1 + ( 2 - z ) -1 3
Þ ³ (a) l < n m (b) l > n m (c) l < n m + 1 (d) l > n m + 1
3 5
Sol. (d) Qm > 1
1é 1 1 1 ù 3
or ê + + ú³
3 ë2 - x 2 - y 2 - z û 5 1m + 3m + 5m + ... + (2n - 1)m
\
1 1 1 9 n
Þ + + ³ æ 1 + 3 + 5 + ... + (2n - 1) ö
m
2- x 2-y 2-z 5 >ç ÷
è n ø
2 2 2 18
or + + ³ m
2- x 2-y 2-z 5 æn ö
ç (1 + 2n - 1) ÷
x y z 18 =ç 2 ÷ = nm
or 1+ +1+ +1+ ³ ç n ÷
2- x 2-y 2-z 5
è ø
x y z 18
or + + ³ -3 \ 1m + 3m + 5m + ... + (2n - 1)m > n m + 1
2- x 2-y 2-z 5
x y z 3 Hence, l > nm + 1
Hence, + + ³ = 06
.
2- x 2-y 2-z 5
Chap 3 Sequences and Series 267

JEE Type Solved Examples :


Single Integer Answer Type Questions
n This section contains 2 examples. The answer to each 3 ( 1 + A1 ) 3 ( 1 + A1 ) ( 3 + 2 A1 )
A1 > , >
example is a single digit integer ranging from 0 to 9 ( 3 + A1 ) ( 3 + A1 ) ( 2 + A1 )
(both inclusive).
Þ A12 > 3 or A1 > 3
l Ex. 25 A sequence of positive terms A1 , A 2 , A3 , ..., An \ A1 = 2 [least integral value of A1]
3 (1 + An )
satisfies the relation An + 1 = . Least integral value
(3 + An ) l Ex. 26 When the ninth term of an AP is divided by its
of A1 for which the sequence is decreasing can be second term we get 5 as the quotient, when the thirteenth
3 ( 1 + An ) 3 ( 1 + A1 ) term is divided by sixth term the quotient is 2 and the
Sol. (2) Q An + 1 = . For n = 1, A 2 =
( 3 + An ) ( 3 + A1 ) remainder is 5, then the second term is
3 (1 + A 2 ) Sol. (7) Let a be the first term and d be the common difference,
For n = 2, A 3 =
(3 + A 2 ) then T 9 = 5 T 2
æ 3 ( 1 + A1 ) ö Þ (a + 8d ) = 5 (a + d )
3 ç1 + ÷
è ( 3 + A1 ) ø 6 + 4 A1 3 + 2 A1 \ 4a = 3d …(i)
= = =
3 ( 1 + A1 ) 4 + 2 A1 2 + A1 and T13 = T 6 ´ 2 + 5
3+
( 3 + A1 ) Þ a + 12d = 2 (a + 5d ) + 5
Q Given, sequence can be written as Þ 2d = a + 5 …(ii)
3 ( 1 + A1 ) ( 3 + 2 A1 ) From Eqs. (i) and (ii), we get
A1, , , ...
( 3 + A1 ) ( 2 + A1 ) a = 3 and d = 4
Given, A1 > 0 and sequence is decreasing, then \ T2 = a + d = 7

JEE Type Solved Examples :


Matching Type Questions
n
This section contains 2 examples. Examples 27 has three (A)Qa1, a 2 , a 3 , ... are in AP.
statements (A, B and C) given in Column I and four \ a1 + a 30 = a 6 + a 25 = a10 + a 21 = l [say]
statements (p, q, r and s) in Column II example 28 has
Q a1 + a 6 + a10 + a 21 + a 25 + a 30 = 120
four statements (A, B, C and D) given in Column I and
five statements (p, q, r, s and t) in Column II. Any given \ 3l = 120
statement in Column I can have correct matching with Þ l = 40
one or more statement(s) given in Column II. 30
30
l Ex. 27
Then, åai = 2
(a1 + a 30 ) = 15 ´ l = 15 ´ 40 = 600
i =1

Column I Column II (B)Qa1, a 2 , a 3 , ... are in AP.


(A) If a1 , a2 , a3 , ... are in AP and a1 + a6 + a10 + a21 \ a1 + a 25 = a 5 + a 21
30 (p) 400
+ a25 + a30 = 120, then å ai is = a 9 + a17 = a13 + a13 = l [say]
i=1
Q a1 + a 5 + a 9 + a13 + a17 + a 21 + a 25 = 112
(B) If a1 , a2 , a3 , ... are in AP and a1 + a5 + a9
l
25
(q) 600 \ 3l + = 112
+ a13 + a17 + a21 + a25 = 112, then å ai is 2
i=1
7l
(C) If a1 , a2 , a3 , ... are in AP and Þ = 112
a1 + a4 + a7 + a10 + a13 (r) 800 2
16
Þ l = 32
+ a16 = 375, then å ai is
i=1 25
25 25
(s) 1000 Then, åai = 2
(a1 + a 25 ) =
2
´ 32 = 400
i =1
Sol. (A) ® (q); (B) ® (p); (C) ® (s)
268 Textbook of Algebra

(C)Qa1, a 2 , a 3 , .... are in AP. For c , a, we get


\ a1 + a16 = a 4 + a13 = a 7 + a10 = l [say] (c + a )
³ ca
Q a1 + a 4 + a 7 + a10 + a13 + a16 = 375 2
\ 3l = 375 \ l = 125 Þ (c + a ) ³ 2 ca …(ii)
16
16 and for a, b, we get
Then, åai = 2
(a1 + a16 ) (a + b )
i =1 ³ ab
2
= 8 ´ l = 8 ´ 125 = 1000
Þ (a + b ) ³ 2 ab …(iii)
l Ex. 28
On multiplying Eqs. (i), (ii) and (iii), we get
Column I Column II
(b + c ) (c + a ) (a + b ) ³ 8abc
(A) If a > 0, b > 0, c > 0 and the minimum value (p) Þ (1 - a ) (1 - b ) (1 - c ) ³ 8abc [Qa + b + c = 1]
of a (b2 + c2 ) + b (c2 + a2 ) + c (a2 + b2 ) is 2 æ1 ö æ1 ö æ1 ö
l abc, then l is Þ ç - 1÷ ç - 1÷ ç - 1÷ ³ 8
èa ø èb ø èc ø
(B) If a, b, c are positive, a + b + c = 1and the (q) \ l =8
æ1 ö æ1 ö æ1 ö
minimum value of ç - 1÷ ç - 1÷ ç - 1÷ is 4 (C)Q AM ³ HM
èa ø èb ø èc ø
l, then l is (s - a ) + (s - b ) + (s - c ) 3
³
3 1 1 1
(C) If a > 0, b > 0, c > 0, s = a + b + c and the (r) + +
s -a s -b s -c
2s 2s 2s
minimum value of + + is 6 3s - (a + b + c ) 3
s-a s-b s-c Þ ³
3 1 1 1
(l - 1), then l is + +
s -a s -b s -c
(D) If a > 0, b > 0, c > 0, a, b, c are in GP and the (s) 3s - s 3
æ aö
l
æ cö
l Þ ³
the minimum value of ç ÷ + ç ÷ is 2, 8 3 æ 1 1 1 ö
è bø è bø ç + + ÷
ès - a s - b s - c ø
then l is
2s 2s 2s
(t) 10 Þ + + ³9
s -a s -b s -c
Sol. (A) ® (r); (B) ® (s); (C) ® (t); (D) ® (p, q, r, s, t) Here, l -1=9
(A)Q AM ³ GM \ l = 10
(D) If a, b, c are in GP.
ab 2 + ac 2 + bc 2 + ba 2 + ca 2 + cb 2
\ Then, a l , b l , c l are also in GP.
6
³ (ab 2 × ac 2 × bc 2 × ba 2 × ca 2 × cb 2 )1/ 6 = abc Then, AM ³ GM
al + c l
³ bl
\ a (b 2 + c 2 ) + b (c 2 + a 2 ) + c (a 2 + b 2 ) ³ 6 abc 2
Þ l =6 Þ a l + c l ³ 2b l
l l
(B)Q AM ³ GM æa ö æc ö
Þ ç ÷ + ç ÷ ³2
(b + c ) èb ø èb ø
For b, c , we get ³ bc
2 \ l ÎR
Þ (b + c ) ³ 2 bc …(i) Hence, l = 2, 4, 6, 8, 10
Chap 3 Sequences and Series 269

JEE Type Solved Examples :


Statement I and II Type Questions
n Directions Example numbers 29 to 32 are l Ex. 31 Consider an AP with a as the first term and d is
Assertion-Reason type examples. Each of these examples the common difference such that Sn denotes the sum to
contains two statements: n terms and a n denotes the nth term of the AP. Given that for
Statement-1 (Assertion) and Statement-2 (Reason)
S m2
Each of these examples also has four alternative choices, some m, n Î N , m = (m ¹ n ).
only one of which is the correct answer. You have to select Sn n2
the correct choice as given below. Statement 1 d = 2a because
(a) Statement-1 is true, Statement-2 is true; Statement-2 a 2m + 1
is a correct explanation for Statement-1 Statement 2 m =
(b) Statement-1 is true, Statement-2 is true; Statement-2 an 2n + 1
is not a correct explanation for Statement-1 Sm m 2
(c) Statement-1 is true, Statement-2 is false Sol. (c) Q = 2
Sn n
(d) Statement-1 is false, Statement-2 is true
Let Sm = m 2k , Sn = n 2k
l Ex. 29. Statement 1 The sum of first n terms of the series
n ( n + 1) \ am = Sm - Sm - 1 = m 2k - (m - 1)2 k
12 - 22 + 32 - 4 2 + 52 -... can be = ± .
2 n (n +1) Þ am = (2m - 1) k
Statement 2 Sum of first n natural numbers is . am 2m - 1
2 Similarly, an = (2n - 1) k \ =
Sol. (a) Clearly, nth term of the given series is negative or an 2n - 1
positive according as n is even or odd, respectively.
Statement-2 is false.
Case I When n is even, in this case the given series is
Also, Q a1 = k , a 2 = 3k , a 3 = 5k , …
12 - 22 + 32 - 4 2 + 52 - 62 + ... + (n - 1)2 - n 2
Given, a1 = a = k
= (12 - 22 ) + (32 - 4 2 ) + (52 - 62 ) + ...+ [(n - 1)2 - n 2 ]
\ a1 = a, a 2 = 3a, a 3 = 5a, …
= (1 - 2) (1 + 2) + (3 - 4 ) (3 + 4 ) + (5 - 6) (5 + 6) + ...
\ Common difference d = a 2 - a1 = a 3 - a 2 = ...
+ (n - 1 - n ) (n - 1 + n )
n ( n + 1) Þ d = 2a
= - (1 + 2 + 3 + 4 + 5 + 6 + ... + (n - 1) + n ) =
2 \ Statement-1 is true.
Case II When n is odd, in this case the given series is
12 - 22 + 32 - 4 2 + 52 - 62 + ... + (n - 2)2 - (n - 1)2 + n 2
l Ex. 32 Statement-1 1, 2, 4, 8, … is a GP, 4, 8, 16, 32, …
is a GP and 1 + 4, 2 + 8, 4 + 16, 8 + 32, ... is also a GP.
= (12 - 22 ) + (32 - 4 2 ) + (52 - 62 )+... + [(n - 2)2 - (n - 1)2 ] + n 2
= (1 - 2) (1 + 2) + (3 - 4 ) (3 + 4 ) + (5 - 6) (5 + 6) + ...
Statement-2 Let general term of a GP with common ratio r
be Tk + 1 and general term of another GP with common ratio r
+ [(n - 2) - (n - 1)] [(n - 2) + (n - 1)] + n 2
be Tk¢ + 1 , then the series whose general term
= - [1 + 2 + 3 + 4 + 5 + 6 + ... + (n - 2) + (n - 1)] + n 2
( n - 1) ( n - 1 + 1) n ( n + 1) Tk¢¢+ 1 = Tk + 1 + Tk¢ + 1 is also a GP with common ratio r.
=- + n2 =
2 2 Sol. (a) 1, 2, 4, 8, …
It is clear that Statement-1 is true, Statement-2 is true and Common ratio r = 2
Statement-2 is correct explanation for Statement-1.
\ Tk + 1 = 1 × (2)k + 1 - 1 = 2k
l Ex. 30 Statement 1 If a , b, c are three positive numbers and 4, 8, 16, 32, ...
æa +b +c ö æ 3abc ö 2 /3 Common ratio, r = 2
in GP, then ç ÷ç ÷ = (abc ) .
è 3 ø è ab + bc + ca ø \ Tk¢ + 1 = 4 × (2)k + 1 - 1 = 4 × 2k
Statement-2 (AM) (HM) =(GM) 2 is true for positive numbers. Then, Tk + 1 + Tk¢ + 1 = 5 × 2k = Tk¢¢+ 1
Sol. (c) If a, b be two real, positive and unequal numbers, then
a+b 2ab 5 × 2k
AM = , GM = ab and HM = Common ratio of Tk¢¢+ 1 = = 2, which is true.
2 a+b 5 × 2k - 1
\ AM) (HM) = (GM) 2
Hence, Statement-1 and Statement-2 both are true and
This result will be true for n numbers, if they are in GP. Statement-2 is the correct explanation of Statement-1.
Hence, Statement-1 is true and Statement-2 is false.
270 Textbook of Algebra

Subjective Type Examples


n In this section, there are 24 subjective solved examples. 2 (2n - 1)
Þ 2a × = 16 (2n - 1)
l Ex. 33 In a set of four numbers, the first three are in GP
( 2 - 1)
and the last three are in AP with a common difference of 6. If Þ 2a + 1 = 16 = 24
the first number is same as the fourth, then find the four Þ a+1= 4
numbers. \ a=3
Sol. Let the last three numbers in AP, be a, a + 6, a + 12.
[Q 6 is the common difference] l Ex. 35 If n is a root of x 2 (1 - ac ) - x (a 2 + c 2 )
If first number is b, then four numbers are - (1 + ac ) = 0 and if n harmonic means are inserted between
b, a, a + 6, a + 12 a and c , find the difference between the first and the last
But given, b = a + 12 means.
\ Four numbers are a + 12, a, a + 6, a + 12 …(i) Sol. Let H 1, H 2 , H 3 , ..., H n , are n harmonic means, then
Since, first three numbers are in GP. a, H 1, H 2 , H 3 , ..., H n ,b are in HP.
Then, a 2 = (a + 12) (a + 6)
1 1 1 1 1 1
\ , , , , ..., , are in AP.
Þ a 2 = a 2 + 18a + 72 a H1 H 2 H 3 Hn b
Þ 18a + 72 = 0 1 1
\ a=-4 [from Eq. (i)] If d be the common difference, then = + ( n + 2 - 1) d
c a
Hence, four numbers are 8, - 4, 2, 8. (a - c )
\ d= …(i)
n ac (n + 1)
l Ex. 34 Find the natural number a for which å f (a + k ) 1 1 1 1
k =1 Þ = + d and = -d
h1 a hn c
= 16 ( 2 n - 1), where the function f satisfies
f ( x + y ) = f ( x ) f (y ) for all natural numbers x , y and a c a c
\ h1 - hn = - = -
further f (1) = 2. 1 + ad 1 - cd 1 + (a - c ) 1 - (a - c )
Sol. Given, f ( x + y ) = f ( x ) f (y ) …(i) c ( n + 1) a ( n + 1)
and f ( 1) = 2 …(ii) ac (n + 1) ac (n + 1) æ 1 1 ö
= - = ac (n + 1) ç - ÷
On putting x = y = 1 in Eq. (i), we get cn + a an + c è cn + a an + c ø
f ( 1 + 1) = f ( 1) f ( 1) = 2 × 2
æ an + c - cn - a ö
\ f ( 2) = 22 …(iii) = ac (n + 1) ç 2 2 2
÷
è acn + (a + c )n + ac ø
Now, on putting x = 1, y = 2 in Eq. (i), we get
ac (a - c ) (n 2 - 1)
f (1 + 2) = f (1) f (2) = 2 × 22 [from Eqs. (ii) and (iii)] = …(ii)
acn 2 + (a 2 + c 2 ) n + ac
\ f ( 3) = 23
But given n is a root of
On putting x = y = 2 in Eq. (i), we get
x 2 (1 - ac ) - x (a 2 + c 2 ) - (1 + ac ) = 0.
f ( 2 + 2) = f ( 2) f ( 2) = 22 × 22 [from Eq. (iii)]
Then, n 2 (1 - ac ) - n (a 2 + c 2 ) - (1 + ac ) = 0
\ f ( 4 ) = 24
M M M or acn 2 + (a 2 + c 2 )n + ac = n 2 - 1,
Similarly, f ( l ) = 2l , l Î N ac (a - c ) (n 2 - 1)
then from Eq. (ii), h1 - hn = = ac (a - c )
\ f (a + k ) = 2a + k , a + k Î N ( n 2 - 1)
n n
Q å f (a + k ) = 16 (2n - 1) Þ å2a + k = 16 (2n - 1) l Ex. 36 A number consists of three-digits which are in GP
k =1 k =1
n
the sum of the right hand and left hand digits exceeds twice
Þ 2a å2k = 16 (2n - 1) the middle digit by 1 and the sum of the left hand and
k =1 middle digits is two third of the sum of the middle and right
Þ 2 (2 + 22 + 23 + ... + 2n ) = 16 (2n - 1)
a 1 hand digits. Find the number.
Chap 3 Sequences and Series 271

Sol. Let the three digits be a, ar and ar 2 , then number is or


20
+ 20r = 50
100a + 10ar + ar 2
…(i) r
or 2 + 2r 2 = 5r or 2r 2 - 5r + 2 = 0
Given, a + ar 2 = 2ar + 1
1
or a(r 2 - 2r + 1) = 1 or (r - 2) (2r - 1) = 0 \ r = 2 or
2
or a ( r - 1) 2 = 1 …(ii)
Hence, the three numbers are 10, 20, 40 or 40, 20, 10.
2 2
Also, given a + ar = (ar + ar ) Þ 3 + 3r = 2r + 2r 2
3 l Ex. 39 If the sum of m terms of an AP is equal to the sum
or 2r 2 - r - 3 = 0 or (r + 1) (2r - 3) = 0 of either the next n terms or the next p terms, then prove that
3 æ 1 1ö
\ r = - 1, æ 1 1ö
2 (m + n ) ç - ÷ = (m + p ) ç - ÷ .
èm pø èm nø
1 1
For r = - 1, a = = ÏI
( r - 1) 2 4 Sol. Let the AP be a, a + d , a + 2d , ...
\ r ¹ -1 Given, T1 + T 2 + ... + Tm = Tm + 1 + Tm + 2 + ... + Tm + n …(i)
3 1 On adding T1 + T 2 + ... + Tm both sides in Eq. (i), we get
For r = ,a = 2
=4 [from Eq. (ii)]
2 æ3 ö 2 (T1 + T 2 + ... + Tm ) = T1 + T 2 + ... + Tm + Tm + 1
ç - 1÷ + ... + Tm + n
è2 ø
3 9 Þ 2Sm = Sm + n
From Eq. (i), number is 400 + 10 × 4 × + 4 × = 469
2 4 m m +n
\ 2 × [ 2a + (m - 1) d ] = [ 2a + (m + n - 1) d ]
n i j 2 2
l Ex. 37 Find the value of the expression å å å1. Let 2a + (m - 1) d = x
i =1 j =1 k =1 m +n
Þ mx = { x + nd }
j
n i n i n
i ( i + 1) 2
Sol. å å å1 = å å j = å 2 Þ (m - n ) x = ( m + n ) nd …(ii)
i =1 j =1 k =1 i =1 j =1 i =1
Again, T1 + T 2 + ... + Tm = Tm + 1 + Tm + 2 + ... + Tm + p
é n 2 n ù
=
1
ê åi + åi ú =
2
êë i = 1 i =1 ú
1
2
[
ån 2 + ån ] Similarly, (m - p ) x = (m + p ) pd …(iii)
û On dividing Eq. (ii) by Eq. (iii), we get
1 é n (n + 1) (2n + 1) n (n + 1) ù m - n (m + n ) n
= ê + ú =
2ë 6 2 û m - p (m + p ) p
n ( n + 1) n ( n + 1) ( n + 2) Þ (m - n ) (m + p ) p = (m - p ) (m + n ) n
= [ 2n + 1 + 3] =
12 6 On dividing both sides by mnp, we get
l Ex. 38 Three numbers are in GP whose sum is 70. If the æ1 1 ö æ1 1ö
(m + p ) ç - ÷ = (m + n ) ç - ÷
extremes be each multiplied by 4 and the mean by 5, then èn m ø èp mø
they will be in AP. Find the numbers. æ1 1ö æ 1 1ö
a Hence, (m + n ) ç - ÷ = (m + p ) ç - ÷
Sol. Let the three numbers in GP be , a, ar . èm p ø èm n ø
r
a
Given, + a + ar = 70 …(i) l Ex. 40 Find the sum of the products of every pair of the
r
4a first n natural numbers.
and , 5a, 4ar are in AP. Sol. We find that
r
4a 10a a S = 1 × 2 + 1 × 3 + 1 × 4 + ... + 2 × 3 + 2 × 4 + ... + 3 × 4
\ 10a = + 4ar or = + ar + 3 × 5 + ... + (n - 1) × n …(i)
r 4 r
5a [1 + 2 + 3 + ... + (n - 1) + n ]2 = 12 + 22 + 32 +
or = 70 - a [from Eq. (i)] Q
2 ... + (n - 1)2 + n 2
or 5a = 140 - 2a or 7a = 140
\ a = 20 + 2 [1 × 2 + 1 × 3 + 1 × 4 + ... + 2 × 3 + 2 × 4 +... + 3 × 4 + 3 × 5
From Eq. (i), we get + ... + (n - 1) × n ]
20 ( å n )2 = å n 2 + 2S [from Eq. (i)]
+ 20 + 20r = 70
r
272 Textbook of Algebra

( å n )2 - å n 2 Let a, ar , ar 2 , K be a GP with common ratio | r | < 1 [Q given


Þ S=
2 infinitely GP]
2
ì n ( n + 1) ü n (n + 1) (2n + 1) and also given S ¥ = 27
í ý -
î 2 þ 6 a
= so, = 27 …(i)
2 1-r
n 2 (n + 1)2 n (n + 1) (2n + 1) and a - ar = f ¢(0)
-
= 4 6 Þ a (1 - r ) = f ¢(0) = 3 [Q f ¢ (0) = 3]
2 \ a (1 - r ) = 3 …(ii)
n ( n + 1) From Eqs. (i) and (ii), we get
= [3n (n + 1) - 2 (2n + 1)] 1 1
24 (1 - r )2 = Þ 1-r = ±
n (n + 1) (3n 2 - n - 2) 9 3
= 1
24 \ r =1±
(n - 1) n (n + 1) (3n + 2) 3
Hence, S= 4 2 4
24 So, r = , Þ r ¹ [Q | r | < 1]
3 3 3
p /4 2
l Ex. 41 If I n = ò tan n x dx , show that Hence, r =
0 3
1 1 1 1
, , , ,Kform an AP. Find its l Ex. 43 Solve the following equations for x and y
I 2 + I 4 I3 + I 5 I 4 + I 6 I 5 + I7
1 1
common difference. log10 x + log10 x + log10 x + ... = y
2 4
Sol. We have,
1 + 3 + 5 + ... + ( 2y - 1) 20
p/ 4 n+2 and = .
ò0 (tan x + tan x ) dx
n
In + In + 2 =
4 + 7 + 10 + ... + (3y + 1) 7 log 10 x
p/ 4
= ò tann x (1 + tan 2 x ) dx Sol. From the first equation
0
p/ 4 ì 1 1 ü
p/ 4 é tann + 1 x ù 1 log10 x í1 + + + ... + ¥ý = y
= ò0 tann x × sec 2 x dx = ê
êë n + 1 úû 0
ú =
n +1
î 2 4 þ
ì ü
1 ï 1 ï
Hence, =n +1 Þ log10 x í ý=y
In + In + 2 ï1 - ï
1
On putting n = 2, 3, 4, 5, ... î 2þ
1 1 1 1 Þ 2 log10 x = y …(i)
\ = 3, = 4, = 5, = 6, ...
I2 + I4 I3 + I5 I4 + I6 I5 + I7 From the second equation
1 1 1 1 1 + 3 + 5 + ... + (2y - 1) 20
Hence, , , , , K are in AP with =
I2 + I4 I3 + I5 I4 + I6 I5 + I7 4 + 7 + 10 + ... + (3y + 1) 7 log10 x
common difference 1. y
(1 + 2y - 1)
2 20
Þ =
l Ex. 42 If the sum of the terms of an infinitely decreasing y 7 log 10 x
( 4 + 3y + 1)
GP is equal to the greatest value of the function 2
f ( x ) = x 3 + 3 x - 9 on the interval [ -5, 3 ] and the difference 2y 20
Þ =
3y + 5 7 log10 x
between the first and second terms is f ¢ (0 ), then show that
2 Þ 7y (2 log10 x ) = 60y + 100
the common ratio of the progression is . Þ 7y (y ) = 60y + 100 [from Eq. (i)]
3 Þ 7y 2 - 60y - 100 = 0
Sol. Given, f ( x ) = x 3 + 3x - 9
\ (y - 10) (7y + 10) = 0
\ f ¢( x ) = 3x 2 + 3 -10
\ y = 10, y ¹
7
Hence, f ¢( x ) > 0 in [ - 5, 3], then f ( x ) is an increasing
[because y being the number of terms in series Þ y Î N ]
function on [ -5, 3] and therefore, f ( x ) will have greatest
From Eq. (i), we have
value at x = 3.
2 log10 x = 10 Þ log10 x = 5
Thus, greatest value of f ( x ) is
\ x = 10 5
f ( x ) = 33 + 3 × 3 - 9 = 27
Hence, required solution is x = 10 5, y = 10
Chap 3 Sequences and Series 273

p Sol. Let 1st term of the r th group be Tr and the 1st terms of
l Ex. 44 If 0 < x < , successive rows are 1, 2, 4, 8, ..., respectively.
2
-1 -1
exp [(sin 2 x + sin 4 x + sin 6 x + ... + ¥) log e 2 ] satisfies the Tr = 1 × 2r = 2r
quadratic equation x 2 - 9 x + 8 = 0, find the value of Hence, the sum of the numbers in the r th group is
sin x - cos x 2r -1
-1 -1
. = {2 × 2r + (2r - 1) × 1}
sin x + cos x 2
-1
p [Q number of terms in r th group is 2r ]
Sol. 0< x <
2 =2 r -2 r
{2 + 2 r -1
- 1}
\ 0 < sin 2 x < 1
Hence, sum of the numbers in the nth group is
Then, sin 2 x + sin 4 x + sin 6 x + K + ¥ 2n - 2 [2n + 2n - 1 - 1].
sin 2 x
= = tan 2 x
1 - sin 2 x l Ex. 46 If a , b, c are in HP, then prove that
2 4 6 a +b c +b
\ exp [(sin x + sin x + sin x + ... + ¥ ) loge 2] + > 4.
2 2a - b 2c - b
= exp (tan 2 x × loge 2) = exp (loge 2tan x
)
2 Sol. Since, a, b, c are in HP.
loge 2 tan x tan 2 x
=e =2 2 1 1
tan 2 x
\ = + …(i)
Let y =2 b a c
Because y satisfies the quadratic equation. a+b c +b
and let P= +
Then, y 2 - 9y + 8 = 0 2a - b 2c - b
So, y = 1, 8
2 2ac 2ac
if y = 1 = 2tan x a+ c +
a+c a+c
Þ
2
2tan x = 20 = + [from Eq. (i)]
2ac 2ac
Þ tan 2 x = 0 2a - 2c -
a+c a+c
\ x =0 [impossible] [Q x > 0]
a + 3c 3a + c 3 æc a ö
Now, if y =8=2 tan 2 x = + =1+ ç + ÷ …(ii)
2a 2c 2 èa c ø
2
Þ 2tan x
= 23 Q AM > GM [Qa ¹ c ]
2
Þ tan x = 3 æc a ö
\ ç + ÷ >2
\ tan x = 3 èa c ø

sin x - cos x tan x - 1 3 -1 3 -1 3 æc a ö


\ = = ´ Þ ç + ÷ >3
sin x + cos x tan x + 1 3 +1 3 -1 2 èa c ø

( 3 - 1) 2 3 + 1 - 2 3 3 æc a ö
= = or 1+ ç + ÷ > 1 + 3 or P > 4
3-1 2 2 èa c ø
sin x - cos x a+b c +b
Hence, =2- 3 Hence, + >4
sin x + cos x 2a - b 2c - b

l Ex. 45 The natural numbers are arranged in the form


l Ex. 47 Find the sum of n terms of the series
given below 1 2 3
1
2 4
+ 2 4
+ + ... .
2 3 1+ 1 + 1 1+ 2 + 2 1 + 3 + 34
2

4 5 6 7 n
Sol. The n th term of the given series is Tn =
8 9 10 11 12 13 14 15 (1 + n 2 + n 4 )
.................................................................................. n
..................................................................................
\Sum of n terms = Sn = å Tn = å ( 1 + n 2 + n 4 )
The rth group containing 2r - 1 numbers. Prove that sum =
n
å (1 + n + n 2 ) (1 - n + n 2 )
of the numbers in the nth group is 2n - 2 [2n + 2n - 1 - 1].
274 Textbook of Algebra

1 æ 1 1 ö l Ex. 49 Find the sum of the first n terms of the series


=
2
å çè 1 - n + n 2 - ÷
1+n +n ø 2
1 + 3 × 2 2 + 3 3 + 3 × 4 2 + 5 3 + 3 × 6 2 + ...
3

1æ 1 1 ö If (i) n is even, (ii) n is odd.


= ç - ÷ [ by property]
2 è1 - 1 + 1 1 + n + n ø
2
Sol. Case I If n is even.
2
(n + n ) n ( n + 1) Let n = 2m
= =
2 (1 + n + n ) 2 2
2 ( n + n + 1) \ S = 13 + 3 × 22 + 33 + 3 × 4 2 + 53 + 3 × 62 +
... + (2m - 1)3 + 3 (2m )2
343
l Ex. 48 The value of xyz is 55 or according as the = {13 + 33 + 53 + ... + (2m - 1)3 } + 3 {22 + 4 2 + 62
55
+ ... + (2m )2 }
series a , x , y , z , b is an AP or HP. Find the values of a and b
m m
given that they are positive integers. = å(2r - 1)3 + 3 × 4 år 2
Sol. If a, x , y , z , b are in AP. r =1 r =1
m m
Then, b = Fifth term = a + (5 - 1) d = å {8r 3 - 12r 2 + 6r - 1} + 12 år 2
where, d is common difference] r =1 r =1
b-a
\ d= m m m m m
4 =8 år 3 - 12 år 2 + 6 år - å1 + 12 år 2
\ x × y × z = (a + d ) (a + 2d ) (a + 3d ) = 55 [given] r =1 r =1 r =1 r =1 r =1

æ b + 3a ö æ 2a + 2b ö æ a + 3b ö m m m
Þ ÷ = 55
ç ÷ç
è 4 øè 4 ø è 4 ø
÷ç =8 år 3 + 6 år - å1
r =1 r =1 r =1
Þ (a + 3b ) (a + b ) (3a + b ) = 55 ´ 32 …(i) 2
m ( m + 1) 2
m ( m + 1)
If they are in HP. = 8× +6 -m
4 2
1 æ1 1ö
The common difference of the associated AP is ç - ÷. = 2m 2 (m + 1)2 + 3m (m + 1) - m
4 èb a ø
(a - b ) = m [ 2m 3 + 4m 2 + 5m + 2]
i.e.
4ab
n é æn ö ù é
3 2
æn ö æn ö nù
1 1 (a - b ) = ê2 ç ÷ + 4 ç ÷ + 5 ç ÷ + 2ú êQm = ú
\ = + 2 ê 2 è ø è 2 ø è 2 ø úû ë 2û
x a 4ab ë
4ab n
Þ x= Hence, S = (n 3 + 4n 2 + 10n + 8) …(i)
a + 3b 8
1 1 2 (a - b ) Case II If n is odd.
\ = + Then, (n + 1) is even in the case
y a 4ab
4ab 2ab Sum of first n terms = Sum of first (n +1) terms - (n + 1) th
Þ y= = term
2a + 2b a + b
( n + 1)
1 1 3 (a - b ) = [(n + 1)3 + 4 (n + 1)2 + 10 (n + 1) + 8] - 3 (n + 1)2
and = + 8
z a 4ab
1
4ab = (n + 1) [n 3 + 3n 2 + 3n + 1 + 4n 2 + 8n + 4 + 10n
Þ z= 8
3a + b
+ 10 + 8 - 24n - 24 ]
4ab 2ab 4ab
\ xyz = × × = 343 [given] 1
(a + 3b ) (a + b ) (3a + b ) Hence, S = (n + 1) [n 3 + 7n 2 - 3n - 1]
8
32 a 3b 3 343
Þ = [from Eq. (i)]
55 ´ 32 55 l Ex. 50 Find out the largest term of the sequence
3 3 1 4 9 16
or a b = 343 , , , , ... .
503 524 581 692
Þ ab = 7
n2
Hence, a = 7, b = 1 Sol. General term can be written as Tn =
500 + 3n 3
or a = 1, b = 7
Chap 3 Sequences and Series 275

1 500 n n
Let Un =
Tn
= 2 + 3n
n
=- å r - å 1 + ( n + 1) 2 f ( n + 1) - 12 f ( 1)
r =1 r =1
dU n 1000 n ( n + 1)
Then, =- +3 =- - n + ( n + 1) 2 f ( n + 1) - f ( 1)
dn n 2
3000 d 2U n n ( n + 3)
and = 4 = ( n + 1) 2 f ( n + 1) - -1 [Q f (1) = 1]
dn 2 n 2
For maxima or minima of U n , we have (n 2 + 3n + 2)
dU n 1000 = ( n + 1) 2 f ( n + 1) -
= 0 Þ n3 = 2
dn 3 Hence, this is the required result.
1/ 3 1/ 3
æ 1000 ö æ 1000 ö
Þ n=ç ÷ (not an integer) and 6< ç ÷ <7
è 3 ø è 3 ø l Ex. 52 If the equation x 4 - 4 x 3 + ax 2 + bx + 1 = 0 has
But n is an integer, therefore for the maxima or minima of four positive roots, find the values of a and b.
æ 1000 ö
1/ 3 Sol. Let x 1, x 2 , x 3 , x 4 are the roots of the equation
U n we will take n as the nearest integer to ç ÷ .
è 3 ø x 4 - 4 x 3 + ax 2 + bx + 1 = 0 …(i)
1/ 3 \ x 1 + x 2 + x 3 + x 4 = 4 and x 1 x 2 x 3 x 4 = 1
æ 1000 ö
Since, ç ÷ is more close to 7 than to 6. Thus, we take x + x2 + x3 + x4 4
è 3 ø Q AM = 1 = =1
4 4
n = 7.
and GM = ( x 1 x 2 x 3 x 4 )1/ 4 = (1)1/ 4 = 1
d 2U n
Further = + ve , then U n will be minimum and i.e., AM = GM
dn 2
therefore, Tn will be maximum for n = 7. which is true only when x 1 = x 2 = x 3 = x 4 = 1
Hence, T 7 is largest term. So, largest term in the given Hence, given equation has all roots identical, equal to 1 i.e.,
49 equation have form
sequence is .
1529 ( x - 1) 4 = 0
Þ x 4 - 4 x 3 + 6x 2 - 4 x + 1 = 0 …(ii)
1 1 1
l Ex. 51 If f (r ) = 1 + + + ... + and f (0 ) = 0, find On comparing Eqs. (i) and (ii), we get
2 3 r
n a = 6, b = - 4
å ( 2r + 1) f (r ). ¥ ¥
r =1 m 2n
n
l Ex. 53 Evaluate å å 3m (n × 3m + m × 3n )
.
Sol. Since, å(2r + 1) f (r ) ¥ ¥
m =1 n =1
r =1 m 2n
n n Sol. Let S = å å 3m (n × 3m + m × 3n )
= å(r 2 + 2r + 1 - r 2 ) f (r ) = å {(r + 1)2 - r 2 } f (r ) m =1 n =1
r =1 r =1 ¥ ¥
1
=
n
å {(r + 1) 2 2
f ( r ) - ( r + 1) f ( r + 1) + ( r + 1) 2 = å å æ 3m ö æ 3m 3n ö
m =1 n =1
r =1 ç ÷ç + ÷
2
f (r + 1) - r f (r )} èm øèm nø
n n
3m 3n
= å( r + 1)2 { f (r ) - f (r + 1)} + å {(r + 1)2 Now, let am =
m
and an =
n
r =1 r =1
¥ ¥
f (r + 1) - r 2 f (r )} 1
( r + 1)
n 2 n -1
Then, S= å åa (am + an )
…(i)
=- å + å (r + 1)2 f (r + 1) + (n + 1)2 m =1 n =1 m

r = 1 ( r + 1) r =1 By interchanging m and n, then


n
é 1 ù ¥ ¥
f ( n + 1) - å r 2 f ( r ) êQ f (r + 1) - f (r ) = 1
r =1 ë
ú
r + 1û S= å åa (an + am )
…(ii)
m =1 n =1 n
n
=- å(r + 1) + {22 f (2) + 32 f (3) + ... + n 2 f (n )} On adding Eqs. (i) and (ii), we get
r =1 2 2 2 ¥ ¥ ¥ ¥
+ (n + 1) f (n + 1) - {1 f (1) + 2 f (2) 1 mn
2S = å åa a
= å å m n
+ 32 f (3) + ... + n 2 f (n )} m =1 n =1 m n m =1 n =1 3 3
276 Textbook of Algebra

2 2 ¥ ¥ ¥
æ ¥ nö é æ1ö 2 3 ù 1 æ3 1ö 3 1 1
æ1ö æ1ö
= çç å n ÷÷ = ê1 ç ÷ + 2 ç ÷ + 3 ç ÷ + ...ú
= å 32 i çè 2 - 3i ÷ø = å 2 × 32 i - å 33i
èn = 13 ø êë è 3 ø è3ø è3ø úû i =0 i =0 i =0

3 9 27 135
= (S ¢ )2 …(iii) = × - =
2 8 26 208
2 3
æ1ö æ1ö æ1ö Hence required sum, S = S1 - S 2 - 3 S 3
where, S ¢ = 1 ç ÷ + 2 ç ÷ + 3 ç ÷ + ... + ¥
è3ø è3ø è3ø 27 27 æ 135 ö 27 ´ 26 - 27 ´ 8 - 3 ´ 135 81
= - -3ç ÷= =
1 æ1ö æ1ö
2 3 8 26 è 208 ø 208 208
S¢ = 1ç ÷ +2ç ÷ + ... + ¥
3 è3ø è3ø
– – – l Ex. 55 Let Sn , n =1, 2, 3, K be the sum of infinite geomet-
1 æ1ö
2 3 1
2 ¢ æ1ö ric series, whose first term is n and the common ratio is .
S = +ç ÷ +ç ÷ + ... + ¥ n +1
3 3 è3ø è3ø
1 Evaluate
= 3 =
1 S1Sn + S 2 Sn - 1 + S3 Sn - 2 + ... + Sn S1
1 2
lim .
1- n ®¥ S12 + S 22 + ... + Sn2
3
3 n
\ S¢= Sol. Q Sn = Þ Sn = n + 1
1
4 1-
2 n +1
æ3ö
From Eq. (iii), we get 2S = ç ÷ \ S1 Sn + S 2 Sn - 1 + S 3 Sn - 2 + ... + Sn S1
è4ø
n n

\ S=
9 = åSr Sn - r + 1 = å(r + 1) (n - r + 2)
r =1 r =1
32
n
¥ ¥ ¥
1
= å [(n + 1) r - r 2 + (n + 2)]
l Ex. 54 Find the value of å å å 3i 3 j 3k
r =1
n n n
i=0 j=0 k =0
(i ¹ j ¹ k )
= ( n + 1) å r - å r 2 + ( n + 2) å 1
r =1 r =1 r =1
¥ ¥ ¥
1
Sol. Let S = å å å [i ¹ j ¹ k ] = ( n + 1) å n - ån 2
+ ( n + 2) × n
i =0 j =0 k =0 3i 3 j 3k
(n + 1) n (n + 1) n (n + 1) (2n + 1)
We will first of all find the sum without any restriction on = - + ( n + 2) n
2 6
i, j , k .
n
¥ ¥ ¥ æ ¥ 1ö
3 = (n 2 + 9n + 14 ) …(i)
1 6
Let S1 = å å å = çå ÷
3i 3 j 3k çèi = 0 3i ÷ø n n n
i =0 j =0 k =0
and S12 + S 22 + ... + Sn2 = åSr2 = å( r + 1)2 = å( r + 1)2 - 12
3 r =1 r =1 r =0
æ3ö 27
=ç ÷ = (n + 1) (n + 2)( 2n + 3)
è2ø 8 = -1
6
Case I If i = j = k n
¥ ¥ ¥
= (2n 2 + 9n + 13) …(ii)
1 6
Let S2 = å å å 3 i 3 j 3k From Eqs. (i) and (ii), we get
i =0 j =0 k =0
S1 Sn + S 2 Sn - 1 + S 3 Sn - 2 + ... + Sn S1
¥ lim
1 1 1 1 27
= å 33i =1+
33
+
36
+ ... =
1
=
26
n ®¥ S12 + S 22 + ... + Sn2
i =0 1- æ 9 14 ö
33 n 2
(n + 9n + 14 ) ç1 + + ÷
è n n2 ø
Case II If i = j ¹ k = lim 6 = lim
n ®¥ n n ®¥ æ 9 13 ö
¥ ¥ ¥ æ ¥ 1 öæ ¥ 1ö (2n 2 + 9n + 13) ç2 + + ÷
1 è n n2 ø
Let S3 = å å å = çå ÷çå ÷
3i 3 j 3k çèi = 0 3 2 i ÷ø çèk = 0 3k ÷ø
6
1+0+0 1
i =0 j =0 k =0
= =
[Qk ¹ i ] 2+0+0 2
Chap 3 Sequences and Series 277

l Ex. 56 The nth term of a series is given by n ( n + 1) 1 æ 1 ö


= + ç 2 - 1÷ [ by property]
n 5 + n3 2 2 èn + n + 1 ø
tn = and if sum of its n terms can be expressed as
n 4 + n 2 +1 n2 n 1 1
= + - + 2
1 2 2 2 2n + 2n + 2
Sn = a n2 + a + , where a n and bn are the nth terms of
2 2
bn + b æn 1 ö 1 1 1
=ç + ÷ - - +
some arithmetic progressions and a , b are some constants, è 2 2 2ø 8 2 æ 1 ö
2
3
çn 2 + ÷ +
b è 2ø 2
prove that n is a constant.
an 2
æn 1 ö 5 1
n +n5 3 =ç + ÷ - +
Sol. Since, tn = è 2 2 2ø 8 æ 1 ö
2
3
n + n2 + 1
4 ç n 2 + ÷ +
è 2ø 2
n
=n- 1
n4 + n2 + 1 but given, Sn = an2 + a +
bn2 + b
1 1
=n + - On comparing, we get
2 ( n 2 + n + 1) 2 ( n 2 - n + 1)
n 1 5 æ 1 ö 3
an = + , a = - , bn = çn 2 + ÷, b =
Sum of n terms Sn = åt n 2 2 2 8 è 2ø 2
1ì æ 1 1 öü
= ån + í å ç 2 - 2 ÷ý \
bn
= 2 , which is constant.
2 îè n + n + 1 n - n + 1ø þ an
#L Sequences and Series Exercise 1 :
Single Option Correct Type Questions
n This section contains 30 multiple choice questions. 1 3 7 15
6. Sum of the first n terms of the series + + + + ...
Each question has four choices (a), (b), (c) and (d) out of 2 4 8 16
which ONLY ONE is correct is equal to
yz xz (a) 2n - n - 1 (b) 1 - 2 -n
1. If the numbers x , y, z are in HP, then , ,
y + z x + z (c) n + 2 -n - 1 (d) 2n - 1

xy 7. If in a DPQR, sin P, sin Q, sin R are in AP, then


are in
x + y (a) the altitudes are in AP (b) the altitudes are in HP
(a) AP (b) GP (c) the medians are in GP (d) the medians are in AP
(c) HP (d) None of these 8. Let a1 , a 2 , ..., a10 be in AP and h 1 , h 2 , ..., h 10 be in HP. If
n
a1 = h 1 = 2 and a10 = h 10 = 3, then a 4 h 7 is
2. If a1 , a 2 , ... are in HP and f k = å ar - ak , then (a) 2 (b) 3
r =1
(c) 5 (d) 6
2 a1 , 2 a2 , 2 a3 , 2 a4 , ... are in p1 - sin 2nx
ì a1 a2 a ü 9. If I n = ò dx , then I 1 , I 2 , I 3 ,... are in
í wherea 1 = , a 2 = , a 3 = 3 , Ký 0 1 - cos 2x
î f 1 f 2 f 3 þ (a) AP (b) GP
(a) AP (b) GP (c) HP (d) None of these
(c) HP (d) None of these
10. If a (b - c ) x 2 + b (c - a ) xy + c (a - b ) y 2 is a perfect
3. ABC is a right angled triangle in which ÐB = 90° and square, the quantities a, b, c are in
BC = a. If n points L 1 , L 2 , ..., L n on AB are such that AB (a) AP (b) GP
is divided in n + 1 equal parts and (c) HP (d) None of these
L 1 M 1 , L 2 M 2 , K , L n M n are line segments parallel to BC 11. The sum to infinity of the series,
and M 1 , M 2 , K , M n are on AC, the sum of the lengths of 2
æ 1ö æ 1ö
L 1 M 1 , L 2 M 2 , ..., L n M n is 1 + 2 ç1 - ÷ + 3 ç1 - ÷ + ... is
a (n + 1 ) a (n - 1 ) è nø è nø
(a) (b)
2 2 (a) n 2 (b) n (n + 1 )
an 2
(c) æ 1ö
2 (c) n ç1 + ÷ (d) None of these
è nø
(d) impossible to find from the given data
12. If log 3 2, log 3 (2 x - 5) and log 3 æç2 x - ö÷ are in AP, x is
7
4. Let S n (1 £ n £ 9 ) denotes the sum of n terms of the series
è 2ø
1 + 22 + 333 + ... + 999... 9, then for 2 £ n £ 9
123 equal to
9 times
(a) 2 (b) 3
1 n
(a) Sn - Sn - 1 = (10 - n 2 + n ) (c) 4 (d) 2, 3
9
1 n 13. Let a, b, c be three positive prime numbers. The
(b) Sn = (10 - n 2 + 2n - 2 )
9 progression in which a , b , c can be three terms (not
(c) 9 (Sn - Sn - 1 ) = n (10n - 1 ) necessarily consecutive), is
(d) None of the above (a) AP (b) GP
2 (c) HP (d) None of these
5. If a, b, c are in GP, then the equations ax + 2bx + c = 0
d e f 14. If n is an odd integer greater than or equal to 1, the value
and dx 2 + 2ex + f = 0 have a common root, if , ,
a b c of n 3 - (n - 1) 3 + (n - 2) 3 - ... + ( - 1)n - 1 1 3 is
are in (n + 1 ) 2 (2n - 1 ) (n - 1 ) 2 (2n - 1 )
(a) AP (b) GP (a) (b)
4 4
(c) HP (d) None of these
(n + 1 ) 2 (2n + 1 )
(c) (d) None of these
4
Chap 03 Sequences and Series 279

15. If the sides of a right angled triangle form an AP, the 22. If a, b, c and d are four positive real numbers such that
sines of the acute angles are abcd = 1, the minimum value of
3 4 1 (1 + a ) (1 + b ) (1 + c ) (1 + d ) is
(a) , (b) 3,
5 5 3 (a) 1 (b) 4
5 -1 5+1 3 1 (c) 16 (d) 64
(c) , (d) ,
2 2 2 2 23. If a, b, c are in AP and (a + 2b - c ) (2b + c - a ) (c + a - b )
16. The sixth term of an AP is equal to 2. The value of the = l abc , then l is
common difference of the AP which makes the product (a) 1 (b) 2
a1 a 4 a 5 least, is given by (c) 4 (d) None of these
8 5
(a) (b) 24. If a1 , a 2 , a 3 , ... are in GP with first term a and common
5 4
2 ratio r, then
(c) (d) None of these
3 a1a 2 a a a a an - 1 an
+ 2 2 3 2 + 2 3 4 2 + ... + 2 is equal
17. If the arithmetic progression whose common difference 2 2
a1 - a 2 a 2 - a 3 a 3 - a 4 an - 1 - an2
is non-zero, the sum of first 3n terms is equal to the sum
of the next n terms. The ratio of the sum of the first 2n to
nr (n - 1 ) r nr (n - 1 ) r
terms to the next 2n terms is (a) (b) (c) (d)
1 2 1 - r2 1 - r2 1 -r 1 -r
(a) (b)
5 3 25. The sum of the first ten terms of an AP is four times the
3
(c) (d) None of these sum of the first five terms, the ratio of the first term to
4
the common difference is
18. The coefficient of x n - 2 in the polynomial (a)
1
(b) 2 (c)
1
(d) 4
( x - 1) ( x - 2) ( x - 3) ... ( x - n ), is 2 4
n (n 2 + 2 ) (3n + 1 ) 26. If cos ( x - y ), cos x and cos ( x + y ) are in HP, the
(a)
24 æy ö
cos x sec ç ÷ is equal to
n (n 2 - 1 ) (3n + 2 ) è2ø
(b)
24 1
(a) ± 2 (b)
n (n 2 + 1 ) (3n + 4 ) 2
(c)
24 1
(c) - (d) None of these
(d) None of the above 2
19. Consider the pattern shown below: 27. If 11 AM’s are inserted between 28 and 10, the number
of integral AM’s is
Row 1 1
(a) 5 (b) 6
Row 2 3 5 (c) 7 (d) 8
Row 3 7 9 11 28. If x , y, z are in GP ( x , y, z > 1), then
Row 4 13 15 17, 19, etc. 1 1 1
, , are in
The number at the end of row 60 is 2x + ln x 4 x + ln y 6x + ln z
(a) 3659 (b) 3519 (a) AP (b) GP
(c) 3681 (d) 3731 (c) HP (d) None of these
100
29. The minimum value of the quantity
20. Let an be the nth term of an AP. If å a 2r = a and
(a 2 + 3a + 1) (b 2 + 3b + 1) (c 2 + 3c + 1)
r =1
,
100 abc
å a 2r - 1 = b, the common difference of the AP is where a, b, c Î R +, is
r =1
11 3
(a) a - b (b) b - a (a) (b) 125
a -b 23
(c) (d) None of these (c) 25 (d) 27
2
21. If a1 , a 2 , a 3 , a 4 , a 5 are in HP, then 30. Let a1 , a 2 , ... be in AP and q 1 , q 2 , ... be in GP. If
a1a 2 + a 2 a 3 + a 3 a 4 + a 4 a 5 is equal to a1 = q 1 = 2 and a10 = q 10 = 3, then
(a) 2 a1a 5 (b) 3a1a 5 (a) a 7 q19 is not an integer (b) a19 q 7 is an integer
(c) 4a1a 5 (d) 6a1a 5 (c) a 7 q19 = a19 q10 (d) None of these
280 Textbook of Algebra

#L Sequences and Series Exercise 2 :


More than One Correct Option Type Questions
n This section contains 15 multiple choice questions. (a) 7th term is 16 (b) 7th term is 18
Each question has four choices (a), (b), (c) and (d) out of 505 405
(c) sum of first 10 terms is (d) sum of first 10 terms is
which MORE THAN ONE may be correct. 4 4
1 1 1 1 1 1 1
31. If a(n ) = 1 + + + + ... + n , then 40. Let E = + + + ..., then
2 3 4 2 -1 12 22 32
(a) a (100 ) < 100 (b) a (100 ) > 100 3
(a) E < 3 (b) E > (c) E < 2 (d) E > 2
(c) a (200 ) > 100 (d) a (200 ) < 100 2
32. If the first and (2n - 1) th term of an AP, GP and HP are 41. Let S n (n ³ 1) be a sequence of sets defined by
equal and their nth terms are a, b and c respectively, then ì 3 5ü ì 8 11 14 ü
S 1 = {0}, S 2 = í , ý, S 3 = í , , ý,
(a) a = b = c (b) a ³ b ³ c î 2 2þ î3 3 3 þ
(c) a + c = b (d) ac - b 2 = 0 ì 15 19 23 27 ü
¥ ¥ S 4 = í , , , ý, ..., then
p î4 4 4 4þ
33. For 0 < f < , if x =
2
å cos 2n f, y = å sin 2n f and
439
n=0 n=0
¥ (a) third element in S 20 is
20
z= å cos 2n f sin 2n f, then
431
n=0 (b) third element in S 20 is
20
(a) xyz = xz + y (b) xyz = xy + z
(c) sum of the elements in S 20 is 589
(c) xyz = x + y + z (d) xyz = yz + x
(d) sum of the elements in S 20 is 609
34. If a, b, c are in AP and a 2 , b 2 , c 2 are in HP, then which of
the following could hold true?
42. Which of the following sequences are unbounded?
2
n n
a æ 1ö æ 2n + 1 ö æ 1ö
(a) - , b, c are in GP (b) a = b = c (a) ç1 + ÷ (b) ç ÷ (c) ç1 + ÷ (d) tann
2 è nø èn+2ø è nø
(c) a 3, b 3, c 3 are in GP (d) None of these
43. Let a sequence {an } be defined by
2 3
35. The next term of the GP x , x + 2, x + 10 is 1 1 1 1
an = + + + ... + , then
729 n +1 n +2 n +3 3n
(a) 0 (b) 6 (c) (d) 54
16 11 19
2 2 (a) a 2 = (b) a 2 =
36. If the sum of n consecutive odd numbers is 25 - 11 , then 12 20
(9n + 5 )
(a) n = 14 (b) n = 16 (c) an + 1 - an =
(3n + 1 ) (3n + 2 ) (3n + 3 )
(c) first odd number is 23 (d) last odd number is 49
-2
37. The GM of two positive numbers is 6. Their AM is A and (d) an + 1 - an =
3 (n + 1 )
HM is H satisfy the equation 90A + 5H = 918, then A
æ 1 ö æ 1 ö
may be equal to 44. Let S n ( x ) = ç x n - 1 + n -1 ÷
+ 2 ç x n - 2 + n - 2 ÷ + ...
(a)
1
(b) 5 (c)
5
(d) 10
èx ø è x ø
5 2 æ 1ö
+ (n - 1) ç x + ÷ + n, then
38. If the sum to n terms of the series è xø
1
1 1 1 1 l (a) S1( x ) = 1 (b) S1( x ) = x +
+ + + ... is - , then x
1 × 3 × 5 × 7 3 × 5 × 7 × 9 5 × 7 × 9 × 11 90 f (n ) 2 2
1 æ x100 - 1 ö 1 æ x100 - 1 ö
(a) f ( 0 ) = 15 (b) f (1 ) = 105 (c) S100( x ) = ç ÷ (d) S100( x ) = 100 ç ÷
640 1 x è x -1 ø
99
x è x -1 ø
(c) f ( l ) = (d) l =
27 3
45. All the terms of an AP are natural numbers and the sum
39. For the series, of the first 20 terms is greater than 1072 and less than
1 1 1162. If the sixth term is 32, then
S =1+ (1 + 2) 2 + (1 + 2 + 3) 2
(1 + 3) (1 + 3 + 5) (a) first term is 7 (b) first term is 12
1
+ (1 + 2 + 3 + 4 ) 2 + ... (c) common difference is 4 (d) common difference is 5
(1 + 3 + 5 + 7 )
Chap 03 Sequences and Series 281

#L Sequences and Series Exercise 3 :


Passage Based Questions
n This section contains 8 passages. Based upon each of the Passage IV
passage 3 multiple choice questions have to be (Q. Nos. 55 to 57)
answered. Each of these questions has four choices (a), (b),
(c) and (d) out of which ONLY ONE is correct. There are two sets A and B each of which consists of three
numbers in GP whose product is 64 and R and r are the
Passage I p 3
(Q. Nos. 46 to 48) common ratios such that R = r + 2. If = , where p and q
q 2
8 16 24
S n be the sum of n terms of the series + + + ... are sum of numbers taken two at a time respectively in the
5 65 325 two sets.
46. The value of lim S n is
n ®¥ 55. The value of p is
1 (a) 66 (b) 72 (c) 78 (d) 84
(a) 0 (b) (c) 2 (d) 4
2
56. The value of q is
47. The seventh term of the series is (a) 54 (b) 56 (c) 58 (d) 60
56 56 56 56
(a) (b) (c) (d) R
57. The value of r + R is r
2505 6505 5185 9605
(a) 5392 (b) 368 (c) 32 (d) 4
48. The value of S 8 , is
(a)
288
(b)
1088
(c)
81
(d)
107 Passage V
145 545 41 245 (Q. Nos. 58 to 60)
Passage II The numbers 1, 3, 6, 10, 15, 21, 28, ... are called triangular
(Q. Nos. 49 to 51) numbers. Let t n denotes the nth triangular number such
Two consecutive numbers from 1, 2, 3, ..., n are removed. that t n = t n - 1 + n, " n ³ 2.
105 58. The value of t 50 is
The arithmetic mean of the remaining numbers is .
4 (a) 1075 (b) 1175 (c) 1275 (d) 1375
49. The value of n lies in 59. The number of positive integers lying between t 100 and
(a) (41, 51) (b) (52, 62) (c) (63, 73) (d) (74, 84) t 101 are
50. The removed numbers (a) 99 (b) 100 (c) 101 (d) 102
(a) are less than 10 (b) lies between 10 to 30 60. If (m + 1) is the nth triangular number, then (n - m) is
(c) lies between 30 to 70 (d) greater than 70
(a) 1 + (m 2 + 2m ) (b) 1 + (m 2 + 2 )
51. Sum of all numbers is
(a) less than 1000 (b) lies between 1200 to 1500 (c) 1 + (m 2 + m ) (d) None of these
(c) greater than 1500 (d) None of these

Passage III Passage VI


(Q. Nos. 52 to 54) (Q. Nos. 61 to 63)
There are two sets A and B each of which consists of three Let A1 , A2 , A3 , ..., Am be arithmetic means between - 3
numbers in AP whose sum is 15 and where D and d are the and 828 and G1 , G2 , G3 , ..., Gn be geometric means
common differences such that D = 1 + d, d > 0. If between 1 and 2187. Product of geometric means is 3 35
p = 7 ( q - p), where p and q are the product of the and sum of arithmetic means is 14025.
numbers respectively in the two series. 61. The value of n is
52. The value of p is (a) 45 (b) 30 (c) 25 (d) 10
(a) 105 (b) 140 (c) 175 (d) 210 62. The value of m is
(a) 17 (b) 34 (c) 51 (d) 68
53. The value of q is
(a) 200 (b) 160 (c) 120 (d) 80 63. The value of G 1 + G 2 + G 3 + ... + G n is
(a) 2044 (b) 1022
54. The value of 7 D + 8d is
(c) 511 (d) None of these
(a) 37 (b) 22 (c) 67 (d) 52
282 Textbook of Algebra

Passage VII Passage VIII (Q. Nos. 67 to 69)


(Q. Nos. 64 to 66) Suppose p is the first of n ( n >1) arithmetic means between
Suppose a, b are roots of ax 2 + bx + c = 0 and g, d are roots two positive numbers a and b and q the first of n harmonic
means between the same two numbers.
of Ax 2 + Bx + C = 0.
67. The value of p is
64. If a , b, g , d are in AP, then common difference of AP is na + b nb + a na - b nb - a
(a) (b) (c) (d)
1 æb B ö 1 æb B ö n+1 n+1 n+1 n+1
(a) ç - ÷ (b) ç - ÷
4 èa A ø 3 èa A ø
68. The value of q is
1 æc B ö 1 æc C ö
(c) ç - ÷ (d) ç - ÷ (n - 1 ) ab (n + 1 ) ab (n + 1 ) ab (n - 1 ) ab
2 èa A ø 3 èa A ø (a) (b) (c) (d)
nb + a nb + a na + b na + b
65. If a, b, c are in GP as well as a , b, g , d are in GP, then
69. Final conclusion is
A, B, C are in 2
æn + 1ö
(a) AP only (b) GP only (a) q lies between p and ç ÷ p
è n -1 ø
(c) AP and GP (d) None of these
æn + 1ö
(b) q lies between p and ç ÷p
66. If a , b, g , d are in GP, then common ratio of GP is è n -1 ø
2
æ bA ö æ aB ö æn + 1ö
(a) ç ÷ (b) ç ÷ (c) q does not lie between p and ç ÷ p
è aB ø è bA ø è n -1 ø
æ bC ö æ cB ö æn + 1ö
(c) ç ÷ (d) ç ÷ (d) q does not lie between p and ç ÷p
è cB ø è bC ø è n -1 ø

#L Sequences and Series Exercise 4 :


Single Integer Answer Type Questions
n
This section contains 10 questions. The answer to each 75. The sequence a1 , a 2 , a 3 , ... is a geometric sequence with
question is a single digit integer, ranging from 0 to 9 common ratio r. The sequence b1 , b 2 , b 3 , ... is also a
(both inclusive).
geometric sequence. If b1 = 1, b 2 = 4 7 - 4 28 + 1, a1 = 4 28
70. Let a, b, c , d be positive real numbers with a < b < c < d . ¥
1 ¥

Given that a, b, c , d are the first four terms of an AP and and å = å bn , then the value of (1 + r 2 + r 4 ) is
n = 1 an n =1
ad p
a, b, d are in GP. The value of is , where p and q are 76. Let (a1 , b1 ) and (a 2 , b 2 ) are the pair of real numbers such
bc q
prime numbers, then the value of q is that 10, a, b, ab constitute an arithmetic progression.
æ 2a a + b1b 2 ö
110
Then, the value of ç 1 2 ÷ is
71. If the coefficient of x in the expansion of Õ (1 + rx ) is è 10 ø
r =1
l (1 + 10) (1 + 10 + 10 2 ), then the value of l is 77. If one root of Ax 3 + Bx 2 + Cx + D = 0, A ¹ 0, is the
arithmetic mean of the other two roots, then the relation
72. A 3-digit palindrome is a 3-digit number (not starting
2B 3 + lABC + mA 2 D = 0 holds good. Then, the value of
with zero) which reads the same backwards as forwards
For example, 242. The sum of all even 3-digit palindromes 2l + m is
1 2 22
is 2n 1 × 3n 2 × 5n 3 × 7 n 4 × 11n 5 , alue of n 1 + n 2 + n 3 + n 4 + n 5 is 78. If | x | > 1, then sum of the series + +
1+ x 1+ x2 1+ x4
73. If n is a positive integer satisfying the equation 23 1
+ + ... upto ¥ is , then the value of l is
2 + (6 × 2 2 - 4 × 2) + (6 × 3 2 - 4 × 3) + ... + (6 × n 2 - 4 × n ) = 140, 1+ x8 x-l
then the value of n is 79. Three non-zero real numbers form an AP and the
squares of these numbers taken in same order form a
74. Let S( x ) = 1 + x - x 2 - x 3 + x 4 + x 5 - x 6 - x 7 GP. If the possible common ratios are (3 ± k ) where
2 +1 ék 8 ù
+ ... + ¥, where 0 < x < 1. If S( x ) = , then the value k Î N , then the value of ê - ú is (where [ ] denotes
2 ë8 k û
of ( x + 1) 2 is the greatest integer function).
Chap 03 Sequences and Series 283

#L Sequences and Series Exercise 5 :


Matching Type Questions
n This section contains 4 questions. Questions 80, 81 and 82 have three statements (A, B and C) and question 83 has four
statements (A, B, C and D) given in Column I and questions 80 and 81 have four statements (p, q, r and s), question 82
has five statements (p, q, r, s and t) and question 83 has three statements (p, q and r) in Column II, respectively. Any
given statement in Column I can have correct matching with one or more statement(s) given in Column II.

80. Column I Column II 82. Column I Column II

(A) a, b, c, d are in AP, then (p) a+ d > b+ c (B) If a1 , a2 , a3 , ... are in AP and (q) a + 2b = 260
a1 + a5 + a10 + a15 + a20 + a24
(B) a, b, c, d are in GP, then (q) ad > bc = 195,
a = a2 + a7 + a18 + a23 and
(C) a, b, c, d are in HP, then (r) 1 1 1 1 b = 2 (a3 + a22 ) - (a8 + a17 ),
+ > +
a d b c then
(s) ad < bc (C) If a1 , a2 , a3 , ... are in AP and (r) a + 2b = 220
a1 + a7 + a10 + a21 +
a24 + a30 = 225,
81. Column I Column II a = a2 + a7 + a24 + a29 and
b = 2 (a10 + a21 ) - (a3 + a28 ),
(A) For an AP a1 , a2 , a3 , ..., an , K; (p) 9
5 then
a1 = ; a10 = 16. If a1 + a2
2 (s) a - b = 5l , l Î I
+ ... + an = 110, then ‘n’ equals
(t) a + b =15m , m ÎI
(B) The interior angles of a convex (q) 10
non-equiangular polygon of 9 sides 83. Column I Column II
are in AP. The least positive integer
that limits the upper value of the (A) If 4 a2 + 9b2 + 16 c2 (p) AP
common difference between the = 2 (3ab + 6bc + 4 ca), where a, b, c
measures of the angles in degrees is are non-zero numbers, then a, b, c are
in
(C) For an increasing GP, (r) 11
a1 , a2 , a3 , ..., an , K; (B) If 17a2 + 13b2 + 5 c2 (q) GP
a6 = 4 a4; a9 - a7 = 192, = (3ab + 15bc + 5 ca), where
if a4 + a5 + a6 + ... + an = 1016, then a, b, c are non-zero numbers, then
n equals a, b, c are in
(s) 12 2 2 2
(C) If a + 9b + 25 c (r) HP
æ 15 5 3 ö
= abc ç + + ÷ ,where a, b, c are
82. Column I Column II è a b cø
non-zero numbers, then a, b, c are in
(A) If a1 , a2 , a3 , ... are in AP and (p) a = 2b
a1 + a4 + a7 + a14 + a17 + (D) If (a2 + b2 + c2 ) p2 - 2p (ab + bc + ca)
a20 = 165, + (a2 + b2 + c2 ) £ 0, where a, b, c, p
a = a2 + a6 + a15 + a19 and
b = 2 (a9 + a12 ) - (a3 + a18 ), are non-zero numbers, then a, b, c are
then in
284 Textbook of Algebra

#L Sequences and Series Exercise 6 :


Statement I and II Type Questions
n Directions (Q. Nos. 84 to 90) are Assertion-Reason type Statement 2 The sum of products of numbers a1 , a 2 , a 3 ,
questions. Each of these questions contains two ..., an taken two at a time is denoted by å å ai a j .
statements: 1£i < j £n
Statement-1 (Assertion) and Statement-2 (Reason) 87. Statement 1 a + b + c = 18 (a, b, c > 0), then the
Each of these questions also has four alternative choices,
maximum value of abc is 216.
only one of which is the correct answer. You have to select
the correct choice as given below. Statement 2 Maximum value occurs when a = b = c
(a) Statement-1 is true, Statement-2 is true; Statement-2 88. Statement 1 If 4a 2 + 9b 2 + 16c 2 = 2 (3ab + 6bc + 4ca ),
is a correct explanation for Statement-1
where a, b, c are non-zero real numbers, then a, b, c are in
(b) Statement-1 is true, Statement-2 is true; Statement-2
GP.
is not a correct explanation for Statement-1
Statement-2 If (a1 - a 2 ) 2 + (a 2 - a 3 ) 2 + (a 3 - a1 ) 2 = 0,
(c) Statement1 is true, Statement-2 is false
then a1 = a 2 = a 3 , " a1 , a 2 , a 3 Î R .
(d) Statement-1 is false, Statement-2 is true
84. Statement 1 4, 8, 16 are in GP and 12, 16, 24 are in HP. 89. Statement 1 If a and b be two positive numbers, where
Statement 2 If middle term is added in three a > b and 4 ´ GM = 5 ´ HM for the numbers. Then, a = 4b.
consecutive terms of a GP, resultant will be in HP. Statement 2 (AM) (HM) = (GM) 2 is true for positive
85. Statement 1 If the nth term of a series is 2n 3 + 3n 2 - 4, numbers.
then the second order differences must be an AP. 90. Statement1 The difference between the sum of the first
Statement 2 Ifnth term of a series is a polynomial of 100 even natural numbers and the sum of the first 100
degreem, thenmth order differences of series are constant. odd natural numbers is 100.
86. Statement 1 The sum of the products of numbers Statement 2 The difference between the sum of the
n
± a1 , ± a 2 , ± a 3 , K , ± an taken two at a time is - å ai2 . first n even natural numbers and sum of the first n odd
i =1
natural numbers is n.

#L Sequences and Series Exercise 7 :


Subjective Type Questions
n
In this section, there are 24 subjective questions. 1 1 1
(ii) 1 - + - + K upto ¥
91. The p th, (2p ) th and ( 4p ) th terms of an AP, are in GP, 22 32 4 2
then find the common ratio of GP. 97. If the arithmetic mean of a1 , a 2 , a 3 , ..., an is a and
92. Find the sum of n terms of the series b1 , b 2 , b 3 , ..., bn have the arithmetic mean b and
(a + b ) + (a 2 + ab + b 2 ) + (a 3 + a 2 b + ab 2 + b 3 ) + ..., ai + bi = 1 for i = 1, 2, 3, ..., n, prove that
where a ¹ 1, b ¹ 1 and a ¹ b. n n
93. The sequence of odd natural numbers is divided into
groups 1; 3, 5; 7, 9, 11; ... and so on. Show that the sum of
å (ai - a )2 + å ai bi = nab
i =1 i =1
the numbers in nth group is n 3 .
94. Let a, b, c are respectively the sums of the first n terms, 98. If a1 , a 2 , a 3 , ... is an arithmetic progression with common
the next n terms and the next n terms of a GP. Show that difference 1 and a1 + a 2 + a 3 + ... + a 98 = 137, then find
a, b, c are in GP. the value of a 2 + a 4 + a 6 + ... + a 98 .
n
95. If the first four terms of an arithmetic sequence are
a, 2a, b and (a - 6 - b ) for some numbers a and b, find the
99. If t 1 = 1, t r - t r - 1 = 2r - 1 , r ³ 2, find åt r .
r =1
sum of the first 100 terms of the sequence.
1 1 1 p2 100. Prove that I 1 , I 2 , I 3 , ... form an AP, if
96. If 2 + 2 + 2 + ... upto ¥ = , find 2
1 2 3 6 p sin 2nx p æ sin nx ö
1 1 1 (i) I n = ò dx (ii) I n = ò ç ÷ dx
(i) 2 + 2 + 2 + ... upto ¥ 0 sin x 0 è sin x ø
1 3 5
Chap 03 Sequences and Series 285

101. Consider the sequence S = 7 + 13 + 21 + 31 + ... + T n , find 108. Show that,


n
the value of T 70 . (1 + 5 -1 ) (1 + 5 -2 ) (1 + 5 -4 ) (1 + 5 -8 ) ... (1 + 5 -2 )
3 3 3 5 n+1
1 æ 1 ö æ 1 ö
102. Find value of æç x + ö÷ + ç x 2 + 2 ÷ +...+ ç x n + n ÷ . = (1 - 5 - 2 )
è ø èx ø è ø 4
x x
n
2n
103. If am be the mth term of an AP, show that 109. Evaluate S = å n
(where a > 1).
n n=0 (a 2 + 1)
a12 - a 22 + a 32 - a 42 + ... + a 22n - 1 - a 22n = (a12 - a 22n ).
(2n - 1) 110. Find the sum to infinite terms of the series
104. If three unequal numbers are in HP and their squares are æ1ö æ2ö æ 2n - 1 ö
tan -1 ç ÷ + tan -1 ç ÷ + ... + tan -1 ç ÷ + ...
in AP, show that they are in the ratio è3ø è9 ø è 1 + 2 2n - 1 ø
1 + 3 : - 2 : 1 - 3 or 1 - 3 : - 2 : 1 + 3.
111. Find the sum to n terms, whose nth term is
105. If a1 , a 2 , a 3 , ..., an are in AP with a1 = 0, prove that tan [ a + (n - 1) b ] tan (a + nb ).
a3 a4 a æ1 1 1 ö n
n n
1
+ + ... + n - a 2 çç + + ... + ÷ 112. If åTr = (n + 1) (n + 2) (n + 3), find å .
a2 a3 an - 1 èa2 a3 an - 2 ÷ø 8
r =1 r = 1 Tr
an - 1 a
= + 2
a2 an - 1 113. If S 1 , S 2 , S 3 denote the sum of n terms of 3 arithmetic
series whose first terms are unity and their common
106. Balls are arranged in rows to form an equilateral triangle.
difference are in HP, prove that
The first row consists of one ball, the second row of two
balls and so on. If 669 more balls are added, then all the 2S 3 S 1 - S 1S 2 - S 2 S 3
n= .
balls can be arranged in the shape of a square and each of S1 - 2 S 2 + S 3
the sides, then contains 8 balls less than each side of the
triangle. Determine the initial number of balls.
114. Three friends whose ages form a GP divide a certain
sum of money in proportion to their ages. If they do that
107. If q 1 , q 2 , q 3 , ..., q n are in AP whose common difference three years later, when the youngest is half the age of
is d, then show that the oldest, then he will receive ` 105 more than he gets
sin d {sec q 1 sec q 2 + sec q 2 sec q 3 + ... now and the middle friend will get ` 15 more than he
+ sec q n - 1 sec q n } = tan q n - tan q 1 . gets now. Find the ages of the friends.

#L Sequences and Series Exercise 8 :


Questions Asked in Previous 13 Year’s Exam
n
This section contains questions asked in IIT-JEE, 117. Let a1 , a 2 , a 3 , ... be terms are in AP, if
AIEEE, JEE Main & JEE Advanced from year 2005 to a1 + a 2 + ... + a p p2 a6
year 2017. = , p ¹ q , then equals
a1 + a 2 + ... + aq q 2 a 21
115. If a, b, c are in AP and | a |, | b |, | c | < 1 and
[AIEEE 2006, 4.5M]
x = 1 + a + a 2 + ... + ¥ 41 7
(a) (b)
y = 1 + b + b 2 + ... + ¥ 11 2
2 11
(c) (d)
z = 1 + c + c 2 + ... + ¥ 7 41
Then, x , y, z will be in [AIEEE 2005, 3M] 118. If a1 , a 2 , ..., an are in HP, then the expression
(a) AP (b) GP (c) HP a1a 2 + a 2 a 3 + ... + an - 1an is equal to [AIEEE 2006, 6M]
(d) None of these
(a) n (a1 - an )
2 3 n
3 æ3ö æ3ö æ3ö (b) (n - 1 ) (a1 - an )
116. If an = - ç ÷ + ç ÷ - ... + ( -1)n - 1 ç ÷ and
4 è 4 ø è 4 ø è4ø (c) na1an
bn = 1 - an , then find the least natural number n 0 such (d) (n - 1 ) a1an
that bn > an , " n ³ n 0 . [IIT-JEE 2006, 6M]
286 Textbook of Algebra

119. Let Vr denotes the sum of the first r terms of an 122. Suppose four distinct positive numbers a1 , a 2 , a 3 , a 4 are
arithmetic progression whose first term is r and the in GP. Let b1 = a1 , b 2 = b1 + a 2 , b 3 = b 2 + a 3
common difference is (2 r - 1). Let T r = Vr + 1 - Vr - 2 and and b 4 = b 3 + a 4 .
Q r = T r + 1 - T r for r = 1, 2, K [IIT-JEE 2007, 4+4+4M] Statement 1 The numbers b1 , b 2 , b 3 , b 4 are neither in
(i) The sum V1 + V2 + ... + Vn is AP nor in GP.
1
(a) n (n + 1 ) (3n 2 - n + 1 ) Statement 2 The numbers b1 , b 2 , b 3 , b 4 are in HP.
12 [IIT-JEE 2008, 3M]
1
(b) n (n + 1 ) (3n 2 + n + 2 ) (a) Statement-1 is true, Statement-2 is true; Statement-2 is a
12 correct explanation for Statement-1
1
(c) n (2n 2 - n + 1 ) (b) Statement-1 is true, Statement-2 is true; Statement-2 is
2 not a correct explanation for Statement-1
1 (c) Statement-1 is true, Statement-2 is false
(d) (2n 3 - 2n + 3 )
3 (d) Statement-1 is false, Statement-2 is true
(ii) Tr is always
123. The first two terms of a geometric progression add upto
(a) an odd number (b) an even number
12 the sum of the third and the fourth terms is 48, if the
(c) a prime number (d) a composite number
terms of the geometric progression are alternately
(iii) Which one of the following is a correct statement? positive and negative, then the first term is
(a) Q1, Q2, Q3, K are in AP with common difference 5 [AIEEE 2008, 3M]
(b) Q1, Q2, Q3, K are in AP with common difference 6 (a) - 12 (b) 12 (c) 4 (d) -4
(c) Q1, Q2, Q3, K are in AP with common difference 11
124. If the sum of first n terms of an AP is cn 2 , then the sum
(d) Q1 = Q2 = Q3 = ...
of squares of these n terms is [IIT-JEE 2009, 3M]
2 2 2 2
120. Let A 1 , G 1 , H 1 denote the arithmetic, geometric and n ( 4n - 1 )c n ( 4n + 1 ) c
(a) (b)
harmonic means respectively, of two distinct positive 6 3
numbers. For n ³ 2, let A n - 1 , G n - 1 and H n - 1 has n ( 4n 2 - 1 )c 2 n ( 4n 2 + 1 ) c 2
(c) (d)
arithmetic, geometric and harmonic means as 3 6
A n , G n , H n , respectively. [IIT-JEE 2007, 4+4+4M] 125. The sum to infinity of the series
2 6 10 14
(i) Which one of the following statement is correct? 1+ + + + + ... is [AIEEE 2009, 4M]
3 32 33 34
(a) G1 > G2 > G3 > ... (b) G1 < G2 < G3 < K
(a) 6 (b) 2 (c) 3 (d) 4
(c) G1 = G2 = G3 = ...
126. Let S k , k = 1, 2, ..., 100, denote the sum of the infinite
(d) G1 < G3 < G5 < ... and G2 > G4 > G6 > ... k -1
(ii) Which of the following statement is correct? geometric series whose first term is and common
k!
(a) A1 > A2 > A3 > K 1 100 2 100
(b) A1 < A2 < A3 < K ratio is . Then, the value of + å | (k 2 - 3k + 1) S k | is
k 100 ! k = 2
(c) A1 > A3 > A5 > ... and A2 < A4 < A6 < ... [IIT-JEE 2010, 3M]
(d) A1 < A3 < A5 < K and A2 > A4 > A6 > K 127. Let a1 , a 2 , a 3 , ..., a11 be real numbers satisfying
(iii) Which of the following statement is correct?
a1 = 15, 27 - 2a 2 > 0 and ak = 2ak - 1 - ak - 2 for
(a) H1 > H 2 > H 3 > ...
a 2 + a 22 + ... + a11
2
(b) H1 < H 2 < H 3 < ... k = 3, 4, K , 11. If 1 = 90 , then the value
11
(c) H1 > H 3 > H 5 > ... and H 2 < H 4 < H 6 < K a + a 2 + ... + a11
(d) H1 < H 3 < H 5 < K and H 2 > H 4 > H 6 > ... of 1 is equal to
11 [IIT-JEE 2010, 3M]
121. If a geometric progression consisting of positive terms, 128. A person is to count 4500 currency notes. Let an denotes
each term equals the sum of the next two terms, then the number of notes he counts in the nth minute. If
the common ratio of this progression equals a1 = a 2 = ... = a10 = 150 and a10 , a11 , ... are in AP with
[AIEEE 2007, 3M]
1 1 common difference - 2, then the time taken by him to
(a) (1 - 5 ) (b) 5 count all notes is [AIEEE 2010, 8M]
2 2
1 (a) 34 min (b) 125 min
(c) 5 (d) ( 5 - 1 )
2 (c) 135 min (d) 24 min
Chap 03 Sequences and Series 287

k (k + 1)
129. The minimum value of the sum of real numbers 4n
-5
a ,a -4
, 3a -3 8
, 1, a and a 10
with a > 0 is [IIT-JEE 2011, 4M]
137. Let S n = å ( -1) 2 × k 2 , then S n can take value(s)
k =1 [JEE Advanced 2013, 4M]
130. A man saves ` 200 in each of the first three months of (a) 1056 (b) 1088 (c) 1120 (d) 1332
his service. In each of the subsequent months his saving 138. A pack contains n cards numbered from 1 to n. Two
increases by ` 40 more than the saving of immediately consecutive numbered cards are removed from the pack
previous month. His total saving from the start of and the sum of the numbers on the remaining cards is
service will be ` 11040 after [AIEEE 2011, 4M (Paper I)]
1224. If the smaller of the numbers on the removed cards
(a) 19 months (b) 20 months is k, then k - 20 is equal to [JEE Advanced 2013, 4M]
(c) 21 months (d) 18 months
100 139. If (10) 9 + 2 (11)1 (10) 8 + 3 (11) 2 (10) 7 + K+ (10)(11) 9
131. Let an be the nth term of an AP, if å a 2r = a and = k (10) 9 , then k is equal to [JEE Main 2014, 4M]
r =1
100 121 441
å a 2r - 1 = b, then the common difference of the AP is (a) 100 (b) 110 (c)
10
(d)
100
r =1 [AIEEE 2011, 4M (Paper II)]
a -b 140. Three positive numbers form an increasing GP. If the
(a) (b) a - b middle terms in this GP is doubled, the new numbers are
200
a -b in AP. Then, the common ratio of the GP is
(c) (d) b - a [JEE Main 2014, 4M]
100
(a) 2 - 3 (b) 2 + 3 (c) 2 + 3 (d) 3 + 2
132. If a1 , a 2 , a 3 ,…be in harmonic progression with a1 = 5 and b
a 20 = 25 . The least positive integer n for which an < 0 is 141. Let a, b, c be positive integers such that is an integer. If
a
(a) 22 (b) 23 [IIT-JEE 2012, 3M] a, b, c are in geometric progression and the arithmetic
(c) 24 (d) 25 a 2 + a - 14
mean of a, b, c is b + 2, the value of is
133. Statement 1 The sum of the series a +1
1 + (1 + 2 + 4 ) + ( 4 + 6 + 9 ) + (9 + 12 + 16) [JEE Advanced 2014, 3M]
+ K + (361 + 380 + 400) is 8000. 142. The sum of first 9 terms of the series
n
Statement 2 å (k 3 3 3
- (k - 1) ) = n for any natural 13 13 + 23 13 + 23 + 33
+ + + ... is
k =1 1 1+3 1+3+5 [JEE Main 2015, 4M]
number n. [AIEEE 2012, 4M] (a) 192 (b) 71 (c) 96 (d) 142
(a) Statement-1 is true, Statement-2 is true; Statement-2 is a
143. If m is the AM of two distinct real numbers l and n
correct explanation for Statement-1
(l , n > 1) and G 1 , G 2 and G 3 are three geometric means
(b) Statement-1 is true, Statement-2 is true; Statement-2 is
not a correct explanation for Statement-1 between l and n, then G 14 + 2G 24 + G 34 equals
[JEE Main 2015, 4M]
(c) Statement-1 is true, Statement-2 is false
(a) 4 l 2m 2n 2 (b) 4 l 2mn
(d) Statement-1 is false, Statement-2 is true
(c) 4 lm 2n (d) 4 lmn 2
134. If 100 times the 100th term of an AP with non-zero
common difference equals the 50 times its 50th term, 144. Suppose that all the terms of an arithmetic progression
then the 150th term of this AP is [AIEEE 2012, 4M] (AP) are natural numbers. If the ratio of the sum of the first
(a) 150 times its 50th term (b) 150 seven terms to the sum of the first eleven terms is6 : 11 and
(c) zero (d) -150 the seventh term lies between 130 and 140, then the
common difference of this AP is [JEE Main 2015, 4M]
135. If x, y, z are in AP and tan -1 x , tan -1 y , tan -1 z are also
in AP, then [JEE Main 2013, 4M]
145. If the 2nd, 5th and 9th terms of a non-eustant AP are in
GP, then the common ratio of this GP is
(a) 2 x = 3y = 6z (b) 6 x = 3y = 2z [JEE Main 2016, 4M]
(c) 6 x = 4y = 3z (d) x = y = z 7 8 4
(a) 1 (b) (c) (d)
4 5 3
136. The sum of first 20 terms of the sequence 0.7, 0.77, 0.777, …,
is [JEE Main 2013, 4M]
146. If the sum of the first ten terms of the series
2 2 2 2
7 7 æ 3ö æ 2ö æ 1ö æ 4ö 16
(a) (99 - 10 -20 ) (b) (179 + 10 -20 ) 2
ç1 ÷ + ç2 ÷ + ç3 ÷ + 4 + ç 4 ÷ + K is m, then
9 81 è 5ø è 5ø è 5ø è 5ø 5
7 7 m equal to [JEE Main 2016, 4M]
(c) (99 + 10 -20 ) (d) (179 - 10 -20 )
9 81 (a) 100 (b) 99 (c) 102 (d) 101
288 Textbook of Algebra

147. Let bi > 1 for i = 1, 2, ..., 101. Suppose loge b1 , loge b 2 , 148. For any three positive real numbers a, b and c ,
loge b 3 , ..., loge b101 are in Arithmetic Progression (AP)
with the common difference loge 2. Suppose 9 (25a 2 + b 2 ) + 25 (c 2 - 3ac ) = 15b (3a + c ). Then
[JEE Main 2017, 4M]
a1 , a 2 , a 3 , K , a101 are in AP. Such that, a1 = b1 and
(a) a, b and c are in GP
a 51 = b 51 . If t = b1 + b 2 + K + b 51 and
(b) b, c and a are in GP
s = a1 + a 2 + K + a 51 , then [JEE Advanced 2016, 3M] (c) b, c and a are in AP
(a) s > t and a101 > b101 (b) s > t and a101 < b101 (d) a, b and c are in AP
(c) s < t and a101 > b101 (d) s < t and a101 < b101

Answers
Exercise for Session 1
1. (c) 2. (d) 3. (b) 4. (c) 5. (a) 31. (a,c) 32. (b,d) 33. (b,c) 34. (a,b) 35. (c,d) 36. (a,c,d)
37. (a,d) 38. (a,b,c) 39. (a,c) 40. (b,c) 41. (a,c) 42. (c,d)
Exercise for Session 2 43. (b,c) 44. (a,c) 45. (a,d)
1. (b) 2. (a) 3. (a) 4. (b) 5. (c) 6. (c) 46. (c) 47. (d) 48. (a) 49. (a) 50. (a) 51. (b)
52. (a) 53. (c) 54. (b) 55. (d) 56. (b) 57. (c)
Exercise for Session 3 58. (c) 59. (b) 60. (d) 61. (d) 62. (b) 63. (d)
1. (b) 2. (d) 3. (b) 4. (c) 5. (d) 64. (a) 65. (b) 66. (b) 67. (a) 68. (b) 69. (c)
70. (3) 71. (5) 72. (8) 73. (4) 74. (2) 75. (7)
Exercise for Session 4
76. (3) 77. (9) 78. (1) 79. (0)
1. (c) 2. (c) 3. (c) 4. (d) 5. (a) 6. (a)
80. (A) ® (r, s); (B) ® (p, r); (C) ® (p, q)
Exercise for Session 5 81. (A) ® (r); (B) ® (p); (C) ® (q)
82. (A) ® (p,r,s,t); (B) ® (p,q,s,t); (C) ® (p,s,t)
1. (c) 2. (a) 3. (a) 4. (c) 5. (b) 6. (b)
7. (b) 8. (b) 9. (a) 10. (b) 83. (A) ® (r); (B) ® (p); (C) ® (r); (D) ® (q)
84. (a) 85. (a) 86. (b) 87. (a) 88. (d) 89. (c)
Exercise for Session 6 90. (a)
1. (b) 2. (d) 3. (b) 4. (c) 5. (a) 6. (a) 1 ïì a2 (1 - an ) b2 (1 - bn ) ïü
91. 2. 92. í - ý
7. (a) 8. (c) 9. (b) 10. (c) (a - b) ïî (1 - a) (1 - b) ïþ
p2 p2
Exercise for Session 7 95. - 5050 96. (i) (ii) 98. 93
8 12
1. (a) 2. (d) 3. (b) 4. (d) 5. (c) 6. (c)
99. 2 n + 1
-n-2 101. 5113
7. (a) 8. (a)
x (1 - x 3n )
3
(1 - x 3n ) 3x (1 - x n ) 3 (1 - x n )
102. + + + n
Exercise for Session 8 1- x 3 3n
(1 - x )
x 3
(1 - x) x (1 - x)
1. (c) 2. (c) 3. (b) 4. (a) 5. (c) 6. (b) 1
106. 1540 109.
7. (c) 8. (d) 9. (a) 10. (a) a-1
sin nb
Exercise for Session 9 - n tan b
p cos (a + nb ) cos a
1. (d) 2. (c) 3. (d) 4. (a) 5. (b) 6. (a) 110. 111.
4 tan b
7. (c)
n (n + 3 )
112. 114. 12, 18, 27 115. (c)
Chapter Exercises 2 (n + 1) (n + 2)
116. (7) 117. (d) 118. (d) 119. (i) (b), (ii) (d), (iii) (b)
1. (a) 2. (d) 3. (c) 4. (c) 5. (a) 6. (c)
120. (i) (c), (ii) (a), (iii) (b) 121. (d) 122. (c) 123. (a) 124. (c)
7. (b) 8. (d) 9. (a) 10. (c) 11. (a) 12. (b) 125. (c) 126. (3) 127. (0) 128. (a) 129. (8) 130. (c)
13. (d) 14. (a) 15. (a) 16. (c) 17. (a) 18. (b) 131. (c) 132. (d) 133. (a) 134. (c) 135. (d) 136. (b)
19. (a) 20. (d) 21. (c) 22. (c) 23. (c) 24. (b) 137. (a,d) 138. (5) 139. (a) 140. (b) 141. (4) 142. (c)
25. (a) 26. (a) 27. (a) 28. (c) 29. (b) 30. (c) 143. (c) 144. (a) 145. (d) 146. (d) 147. (b) 148. (c)
AL1 L1M1 a
3. Q = Þ L1M1 =

Solutions
AB BC n+1

2a
Similarly, L2M2 = A
n+1
3a
L3M3 = M1
n+1 L1
1. Q x, y , z are in HP. L2 M2
M M M L3 M3
1 1 1
\ , , are in AP. na
x y z Ln Mn =
1 1 1 1 n+1
\ - = - …(i) Ln Mn
x y y z L1M1 + L2M2 + ¼ + Ln Mn
B a C
a
yz 1 = (1 + 2 + 3 + ¼ + n )
= =a [say] (n + 1 )
y + z 1
+
1
a n (n + 1 ) na
y z = × =
(n + 1 ) 2 2
zx 1
= =b [say] 4. Q Sn = 1 + 22 + 333 + ¼ + nnnn
x + z 1 1 142¼ 3n
4
+ n terms
z x
\ Sn - Sn - 1 = nnnn ¼ n = n (111 ¼ 1 )
xy 1 1424 3 14243
and = =c [say] n times n times
x + y 1
+
1
n (10n - 1 )
x y = n (10n - 1 + 10n - 2 + ¼ + 10 + 1 ) =
1 1 10 - 1
-
1 1 1 1 1 1 \ 9 (Sn - Sn - 1 ) = n (10n - 1 )
+ + -
a -b y z z x x y a 5. Given that a, b, c are in GP.
Q = = =
b -c 1
-
1 1 1 a
-
1 1 1 1 Then, b 2 = ac …(i)
+ + y z
z x x y and equations ax 2 + 2bx + c = 0
[from Eq. (i)] anddx 2 + 2ex + f = 0 have a common root. …(A)
2
yz zx xy Now, ax + 2bx + c = 0
Hence, , , are in AP.
y + z z + x x + y Þ 2
ax + 2 ac x + c = 0 [ by Eq. (i)]
2. Qa1, a 2, a 3, ¼are in HP. Þ ( ax + c ) = 0 Þ 2
ax + c = 0
1 1 1 c
Þ , , , ¼are in AP. …(i) Þ x=- [repeated]
a1 a 2 a 3 a
n
Q fk = å ar - ak æ
By the condition (A), ç -
cö 2
÷ be the root of dx - 2ex + f = 0
r =1 è aø
n
Þ ak + fk = å ar = l [say] So, it satisfy the equation
r =1 2
æ cö æ cö
Þ a1 + f1 = a 2 + f 2 = a 3 + f 3 = ¼ = l d ç- ÷ + 2e ç - ÷+ f =0
è aø è aø
l l l
From Eq. (i), , , , ¼are also in AP. dc c d 2e f
a1 a 2 a 3 Þ - 2e + f =0 Þ - + =0
a a a ac c
a + f1 a 2 + f 2 a 3 + f 3
Þ 1 , , , ¼ are also in AP. d 2e f d f æe ö
a1 a2 a3 Þ - + = 0 Þ + =2 ç ÷
a b c a c èb ø
Subtracting from each term by 1, we get
d e f
f1 f 2 f 3 So, , , are in AP.
, , , ¼are also in AP. a b c
a1 a 2 a 3
1 3 7 15
1 1 1 6. Q Sn = + + + + ¼ n up to terms
Þ , , , ¼are in AP. 2 4 8 16
a1 a 2 a 3
æ 1ö æ 1ö æ 1ö
\ a1, a 2, a 3, ¼are in HP. = ç1 - ÷ + ç1 - ÷ + ç1 - ÷ + ¼ n up to terms
è 2 ø è 4 ø è 8ø
\ 2 a1 , 2 a 2 , 2 a 3 , ¼are not in AP/GP/HP.
æ 1ö æ 1ö æ 1ö æ 1ö
= ç1 - ÷ + ç1 - 2 ÷ + ç1 - 3 ÷ + ¼ + ç1 - n ÷
è 2ø è 2 ø è 2 ø è 2 ø
290 Textbook of Algebra

1æ 1 1 1 ö 1 1 1 1
=n - ç1 + + 2 + ¼ + n - 1 ÷ - -
2 è 2 2 2 ø h h1 3 2 -1 -1
D = 10 = = -
é 10 - 1 9 9 ´ 6 54
æ1ö ù
n
1 ê1 - ç ÷ ú 1 1 1 1 æ -1 ö 1 1
ê è2ø ú = + 6D Þ = + 6ç ÷ = -
=n - + ë û
1 So,
h7 h1 l7 2 è 54 ø 2 9
2 æ 1ö
ç1 - ÷ 1 7 18
è 2ø = Þ h7 =
h7 18 7
é a (1 - r n ) ù
ê by sum GP ,Sn = , if 0 < r < 1 ú 7 18
a 4h7 = ´ =6
ë 1 -r û
So,
3 7
1 p 1 - sin 2nx p 1 - sin 2nx
=n -1 + = n - 1 + 2 -n 9. Q I n = ò dx Þ I n = ò dx
2n 0 1 - cos 2 x 0 2 sin 2 x

7. Let triangle be the area of DPQR. Þ I n + 1 + I n - 1 - 2I n


P [1 - sin 2 (n + 1 ) x + 1 - sin 2 (n - 1 ) x - 2
1 p + 2 sin 2nx ]
r h1
q
=
2 ò0 sin 2 x
dx
h2
1 p [sin 2nx- sin 2 (n + 1 ) x ] + [sin 2nx - sin 2 (n - 1 ) x ]
h3 =
2 ò0 sin 2 x
dx
Q R
p 1 p - 2 cos (2n + 1 ) x sin ( x ) + 2 cos (2n - 1 ) x sin x
=
2 ò0 sin 2 x
dx
1 p sin x [cos (2n - 1 ) x - cos (2n + 1 ) x ]
\ D= ´ p ´ h1 [h1, h2, h3 are altitudes]
2 =ò dx
0 sin 2 x
2D
Þ h1 = …(i) p 2 sin 2nx sin x p 2
p =ò dx = 2 ò sin 2nx dx = [ - cos 2nx ]p0
0 sin x 0 2n
2D
Similarly, h2 = …(ii) 1
q = - (1 - 1 ) = 0
n
2D
and h3 = …(iii) \ I n + 1 + I n - 1 = 2I n
r
Þ I n - 1 + I n , I n + 1 are in AP.
According to the question, sin P , sin Q, sin R are in AP.
\ I 1, I 2, I 3,¼are in AP.
Then, kp, kq, kr are in AP [by sine rule]
10. Given that,
Þ p, q, r are in AP.
2D 2D 2D a (b - c ) x 2 + b (c - a ) xy + c (a - b ) y 2 is perfect square.
Þ , , are in AP. [ by Eqs. (i), (ii) and (iii)]
h1 h2 h3 \ b 2 (c - a ) 2 = 4a (b - c ) × c (a - b )

Þ
1 1 1
, , are in AP. Þ b 2 (c - a ) 2 = 4ac (a - b ) (b - c )
h1 h2 h3 Þ [a (b - c ) + c (a - b )]2 = 4ac (a - b ) (b - c )
Þ h1, h2, h3 are in HP. [Q a (b - c ) + b (c - a ) + c (a - b ) = 0 ]
Þ Altitudes are in HP.
Þ [a (b - c ) - c (a - b )]2 = 0
8. Given that, a1, a 2, ¼, a10 be in AP.
Þ a (b - c ) - c (a - b ) = 0
Let d be the common difference of AP. Þ ab - ac - ca + bc = 0 Þ b (a + c ) = 2ac
a - a1
\ d = 10 Þ b=
2ac
10 - 1 a+b
3 -2
d = [given that, a1 = h1 = 2 and a10 = h10 = 3] Þ a, b, c are in HP.
9 2
æ 1ö æ 1ö
1 11. Let S = 1 + 2 ç1 - ÷ + 3 ç1 - ÷ + ¼ + ¥
d = è n ø è nø
9
3 1 7 æ 1ö æ 1ö æ 1ö
\ a 4 = a1 + 3d = 2 + = 2 + = ç1 - ÷ S = ç1 - ÷ + 2 ç1 - ÷ + ¼ + ¥
9 3 3 è nø è nø è nø
Now, h1, h2, ¼, h10 be in HP. - - -
So, common difference of respective AP. æ 1ö æ 1ö æ 1ö
2
S ç1 - 1 + ÷ = 1 + ç1 - ÷ + ç1 - ÷ + ¼ + ¥
è n ø è n ø è nø
Chap 03 Sequences and Series 291

2
S 1 é a ù é n - 1 æn - 1 öù
Þ = êS ¥ = by GP ú
é n (n + 1 ) ù
2 ê 2 çè 2 + 1 ÷ø ú
n æ 1ö ë 1 - r û =ê
1 - ç1 - ÷
è nø ë 2 úû - 16 × ê 2
ú
ê ú
n êë úû
Þ S=
1 n 2 (n + 1 ) 2 4 (n - 1 ) 2 (n + 1 ) 2 (n + 1 ) 2 2
= - = [n - (n - 1 ) 2 ]
n 4 16 4
Þ S = n2 (n - 1 ) 2 (2n - 1 ) (n + 1 ) 2
= × (2n - 1 ) (1 ) =
æ 7ö 4 4
12. Q log 3 2, log 3 (2 x - 5) and log 3 ç2 x - ÷ are in AP. …(i)
è 2ø 15. Let the sides of right angled triangle be
7 (a - d ), a, (a + d ) (a > d ).
For defined, 2 x - 5 > 0 and 2 x - >0 A
2
x
\ 2 >5 …(ii)
7 (a + b )
From Eq. (i), 2, 2 x - 5, 2 x - are in GP. (a – d )
2
æ 7ö
\ (2 - 5 ) 2 = 2 × ç2 x - ÷
x
è 2ø C
B
2x x a
Þ 2 - 12 × 2 + 32 = 0
By Pythagoras theorem,
Þ (2 x - 8 ) (2 x - 4 ) = 0
(a + d ) 2 = a 2 + (a - d ) 2
\ 2 x = 8, 4 a 2 + d 2 + 2ad = a 2 + a 2 + d 2 - 2ad
x 3 x
Þ 2 =8 =2 ,2 ¹ 4 [fromEq. (ii)] a 2 = 4ad
\ x =3 a = 4d [since a ¹ 0] …(i)
13. Qa, b, c are positive prime numbers. a 4d 4
According to the question, sin A = = =
Let a , b , c are 3 terms of AP. [not necessarily consecutive] a + d 5d 5
a - d 3d 3
Then, a = A + (p - 1) D …(i) sin C = = =
a + d 5d 5
b = A + (q - 1 ) D …(ii)
16. T6 = 2
c = A + (r - 1 ) D …(iii) Let d be common difference of AP and a be the first term of
[A and D be the first term and common difference of AP] AP.
a - b = (p - q ) D …(iv) T6 = 2
Þ a + 5d = 2 …(i)
b - c = (q - r ) D …(v)
Let A = a1a 4a 5
c - a = (r - p ) D …(vi) A = a (a + 3d ) (a + 4d )
On dividing Eq. (iv) by Eq. (v), we get [using Tn = a + (n - 1 ) d and from Eq. (i) a = 2 - 5d ]
A = (2 - 5d ) (2 - 2d ) (2 - d )
a - b p -q
= …(vii) A = 8 - 32d + 34d 2 - 10d 3
b - c q -r
dA
Since, p, q, r are natural numbers and a, b, c are positive prime For max and min values of A, =0
numbers, so dd
Eq. (vii) does not hold. - 30d 2 + 68d - 32 = 0 Þ 15d 2 - 34d + 16 = 0
So, a , b and c cannot be the 3 terms of AP. 15d 2 - ( 24d + 10d ) + 16 = 0
[not necessarily consecutive]
15d 2 - 24d - 10d + 16 = 0
Similarly, we can show that a , b , c cannot be any 3 terms
3d ( 5d - 8 ) - 2 ( 5d - 8 ) = 0
of GP and HP. [not necessarily, consecutive]
( 5d - 8 ) ( 3d - 2 ) = 0
14. Given that n is an odd integer greater than or equal to 1.
8 2
Sn = n 3 - (n - 1 ) 3 + (n - 2 ) 3 - ¼ + ( - 1 )n - 11 3 d = or d =
5 3
= 1 3 - 2 3 + ¼ + (n - 2 ) 3 - (n - 1 ) 3 + n 3 2 d A 2
For d = , >0
[Qn is odd integer, so (n - 1 ) is even integer] 3 dd 2
æ n -1 ö 2
= (1 + 2 + ¼ + n 3 ) - 2 × 2 3 ç1 3 + 2 3 + ¼ +
3 3
terms÷ So, A is least for d = .
è 2 ø 3
292 Textbook of Algebra

17. Given, common difference ¹ 0 On subtracting Eq. (ii) from Eq. (i), we get
S 3n = S 4n - S 3n (a 2 - a1 ) + (a 4 - a 3 ) + ¼ + (a 200 - a199 ) = a - b
Þ 2 × S 3n = S 4n [ let Sn = Pn 2 + Qn] d + d + ¼ up to 100 terms = a - b
Þ 2 × [ P (3n ) 2 + Q(3n )] = P ( 4n ) 2 + Q( 4n ) [beacause an be the nth term of AP with common difference d ]
100 d = a - b
Þ 2 Pn 2 + 2 Qn = 0
a -b
or Q = - nP …(i) d =
100
2
S 2n P (2n ) + Q(2n ) 21. Given that, a1, a 2, a 3, a 4 , a 5 are in HP.
\ =
S 4n - S 2n [ P ( 4n ) 2 + Q( 4n )] - [ P (2n ) 2 + Q(2n )] 1 1 1 1 1
\ , , , , are in AP.
2n (2nP + Q ) 2nP + Q a1 a 2 a 3 a 4 a 5
= = 1 1 1 1 1 1 1 1
12 Pn 2 + 2nQ 6nP + Q Þ - = - = - = - =d [say]
a 2 a1 a 3 a 2 a 4 a 3 a 5 a 4
2nP - nP 1
= = [from Eq. (i)] \ a1 - a 2 = a1a 2 d Þ a 2 - a 3 = a 2 a 3d
6nP - nP 5
a 3 - a 4 = a 3a 4d Þ a 4 - a 5 = a 4 a 5d
18. Let f ( x ) = ( x - 1) ( x - 2) ( x - 3) ¼ ( x - n ) On adding all, we get
= xn - S1xn - 1 + S 2 xn - 2 - ¼ + ( - 1 )n (1 × 2 × 3 ¼ n ) æ 1 1ö
a1 - a 5 ç - ÷
a a1 ÷
So, coefficient of xn - 2 in f ( x ) = S 2 = (1 × 2 + 1 × 3 + ¼) a1a 2 + a 2 a 3 + a 3a 4 + a4 a 5 = = a1a 5 ç 5 = 4 a1a 5
d ç d ÷
ç ÷
= Sum of product of first n natural number taken 2 at time è ø
1 22. Q(1 + a ) (1 + b ) (1 + c ) (1 + d )
= [(1 + 2 + ¼ + n ) 2 - (1 2 + 2 2 + ¼ + n 2 )]
2 = 1 + a + b + c + d + ab + ac + ad + bc + bd + cd
1 éì n (n + 1 )ü n (n + 1 ) (2n + 1 ) ù
2
+ abc + abd + cda + cdb + abcd [16 terms]
= êí ý - ú
2 êëî 2 þ 6 úû \ AM ³ GM
(1 + a ) (1 + b ) (1 + c ) (1 + d )
1 n (n + 1 ) é n (n + 1 ) 2n + 1 ù ³ (a 8b 8c 8d 8 )1/16
= × êë - 16
2 2 2 3 úû
= (abcd )1/2 = (1 )1/2 = 1 [Q abcd = 1 ]
1 n (n + 1 ) é 3n 2 + 3n - 4n - 2 ù (1 + a ) (1 + b ) (1 + c ) (1 + d )
= × ê ú Þ ³1
2 2 ë 6 û 16
n (n + 1 ) (3n 2 - n - 2 ) n (n + 1 ) (3n + 2 )(n - 1 ) Þ (1 + a ) (1 + b ) (1 + c ) (1 + d ) ³ 16
= =
24 24 \Minimum value of (1 + a ) (1 + b ) (1 + c ) (1 + d ) is 16.
n (n 2 - 1 ) (3n + 2 ) 23. Qa, b, c are in AP.
=
24 \ 2b = a + c …(i)
19. If last term of nth row is Tn , then Now, (a + 2b - c ) (2b + c - a ) (c + a - b )
Let S = 1 + 5 + 11 + 19 + ¼ + Tn = (a + a + c - c ) (a + c + c - a ) (2b - b ) [from Eq. (i)]
S = 1 + 5 + 11 + ¼ + Tn - 1 + Tn = (2a ) (2c ) (b ) = 4abc
- - - - - - \ l=4
0 = 1 + 4 + 6 + 8 + ¼ + n terms - Tn 24. a1, a 2,¼, an are in GP with first term a and common ratio r.
aa aa an - 1 an
Tn = 1 + 2 ( 2 + 3 + 4 + ¼+ (n - 1 ) terms) Sn = 2 1 2 2 + 2 2 3 2 + ¼ + 2 …(i)
(n - 1 ) a1 - a 2 a 2 - a 3 an - 1 - an2
=1 + 2 [2 × 2 + (n - 2 ) × 1] 144444424444443
2 (n - 1) times

= 1 + (n - 1 ) (n + 2 ) an - 1 an an - 1 an
Tn = =
= 1 + n2 + n - 2 an2 - 1 - an2 (an - 1 - an ) (an - 1 + an )
Þ Tn = n 2 + n - 1 1
=
\ 2
T60 = (60 ) + 60 - 1 = 3600 + 59 = 3659 æ a ö æ an - 1 ö
ç1 - n ÷ ç1 + ÷
100 è an - 1 ø è an ø
20. Given that, å a 2r = a
1 r
r =1 = = [ by GP]
Þ a 2 + a 4 + ¼ + a 200 = a …(i) æ 1 ö (r + 1 ) (1 - r )
(1 - r ) ç1 + ÷
100 è rø
and å a 2r - 1 = b n n
r (n - 1 ) r
r =1 \ Sn = åTn = å 2
=
n = 2 (1 - r ) (1 - r 2 )
Þ a1 + a 3 + ¼ + a199 = b n=2
Chap 03 Sequences and Series 293

25. According to the question, for AP 1 1


a+³2 Þ a + + 3 ³5
S10 = 4 S 5 a b
10 5 é n ù 1 1
(2a + 9d ) = 4 × (2a + 4d ) ê by Sn = [2a + (n - 1 ) d ]ú Similarly, b + ³ 2 Þ b + + 3 ³ 5
2 2 ë 2 û b b
1 1
10a + 45d = 20a + 40d and c + ³2 Þ c + + 3 ³5
c c
a 1
Þ 10a = 5d Þ = æ 1 öæ 1 öæ 1 ö
d 2 \ ça + + 3 ÷ çb + + 3 ÷ çc + + 3 ÷ ³ 125
è a øè b øè c ø
26. Q cos ( x - y ), cos x, cos ( x + y ) are in HP.
So, A ³ 5 × 5 × 5 Þ A ³ 125
2 cos ( x - y ) cos ( x + y ) Minimum value of A is 125.
\ cos x =
cos ( x - y ) + cos ( x + y )
30. a1, a 2,¼are
, in AP and q1, q 2, ¼ are in GP.
2 (cos2 x - sin 2 y ) a1 = q1 = 2 and a10 = q10 = 3
Þ cos x =
2 cos x cos y Let d be the common diference of AP
Þ cos2 x cos y = cos2 x - sin 2 y 3 -2 1
i.e., d = =
9 9
Þ cos2 x (1 - cos y ) = sin 2 y
1 8
= (1 + cos y ) (1 - cos y ) Then, a 7 = a1 + 6d = 2 + 6d = 2 + 6 ´ =
9 3
Þ cos2 x = (1 + cos y ) [Q1 - cos y ¹ 0] a19 = a1 + 18d = 2 + 18d
y 1 36
Þ cos2 x = 2 cos2 = 2 + 18 ´ = =4
2 9 9
1/ 9
æy ö æ3ö
Þ cos2 x sec 2 ç ÷ = 2 Let r be the common ratio of GP i.e., r = ç ÷
è2ø è2ø
æy ö Then, q 7 = q1r 6 = 2r 6
\ cos x sec ç ÷ = ± 2
è2ø 6´
1
2 /3
æ3ö 9 æ3ö
27. Let 11 AM’s are A1, A2, A 3,¼, A 11. = 2× ç ÷ =2 ç ÷
è2ø è2ø
Given, 28, A1, A2, A3, ¼, A11, 10 are in AP. 9´
1
9 9 æ3ö 9
10 - 28 3 q10 = q1r = 2r = 2 × ç ÷ =3
\ d = =- è2ø
12 2
q19 = q1 × r 18 = 2 × r 18
3
\ A i = 28 + id = 28 - i 18 ´
1
18 / 9
2 æ3ö 9 æ3ö 9
= 2× ç ÷ = 2ç ÷ =
It is clear that A2, A4 , A6, A8, A10 are integral AM’s. è2ø è2ø 2
Hence, number of integral AM’s are 5. 8 9
(a) a 7q19 = ´ = 12, which is an integer.
28. Q x, y , z are in GP [ x, y , z > 1 ] 3 2
2 /3 2 /3
\ ln x, ln y , ln z are in AP æ3ö æ3ö
(b) a19q 7 = 4 ´ 2 ´ ç ÷ =8 ç ÷ ,
è2ø è2ø
and 2 x, 4 x, 6 x are also in AP. [ x > 1]
By property, which is not an integer.
2 x + ln x, 4 x + ln y , 6 x + ln z are also in AP. 8 9
(c) a 7q19 = ´ = 12; a19q10 = 4 ´ 3 = 12
1 1 1 3 2
\ , , are in HP. 1 1 1
2 x + ln x 4 x + ln y 6 x + ln z 31. Qa(n ) = 1 + + + ¼ + n
2 3 2 -1
(a 2 + 3a + 1 ) (b 2 + 3b + 1 ) (c 2 + 3c + 1 ) æ1 1ö æ 1 1 1 1ö
29. Let A = =1 + ç + ÷ + ç + + + ÷
abc è2 3ø è 4 5 6 7ø
æ a 2 + 3a + 1 ö æ b 2 + 3b + 1 ö æ c 2 + 3c + 1 ö æ1 1ö 1
=ç ÷ç ÷ç ÷ + ç + ¼+ ÷ + ¼+ n
è a øè b øè c ø è8 15 ø 2 -1
æ 1ö æ 1ö æ 1ö æ1 1 ö æ1 1 1 1 ö 1
= ça + 3 + ÷ çb + 3 + ÷ çc + 3 + ÷, =1+ ç + 2 ÷+ç + + + ÷ + ¼+ n
è aø è bø è cø è 2 2 - 1 ø è 22 5 6 23 - 1 ø 2 -1
where a, b, c Î R + . \ a(n ) < 1 + 1 + ¼+ n terms
1 Þ a(n ) < n
Applying AM ³ GM on a and ,
a Þ a (100 ) < 100
294 Textbook of Algebra

1 æ1 1 ö æ1 1 1 1ö 1 xy
Also, a(n ) = 1 + + ç + ÷ + ç + + + ÷+ ¼ + n Þ z=
2 è3 4ø è5 6 7 8ø 2 -1 xy - 1
Þ xyz = z + xy
1 æ 1 1ö æ 1 1 1 1ö xyz = z + x + y
=1 + + ç 1 + 2÷ + ç 2 + + + 3÷ and [from Eq. (iii)]
2 è2 + 1 2 ø è2 + 1 6 7 2 ø a+c
34. Qa, b, c are in AP Þ b = …(i)
æ 2
1 1ö 1
+ ¼ + ç n -1 + ¼+ n ÷ - n 2 2 2
anda , b , c are in HP.
è2 +1 2 ø 2
2a 2c 2
1 2 4 2 1 n -1 Þ b2 = …(ii)
a(n ) > 1 + + + + ¼+ n - n a2 + c2
2 4 8 2 2
Þ b 2 {a 2 + c 2 } = 2a 2c 2
æ 1ö n n
a(n ) > ç1 - n ÷ + Þ a(n ) > Þ b 2 {(a + c ) 2 - 2ac } = 2a 2c 2 [from Eq. (i)]
è 2 ø 2 2
\ a (200 ) > 100 Þ b 2 ( 4b 2 - 2ac ) = 2a 2c 2
32. Þ 2b 4 - ac (b 2 ) - a 2c 2 = 0

1 n 2n – 1 Þ (b 2 - ac ) (2b 2 + ac ) = 0
middle term
If b 2 - ac = 0
In a AP of (2n - 1 ) terms, nth term = a a, b, c are in GP.
In a GP of (2n - 1 ) terms, nth term = b But given a, b, c are in AP.
In a HP of (2n - 1 ) terms, nth term = c \ a =b =c
a, b, c will be arithmetic mean, geometric mean, harmonic and if 2b 2 + ac = 0
mean, respectively.
-a
So, a ³ b ³ c and b 2 = ac then , b, c are in GP.
2
p
33. Q 0 < f < 35. According to the question, x, x 2 + 2 and x 3 + 10 are in GP.
2
So, ( x 2 + 2 ) 2 = x ( x 3 + 10 )
\ 0 < sin f < 1 and 0 < cos f < 1
¥ Þ x 4 + 4 + 4 x 2 - x 4 - 10 x = 0
\ x= å cos2n f = 1 + cos2 f + cos4 f + ¼ + ¥
Þ 4 x 2 -10 x + 4 = 0
n=0

1 1 Þ 2x 2 - 5x + 2 = 0
= =
1 - cos2 f sin 2 f Þ 2
2x - 4x - x + 2 = 0
1 Þ 2x (x - 2) - 1 (x - 2) = 0
or sin 2 f = …(i)
x Þ ( x - 2 ) (2 x - 1 ) = 0
¥
1
and y = å sin 2n f = 1 + sin 2 f + sin 4 f + ... + ¥ Þ x = 2 or x =
n =0 2
1 1 For x = 2, first 3 terms are 2, 6, 18.
= = So, 4th term of GP = 2 × (3 ) 3 = 54
1 - sin 2 f cos2 f
1 1 1 9 81
or cos2 f = …(ii) For x = , first 3 terms are , , .
y 2 2 4 8
3
From Eqs. (i) and (ii), 1 æ9ö 1 729 729
So, T4 = ç ÷ = ´ =
1 1 2 è2ø 2 8 16
sin 2 f + cos2 f = +
x y 36. Let n consecutive odd numbers be
1 1 2k + 1, 2k + 3, 2k + 5, ..., 2k + 2n - 1
1= +
x y According to question, sum of these n numbers
n
\ xy = x + y …(iii) = [ 2k + 1 + 2k + 2n - 1 ] = n (2k + n )
¥ 2
and z= å cos2n f sin2n f = n 2 +2kn = (n + k ) 2 - k 2
n=0
Given that, (n + k ) 2 - k 2 = 25 2 - 11 2
= 1 + cos2 f sin 2 f + cos4 f sin 4 f + ...
Þ n + k = 25 and k = 11 Þ n = 14 and k = 11
1 1
= 2 2
= [from Eqs. (i) and (ii)] So, first term = 2k + 1 = 23
1 - sin f cos f 1 - 1
Last term = 2k + 2n - 1 = 22 + 28 - 1 = 22 + 27 = 49
xy
Chap 03 Sequences and Series 295

37. QG = 6 and G 2 = AH 1 1 1
40. E= + + + ...
36 12 22 32
Þ H = 1 1
A E <1 + + + ...
Given, 90 A + 5 H = 918 (1 ) (2 ) (2 ) (3 )
36 10 æ 1ö æ1 1ö
Þ 90 A + 5 ´ = 918 Þ 5 A + = 51 E < 1 + ç1 - ÷ + ç - ÷ + ...
A A è 2ø è2 3ø
Þ 5 A 2 - 51 A + 10 = 0 Þ( A - 10 ) ( 5 A - 1 ) = 0 E <2 …(i)
1 1 1
\ A = 10, E >1 + + + ...
5 (2 ) (3 ) (3 ) ( 4 )
1 æ1 1ö æ1 1 ö
38. Q Tn = E > 1 + ç - ÷ + ç - ÷ + ...
(2n -1 ) (2n + 1 ) (2n + 3 ) (2n + 5 ) è2 3ø è3 4ø
n 1 3
\ Sn = å Tn a E >1 + ; E >
2 2
n =1
41. QS1 = { 0}
1 n (2n + 5 ) - (2n - 1 )
Sn = å
6 n = 1 (2n - 1 ) (2n + 1 ) (2n + 3 ) (2n + 5 )
ì 3 5ü
S2 = í , ý
î 2 2þ
1 n æ 1
= åç
6 n = 1 è (2n - 1 ) (2n + 1 ) (2n + 3 )
ì 8 11 14ü
S3 = í , , ý
î3 3 3 þ
1 ö ì 15 19 23 27ü
- ÷ S4 = í , , , ý
(2n + 1 ) (2n + 3 ) (2n + 5 ) ø î 4 4 4 4þ
1æ 1 1 ö M M M
= ç - ÷
6 è 1 × 3 × 5 (2n +1 ) (2n + 3 ) (2n + 5 ) ø Let S = 3 + 8 + 15 + ... + T19
1 S = 3 + 8 + ... + T18 + T19
1 - - - - -
= - 6
90 (2n + 1 ) (2n + 3 ) (2n + 5 ) 0 = 3 + 5 + 7 + ... + 19 terms - T19
1 T19 = 3 + 5 + 7 + ... + 19 terms
\ l= 19 19
6 \ T19 = (6 + 18 ´ 2 ) = ´ 42 = 399
2 2
and f (n ) = (2n + 1 ) (2n + 3 ) (2n + 5 )
ì 399 419 439 ü
\ f ( 0 ) = 15 S 20 = í , , , ...ý
f (1 ) = 105 î 20 20 20 þ
439
æ1ö æ1 ö æ1 ö æ1 ö 640 \ Third element of S 20 =
and f ( l ) = f ç ÷ = ç + 1 ÷ ç + 3 ÷ ç + 5 ÷ = 20
è6ø è3 ø è3 ø è3 ø 27
20 1
1 1 Sum of elements of S 20 = ´ [ 2 ´ 399 + 19 ´ 20 ]
39. QS = 1 + (1 + 2 ) 2 + (1 + 2 + 3 ) 2 + ... 2 2
(1 + 3 ) (1 + 3 + 5 )
= 399 + 190 = 589
1
Tn = × (1 + 2 + 3 + 4 + ... n terms) 2 æ 1ö
n
(1 + 3 + 5 + 7 + ...n terms) 42. (a) QS = ç1 + ÷
è nø
2
1 æ n (n + 1 ) ö (n + 1 ) 2 2 3
= ×ç ÷ = æ1ö æ1ö æ1ö æ1ö
n
én ù è 2 ø 4 S = 1 + nC1 ç ÷ + nC 2 ç ÷ + nC 3 ç ÷ + ... + nCn ç ÷
êë 2 [2 × 1 + (n - 1 ) × 2 ]úû èn ø èn ø èn ø ènø
1 n (n - 1 ) æ 1 ö n (n - 1 ) (n - 2 ) æ1ö
(7 +1 ) 2 64 S = 1 + n× + ç 2÷ + ç 3÷
(a) T7 = = = 16 n 2! èn ø 3! èn ø
4 4
n (n - 1 ) ... 1 æ 1 ö
10
æn + 1ö 1
2 10
+ ... + ç n÷
(b) S10 = å ç ÷ = å (n + 2n + 1 )
2
n! èn ø
n =1
è 2 ø 4 n =1
1 æ 1ö 1 æ 1ö æ 2ö
S = 1 + 1 + ç1 - ÷ + ç1 - ÷ ç1 - ÷ + ...
1 æ 10 10 10 ö 2! è n ø 3! è nø è nø
= ç å n 2 + 2 å n + å 1÷
4 ç ÷ 1 æ 1ö æ 2ö æ n - 1ö
èn = 1 n =1 n =1 ø +
ç1 - ÷ ç1 - ÷ ... ç1 - ÷
1 æ 10 ´ 11 ´ 21 2 ´ 10 ´ 11 n! è nø è nø è n ø
ö
= ç + + 10 ÷ 1 1 1
4 è 6 2 ø S < 1 + 1 + + + ... +
1 505 2! 3! n!
= ( 385 + 110 + 10 ) =
4 4
296 Textbook of Algebra

1 1 1 4
S <1 + 1 + + + ... + 1 1 1 1 1 20 + 15 + 12 + 10
1 ×2 1 ×2 ×3 1 × 2 × 3 ... n
a2 = å 2 + a
= + + + =
3 4 5 6 60
a =1
1 1 1
S <1 + 1 + + + ... + =
57 19
=
2 2 ×2 2 × 2 ... 2 60 20
1 æ 1 1 ö
1- n Now, an + 1 - an = ç +
1
+ ... +
2 Þ S < 1 + 2 æç1 - 1 ö÷ ÷
S <1 + èn + 2 n + 3 3n + 3 ø
1 è 2n ø
1-
2 æ 1 1 1ö
1 -ç + + ... + ÷
S < 3 - n - 1 \S < 3, " n èn + 1 n + 2 3n ø
2 1 1 1 1
= + + -
1 æ 1ö 1 æ 1ö æ 2ö 3n + 1 3n + 2 3n + 3 n + 1
Also, S = 1 + 1 + ç1 - ÷ + ç1 - ÷ ç1 - ÷ + ...
2! è n ø 3! è nø è nø 1 1 2
= + -
1 æ 1ö æ 2ö æ n - 1ö 3n + 1 3n + 2 3 (n + 1 )
+ ç1 - ÷ ç1 - ÷ ... ç1 - ÷
n! è nø è nø è n ø
9n 2 + 15n + 6 + 9n 2 + 12n + 3 - 18n 2 - 18n - 4
S > 1 + 1; S > 2 =
(3n + 1 ) (3n + 2 ) (3n + 3 )
\ S is bounded.
2n + 1 9n + 5
(b) Q an = =
n+2 (3n + 1 ) (3n + 2 ) (3n + 3 )
æ 1 ö æ n-2 1 ö
3
For n1 = 1, a1 = = 1, 44. Sn ( x ) = ç xn - 1 + ÷ + 2 çx + n -2÷
3 è xn - 1 ø è x ø
5 æ 1ö
for n = 2 , a 2 = = 1.25 + ... + (n - 1 ) ç x + ÷ + n
è xø
4
M M M Let S ¢ = xn - 1 + 2 xn - 2 + 3 xn - 3 + ... + (n - 1 ) x
Now, an + 1 - an > 0 Þ an + 1 > an S¢
= xn - 2 + 2 xn - 3 + ... + (n - 2 ) x + (n - 1 )
\ an represents the increasing sequence x
æ 1ö - - - - - -
n ç2 + ÷
2n + 1 è nø 2 æ 1ö
lim an = lim = lim = =2 S ¢ ç1 - ÷ = xn - 1 + xn - 2 + xn - 3 + ... + x - (n - 1 )
n®¥ n®¥ n + 2 n®¥ æ 2ö 1 è xø
n ç1 + ÷
è nø
(x - 1) x × (xn - 1 - 1)
\ {an } is bounded sequence. S¢ = - (n - 1 )
x (x - 1)
n2
æ 1ö x2 (n - 1 ) x
(c) Q an = ç1 + ÷ Þ S¢ = (xn - 1 - 1) -
è nø 2
(x - 1)
(x - 1)
For n = 1, a1 = 2, 1 2 (n - 1 )
4 4 S ¢¢ = n - 1 + n - 2 + ... +
æ 1ö æ3ö 3 4 81 x x x
for n = 2, a 2 = ç1 + ÷ = ç ÷ = 4 = = 5.06
è 2ø è2ø 2 16 1
Þ S ¢¢ = n [ x + 2 x 2 + ... + (n - 1 ) xn - 1 ]
[approximate] x
1
æ 1ö
n2 lim ´ n2 lim n 1 [(n - 1 ) xn - nxn - 1 + 1 ]
lim ç1 + ÷ = en ®¥ n = en ® ¥ = e¥ = ¥ = n [similarly as above]
n®¥è nø x (x - 1)2
\ {an } represents unbounded sequence. \ Sn ( x ) = S ¢ + S ¢¢ + n
2
(d) Qan = tan n 1 æ xn - 1 ö
Þ Sn ( x ) = ç ÷ …(i)
n3 2 x (n - 1) è x -1 ø
an = n + + n 5 + ... + ¥
3 15
For n = 1, S1 ( x ) = 1
and we know that - ¥ < tan n < ¥ 2
So, {an } is unbounded sequence. 1 æ x100 - 1 ö
S100 ( x ) = ç ÷
1 1 1 1 x 99 è x - 1 ø
43. Q an = + + + ... +
n+1 n+2 n+3 3n 45. Let the AP start with n and common difference d, then
1 1 1 1 according to question,
an = + + + ... +
n+1 n+2 n+3 n + 2n n + 5d = 32
2n n = 32 - 5d …(i)
1
an = å n+a
and1072 < n + (n + d ) + ... + (n + 19d ) < 1162
a =1
Chap 03 Sequences and Series 297

19 ´ 20 Þ 1 £ 16t 2 - 95t + 1 < 8t + 2


1072 < 20n + d < 1162
2 Þ t = 6 Þn = 50 and p = 7
1072 < 640 - 100d + 190d < 1162
49. Hence, the value of n lies in ( 41,51).
432 < 90d < 522
50. Hence, removed numbers are 7 and 8.
4.8 < d < 5.8
50 (50 + 1 )
Let d is natural number, so d = 5 51. Sum of all numbers = = 1275
2
\ n = 32 - 5 ´ 5 = 7
Sol. (Q. Nos. 52 to 54)
First term is 7 and common difference is 5.
Let A = { A - D, A, A + D }; B = {a - d , a, a + d }
Sol. (Q. Nos. 46 to 48) According to the question,
8 16 24 A - D + A + A + D = 15
Let Sn = + + + ...
5 65 325 Þ 3 A = 15
8r 8r Þ A =5 ...(i)
Tr = 4 =
4r + 1 (2r 2 + 2r + 1 ) (2r 2 - 2r + 1 ) and a - d + a + a + d = 15
é (2r 2 + 2r + 1 ) - (2r 2 - 2r + 1 ) ù Þ a =5 …(ii)
=2 ê ú
2 2
ë (2r + 2r + 1 ) (2r - 2r + 1 ) û and D =1 + d …(iii)
p = (A - D ) A (A + D )
é 1 1 ù
=2 ê 2 - 2 ú p = A (A 2 - D 2 ) …(iv)
ë 2r - 2r + 1 2r + 2r + 1 û 2
p = 5 (25 - D ) …(v)
n
46. lim Sn = lim
n®¥ n®¥
å Tr Similarly, q = 5 (25 - d ) 2

r =1
Given that, p = 7 (q - p )
æn
1 1 ö 8 p = 7q
= lim å 2 ç 2 - 2 ÷
r = 1 è 2r - 2r + 1 2r + 2r + 1 ø
n®¥
From Eqs. (iv) and (v), we get
æ 1 ö 8 ´ 5 (25 - D 2 ) = 7 ´ 5 (25 - d 2 )
= 2 lim ç1 - 2 ÷ = 2 (1 - 0 ) = 2
n®¥è 2n + 2n + 1 ø 200 - 8 D 2 = 175 - 7d 2
8 ´7 56 25 = 8 D 2 - 7d 2
47. T7 = 4
=
4 ´ 7 + 1 9605 25 = 8 (1 + d ) 2 - 7d 2 [from Eq. (iii)]
8 8 æ 1 1 ö 25 = 8 + 8d + 16d - 7d 2
2
48. S 8 = å Tr = 2 å ç 2 - 2 ÷
r =1 r =1 è2 r - 2 r + 1 2 r + 2 r + 1ø 17 - d 2 - 16d = 0
æ 1 ö æ 1 ö 288 d 2 + 16d -17 = 0
= 2 ç1 - ÷ = 2 ç1 - ÷=
è 2
2 (8 ) + 2 (8 ) + 1 ø è 145 ø 145 (d + 17 ) (d - 1 ) = 0
Sol. (Q. Nos. 4-6) d = - 17 or d = 1
Let p and ( p + 1 ) be removed numbers from 1, 2, 3, ... n, then Þ d =1 [Q d > 0]
Sum of the remaining numbers Þ D =2
n (n + 1 ) 52. p = 5 (25 - D 2 ) = 5 (25 - 4) = 5 (21) = 105
= - (2 p + 1 )
2
53. q = 5 (25 - d 2 ) = 5 (25 - 1) = 120
From given condition,
n (n + 1 ) 54. 7D + 8d = 14 + 8 = 22
- (2 p + 1 )
105 2 Sol. (Q. Nos. 55 to 57)
=
4 (n - 2 ) ìA ü
Let A = í , A, ARý
Þ 2n 2 - 103n - 8 p + 206 = 0 îR þ
Since, n and p are integers, so n must be even. ìa ü
B = í , a, ar ý
Let n = 2r, we get îr þ
4r 2 + 103 (1 - r ) A
According to the question, × A × AR = 64
p=
4 R
3
Since, p is an integer, then (1 - r ) must be divisible by 4. Þ A = 64 Þ A = 4 …(i)
Let r = 1 + 4t , we get a
× a × ar = 64 Þ a 3 = 64 Þa = 4 …(ii)
n = 2 + 8t and p = 16t 2 - 95t + 1 r
and R =r + 2 …(iii)
Now, 1 £ p <n
298 Textbook of Algebra

p=
A
× A + A × AR + AR ×
A 60. According to the question, (m + 1) is the nth triangular number,
R R then
A2 16 n (n + 1 )
= 2
+A R+A = 2
+ 16 R + 16 =m + 1
R R 2
a a n 2 + n - 2 (m + 1 ) = 0
q = × a + a × ar + ar ×
r r - 1 ± 1 + 8 (m + 1 )
n=
a2 16 2
= + a 2r + a 2 = + 16r + 16
r r - 1 + (8m + 9 )
=
p 3 2
Given that, =
q 2 - 1 + 8m + 9 - 2m
\ n -m =
(16 + 16 R 2 + 16 R ) r 3 2
So, =
(16 + 16r 2 +16r ) R 2 Sol. (Q. Nos. 61 to 63)
2
(1 + R + R ) r 3 A1, A2, A3, ..., Am are arithmetic means between - 3 and 828.
= (a + b )
(1 + r 2 + r ) R 2 So, A1 + A2 + ... + Am = m
2
From Eq. (iii), R =r + 2
æ - 3 + 288 ö
(1 + r 2 + 4 + 4r + r + 2 ) r 3 Þ A1 + A2 + ... + Am = m ç ÷
Þ = è 2 ø
(1 + r + r 2 ) (r + 2 ) 2
æ 825 ö
Þ 14025 = m ç ÷
r 3 + 5r 2 + 7r 3 è 2 ø
Þ =
r + 3r 2 + 3r + 2 2
3
[given that sum of AM’s = 14025]
Þ r 3 - r 2 - 5r + 6 = 0 Þ m = 17 ´ 2
Þ (r - 2 ) (r 2 + r - 3 ) = 0 \ m = 34 …(i)
Now, G1, G2, ..., Gn be the GM’s between 1 and 2187.
- 1 ± 13
Þ r = 2 or r = \ G1G2 G3 ... Gn = (ab )n / 2
2
So, R=4 Þ 3 35 = (1 ´ 2187 )n / 2 Þ 3 35 = 3 7n / 2
æ1 ö æ1 ö 16 35 =
7n
55. p = 16 ç + R + 1÷ = 16 ç + 4 + 1÷ = (21) = 84 So,
èR ø è4 ø 4 2
æ1 ö æ1 ö 16 Þ n = 10 …(ii)
56. q = 16 ç + r + 1÷ = 16 ç + 2 + 1÷ = ´ 7 = 8 ´ 7 = 56
èr ø è2 ø 2 61. n = 10 [by Eq. (ii)]
57. r R + Rr = ( 4) 2 + (2) 4 = 16 + 16 = 32 62. m = 34 [by Eq. (i)]
2 3 n
Sol. (Q. Nos. 58 to 60) 63. G1 + G2 + ... + Gn = r + r + r + ... + r
Given sequence, 1, 3, 6, 10, 15, 21, 28, ... (1 - r 10 )
where tn = tn - 1 + n, " n ³ 2 = r + r 2 + r 3 + ... + r 10 = r
1 -r
So, tn = [tn - 2 + (n - 1 )] + n é 1/n + 1 1/11 ù
æl ö æ 2187 ö
= tn - 3 + (n - 2 ) + (n - 1 ) + n êQ r = ç ÷ =ç ÷ = 3 7/11 ú
êë è a ø è 1 ø úû
M M M
tn = t1 + 2 + 3 + ... + (n - 1 ) + n (1 - 3 70 /11 )
= 3 7 /11
tn = 1 + 2 + 3 + ... + n (1 - 3 7/11 )
n (n + 1 )
tn = …(i) Solution (Q. Nos. 64 to 66)
2
50 ´ 51 b c b 2 - 4ac
58. t 50 = = 25 ´ 51 = 1275 Q a + b = - , ab = , a - b =
2 a a a
100 ´ 101 B C B 2 - 4 AC
59. t100 = = 5050 and g + d=- , gd = , g - d =
2 A A A
101 ´ 102
t101 = = 101 ´ 51 = 5151 64. Since, a, b, g are in AP.
2
Let b = a + D , g = a + 2D and d = a + 3D
Number of positive integers lying between t100 and t101
-b b
= 5151 - 5050 - 1 Q a+b= Þa+ a+ D=-
a a
= 101 - 1 = 100
b
or 2a + D = - …(i)
a
Chap 03 Sequences and Series 299

B B p æ an + b ö é a + bn ù
and g + d=- Þ 2 a + 5D = - …(ii) 69. Again, =ç ÷´
A A q è n + 1 ø êë ab (n + 1 ) úû
From Eqs. (i) and (ii), we get
a 2n + abn 2 + b 2n + ab
æ B bö 1 æb B ö =
4D = ç - + ÷ or D = ç - ÷ ab (n + 1 ) 2
è A aø 4 èa A ø
æa b ö
65. Since, a, b, g, d, ... are in GP. n ç + ÷ + (n 2 + 1 )
èb a ø
b g d =
\ = = (n + 1 ) 2
a b g 2
b d a b æa b ö æ a bö
Þ = Þ = n ç + - 2÷ n ç - ÷
a g g d p èb a ø è b aø
Þ -1 = =
a+b ab q (n + 1 ) 2 (n + 1 ) 2
Þ = p p
g+d gd So, -1 > 0 Þ >1 Þ p >q …(iii)
q q
b c
-
a a b 2A 2 c A From Eqs. (i) and (ii), we get
Þ = Þ = 2
B C a 2B 2 aC æn + 1ö
- q<p<ç ÷ p
A A èn - 1ø
ac A 2 c A
Þ = Þ B 2 = AC 70. a, b, c, d are positive real numbers with
aB 2 C
a <b <c <d …(A)
Hence, A, B, C are in GP.
According to the question, a, b, c, d are in AP.
66. Since, a, b, g, d, ... are in GP.
Þ b = a + a, c = a + 2 a and d = a + 3 a …(i)
b g d
\ r= = = a be the common difference
a b g
and a, b, d are in GP.
b
Þ a + b = a + ar = - Þ b 2 = ad …(ii)
a
b From Eqs. (i) and (ii), we get
Þ a (1 + r ) = - …(i) (a + a ) 2 = a (a + 3 a )
a
B Þ a 2 + a 2 + 2aa = a 2 + 3aa
and g + d = ar 2 + ar 3 = -
A Þ a 2 = aa
2 B
Þ ar (1 + r ) = - …(ii) Þ a (a - a ) = 0
A
Þ a = 0 or a = a
From Eqs. (i) and (ii), we get
a ¹ 0 by (A), so a = a
Ba
r2 = From Eq. (i), b = 2a, c = 3a and d = 4a
bA
ad a × 4a 2 æ p ö
aB = = =ç ÷
\ r= bc 2a × 3a 3 è q ø
bA
Sol. (Q. Nos. 67 to 69) where, p and q are prime numbers.
For n > 1, we have n + 1 > n - 1 So, q =3
110
2
n+1 æn + 1ö 71. Q å (1 + rx ) = (1 + x ) (1 + 2x ) (1 + 3x )...(1 + 110x )
Þ >1 Þ p ç ÷ >p [Q p > 0] …(i) r =1
n -1 èn - 1ø
Now, p =a + d = 1110 + ( x + 2 x + 3 x + ... + 110 x ) 1109 + ...
Since, a, p, b, are in AP. So, coefficient of x in
b -a 110
110 ´ 111
And d =
n+1 å (1 + rx ) = (1 + 2 + 3 + ... + 110 ) =
2
= 55 ´ 111
r =1
(b - a ) na + b = 6105
67. p =a + =
n+1 n+1 Now, l (1 + 10 ) (1 + 10 + 10 2 ) = l (11 ) (111 )
1 1
- Þ l (111 ) (11 ) = 6105 Þ l = 5
1 1 1
68. = +D= + b a 72. Let number of the form palindrome be aba.
q a a n+1
ab (n + 1 ) Now, If aba is even, then a may be 2, 4, 6, 8 and b take values
Þ q= 0, 1, 2, ..., 9 .
a + bn
So, total number of palindrome (even) = 10 ´ 4 = 40
300 Textbook of Algebra

To find the sum of all even 3 digit plaindrome 1


Þ ( 2 + 1) x 2 - 2x + =0
So, sum of number start with 2 2+1
= (200 + 2 ) ´ 10 + ( 0 + 1 + 2 + 3 + ... + 9 ) ´ 10 Þ [( 2 + 1 ) x ]2 - 2 ( 2 + 1 ) x + 1 = 0
= 2020 + 450 = 2470
Þ [( 2 + 1 ) x - 1 ]2 = 0
Sum of number start with 4 = ( 404 ) ´ 10 + 450
1
Similarly, sum of number start with 6 = (606 ) ´ 10 + 450 Þ x= [repeated]
2+1
Similarly, sum of number start with 8 = (808 ) ´ 10 + 450
\Total sum = (202 + 404 + 606 + 808 ) ´ 10 + 450 ´ 4 So, x = 2 -1
= 20200 + 1800 = 22000 \ (x + 1)2 = 2
= 2 4 ´ 5 3 ´ 11 75. a1, a 2, a 3, ... are in GP with common ratio r
4 3 1
On comparing 2 ´ 5 ´ 11 with and b1, b2, b3, ... is also a GP i.e. b1 = 1
2 n1
´3 n2
´5 n3
´7 n4 n5
´ 11 , b2 = 4 7 - 4 28 + 1, a1 = 4 28
¥ ¥
n1 = 4, n2 = 3, n3 = 0, n4 = 0, n5 = 1 1
Now, n1 + n2 + n3 + n4 + n5 = 8
and å an = åbn
n =1 n =1
73. Q2 + ( 6 × 2 - 4 × 2) + ( 6 × 3 2 - 4 × 3)
2
1 1 1
+ ... + (6 × n 2 - 4 × n ) = 140 + + + ... + ¥ = b1 + b2 + b3 + ... + ¥
a1 a 2 a 3
Þ 2 + 6 (2 2 + 3 2 + ... + n 2 ) - 4 × (2 + 3 + ... + n ) = 140 1 1 1
Þ 4 + + + ... + ¥
æ n (n + 1 ) (2n - 1 ) ö 28 4 28 r 4 28 r 2
Þ 2+6ç - 1÷ - 4
è 6 ø
= 1 + ( 4 7 - 4 28 + 1 ) + ( 4 7 - 4 28 + 1 ) 2 + ... + ¥
æ n (n + 1 ) ö 1
ç - 1 ÷ = 140
è 2 ø 4 1
Þ 28 =
Þ 2 + n (n + 1 ) (2n + 1 ) - 6 - 2n (n + 1 ) + 4 = 140 1 1 - 4 7 + 4 28 - 1
1-
Þ n (n + 1 ) (2n + 1 ) - 2n (n + 1 ) - 140 = 0 r
Þ 2n 3 + 3n 2 + n - 2n 2 - 2n - 140 = 0 Þ
r
=4 4
1
Þ 2n 3 + n 2 - n - 140 = 0 (r - 1 ) 28
4
7 ( 4 - 1)
r 1 1
Þ (n - 4 ) (2n 2 + 9n + 35 ) = 0 Þ =
(r - 1 ) 4 4 ( 4 4 - 1 )
Þ n = 4 or 2n 2 + 9n + 35 = 0
Let 4
4 = a, we get
Þ 2n 2 + 9n + 35 = 0
r 1
- 9 ± 81 - 280 Þ =
Þ n= (r - 1 ) a a - 1
4
Þ ra - r = r a - a Þ r = a
9 ± - 199
\ n= [complex values] Þ r =44
4
Only positive integer value of n is 4. Now, 1 + r 2 + r 4 = 1 + (4 4 )2 + (4 4 )4
74. S ( x ) = 1 + x - x 2 - x 3 + x 4 + x 5 - x 6 - x 7+ ... + ¥ = 1 + 41/2 + 4 = 1 + 2 + 4 = 7
where x Î( 0, 1 ) 76. Let a = 10 + D …(i)
S ( x ) = (1 + x ) - x 2 (1 + x ) + x 4 (1 + x ) - x 6 (1 + x ) + ... + ¥ b = 10 + 2 D …(ii)
Þ S ( x ) = (1 + x ) [1 - x 2 + x 4 - x 6 + ... + ¥ ] ab = 10 + 3 D …(iii)
On substituting the values of a and b in Eq. (iii), we get
æ 1 ö é a ù
Þ S ( x ) = (1 + x ) ç ÷ êQ S ¥ = 1 - r for GP ú (10 + D ) (10 + 2 D ) = (10 + 3 D )
è1 + x2 ø ë û Þ 2 D 2 + 27 D + 90 = 0
2+1 15
According to the question, S ( x ) = \ D = -6,D = -
2 2
1+x 2+1 \ a1 = 10 - 6 = 4 ,
So, =
1 + x2 2 15 5
a 2 = 10 - =
Þ 2 + 2x = ( 2 + 1)x 2 + 2 + 1 2 2
and b1 = 10 - 12 = - 2 , b2 = 10 - 15 = - 5
Þ ( 2 + 1) x 2 - 2x - 2 + 2 + 1 = 0
æ 2a1a 2 + b1b2 ö æ 2 ´ 10 + 10 ö
Þ ( 2 + 1) x 2 - 2x + 2 - 1 = 0 Now, ç ÷=ç ÷ =3
è 10 ø è 10 ø
Chap 03 Sequences and Series 301

77. Given equation, Ax 3 + Bx 2 + Cx + D = 0 …(i) (a + d ) 2


\ r2 =
where, A¹0 (a - d ) 2
a+g a 2 + d 2 + 2ad
Let roots are a, b, g, then b = …(ii) Þ r2 =
2 a 2 + d 2 - 2ad
Given relation, 2 B + l ABC + m A 2D = 0
3
…(iii)
2d 2 + d 2 + 2 2d 2
b Þ r2 =
From Eq. (i), a + b + g = - 2d 2 + d 2 - 2 2 d 2
A
[from Eq. (i) for a = 2 d ]
B
Þ 3b = - [from Eq. (ii)]
A (3 + 2 2 ) d 2
Þ r2 =
B (3 - 2 2 ) d 2
Þ b= -
3A (3 + 2 2 ) (3 + 2 2 )
Þ r2 =
Now, b satisfy Eq. (i), so 9 -8
3 2
æ- Bö æ- Bö æ- Bö Þ r 2 = (3 + 2 2 ) 2
Aç ÷ +Bç ÷ +C ç ÷+D=0
è 3A ø è 3A ø è 3A ø
Þ r 2 = (3 + 8 ) 2
- B3 B3 BC
Þ 2
+ 2 - +D=0 \ r = ± (3 + 8 )
27 A 9A 3A
Þ r =3 + 8 [Q r is positive]
2 B 3 BC
Þ - +D=0 Similarly, for a = - 2 d , we get
27 A 2 3 A
Þ 2 B 3 - 9 ABC + 27 DA 2 = 0 r = ± (3 - 8 )
Compare with Eq. (iii), we get Þ r = (3 - 8 ) [Q r is positive]
l = - 9 , m = 27 Compare r with 3 ± k, we get
2 l + m = - 18 + 27 = 9 k =8
æ 1 2 22 ö ék 8 ù é8 8ù
78. Let P = lim ç + + + ... upto n terms÷ \ êë 8 - k úû = êë 8 - 8 úû
n ® ¥ è1 + x 1+x 2
1+x 4
ø
n æ ö = [1 - 1 ] = [ 0 ] = 0
2r 2r 2r
= lim
n®¥
å çç r
+ r
- r
÷
÷ 80. (A) a, b, c, d are in AP [a, b, c, d are positive real numbers]
r =0 è 1 + x2 1 - x2 1 - x2 ø
By AM > GM, for a, b, c
n æ 2r + 1 2r ö b > ac
= lim å ç - ÷
n®¥ ç 2 r + 1 r ÷
r =0 è 1 - x 1 - x2 ø Þ b 2 > ac …(i)
æ 2n + 1 1 ö÷ Now, applying for b, c, d
= lim ç - c > bd Þ c 2 > bd
n®¥ç 1 - x ÷ø …(ii)
n + 1
è1 - x2
From Eqs. (i) and (ii), we get
2n +1
n+1 b 2c 2 > (ac ) (bd ) Þ bc > ad
x2 1 1
= lim - =0- Again, applying AM > HM for a,b, c
n®¥ 1
-1 1 -x 1-x
2 1 1 2
2n + 1 b> Þ + > …(iii)
x 1 1 a c b
1 1 +
= = [given] a c
x -1 x - l
For last 3 terms b, c, d
\ l =1 2 1 1 2
c> Þ + > …(iv)
79. Let number of AP are (a - d ), a, (a + d ) . 1 1
+ b d c
According to the question, (a - d ) 2, a 2, (a + d ) 2 are in GP. b d
From Eqs. (iii) and (iv), we get
\ (a 2 ) 2 = (a - d ) 2 (a + d ) 2 1 1 1 1 2 2
+ + + > +
Þ a 4 = (a 2 - d 2 ) 2 a c b d b c
Þ a 4 = a 4 + d 4 - 2a 2d 2 1 1 1 1
Þ + > +
a d b c
Þ a 2 (a 2 - 2d 2 ) = 0
(B) a, b, c, d are in GP.
Þ a ¹ 0, so a 2 = 2d 2 For a, b, c applying AM > GM,
Þ a = ± 2d …(i) a+c
Þ > b Þ a + c > 2b …(i)
Let common ratio of GP is r. 2
302 Textbook of Algebra

Similarly, for b, c, d 3
Þ 5n + (n 2 - n ) = 220
b + d > 2c ...(ii) 2
From Eqs. (i) and (ii), we get Þ 3n 2 + 7n - 440 = 0
a + b + c + d > 2 b + 2c Þ a + d > b + c Þ 3n 2 + 40n - 33n - 440 = 0
Now, applying GM > HM for a, b c
Þ n (3n + 40 ) - 11 (3n + 40 ) = 0
2 ac
b> Þ (3n + 40 ) (n - 11 ) = 0
a+c
40
1 1 2 So, n=- or n = 11
Þ + > …(iii) 3
c a c \ n = 11 [n Î N ]
Similarly, for b, c, d , we get (B) Let first angle = a [in degrees]
1 1 2
+ > …(iv) Common difference = d [in degrees]
d b c
Number of sides n = 9
On adding Eqs. (iii) and (iv), we get
\Sum of interior angles = (n - 2 ) ´ 180°
1 1 1 1 æ 1 1ö
+ + + >2 ç + ÷ Þ
n
[ 2a + (n - 1 ) d ] = (n - 2 ) ´ 180°
a b c d èb c ø
2
1 1 1 1 9
Þ + > + Þ ( 2 a + 8d ) = 7 ´ 180°
a d b c 2
(C) a, b, c, d are in HP. Þ a + 4d = 140°
Applying AM > HM for a, b, c and largest angle T9 = a + 8d < 180°
a+c Þ 4d < 40
>b
2 Þ d < 10
Þ a + c > 2b …(i) \ d =9
Similarly, for last 3 terms b, c, d (C) Given increasing GP,
Þ b + d > 2c …(ii) a1, a 2, ..., an , ...
On adding Eqs. (i) and (ii), we get where a 6 = 4a 4
a + b + c + d > 2b + 2c a1r 5 = 4a1r 3 [r is the common ratio]
Þ a + d >b + c Þ r2 = 4
Again, applying GM > HM for a, b, c
Þ r =2 [Q increasing GP]
ac > b
and a 9 - a 7 = 192
Þ ac > b 2 …(iii) a1 (r 8 - r 6 ) = 192
Similarly, for b, c, d a1 (256 - 64 ) = 192
Þ bd > c 2 …(iv) 192
a1 =
On multiplying Eqs. (iii) and (iv), we get 192
abcd > b 2c 2 a1 = 1
ad > bc Then, a 2 = 2 , a 3 = 4 and a 4 + a 5 + ... + an = 1016
(a1 + a 2 + ... + an ) - (a1 + a 2 + a 3 ) = 1016
81. (A) a1, a 2, a 3,..., an , ... are in AP
1 (2n - 1 )
5 = 1016 + 7
and a1 = , a10 = 16 2 -1
2
\ a1 + a 2 + ... + an = 110 2n = 1023 + 1 = 1024 = 210
n \ n = 10
Þ (a1 + an ) = 110
2 82. (A) a1, a 2, ... are in AP.
n é5 5 ù a1 + a 4 + a 7 + a14 + a17 + a 20 = 165 [In an AP, sum of the
Þ + + (n - 1 ) d ú = 110 …(i)
2 êë 2 2 û terms equidistant from the 1st and last is equal to sum of
5 1st and last terms]
16 -
a - a1 2 = 27 = 3 Þ 3 (a1 + a 20 ) = 165
Now, d = 10 = …(ii)
10 - 1 9 9 ´2 2 Þ a1 + a1 +19d = 55
From Eqs. (i) and (ii), we get d is the common difference of AP.
né 3ù 2a1 + 19d = 55 …(i)
5 + (n - 1 ) ú = 110
2 êë 2û Now, a = a 2 + a 6 + a15 + a19
a = 2 (a 2 + a19 )
Chap 03 Sequences and Series 303

a = 2 (a1 + d + a1 + 18d ) and 3b = 4c and 4c = 2a


a = 2 (2a1 + 19d ) …(ii) Þ
3 4
a = b and b = c and c = a
1
and b = 2 (a 9 + a12 ) - (a 3 + a18 ) 2 3 2
b = 2 (a1 + 8d + a1 + 11d ) - (a1 + 2d + a1 + 17d ) 3 4 3
Þ a = b and b = c and c = b
b = 2 (2a1 + 19d ) - (2a1 + 19d ) 2 3 4
b = 2a1 + 19d …(iii) 3 3
So, a, b, c are b, b, b
From Eqs. (i) and (iii), we get 2 4
a = 2b 2 1 4
Reciprocal of the terms , , ,
From Eqs. (i), (ii) and (iii), we get 3b b 3b
a + 2 b = 4 (2a1 +19d ) = 4(55 ) = 220 [from Eq.(i)] which is in AP.
a + b = 3 (2a1 + 19d ) So, these a, b, c are in HP.
= 3 ´ 55 = 165 = 15 ´ 11 = 15m, where m Î I (B) 17a 2 + 13b 2 + 5c 2 = 3ab + 15bc + 5ca
a - b = 2a1 + 19d Þ 34a 2 + 26a 2 + 10c 2 - 6ab - 30bc - 10ca = 0
= 55 = 5 ´ 11 = 5l, where l Î I Þ (3a - b ) 2 + (5b - 3c ) 2 + (c - 5a ) 2 = 0
(B) a1, a 2, ... are in AP.
Þ 3a - b = 0 and 5b - 3c = 0 and c - 5a = 0
a1 + a 5+ a10 + a15 + a 20 + a 24 = 195 a b c
3 (a1 + a 24 ) = 195 Þ = = =l [say]
1 3 5
Þ a1 + a 24 = 65 ...(i) \ a = l, b = 3 l, c = 5 l
Þ 2a1 + 23d = 65 Hence, a, b, c are in AP.
Now, a = a 2 + a 7 + a18 + a 23 æ 15 5 3 ö
= 2 (a 2 + a 23 ) = 2 (2a1 + 23d ) (C) a 2 + 9b 2 + 25c 2 = abc ç + + ÷
èa b cø
a = 130 [from Eq. (i)]
Þ (a ) 2 + (3b ) 2 + (5c ) 2 -15bc - 5ac - 3ab = 0
b = 2 (a 2 + a 22 ) - (a 8 + a17 )
1
= 2 (2a1 + 23d ) - (2a1 + 23d ) Þ {(a - 3b ) 2 + (3b - 5c ) 2 + (5c - a ) 2 } = 0
2
= 130 - 65 = 65
Þ a - 3b = 0 and 3b - 5c = 0 and 5c - a = 0
Then, a = 2 b
Þ a = 3b and 3b = 5c and 5c = a
a + 2 b = 130 + 130 = 260
5 a
a + b = 195 = 15 ´ 13 = 15 m, where m = 13 Þ a = 3b and b = c and c =
3 5
and a - b = 130 - 65 = 65
5 3
= 5 ´ 13 = 5l, where l = 13 Þ a = 3b and b = c and c = b
3 5
(C) a1, a 2,... are in AP.
3b
a1 + a 7 + a10 + a 21 + a 24 + a 30 = 225 So, a,b, c are of the form 3b, b, .
5
3 (a1 + a 30 ) = 225 3b 1 1 5
2a1 + 29d = 75 …(i) Reciprocal of 3b, b, are , , , which are in AP.
5 3b b 3b
Now, a = a 2 + a 7 + a 24 + a 29 é 1 1 2 5 1 2ù
a = 4a1 + 58d = 2 (2a1 + 29d ) êëQ b - 3b = 3b and 3b - b = 3b úû
= 2 ´ 75 = 150
(D) (a 2 + b 2 + c 2 ) p 2 - 2 p (ab + bc + ca ) + a 2 + b 2 + c 2 £ 0
a = 150 …(ii)
and b = 2 (a10 + a 21 ) - (a 3 + a 28 ) Þ (a 2p 2 + b 2 - 2abp ) + (b 2p 2 + c 2 - 2 pbc )
= 2 (2a1 + 29d ) - (2a1 + 29d ) = 150 - 75 + (c 2p 2 + a 2 - 2acp ) £ 0
b = 75 …(iii) Þ (ap - b ) 2 + (bp - c ) 2 + (cp - a ) 2 £ 0
Then, a = 2b Þ (ap - b ) 2 + (bp - c ) 2 + (cp - a ) 2 = 0
a + 2b = 150 + 150 = 300 and a - b = 150 - 75 = 75
Þ ap - b = 0 and bp - c = 0 and cp - a = 0
= 5 ´ 15 = 5l, where l = 15 b c a
and a + b = 150 + 75 = 225 = 15 ´ 15 = 15m, where m = 15 Þ p = and p = and p =
a b c
83. (A) 4a 2 + 9b 2 + 16c 2 = 2 (3ab + 6bc + 4ca ) b c a
Þ = =
(2a ) 2 + (3b ) 2 + ( 4c ) 2 - (2a ) (3b ) ( 4c ) - (2a ) ( 4c ) = 0 a b c
1 Þa, b, c are in GP.
{(2a - 3b ) 2 + (3b - 4c ) 2 + ( 4c - 2a ) 2 } = 0
2 84. If a, b, c are in GP.
Þ 2a - 3b = 0 and 3b - 4c = 0 Then, b 2 = ac
and 4c - 2a = 0 Þ2a = 3b
If middle term is added, then a + b, 2b and c + b are in GP.
304 Textbook of Algebra

I - II a + b - 2b Statement-2 If
= [here, I = a + b, II = 2b, III = c + b]
II - III 2b - (c + b ) (a1 - a 2 ) 2 + (a 2 - a 3 ) 2 + (a 3 - a1 ) 2 = 0
a - b ab - b 2 ab - ac Þ a1 - a 2 = 0 and a 2 - a 3 = 0 and a 3 - a1 = 0
= = = [Q b 2 = ac ]
b - c b 2 - bc ac - bc Þ a1 = a 2 = a 3, " a1, a 2, a 3 Î R
a (b - c ) (a + b ) (b + c ) So, Statement-2 is true.
= a+b 2ab
c (a - b ) (a + b ) (b + c ) 89. Q A = , G = ab and H =
2 a+b
a (b 2 - c 2 ) (a + b )
= Given, 4G = 5 H …(i)
c (a 2 - b 2 ) (b + c )
and G 2 = AH
a (ac - c 2 ) (a + b ) a + b I
= ; = G2
c (a - ac ) (b + c ) b + c III
2
\ H = …(ii)
A
Hence, a + b, 2b, b + c are in HP. From Eqs. (i) and (ii), we get
Hence, both statements are true and Statement-2 is correct
5G 2
explanation for Statement-1. 4G = Þ 4 A = 5G
A
85. QTn = 2n 3 + 3n 2 - 4
Þ 2 (a + b ) = 5 ab
Sequence is 1, 24, 77, 172, 321, ...
First order difference 23, 53, 95, 149, ... Þ 4 (a 2 + b 2 + 2ab ) = 25ab
Second order difference 30, 42, 54, ... Þ 4a 2 - 17ab + 4b 2 = 0
which are in AP. Þ (a - 4b ) ( 4a - b ) = 0
\Statenemt-1 is true. a = 4b, 4a - b ¹ 0 [Q a > b]
QTn is of three degree and third order difference will be constant. \Statement-1 is true.
Statement-2 is true, which is correct explanation for Statement-2 is true only for two numbers, if numbers more
Statement-1. than two, then this formula (AM) (HM) = (GM) 2 is true, if
86. Statement-1 Let S be the required sum of product of numbers. numbers are in GP.
2 Statement-2 is false for positive numbers.
æ n ö n
ç å xi ÷ = å xi2 + 2 å å xi x j 90. Statement-1 Sum of first 100 even natural numbers
ç ÷
è i =1 ø i =1 1 £i < j £n 2 (100 ´ 101 )
n
E1 = 2 + 4 + ... + 200 = = 10100
2
\ (a1 - a1 + a 2 - a 2 + ... + an - an ) 2 = 2 åai2 + 2S
i =1
Sum of 100 odd natural numbers = 1 + 3 + ... + 199
100
n O= (1 + 199 ) = 10000
\ S = - åai2 2
i =1 \ E - O = 100
\Statement-1 is true. So, Statement-1 is true.
Statement-2 is true but not correct explanation for Statement-2 Sum of first n natural even numbers
Statement-1. 2n (n + 1 )
E = 2 + 4 + ... + 2n = = n2 + n
87. Statement-1 a + b + c = 18, a, b, c > 0 2
Sum of first n odd natural numbers
Applying AM ³ GM for a, b, c
a+b+c 3 O = 1 + 3 + ... + (2n - 1 )
³ abc Þ 3 abc £ 6 Þ abc £ 216 n
3 = [1 + 2n - 1 ] = n 2
2
Maximum value of abc is 216 which occurs at a = b = c .
Statement-2 is the correct explanation for Statement-1. So, E - O = n2 + n - n2 = n
88. Statement-1 Statement-2 is true and correct explanation for Statement-1.
4a 2 + 9b 2 + 16c 2 - 2 (3ab + 6bc + 4ca ) = 0 91. Let Tn = An + B
Þ (2a ) 2 + (3b ) 2 + ( 4c ) 2 - (2a ) (3b ) - (3b ) ( 4c ) - (2a ) ( 4c ) = 0 \ Tp = Ap + B,
1 T2p = 2 Ap + B, T4 p = 4 Ap + B
Þ {(2a - 3b ) 2 + (3b - 4c ) 2 + ( 4c - 2a ) 2 } = 0 \ Tp , T2p , T4 p are in G P.
2
Þ 2a - 3b = 0 and 3b - 4c = 0 and 4c - 2a = 0 \ (2 Ap + B ) 2 = ( Ap + B ) ( 4 Ap + B )
4c a 3b 4c 3b Þ ABp = 0
Þ and b = and c = Þ a = and b = and c =
3 2 2 3 4 \ B = 0, A ¹ 0, p ¹ 0
3b 3b T2p 2 Ap + 0
Then, a, b, c are of the form , b, , which are in HP. Þ Common ratio, r = = =2
2 4 Tp Ap + 0
So, Statement-1 is false.
Chap 03 Sequences and Series 305

92. a ¹ 1, b ¹ 0 and a ¹ b p2 1 p2 3 p2 p2
= - = ´ =
2 2 3 2 2 3
Let S = (a + b ) + (a + ab + b ) + (a + a b + ab + b ) + ¼+ n 6 4 6 4 6 8
terms 1 1 1 æ1 1 ö
(ii) 1 - 2 + 2 - 2 + ¼ + ¥ = ç 2 _ 3 + ¼÷
1 2 3 4 è1 3 ø
= [(a 2 - b 2 ) + (a 3 - b 3 ) + (a 4 - b 4 ) + ¼+ n terms]
(a - b ) 1 æ1 1 ö
- 2 ç 2 - 2 + ¼ + ¥÷
1 2 è1 2 ø
= [a 2(1 + a + ¼ + n terms)
(a - b ) p2 1 p2 p2
- b 2 (1 + b + b 2 + ¼ + n terms)] = - ´ = [by part (i)]
8 4 6 12
1 é 2 1 × (an - 1 ) 1 × (bn - 1 ) ù 97. Saibi = Sai (1 - ai ) = na - Sai2
= êa × - b2 × ú
(a - b ) ë (a - 1 ) (b - 1 ) û = na - S(ai - a + a ) 2
1 é 2 (1 - an ) (1 - bn ) ù = na - S[(ai - a ) 2 + a 2 + 2a (ai - a )]
= êa - b2 ú
(a - b ) ë (1 - a ) (1 - b ) û = na - S[(ai - a ) 2 - Sa 2 - 2aS(ai - a )
93. Sequence of natural number is divided into group. \ Saibi + S(ai - a ) 2 = na - na 2 - 2a (na - na )
1, 3, 5, 7, 9, 11, … éQ Sbi = S1 - Sai ù
\nth row contains n elements ê \ nb = n - na ú
= na (1 - a ) = nab
1st element of nth row = n 2 - (n - 1 ) ê ú
êë or a + b = 1 úû
Least element of nth row = n 2 + (n - 1 )
98. a1 + a 2 + ¼ + a 98 = 137
\Sum of the element in the nth row 98
n n (a1 + a 98 ) = 137
= (a + l ) = [n 2 - (n - 1 ) + n 2 + (n - 1 )] 2
2 2 137 137
n 2 n a1 + a 2 + 97 = ; 2a1 + 97 =
= [n - n + 1 + n 2 + n - 1 ] = [ 2n 2 ] = n 3 49 49
2 2
137 1 (137 - 4753
a (r n - 1 ) 2a1 = - 97; a1 =
94. a = Sn = …(i) 49 2 49
r -1 4616 2308
a1 = - ; a1 = …(i)
a(r 2n - 1 ) a (r n - 1 ) a (r n - 1 ) n 2 ´ 49 49
b = S 2n - Sn = - = (r ) …(ii)
(r - 1 ) (r - 1 ) (r - 1 ) Now, a 2 + a 4 +¼ + a 98 = (a1 + 1 ) + (a1 + 3 ) + ¼ + (a1 + 97 )
a (r 3n - 1 ) a (r 2n - 1 ) [Q d = 1 ]
c = S 3n - S 2n = -
(r - 1 ) (r - 1 ) = 49a1 + (1 + 3 + ¼+ 97 )
2308 49
a (r n - 1 ) 2n a (r n - 1 ) n 2 = - 49 ´ + (1 + 97 )
= (r + r n + 1 - r n - 1 ) = × (r ) 49 2
(r - 1 ) (r - 1 )
= - 2308 + 49 2
…(iii)
From Eqs. (i), (ii) and (iii), b 2 = ac , so a, b, c are in GP. = - 2308 + 2401 = 93
99. t1 = 1 and tr - tr - 1 = 2r - 1, r ³ 2
95. First four terms of an AP are a, 2a, b and (a - 6 - b ).
t 2 - t1 = 2
So, 2a - a = a - 6 - b - b
Þ a = a - 6 - 2b t3 - t2 = 22
Þ - 2b = 6 Þb = - 3 t4 - t3 = 23
and 2a - a = b - 2a M M M
Þ b = 3a Þ a = - 1 tn - tn - 1 = 2n - 1
\First terms a = - 1 and d = a = - 1
Addiing columnwise, we get
100
S100 = [ 2a + (100 - 1 ) d ] tn - t1 = 2 + 2 2 + ¼ + 2n - 1
2
= 50 [ - 2 + 99 ( - 1 )] tn = 1 + 2 + 2 2 + ¼ + 2n - 1
= 50 ( - 2 - 99 ) = 50 ( - 101 ) = - 5050 1 × (2n - 1 )
tn = Þ tn = 2n - 1
1 1 1 p2 2 -1
96. (i) 2 + 2 + 2 + ¼+ ¥ = ...(i) n
1 2 3
1 1 1
6 So, åtr = t1 + t2 + ¼ + tn = (2 - 1) + (22 - 1) + ¼ + (2n - 1)
r =1
\ + + + ¼+ ¥ 2 × (2n - 1 )
12 32 52 = (2 + 2 2 + ¼ + 2n ) - n = - n = 2n + 1 - 2 - n
æ1 1 1 1 ö æ1 1 1 ö (2 - 1 )
= ç 2 + 2 + 2 + 2 + ¼+ ¥ ÷ - ç 2 + 2 + 2 + ¼ + ¥ ÷ = 2n + 1 - n - 2
è1 2 3 4 ø è2 4 6 ø
306 Textbook of Algebra

p sin 2nx p sin 2n ( p - x )


100. (i) I n = ò dx = ò dx x 3 (1 - x 3n ) (1 - x 3n ) 3 x (1 - xn ) 3 (1 - xn )
sin x
0 0 sin x = + 3n + + n
3
(1 - x ) x (1 - x ) 3
(1 - x ) x (1 - x )
p sin (2np - 2nx )
=ò dx 103. Let d be the common difference of AP.
0 sin x
LHS = a12 - a 22 + a 32 - a 42 + ¼ + a 22n - 1 - a 22n
p sin 2nx
In = - ò dx (a1 - a 2 ) (a1 + a 2 ) + (a 3 - a 4 ) (a 3 + a 4 )
0 sin x
+ ¼ + (a 2n - 1 - a 2n ) (a 2n - 1 + a 2n )
I n = - I n Þ 2I n = 0 Þ I n = 0 = - d (a1 + a 2 + ¼ + a 2n - 1 + a 2n )
\ I1 = I 2 = I 3 = ¼ = 0 = - d [(a1 + a 2n ) + (a 2 + a 2n - 1 ) + ¼ + (an + an + 1 )]
which is a constant series. = - dn (a1 + a 2n )
\ This series is AP with common difference 0 and first 2
(a 2 - a 22n ) -dn (a1 - a 2n )
2

term o. = - dn 1 = [Q a 2n = a1 + (2n - 1 ) d ]
(a1 - a 2n ) (1 - 2n ) d
p sin 2 nx
(ii) I n = ò dx n
0 sin 2 x = (a12 - a 22n )
2n - 1
sin 2 nx
Let f (x ) = 104. Let a, b, c (unequal number) are in HP
sin 2 x
Hence, f ( p - x ) = f ( x ) 2ac
\ b=
p/ 2 sin 2 nx a+c
So, In = 2 ò dx
0 sin 2 x b ac
Þ = =l [say]
Now, I n + 1 + I n - 1 - 2 I n 2 a+c
p/ 2 ì (sin 2(n + 1 ) x - sin 2 nx ) Þ b = 2l and ac = l (a + c ) …(i)
= 2ò í 2 2 2
0 sin 2 x Now, a , b , c are in AP
î
(sin 2(n - 1 ) x - sin 2 nx )ü a2 + c2
+ ý dx So, b2 = Þ2b 2 = a 2 + c 2
sin 2 x þ 2
sin (2n + 1 ) x sin x - sin (2n - 1 ) x sin x
p/ 2 Þ 2(2 l ) 2 = (a + c ) 2 - 2ac
=2 ò dx
0 sin 2 x Þ (a + c ) 2 - 2 l (a + c ) - 8 l2 = 0
p/ 2 sin (2n + 1 ) x - sin (2n - 1 ) x
=2 ò dx Þ (a + c - 4 l ) (a + c + 2 l ) = 0
0 sin x Þ a + c = 4l or a + c = - 2l
p/ 2 2 cos 2nx sin x
=2 ò dx Case I If a + c = 4l
0 sin x \ ac = 4 l2 [from Eq. (i)]
p /2 4 2
= 4 ò cos 2nx dx = [sin 2nx ]p0 /2 = × 0 = 0 Þ 2 2
(a - c ) = (a + c ) - 4ac
0 2n n
\ I n + 1 + I n - 1 = 2 I n \I 1, I 2, I 3, ¼are in A P. Þ (a - c ) 2 = 16 l2 - 16 l2
101. S = 7 + 13 + 21 + 31 + ¼ + Tn Þ (a - c ) 2 = 0 Þa = c
S= 7 + 13 + 21 + ¼ + Tn -1 + Tn Let given that a, b, c are distinct, so a + c = 4l is not valid.
- - - - - - - Case II If a + c = - 2l
0 = 7 + 6 + 8 + 10 + ¼+ n terms - Tn
Þ ac = - 2 l2 [from Eq. (i)]
Tn = 7 + 6 + 8 + 10 + ¼+ n terms
\ (a - c ) 2 = (a + c ) 2 - 4ac
Tn = 7 + {6 + 8 + 10 + ¼ + (n - 1 ) terms}
(n - 1 ) Þ (a - c ) 2 = 4 l2 + 8 l2 Þ (a - c ) = ± 2 3l ...(ii)
Tn = 7 + (12 + (n - 2 ) 2 )
2 If a - c = 2 3l, ...(iii)
(n - 1 ) a + c = 2l
Tn = 7 + (8 + 2n ) then
2 From Eqs. (ii) and (iii), we get
Tn = 7 + (n - 1 ) ( 4 + n )
a = ( 3 - 1 ) l and c = - (1 + 3 ) l
T70 = 7 + 69 ´ 74 = 7 + 5106 = 5113
3 3 n 3 3 \ a : b : c = ( 3 - 1)l : 2l : - ( 3 + 1)l
æ 1ö æ 2 1 ö æ n 1 ö æ n 1 ö
102. ç x + ÷ + çx + 2 ÷ + ¼ + çx + n ÷ = å çx + n ÷ a : b : c = ( 3 - 1) : 2 : - ( 3 + 1)
è x ø è x ø è x ø n =1
è x ø
n Þ a : b : c = (1 - 3 ) : - 2 : ( 3 + 1 )
æ 1 æ 1 öö
= å çè x 3n + x 3n + 3 çè xn + xn ÷ø ÷ø If a - c = - 2 3l, …(iv)
n =1
n n
1 n n
1 then a + c = - 2l …(v)
= å x 3n + å x 3n +3 å xn + 3 å xn
n =1 n =1 n =1 n =1
Chap 03 Sequences and Series 307

n
From Eqs. (iv) and (v), we get 108. LHS = (1 + 5 - 1 ) (1 + 5 -2 ) (1 + 5 - 4 ) ¼ (1 + 5 - 2 )
a = - ( 3 + 1 ) l and c = ( 3 - 1 ) l æ 1ö æ 1öæ 1ö æ 1 ö
= ç1 + ÷ ç1 + 2 ÷ ç1 + 4 ÷ ¼ ç1 + n ÷
\ a : b : c = - ( 3 + 1) l : 2l : ( 3 - 1)l è 5ø è 5 ø è 5 ø è 52 ø
Þ a : b : c = (1 + 3 ) : - 2 : (1 - 3 ) æ 1ö
ç1 - ÷
105. a1, a 2, a,¼, an are in AP with a1 = 0 and common difference d è 5ø æ 1ö æ 1öæ 1ö æ 1 ö
= ç1 + ÷ ç1 + 2 ÷ ç1 + 4 ÷ ¼ ç1 + n ÷
æ 1ö è 5ø è 5 øè 5 ø è 52 ø
[d ¹ 0 ] ç1 - ÷
è 5ø
\ a 2 = d , a 3 = 2d , ¼, an = (n - 1 ) d
éæ
5 1öæ 1öæ 1ö æ 1 öù
a a a a æ1 1 1 ö = ê çè1 + 2 ÷ø çè1 + 2 ÷ø çè1 + 4 ÷ø ¼ ç1 + 2 n ÷ ú
LHS = 3 + 4 + 5 + ¼ + n - a 2 ç + +¼+ ÷ 4
ë 5 5 5 è 5 øû
a2 a3 a4 an - 1 è a2 a3 an - 2 ø
M M M
1 1 (an - 1 - a 2 ) a 5 æ 1 ö 5
= (a 3 - a 2 ) + (a 4 - a 2 ) + ¼ + + n - 2n + 1
a2 a3 an - 2 an - 1 = ç1 - n + 1 ÷ = (1 - 5 ) = RHS
4è 52 ø 4
1 1 [(n - 2 ) d - d ] (n - 1 ) d ¥
= (2d - d ) + (3d - d ) + ... + + 2n
d 2d (n - 3 ) d (n - 2 ) d 109. S = å n
, (a > 1 )
n=0 a2 + 1
n-1
= [1 + 1 + ¼ + (n - 3 ) times ] + n
2n
n-2 Sn = å n
(n - 1 ) (n - 2 ) + 1 a2 + 1
n =0
= (n - 3 ) + = (n - 3 ) +
n -2 (n - 2 ) 1 2 4 8 2n
= + 2 + 4 + 8 + ¼+ n
1 1 a+1 a +1 a +1 a +1 a2 + 1
= (n - 3 ) + 1 + =n -2 +
n -2 n -2 1 2 4 8 2n
= + + + + ¼+
a (n - 2 ) d d an - 1 a 1+a 1+a 2
1+a 4
1+a 8
1 + a2
n
= + = + 2 = RHS
d (n - 2 ) d a2 an - 1 æ 1 1 ö 1 2 4 2n
= ç- + ÷+ + + + ¼+
106. Let one side of equilateral triangle contains n balls. Then è 1 -a 1 -aø 1 + a 1 + a 2
1+a 4
1 + a2
n

n (n + 1 )
Number of balls (initially) = 1 + 2 + 3 + ¼ + n = 1 æ 1 1 ö 2 4 2n
2 = +ç + ÷+ + + ¼ +
n (n + 1 ) a - 1 è1 -a 1 + a ø 1 + a2 1 + a4 1 + a2
n

According to the question, + 669 = (n - 8 ) 2


2 1 æ 2 2 ö 4 2n
Þ n 2 + n + 1338 = 2n 2 - 32n + 128 = +ç + ÷ + + ¼ +
a - 1 è1 - a2 1 + a2 ø 1 + a4 1 + a2
n

Þ n 2 - 33n - 1210 = 0 M M M M
Þ (n - 55 ) (n + 22 ) = 0 Þ n = 55 or n = - 22 1 2n + 1
Sn = +
which is not possible a - 1 1 - a2n + 1
\ n = 55 æ 1
n (n + 1 ) 55 ´ 56 2n + 1 ö÷
So, = = 55 ´ 28 = 1540 S = lim Sn = lim ç +
n ® ¥ ça - 1
n+1 ÷
n®¥
2 2 è 1 - a2 ø
107. q1, q2, q3, ¼, qn are in AP. æ 2n + 1 ö
ç n+1 ÷
q2 - q1 = q3 - q2 = ¼ = qn - qn - 1 = d 1 a2 ÷= 1 + 0 = 1
So, = lim ç +
n ® ¥ ça - 1
- 1 ÷÷ a - 1 0 - 1 a - 1
\LHS = sin d [sec q1 sec q2 + sec q2 sec q3 + ¼ 1
ç 2n + 1
+ sec qn - 1 sec qn ] è a ø
é 1 1 æ 2n - 1 ö æ 2n -1 ö
= sin d ê + 110. Tn = tan - 1 ç ÷ = tan - 1 ç ÷
ë cos q1 cos q2 cos q2 cos q3 è1 + 2 2n - 1
ø n
è 1 + 2 ×2 n -1
ø
ù
1
+ ¼+ ú æ 2n - 2n -1 ö
cos qn - 1 cos qn û = tan - 1 ç ÷ = tan - 1 2 n - tan - 1 2 n - 1
è 1 + 2 n ×2 n - 1 ø
sin d sin d sin d
= + + ¼+ Sn = T1 + T2 + ¼ + Tn
cos q1 cos q2 cos q2 cos q3 cos qn - 1 cos qn
= (tan - 1 21 - tan - 1 2 0 ) + (tan - 1 2 2 - tan - 1 21 ) + ¼
sin ( q2 - q1 ) sin ( q3 - q2 ) sin ( qn - qn - 1 )
= + + ¼+ + (tan - 1 2n - tan - 1 2n - 1 )
cos q1 cos q2 cos q2 cos q3 cos qn -1 cos qn = (tan - 1 2n - tan - 1 1 )
= (tan q2 - tan q1 ) + (tan q3 - tan q2 ) p
Sn = tan - 1 2n -
+ ¼ + (tan qn - tan qn - 1 ) 4
= tan qn - tan q1 = RHS æ pö p p p
S = lim Sn = lim ç tan - 1 2n - ÷ = - =
n®¥ n®¥è 4ø 2 4 4
308 Textbook of Algebra

111. Tn = tan [ a + (n - 1) b} tan ( a + nb) 2 1 1


\ = +
tan b = tan [( a + nb ) - { a + (n - 1 ) b }] d 2 d1 d 3
2 1 1
tan ( a + nb ) - tan ( a + (n - 1 ) b }] Þ = +
tan b = 2 (S 2 - n ) 2 (S1 - n ) 2 (S 3 - n )
1 + tan ( a + nb ) tan { a + (n - 1) b }]
n (n - 1 ) n (n - 1 ) n (n - 1 )
\1 + Tn = cot b [tan ( a + nb ) - tan { a + (n - 1 ) b }]
2 1 1
Tn = cot b [tan ( a + nb ) - tan { a + (n - 1 ) b }] - 1 Þ = +
S 2 - n S1 - n S 3 - n
For n = 1,
2 S + S1 - 2n
T1 = cot b [tan ( a + b ) - tan a ] - 1 Þ = 3
S 2 - n (S1 - n ) (S 3 - n )
For n = 2,
T2 = cot b [tan ( a + 2b ) - tan ( a + b )] - 1 Þ2 [S1S 3 - (S1 + S 3 ) n + n 2 ] = (S 2 - n ) (S1 + S 3 - 2n )
For n = 3, Þ2S1S 3 - 2(S1 + S 3 ) n + 2n 2
T3 = cot b [tan ( a + 3b ) - tan ( a + 2b )] - 1 = S1S 2 + S 2 S 3 - 2nS 2 - n (S1 + S 3 ) + 2n 2
M M M M Þ 2S1S 3 - S 2 S 3 - S1S 2 = n (S1 + S 3 - 2S 2 )
For n = n, (2S S - S 2 S 3 - S1S 2 )
Þ n= 1 3
Tn = cot b [tan ( a + nb ) - tan ( a + (n - 1 ) b }] - 1 (S1 - 2S 2 + S 3 )
Sum columnwise, 114. Let their ages be a, ar , ar 2.
Sn = T1 + T2 + T3 + ...+ Tn = cot b [tan ( a + nb ) - tan a ] - n
After 3 yr, their ages will be a + 3, ar + 3, ar 2 + 3.
sin ( a + nb ) sin a sin ( a + nb - a )
+ Given, 2 (a + 3 ) = ar 2 + 3 …(i)
cos ( a + nb ) cos a cos a cos ( a + nb )
= -n= -n Let x rupees be the sum of the money divided.
tan b tan b
sin nb And let y = a + ar + ar 2 …(ii)
= - n tan b 2
cos ( a + nb ) cos a Then, y + 9 = a + 3 + (ar + 3 ) + (ar + 3 )
tan b x (a + 3 ) x a
We have, = + 105
n (y + 9 ) y
n (n + 1 ) (n + 2 ) (n + 3 )
112. Sn = å Tr = æa + 3 a ö
r =1 8 Þ xç - ÷ = 105 …(iii)
èy + 9 y ø
r (r + 1 ) (r + 2 ) (r + 3 ) (r - 1 ) r (r + 1 ) (r + 2 )
Tr = Sr - Sr - 1 = - x (ar + 3 ) xar
8 8 Also, = + 15
r (r + 1 ) (r + 2 ) (y + 9 ) y
=
2 é ar + 3 ar ù
æ 1 ö Þ xê - ú = 15 …(iv)
1 2 (r + 2 ) - r 1
= = =ç - ÷ ëy +9 y û
Tr r (r + 1 )(r + 2 ) r (r + 1 ) (r + 2 ) è r (r + 1 ) (r + 1 )(r + 2 ) ø
On dividing Eq. (iii) by Eq. (iv), we get
n
1 n
æ 1 1 ö y (a + 3 ) - a (y + 9 ) y - 3a
å Tr = å çè r (r + 1) - (r + 1) (r + 2) ÷ø y (ar + 3 ) - ar (y + 9 )
=7 Þ
y - 3ar
=7
r =1 r =1
n ì a (7r - 1 )
æ1 1 ö æ 1 1 öü Þ 6y = 21ar - 3a Þ y =
= å íç - ÷-ç - ÷ý 2
r =1 î è r r + 1 ø è r + 1 r + 2 øþ
From Eq. (ii),
æ1 1 ö æ1 1 ö a(7r - 1 )
=ç - ÷-ç - ÷ = a + ar + ar 2
è1 n + 1ø è2 n + 2ø 2
1 1 1 n (n + 3 ) Þ 5ar = 3a + 2ar 2 …(v)
= + - =
2 n + 2 n + 1 2 (n + 1 ) (n + 2 ) From Eqs. (i) and (ii),
3
113. Let d1, d 2 and d 3 be the common differences of the 3 arithmetic a = 12, r =
2 2
progressions. 3 æ3ö
n Let ages of these friends are 12, 12 ´ , 12 ´ ç ÷ i.e. 12, 18, 27.
\ Si = [2 ´ a + (n - 1 ) di ], " i = 1, 2, 3 2 è2ø
2
1 1 1
n 115. Clearly, x = ,y = and z = .
Þ Si = [2 + (n - 1 ) di ] 1 -a 1 -b 1 -c
2
n (n - 1 ) 2 (Si - n ) Since, a, b, c are in AP.
Þ Si = n + di Þ di = Þ 1 - a, 1 - b, 1 - c are also in AP.
2 n (n - 1 )
1 1 1
Given that d1, d 2, d 3 are in HP. Þ , , are in HP.
1 1 1 1 -a 1 -b 1 -c
\ , , are in AP.
d1 d 2 d 3 \x, y , z are in HP.
Chap 03 Sequences and Series 309

1
1 é æ n (n + 1 ) ö
2
116. Q Bn = 1 - An > An Þ An < n (n + 1 ) (2n + 1 ) n (n + 1 ) ù
2 = ê2 ç ÷ - + úû
2 êë è 2 ø 6 2
3 æ

ç1 - æç - ö÷ ÷÷
3
1
4 çè è 4ø ø 1 æ 3ö
n
1 = n (n + 1 ) (3n 2 + n + 2 )
Now, An = < Þ ç- ÷ > - 12
3 2 è 4ø 6 1 1
1+ (ii) Vr + 1 - Vr = (r + 1 ) 3 - r 3 - [(r + 1 ) 2 - r 2 ] + ( 1 )
4 2 2
Obviously, it is true for all even values of n. = 3r 2 + 2r + 1
3 1 \ Tr = 3r 2 + 2r - 1
But for n = 1, - < -
4 6 = (r + 1 )(3r - 1 ), which is a composite number.
æ 3ö
3
27 1 (iii) Since, Tr = 3r 2 + 2r - 1
n = 3 ç- ÷ = - <- \ Tr + 1 = 3 (r + 1 ) 2 + 2 (r + 1 ) - 1
è 4ø 24 6
5 \ Qr = Tr + 1 - Tr = 3 [2r + 1 ] + 2[1 ]
æ 3ö 243 1
n = 5, ç - ÷ = - >- Þ Qr = 6r + 5
è 4ø 1024 6 Þ Qr + 1 = 6(r + 1 ) + 5
which is true for n = 7 obviously, n0 = 7 Common difference = Qr + 1 - Qr = 6
Aliter Bn = 1 - An > An a+b 2ab
æ
120. (i) A 1 = ; G 1 = ab ; H1 =
3 ö
n
a+b
çç1 - æç - ö÷ ÷÷
2
1 3 è è 4ø ø 1 æ 3ö
n
1 An - 1 + Hn - 1 2 An - 1Hn - 1
Þ An < Þ < Þ ç- ÷ > - An = , Gn = An - 1Hn - 1 and Hn =
2 4 3 2 è 4ø 6 2 An - 1 + Hn - 1
1+
4 Clearly, G1 = G2 = G3 = ¼ = ab
Obviously, it is true for all even values of n.
3 1 (ii) A2 is AM of A1 and H1 and A1 > H1
But for n = 1, - < - Þ A1 > A2 > H1
4 6
3 A3 is AM of A2 and H 2 and A2 > H 2
æ 3ö 27 1
n = 3, ç - ÷ = - <- Þ A2 > A3 > H 2
è 4ø 64 6
5 \ A1 > A2 > A3 > ¼
æ 3ö 243 1
(iii) As above A1 > H 2 > H1, A1 > H 3 > H 2
n = 5, ç - ÷ = - <-
è 4ø 1024 6
\ H1 < H 2 < H 3 < ¼
7
æ 3ö 2187 1 121. Let geometric progression is a, ar , ar 2,¼, [a, r > 0 ]
and for n = 7 Þ ç- ÷ = - >-
è 4ø 12288 6 2
Q a = ar + ar
Hence, minimum natural number n0 = 7. -1 ± 5
p Þ r2 + r - 1 = 0 Þ r=
[2a1 + ( p - 1 ) d ] 2
2 p2 5 -1
117. Q = 2 \ r=
q 2
[2a1 + (q - 1 ) d ] q
2
æ p - 1ö 122. b1 = a1, b2 = b1 + a 2 = a1 + a 2, b3 = b2 + a 3 = a1 + a 2 + a 3
a1 + ç ÷d and b4 = b3 + a 4 = a1 + a 2 + a 3 + a 4
2a1 + ( p - 1 ) d p è 2 ø p
Þ = Þ =
2a1 + (q - 1 ) d q æq - 1ö q Hence, b1, b2, b3, b4 are neither in AP nor in GP and nor in HP
a1 + ç ÷d
è 2 ø 123. Let a, ar , ar 2, ¼
a a 6 11
For 6 , p = 11 and q = 41 Þ = a + ar = 12 …(i)
a 21 a 21 41
and ar 2 + ar 3 = 48 …(ii)
1 1 1 1 1 1
118. - = - =¼= - =d [say] On dividing Eq. (ii) by Eq. (i), we get
a 2 a1 a 3 a 2 an an - 1
an - 1 - an r 2 = 4, if r ¹ - 1
a - a2 a - a3
Then, a1a 2 = 1 , a 2a 3 = 2 , ¼, an - 1an = \ r = -2
d d d
a1 - an [Q terms are alternatively positive and negative]
\ a1a 2 + a 2 a 3 + ¼ + an - 1an =
d Now, from Eq. (i), a = - 12
1 1 a1 - an 124. Q Sn = cn 2
Also, = + (n - 1 ) d Þ = (n - 1 ) a1an
an a1 d
\ tn = Sn - Sn - 1 = c (2n - 1 )
\ a1a 2 + a 2 a 3 + ¼ + an - 1an = (n - 1 ) a1an
Stn2 = c 2 S(2n - 1 ) 2
r 1
119. (i) Vr = [(2 r + (r - 1 ) (2 r - 1 )] = (2r 3 - r 2 + r )
2 2 = c 2 S ( 4n 2 - 4n + 1 ) = c 2 { 4 Sn 2 - 4 Sn + S1}
n
1é n 3 n 3 n ù ì 4n (n + 1 ) (2n + 1 ) 4n (n + 1 ) ü
\ åVr = ê2 år - år + år ú = c2 í - + ný
2 êë r = 1 î 6 2 þ
r =1 r =1 r =1 úû
310 Textbook of Algebra

ì2 ü Þ (7d + 9 ) (d + 3 ) = 0
= c 2n í ( 2n 2 + 3n + 1 ) - 2n - 2 + 1ý 9
î3 þ \ d = - 3, d ¹ - [Q 27 - 2a 2 > 0 ]
2 2 2 7
c n n ( 4n - 1 ) c
= ( 4n 2 - 1 ) = 11
{2a1 + (11 - 1 ) d }
3 3 a1 + a 2 + a 3 + ¼ + a11
2 6 10 14 \ = 2
125. Let S = 1 + + 2 + 3 + 4 + ¼ …(i) 11 11
3 3 3 3 = a1 + 5d = 15 - 15 = 0
1 1 2 6 10 128. Till 10th minute, number of counted notes = 1500
\ S= + + 3 + 4 +¼ …(ii)
3 3 32 3 3 n
\3000 = {2 ´ 148 + (n - 1 ) ´ - 2 } = n (148 - n + 1 )
On subtracting Eq. (ii) from Eq. (i), we get 2
2 1 4 4 4 Þ n 2 - 149n + 3000 = 0
S =1 + + 2 + 3 + 4 + ¼
3 3 3 3 3 Þ (n - 125 ) (n - 24 ) = 0
ì 1 ü \ n = 125, 24
4 æ1 1 1 ö 4 ï 2 ï 4 2 n = 125 is not possible.
= + 4 ç 2 + 3 + 4 + ¼÷ = + 4 í 3 ý= + = 2
3 è ø 3 1 \ n = 24
3 3 3 ï1 - ï 3 3
î 3þ \Total time = 10 + 24 = 34 min
\ S =3
k -1 129. Q AM ³ GM
a k 1 a - 5 + a - 4 + a - 3 + a - 3 + a - 3 + 1 + a 8 + a10
126. Sk = = k! = = \
1 - r 1 - 1 k ! (k - 1 )! 8
k ³ (a - 5 × a - 4 × a - 3 × a - 3 × a - 3 × 1 × a 8 × a10 )1/8 = (1 )1/8 = 1
100 100
1 a - 5 + a - 4 + 3a - 3 + 1 + a 8 + a10
Now, å |(k 2 - 3k + 1 ) Sk | = å | (k 2 - 3k + 1 ) × | Þ ³1
k=2 k=2 (k - 1 )! 8
-5 -4 -3 8 10
\ a + a + 3a + 1 + a + a ³ 8
100
(k - 1 ) k Þ Required minimum value = 8
=å =
k=2 (k - 2 ) (k - 1 )! 130. Let the time taken to save ` 11040 be (n + 3) months.
1 2 2 3 3 4 99 100 For first 3 months, he saves ` 200 each month.
= - + - + - + ¼+ - \In (n + 3 ) month,
0! 1! 1! 2! 2! 3! 98 ! 99 !
n
3 ´ 200 + [2 (240 ) + (n - 1 ) ´ 40 ] = 11040
æ2 1 ö æ2 3ö æ3 4ö æ 99 100 ö 2
= ç - ÷ + ç - ÷ + ç - ÷ + ¼+ ç - ÷
è 1! 0!ø è 1! 2!ø è 2! 3!ø è 98 ! 99 ! ø n
Þ 600 + [ 40 (12 + n - 1 )] = 11040
100 (100 ) 2 2
=3 - =3 - Þ 600 + 20n (n + 11 ) = 11040
99! 100 !
Þ n 2 + 11n - 522 = 0
(100 ) 2 100 Þ (n - 18 ) (n + 29 ) = 0
\ + å |(k 2 - 3k + 1 ) Sk | = 3
100 ! k = 2 \ n = 18, neglecting n = - 29
ak - 2 + ak \Total time = n + 3 = 21 months
127. Q ak = 2ak - 1 - ak - 2 or ak - 1 = 131. Given, a 2 + a 4 + a 6 + ¼ + a 200 = a …(i)
2
\ a1, a 2, a 3, ¼are in AP. and a1 + a 3 + a 5 + ¼ + a199 = b …(ii)
11
a 2 + a 22 + a 32 + ¼ + a11
2 On subtracting Eq. (ii) from Eq. (i), we get
Q 1
11
= 90 Þ å ai2 = 11 ´ 90 (a 2 - a1 ) + (a 4 - a 3 ) + (a 6 - a 5 ) + ¼+
k =1
11 (a 200 - a199 ) = a - b
Þ å 2
(a1 + (i - 1 ) d ) = 11 ´ 90 Þ d + d + d +¼+ d = a - b Þ 100d = a - b
k =1
( a - b)
11 \ d =
Þ å {a12 + 2a1d (i - 1) + d 2 (i - 1)2 } = 11 ´ 90 100
k =1 132. Qa1, a 2, a 3, ¼are in HP.
Þ 11 ´ a12 + 2a1d ( 0 + 1 + 2 + 3 + ¼ + 10 ) \
1 1 1
, , , ¼are in AP.
+ d 2 ( 0 2 + 1 2 + 2 2 + ¼ + 10 2 ) = 11 ´ 90 a1 a 2 a 3
æ 10 × 11 ö Let D be the common difference of this AP, then
2 æ 10 × 11 × 21 ö
Þ 11 ´ 15 2 + 2 ´ 15 ´ d × ç ÷ +d ×ç ÷ 1 1
è 2 ø è 6 ø = + (20 - 1 ) D
a 20 a1
= 11 ´ 90
1 1
Þ 385d 2 + 1650d + 1485 = 0 [Q a1 = 15 ] -
25 5 =- 4
Þ 7d 2 + 30d + 27 = 0 Þ D=
19 25 ´ 19
Chap 03 Sequences and Series 311

1 1 1 4 (n - 1 ) æ 95 - 4n + 4 ö \x, y , z are in GP. ...(iii)


and = + (n - 1 ) D = - =ç ÷
an a1 5 25 ´ 19 è 25 ´ 19 ø From Eqs. (i) and (ii) x, y , z are in AP and also in GP, then
x = y = z.
æ 99 - 4n ö
=ç ÷<0 [Q an < 0 ] 136. S = 0.7 + 0.77 + 0.777 + K upto 20 terms
è 25 ´ 19 ø
7
Þ 99 - 4n < 0 Þ n > 24 .75 = (0.9 + 0.99 + 0.999 + K upto 20 terms)
9
Hence, the least positive integer is n = 25. 7
= [(1 - 0.1 ) + (1 - 0.01 ) + (1 - 0.001 ) + K upto 20 terms]
133. Q (1 ) = (1 - 0 ) (1 2 + 1 × 0 + 0 2 ) = 1 3 - 0 3 9
7
(1 + 2 + 4 ) = (2 - 1 ) (2 2 + 2 × 1 + 1 2 ) = 2 3 - 1 3 = [(1 + 1 + 1 + ¼ upto 20 times)
9
( 4 + 6 + 9 ) = (3 - 2 ) (3 2 + 3 × 2 + 2 2 ) = 3 3 - 2 3 æ1 1 1 öù
- ç + 2 + 3 + ¼ upto 20 terms÷ ú
M M M è 10 10 10 øû
é æ 20 ö ù
çç 1 - æç ö÷ ÷÷ ú
(361 + 380 + 400 ) = (20 - 19 ) (20 2 + 20 × 19 + 19 2 ) = 20 3 - 19 3 1 1
ê
7ê 10 è è 10 ø ø ú 7 é 180 - 1 + 10 -20 ù
Required sum = ê 20 -
9 1 ú = 9ê 9
ú
= (1 3 - 0 3 ) + (2 3 - 1 3 ) + (3 3 - 2 3 ) +¼ + (20 2 - 19 3 ) = 20 3 = 8000 ê 1- ú ë û
n n ê 10 ú
Also, åk 3 - (k - 1 ) 3 = ë û
å {k - (k - 1)} {k 2 + k(k - 1) + (k - 1)2 } =
7
(179 + 10 - 20 )
k =1 k =1
n
81
= å(3k 2
- 3k + 1 ) = 3 Sn 2 - 3 Sn + S1 137. Sn = - 1 2 - 2 2 + 3 2 + 4 2 - 5 2 - 6 2 + 7 2 + 8 2 -¼
k =1
+ ( 4n - 1 ) 2 + ( 4n ) 2
3n(n + 1 ) (2n + 1 ) 3n (n + 1 )
= - +n = (3 2 - 1 2 ) + ( 4 2 - 2 2 ) + (7 2 - 5 2 ) + (8 2 - 6 2 ) + ¼
6 2
n + [{( 4n - 1 ) 2 - ( 4n - 3 ) 2 } + {( 4n ) 2 - ( 4n - 2 ) 2 }]
= (2n 2 + 3n + 1 - 3n - 3 + 2 ) = n 3
2 = 4 [2 + 3 + 6 + 7 + 10 + 11 + ¼ + ( 4n - 2 ) + ( 4n - 1 )]
Both statements are correct and Statement-2 is the correct = 8 {(1 + 3 + 5 + ¼ + (2n - 1 )} + 4 {3 + 7 + 11
explanation of Statement-1. + ¼ + ( 4n - 1 )}
134. Let a be the first term and d be the common difference. Then, = 16n 2 + 4n = 4n ( 4n + 1 ), n Î N
100 T100 = 50 T50 Satisfied by (a) and (d), where n = 8, 9, respectively.
Þ 100 (a + 99d ) = 50 (a + 49d ) 138. Let two consecutive numbers are k and k + 1 such that
Þ 2 (a + 99d ) = (a + 49d ) Þ a + 149 d = 0 1 £ k £ n - 1, then
\ T150 = 0 (1 + 2 + 3 + ¼ + n ) - (k + k + 1 ) = 1224
135. Q x, y , z are in AP. n (n + 1 ) n 2 + n - 2450
Þ - (2k + 1 ) = 1224 or k =
Let x = y - d , z = y + d ...(i) 2 4
Also, given tan - 1 x, tan - 1 y , tan - 1 z are in AP. n 2 + n - 2450
Now, 1£ £ n - 1 Þ 49 < n < 51
\ 2 tan - 1 y = tan - 1 x + tan - 1 z 4
\ n = 50 Þ k = 25
æ 2y ö æx+z ö
Þ tan - 1 ç ÷ = tan - 1 ç ÷ Hence, k - 20 = 25 - 20 = 5
è1 - y ø 2
è 1 - xz ø 139. The given series can be written as
2y x+z 2y 2y 2 8 9
Þ = Þ = æ 11 ö æ 11 ö æ 11 ö æ 11 ö
1 - y 2 1 - xz 1 - y 2 1 - (y 2 - d 2 ) k = 1 + 2 ç ÷ + 3 ç ÷ + ¼ + 9 ç ÷ + 10 ç ÷ …(i)
è 10 ø è 10 ø è 10 ø è 10 ø
2 2 2
Þ y =y -d [from Eq. (i)] æ 11 ö
On multiplying both sides by ç ÷ , then
\ d =0 è 10 ø
From Eq. (i), x = y and z = y 11k æ 11 ö
2 3 9 10
æ 11 ö æ 11 ö æ 11 ö æ 11 ö
\ x =y =z = ç ÷ + 2 ç ÷ + 3 ç ÷ + ¼ + 9 ç ÷ + 10 ç ÷ …(ii)
10 è 10 ø è 10 ø è 10 ø è 10 ø è 10 ø
Aliter
Q x, y , z are in AP. ...(i) Now, on subtracting Eq. (ii) from Eq. (i), then
\ 2y = x + z ...(ii) 2 2 10
k æ 11 ö æ 11 ö æ 11 ö æ 11 ö
Also, tan - 1 x, tan - 1 y , tan - 1 z are in AP. - = 1 + ç ÷ + ç ÷ + ¼ + ç ÷ - 10 ç ÷
10 è 10 ø è 10 ø è 10 ø è 10 ø
\ 2 tan - 1 y = tan - 1 x + tan - 1 z 1444442444443
10 times
æ 2y ö æx+z ö
Þ tan - 1 ç ÷ = tan - 1 ç ÷ ìï æ 11 ö 10 üï
è1 - y ø 2
è 1 - xz ø 1 × í ç ÷ - 1ý
ï è 10 ø ïþ æ 11 ö
10
2y x+z 2y = î - 10 ç ÷
Þ = = [from Eq. (ii)] æ 11 ö è 10 ø
1 - y 2 1 - xz 1 - xz ç - 1÷
è 10 ø
Þ y 2 = zx
312 Textbook of Algebra

ïì æ 11 ö ïü
10 10
æ 11 ö or a = 9d and 130 < a 7 < 140
Þ k = - 100 × í ç ÷ - 1ý + 100 ç ÷ = 100
è ø è 10 ø
îï 10 þï Þ 130 < a1 + 6d < 140 Þ 130 < 15d < 140
2
140. Let a, ar , ar are in GP. Q GP is increasing. 2 1
\ 8 <d <9 Þ d =9 (Qa, d Î N )
\ r >1 3 3
New numbers a, 2ar , ar 2 are in AP. 145. Q a + d , a + 4d , a + 8d are in GP (d ¹ 0)
\ 4ar = a + ar 2 Þ r 2 - 4r + 1 = 0 \ (a + 4d ) 2 = (a + d ) (a + 8d )
4 ± (16 - 4 )
Hence, r= =2 + 3 [Qr > 1] Þ a = 8d
2 a + 4d 8d + 4d 4
b c \Common ratio = = = (Qa = 8d )
141. Let = =r a+d 8d + d 3
a b
\ b = ar , c = ar 2 Aliter
a+b+c b c æb ö 6 Let the GP be a, ar , ar 2 and terms of AP and A + d , A + 4d ,
Given, =b + 2 Þ 1 + + =3 ç ÷ +
3 a a èa ø a A + 8d , then
6 ar 2 - ar ( A + 8d ) - ( A + 4d ) 4
Þ 1 + r + r 2 = 3r + r= = =
a ar - a ( A + 4d ) - ( A + d ) 3
Now, for a = 6, only we get r = 0, 2 [rational] 2 2 2 2 2
æ8ö æ 12 ö æ 16 ö æ 20 ö æ 44 ö
So, r =2 146. ç ÷ + ç ÷ + ç ÷ + ç ÷ +K+ ç ÷
è5ø è5ø è5ø è5ø è5ø
Þ (a, b, c ) = (6, 12, 24 )
2 16 2
a + a - 14 36 + 6 - 14 = (2 + 3 2 + 4 2 + 5 2 + K 11 2 )
\ = =4 25
a+1 6+1 2
æ n (n + 1 ) ö 16 æ 11 . (11 + 1 ) . (22 + 1 ) ö
ç ÷ = ç - 1÷
13 + 23 + 33 + ¼ + n 3 è 2 ø (n + 1 ) 2 25 è 6 ø
142. Tn = = =
1 + 3 + 5 + ¼ + (2n - 1 ) n (1 + 2n - 1 ) 4 16 16 16
1 2 2 = ´ 505 = ´ 101 = m (given)
= (n + 2n + 1 ) 25 5 5
4 \m = 101
1 1 é n (n + 1 )(2n + 1 ) 2n(n + 1 ) ù
\ Sn = ( Sn 2 + 2 Sn + S1 ) = ê + + nú 147. Q loge b1, loge b2, loge b3, ..., loge b101 are in AP.
4 4ë 6 2 û
1 Þ b1, b2, b3, K, b101 are in GP with common ratio 2.
S9 = [285 + 90 + 9 ] = 96 (Q common difference = loge 2)
4
l +n Also, a1, a 2, a 3, K a101 are in AP.
143. Given, m = Þl + n = 2m …(i)
2 where, a1 = b1 and a 51 = b51
and l , G1, G2, G3, n are in GP. \ b2, b3, K, b50 are GM’s and a 2, a 3, ...., a 50 are AM’s between b1
G1 G2 G3 n and b51.
\ = = =
l G1 G2 G3 GM < AM
Q
Þ G1G3 = ln, G12 = lG2, G22 = G3G1, G32 = nG2 …(ii) Þ b2 < a 2, b3 < a 3, K, b50 < a 50
Now, G14 + 2 G24 + G34 = l 2 G22 + 2 G24 + n 2 G22 \ b1 + b2 + b3 + K + b51 < a1 + a 2 + a 3 + K + a 51
= G22 2
(l + 2G22 +n ) 2
[from Eq. (ii)] Þ t <s
Also, a1, a 2, a 3, K, a101 are in AP and b1, b2, b3, .... b101 are in GP.
= G3G1 (l 2 + 2G3G1 + n 2 ) [from Eq. (ii)]
Q a1 = b1 and a 51 = b51
= ln (l 2 + 2 ln + n 2 ) [from Eq. (ii)]
\ b101 > a101
= ln (l + n ) 2 = ln (2m ) 2 [from Eq. (i)] = 4lm 2n
148. (15a ) 2 + (3b ) 2 + (5c ) 2 - 45ab - 15bc - 75ac = 0
144. Let first term = a and common difference = d 1
sum of seven terms 6 Þ {(15a - 3b ) 2 + (3b - 5c ) 2 + (5c - 15a ) 2 } = 0
Q = 2
sum of eleven terms 11 Þ (15a - 3b ) 2 + (3b - 5c ) 2 + (5c - 15a ) 2 = 0
7 7
(a1 + a 7 ) (2a + 6d ) or 15a - 3b = 0, 3b - 5c = 0, 5c - 15a = 0
2 6 6
Þ = Þ 2 = b = 5a, c = 3a
11 11 11 11 Q
(a1 + a11 ) (2a + 10d )
2 2 Þ 5a, 3a, a are in AP i.e. b, c, a are in AP.
CHAPTER

04
Logarithms and
Their Properties
Learning Part
Session 1
● Definition

● Characteristic and Mantissa

Session 2
● Principle Properties of Logarithm

Session 3
● Properties of Monotonocity of Logarithm

● Graphs of Logarithmic Functions

Practice Part
● JEE Type Examples
● Chapter Exercises

Arihant on Your Mobile !


Exercises with the #L
symbol can be practised on your mobile. See inside cover page to activate for free.
314 Textbook of Algebra

The technique of logarithms was introduced by John Napier (1550-1617). The logarithm is a form of indices which is
used to simplify the algebraic calculations. The operations of multiplication, division of a very large number becomes
quite easy and get converted into simple operations of addition and subtraction, respectively. The results obtained are
correct upto some decimal places.

Session 1
Definition, Characteristic and Mantissa
Definition Corollary II The function defined by
f ( x ) = log a x , a > 0, a ¹ 1 is called logarithmic function. Its
The logarithm of any positive number, whose base is a
domain is (0, ¥) and range is R (set of all real numbers).
number ( > 0 ) different from 1, is the index or the power to
which the base must be raised in order to obtain the given Corollary III a x > 0, " x Î R
number. (i) If a > 1 , then a x is monotonically increasing.
x
i.e. if a = b (where a > 0, ¹ 1), then x is called the
For example, 5 2.7 > 5 2.5 , 3 222 > 3 111
logarithm of b to the base a and we write log a b = x ,
clearly b > 0. Thus, log a b = x Û a x = b, a > 0, a ¹ 1 and (ii) If 0 < a < 1 , then a x is monotonically decreasing.
b > 0. æ 1ö
2.7
æ 1ö
2.5
For example, ç ÷ <ç ÷ , (0.7 ) 222 < (0.7 ) 212
If a = 10, then we write log b rather than log 10 b. If a = e , è5ø è5ø
we write ln b rather than log e b. Here, ‘e’ is called as
Napier’s base and has numerical value equal to 2.7182. Corollary IV
Also, log 10 e is known as Napierian constant. (i) If a > 1, then a - ¥ = 0
i.e. log 10 e = 0 . 4343 \ log a 0 = - ¥ (if a > 1)
\ ln b = 2.303 log 10 b (ii) If 0 < a < 1 , then a ¥ = 0
é 1
êsince, ln b = log 10 b ´ log e 10 = log e ´ log 10 b \ log a 0 = + ¥ (if 0 < a < 1)
ë 10
Corollary V (i) log a b ® ¥, if a > 1, b ® ¥
1 ù
= log 10 b = 2.303 log 10 bú (ii) log a b ® - ¥, if 0 < a < 1 , b ® ¥
0.4343 û

Remember Remark
1. ‘log’ is the abbreviation of the word ‘logarithm’.
(i) log 2 = log 10 2 = 0.3010
2. Common logarithm (Brigg’s logarithms) The base is 10.
(ii) log 3 = log 10 3 = 0.4771 3. If x < 0, a > 0 and a ¹ 1, then log a x is an imaginary.
(iii) ln 2 = 2.303log2 = 0.693 ì+ ve, x >1
ï
4. If a > 1, log a x = í 0, x =1
(iv) ln 10 = 2.303 ï - ve, 0 < x < 1
î
Corollary I From the definition of the logarithm of the ì+ ve, 0 < x < 1
number b to the base a, we have an identity ï
And if 0 < a < 1,log a x = í 0, x =1
a loga b = b, a > 0, a ¹ 1 and b > 0 ï - ve, x >1
î
which is known as the Fundamental Logarithmic 5. log a 1 = 0 ( a > 0, a ¹ 1)
Identity. log a a = 1 ( a > 0, a ¹ 1) and log ( 1/ a) a = -1 ( a > 0, a ¹ 1)
Chap 04 Logarithms and Their Properties 315

y Example 1. Find the value of the following : y Example 2. Find the value of the following:
(i) log 9 27 (ii) log 3 2 324 (i) log tan 45 °cot 30° (ii) log (sec 2 60 ° - tan 2 60 °)cos60°
(iii) log 1/ 9 (27 3 ) (iv) log ( 5 + 2 6 ) (5 - 2 6 ) (iii) log (sin 2 30 ° + cos 2 30 °) 1 (iv) log 30 1
(v) log 0 . 2 0008
. (vi) 2 2log 4 5 Sol. (i) Here, base = tan45° = 1 tan
ì1
-log 2.5 í +
1
+
1 ü
+ Ký \ log is not defined.
î
ï 3 32 33 þ log 20 ( 0. 3 )
(vii) (0.4) (viii) (0.05) (ii) Here, base = sec2 60° - tan 2 60° = 1
Sol. (i) Let x = log 9 27 \ log is not defined.
Þ 9 x = 27 Þ 32 x = 33 Þ 2x = 3 (iii) Q log (sin 2 30 ° + cos 2 30 ° ) 1 = log1 1 ¹ 1
3 Q Here, base = 1
\ x=
2 \ log is not defined.
(ii) Let x = log 3 (iv) log 30 1 = 0
2 324
Þ (3 2 ) = 324 = 2 ×3 Þ (3 2 )x = (3 2 )4
x 2 4

\
(iii) Let
x=4
x = log1 / 9 (27 3 )
Characteristic and Mantissa
The integral part of a logarithm is called the
x
æ1ö -2 x characteristic and the fractional part (decimal part) is
Þ ç ÷ = 27 3 Þ 3 = 37 / 2 Þ - 2x = 7 / 2
è9 ø called mantissa.
\ x=-
7 i.e., log N = Integer + Fractional or decimal part ( +ve)
4 ¯ ¯
(iv) Q (5 + 2 6 ) (5 - 2 6 ) = 1 Characteristic Mantissa
1 The mantissa of log of a number is always kept positive.
or 5 + 2 6 = …(i)
5 -2 6 i.e., if log564 = 2.751279, then 2 is the characteristic and
Now, let x = log (5 + 2 6 ) (5 - 2 6) 0.751279 is the mantissa of the given number 564.
= log1/ (5 - 2 And if log 0.00895 = - 2.0481769
6)5 - 2 6 = -1 [from Eq. (i)]
= - 2 - 0.0481769
(v) Let x = log 0.2 0.008
= ( -2 - 1) + (1 - 0.0481769 )
Þ (0.2)x = 0.008 Þ (0.2)x = (0.2)3 Þ x = 3
= - 3 + 0.9518231
(vi) Let x = 22 log 4 5 = 4 log 4 5 = 5
ì1 1 1 ü
Hence, -3 is the characteristic and 0.9518231
- log 2.5 í + + + Ký
(vii) Let x = (0.4) îï 3 3 2 3 3 þ (not 0.0481769) is mantissa of log 0.00895.
ì 1 ü In short, -3 + 0.9518231 is written as 3.9518231 .
ï ï æ 1ö æ 1ö
- log 2.5 í 3 ý - log 2.5 ç ÷ log 5 / 2 ç ÷
æ4 ö ï 1- 1 ï æ2ö è 2ø æ5ö è 2ø 1
=ç ÷ ïî 3þ =ç ÷ =ç ÷ = Remark
è 10 ø è5ø è2ø 2
1. If N > 1, the characteristic of log N will be one less than the
log ( 0. 3 ) log (l ) number of digits in the integral part of N.
(viii) Let x = (0.05) 20 = (0.05) 20 …(i)
. = 2.3723227
For example, If log 23568
where, l = 0.3
Here, N = 23568 .
Then, l = 0.33333 ... …(ii) \ Number of digits in the integral part of N = 3
Þ 10l = 3.33333 ... …(iii) Þ Characteristic of log 23568 . = N -1= 3-1=2
On subtracting Eq. (ii) from Eq. (iii), we get 2. If 0 < N < 1, the characteristic of log N is negative and
1 numerically it is one greater than the number of zeroes
9l = 3 Þ l = immediately after the decimal part in N.
3
æ 1ö
log 20 ç ÷ For example, If log0 .0000279 = 5 .4456042
è 3ø
Now, from Eq. (i), x = (0.05) Here, four zeroes immediately after the decimal point in the
-1 1
log
(20)1/2
(3 ) - log 20 3 number 0.0000279 is ( 4 + 1), i.e. 5.
æ1ö æ1ö 1/ 2
=ç ÷ =ç ÷ 3. If the characteristics of log N be n, then number of digits in N
è 20 ø è 20 ø is ( n + 1) (Here, N > 1).
2
= 20(2 log 20 3 ) = 20log 20 3 = 32 = 9 4. If the characteristics of log N be -n, then there exists ( n - 1)
number of zeroes after decimal part of N (here, 0 < N < 1).
316 Textbook of Algebra

y Example 3. If log 2 = 0.301 and log 3 = 0.477, find Sol. Let P = (0.036)16 Þ log P = 16log (0.036)

the number of digits in 6 20 . æ 36 ö æ 22 × 32 ö


= 16 log ç ÷ = 16 log ç ÷
Sol. Let P = 6 20
= ( 2 ´ 3) 20 è 1000 ø è 1000 ø
\ log P = 20 log(2 ´ 3) = 20 {log 2 + log 3} = 16 {log 22 + log 32 - log 103 }
= 20 {0.301 + 0.477}
= 16 {2 log 2 + 2 log 3 - 3}
= 20 ´ 0.778 = 15.560
= 16 {2 ´ 0.301 + 2 ´ 0.477 - 3}
Since, the characteristic of log P is 15, therefore the number
of digits in P will be 15 + 1, i.e. 16. = 16 {1.556 - 3} = 24.896 - 48
= -48 + 24 + 0.896
y Example 4. Find the number of zeroes between the
decimal point and first significant digit of (0.036)16 , = -24 + 0.896 = 24 + 0.896

where log 2 = 0.301 and log 3 = 0.477. \ The required number of zeroes = 24 - 1 = 23.

#L Exercise for Session 1


1. The value of log2 3 1728 is
(a) 6 (b) 8
(c) 3 (d) 5

2. The value of log( 8 - 3 7 )(8 + 3 7 ) is


(a) -2 (b) -1
(c) 0 (d) Not defined
ì1 1 ü
log 2.5í + + Ký
î 3 32 þ
3. The value of (0.16) is
(a) 2 (b) 4
(c) 6 (d) 8

4. If log 2 = 0.301, the number of integers in the expansion of 417 is


(a) 9 (b) 11
(c) 13 (d) 15

5. If log 2 = 0.301, then the number of zeroes between the decimal point and the first significant figure of 2-34 is
(a) 9 (b) 10
(c) 11 (d) 12
Session 2
Principle Properties of Logarithm

Principle Properties (vii) log a x 2 ¹ 2 log a x , a > 0, a ¹ 1


of Logarithm Since, domain of log a ( x 2 ) is R ~ {0 } and domain of
log a x is (0, ¥) are not same.
Let m and n be arbitrary positive numbers,
a > 0, a ¹ 1, b > 0, b ¹ 1 and a, b be any real numbers, then (viii)a logb a = a , if b = a 2 , a > 0, b > 0, b ¹ 1
(i) log a (m n ) = log a m + log a n (ix) a logb a = a 2 , if b = a , a > 0, b > 0, b ¹ 1
In general, log a ( x 1 x 2 x 3 K x n ) = log a x 1
y Example 5. Solve the equation 3 × x log 5 2 + 2log 5 x = 64.
+ log a x 2 + log a x 3 + K + log a x n
Sol. Q 3 × x log 5 2 + 2log 5 x = 64
(where, x 1 , x 2 , x 3 , ..., x n > 0)
Þ 3 × 2log 5 x + 2log 5 x = 64 [by extra property (i)]
Or
log 5 x
æ n ö n Þ 4 ×2 = 64
log a ç Õ x i ÷ = å log a x i , "x i > 0
èi =1 ø i = 1 Þ 2log 5 x = 4 2 = 24
where, i = 1 , 2, 3, ..., n. \ log 5 x = 4
æm ö Þ x = 54 = 625
(ii) log a ç ÷ = log a m - log a n
ènø
y Example 6. If 4 log16 4 + 9log 3 9 = 10log x 83 , find x .
(iii) log a m a = a log a m
Sol. Q 4 log16 4 = 4 = 2 [by extra property (ix)]
1 log a m
(iv) log ab m = log a m (v) log b m = and 9 log 3 9 2
= 9 = 81 [by extra property (viii)]
b log a b
\ 4 log16 4 + 9 log 3 9 = 2 + 81 = 83 = 10log x 83
Remark Þ log10 83 = log x 83
1
1. log b a × log a b = 1 Û log b a = \ x = 10
log a b
2. log b a × log c b × log a c = 1 y Example 7. Prove that a log a b
-b
log b a
= 0.
3. log y x × log z y × log a z = log a x
(loga b ) loga b ´ loga b ´ logb a
x
4. eln a = ax Sol. Q a =a
loga b × (logb a )
=a
Extra Properties of Logarithm =b logb a
[by extra property (ii)]
logb x logb a
(i) a =x , b ¹ 1, a, b, x are positive numbers. Hence, a (loga b )
-b (logb a )
=0
loga x
(ii) a = x , a > 0, a ¹ 1, x > 0
log 2 24 log 2 192
1 y Example 8. Prove that - = 3.
(iii) log ak x = log a x , a > 0, a ¹ 1, x > 0 log 96 2 log 12 2
k
log 2 24 log 2 192
(iv) log a x 2k = 2k log a | x |, a > 0, a ¹ 1, k Î I Sol. LHS = -
log 96 2 log12 2
1
(v) loga 2k x = log|a | x , x > 0, a > 0, a ¹ ± 1 and k Î I ~ {0} = log 2 24 ´ log 2 96 - log 2 192 ´ log 2 12
2k
Now, let 12 = l, then
b
(vi) log a a x b = log a x , x > 0, a > 0, a ¹ 1, a ¹ 0 LHS = log 2 2l ´ log 2 8l - log 2 16l ´ log 2 l
a
318 Textbook of Algebra

= (log 2 2 + log 2 l ) (log 2 8 + log 2 l ) y Example 9. Solve for a, l, if


- (log 2 16 + log 2 l )log 2 l log l a× log 5 l × log l 25 = 2.
3
= (log 2 2 + log 2 l ) (log 2 2 + log 2 l ) Sol. Here, l > 0, l ¹ 1
We have, log l a × {log 5 l × log l 25} = 2
- (log 2 24 + log 2 l )log 2 l
Þ (log l a ){log 5 25} = 2
= (1 + log 2 l ) (3 log 2 2 + log 2 l )
Þ (log l a ){log 5 52 } = 2
- ( 4 log 2 2 + log 2 l )log 2 l
Þ (log l a ){2 log 5 5} = 2
= (1 + log 2 l ) (3 + log 2 l ) - log 2 l ( 4 + log 2 l )
Þ (log l a ){ 2 } = 2
=3
\ log l (a ) = 1 or a = l
= RHS

#L Exercise for Session 2


1 If a = log24 12, b = log48 36 and c = log36 24, 1 + abc is equal to
(a) 2ab (b) 2bc (c) 2ca (d) ba + bc

2 The value of log4[log2 {log2(log3 81)}] is equal to


(a) -1 (b) 0 (c) 1 (d) 2

3 log2log2 ( ... 2 ) is equal to


14243
n times

(a) 0 (b) 1 (c) n (d) -n

4 If a = log3 5, b = log1725, which one of the following is correct?


(a) a < b (b) a = b (c) a > b (d) None of these
-2
5 The value of log0.75 log2 (0.125) is equal to
(a) -1 (b) 0 (c) 1 (d) None of these
Session 3
Properties of Monotonocity of Logarithm,
Graphs of Logarithmic Functions

Properties of Monotonocity Graphs of Logarithmic


of Logarithm Functions
1. Graph of y = log a x , if a > 1 and x > 0
1. Constant Base Y

ì x > y > 0, if a > 1


(i) log a x > log a y Û í
î0 < x < y , if 0 < a < 1 X′
O
X
(1, 0)
ì 0 < x < y , if a > 1
(ii) log a x < log a y Û í
îx > y > 0, if 0 < a < 1 Y′

ìx > a p , if a > 1 2. Graph of y = log a x , if 0 < a < 1 and x > 0


(iii) log a x > p Û í p Y
î0 < x < a , if 0 < a < 1
ì 0 < x < a p , if a > 1
(iv) log a x < p Û í p X′
(1, 0)
X
îx > a , if 0 < a < 1 O

2. Variable Base Y′

(i) log x a is defined, if a > 0, x > 0, x ¹ 1. Remark


1. If the number x and the base ‘a’ are on the same side of the
(ii) If a > 1 , then log x a is monotonically decreasing in unity, then the logarithm is positive.
(0, 1) È (1, ¥). Case I y = log ax , a > 1, x > 1 Case II y = log a x , 0 < a < 1, 0 < x < 1
Y Y
(iii) If 0 < a < 1 , then log x a is monotonically increasing in
(0, 1) È (1, ¥).
X′ X X′ X
Very Important Concepts O (1, 0) O (1, 0)

(i) If a > 1 , p > 1 , then log a p > 0 Y′ Y′


(ii) If 0 < a < 1, p > 1, then log a p < 0 2. If the number x and the base a are on the opposite sides of
the unity, then the logarithm is negative.
(iii) If a > 1, 0 < p < 1, then log a p < 0 Case I y = log a x, a > 1, 0 < x < 1
(iv) If p > a > 1, then log a p > 1 Case II y = log a x, 0 < a < 1, x > 1
Y Y
(v) If a > p > 1, then 0 < log a p < 1
(vi) If 0 < a < p < 1, then 0 < log a p < 1 (1, 0) (1, 0)
X′ X X′ X
O O
(vii) If 0 < p < a < 1, then log a p > 1

Y′ Y′
320 Textbook of Algebra

3. Graph of y = log a | x | y Example 10. Arrange in ascending order


Y Y log 2 ( x ), log 3 ( x ), log e ( x ), log 10 ( x ), if
a>1 0<a<1 (i) x > 1 (ii) 0 < x < 1.
Sol. Q2 < e < 3 < 10
X′ X X′
(–1, 0) (1, 0)
(–1, 0) O (1, 0) X
O (i) For x > 1, log x 2 < log x e < log x 3 < log x 10
1 1 1 1
Þ < < <
Y′ Y′ log 2 ( x ) loge ( x ) log 3 ( x ) log10 ( x )

Remark Þ log 2 ( x ) > loge ( x ) > log 3 ( x ) > log10 ( x )


Graphs are symmetrical about Y-axis. Hence, ascending order is
4. Graph of y = | log a | x || log10 ( x ) < log 3 ( x ) < loge ( x ) < log 2 ( x )
Y
Y (ii) For 0 < x < 1, log x 2 > log x e > log x 3 > log x 10
1 1 1 1
a>1
0<a<1 Þ > > >
log 2 ( x ) loge ( x ) log 3 ( x ) log10 ( x )
X′ X X′ \ log 2 ( x ) < loge ( x ) < log 3 ( x ) < log10 ( x )
(–1, 0) O (1, 0) X
(–1, 0) O (1, 0)
which is in ascending order.

Y′ Y′ y Example 11. If log 11 = 1.0414, prove that 1011 > 1110 .


Remark Sol. Q log 1011 = 11 log 10 = 11
Graphs are same in both cases i.e., a > 1and 0 < a < 1.
and log 1110 = 10 log 11 = 10 ´ 1.0414 = 10.414
5. Graph of | y | = log a | x ||
It is clear that, 11 > 10.414
Y Þ log 1011 > log 1110 [Q here, base = 10]
a > 0 and a ≠ 1 Þ 11
10 > 11 10

(–1, 0) (1, 0)
X′ X y Example 12. If log 2 ( x - 2) < log 4 ( x - 2), find the
O
interval in which x lies.
Sol. Here, x - 2 > 0
Y′
Þ x >2 …(i)
1
and log 2 ( x - 2) < log 2 2 ( x - 2) = log 2 ( x - 2)
6. Graph of y = log a [ x ], a > 1 and x ³ 1 2
1
(where [ × ] denotes the greatest integer function) Þ log 2 ( x - 2) < log 2 ( x - 2)
2
Since, when 1 £ x < 2,[ x ] = 1 Þ log a [ x ] = 0 Þ
1
log 2 ( x - 2) < 0 Þ log 2 ( x - 2) < 0
when 2 £ x < 3,[ x ] = 2 Þ log a [ x ] = log a 2 2
Þ x - 2 < 20 Þ x - 2 < 1
when 3 £ x < 4,[ x ] = 3 Þ log a [ x ] = log a 3 and so on. Þ x <3 …(ii)
Y From Eqs. (i) and (ii), we get
2 < x < 3 or x Î(2, 3)

y Example 13. Prove that log n (n + 1) > log (n +1) (n + 2)


for any natural number n > 1.
loga 4
loga 3
loga 2 n +1 1 1 æn + 2ö
Sol. Since, =1+ >1+ =ç ÷
X′ X n n n + 1 èn + 1ø
0 1 2 3 4 5
For n > 1,
æn + 1ö æn + 1ö æn + 2ö
Y′ logn ç ÷ > logn + 1 ç ÷ > logn + 1 ç ÷
è n ø è n ø èn + 1ø
Chap 04 Logarithms and Their Properties 321

Þ logn (n + 1) - logn n > log (n + 1)(n + 2) - log (n + 1)(n + 1) y Example 14. Find the least value of the
Þ logn (n + 1) - 1 > log (n + 1)(n + 2) - 1 expression 2 log 10 x - log x 0.01, where x > 0, x ¹ 1.
\ logn (n + 1) > log (n + 1)(n + 2) Hence proved.
. = 2 log10 x - log x (10-2 )
Sol. Let P = 2 log10 x - log x 001
= 2(log10 x + log x 10)
How to Find Minimum Value of ³2 ×2 = 4 [by above article]
l1 log a x + l2 log x a, a > 0, x > 0, \ P³4
a ¹ 1, x ¹ 1 and l1, l2 Î R+ Hence, the least value of P is 4.
AM ³ GM
Q
y Example 15. Which is smaller 2 or (log p 2 + log 2 p )?
l1 loga x + l 2 log x a
Þ ³ ( l1 loga x ) ( l 2 log x a ) = l1l 2 Sol. Let P = log p 2 + log 2 p > 2 [by above article] [Q p ¹ 2]
2
Þ l1 loga x + l 2 log x a ³ 2 l1l 2 \ P >2
Þ (log p 2 + log 2 p ) > 2
Hence, the minimum value of l1 loga x + l 2 log x a is 2 l1l 2 .
Hence, the smaller number is 2.

#L Exercise for Session 3


1 If log0.16(a + 1) < log0.4(a + 1), then a satisfies
(a) a > 0 (b) 0 < a < 1 (c) -1 < a < 0 (d) None of these
1

2 The value of x satisfying the inequation x log10 x × log10 x < 1, is


(a) 0 < x < 10 (b) 0 < x < 1010 (c) 0 < x < 101/ 10 (d) None of these

3 If logcosec x sin x > 0, then


(a) x > 0 (b) x < 0 (c) -1 < x < 1 (d) None of these

4 The value of log10 3 lies in the interval


(a) æç ,

(b) æç 0, ö÷ (c) æç 0, ö÷
2 1 2
÷ (d) None of these
è5 2ø è 2ø è 5ø
2 3
3 æ3ö æ3ö
5 The least value of n in order that the sum of first n terms of the infinite series 1 + + ç ÷ + ç ÷ + ..., should
4 è4ø è4ø
differ from the sum of the series by less than 10-6, is (given, log 2 = 0.30103, log 3 = 0.47712)
(a) 14 (b) 27 (c) 53 (d) 57

Shortcuts and Important Results to Remember


1 For a non-negative number ‘a’ and n ³ 2, n Î N,
5 log 2 log 2 ... 2 = - n
n
a = a1/ n . 14
4244 3
n times
2 The number of positive integers having base a and log a b log b a
characteristic n is an + 1 - an . 6 a =b
7 Logarithms to the base 10 are called common logarithms
3 Logarithm of zero and negative real number is not
(Brigg’s logarithms).
defined.
8 If x = logc b + log b c , y = log a c + logc a,
4 |log b a + log a b| ³ 2, " a > 0, a ¹ 1, b > 0, b ¹ 1.
z = log a b + log b a, then x 2 + y 2 + z 2 - 4 = xyz.
322 Textbook of Algebra

JEE Type Solved Examples :


Single Option Correct Type Questions
n This section contains 8 multiple choice examples. Each The pairs (a, b) are
example has four choices (a), (b), (c) and (d) out of which (3, 32 ), ( 4, 4 2 ), (5, 52 ), (6, 62 ),…, (44, 44 2 ) and (3, 33 ), (4, 4 3 ),
ONLY ONE is correct.
(5, 53 ),…, (12, 123 ).
4
æ æ kp ö ö
l Ex. 1 The expression log 2 5 - å log 2 ç sin ç ÷ ÷ reduces Hence, there are 42 + 10 = 52 pairs.
k =1 è è 5 øø
p
to , where p and q are co-prime, the value of p 2 + q 2 is l Ex. 3 The lengths of the sides of a triangle are log 10 12 ,
q log 10 75 and log 10 n, where n Î N . If a and b are the least
(a) 13 (b) 17 (c) 26 (d) 29 and greatest values of n respectively, the value of b - a is
4
æ æ kp ö ö divisible by
Sol. (b) Let p = log 2 5 - å log 2 çsin ç ÷ ÷
k =1 è è 5 øø (a) 221 (b) 222 (c) 223 (d) 224
Sol. (c) In a triangle,
ì æ æ p öö æ æ 2p ö ö
= log 2 5 - í log 2 çsin ç ÷ ÷ + log 2 çsin ç ÷ ÷ log10 12 + log10 75 > log10 n Þ n < 12 ´ 75 = 900
î è è 5 ø ø è è 5 øø
\ n < 900 …(i)
æ æ 3p ö ö æ æ 4 p ö öü
+ log 2 çsin ç ÷ ÷ + log 2 çsin ç ÷ ÷ý and log10 12 + log10 n > log10 75
è è 5 øø è è 5 ø øþ
75 25
Þ n> =
ì p 2p 3p 4pü 12 4
= log 2 5 - log 2 ísin × sin × sin × sin ý
î 5 5 5 5 þ 25
\ n> …(ii)
ì æpö æ 2p öü 4
= log 2 5 - log 2 ísin 2 ç ÷ × sin 2 ç ÷ý
î è 5 ø è 5 øþ 25
From Eqs. (i) and (ii), we get < n < 900
ì (1 - cos 72° )(1 - cos 144° )ü 4
= log 2 5 - log 2 í ý
î 4 þ \ n = 7, 8, 9, 10, …, 899
- ° + ° Hence, a = 7, b = 899
ì ( 1 sin 18 )( 1 cos 36 ) ü
= log 2 5 - log 2 í ý \ b - a = 892 = 4 ´ 223
î 4 þ
Hence, b - a is divisible by 223.
ìæ 5 - 1öæ 5 + 1öü
ï ç1 - ÷ ç1 + ÷ï
ï è 4 ø è 4 øï æ 1 + log 3 (abc ) ö
= log 2 5 - log 2 í ý l Ex. 4 If 5 log abc (a 3 + b 3 + c 3 ) = 3 lç ÷ and
ï 4 ï è log 3 (abc ) ø
ïî ïþ m
(abc ) a + b + c =1 and l = , where m and n are relative primes,
ì (5 - 5 )(5 + 5 )ü æ5ö n
= log 2 5 - log 2 í ý = log 2 5 - log 2 ç ÷
î 64 þ è 16 ø the value of | m + n| + | m - n| is
(a) 8 (b) 10 (c) 12 (d) 14
æ 16 ö 4 p
= log 2 ç5 ´ ÷ = log 2 24 = = [given] Sol. (b) Q (abc ) a +b +c
= 1 = (abc ) 0
è 5ø 1 q
\ p = 4, q = 1 \ a + b + c = 0 Þ a 3 + b 3 + c 3 = 3abc
Hence, p 2 + q 2 = 4 2 + 12 = 17 Now, LHS = 5 logabc (a 3 + b 3 + c 3 ) = 5 logabc (3abc ) …(i)
æ 1 + log 3 (abc ) ö æ log 3 (3abc ) ö
l Ex. 2 If 3 £ a £ 2015, 3 £ b £ 2015 such that and RHS = 3l ç ÷ = 3l ç ÷
è log 3 (abc ) ø è log 3 (abc ) ø
log a b + 6 log b a = 5, the number of ordered pairs (a, b) of
integers is = 3l logabc (3abc ) …(ii)
(a) 48 (b) 50 (c) 52 (d) 54 From Eqs. (i) and (ii), we get
Sol. (c) Let x = loga b …(i) 5 logabc (3abc ) = 3l logabc (3abc )
6 5 m
Þ x + = 5 Þ x 2 - 5x + 6 = 0 Þ x = 2, 3 \ l= = [given]
x 3 n
From Eq. (i), we get loga b = 2, 3 Þ m = 5, n = 3
Hence, | m + n | + | m - n | = 8 + 2 = 10
Þ b = a 2 or a 3
Chap 04 Logarithms and Their Properties 323

log 4 3 log 4 3 log 4 3 Also, b - a = 36 - a is a square for a = 35, 32, 27, 20, 11
lEx. 5 If a logb c = 3 × 3 log 4 3 × 3 log 4 3 × 3 log 4 3 ... ¥,
b 2 362
where a, b, c ÎQ, the value of abc is Now, c = = is an integer for a = 27
a a
(a) 9 (b) 12 (c) 16 (d) 20
\ a = 27, b = 36, c = 48
logb c 1+ log 4 3 + (log 4 3 ) 2 + (log 4 3 ) 3 + ... ¥
Sol. (c) a =3 Hence, a + b - c = 27 + 36 - 48 = 15
= 31/ (1 - log 4 3 ) = 31/ log 4 (4 / 3 ) = 3log 4 /3 4
æ bcd ö æ acd ö
\
4
a = 3, b = , c = 4
l Ex. 8 If x = log 2a ç ÷ , y = log 3b ç ÷,
3 è 2 ø è 3 ø
4 æ abd ö æ abc ö
Hence, abc = 3 × × 4 = 16 z = log 4c ç ÷ and w = log 5d ç ÷ and
3 è 4 ø è 5 ø
l Ex. 6 Number of real roots of equation 1 1 1 1
+ + + = log abcd N + 1, the value of N is
3 log 3 ( x
2 - 4x +3)
= ( x - 3 ) is x +1 y +1 z +1 w +1
(a) 0 (b) 1 (c) 2 (d) infinite (a) 40 (b) 80
2 - 4x + 3) (c) 120 (d) 160
Sol. (a) Q 3log 3 (x = ( x - 3) …(i)
æ bcd ö
2 Sol. (c) Q x = log 2a ç ÷
Eq. (i) is defined, if x - 4 x + 3 > 0 è 2 ø
Þ ( x - 1)( x - 3) > 0
æ 2abcd ö
Þ x < 1 or x > 3 …(ii) Þ x + 1 = log 2a ç ÷ = log 2a (abcd )
è 2 ø
Eq. (i) reduces to x 2 - 4 x + 3 = x - 3 Þ x 2 - 5x + 6 = 0
1
\ x = 2, 3 …(iii) \ = logabcd 2a
x +1
From Eqs. (ii) and (iii), use get x Î f 1 1
\ Number of real roots = 0 Similarly, = logabcd 3b, = logabcd 4c
y +1 z +1
l Ex. 7 If log 6 a + log 6 b + log 6 c = 6, where a, b, c Î N and 1
and = logabcd 5d
a, b, c are in GP and b - a is a square of an integer, then the w +1
value of a + b - c is 1 1 1 1
(a) 21 (b) 15 (c) 9 (d) 3 \ + + + = logabcd (2a × 3b × 4c × 5d )
x +1 y +1 z +1 w +1
Sol. (b) Q log 6 a + log 6 b + log 6 c = 6
= logabcd (120abcd )
Þ log 6 (abc ) = 6
= logabcd 120 + 1
Þ abc = 66
Þ b 3 = 66 [Qb 2 = ac ] = logabcd N + 1 [given]
Hence, N = 120
Þ b = 36

JEE Type Solved Examples :


More than One Correct Option Type Questions
n
This section contains 4 multiple choice examples. Each Þ 3ab(a + b ) = 0 Þ a = 0 or b = 0 or a + b = 0
example has four choices (a), (b), (c) and (d) out of which Þ log10 x + 2 = 0 or log10 x - 1 = 0
more than one may be correct.
or 2 log10 x + 1 = 0
l Ex. 9 The equation Þ x = 10-2 or x = 10 or x = 10-1/ 2
1 1
(log 10 x + 2 ) 3 + (log 10 x - 1) 3 = ( 2 log 10 x + 1) 3 has Hence, x= or x = 10 or x =
100 10
(a) no natural solution (b) two rational solutions
(c) no prime solution (d) one irrational solution log 5 9 × log 7 5 × log 3 7 1
Sol. (b, c, d) Let log10 x + 2 = a and log10 x - 1 = b
l Ex. 10 The value of + is
log 3 6 log 4 6
\ a + b = 2 log10 x + 1, then given equation reduces to
co-prime with
a 3 + b 3 = (a + b )3
(a)1 (b) 3 (c) 4 (d) 5
324 Textbook of Algebra

log 5 9 × log 7 5 × log 3 7 1 (a) x 1 + x 2 = 101 (b) y 1 + y 2 = 25


Sol. (a, b, d) Let P = +
log 3 6 log 4 6 (c) x 1x 2 = 100 (d) z 1z 2 = 100
log 3 9 Sol. (a, b, c, d) Let log10 x = a, log10 y = b and log10 z = c
= + log 6 4 = log 69 + log 6 4
log 3 6 Then, given equations reduces to
= log 6 (36) = log 6 ( 6 )4 = 4 Þ P = 4 a + b - ab = 4 - log10 2000 = log10 5 …(i)
b + c - bc = 1 - log10 2 = log10 5 …(ii)
which is co-prime with 1, 3, 4 and 5.
and c + a - ca = 0 …(iii)
l Ex. 11 Which of the following quantities are irrational From Eqs. (i) and (ii), we get
for the quadratic equation a + b - ab = b + c - bc
(log 10 8 ) x 2 - (log 10 5 ) x = 2(log 2 10 ) -1 - x ? Þ (c - a ) - b(c - a ) = 0
(a) Sum of roots (b) Product of roots Þ (c - a )(1 - b ) = 0
(c) Sum of coefficients (d) Discriminant 1 - b ¹ 0, c - a = 0 Þ c = a
Sol. (c, d) Q (log10 8)x 2 - (log10 5)x = 2(log 2 10)-1 - x From Eq. (iii), we get
2a - a 2 = 0 Þ a = 0, 2
Þ (3 log10 2)x 2 + (1 - log10 5)x - 2 log10 2 = 0
Then, c = a Þ c = 0, 2
Þ (3 log10 2)x 2 + (log10 2)x - 2 log10 2 = 0
and b = log10 5, 2 - log10 5
1
Now, Sum of roots = - = Rational \ log10 x = 0, 2 Þ x = 100 , 102
3
2 Þ x = 1, 100
Product of roots = - = Rational
3 Þ x 1 = 1, x 2 = 100
Sum of coefficients = 3 log10 2 + log10 2 - 2 log10 2 and log10 y = log10 5, 2 - log10 5
= 2 log10 2 = Irrational = log10 5, log10 20
Discriminant = (log10 2)2 + 24(log10 2)2 Þ y = 5, 20
= 25 (log10 2)2 = Irrational Þ y1 = 5, y 2 = 20
and log10 z = 0, 2 Þ z = 100 , 102
l Ex. 12 The system of equations
Þ z = 1, 100
log 10 ( 2000 xy ) - log 10 x × log 10 y = 4
Þ z1 = 1, z 2 = 100
log 10 ( 2yz ) - log 10 y × log 10 z = 1
and log 10 ( zx ) - log 10 z × log 10 x = 0 Finally, x 1 + x 2 = 1 + 100 = 101, y1 + y 2 = 5 + 20 = 25,
has two solutions (x 1 , y 1 , z 1 ) and (x 2 , y 2 , z 2 ), then x 1x 2 = 1 ´ 100 = 100 and z1z 2 = 1 ´ 100 = 100

JEE Type Solved Examples :


Passage Based Questions
2 -6 2
This section contains 2 solved passages based upon each 15. If x 2 t + y 6 - 2t = 6, the value of t 1 t 2 t 3 t 4 is
n

of the passage 3 multiple choice examples have to be


answered. Each of these examples has four choices (a), (b), (a) 1 (b) 2 (c) 4 (d) 8
(c) and (d) out of which ONLY ONE is correct. Sol. (Ex. Nos. 13-15)
Passage I Q log10 ( x - 2) + log10 y = 0
(Ex. Nos. 13 to 15) \ x - 2 > 0, y > 0
Suppose that log 10 ( x - 2 ) + log 10 y = 0 and Þ x > 2, y > 0 …(i)
x + (y - 2 ) = ( x + y ) . and log10 {( x - 2)y } = 0
13. The value of x is Þ ( x - 2)y = 100 = 1
(a) 2 + 2 (b) 1 + 2 (c) 2 2 (d) 4 - 2 \ ( x - 2)y = 1 …(ii)
14. The value of y is Also, given that x + (y - 2) = ( x + y )
(a) 2 (b) 2 2 (c) 1 + 2 (d) 2 + 2 2 \ x ³ 0, y - 2 ³ 0, x + y ³ 0
Chap 04 Logarithms and Their Properties 325

Þ x ³ 0, y ³ 2 …(iii) Passage II
On squaring both sides, we get (Ex. Nos. 16 to 18)
x + y - 2 + 2 x ( y - 2) = x + y log p {logq (logr x )}
If 10 = 1 and log q {log r (log p x )} = 0.
Þ x y -2 =1
Þ x ( y - 2) = 1 …(iv)
16. The value of x is
(a) q r (b) r q (c) r p (d) rq
From Eqs. (i) and (iii),we get
log p [logq (logr x )]
x > 2, y ³ 2 Sol. (b) Q 10 = 1 = 100
and from Eqs. (ii) and (iv), we get y = x Þ log p {logq (logr x )} = 0
From Eq. (ii), ( x - 2)x = 1 Þ logq (logr x ) = 1 Þ logr x = q
Þ x 2 - 2x - 1 = 0 Þ x = rq …(i)

2± 4 + 4 and logq {logr (log p x )} = 0


\ x= [neglect -ve sign, since x > 2] Þ logr (log p x ) = 1 Þ log p x = r
2
13. (b) x = ( 2 + 1). \ x = pr …(ii)
q r
14. (c) y = x = 1 + 1 From Eqs. (i) and (ii), we get x =r = p

15. (d) Q x 2t
2-6
+ y 6 - 2t = 6
2 17. The value of p is
2-6 2-6 (a) r q / r (b) rq (c) 1 (d) r r /q
Þ x 2t + ( x -1 )2t =6
Sol. (a) Q r q = pr …(iii)
2 t2 - 3 -2 t 2 - 3
Þ (x ) + (x ) =6 q /r
Þ p =r
2-3 2-3
Þ (3 + 2 2 )t + (3 - 2 2 )t =6
18. The value of q is
Now, we get t2 - 3 = ± 1
(a) r p / r (b) p log p r (c) r logr p (d) r r /p
2
Þ t = 4, 2
Sol. (c) From Eq. (iii),
\ t = ± 2, ± 2 æ log p ö
q log r = r log p Þ q = r ç ÷ = r logr p
\ t 1 t 2 t 3 t 4 = ( 2) ( - 2) ( 2 ) ( - 2 ) = 8 è log r ø

JEE Type Solved Examples :


Single Integer Answer Type Questions
n
This section contains 2 examples. The answer to each lEx. 20 If (31.6) a = (0.0000316) b = 100, the value of
example is a single digit integer ranging from 0 to 9 1 1
(both inclusive). - is
a b
l Ex. 19 If x 1 and x 2 are the solutions of the equation
log10 x x x Sol. (3) Q (31.6)a = (0.0000316)b = 100
x = 100 x such that x 1 >1 and x 2 < 1, the value of 1 2 is
2 Þ a lo g10 (31.6) = b l og10 (0.0000316) = log10 100
log x
Sol. (5) Q x 10 = 100x
Þ a log10 (31.6) = b log10 (31.6 ´ 10-6 ) = 2
Taking logarithm on both sides on base 10, then we get
Þ a log10 (31.6) = b log10 (31.6) - 6b = 2
log10 x × log10 x = log10 100 + log10 x 2
Þ = log10 (31.6)
Þ (log10 x )2 - log10 x - 2 = 0 a
Þ (log10 x - 2) (log10 x + 1) = 0 2
and = log10 (31.6) - 6
\ log10 x = 2, - 1 Þ x = 102 , 10-1 b
2 2
1 \ - =6
\ x 1 = 100, x 2 = a b
10
x 1x 2 1 1
\ =5 Þ - =3
2 a b
326 Textbook of Algebra

JEE Type Solved Examples :


Matching Type Questions
n This section contains 2 examples. Examples 24 and 25 1
\ log 5 x = - , 2
have three statements (A, B and C) given in Column I and 2
four statements (p, q, r and s) in Column II. Any given 1
Þ x = 5-1 / 2 , 52 or x =
, 25
statement in Column I can have correct matching with 5
one or more statement(s) given in Column II. 1
\ Product of the values of x = ´ 25 = 5 5
l Ex. 21 5
(C) Q logb a = - 3 and logb c = 4
Column I Column II 3
\ logc a = - …(i)
(A) If x1 and x 2 satisfy the equation 4
( x + 1 ) log10 ( x + 1) = 100 ( x + 1 ), then (p) irrational and a3x = c x - 1
the value of( x1 + 1 ) ( x 2 + 1 ) + 5 is Þ 3x log a = ( x - 1)log c
(B) The product of all values of x Þ 3x logc a = x - 1
which make the following (q) rational 3
statement true Þ 3x ´ - = x - 1 [from Eq. (i)]
(r) prime 4
(log 3 x ) (log 5 9 ) - log x 25 + log 3 2 4
= log 3 54, is Þ - 9 x = 4 x - 4 or x =
13
(C) If logb a = - 3, logb c = 4 and if the (s) composite \ q = 13 [prime and rational]
value of x satisfying the equation
a 3 x = c x - 1 is expressed in the form l Ex. 22
p / q, where p and q are relatively
prime, then q is Column I Column II
(t) twin prime (A) If a and b are the roots of (p) divisible by 2
ax 2 + bx + c = 0, where
Sol. A ® (q, s, t), B ®(p), C ® (q, r) a = 2 log 2 3 - 3 log 3 2,
+ 1)
(A) ( x + 1)log10 (x = 100 ( x + 1) b =1 + 2 log 2 3
-3 log 3 2

Taking logarithm on both sides on base 10, then we get (q) divisible by 4
and c = log 2 log 2 2 ,
log10 ( x + 1) × log10 ( x + 1) = log10 100 + log10 ( x + 1)
then HM of a and b is
Þ {log10 ( x + 1)} 2 = 2 + log10 ( x + 1)
(B) The sum of the solutions of the (r) divisible by 6
Þ {log10 ( x + 1)} 2 - log10 ( x + 1) - 2 = 0 equation
Þ {log10 ( x + 1) - 2} {log10 ( x + 1) + 1} = 0 | x - 1 | log 2 x
2 - 2 log 4
x = ( x - 1 ) 7 is (s) divisible by 8
\ log10 ( x + 1) = 2, - 1
(C) If 5 (log y x + log x y ) = 26, xy = 64, (t) divisible by 10
Þ ( x + 1) = 102 , 10-1 then the value of | x - y | is
\ ( x 1 + 1) ( x 2 + 1) = 102 ´ 10-1 = 10
Sol. A ® (p, q, r), B ® (p, r), C ® (p, r, t)
Þ ( x 1 + 1) ( x 2 + 1) + 5 = 10 + 5 -6
(A) Q a = 3 - 2 = 1, b = 1, c = log 2 log 2 22
= 15 = 3 ´ 5
(B) Q(log 3 x ) (log 5 9 ) - log x 25 + log 3 2 = log 3 54 = log 2 (2-6 ) = - 6
Þ 2 log 5 x - 2 log x 5 = log 3 54 - log 3 2 The equation reduces to x 2 + x - 6 = 0
= log 3 (27 ) = 3 \ a + b = - 1, ab = - 6
2ab 2( - 6)
Let log 5 x = l, then \ HM = = = 12
2 a +b ( - 1)
2l - = 3
l (B) Obviously, x = 2 is a solution. Since, LHS is positive,
Þ 2l2 - 3l - 2 = 0 x - 1 > 0. The equation reduces to
Þ 2l2 - 4 l + l - 2 = 0 log 2 x 2 - 2 log x 4 = 7
1 4
2l ( l - 2) + 1( l - 2) = 0 Þ l = - , 2 Þ 2l - = 7, where l = log 2 x
2 l
Chap 04 Logarithms and Their Properties 327

1 a 1 26
Þ 2l2 - 7 l - 4 = 0 Þ l = 4, - Let = l, then l + =
2 b l 5
1 Þ 5l2 - 26l + 5 = 0
\ Þ x = 2 4 , 2-1 / 2
log 2 x = 4, -
2
Þ 5l2 - 25l - l + 5 = 0
1
Þ x = 16, Þ ( l - 5) ( 5l - 1) = 0
2
1
1 Þ l = 5,
Þ x = 16, x ¹ [Q x > 1] 5
2
a 1 a
\ Solutions are x = 2, 16 \ = 5, Þ =5
b 5 b
\ Sum of solutions = 2 + 16 = 18
Þ a = 5b …(i)
(C) If a = log x , b = log y
and a + b = log x + log y = log( xy ) = log(64 )
a b
\ log y x + log x y = + \ a + b = 6 log 2 …(ii)
b a
From Eqs. (i) and (ii), we get
\ 5(log y x + log x y ) = 26 b = log2 and a = 5 log 2
a b 26 Þ y = 2, x = 32 or y = 32, x = 2
Þ + =
b a 5 \ | x - y | = 30

JEE Type Solved Examples :


Statement I and II Type Questions
n
Directions Example numbers 23 to 24 are l Ex. 24 Statement-1 If p, q Î N satisfy the equation
Assertion-Reason type examples. Each of these examples x x = ( x ) x and q > p, then q is a perfect number.
contains two statements: Statement-2 If a number is equal to the sum of its factor,
Statement-1 (Assertion) and Statement-2 (Reason)
then number is known as perfect number.
Each of these examples also has four alternative choices,
x
only one of which is the correct answer. You have to select Sol. (d) Q x = ( x )x
the correct choice as given below. Taking logarithm on both sides on base e, then
(a) Statement-1 is true, Statement-2 is true; Statement-2
x
is a correct explanation for Statement-1 ln ( x ) = ln ( x )x
(b) Statement-1 is true, Statement-2 is true; Statement-2 x
is not a correct explanation for Statement-1 Þ x ln x = x ln x Þ x ln x = ln x
2
(c) Statement-1 is true, Statement-2 is false
æ xö
(d) Statement-1 is false, Statement-2 is true Þ ln x ç x - ÷ = 0
20 è 2ø
æ 1 ö
l Ex. 23 Statement-1 If N = ç ÷ , then N contains æ xö
7 digits before decimal. è 0.4 ø Þ ln x × x × ç1 - ÷ =0
è 2 ø
Statement-2 Characteristic of the logarithm of N to the
x
base 10 is 7. Þ ln x = 0, x = 0, 1 - =0
æ 1 ö
20
æ 10 ö
20 2
Sol. (d) Q N =ç ÷ = ç 2÷ \ x = 1, 0, 4
è 0.4 ø è2 ø
Q x ÎN
Þ log10 N = 20(1 - 2 log10 2) = 20 (1 - 2 ´ 0.3010)
\ x = 1, 4 Þ p = 1 and q = 4
= 20 ´ 0.3980 = 7.9660
Q 4 =1´2´2 Þ 4 ¹1+2+2
Since, characteristic of log10 N is 7, therefore the number of
digits in N will be 7 + 1, i.e. 8. \ q is not a perfect number.
Hence, Statement-1 is false and Statement-2 is true. Hence, Statement-1 is false and Statement-2 is true.
328 Textbook of Algebra

Subjective Type Questions


n In this section, there are 21 subjective solved examples.
and 8>7
l Ex. 25 Prove that log 3 5 is an irrational. Þ log 8 > log 7 …(ii)
Sol. Let log 3 5 is rational. From Eqs. (i) and (ii), we get
p log 11 × log 8 > log 7 × log 5
\ log 3 5 = , where p and q are co-prime numbers.
q log 11 log 5
Þ > Þ log 7 11 > log 8 5
log 7 log 8
Þ 5 = 3p /q Þ 3p = 5q
which is not possible, hence our assumption is wrong.
Hence, log 3 5 is an irrational.
l Ex. 30 Given, a 2 + b 2 = c 2 . Prove that
log b + c a + log c - b a = 2 log c + b a × log c - b a, " a > 0, a ¹ 1
l Ex. 26 Find the value of the expression c - b > 0, c + b > 0
(log 2 ) 3 + log 8 × log 5 + (log 5 ) 3 .
c - b ¹ 1, c + b ¹ 1.
Sol. Q log 2 + log 5 = log(2 × 5) = log10 = 1 …(i) Sol. LHS = logb + c a + logc - b a
3
Þ (log 2 + log 5) = 1
1 1
= +
Þ (log 2)3 + (log 5)3 + 3 log 2 log 5(log 2 + log 5) = 13 loga (c + b ) loga (c - b )
loga (c + b ) + loga (c - b )
=
loga (c + b )loga (c - b )
Þ (log 2)3 + (log 5)3 + log 23 log 5(1) = 1 [from Eq. (i)] loga (c 2 - b 2 )
=
Þ (log 2)3 + log 8 log 5 + (log 5)3 = 1 loga (c + b ) × loga (c - b )
loga a 2
log 3 5 (log 3 5 ) 2 = [Qc 2 - b 2 = a 2 ]
l Ex. 27 If l = 81, find the value of l . loga (c + b ) × loga (c - b )
Sol. Q llog 3 5 = 81 =
2loga a
loga (c + b ) × loga (c - b )
\ ( llog 3 5 )log 3 5 = (81)log 3 5
2
Þ
2 4
l(log 3 5 ) = 34 log 3 5 = 3log 3 5 = 54 = 625 =
loga (c + b ) × loga (c - b )
l Ex. 28 Find the product of the positive roots of the = 2logc + b a × log c - b a = RHS
equation ( 2009 )( x ) log 2009 x = x 2 . l Ex. 31 Let a > 0, c > 0, b = ac , a , c and ac ¹1, N > 0.
Sol. Given, (2009 )( x ) log 2009 x
=x 2
log a N log a N - log b N
Prove that = .
Taking logarithm both sides on base 2009, then log c N log b N - log c N
log 2009 (2009 ) + log 2009 x × log 2009 x = log 2009 x 2 1 1
-
loga N - logb N log N a log N b
Sol. RHS = =
logb N - logc N 1
-
1
log N b log N c
1
Þ + (log 2009 x )2 = 2 log 2009 x [ for x >0] (log N b - log N a ) log N c
2 = ×
(log N c - log N b ) log N a
1
Þ (log 2009 x )2 - 2 log 2009 x + = 0
2 æb ö
log N ç ÷
è a ø loga N loga N
If roots are x 1 and x 2 , then log 2009 x 1 + log 2009 x 2 = 2 = × = = LHS
æ c ö logc N logc N
Þ log 2009 ( x 1x 2 ) = 2 or x 1x 2 = (2009 )2 log N ç ÷
èb ø
l Ex. 29 Prove that log 7 11 is greater than log 8 5. é 2 b cù
êëQb = ac Þ b = ac Þ a = b úû
Sol. Q 11 > 5
Þ log 11 > log 5 …(i)
Chap 04 Logarithms and Their Properties 329

l Ex. 32 If a x = b, b y = c , c z = a , x = log b a k1 , y = log c b k 2 , \ (b + c ) ln a + (c + a ) ln b + (a + b ) ln c = 0

z = log a c k 3 , find the minimum value of 3k 1 + 6k 2 + 12k 3 . Þ ln ab + c + ln bc + a + ln c a + b = 0

Sol. Q a = c z = (b y )z [Qc = b y ] Þ ln {ab + c × bc + a × c a + b } = 0

= b yz = (a x )yz = a xyz [Qb = a x ] Þ ab + c × bc + a × c a + b = e 0 = 1 …(i)


\ xyz = 1 Again, AM ³ GM
k1 k2
Also, xyz = logb a × logc b × loga c k3 ab + c + bc + a + c a + b
Þ ³ (ab + c × bc + a × c a + b )1 / 3
= k1 × k 2 × k 3 × logb a × logc b × loga c 3
1 = k1k 2k 3 = (1)1 / 3 = 1 [from Eq. (i)]
b +c c +a a +b
Q AM ³ GM or a +b +c ³3
3k1 + 6k 2 + 12k 3
\ ³ (3k1 × 6k 2 × 12k 3 )1 / 3
3 log 1 / 5 (1 / 2) æ 4 ö
= (3 × 6 × 12 × k1k 2k 3 )1 / 3
l Ex. 35 Simplify 5 + log 2 ç ÷
è 7 + 3ø
= (3 × 6 × 12)1 / 3 [Qk1k 2k 3 = 1]
æ 1 ö
3
= (2 × 3 )3 1/ 3
=6 + log 1 / 2 ç ÷.
è 10 + 2 21 ø
or 3k1 + 6k 2 + 12k 3 ³ 18 æ 1ö
log1 / 5 ç ÷
è 2ø
\ Minimum value of 3k1 + 6k 2 + 12k 3 is 18. Sol. Q 5 = 5log 5 (2 ) = 2
æ 4 ö æ 4( 7 - 3) ö
l Ex. 33 If x = 1 + log a bc , y = 1 + log b ca , z = 1 + log c ab, log 2ç ÷ = log 2 ç ÷
è 7 + 3ø è( 7 + 3) ( 7 - 3)ø
prove that xyz = xy + yz + zx .
log bc log b + log c = log 2 ( 7 - 3 )
Sol. Q x = 1 + loga bc = 1 + =1+
log a log a = log 21 / 2 ( 7 - 3 )1
log a + log b + log c 1
= = log 2 ( 7 - 3 )
log a 1/2
1 log a = log 2 ( 7 - 3 )2 = log 2 (10 - 2 21 )
or = …(i)
x log a + log b + log c
æ 1 ö
1 log b and log1 / 2 ç ÷ = log 2 (10 + 2 21 )
Similarly, = …(ii) è 10 + 2 21 ø
y log a + log b + log c
Hence,
1 log c
and = …(iii) log1 / 5(1 / 2 ) æ 4 ö æ 1 ö
z log a + log b + log c 5 + log 2ç ÷ + log1/ 2 ç ÷
è 7 + 3)ø è 10 + 2 21 ø
On adding Eqs. (i), (ii) and (iii), we get
= 2 + log 2 (10 - 2 21 ) + log 2 (10 + 2 21 )
1 1 1
+ + =1
x y z = 2 + log 2 {(10 - 2 21 ) (10 + 2 21 )}
or xyz = xy + yz + zx = 2 + log 2 (100 - 84 ) = 2 + log 2 (2)4 = 2 + 4 = 6

ln a ln b ln c Ex. 36 Find the square of the sum of the roots of the


l Ex. 34 If = = , prove that l

(b - c ) (c - a ) (a - b ) equation log 2 x × log 3 x × log 5 x = log 2 x × log 3 x


a b + c × b c + a × c a + b =1 + log 3 x × log 5 x + log 5 x × log 2 x.
Sol. Let log 2 x = A , log 3 x = B and log 5 x = C , then the given
Also, prove that a b + c + b c + a + c a + b ³ 3. equation can be written as
Sol. Since, a > 0, b > 0, c > 0 æ1 1 1ö
ABC = AB + BC + CA = ABC ç + + ÷
ln a ln b ln c èC A B ø
= =
(b - c ) (c - a ) (a - b ) æ1 1 1 ö
Þ ABC ç + + - 1÷ = 0
(b + c ) ln a + (c + a ) ln b + (a + b ) ln c èA B C ø
=
0 1 1 1
or A = 0, B = 0, C = 0, + + -1=0
[using ratio and proportion] A B C
330 Textbook of Algebra

1
log 2 x = 0, log 3 x = 0, log 5 x = 0, log x 2 + log x 3 + log x 5 = 0 2a +
1444442444443 14444244443 2 log 2 3 + log 2 7 c
x >0 x > 0, x ¹ 1 = =
2 + log 2 5 + log 2 7 2 + ab + 1
or x = 20 , x = 30 , x = 50 , log x (2 × 3 × 5) = 0 c
or x = 1, x = 1, x = 1, x = 30 [from Eqs. (i), (ii) and (iii)]
æ 2ac + 1 ö
\ Roots are 1 and 30. =ç ÷
è 2c + abc + 1 ø
Hence, the required value
= (1 + 30)2 = (31)2 = 961 l Ex. 39 Show that the sum of the roots of the equation
x + 1 = 2 log 2 ( 2 x + 3 ) - 2 log 4 (1980 - 2 - x ) is log 2 11.
l Ex. 37 Given that log 2 a = l, log 4 b = l2 and
Sol. Given,
2 æ a 2b 5 ö
log c 2 (8 ) = , write log 2 ç ÷ as a function of ‘l’ x + 1 = 2 log 2 (2x + 3) - 2 log 4 (1980 - 2- x )
l3 + 1 è c4 ø
= 2 log 2 (2x + 3) - 2 log 2 2 (1980 - 2- x )1
(a , b, c > 0, c ¹ 1).
1
Sol. Q log 2 a = l Þ a = 2l = 2 log 2 (2x + 3) - 2 × log 2 (1980 - 2- x )
2
Þ log 4 b = l2 = log 2 (2x + 3)2 - log 2 (1980 - 2- x )
2 2
Þ b = 4l = 2 2l ì ( 2x + 3) 2 ü
= log 2 í -x ý
and logc 2 (8) =
2 î 1980 - 2 þ
3
l +1
( 2x + 3) 2
3 2 or 2x + 1 =
Þ logc 2 = 3 1980 - 2- x
2 l +1
Þ 1980(2x + 1 ) - 2 = 22 x + 9 + 6 × 2x
4
Þ logc 2 = Þ 22 x - 3954 × 2x + 11 = 0 …(i)
3( l3 + 1)
ìï 3 ( l3 + 1) üï If x 1, x 2 are the roots of Eq. (i), then
í ý
3( l + 1)3
ïî 4 ïþ 2x 1 × 2x 2 = 11 or 2x 1 + x 2 = 11
or log 2 c = or c = 2
4 Þ x 1 + x 2 = log 2 11
æ a 2b 5 ö
\ log 2 ç 4 ÷ = log 2 (a 2b 5c -4 )
è c ø
l Ex. 40 Solve the following equations for x and y
1
2
= log 2 {2 2 l × 2 10 l × 2-3 (l
3 + 1)
} log 100 | x + y | = , log 10 y - log 10 | x | = log 100 4 .
2
2 l + 10 l2 - 3 ( l3 + 1) 1
= log 2 {2 } Sol. Q log100 | x + y | =
2 3 2
= 2l + 10l - 3( l + 1)
Þ | x + y | = (100)1/ 2 = 10
l Ex. 38 Given that log 2 3 = a , log 3 5 = b, log 7 2 = c , Þ | x + y | = 10 …(i)
express the logarithm of the number 63 to the base 140 in and log10 y - log10 | x | = log100 4, y > 0
terms of a , b and c. æy ö 2
Sol. Q log 2 3 = a …(i) Þ log10 ç ÷ = log10 2 22 = log10 2
è|x |ø 2
log 2 5 log 2 5
Þ b = log 3 5 = = [from Eq. (i)] æy ö y
log 2 3 a Þ log10 ç ÷ = log10 2 Þ =2
è|x |ø |x |
\ log 2 5 = ab ...(ii)
Þ y = 2| x | …(ii)
and log 7 2 = c
From Eqs. (i) and (ii),we get
1 1
Þ = c or log 2 7 = …(iii) | x + 2| x || = 10 …(iii)
log 2 7 c
Case I If x > 0, then | x | = x
log 2 63 log 2 (32 ´ 7 ) From Eq. (iii),
Now, log140 63 = =
log 2 140 log 2 (22 ´ 5 ´ 7 ) | x + 2x | = 10
Chap 04 Logarithms and Their Properties 331

10 ì 4 ü
Þ 3| x | = 10 Þ | x | = Þ log( x - 1)í2 log 3 x - - 7ý = 0
3 î log 3 x þ
10 20 Þ log( x - 1){2(log 3 x )2 - 7 log 3 x - 4 } = 0
\ x=
,y = [from Eq. (ii)]
3 3
Þ log( x - 1) (log 3 x - 4 ) (2 log 3 x + 1) = 0
Case II If x < 0, then | x | = - x 1
Þ log( x - 1) = 0, log 3 x = 4, log 3 x = -
From Eq. (iii), 2
| x - 2x | = 10 Þ x - 1 = (10)0 , x = 34 , x = 3-1 / 2
Þ | - x | = 10 Þ | x | = 10 1
Þ x - 1 = 1, x = 81, x =
\ - x = 10 3
Þ x = - 10 é 1 ù
\ x = 2, 81 êëQ x ³ 2, \ x ¹ 3 úû
From Eq. (ii), y = 20
ì 10 20ü
Hence, solutions are í , ý, { -10, 20}. l Ex. 43 Find all real numbers x which satisfy the
î3 3þ
equation 2 log 2 log 2 x + log 1/ 2 log 2 ( 2 2 x ) = 1.
l Ex. 41 Solve the following equation for x Sol. Given,
6 loga x ×log10 a×loga 5 2 log 2 log 2 x + log1/ 2 log 2 (2 2x ) = 1
a - 3 log10 ( x / 10 ) = 9 log100 x + log 4 2 .
5 Þ 2 log 2 log 2 x - log 2 log 2 (2 2x ) = 1
6 log x × log 10 a× loga 5 log x + log 4 2 Þ 2 log 2 log 2 x - log 2 {log 2 (2 2 ) + log 2 x } = 1
Sol. Q × a a - 3log 10 (x / 10 ) = 9 100
5 ì3 ü
æ1 1ö Þ 2 log 2 log 2 x - log 2 í + log 2 x ý = 1
6 log10 5 2 ç log10 x
è
+ ÷
2ø î2 þ
Þ ×x - 3(log10 x - 1) = 3 2 [by property]
5 æ3 ö
Let log 2 x = l, then 2 log 2 l - log 2 ç + l ÷ = 1
6 log10 x 3log10 x è2 ø
Þ ×5 - = 3log10 x + 1 [by property]
5 3 æ3 ö
Þ log 2 l2 - log 2 ç + l ÷ = 1
è2 ø
Let log10 x = l, then
ì ü
6 l 3l ï l2 ï l2
Þ ×5 - = 3 × 3l Þ log 2 í =1 Þ = 21
5 3 3 ý 3
ï + lï +l
6 l æ1 ö 10 î2 þ 2
Þ × 5 = 3l ç + 3÷ = × 3l
5 è3 ø 3 Þ l2 = 3 + 2l Þ l2 - 2l - 3 = 0
Þ 5 l-2
=3 l-2
which is possible only, where l = 2. Þ ( l - 3) ( l + 1) = 0
\ l = 3, - 1
Þ log10 x = 2 or log 2 x = 3, - 1
\ x = 102 = 100 Þ x = 23 , 2-1
1
Þ x = 8, …(i)
l Ex. 42 Find the value of x satisfying the equation 2
2 - 2 log 9
| x - 1 |log 3 x x = ( x - 1) 7 . But the given equation is valid only when,
Sol. The given equation is, x > 0, 2 2x > 0, log 2 x > 0, log 2 (2 2x ) > 0
2
|x - 1| log 3 x - 2 log x 9 = ( x - 1) 7 …(i) Þ x > 0, x > 0, x > 1, x >
1
2 2
This equation is defined for
Hence,x > 1
x 2 > 0, x > 0, x ¹ 1 and x - 1 ³ 1
From Eq. (i), the solution of the given equation is x = 8.
Þ x ³ 2, then Eq. (i) reduces to
2 - 2 log 9
( x - 1)log 3 x x = ( x - 1) 7 l Ex. 44 Solve for x ,
Taking log on both sides, then log 3 / 4 log 8 ( x 2 + 7 ) + log 1 / 2 log 1 / 4 ( x 2 + 7 ) -1 = - 2.
(log 3 x 2 - 2 log x 9 )log( x - 1) = 7 log( x - 1) Sol. Given,
Þ 2
log( x - 1){log 3 x - 2 log x 9 - 7 } = 0 log 3 / 4 log 8 ( x 2 + 7 ) + log1/ 2 log1/ 4 ( x 2 + 7 )-1 = - 2
332 Textbook of Algebra

Þ log 3 / 4 log 2 3 ( x 2 + 7 ) + log 2 -1 log 2 -2 ( x 2 + 7 )-1 = - 2 æ 1 ö æ 1 ö


2

ç loga b + + 2÷ ç | loga b | + ÷
ì1 ü ì1 ü log b | log b |
Þ log 3 / 4 í log 2 ( x 2 + 7 )ý- log 2 í log 2 ( x 2 + 7 )ý = - 2 = ç a ÷ = ç a ÷
î3 þ î2 þ ç 4 ÷ ç 2 ÷
ç ÷ ç ÷
è ø è ø
Let log 2 ( x 2 + 7 ) = 6l ...(i)
Then, log 3 / 4 (2l ) - log 2 (3l ) = - 2 1æ 1 ö
= ç | loga b | + ÷
log 2 (2l ) 2è | loga b | ø
Þ - log 2 (3l ) = - 2
log 2 (3 / 4 )
and loga 4 (b / a ) + logb 4 (a / b )
1 + log 2 l
Þ - (log 2 3 + log 2 l ) = - 2
log 2 3 - log 2 4 1 æb ö 1 æa ö
= loga ç ÷ + logb ç ÷
1 + log 2 l 4 èa ø 4 èb ø
Þ - (log 2 3 + log 2 l ) = - 2
log 2 3 - 2 1
= (loga b - 1 + logb a - 1)
Again, let log 2 l = A and log 2 3 = B, then 4
1+ A 1
- (B + A) = - 2 loga b + -2
B-2 loga b
=
Þ 1 + A - B 2 - AB + 2B + 2A = - 2B + 4 4

Þ A ( 3 - B ) = B 2 - 4 B + 3 = ( B - 1) ( B - 3) 1
| loga b | -
| loga b |
Þ A = - ( B - 1) =
2
[Q B - 3 ¹ 0, i.e. log 2 3 ¹ 3]
Þ A + B = 1 Þ log 2 l + log 2 3 = 1 \ loga 4 ab + logb 4 ab - loga 4 b / a + logb 4 (a / b )
Þ log 2 (3l ) = 1 P (say)
Þ 3l = 2 1 ìï 1 1 üï
1 = í | loga b | + + | loga b | - ý
Þ 3 × log 2 ( x 2 + 7 ) = 2 [from Eq. (i)] 2 ïî | loga b | | loga b | ïþ
6
Case I If b ³ a > 1 , then
Þ log 2 ( x 2 + 7 ) = 4
1ì 1 1 ü
Þ x 2 + 7 = 24 = 16 or x 2 = 9 P = í | loga b | + - | loga b | + ý
2î | loga b | | loga b | þ
\ x = ±3
1
=
loga b
l Ex. 45 Prove that
loga b
( loga 4 ab + logb 4 ab - loga 4 b /a + logb 4 a /b ) loga b \ 2P = 21 = 2
2
ì 2, b ³ a > 1. Case II If 1 < b < a, then
= í log b .
a ,1 < b < a
î2 1ì 1 1 ü
P = í | loga b | + + | loga b | - ý
1 1 2î | loga b | | loga b | þ
Sol. Since, loga 4 ab + logb 4 ab = loga (ab ) + logb (ab )
4 4
= | loga b |
1
= (1 + loga b + logb a + 1) loga b
4 \ 2P = 2loga b
#L Logarithms and Their Properties Exercise 1 :
Single Option Correct Type Questions
n This section contains 20 multiple choice questions. 11. If log 0. 3 ( x − 1) < log 0.09 ( x − 1), then x lies in the interval
Each question has four choices (a), (b), (c) and (d) out of
(a) ( − ∞, 1 )
which ONLY ONE is correct
(b) (1, 2 )
1. If log10 2 = 0.3010..., the number of digits in the number (c) (2, ∞ )
2000 2000 is (d) None of the above
(a) 6601 (b) 6602 (c) 6603 (d) 6604
12. The value of a x − b y is (where x = loga b and
2. There exist a positive number λ, such that y = logb a , a > 0, b > 0 and a, b ≠ 1)
log 2 x + log 4 x + log 8 x = log λ x , for all positive real (a) 1 (b) 2
numbers x . (c) 0 (d) −1
If λ = b a , where a, b ∈ N , the smallest possible value of
(a + b ) is equal to 13. If x = 1 + loga bc , y = 1 + logb ca, z = 1 + logc ab, then
(a) 12 (b) 63 (c) 65 (d) 75 xyz
is equal to
3. If a, b and c are the three real solutions of the equation xy + yz + zx
(a) 0 (b) 1
2
x log10 x + log10 x +3
2 3
= (c) −1 (d) 2
1 1
− logb ( logb N)
x +1 −1 x + 1 +1 14. The value of a logb a
is
where, a > b > c , then a, b, c are in (a) loga N (b) logb N
(a) AP (b) GP (c) log N a (d) log N b
(c) HP (d) a −1 + b −1 = c −1
n −1 100 15. The value of 49 A + 5 B , where A = 1 − log 7 2 and
4. If f (n ) = ∏ logi (i + 1), the value of
i=2
∑ f (2k ) equals B = − log 5 4 is
k =1 (a) 10.5 (b) 11.5
(a) 5010 (b) 5050 (c) 5100 (d) 5049 (c) 12.5 (d) 13.5

5. If log 3 27 ⋅ log x 7 = log 27 x ⋅ log 7 3, the least value of x, is 16. The number of real values of the parameter λ for which
(a) 7 −3 (b) 3 −7 (c) 7 3 (d) 3 7 (log16 x ) 2 − log16 x + log16 λ = 0 with real coefficients
6. If x = log 5 (1000) and y = log 7 (2058), then will have exactly one solution is
(a) 1 (b) 2
(a) x > y (b) x < y (c) 3 (d) 4
(c) x = y (d) None of these
17. The number of roots of the equation x logx (x + 3 ) = 16 is
2

7. If log 5 120 + ( x − 3) − 2 log 5 (1 − 5 x − 3 )


(a) 1 (b) 0
= − log 5 (0.2 − 5 x − 4 ), then x is (c) 2 (d) 4
(a) 1 (b) 2 (c) 3 (d) 4
(2 x + 1)
18. The point on the graph y = log 2 log 6 {2 + 4 },
8. If x n > x n −1 > ... > x 2 > x 1 > 1, the value of
..x 1 whose y-coordinate is 1 is
..
xn. − 1 (a) (1, 1 ) (b) (6, 1 )
log x1 log x 2 log x 3 K log xn x n is
(c) ( 8, 1 ) (d) (12, 1 )
(a) 0 (b) 1
(c) 2 (d) undefined 19. Given, log2 = 0.301 and log 3 = 0.477, then the number of
x (y + z − x ) y (z + x − y ) z ( x + y − z ) digits before decimal in 312 × 2 8 is
9. If = = ,
log x log y log z (a) 7 (b) 8
then x y = z y is equal to
y x y z (c) 9 (d) 11
(a) z x x z (b) x zy x (c) x yy z (d) x xy y 20. The number of solution(s) for the equation
1 1 2 log x a + logax a + 3 loga 2 x a = 0, is
1 − loga x 1 − loga y
10. If y = a and z = a , then x is equal to (a) one (b) two
1 1 1 1
1 + log a z 2 + log a z 1 − log a z 2 − log a z (c) three (d) four
(a) a (b) a (c) a (d) a
334 Textbook of Algebra

#L Logarithms and Their Properties Exercise 2 :


More than One Correct Option Type Questions
n This section contains 9 multiple choice questions. Each 26. If loga x = α, logb x = β, logc x = γ and logd x = δ, x ≠ 1
question has four choices (a), (b), (c) and (d) out of which and a, b, c , d ≠ 0, > 1, then logabcd x equals
MORE THAN ONE may be correct. α+β+γ+δ α+β+γ+δ
(a) ≤ (b) ≥
(log 2 x )2 − 6 log 2 x + 11
21. If x = 64, then x is equal to 16 16
1 1
(a) 2 (b) 4 (c) 6 (d) 8 (c) −1 (d)
α + β −1 + γ −1 + δ −1 αβγδ
22. If log λ x ⋅ log 5 λ = log x 5, λ ≠ 1, λ > 0, then x is equal to
27. If log10 5 = a and log10 3 = b, then
1
(a) λ (b) 5 (c) (d) None of these (a + b )
5 (a) log10 8 = 3 (1 − a ) (b) log 40 15 =
(3 − 2a )
23. If S = {x : log x 3x , where log 3 x > − 1}, then 1 − a
(c) log 243 32 =   (d) All of these
 b 
(a) S is a finite set (b) S ∈ φ
1  28. If x is a positive number different from 1, such that
(c) S ⊂ ( 0, ∞ ) (d) S properly contains  , ∞
3  loga x , logb x and logc x are in AP, then
2 (log a ) (log c ) a+c
(a) log b = (b) b =
24. If x satisfies log 2 (9 x −1 + 7 ) = 2 + log 2 (3 x −1 + 1), then (log a + log c ) 2
(a) x ∈ Q (c) b = ac (d) c 2 = (ac ) log a b
(b) x ∈ N
(c) x ∈{ x ∈ Q : x < 0 } 29. If | a | < | b |, b − a < 1 and a, b are the real roots of the
(d) x ∈ Ne (set of even natural numbers) equation x 2 − | α | x − | β | = 0, the equation
x
25. log p log p p p p
... p p , p > 0 and p ≠ 1 is equal to log| b | − 1 = 0 has
14243 a
n times (a) one root lying in interval ( − ∞, a )
(a) n (b) −n (b) one root lying in interval (b, ∞ )
1 (c) one positive root
(c) (d) log 1 /p( pn )
n (d) one negative root

#L Logarithms and Their Properties Exercise 3 :


Passage Based Questions
n
This section contains 4 passages. Based upon each of the 32. If a1 = 6, a 2 = 4 and a 3 = 3, the difference of largest and
passage 3 multiple choice questions have to be smallest integral values of N, is
answered. Each of these questions has four choices (a), (b), (a) 2 (b) 8
(c) and (d) out of which ONLY ONE is correct. (c) 14 (d) 20

Passage I Passage II
(Q. Nos. 30 to 32) (Q. Nos. 33 to 35)
Let log 2 N = a1 + b1 , log 3 N = a 2 + b2 and Let ‘S ’ denotes the antilog of 0.5 to the base 256 and ‘K ’
log 5 N = a 3 + b3 , where a1 , a 2 , a 3 ∈ I and denotes the number of digits in 610 (given
b1 , b2 , b3 ∈[ 0, 1). log 10 2 = 0.301, log 10 3 = 0.477) and G denotes the number of
30. If a1 = 5 and a 2 = 3, the number of integral values of N is positive integers, which have the characteristic 2, when the
base of logarithm is 3.
(a) 16 (b) 32 (c) 48 (d) 64
31. If a1 = 6, a 2 = 4 and a 3 = 3, the largest integral value of 33. The value of G is
(a) 18 (b) 24 (c) 30 (d) 36
N is
(a) 124 (b) 63 34. The value of KG is
(c) 624 (d) 127 (a) 72 (b) 144 (c) 216 (d) 288
Chap 04 Logarithms and Their Properties 335

35. The value of SKG is Passage IV (Q. Nos. 39 to 41)


(a) 1440 (b) 17280
Let G , O , E and L be positive real numbers such that
(c) 2016 (d) 2304
log (G ⋅ L ) + log (G ⋅ E ) = 3, log ( E ⋅ L ) + log ( E ⋅ O ) = 4,
Passage III log (O ⋅ G ) + log (O ⋅ L ) = 5 (base of the log is 10).
(Q. Nos. 36 to 38) 39. If the value of the product (GOEL ) is λ, the value of
Suppose ‘U’ denotes the number of digits in the number
( 60)100 and ‘M’ denotes the number of cyphers after decimal, log λ log λ log λK is
−296 (a) 3 (b) 4
before a significant figure comes in ( 8) . If the fraction
(c) 5 (d) 7
U/M is expressed as rational number in the lowest term as p /q
(given log 10 2 = 0.301and log 10 3 = 0.477 ). 40. If the minimum value of 3G + 2L + 2O + E is 2 λ 3 µ 5 ν ,
36. The value of p is where λ , µ and ν are whole numbers, the value of
(a) 1 (b) 2 (c) 3 (d) 4 ∑ (λ µ + µ λ ) is
37. The value of q is (a) 7 (b) 13
(c) 19 (d) None of these
(a) 5 (b) 2
41. If log   and log   are the roots of the equation
(c) 3 (d) 4 G O
38. The equation whose roots are p and q, is O  E
(a) x 2 − 3 x + 2 = 0 (b) x 2 − 5 x + 6 = 0 (a) x + x = 0
2
(b) x 2 − x = 0
(c) x 2 − 7 x + 12 = 0 (d) x 2 − 9 x + 20 = 0 (c) x 2 − 2 x + 3 = 0 (d) x 2 − 1 = 0

#L Logarithms and Their Properties Exercise 4 :


Single Integer Answer Type Questions
n
This section contains 10 questions. The answer to each
question is a single digit integer, ranging from 0 to 9
(both inclusive). 46. If x > 2 is a solution of the equation
| log x − 2 | + | log 3 x − 2 | = 2, then the value of x is
42. If x , y ∈ R + and log10 (2x ) + log10 y = 2 and 3

m
log10 x 2 − log10 (2y ) = 4 and x + y = , where m and n
47. Number of integers satisfying the inequality
n log 2 x − 2 log12/ 4 x + 1 > 0, is
are relative prime, the value of m − 3n 6 is
48. The value of b (> 0) for which the equation
43. A line x = λ intersects the graph of y = log 5 x and
2 log1 / 25 (bx + 28) = − log 5 (12 − 4 x − x 2 ) has coincident
y = log 5 ( x + 4 ). The distance between the points of
intersection is 0.5. Given λ = a + b, where a and b are roots, is
log 2
integers, the value of (a + b ) is 2 21 / 4
− 3 log27 125 − 4
49. The value of is
44. If the left hand side of the equation 7 4 log49 2 − 3
a (b − c ) x 2 + b (c − a ) xy + c (a − b )y 2 = 0 is a perfect 50. If x 1 and x 2 ( x 2 > x 1 ) are the integral solutions of the
square, the value of equation
2
 log(a + c ) + log(a − 2b + c )  5

+
 , (a, b, c ∈ R , a > c ) is (log 5 x ) 2 + log 5 x   = 1, the value of | x 2 − 4 x 1 | is
− x
 log( a c ) 
1
45. Number of integers satisfying the inequality 51. If x = log λ a = loga b = log b c and
| x + 2| 2
 1 2 − |x | log λ c = nx n +1 , the value of n is
  > 9 is
 3
336 Textbook of Algebra

#L Logarithms and Their Properties Exercise 5 :


Matching Type Questions
n This section contains 3 questions. Questions 52 to 54 have four statements (A, B, C and D) given in Column I and four
statements (p, q, r and s) in Column II. Any given statement in Column I can have correct matching with one or more
statement(s) given in Column II.

52. Column I Column II Column I Column II


(D) If (52.6 ) = ( 0.00526 ) = 100, the value of
a b
log 3 243 (p) positive integer (s) 4
(A)
log 2 32 1 1
− is
a b
2 log 6 (q) negative integer
(B)
(log 12 + log 3 )
54. Column I Column II
−2
 1 (r) rational but not
(C) log1 / 3   integer  2 (x − 2)  (p)  1 
 9 (A)  0, 
If log1 /x   ≥ 1, then x can  3
log 5 16 − log 5 4 (s) prime ( x + 1 ) ( x − 5 ) 
(D)
log 5 128 belongs to
3 (q) (1, 2 ]
(B) If log 3 x − log 23 x ≤ log (1/2 2 ) 4, then x
53. Column I Column II 2
can belongs to
(A) The expression log 02.5 8 has the value (p) 1
equal to (C) If log1 / 2( 4 − x ) ≥ log1/2 2 − log1 / 2( x − 1 ), (r) [3, 4)
then x belongs to
(B) The value of the expression (q) 2
(log10 2 ) 3 + log10 8 ⋅ log10 5 + (log10 5 ) 3 + 3, is (D) Let α and β are the roots of the quadratic (s) (3, 8)
equation
(C) Let N = log 2 15 ⋅ log1 / 6 2 ⋅ log 3   . The
1
(r) 3 ( λ2 − 3 λ + 4 ) x 2 − 4 (2 λ − 1 ) x + 16 = 0,
 6
if α and β satisfy the condition β > 1 > α ,
value of [ N ] is (where [ ⋅ ] denotes the
greatest integer function) then p can lie in

#L Logarithms and Their Properties Exercise 6 :


Statement I and II Type Questions
n
Directions Question numbers 55 to 60 are Assertion- Statement-2 a loga N = N , where a > 0, a ≠ 1 and N > 0.
Reason type questions. Each of these questions contains 7 4
57. Statement-1   <  
two statements: 1 1
Statement-1 (Assertion) and Statement-2 (Reason)  3  3
Each of these questions also has four alternative choices,
 1  1
only one of which is the correct answer. You have to select ⇒ 7 log   < 4 log   ⇒ 7 < 4
the correct choice as given below.  3  3
(a) Statement-1 is true, Statement-2 is true; Statement-2 Statement-2 If ax < ay, where a < 0, x , y > 0, then x > y.
is a correct explanation for Statement-1
58. Statement-1 The equation x logx (1− x ) = 9 has two
2

(b) Statement-1 is true, Statement-2 is true; Statement-2


is not a correct explanation for Statement-1 distinct real solutions.
(c) Statement-1 is true, Statement-2 is false Statement-2 a loga b = b, when a > 0, a ≠ 1, b > 0.
(d) Statement-1 is false, Statement-2 is true
59. Statement-1 The equation (log x ) 2 + log x 2 − 3 = 0 has
55. Statement-1 log10 x < log 3 x < loge x < log 2 x two distinct solutions.
( x > 0, x ≠ 1). Statement-2 log x 2 = 2 log x .
Statement-2 If 0 < x < 1, then log x a > log x b ⇒0 < a < b .
3
+1)
60. Statement-1 log x 3 ⋅ log x3 = log 81 (3) has a solution.
/9
56. Statement-1 The equation 7 log7 (x − x 2 = 1 has two Statement-2 Change of base in logarithms is possible.
distinct real roots.
Chap 04 Logarithms and Their Properties 337

#L Logarithms and Their Properties Exercise 7 :


Subjective Type Questions
n In this section, there are 27 subjective questions. 69. Find a rational number, which is 50 times its own
61. (i) If log 7 12 = a, log12 24 = b, then find value of log 54 168 in logarithm to the base 10.
terms of a and b. 70. Find the value of the expression
(ii) If log 3 4 = a, log 5 3 = b, then find the value of log 3 10 in 2 3
terms of a and b. 6
+ .
log 4 (2000) log 5 (2000) 6
ln a ln b ln c
62. If = = , prove the following. 71. Find the value of x satisfying
b −c c −a a −b
loga {1 + logb {1 + logc (1 + log p x )}} = 0.
(i) abc = 1
(3 − 6 ) − 6 log 8 ( 3 −
(ii) aa ⋅ bb ⋅ cc = 1
72. Find the value of 4 5 log4 2 2)
.
2
+ bc + c 2 2
+ ca + a 2 2
+ ab + b 2 73. Solve the following inequations.
(iii) ab ⋅ bc ⋅ ca =1
(i) log ( 2x + 3) x 2 < 1
(iv) a + b + c ≥ 3
(ii) log 2x ( x 2 − 5 x + 6 ) < 1
(v) aa + bb + cc ≥ 3
2
+ bc + c 2 2
+ ca + a 2 2
+ ab + b 2
(iii) log 2(2 − x ) < log1 / 2( x + 1 )
(vi) ab + bc + ca ≥3
(iv) log x 2 ( x + 2 ) < 1
( x − 1)
63. Prove that log10 2 lies between and ⋅
1 1 (v) 3 log 3 < 3 log 3 ( x − 6) + 3
3 4 (vi) log1 /2( 3 x − 1 ) 2 < log1 /2( x + 5 ) 2
64. If log2 = 0.301 and log 3 = 0.477, find the number of (vii) log10 x + 2 ≤ log10
2
x
integers in (viii) log10( x 2 − 2 x − 2 ) ≤ 0
(i) 5 200 (ii) 6 20
 3
(iii) the number of zeroes after the decimal is 3 −500. (ix) log x 2 x −  > 2
 4
65. If log2 = 0.301 and log3 = 0.477, find the value of (x) log1 / 3 x < log1 / 2 x
log (3.375). (xi) log 2x + 3 x 2 < log 2x + 3(2 x + 3 )
66. Find the least value of log 2 x − log x (0.125) for x > 1. 5
(xii) log 22 x + 3 log 2 x ≥ log 4 2 16
2
67. Without using the tables, prove that (xiii) ( x + x + 1 ) < 1
2 x

1 1
+ > 2. (xiv) log ( 3x 2 + 1) 2 <
1
log 3 π log 4 π 2
2
− 3 log10 x + 1
68. Solve the following equations. (xv) x (log10 x ) > 1000
1 + log10 x
(i) x = 10 x (xvi) log 4 {14 + log 6( x − 64 )} ≤ 2
2

(ii) log 2( 9 + 2 ) = 3 x
(xvii) log 2(9 − 2 x ) ≤ 10 log10 ( 3 − x )
(iii) 2 ⋅ x log 4 3 + 3 log 4 x = 27  2x + 3
(xviii) loga   ≥ 0 for
 x 
(iv) log 4 log 3log 2 x = 0
log10 x + 5
(a) a > 1, (b) 0 < a < 1
(v) x 3 = 10 5 + log10 x (xix) 1 + log 2( x − 1 ) ≤ log x − 1 4
(xx) log 5x + 4 ( x 2 ) ≤ log 5x + 4 (2 x + 3 )
 1 
(vi) log 3  log 9 x + + 9 x  = 2 x
 2 
74. Solve log x (ax )1 / 5 + loga (ax )1 / 5
(vii) 4 log10 x + 1 − 6 log10 x − 2 ⋅ 3 log10 x +2
2
=0 1/ 5 1/ 5
x a
log10( x − 3 ) 1 + loga   + log x   = a.
(viii) = a x
log10( x − 21 ) 2
2

75. It is known that x = 9 is root of the equation,


(ix) x log 2 x + 4 = 32 8ax
log π ( x 2 + 15a 2 ) − log π (a − 2) = log π
(x) loga x = x, where a = x log 4 x a −2
(xi) log 2 sin x (1 + cos x ) = 2 find the other roots of this equation.
338 Textbook of Algebra

76. Solve log 4 (log 3 x ) + log 1/ 4 (log1 / 3 y ) = 0 and 83. Find the solution set of the inequality
17 9
x2 +y2 = ⋅ 2 log1/ 4 ( x + 5) > log 1 (9 ) + log (x + 5 ) (2).
4 4 3 3
77. Find the real value(s) of x satisfying the equation 84. Solve log 3 ( x + | x − 1 | ) = log 9 ( 4 x − 3 + 4 | x − 1 | ).
log 2 x ( 4 x ) + log 4 x (16x ) = 4.
85. In the inequality
78. Find the sum and product of all possible values of x 2
which makes the following statement true  x 5
(log 2 x ) 4 −  log1 /2  − 20 log 2 x + 148 < 0
 4  4 
log 6 54 + log x 16 = log 2 x − log 36   .
9 holds true in (a, b ), where a, b ∈ N . Find the value of
ab (a + b ).
79. Solve the equation
3 86. Find the value of x satisfying the equation
log 4 ( x + 2) 3 + 3 = log 4 ( 4 − x ) 3 + log 4 ( x + 6) 3 .
2
(log 3 3 3x + log x 3
3x ) ⋅ log 3 x 3

 1
80. Solve log 2 ( 4 x + 1 + 4 ) ⋅ log 2 ( 4 x + 1) = log1/  . 
2
 8 x  3 
+  log 3 3   + log x 3    log 3 x
3
= 2.
x+ y   3 x
81. Solve the system of equations 2 = 256 and
 3 87. If P is the number of natural numbers whose logarithm
log10 xy − log10   = 1.
 2 to the base 10 have the characteristic P and Q is the
number of natural numbers reciprocals of whose 3
82. Solve the system of equations logarithms to the base 10 have the characteristic − q,
log 2 y = log 4 ( xy − 2), log 9 x 2 + log 3 ( x − y ) = 1. show that log10 P − log10 Q = p − q + 1.

#L Logarithms and Their Properties Exercise 8 :


Questions Asked in Previous 13 Year’s Exam
n
This section contains questions asked in IIT-JEE, 90. The value of
AIEEE, JEE Main & JEE Advanced from year 2005 to  1 1 1 1 
year 2017. 6 + log 3 / 2  4− 4− 4− K  is
3 2 3 2 3 2 3 2 
88. Let a = log 3 log 3 2 and an integer k satisfying 
−a [IIT-JEE 2012, 4M]
1 < 2 (− k + 3 )
< 2, then k equals to [IIT-JEE 2008, 1.5M]
x −1
(a) 0 (b) 1 91. If 3 = 4
x
, then x equals [JEE Advanced 2013, 3M]
(c) 2 (d) 3 2 log 3 2 2
(a) (b)
89. Let ( x 0 , y 0 ) be solution of the following equations 2 log 3 2 − 1 2 − log 2 3
(2x ) ln2 = (3y ) ln 3 and 3 ln x = 2 ln y , then x 0 is 1 2 log 2 3
(c) (d)
[IIT-JEE 2011, 3M] 1 − log 4 3 2 log 2 3 − 1
1 1 1
(a) (b) (c) (d) 6
6 3 2
Answers
Exercise for Session 1 69. 100
1
70. 71. 1 72. 9
6
1. (a) 2. (b) 3. (b) 4. (b) 5. (b)
73. (i) x ∈  − , 3 ∪ {− 1, 0 } (ii) x ∈  0,  ∪ (1, 2) ∪ (3, 6)
3 1
 2   2
Exercise for Session 2
 1 − 5  1 + 5 
1. (b) 2. (b) 3. (d) 4. (c) 5. (c) (iii) x ∈  − 1,  ∪ , 2
 2   2 
Exercise for Session 3 (iv) x ∈ (− 2, 1) ∪ (2, ∞ ) ~ {−1, 0} (v) x > 6
1. (c) 2. (c) 3. (d) 4. (a) 5. (c) (vi) x ∈ ( − ∞ , − 5) ∪ (− 5, − 1) ∪ (3, ∞ )
(vii) x ∈ (0, 10−1 ] ∪ [102 , ∞ )
Chapter Exercises (viii) x ∈ [ −1, 1 − 3 ) ∪ (1 + 3 , 3]
(ix) x ∈  ,  ∪  1, 
1. (c) 2. (d) 3. (b) 4. (b) 5. (a) 6. (a) 3 1 3
(x) x ∈ (0, 1)
7. (a) 8. (b) 9. (a) 10. (c) 11. (c) 12. (c)  8 2  2
13. (b) 14. (b) 15. (c) 16. (b) 17. (b) 18. (d)
(xi) x ∈  − , − 1 ∪ (−1, 3) (xii) x ∈  0,  ∪ [ 2, ∞ )
3 1
19. (c) 20. (b)
 2   16 
21. (a, b, d) 22. (b, c) 23. (c, d) 24. (a, b) 25. (b, d)
(xiii) x ∈ (− ∞ , − 1) (xiv) x ∈ (− ∞ , − 1) ∪ (1, ∞ )
26. (a, c) 27. (a, b, c, d) 28. (a, d) 29. (c, d)
(xv) x ∈ (1000, ∞ ) (xvi) x ∈ [ − 10, − 8) ∪ (8, 10]
30. (b) 31. (d) 32. (a) 33. (a) 34. (b) 35. (d) (xvii) x ∈ ( − ∞ , 0]
36. (b) 37. (c) 38. (b) 39. (b) 40. (a) 41. (d)
(xviii) (a) x ∈ (− ∞ , − 3] ∪ (0, ∞ ) (b) x ∈  −3, − 
3
42. (9) 43. (6) 44. (4) 45. (3) 46. (9) 47. (3)  2
48. (4) 49. (7) 50. (1) 51. (2)

(xx) x ∈  − , −  ∪ [ −1, 0) ∪ (0, 3]
3 3
52. (A) → (p, s), (B) → (p), (C) → (q), (D) → (r) (xix) x ∈ (2, 3]
 5 2
53. (A) → (r), (B) → (s), (C) → (q), (D) → (q)
54. (A) → (q), (B) → (p), (C) → (q, r), (D) → (s) 74. x = a 4/ 5a 2 75. x =15 for a = 3
55. (d) 56. (d) 57. (d) 58. (d) 59. (c) 60. (d) 1 1
76. x = 2 or , y = or 2
ab + 1 ab + 2 2 2
61. (i) (ii) 64. (i) 140 (ii)16 (iii) 238 65. 0.528
a (8 − 5 b ) 2b 9
77. x = 1, 2 −3/ 2 78. Sum = , Product = 2
1 2
66. 2 3 68. (i) 10, (ii) x ∈ φ
10 79. x = 2 80. x = 0 81. (9, 25) and (25, 9)
(iii) x = 16 (iv) x = 8 (v) {10−5 , 103}
82. x = 3, y = 2 83. x ∈ (−5,−4) ∪ (−3,−1)
1 1 1
(vi) x = (vii) x = (viii) x = 5 (ix) x = 2 or 25
3 100 32 84. x = 85. 3456 86. x ∈ (1, 3] 88. (b)
π 64
(x) x = 2 (xi) x = 89. (c) 90. (4) 91. (a, b, c)
3
340 Textbook of Algebra

Solutions
n -1
log(i + 1 ) log(n )
4. f (n ) = Õ = = log 2 n
i=2 log(i ) log(2 )
\ f (2k ) = k
100 100 100 × (100 + 1 )
Then, å f (2k ) = kå= 1k = 2
= 5050
k =1
1. log10 2 = 0.3010
5. log 3 27 × log x 7 = log 27 x × log 7 3 ... (i)
Let y = 2000 2000
Eq. (i) valid for x > 0, x ¹ 1
log10 y = 2000 log10 2000 = 2000 ´ (log10 2 + 3 )
On solving Eq. (i),
= 2000 ´ 3.3010 = 6602 1
So, the number of digits in 2000 2000 = 6602 + 1 = 6603. log 3(3 3 ) × log x 7 = log 3 x × log 7 3
3
2. Q l > 0 and l ¹ 1 and x > 0 Þ 9 × log x 7 = log 7 x
log 2 x + log 4 x + log 8 x = log l x Þ 9 = (log 7 x ) 2
1 1 Þ log 7 x = ± 3
Þ log 2 x + log 2 x + log 2 x = log l x
2 3 Þ x = 7 3 or x = 7 -3
11
Þ log 2 x = log l x 1
6 Then, the least value of x is 3 i.e., 7 -3.
7
11 11
Þ = 6. Q x = log 5(5 3 ´ 8) = 3 + log 5 8
6 log x 2 log x l
Þ x - 3 = log 5 8 …(i)
Þ 11 log x l - 6 log x 2 = 0
æ l11 ö and y = log 7 (7 3 ´ 6 ) = 3 + log 7 6
l11
Þ log x ç 6 ÷ = 0 Þ 6 = 1 Þ y - 3 = log 7 6 …(ii)
è2 ø 2
Q 8 > 6 and 7 > 5
Þ l11 = 2 6 Þ l = 2 6 /11
Þ log 8 > log 6 and log 7 > log 5
Þ l = (2 6 )1/11 …(i)
or (log 8 )(log 7 ) > (log 6 ) (log 5 )
Given that, l = b a and a, b Î N Þ log 5 8 > log 7 6
1
Þ x -3 >y -3 [from Eqs. (i) and (ii)]
Þ l = ab …(ii)
\ x >y
From Eqs. (i) and (ii), we get 7. Q log 5 120 + ( x - 3) - 2 log 5 (1 - 5 x - 3 ) = - log 5( 0 .2 - 5 x - 4 )
a = 2 6 and b = 11
Þ log 5(5 ´ 24 ) + ( x - 3 )
Þ a + b = 64 + 11 = 75 æ 1 - 5x - 3 ö
2
log10 x + log10 x 3 + 3 2 = log 5(1 - 5 x - 3 ) 2 - log 5 ç ÷
3. x = è 5 ø
1 1
-
x + 1 -1 x + 1 +1 Þ 1 + log 5 24 + ( x - 3 ) = log 5 {5 × (1 - 5 x - 3 )}
Given, a, b and c are real solution Eq. (i) and a > b > c and for Þ 1 + log 5(24 × 5 x - 3 ) = 1 + log 5(1 - 5 x - 3 )
Eq. (i) to be defined x > 0, x > - 1 Þ x > 0 from Eq. (i), Þ 24 × 5 x - 3 = 1 - 5 x - 3
log 2 x + 3 log10 x + 3 2x
x 10 = Þ 25 × 5 x - 3 = 1
2
On taking logarithm both sides on base 10, then Þ 5x - 1 = 50
2
(log10 x + 3 log10 x + 3 ) log10 x = log10 x \ x -1 = 0 Þ x =1
2 8. Given, xn > xn -1 > ××× > x 2 > x1 > 1
Þ (log10 x + 3 log10 x + 2 ) log10 x = 0
x
x N 1
Þ(log10 x + 1 )(log10 x + 2 ) log10 x = 0 \ log x1 log x 2 log x 3 ××× log xn xn n - 1 x1
x n - 2N
\ log10 x = - 2, -1, 0 = log x1 log x 2 log x 3 ××× log xn -1xn -1
\ x = 10 -2, 10 -1, 10 0 = log x1 x1 = 1 (Q loga a = 1 )
1 1 x (y + z - x ) y (z + x - y ) z ( x + y - z ) 1
x= , ,1 9. Let = = =
100 10 log x log y log z n
1 1
So, a, b, c can take values a = 1, b = , c = (Q a > b > c ) Then, log x = nx (y + z - x ) …(i)
10 100
log y = ny (z + x - y ) …(ii)
\ a, b, c ÎGP
and log z = nz ( x + y - z ) …(iii)
Chap 04 Logarithms and Their Properties 341

logb (logb N )
\ y log x + x log y = y log z + z log y
14. a logb a
= a loga (logb N ) = logb N
= z log x + x log z
Þ log( x y × y x ) = log(y z × z y ) = log( x z × z x ) 15. 49 A + 5 B = ?
A = 1 - log 7 2
Þ x y ×y x = y z ×z y = z x × x z
1
A = log 7 7 - log 7 2
1 - loga x 7 7 49
10. Q y = a A = log 7 Þ 7 A = Þ 49 A =
1 2 2 4
Þ loga y = …(i)
1 - loga x æ1ö B 1
and B = - log 5 4 = log 5 ç ÷ Þ 5 =
1 è 4ø 4
and z = a 1 - loga y A B 49 1 50
\ 49 + 5 = + = = 12.5
1 4 4 4
or loga z = …(ii)
1 - loga y 16. (log16 x ) 2 - log16 x + log16 l = 0 ...(i)
From Eqs. (i) and (ii), we get 2
æ 1ö 1
1 1 Eq. (i) defined for x > 0, l > 0 ç log16 x - ÷ - + log16 l = 0
loga z = =1- è 2ø 4
æ 1 ö loga x
1-ç ÷ For exactly one solution,
è 1 - loga x ø 1
log16 x - = 0
1 1 2
Þ = (1 - loga z ) Þ loga x =
loga x (1 - loga z ) 1 1
\ - + log16 l = 0 Þ log16 l =
1 4 4
\ x = a 1 - loga z or 1/ 4
l = (16 ) = 2
11. log 0.3( x - 1) < log 0.09( x - 1) …(i) 17.
2
x log x ( x + 3) = 16 …(i)
Eq. (i) defined for x > 1, …(ii) From Eq. (i), x > 0 and x ¹ 1 ... (ii)
Þ log 0.3 ( x - 1 ) - log ( 0.3)2 ( x - 1 ) < 0
By Eq. (i), ( x + 3 ) 2 = 16
1
Þ log 0.3( x - 1 ) - log 0.3 ( x - 1 ) < 0 Þ x+3=±4
2
1 Þ x = 1 or x = - 7
Þ log 0.3( x - 1 ) < 0 From Eq. (ii), no values of x satisfy Eq. (i).
2
Þ log 0.3 ( x - 1 ) < 0 \ Number of values of x satisfy Eq. (i)
Þ ( x - 1 ) > ( 0.3 )0 \ Number of roots = 0
2x + 1
[Q base of log is lie in (0, 1)] 18. Given, y = log 2 log 6(2 + 4) …(i)
Þ x >2 …(iii) From Eq. (i) to be defined,
From Eqs. (ii) and (iii), we get 1
2x + 1 > 0 Þ x > - ... (ii)
x > 2 Þ x Î(2, ¥ ) 2
12. Q a x = a loga b We find value of x for which y = 1
2x + 1
=a a
log b × loga b logb a
=a
loga b logb a = b logb a
= by \ 1 = log 2 log 6 (2 + 4)
\ a - by = 0
x
Þ log 6(2
2x + 1
+ 4) = 2
13. Q x = 1 + loga bc = loga a + loga bc = loga (abc ) Þ 2
2x + 1
+ 4 = 36
1
\ = logabc a …(i) Þ 2
2x + 1
= 32 = 2 5 Þ 2 x + 1 = 5
x
1 Þ 2 x + 1 = 25 Þ x = 12
Similarly, = logabc b …(ii)
y So, required point is (12, 1).
1 19. Given that, log2 = 0.301
and = logabc c …(iii)
z log3 = 0.477
On adding Eqs. (i), (ii) and (iii), we get Let y = 312 ´ 2 8
1 1 1
+ + = logabc abc = 1 log y = 12 log 3 + 8 log 2
x y z
xy + yz + zx xyz = 12 ´ ( 0.477 ) + 8 ( 0.301 ) = 8.132
Þ = 1 or =1
xyz xy + yz + zx So, number of digits before decimal in 312 ´ 2 8 = 8 + 1 = 9
342 Textbook of Algebra

20. Given, equation 2 log x a + logax a + 3 loga 2 x a = 0 ...(i) Þ (3 x - 1 - 3 ) (3 x - 1 - 1 ) = 0


2 1 3 Þ x - 1 = 1 or x - 1 = 0
Þ + + =0 …(ii)
loga x 1 + loga x 2 + loga x Þ x = 2 or x = 1
Let loga x = t 25. y = log p log p ( p p ×××p p ) [ p > 0, p ¹ 1 ]
Then, Eq. (ii), 1424 3
n times
2 1 3
+ + = 0 Þ 6t 2 + 11t + 4 = 0 ì 1ü ì ü
t 1+t 2+t ï ï ï1 ï
= log p í log p ( p ×××p p ) p ý = log p í log p ( p p ×××p p )ý
4 1 ï 14243ï ï p 14243ï
Þ t = - or - î (n - 1) times þ î (n - 1) times þ
3 2
ì ü
So, x = a -4 / 3 or x = a -1 / 2 ï1 1 ï
= log p í × log p ( p p ×××p p )ý
Two value of x possible for which Eq. (i) is defined and satisfy. ï p p 1 4 2 4 3 ï
î (n - 2 ) times þ
21. Decimal on x > 0 and x ¹ 1.
æ 1 ö
Taking logarithm on both sides on base 2, we get = log p ç n ÷ = - n, log1/p pn = -n
èp ø
{(log 2 x ) 2 - 6 log 2 x + 11 } log 2 x = 6
Let log 2 x = t 26. loga x = a, logb x = b, logc x = g, logd x = d
\ 3 2
t - 6t + 11t - 6 = 0 Þ log x a = a -1 …(i)
-1
Þ (t - 1 ) (t - 2 ) (t - 3 ) = 0 Þ t = 1, 2, 3 Þ log x b = b …(ii)
Þ log 2 x = 1, 2, 3 Þ log x c = g -1 …(iii)
Þ x = 2, 2 2, 2 3 Þ log x d = d-1 …(iv)
22. log l x × log 5 l = log x 5 …(i) On adding Eqs. (i), (ii), (iii) and (iv) , we get
l ¹ 1, l > 0 and x > 0, x ¹ 1 1 1 1 1
log x (abcd ) = + + + …(v)
Þ log 5 x = log x 5 Þ (log 5 x ) 2 = 1 a b g d
Þ log 5 x = ± 1 Þ x = 51 and 5 -1 1
\ logabcd x = -1
1 a + b + g - 1 + d- 1
-1

\ x = 5 and
5 For a, b, g, d
a+b+g+d 4
23. S = { x : log x 3x : log 3 x > - 1} AM ³ HM Þ ³ -1
4 a + b - 1 + g - 1 + d- 1
log 3 x > - 1
1 a+b+g+d
1 or £
Þ x> …(i) a -1 + b - 1 + g - 1 + d- 1 16
3
a+b+g+d
Let y = log x 3 x , x ¹ 1 or logabcd x £ [from Eq. (v)]
16
To be defined y, 3 x > 0 Þ x > 0 …(ii)
27. Q log10 5 = a and log10 3 = b …(i)
and log x 3 x ³ 0 …(iii)
æ 10 ö
From Eqs. (i) and (iii), \ log10 2 = log10 ç ÷ = 1 - a …(ii)
è5ø
æ1 ö
for x Î ç , 1÷ Þ 3x £ 1 Option (a)
è3 ø
1 \ log10 8 = 3 log10 2 = 3 (1 - a ) [from Eq. (ii)]
Þ 3x £ 1 Þ x£ log10 15 log10(5 ´ 3 )
3 Option (b) log 40 15 = =
No solution for this case. log10 40 log10 (2 3 ´ 5 )
1 log10 5 + log10 3
Now, for x > 1, from Eq. (iii), 3 x ³ 1 Þ x ³ =
3 log10 2 3 + log10 5
\ x >1 a+b (a + b )
24. Given equation, = =
3 (1 - a ) + a (3 - 2a )
log 2(9 x - 1 + 7 ) = 2 + log 2(3 x - 1 + 1 )
5 log10 2
{3 2 ( x - 1) + 7 } Option (c) log 243 32 = log 35 2 5 = log 5 2 =
Þ log 2 ( x - 1) =2 5 log10 2
3 +1 1 -a
= [from Eqs. (i) and (ii)]
Þ 3 2 ( x - 1) + 7 = 4 × {3 ( x - 1) + 1 } a
Þ {3 ( x - 1) } 2 - 4 × 3 ( x - 1) + 3 = 0 Hence, all options are correct.
Chap 04 Logarithms and Their Properties 343

28. Q x > 0 and x ¹ 1 Sol. (Q. Nos. 33 to 35)


Given, loga x , logb x and logc x are in AP. S = Antilog of (0.5) to the base 256
Þ 2 logb x = loga x + logc x Þ log 256 S = 0.5
2 log x log x log x S = (256 ) 0.5 = (2 8 )1/2
Þ = +
log b log a log c S = 24
2 1 1 é log x ¹ 0 ù S = 16 …(i)
Þ = + ê\ x ¹ 1 ú
log b log a log c ë û K = Number of digits in 610

Þ log b =
2 (log a ) (log c ) [Q log10 2 = 0.301, log10 3 = 0.477 ]
(log a + log c ) Let a = 610
log b 2 log c log a = 10 log10 6 = 10 ( 0.301 + 0.477 )
Also, =
log a log a + log c = 10 ( 0.778 )
log c 2 log(610 ) = 7.78
Þ loga b = = log (ac ) c 2
log(ac ) So, x = 7 + 1, x = 8
\ c 2 = (ac ) loga b Number of positive integers which have characteristic 2, when the
base of logarithm is 3
29. | a | < |b |, b - a < 1
= 3 2 + 1 - 3 2 = 18
a, b Î x 2 - | a | x - | b | = 0 …(i)
\ G = 18
a + b = | a| ü
So, ý …(ii) 33. The value of G = 18
ab = - |b|þ
34. The value of KG = 8 ´ 18 = 144
x x
Given equation, log| b| - 1 = 0, log| b| =1 35. The value of SKG = 16 ´ 8 ´ 18 = 16 ´ 144 = 2304
a a
x Sol. (Q. Nos. 36 to 38)
Þ = | b |1
a U = Number of digits in (60 )100
Þ | x | = | ab | Let a = (60 )100
Þ | x | = | b| [from Eq. (ii)] log10 a = 100 log10 60 = 100 (1 + log10 2 + log10 3 )
\ x=±b = 100 (1.778 )
Sol. (Q. Nos. 30 to 32) log10 a = 177.8
Q log 2 N = a 1 + b 1 So, U = 177 + 1 Þ U = 178 …(i)
Þ b1 = log 2 N - a1 M = Number of cyphers after decimal, before a significant
Given, 0 £ b1 < 1 Þ 0 £ log 2 N - a1 < 1 figure comes in (8 ) -296
Þ a1 £ log 2 N < 1 + a1 Let b = (8 ) -296
Þ 2a 1 £ N < 21 + a 1 …(i) log10 b = ( -296 ) log10 8 = ( -296 ) ´ 3 log10 2
1+a2
Similarly, 3a 2 £ N 3 …(ii) log10 b = ( -296 ) ´ 3 ´ ( 0.301 )
and 5a 3 £ N < 51 + a 3 …(iii) = - 267.288 = - 267 - 0.288
= - 267 - 1 + (1 - 0.288 ) = - 268 + 0.712
30. Here, a1 = 5 and a 2 = 3, then from Eqs. (i) and (ii),
log10 b = 268.712
2 5 £ N < 2 6 and 3 3 £ N < 3 4
\ M = 268 - 1 = 267
\ Common values of N are 32, 33, 34, K, 63 U 178
Now, =
Number of integral values of N are 32. M 267
31. Here, a1 = 6, a 2 = 4 and a 3 = 3, then from Eqs. (i), (ii) and (iii), According to the question,
2 6 £ N < 2 7, 3 4 £ N < 3 5 and 5 3 £ N < 5 4 U 2
=
M 3
Þ 64, 65, 66, ..., 127, 81, 82, 83,..., 242 and 125, 126, ..., 624
U p
\ Largest common value = 127 Þ =
M q
32. Here, a1 = 6, a 2 = 4 and a 3 = 3
So, p =2
From question number 31, we get and q =3
64, 65, 66,..., 127; 81, 82, 83, ..., 242 and 125, 126, ...,624 36. The value of p = 2.
\ Largest common value = 127
37. The value of q = 3.
and smallest common value = 125
\ Difference = 127 - 125 = 2 38. The equation whose roots are p and q is x 2 - 5x + 6 = 0.
344 Textbook of Algebra

Sol. (Q. Nos. 39 to 41) From Eqs. (i) and (ii), x 3 = 10 6 Þ x = 100
According to question, G,O, E , L > 0 and are real numbers. 1
From Eq. (i), y =
Such that, 2
log10(G × L ) + log10(G × E ) = 3 Þ log10 G 2LE = 3 1 201 m
\ x + y = 100 + = = (given)
Þ G 2LE = 10 3 ...(i) 2 2 n
and log10 E × L + log10 E × O = 4 \ m = 201 and n = 2
Þ log10 E 2 × L × O = 4 Þ m - 3n 6 = 201 - 3 (2 ) 6 = 201 - 192 = 9

Þ E 2 × L × O = 10 4 ...(ii) 43. Solving, x = l and y = log 5 x, we get


and log10(O × G ) + log10(O × L ) = 5 A º ( l, log 5 l ), l > 0
Þ log10 O 2 GL = 5 Þ O 2 GL = 10 5 ...(iii) and solving x = l and y = log 5( x + 4 ), we get
From Eqs. (i), (ii) and (iii), we get B º { l, log 5( l + 4 )}, l > - 4
G 3O 3E 3L3 = 1012 Given, AB = 0.5
GOEL = 10 4 ...(iv) Þ log 5 ( l + 4 ) - log 5 l = 0.5
Þ l = 10 4 l+4
Þ = (5 )1/2 = 5
39. Now, let l
4 ( 5 + 1)
1
+
1 1
+ + ××× Þ l= =4
y = log l log l log l ××× = (log l ) 2 4 8 5 -1 4
1/ 2 =1 + 5 =a + b [given]
1 - 1/ 2
= (log l ) = (log l ) \ a = 1 and b = 5
4
= log 10 = 4 log 10 = 4 Then, a + b =1 + 5 =6
44. \ a (b - c ) x 2 + b (c - a ) xy + c (a - b ) y 2 = b, y ¹ 0 …(i)
40. Minimum of 3G + 2L + 2O + E = 2 l3m5 u
2
where l, m, u ÎW æxö æxö
a (b - c ) ç ÷ + b (c - a ) ç ÷ + c (a - b ) = 0
Apply AM ³ GM for 3G, 2 L, 2O, E èy ø èy ø
3G + 2 L + 2O + E 8 3 x
³ G ´ L2 ´ O 2 ´ E Let =X
8 y
So, 8 ´ 8 G 3L 2O 2E = 2 l3m5 u …(v) Þ a (b - c ) X 2 + b (c - a ) X + c (a - b ) = 0
(equality hold, if G = L = O = E ) Q a (b - c ) + b (c - a ) + c (a - b ) = 0
From Eqs. (i) and (iii) of Q. 10, we get \ X =1
G 3L2O 2E = 10 8 Q Eq. (i) is perfect square.
\ Roots are equal.
From Eq. (v), 8 ´ (10 8 )1/8 = 2 l3m5 u c (a - b )
\ 1 ´1 =
a (b - c )
8 ´ 10 = 2 l3m5 u
2ac
Þ b= …(ii)
2 4 ´ 51 = 2 l3m5 u a+c
\ l = 4, u = 1, m = 0 Now, log(a + c ) + log(a - 2b + c )
m l
S( l + m ) = ( 4 0 + 0 4 ) + ( 01 + 1 0 ) + (1 4 + 41 ) = log {(a + c ) 2 - 2b (a + c )}
= (1 + 0 ) + ( 0 + 1 ) + 1 + 4 = 7 = log {(a + c ) 2 - 4ac } [from Eq. (ii)]
æG ö æO ö æG ö 2
= log(a - c ) = 2 log(a - c )
41. log10 ç ÷ + log10 ç ÷ = log10 ç ÷ = log10 1 = 0
èO ø èEø èEø log(a + c ) + log(a - 2b + c )
[divide Eq. (iv) and Eq. (ii) of Q. 39] Þ =2
log(a - c )
G O æ1ö 2
P = log10 × log10 = log ç ÷ log(10 ) = - 1 ì log(a + c ) + log(a - 2b + c )ü
O E è 10 ø \ í ý =4
î log(a - c ) þ
[by dividing Eq. (i) by Eq. (ii) and dividing Eq. (iii) by Eq. (iv) in
Q. 10] 45. According to the question, x Î I
2 2 | x + 2|
= x - 0 × x + ( -1 ) = 0 = x - 1
æ 1 ö 2 - | x|
2 Given equation, ç ÷ > 9 [ x ¹ ± 2] …(i)
42. log10(2x ) + log10 y = 2 Þ 2xy = 10 …(i) è3ø
| x + 2|
and log10 x 2 - log10 2y = 4 | x + 2|
2 - | x|
Þ 3 > 32 Þ - >2
x2 2 -| x |
Þ = 10 4 …(ii)
2y
Chap 04 Logarithms and Their Properties 345

| x + 2| Þ y =2
Þ -2 > 0
|x | - 2 Þ log 3 x = 2
| x + 2| - 2 | x | + 4 \ x = 32 = 9 [impossible]
Þ >0 …(ii)
| x| - 2 Case III y ³ 2, then x ³ 9
x - 2 + 2x + 4 From Eq. (ii), 2 (y - 1 ) + (y - 2 ) = 2
Case I If x < - 2, - >0
-x - 2 \ y = 2, log 3 x = 2
++ \ x =9 [acceptable]
x
– – – – 0 47. Given equation is
log 2 x - 2 log12/4 x + 1 > 0 …(i)
++++
x+2
– – –2 From Eq. (i), x>0
1 2
Eq. (i) Þ log 2 x - log 22 x + 1 > 0
x+2 2 (- 2)2
> 0 Þ -1 > 0
-(x + 2) 1 1
Þ log 2 x - log 22 x + 1 > 0
which is not possible. 2 2
Case II -2 < x < 0, then Eq. (ii) Þ (log 2 x ) 2 - (log 2 x ) - 2 < 0
x + 2 + 2x + 4 3x + 6 Þ (log 2 x - 2 ) (log 2 x + 1 ) < 0
Þ >0 Þ >0
-x - 2 -(x + 2) Þ - 1 < log 2 x < 2
-3 ( x + 2 ) Þ 2 -1 < x < 2 2
> 0-3 > 0
(x + 2) 1
Þ <x<4
which is not possible. 2
Case III when x > 0 Þ x Î I , so x = 1, 2, 3
From Eq. (ii), So, number of integer value of x is 3.
x + 2 - 2x + 4 -x + 6 48. Given that, b > 0
>0 Þ >0
x -2 x -2
2 log1/25 (bx + 28 ) = - log 5 (12 - 4 x - x 2 ) …(i)
x -6
<0 2
x -2 log 5 (bx + 28 ) = - log 5 (12 - 4 x - x 2 )
(- 2)
2 < x <6
Þ bx + 28 = 12 - 4 x - x 2
So, the integer values of x = 3, 4, 5
and bx + 28 > 0
So, the number of integer values of x is 3.
and 12 - 4 x - x 2 > 0
46. x > 2
Þ x 2 + ( 4 + b ) x + 16 = 0 …(ii)
| log 3 x - 2 | + | log 3 x - 2 | = 2
| 2 log 3 x - 2 | + | log 3 x - 2 | = 2 - 28
and x> and - 6 < x < 2
2 | log 3 x - 1 | + | log 3 x - 2 | = 2 b
Since, Eq. (i) has coincident roots, so discriminant Eq. (ii) is zero.
Let log 3 x = y …(i)
( 4 + b ) 2 - 64 = 0
Then, Eq. (i) Þ2 | y - 1 | + | y - 2 | = 2 …(ii)
b+4=±8
+ b = 4 or b = - 12
y–2
– – 2
Since, b > 0 so b = 4
+ + for this value x > - 7 and - 6 < x < 2
y–1 3 3
– 1 2 log1 / 4 2 - 3 log 27 125 - 4 2 4 log 2 2 - 3 log 3 5
-4
49. =
7 4 log 49 2 - 3 4 log 2 21
7 7
Case I y < 1, then x < 3
Eq. (ii) becomes - 2y + 2 - y + 2 = 2 24 - 5 - 4 16 - 9
= = =7
2 2 log 7 2 22 - 3
- 3y = - 2, y = 7 -3
3
æ5ö 1
2 50. (log 5 x ) 2 + log 5x ç ÷ = 1, x > 0, x ¹
log 3 x = [from Eq. (i)] èxø 5
3
æ5ö
Þ x = 32 / 3 log 5 ç ÷
èxø 1 - log 5 x
which is less than 2, so not acceptable. Þ (log 5 x ) 2 + = 1 Þ (log 5 x ) 2 + =1
log 5 (5 x ) 1 + log 5 x
Case II 1 < y < 2, then 3 < x < 9
Let log 5 x = t, then
From Eq. (ii), 2 (y - 1 ) - (y - 2 ) = 2
346 Textbook of Algebra

1 -t 9 < 15 < 27
t2 + =1
1+t 2 < log 3 15 < 3
Þ t 3 + t 2 - 2t = 0 So, [ N ] = 2 (q)
Þ t (t + 2 ) (t - 1 ) = 0 Þ t = - 2, 0, 1 (D) (52.6 )a = ( 0.00526 )b = 100
Þ x = 5 -2, 5 0, 51 (52.6 )a = 100 and ( 0.00526 )b = 100
2
1
Þ x = , 1, 5 52.6 = 10a …(i)
25
b - 4b 2
x1, x 2 Î I (52.6 ) ´ 10 = 10
\ x1 = 1, x 2 = 5 (52.6 ) = 10 2 + 4 b
b

\ | x 2 - 4 x1 | = | 5 - 4 | = 1 æ2 + 4b ö
ç ÷
1 è b ø
51. Given, x = log l a = loga b = logb c and log l c = nxn + 1 Þ 52.6 = 10 …(ii)
2 From Eqs. (i) and (ii), we get
x = log l a = loga b = logb c and log l c = nxn +1 ...(i) 2 2 + 4b

From Eq. (i), log l a ´ loga b + logb c = x 3 10a = 10 b


1 2 2
log l c = x 3, log l c = x 3 Þ = +4
2 a b
1 1
log l c = 2 x 3 Þ - = 2 (q)
a b
Compare with log l c = nxn + 1 2 (x - 2)
54. (A) Given that, log1/x ³1 …(i)
Þ n =2 (x + 1) (x - 5)
log 3 243 log 3 3 5 5 ´2 (x - 2)
52. (A) = = = 2 (p,s) for log to be defined > 0,
log 2 32 - 1 log 2 5 5 (x + 1) (x - 5)
2
2
then x Î ( - 1, 2 ) È (5, ¥ )
2 log 6 2 log 6 2 log 6
(B) = = = 1 (p)
log 12 + log 3 log 36 2 log 6 + +
-2 – –1 2 – 5
æ1ö
(C) log1/3 ç ÷ = - log 3 3 4 = - 4 (q)
è9ø Let x > 0 and x ¹ 1
æ 16 ö So, x Î ( 0, 1 ) È (1, 2 ) È (5, ¥ )
log 5 ç ÷
log 5 16 - log 5 4 è4ø log 5 (2 ) 2 2 Case I x Î( 0, 1 ) …(ii)
(D) = 7
= = (r)
log 5 128 log 5 (2 ) log 5 (2 ) 7 7 1
>1
x
53. (A) log (20× 5) 2 8 = log12/2 8 = (log 2-1 2 3 ) 2 2 (x - 2)
\ By Eq. (i), log 1 ³1
2 ( x + 1) (x - 5)
æ 3 ö x
= ç log 2 2 ÷ = ( - 3 ) 2 = 9 = 3 (r) 2 (x - 2) 1
è-1 ø Þ ³
(x + 1) (x - 5) x
(B) (log10 2 ) 3 + log10 8 × log10 5 + (log10 5 ) 3 2 (x - 2) 1
Þ - ³0
= (log10 2 ) 3 + 3 log10 2 log10 5 + (log10 5 ) 3 (x + 1) (x - 5) x
= (log10 2 ) 3 + 3 × log10 2 × log10 5 × (log10 2 + log10 5 ) 2x (x - 2) - (x + 1) (x + 5)
Þ ³0
x (x + 1) (x - 5)
+ (log10 5 ) 3
2x 2 - 4x - x 2 + 4x + 5
[Q log10 2 + log10 5 = log10 10 = 1 ] Þ ³0
x (x + 1) (x - 5)
x2 + 5
= (log10 2 + log10 5 ) 3 = (log10 10 ) 3 = (1 ) 3 = 1 Þ ³0
x (x + 1) (x - 5)
3 + (log10 2 ) 3 + log10 8 × log10 5 + (log10 5 ) 3 Þ x (x + 1) (x - 5) > 0
= 3 + 1 = 4 (s)
–1 + +
1
(C) N = log 2 15 × log1/6 2 × log 3 – 0 – 5
6
= log 2 15 ( - log 6 2 ) ( - log 3 6 ) Þ x Î ( - 1, 0 ) È (5, ¥ )
log 15 log 2 log 6 But by Eq. (ii), x Î( 0, 1 )
= ´ ´ = log 3 15
log 2 log 6 log 3 So, no solution for this case.
Chap 04 Logarithms and Their Properties 347

Case II Let x Î (1, 2 ) È (5, ¥ ) …(iii) From Eqs. (ii) and (iii),
1 x Î (1, 2 ] È [3, 4 ) (q, r)
<1
x
(D) Given equation is
2 (x - 2)
Eq. (i) Þ log 1 ³1 ( l2 - 3 l + 4 ) x 2 - 4 (2 l - 1 ) x + 16 = 0 …(i)
x
(x + 1) (x - 5)
2 2 2
2 (x - 2) æ3ö æ3ö æ 3ö 7
£
1 l2 - 3 l + 4 = l2 - 3 l + ç ÷ - ç ÷ + 4 = ç l - ÷ +
è2ø è2ø è 2 ø 4
(x + 1) (x - 5) x
2 (x - 2) 1 So, l2 - 3 l + 4 > 0, " l Î R
Þ - £0
(x + 1) (x - 5) x
1
x2 + 5
Þ £0 [by case I] α β
x (x + 1) (x - 5)
Þ x (x + 1) (x - 5) < 0
Þ x Î ( - ¥, - 1 ) È ( 0, 5 ) …(iv) and D>0
Eq. (iii), x Î (1, 2 ) È (5, ¥ ) 15
Þ We get l> …(ii)
8
+ +
Let f ( x ) = ( l2 - 3 l + 4 ) x 2 = 4 (2 l - 1 ) x + 16
– –1 0 – 5
\ f (1 ) < 0 by graph of f ( x )
From Eqs. (iii) and (iv), x Î(1, 2 ] (q)
l2 - 11 l + 24 < 0
3
(B) log 3 x - log 23 x £ log 1 4 …(i) (l - 3) (l - 8) < 0
2 2 2
3 < l <8 …(iii)
defined, when x > 0
From Eqs. (ii) and (iii), we get
3 æ - 2ö
log 3 x - log 32 x ³ ´ç ÷ ´2 ´1 3 < l < 8 Þ l Î(3, 8 ) (s)
2 è 3 ø
55. If 0 < a < b
Þ log 3 x - log 32 x + 2 £ 0
Statement-1 If x > 1
Þ log 23 x - log 3 x - 2 ³ 0
Þ log x a < log x b
Þ (log 3 x - 2 ) (log 3 x + 1 ) ³ 0 \Statement-2 If 0 < x < 1
Þ log 3 x £ - 1 Þ log x a > log x b
or log 3 x ³ 2 \ Statement-2 is true, also
1
Þ x £ or x ³ 9 10 > 3 > e > 2
3
If x > 1,
From Eq. (i), x > 0
then log x 10 > log x 3 > log x e > log x 2
æ 1ù 1 1 1 1
So, x Î ç 0, ú È [9, ¥ ) (p) Þ < < <
è 3û
log x 10 log x 3 log x e log x 2
(C) log 1 ( 4 - x ) ³ log 1 2 - log 1 ( x - 1 ) …(i) Þ log10 x < log 3 x < loge x < log 2 x
2 2 2
and for 0 < x <1
(4 - x ) (x - 1)
Þ log 1 ³0 We get, log10 x > log 3 x > loge x > log 2 x
2
2 It is clear that for x > 0, x ¹ 1
(x - 4) (x - 1) Statement-1 is false.
Þ - £1
2
Þ (x - 4) (x - 1) ³ - 2 ( x 3 + 1)
56. Statement-1 7 log 7 - x2 = 1 …(i)
Þ x 2 - 5x + 4 + 2 ³ 0
x + 1 - x = 1 ì for this x + 1 > 0
3 2 3
Þ x 2 - 5x + 6 ³ 0 ï
x3 - x2 = 0 íÞ x3 > - 1
x 2 ( x - 1 ) = 0 ïî Þ x > - 1
+ +
2 – 3
x = 0 (repeated) or x = 1
( x - 3 )( x - 2 ) ³ 0 Thus, Eq. (i) has 2 repeated roots.
x £ 2 or x ³ 3 …(ii) \Statement-1 is false.
From Eq. (i) to be defined, 4 - x > 0 and x - 1 > 0 Statement-2 a loga N = N , a > 0, a ¹ 1 and N > 0
x < 4 and x > 1 …(iii)
which is true.
348 Textbook of Algebra

æ1ö
7
æ1ö
4 log 12 2 log 2 + log 3
57. Statement-1 ç ÷ < ç ÷ . Taking log on both sides, 61. (i) Qa = log 7 12 = =
è3ø è3ø log 7 log 7
æ1ö
7
æ1ö
4 2 + log 2 3
loge ç ÷ < loge ç ÷ a= …(i)
è3ø è3ø log 2 7
1 1 log 24 3 log 2 + log 3
7 loge < 4 loge and b = log12 24 = =
3 3 log 12 2 log 2 + log 3
1 3 + log 2 3
Now, loge < 0 [Q2 < e < 3] = …(ii)
3 2 + log 2 3
So, 7>4
Let log 2 3 = l and log 2 7 = m
Statement-1 is false. ...(i)
2+l
Statement-2 ax < ay From Eq. (i), a =
m
and a < 0, x > 0, y > 0
3+l
Eq. (i) divide by a, we get x > y and from Eq. (ii), b = , we get
2+l
Statement-2 is true.
3 - 2b 1
58. Statement-1 x log x (1 - x ) 2
=9 l= and m =
b -1 a (b - 1 )
ì Eq. ( i) is defined, if
(1 - x ) 2 = 9 í \ log 54 168 =
log 168 log (2 3 ´ 3 ´ 7 )
=
î x ¹ 1, x > 0
log 54 log ( 3 3 ´ 2 )
1-x=±3 3 log 2 + log 3 + log 7
\ x = - 2 or 4 =
3 log 3 + log 2
x=4 [acceptable]
3 + log 2 3 + log 2 7 3 + l + m
\ Eq. (i) has only one solution. = =
3 log 2 3 + 1 3l + 1
Statement-1 is false.
3 - 2b 1
Statement-2 a loga b = b, where a > 0, a ¹ 1, b > 0 3+ +
b - 1 a (b - 1 )
which is true. =
3 (3 - 2b )
59. Statement-1 (log x ) 2 + log x 2 - 3 = 0 …(i) +1
b -1
2
Þ (log x ) + 2 log x - 3 = 0 (ab + 1 )
=
Þ (log x + 3 ) (log x - 1 ) = 0 a (8 - 5b )
Þ log x = - 3 or log x = 1 (ii) Q a = log 3 4 and b = log 5 3
Þ x = 10 -3 or x = 10 \ ab = log 5 4 …(i)
Eq. (i) is defined for x > 0. log 5 10 2 log 5 10
So, Eq. (i) has 2 distinct solutions. Now, log 3 10 = =
log 5 3 2 log 5 3
Statement-2 log x 2 ¹ 2 log x
log 5 (100 ) log 5 ( 4 ´ 25 )
Q LHS has domain x Î R and RHS has domain x Î ( 0, ¥ ) = =
2b 2b
\ Statement-2 is false. log 5 4 + 2 ab + 2
= = [from Eq. (i)]
60. Statement-1 2b 2b
log x 3 × log x / 9 3 = log 81 3 …(i) ln a ln b ln c
Eq. (i) holds, if x > 0, x ¹ 1, x ¹ 9 62. Q = = [by using law of proportion]
b -c c -a a -b
1 1 1
By Eq. (i), × = ln a ln b ln c
log 3 x (log 3 x + 2 ) 4 (i) Q = =
b -c c -a a -b
(log 3 x ) 2 + 2 log 3 x - 4 = 0
ln a + ln b + ln c ln (abc )
(log 3 x ) 2 + 2 log 3 x + 4 = 8 = =
b -c + c -a + a -b 0
(log 3 x + 2 ) 2 = 8
Þ ln (abc ) = 0 Þ abc = 1
log 3 x + 2 = ± 2 2 ln a ln b ln c a ln a + b ln b + c ln c
(ii) + + =
log 3 x = 2 ( - 1 ± 2 ) b - c c - a a - b a (b - c ) + b (c - a ) + c (a - b )
\ x = 32 (- 1 ± 2)
ln aa + ln bb + ln cc ln (aa × bb × cc )
= =
Two values of x satisfying Eq. (i) 0 0
a b c
So, Statement-1 is false. Þ ln (a b c ) = 0
Statement-2 Change of bases in logarithm is possible. Þ aabbcc = 1
\ Statement-2 is true.
Chap 04 Logarithms and Their Properties 349

(iii)
ln a
=
ln b
=
ln c (ii) a = 6 20
b -c c -a a -b \ log a = 20 log 6 = 20 (log 2 + log 3 )
[(b 2 + bc + c 2 ) ln a + (c 2 + ca + a 2 ) ln b = 20 (0.310 + 0.477)
+ (a 2 + ab + b 2 ) ln c ] = 20 ´ 0.778 = 15.560
=
[(b 2 + bc + c 2 ) (b - c ) + (c 2 + ca + a 2 ) (c - a ) So, number of integers in 6 20 = 15 + 1 = 16
+ (a 2 + ab + b 2 ) (a - b )] (iii) Let a = 3 - 500
2
+ bc + c 2 2 2 2 2 log a = - 500 log 3 = - 500 ´ (0.477) = - 238.5
ln ab + ln bc + ca + a + ln ca + ab + b
= = - 239 + 0.5 = 239.5
(b - c ) + (c 3 - a 3 ) + (a 3 - b 3 )
3 3

2
So, number of zeroes after the decimal in
+ bc + c 2 2
+ ca + a 2 2
+ ab + b 2
ln (ab × bc × ca ) 3 -500 = 239 - 1 = 238
=
0
2 65. Given that, log10 2 = 0.301
+ bc + c 2 2
+ ca + a 2 2
+ ab + b 2
Þ ln (ab × bc × ca )=0
and log103 = 0.477
2
+ bc + c 2 2
+ ca + a 2 2
+ ab + b 2
\ ab × bc × ca =1 log 3.375 = log(3375 ) - log 10 3 = log 5 3 ´ 3 3 - 3 log 5 ´ 2
(iv)Q AM ³ GM = 3 log 5 + 3 log 3 - 3 log 5 - 3 log 2
a+b+c = 3 (0.477) - 3 (0.301) = 3 (0.176)
\ ³ (abc )1 / 3 = (1 )1 / 3 = 1 [from Eq. (i)]
3 = 0.528
a+b+c æ1ö
\ ³1 Þ a + b + c ³3 66. Let P = log 2 x - log x (0.125) = log 2 x - log x ç ÷
3 è8ø
(v) Q AM ³ GM = log 2 x + 3 log x 2
aa + b b + c c \ AM ³ GM
Þ ³ (aa × bb × cc )1 / 3
3 log 2 x + 3 log x 2
Þ ³ (log 2 x ) (3 log x 2 ) = 3
= (1 )1/3 = 1 [from Eq. (ii)] 2
a
a +b +c b c P
Þ ³ 1 Þ aa + b b + c c ³ 3 \ ³ 3
3 2
2
+ bc + c 2 2
+ ca + a 2 2
+ ab + b 2 Þ P ³2 3
ab + bc + ca
(vi)Q AM ³ GM \ Least value of log 2 x - log x (0.125) is 2 3.
3
2
+ bc + c 2 2
+ ca + a 2 2
+ ab + b 2 1 1
³ (ab × bc × ca )1 / 3 67. Let y = + = log p 3 + log p 4
1/ 3
log 3 p log 4 p
= (1 ) [from Eq. (iii)]
= log p 12
=1 2
2
Now, 12 > p
+ bc + c 2 2
+ ca + a 2 2
+ ab + b 2
ab + bc + ca
Þ ³1 log p 12 > log p p 2 \ y > 2
3 1 + log x
b 2 + bc + c 2 c 2 + ca + a 2 2
+ ab + b 2 68. (i) \ x 10 = 10 x …(i)
Þ a +b + ca ³3
log x
1 1 Þ x×x 10 = 10 x
63. To prove log10 2 lies between and log x
3 4 Þ x [x 10 - 10 ] = 0
212 = 4096 log x
x ¹ 0, so x 10 - 10 = 0
1000 < 4096 < 10000 log
Þ x 10 x = 10
10 3 < 212 < 10 4
Taking logarithm to the base 10, Þ log10 x = log x 10
log10 10 3 < log10 212 < log10 10 4 Þ (log10 x ) 2 = 1
1 1 Þ log10 x = ± 1
3 < 12 log10 2 < 4 Þ < log10 2 <
4 3 Þ x = 10 ± 1
64. log 2 = 0.301 Þ x = 10 or
1
[Q x > 0]
log 3 = 0.477 10
(i) Let a = 5 200 (ii) log 2 (9 + 2 x ) = 3

log a = 200 log 5 = 200 (log 10 - log 2 ) = 200 (1 - 0.301) Þ 9 + 2x = 8


= 200 ´ 0.699 = 139.8 Þ 2x = - 1
200
So, number of integers in 5 = 139 + 1 = 140. which is not possible, so x Î f.
350 Textbook of Algebra

(iii) 2 × x log 4 3 + 3 log 4 x = 27 Þ 9 m (2 m + 1 ) - 4 (2 m + 1 ) = 0


éQ a log c log a ù 1 4
2 ×3 log 4 x
+3 log 4 x
= 27 b =c b \ m = - ,m =
ëê ûú 2 9
( x + 1) 1
3 log 4 = 33 m¹-
2
log 4 ( x + 1 ) = 3 4
log 4 x = 2 \ m=
9
x = 16 l -2
æ3ö æ3ö
(iv) log 4 log 3 log 2 x = 0 ...(i) ç ÷ = ç ÷ Þ l = -2
è2ø è2ø
Defined for x > 0, log 2 x > 0 and log 3 log 2 x > 0
1
Þ x > 0, x > 1, x > 3 Hence, x = 10 l = 10 -2
100
\ x >3 log10 ( x - 3 ) 1
log 3 log 2 x = 1 (viii) =
log10 ( x 2 - 21 ) 2
log 2 x = 3, x = 8
which satisfy Eq. (i). is defined for x > 1 and x 2 > 21.
log10 x + 5 \ x > 21 …(i)
x
(v) x 3 = 10 5 + log10 …(i) Þ 2 log10 ( x - 3 ) = log10 ( x 2 - 21 )
Defined for x > 0 Þ log10 ( x - 3 ) 2 = log10 ( x 2 - 21 )
Let log10 x = y
Þ ( x - 3 ) 2 = x 2 - 21
Þ x = 10 y
x 2 - 6 x + 9 = x 2 - 21
æ y + 5ö
y× ç ÷
By Eq. (i), 10 è 3 ø
= 10 5+ y \ x =5
satisfy Eq. (i), hence x = 5.
Þ y 2 + 5y = 15 + 3y
(ix) x log 2 x + 4 = 32
2
Þ y + 2y - 15 = 0
Defined for x > 0,
Þ (y + 5 ) (y - 3 ) = 0 log 2 x + 4 = log x 2 5
Þ y = - 5 or y = 3
5
1 log 2 x + 4 =
Þ x = 5 or x = 10 3 log 2 x
10
(log 2 x ) 2 + 4 log 2 x - 5 = 0
\ x = {10 -5, 10 3}
(log 2 x + 5 ) (log 2 x - 1 ) = 0
æ 1 ö
(vi) log 3 ç log 9 x + + 9 x ÷ = 2 x Þ log 2 x = - 5 or log 2 x = 1
è 2 ø
Þ x = 2 -5 or x = 21
Defined for x > 0, 1
1 \ x = or x = 2
log 9 x + + 9 x = 9 x 32
2 which satisfy Eq. (i).
1
1 - (x) loga x = x ...(i)
Þ log 9 x = - Þ x =9 2
2 and a = x log 4 x ...(ii)
Þ x = 3 -1 Defined for x > 0
1 From Eq. (i), x = ax
\ x=
3
2
a x = x , a = x 1/x
(vii) 4 log10 x + 1 - 6 log10 x - 2 × 3 log10 x + 2
=0 ...(i) 1
From Eq. (ii), xx = x log 4 x
Þ 2 2 log10 x + 2 - (2 ´ 3 ) log10 x - 2 × 3 2 log10 x + 2 = 0
1
Let log10 x = l, then Þ = log 4 x
x
2 2 l + 2 - (2 ´ 3 ) l - 2 × 3 2 l + 2 = 0
l 2l
Þ x = log x 4 Þ x x = 4
æ3ö æ3ö
Þ 22 - ç ÷ - 2 × 32 × ç ÷ =0 \ x =2
è2ø è2ø
l
(xi) log 2 sin x (1 + cos x ) = 2 …(i)
æ3ö Defined for 1 + cos x > 0, 2 sin x > 0
Let ç ÷ =m
è2ø
and 2 sin x ¹ 1, then
\ 18 m 2 + m - 4 = 0
1 + cos x = 2 sin 2 x
Þ 18 m 2 + 9 m - 8 m - 4 = 0
1 + cos x = 2 - 2 cos2 x
Chap 04 Logarithms and Their Properties 351

2 cos2 x + cos x - 1 = 0 x 2 - 2x - 3 > 0


(2 cos x - 1 ) (cos x + 1 ) = 0 (x - 3) (x + 1) > 0
1 + cos x ¹ 0 x < - 1 or x > 3
1 æ 3 ö
So, cos x = \ x Î ç - , - 1÷ …(ii)
2 è 2 ø
p
x= , Eq. (i) is defined for that value of x . Case II 2 x + 3 > 1 Þ x > - 1
3
Eq. (i), x 2 < 2 x + 3
69. Let rational number be x , then
(x - 3) (x + 1) < 0 - 1 < x < 3
x = 50 log10 x Þ 2 x = 100 × log10 x
Þ x Î ( - 1, 3 ] …(iii)
Taking logarithm to the base 10, then
Eq. (i), x¹0 …(iv)
log10 2 + log10 x = 2 + log10 (log10 x )
æ 3 ö
Let log10 x = l Eqs. (ii), (iii) and (iv), x Î ç - , 3 ÷ È { - 1, 0 }
è 2 ø
\ log10 2 + l = 2 + log10 ( l )
æ lö (ii) log 2 x ( x 2 - 5 x + 6 ) < 1 …(i)
Þ log10 ç ÷ = l - 2
è2ø For Eq. (i) to be defined 2 x > 0 and 2 x ¹ 1
which is true for l = 2. 1ü
So, x > 0 and x ¹ ï
\ log10 x = 2 2ý …(ii)
Þ x = 10 2 = 100 and x 2 - 5 x + 6 > 0 Þ x < 2 or x > 3 ïþ
2 3 1
70. Let y = + Case I 0 < 2 x < 1 Þ 0 < x < …(iii)
log 4 (2000 ) 6 log 5 (2000 ) 6 2
= 2 log ( 2000) 6 4 + 3 log ( 2000) 6 5 From Eq. (i), log 2 x (x 2 - 5x + 6) < 1
= log ( 2000) 6 4 2 + log ( 2000) 6 5 3 x 2 - 5x + 6 < 2x
= log ( 2000) 6 ( 4 2 ´ 5 3 ) x 2 - 7x + 5 > 0
1 1 (x - 6) (x - 1) > 0
= log 2000 2000 =
6 6 x < 1 or x > 6 …(iv)
71. loga [1 + logb {1 + logc (1 + log p x )}] = 0 ...(i) From Eqs. (iii), (iv) and (ii)
Þ 1 + logb {1 + logc (1 + log p x )} = 1 æ 1ö
x Î ç 0, ÷ …(A)
Þ logb {1 + logc (1 + log p x )} = 0 è 2ø
Þ 1 + logc (1 + log p x ) = 1 1
Þ logc (1 + log p x ) = 0 Case II 2 x > 1 Þ x > …(v)
2
Þ 1 + log p x = 1
From Eq. (i), log 2 x ( x 2 - 5 x + 6 ) < 1
Þ log p x = 0
Þ x = p0 Þ x 2 - 5x + 6 < 2x
Þ x =1 Þ x 2 - 7x + 6 < 0
Eq. (i) is satisfied for this value of x .
Þ 1 < x <6 …(vi)
72. Q5 log 4 2 (3 - 6 ) - 6 log 8 ( 3 - 2 )
From Eqs. (ii), (v) and (vi),
= 5 log 2 5 / 2 (3 - 6 ) - 6 log 2 3 ( 3 - 2 ) x Î (1, 2 ) È (3, 6 ) …(B)
=5 ´
1 1
log 2 (3 - 6 ) - 6 ´ log 2 ( 3 - 2 ) æ 1ö
From Eqs. (A) and (B), x Î ç 0, ÷ È (1, 2 ) È (3, 6 )
5/ 2 3 è 2ø
= log 2 (3 - 6 ) 2 - log 2 ( 3 - 2 ) 2 (iii) log 2 (2 - x ) < log1 / 2 ( x + 1 ) …(i)
2 2 From Eq. (i) to be defined 2 - x > 0 Þ x < 2
æ 3- 6 ö æ 3 ( 3 - 2)ö
= log 2 ç ÷ = log 2 ç ÷ = log 2 3 and x + 1 > 0Þx > -1
è 3 - 2ø è ( 3 - 2) ø
So, x Î ( - 1, 2 ) …(ii)
5 log 4 2 ( 3 - 6 ) - 6 log 8 ( 3 - 2 )
\ =4 Now, from Eq. (i), log 2 (2 - x ) + log 2 ( x + 1 ) < 0
=4
log 2 3
=2
2 log 2 3
=2
log 2 9
=9 (2 - x ) ( x + 1 ) < 1
2 (x - 2) (x + 1) + 1 > 0
73. (i) log 2 x + 3 x < 1 …(i)
x2 - x -2 + 1 > 0
3
Case I 0 < 2 x + 3 < 1, i.e. - < x < -1 x2 - x -1 > 0
2
Eq. (i), x 2 > 2x + 3
352 Textbook of Algebra

1- 5 (vii) 2
log10 x + 2 £ log10 x …(i)
Þ x<
2 From Eq. (i), x>0 …(ii)
1+ 5 2
log10 x + log10 x - 2 ³ 0
or x> …(iii)
2
(1010 x - 2 ) (log10 x + 1 ) ³ 0
From Eqs. (ii) and (iii),
log10 x £ - 1 or log10 x ³ 2
æ 1 - 5 ö æ1 + 5 ö 1
x Î ç - 1, ÷ Èç , 2÷ x £ or x ³ 100 …(iii)
è 2 ø è 2 ø 10
(iv) log x 2 ( x + 2 ) < 1 ...(i) From Eqs. (ii) and (iii),
From Eq. (i) to be defined, x + 2 > 0 Þ x > - 2 ü æ 1ù
ý …(A) x Î ç 0, ú È [100, ¥ )
and x Î R, x ¹ 0 and x ¹ 1 è 10 û
þ
Case I x Î ( - 1, 1 ) ~ { 0 } …(ii) or x Î ( 0, 10 -1 ] È [10 2, ¥ )
Eq. (i), (x + 2) > x 2 2
(viii) log10 ( x - 2 x - 2 ) £ 0 …(i)
2
x - x -2 < 0 From Eq. (i), x 2 - 2x - 2 > 0
2
(x - 2) (x + 1) < 0 x - 2x + 1 - 3 > 0
x -1 < x <2 …(iii) (x - 1)2 - ( 3 )2 > 0
From Eqs. (ii), (iii) and (A), [ x - (1 + 3 )] [ x - (1 - 3 )] > 0
x Î ( - 1, 0 ) È ( 0, 1 ) …(B) \ x Î ( - ¥, 1 - 3 ) È (1 + 3, ¥ ) …(ii)
Case II x Î ( - ¥, - 1 ) È (1, ¥ ) …(iv) x 2 - 2x - 2 £ 1
Eq. (i), x + 2 < x2 x 2 - 2x - 3 £ 0
x2 - x - 2 > 0 (x - 3) (x + 1) £ 0
x < - 1 or x > 2 …(v) -1 £ x £3 …(iii)
From Eqs. (iv), (v) and (A), From Eqs. (ii) and (iii), we get
x Î ( - 2, - 1 ) È (2, ¥ ) …(C) x Î [ - 1, 1 - 3 ) È (1 + 3, 3 ]
From Eqs. (B) and (C), æ 3ö
(ix) log x ç2 x - ÷ > 2 …(i)
x Î ( - 2, 1 ) È (2, ¥ ) ~ { - 1, 0 } è 4ø
log 3 x -1
< 3 log 3 ( x - 6) + 3 3
(v) 3 …(i) From Eq. (i) to be defined x > 0, x ¹ 1, 2 x - >0
4
From Eq. (i) to be defined 3
x -1 > 0 Þ x >1 …(ii) x > 0, x ¹ 1, x > …(ii)
8
and x - 6 > 0 Þ x > 6 …(iii) æ 3ö
From Eq. (i), log x ç2 x - ÷ > 2
From Eqs. (ii) and (iii), x > 6 …(iv) è 4ø
Eq. (i), x - 1 - ( x - 6 ) - 3 < 0
x -1 - x + 3 < 0 Case I 0 < x <1 …(iii)
3
x - 1 < (x - 3) x - 1 < (x - 3)2 2x - < x 2
4
x 2 + 9 - 6x - x + 1 > 0 8x - 3 - 4x 2 < 0
x 2 - 7 x + 10 > 0 4x 2 - 8x + 3 > 0
(x - 5) (x - 2) > 0 2
4x - 6x - 2x + 3 > 0
x < 2 or x > 5 …(v) (2 x - 1 ) (2 x - 3 ) > 0
From Eqs. (iv) and (v), x > 6 1 3
x < or x > …(iv)
(vi) log1/2 (3 x - 1 ) 2 < log1/2 ( x + 5 ) 2 …(i) 2 2
1 From Eqs. (ii), (iii) and (iv),
From Eq. (i) to be defined x ¹ , x ¹ - 5 …(ii)
3 æ3 1ö
x Îç , ÷ …(v)
2
Eq. (i), (3 x - 1 ) > ( x + 5 ) 2 è8 2ø
(3 x - 1 - x - 5 ) (3 x - 1 + x + 5 ) > 0 Case II x >1 ...(vi)
(2 x - 6 ) ( 4 x + 4 ) > 0 3
Eq. (i) Þ 2x - > x 2
(x - 3) (x + 1) > 0 4
8x - 3 > 4x 2
x < - 1 or x > 3 …(iii)
From Eqs. (ii) and (iii), 4x 2 - 8x + 3 < 0
x Î ( - ¥, - 5 ) È ( - 5, - 1 ) È (3, ¥ )
Chap 04 Logarithms and Their Properties 353

1 3 1
<x< …(vii) log 2 x £ - 4 or log 2 x ³ 1 Þ x £
2 2 16
From Eqs. (ii), (vi) and (vii), we get or x ³2 …(ii)
æ 3ö From Eq. (i), x>0 …(iii)
x Î ç1, ÷ …(viii)
è 2ø æ 1ù
From Eqs. (ii) and (iii), x Î ç 0, ú Î [2, ¥ )
From Eqs. (v) and (viii), we get è 16 û
æ3 1ö æ 3ö (xiii)Q ( x 2 + x + 1 ) x < 1
x Î ç , ÷ È ç1, ÷
è8 2ø è 2ø
Taking logarithm on both sides, then
(x) log1/3 x < log1/2 x ( x > 0 ) x log ( x 2 + x + 1 ) < 0
Þ log 3 x > log 2 x
Q x 2 + x + 1 > 0, " x Î R
log x log x
Þ - >0 Case I If x>0 …(i)
log 3 log 2
Then, log ( x 2 + x + 1 ) < 0
æ log 3 - log 2 ö
log x ç ÷<0 \ x2 + x + 1 < 1
è log 2 log 3 ø
Þ x (x + 1) < 0
Þ log x < 0 Þ x < 1
Þ -1 < x < 0 …(ii)
So, x Î( 0, 1 )
2 From Eqs. (i) and (ii), x Î f
(xi) log 2x + 3 x < log 2x + 3 (2 x + 3 ) …(i)
Case II If x < 0 …(iii)
From Eq. (i) to be defined, Then, log ( x 2 + x + 1 ) > 0
2x + 3 > 0
3 Þ x2 + x + 1 > 1
x>-
2 Þ x (x + 1) > 0
2x + 3 ¹ 1 \ x Î ( - ¥, - 1 ) È ( 0, ¥ ) …(iv)
x ¹ -1 From Eqs. (iii) and (iv), we get
x Î R - { 0} …(A)
2
x Î ( - ¥, - 1 )
From Eq. (i), log 2x + 3 x < 1 …(ii)
1
3 (xiv) log ( 3x 2 + 1) 2 <
Case I 0 < 2x + 3 < 1 Þ - < x < -1 …(iii) 2
2 2 < (3 x 2 + 1 )1/2
From Eq. (ii), log 2x + 3 x 2 < 1 (3 x 2 + 1 > 1, " x Î R )
Þ x 2 > 2x + 3 Þ x 2 - 2x - 3 > 0 4 < 3x 2 + 1
Þ (x - 3) (x + 1) > 0 3x 2 > 3
Þ x < - 1 or x > 3 …(iv) x2 > 1
æ 3 ö x < - 1 or x > 1
From Eqs. (A), (iii) and (iv), x Î ç - , - 1 ÷ …(B)
è 2 ø Þ x Î ( - ¥, - 1 ) È (1, ¥ )
Case II If 2 x + 3 > 1 Þ x > - 1 …(v) (log x ) 2 - 3 log x+1
(xv) x 10 10 > 1000 …(i)
log 2x + 3 x 2 < 1
From Eq. (i) to be defined, x > 0 and x ¹ 1
x 2 < 2x + 3 Let log10 x = y Þ x = 10 y
x 2 - 2x - 3 < 0 2
- 3y + 1)
From Eq. (i), 10 y ( y > 10 3
(x - 3) (x + 1) < 0
Þ y 3 - 3y 2 + y - 3 > 0
Þ -1 < x <3 …(vi)
So, Eqs. (A), (v) and (vi), x Î ( - 1, 3 ) …(C) Þ y 2 (y - 3 ) + 1 (y - 3 ) > 0
From Eqs. (B) and (C), Þ (y - 3 ) (y 2 + 1 ) > 0
æ 3 ö Þ y >3
x Î ç - , - 1 ÷ È ( - 1, 3 )
è 2 ø Þ log10 x > 3
5 Þ x > 1000
(xii) log 22 x + 3 log 2 x ³ log 4 2 16 …(i)
2 Þ x Î (1000, ¥ )
5 2 log 4 {14 + log 6( x 2 - 64 )} £ 2
log 22 x + 3 log 2 x - ´ log 2 16 ³ 0 (xvi) …(i)
2 5 2
2 14 + log 6 ( x - 64 ) £ 16
log 2 x + 3 log 2 x - 4 ³ 0
log 6 ( x 2 - 64 ) £ 2
(log 2 x + 4 ) (log 2 x - 1 ) ³ 0
354 Textbook of Algebra

x 2 - 64 £ 36 x+3
Þ £0
x
x 2 £ 100
Þ -3 £ x £ 0
- 10 £ x £ 10 …(ii)
2
Þ x Î [ - 3, 0 ] …(v)
From Eq. (i), x - 64 > 0
é 3ö
From Eqs. (ii) and (v), we get x Î ê - 3, - ÷
Þ x < - 8 or x > 8 …(iii) ë 2ø
From Eqs. (ii) and (iii), (xix) 1 + log 2 ( x - 1 ) £ log ( x - 1) 4 …(i)
x Î [ - 10, - 8 ) È (8, 10 ] From Eq. (i) to be defined, x - 1 > 0 Þ x > 1
log (3 - x )
(xvii) log 2 (9 - 2 x ) £ 10 10 ...(i) and x -1 ¹1 Þ x ¹2 …(ii)
From Eq. (i) to be defined, By Eq. (i), 1 + log 2 ( x - 1 ) £ 2 log ( x - 1) 2
9 - 2x > 0 Þ 9 > 2x Let log 2 ( x - 1 ) = l, then
2
Þ 2 x < 9 Þ x < log 2 9 1+l£
l
3 - x > 0 Þ x <3
l2 + l - 2
Then, x <3 …(ii) Þ £0
l
From Eq. (i), log 2 (9 - 2 x ) £ 3 - x (l + 2) (l - 1)
Þ £0
Þ 9 - 2x £ 23 - x l
Þ 9 - 2x - 8 × 2-x £ 0 + +
x 2 x – –2 0 – 1
Þ (2 ) - 92 + 8 ³ 0
Þ (2 x - 8 ) (2 x - 1 ) ³ 0 Þ l £ - 2 or 0 < l £ 1
Þ x
2 £ 1 or 2 ³ 8 x Þ log 2 ( x - 1 ) £ - 2
Þ x £ 0 or x ³ 3 …(iii) or 0 < log 2 ( x - 1 ) £ 1
From Eqs. (ii) and (iii), x £ 0 Þ x Î ( - ¥, 0 ] Þ x - 1 £ 2 -2 or 2 0 < x - 1 £ 2 1
æ 2x + 3 ö Þ
5
x £ or 2 < x £ 3
(xviii) loga ç ÷³0 …(i) …(iii)
è x ø 4
From inequation (a), a > 1 From Eqs. (ii) and (iii), we get
2x + 3 x Î(2, 3 ]
By Eq. (i), >0 2
x (xx) log 5x + 4 x £ log 5x + 4 (2 x + 3 ) …(i)
é æ 3ö ù 4
ê x - çè - 2 ÷ø ú From Eq. (i) to be defined, 5 x + 4 > 0 Þ x > -
5
Þ ê ú>0
ê x-0 ú 5x + 4 ¹ 1 Þ x ¹ -
3
êë úû 5
3 3
Þ x < - or x > 0 …(ii) 2x + 3 > 0 Þ x > -
2 2
æ 3ö and x Î ( - ¥, ¥ ) - { 0 }
From Eq. (i), loga ç2 + ÷ ³ 0
è xø æ 4 3ö æ 3 ö
Þ x Î ç - , - ÷ È ç - , 0 ÷ È ( 0, ¥ ) …(ii)
3 è 5 5ø è 5 ø
2 + ³1
x From Eq. (i), log 5x + 4 x 2 £ log 5x + 4 (2 x + 3 )
3+x
³0 x2
x log 5x + 4 £0 …(iii)
2x + 3
x - (- 3)
³0 Case I 0 < 5x + 4 < 1
x-0
4 3
x £ - 3 or x ³ 0 …(iii) Þ - <x<- …(iv)
5 5
From Eqs. (ii) and (iii),
x2
x £ - 3 or x > 0 …(iv) From Eq. (iii), ³1
2x + 3
Þ x Î ( - ¥, - 3 ] È ( 0, ¥ )
From inequation in (b), 0 < a < 1 x 2 - 2x - 3
³0
2x + 3 2x + 3
From Eq. (i), £1
x
Chap 04 Logarithms and Their Properties 355

(x - 3) (x + 1) 5
³0 loga x = a 2
é æ 3öù 4
ê x - ç - ÷
ë è 2 ø úû 5 2
a
3 x = a4

2 + + Case II 1 < x < a …(v)
– –1 – 3 1 1
By Eq. (iii), y + - y + = 5a
y y
æ 3 ù
x Î ç - , - 1 ú È [3, ¥ ) …(v) 2
è 2 û = 5a
y
From Eqs. (ii), (iv) and (v), x Î f …(vi)
2
Case II 5x + 4 > 1 Þ x > -
3
…(vii) y =
5 5a
2
x2 | loga x | =
From Eq. (iii), £1 5a
2x + 3
4
Þ loga x = 2
é ù 5a
ê ú
ê ( x - 3 ) ( x + 1 ) ú£0
2
x = a 4 / 5a
ê ì æ 3 öü ú
ê í x - çè - 2 ÷øý ú 75. Given equation,
ë î þ û 8ax
3 log p ( x 2 + 15a 2 ) - log p (a - 2 ) = log p …(i)
Þ x < - or x Î [ - 1, 3 ] …(viii) a -2
2
Eq. (i) is defined, if a - 2 > 0 Þa > 2 …(ii)
From Eqs. (ii), (vii) and (viii), 8ax
æ 3 3ö >0
x Î ç - , - ÷ È [ - 1, 0 ) È ( 0, 3 ] …(ix) a -2
è 5 2ø
By Eq. (ii), a >2
From Eqs. (vi) and (ix), we get
So, ax > 0, then x > 0
æ 3 3ö
x Î ç - , - ÷ È [ - 1, 0 ) È ( 0, 3 ] Eq. (i) for x = 9, a > 0
è 5 2ø
( x 2 + 15a 2 ) 8ax
74. Given equation is log p = log p
(a - 2 ) a -2
log x (ax )1/5 + loga (ax )1/5
x 2 + 15a 2 = 8ax …(iii)
1/ 5 1/ 5
æxö æa ö ( x - 3a )( x - 5a ) = 0
+ loga ç ÷ + log x ç ÷ = a …(i)
èa ø èxø \ x = 3a and x = 5a
1 For a = 3, x = 9 and x = 15
1 + log x a + 1 + loga x
5 Þ x = 15 for a = 3
1 76. Given that,
+ loga x - 1 + log x a - 1 = a
5 log 4 (log 3 x ) + log1/4 (log1/3 y ) = 0 …(i)
1 1
Þ log 2 log 3 x - log 2( - log 3 y ) = 0
1 1 2 2
loga x + +2+ loga x + - 2 = 5a
loga x loga x 1é æ log 3 x ö ù
Þ ê log 2 ç ÷ú = 0
1 1 2ë è - log 3 y ø û
| loga x | + + | loga x | - = 5a …(ii)
| loga x | | loga x | log 3 x
Þ - =1
log 3 y
Let | loga x | = y [y ³ 0 ]
Þ log 3 x = - log 3 y
1 1
y + + y - = 5a …(iii) æ1ö
y y Þ log 3 x = log 3 ç ÷
èy ø
1 1
Case I x ³ a > 1 Eq. (iii) Þy + + y - = 5a 1
y y Þ x= …(ii)
y
Þ 2y = 5a
17
Also, given that, x2 + y 2 =
2 | loga x | = 5a 4
5 1 17
| loga x | = a x2 + 2 =
2 x 4
356 Textbook of Algebra

æ 1ö
2
17 i.e., -2 < x < 4 …(ii)
çx + ÷ = +2
è xø 4 From Eq. (i),
1 5 3 1 1
x+ = [by Eq. (i) x > 0, y > 0] ´ 2 ´ log 2 |( x + 2 )| + 3 = ´ 3 log 2( 4 - x )
x 2 2 2 2
1 1 1
x + =2 + + ´ 3 log 2 ( x + 6 )
x 2 2
1 Þ log 2 ( x + 2 ) + 2 = log 2 ( 4 - x ) + log 2 ( x + 6 )
\ x = 2 or
2 Þ log 2 { 4 ( x + 2 )} = log 2 {( 4 - x ) ( x + 6 )}
1 Þ 4 (x + 2) = (4 - x ) (x + 6)
For these values of x, y = or 2 [by Eq. (ii)]
2
4 x + 8 = - x 2 - 2 x + 24
77. log 2x 4x + log 4 x 16x = 4 …(i)
1 1 x 2 + 6 x - 16 = 0
From Eq. (i) is defined for x > 0, x ¹ , x ¹ …(ii)
2 4 (x + 8) (x - 2) = 0
log 4 x log 16 x \ x = - 8, x = 2 …(iii)
Þ + =4
log 2 x log 4 x From Eqs. (ii) and (iii), we get x = 2
2 log 2 + log x 4 log 2 + log x æ 1 ö
Þ + =4 80. log 2 ( 4 x + 1 + 4) × log 2( 4 x + 1) = log1/ 2 ç ÷ …(i)
log 2 + log x 2 log 2 + log x è 8ø

On dividing by log 2, then Eq. (i) defined, for 4 x + 1 > 0 which is true for all x Î R.
2 + log 2 x 4 + log 2 x log 2 [ 4( 4 x + 1 )] × log 2( 4 x + 1 ) = log 8 =3
+ =4 2
1 + log 2 x 2 + log 2 x x x
[2 + log 2 ( 4 + 1 )] log 2 ( 4 + 1 ) = 3
Let log 2 x = l, then
Let log 2( 4 x + 1 ) = y
(2 + l ) 2 + (1 + l ) ( 4 + l ) = 4 (1 + l ) (2 + l )
\ (y + 2 ) y = 3
Þ 2 l2 + 9 l + 8 = 4 l2 + 12 l + 8 2
y + 2y - 3 = 0
Þ 2 l2 + 3 l = 0
(y + 3 ) (y - 1 ) = 0
3 y = 1 or y = - 3
\ l = 0, l = -
2 \ log 2 ( 4 x + 1 ) = 1 or log 2 ( 4 x + 1 ) = - 3
3
Þ log 2 x = 0, log 2 x = - 1
2 4 x + 1 = 2 or 4x + 1 =
8
\ x = 2 0, x = 2 - 3 / 2 1
4 x = 1 or x
4 = -1
or x = 1, x = 2 -3/2 8
78. Given equation, x 7
x = 0 or 4 = - which is not possible.
4 8
log 6 54 + log x 16 = log 2 x - log 36 …(i)
9 \ x=0
x +
Eq. (i) holds, if x > 0, x ¹ 1 81. 2 y
= 256
From Eq. (i), Þ 2 x + y
= 28
2
1 + log 6 9 + 4 log x 2 = 2 log 2 x - log 6 Þ x + y =8 …(i)
3
2 3
Þ 1 + log 6 9 + log 6 + 4 log x 2 - 2 log 2 x = 0 Also, given that, log10 xy - log10 =1 …(ii)
3 2
Þ 2 + 4 log x 2 - 2 log 2 x = 0 which is defined, xy > 0
Þ (log 2 x ) 2 - log 2 x - 2 = 0 æ 3ö
So, Eq. (ii) Þ log10 xy = log10 ç10 ´ ÷
è 2ø
Þ log 2 x = 2 or log 2 x = - 1
1 Þ xy = 15
Þ x = 4 or x =
2 Þ xy = 225 …(iii)
1 9 From Eq. (i), x + y + 2 xy = 64
Sum of the values of x satisfy Eq. (i) = 4 + =
2 2
1 x + y = 64 - 30
Product of the values of x satisfy Eq. (i) = 4 ´ = 2
2 x + y = 34
3
79. Let log 4 ( x + 2) 3 + 3 = log 4 ( 4 - x ) 3 + log 4 ( x + 6) 3 …(i) From Eq. (iii), xy = 225
2 After solving, we get x = 9 or x = 25, then y = 25 or y = 9
Eq. (i) holds, if 4 - x > 0 and x + 0 > 0, x + 2 > 0 Hence, solutions are (9, 25) and (25, 9).
Chap 04 Logarithms and Their Properties 357

82. Given, log 2 y = log 4 ( xy - 2) …(i) From Eqs. (ii), (iv) and (v),
Eq. (i) defined for y > 0 and xy - 2 > 0 …(ii) x Î ( - 5, - 4 ) È ( - 3, - 1 )
xy > 0 …(iii) 84. log 3 ( x + | x - 1 | ) = log 9 ( 4 x - 3 + 4 | x - 1 | ) …(i)
From Eqs. (ii) and (iii) Þ y > 0, x > 0 From Eq. (i) is defined, if x ³ 0
By Eq. (i), y = xy - 2 then log 3 ( x + | x - 1 | ) = log 32 ( 4 x - 3 + 4 | x - 1 | )
y 2 - xy + 2 = 0 …(iv)
Þ 2( x + | x - 1 | ) = 4 x - 3 + 4 | x - 1 |
y (x - y ) = 2 …(v)
Also given that, Þ 3 - 2 x = 2 | x - 1|
2
log 9 x + log 3 ( x - y ) = 1 …(vi) On squaring both sides, then
which is defined for x Î R - { 0 } and x - y > 0 9 + 4 x - 12 x = 4 x + 4 - 4 x
Þ x >y
Þ 8 x =5
By Eq. (vi), x( x - y ) = 3 Þ x 2 - xy = 3 …(vii)
25
and x (x - y ) = 3 ...(viii) \ x=
64
Form Eqs. (iv) and (vii), y 3 + 2 = x 2 - 3 2
æ x 5ö
2
x -y =5 2
…(ix) 85. (log 2 x ) 4 - ç log1/2 ÷ - 20 log 2 x + 148 < 0
è 4 ø
On dividing Eq. (v) by Eq. (viii),
y 2 2x From Eq. (i), x > 0
= Þ y = …(x)
x 3 3 Þ (log 2 x ) 4 - (5 log 2 x - 2 ) 2 - 20 log 2 x + 148 < 0 ...(i)
From Eqs. (ix) and (x),
(log 2 x ) 4 - 25 log 22 x - 4 + 20 log x x - 20 log 2 x + 148 < 0
x = 3 and y = 2
83. Given that, (log 2 x ) 4 - 25 log 22 x + 144 < 0
9 {(log 2 x ) 2 - 16 } {(log 2 x ) 2 - 9 } < 0
2 log1/4 ( x + 5 ) > log1/3 3 9 + log x + 5 2 …(i)
4
9 < (log 2 x ) 2 < 16
By Eq. (i), x + 5 > 0 Þ x > - 5
x + 5 ¹1 Þ x ¹ - 4 3 < log 2 x < 4 or - 4 < log 2 x < - 3
So, x Î ( - 5, - 4 ) È ( - 4, ¥ ) …(ii) 8 < x < 16 …(ii)
Now, by Eq. (i) 1 1
or <x< …(iii)
2 9 æ - 2ö 16 8
log 2 ( x + 5 ) - ´ ç ÷ log 3 9 - 2 log x + 5 2 > 0
-2 4 è 3 ø According to the question in Eq. (i) holds, for x Î(a, b )
- log 2 ( x + 5 ) + 3 - 2 log x + 5 2 > 0 where a, b Î N
2 So, from Eq. (ii), a = 8, b = 16
- log 2 ( x + 5 ) - +3>0 …(iii)
log 2 ( x + 5 ) \ ab (a + b ) = 8 ´ 16 (8 + 16 )
= 144 ´ 24 = 3456
Now, let log 2 ( x + 5 ) = y , then Eq. (iii) becomes
2 86. (log 3 3 3 x ) + (log x 3 3 x ) log 3 x 3
-y - + 3 > 0
y æ x 3ö
2 + ç log 3 3 + log x 3 ÷ log 3 x 3 = 2 …(i)
- y + 3y - 2 è 3 xø
Þ >0
y
y 2 - 3y + 2 Eq. (i) is defined for x > 0, x ¹ 1
Þ <0
y 1
From Eq. (i), (1 + log 3 x + 1 + log x 3 ) 3 log 3 x
(y - 2 ) (y - 1 ) 3
Þ <0
y 1
+ (log 3 x - 1 + log x 3 - 1 ) 3 log 3 x
+ + 3
– 0 1 – 2 æ 1 ö æ 1 ö
= 2 ç log 3 x + + 2÷ + ç log 3 x + - 2÷
è log 3 x ø è log 3 x ø
Þ y < 0 or 1 < y < 2
= 2 | log x 3 |
Þ log 2 ( x + 5 ) < 0 or 1 < log 2 ( x + 5 ) < 2
Þ x + 5 < 1 or 2 < x + 5 < 4 1 1
Þ | log 3 x | + + | log 3 x | - = 2 | log x 3 |
Þ x<-4 …(iv) | log 3 x | | log 3 x |
or -3 < x < -1 …(v) …(ii)
358 Textbook of Algebra

1 Taking log with base e on both sides, then


Case I If x ³ 3, log 3 x - >0
log 3 x ln x × ln 3 = ln y × ln 2 …(ii)
2 log 3 x = 2 log x 3 From Eqs. (i) and (ii), we get
æ ln x × ln 3 ö
1 ln 2 (ln 2 + ln x ) = ln 3 ç ln 3 + ÷
(log 3 x ) 2 = 1 Þ x = 3 or , so x = 3 è ln 2 ø
3
2 æ (ln 3 ) 2 ö
Case II If 1 < x < 3, = 2 log x 3 Þ ln x ç - ln 2 ÷ = - ((ln 3 ) 2 - (ln 2 ) 2 )
log 3 x è ln 2 ø
Þ log 3 x × log x 3 = 1 æ1ö
\ ln x = - ln 2 = ln ç ÷
Þ 1 =1 è2ø
which is true, for all x Î(1, 3 ]. 1
Þ x=
So, x Î(1, 3 ] 2
87. P = Number of natural numbers, whose logarithms to the base 1
\ x0 =
10 have characteristic p. 2
Let ‘x ’ represent the natural number, i.e. 1 1 1 1
x = l ´ 10 p [ l = a1 × a 2 × a 3 ¼] 90. Let S = 4- 4- 4- ¼¥
3 2 3 2 3 2 3 2
So, P = Number of natural numbers which have ( p + 1 ) digits
1
= 9 × 10 p - 1 + 1 = 9 × 10 p Þ S= 4 -S
3 2
Q = Number of natural numbers which have (q ) digits. or (3 2 S ) 2 = 4 - S
Q = 9 × 10q - 1 - 1 + 1 = 9 × 10q - 1
Þ 18 S 2 + S - 4 = 0
p q -1
So, log10 P - log10 Q = log10(9 × 10 ) - log10(9 × 10 )
Þ (9 S - 4 ) (2 S + 1 ) = 0
= (log10 9 + p ) - (log10 9 + q - 1 ) \ 9S - 4 = 0 [Q2S + 1 ¹ 0]
= p -q + 1 -2
4 æ3ö
88. Q a = log 3 log 3 2 or S= =ç ÷
9 è2ø
Þ 3a = log 3 2
Þ log 3/2 S = - 2 Þ6 + log 3/2 S = 6 - 2 = 4
\ 3 -a = log 2 3 Hence,
-a
Now, 1 < 2 -k + 3 < 21 æ 1 1 1 1 ö
6 + log 3/2 ç 4- 4- 4- ¼÷ = 4
-a ç3 2 3 2 3 2 3 2 ÷
Þ 2 0 < 2 -k + 3 < 21 è ø
\ 0 < -k + 3 -a < 1 91. (3 / 4) x = 1 / 4
Þ 0 < - k + log 2 3 < 1 Taking log with base 2
Þ 0 > k - log 2 3 > - 1 Þ x (log 2 3 - 2 ) = -2
Þ log 2 3 - 1 < k < log 2 3 2 1
\ x= = Þ (b, c)
\ k =1 2 - log 2 3 1 - log 4 3
89. Q(2x ) ln 2 = (3y ) ln 3 and taking log with base 3
Taking log with base e on both sides, then Þ x (1 - log 3 4 ) = -2 log 3 2
ln 2 ( ln 2 + ln x ) = ln 3 (ln 3 + ln y ) …(i) 2 log 3 2
\ x=
2 log 3 2 - 1
and 3 ln x = 2 ln y
CHAPTER

05
Permutations and
Combinations
Learning Part
Session 1
● Fundamental Principle of Counting

● Factorial Notation

Session 2
● Divisibility Test

● Principle of Inclusion and Exclusion

● Permutation

Session 3
● Number of Permutations Under Certain Conditions

● Circular Permutations

● Restricted Circular Permutations

Session 4
● Combination

● Restricted Combinations

Session 5
● Combinations from Identical Objects

Session 6
● Arrangement in Groups

● Multinomial Theorem

● Multiplying Synthetically

Session 7
● Rank in a Dictionary

● Gap Method [when particular objects are never together]

Practice Part
● JEE Type Examples
● Chapter Exercises

Arihant on Your Mobile !


Exercises with the #L
symbol can be practised on your mobile. See inside cover page to activate for free.
360 Textbook of Algebra

In everyday life, we need to know about the number of ways of doing certain work from given number of available
options. For example, Three persons A,B and C are applying for a job in which only one post is vacant. Clearly, vacant
post can be filled either by A or B or C i.e., total number of ways doing this work is three.
Again, let two persons A and B are to be seated in a row, then only two possible ways of arrangement is AB or BA.In
two arrangements, persons are same but their order is different. Thus, in arranging things, order of things is important.

Session 1
Fundamental Principle of Counting, Factorial Notation

Fundamental Principle (ii) Addition Principle


of Counting If an operation can be performed in ‘m’ different ways and
another operation, which is independent of the first
operation, can be performed in ‘n’ different ways. Then,
(i) Multiplication Principle either of the two operations can be performed in (m + n )
If an operation can be performed in ‘m’ different ways, ways. This can be extended to any finite number of
following which a second operation can be performed in ‘n’ mutually exclusive operation.
different ways, then the two operations in succession can be Note For OR → ‘+’ (Addition)
performed inm × n ways. This can be extended to any finite
y Example 3. There are 25 students in a class in which
number of operations.
Note For AND → ‘×’ (multiply)
15 boys and 10 girls. The class teacher select either a
boy or a girl for monitor of the class. In how many
y Example 1. A hall has 12 gates. In how many ways ways the class teacher can make this selection?
can a man enter the hall through one gate and come Sol. Since, there are 15 ways to select a boy, so there are 10
out through a different gate? ways to select a girl.
Sol. Since, there are 12 ways of entering into the hall. After Hence, by the fundamental principle of addition, either a
entering into the hall, the man come out through a differ- boy or a girl can be performed in 15 + 10 = 25 ways.
ent gate in 11 ways.
Hence, by the fundamental principle of multiplication, total y Example 4. There are 4 students for Physics, 6
number of ways is 12 × 11 = 132 ways. students for Chemistry and 7 students for Mathematics
gold medal. In how many ways one of these gold
y Example 2. There are three stations A, B and C, five medals be awarded?
routes for going from station A to station B and four Sol. The Physics, Chemistry and Mathematics student’s gold
routes for going from station B to station C. Find the medal can be awarded in 4, 6 and 7 ways, respectively.
number of different ways through which a person can Hence, by the fundamental principle of addition, number
go from A to C via B. ways of awarding one of the three gold medals.
Sol. Since, there are five routes for going from A to B. So, = 4 + 6 + 7 = 17 ways.
there are four routes for going from B to C.

A
B
C
Factorial Notation
Hence, by the fundamental principle of multiplication, total Let n be a positive integer. Then, the continued product of
number of different ways first ‘n’ natural numbers is called factorial n, to be denoted
=5× 4 [i.e., A to B and then B to C ] by n ! or n i.e., n ! = n(n − 1)(n − 2 ) K 3 ⋅ 2 ⋅ 1
= 20 ways Note When n is negative or a fraction, n! is not defined.
Chap 05 Permutations and Combinations 361

Some Important Properties Exponent of prime p in n!


(i) n ! = n (n − 1) ! = n (n − 1)(n − 2 ) ! Exponent of prime p in n ! is denoted by E p (n !), where p is
(ii) 0 ! = 1 ! = 1 prime number and n is a natural number. The last integer
n 
(iii) (2n ) ! = 2 n n ![1 ⋅ 3 ⋅ 5 K(2n − 1)] amongst 1, 2, 3, …, (n − 1), n which is divisible by p is   p,
p
n!
(iv) = n(n − 1)(n − 2 ) K (r + 1) [r < n ] where [ ⋅ ] denotes the greatest integer function.
r! ∴ E p (n !) = E p (1 ⋅ 2 ⋅ 3 K(n − 1) ⋅ n )
n! 
(v) = n(n − 1) (n − 2 ) K (n − r + 1) [r < n ] n  
(n − r ) ! = E p  p ⋅ 2 p ⋅ 3 pK(n − 1) p ⋅   p 
  p  
1 1 λ 
(vi) + = , then λ = (n + 2 ) 2
n ! (n + 1) ! (n + 2 ) ! [because the remaining natural numbers from 1 to n are
(vii) If x ! = y ! ⇒ x = y or x = 0, y = 1 not divisible by p]
or x = 1, y = 0 n    n 
=   + E p 1 ⋅ 2 ⋅ 3 K   …(i)
p   p 
y Example 5. Find n, if (n + 2)! = 60 × (n − 1)!.
n 
Sol. Q(n + 2)! = (n + 2)(n + 1)n (n − 1)! Now, the last integer amongs 1, 2, 3, …,   which is
p
(n + 2)!
⇒ = (n + 2)(n + 1)n n 
(n − 1)! p  n 
⇒ 60 = (n + 2)(n + 1)n [given] divisible by p is   =  2 . Now, from Eq. (i), we get
⇒ 5 × 4 × 3 = ( n + 2) × ( n + 1) × n
 p   p 
 
∴ n =3
n   n  
E p (n !) =   + E p  p, 2 p, 3 p, K,  2  p 
n
y Example 6. Evaluate ∑r × r! . p   p  
r =1
n 
n n n [because the remaining natural numbers from 1 to   are
Sol. We have, ∑r × r ! = ∑ {(r + 1) − 1}r ! = ∑(r + 1)! − r ! not divisible by p]
p
r =1 r =1 r =1

= (n + 1)! − 1! n   n    n 
∴ E p (n !) =   +  2  + E p  1 ⋅ 2 ⋅ 3 K  2  
[ put r = n in (r + 1)! and r = 1 is r !]  p   p    p  
= (n + 1)! − 1
Similarly, we get
n
n   n   n  n 
y Example 7. Find the remainder when ∑ r ! is divided E p (n !) =   +  2  +  3  + K +  s 
r =1  p   p   p   p 
by 15, if n ≥ 5.
Sol. Let where, s is the largest natural number such that
n
N = ∑ r ! = 1! + 2! + 3! + 4 ! + 5! + 6! + 7 ! + K + n ! ps ≤ n < ps + 1
r =1 Note Number of zeroes at the end of n! = E5 ( n!).
= (1! + 2! + 3! + 4 !) + (5! + 6! + 7 ! +K + n !)
= 33 + (5! + 6! + 7 ! + K+n !) y Example 8. Find the exponent of 3 in 100! .
N 33 (5! + 6! + 7 ! + K+ n !) Sol. In terms of prime factors 100! can be written as 2a ⋅ 3b ⋅ 5c ⋅ 7d K
⇒ = +
15 15 15 Now, b = E 3 (100!)
3
=2+ + Integer [as 5!, 6!, K are divisible by 15] 100 100 100 100
15 =   +  4  +  3  +  4  +K
 3  3  3  3 
3
= + Integer = 33 + 11 + 3 + 1 + 0 + ... = 48
15
Hence, exponent of 3 is 48.
Hence, remainder is 3.
362 Textbook of Algebra

Aliter ∴ 100! = 297 ⋅ 3b ⋅ 524 ⋅ 7d K = 273 ⋅ 3b ⋅ (2 × 5)24 ⋅ 7d K


Q 100! = 1 × 2 × 3 × 4 × 5 × K × 98 × 99 × 100 = 273 ⋅ 3b ⋅ (10)24 ⋅ 7d K
= (1 × 2 × 4 × 5 × 7 × K × 98 × 100) Hence, number of zeroes at the end of 100! is 24.
(3 × 6 × 9 × K × 96 × 99 )
or Exponent of 10 in 100! = min (97, 24) = 24.
= k × 3 (1 × 2 × 3 × K × 32 × 33)
33
Aliter
= [k (1 × 2 × 4 × 5 × K × 31 × 32)] Number of zeroes at the end of 100!
(3 × 6 × 9 × K × 30 × 33) 100 100
= 333 k1 × 311(1 × 2 × 3 × K × 10 × 11) = E 5 (100!) =   +  2  + K
 5  5 
= 344 [k1 (1 × 2 × 4 × 5 × K × 10 × 11)](3 × 6 × 9 ) = 20 + 4 + 0 + K = 24
= k 2 × 344 × 34 × 2 = k 3 × 348
y Example 11. For how many positive integral values of
Hence, exponent of 3 in 100! is 48. n does n! end with precisely 25 zeroes?
y Example 9. Prove that 33! is divisible by 219 and Sol. Q Number of zeroes at the end of n ! = 25 [given]
what is the largest integer n such that 33! is divisible ⇒ E 5 ( x !) = 25
by 2n ? n   n   n 
⇒  5  + 25 + 125 + K = 25
Sol. In terms of prime factors, 33! can be written as
2a ⋅ 3b ⋅ 5c ⋅ 7d ⋅ K
It’s easy to see that n = 105 is the smallest satisfactory value
33 33 33 33 33 of n. The next four values of n will also work (i.e., n = 106,
Now, E 2 (33!) =   +  2  +  3  +  4  +  5  + K
 2  2  2  2  2  107, 108, 109). Hence, the answer is 5.
= 16 + 8 + 4 + 2 + 1 + 0 + K
y Example 12. Find the exponent of 80 in 180!.
= 31
Sol. Q 80 = 24 × 5
Hence, the exponent of 2 in 33! is 31. Now, 33! is divisible
by 231 which is also divisible by 219 . 180 180 180 180
∴ E 2 (180!) =   +  2  +  3  +  4 
∴ Largest value of n is 31.  2  2  2  2 
180 180 180
y Example 10. Find the number of zeroes at the end +  5  +  6  +  7  +K
2  2  2 
of 100!.
= 90 + 45 + 22 + 11 + 5 + 2 + 1 = 176
Sol. In terms of prime factors, 100! can be written as
180 180 180
2a ⋅ 3b ⋅ 5c ⋅ 7d K and E 5 (180!) =   +  2  +  3  + K
 5  5  5 
100  100  100  100  100  100 
Now, E 2(100 !) =  + + + + + = 36 + 7 + 1 + 0 + K
 2   2 2   2 3   2 4   2 5   2 6 
= 44
= 50 + 25 + 12 + 6 + 3 + 1 = 97 176 
Now, exponent of 16 in 180! is  = 44, where [ ⋅ ] denotes the
100 100
and E 5 (100!) =   +  2   4 
 5  5  greatest integer function. Hence, the exponent of 80 in 180! is 44.
= 20 + 4 = 24
Chap 05 Permutations and Combinations 363

#L Exercise for Session 1


1. There are three routes: air, rail and road for going from Chennai to Hyderabad. But from Hyderabad to
Vikarabad, there are two routes, rail and road. The number of routes from Chennai to Vikarabad via
Hyderabad is
(a) 4 (b) 5 (c) 6 (d) 7

2. There are 6 books on Mathematics, 4 books on Physics and 5 books on Chemistry in a book shop. The
number of ways can a student purchase either a book on Mathematics or a book on Chemistry, is
(a) 10 (b) 11 (c) 9 (d) 15
1 1 λ
3. If a, b and c are three consecutive positive integers such that a < b < c and + = , the value of λ is
a! b ! c!
(a) a (b) b (c) c (d) a + b + c

4. If n!, 3 × n ! and (n + 1)! are in GP, then n ! , 5 × n ! and (n + 1)! are in


(a) AP (b) GP (c) HP (d) AGP
n
5. Sum of the series ∑ (r 2 + 1) r! is
r =1

(a) (n + 1) ! (b) (n + 2) ! − 1 (c) n ⋅ (n + 1) ! (d) n ⋅ (n + 2) !


α β γ δ θ φ
6. If 15 ! = 2 ⋅ 3 ⋅ 5 ⋅ 7 ⋅ 11 ⋅ 13 , the value of α − β + γ − δ + θ − φ is
(a) 4 (b) 6 (c) 8 (d) 10

7. The number of naughts standing at the end of 125! is


(a) 29 (b) 30 (c) 31 (d) 32

8. The exponent of 12 in 100! is


(a) 24 (b) 25 (c) 47 (d) 48

9. The number 24! is divisible by


(a) 6 24 (b) 24 6 (c) 1212 (d) 48 5

10. The last non-zero digit in 20! is


(a) 2 (b) 4 (c) 6 (d) 8

11. The number of prime numbers among the numbers 105 ! + 2,105 ! + 3,105 ! + 4, …, 105 ! + 104
and 105 ! + 105 is
(a) 31 (b) 32 (c) 33 (d) None of these
Session 2
Divisibility Test, Principle of Inclusion and Exclusion,
Permutation
Divisibility Test Principle of Inclusion
In decimal system all numbers are formed by the digits 0, and Exclusion
1, 2, 3, …, 9. If a b c d e is a five-digit number in decimal
1. If A and B are finite sets, from the Venn diagram (i), it
system, then we can write that.
is clear that
a b c d e = 10 4 ⋅ a + 10 3 ⋅ b + 10 2 ⋅ c + 10 ⋅ d + e. U
Number a b c d e will be divisible
A B
(1) by 2, if e is divisible by 2.
(2) by 4, if 2d + e is divisible by 4. (i)
(3) by 8, if 4c + 2d + e is divisible by 8.
n (A ∪ B ) = n (A) + n (B ) − n (A ∩ B )
(4) by 2t , if number formed by last t digits is divisible by 2 t .
and n ( A ′ ∩ B ′ ) = n (U ) − n ( A ∪ B )
For example, Number 820101280 is divisible by 2 5
2. If A , B and C are three finite sets, then from the Venn
because 01280 is divisible by 2 5 . diagram (ii), it is clear that
(5) by 5, if e = 0 or 5.
U
(6) by 5t , if number formed by last t digits is
B
divisible by 5 t . A

For example, Number 1128375 is divisible by 5 3


because 375 is divisible by 5 3 .
C
(7) by 3, if a + b + c + d + e (sum of digits) is divisible
by 3. (ii)
(8) by 9, if a + b + c + d + e is divisible by 9. n ( A ∪ B ∪ C ) = n ( A ) + n ( B ) + n (C ) − n ( A ∩ B )
(9) by 6, if e = even and a + b + c + d + e is divisible by 3. − n ( B ∩ C ) −n (C ∩ A ) + n ( A ∩ B ∩ C )
(10) by 18, if e = even and a + b + c + d + e is divisible and n ( A ′ ∩ B ′ ∩ C ′ ) = n (U ) − n ( A ∪ B ∪ C )
by 9.
Note If A1,A2,A 3, …, An are finite sets, then
(11) by 7, if abcd − 2e is divisible by 7.
n ( A1 ∪ A2 ∪ ... ∪ An ) = Σn( Ai ) − Σn( Ai ∩ Aj ) + Σn( Ai ∩ Aj ∩ Ak ) −
For example, Number 6552 is divisible by 7 because ... + ( −1) n Σn( A1 ∩ A2 ∩ ... ∩ An )
655 − 2 × 2 = 651 = 93 × 7 is divisible by 7. and n( A′1 ∩ A′ 2 ∩ ... ∩ A′ n ) = n( U ) − n( A1 ∪ A2∪ ... ∪ An ).
(12) by 11, if a1
4+2+3e
c4 − +d
b{
Sum of digits at
y Example 13. Find the number of positive integers
Sum of digits
odd places at even places from 1 to 1000, which are divisible by atleast 2, 3 or 5.
is divisible by 11. Sol. Let Ak be the set of positive integers from 1 to 1000,
For example, Number 15222163 is divisible by 11 which is divisible by k. Obviously, we have to find
because n ( A 2 ∪ A 3 ∪ A 5 ). If [ ⋅ ] denotes the greatest integer
function, then
(1 + 2 + 2 + 6 ) − (5 + 2 + 1 + 3 ) = 0 is divisible by 11. 1000
n( A 2 ) =  = [500] = 500
(13) by 13, if abcd + 4e is divisible by 13.  2 
For example, Number 1638 is divisible by 13 because 1000
n( A 3 ) =  = [333.33] = 333
163 + 4 × 8 = 195 = 15 × 13 is divisible by 13.  3 
Chap 05 Permutations and Combinations 365

1000
n( A 5 ) =  = [200] = 200 Note
 5  (i) The number of permutations of n different things taken all at
1000 a time = n Pn = n!
n( A 2 ∩ A 3 ) =  = [16667
. ] = 166
 6  P0 = 1 , n P1 = n and n Pn − 1 = n Pn = n!
n
(ii)
1000 n
Pr = n( n − 1 Pr = n( n − 1)( n − 2 Pr
= [6667
. ] = 66
(iii) − 1) − 2)
n( A 3 ∩ A 5 ) = 
 15  = n( n − 1)( n − 2)( n − 3 Pr − 3) = K
1000
n( A 2 ∩ A 5 ) =  = [100] = 100 n −1
Pr = ( n − r ) n −1
 10 
(iv) Pr −1
n
1000 Pr
and n ( A 2 ∩ A 3 ∩ A 5 ) =  = [33.33] = 33 (v) = ( n − r + 1)
 30 
n
Pr − 1

From Principle of Inclusion and Exclusion


y Example 14. If 56
Pr + 6 : 54
Pr + 3 = 30800 : 1, find r P2 .
n( A 2 ∪ A 3 ∪ A 5 ) = n( A 2 ) + n( A 3 ) + n( A 5 )
− n (A2 ∩ A3 ) 56
Pr +6 30800
Sol. We have, =
−n ( A 3 ∩ A 5 ) − n ( A 2 ∩ A 5 ) + n ( A 2 ∩ A 3 ∩ A 5 ) 54
Pr +3 1
= 500 + 333 + 200 − 166 − 66 − 100 + 33 = 734 54
Pr +4 30800
Hence, the number of positive integers from 1 to 1000, ⇒ (56)(55) 54 = [from note (iii)]
which are divisible by atleast 2, 3 or 5 is 734. Pr +3 1
Note 54
Pr +4
The number of positive integers from 1 to 1000, which are not ⇒ 54
= 10
divisible by 2, 3 or 5 is n ( A′2 ∩ A′3 ∩ A′3 ). Pr +3
∴n( A′ 2 ∩ A′ 3 ∩ A′ 5 ) = n( U ) − n( A2 ∪ A3 ∪ A5 ) [here , n( U ) = 1000] ⇒ 54 − (r + 4 ) + 1 = 10 [from note (v)]
= 1000 − 734 = 266
r = 41
∴ r
P2 = P2 = 41 ⋅ 40 = 1640
41

Permutation 11 (n − 1) n + 3
n+5
Each of the different arrangements which can be made by y Example 15. If Pn + 1 = ⋅ Pn , find n.
taking some or all of a number of things is called a 2
n+5
permutation. In permutation, order of the arrangement is Pn + 1 11(n − 1)
Sol. We have, =
important. n+3
Pn 2

Important Results (n + 5)(n + 4 ) ⋅n + 3 Pn − 1 11(n − 1)


⇒ n+3
= [from note (iii)]
1. The number of permutations of n different Pn 2
things, taking r at a time is denoted by n Pr or (n + 5)(n + 4 ) 11(n − 1)
= [from note (v)]
P (n , r ) or A (n , r ), then ( n + 3 − n + 1) 2
n
Pr = n (n − 1 )(n − 2) K (n − r + 1 ) ⇒ (n + 5)(n + 4 ) = 22(n − 1)
n! ⇒ n 2 − 13n + 42 = 0
=
(n − r ) !
⇒ (n − 6)(n − 7 ) = 0
where, n ∈ N , r ∈W and 0 ≤ r ≤ n. ∴ n = 6, 7
Proof LHS = n Pr = Number of ways of filling up r
y Example 16. If m + n P2 = 90 and m −n
P2 = 30, find the
vacant places simultaneously from n different things
values of m and n.
1 2 3 r m +n
Sol. Q P2 = 90 = 10 × 9 = 10
P2
n ways (n–1) ways (n–2) ways (n–r+1) ways ∴ m + n = 10 ...(i)
m −n
= n (n − 1) (n − 2 )...(n − r + 1) and P2 = 30 = 6 × 5 = P2 6

n (n − 1)(n − 2 )...(n − r + 1) × (n − r ) ∴ m −n =6 ...(ii)


=
(n − r ) ! From Eqs. (i) and (ii), we get
n!
= = RHS m = 8 and n = 2.
(n − r ) !
366 Textbook of Algebra

y Example 17. Find the value of r, if Note Total number of letters in English alphabet = 26
(i) Number of vowels = 5
(i) 11 Pr = 990
i.e., A, E, I, O, U [strong vowels]
(ii) 8
P5 + 5 ⋅ 8 P4 = 9 Pr (ii) W and Y an half vowels. [weak vowels]
(iii) 22
Pr + 1 : 20
Pr + 2 = 11 : 52 (iii) Number of consonants = 21 [except vowels]
i.e., B, C, D, F, G, ..., Y, Z
Sol. (i) Q 11 Pr = 990 = 11 × 10 × 9 = 11P3 (iv) Words which contains all vowels are
∴ r =3 EDUCATION, EQUATION, ...
(ii) Q 8
P5 + 5 ⋅ 8 P4 = 9 Pr (iv) Words which do not contains any vowels are
SKY, FLY, TRY, ...
 8P 
⇒ 8
P4  8 5 + 5 = 9 Pr
 P4  y Example 20. How many different signals can be
given using any number of flags from 4 flags of
⇒ 8
P4 (8 − 5 + 1 + 5) = 9 Pr [from note (v)]
different colours?
⇒ 9 ⋅ P4 = Pr
8 9
Sol. The signals can be made by using one or more flags at a
⇒ 9
P5 = 9 Pr [from note (iii)] time. Hence, by the fundamental principle of addition, the
total number of signals
∴ r =5
= 4 P1 + 4 P2 + 4 P3 + 4 P4
(iii) Q 22
Pr +1 :
20
Pr +2 = 11 : 52
= 4 + ( 4 × 3) + ( 4 × 3 × 2) + ( 4 × 3 × 2 × 1)
22
Pr +1 11 = 4 + 12 + 24 + 24 = 64
⇒ 20
=
Pr +2 52
y Example 21. Find the total number of 9-digit
22 ⋅ 21 ⋅ 20
Pr −1 11 numbers which have all different digits.
⇒ =
(19 − r ) ⋅ (20 − r ) ⋅ (21 − r ) ⋅ 20
Pr −1 52 Sol. Number of digits are 10 (0, 1, 2, 3, 4, 5, 6, 7, 8, 9)
[from note (iii) and (iv)] Total number of 9-digit numbers = 10
P9
⇒ (21 − r ) ⋅ (20 − r ) ⋅ (19 − r ) = 52 × 2 × 21 Out of these, the number of numbers having zero at the
= 14 × 13 × 12 first place = 9 P8
∴ r =7 Hence, required number of numbers = 10
P9 − 9 P8

y Example 18. Prove that = 10 × 9 P8 − 9 P8 = 9 × 9 P8


n +1
1
P1 + 2 ⋅ 2P2 + 3 ⋅ 3P3 + ... + n ⋅ nPn = Pn + 1 − 1. =9 ×
9!
= 9 × 9!
1!
Sol. LHS = 1P1 + 2 ⋅ 2 P2 + 3 ⋅ 3 P3 + ... + n ⋅ n Pn
Note Total number of n digit numbers ( 1 ≤ n ≤ 10 ) ,
n n
which have all different digits = Pn − 9 Pn − 1
∑r ⋅ ∑ {(r + 1) − 1)} ⋅
10
= r
Pr = r
Pr
r =1 r =1
n y Example 22. A 5-digit number divisible by 3 is to be
= ∑ {(r + 1) ⋅ r Pr − r Pr )} formed using the numbers 0, 1, 2, 3, 4 and 5 without
r =1
repetition. Find total number of ways in which this can
n
be done.
= ∑(r + 1 Pr + 1 − r Pr ) [from note (iii)]
Sol. A number will be divisible by 3, if sum of the digits in
r =1
n +1 n +1
number be divisible by 3.
= Pn + 1 − 1P1 = Pn + 1 − 1
Here, 0 + 1 + 2 + 3 + 4 + 5 = 15 , which is divisible by 3.
= RHS Therefore, the digit that can be left out, while the sum still
is multiple of 3, is either 0 or 3.
y Example 19. Determine the number of permutations If 0 left out
of the letters of the word ‘SIMPLETON’ taken all at Then, possible numbers = 5 P5 = 5! = 120
a time.
If 3 left out
Sol. There are 9 letters in the word ‘SIMPLETON’ and all the
9 letters are different. Hence, the number of permutations Then, possible numbers = 5 P5 − 4 P4 = 5! − 4 ! = 120 − 24 = 96
taking all the letters at a time Hence, required total numbers = 120 + 96 = 216
= 9 P9 = 9 ! = 362880
Chap 05 Permutations and Combinations 367

y Example 23. A 5-digit number is formed by the digits Proof Let the required number of permutations be x .
1, 2, 3, 4, 5 without repetition. Find the number of the Since, p different things can be arranged among
numbers, thus formed divisible by 4. themselves in p ! ways, therefore if we replace p
Sol. Let a 5-digit number be abcde. identical things by p different things, which are also
different from the rest of things, the number of
It will be divisible by 4, if 2d + e is divisible by 4.
permutations will become x × p !
⇒ 2d + e is divisible by 4 ∴ e must be even.
123 Again, if we replace q identical things by q different
Even
things, the number of permutations will become
 e
⇒ 2 d +  is divisible by 4 ( x × p !) × q !
 2
1424 3 Again, if we replace r identical things by r different
Should be even
things, the number of permutations will become
Then, e = 2, d = 1, 3, 5 ( x × p ! × q !) × r ! . Now, all the n things are different
 Total four cases
and e = 4, d = 2  and therefore, number of permutations should be n ! .
∴ Required number of ways = 4 × {
3! = 24 Thus, x × p ! × q ! × r ! = n !
Number of ways n!
∴ x=
filling abc after filling de. p!q!r !
Aliter A number will be divisible by 4, if the last two digits
Remark
of the number is divisible by 4, then for divisible by 4, last
The above theorem can be extended further i.e. if there are
two digits 12 or 24 or 32 or 52
n things taken all at a time, p1 are alike of one kind, p2 are alike of
second kind,p3 are alike of third kind, ..., pr are alike of rth kind
such that p1 + p2 + p3 + K + pr = n, the number of permutations
3! ways 4 ways n!
of these n things is .
p1 ! p2 ! p3 ! ... pr !
Hence, the number formed is divisible by 4 = 3! × 4 = 24.

y Example 24. Find the number of permutations of y Example 25. How many words can be formed with
letters ab c d e f g taken all together if neither ‘beg’ the letters of the word ‘ARIHANT’ by rearranging them?
Sol. Here, total letters 7, in which 2A’s but the rest are
nor ‘cad’ pattern appear. 7!
Sol. The total number of permutations without any restriction is 7! different. Hence, the number of words formed = = 2520
2!
n (U ) = 7 ! = 5040
Let n ( A ) be the number of permutations in which ‘beg’ y Example 26. Find the number of permutations of the
pattern always appears letters of the words ‘DADDY DID A DEADLY DEED’.
b e gac d f Sol. Here, total letters 19, in which 9D ’s, 3A’s, 2Y’s, 3E’s and
i.e., n ( A ) = 5! = 120 rest occur only once.
19 !
and let n ( B ) be the number of permutations in which ‘cad’ ∴ Required number of permutations =
pattern always appears 9 ! × 3! × 2! × 3!
c a db e f g
i.e., n ( B ) = 5! = 120 y Example 27. How many words can be formed with
Now, n ( A ∩ B ) = Number of permutations in which ‘beg’ the letters of the words
and ‘cad’ pattern appear (i) HIGH SCHOOL and
begcad f (ii) INTERMEDIATE by rearranging them?
i.e., n ( A ∩ B ) = 3! = 6 Sol. (i) Here, total letters are 10, in which 3H’s and 2O’s, but
Hence, the number of permutations in which ‘beg’ and ‘cad’ the rest are different. Hence, the number of words
patterns do not appear is n ( A ′ ∩ B ′ ). 10!
formed =
or n ( A ′ ∩ B ′ ) = n (U ) − n ( A ∪ B ) 3! 2!
= n (U ) − [n ( A ) + n ( B ) − n ( A ∩ B )]
(ii) Here, total letters are 12, in which 2I’s, 2T’s and 3E’s
= 5040 − 120 − 120 + 6 = 4806
but the rest are different. Hence, the number of words
2. The number of permutations of n things taken all 12!
formed = Note [For Remember]
at a time, p are alike of one kind, q are alike of 2! 2! 3!
second kind and r are alike of a third kind and High School = 10 th class = Total number of letters are 10
n!
the rest n − ( p + q + r ) are all different is Intermediate = 12 th class = Total number of letters
p!q !r ! are 12
368 Textbook of Algebra

3. The number of permutations of n different y Example 31. In how many ways can 4 prizes be distributed
things taken r at a time when each thing may be among 5 students, if no student gets all the prizes?
repeated any number of times is n r . Sol. The number of ways in which the 4 prizes can be given
Proof Since, the number of permutations of n away to the 5 students, if a student can get any number
different things taken r at a time = Number of ways in of prizes = 54 = 625.
which r blank places can be filled by n different Again, the number of ways in which a student gets all the 4
things. prizes = 5, since there are 5 students and any one of them
may get all the four prizes.
Clearly, the first place can be filled in n ways. Since, Therefore, the required number of ways in which a student
each thing may be repeated, the second place can be does not get all the prizes = 625 − 5 = 620.
filled in n ways. Similarly, each of the 3rd, 4th, K, r th
place can be filled in n ways. y Example 32. Find the number of n-digit numbers, which
By multiplication principle, the number of contain the digits 2 and 7, but not the digits 0, 1, 8, 9.
permutations of n different things taken r at a time Sol. The total number without any restrictions containing
when each thing may be repeated any number of digits 2, 3, 4, 5, 6, 7 is n (U ) = 6n .
times The total number of numbers that contain 3, 4, 5, 6, 7 is
= n × n × n × K × r factors n ( A ) = 5n .
The total number of numbers that contain 2, 3, 4, 5, 6 is
= nr
n ( B ) = 5n .
Corollary When r = n The total number of numbers that contain 3, 4, 5, 6 is
i.e., the number of permutations of n different things, n ( A ∩ B ) = 4n .
taken all at a time, when each thing may be repeated The total number of numbers that do not contain digits 2
any number of times in each arrangements is n n . and 7 is n ( A ∪ B )
i.e., n ( A ∪ B ) = n ( A ) + n ( B ) − n ( A ∩ B )
y Example 28. A child has four pockets and three = 5n + 5n − 4 n = 2 (5n ) − 4 n
marbles. In how many ways can the child put the Hence, the total number of numbers that contain 2 and 7 is
marbles in its pockets? n( A ′ ∩ B ′ )
∴ n ( A ′ ∩ B ′ ) = n (U ) − n ( A ∪ B ) = 6n − 2 ⋅ (5n ) + 4n
Sol. The first marble can be put into the pocket in 4 ways, so
can the second. Thus, the number of ways in which the
child can put the marbles = 4 × 4 × 4 = 4 3 = 64 ways y Example 33. Show that the total number of
permutations of n different things taken not more than
y Example 29. There are m men and n monkeys r at a time, when each thing may be repeated any
(n > m ). If a man have any number of monkeys. In how n(n r − 1)
number of times is .
many ways may every monkey have a master? (n − 1)
Sol. The first monkey can select his master by m ways and Sol. Given, total different things = n
after that the second monkey can select his master again The number of permutations of n things taken one at a time
by m ways, so can the third and so on.
= n P1 = n , now if we taken two at a time (repetition is
All monkeys can select master = m × m × m K upto n
factors = (m )n ways allowed), then first place can be filled by n ways and second
place can again be filled in n ways.
y Example 30. How many four digit numbers can be ∴The number of permutations of n things taken two at a time
formed by using the digits 1, 2, 3, 4, 5, 6, 7, if atleast = n P1 × n P1 = n × n = n 2
one digit is repeated ? Similarly, the number of permutations of n things taken
Sol. The numbers that can be formed when repetition of digits three at a time = n 3
is allowed are 7 4 = 2401.
M M M M M
The numbers that can be formed when all the digits are The number of permutations of n things taken r at a
distinct when repetition is not allowed are 7 P4 = 840. time = n r . Hence, the total number of permutations
Therefore, the numbers that can be formed when atleast = n + n 2 + n 3 + K + nr
one digit is repeated = 7 4 − 7 P4 n ( n r − 1)
= [sum of r terms of a GP]
= 2401 − 840 = 1561 ( n − 1)
Chap 05 Permutations and Combinations 369

#L Exercise for Session 2


1. If nP5 = 20 ⋅ nP3, then n equals
(a) 4 (b) 8 (c) 6 (d) 7
2. If P5 + 5 ⋅ P4 = Pr , then n + r equals
9 9 n

(a) 13 (b) 14 (c) 15 (d) 16


m
m+ n m−n P3
3. If P2 = 56 and P3 = 24, then n
equals
P2
(a) 20 (b) 40 (c) 60 (d) 80
2n + 1 2n − 1
4. If Pn − 1 : Pn = 7 : 10, then P3 equals
n

(a) 60 (b) 24 (c) 120 (d) 6


5. In a train, five seats are vacant, the number of ways three passengers can sit, is
(a) 10 (b) 20 (c) 30 (d) 60
6. If a denotes the number of permutations of x + 2 things taken all at a time, b the number of permutations of x
things taken 11 at a time and c the number of permutations of ( x − 11) things taken all at a time such that
a = 182 bc, the value of x is
(a) 10 (b) 12 (c) 15 (d) 18
7. The number of nine non-zero digits such that all the digits in the first four places are less than the digit in the
middle and all the digits in the last four places are greater than that in the middle, is
(a) 48 (b) 7560 (c) 10080 (d) 576
8. Total number of words that can be formed using all letters of the word ‘DIPESH’ that neither begins with ‘I’ nor
ends with ‘D’ is equal to
(a) 504 (b) 480 (c) 624 (d) 696
9. The number of all five digit numbers which are divisible by 4 that can be formed from the digits 0, 1, 2, 3, 4
(without repetition), is
(a) 36 (b) 30 (c) 34 (d) None of these
10. The number of words can be formed with the letters of the word ‘MATHEMATICS’ by rearranging them, is
11! 11! 11!
(a) (b) (c) (d) 11!
2! 2! 2! 2! 2! 2!

11. Six identical coins are arranged in a row. The number of ways in which the number of tails is equal to the
number of heads, is
(a) 9 (b) 20 (c) 40 (d) 120
12. A train time table must be compiled for various days of the week so that two trains twice a day depart for three
days, one train daily for two days and three trains once a day for two days. How many different time tables can
be compiled?
(a) 140 (b) 210 (c) 133 (d) 72
13. Five persons entered the lift cabin on the ground floor of an 8 floor house. Suppose each of them can leave
the cabin independently at any floor beginning with the first. The total number of ways in which each of the five
persons can leave the cabin at any one of the floor, is
(a) 57 (b) 75 (c) 35 (d) 2520
14. Four die are rolled. The number of ways in which atleast one die shows 3, is
(a) 625 (b) 671 (c) 1256 (d) 1296
15. The number of 4-digit numbers that can be made with the digits 1, 2, 3, 4 and 5 in which atleast two digits are
identical, is
(a) 45 − 5 ! (b) 505 (c) 600 (d) 120

16. There are unlimited number of identical balls of three different colours. How many arrangements of atmost 7
balls in a row can be made by using them?
(a) 2187 (b) 343 (c) 399 (d) 3279
Session 3
Number of Permutations Under Certain Conditions,
Circular Permutations, Restricted Circular Permutations

Number of Permutations Sol. The word ‘INSURANCE’ has nine different letters,
Under Certain Conditions combine the vowels into one bracket as (IUAE) and treat-
ing them as one letter we have six letters viz.
(i) Number of permutations of n different things, taken r (IUAE) N S R N C and these can be arranged among
at a time, when a particular thing is to be always 6!
themselves in ways and four vowels within the bracket
included in each arrangement, is r ⋅ n − 1 Pr − 1 2!
can be arranged themselves in 4 ! ways.
Corollary Number of permutations of n different 6!
∴ Required number of words = × 4 ! = 8640
things, taken r at a time, when p particular things is 2!
to be always included in each arrangement, is
p ! (r − ( p − 1)) n − p Pr − p . y Example 36. How many words can be formed with
the letters of the word ‘PATALIPUTRA’ without
(ii) Number of permutations of n different things, taken r changing the relative positions of vowels and
at a time, when a particular thing is never taken in consonants?
each arrangement, is
n −1 Sol. The word ‘PATALIPUTRA’ has eleven letters, in which
Pr . 2P’s, 3A’s, 2T’s, 1L, 1U, 1R and 1I. Vowels are AAIUA
(iii) Number of permutations of n different things, taken 5!
These vowels can be arranged themselves in = 20 ways.
all at a time, when m specified things always come 3!
together, is The consonants are PTLPTR these consonants can be
m ! × (n − m + 1) ! arranged themselves in
6!
= 180 ways
(iv) Number of permutations of n different things, taken 2!2!
all at a time, when m specified things never come ∴ Required number of words
together, is = 20 × 180 = 3600 ways.
n ! − m ! × (n − m + 1) !
y Example 37. Find the number of permutations
y Example 34. How many permutations can be made that can be had from the letters of the
out of the letters of the word ‘TRIANGLE’ ? How many word ‘OMEGA’
of these will begin with T and end with E ? (i) O and A occuping end places.
Sol. The word ‘TRIANGLE’ has eight different letters, which (ii) E being always in the middle.
can be arranged themselves in 8! ways.
∴ Total number of permutations = 8 ! = 40320
(iii) Vowels occuping odd places.
Again, when T is fixed at the first place and E at the last (iv) Vowels being never together.
place, the remaining six can be arranged themselves in Sol. There are five letters in the word ‘OMEGA’.
6 ! ways. (i) When O and A occuping end places
∴The number of permutations which begin with T and end with i.e., M E G (OA)
E = 6! = 720. the first three letters (M, E, G) can be arranged
themselves by 3 ! = 6 ways and last two letters (O, A)
y Example 35. In how many ways can the letters of can be arranged themselves by 2 ! = 2 ways.
the word ‘INSURANCE’ be arranged, so that the vowels ∴ Total number of such words
are never separate?
= 6 × 2 = 12 ways.
Chap 05 Permutations and Combinations 371

(ii) When E is the fixed in the middle, then there are four Similarly, if n different things are arranged along a circle
places left to be filled by four remaining letters O, M, for each circular arrangement number of linear
G and A and this can be done in 4 ! ways. arrangements is n.
∴ Total number of such words = 4 ! = 24 ways.
Therefore, the number of linear arrangements of n
(iii) Three vowels (O, E, A) can be arranged in the odd different things = n × number of circular arrangements of
places in 3 ! ways (1st, 3rd and 5th) and the two n different things
consonants (M, G) can be arranged in the even places
in 2 ! ways (2nd and 4th) (ii) Arrangements of beads or flowers
∴ Total number of such words (all different) around a circular
= 3 ! × 2 ! = 12 ways. necklace or garland
(iv) Total number of words = 5 ! = 120 Consider five beads A, B, C , D and E in a necklace or five
Combine the vowels into one bracket as (OEA) and flowers A, B, C , D and E in a garland, etc. If the necklace or
treating them as one letter, we have garland on the left is turned over, we obtain the
(OEA), M, G and these can be arranged themselves in arrangement on the right i.e. anti-clockwise and clockwise
3 ! ways and three vowels with in the bracket can be order of arrangement is not different we will get
arranged themselves in 3 ! ways. arrangements as follows:
∴ Number of ways when vowels come together We see that arrangements in figures are not different.
= 3 ! × 3 ! = 36 ways.
Flip to right
Hence, number of ways when vowels being
never together = 120 − 36 = 84 ways. D C C D

Circular Permutations E B B E

A A
(i) Arrangements round a circular table
Consider five persons A, B, C , D and E on the Then, the number of circular permutations of n different
1
circumference of a circular table in order which has no things taken all at a time is (n − 1) !, if clockwise and
2
head now, shifting A, B, C , D and E one position in anti-clockwise orders are taken as not different.
anti-clockwise direction we will get arragements as
follows y Example 38. Find the number of ways in which 12
different beads can be arranged to form a necklace.
E D C
Sol. 12 different beads can be arranged among themselves in a
A D E C D B circular order in (12 − 1) ! = 11 ! ways. Now, in the case of
necklace, there is no distinction between clockwise and
B C A B E A anti-clockwise arrangements. So, the required number of
1
(i) (ii) (iii) arrangements = (11 !).
B A 2
C A B E
y Example 39. Consider 21 different pearls on a
necklace. How many ways can the pearls be placed in
D E C D
on this necklace such that 3 specific pearls always
(iv) (v)
remain together?
We see that, if 5 persons are sitting at a round table, they Sol. After fixing the places of three pearls, treating 3 specific
can be shifted five times and five different arrangements. pearls = 1 unit. So, we have now
Thus, obtained will be the same, because anti-clockwise 18 pearls + 1 unit = 19 and the number of arrangement will
order of A, B, C , D and E does not change. be (19 − 1) ! = 18 !
Also, the number of ways of 3 pearls can be arranged
But if A, B, C , D and E are sitting in a row and they are
between themselves is 3 ! = 6.
shifted in such an order that the last occupies the place of
Since, there is no distinction between the clockwise and
first, then the five arrangements will be different. Thus, if anti-clockwise arrangements.
there are 5 things, then for each circular arrangement 1
number of linear arrangements is 5. So, the required number of arrangements = 18 !⋅ 6 = 3 (18 !).
2
372 Textbook of Algebra

Restricted Circular y Example 42. In how many different ways can five
boys and five girls form a circle such that the boys and
Permutations girls alternate?
Sol. After fixing up one boy on the table, the remaining can
Case I If clockwise and anti-clockwise orders are taken as
be arranged in 4 ! ways but boys and girls are to alternate.
different, then the number of circular permutations of n There will be 5 places, one place each between two boys
different things taken r at a time. these five places can be filled by 5 girls in 5 ! ways.
n B1
Pr 1 n!
= = ⋅
r r (n − r ) ! B5
B2
Note For checking correctness of formula, put r = n, then we get
( n − 1) ! [result (5) (i)]
B4
y Example 40. In how many ways can 24 persons be B3
seated round a table, if there are 13 sets ?
Hence, by the principle of multiplication, the required
Sol. In case of circular table, the clockwise and anti-clockwise
number of ways = 4 ! × 5 ! = 2880.
orders are different, the required number of circular
24
P13 24 ! y Example 43. 20 persons were invited to a party. In
permutations = = .
13 13 × 11 ! how many ways can they and the host be seated at a
⇒ n ! = n × number of circular arrangements of n circular table ? In how many of these ways will two
different things particular persons be seated on either side of the host?
⇒ Number of circular arrangements of n different things Sol. I Part Total persons on the circular table
n! = 20 guest + 1 host = 21
= = ( n − 1) !
n They can be seated in (21 − 1) ! = 20 ! ways.
Hence, the number of circular permutations of n different
things taken all at a time is (n − 1) !, if clockwise and II Part After fixing the places of three persons
anti-clockwise orders are taken as different. (1 host + 2 persons).
Treating (1 host + 2 persons) = 1 unit, so we have now
y Example 41. Find the number of ways in which three {(remaining 18 persons + 1 unit) = 19} and the number of
Americans, two British, one Chinese, one Dutch and arrangement will be (19 − 1) ! = 18 ! also these two particular
one Egyptian can sit on a round table so that persons
persons can be seated on either side of the host in 2 ! ways.
of the same nationality are separated.
P20 H P1
Sol. The total number of persons without any restrictions is P19 P2
n (U ) = (8 − 1)! P18 P3
= 7! = 5040 P17 P4
P5
When, three Americans ( A1, A 2 , A 3 ) are sit together, P16
P6
n ( A ) = 5! × 3! P15
P14 P7
= 720
P13 P8
When, two British ( B1, B 2 ) are sit together P12 P P P9
11 10
n ( B ) = 6! × 2!
Hence, the number of ways of seating 21 persons on the
= 1440 circular table such that two particular persons be seated on
When, three Americans ( A1, A 2 , A 3 ) and two British ( B1, B 2 ) either side of the host = 18 ! × 2 ! = 2 × 18 !
are sit together n ( A ∩ B ) = 4 ! × 3! × 2! = 288
Case II If clockwise and anti-clockwise orders are taken as
∴ n ( A ∪ B ) = n ( A ) + nB ) − n ( A ∩ B ) not different, then the number of circular permutations of n
= 720 + 1440 − 288= 1872 n
P 1 n!
different things taken r at a time = r = ⋅
Hence, n ( A ∩ B ′ ) = n (U ) − n ( A ∪ B ) 2r 2r (n − r ) !
Note
= 5040 − 1872
For checking correctness of formula put r = n, then we get
= 3168 ( n − 1)!
[result (5) (ii)]
2
Chap 05 Permutations and Combinations 373

y Example 44. How many necklace of 12 beads each can be made from 18 beads of various colours?
Sol. In the case of necklace, there is no distinction between the clockwise and anti-clockwise arrangements, the required
number of circular permutations.
18
P12 18 ! 18 × 17 × 16 × 15 × 14 × 13 ! 119 × 13 !
= = = =
2 × 12 6 ! × 24 6 × 5 × 4 × 3 × 2 × 1 × 24 2

#L Exercise for Session 3


1. How many words can be formed from the letters of the word ‘COURTESY’ whose first letter is C and the last
letter is Y ?
(a) 6! (b) 8! (c) 2 (6) ! (d) 2 (7) !

2. The number of words that can be made by writing down the letters of the word ‘CALCULATE’ such that each
word starts and ends with a consonant, is
3 5
(a) (7) ! (b) 2 (7) ! (c) (7) ! (d) 3 (7) !
2 2

3. The number of words can be formed from the letters of the word ‘MAXIMUM’, if two consonants cannot occur
together, is
(a) 4! (b) 3 ! × 4 !
4!
(c) 3! (d)
3!

4. All the letters of the word ‘EAMCET’ are arranged in all possible ways. The number of such arrangements in
which two vowels are not adjacent to each other, is
(a) 54 (b) 72
(c) 114 (d) 360

5. How many words can be made from the letters of the word ‘DELHI’, if L comes in the middle in every word?
(a) 6 (b) 12 (c) 24 (d) 60

6. In how many ways can 5 boys and 3 girls sit in a row so that no two girls are sit together?
(a) 5 ! × 3 ! (b) 4 P3 × 5 ! (c) 6P3 × 5 ! (d) 5P3 × 3 !

7. There are n numbered seats around a round table. Total number of ways in whichn1 (n1 < n ) persons can sit
around the round table, is equal to
(a) nCn 1 (b) n Pn 1 (c) nC n1 − 1 (d) n Pn1 − 1

8. In how many ways can 7 men and 7 women can be seated around a round table such that no two women can
sit together?
(a) 7! (b) 7 ! × 6 ! (c) (6 !)2 (d) (7 !)2

9. The number of ways that 8 beads of different colours be string as a necklace, is


(a) 2520 (b) 2880 (c) 4320 (d) 5040

10. If 11 members of a committee sit at a round table so that the President and secretary always sit together, then
the number of arrangements, is
(a) 9 ! × 2 (b) 10! (c) 10 ! × 2 (d) 11!

11. In how many ways can 15 members of a council sit along a circular table, when the secretary is to sit on one
side of the Chairman and the deputy secretary on the other side?
(a) 12 ! × 2 (b) 24 (c) 15 ! × 2 (d) 30
Session 4
Combination, Restricted Combinations
Combination (ii) n C r = 0, if r > n
Each of the different groups or selections which can be (iii) n C 0 = n C n = 1 , n C 1 = n
made by some or all of a number of given things without
reference to the order of the things in each group is called (iv) n Pr = n C r , if r = 0 or 1
a combination. n
(v) n C r = n C n − r , if r >
2
Important Result (vi) If n C x = n C y ⇒ x = y or x + y = n
(1) The number of combinations of n different n +1
(vii) n C r + n C r − 1 = Cr [Pascal’s rule]
things taken r at a time is denoted by n C r or
n  n n −1
C (n , r ) or   . (viii) n C r = ⋅ Cr − 1
r  r
Then, (ix) n ⋅ n − 1 C r − 1 = (n − r + 1) ⋅ n C r − 1
n! n
n −r +1
n
Cr = [ 0 ≤ r ≤ n] (x)
Cr
=
r !( n − r )! n r
Cr − 1
n
Pr
= n
r! (xi) (a) If n is even , n C r is greatest for r =
2
n ( n − 1) ( n − 2) K ( n − r + 1)
= , n ∈ N and r ∈W n −1 n +1
r ( r − 1) ( r − 2) K 2 ⋅ 1 (b) If n is odd , n C r is greatest for r = or
2 2
Proof Let the number of combinations of n different
(xii) n C 0 + n C 1 + n C 2 + K + n C n = 2 n
things taken r at a time be n C r .
(xiii) n C 0 + n C 2 + n C 4 +K
Now, each combination consists of r different things and
these r things can be arranged among themselves in r ! = n C 1 + n C 3 + n C 5 + K = 2n − 1
ways.
(xiv) 2n + 1 C 0 + 2n + 1
C1 + 2n + 1
C 2 +K + 2n + 1
C n = 2 2n
Thus, for one combination of r different things, the
n +1 n +2 n+3
number of arrangements is r ! . (xv) n C n + Cn + C2 + Cn + K
2n − 1
Hence, for n C r combinations, number of arrangements is + Cn = 2n
Cn + 1
r ! × Cr
n
…(i)
y Example 45. If 15
C 3r = 15C r + 3 , find r C 2 .
But number of permutations of n different things taken r at
a time is n Pr . …(ii) Sol. We have, 15
C 3r = 15C r +3

From Eqs. (i) and (ii), we get ⇒ 3r = r + 3


n! or 3r + r + 3 = 15
r ! × n C r = n Pr = ⇒ 2r = 3 or 4r = 12
(n − r ) !
3
n! ⇒ r = or r = 3
∴ n
Cr = , r ∈W and n ∈ N 2
r ! (n − r ) ! 3
but r ∈W , so that r ≠
Note the following facts: 2
(i) n C r is a natural number ∴ r =3
Then, r
C 2 = 3C 2 = 3C 1 = 3
Chap 05 Permutations and Combinations 375

y Example 46. If n C 9 = n C 7 , find n. ⇒


n
=2
r
Sol. We have, n
C 9 = nC 7 ⇒ n = 9 + 7 [Q9 ≠ 7]
⇒ n = 2r …(ii)
∴ n = 16 On solving Eqs. (i) and (ii), we get n = 10 and r = 5.

y Example 47. Prove that y Example 50. If n C r − 1 = 36, n C r = 84 and


 n  n   n   n + 2 n
C r + 1 = 126, find r.
  +2 +  = 
r   r − 1  r − 2  r  n
Cr 84
Sol. Here, =
n  n
Cr − 1 36
Sol. ∴   = n C r
r 
n −r +1 7  n Cr n − r + 1
n   n   n  ⇒ = Q n = 
∴ LHS =   + 2   +  r 3  C r − 1 r 
r  r − 1 r − 2
⇒ 3n − 3r + 3 = 7 r
= n Cr + 2 n Cr −1 + n Cr −2
⇒ 10r − 3n = 3 …(i)
= (n C r + n C r − 1) + (n C r −1 + n Cr − 2) n
Cr +1 n − (r + 1) + 1 126  n Cr n − r + 1
n +1 n +1 n+2 and = = Q n = 
= Cr + Cr −1 = Cr n
( r + 1)
Cr 84  C r − 1 r 
n + 2 n −r 3
=  = RHS ⇒ =
 r  r +1 2

y Example 48. If 2n
C 3 : n C 3 = 11 : 1, find the value of n . ⇒ 2n − 2r = 3r + 3
⇒ 5r − 2n = − 3
Sol. We have,
or 10r − 4 n = − 6 …(ii)
2n
C 3 : n C 3 = 11 : 1
2n
On subtracting Eq. (ii) from Eq. (i), we get
C3 11
⇒ n
= n =9
C3 1
From Eq. (i), we get
2n ( 2n − 1) (2n − 2)
10r − 27 = 3 ⇒ 10r = 30
1⋅2⋅3 4 ( 2n − 1)
⇒ = 11 ⇒ = 11 ∴ r =3
n ( n − 1) ( n − 2) ( n − 2)
1⋅2⋅3
y Example 51. Prove that product of r consecutive
⇒ 8 n − 4 = 11n − 22 ⇒ 3n = 18 positive integers is divisible by r ! .
∴ n =6 Sol. Let r consecutive positive integers be (m),
n +1 n −1
(m + 1), (m + 2), K , (m + r − 1), where m ∈ N .
y Example 49. If C r + 1 : nC r : C r − 1 = 11 : 6 : 3 , ∴ Product = m (m + 1) (m + 2) K (m + r − 1)
find the values of n and r. (m − 1)! m (m + 1) (m + 2) K (m + r − 1)
n +1
=
Cr +1 11 (m − 1)!
Sol. Here, =
n
Cr 6 (m + r − 1)! r ! ⋅ (m + r − 1)!
= =
n + 1 C r 11 n
 n n n −1  (m − 1)! r ! (m − 1)!
⇒ ⋅ = Q C r = r ⋅ Cr
r + 1 n Cr 6
− 1
 [Q m + r − 1C r is natural number]
n + 1 11 = r!⋅ m +r −1
Cr ,
⇒ =
r +1 6 which is divisible by r ! .
⇒ 6n + 6 = 11r + 11
⇒ 6n − 11r = 5 …(i)
y Example 52. Evaluate
3 5

and n −1
n
Cr
=
6 47
C4 + ∑ 50 − j C 3 + ∑ 56 − k C 53 − k .
Cr 3 j =0 k =0
−1
3 5
n −1

n

Cr −1
=
6  n n n −1  Sol. We have, 47
C4 + ∑ 50 − j C 3 + ∑ 56 − k C 53 − k
r n −1
Cr 3 Q C r = r ⋅ Cr − 1
 j =0 k =0
−1
376 Textbook of Algebra

3 5
= 47
C4 + ∑ 50 − j C 3
j =0
+ ∑ 56 − k C 3 [Q nC r
k =0
= nC n − r ] Restricted Combinations
(i) The number of selections (combinations) of r objects
= 47
C 4 + ( 50C 3 + 49
C3 + 48
C3 + 47
C3)
out of n different objects, when
+ ( 56C 3 + 55
C3 + 54
C3 + 53
C3 + 52
C 3 + 51C 3 ) (a) k particular things are always included= n − k C r − k .
= 47
C4 + 47
C3 + 48
C3 + 49
C3 + 50
C 3 + 51C 3 n −k
(b) k particular things are never included = Cr .
+ 52
C3 + 53
C3 + 54
C3 + 55
C3 + 56
C3 (ii) The number of combinations of r things out of n
= ( C4 + 47 47
C3) + 48
C3 + 49
C3 + 50
C3 + C3 51 different things, such that k particular things are not
together in any selection = n C r − n − k C r − k
+ 52
C3 + 53
C3 + 54
C3 + 55
C3 + 56
C3
(iii) The number of combinations of n different objects
= 48
C4 + 48
C3 + 49
C3 + 50
C 3 + 51C 3 + K + 56
C3 taking r at a time when, p particular objects are
= 49
C4 + 49
C3 + 50
C3 + K + 56
C3 always included and q particular objects are always
excluded = n − p −q C r − p
M M M M M
Note
= 56
C4 + 56
C3 = 57
C4 (i) The number of selections of r consecutive things out of
n things in a row = n − r + 1.
y Example 53. Prove that the greatest value of (ii) The number of selections of r consecutive things out of
n, if r < n
2n
C r ( 0 ≤ r ≤ 2n ) is 2n C n (for 1 ≤ r ≤ n). n things along a circle =  .
1, if r = n
2n
Cr  n Cr
2n − r + 1 n − r + 1
Sol. We have, = Q n = 
2n
C r −1  C r − 1
r r  y Example 55. In how many ways can a cricket, eleven
players by chosen out of a batch 15 players, if
2 ( n − r ) + ( r + 1) 1 + 2( n − r ) + 1
= = >1 (i) a particular is always chosen.
r r
2n
(ii) a particular player is never chosen?
Cr
⇒ 2n
>1 [for 1 ≤ r ≤ n] Sol. (i) Since, particular player is always chosen. It means
Cr −1 that 11 − 1 = 10 players are selected out of the
remaining 15 − 1 = 14 players.
∴ 2n
Cr −1 < 2n
Cr
∴ Required number of ways = 14C 10 = 14C 4
On putting r =1, 2, 3, …, n,
14 ⋅ 13 ⋅ 12 ⋅ 11
then 2n
C0 < 2n
C 1, 2n
C1 < 2n
C 2 , K, 2n
Cn − 1 < 2n
Cn = = 1001
1⋅2⋅3⋅ 4
On combining all inequalities, we get
(ii) Since, particular player is never chosen. It means that
⇒ 2n
C0 < 2n
C1 < 2n
C2 < 2n
C3 < K < 2n
Cn − 1 < 2n
Cn 11 players are selected out of the remaining 15 − 1 = 14
players.
but 2n
Cr = 2n
C 2n − r , it follows that
∴ Required number of ways = 14C 11 = 14C 3
2n
C 2n < 2n
C 2n − 1 < C 2n − 2 <
2n 2n
C 2n − 3 < K < Cn + 1 <
2n 2n
Cn
14 ⋅ 13 ⋅ 12
Hence, the greatest value of 2n
C r is 2n
Cn . = = 364
1 ⋅ 2⋅ 3

y Example 54. Thirty six games were played in a y Example 56. How many different selections of 6
football tournament with each team playing once books can be made from 11 different books, if
against each other. How many teams were there?
(i) two particular books are always selected.
Sol. Let the number of teams be n.
(ii) two particular books are never selected?
Then number of matches to be played is n C 2 = 36 Sol. (i) Since, two particular books are always selected. It
9 ×8 9 means that 6 − 2 = 4 books are selected out of the
⇒ n
C2 = = C2
1×2 remaining 11 − 2 = 9 books.
9 ⋅8⋅7 ⋅6
⇒ n =9 ∴ Required number of ways = 9C 4 = = 126.
1⋅2⋅3⋅ 4
Chap 05 Permutations and Combinations 377

(ii) Since, two particular books are never selected. It y Example 59. A question paper consists of two
means that 6 books are selected out of the remaining sections having respectively, 3 and 5 questions. The
11 − 2 = 9 books. following note is given on the paper ‘‘It is not
∴ Required number of ways = 9C 6 necessory to attempt all the questions one question
9 ⋅8⋅7 from each section is compulsory’’. In how many
= 9C 3 = = 84.
1⋅2⋅3 ways can a candidate select the questions?
Sol. Here, we have two sections A and B (say), the section A
y Example 57. A person tries to form as many different has 3 questions and section B has 5 questions and one
parties as he can, out of his 20 friends. Each party question from each section is compulsory, according to
should consist of the same number. How many friends the given direction.
should be invited at a time? In how many of these ∴ Number of ways selecting one or more than one question
parties would the same friends be found? from section A is 2 3 − 1 = 7
Sol. Let the person invite r number of friends at a time. Then, and number of ways selecting one or more than one
the number of parties are 20 C r , which is maximum, question from section B is 2 5 − 1 = 31
when r = 10. Hence, by the principle of multiplication, the required
If a particular friend will be found in p parties, then p is the number of ways in which a candidate can select the
number of combinations out of 20 in which this particular questions
friend must be included. Therefore, we have to select 9 = 7 × 31 = 217.
more from 19 remaining friends.
Hence, p = 19C 9 y Example 60. A student is allowed to select atleast
(2) The number of ways (or combinations) of n one and atmost n books from a collection of (2n + 1)
different things selecting atleast one of them is books. If the total number of ways in which he can
select books is 63, find the value of n.
2n − 1 . This can also be stated as the total
Sol. Given, student select atleast one and atmost n books from
number of combinations of n different things. a collection of (2n + 1) books. It means that he select one
Proof For each things, there are two possibilities, whether book or two books or three books or … or n books.
it is selected or not selected. Hence, by the given hypothesis.
2n + 1 2n + 1 2n + 1
Hence, the total number of ways is given by total C1 + C2 + C 3 + ... + 2 n + 1 C n = 63 …(i)
possibilities of all the things which is equal to Also, the sum of binomial coefficients, is
2 × 2 × 2 × ... × n factors = 2 n 2n + 1
C0 + 2n + 1
C1 + K + 2n + 1
Cn + 2n + 1
Cn + 1
2n + 1
But, this includes one case in which nothing is selected. +K+ Cn + 1
2n + 1 2n + 1
= ( 1 + 1) =2
Hence, the total number of ways of selecting one or more
2n + 1
of n different things = 2 n − 1 ⇒ C 0 + 2 ( 2 n + 1C 1 + 2n + 1
C2 + K + 2n + 1
Cn )
2n + 1 2n + 1
Aliter Number of ways of selecting one, two, three, …, n + C 2n + 1 = 2 [Q C r = C n − r ]
n

2n + 1 2n + 1
things from n different things are ⇒ 1 + 2 × 63 + 1 = 2 ⇒ 128 = 2
2n + 1
n n n n
C 1 , C 2 , C 3 , ..., C n , respectively. ⇒ 2 =2
7
⇒ 7 = 2n + 1
∴ n =3
Hence, the total number of ways or selecting atleast one
thing is y Example 61. There are three books of Physics, four
n
C1 + nC2 + nC 3 + K + nCn of Chemistry and five of Mathematics. How many
= ( n C 0 + n C 1 + n C 2 + K + n C n ) − n C 0 = 2n − 1 different collections can be made such that each
collection consists of
y Example 58. Mohan has 8 friends, in how many (i) one book of each subject,
ways he invite one or more of them to dinner? (ii) atleast one book of each subject,
Sol. Mohan select one or more than one of his 8 friends. So, (iii) atleast one book of Mathematics.
required number of ways
Sol. (i) 3 C 1 × 4C 1 × 5C 1 = 3 × 4 × 5 = 60
= 8C 1 + 8C 2 + 8C 3 + K + 8C 8 (ii) (23 − 1) × (24 − 1) × (25 − 1) = 7 × 15 × 31 = 3255
= 2 − 1 = 255.
8
(iii) (25 − 1) × 27 = 31 × 128 = 3968
378 Textbook of Algebra

#L Exercise for Session 4


1. If 43Cr − 6 = 43C3r + 1, the value of r is
(a) 6 (b) 8 (c) 10 (d) 12
2. If 18
C15 + 2 ( C16 ) +
18
C16 + 1 = C3, the value of n is
17 n

(a) 18 (b) 20 (c) 22 (d) 24


3. If 20
Cn + 2 = nC16 , the value of n is
(a) 7 (b) 10 (c) 13 (d) None of these
5
4. If 47C4 + ∑ 52 − r C3 is equal to
r =1

(a) 47C6 (b) 52


C5 (c) 52
C4 (d) None of these
n+1
5. If C3 + C4 >
n n
C3 then
(a) n > 6 (b) n < 6 (c) n > 7 (d) n < 7

6. The Solution set of 10


Cx −1 > 2 . Cx is
10

(a) {1, 2, 3} (b) {4, 5, 6} (c) {8, 9, 10} (d) {9, 10, 11}
7. If 2n
C2 : C2 = 9 : 2 and Cr = 10, then r is equal to
n n

(a) 2 (b) 3 (c) 4 (d) 5


8. If 2n
C3 : nC2 = 44 : 3 , for which of the following value of r, the value of nCr will be 15.
(a) r = 3 (b) r = 4 (c) r = 5 (d) r = 6
9. If Cr = Cr − 1 and Pr = Pr + 1, the value of n is
n n n n

(a) 2 (b) 3 (c) 4 (d) 5


10. If nPr = 840, nCr = 35, the value of n is
(a) 1 (b) 3 (c) 5 (d) 7
11. If P3 + Cn − 2 = 14n, the value of n is
n n

(a) 5 (b) 6 (c) 8 (d) 10


12. There are 12 volleyball players in all in a college, out of which a team of 9 players is to be formed. If the captain
always remains the same, in how many ways can the team be formed ?
(a) 36 (b) 99 (c) 108 (d) 165
13. In how many ways a team of 11 players can be formed out of 25 players, if 6 out of them are always to be
included and 5 are always to be excluded
(a) 2002 (b) 2008 (c) 2020 (d) 8002
14. A man has 10 friends. In how many ways he can invite one or more of them to a party?
(a) 10! (b) 210 (c) 10 ! − 1 (d) 210 − 1

15. In an examination, there are three multiple choice questions and each question has four choices. Number of
ways in which a student can fail to get all answers correct, is
(a) 11 (b) 12 (c) 27 (d) 63
16. In an election, the number of candidates is 1 greater than the persons to be elected . If a voter can vote in
254 ways, the number of candidates is
(a) 6 (b) 7 (c) 8 (d) 10
17. The number of groups that can be made from 5 different green balls, 4 different blue balls and 3 different red
balls, if atleast one green and one blue ball is to be included
(a) 3700 (b) 3720 (c) 4340 (d) None of these
18. A person is permitted to select atleast one and atmost n coins from a collection of (2n + 1) distinct coins. If the
total number of ways in which he can select coins is 255, thenn equals
(a) 4 (b) 8 (c) 16 (d) 32
Session 5
Combinations from Identical Objects
Combinations from ∴ The total number of selections when all the three kinds of
fruits are selected = 5 × 6 × 7 = 210
Identical Objects But, in one of these selection number of each kind of fruit is
(i) The number of combinations of n identical objects zero and hence this selection must be excluded.
taking r objects (r ≤ n ) at a time = 1. ∴ Required number = 210 − 1 = 209
(ii) The number of combinations of zero or more objects
from n identical objects = n + 1.
(iii) The total number of combinations of atleast one out
Combinations when both Identical
of a 1 + a 2 + a 3 + K + a n objects, where a 1 are alike of and Distinct Objects are Present
one kind, a 2 are alike of second kind, a 3 are alike of The number of combinations (selections) of one or more
third kind, …, a n are alike of nth kind objects out of a 1 + a 2 + a 3 + K + a n objects, where a 1
= (a 1 + 1) (a 2 + 1) (a 3 + 1) K (a n + 1) − 1 are alike of one kind, a 2 are alike of second kind, a 3 are
alike of third kind, …, a n are alike of nth kind and k
y Example 62. How many selections of atleast one red are distinct.
ball from a bag containing 4 red balls and 5 black = {(a 1 + 1) (a 2 + 1) (a 3 + 1) L (a n + 1)}
balls, balls of the same colour being identical?
(k C 0 + k C1 + k C2 + K + k Ck ) − 1
Sol. Number of selections of atleast one red ball from 4 identi-
cal red balls = 4 = (a 1 + 1) (a 2 + 1) (a 3 + 1) + K + (a n + 1) 2 k − 1
Number of selections of any number of black balls from 5
identical black balls y Example 65. Find the number of ways in which one
=5+1=6 or more letters can be selected from the letters
∴ Required number of selections of balls AAAAA BBBB CCC DD EFG.
= 4 × 6 = 24 Sol. Here, 5A’s are alike, 4 B ’s are alike, 3 C ’s are alike, 2D ’s
are alike and E, F, G are different.
y Example 63. There are p copies each of n different ∴ Total number of combinations
books. Find the number of ways in which a non-empty = ( 5 + 1) ( 4 + 1) ( 3 + 1) ( 2 + 1) 2 3 − 1
selection can be made from them. = 6 ⋅5 ⋅ 4 ⋅3 ⋅8 − 1
Sol. Since, copies of the same book are identical. = 2879
∴ Number of selections of any number of copies of a book [excluding the case, when no letter is selected]
is p + 1. Similarly, in the case for each book.
Explanation Selection from ( AAAAA ) can be made by 6
Therefore, total number of selections is ( p + 1)n .
ways such include no A, include one A, include two A,
But this includes a selection, which is empty i.e., zero copy include three A, include four A, include five A. Similarly,
of each book. Excluding this, the required number of selections from ( BBBB ) can be made in 5 ways, selections
non-empty selections is ( p + 1)n − 1. from (CCC ) can be made in 4 ways, selections from ( DD )
can be made in 3 ways and from E, F, G can be made in
y Example 64. There are 4 oranges, 5 apples and 2 × 2 × 2 ways.
6 mangoes in a fruit basket and all fruits of the same
kind are identical. In how many ways can a person make
a selection of fruits from among the fruits in the basket?
Number of Divisors of N
Sol. Zero or more oranges can be selected out of 4 identical Every natural number N can always be put in the form
oranges = 4 + 1 = 5 ways. N = p 1α 1 ⋅ p 2α 2 ⋅ p 3α 3 ... p kα k , where p 1 , p 2 , p 3 , ..., p k are
Similarly, for apples number of selection = 5 + 1 = 6 ways distinct primes and α 1 , α 2 , α 3 , K, α k ∈W .
and mangoes can be selected in 6 + 1 = 7 ways.
380 Textbook of Algebra

(i) The total number of divisors of N including 1 and N = ( 3 + 1) ( 2 + 1) ( 2 + 1) ( 1 + 1) − 2


= (α 1 + 1) (α 2 + 1) (α 3 + 1) K (α k + 1) = 72 − 2 = 70

(ii) The total number of divisors of N excluding 1 and N And sum of all these divisors (proper)
= (α 1 + 1) (α 2 + 1) (α 3 + 1) K (α k + 1) − 2 = ( 20 + 21 + 22 + 23 ) ( 30 + 31 + 32 )

(iii) The total number of divisors of N excluding either 1 (7 0 + 71 + 7 2 ) (110 + 111 ) − 1 − 38808
or N = (α 1 + 1) (α 2 + 1) (α 3 + 1) K (α k + 1) − 1 = (15) (13) (57 ) (12) − 38809
(iv) Sum of all divisors = ( p 10 + p 11 + p 12 + p 13 + K + p 1∞1 ) = 133380 − 38809
( p 20 + p 21 + p 22 + p 23 + K + p 2α 2 ) ... = 94571

( p k0 + p k1 + p k2 + p k3 + K + p kα k ) y Example 67. Find the number of even proper divisors


 1 − p α1 + 1   1 − p α 2 + 1   1 − p αk + 1  of the number 1008.
=  1  ⋅ 2  ...  k  Sol. Q1008 = 24 × 32 × 71
−   −   − p k 
 1 p 1   1 p 2   1
∴ Required number of even proper divisors
(v) Sum of proper divisors (excluding 1 and the = Total number of selections of atleast one 2 and
expression itself) any number of 3’s or 7’s.
= Sum of all divisors − ( N + 1) = 4 × (2 + 1) × (1 + 1) − 1 = 23

(vi) The number of even divisors of N are possible only if y Example 68. Find the number of odd proper divisors
p 1 = 2 , otherwise there is no even divisor. of the number 35700. Also, find sum of the odd
∴ Required number of even divisors proper divisors.
Sol. Q35700 = 22 × 31 × 52 × 71 × 171
= α 1 (α 2 + 1) (α 3 + 1) + K + (α k + 1)
∴ Required number of odd proper divisors
(vii) The number of odd divisors of N
= Total number of selections of zero 2 and any
Case I If p 1 = 2 , the number of odd divisors number of 3’s or 5’s or
= (α 2 + 1) (α 3 + 1) K (α k + 1) 7’s or 17’s
= (1 + 1) (2 + 1) (1 + 1) (1 + 1 ) − 1 = 23
Case II If p 1 ≠ 2 , the number of odd divisors
∴ The sum of odd proper divisors
= (α 1 + 1) (α 2 + 1) (α 3 + 1) K (α k + 1)
= (30 + 31 ) (50 + 51 + 52 ) (7 0 + 71 ) (17 0 + 71 ) − 1
(viii) The number of ways in which N can be resolved as a = 4 × 31 × 8 × 18 − 1
product of two factors
= 17856 − 1 = 17855

 1 (α + 1) (α + 1) K (α + 1) , if N is not a
y Example 69. If N = 10800, find the
 2 1 2 k perfect square
= (i) the number of divisors of the form
1
 {(α 1 + 1) (α 2 + 1) K (α k + 1) + 1} , if N is a 4m + 2 , ∀ m ∈W .
2 perfect square (ii) the number of divisors which are multiple of 10.
(ix) The number of ways in which a composite number N (iii) the number of divisors which are multiple of 15.
can be resolved into two factors which are relatively Sol. We have, N = 10800 = 24 × 33 × 52
prime ( or coprime) to each other is equal to 2 n − 1 , (i) Q( 4 m + 2) = 2(2m + 1), in any divisor of the form
where n is the number of different factors (or different 4 m + 2, 2 should be exactly 1.
primes) in N . So, the number of divisors of the form
( 4 m + 2) = 1 × (3 + 1) × (2 + 1) = 1 × 4 × 3 = 12
y Example 66. Find the number of proper factors of
the number 38808. Also, find sum of all these divisors. (ii) ∴ The required number of proper divisors
Sol. The number 38808 = 23 ⋅ 32 ⋅ 7 2 ⋅ 11 = Total number of selections of atleast one 2 and one
5 from 2, 2, 2, 2, 3, 3, 3, 5, 5
Hence, the total number of proper factors (excluding 1 and
itself i.e., 38808) = 4 × (3 + 1) × 2 = 32
Chap 05 Permutations and Combinations 381

(iii) ∴ The required number of proper divisors Proof The number of ways in which (m + n ) distinct
= Total number of selections of atleast one 3 and one objects are divided into two groups of the sizem and n
5 from 2, 2, 2, 2, 3, 3, 3, 5, 5
= The number of ways m objects are selected out of
= ( 4 + 1) × 3 × 2 = 30 (m + n ) objects to form one of the groups, which can
be done in m + n C m ways. The other group of n objects
y Example 70. Find the number of divisors of the
is formed by the remaining n objects.
number N = 2 3⋅ 3 5 ⋅ 5 7 ⋅7 9 ⋅9 11 , which are
perfect square.
m+n
Sol. QN = 2 3 ⋅ 3 5⋅ 5 7 ⋅ 7 9⋅ 9 11
= 2 3 ⋅ 3 5 ⋅ 5 7 ⋅ 7 9 ⋅ 3 22
= 2 3⋅ 3 27⋅ 5 7 ⋅ 7 9
m n
For perfect square of N , each prime factor must occur even
number of times. (Tree diagram)
2 can be taken in 2 ways (i.e., 20 or 22 ) (m + n )
m +n
3 can be taken in 14 ways (i.e., 30 or 32 or 34 or 36 or 38 or = C m ⋅n C n =
m!n!
310 or 312 or 314 or 316 or 318 or 320 or 322 or 324 or 326 )
Corollary I The number of ways to distribute (m + n )
5 can taken in 4 ways (i.e., 50 or 52 or 54 or 56 )
and 7 can taken in 5 ways
distinct objects among 2 persons in the groups
(i.e., 7 0 or 7 2 or 7 4 or 7 4 or 7 6 or 7 8 ) containing m and n objects
Hence, total divisors which are perfect squares = (Number of ways to divide) × (Number of groups)
= 2 × 14 × 4 × 5 = 560 (m + n )
= × 2!
m!n!
y Example 71. In how many ways the number 10800
can be resolved as a product of two factors? Corollary II The number of ways in which (m + n + p )
Sol. Let N = 10800 = 2 4 × 3 3 × 5 2 distinct objects can be divided into three unequal
groups containingm, n and p objects, is
Here, N is not a perfect square [Q power of 3 is odd]
1
Hence, the number of ways = ( 4 + 1) (3 + 1) (2 + 1) = 30
2 m+n+p

y Example 72. In how many ways the number 18900


can be split in two factors which are relatively prime
(or coprime)? m p
n
Sol. Let N = 18900 = 2 ⋅ 3 ⋅ 5 ⋅ 7
2 3 2 1
(Tree diagram)
Relatively prime or coprime Two numbers not neces-
sarily prime are said to be relatively prime or coprime, if
m +n +p (m + n + p ) !
their HCF (highest common factor) is one as 2, 3, 5, 7 are C m ⋅n + p C n ⋅p C p =
relatively prime numbers. m!n! p!
∴ n=4 [number of different primes in N ] Corollary III The number of ways to distribute
Hence, number of ways in which a composite number N (m + n + p ) distinct objects among 3 persons in the
can be resolved into two factors which are relatively groups containing m, n and p objects
prime or coprime = 24 − 1 = 23 = 8
= (Number of ways to divide) × (Number of groups)
Division of Objects Into Groups (m + n + p ) !
= × 3!
m!n! p!
(a) Division of Objects Into Groups of Unequal Size
Theorem Number of ways in which (m + n ) Corollary IV The number of ways in which
distinct objects can be divided into two unequal ( x 1 + x 2 + x 3 + L + x n ) distinct objects can be
(m + n ) ! divided into n unequal groups containing x 1 ,
groups containing m and n objects is . x 2 , x 3 , K, x n objects, is
m! n !
382 Textbook of Algebra

52
x1 + x2 + x3 +...+xn

13 13
13 13
x1 xn (Tree diagram)
.
x2 x3 ..
(Tree diagram) 52! 52!
4
× 4! =
4 ! (13!) (13!)4
(x 1 + x 2 + x 3 + L + x n ) ! Aliter Each player will get 13 cards. Now, first player can
.
x 1 ! x 2 ! x 3 !L x n ! be given 13 cards out of 52 cards in 52 C 13 ways. Second
Corollary V The number of ways to distribute player can be given 13 cards out of remaining 39 cards (i.e.,
( x 1 + x 2 + x 3 + K + x n ) distinct objects among n 52 − 13 = 39) in 39
C 13 ways. Third player can be given 13
persons in the groups containing x 1 , x 2 , K , x n cards out of remaining 26 cards (i.e., 39 − 13 = 26 ) in 26
C 13
objects ways and fourth player can be given 13 cards out of
remaining 13 cards (i.e., 26 − 13 = 13) in 13 C 13 ways.
= (Number of ways to divide) × (Number of groups)
( x + x 2 + x 3 + ... + x n ) ! Hence, required number of ways
= 1 × n! = C 13 ×
52 39
C 13 × 26
C 13 × 13C 13
x 1 ! x 2 ! x 3 !K x n !
52! 39 ! 26! 52!
(b) Division of Objects Into Groups of Equal Size = × × × 1=
13! 39 ! 13! 26! 13! 13! (13!)4
The number of ways in which mn distinct objects can
be divided equally into m groups, each containing n (ii) Here, order of group is not important, then the
objects and number of ways in which 52 different cards can be
(i) If order of groups is not important is. divided equally into 4 groups is

 (mn ) !  1 52
= × .
 (n !) m  m !
(ii) If order of groups is important is. 13 13
 (mn ) ! 1  (mn ) ! 13 13
 ×  ×m ! = .
 (n !) m m ! (n !) m (Tree diagram)

Note Division of 14n objects into 6 groups of 2n, 2n, 2n, 2n, 3n, 3n, 52!
 ( 14 n)!  4 ! (13!)4
 
 ( 2n)! ( 2n)! ( 2n)! ( 2n)! ( 3n)! ( 3n)!  ( 14 n) 1
size is = 4 2
× Aliter Each group will get 13 cards. Now, first group can be
4 ! 2! (( 2n)!) (( 3n)!) 4 ! 2! given 13 cards out of 52 cards in 52 C 13 ways. Second group
Now, the distribution ways of these 6 groups among 6 persons is can be given 13 cards out of remaining 39 cards (i.e.,
( 14 n)!
×
1
× 6!=
( 14 n)!
× 15
52 − 13 = 39) in 39 C 13 ways. Third group can be given 13
4 2
[( 2n)!)] [( 3n)!] 4 ! 2! [( 2n)!]4 [( 3n)!] 2 cards out of remaining 26 cards
(i.e., 39 − 13 = 26) in 26 C 13 ways and fourth group can be
y Example 73. In how many ways can a pack of 52 given 13 cards out of remaining 13 cards (i.e., 26 − 13 = 13)
in 13 C 13 ways. But the all (four) groups can be interchanged
cards be
in 4! ways. Hence, the required number of ways
(i) distributed equally among four players in order?
1
(ii) divided into four groups of 13 cards each? = 52 C 13 × 39C 13 × 26C 13 × 13C 13 ×
4!
(iii) divided into four sets, three of them having 17 cards 52! 39 ! 26! 1 52!
= × × ×1× =
each and fourth just one card? 13! 39 ! 13! 26! 13! 13! 4 ! (13!)4 4 !
Sol. (i) Here, order of group is important, then the numbers
(iii) First, we divide 52 cards into two sets which contains
of ways in which 52 different cards can be divided
1 and 51 cards respectively, is
equally into 4 players is
52!
1! 51!
Chap 05 Permutations and Combinations 383

12! × 2 12 ⋅ 11 ⋅ 10 ⋅ 9 ⋅ 8 ⋅ 7 !⋅ 2
52 = = = 1584.
5! × 7 ! 5 ⋅ 4 ⋅3 ⋅2 ⋅1 ⋅7 !
II Part Here, order is not important, then the number of
17 1 ways in which 12 different balls can be divided into three
17 17 groups of 5,4 and 3 balls respectively, is
12! 12 ⋅ 11 ⋅ 10 ⋅ 9 ⋅ 8 ⋅ 7 ⋅ 6 ⋅ 5!
Now, 51 cards can be divided equally in three sets each = = = 27720
contains 17 cards (here order of sets is not important) 5! 4 ! 3! 5! ⋅ 4 ⋅ 3 ⋅ 2 ⋅ 1 ⋅ 3 ⋅ 2 ⋅ 1
51!
in ways.
3! (17 !)3 12
Hence, the required number of ways
52! 51!
= ×
1! 51! 3 !(17 !)3
5 2
52! 52! 4
= =
1! 3!(17 )3 (17 !) 3 3! (Tree diagram)

Aliter First set can be given 17 cards out of 52 cards in Aliter First group can be given 5 balls out of 12 balls in
52 12
C 17 . Second set can be given 17 cards out of remaining 35 C 5 ways. Second group can be given 4 balls out of
cards (i.e., 52 − 17 = 35) in 35 C 17 . Third set can be given 17 remaining 7 balls (12 − 5 = 7 ) in 7 C 4 and 3 balls can be
cards out of remaining 18 cards (i.e., 35 − 17 = 18) in 18 C 17 given out of remaining 3 balls in 3 C 3 .
and fourth set can be given 1 card out of 1 card in 1C 1. But
Hence, the required number of ways (here order of groups
the first three sets can interchanged in 3! ways. Hence, the are not important)
total number of ways for the required distribution
= 12
C 5 × 7C 4 × 3C 3
1
= 52 C 17 × 35C 17 × 18C 17 × 1C 1 × !
3 12! 7!
= × ×1
52! 35! 18! 1 (52)! 5! 7 ! 4 ! 3!
= × × ×1× =
17 ! 35! 17 ! 1! 17 ! 18! 3! (17 !)3 3! 12!
=
5! 4 ! 3!
y Example 74. In how many ways can 12 different balls
be divided between 2 boys, one receiving 5 and the y Example 75. In how many ways can 16 different
other 7 balls? Also, in how many ways can these 12 balls books be distributed among three students A, B, C so
be divided into groups of 5, 4 and 3 balls, respectively? that B gets 1 more than A and C gets 2 more than B ?
Sol. I Part Here, order is important, then the number of ways Sol. Let A gets n books, then B gets n + 1 and C gets n + 3.
in which 12 different balls can be divided between two
Now, n + (n + 1) + (n + 3) = 16
boys which contains
5 and 7 balls respectively, is ⇒ 3n = 12
∴ n=4
12
16

5 7
(Tree diagram)
4 7
5
12! 12 ⋅ 11 ⋅ 10 ⋅ 9 ⋅ 8 ⋅ 7 !
= × 2! = ⋅ 2 = 1584 (Tree diagram)
5! 7 ! (5⋅ 4 ⋅ 3 ⋅ 2⋅ 1)7 !
⇒ A , B, C gets 4, 5 and 7 books, respectively.
Aliter First boy can be given 5 balls out of 12 balls in 12 C 5 .
Second boy can be given 7 balls out of 7 balls (i.e., Hence, the total number of ways for the required
12 − 5 = 7) but there order is important boys interchange by distribution
(2 types), then required number of ways 16!
=
12! 4 ! 5! 7 !
= 12 C 5 × 7C 7 × 2! = × 1 × 2!
5! 7 !
384 Textbook of Algebra

y Example 76. In how many ways can 9 different The number of division ways for tree diagrams (i), (ii) and
books be distributed among three students if each (iii) are
9! 1 9! 9! 1
receives atleast 2 books? × , and × , respectively.
Sol. If each receives atleast 2 books, then the division as (2!)2 (5!) 2! 2! 3! 4 ! (3!)3 3!
shown by tree diagrams Hence, the total number of ways of distribution of these
groups among 3 students is
9 9 9  9! 1 9! 9! 1
 2 × + + 3
×  × 3!
(2!) (5!) 2 ! 2 ! 3 ! 4 ! (3!) 3 !

2 2 3 = [378 + 1260 + 280] × 6


2 5 3 4 3 3
= 11508
(i) (ii) (iii)

#L Exercise for Session 5


1. There are 3 oranges, 5 apples and 6 mangoes in a fruit basket (all fruits of same kind are identical). Number of
ways in which fruits can be selected from the basket, is
(a) 124 (b) 125 (c) 167 (d) 168
2. In a city no two persons have identical set of teeth and there is no person without a tooth. Also, no person has
more than 32 teeth. If we disguard the shape and size of tooth and consider only the positioning of the teeth,
the maximum population of the city is
(a) 2 32 (b) (32)2 − 1 (c) 232 − 1 (d) 231

3. If a1, a 2, a 3, L , a n + 1 be (n + 1) different prime numbers, then the number of different factors (other than 1) of
a1m ⋅ a 2 ⋅ a 3, K , a n + 1, is
(a) m + 1 (b) (m + 1) 2 n (c) m ⋅ 2n + 1 (d) None of these

4. Number of proper factors of 2400 is equal to


(a) 34 (b) 35
(c) 36 (d) 37

5. The sum of the divisors of 2 5 ⋅ 34 ⋅ 52 is


(a) 32 ⋅ 71 ⋅ 112 (b) 32 ⋅ 71 ⋅ 112 ⋅ 31
(c) 3 ⋅ 7 ⋅ 11⋅ 31 (d) None of these

6. The number of proper divisors of 2p ⋅ 6q ⋅ 21r , ∀ p, q , r ∈ N, is


(a) (p + q + 1) (q + r + 1) (r + 1) (b) (p + q + 1) (q + r + 1) (r + 1) − 2
(c) (p + q ) (q + r ) r − 2 (d) (p + q ) (q + r )r

7. The number of odd proper divisors of 3p ⋅ 6q ⋅ 15r , ∀ p, q , r ∈ N, is


(a) (p + 1) (q + 1) (r + 1) − 2 (b) (p + 1) (q + 1) (r + 1) − 1
(c) (p + q + r + 1) (r + 1) − 2 (d) (p + q + r + 1) (r + 1) − 1

8. The number of proper divisors of 1800, which are also divisible by 10, is
(a) 18 (b) 27 (c) 34 (d) 43

9. Total number of divisors of 480 that are of the form 4n + 2, n ≥ 0, is equal to


(a) 2 (b) 3 (c) 4 (d) 5

10. Total number of divisors of N = 2 ⋅ 3 ⋅ 5 5 4 10


⋅ 7 that are of the form 4n + 2, n ≥ 1, is equal to
6

(a) 54 (b) 55 (c) 384 (d) 385


Chap 05 Permutations and Combinations 385

11. Total number of divisors of N = 3 5 ⋅ 5 7⋅ 79 that are of the form 4n + 1, n ≥ 0 is equal to


(a) 15 (b) 30 (c) 120 (d) 240

12. Number of ways in which 12 different books can be distributed equally among 3 persons, is
12 ! 12 ! 12 ! 12 !
(a) (b) (c) (d)
(4 !)3 (3 !)4 (4 !)4 (3 !)3

13. Number of ways in which 12 different things can be distributed in 3 groups, is


12 ! 12 ! 12 ! 12 !
(a) (b) (c) (d)
(4 !)3 3 ! (4 !)3 4 ! (3 !)3 (3 !)4

14. Number of ways in which 12 different things can be distributed in 5 sets of 2, 2, 2, 3, 3, things is
12 ! 12 ! 5 ! 12 ! 12 ! 5 !
(a) 2 3
(b) 2 3
(c) 3 4
(d)
(3 !) (2 !) (3 !) (2 !) (3 !) (2 !) (3 !)2 (2 !)4

15. Number of ways in which 12 different things can be divided among five persons so that they can get 2, 2, 2, 3,
3 things respectively, is
12 ! 12 ! 5 ! 12 ! 12 ! 5 !
(a) (b) (c) (d)
(3 !)2 (2 !)3 (3 !)2 (2 !)3 (3 !)2 (2 !)4 (3 !)2 (2 !)4

16. The total number of ways in which 2n persons can be divided into n couples, is
2n ! 2n ! 2n !
(a) (b) (c) (d) None of these
(n !)2 (2n !)n n ! (2n !)2

17. n different toys have to be distributed among n children. Total number of ways in which these toys can be
distributed so that exactly one child gets no toy, is equal to
(a) n ! (b) n ! nC2 (c) (n − 1) ! nC2 (d) n ! n −1C2

18. In how many ways can 8 different books be distributed among 3 students if each receives atleast 2 books?
(a) 490 (b) 980 (c) 2940 (d) 5880
Session 6
Arrangement in Groups, Multinomial Theorem,
Multiplying Synthetically

Arrangement in Groups All 5 balls can be arranged by 5! ways and boxes can be
3!
(a) The number of ways in which n different things arranged in each system by .
2!
can be arranged into r different groups is 3! 3!
Hence, required number of ways = 5! × + 5! ×
r (r + 1 )(r + 2) K (r + n − 1 ) or n !⋅ n − 1 C r − 1 2! 2!
according as blank groups are or are not admissible. = 120 × 3 + 120 × 3 = 720
Proof (b) The number of ways in which n different things
(i) Let n letters a 1 , a 2 , a 3 , K, a n be written in a row can be distributed into r different groups is
in any order. All the arrangements of the letters r n −r C 1 (r − 1 ) n + r C 2 (r − 2) n − K + ( −1 ) r −1 ⋅ r C r −1
in r, groups, blank groups being admissible, can Or
be obtained thus, place among the letters (r − 1)
r

marks of partition and arrange the (n + r − 1) ∑ ( −1 ) p ⋅ r C p ⋅ (r − p )n


p=0
things (consisting of letters and marks) in all Or
possible orders. Since, (r − 1) of the things are
Coefficient of x in n !(e x − 1 ) r .
n
alike, the number of different arrangements is
(n + r − 1) ! Here, blank groups are not allowed.
= r (r + 1)(r − 2 ) K(r + n − 1).
(r − 1) ! Proof In any distribution, denote the groups by
(ii) All the arrangements of the letters in r groups, g 1 , g 2 , g 3 , K, g r and consider the distributions in
none of the groups being blank, can be obtained which blanks are allowed.
as follows: The total number of these is r n .
(I) Arrange the letters in all possible orders. This can be The number in which g 1 is blank, is (r − 1) n .
done in n ! ways.
Therefore, the number in which g 1 is not blank, is
(II) In every such arrangement, place (r − 1) marks of
partition in (r − 1) out of the (n − 1) spaces between r n − (r − 1) n
the letters. This can be done in n − 1 C r − 1 ways. of these last, the number in which g 2 is blank, is
(r − 1) n − (r − 2 ) n
Hence, the required number is n !⋅ n − 1 C r − 1 .
Therefore, the number in which g 1 , g 2 are not blank, is
y Example 77. In how many ways 5 different balls can r n − 2 (r − 1) n + (r − 2 ) n
be arranged into 3 different boxes so that no box of these last, the number in which g 3 is blank, is
remains empty? (r − 1) n − 2 (r − 2 ) n + (r − 3 ) n
Sol. The required number of ways = 5!⋅ 5 − 1 C 3 − 1 = 5!⋅ 4C 2 Therefore, the number in which g 1 , g 2 , g 3 are not
 4 ⋅ 3 blank, is
= (120) ⋅   = 720
 1 ⋅2  r n − 3 (r − 1) n + 3 (r − 2 ) n − (r − 3 ) n
Aliter
This process can be continued as far as we like and it
Each box must contain atleast one ball, since no box
is obvious that the coefficients are formed as in a
remains empty. Boxes can have balls in the following
systems binomial expansion.
Hence, the number of distributions in which no one
Box I II III Box I II III
of x assigned groups is blank, is
Number of 1 1 3 Or Number of 1 2 2
balls balls r n − x C 1(r − 1)n + x C 2 (r − 2)n − K + ( −1)x (r − x )n
Chap 05 Permutations and Combinations 387

when x = r , then Box I II III Box I II III


r n − r C 1 (r − 1) n + r C 2 (r − 2 ) n −K + ( −1) r − 1 ⋅ r C r − 1 Number of 1 1 3 Or Number of 1 2 2
balls balls
(r − (r − 1)) n + ( −1) r ⋅ r C r (r − r ) n
Or The number of ways to distribute the balls in I system
r − C 1 (r − 1) + C 2 (r − 2)n − K + ( −1)r − 1 ⋅ r C r − 1
n r n r
= 5C 1 × 4C 1 × 3C 3
Aliter ∴Thetotalnumberofwaystodistribute 1,1,3ballstotheboxes
3!
By Principle of Inclusion and Exclusion = 5C 1 × 4C 1 × 3C 3 × = 5 × 4 × 1 × 3 = 60
2!
Let Ai denotes the set of distribution of things, if ith and the number of ways to distribute the balls in II system
group gets nothing. Then, n( Ai ) = (r − 1) n = 5C 1 × 4C 2 × 2C 2
[as n things can be distributed among (r − 1) groups ∴ The total number of ways to distribute 1, 2, 2 balls to the
in (r − 1) n ways] boxes 3!
= 5C 1 × 4C 2 × 2C 2 ×
Then, n ( Ai ∩ A j ) represents number of distribution 2!
ways in which groups i and j get no object. Then, = 5 × 6 × 1 × 3 = 90
∴ The required number of ways = 60 + 90 = 150
n ( Ai ∩ A j ) = (r − 2 ) n
Also, n ( Ai ∩ A j ∩ Ak ) = (r − 3 ) n y Example 79. In how many ways can 5 different
This process can be continued, then the required books be tied up in three bundles?
number is 1 5 3
Sol. The required number of ways = ( 3 − C 1 ⋅ 25 + 3C 2 ⋅ 15 )
n ( A1 ′ A2 ′ ∩ K ∩ Ar ′ ) 3!
= n (U ) − n( A1 ∪ A2 ∪ K ∪ Ar ) 150
= = 25
= r n − { ∑ n( Ai ) − ∑ n ( Ai ∩ A j )
6

y Example 80. If n ( A ) = 5 and n (B ) = 3, find number


+ ∑ n ( Ai ∩ A j ∩ Ak ) K
+ ( −1) n ∑ n ( A1 ∩ A2 ∩ K ∩ Ar )}
of onto mappings from A to B.
Sol. We know that in onto mapping, each image must be
= r n − { r C 1 (r − 1) n − r C 2 (r − 2 ) n assigned atleast one pre-image.
This is equivalent to number of ways in which 5 different
+ r C 3 (r − 3 ) n − K+ ( −1) r ⋅ r C r − 1 }
balls (pre-images) can be distributed in 3 different boxes
= r n − r C 1 (r − 1) n + r C 2 (r − 2 ) n (images), if no box remains empty. The total number of
onto mappings from A to B
− r C 3 (r − 3 ) n + K + ( −1) r − 1 ⋅ r C r − 1 ⋅ 1
= 35 − 3C 1(3 − 1)5 + 3C 2 (3 − 2)5
r
p
Note Coefficient of x r in epx = . = 243 − 96 + 3 = 150
r!
(c) The number of ways in which n identical things
y Example 78. In how many ways 5 different balls can be can be distributed into r different groups is
distributed into 3 boxes so that no box remains empty? n +r −1
C r − 1 or n −1
Cr − 1
Sol. The required number of ways
According, as blank groups are or are not
= 35 − 3C 1(3 − 1)5 + 3C 2 (3 − 2)5 − 3C 3 (3 − 3)5
admissible.
= 243 − 96 + 3 − 0 = 150 Proof
Or If blank groups are not allowed Any such
Coefficient of x 5 in 5!(e x − 1)3 distribution can be effected as follows: place the n
things in a row and put marks of partition in a
= Coefficient of x 5 in 5!(e 3 x − 3e 2 x + 3e x − 1)
selection of (r − 1) out of the (n − 1) spaces between
 35 25 1 them. This can be done in n − 1 C r − 1 .
= 5!  − 3 × + 3 ×  = 35 − 3⋅25 + 3 = 243 − 96 + 3 = 150
 5! 5! 5! 
If blank groups are allowed The number of
Aliter distribution is the same as that of (n + r ) things of the
Each box must contain atleast one ball, since number box same sort into r groups with no blank groups. For
remains empty. Boxes can have balls in the following such a distribution can be effected thus, put one of the
systems
388 Textbook of Algebra

20 + 4 − 1
(n + r ) things into each of the r groups and distribute Hence, total number of Sols = C4 −1
the remaining n things into r groups, blank lots being 23 ⋅ 22 ⋅ 21
allowed. Hence, the required number is n + r − 1 C r − 1 . = C3 =
23
= 1771
1 ⋅2 ⋅3
Aliter The number of distribution of n identical (ii) Since, x + y + z + w = 20 …(i)
Here, x ≥ 1, y ≥ 1, z ≥ 1, w ≥ 1
things into r different groups is the coefficient of x n
or x− 1 ≥ 0, y − 1 ≥ 0, z − 1 ≥ 0, w − 1 ≥ 0
in (1 + x + x 2 + K + ∞ ) r or in ( x + x 2 + x 3 + K + ∞ ) r
Let x1 = x − 1 ⇒ x = x1 + 1
according as blank groups are or are not allowed. y1 = y − 1 ⇒ y = y1 + 1
These expressions are respectively equal to z1 = z − 1 ⇒ z = z1 + 1
(1 − x ) −r and x r (1 − x ) −r w1 = w − 1 ⇒ w = w1 + 1
Then, from Eq. (i), we get
Hence, coefficient of x n in two expressions are
x 1 + 1 + y1 + 1 + z1 + 1 + w 1 + 1 = 20
n +r −1 n −1
C r − 1 and C r − 1 , respectively. ⇒ x 1 + y1 + z1 + w 1 = 16
and x 1 ≥ 0, y1 ≥ 0, z1 ≥ 0, w 1 ≥ 0
y Example 81. In how many ways 5 identical balls can Hence, total number of Solutions = 16 + 4 − 1C 4 − 1
be distributed into 3 different boxes so that no box
19 ⋅ 18 ⋅ 17
remains empty? = 19C 3 = = 57 ⋅ 17 = 969
1 ⋅2 ⋅3
5 −1 4 ⋅3 Aliter
Sol. The required number of ways = C 3 − 1 = 4C 2 = =6
1 ⋅2 Part (ii) Q x + y + z + w = 20
Aliter Each box must contain atleast one ball, since no x ≥ 1, y ≥ 1, z ≥ 1, w ≥ 1
box remains empty. Boxes can have balls in the following Hence, total number of solutions
20 − 1
systems. = C 4 − 1 = 19C 3 = 969
Box I II III Box I II III
Number 1 1 3 Or Number of 1 2 2
y Example 84. How many integral solutions are there
of balls balls to x + y + z + t = 29, when x ≥ 1, y > 1, z ≥ 3 and t ≥ 0 ?
Sol. Since, x + y + z + t = 29 …(i)
Here, balls are identical but boxes are different the number
of combinations will be 1 in each systems. and x, y, z, t are integers
3! 3! ∴ x ≥ 1, y ≥ 2, z ≥ 3, t ≥ 0
∴ Required number of ways = 1 × + 1 × = 3 + 3 = 6
2! 2! ⇒ x − 1 ≥ 0, y − 2 ≥ 0, z − 3 ≥ 0, t ≥ 0
Let x 1 = x − 1, x 2 = y − 2, x 3 = z − 3
y Example 82. Four boys picked up 30 mangoes. In or x = x 1 + 1, y = x 2 + 2, z = x 3 + 3 and then x 1 ≥ 0, x 2 ≥ 0,
how many ways can they divide them, if all mangoes x 3 ≥ 0, t ≥ 0
be identical? From Eq. (i), we get
Sol. Clearly, 30 mangoes can be distributed among 4 boys such x 1 + 1 + x 2 + 2 + x 3 + 3 + t = 29
that each boy can receive any number of mangoes. ⇒ x 1 + x 2 + x 3 + t = 23
30 + 4 − 1
Hence, total number of ways = C4 −1 Hence, total number of solutions = 23 + 4 − 1C 4 − 1
33 ⋅ 32 ⋅ 31 26 ⋅ 25 ⋅ 24
= 33
C3 = = 5456 = 26
C3 = = 2600
1 ⋅2 ⋅3 1 ⋅2 ⋅3
Aliter
y Example 83. Find the positive number of solutions of Q x + y + z + t = 29 …(i)
x + y + z + w = 20 under the following conditions and x ≥ 1, y − 1 ≥ 1, z − 2 ≥ 1, t + 1 ≥ 1
(i) Zero value of x , y, z and w are included. Let x 1 = x , y1 = y − 1, z1 = z − 2, t 1 = t + 1
(ii) Zero values are excluded. or x = x 1, y = y1 + 1, z = z1 + 2, t = t 1 − 1
Sol. (i) Since, x + y + z + w = 20 and then x 1 ≥ 1, y1 ≥ 1, z1 ≥ 1, t 1 ≥ 1
From Eq. (i), x 1 + y1 + 1 + z1 + 2 + t 1 − 1 = 29
Here, x ≥ 0, y ≥ 0, z ≥ 0, w ≥ 0
⇒ x 1 + y1 + z1 + t 1 = 27
The number of Sols of the given equation in this case Hence, total number of solutions = 27 − 1
C4 −1 = 26
C3
is same as the number of ways of distributing 20
things among 4 different groups. 26 ⋅ 25 ⋅ 24
= = 2600
1 ⋅2 ⋅3
Chap 05 Permutations and Combinations 389

y Example 85. How many integral Solutions are there = Coefficient of x n in the expansion of
to the system of equations x 1 + x 2 + x 3 + x 4 + x 5 = 20 x lr (1 + x + x 2 + K + x m − l ) r
and x 1 + x 2 = 15, when x k ≥ 0?
= Coefficient of x n − lr in the expansion of
Sol. We have, x 1 + x 2 + x 3 + x 4 + x 5 = 20 …(i)
(1 + x + x 2 + K + x m − l ) r
and x 1 + x 2 = 15 …(ii)
Then, from Eqs. (i) and (ii), we get two equations = Coefficient of x n − lr in the expansion of
r
x3 + x4 + x5 = 5 …(iii)  1 ⋅ (1 − x m − l + 1 ) 
 
x 1 + x 2 = 15 …(iv)  (1 − x ) 
and given x 1 ≥ 0, x 2 ≥ 0, x 3 ≥ 0, x 4 ≥ 0 and x 5 ≥ 0
[sum of m − l + 1 terms of GP]
Then, number of solutions of Eq. (iii)
n − lr
= 5 + 3 −1
C 3 − 1 = 7C 2
= Coefficient of x in the expansion of

7 ⋅6 (1 − x m − l + 1 ) r (1 − x ) −r
= = 21
1 ⋅2
y Example 87. In how many ways can three persons,
and number of solutions of Eq. (iv) each throwing a single dice once, make a sum of 15 ?
15 + 2 − 1
= C 2 − 1 = 16C 1 = 16 Sol. Number on the faces of the dice are 1, 2, 3, 4, 5, 6 (least
Hence, total number of solutions of the given system of number 1, greatest number 6)
equations Here, l = 1, m = 6, r = 3 and n = 15
= 21 × 16 = 336 ∴ Required number of ways = Coefficient of x 15 − 1 × 3 in the
expansion of (1 − x 6 )3 (1 − x )−3
y Example 86. Find the number of non-negative
integral solutions of 3x + y + z = 24. = Coefficient of x 12 in the expansion of
Sol. We have, (1 − 3x 6 + 3x 12 )(1 + 3C 1 x + 4C 2 x 2 + K + 8C 6 x 6 + K
3x + y + z = 24 and given x ≥ 0, y ≥ 0, z ≥ 0 + 14C 12 x 12 + K )
Let x =k = 14C 12 − 3 × 8C 6 + 3 = 14C 2 − 3 × 8C 2 + 3
∴ y + z = 24 − 3k …(i) = 91 − 84 + 3 = 10
Here, 24 ≥ 24 − 3k ≥ 0[Q x ≥ 0]
Hence, 0 ≤ k ≤ 8 y Example 88. In how many ways in which an
The total number of integral solutions of Eq. (i) is examiner can assign 30 marks to 8 questions, giving
24 − 3k + 2 − 1
C2 −1 = 25 − 3k
C 1 = 25 − 3k not less than 2 marks to any question.
Sol. If examiner given marks any seven question 2 (each)
Hence, the total number of Sols of the original equation marks, then marks on remaining questions given by
8 8 8
examiner = − 7 × 2 + 30 = 16
= ∑(25 − 3k ) = 25 ∑1 − 3 ∑k If x i are the marks assigned to ith question, then
k =0 k =0 k =0
x 1 + x 2 + x 3 + K + x 8 = 30 and 2 ≤ x i ≤ 16
8 ⋅9
= 25 ⋅ 9 − 3 ⋅ = 225 − 108 = 117 for i = 1, 2, 3, K, 8.
2
Here, l = 2,m = 16, r = 8 and n = 30
(d) The number of ways in which n identical things ∴ Required number of ways
can be distributed into r groups so that no group 30 − 2 × 8
= Coefficient of x in the expansion of
contains less than l things and more than m
things (l < m ) is coefficient of x n − lr in the (1 − x 16 − 2 + 1 )8 (1 − x )−8
expansion of (1 − x m − l + 1 ) r (1 − x ) −r . = Coefficient of x 14 in the expansion of

Proof Required number of ways (1 − x 15 )8 (1 + 8C 1x + 9C 2 x 2 + K+ 21C 14 x 14 + K )

= Coefficient of x n in the expansion of = Coefficient of x 14 in the expansion of


(1 + 8C 1x + 9C 2 x 2 + K + 21C 14 x 14 + K )
(x l + x l + 1 + x l + 2 + K + x m )r
= 21C 14 = 21C 7
[Q no group contains less than l things and
Note Coefficient of x r in the expansion of ( 1 − x ) − n is n + r −1
Cr .
more than m things, here r groups]
390 Textbook of Algebra

(e) If a group has n things in which p are identical, [Q the remaining (n − 1) letters can be placed in (n − 1)
then the number of ways of selecting r things envelopes is (n − 1) ! ]
from a group is and n ( Ai ∩ A j ) = 1 × 1 × (n − 2 ) ! [Qi and j can be placed in
r r their corresponding envelopes and remaining (n − 2 )
∑ n−p
C r or ∑ n − p C r ,according asr ≤ p or letters can be placed in (n − 2 ) envelopes in (n − 2 ) ! way]
r =0 r =r − p
r ≥ p. Also, n ( Ai ∩ A j ∩ Ak ) = 1 × 1 × 1 × (n − 3 ) !
Hence, the required number is
y Example 89. A bag has contains 23 balls in which 7 n ( A1 ′ ∩ A2 ′ ∩ A 3 ′ ∩ K ∩ An ′ )
are identical. Then, find the number of ways of
= n (U ) − n ( A1 ∪ A2 ∪ A 3 ∪ K ∪ An )
selecting 12 balls from bag.
= n ! − {∑ n ( Ai ) − ∑ n( Ai ∩ A j )
Sol. Here, n = 23, p = 7, r = 12 (r > p )
12 + ∑ n ( Ai ∩ A j ∩ Ak ) −K + ( −1) n
∴ Required number of selections = ∑ 16
Cr
r =5 ∑ n ( A1 ∩ A2 ∩ A 3 ∩ K ∩ An )
= C 5 + C 6 + C 7 + C 8 + C 9 + C 10 + C 11 + C 12
16 16 16 16 16 16 16 16
= n ! − { n C 1 × (n − 1) ! − n C 2 × (n − 2 ) !
= ( 16C 5 + 16C 6 ) + ( 16C 7 + 16C 8 ) + ( 16C 9 + 16C 10 ) + n C 3 × (n − 3 ) ! −K + ( −1) n − 1 × n C n × 1 ! }
+ ( 16C 11 + 16C 12 ) n × (n − 1) ! n(n − 1)
n +1 = n! −  − × (n − 2 ) !
= 17C 6 + 17C 8 + 17C 10 + 17C 12 [Q n C r + n C r −1 = Cr ]  1! 2!
n(n − 1)(n − 2 ) 
= C 11 + C 9 + C 10 + C 12 [Q C r = n C n − r ] × (n − 3 ) ! − K + ( −1) n − 1 × 1
17 17 17 17 n
+
3! 
= ( 17C 11 + 17C 12 ) + ( 17C 9 + 17C 10 )
n ! n ! n ! 
= n ! −  − + − K + ( −1) n ⋅ 1
= 18C 12 + 18C 10 = 18C 6 + 18C 8  1! 2 ! 3 ! 
Derangements Any change in the order of the things in  1 1 1 ( −1) n 

a group is called a derangement. = n ! 1 − + − + K + 


 1! 2 ! 3 ! n! 
Or
When ‘n’ things are to be placed at ‘n’ specific places but Maha Short Cut Method
none of them is placed on its specified position, then we  1 1 1 ( −1) n 
say that the ‘n’ things are deranged. If D n = n ! 1 − + − + K + 
 1! 2 ! 3 ! n! 
Or
Then, D n + 1 = (n + 1 ) D n + ( −1 ) n + 1 , ∀ x ∈ N
Assume a 1 , a 2 , a 3 , K, a n be n distinct things such that their
positions are fixed in a row. If we now rearrange a 1 , a 2 , and D n + 1 = n ( D n + D n − 1 ), ∀ x ∈ N − {1 }
a 3 , K, a n in such a way that no one occupy its original where D 1 = 0
position, then such an arrangement is called a For n = 1, from result I
derangement. D 2 = 2 D 1 + ( −1) 2 = 0 + 1 = 1
Consider ‘n’ letters and ‘n’ corresponding envelops. The For n = 2, from result I
number of ways in which letters can be placed in the D 3 = 3 D 2 + ( −1) 3 = 3 × 1 − 1 = 2
envelopes (one letter in each envelope) so that no letter is
placed in correct envelope is For n = 3, from result I
 1 1 1 1 D 4 = 4 D 3 + ( −1) 4 = 4 × 2 + 1 = 9
n !  1 − + − + K + ( −1) n 
 1! 2 ! 3 ! n ! For n = 4, from result I
Proof n letters are denoted by 1, 2, 3, K, n. Let Ai denote D 5 = 5 D 4 + ( −1) 5 = 5 × 9 − 1 = 44
the set of distribution of letters in envelopes (one letter in For n = 5, from result I
each envelop) so that the
i th letter is placed in the corresponding envelope, then D 6 = 6 D 5 + ( −1) 6 = 6 × 44 + 1 = 265
n ( Ai ) = 1 × (n − 1) ! Note D1 = 0, D2 = 1, D3 = 2, D4 = 9, D5 = 44, D6 = 265 [Remember]
Chap 05 Permutations and Combinations 391

Remark The number of ways to put 2 letters in 2


If r things goes to wrong place out of n things, then ( n − r ) things addressed envelopes so that all are in wrong
goes to original place (here r < n). envelopes
If Dn = Number of ways, if all n things goes to wrong places.
= The number of ways without restriction − The
and Dr = Number of ways, if r things goes to wrong places.
If r goes to wrong places out of n, then ( n − r ) goes to correct
number of ways in which all are in correct envelopes
places. − The number of ways in which 1 letter is in the
Then, Dn = n Cn − r Dr correct envelope
where, Dr = r !  1 − + ! − + K + ( −1) r 
1 1 1 1 = 2! − 1 − 0 = 2 − 1
 1! 2! 3! r !
n =1 …(ii) [from Eq. (i)]
If atleast p things goes to wrong places, then Dn = ∑ Cn − r ⋅ Dr
n

r =p
The number of ways to put 3 letters in 3
addressed envelopes so that all are in wrong
y Example 90. A person writes letters to six friends envelopes
and addresses the corresponding envelopes. In how = The number of ways without restriction − The
many ways can the letters be placed in the envelopes number of ways in which all are in correct envelopes
so that (i) atleast two of them are in the wrong − The number of ways in which 1 letter is in the
envelopes. (ii) all the letters are in the wrong correct envelope − The number of ways in which 2
envelopes. letter are in correct envelope
Sol. (i) The number of ways in which atleast two of them in = 3 ! − 1 − 3 C 1 × 1 − 0 [from Eqs. (i) and (ii)]
the wrong envelopes
6 =2
= ∑ 6
C 6 − r ⋅ Dr [ 3C 1 means that select one envelope to put the letter
r =2
correctly]
= 6
C4 × D 2 + 6C 3 × D 3 + 6C 2 × D 4 + 6C 1
The number of ways to put 4 letters in 4
× D 5 + 6C 0 × D 6 addressed envelopes so that all are in wrong
= 15D 2 + 20D 3 + 15D 4 + 6D 5 + D 6 [from note] envelopes
= 15 × 1 + 20 × 2 + 15 × 9 + 6 × 44 + 265 = The number of ways without restriction − The
= 719
number of ways in which all are in correct envelopes
− The number of ways in which 1 letter is in the
(ii) The number of ways in which all letters be placed in correct envelope − The number of ways in which 2
wrong envelopes = D 6 = 265 [from note] letters are in correct envelopes − The number of ways
Aliter in which 3 letters are in correct envelopes
(i) The number of all the possible ways of putting 6 = 4 ! − 1 − 4C 1 × 2 − 4C 2 × 1 − 4C 3 × 0
letters into 6 envelopes is 6 ! . [from Eqs. (i), (ii) and (iii)]
Number of ways to place all letters correctly into = 24 − 1 − 8 − 6 − 0 = 9 …(iv)
corresponding envelopes = 1
The number of ways to put 5 letters in 5
and number of ways to place one letter in the wrong addressed envelopes so that all are in wrong
envelope and other 5 letters in the write envelope = 0 envelopes
[Q It is not possible that only one letter goes in the = The number of ways without restriction − The
wrong envelope, when if 5 letters goes in the right number of ways in which all are in correct envelopes −
envelope, then remaining one letter also goes in the The number of ways in which 1 letter is in the correct
write envelope] envelopes − The number of ways in which 2 letters are
Hence, number of ways to place atleast two letters in correct envelopes − The number of ways in which 3
goes in the wrong envelopes letters are in correct envelopes − The number of ways
in which 4 letters are in correct envelopes
= 6 ! − 0 − 1 = 6 ! − 1 = 720 − 1 = 719
= 5 ! − 1 − 5C 1 × 9 − 5C 2 × 2 − 5C 3 × 1 − 5C 4 × 0
(ii) The number of ways 1 letter in 1 address envelope, so
that one letter is in wrong envelope = 0 …(i) [from Eqs. (i), (ii), (iii) and (iv)]
[because it is not possible that only one letter goes in = 120 − 1 − 45 − 20 − 10 − 0 = 44
the wrong envelope]
392 Textbook of Algebra

The number of ways to put 6 letters in 6 addressed y Example 91. In how many ways the sum of upper
envelopes so that all are in wrong envelopes faces of four distinct die can be five?
= The number of ways without restriction − The Sol. Here, the number of required ways will be equal to the
number of ways in which all are in correct envelopes number of solutions of x 1 + x 2 + x 3 + x 4 = 5 i.e., 1 ≤ x i ≤ 6
− The number of ways in which 1 letter is in the for i = 1, 2, 3, 4.
correct envelope − The number of ways in which 2 Since, upper limit is 6, which is greater than required
sum, so upper limit taken as infiite. So, number of Sols is
letters are in correct envelopes − The number of ways
equal to coefficient of α 5 in the expansion of
in which 3 letters are in correct envelopes − The (1 + α + α 2 + K + ∞ )4
number of ways in which 4 letters are in correct
envelopes − The number of ways in which 5 letters = Coefficient of α 5 in the expansion of (1 − α )−4
are in correct envelopes. = Coefficient of α 5 in the expansion of
= 6 ! − 1 − 6 C 1 × 44 − 6 C 2 × 9 − 6 C 3 × 2 (1 + 4C 1α + 5C 2 α 2 + K )
− 6C 4 × 1 − 6C 5 × 0 8 ⋅7 ⋅6
= 8C 5 = 8C 3 = = 56
[from Eqs. (i), (ii), (iii), (iv) and (v)] 1 ⋅2 ⋅3
= 720 − 1 − 264 − 135 − 40 − 15 = 720 − 455 = 265 Case II If the upper limit of a variable is less than
the sum required and the lower limit of all variables
Multinomial Theorem is non-negative, then the upper limit of that variable
is that given in the problem.
(i) If there are l objects of one kind, m objects of second
kind, n objects of third kind and so on, then the y Example 92. In an examination, the maximum
number of ways of choosing r objects out of these marks each of three papers is 50 and the maximum
objects (i.e., l + m + n +K) is the coefficient of x r in mark for the fourth paper is 100. Find the number
the expansion of of ways in which the candidate can score 60%
(1 + x + x 2 + x 3 + K + x l )(1 + x + x 2 + K + x m ) marks in aggregate.
(1 + x + x 2 + K + x n ) Sol. Aggregate of marks = 50 × 3 + 100 = 250
Further, if one object of each kind is to be included, 60
then the number of ways of choosing r objects out of ∴ 60% of the aggregate = × 250 = 150
100
these objects (i.e., l + m + n +K) is the coefficient of
Let the marks scored by the candidate in four papers be x 1,
x r in the expansion of x 2 , x 3 and x 4 . Here, the number of required ways will be
( x + x 2 + x 3 + K + x l )( x + x 2 + x 3 + K + x m ) equal to the number of Sols of x 1 + x 2 + x 3 + x 4 = 150 i.e.,
(x + x 2 + x 3 + K + x n ) K 0 ≤ x 1, x 2 , x 3 ≤ 50 and 0 ≤ x 4 ≤ 100.
Since, the upper limit is 100 < required sum (150).
(ii) If there are l objects of one kind, m objects of second
kind, n objects of third kind and so on, then the The number of solutions of the equation is equal to
number of possible arrangements/permutations of r coefficient of α150 in the expansion of
objects out of these objects (i.e., l + m + n +K) is the (α 0 + α1 + α 2 + K + α 50 )3 (α 0 + α1 + α 2 + K + α100 )
coefficient of x r in the expansion of
= Coefficient of α150 in the expansion of
 x x2 xl   x x2 xm 
r ! 1 + + + K +  1 + + +K +  (1 − α 51 )3 (1 − α10 )(1 − α )−4
 1! 2 ! l !   1! 2 ! m! 
 x x2 xn  = Coefficient of α150 in the expansion of
1 + + +K +  K.
 1! 2 ! n!  (1 − 3α 51 + 3α102 )(1 − α101 )(1 + 4C 1α + 5C 2 α 2 + K + ∞ )
= Coefficient of α150 in the expansion of
Different Cases of (1 − 3α 51 − α101 + 3α102 )(1 + 4C 1α + 5C 2 α 2 + K + ∞ )
Multinomial Theorem = C 150 − 3 ×
153 102
C 99 − C 49 + 3 × 51C 48
52

Case I If upper limit of a variable is more than or = C3 − 3 ×


153 102
C3 − 52
C 3 + 3 × 51C 3
equal to the sum required, then the upper limit of = 110556
that variable can be taken as infinite.
Chap 05 Permutations and Combinations 393

Very Important Trick … on multiplying by 1 + α + α 2 + α 3 → To each coefficient


On multiplying p 0 + p 1 α + p 2 α 2 + p 3 α 3 + K + p n α n add 3 preceding coefficients

by (1 + α ), we get 1 4 7 8 7 4 1 ...
p 0 + (p 0 + p1 ) α + (p1 + p2 ) α 2 + (p2 + p 3 ) α 3 + K …on multiplying by 1 + α + α → To each coefficient add 2
2

n −1 n +1
+ (pn − 2 + pn − 1 )α + (pn − 1 + pn )α + pnα n
preceding coefficients.
i.e., we just add coefficient of α r with coefficient of α r − 1 1 5 12 19 22 19 12 ...
(i.e., previous term) to get coefficient α r in product. …on multiplying by 1 + α + α 2 → To each coefficient add 2
Now, coefficient of α = p r − 1 + p r
r
preceding coefficients.
On multiplying p 0 + p 1 α + p 2 α 2 + p 3 α 3 + K + p n α n by ... ... ... ... ... ... 53 ...
(1 + α + α 2 )
Hence, required coefficient is 53.
we get, p 0 + ( p 0 + p 1 ) α + ( p 0 + p 1 + p 2 ) α 2
y Example 94. Find the number of different
+ (p1 + p2 + p 3 ) α 3 + (p2 + p 3 + p 4 ) α 4 + K
selections of 5 letters which can be made from
i.e., to find coefficient of α r in product and add this with 2 5A’s, 4B ’s, 3C’s, 2D’s and 1E
preceding coefficients. Sol. All selections of 5 letters are given by 5th degree terms in
Now, coefficient of α r = p r − 2 + p r − 1 + p r (1 + A + A 2 + A 3 + A 4 + A 5 )(1 + B + B 2 + B 3 + B 4 )
Similarly, in product of p 0 + p 1 α + p 2 α 2 + K with (1 + C + C 2 + C 3 )(1 + D + D 2 )(1 + E )
(1 + α + α 2 + α 3 ), the coefficient of α r in product will be ∴ Number of 5 letter selections
p − 3 + pr − 2 + pr − 1 + pr = Coefficient of α 5 in (1 + α + α 2 + α 3 + α 4 + α 5 )
1r 444 24443
3 preceding coefficients (1 + α + α 2 + α 3 + α 4 )(1 + α + α 2 + α 3 )
and in product of p 0 + p 1 α + p 2 α 2 + K with (1 + α + α 2 )(1 + α )
( 1 + α + α 2 + α 3 + α 4 ), the coefficient of α r in product Multiplying synthetically
will be p r − 4 + p r − 3 + p r − 2 + p r − 1 + p r 1 α α 2
α3 α4 α 5 ...
14444 4244444 3
4 preceding coefficients 1 1 1 1 1 1

Finally, in product of p 0 + p 1 α + p 2 α + K with 2 –––––––––––––––––––––––––––––– × 1 + α + α + α 2 + α 3+ α 4

(1 + α + α 2 + α 3 + K + upto ∞ ), the coefficient of α r in 1 2 3 4 5 5 ...


–––––––––––––––––––––––––––––– × 1 + α + α 2 + α 3
product will be p 0 + p 1 + p 2 + K + p r − 1 + p r
1444424444 3 1 3 6 10 14 17 ...
all preceding coefficients
–––––––––––––––––––––––––––––– ×1 + α + α 2
y Example 93. Find the coefficient of α in the 6
1 4 10 19 30 41 ...
product (1 + α + α 2 )(1 + α + α 2 )(1 + α + α 2 + α 3 ) –––––––––––––––––––––––––––––– × 1 + α
(1 + α )(1 + α )(1 + α ). 1 5 14 29 49 71
Sol. The given product can be written as Hence, required coefficient is 71.
(1 + α + α 2 )(1 + α + α 2 )(1 + α + α 2 + α 3 )(1 + α )3
y Example 95. Find the number of combinations and
or (1 + α + α 2 )(1 + α + α 2 )(1 + α + α 2 + α 3 )
permutations of 4 letters taken from the word
( 1 + 3α + 3α 2 + α 3 ) EXAMINATION.
Sol. There are 11 letters

Multiplying Synthetically A, A, N, N, X, M, T, O.
Then, number of combinations
= coefficient of x 4 in (1 + x + x 2 )3 (1 + x )5
1 α α 2
α 3
α 4
α 5
α 6
...
[Q2A’s, 2I ’s, 2N’s, 1E, 1X, 1M, 1T and 1O]
1 3 3 1 0 0 0
= Coefficient of x 4 in {(1 + x )3 + x 6 + 3(1 + x )2 x 2
394 Textbook of Algebra

+ 3(1 + x )x 4 }(1 + x )5 A X A O
= Coefficient of x in 4
7 ×6
∴ Number of selections = 3 C 1 × 7 C 2 = 3 × = 63
{(1 + x ) + x (1 + x ) + 3x (1 + x ) + 3x (1 + x ) }
8 6 5 2 7 4 6 1×2
4!
8 ⋅7 ⋅6 ⋅5 7 ⋅6 and number of permutations = 63 ⋅ = 756
= 8C 4 + 0 + 3 ⋅ 7C 2 + 3 = + 3⋅ + 3 = 70 + 63 + 3 2!
1 ⋅2 ⋅3 ⋅ 4 1 ⋅2 From Case I, II and III
= 136
The required number of combinations = 70 + 3 + 63 = 136
and number of permutations
3 and number of permutations = 1680 + 18 + 756 = 2454
 x x2  x
5
= Coefficient of x 4 in 4 ! 1 + +  1 +  Note Number of combinations and permutations of 4 letters
 1! 2!   1!  taken from the word MATHEMATICS are 136 and 2454
3 respectively, as like of EXAMINATION.
 x 2
= Coefficient of x 4 in 4 ! 1 + x +  (1 + x )
5
 2
Number of Solutions with the
= Coefficient of x 4 in
 x6 3 3 
Help of Multinomial Theorem
4 ! (1 + x )3 + + (1 + x )2 x 2 + x 4 (1 + x ) (1 + x )5 Case I If the equation
 8 2 4 
α + 2 β + 3γ + K + q θ = n …(i)
= Coefficient of x 4 in
(a) If zero included, the number of solution of Eq. (i)
 x6 3 3 
4 ! (1 + x )8 + (1 + x )5 + x 2 (1 + x )7 + x 4 (1 + x )6  = Coefficient of x n in (1 + x + x 2 + K )
 8 2 4 
(1 + x 2 + x 4 + K )(1 + x 3 + x 6 + K ) K
 3 3  8 ⋅7 ⋅6 ⋅5 3 7 ⋅6 3 
= 4 !  8 C 4 + 0 + ⋅ 7C 2 +  = 24  + ⋅ +  (1 + x q + x 2q + K )
 2 4 1 ⋅2 ⋅3 ⋅ 4 2 1 ⋅2 4 
= 8 ⋅ 7 ⋅ 6 ⋅ 5 + 6(3 ⋅ 7 ⋅ 6) + 6 ⋅ 3 = 1680 + 756 + 18 = 2454 = Coefficient of x n in
Aliter There are 11 letters: (1 − x ) −1 (1 − x 2 ) −1 (1 − x 3 ) −1 K (1 − x q ) −1
A, A, I, I, N, N, E, X, M, T, O (b) If zero excluded, then the number of solutions of Eq. (i)
The following cases arise: = Coefficient of x n in ( x + x 2 + x 3 + K )
Case I All letters different The required number of ( x 2 + x 4 + x 6 + K )( x 3 + x 6 + x 9 + K )
choosing 4 different letters from 8 different (A, I, N, E, X, K ( x q + x 2q + K )
M, T, O) types of the letters
8 ⋅7 ⋅6 ⋅5 = Coefficient of x n in x 1 + 2 + 3 + K + q (1 − x ) −1
= 8C 4 = = 70
1⋅2 ⋅3 ⋅ 4 (1 − x 2 ) −1 (1 − x 3 ) −1 K (1 − x q ) −1
and number of permutations = 8 P 4 = 8 ⋅ 7 ⋅ 6 ⋅ 5 = 1680 q (q + 1 )
n−
= Coefficient of x 2 in
Case II Two alike of one type and two alike of
another type This must be 2A’s, 2I’s or 2I’s, 2N’s, or (1 − x ) (1 − x ) (1 − x 3 ) −1 K(1 − x q ) −1
−1 2 −1

2N’s, 2A’s.
y Example 96. Find the number of non-negative
∴ Number of selections = 3 C 2 = 3 integral solutions of x 1 + x 2 + x 3 + 4 x 4 = 20.
For example, [for arrangements] Sol. Number of non-negative integral solutions of the
given equation
A A I I
= Coefficient of x 20 in (1 − x )−1(1 − x )−1(1 − x )−1(1 − x 4 )−1
4! = Coefficient of x 20 in (1 − x )−3 (1 − x 4 )−1
and number of permutations = 3 ⋅ = 18
2! 2!
= Coefficient of x 20 in (1 + 3C 1 + 4C 2 x 2 + 5C 3 x 3 + 6C 4 x 4
Case III Two alike and two different This must be
+ K + 10C 8 x 8 + K + 14C 12 x 12 + K + 18C 16 x 16 + K
2A’s or 2I’s or 2N’s
+ C 20 x 20 + K )(1 + x 4 + x 8 + x 12 + x 16 + x 20 + K )
22
and for each case 7 different letters.
= 1 + 6C 4 + 10C 8 + 14C 12 + 18C 16 + 22
C 20
For example, for 2A’s, 7 differents’s are I, N, E, X, M, T, O
For example, [for arrangements] = 1 + 6C 2 + 10C 2 + 14C 2 + 18C 2 + 22
C2
Chap 05 Permutations and Combinations 395

 6 ⋅ 5  10 ⋅ 9   14 ⋅ 13  18 ⋅ 17   22 ⋅ 21 Sol. The number of ways of distributing blankets is equal to


=1+   +   +  +  +  the number of solutions of the equation 3x + 2y + z = 15,
 1 ⋅ 2  1 ⋅ 2   1 ⋅ 2   1 ⋅ 2   1 ⋅ 2 
where x ≥ 1, y ≥ 1, z ≥ 1 which is equal to coefficient
= 1 + 15 + 45 + 91 + 153 + 231 = 536 of α15 in
y Example 97. Find the number of positive unequal (α 3 + α 6 + α 9 + α12 + α15 + ...)
integral solutions of the equation x + y + z + w = 20. × (α 2 + α 4 + α 6 + α 8 + α10 + α12 + α14 +...)
Sol. We have, x + y + z + w = 20 …(i) × (α + α 2 + α 3 + ... + α15 + ...)
Assume x < y < z < w . Here, x, y, z , w ≥ 1 = Coefficient of α 9 in (1 + α 3 + α 6 + α 9 )
Now, let x = x 1, y − x = x 2 , z − y = x 3 and w − z = x 4 × (1 + α 2 + α 4 + α 6 + α 8 )
∴ x = x 1, y = x 1 + x 2 , z = x 1 + x 2 + x 3 and
× (1 + α + α 2 + α 3 + α 4 + α 5 + α 6 + α 7 + α 8 + α 9 )
w = x1 + x 2 + x 3 + x 4
[neglecting higher powers]
From Eq. (i), 4 x 1 + 3x 2 + 2x 3 + x 4 = 20
= Coefficient of α 9 in (1 + α 2 + α 4 + α 6 + α 8 + α 3
Then, x 1, x 2 , x 3 , x 4 ≥ 1
Q 4 x 1 + 3x 2 + 2x 3 + x 4 = 20 …(ii) + α 5 + α 7 + α 9 + α 6 + α 8 + α 9 ) × (1 + α + α 2
∴ Number of positive integral solutions of Eq. (ii) α3 + α4 + α5 + α6 + α7 + α8 + α9 )
= Coefficient of x 20 − 10 in [neglecting higher powers]
(1 − x 4 )−1(1 − x 3 )−1(1 − x 2 )−1 (1 − x )−1 = 1 + 1 + 1 + 1 + 1 + 1 + 1 + 1 + 1 + 1 + 1 + 1 = 12

= Coefficient of x 10 in Case II If the inequation


(1 − x 4 )−1(1 − x 3 )−1 (1 − x 2 )−1 (1 − x )−1 x 1 + x 2 + x 3 +K + x m ≤ n …(i)
[when the required sum is not fixed]
= Coefficient of x 10 in (1 + x 4 + x 8 + x 12 + ...)
In this case, we introduce a dummy variable x m + 1 .
× (1 + x 3 + x 6 + x 9 + x 12 + K ) ×
So that,
(1 + x 2 + x 4 + x 6 + x 8 + x 10 + ...) × (1 + x + x 2 + x 3 + x 4
x 1 + x 2 + x 3 + K + x m + x m + 1 = n,
+ x 5 + x 6 + x 7 + x 8 + x 9 + x 10 + K )
xm +1 ≥ 0 …(ii)
= Coefficient of x 10 in
(1 + x 3 + x 6 + x 9 + x 4 + x 7 + x 10 + x 8 ) Here, the number of Sols of Eqs. (i) and (ii) will be same.
× (1 + x 2 + x 4 + x 6 + x 8 + x 10 )(1 + x + x 2 + x 3 y Example 99. Find the number of positive integral
+ x 4 + x 5 + x 6 + x 7 + x 8 + x 9 + x 10 ) solutions of the inequation 3x + y + z ≤ 30.
[neglecting higher powers] Sol. Let dummy variable w , then
= Coefficient of x 10 in 3x + y + z + w = 30, w ≥ 0 …(i)
(1 + x 2 + x 4 + x 6 + x 8 + x 10 + x 3 + x 5 + x 7 + x 9 + x 6 Now, let a = x − 1, b = y − 1, c = z − 1, d = w , then
+ x 8 + x 10 + x 9 + x 4 + x 6 + x 8 + x 10 + x 7 + x 9 + x 10 3a + b + c + d = 25, where a, b, c, d ≥ 0 …(ii)
∴ Number of positive integral solutions of Eq. (i)
+ x 8 + x 10 )(1 + x + x 2 + x 3 + x 4 + x 5 + x 6 + x 7
= Number of non-negative integral solutions of Eq. (ii)
+ x 8 + x 9 + x 10 ) [neglecting higher powers] = Coefficient of α 25 in (1 + α 3 + α 6 + K )
=1+1+1+1+1+1+1+1+1+1+1+1+1 (1 + α + α 2 + K )3
+ 1 + 1 + 1 + 1 + 1 + 1 + 1 + 1 + 1 + 1 = 23 = Coefficient of α 25
in (1 + α + α + K )(1 − α )−3
3 6

But x, y, z and w can be arranged in 4 P4 = 4 ! = 24 = Coefficient of α 25 in


Hence, required number of Sols = (23)(24 ) = 552 (1 + α 3 + α 6 + K )(1 + 3C 1α + 4C 2 α 2 + K )
= C 25 +
27 24
C 22 + 21C 19 + 18C 16 + 15C 13 + 12C 10 + 9C 7
y Example 98. In how many ways can 15 identical
blankets be distribted among six beggars such that + 6C 4 + 3C 1
everyone gets atleast one blanket and two particular = C2 +
27
C 2 + 21C 2 + 18C 2 + 15C 2 + 12C 2 + 9C 2
24

beggars get equal blankets and another three


+ 6C 2 + 3C 1
particular beggars get equal blankets.
= 351 + 276 + 210 + 153 + 105 + 66 + 36 + 15 + 3 = 1215
Aliter
396 Textbook of Algebra

From Eq. (ii), 3a + b + c + d = 25, where a, b, c, d ≥ 0 Total number of cases = 66 = 26 × 36 = 64 × 729 = 46656
Clearly, 0 ≤ a ≤ 8, if a = k , then and number of ways to get the sum less than or equal to 15,
b + c + d = 25 − 3k …(iii) which is 4501 [from Example 100]
Hence, number of non-negative integral solutions of Hence, the number of ways to get a sum greater than 15 is
Eq. (iii) is 46656 − 4501 = 42155
25 − 3k + 3 − 1 (27 − 3k )(26 − 3k )
C 3 − 1 = 27 − 3k C 2 = Case IV If the equation
2
3 2 x 1 x 2 x 3 K x n = 2 α1 ⋅ 3 α 2 ⋅ 5 α 3 K
= (3k − 53k + 234 )
2 where α 1 , α 2 , α 3 , K are natural numbers.
Therefore, required number is In this case number of positive integral solutions
3 8 ( x 1 , x 2 , x 3 , K, x n ) are
∑(3k 2 − 53k + 234 )
2k = 0 ( α 1 + n − 1 C n − 1 )( α 2 + n − 1 C n − 1 )( α 3 + n − 1 C n − 1 ) K
3   8 × 9 × 17  8 × 9 
= 3 ⋅   − 53 ⋅   + 234 × 9 = 1215
2  6   2   y Example 102. Find the total number of positive
integral solutions for ( x , y , z ) such that xyz = 24.
y Example 100. In how many ways can we get a sum
Sol. Q xyz = 24 = 23 × 31
of atmost 15 by throwing six distinct dice ?
Sol. Let x 1, x 2 , x 3 , x 4 , x 5 and x 6 be the number that appears Hence, total number of positive integral solutions
on the six dice. = ( 3 + 3 −1C 3 − 1 )( 1 + 3 − 1C 3 − 1 )
The number of ways = Number of solutions of the inequation = 5C 2 × 3C 2 = 30
x 1 + x 2 + x 3 + x 4 + x 5 + x 6 ≤ 15 Aliter
Introducing a dummy variable x 7 ( x 7 ≥ 0), the inequation Q xyz = 24 = 23 × 31
becomes an equation
Now, consider three boxes x , y , z .
x 1 + x 2 + x 3 + x 4 + x 5 + x 6 + x 7 = 15
3 can be put in any of the three boxes.
Here, 1 ≤ x i ≤ 6 for i = 1, 2, 3, 4, 5, 6 and x 7 ≥ 0.
Also, 2, 2, 2 can be distributed in the three boxes in
Therefore, number of solutions 3 + 3 −1
C 3 −1 = 5C 2 = 10 ways. Hence, the total number of
= Coefficient of x 15 in ( x + x 2 + x 3 + x 4 + x 5 + x 6 )6 positive integral solutions = the number of distributions
× (1 + x + x 2 + K ) which is given by 3 × 10 = 30.
−7
= Coefficient of x in (1 − x ) (1 − x )
9 6 6

= Coefficient of x 9 in (1 − 6x 6 )(1 + 7C 1x + 8C 2 x 2 + K ) Geometrical Problems


[neglecting higher powers]
(a) If there are n points in a plane out of these points no
= 15C 9 − 6 × 9C 3 = 15C 6 − 6 × 9C 3
three are in the same line exceptm points which are
= 5005 − 504 = 4501 collinear, then
Case III If the inequation (i) Total number of different lines obtained by
x 1 + x 2 + x 3 +K + x n ≥ n joining these n points is n C 2 − m C 2 + 1
[when the values of x 1 , x 2 ,…, x n are restricted] (ii) Total number of different triangles formed by
In this case first find the number of solutions of joining these n points is n C 3 − m C 3
x 1 + x 2 + x 3 + K + x n ≤ n − 1 and then subtract it (iii) Total number of different quadrilateral formed
from the total number of solutions. by joining these n points is
y Example 101. In how many ways can we get a sum
n
C 4 − ( m C 3 ⋅n C 1 + m C 4 ⋅ n C 0 )
greater than 15 by throwing six distinct dice?
y Example 103. There are 10 points in a plane out of
Sol. Let x 1, x 2 , x 3 , x 4 , x 5 and x 6 be the number that appears
these points no three are in the same straight line
on the six dice.
except 4 points which are collinear. How many
The number of ways = Number of solutions of the
inequation (i) straight lines (ii) trian-gles
x 1 + x 2 + x 3 + x 4 + x 5 + x 6 > 15 (iii) quadrilateral, by joining them?
Here, 1 ≤ x i ≤ 6, i = 1, 2, 3, 4, 5, 6 Sol. (i) Required number of straight lines
Chap 05 Permutations and Combinations 397

10 ⋅ 9 4 ⋅ 3 n ( n − 1) n (n − 3) 14 × 11
= 10C 2 − 4C 2 + 1 = − + 1 = 45 − 6 + 1 = 40 ⇒ − n = 77 ⇒ =
1⋅2 1 ⋅2 2 2 2
we get, n = 14
(ii) Required number of triangles
(c) n straight lines are drawn in a plane such that no two
10 ⋅ 9 ⋅ 8 4
= 10C 3 − 4C 3 = − C 1 = 120 − 4 = 116 of them are parallel and no three of them are
1 ⋅2 ⋅3 concurrent. Then, number of parts into which these
(iii) Required number of quadrilaterals lines divides the plane is equal to
= 10C 4 − ( 4 C 3 ⋅ 6C 1 + 4C 4 ⋅ 6C 0 )
n
(n 2 + n + 2 )
1 + ∑ k, .e.
10 ⋅ 9 ⋅ 8 ⋅ 7 k =1 2
= − ( 4C 1 ⋅6 C 1 + 1.1)
1 ⋅2 ⋅3 ⋅ 4
y Example 107. If n lines are drawn in a plane such
= 210 − ( 4 × 6 + 1) = 210 − 25 = 185
that no two of them are parallel and no three of them
(b) If there are n points in a plane out of these points no are concurrent, such that these lines divide the plane
any three are collinear, then in 67 parts, then find number of different points at
(i) Total points of intersection of the lines joining which these lines will cut.
these n points = p C 2 , where p = n C 2 Sol. Given number of straight lines = n , then
(ii) If n points are the vertices of a polygon, then n
n2 + n + 2
n(n − 3 )
1+ ∑k = 67 ⇒
2
= 67
total number of diagonals = n C 2 − n = k =1
2 ⇒ n + n − 132 = 0 ⇒ (n + 12) (n − 11) = 0
2

y Example 104. How many number of points of ∴ n = 11, n ≠ − 12


intersection of n straight lines, if n satisfies 11 ⋅ 10
Hence, required number of points = n C 2 = 11C 2 =
n +5 11(n − 1) n +3 2
Pn +1 = × Pn ? = 55
2
(d) If m parallel lines in a plane are intersected by a
n+5 11(n − 1) n + 3
Sol. We have, Pn +1 = × Pn family of other n parallel lines. Then, total number of
2
parallelograms so formed
(n + 5)! 11(n − 1) (n + 3)!
⇒ = × mn (m − 1) (n − 1)
4! 2 3! = mC 2 ⋅n C 2 i.e.,
4
(n + 5) (n + 4 ) 11(n − 1)
⇒ =
4 2 y Example 108. Find number of rectangles in a chess
⇒ n − 13n + 42 = 0 ⇒ (n − 6) (n − 7 ) = 0
2 board, which are not a square.
⇒ n = 6 or n = 7 Sol. Number of rectangles = 9C 2 × 9C 2 = (36)2 = 1296

The number of points of intersection of lines is 6 C 2 or 7 C Number of squares = 8 × 8 + 7 × 7 + 6 × 6 + … + 1 × 1


= 204
= 15 or 21
∴ Required number = 1296 − 204 = 1092
y Example 105. The interior angles of a regular 1 2 3 4 5 6 7 8 9
polygon measure 150° each. Then, find the number of
2
diagonals of the polygon.
3
Sol. Each exterior angle = 30°
4
π
360 × 5
360° 180
∴ Number of sides = = = 12 6
30° π
30 × 7
180 8
∴ Number of diagonals = C 2 − 12 = 66 − 12 = 54
12
9
Square can be formed as follows :
y Example 106. In a polygon the number of diagonals is
To form the smallest square, select any two consecutive
77. Find the number of sides of the polygon. lines from the given (here 9) vertical and horizontal lines.
Sol. Let number of sides of the polygon = n, then n C 2 − n = 77 This can be done in 8 × 8 ways (1-2, 2-3, 3-4, …, 8-9)
398 Textbook of Algebra

Again, to form the square consists of four small squares, Sol. The required number of ways
select the lines as follows (1-3, 2-4, 3-5,…, 7-9) from both = Coefficient of x 6 in ( 2 C 1x + 2C 2 x 2 ) ( 4 C 1x + 4C 2 x 2
vertical and horizontal lines, thus 7 × 7 squares are
obtained. Proceed in the same way) + 4C 3 x 3 + 4C 4 x 4 ) ( 2 C 1x + 2C 2 x 2 )
Note If n parallel lines are intersected by another n parallel lines, = Coefficient of x 3 in (2 + x )2 ( 4 + 6x + 4 x 2 + x 3 )
( n − 1) n ( 2n − 1)
then number of rhombus = ∑ ( n − 1) =
2
6 = Coefficient of x 3 in ( 4 + 4 x + x 2 ) ( 4 + 6x + 4 x 2 + x 3 )
(e) Number of Rectangles and Squares = 4 + 16 + 6
(i) Number of rectangles of any size in a square of = 26
n
n × n is ∑ r 3 and number of squares of any Aliter
r =1 In the given figure there are 8 squares and we have to place
n 6X ’s this can be done in
size is ∑ r 2 . 8 ⋅7
r =1
8
C 6 = 8C 2 = = 28 ways
1⋅2
(ii) In a rectangle of n × p (n < p ) number of
But these include the possibility that either headed row or
np
rectangles of any size is (n + 1) ( p + 1) and lowest row may not have any X . These two possibilities are
4 to be excluded.
number of squares of any size is ∴ Required number of ways = 28 − 2 = 26
n
∑ (n + 1 − r ) ( p + 1 − r ).
r =1 X X

y Example 109. Find the number of rectangles X X X X OR X X X X


excluding squares from a rectangle of size 9 × 6.
Sol. Here, n = 6 and p = 9 X X

∴ Number of rectangles excluding square


6 ⋅9 6
y Example 111. In how many ways the letters of the
= (6 + 1) (9 + 1) − ∑ (7 − r ) (10 − r )
4 r =1 word DIPESH can be placed in the squares of the
6
adjoining figure so that no row remains empty?
= 945 − ∑(70 − 17r + r 2 ) = 945 − 154 = 791
r =1

(f) If there are n rows, first row has α 1 squares, 2nd row
has α 2 squares, 3rd row has α 3 squares, ... and nth
row has α n squares. If we have to filled up the
squares with β X s such that each row has atleast
one X. The number of ways = Coefficient of x β in
( α1 C1 x + α2
C 2 x 2 +… + α 1 C α 1 x α 1 )
× ( α 2 C 1 x + α 2 C 2 x 2 +… + α2
Cα2 x α2 )
× ( α 3 C 1 x + α 3 C 2 x 2 +… + α 3 C α 3 x α 3 ) ×
… × ( α n C 1 x + α n C 2 x 2 +… + α n C α n x α n ) Sol. If all letters are same, then number of ways
= Coefficient of x 6 in ( 3C 1x + 3C 2 x 2 + 3C 3 x 3 )3 ( 1C 1x )2
y Example 110. Six X ’s have to be placed in the = Coefficient of x in (3 + 3x + x 2 )3
squares of the figure below, such that each row
contains atleast one X. In how many different ways = Coefficient of x in (3 + 3x )3
can this be done? [neglecting higher degree term]
= 27 × 3C 1 = 81
But in DIPESH all letters are different.
∴ Required number of ways = 81 × 6!
Chap 05 Permutations and Combinations 399

#L Exercise for Session 6


1. If number of ways in which 7 different balls can be distributed into 4 different boxes, so that no box remains
empty is 100 λ, the value of λ is
(a) 18 (b) 108 (c) 1008 (d) 10008

2. If number of ways in which 7 different balls can be distributed into 4 boxes, so that no box remains empty is
48 λ, the value of λ is
(a) 231 (b) 331 (c) 431 (d) 531

3. If number of ways in which 7 identical balls can be distributed into 4 boxes, so that no box remains empty is4λ,
the value of λ is
(a) 5 (b) 7 (c) 9 (d) 11

4. Number of non-negative integral solutions of the equation a + b + c = 6 is


(a) 28 (b) 32 (c) 36 (d) 56

5. Number of integral solutions of a + b + c = 0, a ≥ − 5, b ≥ − 5 and c ≥ − 5, is


(a) 272 (b) 136 (c) 240 (d) 120

6. If a, b and c are integers and a ≥ 1, b ≥ 2 and c ≥ 3. If a + b + c = 15, the number of possible solutions of the
equation is
(a) 55 (b) 66 (c) 45 (d) None of these

7. Number of integral solutions of 2x + y + z = 10 (x ≥ 0, y ≥ 0, Z ≥ 0) is


(a) 18 (b) 27 (c) 36 (d) 51

8. A person writes letters to six friends and addresses the corresponding envelopes. Let x be the number of ways
so that atleast two of the letters are in wrong envelopes and y be the number of ways so that all the letters are
in wrong envelopes. Then, x − y is equal to
(a) 719 (b) 265 (c) 454 (d) None of these

9. A person goes for an examination in which there are four papers with a maximum ofm marks from each paper.
The number of ways in which one can get 2 m marks, is
(b)   (m + 1) (2m 2 + 4m + 1)
2 m+ 3 1
(a) C3
 3

(c)   (m + 1) (2m 2 + 4m + 3)
1
(d) None of these
 3

10. The number of selections of four letters from the letters of the word ASSASSINATION, is
(a) 72 (b) 71 (c) 66 (d) 52

11. The number of positive integral solutions of 2x1 + 3x 2 + 4x 3 + 5x4 = 25, is


(a) 20 (b) 22 (c) 23 (d) None of these

12. If a, b , and c are positive integers such that a + b + c ≤ 8, the number of possible values of the ordered triplet (
a, b , c ) is
(a) 84 (b) 56 (c) 83 (d) None of these

13. The total number of positive integral solutions of 15 < x1 + x 2 + x 3 ≤ 20 is equal to


(a) 685 (b) 785 (c) 1125 (d) None of these

14. The total number of integral solutions for ( x , y , z ) such that xyz = 24, is
(a) 36 (b) 90 (c) 120 (d) None of these

15. There are 12 points in a plane in which 6 are collinear. Number of different straight lines that can be drawn by
joining them, is
(a) 51 (b) 52 (c) 132 (d) 18
400 Textbook of Algebra

16. 4 points out of 11 points in a plane are collinear. Number of different triangles that can be drawn by joining
them, is
(a) 165 (b) 161 (c) 152 (d) 159

17. The number of triangles that can be formed with 10 points as vertices, n of them being collinear, is 110. Then, n
is
(a) 3 (b) 4 (c) 5 (d) 6

18. ABCD is a convex quadrilateral. 3, 4, 5 and 6 points are marked on the sides AB, BC, CD and DA, respectively.
The number of triangles with vertices on different sides, is
(a) 270 (b) 220 (c) 282 (d) None of these

19. There are 10 points in a plane of which no three points are collinear and 4 points are concyclic. The number of
different circles that can be drawn through atleast 3 points of these points, is
(a) 116 (b) 120 (c) 117 (d) None of these

20. 4 points out of 8 points in a plane are collinear. Number of different quadrilateral that can be formed by joining
them, is
(a) 56 (b) 60 (c) 76 (d) 53

21. There are 2n points in a plane in which m are collinear (n > m > 4). Number of quadrilateral formed by joining
these lines
(a) is equal to C4 − mC4
2n
(b) is greater than C4 − mC4
2n

(c) is less than 2n


C4 − mC4 (d) None of these

22. In a polygon the number of diagonals is 54. The number of sides of the polygon, is
(a) 10 (b) 12 (c) 9 (d) None of these

23. In a polygon no three diagonals are concurrent. If the total number of points of intersection of diagonals interior
to the polygon be 70, then the number of diagonals of the polygon, is
(a) 20 (b) 28 (c) 8 (d) None of these

24. n lines are drawn in a plane such that no two of them are parallel and no three of them are concurrent. The
number of different points at which these lines will cut, is
n −1
(a) ∑k (b) n (n − 1) (c) n 2 (d) None of these
k =1

25. Six straight lines are drawn in a plane such that no two lines are parallel and no three lines are concurrent. Then,
the number of parts into which these lines divide the plane, is
(a) 15 (b) 22 (c) 29 (d) 36

26. A parallelogram is cut by two sets of m lines parallel to its sides. The number of parallelogram thus formed, is
(a) ( mC2 )2 (b) ( m + 1C2 )2 (c) ( m + 2C2 )2 (d) None of these

27. The number of rectangles excluding squares from a rectangle of size 11 × 8 is 48λ, then the value of λ is
(a) 13 (b) 23 (c) 43 (d) 53

28. The number of ways the letters of the word PERSON can be placed in the squares of the figure shown so that
no row remains empty, is

R1

R2

R3

(a) 24 × 6 ! (b) 26 × 6 ! (c) 26 × 7 ! (d) 27 × 7 !


Session 7
Rank in a Dictionary, Gap Method

Rank in a Dictionary y Example 113. If letters of the word are arranged as in


The dictionary format means words are arranged in dictionary, find the rank of the following words.
alphabetical order. (i) INDIA (ii) SURITI (iii) DOCOMO
Following Examples will help you learn how to find the Sol. (i) The letters in alphabetical order are A, D, I, I, N
rank in the dictionary. ∴ The first word is ADIIN
4!
Number of words beginning with A = = 12
y Example 112. If the letters of the word are arranged 2!
as in dictionary, find the rank of the following words. Number of words beginning with D =
4!
= 12
(i) RAJU (ii) UMANG 2!
(iii) AIRTEL Number of words beginning with IA = 3 ! = 6
Sol. (i) In a dictionary, the letters in alphabetical order are Number of words beginning with ID = 3 ! = 6
A, J, R, U Number of words beginning with II = 3 ! = 6
∴ The first word is AJRU. Number of words beginning with INA = 2 ! = 2
Number of words beginning with INDA = 1 ! = 1
Number of words beginning with A = Number of Number of words beginning with INDIA = 1
ways arranging J, R, U = 3 ! = 6
∴ Rank of the word INDIA
Number of words beginning with J = 3 ! = 6
= 12 + 12 + 6 + 6 + 6 + 2 + 1 + 1 = 46
The next word begin with R and it is RAJU.
(ii) The letters in alphabetical order are I, I, R, S, T, U
∴ Number of words before RAJU = 12 ∴ The first word is IIRSTU
∴ Rank of word RAJU = 13 Number of words beginning with I = 5 ! = 120
5!
(ii) The letters in alphabetical order are A, G, M, N, U Number of words beginning with R = = 60
2!
∴ The first word is AGMNU Number of words beginning with SI = 4 ! = 24
Number of words beginning with A = 4 ! = 24 4!
Number of words beginning with SR = = 12
Number of words beginning with G = 4 ! = 24 2!
Number of words beginning with M = 4 ! = 24 4!
Number of words beginning with N = 4 ! = 24 Number of words beginning with ST = = 12
2!
Number of words beginning with UA = 3 ! = 6 Number of words beginning with SUI = 3 ! = 6
Number of words beginning with UG = 3 ! = 6 Number of words beginning with SURII = 1 ! = 1
Number of words beginning with UMAG = 1 ! = 1 Number of words beginning with SURITI = 1
Number of words beginning with UMANG = 1 ∴ Rank of the word SURITI
∴ Rank of the word = 120 + 60 + 24 + 12 + 12 + 6 + 1 + 1 = 236
UMANG = 24 + 24 + 24 + 24 + 6 + 6 + 1 + 1 = 110 (iii) The letters in alphabetical order are C, D, M, O, O, O
(iii) The letters in alphabetical order are A, E, I, L, R, T ∴ The first word is CDMOOO
5!
∴ The first word is AEILRT Number of words beginning with C = = 20
3!
Number of words beginning with AE = 4 ! = 24 4!
Number of words beginning with DC = =4
Number of words beginning with AIE = 3 ! = 6 3!
Number of words beginning with AIL = 3 ! = 6 4!
Number of words beginning with DM = =4
Number of words beginning with AIRE = 2 ! = 2 3!
Number of words beginning with AIRL = 2 ! = 2 2!
Number of words beginning with DOCM = =1
Number of words beginning with AIRTEL = 1 2!
Number of words beginning with DOCOMO = 1
∴ Rank of the word AIRTEL= 24 + 6 + 6 + 2 + 2 + 1 = 41
∴ Rank of the word DOCOMO = 20 + 4 + 4 + 1 + 1 = 30
402 Textbook of Algebra

Gap Method y Example 117. Find the number of ways in which 5


girls and 5 boys can be arranged in a row, if boys and
[when particular objects are girls are alternate.
never together] Sol. First five girls can be arranged in 5 ! ways
i.e., × G × G × G × G × G or G × G × G × G × G ×
y Example 114. There are 10 candidates for an
Now, if girls and boys are alternate, then boys can occupy
examination out of which 4 are appearing in places with ‘ × ’ as shows above.
Mathematics and remaining 6 are appearing in Hence, total number of arrangements is
different subjects. In how many ways can they be 5 ! × 5 ! + 5 ! × 5 ! = 2 × (5 !)2
seated in a row so that no two Mathematics Use of Set Theory
candidates are together? A set is well defined collection of distinct objects.
Sol. In this method first arrange the remaining candidates
Subset
Here, remaining candidates = 6
If every element of a set A is also an element of a set B,
×0×0×0×0×0×0× then A is called the subset B, we write
× : Places available for Mathematics candidates
A ⊂ B ⇔ {x ∈ A ⇒ x ∈ B }
0 : Places for others Union
Remaining candidates can be arranged in 6 ! ways. There
are seven places available for Mathematics candidates so The union of two sets A and B is the set of all those
that no two Mathematics candidates are together. Now, elements which are either in A or in B or in both. This set is
four candidates can be placed in these seven places denoted by A ∪ B or A + B.
in 7 P4 ways. Symbolically, A ∪ B = { x : x ∈ A or x ∈ B }
Hence, the total number of ways = 6 ! × P4 = 720 × 840
7
Intersection
= 604800 The intersection of two sets A and B is the set of all
elements which are common in A and B. This set is denoted
y Example 115. In how many ways can 7 plus (+ ) and 5 by A ∩ B or AB.
minus ( − ) signs be arranged in a row so that no two Symbolically, A ∩ B = { x : x ∈ A and x ∈ B }
minus ( − ) signs are together?
Sol. In this method, first arrange the plus ( + ) signs. y Example 118. A is a set containing n elements. A
Here, minus ( − ) signs = 5 subset P1 of A is chosen. The set A is reconstructed
0+0+0+0+0+0+0+0 by replacing the elements of P1 . Next, a subset P2 of
We can put minus ( − ) sign in any of the 8 places in the A is chosen and again the set is reconstructed by
above arrangement i.e., we have to select 5 places out of 8 replacing the elements of P2 . In this way m (> 1)
which can be done is 8 C 5 ways = 8C 3 ways = 56 ways.
subsets P1 , P2 , ..., Pm of A are chosen. Find the
y Example 116. Find the number of ways in which 5 number of ways of choosing P1 , P2 , ..., Pm , so that
girls and 5 boys can be arranged in a row, if no two
(i) P1 ∩ P2 ∩ P3 ∩ ... ∩ Pm = φ
boys are together.
Sol. In this example, there is no condition for arranging the (ii) P1 ∪ P2 ∪ P3 ∪ ... ∪ Pm = A
girls. Now, 5 girls can be arranged in 5 ! ways. Sol. Let A = {a1, a 2 , a 3 , ..., an }
×G ×G ×G ×G ×G × (i) For each ai (1 ≤ i ≤ n ), we have either a ∈ P j or
When girls are arranged, six gaps are generated as shown ai ∉ P j (1 ≤ j ≤ m ). i.e., there are 2m choices in which
above with ‘× ’. ai (1 ≤ i ≤ n ) may belong to the P j ’s.
Now, boys must occupy the places with ‘×’ marked, so that
no two boys are together. Out of these, there is only one choice, in which ai ∈ P j
for all j = 1, 2, ..., m which is not favourable for
Therefore, five boys can be arranged in these six gaps in
6
P5 ways. P1 ∩ P2 ∩ P3 ∩ ... ∩ Pm to be φ. Thus,
ai ∉ P1 ∩ P2 ∩ ... ∩ Pm in (2m − 1) ways. Since, there
Hence, total number of arrangement is 5 ! × 6 P5 .
are n elements in the set A, the total number of
choices is (2m − 1)n .
Chap 05 Permutations and Combinations 403

(ii) There is exactly one choice, in which, ai ∈ P j for all y Example 120. Find the sum of the digits in the unit’s
j = 1, 2, 3, ..., m which is not favourable for place of all numbers formed with the help of 3, 4, 5, 6
P1 ∪ P2 ∪ P3 ∪ ... ∪ Pm to be equal to A. Thus, ai can taken all at a time.
belong to P1 ∪ P2 ∪ P3 ∪ ... ∪ Pm in (2m − 1) ways. Sol. Sum of the digits in the unit’s place
Since, there are n elements in the set A, the number of = ( 4 − 1) ! (3 + 4 + 5 + 6) = 6 × 18 = 108
ways in which P1 ∪ P2 ∪ P3 ∪ ... ∪ Pm can be equal to (ii) The sum of all digit numbers that can be formed using
A is (2m − 1)n . the digits a1, a 2 , ..., an (repetition of digits not allowed)
(10n − 1)
is = (n − 1) ! (a1 + a 2 + K + an )
y Example 119. A is a set containing n elements. A 9
subset P of A is chosen. The set A is reconstructed y Example 121. Find the sum of all five digit
by replacing the elements of P. A subset of A is numbers that can be formed using the digits 1, 2, 3,
again chosen. Find the number of ways of choosing 4 and 5 (repetition of digits not allowed)
P and Q, so that  105 − 1
Sol. Required sum = (5 − 1) ! (1 + 2 + 3 + 4 + 5)  
(i) P ∩ Q contains exactly r elements.  9 
= 24 ⋅ 15 ⋅ 11111 = 3999960
(ii) P ∩ Q contains exactly 2 elements. Aliter
(iii) P ∩ Q = φ Since, one of the numbers formed with the 5 digits a, b, c , d
Sol. Let A = {a1, a 2 , a 3 , ..., an } and e is 104 a + 103 b + 102 c + 10d + e ;
(i) The r elements in P and Q such that P ∩ Q can be Hence, 104 a will occur altogether in 4 ! ways similarly each
chosen out of n is n C r ways a general element of A of 104 b, 104 c , 104 d , 104 e will occur in 4 ! ways.
must satisfy one of the following possibilities [here, Hence, if all the numbers formed with the digits be written
general element be ai (1 ≤ i ≤ n )] one below the other, thus
(i) ai ∈ P and ai ∈ Q
104 ⋅ a + 103 ⋅ b + 102 ⋅ c + 10 ⋅ d + e
(ii) ai ∈ P and ai ∉Q
(iii) ai ∈ P and ai ∈ Q 104 ⋅ b + 103 ⋅ c + 102 ⋅ d + 10 ⋅ e + a
(iv) ai ∉ P and ai ∉Q 104 ⋅ c + 103 ⋅ d + 102 ⋅ e + 10 ⋅ a + b
Let a1, a 2 , ..., ar ∈ P ∩ Q
104 ⋅ d + 103 ⋅ e + 102 ⋅ a + 10 ⋅ b + c
There is only one choice each of them (i.e., (i) choice)
and three choices (ii), (iii) and (iv) for each of 104 ⋅ e + 103 ⋅ a + 102 ⋅ b + 10 ⋅ c + d
remaining (n − r ) elements. Hence, the required sum
Hence, number of ways of remaining elements = 3n − r
= 4 ! × (a + b + c + d + e ) × (104 + 103 + 102 + 10 + 1)
Hence, number of ways in which P ∩ Q contains
= 4 ! × (1 + 2 + 3 + 4 + 5) (11111) = 3999960
exactly r elements = n C r × 3n − r
(i) Put r = 2, then n C 2 × 3n − 2 Difference between Permutation and Combination
(iii) Put r = 0, then n C 0 × 3n = 3n Problems of permutations Problems of combinations
1. Arrangements Selections, choose
Sum of digits
2. Standing in a line, seated in a row Distributed group is formed
(i) The sum of the digits in the unit’s place of all numbers 3. Problems on digits Committee
formed with the help of a1, a 2 , ..., an taken all at a time
is (n − 1) ! (a1 + a 2 + K + an ) 4. Problems on letters from a word Geometrical problems
(repetition of digits not allowed)
404 Textbook of Algebra

#L Exercise for Session 7


1. The letters of the word ‘‘DELHI’’ are arranged in all possible ways as in a dictionary, the rank of the word
‘‘DELHI’’ is
(a) 4 (b) 5
(c) 6 (d) 7

2. The letters of the word ‘‘KANPUR’’ are arranged in all possible ways as in a dictionary, the rank of the word
‘‘KANPUR’’ from last is
(a) 121 (b) 122
(c) 598 (d) 599

3. The letters of the word ‘‘MUMBAI’’ are arranged in all possible ways as in a dictionary, the rank of the word
‘‘MUMBAI’’ is
(a) 297 (b) 295
(c) 299 (d) 301

4. The letters of the word ‘‘CHENNAI’’ are arranged in all possible ways as in a dictionary, then rank of the word
‘‘CHENNAI’’ from last is
(a) 2016 (b) 2017
(c) 2018 (d) 2019

5. If all permutations of the letters of the word ‘‘AGAIN’’ are arranged as in a dictionary, then 50th word is
(a) NAAGI (b) NAGAI
(c) NAAIG (d) NAIAG
Shortcuts and Important Results to Remember
1 When two dice are thrown, the number of ways of getting 11 Highest power of prime p in n Cr , since
a total r (sum of numbers on upper faces), is n!
n
Cr =
(i) r − 1, if 2 ≤ r ≤ 7 r ! (n − r ) !
(ii) 13 − r, if 8 ≤ r ≤ 12
If Hp (n !) = α,
2 When three dice are thrown, the number of ways of
getting a total r (sum of numbers on upper faces), is Hp (r !) = β

(i) r −1
C2 , if 3 ≤ r ≤ 8 and Hp {(n − r ) !} = γ

(ii) 25, if r = 9 Then, Hp (n Cr ) = α − ( β + γ )


(iii) 27, if r = 10, 11 12 Highest power of prime p in n Pr , since
(iv) 25, if r = 12 n!
(v) 20 − r
C2 , if 13 ≤ r ≤ 18
n
Pr =
(n − r ) !
3 The product of k consecutive positive integers is divisible If Hp (n !) = λ , Hp {(n − r ) !} = µ. Then, Hp (n Pr ) = λ − µ
by k !.
4 Number of zeroes in n ! = E5 (n !) 13 If there are n rows. Ist row has m1squares, IInd row has m2
squares, IIIrd row has m3 squares and so on. If we placed
5 n straight lines are drawn in the plane such that no two
λ X’s in the squares such that each row contains atleast
lines are parallel and no three lines are concurrent. Then,
one X. Then the number of ways = Coefficient of x λ in
the number of parts into which these lines divides the
(n 2 + n + 2 ) ( m 1 C1 x + m 1 C2 x 2 + ...+ m1
Cm 1 x m 1 )
plane is equal to .
2 × ( m2 C1 x + m2 C2 x 2 + m2
C3 x 3 + ...+ m2
Cm 2 x m2 ) ×
6 n
Cr is divisible by n only, if n is a prime number ( m3 C1 x + m3 C2 x 2 + ... + m3
Cm3 x m3 ) ×...
(1 ≤ r ≤ n − 1).
1 1 1
7 The number of diagonals in n-gon (n sides closed If + = , ∀ x, y, n ∈ N
x y n
n(n − 3)
polygon) is .
2 ⇒ ( x − n) ( y − n) = n2
8 In n-gon no three diagonals are concurrent, then the total ∴ x = n + λ,
number of points of intersection of diagonals interior to
n2
the polygon is n C4 . y =n+ ,
λ
9 Consider a polygon of n sides, then number of triangles in
where λ is divisor of n 2 .
which no side is common with that of the polygon are
1 Then, number of integral solutions ( x, y ) is equal to
n (n − 4) (n − 5).
6 number of divisors of n 2 .
10 If m parallel lines in a plane are intersected by a family of If n = 3, n 2 = 9 = 32 , the equation has 3 solutions.
other n parallel lines. The total number of parallelograms ( x, y ) = (4, 12 ), (6, 6), (12, 4)
mn(m − 1)(n − 1)
so formed = mC2 ⋅n C2 =
4
JEE Type Solved Examples :
Single Option Correct Type Questions
n This section contains 10 multiple choice examples. Sol. (c) Qa is a factor of b and c ⇒ a divides 60
Each example has four choices (a), (b), (c) and (d) out of ∴ a = 1, 2, 3, 4, 5, 6, 10, 12, 15, 30 [Qa ≠ 60]
which ONLY ONE is correct. and b = ma , c = na, when m, n ≥ 1
Q a + b + c = 60
l Ex. 1 Number of words of 4 letters that can be formed  60 
⇒ a + ma + na = 60 ⇒ m + n =  − 1
with the letters of the word IIT JEE, is a 
60
−1−1  60 
(a) 42 (b) 82 (c) 102 (d) 142 ∴ Number of solutions = a C 2 − 1 =  − 2
Sol. (c) There are 6 letters I, I, E, E, T, J a 
The following cases arise: Hence, total number of solutions for all values of a
Case I All letters are different = 58 + 28 + 18 + 13 + 10 + 8 + 4 + 3 + 2 + 0 = 144
4
P4 = 4 ! = 24
lEx. 4 The number of times the digit 3 will be written
Case II Two alike and two different
when listing the integers from 1 to 1000, is
4!
2
C 1 × 3C 2 × = 72 (a) 269 (b) 271 (c) 300 (d) 302
2!
Sol. (c) Since, 3 does not occur in 1000. So, we have to count
Case III Two alike of one kind and two alike of another the number of times 3 occurs, when we list the integers
kind 4! from 1 to 999.
2
C2 × =6
2! 2! Any number between 1 and 999 is of the form xyz, where
Hence, number of words = 24 + 72 + 6 = 102 0 ≤ x , y , z ≤ 9.
Aliter Let us first count the number in which 3 occurs exactly
once. Since, 3 can occur at one place in 3 C 1 ways, there are
Number of words = Coefficient of x 4 in
2
3
C 1 × 9 × 9 = 243 such numbers. Next 3 can occur in exactly
 x x2
4 ! 1 + +  (1 + x )
2
two places in 3 C 2 × 9 = 27 such numbers. Lastly, 3 can
 1! 2! 
occur in all three digits in one number only. Hence, the
= Coefficient of x 4 in 6[(1 + x )2 + 1]2 (1 + x )2 number of times, 3 occurs is 1 × 243 + 2 × 27 + 3 × 1 = 300
= Coefficient of x 4 in 6[(1 + x )6 + 2(1 + x )4 + (1 + x )2 ] l Ex. 5 Number of points having position vector
= 6[ 6C 4 + 2 ⋅4 C 4 + 0] = 6(15 + 2) = 102
ai + b$j + ck$ , where a , b, c ∈ {1, 2, 3, 4, 5 } such that 2 a + 3 b + 5 c
$
l Ex. 2 Let y be element of the set A = {1, 2, 3, 5, 6, 10, 15, is divisible by 4, is
30} and x 1 , x 2 , x 3 be integers such that x 1 x 2 x 3 = y , the (a) 70 (b) 140
number of positive integral solutions of x 1 x 2 x 3 = y , is (c) 210 (d) 280
(a) 27 (b) 64 (c) 81 (d) 256 Sol. (a) Q2a + 3b + 5c = 2a + ( 4 − 1)b + ( 4 + 1)c
Sol. (b) Number of solutions of the given equations is the same = 2a + 4k + ( −1)b + (1)c
as the number of solutions of the equation
= 2a + 4k + ( −1)b + 1
x 1 x 2 x 3 x 4 = 30 = 2 × 3 × 5
Here, x 4 is infact a dummy variable. I. a = 1, b = even, c = any number
II. a ≠ 1, b = odd, c = any number
If x 1 x 2 x 3 = 15, then x 4 = 2 and if x 1x 2 x 3 = 5, then x 4 = 6, etc.
∴ Required number of ways = 1 × 2 × 5 + 4 × 3 × 5 = 70
Thus, x1 x 2 x 3 x 4 = 2 × 3 × 5
[Q even numbers = 2, 4; odd numbers = 1, 3, 5 and any
Each of 2, 3 and 5 will be factor of exactly one of x 1, x 2 , x 3 ,
numbers = 1, 2, 3, 4, 5]
x 4 in 4 ways.
∴ Required number = 4 3 = 64 l Ex. 6 Number of positive unequal integral solutions of
the equation x + y + z =12 is
l Ex. 3 The number of positive integer solutions of (a) 21 (b) 42 (c) 63 (d) 84
a + b + c = 60, where a is a factor of b and c, is Sol. (b) We have, x + y + z = 12 …(i)
(a) 184 (b) 200 (c) 144 (d) 270 Assume x < y < z . Here, x , y , z ≥ 1
Chap 05 Permutations and Combinations 407

∴ Solutions of Eq. (i) are Therefore, for one element ai of A, we have four choices (i),
(1, 2, 9), (1, 3, 8), (1, 4, 7), (1, 5, 6), (2, 3, 7), (2, 4, 6) and (3, 4, 5). (ii), (iii) and (iv).
Number of positive integral solutions of Eq. (i) = 7 but ∴ Total number of cases for all elements = 4n
x , y , z can be arranged in 3! = 6
and for one element ai of A, such that ai ∈ P ∪ Q , we have
Hence, required number of solutions = 7 × 6 = 42 three choices (i), (ii) and (iii).
Aliter
∴ Number of cases for all elements belong to P ∪ Q = 3n
Let x = α, y − x = β, z − y = γ
∴ x = α, y = α + β, z = α + β + γ Hence, number of ways in which atleast one element of A
From Eq. (i), 3α + 2β + γ = 12; α, β, γ ≥ 1 does not belong to
∴ Number of positive integral solutions of Eq. (i) P ∪ Q = 4n − 3n .
= Coefficient of λ12 in
( λ 3 + λ 6 + λ 9 + λ12 + ...) l Ex. 9 Let N be a natural number. If its first digit (from the
( λ + λ + λ + λ + λ + λ + ...)
2 4 6 8 10 12 N
left) is deleted, it gets reduced to . The sum of all the digits
( λ + λ 2 + λ 3 + ... + λ12 ) 29
of N is
= Coefficient of λ 6 in (1 + λ 3 + λ 6 )(1 + λ 2 + λ 4 + λ 6 )
(a) 14 (b) 17
(1 + λ + λ 2 + λ 3 + λ 4 + λ 5 + λ 6 ) (c) 23 (d) 29
= Coefficient of λ 6 in (1 + λ 2 + λ 4 + λ 6 + λ 3 + λ 5 + λ 6 ) Sol. (a) Let N = an an − 1 an − 2 ... a 3 a 2 a1 a 0
× (1 + λ + λ 2 + λ 3 + λ 4 + λ 5 + λ 6 ) = a 0 + 10a1 + 102 a 2 + ... + 10n −1an −1 + 10n an …(i)
= 1+ 1 + 1 + 1 + 1 + 1 + 1 = 7 N
but x,y, z can be arranged in 3! = 6 Then, = an − 1 an − 2 an − 3 ... a 3 a 2 a1 a 0
29
Hence, required number of solutions = 7 × 6 = 42
= a 0 + 10a1 + 102 a 2 + … + 10n − 2 an − 2 + 10n − 1an − 1
l Ex. 7 12 boys and 2 girls are to be seated in a row such or N = 29 (a 0 + 10a1 + 102 a 2 + ...
that there are atleast 3 boys between the 2 girls. The number
+ 10n − 2 an − 2 + 10n − 1an − 1 ) …(ii)
of ways this can be done is λ ×12 ! , the value of λ is
(a) 55 (b) 110 (c) 20 (d) 45 From Eqs. (i) and (ii), we get
Sol. (b) Let P = Number of ways, 12 boys and 2 girls are 10n ⋅ an = 28(a 0 + 10a1 + 102 a 2 + ... + 10n − 1an − 1 )
seated in a row
= 14 ! = 14 × 13 × 12! = 182 × 12! ⇒ 28 divides 10n ⋅ an ⇒ an = 7, n ≥ 2 ⇒ 52 = a 0 + 10a1
P1 = Number of ways, the girls can sit together The required N is 725 or 7250 or 72500, etc.
= (14 − 2 + 1) × 2! × 12! = 26 × 12! ∴ The sum of the digits is 14.
P2 = Number of ways, one boy sits between the girls
= (14 − 3 + 1) × 2! × 12! = 24 × 12! l Ex. 10 If the number of ways of selecting n cards out of
P3 = Number of ways, two boys sit between the girls unlimited number of cards bearing the number 0, 9, 3, so that
= (14 − 4 + 1) × 2! × 12! = 22 × 12! they cannot be used to write the number 903 is 93, then n is
∴ Required number of ways = (182 − 26 − 24 − 22) × 12! equal to
= 110 × 12! = λ × 12! [given] (a) 3 (b) 4
∴ λ = 110 (c) 5 (d) 6
Sol. (c) We cannot write 903.
l Ex. 8 A is a set containing n elements. A subset P of A is If in the selection of n cards, we get either
chosen. The set A is reconstructed by replacing the elements
(9 or 3), (9 or 0), (0 or 3), (only 0), (only 3) or (only 9).
of P. A subset Q of A is again chosen, the number of ways of
choosing so that (P ∪ Q ) is a proper subset of A, is For (9 or 3) can be selected = 2 × 2 × 2 × ... × n factors = 2n
(a) 3n (b) 4n (c) 4n − 2n (d) 4n − 3n Similarly, (9 or 0) or (0 or 3) can be selected = 2n
Sol. (d) Let A = {a1, a 2 , a 3 , ..., an } In the above selection (only 0) or (only 3) or (only 9) is
repeated twice.
a general element of A must satisfy one of the following
possibilities. ∴ Total ways = 2n + 2n + 2n − 3 = 93
[here, general element be ai (1 ≤ i ≤ n )] ⇒ 3 ⋅ 2n = 96 ⇒ 2n = 32 = 25
(i) ai ∈ P , ai ∈ Q (ii) ai ∈ P , ai ∉Q ∴ n =5
(iii) ai ∉ P , ai ∈ Q (iv) ai ∉ P , ai ∉Q
JEE Type Solved Examples :
More than One Correct Option Type Questions
n This section contains 5 multiple choice examples. Each (a) n − 2 C 3 (b) n − 3 C 3 + n−3
C2
example has four choices (a), (b), (c) and (d) out of which (n − 2)(n − 3 )(n − 4 )
more than one may be correct. (c) (d) n C 2
6
l Ex. 11 In a plane, there are two families of lines Sol. (a, b, c) Let a 0 be the number of objects to the left of the
y = x + r , y = − x + r , where r ∈{0, 1, 2, 3, 4 }. The number of first object chosen, a1 be the number of objects between
squares of diagonals of the length 2 formed by the lines is the first and the second, a 2 be the number of objects
3 between the second and the third and a 3 be the number of
(a) 9 (b) 16(c) ⋅ 4C 2 (d) 5C 2 + 3P2
2 objects to the right of the third object. Then,
Sol. (a, c) There are two sets of five parallel lines at equal a 0 , a 3 ≥ 0 and a1, a 2 ≥ 1
distances. Clearly, lines like l1, l 3 , m1 and m 3 form a square a0 a1 a2 a3
whose diagonal’s length is 2.
Y
also a 0 + a1 + a 2 + a 3 = n − 3
5
m

Let a = a 0 + 1, b = a 3 + 1, then a ≥ 1, b ≥ 1 such that


4
m
3
m

a + a1 + a 2 + b = n − 1
2
m

4
1
m

The total number of positive integral solutions of this


3
equation isn −1−1C 4 −1 = n − 2 C 3 = n − 3C 3 + n − 3C 2
2
(n − 2)(n − 3)(n − 4 )
1 =
1 ⋅2 ⋅3
X′ X
O l5 l Ex. 14 Given that the divisors of n = 3 p ⋅ 5 q ⋅ 7 r are of the
l4
Y′ l3 form 4 λ + 1, λ ≥ 0. Then,
l2
l1 (a) p + r is always even (b) p + q + r is even or odd
3
∴ The number of required squares = 3 × 3 = 9 = ⋅4 C 2 (c) q can be any integer (d) if p is even, then r is odd
2
[Q choices are (l1, l 2 ), (l 2 , l 4 ) and (l 3 , l 5 ) for one set, etc.] Sol. (a, b, c)
Q 3p = ( 4 − 1) p = 4 λ 1 + ( − 1) p ,
l Ex. 12 Number of ways in which three numbers in AP
5q = ( 4 + 1)q = 4 λ 2 + 1
can be selected from 1, 2, 3, ..., n, is and 7r = (8 − 1)r = 8λ 3 + ( −1)r
2
 n − 1 n (n − 2 )
(a)   , if n is even (b) , if n is even Hence, both p andr must be odd or both must be even. Thus,
 2  4 p + r is always even. Also, p + q + r can be odd or even.
(n − 1)2 n (n + 1)
(c) , if n is odd (d) , if n is odd l Ex. 15 Number of ways in which 15 identical coins can be
4 2
put into 6 different bags
Sol. (b, c) If a, b, c are in AP, then a + c = 2b
(a) is coefficient of x 15 in x 6 (1 + x + x 2 + ... ∞ )6 , if no bag
a and c both are odd or both are even. remains empty
Case I If n is even (b) is coefficient of x 15 in (1 − x )−6
Let n = 2m in which m are even and m are odd numbers. (c) is same as number of the integral solutions of
m ( m − 1) a + b + c + d + e + f = 15
∴ Number of ways = mC 2 + mC 2 = 2 ⋅ mC 2 = 2 ⋅
2 (d) is same as number of non-negative integral solutions of
n n  n ( n − 2) 6
=  − 1 =
2 2  4
[Qn = 2m ] ∑ x i = 15
i =1
Case II If n is odd
Sol. (a, b, d) Let bags be x 1, x 2 , x 3 , x 4 , x 5 and x 6 , then
Let n = 2m + 1 in which m are even and m + 1 are odd numbers.
x 1 + x 2 + x 3 + x 4 + x 5 + x 6 = 15.
∴ Number of ways = mC 2 + m +1C 2
∴ For no bags remains empty, number of ways
m ( m − 1) ( m + 1) m ( n − 1) 2
= + = m2 = [Qn = 2m + 1] = Coefficient of x 15 in ( x 1 + x 2 + x 3 + ... ∞ )6
2 2 4
= Coefficient of x 15 in x 6 (1 + x + x 2 + ... ∞ )6
l Ex. 13 If n objects are arranged in a row, then number of
= Coefficient of x 9 in (1 − x )−6
ways of selecting three of these objects so that no two of
In option (c), it is not mentioned that solution is positive integral.
them are next to each other, is
Chap 05 Permutations and Combinations 409

JEE Type Solved Examples :


Passage Based Questions
n This section contains 3 solved passages based upon each Hence, required number of ways
of the passage 3 multiple choice examples have to be 3!  4 ! 4 ! 4! 
answered. Each of these examples has four choices (a), (b), = 5C 3 × × + + 
2!  3! 3! 2! 2! 
(c) and (d) out of which ONLY ONE is correct.
= 30( 4 + 4 + 6) = 420
Passage I
(Ex. Nos. 16 to 18) Passage II (Ex. Nos. 19 to 21)
All the letters of the word ‘AGAIN’ be arranged and the Consider a polygon of sides ‘n’ which satisfies the equation
words thus formed are known as ‘Simple Words’. Further 3 ⋅ n P4 = n −1 P5 .
two new types of words are defined as follows:
19. Rajdhani express travelling from Delhi to Mumbai
(i) Smart word: All the letters of the word ‘AGAIN’ are
has n stations enroute. Number of ways in which a
being used, but vowels can be repeated as many times
train can be stopped at 3 stations if no two of the
as we need.
stopping stations are consecutive, is
(ii) Dull word: All the letters of the word ‘AGAIN’ are (a) 20 (b) 35 (c) 56 (d) 84
being used, but consonants can be repeated as many
times as we need. 20. Number of quadrilaterals that can be formed using the
vertices of a polygon of sides ‘n’ if exactly 1 side of the
16. If a vowel appears in between two similar letters, the quadrilateral is common with side of the n-gon, is
number of simple words is (a) 96 (b) 100 (c) 150 (d) 156
(a) 12 (b) 6 (c) 36 (d) 14
21. Number of quadrilaterals that can be made using the
17. Number of 7 letter smart words is vertices of the polygon of sides ‘n’ if exactly two
(a) 1500 (b) 1050 (c) 1005 (d) 150 adjacent sides of the quadrilateral are common to the
18. Number of 7 letter dull words in which no two vowels sides of the n-gon, is
are together, is (a) 50 (b) 60 (c) 70 (d) 80
n −1
(a) 402 (b) 420 (c) 840 (d) 42 Sol. Q 3 ⋅ P4 =
n
P5
Sol. It is clear that n ≥ 6.
16. (b) ∴ 3 ⋅ n (n − 1)(n − 2)(n − 3) = (n − 1)(n − 2)(n − 3)
N A-I-A G (n − 4 )(n − 5)
⇒ (n − 1)(n − 2)(n − 3)(n 2 − 12n + 20) = 0
∴ Required number of simple words = 3! = 6
⇒ (n − 1)(n − 2)(n − 3)(n − 10)(n − 2) = 0
17. (b)
∴ n = 10, n ≠ 1, 2, 3 [Qn ≥ 6]
A G A I N A A
⇒ n = 10
I I
19. (d) Let a 0 be the number of stations to the left of the
A I
station I chosen, a1 be the number of stations between the
∴ Number of 7 letter smart words station I and station II, a 2 be the number of stations
between the station II and station III and a 3 be the
7! 7! 7!
= + + = 210 + 420 + 420 = 1050 number of stations to the right of the third station. Then,
4 ! 2! 3! 3! 2! a 0 , a 3 ≥ 0 and a1, a 2 ≥ 1
18. (b) Now, 3 vowels A, I, A are to be placed in the five Also, a 0 + a1 + a 2 + a 3 = n + 1 − 3
available places. Let a = a 0 + 1, b = a 3 + 1, then a, b ≥ 1 such that
× N × G × N × N × a + a1 + a 2 + b = n
 OR ∴ Required number of ways = n −1C 4 −1 = 9C 3 [here, n = 10]

× N × G × G × G × = 84
 OR 20. (c) Number of quadrilaterals of which exactly one side is
 the side of the n-gon
× N × G × G × N ×
= n × n − 4C 2 = 10 × 6C 2 = 150 [Qn = 10]
410 Textbook of Algebra

21. (a) Number of quadrilaterals of which exactly two adjacent Sol.


sides of the quadrilateral are common to the sides of the (2016)!
n-gon 22. (c) Q N
C N /2 = 2016
C 1008 =
[(1008)!]2
n−5
=n × C 1= n (n − 5) = 10 × 5 [Qn = 10]
2016 2016 2016 2016
= 50 E 5 (2016!) =  + + +
 5   5   5   5 
2 3 4

Passage III = 403 + 80 + 16 + 3 = 502


(Ex. Nos. 22 to 23) 1008 1008 1008 1008
and E 5 (1008!) =  + + +
Consider the number N = 2016.  5   5   5   5 
2 3 4

22. Number of cyphers at the end of N C N /2 is = 201 + 40 + 8 + 1 = 250


2016
(a) 0 (b) 1 Hence, the number of cyphers at the end of C 1008
(c) 2 (d) 3 = 502 − 250 − 250 = 2
23. Sum of all even divisors of the number N is 23. (b) Q N = 2016 = 25 ⋅ 32 ⋅ 71
(a) 6552 (b) 6448 ∴ Sum of all even divisors of the number N
(c) 6048 (d) 5733
= (2 + 22 + 23 + 24 + 25 )(1 + 3 + 32 )(1 + 71 ) = 6448

JEE Type Solved Examples :


Single Integer Answer Type Questions
n
This section contains 2 examples. The answer to each l Ex. 25 If λ be the number of 3-digit numbers are of the
example is a single digit integer ranging from 0 to 9 λ
(both inclusive). form xyz with x < y , z < y and x ≠ 0, the value of is
30
 18   18  18   20  Sol. (8) Since, x ≥ 1 , then y ≥ 2 [Q x < y ]
l Ex. 24 If   +2  +   ≥   , then the
r − 2  r − 1  r  13  If y = n , then x takes values form 1 to n − 1 and z can take
number of values of r are the values from 0 to n − 1 (i.e., n values).
Thus, for each values of y (2 ≤ y ≤ 9 ), x and z take n (n − 1)
 18   18  18 20 values.
Sol. (7) We have,   + 2  + ≥ 
r − 2 r − 1  r  13 Hence, the 3-digit numbers are of the form xyz
9 9
It means that 18C r − 2 + 2 ⋅ 18C r −1 + 18C r ≥ 20
C 13 = ∑ n ( n − 1) = ∑ n ( n − 1) [Q at n = 1, n (n − 1) = 0]
n =2 n =1
⇒ ( C r − 2 + C r −1 ) + ( C r −1 + C r ) ≥
18 18 18 18 20
C7
9 9
⇒ 19
C r −1 + 19C r ≥ 20
C7 = ∑n 2 − ∑n
n =1 n =1
⇒ 20
Cr ≥ 20
C7 9 (9 + 1)(18 + 1) 9 (9 + 1)
= −
or 20
Cr ≥ 20
C 13 6 2
= 285 − 45
⇒ 7 ≤ r ≤ 13
= 240 = λ [given]
∴ r = 7, 8, 9, 10, 11, 12, 13 λ
∴ =8
Hence, the number of values of r are 7. 30
Chap 05 Permutations and Combinations 411

JEE Type Solved Examples :


Matching Type Questions
n This section contains 2 examples. Examples 26 and 27 have four statements (A, B and C) given in Column I and four
statements (p, q, r and s) in Column II. Any given statement in Column I can have correct matching with one or more
statement(s) given in Column II.

l Ex. 26 l Ex. 27
Column I Column II
Column I Column II
(A) Four dice (six faced) are rolled. The number of (p) 210
(A) The sum of the factors of 8! which are odd (p) 384 possible outcomes in which atleast one die shows
and are the form 3 λ + 2, λ ∈ N, is 2, is
(B) The number of divisors of n = 2 7 ⋅ 3 5 ⋅ 5 3 (q) 240 (B) Let A be the set of 4-digit numbers a1a 2 a 3 a 4 , where (q) 480
which are the form 4 λ + 1, λ ∈ N, is a1 > a 2 > a 3 > a 4 . Then, n( A ) is equal to
(C) The total number 3-digit numbers, the sum of (r) 671
(C) Total number of divisors of (r) 11
whose digits is even, is equal to
n = 2 5 ⋅ 3 4 ⋅ 510 ⋅ 7 6 which are the form
4 λ + 2, λ ≥ 1, is (D) The number of 4-digit numbers that can be formed (s) 450
from the digits 0, 1, 2, 3, 4, 5, 6, 7, so that each
(D) Total number of divisors of n = 3 5 ⋅ 5 7 ⋅ 7 9 (s) 40 number contains digit 1, is
which are the form 4 λ + 1, λ ≥ 0, is
Sol. (A) → r; (B) → p; (C) → s; (D) → q
(A) The number of possible outcomes with 2 on atleast one die
Sol. (A) → s; (B) → r; (C) → p; (D) → q
= The total number of outcomes with 2 on atleast one die
(A) Here, 8! = 27 ⋅ 32 ⋅ 51 ⋅ 71 = (The total number of outcomes) − (The number of
outcomes in which 2 does not appear on any dice)
So, the factors may be 1, 5, 7, 35 of which 5 and = 64 − 54 = 1296 − 625 = 671
35 are of the form 3λ + 2.
(B) Any selection of four digits from the 10 digits 0, 1, 2, 3,…, 9
∴ Sum is 40. gives one number. So, the required number of numbers
(B) Number of odd numbers = (5 + 1)(3 + 1) = 24 is 10 C 4 i.e., 210.
Required number = 12, but 1 is included. (C) Let the number be n = pqr . Since, p + q + r is even, p can
be filled in 9 ways and q can be filled in 10 ways.
∴ Required number of numbers = 12 − 1 = 11 of
r can be filled in number of ways depending upon what is
the form 4 λ + 1.
the sum of p and q.
(C) Here, 4 λ + 2 = 2(2λ + 1) If ( p + q ) is odd, then r can be filled with any one of five
odd digits.
∴ Total divisors = 1 ⋅ 5 ⋅ 11 ⋅ 7 − 1 = 384
If (p + q ) is even, then r can be filled with any one of five
[Q one is subtracted because there will be case even digits.
when selected powers of 3, 5 and 7 are zero] In any case, r can be filled in five ways.
(D) Here, any positive integer power of 5 will be in Hence, total number of numbers is 9 × 10 × 5 = 450
the form of 4 λ + 1 when even powers of 3 and (D) After fixing 1 at one position out of 4 places, 3 places can
7 will be in the form of 4 λ + 1 and odd powers of be filled by 7 P3 ways. But for some numbers whose fourth
3 and 7 will be in the form of 4 λ − 1. digit is zero, such type of ways is 6 P2 . Therefore, total
∴ Required divisors = 8(3 ⋅ 5 + 3 ⋅ 5) = 240 number of ways is 7 P3 − 6 P2 = 480
412 Textbook of Algebra

JEE Type Solved Examples :


Statement I and II Type Questions
n Directions Example numbers 28 and 29 are l Ex. 29 Statement-1 If f : {a 1 , a 2 , a 3 , a 4 , a 5 } →
Assertion-Reason type examples. Each of these examples
contains two statements: {a 1 , a 2 , a 3 , a 4 , a 5 }, f is onto and f ( x ) ≠ x for each
Statement-1 (Assertion) and Statement-2 (Reason) x ∈{a 1 , a 2 , a 3 , a 4 , a 5 }, is equal to 44.
Each of these examples also has four alternative choices,
only one of which is the correct answer. You have to select l Statement-2 The number of derangement for n objects is
the correct choice as given below.
( −1) r
n
(a) Statement-1 is true, Statement-2 is true; Statement-2 n!∑ .
is a correct explanation for Statement-1
r =0 r !
(b) Statement-1 is true, Statement-2 is true; Statement-2
is not a correct explanation for Statement-1 n
( −1)r  1 1 1 ( −1)n 
Sol. (a) Q Dn = n ! ∑ = n ! 1 − + − + ... + 
(c) Statement-1 is true, Statement-2 is false  n! 
r =0 r ! 1! 2! 3!
(d) Statement-1 is false, Statement-2 is true
 1 1 1 1 1
∴ D 5 = 5! 1 − + − + − 
l Ex. 28 Statement-1 Number of rectangles on a chess-  1! 2! 3! 4 ! 5! 
board is 8 C 2 × 8 C 2 . 1 1 1 1 
= 120  − + − 
Statement-2 To form a rectangle, we have to select any two  2 6 24 120
of the horizontal lines and any two of the vertical lines. = 6 − 20 + 5 − 1
Sol. (d) In a chessboard, there are 9 horizontal lines and 9 = 65 − 21
vertical lines. = 44
∴ Number of rectangles of any size are 9 C 2 × 9C 2 . Hence, Statement-1 is true, Statement-2 is true and
Hence, Statement-1 is false and Statement-2 is true. Statement-2 is a correct explanation for Statement-1.

Subjective Type Examples


n
In this section, there are 17 subjective solved examples. From wavy curve method
x ∈ ( −2,2) ∪ (3,11)
l Ex. 30 Solve the inequality
x −1 x −1 5 x −2 but x ∈N
C4 − C3 − A 2 < 0, x ∈N .
4 ∴ x = 1, 4, 5, 6, 7, 8, 9, 10 …(i)
x −1 5 x −2 From inequality,
Sol. We have, C 4 − x −1C 3 − A2 < 0
4 x − 1 ≥ 4, x − 1 ≥ 3, x − 2 ≥ 2
( x − 1)( x − 2)( x − 3)( x − 4 ) ( x − 1)( x − 2)( x − 3) or x ≥ 5, x ≥ 4, x ≥ 4
⇔ −
1⋅2⋅3⋅4 1⋅2⋅3 Hence, x ≥5 …(ii)
5 From Eqs. (i) and (ii), solutions of the inequality are
− ⋅ ( x − 2)( x − 3) < 0
4 x = 5, 6, 7, 8, 9, 10.
⇔ ( x − 1)( x − 2)( x − 3)( x − 4 ) − 4( x − 1)( x − 2)( x − 3)
−30( x − 2)( x − 3) < 0 l Ex. 31 Find the sum of the series
⇔ ( x − 2)( x − 3){( x − 1)( x − 4 ) − 4( x − 1) − 30} < 0
(1 + 1)1 ! + ( 2 2 + 1) 2 ! + (3 2 + 1) 3 ! +...+(n 2 + 1)n !.
2

+ + + Sol. Let Sn = (12 + 1)1! + (22 + 1)2! + (32 + 1)3! +...+(n 2 + 1)n !
–2 – 2 3 11
∴ nth term Tn = (n 2 + 1)n !

⇔ ( x − 2)( x − 3){ x 2 − 9 x − 22} < 0 = {(n + 1)(n + 2) − 3(n + 1) + 2}n !


⇔ ( x − 2)( x − 3)( x + 2)( x − 11) < 0 Tn = (n + 2)! − 3(n + 1)! + 2n !
Chap 05 Permutations and Combinations 413

Putting n = 1, 2, 3, 4,...,n  1 − x 10 
6

= Coefficient of x 18 in  
Then, T1 = 3! − 3 ⋅ 2! + 2 ⋅ 1!  1− x 
T 2 = 4 ! − 3 ⋅ 3! + 2 ⋅ 2!
= Coefficient of x 18 in (1 − x 10 )6 (1 − x )− 6
T 3 = 5! − 3 ⋅ 4 ! + 2 ⋅ 3!
T 4 = 6! − 3 ⋅ 5! + 2 ⋅ 4 ! = Coefficient of x 18 in (1 − 6 x 10 )(1 + 6C 1x + 7C 2 x 2 + ...
LLLLLLL + 13C 8 x 8 + ... + C 18 x 18 + ...)
23

LLLLLLL = 23
C 18 − 6 ⋅13 C 8 = 23
C 5 − 6 ⋅13 C 5
Tn −1 = (n + 1)! − 3n ! + 2(n − 1)!
23 ⋅ 22 ⋅ 21 ⋅ 20 ⋅ 19 13 ⋅ 12 ⋅ 11 ⋅ 10 ⋅ 9
Tn = (n + 2)! − 3(n + 1)! + 2n ! = − 6⋅
1 ⋅2 ⋅3 ⋅ 4 ⋅5 1 ⋅2 ⋅3 ⋅ 4 ⋅5
∴ Sn = T1 + T 2 + T 3 +...+Tn
= 33649 − 7722 = 25927
= (n + 2)! − 2(n + 1)! [the rest cancel out]
= (n + 2)(n + 1)! − 2(n + 1)!
l Ex. 34 How many different car licence plates can be
= (n + 1)!(n + 2 − 2)
constructed, if the licences contain three letters of the
= n (n + 1)!
English alphabet followed by a three digit number,
l Ex. 32 Find the negative terms of the sequence (i) if repetition are allowed?
n+4
P4 143 (ii) if repetition are not allowed?
xn = − . Sol. (i) Total letters = 26 (i.e., A , B, C , ..., X , Y , Z )
Pn + 2 4Pn
and total digit number = 10 (i.e., 0, 1, 2, ..., 9)
Sol. We have, If three letters on plate is represented by, then first
n+4
P4 143 place can be filled = 26
xn = −
Pn + 2 4 Pn Second place can again be filled = 26
[Q repetition are allowed]
(n + 4 )(n + 3)(n + 2)(n + 1) 143
∴ xn = − and third place can again be filled = 26
(n + 2)! 4n !
(n + 4 )(n + 3)(n + 2)(n + 1) 143
= −
(n + 2)(n + 1)n ! 4n !

(n + 4 )(n + 3) 143 ( 4n 2 + 28n − 95) Hence, three letters can be filled = 26 × 26 × 26


= − =
n! 4n ! 4n ! = (26)3 ways
Q x n is negative and three digit numbers on plate by 999 ways
( 4n + 28n − 95)
2
(i.e., 001, 002, ..., 999)
∴ <0
4n ! Hence, by the principle of multiplication, the required
which is true for n = 1,2. number of ways = (26)3 (999 ) ways
63 23
Hence, x 1 = − and x 2 = − are two negative terms. (ii) Here, three letters out of 26 can be filled = 26
P3
4 8
[Q repetition are not allowed]
l Ex. 33 How many integers between 1 and 1000000 have
and three digit can be filled out of 10 = 10
P3
the sum of the digits equal to 18?
Sol. Integers between 1 and 1000000 will be 1, 2, 3, 4, 5 or 6 [Q repetition are not allowed]
digits and given sum of digits = 18 Hence, required number of ways = ( 26 P3 ) ( 10 P3 ) ways.
Thus, we need to obtain the number of solutions of the
equation l Ex. 35 A man has 7 relatives, 4 of them are ladies and 3
x 1 + x 2 + x 3 + x 4 + x 5 + x 6 = 18 …(i) gentlemen, has wife, has also 7 relatives, 3 of them are ladies
where, 0 ≤ x i ≤ 9, i = 1, 2, 3, 4, 5, 6 and 4 are gentlemen. In how many ways can they invite a
Therefore, the number of solutions of Eq. (i), will be dinner party of 3 ladies and 3 gentlemen so that there are
= Coefficient of x 18 in ( x 0 + x 1 + x 2 + x 3 + ... + x 9 )6 3 of them man’s relatives and 3 of the wife’s relatives?
414 Textbook of Algebra

Sol. The four possible ways of inviting 3 ladies and 3 gentle- and last digit can be filled in 3 ways.
men for the party with the help of the following table : ∴ By the principle of multiplication, the required
Man’s relatives Wife’s relatives number of ways
= 120 × 3 = 360
4 3 3 4 Number of ways
Ladies Gentlemen Ladies Gentlemen (iii) For the number exactly divisible by 4, then last two
digit must be divisible by 4, the last two digits are viz.,
0 3 3 0 4
C 0 × 3C 3 × 3C 3 × 4C 0 = 1
12, 16, 24, 32, 36, 52, 56, 64, 72, 76. Total 10 ways.
1 2 2 1 4
C1 × 3C 2 × 3C 2 × 4C1 = 144 Now, the remaining two first places on the left of
4-digit numbers are to be filled from the remaining
5-digits and this can be done in 5 P2 = 20 ways.
2 1 1 2 4
C 2 × 3C1 × 3C1 × 4C 2 = 324
Hence, by the principle of multiplication, the required
number of ways
3 0 0 3 4
C 3 × 3C 0 × 3C 0 × 4C 3 = 16 = 20 × 10 = 200
(iv) For the number exactly divisible by 25, then last two
∴ Required number of ways to invite = 1 + 144 + 324 + 16 digit must be divisible by 25, the last two digits are
= 485 viz., 25, 75. Total 2 ways.
Now, the remaining two first places on the left of
l Ex. 36 A team of ten is to be formed from 6 male doctors
4-digit number are to be filled from the remaining
and 10 nurses of whom 5 are male and 5 are female. In how
5-digits and this can be done in 5 P2 = 20 ways.
many ways can this be done, if the team must have atleast 4
doctors and atleast 4 nurses with atleast 2 male nurses and Hence, by the principle of multiplication, the required
atleast 2 female nurses? number of ways
Sol. = 20 × 2 = 40
6 5 Male 5 Female Number of ways of
Doctors nurses nurses selection
l Ex. 38 India and South Africa play One Day Interna-
tional Series until one team wins 4 matches. No match ends
4 4 2 6
C 4 × 5C 4 × 5C 2 = 750
in a draw. Find in how many ways the series can be won?
4 3 3 6
C 4 × 5C 3 × 5C 3 = 1500 Sol. Taking I for India and S for South Africa. We can arrange
I and S to show the wins for India and South Africa,
4 2 4 6
C 4 × C 2 × C 4 = 750
5 5
respectively.
5 3 2 6
C 5 × 5C 3 × 5C 2 = 600 For example. , ISSSS means first match is won by India which
is followed by 4 wins by South Africa. This is one way in
5 2 3 6
C 5 × 5C 2 × 5C 3 = 600 which series can be won.
6 2 2
6
C 6 × 5C 2 × 5C 2 = 100 Suppose, South Africa wins the series, then last match is
always won by South Africa.
∴ Total ways = 4300
Wins of I Wins of S Number of ways
l Ex. 37 A number of four different digits is formed with (i) 0 4 1
the help of the digits 1,2,3,4,5,6,7 in all possible ways.
(ii) 1 4 4!
(i) How many such numbers can be formed? =4
3!
(ii) How many of these are even?
(iii) How many of these are exactly divisible by 4? (iii) 2 4 5!
= 10
(iv) How many of these are exactly divisible by 25? 2 !3 !
Sol. Here total digit = 7 and no two of which are alike
(iv) 3 4 6!
(i) Required number of ways = Taking 4 out of 7 = 20
3 !3 !
= 7 P4 = 7 × 6 × 5 × 4 = 840
(ii) For even number last digit must be 2 or 4 or 6. Now ∴ Total number of ways = 35
the remaining three first places on the left of 4-digit In the same number of ways, India can win the series.
numbers are to be fillled from the remaining 6-digits ∴ Total number of ways in which the series can be won
and this can be done in
= 35 × 2 = 70
6
P3 = 6 ⋅ 5 ⋅ 4 = 120 ways
Chap 05 Permutations and Combinations 415

l Ex. 39 Let n and k be positive integers such that Sol. 12, 21 ... 122222222 are form the required numbers we can
k (k + 1) assume all of them to be nine digit in the form
n≥ . Find the number of solutions ( x 1 , x 2 ,..., x k ), a1,a 2 ,a 3 ,a 4 ,a 5 ,a 6 ,a 7 ,a 8 ,a 9 and can use 0 for a1 ;a 2 and a 0 and
2 a 0 , a1,a 2 and a 3 ... and so on to get 8-digit, 7-digit, 6-digit
x 1 ≥ 1, x 2 ≥ 2, ..., x k ≥ k all numbers etc. a1 can assume one of the 2 values of 0 or 1.
integers satisfying x 1 + x 2 + ... + x k = n. a 2 ,a 3 ,a 4 ,a 5 ,a 6 ,a 7 ,a 8 can assume any of 3 values 0, 1, 2.
Sol. We have, x 1 + x 2 + ... + x k = n …(i) The number for which
a1 = a 2 = a 3 = a 4 = a 5 = a 6 = a 7 = a 8 = a 9 = 0 must be
Now, let y1 = x 1 − 1 , y 2 = x 2 − 2, ...,yk = x k − k
eleminated. The sum of first 8-digits i.e., a1 + a 2 +...+a 8 can
∴ y1 ≥ 0, y 2 ≥ 0, ...,yk ≥ 0 be in the form of 3n − 2 or 3n − 1 or 3n.
On substituting the values x 1, x 2 ,..., x k in terms of y1,y 2 ,...,yk In each case a 9 can be chosen from 0,1,2 in only 1 way so
in Eq. (i), we get that the sum of all 9-digits in equal to 3n.
y1 + 1 + y 2 + 2 + ... + yk + k = n ∴ Total numbers = 2 × 37 × 1 − 1 = 4374 − 1 = 4373.
⇒ y1 + y 2 + ... + yk = n − (1 + 2 + 3 + ... + k )
k ( k + 1)
l Ex. 42 There are n straight lines in a plane such that n1 of
∴ y1 + y 2 + ... + yk = n − = A (say) …(ii)
2 them are parallel in one direction, n 2 are parallel in different
The number of non-negative integral solutions of the Eq. direction and so on, nk are parallel in another direction such
(ii) is that n1 + n 2 + ... + nk = n. Also, no three of the given lines
(k + A − 1)! meet at a point. Prove that the total number of points of
= k + A −1 C A = intersection is
A !(k − 1)!
1  2 k 
2
where, A =n−
k ( k + 1) n − ∑ nr .
2 2  r =1 
Sol. Total number of points of intersection when no two of n
l Ex. 40 Find the number of all whole numbers formed on given lines are parallel and no three of them are concur-
the screen of a calculator which can be recognised as rent, is n C 2 . But it is given that there are k sets of
numbers with (unique) correct digits when they are read n1, n 2 , n 3 , ..., nk parallel lines such that no line in one set is
inverted. The greatest number formed on its screen is 999999. parallel to line in another set.
Sol. The number can use digits 0, 1, 2, 5, 6, 8 and 9 because Hence, total number of points of intersection
they can be recognised as digits when they are see
inverted. = n C 2 − (n 1 C 2 + n2
C2 + K + nk
C2)
A number can’t begin with ,therefore all numbers having n (n − 1)  n1(n1 − 1) n 2 (n 2 − 1) n ( n − 1) 
= − + +K+ k k 
at unit’s digit should no be counted. (when those numbers 2  2 2 2 
will be read inverted they will begin with).
n ( n − 1) 1
= − {(n12 + n 22 + K + nk2 ) − (n1 + n 2 + K + nk )}
No. of digits Total numbers 2 2
1 7 n ( n − 1) 1  k 2 
= −  ∑ nr − n 
2 6 2 = 36 2 2 r = 1 
n 2
1 k
1  k 
3 6 × 7 × 6 = 252 = − ∑ nr2 = n 2 − ∑ nr2 
2 2r = 1 2 r =1 
4 6 × 7 2 × 6 = 1764

5 6 × 7 3 × 6 = 12348 l Ex. 43 There are p intermediate stations on a railway line


from one terminus to another. In how many ways a train can
6 6 × 7 4 × 6 = 86436
stop at 3 of these interediate stations, if no two of these stop-
∴ Total = 100843 ping stations are to be consecutive?

Sol. A B
l Ex. 41 How many different numbers which are smaller S1 S2 S3 Sn –1 Sn Sn +1 Sp –1 Sp
than 2 × 10 8 and are divisible by 3, can be written by means Let there be p intermediate stations between two terminus
of the digits 0, 1 and 2? stations A and B as shown above.
416 Textbook of Algebra

Number of ways the train can stop in three intermediate l Ex. 45 There are 2n guests at a dinner party. Supposing
stations = p C 3 that the master and mistress of the house have fixed seats
These are comprised of two exclusive cases viz. opposite one another and that there are two specified guests
(i) atleast two stations are consecutive. who must not be placed next to one another. Find the
(ii) now two of which is consecutive. number of ways in which the company can be placed.
Now, there are ( p − 1) pairs of consecutive intermediate Sol. Let the M and M ′ represent seats of the master and
stations. mistress respectively and let a1, a 2 , a 3 , K, a 2n represent the
2n seats.
In order to get a station trio in which atleast two stations an + 1
an
an + 2
are consecutive, each pair can be associated with a third M´
station in ( p − 2) ways. Hence, total number of ways in an + 3
which 3 stations consisting of atleast two consecutive a4
stations, can be chosen in ( p − 1) ( p − 2) ways. Among
these, each triplet of consecutive stations occur twice. a3

For example , the pair (Sn , Sn − 1 ) when combined with Sn + 1 a2


and the pair (Sn , Sn + 1 ) when combined with Sn − 1 gives the 2n a1
M
same triplet and is counted twice. So, the number of three
consecutive stations trio should be subtracted. Let the guests who must not be placed next to one another
be called P and Q.
Now, number of these three consecutive stations trio is
( p − 2). Now, put P at a1 and Q at any position, other than a 2 , say at
a 3 , then remaining (2n − 2) guests can be arranged in the
Hence, the number of ways the triplet of stations consisting remaining (2n − 2) positions in (2n − 2) ! ways. Hence, there
of atleast two consecutive stations can be chosen in will be altogether (2n − 2) (2n − 2) ! arrangements of the
= {( p − 1) ( p − 2) − ( p − 2)} ways guests, when P is at a1.
= ( p − 2)2 ways The same number of arrangements when P is at an or an + 1
Therefore, number of ways the train can stop in three or a 2n . Thus, for these positions (a1, an , an + 1, a 2n ) of P, there
consecutive stations are altogether 4 (2n − 2) (2n − 2) ! ways. ...(i)
p ( p − 1) ( p − 2) If P is at a 2 , then there are altogether (2n − 3) positions for
= p C 3 − ( p − 2) 2 = − ( p − 2) 2 Q. Hence, there will be altogether (2n − 3) (2n − 2) !
1 ⋅2 ⋅3
arrangements of the guests, when P is at a 2 .
 p 2 − p − 6p + 12 ( p − 2) ( p 2 − 7 p + 12)
= ( p − 2)  = The same number of arrangements can be made when P is
 6  6 at any other position excepting the four positions
( p − 2) ( p − 3) ( p − 4 ) ( p − 2 ) a1, an , an + 1, a 2n .
= = C3 Hence, for these (2n − 4 ) positions of P, there will be
1 ⋅2 ⋅3
altogether
l Ex. 44 How many different 7-digit numbers are there and (2n − 4 ) (2n − 3) (2n − 2) ! arrangements of the guests ...(ii)
sum of whose digits is even? Hence, from Eqs. (i) and (ii), the total number of ways of
Sol. Let us consider 10 successive 7-digit numbers arranging the guests
a1a 2a 3a 4a 5a 6 0, = 4 (2n − 2) (2n − 2) ! + (2n − 4 ) (2n − 3) (2n − 2) !
a1a 2a 3a 4a 5a 6 1, = ( 4n 2 − 6n + 4 ) (2n − 2) !
a1a 2a 3a 4a 5a 6 2,
.....................
l Ex. 46 Find the number of triangles whose angular points are
a1a 2a 3a 4a 5a 6 9
at the angular points of a given polygon of n sides, but none of
whose sides are the sides of the polygon.
where, a1, a 2 , a 3 , a 4 , a 5 and a 6 are some digits. We see that
half of these 10 numbers, i.e. 5 numbers have an even sum Sol. A polygon of n sides has n angular points. Number of
of digits. triangles formed from these n angular points = n C 3 .
The first digit a1 can assume 9 different values and each of These are comprised of two exclusive cases viz.
the digits a 2 , a 3 , a 4 , a 5 and a 6 can assume 10 different (i) atleast one side of the triangle is a side of the polygon.
values. (ii) no side of the triangle is a side of the polygon.
The last digit a 7 can assume only 5 different values of D
which the sum of all digits is even.
∴ There are 9 × 105 × 5 = 45 × 105 , 7-digit numbers the sum C
of whose digits is even.
A B
Chap 05 Permutations and Combinations 417

Let AB be one side of the polygon. If each angular point of ( m + p − 1) !


m ( m + 1) ( m + 2) K ( m + p − 1) =
the remaining (n − 2) points are joined with A and B, we get ( m − 1) !
a triangle with one side AB.
( m + p − 1) !
∴ Number of triangles of which AB is one side = (n − 2) =p!
( m − 1) ! p !
Likewise, number of triangles of which BC is one side
= (n − 2) and of which atleast one side is the side of the = p !⋅m + p − 1 C p
polygon = n (n − 2).
Since, m + p − 1C p is an integer.
Out of these triangle, some are counted twice. For example ,
the triangle when C is joined with AB is ∆ABC, is taken m + p −1 m ( m + 1) ( m + 2) K ( m + p − 1)
when AB is taken as one side. Again triangle formed when ∴ Cp =
p!
A is joined with BC is counted when BC is taken as one
side. Now, (n !) ! is the product of the positive integers from 1 to
n ! . We write the integers from 1 to n ! is (n − 1) ! rows as
Number of such triangles = n
follows:
So, the number of triangles of which one side is the side of
the triangle 1 2 3 ... n
= n ( n − 2) − n = n ( n − 3) n+1 n+2 n+3 ... 2n
Hence, the total number of required triangles
2n + 1 2n + 2 2n + 3 ... 3n
1
= n C 3 − n ( n − 3) = n ( n − 4 ) ( n − 5) : : : : :
6
n!−n + 1 n!−n + 2 n!−n + 3 .... n!
l Ex. 47 Prove that (n !)! is divisible by (n !) (n − 1) ! .
Each of these (n − 1) ! rows contain n consecutive positive
Sol. First we show that the product of p consecutive positive
integers. The product of the consecutive integers in each
integers is divisible by p ! . Let the p consecutive integers
row is divisible by n ! . Thus, the product of all the integers
be m, m + 1, m + 2, ..., m + p − 1. Then,
from 1 to n ! is divisible by (n !)(n − 1) ! .
#L Permutations and Combinations Exercise 1 :
Single Option Correct Type Questions
n This section contains 30 multiple choice questions. 10. If the permutations of a, b, c, d and e taken all together
Each question has four choices (a), (b), (c) and (d) out of are written down in alphabetical order as in dictionary
which ONLY ONE is correct and numbered, the rank of the permutation debac is
1. A lady gives a dinner party to 5 guests to be selected (a) 90 (b) 91 (c) 92 (d) 93
from nine friends. The number of ways of forming the 11. On a railway there are 20 stations. The number of
party of 5, given that two of the friends will not attend different tickets required in order that it may be possible
the party together, is to travel from every station to every station, is
(a) 56 (b) 126 (c) 91 (d) None of these (a) 210 (b) 225 (c) 196 (d) 105
2. If a, b, c and d are odd natural numbers such that 12. A set containing n elements. A subset P of A is chosen.
a + b + c + d = 20, the number of values of the ordered The set A is reconstructed by replacing the element of P.
quadruplet (a, b, c , d ) is A subset Q of A is again chosen the number of ways of
(a) 165 (b) 455 (c) 310 (d) None of these choosing P and Q, so that P ∩ Q = φ, is
3. If l = LCM of 8!, 10! and 12! and h = HCF of 8!, 10! and (a) 2 2n − 2nCn (b) 2n (c) 2n − 1 (d) 3n
l 13. The straight lines I 1 , I 2 , I 3 are parallel and lie in the
12!, then is equal to
h same plane. A total number of m points on I 1 ; n points
(a) 132 (b) 11800
on I 2 ; k points on I 3 , the maximum number of triangles
(c) 11880 (d) None of these
formed with vertices at these points is
4. The number of positive integers satisfying the inequality (a) m + n + k C 3 (b) m + n + k C 3 − mC 3 − nC 3 − kC 3
n +1 n +1
Cn − 2 − C n − 1 ≤ 100 is (c) m C 3 + nC 3 + kC 3 (d) None of these
(a) 9 (b) 8
14. Let A be a set of n(≥ 3) distinct elements. The number of
(c) 5 (d) None of these
triplets ( x , y, z ) of the set A in which atleast two
5. The number of ways in which a score of 11 can be made coordinates are equal to
from a through by three persons, each throwing a single (a) n P3 (b) n 3 − n P3
die once, is (c) 3n 2 − 2n (d) 3n 2(n − 1 )
(a) 45 (b) 18 (c) 27 (d) 68
15. The total number of five-digit numbers of different
6. The number of positive integers with the property that digits in which the digit in the middle is the largest, is
they can be expressed as the sum of the cubes of 2
(a) 2 2 ⋅ 3 2 ⋅ 7 2 (b) 2 3 ⋅ 3 ⋅ 7 3
positive integers in two different ways is
(c) 2 2 ⋅ 3 3 ⋅ 7 2 (d) 2 3 ⋅ 3 2 ⋅ 7 3
(a) 1 (b) 100 (c) infinite (d) 0
16. The total number of words that can be formed using all
7. In a plane there are 37 straight lines, of which 13 pass
letters of the word ‘RITESH’ that neither begins with I
through the point A and 11 pass through the point B.
nor ends with R, is
Besides, no three lines pass through one point, no line
(a) 504 (b) 480
passes through both points A and B and no two are parallel, (c) 600 (d) 720
the number of intersection points the lines have, is
(a) 535 (b) 601 (c) 728 (d) 963 17. A man has three friends. The number of ways he can
invite one friend everyday for dinner on six successive
8. If a denotes the number of permutations of x + 2 things nights, so that no friend is invited more than three
taken all at a time, b the number of permutations of x times, is
things taken 11 at a time and c the number of (a) 360 (b) 420 (c) 170 (d) 510
permutations of x − 11 things taken all at a time such
18. The number of three digit numbers of the form xyz such
that a = 182bc , the value of x is
(a) 15 (b) 12 (c) 10 (d) 18 that x < y, z ≤ y and x ≠ 0, is
(a) 240 (b) 244 (c) 276 (d) 285
9. The number of numbers less than 1000 that can be
formed out of the digits 0, 1, 2, 3, 4 and 5, no digit being 19. The letters of the word ‘MEERUT’ are arranged in all
repeated, is possible ways as in a dictionary, then the rank of the
(a) 130 (b) 131 (c) 156 (d) 158 word ‘MEERUT’ is
(a) 119 (b) 120 (c) 121 (d) 122
Chap 05 Permutations and Combinations 419

20. The number of ways in which 10 candidates A 1 , A 2 , … , 27. Let x 1 , x 2 , x 3 , …, x k be the divisors of positive integer
A 10 can be ranked so that A 1 is always above A 2 , is ‘n’ (including 1 and x ). If x 1 + x 2 + … + x k = 75, then
10 ! k
1
(a) 10! (b)
2 ∑ x i is equal to
(c) 9! (d) None of these i =1
k2 75 n2 75
21. Let A be the set of four digit numbers a1a 2 a 3 a 4 , where (a) (b) (c) (d)
75 k 75 n
a1 > a 2 > a 3 > a 4 , then n( A ) is
(a) 126 (b) 84 28. The total number of functions ‘f ’ from the set {1, 2, 3}
(c) 210 (d) None of these into the set {1, 2, 3, 4, 5} such that f (i ) ≤ f ( j ), ∀ i < j, is
(a) 35 (b) 30 (c) 50 (d) 60
22. The number of distinct rational numbers x such that
p 29. Ten persons numbered 1, 2, 3, …, 10 play a chess
0 < x < 1 and x = , where p, q ∈{1, 2, 3, 4, 5, 6} is
q tournament, each player playing against every player
(a) 15 (b) 13 (c) 12 (d) 11 exactly one game. It is known that no game ends in a
draw. Let w 1 , w 2 , w 3 , …, w 10 be the number of games
23. The total number of integral solutions of xyz = 24 is won by player 1, 2, 3, …, 10 respectively and l 1 , l 2 , l 3 , …,
(a) 30 (b) 36 (c) 90 (d) 120 l 10 be the number of games lost by the players 1, 2, 3, …,
24. If ABCD is a convex quadrilateral with 3, 4, 5 and 6 10 respectively, then
points, marked on sides AB, BC, CD and DA respectively, (a) Σwi2 = 81 − Σli2 (b) Σwi2 = 81 + Σli2
then the number of triangles with vertices on different (c) Σwi2 = Σli2 (d) None of these
sides, is
30. In the next world cup of cricket there will be 12 teams,
(a) 220 (b) 270 (c) 282 (d) 342
divided equally in two groups. Teams of each group will
play a match against each other. From each group 3 top
25. The number of ways can a team of six horses be selected teams will qualify for the next round. In this round each
out of a stud of 16, so that there shall always be three team will play against others once.
out of A, B, C, D, E, F but never AD, BE or CF together, is Four top teams of this round will qualify for the semi- final
(a) 720 (b) 840 (c) 960 (d) 1260
round, where each team will play against the others once.
26. The number of polynomials of the form Two top teams of this round will go to the final round,
x 3 + ax 2 + bx + c that are divisible by x 2 + 1 , where a, where they will play the best of three matches. The
b, c ∈{1, 2, 3, 4, 5, 6, 7, 8, 9, 10}, is minimum number of matches in the next world cup will be
(a) 10 (b) 15 (c) 5 (d) 8 (a) 54 (b) 53 (c) 38 (d) 37

#L Permutations and Combinations Exercise 2 :


More than One Correct Option Type Questions
n
This section contains 10 multiple choice questions. 34. Let n be 4-digit integer in which all the digits are
Each question has four choices (a), (b), (c) and (d) out of different. If x is the number of odd integers and y is the
which MORE THAN ONE may be correct.
number of even integers, then
31. If 300 ! = 3m × an integer, then (a) x < y (b) x > y
(a) m = 148 (b) m = 150
(c) x + y = 4500 (d) | x − y | = 56
(c) It is equivalent to number of n in 150 ! = 2n − 2 × an integer
(d) m = 150C 2 35. Let S = {1, 2, 3, …, n }. If X denotes the set of all subsets of
α β γ δ S containing exactly two elements, then the value of
32. If 102 ! = 2 ⋅ 3 ⋅ 5 ⋅ 7 …, then
(a) α = 98 (b) β = 2 γ + 1 (c) α = 2 β (d) 2 γ = 3 δ
∑ (min. A ) is given by
33. The number of ways of choosing triplet ( x , y, z ) such A ∈X
that z ≥ max {x , y } and x, y, z ∈ {1, 2, 3, …, n, n + 1}, is (a) n + 1C 3 (b) n C 3
n(n + 1 )(2n + 1 ) n (n 2 − 1 ) n (n 2 − 3n + 2 )
(a) n + 1C 3 + n+ 2
C3 (b) (c) (d)
6 6 6
(c) 1 2 + 2 2 + 3 2 + … + n 2 (d) 2(n + 2C 3 ) − n + 1C 2
420 Textbook of Algebra

36. Let p = 2520, x = number of divisors of p which are (b) maximum number of fights is 191
(c) maximum number of fights occur when x = 7
multiple of 6, y = number of divisors of p which are
(d) maximum number of fights occur when x = 8
multiple of 9, then
(a) x = 12 (b) x = 24 (c) y = 12 (d) y = 16 39. Let N denotes the number of ways in which 3n letters
37. If N denotes the number of ways of selecting r objects can be selected from 2n A’s, 2nB’s and 2nC ’s. Then,
out of n distinct objects (r ≥ n ) with unlimited repetition (a) 3 | ( N − 1 )
but with each object included atleast once in selection, (b) n | ( N − 1 )
then N is (c) (n + 1 ) | ( N − 1 )
(a) r − 1Cr − n (b) r − 1Cn (d) 3n(n + 1 ) | ( N − 1 )
(c) r − 1Cn − 1 (d) r − 1Cr − n − 1 40. If α = x 1 x 2 x 3 and β = y 1y 2 y 3 are two 3-digit numbers,
38. There are three teams x, x + 1 and y childrens and total then the number of pairs of α and β can be formed so
number of childrens in the teams is 24. If two childrens that α can be subtracted from β without borrowing, is
(a) 2 ! × 10 ! × 10 ! (b) ( 45 )(55 ) 2
of the same team do not fight,then
(a) maximum number of fights is 190 (c) 3 2 ⋅ 5 3 ⋅ 11 2 (d) 136125

#L Permutations and Combinations Exercise 3 :


Passage Based Questions
n
This section contains 5 passages. Based upon each of 46. The number of words in which the consonants appear in
the passage 3 multiple choice questions have to be alphabetic order, is
answered. Each of these questions has four choices (a), (a) 42 (b) 40 (c) 420 (d) 480
(b), (c) and (d) out of which ONLY ONE is correct.
Passage III (Q. Nos. 47 to 49)
Passage I (Q. Nos. 41 to 43)
Different words are being formed by arranging the letters
Consider the word W = TERRORIST. of the word ‘ARRANGE’.
41. Number of four letter words that can be made using 47. The number of words in which the two R’s are not
only the letters from the word W, if each word must together, is
contains atleast one vowel, is (a) 1260 (b) 960
(a) 588 (b) 504 (c) 294 (d) 600 (c) 900 (d) 600
42. Number of arrangements of the word W, if no two R’s 48. The number of words in which neither two R’s nor two
are together, is A’s come together, is
(a) 11460 (b) 10400 (a) 1260 (b) 900
(c) 12600 (d) 9860 (c) 660 (d) 240
43. Number of arrangements of the word W, if R’s as well as 49. The rank of the word ‘ARRANGE’ in the dictionary is
T’s are separated, is (a) 340 (b) 341 (c) 342 (d) 343
(a) 9860 (b) 1080 (c) 10200 (d) 11400
Passage IV (Q. Nos. 50 to 52)
Passage II (Q. Nos. 44 to 46) Let S ( n) denotes the number of ordered pairs ( x, y)
1 1 1
Different words are formed by arranging the letters of the satisfying + = , ∀ x, y, n ∈ N .
word ‘SUCCESS’. x y n
44. The number of words in which C’s are together but S’s 50. S (10) equals
are separated, is (a) 3 (b) 6 (c) 9 (d) 12
(a) 120 (b) 96
(c) 24 (d) 420 51. S(6) + S(7 ) equals
(a) S (3 ) + S ( 4 ) (b) S (5 ) + S (6 ) (c) S (8 ) + S (9 ) (d) S (1 ) + S (11 )
45. The number of words in which no two C’s and no two 10
S’s are together, is
(a) 120 (b) 96
52. ∑ S(r ) equals
r =1
(c) 24 (d) 180
(a) 47 (b) 48 (c) 49 (d) 50
Chap 05 Permutations and Combinations 421

Passage V (Q. Nos. 53 to 55) 54. The number of solutions of the equation f (n ) = n, where
Let f ( n) denotes the number of different ways, the positive n ∈ N is
integer n can be expressed as the sum of the 1’s and 2’s. (a) 1 (b) 2 (c) 3 (d) 4
For example, f ( 4) = 5 55. In a stage show, f ( 4 ) superstars and f (3) junior artists
i.e., 4 =1 +1 +1 +1 participate. Each one is going to present one item, then
the number of ways the sequence of items can be
=1 +1 + 2 =1 + 2 +1= 2 +1 +1 = 2 + 2 planned, if no two junior artists present their items
53. The value of f { f (6)} is consecutively, is
(a) 376 (b) 377 (c) 321 (d) 370 (a) 144 (b) 360 (c) 4320 (d) 14400

#L Permutations and Combinations Exercise 4 :


Single Integer Answer Type Questions
n This section contains 10 questions. The answer to each question is a single digit integer, ranging from
0 to 9 (both inclusive).
56. The ten’s digit of 1 ! + 2 ! + 3 ! + … + 97 ! is 62. If 2λ is the number of ways of selecting 3 member
100 subset of {1, 2, 3, …, 29}, so that the numbers form
57. The exponent of 7 in C 50 is of a GP with integer common ratio, then the value of
58. Let Pn denotes the number of ways in which three people λ is
can be selected out of n people sitting in a row, in two of 63. In a certain test, there are n questions. In this test, 2n − k
them are consecutive. If Pn + 1 − Pn = 15, the value of n is students gave wrong answers to atleast k questions,
59. If the letters of the word are arranged as in a dictionary. where k = 1, 2, 3, …, n. If the total number of wrong
answers given is 127, then the value of n is
m and n are the rank of the words BULBUL and NANNU
respectively, then the value of m − 4n is 64. A 7-digit number made up of all distinct digits 8, 7, 6, 4,
2, x and y, is divisible by 3. The, possible number of
60. An n-digit number is a positive number with exactly n
ordered pair ( x , y ) is
digits. Nine hundred distinct n-digit numbers are to be
formed using only the three digits 2, 5 and 7. The 65. There are five points A, B, C, D and E. No three points
smallest value of n for which this is possible, is are collinear and no four are concyclic. If the line AB
intersects of the circles drawn throught the five points.
61. If a, b, c are three natural numbers in AP such that The number of points of intersection on the line apart
a + b + c = 21 and if possible number of ordered triplet
from A and B is
(a, b, c ) is λ, then the vlaue of ( λ − 5) is

#L Permutations and Combinations Exercise 5 :


Matching Type Questions
n
This section contains 5 questions. Questions 66 to 70 have four statements (A, B, C and D) given in Column I and
four statements (p, q, r and s) in Column II. Any given statement in Column I can have correct matching with one
or more statement(s) given in Column II.
66.
Column I Column II
n+ 4 n+ 3
(A) Cn + 1 − Cn = 15 (n + 2 ), then n equals (p) 19

(B) 11 ⋅n P4 = 20 ⋅n − 2 P4 , then n equals (q) 27

(C) 2n
C 3 = 11 ⋅n C 3, then n equals (r) 16
n+ 2 n−2
(D) C8 : P4 = 57 : 16, then n equals (s) 6
422 Textbook of Algebra

67. 69. Consider a 6 × 6 chessboard. Then, match the following


Column I Column II columns.
(A) Number of increasing permutations of (p) nm
m symbols are there from the n set Column I Column II
numbers (a1 , a2 , K , an ), where the (A) Number of rectangles is (p) 10
C5
order among the numbers, is given by
a1 < a2 < a3 < K < am − 1 < am is (B) Number of squares is (q) 441
m
(B) There are m men and n monkeys. (q) Cn
Number of ways in which every (C) Number of ways three squares can be (r) 91
monkey has a master, if a man can selected, if they are not in same row
have any number of monkeys is or column is
n
(C) Number of ways in which r red balls (r) Cm (D) In how many ways eleven ‘+’ sign (s) 2400
and (m − 1) green balls can be can be arranged in the squares, if no
arranged in a line, so that no two red row remains empty
balls are together is (balls of the same
colour are alike)
70. 5 balls are to be placed in 3 boxes. Each box can hold all
(D) Number of ways in which ‘m’ (s) mn the 5 balls. Number of ways in which the balls can be
different toys can be distributed in n
children, if every child may received placed so that no box remain empty, if
any number of toys is
Column I Column II

68. (A) balls are identical but boxes are (p) 2


Column I Column II different

(A) Number of straight lines joining any two of (p) 30 (B) balls are different but boxes are (q) 25
10 points of which four point are collinear is identical
(B) Maximum number of points of intersec- (q) 60 (C) balls as well as boxes are identical (r) 50
tion of 10 straight lines in the plane is
(C) Maximum number of points of intersection (r) 40 (D) balls as well as boxes are identical (s) 6
of 6 circles in the plane is but boxes kept in a row
(D) Maximum number of points of intersection (s) 45
of 6 parabolas is

#L Permutations and Combinations Exercise 6 :


Statement I and II Type Questions
n
Directions (Q. Nos. 71 to 82) are Assertion-Reason Statement-2 Product of three consecutive integers is
type questions. Each of these questions contains two divisible by 6.
statements:
Statement-1 (Assertion) and Statement-2 (Reason) 72. Statement-1 A number of four different digits is
Each of these questions also has four alternative formed with the help of the digits 1, 2, 3, 4, 5, 6, 7 in all
choices, only one of which is the correct answer. You possible ways. The number of ways which are exactly
have to select the correct choice as given below. divisible by 4 is 200.
(a) Statement-1 is true, Statement-2 is true; Statement-2 Statement-2 A number divisible by 4, if units place
is a correct explanation for Statement-1 digit is also divisible by 4.
(b) Statement-1 is true, Statement-2 is true; Statement-2
is not a correct explanation for Statement-1 73. Statement-1 The number of divisors of 10! is 280.
(c) Statement1 is true, Statement-2 is false Statement-2 10! = 2 p ⋅ 3q ⋅ 5r ⋅ 7 s , where p , q , r , s ∈ N .
(d) Statement-1 is false, Statement-2 is true
74. Statement-1 Number of permutations of ‘n’ dissimilar
71. Statement-1 The smallest positive integer n such that things taken ‘n’ at a time is n!.
n ! can be expressed as a product of n-3 consecutive
Statement-2 If n( A ) = n( B ) = n, then the total number
integers, is 6.
of functions from A to B are n !.
Chap 05 Permutations and Combinations 423

75. Statement-1 If N the number of positive integral 79. Statement-1 The sum of the digits in the ten’s place of
solutions of x 1 x 2 x 3 x 4 = 770, then N is divisible by 4 all numbers formed with the help of 3, 4, 5, 6 taken all at
distinct prime numbers. a time is 108.
Statement-2 Prime numbers are 2, 3, 5, 7, 11, 13, …. Statement-2 The sum of the digits in the ten’s places =
The sum of the digits is the units’s place.
76. Statement-1 The total number of ways in which three
distinct numbers in AP, can be selected from the set {1,
80. Statement-1 There are p ≥ 8 points in space no four of
2, 3, …, 21}, is equal to 100. which are in the same with exception of q ≥ 3 points
which are in the same plane, then the number of planes
Statement-2 If a, b, c are in AP, then a + c = 2b. each containing three points is p C 3 − q C 3 .
77. Statement-1 The number of even divisors of the Statement-2 3 non-collinear points always determine
numbers N = 12600 is 54. unique plane.
Statement-2 0, 2, 4, 6, 8, … are even integers. 81. Statement-1 The highest power of 3 in 50 C 10 is 4.
78. Statement-1 A 5-digit number divisible by 3 is to be Statement-2 If p is any prime number, then power of
   
formed using the digits 0, 1, 2, 3, 4, 5 and 6 without p in n ! is equal to  n  +  n  +  n  + K, where [⋅]
2 3
repetition, then the total number of ways this can be done is  p   p   p 
216. denotes the greatest integer function.
82. Statement-1 A convex quindecagon has 90 diagonals.
Statement-2A number is divisible by 3, if sum of its
digits is divisible by 3. Statement-2 Number of diagonals in a polygon is
n
C 2 − n.

#L Permutations and Combinations Exercise 7 :


Subjective Type Questions
n
In this section, there are 17 subjective questions. 91. Every man who has lived on earth has made a certain
83. Given that C n −r +3 C n − r + 1 + 3.
n n number of handshakes. Prove that the number of men
who have made an odd number of handshakes is even.
n
C n − r + 2 + n C n − r + 3 = x C r . Find x 92. A train is going from Cambridge to London stops at nine
intermediate stations. Six persons enter the train during
84. Solve the equation 3 x +1 C 2 + P2 x = 4 x A 2 , x ∈ N . the journey with six different tickets. How many
85. How many positive terms are there in the sequence ( x n ) different sets of tickets they have had?
195 n + 3 A 3 93. n different things are arranged around a circle. In how
if x n = − ,n ∈ N?
4 Pn Pn +1 many ways can 3 objects be selected when no two of the
n −1
selected objects are consecutive?
86. Prove that C 3 + n −1 C 4 > n C 3 if n > 7.
94. A boat ’s crew consists of 8 men, 3 of whom can only
87. In how many ways can a mixed doubles game in tennis row on one side and 2 only on the other. Find the
be arranged from 5 married couples, if no husband and number of ways in which the crew can be arranged.
wife play in the same game? 95. In how many different ways can a set A of 3n elements
be partitioned into 3 subsets of equal number of
88. In how many ways, we can choose two teams of mixed elements ? (The subsets P, Q, R form a partition if
double for a tennis tournament from four couples such P ∪ Q ∪ R = A, P ∩ R = φ, Q ∩ R = φ, R ∩ P = φ.)
that if any couple participates, then it is in the same
96. A square of n units by n units is divided into n 2 squares
team?
each of area 1 sq unit. Find the number of ways in which
89. A family consists of a grandfather, 5 sons and daughters 4 points (out of (n + 1) 2 vertices of the squares) can be
and 8 grand child. They are to be seated in a row for chosen so that they form the vertices of a square.
dinner. The grand children wish to occupy the 4 seats at
97. How many sets of 2 and 3 (different) numbers can be
each end and the grandfather refuses to have a grand formed by using numbers between 0 and 180 (both
child on either side of him. In how many ways can the including) so that 60 is their average?
family be made to sit?
98. There are n straight lines in a plane, no two of which are
90. A tea party is arranged for 16 people along two sides of parallel and no three passes through the same point. Their
a large table with 8 chairs on each side. Four men sit on point of intersection are joined. Show that the number of
one particular side and two on the other side. In how 1
many ways can they be seated? fresh lines thus introduced, is n (n − 1) (n − 2) (n − 3).
8
424 Textbook of Algebra

99. 6 balls marked as 1, 2, 3, 4, 5 and 6 are kept in a box. If this sum is even, then 1 point is given to the player.
Two players A and B start to take out 1 ball at a time The first player to get 2 points is declared winner. At the
from the box one after another without replacing the start of the game, the sum is 0. If A starts to take out the
ball till the game is over. The number marked on the ball ball, find the number of ways in which the game can be
is added each time to the previous sum to get the sum of won.
numbers marked on the balls taken out.

#L Permutations and Combinations Exercise 8 :


Questions Asked in Previous 13 Year’s Exam
n This section contains questions asked in IIT-JEE, 105. The set S = {1, 2, 3, ..., 12} to be partitioned into three sets
AIEEE, JEE Main & JEE Advanced from year 2005 A, B, C of equal size. Thus,
to year 2017.
A ∪ B ∪ C = S, A ∩ B = B ∩ C = A ∩ C = φ. The
100. There is a rectangular sheet of dimension number of ways to partition S, is [AIEEE 2007, 3M]
(2m − 1) × (2n − 1), (where m > 0, n > 0). It has been (a)
12 !
(b)
12 !
divided into square of unit area by drawing line 3 !( 4 !) 3 3 !(3 !) 4
perpendicular to the sides. Find the number of rectangles 12 ! 12 !
(c) (d)
having sides of odd unit length. [IIT-JEE 2005, 3M] ( 4 !) 3 (3 !) 4
2m –1
106. Consider all possible permutations of the letters of the
word ENDEANOEL. Match the statements/ expressions
in Column I with the statements/ expressions in
2n –1
Column II. [IIT-JEE 2008, 6M]
Column I Column II
(A) The number of permutations (p) 5!
(a) (m + n + 1 ) 2 (b) mn(m + 1 )(n + 1 )
containing the word ENDEA is
(c) 4m + n − 2 (d) m 2n 2
(B) The number of permutations in (q) 2 × 5!
101. If the letters of the word SACHIN arranged in all which the letters E occurs in the
possible ways and these words are written out as in first and the last positions, is
dictionary, then the word SACHIN appears at series (C) The number of permutations in (r) 7 × 5!
number [AIEEE 2005, 3M] which none of the letters D, L, N
(a) 603 (b) 602 (c) 601 (d) 600 occurs in the last five positions, is
102. If r, s, t are prime numbers and p, q are the positive (D) The number of permutations in (s) 21 × 5 !
2 4 2 which the letters A, E, O occur only
integers such that LCM of p, q is r t s , then the in odd positions, is
number of ordered pair ( p , q ) is [IIT-JEE 2006, 3M]
(a) 252 (b) 254 (c) 225 (d) 224 107. How many different words can be formed by Jumbling
103. At an election, a voter may vote for any number of the letters in the word MISSISSIPPI in which no two S’s
candidates, not greater than number to be elected. There are adjacent? [AIEEE 2008, 3M]
are 10 candidates and 4 are to be selected. If a voter (a) 6 ⋅ 7 ⋅8 C 4 (b) 6 ⋅ 8 ⋅7 C 4
votes for atleast one candidate, then number of ways in (c) 7 ⋅6 C 4 ⋅8 C 4 (d) 8 ⋅6 C 4 ⋅7 C 4
which he can vote, is [AIEEE 2006, 4.5M]
108. In a shop, there are five types of ice-creams available. A
(a) 5040 (b) 6210
(c) 385 (d) 1110
child buys six ice-creams. [AIEEE 2008, 3M]
Statement-1 The number of different ways the child
104. The letters of the word COCHIN are permuted and all
can buy the six ice-creams is 10 C 4 .
the permutations are arranged in an alphabetical order
as in English dictionary. The number of words that Statement-2 The number of different ways the child
appear before the word COHIN is [IIT-JEE 2007, 3M] can buy six ice-creams is equal to the number of
(a) 360 (b) 192 different ways to arranging 6A’s and 4B’s in a row.
(c) 96 (d) 48 [AIEEE 2008, 3M]
Chap 05 Permutations and Combinations 425

(a) Statement-1 is true, Statement-2 is true; Statement-2 is a 115. The value of b 6 , is [IIT-JEE 2012, 3+3M]
correct explanation for Statement-1
(a) 7 (b) 8 (c) 9 (d) 11
(b) Statement-1 is true, Statement-2 is true; Statement-2 is
not a correct explanation for Statement-1 116. Which of the following is correct?
(c) Statement-1 true, Statement-2 is false (a) a17 = a16 + a15 (b) c17 ≠ c16 + c15
(d) Statement-1 is false, Statement-2 is true (c) b17 ≠ b16 + c16 (d) a17 = c17 + b16

109. The number of 7-digit integers, with sum of the digits 117. Assuming the balls to be identical except for difference
equal to10 and formed by using the digits 1, 2 and 3 in colours, the number of ways in which one or more
only, is [IIT-JEE 2009, 3M] balls can be selected from 10 white, 9 green and 7 black
(a) 55 (b) 66 (c) 77 (d) 88 balls, is [AIEEE 2012, 4M]
(a) 630 (b) 879 (c) 880 (d) 629
110. From 6 different novels and 3 different dictionaries, 4
novels and 1 dictionary are to be selected and arranged 118. Let T n be the number of all possible triangles formed
in a row on a shelf so that the dictionary is always in the by joining vertices of an n-sided regular polygon. If
middle. Then, the number of such arrangements is T n + 1 − T n = 10, the value of n is [JEE Main 2013, 4M]
[AIEEE 2009, 4M] (a) 5 (b) 10 (c) 8 (d) 7
(a) atleast 1000 119. Consider the set of eight vectors
(b) less than 500
V = [a i$ + b $j + c k$: a, b, c ∈ {−11
, }] .Three non-coplanar
(c) atleast 500 but less than 750
(d) atleast 750 but less than 1000 vectors can be chosen from V in 2 p ways, then p is
[JEE Advanced 2013, 4M]
111. There are two urns. Urn A has 3 distinct red balls and
urn B has 9 distinct blue balls. From each urn, two balls 120. Let n 1 < n 2 < n 3 < n 4 < n 5 be positive integers such that
are taken out at random and then transferred to the n 1 + n 2 + n 3 + n 4 + n 5 = 20, the number of such distinct
other. The number of ways in which this can be done, is arrangements (n 1 , n 2 , n 3 , n 4 , n 5 ) is
[AIEEE 2010, 4M] [JEE Advanced 2014, 3M]
(a) 36 (b) 66 (c) 108 (d) 3
121. For n ≥ 2 be an integer. Take n distinct points on a circle
112. Statement-1 The number of ways distributing 10 and join each pair of points by a line segment. Colour
identical balls in 4 distinct boxes such that no box is the line segment joining every pair of adjacent points by
empty, is 9 C 3 . blue and the rest by red. If the number of red and blue
Statement-2 The number of ways of choosing any 3 line segments are equal, the value of n is
[JEE Advanced 2014, 3M]
places from 9 different places is 9 C 3 [AIEEE 2011, 4M]
(a) Statement-1 is true, Statement-2 is true; Statement-2 is a 122. Six cards and six envelopes are numbered 1, 2, 3, 4, 5, 6 and
correct explanation for Statement-1 cards are to be placed in envelopes, so that each
(b) Statement-1 is true, Statement-2 is true; Statement-2 is envelope contains exactly one card and no card is placed
not a correct explanation for Statement-1 in the envelope bearing the same number and moreover
(c) Statement-1 is true, Statement-2 is false the card numbered 1 is always placed in envelope
(d) Statement-1 is false, Statement-2 is true numbered 2. Then the number of ways it can be done, is
[JEE Advanced 2014, 3M]
113. There are 10 points in a plane, out of these 6 are (a) 264 (b) 265
collinear. If N is the number of triangles formed by (c) 53 (d) 67
joining these points, then [AIEEE 2011, 4M]
123. The number of integers greater than 6000 that can be
(a) N > 190 (b) N ≤ 100
formed using the digits 3,5,6,7 and 8 without repetition,
(c) 100 < N ≤ 140 (d) 140 < N ≤ 190
is [JEE Main 2015, 4M]
114. The total number of ways in which 5 balls of different (a) 120 (b) 72
colours can be distributed among 3 persons, so that each (c) 216 (d) 192
person gets atleast one ball is [IIT-JEE 2012, 3M]
124. Let n be the number of ways in which 5 boys and 5 girls
(a) 75 (b) 150 (c) 210 (d) 243
can stand in a queue in such away that all the girls stand
n
Directions (Q. Nos. 115 to 116) Let an denotes the consecutively in the queue. Let m be the number of ways
number of all n-digits positive integer formed by the in which 5 boys and 5 girls can stand in a queue in such
digits 0, 1 or both such that no consecutive digits in a way that exactly four girls stand consecutively in the
them are 0. Let bn be the number of such n-digit m
queue, the value of is
integers ending with digit 1 and cn be the number of n [JEE Advanced 2015, 3M]
such n digits integers ending with digit 0.
426 Textbook of Algebra

125. If all the words (with or without meaning having five (a) 380 (b) 320
letters, formed using the letters of the word SMALL and (c) 260 (d) 95
arranged as in a dictionary, then the position of the 127. A man X has 7 friends, 4 of them are ladies and 3 are
word SMALL is [JEE Main 2016, 4M] men. His wife Y also has 7 friends, 3 of them are ladies
(a) 59th (b) 52nd (c) 58th (d) 46th and 4 are men. Assume X and Y have no common
126. A debate club consists of 6 girls and 4 boys. A team of 4 friends. Then the total number of ways in which X and Y
members is to be selected from this club including the together can throw a party inviting 3 ladies and 3 men,
selection of a captain (from among these 4 members) for so that 3 friends of each of X and Y are in this party, is
the team. If the team has to include at most one boy, [JEE Main 2017, 4M]
then the number of ways of selecting the team is (a) 484 (b) 485
[JEE Advanced 2016, 3M] (c) 468 (d) 469

Answers
Exercise for Session 1 Chapter Exercises
1. (c) 2. (b) 3. (c) 4. (a) 5. (c) 6. (b) 1. (c) 2. (a) 3. (c) 4. (b) 5. (c) 6. (c)
7. (c) 8. (c) 9. (b) 10. (b) 11. (d) 7. (a) 8. (b) 9. (b) 10. (d) 11. (a) 12. (d)
13. (b) 14. (b) 15. (c) 16. (a) 17. (d) 18. (c)
Exercise for Session 2
19. (d) 20. (b) 21. (c) 22. (d) 23. (d) 24. (d)
1. (b) 2. (c) 3. (c) 4. (d) 5. (d) 6. (b) 25. (c) 26. (a) 27. (d) 28. (a) 29. (c) 30. (b)
7. (d) 8. (a) 9. (b) 10. (c) 11. (b) 12. (b) 31. (a, c) 32. (a,b,c,d) 33. (a,b,c,d) 34. (a,d) 35. (a, c)
13. (b) 14. (b) 15. (b) 16. (d)
36. (b,d) 37. (a, c) 38. (b,c) 39. (a,b,c,d) 40. (b,c,d)

Exercise for Session 3 41. (a) 42. (c) 43. (c) 44. (c) 45. (b) 46. (a)
47. (c) 48. (c) 49. (c) 50. (c) 51. (c) 52. (b)
1. (a) 2. (c) 3. (a) 4. (b) 5. (c) 6. (c)
7. (b) 8. (b) 9. (a) 10. (a) 11. (a) 53. (b) 54. (c) 55. (d) 56. (1) 57. (0) 58. (8)
59. (3) 60. (7) 61. (8) 62. (6) 63. (7) 64. (8)
Exercise for Session 4 65. (8) 66. (A) → (q); (B) → (r); (C) → (s); (D) → (p)
1. (d) 2. (b) 3. (d) 4. (c) 5. (a) 6. (c) 67. (A) → (r); (B) → (s); (C) → (q); (D) → (p)
7. (a) 8. (b) 9. (b) 10. (d) 11. (a) 12. (d) 68. (A) → (r); (B) → (s); (C) → (p); (D) → (q)
69. (A) → (s); (B) → (r); (C) → (s); (D) → (p)
13. (a) 14. (d) 15. (d) 16. (c) 17. (b) 18. (a)
70. (A) → (s); (B) → (q); (C) → (p); (D) → (s)
Exercise for Session 5 71. (b) 72. (c) 73. (d) 74. (c) 75. (d) 76. (b)
1. (c) 2. (c) 3. (d) 4. (a) 5. (b) 6. (b) 77. (b) 78. (d) 79. (a) 80. (d) 81. (d) 82. (a)
7. (d) 8. (a) 9. (c) 10. (c) 11. (d) 12. (a) 83. x = n + 3 84. x = 3 85. 4 87. 60 88. 42
89. 11520 90. 8 P4 × 8 P2 × 10 ! 92. 45C6
13. (b) 14. (c) 15. (d) 16. (c) 17. (b) 18. (c)
n(n − 4)(n − 5) (3n)! n2 (n + 1)
93. 94. 1728 95. 96.
Exercise for Session 6 6 6(n!) 3
2
1. (c) 2. (c) 3. (a) 4. (a) 5. (b) 6. (a) 97. 4530 99. 96 100. (d) 101. (c) 102. (d)
7. (c) 8. (c) 9. (c) 10. (a) 11. (d) 12. (b) 103. (c) 104. (c) 105. (c)
13. (a) 14. (c) 15. (b) 16. (b) 17. (c) 18. (d) 106. (A) → (p); (B) → (s); (C) → (q); (D) → (q)
19. (c) 20. (d) 21. (c) 22. (b) 23. (a) 24. (a)
107. (c) 108. (a) 109. (c) 110. (a) 111. (c) 112. (a)
25. (b) 26. (c) 27. (c) 28. (b)
113. (b) 114. (b) 115. (b) 116. (a) 117. (b) 118. (a)
Exercise for Session 7 119. (5) 120. (7) 121. (5) 122. (c) 123. (d) 124. (5)
125. (c) 126. (a) 127. (b)
1. (b) 2. (d) 3. (a) 4. (c) 5. (c)
b → 4 ! = 24 dc → 3 ! = 6

Solutions c → 4 ! = 24
da → 3 ! = 6

11. Number of different tickets


dea → 2 ! = 2
debac → 1
∴ The rank of debac = 24 + 24 + 24 + 6 + 6 + 6 + 2 + 1 = 93

= 20 + 19 + 18 + 17 + … + 3 + 2 + 1 = 210
1. 9C 5 − 9−2
C 5− 2 = 9C 4 − 7C 3 12. Let A = {a1, a 2, a 3, …, an }
= 126 − 35 = 91 (i) ai ∈ P , ai ∈ Q (ii) ai ∈ P , ai ∉ Q
(iii) ai ∉ P , ai ∈ Q (iv) ai ∉ P , ai ∉ Q, where 1 ≤ i ≤ n
2. Let a = 2x − 1, b = 2y − 1, c = 2z − 1, d = 2w − 1
Q P ∩Q =φ [cases in favour 3 i.e., (ii), (iii), (iv)]
where, x , y, z, w ∈ N ∴ Required number of ways = 3n
Then, a + b + c + d = 20 13. Total points on all three lines = m + n + k
⇒ x + y + z + w = 12 m+n+k
∴ Maximum number of triangles = C 3 − mC 3 − nC 3 − kC 3
12 − 1
∴ Number of ordered quadruplet = C4 − 1
14. Required number of triplets = Total number of triplets without
11 ⋅ 10 ⋅ 9 restrictions − Number of triplets with all different coordinates
= C3 =
11
= 165
1 ⋅2 ⋅3 = n 3 − n P3
3. l = LCM of 8!, 10! and 12! = 12!
15. Let middle largest digit be r, then digits available for remaining
and h = HCF of 8!, 10! and 12! = 8! four places are 0, 1, 2, 3, …, r − 1.
l 12 ! Number of ways filling remaining four places
∴ = = 12 ⋅ 11 ⋅ 10 ⋅ 9 = 11880 9 9
h 8! = ∑ (r P4 − r − 1P3 ) = ∑ (r − 1 ) × r − 1P3
4. n + 1Cn − 2 − n + 1Cn − 1 ≤ 100 r =4 r =4

n+1 = 3 × 3P3 + 4 × 4 P3 + 5 × 5P3


⇒ C 3 − n + 1C 2 ≤ 100
+ 6 × 6P3 + 7 × 7P3 + 8 × 8P3
(n + 1 )n(n − 1 ) (n + 1 )n
⇒ − ≤ 100 = 5292 = 2 ⋅ 3 ⋅ 7
2 3 2
6 2
16. Required number of words = 6 ! − 5 ! − 5 ! + 4 ! = 504
⇒ n(n + 1 )(n − 4 ) ≤ 600
It is true for n = 2, 3, 4, 5, 6, 7, 8, 9 17. Let x, y, z be the friends and a, b, c denote the case when x is
invited a times, y is invited b times and z is invited c times.
5. Coefficient of x11 in ( x + x 2 + x 3 + x 4 + x 5 + x 6 ) 3 Now, we have the following possibilities (a, b, c ) = (1, 2, 3 ) or (2,
2, 2) or (3, 3, 0) [grouping of 6 days of week]
= Coefficient of x 8 in
Hence, the total number of ways
(1 + x + x + x 3 + x 4 + x 5 ) 3
2
6! 6! 3! 6! 3!
= × 3! + × + ×
= Coefficient of x 8 in(1 − x 6 ) 3(1 − x ) −3 1! 2! 3! 2! 2! 2! 3! 3! 3! 0! 2!
= Coefficient of x 8 in (1 − 3 x 6 )(1 + 3C1x + … ) = 360 + 90 + 60 = 510
18. If y = n, then x takes values from 1 to n − 1 and z can take
= 10C 2 − 3 × 4C 2 = 45 − 18 = 27
values from 0 to n (i.e., (n + 1) values). Thus, for each value of
6. n C 2, n ∈ N , infinite numbers. y (2 ≤ y ≤ 9 ), x and z take (n − 1 )(n + 1 ) values.
7. Q 13 lines pass through A and 11 lines pass through B. Hence, the 3-digit numbers are of the form xyz
9 9
∴ Number of intersection points = ∑ (n − 1 )(n + 1 ) = ∑ (n 2 − 1 ) = 276
x =2 x =2
= 37C 2 − 13C 2 − 11C 2 + 2 [Q two points A andB ]
= 535 19. The letters in alphabetical order are EEMRTU
E → 5 ! = 120
8. Q a = 182bc
MEERTU → 1 ! = 1
⇒ ( x + 2 )! = 182 × x P11 × ( x − 11 )! MEERUT → 1
⇒ ( x + 2 )! = 182 × x ! ∴ Rank of MEERUT = 120 + 1 + 1 = 122
⇒ ( x + 2 )( x + 1 ) = 14 × 13 20. The candidates can be ranked in 10! ways. In half of these
ways, A 1 is above A2 and in another half, A2 is above A1. So,
⇒ x + 1 = 13
required number of ways = 10 !.
∴ x = 12 2
9. 6 + 5 × 5 + 5 × 5 × 4 = 131 21. Any selection of four digits from the ten digits 0, 1, 2, 3, …, 9
gives one number.
10. The letters in alphabetical order are abcde So, the required number of numbers = 10C 4 = 210
a → 4 ! = 24 db → 3 ! = 6
428 Textbook of Algebra

22. As 0 < x < 1, we have p < q ⇒ wi = 9 − li ⇒ wi 2 = 81 − 18 li + li 2


The number of rational numbers = 5 + 4 + 3 + 2 + 1 = 15 ⇒ Σ wi2 = Σ 81 − 18 Σ li + Σ li 2
When p, q have a common factor, we get some rational = 81 × 10 − 18 × 45 + Σli2 = Σli2
numbers which are not different from those already counted.
2 2 3 4 30. The number of matches in the first round = 6C 2 + 6C 2 = 30
There are 4 such numbers , , , .
4 6 6 6 The number of matches in the next round = 6C 2 = 15
∴ The required number of rational numbers = 15 − 4 = 11 and the number of matches in the semi-final round = 4C 2 = 6
23. We have, ∴ The required number of matches = 30 + 15 + 6 + 2 = 53
24 = 2 × 3 × 4 = 2 × 2 × 6
[Q for ‘best of three’ atleast two matches are played]
=1 ×6 × 4 =1 ×3 ×8
300  300  300  300  300 
= 1 × 2 × 12 = 1 × 1 × 24 31. E 3(300 !) = + + + +
 3   3 2   3 3   3 4   3 5 
The number of positive integral solutions of xyz = 24 is
= 100 + 33 + 11 + 3 + 1 = 148
3! 3!
3 ! + + 3 ! + 3 ! + 3 ! + = 30 ∴ m = 148
2! 2!
150  150  150  150 
and number of integral solutions having two negative and E 2(150 !) = + + +…+
 2   2 2   2 3   2 7 
factors is 3C 2 × 30 = 90.
= 75 + 37 + 18 + 9 + 4 + 2 + 1 = 146
Hence, number of integral solutions = 30 + 90 = 120
∴ n − 2 = 146
24. Total number of triangles = Number of triangles with vertices
⇒ n = 148
on sides [( AB, BC , CD ) + ( AB, BC , DA ) + ( AB, CD, DA )
32. Q E 2(102 !) = 98 , E 3(102 !) = 49,
+ ( BC , CD, DA )]
E 5(102 !) = 24 and E 7(102 !) = 16
= 3C1 × 4C1 × 5C1 + 3C1 × 4C1 × 6C1 + 3C1 × 5C1 × 6C1
∴ α = 98, β = 49, γ = 24 and δ = 16
+ 4C1 × 5C1 × 6C1 33. Triplets with
= 60 + 72 + 90 + 120 = 342 (i) x = y < z (ii) x < y < z
25. Q 16 horses = 10 horses + ( A, B, C , D, E , F ) (iii) y < x < z
∴ The number of ways = 10C 3 × (Number of ways of choosing can be chosen in n + 1C 2, n +1C 3,, n + 1C 3 ways.
out of A, B, C, D, E, F , so that AD, BE and CF are not together) n+1 n+1 n+1
∴ C2 + C3 + C 3 = n + 2C 3 + n+1
C3
= 10C 3 × (One from each of pairs AD, BE, CF )
= 2( n + 2C 3 ) − n + 1C 2
= 10C 3 × 2C1 × 2C1 × 2C1 = 960
n (n + 1 )(2n + 1 )
26. We have, i 3 + ai 2 + bi + c = 0 =
6
and ( −i ) 3 + a( −i ) 2 + b( −i ) + c = 0 34. When x is odd
⇒ (c − a ) + (b − 1 )i = 0 Unit’s place filled by 1, 3, 5, 7, 9.
and (c − a ) − i (b − 1 ) = 0 ∴ x = 8 × 8 × 7 × 5 = 2240
⇒ b = 1, a = c When x is even
Thus, total number of such polynomials = 10C1 = 10 Unit’s place filled by 0, 2, 4, 6, 8.
k ∴ y = 8 × 8 × 7 × 4 + 9 × 8 × 7 × 1 = 2296
k ∑ xi ⇒ x < y and | x − y | = 56
1 1 1 1 i =1 75
27. ∑ = + +…+ = =
i =1
xi x1 x 2 xk n n 35. ∑ min A
A ∈X
[as LCM of x1, x 2, x 3, …, xk is n] n −1 n −1 n −1
28. Let ‘1’ be associated with ‘r’, r ∈{1, 2, 3, 4, 5}, then ‘2’ can be ∑ r (n − r ) = n ∑ r − ∑r 2
associated with r, r + 1, … ,5. Let ‘2’ be associated with ‘j’ then r =1 r =1 r =1
3 can be associated with j, j + 1, …, 5. Thus, required number n(n − 1 )n (n − 1 )n(2n − 1 )
of functions = −
2 6
5  5  5
(6 − r ) (7 − r ) (n + 1 ) ⋅ n ⋅ (n − 1 ) n + 1 n(n 2 − 1 )
∑  ∑ (6 − j ) = ∑ 2
= 35 =
1 ⋅2 ⋅3
= C3 =
r =1  j =r  r =1 6
29. Clearly, each player will play 9 games. 36. Q p = 2520 = 2 3 ⋅ 3 2 ⋅ 51 ⋅ 71
∴ Total number of games = 10C 2 = 45 = 6 ⋅ 2 2 ⋅ 31 ⋅ 51 ⋅ 71 = 9 ⋅ 2 3 ⋅ 51 ⋅ 71
Clearly, wi + li = 9 and Σwi = Σ li = 45 ∴ x = (2 + 1 )(1 + 1 )(1 + 1 )(1 + 1 ) = 24
and y = (3 + 1 )(1 + 1 )(1 + 1 ) = 16.
Chap 05 Permutations and Combinations 429

37. Q x1 + x 2 + x 3 + … + xn = r , ∀xi ≥ 1 , (1 ≤ i ≤ n ) 43. × E × T T × O × I × S ×


Total number of such solutions = r − 1Cn − 1 = r − 1Cr − n Number of ways having TT together and RRR separated
= 6C 3 × 5 ! = 2400
38. Q x + x + 1 + y = 24
Hence, number of arrangements of the word W, if R’s as well
⇒ y = 23 − 2 x as T’s are separated = 12600 − 2400 = 10200
Let N = Total number of fights subject to the condition that 44. × U × CC × E ×
any two children of one team do not fight. Hence, required number of ways = 4C 3 × 3 ! = 24
∴ N = 24C 2 − ( xC 2 + x + 1C 2 + yC 2 )
45. × U × C × C × E ×
x+1 23 − 2x
= 24
C2 − ( x C2 + C2 + C2 ) There are five available places for three SSS.
4!
= 23 − 3 x 2 + 45 x ∴ Total number of ways no two S’s together = 5C 3 × = 120
2!
dN
∴ = 0 − 6 x + 45 Hence, number of words having CC separated and SSS
dx separated = 120 − 24 = 96
dN 7!
For maximum or minimum, put = 0 ⇒ x = 7.5 46. Total number of ways = = 420
dx 2 !3 !
⇒ x =7 [Q x ∈ I ] Consonants in SUCCESS are S, C, C, S, S
5!
d 2N Number of ways arranging consonants = = 10
Now, <0 2 !3 !
dx 2 Hence, number of words in which consonants appear in
∴ N will be maximum when x = 7 420
alphabetic order = = 42
and N = 23 − 3(7 ) 2 + 45 × 7 = 191 10
39. Q x + y + z = 3n 47. ×A × A × N × G × E ×
5!
Hence, required number of ways = 6C 2 × = 900
⇒ N = Coefficient of α 3n in (1 + α + α 2 + … + α 2n ) 3 2!
= Coefficient of α 3n in (1 − α 2n + 1 ) 3(1 − α ) −3 48. The number of ways in which two A’s are together
= Coefficient of α 3n in (1 − 3 α 2n + 1 ) (1 + 3C1 α + ... ) i.e., × A A × N × G × E × is 5C 2 × 4 ! = 240
3n + 2 Hence, number of ways in which neither two R’s no two A’s
= C 3n − 3 ⋅n + 1 Cn − 1
come together = 900 − 240 = 660
3n + 2
= C 2 − 3 ⋅n + 1 C 2 = 3n 2 + 3n + 1 49. The letters in alphabetical order are AAEGNRR
∴ N − 1 = 3n (n + 1 ) 5!
AA → = 60ARA → 4 ! = 24
40. Since, α can be subtracted from β without borrowing, if yi ≥ xi , 2!
for i = 1, 2, 3. 5!
AE → = 60 ARE → 4 ! = 24
Let xi = λ 2!
If i = 1, then λ = 1, 2, 3, …, 9 5!
and if i = 2 and 3, then λ = 0, 1, 2, 3, …, 9 AG → = 60ARG → 4 ! = 24
2!
Hence, total number of ways of choosing the pair α, β 5!
 9   9 
2 AN → = 60ARN → 4 ! = 24
2!
=  ∑ (10 − λ )  ∑ (10 − λ ) = ( 45 )(55 ) 2
    ARRAE → 2 ! = 2 ARRAG → 2 ! = 2
λ =1  λ = 0 
ARRANEG → 1 ARRANGE → 1
41. There are 9 letters
∴ Rank in dictionary
T, T, R, R, R, E, O, I, S
= 60 + 60 + 60 + 60 + 24 + 24 + 24 + 24 + 2 + 2 + 1 + 1 = 342
λ = Number of four lettered words (no restriction)
= Coefficient of x 4 in n
Sol. (Q. Nos. 50-52)
 x x2  x x2 x3 1 1 1
+ = ⇒ ( x − n ) (y − n ) = n 2
4! 1 + +  1 + + +  (1 + x ) 4 = 626 x y n
 1! 2!   1! 2! 3!  n2
∴ x = n + λ, y = n +
µ = Number of four lettered words (no vowels) λ
= Coefficient of x 4 in where, λ is divisor of n . 2

 x x2  x x2 x3 The number of integral solutions ( x , y ) is equal to the number


4! 1 + +  1 + + +  (1 + x ) = 38
 1! 2!   1! 2! 3!  of divisors of n 2.
If n = 3,n 2 = 9 = 3 2, then the equation has 3 solutions.
∴ Required ways (atleast one vowel) = 626 − 38 = 588
42. × T × E × O × I × S × T × ( x, y ) = ( 4, 12 ), (6, 6 ), (12, 4 )
There are 7 available places for RRR. 50. Q 10 2 = 2 2 ⋅ 5 2
6! ∴ S(10 ) = 3 × 3 = 9
∴ Required ways = 7 C 3 × = 12600
2!
430 Textbook of Algebra

51. Q 62 = 22 ⋅ 32 4!
A→ = 4 NANNU → 1
⇒ S(6 ) = 3 × 3 = 9 and 7 2
⇒ S(7 ) = 3 3!
∴ n = 4 + 1 =5
∴ S (6 ) + S (7 ) = 12
Hence, m − 4n = 23 − 20 = 3
Also, 82 = 26
60. Each of the n digits can be anyone of the three 2, 5 or 7.
⇒ S(8 ) = 7 and 9 2 = 3 4 ⇒ S(9 ) = 5
∴ The number of n-digit numbers is 3n .
∴ S (8 ) + S (9 ) = 12
⇒ 3n > 900
⇒ S (6 ) + S (7 ) = S (8 ) + S (9 ) = 12
⇒ n = 7, 8, 9 …
52. Q 1 2 → S(1) = 1, 2 2 → S(2) = 3, 3 2 → S(3) = 3,
Hence, smallest value of n is 7.
4 2 → 2 4 → S( 4 ) = 5, 5 2 → S(5 ) = 3, S(6 ) = 9 61. a + b + c = 21 ⇒3b = 21 ⇒b = 7 [Qa + c = 2b]
S(7 ) = 3, S(8 ) = 7, S(9 ) = 5 and S(10 ) = 9 [from above]
10
⇒ a + b + c = 21 ⇒ a + c = 14
∴ ∑ S(r ) = S(1) + S(2) + S(3) + S(4) + S(5) ⇒ λ = 14 − 1C 2 − 1 = 13C1 = 13
r= 1
+ S (6 ) + S (7 ) + S (8 ) + S (9 ) + S (10 ) Hence, λ − 5 = 13 − 5 = 8
= 1 + 3 + 3 + 5 + 3 + 9 + 3 + 7 + 5 + 9 = 48 62. 2λ = Number of selecting 3 member subsets of {1, 2, 3, …, 29}
53. Q f (6) = 6C 0 + 5C1 + 4C 2 + 3C 3 = 13 which are in
∴ f { f (6 )} = f (13 ) = C 0 + C1 + C 2 + C 3 + C 4 + C 5 + C 6
13 12 11 10 9 8 7 GP with common ratio (2 or 3 or 4 or 5).
29  29   29  29 
= 1 + 12 + 55 + 120 + 126 + 56 + 7 = 377 = 2 + 2 + 2 + 2
 2   3   4   5 
54. Q f (1) = C 0 = 1, f (2) = 2C 0 + 1C1 = 2, f (3) = 3C 0 + 2C1 = 3,
1
= 7 + 3 + 1 + 1 = 12
f (4) = 5 [given] ∴ λ =6
and f (5 ) = 5C 0 + 4C1 + 3C 2 = 8 63. The number of students answering exactly i (1 ≤ i ≤ n − 1)
Thus, we say that f (n ) > n for n = 4, 5, 6, … questions wrongly is 2n − i − 2n − i − 1. The number of students
Hence, number of solutions for f (n ) = n is 3. answering all n questions wrongly is 2 0.
55. Number of superstars = f ( 4) = 5 Hence, the total number of wrong answers
and number of junior artists = f (3 ) = 3 n −1

× S1 × S 2 × S 3 × S 4 × S 5 × [Si for superstars] ∑ i (2n − i − 2n − i − 1 + n(20 ) = 127


i =1
∴Required number of ways = 6C 3 × 5 ! × 3 ! = 14400
⇒2n − 1 + 2n − 2 + 2n − 3 + K + 21 + 2 0 = 127
56. For n ≥ 10, the number of zeros in n! ≥ 2
⇒ 2n − 1 = 127
∴ 1 ! + 2 ! + 3 ! + 4 ! + … + 97 ! = … 13
∴ Ten’s digit = 1 ⇒ 2n = 128 = 2 7

57. Q 100
C 50 =
100 ! ∴ n =7
(50 !) 2 64. The sum of digits is divisible by 3.
100  100  i.e., 8 + 7 + 6 + 4 + 2 + x + y or 27 + x + y is divisible by 3
∴ E 7(100 !) = + = 14 + 2 = 16
 7   7 2  ∴ x + y must be divisible by 3.
50  50  Then, possible ordered pairs are
and E 7(50 !) = + =7 + 1 =8
 7   7 2  (0, 3), (3, 0), (1, 5), (5, 1), (0, 9), (9, 0), (3, 9), (9, 3)
∴ E 7 in ( 100C 50 ) = 16 − 2 × 8 = 0 ∴ Number of ordered pairs = 8
58. n −1
C 3 − n − 2C 3 = 15
65. Number of circles through ACD, ACE, ADE intersect the line
AB = 3 and
⇒ n =8 [Q Pn = n − 2C 3 ] Number of circles through BCD, BCE, BDE intersect the line
59. For BULBUL, the letters in alphabetical order are BBLLUU AB = 3 and
4! Number of circles through CDE intersects the line AB = 2
BB → = 6 BULBLU → 1
2 !2 ! Hence, number of points of intersection = 3 + 3 + 2 = 8
4! 66. (A) n + 4 Cn + 1 − n + 3Cn = 15 (n + 2)
BL → = 12 BULBUL → 1
2! n+ 3 n+ 3
⇒ Cn + 1 + Cn − n + 3Cn = 15(n + 2 )
3!
BU → = 39 ⇒ n+ 3
Cn + 1 = 15(n + 2 )
2!
n+ 3
∴ m = 6 + 12 + 3 + 1 + 1 = 23 ⇒ C 2 = 15(n + 2 )
For NANNU (n + 3 )
The letters in alphabetical order are ANNNU ⇒ = 15 ⇒ n = 27
2
Chap 05 Permutations and Combinations 431

(B) 11 ⋅ n P4 = 20 . n − 2P4 71. Statement-1 is True


⇒11. n (n − 1 )(n − 2 )(n − 3 ) = 20(n − 2 )(n − 3 )(n − 4 )(n − 5 ) Q 6 ! = 720 = 8 × 9 × 10 i.e., Product of 6 − 3 = 3 consecutive
integers and Statement-2 is also true, but Statement-2 is not a
⇒ 11n(n − 1 ) = 20(n − 4 )(n − 5 ) [Qn ≠ 2, 3] correct explanation for Statement-1
⇒ 9n − 169n + 400 = 0
2
72. For the number exactly divisible by 4, then last two digits must
25 be divisible by 4, the last two digits are 12, 16, 24, 32, 36, 52, 56,
∴ n = 16 ⇒ n ≠ 64, 72, 76. Total 10 ways.
9
(C) 2n
C 3 = 11 ⋅ n C 3 Now, the remaining two first places on the left of 4 digit
numbers are to be filled from remaining 5 digits and this can
2n(2n − 1 )(2n − 2 ) 11 ⋅ n (n − 1 )(n − 2 )
⇒ = be done in 5P2 = 20 ways.
1 ⋅2 ⋅3 1 ⋅2 ⋅3
∴ Required number of ways = 20 × 10 = 200
∴ n =6
n+ 2
Hence, Statement-1 is true and Statement-2 is false.
C 8 57
(D) n + 2C 8 : n − 2P4 = 57 : 16 ⇒ n−2
= 73. Q10 ! = 2 8 ⋅ 3 4 ⋅ 5 2 ⋅ 71
P4 16
∴ Total number of divisors
(n + 2 ) n + 1 (n ) (n − 1 ) n − 2C 4 57 = (8 + 1 )( 4 + 1 )(2 + 1 )(1 + 1 ) = 270
⇒ ⋅ ⋅ ⋅ ⋅ n−2 =
8 7 6 5 P4 16 Hence, Statement-1 is false and Statement-2 is true.
⇒ (n + 2 )(n + 1 ) n(n − 1 ) = 21 ⋅ 20 ⋅ 19 ⋅ 18 74. Q Number of permutations of n dissimilar things taken n at a
∴ n = 19 time =n Pn = n!
n ∴ Statement-1 is true and Statement-2 is false.
P
67. (A) m = nCm [Q number of function = nn ]
m! 75. Q x1 x 2 x 3 x 4 = 2 ⋅5 ⋅ 7 ⋅11
(B) Required ways = m × m × m × K × m = mn Each of 2, 5, 7, 11 can assign in 4 ways.
144424443
n times
∴ Required number of solutions = 4 × 4 × 4 × 4 = 4 4 = 2 8 = 256
(C) × G × G × G × G × K × G ×
Hence, Statement-1 is false and Statement-2 is true.
Here, number of gaps for red balls = m − 1 + 1 = m
76. Qa + c = 2b
∴ Required ways = mCn
i.e., sum of two numbers is even, then both numbers are even
n ×n ×n ×K×n or odd. In {1, 2,3, 4,...,21 }, 11 numbers are odd and 10 numbers
(D) Required ways = = nm
m times are even.
68. (A) Required lines = 10C 2 − 4C 2 + 1 = 40 Then, total number of ways =11C 2 + 10C 2 = 55 + 45 = 100
(B) Maximum number of points = 10C 2 = 45 Hence, both statements are true but Statement-2 is not a
correct explanation for Statement-1.
(C) Maximum number of points = C 2 × 2 = 30 6

77. Q N = 12600 = 2 3 ⋅ 3 2 ⋅ 5 2 ⋅ 71
(D) Maximum number of points = 6C 2 × 4 = 60
∴ Number of even divisors = 3 ⋅ (2 + 1 ) ⋅ (2 + 1 ) ⋅ (1 + 1 ) = 54
69. (A) Number of rectangles = 7C 2 × 7C 2 = 441 Both statements are true but Statement-2 is not a correct
[select two vertical and two horizontal lines] explanation for Statement-1.
(B) Number of squares = 1 2 + 2 2 + 3 2 + 4 2 + 5 2 + 6 2 = 91 78. We know that a number is divisible by 3, if the sum of its digits
is divisible by 3. Now, out of 0, 1, 2, 3, 4, 5, 6 if we take 1,2,4,5,6
(C) First square can be selected in 36 ways, second square or 1,2,3,4,5 or 0,3,4,5,6 or 0,2,3,4,6 or 0,1,3,5,6 or 0,1,2,4,5 or
can be selected in (36 − 6 − 5) = 25 ways and third 0,1,2,3,6
square can be selected in (25 − 5 − 4 ) = 16 ways. ∴ Total number of ways = 2 × 5P5 + 5 × ( 5P5 − 4 P4 )
∴ Required ways = 36 × 25 × 15 = 2400
= 240 + 480
(D) a1 + a 2 + a 3 + a 4 + a 5 + a 6 = 11
= 720
where, a1, a 2, a 3, a 4 , a 5, a 6, ≥ 1
Statement-1 is false, Statement-2 is true.
∴ Required ways = 11 − 1C 6 − 1 = 10C 5
79. The sum of the digits in the ten’s place
70. (A) 3 1 1 or 2 1 1 Required ways = The sum of the digits in the unit’s place
3! 3!
= 1 × 1 ×1 × + 1 ×1 ×1 × =6 = ( 4 − 1 )!(3 + 4 + 5 + 6 ) = 108
2! 2!
Both statements are true and Statement-2 is a correct
(B) 3 1 1 or 2 1 1 Required ways explanation for Statement-1.
5
C × 2C1 × 1C1 5
C 2 × 3C 2 × 1C1
= 3 + = 25 80. Number of planes each containing three points
2 2
= pC 3 −q C 3 + 1
(C) 3 1 1 or 2 1 1 Required ways = 1 × 1 × 1 + 1 × 1 × 1 = 2
∴ Statement-1 is false and Statement-2 is always true.
(D) 3 1 1 or 2 1 1 Required ways = 3 + 3 = 6
432 Textbook of Algebra

50 ! n! n!
81. Q 50
C10 = ⇔ >
10 ! 40 ! 4 !(n − 4 )! 3 !(n − 3 )!
50  50  50  50  1 1
∴ E 3(50 !) = + + + +... ⇔ > [Qm ! = m(m − 1 )!]
 3   9  27  81  4(n − 4 )! (n − 3 )(n − 4 )!
= 16 + 5 + 1 + 0 + ... = 22 ⇔ n −3 > 4 ⇔ n > 7
 40   40   40   40  87. Now, let sides of game be A and B. Given 5 married couples,
E 3( 40 !) = + + + +...
 3   9   27   81  i.e., 5 husbands and 5 wives. Now 2 husbands for two sides A
and B be selected out of 5 = 5C 2 = 10 ways.
= 13 + 4 + 1 + 0 = 18
After choosing the two husbands their wives are to be
10  10  10 
and E 3(10 !) = + + + ... = 3 + 1 + 0 = 4 excluded (since no husband and wife play in the same game).
 3   9  27  So, we are to choose 2 wives out of remaining 5 − 2 = 3 wives
Hence, highest power of 3 in 50C10 = 22 − (18 + 4 ) = 0 i.e., 3C 2 = 3 ways. Again two wives can interchange their sides
A and B in 2 ! = 2 ways.
∴ Statement-1 is false, Statement-2 is true.
By the principle of multiplication.
82. Number of diagonals in quindecagon =15C 2 − 15 = 105 − 15 = 90 The required number of ways = 10 × 3 × 2 = 60.
Both statements are true and Statement-2 is a correct 88. Case I When no couple is chosen
explanation for Statement-1.
We can choose 2 men in 4C 2 = 6 ways and hence two teams
83. We have, nCn −r +3nCn −r + 1 +3nCn −r + 2 + nCn −r + 3 =n Cr can be formed in 2 × 6 = 12 ways.
⇔ (n Cn −r + nCn −r + 1 ) + 2(n Cn −r + 1 + nCn −r + 2 ) + (n Cn −r + 2 + nCn −r + 3 ) = x Cr Case II When only one couple is chosen
⇔ n +1
Cn −r + 1 + 2.n + 1Cn −r + 2 + n + 1Cn −r + 3 = xCr A couple can be chosen in 4C1 = 4 ways and the other team can
be chosen in 3C1 × 2C1 = 6 ways. Hence, two teams can be
⇔ (n + 1Cn −r + 1 + n + 1Cn −r + 2 ) + (n + 1Cn −r + 2 + n + 1Cn −r + 3 ) = xCr
formed in 4 × 6 = 24 ways.
n+ 2
⇔ Cn −r + 2 + n + 2Cn −r + 3 = x Cr Case III When two couples are chosen
n+ 3
⇔ Cn −r + 3 = x Cr Then team can be chosen in 4C 2 = 6 ways.
n+ 3
⇔ Cr = x Cr [Q nCr =n Cn −r ] Hence, total ways = 12 + 24 + 6 = 42.
Hence, x =n+3 89. The total number of seats
84. We have, x +1
3 C 2 + P2 ⋅ x = 4 x A2 = 1 grandfather + 5 sons and daughters +8 grand children = 14
3( x + 1 ) x The grand children with to occupy the 4 seats on either side of
⇔ + 2 ! x = 4 ⋅ x( x − 1 ) the table 4! ways = 24 ways
1 ⋅2
and grandfather can occupy a seat in (5 − 1 ) ways = 4 ways
⇔ 3x 2 + 3x + 4x = 8x 2 − 8x [Since 4 gaps between 5 sons and daughters]
⇔ 5 x 2 − 15 x = 0 and the remaining seat can be occupied in 5! ways
⇔ 5 x( x − 3 ) = 0 = 120 ways [5 seats for sons and daughters]
∴ x≠0 [Q x ∈ N ] Hence, required number of ways, By the principle of
Hence, x = 3 is the solution of the given equation. multiplication law = 24 × 4 × 120 = 11520
195 n + 3A3 90. There are 8 chairs on each side of the table. Let sides be
85. We have, xn = − represented by A and B. Let four persons sit on side A, then
4 Pn Pn + 1 number of ways arranging 4 persons on 8 chairs on side
195 (n + 3 )(n + 2 )(n + 1 ) A = 8P4 and then two persons sit on side B, then number of
∴ xn = −
4 ⋅n! (n + 1 )! ways arranging 2 persons on 8 chairs on side B = 8P2 and
195 (n + 3 )(n + 2 ) arranging the remaining 10 persons in remaining 10 chairs in
=− − 10! ways.
4 ⋅n! n!
Hence, the total number of ways in which the persons can be
195 − 4n 2 − 20n − 24 171 − 4n 2 − 20n 8 !⋅ 8 ! 10 !
= = arranged = 8P4 × 8P2 × 10 ! =
4 ⋅ n! 4 ⋅ n! 4! 6!
Q xn is positive. 91. The total number of handshake participations by all men what
171 − 4n 2 − 20n so ever is an even number, which is twice the number of
∴ >0 handshakes.
4 ⋅ n!
⇒ 4n 2 + 20n − 171 < 0 The sum of all participations by men having an even number
of handshakes is an even number, which is the sum of several
which is true for n = 1, 2, 3, 4
even numbers.
Hence, the given sequence ( xn ) has 4 positive terms.
The sum of all participations by men having an odd number of
n −1
86. We have, C 3 + n −1C 4 >n C 3 [Q nCr + nCr −1 =n + 1Cr ] handshakes is an even number, which is an even number
⇔ n
C 4 >n C 3 minus an even number.
Chap 05 Permutations and Combinations 433

The number of men having an odd number of handshakes 95. The required number of ways = The number of ways in which
must be even for the sum of the odd numbers of their 3n different things can be divided in 3 equal groups = The
participations be even. number of ways to distribute 3n different things equally among
92. 3n ! 3n !
C L three persons = 3
=
S1 S2 S3 S4 S5 S6 S7 S8 S9 3 !(n !) 6(n !) 3
For S1, 9 different tickets are available, one for each of the 96. Number of squares of area n 2 square units = 1 2
remaining 9 stations, similarly at S 2, 8 different tickets are Number of squares of area (n − 1 ) 2 square units = 2 2
available and so on.
Thus, total number of different tickets Number of squares of area (n − 2 ) 2 square units = 3 2
= 9 + 8 + 7 + 6 + 5 + 4 + 3 + 2 + 1 = 45 ............................................................................
So, the six different tickets must be any six of these 45 and Number of squares of area 1 2 square units = n 2
there are evidently as many different sets of 6 tickets as there n(n + 1 )(2n + 1 )
Adding gives N1 = 1 2 + 2 2 + 3 2 + ...+ n 2 =
are combinations of 45 things taken 6 at a time. 6
Hence, the required number = 45C 6. When n is even
93. Let the object be denoted by a1, a 2, a 3, ..., an arranged in a circle, n2
Number of squares of area square units = 1 2
we have to select 3 objects so that no two of them are 2
consecutive. For this, we first find the number of ways in (n − 2 ) 2
which 2 or 3 objects are consecutive. Now, number of ways in Number of squares of area square units = 3 2
which 2 or 3 objects are consecutive, is obtained as follows 2
with a1. The number of such triples is .............................................................................
a1a 2a 3, a1a 2a 4 , a1a 2a 5, ..., a1a 2an − 1. 22
Number of squares of area square units = (n − 1 ) 2
[Since, we have excluded a1a 2an , so it will be repeated again. If 2
we start with an , then we shall get triples : ana1a 2, ana1a 3] n(n − 1 )(n + 1 )
So, number of such triples when we start with a1, is (n − 3 ). Adding gives N 2 = 1 2 + 3 2 + 5 2 + ...+ (n − 1 ) 2 =
6
Similarly, with a 2, a 3, a 4 , K, we shall get the numbers of triples
When n is odd
that is (n − 3 ).
(n − 1 ) 2
But total number of triples is n C 3. Number of squares of area square units = 2 2
2
Hence, required number of ways = nC 3 − n (n − 3 ) (n − 3 ) 2
Number of squares of area square units = 4 2
n (n − 1 ) (n − 2 ) n 2
= − n (n − 3 ) = [n 2 − 3n + 2 − 6n + 18 ]
1 ⋅2 ⋅3 6 (n − 5 ) 2
Number of squares of area square units = 6 2
n 2 n 2
= (n − 9n + 20 ) = (n − 4 ) (n − 5 )
6 6 .............................................................................
94. Let the men P ,Q , R,S ,T ,U ,V ,W and suppose P ,Q, R remain only 22
Number of squares of area square units = (n − 1 ) 2
on one side and S ,T on the other as represented in figure. 2
n(n − 1 )(n + 1 )
P Q R Adding gives N 2 = 2 + 4 + 6 2 + ...+ (n − 1 ) 2 =
2 2
6
S T ∴Total number of squares formed which can be obtained by
taking 4 points out of (n + 1 ) 2 points = N1 + N 2
Then, since 4 men must row on each side, of the remaining 3,
n(n + 1 )(2n + 1 ) n(n − 1 )(n + 1 ) n 2(n + 1 )
one must be placed on the side of P ,Q, R and the other two on = + =
the side of S , T and this can evidently be done in 3 ways, for we 6 6 2
can place any one of the three on the side of P ,Q, R. 97. (i) Set of 2 numbers
Now, 3 ways of distributing the crew let us first consider one, a +b
Let a and b be 2 numbers = 60 ⇒ a + b = 120
say that in which U is on the side of P , Q, R as shown in figure. 2
Q R U
a and b both cannot be equal to or greater than 60
P
[Q 60 cannot be used twice]
S T V W Let 0 ≤ a ≤ 59 and 61 ≤ b ≤ 120
Now, P , Q, R, U can be arranged in 4! ways and S ,T ,V ,W can be The total number of ways in which a can be chosen
arranged in 4! ways. Hence, total number of ways arranging = 60C1 = 60
the men = 4 ! × 4 ! = 576
Hence, the number of ways of arranging the crew The value of b depends on the value of a and there is 1
= 3 × 576 value of b corresponding to 1 of a.
= 1728 ∴ Total number of sets having 2 numbers = 60
434 Textbook of Algebra

(ii) Set of 3 numbers (n − 1 ) straight lines in (n − 1 ) different points. So, the


Let a, b, c be the three numbers aggregate number of points contained in the n straight lines is
a +b +c n (n − 1 ). But in making up this aggregate, each point has
Then, = 60 ⇒a + b + c = 180 evidently been counted twice. For instance, the point P has
3
been counted once among the points situated on AB and again
Case I Let 0 ≤ a ≤ 59, 0 ≤ b ≤ 59 and c ≥ 60 among those
a can be chosen in 60C1 = 60 ways on CD.
b can be chosen in 59C1 = 59 ways n (n − 1 )
Hence, the actual number of points =
[Qb cannot use the value of a] 2
∴ Number of ways in which a and b can be chosen Now, we have to find the number of new lines formed by
= 60 × 59 = 3540 joining these points. The number of new lines passing through
P is evidently equal to the number of points lying outside the
Now, 1 ≤ a + b ≤ 117 and there is only one value of c for 1
value of a + b so that a + b + c = 180. lines AB and CD for getting a new line joining P with each of
these points only.
∴ Number of ways in which a,b, c can be chosen
Since, each of the lines AB and CD contained (n − 2 ) points
= 60 × 59 = 3540
besides the point P, the number of points situated on AB
Case II a = 60 and CD
∴ b + c = 120
= 2 (n − 2 ) − 1
The number of ways in which b and c can assume values
= 60 [from Eq. (i)] = (2n − 3 )
∴ Number of ways in which a, b, c can be chosen = 60 ∴ The number of points outside AB and CD
Case III 61 ≤ a ≤ 90, 61 ≤ b ≤ 90 and c ≤ 60
n (n − 1 )
a can assume values in 30C1 = 30 ways = − (2n − 3 )
2
b can assume values in 29C1 = 29 ways The number of new lines passing through P and similarly
The value of c depends on the value of a and b through each other points.
∴ Number of ways in which a,b, c can be chosen ∴The aggregate number of new lines passing through the
= 30 × 29 = 870 points
n (n − 1 ) n (n − 1 ) 
∴Total number of ways in which sets of 3 numbers can be =  − (2n − 3 ) 
2  2 
chosen
= 3540 + 60 + 870 = 4470 But in making up this aggregate, every new line is counted
twice. For instance, if Q is one of the points outside AB and CD,
∴ Total number of ways in which sets of 2 and 3 numbers
the line PQ is counted once among the lines passing through P
can be chosen
and again among these passing through Q.
= 4470 + 60 = 4530
Hence, actual number of fresh lines introduced
98. Let AB be any one of n straight lines and suppose it is
1 n (n − 1 ) n (n − 1 ) 
intersected by some other straight line CD at P. =   − (2n − 3 ) 
P 2 2  2 
A B
1
= n (n − 1 ) (n − 2 ) (n − 3 )
Then, it is clear that AB contains (n − 1 ) of the points of 8
intersection because it is intersected by the remaining (n − 1 )

99. Denoting A1, B1, A 2 and B 2 for their taking out the ball, a chart is made to denote the winner.

S. A1 B1 A2 B2 Number of ways
No.

1. Points Number on 1 Even (1 of 3) Even 1 Even 0 Odd (1 of 3) Odd 2 Odd (1 of 2) Even 3


C1 × 2C1 × 3C1 × 2C1 = 36
the ball Sum (1 of 2) Even
2. Points Number on −1 Odd (1 of 3)Odd 1 Odd 0 Even (1 of 3)Even 2 Even (1 of 2) Even 3
C1 × 2C1 × 3C1 × 2C1 = 36
the ball Sum (1 of 2) Even
3. Points Number on 1 Even (1 of 3) Even 2 Odd 0 Odd (1 of 3) Even 3
C1 × 3C1 × 2C1 = 18
the ball Sum (1 of 3) Odd
4. Points Number on 1 Even (1 of 3) Even 1 Even 2 Even (1 of 1)Even 3
C1 × 2C1 × 1C1 = 6
the ball Sum (1 of 2) Even

∴ Total number of ways in which the game can be won when A starts the game = 36 + 36 + 18 + 6 = 96
Chap 05 Permutations and Combinations 435

7
100. Along horizontal side one unit can be taken in (2m − 1) ways 1 − x3
and 3 unit side can be taken in (2m − 3 ) ways. The number of ⇒ Coefficient of x 3 in   = (1 − x 3 ) 7(1 − x ) −7
 1−x 
ways of selecting a side horizontally is
m ⇒ Coefficient of x 3 in (1 − 7x 3 )(1 + 7C1x + 8C 2x 2 + 9C 3x 3 +... )
(2m − 1 + 2m − 3 + 2m − 5 + ...+ 3 + 1 ) = (2m − 1 + 1 ) = m 2
2 9 ⋅8 ⋅7
= 9C 3 − 7 = − 7 = 77
2m–1
1 ⋅2 ⋅3
Aliter
The digits are 1, 1, 1, 1, 1, 2, 3, or 1, 1, 1, 1, 2, 2, 2
2 n-1
7! 7!
Hence, number of seven digit numbers formed = +
5 ! 4 !3 !
= 42 + 35 = 77
Similarly, the number of ways along vertical side is 110. 4 novels can be selected from 6 novels in 6C 4 ways. 1
n
(2n − 1 + 2n − 3 + ...+ 5 + 3 + 1 ) = (2n − 1 + 1 ) = n 2 dictionary can be selected from 3 dictionaries in 3C1 ways.
2
As the dictionary selected is fixed in the middle, the remaining
∴ Total number of rectangles = m 2n 2 4 novels can be arranged in 4! ways.
101. Words starting with A, C, H, I, N are each equals to 5! ∴ The required number of ways of arrangement.
∴ Total words = 5 × 5 ! = 600 = 6C 4 × 3C1 × 4 ! = 1080
The first word starting with S is SACHIN. 9 ×8
111. Total number of ways = 3C 2 × 9C 2 = 3C1 × 9C 2 = 3 ×
∴ SACHIN appears in dictionary at serial number 601. 1 ×2
102. Required number of ordered pair ( p,q ) is = 3 × 9 × 4 = 108
(2 × 3 − 1 )(2 × 5 − 1 )(2 × 3 − 1 ) − 1 = 224 112. The number of ways of distributing 10 identical balls in 4
different boxes such that no box is empty =10−1C 4 −1 = 9C 3
103. 10C1 + 10C 2 + 10C 3 + 10C 4 = 10 + 45 + 120 + 210 = 385 Statement-1 is true.
104. In a word COCHIN, the second place can be filled in 4C1 ways The number of ways of choosing any 3 places from 9 different
and the remaining four alphabets can be arranged in 4! ways in places = 9C 3
four different places. The next 97th word will be COCHIN. Statement-2 is true.
Hence, the number of words that appear before the word Both statements are true but statement-2 is not a correct
COCHIN is 96. explanation for statement-1.
105. 12 different objects are to be divided into 3 groups of equal Aliter Let a, b, c, d are the balls in four boxes, then
size, which are named as A, B and C. a + b + c + d = 10 and a ≥ 1, b ≥ 1, c ≥ 1, d ≥ 1[Q no box is empty]
12 ! ∴Number of solutions = 10−1C 4 −1 = 9C 3
Number of ways =12C 4 × 8C 4 × 4 C 4 =
( 4 !) 3 113. Number of triangles = 10C 3 − 6C 3
106. (A) → (p);(B) → (s);(C) → (q);(D) → (q) 10 ⋅ 9 ⋅ 8 6 ⋅ 5 ⋅ 4
⇒ N = − ⇒ N = 120 − 20 ⇒ N = 100
(A) ENDEA, N, O, E, L are five different letters, then 1 ⋅2 ⋅3 1 ⋅2 ⋅3
permutations = 5!. ∴ N ≤ 100
(B) If E is in the first and last position, then permutations 114. Q Each person gets atleast one ball.
7! 7 × 6 × 5!
= = = 21 × 5 ! ∴ 3 persons can have 5 balls in the following systems
2! 2
4! Person I II III Person I II III
(C) For first four letters = = 4 × 3 = 12 and for last five 1 1 3
or
1 2 2
2! No. of balls No. of balls
5! 5! 5!
letters = = , then permutations = 12 × = 2 × 5 !
3! 6 6 The number of ways to distribute the balls in first system
5!
(D) For A, E and O = and for others = = 12, then
4! = 5C1 × 4 C1 × 3C 3
3! 2! ∴ The total number of ways to distribute 1,1,3 balls to the
5! 5!
permutations = × 12 = × 12 = 2 × 5 !. 3!
persons = 5C1 × 4 C1 × 3C 3 × = 60
3! 6 2!
107. Other than S seven letters M,I,I,I,P,P,Ican be arranged in and the number of ways to distribute the balls in second
7! 7 ⋅6 ⋅5 system = 5C1 × 4C 2 × 2C 2
= = 7⋅6C 2 = 7⋅6C 4
2!4! 1 ⋅2
Hence, the total number of ways to distribute 1,2,2 balls to the
Now, four S can be placed in 8 spaces in 8C 4 ways. 3!
persons = 5C1 × 4 C 2 × 2C 2 × = 90
Hence, required number of ways = 7 ⋅ 6C 4 ⋅8C 4 2!
108. x1 + x 2 + x 3 + x 4 + x 5 = 6 ⇒ 6+ 5−1
C 5−1 = 10C 4 ∴ The required number of ways = 60 + 90 = 150
Aliter The required number of ways
109. Coefficient of x10 in ( x + x 2 + x 3 ) 7
= 3 5 − 3C1(3 − 1 ) 5 + 3C 2(3 − 2 ) 5 − 3C 3(3 − 3 ) 5
⇒ Coefficient of x 3 in (1 + x + x 2 ) 7
= 243 − 96 + 3 − 0 = 150
436 Textbook of Algebra

115. Qan = number of all n-digit positive integers formed by the 121. Number of adjacent lines = n
digits 0,1 or both such that no consecutive digits in them n(n − 3 )
are zero. Number of non-adjacent lines =n C 2 − n =
2
and bn = number of such n-digit integers ending with 1 n(n − 3 ) n(n − 5 )
∴ =n ⇒ = 0 ⇒ n = 0 or 5
cn = number of such n-digit integers ending with 0. 2 2
Clearly an = bn + cn [Qan can end with 0 or 1] But n ≥ 2 ⇒n = 5
Also, bn = an −1 and cn = an −2 122. Q Card numbered 1 is always placed in envelope numbered 2,
[Qif last digit is 0, second last has to be 1] we can consider two cases.
∴ We get an = an −1 + an − 2, n ≥ 3 Case I Card numbered 2 is placed in envelope numbered 1,
then it is derangement of 4 objects, which can be done in
Also, a1 = 1, a 2 = 2
 1 1 1 1
By the recurring formula a 3 = a 2 + a1 = 3 4 ! 1 − + − +  = 9 ways
 1 ! 2 ! 3 ! 4 !
a4 = a3 + a2 = 3 + 2 = 5
Case II Card numbered 2 is not placed in envelope numbered
a5 = a4 + a3 = 5 + 3 = 8
1, then it is derangement of 5 objects, which can be done in
Also, b6 = a 5 = 8  1 1 1 1 1
116. By recurring formula, a17 = a16 + a15 is correct. 5 ! 1 − + − + −  = 44 ways
 1 ! 2 ! 3 ! 4 ! 5 !
Also, C17 ≠ C16 + C15 ⇒ a15 ≠ a14 + a13 [QCn = an −2] ∴ Total ways = 9 + 44 = 53 ways
∴Incorrect. Similarly, other parts are also incorrect.
123. Four digit numbers can be arranged in 3 × 4 ! = 72 ways and five
117. Required number of ways digit numbers can be arranged in 5 ! = 120 ways
= (10 + 1 )(9 + 1 )(7 + 1 ) − 1 = 880 − 1 = 879
∴ Number of integers = 72 + 120 = 192
118. Q Tn + 1 − Tn = 10 ⇒ n + 1C 3 −n C 3 = 10 ⇒ nC 2 + nC 3 −n C 3 = 10 124. n = 5 ! × 6 !
20 5 ⋅ 4 5
⇒ n
C 2 = 10 = = = C2 ⇒ n = 5 For m: 5 boys can stand in a row in 5!, creating 6 alternate
2 1 ⋅2
space for girls. A group of 4 girls can be selected in 5C 4 ways.
119. Given 8 vectors are (1,1,1),( −1,1,1),(1,−1,1),(1,1,−1), ( −1,−1,1), A group of 4 and single girl can be arranged at 2 places out of
(1,−1,−1 ), ( −1,1,−1 ),( −1, − 1,−1 ) there are 4 diagonals of a cube. 6 in 6P2 ways. Also, 4 girls can arrange themselves in 4! ways.
Now, for 3 non-coplanar vectors first we select 3 groups of
diagonals and its opposite in 4C 3 = 4 ways. Then one vector ∴ m = 5 ! × 6P2 × 5C 4 × 4 ! = 5 ! × 30 × 5 × 4 ! = 5 ! × 6 ! × 5
from each group can be selected in 2 × 2 × 2 = 8 ways. m 5!× 6!× 5
⇒ = =5
∴ Total ways = 4 × 8 = 32 = 2 5 = 2 p (given) n 5!× 6!
Hence, p =5 4! 4!
125. Words starting with A, L, M = + 4 !+ = 48
120. If n1,n2,n3,n4 take minimum values 1,2,3, 4 respectively, then n5 2! 2!
will be maximum 10. 3!
Words starting with SA, SL = + 3 ! = 9
∴ Corresponding to n5 = 10, there is only one solution 2!
n1 = 1, n2 = 2, n3 = 3, n4 = 4 Rank of the word SMALL = 48 + 9 + 1 = 58
Corresponding to n5 = 9, we can have, 126. Either one boy will be selected or no boy will be selected. Also
out of four members one captain is to be selected.
n1 = 1, n2 = 2, n3 = 3, n4 = 5 i.e., one solution
∴ Required number of ways = ( 4C1 × 6C 3 + 6C 4 ) × 4C1
Corresponding to n5 = 8, we can have,
= ( 4 × 20 + 15 ) × 4 = 95 × 4 = 380
n1 = 1, n2 = 2, n3 = 3, n4 = 6
127. X Y
or n1 = 1, n2 = 2, n3 = 4, n4 = 5 i.e., two solutions
Corresponding to n5 = 7, we can have
4L 3M 3L 4M
n1 = 1, n2 = 2, n3 = 4, n4 = 6
3 0 0 3 = 4C 3 × 3C 0 × 3C 0 × 4C 3 = 16
or n1 = 1, n2 = 3, n3 = 4, n4 = 5 i.e., two solutions
2 1 1 2 = 4C 2 × 3C1 × 3C1 × 4C 2 = 324
Corresponding to n5 = 6, we can have
n1 = 2, n2 = 3, n3 = 4, n4 = 5 i.e., one solution 1 2 2 1 = 4C1 × 3C 2 × 3C 2 × 4C1 = 144
Thus, there can be 7 solutions. 0 3 3 0 = 4C 0 × 3C 3 × 3C 3 × 4C 0 = 1
485
CHAPTER

06
Binomial Theorem
Learning Part
Session 1
● Binomial Theorem for Positive Integral Index

● Pascal’s Triangle

Session 2
● General Term

● Middle Terms

● Greatest Term

● Trinomial Expansion

Session 3
● Two Important Theorems

● Divisibility Problems

Session 4
● Use of Complex Numbers in Binomial Theorem

● Multinomial Theorem

● Use of Differentiation

● Use of Integration

● When Each Term is Summation Contains the Product of Two Binomial Coefficients or Square of Binomial

Coefficients
● Binomial Inside Binomial

● Sum of the Series

Practice Part
● JEE Type Examples
● Chapter Exercises

Arihant on Your Mobile !


Exercises with the #L
symbol can be practised on your mobile. See inside cover page to activate for free.
Session 1
Binomial Theorem for Positive Integral Index,
Pascal’s Triangle
n
An algebraic expression consisting of two dissimilar terms Cr x n - r ar
2. Let S = ( x + a) n = å
n
with positive or negative sign between them is called a r =0
binomial expressions. Replacing r by n - r , we have
n n
a p q
For example, x + a, x 2 a - , - ,5 - x, S = ( x + a) n = å
n
Cn - r x n - ( n - r ) an - r = å
n
Cn - r x r an - r
2
x x x4 r =0 r =0

1 = n Cn an + n Cn - 1 an - 1 x + n Cn - 2 an - 2 x + ... + n C0 x n
2
( x 2 + 1) 1 / 3 - , etc., are called binomial
( x 3 + 1) Thus, replacing r by n - r , we are infact writing the binomial
expansion in reverse order.
expressions.

Remarks Some Important Points


1. An algebraic expression consisting of three dissimilar terms 1. Replacing a by (- a) in Eq. (i), we get
3
is called a trinomial. e.g. a + 2b + c, x - 2 y + 3z,2a - + g ,
b (x - a )n = nC 0 x n - 0 a 0 - nC1 x n - 1 a 1
etc. are called the trinomials.
+ n C 2 x n - 2 a 2 - ... +...+ ( - 1) r n C r x n - r a r
2. In general, expressions consisting more than two dissimilar
terms are known as multinomial expressions. + ... + ( - 1) n n C n x 0 a n …(ii)
n
or ( x - a ) n = å ( - 1) r n
Cr x n - r a r
Binomial Theorem for r =0

Positive Integral Index 2. On adding Eqs. (i) and (ii), we get


(x + a )n + (x - a )n = 2 { n C 0 x n - 0 a 0
If x , a Î C and n Î N , then
+ n C 2 x n - 2 a 2 + n C 4 x n - 4 a 4 + ... }
( x + a ) n = n C 0 x n - 0 a 0 + n C 1 x n -1 a 1 + n C 2 x n - 2 a 2 + ...
= 2 {Sum of terms at odd places}
+ nCr x n - r a r
The last term is n C n a n or n C n - 1 x a n - 1 ,
+ ... + n C n - 1 x 1 a n - 1 + n C n x 0 a n …(i)
according as n is even or odd, respectively.
n
or ( x + a) = n
å n
Cr x n -r
a r 3. On subtracting Eq. (ii) from Eq. (i), we get
r =0 (x + a )n - (x - a )n = 2 { n C1 x n - 1 a 1
n n n n
Hence, C 0 , C 1 , C 2 ,..., C n are called binomial coefficients. + n C 3 x n - 3 a 3 + n C 5 x n - 5 a 5 + ... }

Remark = 2 {Sum of terms at even places}


1. In each term, the degree is n and the coefficient of x n - r ar is The last term is n C n - 1 x a n - 1 or n C n a n ,
equal to the number of ways x, x, x, ... , x , a, a, a, ... , a according as n is even or odd, respectively.
14243 14243
can be arranged, which ( n - r ) times r times
n! 4. Replacing x by 1 and a by x in Eq. (i), we get
is given by = n Cr
( n - r )! r ! (1 + x ) n = n C 0 x 0 + n C 1 x 1 + n C 2 x 2
5! 5 0 5! 4 5! 3 2
For example, ( x + a)5 = x a + x a+ x a + ... + n C r x r + ... + n C n - 1 x n -1
5! 0 ! 4 ! 1! 3! 2!
5! 2 3 5! 5! 0 5 + n C n x n ...(iii)
+ x a + x a4 + x a n
2! 3! 1! 4 ! 0 ! 5!
5 5 5 4 5 3 2
= C0 x + C1 x a + C2 x a + C3 x a + C4 x a + C5 a 5 2 3 5 4 5 5
or (1 + x ) n = å n Cr xr
r =0
Chap 06 Binomial Theorem 439

5. Replacing x by (- x) in Eq. (iii), we get = P + Q (given) …(i)


and ( x - a )n = n C 0 x n - 0 a 0 - n C 1 x n - 1 a1 +n C 2 x n - 2 a 2
(1 - x ) n = n C 0 x 0 - n C 1 x 1 + n C 2 x 2
- n C 3 x n - 3 a 3 + ... + n C n x n - n an
- ... + ( - 1) r n C r x r + ... + n C n ( - 1) n x n
= (n C 0 x n + n C 2 x n - 2 a 2 + n C 4 x n - 4 a 4 + ...)
n
or (1 - x ) n = å ( - 1) r n
Cr x r - (n C 1 x n - 1 a + n C 3 x n - 3 a 3 +n C 5 x n - 5 a 5 + ...)
r =0
= P - Q (given) ...(ii)
5
y Example 1. Expand æç 2a - ö÷ by binomial theorem.
3 (i) P 2 - Q 2 = ( P + Q ) ( P - Q )
è bø
= ( x + a )n × ( x - a )n
Sol. Using binomial theorem, we get
5 0 1 = ( x 2 - a 2 )n [from Eqs. (i) and (ii)]
æ 3ö 5 5-0 æ 3ö 5 5 -1 æ 3ö
ç2a - ÷ = C 0 (2a ) ç - ÷ + C 1(2a ) ç- ÷
è bø è bø è bø (ii) ( x + a )2n - ( x - a )2n = [( x + a )n ]2 - [( x - a )n ]2
2 3
æ 3ö æ 3ö = ( P + Q )2 - ( P - Q )2
+ 5C 2 (2a )5 - 2 ç - ÷ + 5C 3 (2a )5 - 3 ç- ÷
è bø è bø = 4PQ [from Eqs. (i) and (ii)]
4 5
5 5-4 æ 3ö 5 5 - 5 æ 3ö
+ C 4 (2a ) ç - ÷ + C 5 (2a ) ç- ÷
è bø è bø y Example 4. Show that (101) 50 > (100) 50 + (99 ) 50 .
2
æ3ö æ3ö Sol. Since, (101)50 - (99 )50 = (100 + 1)50 - (100 - 1)50
= 5C 0 (2a )5 - 5C 1 (2a )4 ç ÷ + 5C 2 (2a )3 ç ÷
èb ø èb ø
3 4 5
= 2 { 50 C 1 (100)49 + 50
C 3 (100)47 + 50
C 5 (100)45 + ...}
æ3ö æ3ö æ3ö
- 5C 3 (2a )2 ç ÷ + 5C 4 (2a )1 5
ç ÷ - C5 ç ÷ =2´ 50
C 1 (100)49 + 2 { 50C 3 (100)47 + 50
C 5 (100)45 + ...}
èb ø èb ø èb ø
240 a 4 720 a 3 1080 a 2 810 a 243 = (100)50 + ( a positive number) > (100)50
= 32a 5 - + - + 4 - 5
b b2 b3 b b Hence, (101)50 - (99 )50 > (100)50

y Example 2. Simplify Þ (101)50 > (100)50 + (99 )50

( x + ( x 2 - 1 )) 6 + ( x - ( x 2 - 1 ) 6 . n
1
2
y Example 5. If an = å n
Cr
, find the
Sol. Let (x - 1) = a r =0
n
Then, ( x + a )6 + ( x - a )6 = 2 { 6 C 0 x 6 - 0 a 0 + 6C 2 x 6 - 2 a 2 r
value of å n
Cr
×
+ 6C 4 x 6 - 4 a 4 + 6C 6 x 6 - 6 a 6 } r =0
n
= 2 { x 6 + 15x 4a 2 + 15x 2a 4 + a 6 } r
[from point (2)] Sol. Let P = å n
Cr
…(i)
6 4 2 2 2 2 2 3 r =0
= 2 { x + 15x ( x - 1) + 15x ( x - 1) + ( x - 1) }
[Qa = x 2 - 1 ] Replacing r by (n - r ) in Eq. (i), we get
n
(n - r ) n
(n - r )
= 2(32x 6 - 48x 4 + 18x 2
- 1) P= å n
Cn - r
= å n
Cr
[Q n C r = n C n - r ] …(ii)
r =0 r =0

y Example 3. In the expansion of ( x + a )n , if sum of On adding Eqs. (i) and (ii), we get
odd terms is P and sum of even terms is Q, prove that n
n n
1
(i) P 2 - Q 2 = ( x 2 - a 2 )n
2P = å n
Cr
=n å n
Cr
= nan [given]
r =0 r =0
(ii) 4 PQ = ( x + a ) 2n - ( x - a ) 2n n
\ P= an
Sol. Q ( x + a )n = n C 0 x n - 0a 0 + n C 1 x n - 1 a1 + n C 2 x n - 2 a 2 2
n
+ n C 3 x n - 3 a 3 + ... + ... + n C n x n - n an r n
Hence, å n
Cr
=
2
an
= (n C 0 x n + n C 2 x n - 2 a 2 +n C 4 x n - 4 a 4 + ...) r =0

+ (n C 1 x n - 1 a1 + n C 3 x n - 3 a 3 + n C 5 x n - 5 a 5 + ...)
440 Textbook of Algebra

Properties of Binomial How to Construct a Pascal’s Triangle


Expansion ( x + a )n Binomial coefficients in the expansion of ( x + a ) 3 are
(i) This expansion has (n + 1) terms. 1 3 3 1

1 3 3 1
(ii) Since, nC r = nC n - r , we have
1 (1 + 3 ) (3 + 3 ) (3 + 1) 1
n
C 0 = n Cn = 1 Then, 1 4 6 4 1
n n
C1 = Cn - 1 = n are the binomial coefficients in the expansion of ( x + a ) 4 .
n n (n - 1)
C2 = n Cn - 2 =
and so on. y Example 6. Find the number of dissimilar terms
2!
in the expansion of (1 - 3x + 3x 2 - x 3 ) 33 .
(iii) In any term, the suffix of C is equal to the index of a
and the index of x = n - (suffix of C ). Sol. (1 - 3x + 3x 2 - x 3 )33 = [(1 - x )3 ]33 = (1 - x )99
(iv) In each term, sum of the indices of x and a is equal to n. Therefore, number of dissimilar terms in the expansion of
(1 - 3x + 3x 2 - x 3 )3 is 100.
Properties of Binomial Coefficient n
r × nC r
n ænö
(i) C r can also be represented by C (n, r ) or ç ÷ . y Example 7. Find the value of å nC .
èr ø r =1 r -1
n
(ii) n C x = nC y , then either x = y or n = x + y . Cr n -r +1
Sol. Q n
=
Cr -1 r
n!
So, n C r = n C n - r = n
r ! (n - r ) ! r × Cr
\ n
= ( n - r + 1)
n +1 Cr
(iii) n C r + nC r - 1 = Cr -1
n
r × nC r n n n
å nC å ( n - r + 1) = å ( n + 1) - å r
n
Cr n -r +1 \ =
(iv) = r =1 r -1 r =1 r =1 r =1
n r
C r -1
n

(v) n C r =
n
× n -1
Cr - 1
= ( n + 1) å1 - (1 + 2 + 3 + ... + n )
r =1
r
n ( n + 1) n ( n + 1)
= ( n + 1) × n - =
2 2
Pascal’s Triangle y Example 8. Let C r stands for n C r , prove that
Coefficients of binomial expansion can also be easily
determined by Pascal’s triangle. (C 0 + C 1 ) (C 1 + C 2 ) (C 2 + C 3 ) ...(C n -1 + C n )

(x + a)0 1 (n + 1)n
= (C 0C 1C 2 ... C n -1 ).
(x + a)1 1 1 n!
(x + a)2 1 2 1 Sol. LHS = (C 0 + C 1 ) (C 1 + C 2 ) (C 2 + C 3 ) ... (C n - 1 + C n )
n n
(x + a)3 3 3
Õ(C r - 1 + C r ) = Õ(n + 1C r )
1 1 n +1
= [Q n C r + n C r -1 = Cr ]
(x + a)4 1 4 6 4 1 r =1 r =1

( x + a )5 n
æn + 1ö n é n n n -1 ù
1 5 10 10 5 1
= Õ çè r ø
÷ Cr -1 êëQ C r = r × Cr - 1ú
û
Pascal triangle gives the direct binomial coefficients. r =1
n n n
1
For example, = Õ ( n + 1) × Õ r × Õ C r - 1
( x + a ) 4 = 1 × x 4 + 4 × x 3 × a + 6 × x 2a 2 r =1 r =1 r =1

3 4 1
+ 4 × x a + 1× a n
= (n + 1) × × (C 0C 1C 2 ... C n - 1 )
n!
= x 4 + 4 x 3a + 6 x 2a 2 + 4 x a 3 + a 4 (n + 1)n
= (C 0 C 1 C 2 ... C n - 1 ) = RHS
n!
Chap 06 Binomial Theorem 441

y Example 9. Find the sum of the series æ 1ö


n
æ 3ö æ
n

n
æ 15 ö
n
= ç1 - ÷ + ç1 - ÷ + ç1 - ÷ + ç1 - ÷
n ì1 3r 7r 15r ü è 2 ø è 4 ø è 8 ø è 16 ø
å ( - 1)r nC r í 2r + 2r
+ 3r
+ 4r
+ ... upto m termsý.
r =0 î 2 2 2 þ + ... upto m terms
n
å ( - 1)
n r n r n 2n 3n 4n
Sol. Q(1 - x ) = Cr x ...(i) æ1ö æ1ö æ1ö æ1ö
r =0
=ç ÷ +ç ÷ +ç ÷ +ç ÷ + ... upto m terms
n ìï æ 1 ö r r r è2ø è2ø è2ø è2ø
æ3ö æ7 ö
Let P= å ( - 1) r n
Cr íç ÷ + ç ÷ + ç ÷
è ø è ø è8ø n é ìï æ 1 ön üï ù
m
2 4
r =0 îï æ1ö ê
üï ç ÷ 1 - íç ÷ ý ú
r
æ 15 ö è2ø ê è2ø ï ú
+ ç ÷ + ... upto m termsý
è 16 ø ïþ ë îï þ û
n r n r =
æ1ö æ3ö æ1ö
n
= å ( - 1)r n
Cr × ç ÷
è2ø
+ å ( - 1) C r × ç ÷
r n
è4ø 1- ç ÷
r =0 r =0
n rn r è2ø
Cr × æç ö÷ + å ( - 1)r nCr × æç ö÷
7 15
å (- 1)
r n
+
è8ø r = 0 è 16 ø (2mn - 1)
r =0 =
+ K upto m terms 2mn (2n - 1)

#L Exercise for Session 1


10
1. The value of å r × 10Cr × 3r × ( -2)10 - r is
r =0

(a) 10 (b) 20 (c) 30 (d) 300


15
æ 1 1ö
2. The number of dissimilar terms in the expansion of ç x + + x 2 + 2 ÷
è
are
x x ø
(a) 61 (b) 121 (c) 255 (d) 16

3. 3 1/ 2 5
The expansion {x + ( x - 1) } + {x - ( x - 1) } is a polynomial of degree 3 1/ 2 5

(a) 5 (b) 6 (c) 7 (d) 8

4. 6
( 2 + 1) - ( 2 - 1) is equal to 6

(a) 101 (b) 70 2 (c) 140 2 (d) 120 2

5. The total number of dissimilar terms in the expansion of ( x + a ) 100


+ (x - a ) 100
after simplification will be
(a) 202 (b) 51
(c) 50 (d) 101

6. The number of non-zero terms in the expansion of (1+ 3 2x )9 + (1- 3 2x )9, is


(a) 0 (b) 5
(c) 9 (d) 10
n
æ C1 ö æ C2 ö æ Cn ö
7. If (1 + x )n = å Cr x r , ç1 +
è
÷ ç1 +
C0 ø è
÷ ... ç1 +
C1 ø è
÷ is equal to
Cn - 1 ø
r =0

n n -1
(n + 1)n - 1
(a) (b)
(n - 1)! (n - 1) !
(n + 1)n (n + 1)n + 1
(c) (d)
n! n!
n+1 n -1
8. If Cr + 1 : nCr : Cr - 1 = 11: 6 : 3, nr is equal to
(a) 20 (b) 30
(c) 0 (d) 50
Session 2
General Term, Middle Terms, Greatest Term,
Trinomial Expansion

General Term (ii) the coefficient of x - 7 in the expansion of


11
The term n C r x n - r a r is the (r + 1) th term from æ 1 ö
ç ax - 2 ÷ .
è bx ø
beginning in the expansion of ( x + a ) n . It is usually called
the general term and it is denoted by Tr + 1 . Also, find the relation between a and b, so that these
i.e., Tr + 1 = nC r x n - r a r coefficients are equal.
11
æ 1 ö
y Example 10. Find the 7th term in the expansion of Sol. (i) Here, Tr +1 = 11C r (ax 2 )11- r ç ÷
è bx ø
13
æ 1 ö
ç4 x - ÷ . a11 - r
= 11C r × × x 22 - 3r …(i)
è 2 xø br

6
æ 1 ö Now, in order to find out the coefficient of x 7 , 22 - 3r
Sol. Seventh term, T 7 = T 6 + 1 = 13C 6 ( 4 x )13 - 6 ç - ÷
è 2 xø must be 7,
1 i.e. 22 - 3r = 7
= 13C 6 × 4 7 × x 7 × 6 3
2 ×x \ r =5
= 13C 6 × 28 × x 4 Hence, putting r = 5 in Eq. (i), we get
a6
8 Required coefficient = 11C 5 .
y Example 11. Find the coefficient of x in the b5
10
æ 1ö æ 1 ö
R
expansion of ç x 2 - ÷ . (ii) Here, T R + 1 = 11C R (ax )11 - R ç - 2 ÷
è xø è bx ø
r
æ 1ö R
Sol. Here, Tr = 10C r ( x 2 )10 - r ç - ÷ æ 1ö
+1
è xø = C R (a )11 - R ç - ÷ × x 11 - 3 R
11
è bø
1
= 10C r x 20 - 2r × ( - 1)r × a11 - R
xr = ( - 1)R × 11C R × × x 11 - 3 R …(ii)
10 r 20 - 3r bR
= C r ( - 1) × x ...(i)
Now, in order to find out the coefficient of x - 7 ,
8
Now, in order to find out the coefficient of x , 20 - 3r must 11 - 3R must be - 7.
be 8. i.e., 11 - 3R = - 7 Þ R = 6. Hence, putting R = 6 in Eq.
i.e. 20 - 3r = 8 (ii), we get
\ r =4 Required coefficient
Hence, putting r = 4 in Eq. (i), we get a5 a5
= ( - 1)6 × 11C 6 × = 11C 5 × [Q n C r = n C n - r ]
10 × 9 × 8 × 7 b 6
b 6
Required coefficient = ( - 1)4 × 10C 4 = = 210
1×2×3× 4 Also given, coefficient of x 7 in
11 11
æ 2 1 ö -7 æ 1 ö
y Example 12. Find ç ax + ÷ = coefficient of x in ç ax - ÷
è bx ø è bx 2 ø
(i) the coefficient of x 7 in the expansion of
11 11 a6 a5
æ 2 1ö Þ C5 × = 11C 5 ×
Þ ab = 1
ç ax + ÷ . b5 b6
è bx ø which is the required relation between a and b.
Chap 06 Binomial Theorem 443

y Example 13. Find the term independent of x in the


9
How to Find Free from Radical Terms or
expansion of æç x 2 - ö÷ .
3 1 Rational Terms in the Expansion of
è2 3x ø ( a1/ p + b1/q )N , " a, b Î Prime Numbers
9 -r r
æ3 ö æ 1 ö -r
Sol. Here, Tr = 9C r ç x 2 ÷ ç- ÷
First, find Tr + 1 = N
C r (a 1 /p ) N (b 1 /q ) r
+1
è2 ø è 3x ø - r ) /p
9 -r r
\ Tr + 1 = N
Cr × a ( N × b r /q
æ3ö æ1ö
= ( - 1)r × 9C r × ç ÷ × ç ÷ × x 18 - 3r …(i) By inspection, putting the values of 0 £ r £ N , when
è2ø è3ø
indices of a and b are integers.
If this term is independent of x, then the index of x must be
zero, i.e., 18 - 3r = 0 Þ r = 6 Remark
Therefore, (r + 1) th term, i.e., 7th term is independent of x 1. If indices of a and b are positive integers.
and its value by putting r = 6 in Eq. (i) Then, free from radical terms = Terms which are integers
3 6
æ3ö æ1ö 1 \ Number of non-integral terms = Total terms - Number of
= ( - 1)6 × 9C 6 × ç ÷ × ç ÷ = 9C 3 × 3 3 integral terms
è2ø è3ø 2 ×3
2. If indices of a and b both are not positive integers.
9 ×8×7 7 Then, free from radical terms = Rational terms - Integral
= =
( 1 × 2 × 3) 23 × 33 18 terms
3. Number of irrational terms = Total terms – Number of
rational terms
( p + 1) th Term From End in the
Expansion of ( x + a )n y Example 16. Find the number of terms in the
expansion of ( 4 9 + 6 8 ) 500 which are integers.
( p + 1) th term from end in the expansion of ( x + a ) n
n
Sol. Since, ( 4 9 + 6 8 )500 = (91/ 4 + 81/ 6 )500 = (31/ 2 + 21/ 2 )500
= ( p + 1) th term from beginning in the expansion of (a + x )
[Qa, b Îprime numbers]
= nCp a n - p x p
\ General term, Tr +1 = 500
C r (31/ 2 )500 - r × (21/ 2 )r
500 - r
y Example 14. Find the 4th term from the end in the 500
7 = Cr × 3 2 × 2r / 2
æx3 2 ö
expansion of ç - 2÷ . = 500
C r × 3250 - r / 2 × 2 r / 2
è 2 x ø 7 Now, 0 £ r £ 500
æx3 2 ö
Sol. 4th term from the end in the expansion of ç - 2÷ For r = 0 , 2, 4, 6, 8, ... , 500, indices of 3 and 2 are positive
è 2 x ø
integers.
= 4th term from beginning in the expansion of
æ 2
7 Hence, number of terms which are integers = 250 + 1 = 251
x3 ö
ç- 2 + ÷
è x 2 ø
y Example 17. Find the sum of all rational terms in
7-3 3
æ 2 ö æx3 ö 7 × 6 × 5 24 x 9 the expansion of ( 31/ 5 + 21/ 3 )15 .
= 7C 3 ç - 2 ÷ ç ÷ = × × = 70x
è x ø è 2 ø 1 × 2 × 3 x 8 23
Sol. The general term in the expansion of (31/ 5 + 21/ 3 )15 is
Tr +1 = 15C r (31/ 5 )15 - r × (21 / 3 )r
y Example 15. Find the (n + 1)th term from the end in
r r
3n 3-
æ 1ö = 15C r × 3 5 ×23
the expansion of ç 2x - ÷ .
è xø Now, 0 £ r £ 15
3n
æ 1ö Forr = 0, 15
Sol. (n + 1)th term from the end in the expansion of ç2x - ÷
è xø Rational terms are T 0 + 1 and T15 + 1.
= (n + 1) th 3term
n
from beginning in the expansion of Then, T 0 + 1 = 15C 0 × 33 × 20 = 27
æ 1 ö
ç - + 2x ÷ and T15 + 1 = 15C 15 × 30 × 25 = 32
è x ø
3n - n
æ 1ö \ Sum of all rational terms = 27 + 32 = 59
= Tn + 1 = 3n C n ç - ÷ (2x )n = 3n C n × 2n × x - n
è xø
444 Textbook of Algebra

y Example 18. Find the number of irrational terms in y Example 20. If a, b , c and d are any four
the expansion of ( 8 5 + 6 2 )100 . consecutive coefficients in the expansion of (1 + x )n ,
Sol. Since, ( 8 5 + 6 2 )100 = (51/ 8 + 21/ 6 )100 then prove that:
\ General term, Tr = 100
C r (51 / 8 )100 - r (21 / 6 )r a c 2b
+1 (i) + = .
= 100
C r (5)(100 - r ) / 8 × (2)r / 6
a+b c + d b + c
2
As, 2 and 5 are coprime. æ b ö ac
(ii) ç ÷ > , if x > 0.
\ Tr + 1 will be rational, if (100 - r ) is a multiple of 8 and r is èb + c ø (a + b ) (c + d )
a multiple of 6.
Sol. Let a, b, c and d be the coefficients of the r th, (r + 1)th,
Also, 0 £ r £ 100
(r + 2)th and (r + 3)th terms respectively, in the expansion
\ r = 0, 6, 12, 18, ..., 96
of (1 + x )n . Then,
Now, 100 - r = 4, 10, 16, ... , 100 …(i) -1
Tr = Tr -1+1 = nC r -1 xr
and 100 - r = 0, 8, 16, 24, ... , 100 …(ii)
The common terms in Eqs. (i) and (ii) are 16, 40, 64 and 88. \ a = nC r -1 …(i)
\ r = 84, 60, 36, 12 gives rational terms. Q Tr = nC r x r
+1
\ The number of irrational terms = 101 - 4 = 97
\ b = nC r …(ii)
n r +1
Problems Regarding Three/Four Q Tr +2 = T (r + 1) + 1 = Cr +1 x

Consecutive Terms or Coefficients \ c = nC r +1 ….(iii)


n r +2
and Tr = T (r = Cr x
(i) If consecutive coefficients are given +3 + 2) + 1 +2

In this case, divide consecutive coefficients pairwise, we \ d = nC r +2 …(iv)


get equations and then solve them. From Eqs. (i) and (ii), we get
a + b = n C r - 1 + n C r = n + 1C r
y Example 19. Let n be a positive integer. If the n +1 n æn + 1ö
coefficients of rth, (r + 1)th and (r + 2)th terms in the = × Cr - 1 = ç ÷a
r è r ø
expansion of (1 + x )n are in AP, then find the a r
relation between n and r. \ = …(v)
a+b n +1
n r -1
Sol. Q Tr = T (r - 1) + 1 = Cr -1 x From Eqs. (ii) and (iii), we get
n r n r +1 n +1
Tr +1 = Cr x and Tr +2 = T (r + 1) +1 = Cr +1 x b + c = nC r + nC r +1 = Cr +1

\ Coefficients of rth, (r + 1)th and (r + 2)th terms in the æn + 1ö n æn + 1ö


expansion of =ç ÷ Cr = ç ÷b
èr + 1ø èr + 1ø
(1 + x )n are n C r - 1, n C r , n C r + 1.
b r +1
Q Given, n C r - 1,
n
Cr , n Cr +1 are in AP. \ = …(vi)
b+c n +1
and n ³r +1 From Eqs. (iii) and (iv), we get
n n
Cr Cr n +1
\
-1
, 1,
+1
are also in AP. c + d = nC r + 1 + nC r +2 = Cr +2
n n
Cr Cr æn + 1ö n æn + 1ö
r n -r =ç ÷ Cr +1 =ç ÷c
Þ , 1, are in AP. èr + 2ø èr + 2ø
n -r +1 r +1 c r +2
r n -r n - 2r +1 n - 2r - 1 \ = …(vii)
Þ 1- = -1 Þ = c +d n +1
n -r +1 r +1 n -r +1 r +1
From Eqs. (v), (vi) and (vii), we get
Þ nr - 2r 2 + r + n - 2r + 1 a b c
, and are in AP.
= n 2 - 2nr - n - nr + 2r 2 + r + n - 2r - 1 a+b b+c c +d
Þ n 2 - 4nr + 4r 2 = n + 2 Þ (n - 2r )2 = n + 2 a c æ b ö
(i) + =2ç ÷
Corollary I For r = 2, n = 7 [Qn ³ 3] a+b c +d èb + c ø

Corollary II For r = 5, n = 7, 14 [Qn ³ 6] a c 2b


or + =
a+b c +d b+c
Chap 06 Binomial Theorem 445

(ii) AM > GM From Eqs. (i) and (vi), we get


æ b ö æ a öæ c ö 5 3
\ C 1 × x 4 × y = 240 Þ 5× x4 × x = 240
ç ÷> ç ÷ç ÷ 2
èb + c ø èa + b ø èc + d ø
5
2 \ x = 32 = 25 Þ x = 2
æ b ö ac
Þ ç ÷ > From Eq. (vi), we get y = 3
èb + c ø (a + b ) (c + d )
Hence, x = 2, y = 3 and n = 5
Remembering Method
Q
a b c d
Middle Terms
a+b b+c The middle term depends upon the value of n.
c+d
(i) When n is even The total number of terms in the
a b c expansion of ( x + a ) n is n + 1 (odd). So, there is only
\ , and are in AP.
a+b b+c c +d æn ö
one middle term, i.e., ç + 1÷ th term is the middle
è2 ø
(ii) If consecutive terms are given
term. It is given byTn /2 + 1 = n C n /2 x n /2 a n /2
In this case, divide consecutive terms pairwise. i.e., If four
(ii) When n is odd The total number of terms in the
consecutive terms are Tr , Tr + 1 , Tr + 2 , Tr + 3 . Then, find expansion of ( x + a ) n is n + 1 (even). So, there are
Tr + 1 Tr + 2 Tr + 3
, , Þ l1 , l2 , l 3 (say). Then, divide l2 by æ n + 1ö æn + 3ö
Tr Tr + 1 Tr + 2 two middle terms, i.e., ç ÷ th and ç ÷ th are
è 2 ø è 2 ø
l1 and l 3 by l2 and solve. two middle terms. They are given by
n +1 n -1
y Example 21. If the 2nd, 3rd and 4th terms in the T n + 1 =T æ n - 1 ö = nC n - 1 × x 2 ×a 2
ç ÷ +1
expansion of ( x + y )n are 240, 720 and 1080 2 è 2 ø 2
n -1 n +1
respectively, find x , y and n. n
and T n + 3 = T æ n + 1 ö = Cn +1 × x 2 ×a 2
Sol. Given, T 2 = T1 + 1 = n C 1 × x n - 1× y = 240 …(i) ç ÷ +1
2 è 2 ø 2
T 3 = T 2 + 1 = n C 2 × x n - 2 × y 2 = 720 …(ii)
and T 4 = T 3 + 1 = n C 3 × x n - 3 × y 3 = 1080 …(iii) y Example 22. Find the middle term in the
12
On dividing Eq. (ii) by Eq. (i), we get æa ö
expansion of ç + bx ÷ .
n
C 2 × x n - 2 × y 2 720 èx ø
=
C1 × x n - 1 × y
n 12
240 æa ö
Sol. The number of terms in the expansion of ç + bx ÷ is 13
æn - 2 + 1ö y y 6 èx ø
Þ ç ÷× =3 Þ = …(iv) æ 12 ö
è 2 ø x x n -1 (odd), its middle term is ç + 1÷ th, i.e., 7th term.
è2 ø
Also, dividing Eq. (iii) by Eq. (ii), we get 6
12 æa ö
n
C3 × x n-3
×y 3
1080 \ Required term, T 7 = T 6 +1 = C 6 ç ÷ (bx )6
= èx ø
n-2
n
C2 × x ×y 2
720 = 12 C 6 a 6 b 6 = 924 a 6 b 6
æn - 3 + 1ö y 3 y 9
Þ ç ÷× = Þ = …(v) y Example 23. Find the middle term in the expansion
è 3 ø x 2 x 2 ( n - 2) 9
æ x3 ö
From Eqs. (iv) and (v), we get of ç 3x - ÷ .
6 9 è 6 ø
= æ
9
x3 ö
n - 1 2 ( n - 2) Sol. The number of terms in the expansion of ç3x - ÷ is
10 (even). So, there are two middle terms, è
6 ø
Þ 12n - 24 = 9 n - 9
Þ 3n = 15 æ9 +1ö æ9 + 3ö
i.e. ç ÷ th and ç ÷ th terms. They are given by T 5
\ n =5 è 2 ø è 2 ø
3 and T 6 .
From Eq. (iv), we get y = x …(vi)
2
446 Textbook of Algebra

4
æ x 3ö Now, on substituting values of n, x and a in Eq. (i), we get
\ T 5 = T 4 +1 = 9C 4 (3x )5 ç - ÷
è 6 ø r £ m + f or r £ m
where, m Î N and 0 < f < 1
9 × 8 × 7 × 6 5 5 x 12 189 17
= ×3 x × 4 = x In the first case,Tm +1 is the greatest term, while in the
1× 2× 3× 4 6 8
second case, Tm and Tm + 1 are the greatest terms and both
5
æ x 3ö are equal (numerically).
and T 6 = T 5 + 1 = 9C 5 (3x )4 ç - ÷
è 6 ø Shortcut Method
x 15 To find the greatest term (numerically) in the expansion of
= - 9C 4 × 34 × x 4 ×
65 (x + a )n .
n
9 × 8 × 7 × 6 4 x 19 21 19 æ xö
=- ×3 × 5 = - x Now, (x + a )n = a n ç1 + ÷
1× 2× 3× 4 6 16 è aø
x
y Example 24. Show that the middle term in the (n + 1)
a
expansion of (1 + x ) 2n is Calculate m=
æ x ö
1 × 3 × 5 ...(2n - 1) n n ç +1 ÷
× 2 x , n being a positive integer. è a ø
n!
Case I If m ÎInteger, then Tm and Tm + 1 are the greatest terms
Sol. The number of terms in the expansion of (1 + x )2n is
and both are equal (numerically).
2n + 1 (odd), its middle term is (n + 1)th term.
Case II If m Ï Integer, then T[m ] + 1 is the greatest term, where
\ Required term = Tn + 1
[ × ] denotes the greatest integer function.
2n ! n (1 × 2 × 3 × 4 . .. (2n -1) × 2n ) n
= 2n C n x n = x = x
n! n! n! n! y Example 25. Find numerically the greatest term in
{1 × 3 × 5 ... (2n - 1)} {2 × 4 × 6... 2n } n the expansion of (2 + 3x ) 9 , when x = 3 / 2.
= x
n! n! Sol. Let Tr be the greatest term in the expansion of
+1
{1 × 3 × 5... (2n - 1)} 2n (1 × 2 × 3... n ) n (2 + 3x )9 , we have
= x
n! n! Tr +1 æ 9 - r + 1 ö 3x æ 10 - r ö 3 3 90 - 9r
=ç ÷ =ç ÷ ´ =
n
{1 × 3 × 5... (2n - 1)} 2 n ! n 1 × 3 × 5... (2n - 1) n n Tr è r ø 2 è r ø 2 2 4r
= x = 2 x
n! n! n! [Q x = 3 / 2]
Tr + 1
\ ³1
Tr
Greatest Term Þ
90 - 9r
³ 1 Þ 90 ³ 13r
If Tr and Tr +1 are the rth and (r + 1) th terms in the 4r
expansion of ( x + a ) n , then 90 12
\ r £ =6
13 13
Tr + 1 n
C r × x n -r × a r æ n - r + 1ö a
= =ç ÷× or r £6
12
Tr n
Cr - 1 × x n -r +1
×a r -1 è r ø x 13
Let numerically, Tr +1 be the greatest term in the above \ Maximum value of r is 6.
expansion. Then, So, greatest term = T 6 + 1 = 9C 6 (2)9 - 6 (3x )6
Tr + 1 æ n - r + 1ö a æ 3ö
6
Tr + 1 ³ Tr or ³1 Þ ç ÷ ³1 = 9C 3 × 23 × ç3 ´ ÷
Tr è r ø x è 2ø
[Qa may be + ve or - ve] 9 × 8 × 7 23 × 312 7 ´ 313
(n + 1) = × =
or r£ …(i) 1 × 2× 3 26 2
æ x ö 9
ç1 + ÷ æ
Aliter Since, (2 + 3x )9 = 29 ç1 +
3x ö
è a ø ÷
è 2 ø
Chap 06 Binomial Theorem 447

( 9 + 1)9 3x \ Greatest term (when r = 2) = 11C 2 (3)9 ( - 5x )2


10 ´
Now, m = =
2
4 [Q x = 3 / 2] é 1ù
= 11C 2 (3)9 ( - 1)2 êëQ x = 5 úû
3x 9
+1 +1
2 4 11 × 10 9
= × 3 = 55 ´ 39
90 12 1×2
= =6 ¹ Integer
13 13 and greatest term (when r = 3) = 11C 3 (3)8 ( - 5x )3
\ The greatest term in the expansion is 11 é 1ù
9 = C 3 ( 3) 8 ( - 1) 3 êëQ x = 5 úû
T[m ] + 1 = T 6 + 1 in (2 + 3x )
11 × 10 × 9 8
æ 32 ö
6
= × 3 = 55 ´ 39
9
= C 6 ( 2) 9-6 6 9
( 3x ) = C 3 × 2 × ç ÷3
[Q x = 3 / 2] 1×2×3
è2ø
9 × 8 × 7 312 7 ´ 313
= ×
1 × 2 × 3 23
=
2
Greatest Coefficient
(i) If n is even, then greatest coefficient is n C n /2 ×
y Example 26. Find numerically the greatest term in (ii) If n is odd, then greatest coefficients are n C (n - 1 ) /2 and
1
the expansion of ( 3 - 5x )11 , when x = . n
C (n + 1 ) /2 ×
5
Sol. Let Tr +1 be the greatest term in the expansion of
11
y Example 27. Show that, if the greatest term in the
(3 - 5x ) , we have expansion of (1 + x ) 2n has also the greatest coefficient,
Tr + 1 æ 11 - r + 1 ö 5x n n+1
=ç ÷ -
Tr è r ø 3 then x lies between and .
n+1 n
æ 12 - r ö 1 12 - r
=ç ÷ - = [Q x = 1 / 5] Sol. In the expansion of (1 + x )2n , the middle term is
è r ø 3 3r
æ 2n ö
Tr + 1 12 - r ç + 1÷ th
\ ³1 Þ ³ 1 Þ 12 ³ 4r è2 ø
Tr 3r
i.e., (n + 1) th term, we know that from binomial expan-
\ r £ 3 Þ r = 2, 3
sion, middle term has greatest coefficient.
So, the greatest terms are T 2 + 1 and T 3 + 1. [Q Terms T1, T 2 , T 3 , ..., Tn , Tn + 1, Tn + 2 , ...]
\Greatest term (when r = 2) = T 2 + 1 = 11C 2 (3)9 ( - 5x )2 \ Tn < Tn + 1 > Tn + 2
11 × 10 9 Tn + 1 2n
Cn × x n 2n - n + 1
= × 3 × (1)2 = 55 ´ 39 [Q x = 1 / 5] Þ = = ×x
1×2 Tn 2n
Cn - 1 × x n - 1 n
and greatest term (when r = 3) = T 3 + 1
Tn + 1 n +1
= 11
C 3 ( 3) 8 ( - 5x ) 3 = 11
C 3 ( 3) 8 ( - 1) 3 [Q x = 1 / 5] Þ > 1 or ×x >1
Tn n
11 × 10 × 9 8 n
= 11C 3 × 38 = × 3 = 55 ´ 39 or x> …(i)
1×2×3 n +1
From above, we say that the values of both greatest terms Tn + 2 2n
Cn + 1 x n + 1 2n - (n + 1) + 1
are equal. and = = ×x
Tn + 1 2n
Cn x n
n +1
Aliter 11 n
æ 5x ö = ×x
Since, (3 - 5x )11 = 311 ç1 - ÷ n +1
è 3 ø Tn + 2 n n +1
5x 1 Þ <1 Þ × x < 1 or x < …(ii)
(11 + 1) - 12 ´ - Tn + 1 n +1 n
3 3 é 1ù
Now, m= = êëQ x = 5 úû From Eqs. (i) and (ii), we get
5x 1
- +1 - +1 n n +1
3 3 <x<
n +1 n
4
=3= Corollary For n = 5
1
+1 5 6
3 <x<
Since, the greatest terms in the expansion are T 3 and T 4 . 6 5
448 Textbook of Algebra

Important Properties of Trinomial Expansion


the Binomial Coefficients 2n
n
In the binomial expansion of (1 + x ) . Let us denote the
For n Î N , (1 + x + x 2 ) n = åar x r
r =0
coefficients n C 0 , n C 1 , n C 2 , ..., n C r , ... , n C n by C 0 , C 1 , C 2 , = a 0 + a1 x + a2 x 2
+ K + an x n + K + a 2n x 2n …(i)
... , C r , ... , C n , respectively.
There are (2n + 1) terms. The middle coefficient is a n
(i) The coefficients of the terms equidistant from which is also the greatest.
the beginning and the end are equal
a 0 = a 2n , a 1 = a 2n - 1 , K, a r = a 2n - r
The (r + 1)th term from the beginning in the
expansion of (1 + x ) n is nC r x r × The coefficients of (1 + x + x 2 ) n for n = 0, 1, 2, … can be
arranged in a triangle.
\ The coefficient of the (r + 1) th term from the 1
beginning is nC r and the (r + 1) th term from the end 1 1 1
n 1 2 3 2 1
in the expansion of (1 + x ) = (r + 1) th term from the 1 3 6 7 6 3 1
beginning in the expansion of ( x + 1) n = nC r x n - r 1 4 10 16 19 16 10 4 1
\ The coefficient of the (r + 1) th term from the end is 1 5 15 30 45 51 45 30 15
n 5 1
Cr . . . . . . . . . . . .
. . . . . . . . . . .
Hence, the coefficients of (r + 1) th term from the
i.e., The rows contains the coefficients for n = 0, 1, 2, 3,K.
beginning and the end are equal.
Each entry other than two entries at the ends is the sum of
(ii) The sum of the binomial coefficients in the three entries above it.
expansion of (1 + x ) n
15 = 1 + 4 + 10, 30 = 16 + 10 + 4, etc.
Q (1 + x ) n = n C 0 + n C 1 x + n C 2 x 2 + n C 3 x 3
Putting x = 1 and x = - 1 in Eq. (i), we get
+ ... + n C n x n
a 0 + a 1 + a 2 + a 3 + ... + a 2n = 3 n
Putting x = 1, we get
2 n = n C 0 + n C 1 + n C 2 + ... + n C n [sum of all coefficients] …(ii)
and a 0 - a 1 + a 2 - a 3 + ... + a 2n = 1 …(iii)
or C 0 + C 1 + C 2 + ... + C n = 2 n
On adding Eqs. (ii) and (iii), we get
\ Sum of binomial coefficients = 2 n
3n + 1
a 0 + a 2 + a 4 + ... + a 2n =
(iii) The sum of the coefficients of the odd terms 2
= The sum of the coefficients of the even terms [sum of coefficients of even powers of x ]
Q (1 + x ) n = n C 0 + n C 1 x + n C 2 x 2 + n C 3 x 3 On subtracting Eq. (ii) from Eq. (i), we get
+ ... + n C n x n
3n - 1
a 1 + a 3 + a 5 + K + a 2n - 1 =
Putting x = - 1, we get 2
0 = n C 0 - n C 1 + n C 2 - n C 3 + n C 4 - n C 5 + ... [sum of coefficients of odd powers of x ]
n
or C 1 + n C 3 + n C 5 + ... = n C 0 + n C 2 + n C 4 + ...
Since, the sum of all the coefficients is 2 n , therefore Putting x = i ( - 1 ) in Eq. (i), we get
2n a 0 + a 1 i + a 2 i 2 + a 3 i 3 + a 4 i 4 + a 5 i 5 + ... + a 2n i 2n = i n
each side is equal to i.e. 2 n -1 .
2 Þ (a 0 - a 2 + a 4 - ... ) + i (a 1 - a + a 5 - ... ) = i n
3
Hence, C 1 + C 3 + C 5 + ... = C 0 + C 2 + C 4 + ... = 2 n -1
(a 0 - a 2 + a 4 - K ) + i(a 1 - a 3 + a 5 - K )
n
Remark æ p pö æn p ö æn p ö
n
1. In the expansion of ( x - 2 y + 3z ) , putting x = y = z = 1, then
= ç cos + i sin ÷ = cos ç ÷ + i sin ç ÷
è 2 2ø è 2 ø è 2 ø
we get the sum of coefficients = ( 1 - 2 + 3) n = 2n .
2. In the expansion of ( 1 + x + x 2 ) n, putting x = 1, we get the sum On comparing real and imaginary parts, we get
of coefficients = ( 1 + 1 + 1) n = 3n. æ np ö
a 0 - a 2 + a 4 - K = cos ç ÷
è 2 ø
Chap 06 Binomial Theorem 449

æ np ö Þ ( 4t + 1) ( t - 1) = 0
and a 1 - a 3 + a 5 - K = sin ç ÷
è 2 ø \ t = 1, t ¹ -
1
Þ 3 3 x / 2 = 1 = 30
4
Putting x = w and w2 (cube roots of unity) in Eq. (i), we get
3x
a 0 + a 1 w + a 2 w2 + a 3 w 3 + a 4 w 4 + ... = 0 …(iv) \ = 0 or x = 0
2
and a 0 + a 1 w2 + a 2 w 4 + a 3 w 6 + a 4 w 8 + ... = 0 …(v)
y Example 30. Find the values of
On adding Eqs. (ii), (iv) and (v) and then dividing by 3, we 1 1 1
get (i) + + + ...
(n - 1)! (n - 3)! 3! (n - 5)! 5!
a 0 + a 3 + a 6 + ... = 3 n - 1
1 1 1 1
Note
n -1
(ii) + + + ... +
(i) a1 + a4 + a7 + K = a2 + a5 + a8 + K = 3 12! 10! 2! 8 ! 4 ! 12!
(ii) a0 + a4 + a8 + K =
1ì n æ np ö ü Sol. (i) Q 1! = 1
í3 + 1 + 2cos ç ÷ ý
4î è 2 øþ
\ The given series can be written as
(iii) a1 + a5 + a9 + K =
1ì n æ np ö ü
í3 - 1 + 2sinç ÷ ý 1 1 1
4î è 2 øþ + + + ... …(i)
1ì np ü (n - 1)! 1! (n - 3)! 3! (n - 5)! 5!
(iv) a0 + a6 + a12 + K = í3n + 1 + 2n + 1 cos æç ö÷ ý
6î è 3 øþ Q Sum of values of each terms in factorial are equal.
2n 2n
r -1 i.e. (n - 1) + 1 = (n - 3) + 3 = (n - 5) + 5 = ... = n
(v) å r × ar = n × 3n (vi) å( -1) × r × ar = - n
r =1 r =1 From Eq. (i),
1 é n! n! n! ù
y Example 28. Find the sum of coefficients in the n! ê (n - 1)! 1! + (n - 3)! 3! + (n - 5)! 5! + ...ú
ë û
expansion of the binomial ( 5p - 4q )n , where n is a
1 n 2n - 1
positive integer. =( C 1 + n C 3 + n C 5 + ...) =
n! n!
Sol. Putting p = q =1 in (5p - 4q )n , the required sum of coeffi-
(ii) Q 0! = 1
cients = (5 - 4 )n = 1n = 1
\The given series can be written as
y Example 29. In the expansion of ( 3 - x / 4 + 3 5x / 4 )n , if 1
+
1
+
1
+ ... +
1
…(ii)
12!0! 10! 2! 8! 4 ! 0! 12!
the sum of binomial coefficients is 64 and the term
with the greatest binomial coefficient exceeds the third Q Sum of values of each terms in factorial are equal
by (n - 1), find the value of x. i.e., 12 + 0 = 10 + 2 = 8 + 4 = ... = 12
Sol. Given sum of the binomial coefficients in the expansion of 1 é 12! 12! 12! 12! ù
(3- x / 4 + 35 x / 4 )n = 64
From Eq. (ii),
12! ê 12! 0! + 10! 2! + 8! 4 ! + ... + 0! 12! ú
ë û
Then, putting 3- x / 4 = 35 x / 4 = 1 1 12 212 - 1 211
12 12 12
n n 6
= ( C0 + C2 + C 4 + ... + C 12 ) = =
\ (1 + 1) = 64 Þ 2 = 2 12! 12! 12!
\ n =6
y Example 31. Prove that the sum of the coefficients
We know that, middle term has the greatest binomial
in the expansion of (1 + x - 3x 2 ) 2163
coefficients. Here, n = 6
æn ö
is - 1.
\ Middle term = ç + 1÷ th term = 4th term = T 4 Sol. Putting x = 1 in (1 + x - 3x 2 )2163 , the required sum of
è2 ø
coefficients = (1 + 1 - 3)2163 = ( - 1)2163 = - 1
and given that T 4 = ( n - 1) + T 3
Þ T 3 + 1 = ( 6 - 1) + T 2 + 1 y Example 32. If the sum of the coefficients in the
Þ 6C 3 (3- x / 4 )3 (3 5 x / 4 )3 = 5 + 6C 2 (3- x / 4 )4 (3 5 x / 4 )2 expansion of (ax 2 - 2x + 1) 35 is equal to the sum of
Þ 20 × 3 3 x = 5 + 15 × 3 3 x / 2 the coefficients in the expansion of ( x - ay ) 35 , find the
Let 3 3x /2 = t value of a.
\ 20 t 2 = 5 + 15 t Sol. Given, sum of the coefficients in the expansion of
(ax 2 - 2x + 1)35
Þ 4t 2 - 3t - 1 = 0
450 Textbook of Algebra

= Sum of the coefficients in the expansion of ( x - ay )35 40

Putting x = y = 1, we get
Putting x = 1, we get 0 = åar
r =0
(a - 1)35 = (1 - a )35 or a 0 + a1 + a 2 + a 3 + a 4 + a 5 + ... + a 39 + a 40 = 0 …(ii)
Þ (a - 1)35 = - (a - 1)35 Putting x = - 1 in Eq. (i), we get
40
Þ 2 (a - 1)35 = 0 ( - 2)20 = å( - 1)r ar
Þ a -1 = 0 r =0

\ a =1 or a 0 - a1 + a 2 - a 3 + a 4 - a 5 + ... - a 39 + a 40 = 220 …(iii)


40 On subtracting Eq. (iii) from Eq. (ii), we get
y Example 33. If (1 + x - 2x 2 ) 20 = å ar x r , then find 2 [a1 + a 3 + a 5 + ... + a 39 ] = - 220
r =0
the value ofa1 + a 3 + a 5 + ... + a 39 . or a1 + a 3 + a 5 + ... + a 39 = - 219
40 Corollary On adding Eqs. (ii) and (iii) and then dividing by
Sol. Q (1 + x - 2x 2 )20 = åar x r …(i) 2, we get a 0 + a 2 + a 4 + ... + a 40 = 219
r =0

#L Exercise for Session 2


1. If the rth term in the expansion of (1 + x )20 has its coefficient equal to that of the (r + 4)th term, then r is
(a) 7 (b) 9 (c) 11 (d) 13
n
æ 1ö 5
2. If the fourth term in the expansion of ç px + ÷ is , then n + p is equal to
è xø 2
9 11 13 15
(a) (b) (c) (d)
2 2 2 2
n
æ 1ö 1
3. If in the expansion of ç 3 2 + 3 ÷ , the ratio of 7th term from the beginning to the 7th term from the end is ,
è 3ø 6
then n is
(a) 3 (b) 5 (c) 7 (d) 9

4. The number of integral terms in the expansion of (51/ 2 + 71/ 8 )1024 is


(a) 128 (b) 129 (c) 130 (d) 131

5. In the expansion of (7 1/ 3
+ 11 )1/ 9 6561
, the number of terms free from radicals is
(a) 715 (b) 725 (c) 730 (d) 750

6. n
If the coefficients of three consecutive terms in the expansion of (1 + x ) are 165, 330 and 462 respectively, the
value of n is
(a) 7 (b) 9 (c) 11 (d) 13

7. If the coefficients of 5th, 6th and 7th terms in the expansion of (1 + x )n are in AP, then n is equal to
(a) 7 only (b) 14 only (c) 7 or 14 (d) None of these
n
æ 1ö
8. If the middle term in the expansion of ç x 2 + ÷ is 924 x 6, the value of n is
è xø
(a) 8 (b) 12 (c) 16 (d) 20
n
æ 2 ö
9. If the sum of the binomial coefficients in the expansion of ç x 2 + 3 ÷ is 243, the term independent of x is equal to
è x ø
(a) 40 (b) 30 (c) 20 (d) 10

10. 2
In the expansion of (1 + x ) (1 + x + x ) ... (1 + x + x + ... + x 2 2n
), the sum of the coefficients is
(a) 1 (b) 2n ! (c) 2n ! + 1 (d) (2n + 1) !
Session 3
Two Important Theorems, Divisibility Problems
Two Important Theorems Now, let ( P - Q ) n = f ¢ , where 0 < f ¢ < 1
Theorem 1 If ( P + Q ) n = I + f , where I and n are Also, I + f = (P + Q )n …(i)
positive integers, n being odd and 0 £ f < 1 , then 0 £ f <1 …(ii)
show that ( I + f ) f = k n , where P - Q 2 = k > 0 and f ¢ = (P - Q )n …(iii)
P - Q < 1. and 0 < f ¢ <1 …(iv)
Proof Given, P -Q <1 \ 0 < ( P - Q )n < 1 On adding Eqs. (i) and (iii), we get
Now, let ( P - Q ) n = f ¢, where 0 < f ¢ < 1 I + f + f ¢ = (P + Q )n + (P - Q )n

Also I + f = ( P + Q )n …(i) = 2 [ n C 0 P n + n C 2 P n - 2 ( Q ) 2 + n C 4 P n - 4 ( Q ) 4 + ... ]


0 £ f <1 …(ii) = 2 (integer) = Even integer …(v)
f ¢ = ( P - Q )n …(iii) [Since, RHS contains even power of Q , so RHS is
an even integer]
and 0 < f ¢ <1 …(iv) \ LHS is also an integer.
On subtracting Eq. (iii) from Eq. (i), we get Q I is an integer.
I + f - f ¢ = ( P + Q )n - ( P - Q )n Þ f + f ¢ is also an integer.
\ f + f ¢ =1 [Q0 < ( f + f ¢ ) < 2 ]
= 2 [ n C 1 ( P ) n - 1 × Q + n C 3 ( P ) n - 3 × Q 3 + ... ]
or f ¢ =1- f
= 2 (integer) = Even integer …(v)
From Eq. (v), I = even integer - 1 = odd integer and
[Since, n is odd, RHS contains even powers
( I + f ) (1 - f ) = ( I + f ) f ¢
of P , so RHS is an even integer]
\ LHS is also an integer. = (P + Q )n (P - Q )n = (P 2 - Q )n = k n
Q I is an integer.
y Example 34. Show that the integral part of
\ (f - f ¢) is also an integer.
( 5 + 2 6 )n is odd, where n is natural number.
Þ f - f ¢ =0 [Q - 1 < ( f - f ¢ ) < 1]
or f =f¢ Sol. (5 + 2 6 )n can be written as (5 + 24 )n
From Eq. (v), I is an even integer and Now, let I + f = (5 + 24 )n …(i)
0£ f <1 …(ii)
(I + f ) f = (I + f ) f ¢ = ( P + Q )n ( P - Q )n
and let f ¢ = (5 - 24 )n …(iii)
= (P - Q 2 )n = k n
0< f ¢<1 …(iv)
Remark On adding Eqs. (i) and (iii), we get
If n is even integer, then ( P + Q) n + ( P - Q) n = I + f + f ¢
I + f + f ¢ = (5 + 24 )n + (5 - 24 )n
Since, LHS and I are integers.
\ (f + f ¢) is also an integer. I + 1 = 2p ,
Þ f + f¢ = 1 [Q0 < ( f + f ¢) < 2] " p Î N = Even integer [from theorem 2]
\ f¢ = 1- f \ I = 2p - 1 = Odd integer
Hence, ( I + f ) ( 1 - f ) = ( I + f ) f ¢ = ( P + Q) n ( P - Q) n
= ( P - Q2 ) n = k n y Example 35. Show that the integral part of
( 5 5 + 11) 2n +1 is even, where n Î N .
Theorem 2 If ( P + Q ) n = I + f , where I and n are
Sol. (5 5 + 11)2n + 1 can be written as ( 125 + 11)2n + 1
positive integers and 0 £ f < 1 , show that ( I + f )
(1 - f ) = k n , where P 2 - Q = k > 0 and P - Q < 1. Now, let I + f = ( 125 + 11)2 n + 1 …(i)

Proof Given, P - Q <1 0£ f <1 …(ii)


and let f ¢ = ( 125 - 11)2n + 1 …(iii)
\ 0 < (P - Q )n < 1
0< f ¢<1 …(iv)
452 Textbook of Algebra

On subtracting Eq. (iii) from Eq. (i), we get Now, let f ¢ = (8 - 63 )n …(iii)
I + f - f ¢ = ( 125 + 11)2 n + 1 - ( 125 - 11)2 n + 1 0< f ¢<1 …(iv)
I + 0 = 2p , " p Î N = Even integer On adding Eqs. (i) and (iii), we get
[from theorem 1] [ x ] + f + f ¢ = (8 + 63 )n + (8 - 63 )n
\ I = 2p = Even integer [ x ] + 1 = 2p , " p Î N = Even integer
[from theorem 2]
y Example 36. Let R = (6 6 + 14 ) 2n + 1 and f = R - [R ], \ [ x ] = 2p - 1 = Odd integer
where [ × ] denotes the greatest integer function. Find i.e., Integral part of x = Odd integer
the value of Rf , n Î N . Q f + f ¢=1 Þ 1- f = f ¢ …(v)
Sol. (6 6 + 14 )2 n + 1 can be written as ( 216 + 14 )2 n + 1 and LHS = x - x 2 + x [ x ] = x - x ( x - [ x ]) = x - xf
given that f = R - [ R ]
[Q x = [ x ] + f ]
and R = (6 6 + 14 )2 n + 1= ( 216 + 14 )2 n + 1 = x (1 - f ) = x f ¢ [from Eq.(v)]
\ [ R ] + f = ( 216 + 14 )2 n + 1 …(i) = (8 + 63 )n (8 - 63 )n [from Eqs.(i) and (iii)]
0£ f <1 …(ii)
= (64 - 63)n = (1)n = 1 = RHS
Let f ¢ = ( 216 - 14 )2 n + 1 …(iii)
0< f ¢<1 …(iv) Remark
On subtracting Eq. (iii) from Eq. (i), we get Sometimes, students find it difficult to decide whether a problem
2n + 1 2n + 1
is on addition or subtraction. Now, if x = [ x ] + f and 0 < f ¢ < 1
[ R ] + f - f ¢ = ( 216 + 14 ) - ( 216 - 14 ) and if [ x ] + f + f ¢= Integer. Then, addition and if
[ R ] + 0 = 2p , " p Î N = Even integer [from theorem 1] [ x ] + f - f ¢ = Integer, the subtraction and values of (f + f ¢) and
(f - f ¢) are 1and 0, respectively.
\ f - f ¢ = 0 or f = f ¢
Now, Rf = Rf ¢ = ( 216 + 14 )2 n + 1 ( 216 - 14 )2 n + 1
= (216 - 196)2 n + 1 = (20)2 n + 1 Divisibility Problems
y Example 37. If (7 + 4 3 )n = s + t, where n and s are Type I
positive integers and t is a proper fraction, show that (i) ( x n - a n ) is divisible by ( x - a ), " n Î N .
(1 - t ) (s + t ) = 1. (ii) ( x n + a n ) is divisible by ( x + a ), " n ÎOnly odd
Sol. (7 + 4 3 )n can be written as (7 + 48 )n natural numbers.
\ s + t = (7 + 48 )n …(i)
y Example 39. Show that
0<t <1 …(ii)
Now, let t ¢ = (7 - 48 )n …(iii) 19921998 - 19551998 - 1938 1998 + 19011998 is divisible by 1998.
0<t¢<1 …(iv) Sol. Here, n = 1998 (Even)
On adding Eqs. (i) and (iii), we get \ Only result (i) applicable.
s + t + t ¢ = (7 + 48 )n + (7 - 48 )n Let P = 19921998 - 19551998 - 19381998 + 19011998
s + 1 = 2p , " p Î N = Even integer [from theorem 2] = (19921998 - 19551998 ) - (19381998 - 19011998 )
\ t + t ¢ = 1 or 1 - t = t ¢ divisible by (1992 - 1955 ) divisible by (1938 - 1901)
Then, (1 - t ) (s + t ) = t ¢ (s + t ) = (7 - 48 )n (7 + 48 )n i. e. 37 i. e. 37
[from Eqs. (i) and (iii)] \ P is divisible by 37.
= ( 49 - 48) = (1)n = 1
n
Also, P = (19921998 - 19381998 ) - (19551998 - 19011998 )
divisible by (1992 - 1938 ) divisible by (1955 - 1901)
y Example 38. If x = (8 + 3 7 )n , where n is a natural i.e., 54 i.e., 54
number, prove that the integral part of x is an odd \ P is also divisible by 54.
integer and also show that x - x 2 + x [ x ] = 1 , where [ × ] Hence, P is divisible by 37 ´ 54, i.e., 1998.
denotes the greatest integer function. y Example 40. Prove that 2222 5555 + 5555 2222 is
Sol. (8 + 3 7 )n can be written as (8 + 63 )n
divisible by 7.
\ x = [x ] + f
Sol. We have, 22225555 + 55552222
or [ x ] + f = (8 + 63 )n …(i)
0£ f <1 …(ii) = (22225555 + 4 5555 ) + (55552222 - 4 2222 ) - ( 4 5555 - 4 2222 ) …(i)
Chap 06 Binomial Theorem 453

The number (22225555 + 4 5555 ) is divisible by 2222 + 4


= 2226 = 7 ´ 318, which is divisible by 7 and the number
How to Find Remainder
(55552222 - 4 2222 ) is divisible by by Using Binomial Theorem
5555 - 4 = 5551 = 7 ´ 793, which is divisible by 7 and the If a, p, n and r are positive integers, then to find the
number remainder when a pn + r is divided by b, we adjust power of
( 4 5555 - 4 2222 ) = 4 2222 ( 4 3333 - 1) = 4 2222 (641111 - 11111 ) is a to a pn + r which is very close to b, say with difference 1
divisible by 64 - 1 = 63 = 7 ´ 9, which is divisible by 7. i.e., b ± 1. Also, the remainder is always positive. When
Therefore, each brackets of Eq. (i) are divisible by 7. Hence, number of the type 5n - 2 is divided by 5, then we have
22225555 + 55552222 is divisible by 7. 5 ) 5n - 2 (n
5n
Type II To show that an Expression -
-2
is Divisible by An Integer We can write - 2 = - 2 - 3 + 3 = - 5 + 3
Solution Process 5n - 2 5n - 5 + 3 3
or = =n -1+
(i) If a, p, n and r are positive integers, first of all write 5 5 5
a pn + r = a pn × a r = (a p ) n × a r Hence, the remainder is 3.
(ii) If we will show that the given expression is divisible y Example 43. If 7 103 is divided by 25, find the
by c. Then, expression a p = {1 + (a p - 1)}, if some
remainder.
power of (a p - 1) has c as a factor. Soln. We have, 7103 = 7 × 7102 = 7 × (7 2 )51 = 7 (49)51 = 7 (50 - 1)51
or a p = {2 + (a p - 2 )}, if some power of (a p - 2 ) has c = 7 [(50)51 - 51C 1 (50)50 + 51C 2 (50)49 - ... - 1]
as a factor.
= 7 [(50)51 - 51C 1 (50)50 + 51C 2 (50)49 - ... + 51C 50 (50)]
or a p = {3 + (a p - 3 )}, if some power of (a p - 3 ) has c
- 7 - 18 + 18
as a factor.
M M M M M M = 7 [ 50 ((50)50 - 51 C 1(50)49 + 51 C 2 (50)48 -...+ 51C 50 )] - 25 + 18
or a p = {k + (a p - k )}, if some power of (a p - k ) has c = 7 [ 50k ] - 25 + 18 , where k is an integer.
= 25 [14k - 1] + 18 = 25p + 18 [where p is an integer]
as a factor.
7103 18
Now, = p + . Hence, the remainder is 18.
y Example 41. If n is any positive integer, show 25 25
that 2 3n + 3 - 7n - 8 is divisible by 49. 5 ...
5

Sol. Given expression y Example 44. Find the remainder, when 5 5


= 23n + 3 - 7n - 8 = 23n × 23 - 7n - 8 ( 24 times 5) is divided by 24.
5
= 8n × 8 - 7n - 8 = 8 (1 + 7 )n - 7n - 8 Sol. Here, 555
5 ...
(23 times 5) is an odd natural number.
= 8 (1 + n C 1 × 7 + n C 2 × 7 2 + ... + n C n × 7n ) - 7n - 8 5 5 ...
Let 5 (23 times 5) = 2m + 1
= 8 + 56n + 8 (n C 2 × 7 2 + ... + n C n × 7n ) - 7n - 8 5
5...
= 49n + 8 (n C 2 × 7 2 + ... + n C n × 7n ) Now, let x = 55 (24 times 5) = 52 m + 1 = 5 × 52 m , where m is
= 49 {n + 8 (n C 2 + ... + n C n × 7n - 2 )} a natural number.
\ x = 5 × (52 )m = 5 ( 24 + 1)m
Hence, 23n + 3 - 7n - 8 is divisible by 49. = 5 [ mC 0 (24 )m + mC 1 (24 )m - 1 + ... + mC m - 1 (24 ) + 1]
y Example 42. If 10n divides the number 101100 - 1, find = 5 (24k + 1) = 24 (5k ) + 5
x 5
the greatest value of n. \ = 5k +
24 24
Sol. We have, 101100 - 1 = (1 + 100)100 - 1 Hence, the remainder is 5.
100 100
=1+ C 1 × 100 + C 2 × 1002 + ... + 100
C 100 100100 - 1 32
= 100
C 1 × 100 + 100 2
C 2 × 100 + K + 100
C 100 × 10100
y Example 45. If 7 divides 32 32 , then find the remainder.
= (100) (100) + 100
C 2 × 1002 + ... + 100
C 100 × 100100 Solution. We have, 32 = 25
= (100)2 [1 + 100
C 2 + ... + 10098 ] \ 3232 = (25 )32 = 2160 = (3 - 1)160
160
= 1002 k, where k is a positive integer = C 0 (3)160 - 160
C 1 (3)159 + ... - 160
C 159 (3) + 1
159 160 158 160
Therefore, 101100 - 1 is divisible by 1002 i.e., 104 . = 3 (3 - C 1 ( 3) + ... - C 159 ) + 1
+
\ n=4 = 3m + 1, m Î I
454 Textbook of Algebra

32
Now, 3232 = 32 3m + 1 = 2 5 (3m + 1) = 215m + 5 (i) For last digit
2 3 ( 5m + 1) 5m + 1 5m + 1
= 2 ×2 = 4 ( 8) = 4 ( 7 + 1) 17 256 = 290[ 128 C 0 (290)127 - 128
C 1(290)126
5m + 1
= 4[ C 0 (7 )5m + 1 + 5m + 1
C 1 (7 )5m + 5m + 1C 2 (7 )5m - 1 + 128
C 2 (290)125 - ... -128 C 127 (1)] + 1
+ ... + 5m + 1C 5m (7 ) + 1]
= 290 (k ) + 1, where k is an integer.
= 4 [ 7 ( 5m + 1C 0 (7 )5m + 5m + 1C 1(7 )5m - 1 +... + 5m + 1C 5m ) + 1]
= 4 [7k + 1] , where k is positive integer = 28k + 4 \ Last digit = 0 + 1 = 1
32 (ii) For last two digits,
3232 4
\ = 4k +
7 7 17 256 = (290)2 [ 128C 0 (290)126 - 128
C 1 (290)125 +
Hence, the remainder is 4. 128
C 2 (290)124 - ... + 128
C 126 (1)] - 128
C 127 (290) + 1
128
= 100 m - C 127 (290) + 1, where m is an integer.
How to Find Last Digit, Last Two Digits, 128
= 100 m - C 1 (290) + 1 = 100 m - 128 ´ 290 + 1
Last Three Digits, ... and so on.
= 100 m - 128 ´ (300 - 10) + 1
Ifa, p, n andr are positive integers, thena pn + r is adjust of the
= 100 (m - 384 ) + 1281
form(10k ± 1) m , wherek andm are positive integers. For last = 100 n + 1281, where n is an integer.
digit, take 10 common. For last two digits, take 100 common, \ Last two digits = 00 + 81 = 81
for last three digits, take 1000 common , ... and so on.
(iii) For last three digits,
i.e. (10k ± 1) m = (10k ) m + m C 1 (10k ) m - 1 ( ± 1)
17 256 = (290)3 [ 128C 0 (290)125 - 128
C 1 (290)124
+ m C 2 (10k ) m - 2 ( ± 1) 2 + ... +
128
m
C m - 2 (10k ) 2 ( ± 1) m -2 + m C m - 1 (10k ) ( ± 1) m - 1 + ( ± 1) m + C 2 (290)123 - ... - 128
C 125 (1)]
128
+ C 126 (290)2 - 128
C 127 (290) + 1
For last digit = 10 l + ( ± 1) m
128 2 128
= 1000 m + C 126 (290) - C 127 (290) + 1
For last two digits = 100 m + m C m - 1 (10k ) ( ± 1) m - 1 + ( ± 1) m
where, m is an integer
For last three digits = 1000 n +m C m -2 (10k ) 2 ( ±1) m -2 +m C m -1 128
= 1000 m + C 2 (290)2 - 128
C 1 (290) + 1
m -1 m
(10k ) ( ± 1) + ( ± 1) and so on where l, m, n Î I.
(128) (127 )
= 1000 m + (290)2 - 128 ´ 290 + 1
y Example 46. Find the last two digits of 3 400
. 2
400 2 200 200 200
= 1000 m + (128) (127 ) (290) (145) - (128) (290) + 1
Sol. We have, 3 = (3 ) = (9) = (10 - 1)
= 1000 m + (128) (290) (127 ´ 145 - 1) + 1
200 200 199
= (10) - C 1 (10) + 200C 2 (10)198 - 200
C 3 (10)197
= 1000 m + (128) (290) (18414 ) + 1
200
+ ... + C 198 (10)2 - 200
C 199 (10) + 1 = 1000 m + 683527680 + 1
200
= 100 m - C 199 (10) + 1, where m Î I = 1000 m + 683527000 + 680 + 1
= 100 m - 200
C 1 (10) + 1 = 100 m - 2000 + 1 = 1000 (m + 683527 ) + 681
= 100 (m - 20) + 1 = 100 p + 1, where p is an integer. \ Last three digits = 000 + 681 = 681

Hence, the last two digits of 3400 is 00 + 1 = 01.


Two Important Results
y Example 47. If the number is 17 256 , find the n
æ 1ö
(i) last digit. (ii) last two digits. (i) 2 £ ç 1 + ÷ < 3, n ³ 1, n Î N
è nø n n
(iii) last three digits of 17 256 . ænö ænö
(ii) If n > 6, then ç ÷ < n ! < ç ÷
è3ø è2ø
Sol. Since, 17 256 = (17 2 )128 = (289 )128 = (290 - 1)128

\ 17 256 = 128C 0 (290)128 - 128


C 1 (290)127 + 128
C 2 (290)126
y Example 48. Find the positive integer just greater
128
than (1 + 00001
. )10000 .
- C 3 (290)125 + ... - 128
C 125 (290)3 + 128
C 126 (290)2 10000
æ 1 ö
128 Sol. (1 + 0.0001)10000 = ç1 + ÷
- C 127 (290) + 1 è 10000 ø
Chap 06 Binomial Theorem 455

n
æ 1ö y Example 50. Find the greater number in 300! and
We know that, 2 £ ç1 + ÷ < 3, n ³ 1, n Î N [Result (i)]
è nø
300 300 .
Hence, positive integer just greater than (1 + 0.0001)10000
is 3.
Sol. Since, (100)150 > 3150
y Example 49. Find the greater number is 100100 and Þ (100)150 × (100)150 > 3150 × (100)150
( 300)! . Þ (100)300 > (300)150
n
æn ö (100)300 > 300300
Sol. Using Result (ii), We know that, ç ÷ < n ! or …(i)
è3ø n
Putting n = 300, we get æn ö
Using result (ii), ç ÷ < n !
(100)300 < (300) ! …(i) è3ø

But (100) 100


< (100) 300
…(ii) Putting n = 300, we get (100)300 < 300 ! …(ii)
From Eqs. (i) and (ii), we get From Eqs. (i) and (ii), we get
(100)100 < (100)300 < (300) ! 300 300 < (100)300 < 300 !
Þ (100)100 < (300) ! Þ 300 300 < 300 !
Hence, the greater number is (300) ! . Hence, the greater number is 300 !.

#L Exercise for Session 3


1. If x = (7 + 4 3 )2n = [ x ] + f , where n Î N and 0 £ f < 1, then x (1 - f ) is equal to
(a) 1 (b) 0 (c) - 1 (d) even integer

2. If (5 + 2 6 )n = I + f ; n, I Î N and 0 £ f < 1, then I equals


1 1 1 1
(a) -f (b) -f (c) -f (d) +f
f 1+ f 1- f 1+ f

1- f 2
3. If n > 0 is an odd integer and x = ( 2 + 1)n, f = x - [ x ], then is
f
(a) an irrational number (b) a non-integer rational number (c) an odd number (d) an even number

4. Integral part of ( 2 + 1)6 is


(a) 196 (b) 197 (c) 198 (d) 199

5. (103) 86
- (86) 103
is divisible by
(a) 7 (b) 13 (c) 17 (d) 23
78
2
6. Fractional part of is
31
2 4 8 16
(a) (b) (c) (d)
31 31 31 31

7. The unit digit of 171983 + 111983 - 71983 is


(a) 1 (b) 2 (c) 3 (d) 0

8. The last two digits of the number (23) 14


are
(a) 01 (b) 03 (c) 09 (d) 27

9. The last four digits of the number 3100 are


(a) 2001 (b) 3211 (c) 1231 (d) 0001

10. The remainder when 2323 is divided by 53 is


(a) 17 (b) 21 (c) 30 (d) 47
Session 4
Use of Complex Numbers in Binomial Theorem, Multinomial
Theorem, Use of Differentiation, Use of Integration, Binomial
Inside Binomial, Sum of the Series

Use of Complex Numbers From Eqs. (i) and (ii), we get

in Binomial Theorem (C 0 - C 2 + C 4 - ...) + i (C 1 - C 3 + C 5 - ...)


æ np ö
= 2 n / 2 cos ç ÷ + i ×2 n / 2 sin ç ÷
æ np ö
If q Î R, n Î N and i = - 1, then è 4 ø è 4 ø
On comparing real and imaginary parts, we get
(cos q + i sin q ) n = n C 0 (cos q ) n - 0 (i sin q ) 0
æ np ö
+ n C 1 (cos q ) n - 1 (i sin q ) 1 C 0 - C 2 + C 4 - ... = 2 n / 2 cos ç ÷ [part (i)]
è 4 ø
n n -2 2 n n-3
+ C 2 (cos q ) (i sin q ) + C 3 (cos q )
æ np ö
3 C 1 - C 3 + C 5 - ... = 2 n / 2 sin ç ÷ [part (ii)]
(i sin q ) + ... è 4 ø
or cos nq + i sin nq = cos n q + i × n C 1 (cos q ) n - 1 sin q We have, (1 + x )n = C 0 + C 1x + C 2 x 2 + C 3 x 3
+ C4 x 4

- n C 2 (cos q ) n - 2 sin2 q - i × n C 3 (cos q ) n - 3 sin 3 q +... + C5 x 5


+ C6 x 6
+ ...
On comparing real and imaginary parts, we get 2
Putting x = 1, w, w (cube roots of unity) and adding, we get
cos nq = cos n q - n C 2 (cos q ) n - 2 sin2 q 3 ( C 0 + C 3 + C 6 + ...) = 2n + (1 + w)n + (1 + w2 )n
n n-4 4
- C 4 (cos q ) sin q - ... = 2n + ( - w2 )n + ( - w)n = 2n + ( - 1)n ( w2n + wn )
n n -1 n n-3 3
andsin nq = C 1 (cos q ) sin q - C 3 (cos q ) sin q ìï 4 pin 2 pin ü
n n ï
= 2 + ( -1) íe 3 +e 3 ý
+ n C 5 (cos q ) n - 5 sin5 q - ... ïî ïþ
y Example 51. If (1 + x )n = C 0 + C 1 x + C 2 x 2
æ np ö
= 2n + ( - 1)n × e npi × 2 cos ç ÷
+ C3 x 3 + C4 x 4
+ ... , find the values of è 3 ø
æ np ö
(i) C 0 - C 2 + C 4 - C 6 + ... = 2n + ( - 1)n × ( - 1)n × 2 cos ç ÷
è 3 ø
(ii) C 1 - C 3 + C 5 - C 7 + ...
æ np ö æ np ö
(iii) C 0 + C 3 + C 6 + ... = 2 n + ( - 1)2 n × 2 cos ç ÷ = 2n + 2 cos ç ÷
è 3 ø è 3 ø
Sol. Q(1 + x )n = C 0 + C 1 x + C 2 x 2 + C 3 x 3 + C 4 x 4
1ì n æ np öü
\ C 0 + C 3 + C 6 + ... = í2 + 2 cos ç ÷ý
+ C 5 x 5 + ... 3î è 3 øþ
Putting x = i , where i = - 1, then
y Example 52. Find the value of
(1 + i )n = C 0 + C 1 i + C 2 i 2 + C 3 i 3 + C 4 i 4 + C 5 i 5 + ... 4n
C 0 + 4n C 4 + 4n C 8 + ... + 4n
C 4n .
= (C 0 - C 2 + C 4 - ...) + i (C 1 - C 3 + C 5 - ...) …(i)
Sol. Q 4 - 0 = 8 - 4 = ... = 4
n
é æ 1 i öù
Also, (1 + i )n = ê 2 ç + ÷ \ Four roots of unity (1)1/ 4 are 1, - 1, i , - i , we have
ë è 2 2 ø úû
n
(1 + x )4n = 4n
C0 + 4n
C 1x + 4n
C 2x 2 + 4n
C 3 x 3 + ...
æ p pö
= 2n / 2 ç cos + i sin ÷ Putting x = 1, - 1, i , - i and then adding, we get
è 4 4ø
4 ( 4n C 0 + 4n
C 4 + 4n C 8 + ...) = 24n + 0 + (1 + i )4n + (1 - i )4n
n/2 æ np np ö
=2 ç cos + i sin ÷ …(ii) = 24n + (2i )2 n + ( - 2i )2 n
è 4 4 ø
Chap 06 Binomial Theorem 457

= 24n + 22 n ( - 1)n + 22 n ( - 1)n y +z =5


=2 4n
+ ( - 1) × 2 n 2n + 1 On adding all, we get 2 ( x + y + z ) = 12
\ x +y +z =6
\ 4n
C0 + 4n
C4 + 4n
C 8 + ... = 24n - 2 + ( - 1)n × 22n - 1
Then, x = 1, y = 3, z = 2
Remark Therefore, the coefficient of a 3b 4c 5 in the expansion of
If ( 1 + x ) n = C0 + C1 x + C2 x 2 + C3 x 3+ K + Cn x n , then (bc + ca + ab )6 or the coefficient of (ab )1 (bc )3 (ca )2 in the
np 6!
(i) C0 + C4 + C8 + C12 + K =
1ì n -1
+ 2n / 2cos æç ö÷ üý expansion of (bc + ca + ab )6 is , i.e. 60.
í2 è 4 øþ 1!3!2!

1ì n -1 np
+ 2n / 2sin æç ö÷ üý
(ii) C1 + C5 + C9 + C13 + K =
Aliter
í2 è 4 øþ
2î Coefficient of a 3b 4c 5 in the expansion of (bc + ca + ab )6
np
(iii) C0 + C6 + C12 + K = ìí2n - 1 cos æç ö÷ + 3n / 2cos æç ö÷ üý
1 np
3î è4 ø è 6 øþ = Coefficient of a 3b 4c 5 in the
6
æ1 1 1ö
expansion of (abc )6 ç + + ÷
èa b c ø
Multinomial Theorem 3
æ1ö æ1ö æ1ö
2 1
= Coefficient of ç ÷ ç ÷ ç ÷ in the expansion of
If n is a positive integer and x 1 , x 2 , x 3 , ... , x k Î C , then èa ø èb ø èc ø
( x 1 + x 2 + x 3 + ... + x k ) n = S
n! æ1 1 1ö 6!
6

(a 1 !)(a 2 !)(a 3 !) ... (a k !) ç + + ÷ is = 60


èa b c ø 3 ! 2 ! 1 !
a a a
x 1 1 x 2 2 x 3 3 ... x k a k
where, a 1 , a 2 , a 3 , ... , a k are all non-negative integers
such that a 1 + a 2 + a 3 + ... + a k = n.
Number of Distinct or Dissimilar
Terms in the Multinomial Expansion
Remark Statement The number of distinct or dissimilar terms in
The coefficient of x1a1 × x 2a 2 × x 3a 3 ... x k a k in the expansion of
the multinomial expansion of ( x 1 + x 2 + x 3 + ... + x k ) n
( x1 + x 2 + x 3 + ... + x k ) n is S
n!
.
( a1 !) ( a2 !) ( a3 !) ... ( ak !) n + k -1
is Ck - 1 .
In Particular
Proof We have, ( x 1 + x 2 + x 3 + ... + x k ) n
(i) (a + b + c ) = S
n!
n
a a b b c g such that
=S
n! a a a a
(a !) (b !) ( g !) x 1 1 x 2 2 x 3 3 ... x k k
(a 1 !) (a 2 !) (a 3 !) ... (a k !)
a +b + g =n
where, a 1 , a 2 , a 3 , ... , a k are non-negative integers such
(ii) (a + b + c + d ) n = S
n!
a a bb c g d d that
(a !) (b !) ( g !) ( d !)
such that a + b + g + d = n a 1 + a 2 + a 3 + ... + a k = n ...(i)
Here, the number of terms in the expansion of
y Example 53. Find the coefficient of a 4 b 3 c 2 d in (x 1 + x 2 + x 3 + K + x k )n
the expansion of (a - b + c - d ) 10. = The number of non-negative integral solutions of the Eq. (i)
4 3 2
Sol. The coefficient of a b c d in the expansion of = n + k - 1C k - 1
10 !
(a - b + c - d )10 is ( - 1)4 = 12600 y Example 55. Find the total number of distinct or
4 !3!2!1!
dissimilar terms in the expansion of
[powers of b and d are 3 and 1 \( -1)3 ( -1) ]
( x + y + z + w )n , n Î N .
y Example 54. Find the coefficient of a 3b 4 c 5
in the Sol. The total number of distinct or dissimilar terms in the
6 expansion of ( x + y + z + w )n is
expansion of (bc + ca + ab ) .
( n + 3) ( n + 2) ( n + 1)
Sol. In this case, write a 3b 4c 5
= (ab )x (bc )y (ca )z say = n + 4 - 1C 4 - 1 = n + 3C 3 =
1×2×3
\ a 3b 4c 5
= a z + x ×b x +y
×c y +z
( n + 1) ( n + 2) ( n + 3)
=
Þ z + x = 3, x + y = 4 6
458 Textbook of Algebra

I. Aliter
We know that, ( x + y + z + w )n = {( x + y ) + (z + w )}n
Coefficient of x r in
= ( x + y )n + n C 1 ( x + y )n - 1 (z + w ) Multinomial Expansion
+ n C 2 ( x + y )n - 2 (z + w )2 + ... + n C n (z + w )n If n is a positive integer and a 1 , a 2 , a 3 , ... , a k Î C , then
\ Number of terms in RHS coefficient of x r in the expansion of (a 1 + a 2 x + a 3 x 2
= (n + 1) + n × 2 + (n - 1) × 3 + ... + 1 × (n + 1) k -1 n
+ ... + a k x ) , is
n
= S
r =0
( n - r + 1) ( r + 1)
S n! a a2 a a
a 1 1 a 2 a 3 3 ... a k k
n n n n (a 1 !) (a 2 !) (a 3 !) ... (a k !)
= S (n + 1) + nr - r 2 = (n + 1)r S= 0 1 + n r S= 0 r - r S= 0 r 2
r =0 where, a 1 , a 2 , a 3 , ... , a k are non-negative integers such
n ( n + 1) n ( n + 1) ( 2 n + 1) that a 1 + a 2 + a 3 + ... + a k = n
= ( n + 1) × ( n + 1) + n × -
2 6 and a 2 + 2 a 3 + 3 a 4 + ... + (k - 1) a k = r
( n + 1) ( n + 2) ( n + 3)
= y Example 57. Find the coefficient of x 7 in the
6
II. Aliter expansion of (1 + 3x - 2x 3 )10 .
( x + y + z + w )n = S
n!
x n1 y n 2 z n 3 w n 4 Sol. Coefficient of x 7 in the expansion of (1 + 3x - 2x 3 )10 is
n1 ! n 2 ! n 3 ! n 4 !
= S 10!
(1)a (3)b ( -2)g
where, n1, n 2 , n 3 ,n 4 are non-negative integers subject to the a ! b! g !
condition n1 + n 2 + n 3 + n 4 = n
where, a + b + g = 10 andb + 3g = 7
Hence, number of the distinct terms
The possible values of a , b and g are given below
= Coefficient of x n in ( x 0 + x 1 + x 2 + ... + x n )4
4 a b g
æ1 - x n + 1 ö
= Coefficient of x n in ç ÷ 3 7 0
è 1- x ø
5 4 1
= Coefficient of x n in (1 - x n + 1 )4 (1 - x )- 4
7 1 2
= Coefficient of x n in (1 - x )- 4 [Q x n + 1 > x n ]
7
( n + 3) ( n + 2) ( n + 1) \ Coefficient of x
= n + 3C n = n + 3C 3 =
6 10 ! 10 !
= ( 1) 3 ( 3) 7 ( - 2) 0 + (1)5 (3)4 ( - 2)1
3!7 !0! 5! 4 !1!
Greatest Coefficient in +
10 !
(1)7 (3)1 ( - 2)2
7 !1!2!
Multinomial Expansion = 262440 - 204120 + 4320 = 62640
The greatest coefficient in the expansion of
n!
( x 1 + x 2 + x 3 + ... + x k ) n is
(q !) k -r
((q + 1) !) r
, where q is
Use of Differentiation
the quotient and r is the remainder when n is divided by k i.e. This method applied only when the numericals occur as
k ) n (q the product of the binomial coefficients, if
r (1 + x ) n = C 0 + C 1 x + C 2 x 2 + C 3 x 3 + ... + C n x n
y Example 56. Find the greatest coefficient in the
expansion of (a + b + c + d )15 .
Solution Process
(i) If last term of the series leaving the plus or minus
Sol.Here, n = 15 and k = 4 [Qa, b, c , d are four terms] sign is m, then divide m by n. If q is the quotient and
4 ) 15 (3 r is the remainder.
12 i.e. m = nq + r or n ) m (q
3 nq
\ q = 3 and r = 3 r
15 ! q
Hence, greatest coefficient = Then, replace x by x in the given series and
(3!)1 ( 4 !) 3 multiplying both sides of the expression by x r .
Chap 06 Binomial Theorem 459

(ii) After this, differentiate both sides w.r.t. x and put y Example 59. If (1 + x )n = C 0 + C 1 x + C 2 x 2

x = 1 or - 1 or i(i = -1 ), etc. According to the given


+ ... + C n x n , prove that
series.
(iii) If product of two numericals (or square of numericals) C 0 + 2 C 1 + 3 C 2 + ... + (n + 1) C n = (n + 2) 2n - 1 .
or three numericals (or cube of numericals), then Sol. Here, last term of C 0 + 2 C 1 + 3C 2 + ... + (n + 1) C n is
differentiate twice or thrice. (n + 1) C n i.e., (n + 1) and last term with positive sign.

y Example 58. If and n + 1 = n ×1 + 1


or n )n + 1 (1
(1 + x ) n = C 0 + C 1 x + C 2 x 2 + ... + C n x n , prove that
-n
C 1 + 2C 2 + 3C 3 + ... + nC n = n × 2n -1 . Here, q = 1 and r = 1 1
Sol. Here, last term of C 1 + 2C 2 + 3C 3 + ... + n C n is nC n i.e., n The given series is
and last term with positive sign. (1 + x )n = C 0 + C 1 x + C 2 x 2
+ ... + C n x n
Then, n = n × 1 + 0 or n ) n (1
Now, replacing x by x 1 and multiplying both sides by x, we
n
get
0
x (1 + x )n = C 0 x + C 1 x 2
+ C2 x 3
+ ... + C n x n + 1
Here, q = 1 and r = 0
Differentiating both sides w.r.t. x, we get
Then, the given series is
2 3 x × n (1 + x )n -1 + (1 + x )n × 1 = C 0 + 2C 1 x + 3C 2 x 2
(1 + x )n = C 0 + C 1 x + C 2 x + C3 x + ... + C n x n
Differentiating both sides w.r.t. x , we get + ... + (n + 1) C n x n
n (1 + x )n - 1 = 0 + C 1 + 2C 2 x + 3C 3 x 2 + ... + nC n x n - 1 Putting x = 1, we get
Putting x = 1, we get n (2)n - 1 + 2n = C 0 + 2C 1 + 3C 2 + K + (n + 1) C n
n × 2n - 1 = C 1 + 2C 2 + 3C 3 + ... + nC n or C 0 + 2C 1 + 3C 2 + ... + (n + 1) C n = (n + 2) 2n - 1
or C 1 + 2C 2 + 3C 3 + ... + nC n = n × 2n - 1 I. Aliter
I. Aliter LHS = C 0 + 2 C 1 + 3 C 2 + ... + (n + 1)C n
C 1 + 2C 2 + 3C 3 + ... + n C n = C 0 + (1 + 1) C 1 + (1 + 2) C 2 + ... + (1 + n ) C n
n ( n - 1) n ( n - 1) ( n - 2) = (C 0 + C 1 + C 2 + ... + C n ) + (C 1 + 2C 2 + ... + n C n )
= n +2 × +3× + ... + n × 1
1 ×2 1 ×2 ×3 [use example 58]
ì ( n - 1) ( n - 2) ü n n -1 n -1
= n í1 + (n - 1) + + ... + 1ý = 2 + n ×2 = ( n + 2) 2 = RHS
î 1 ×2 þ
II. Aliter
Let n - 1 = N , then
LHS = C 0 + 2 C 1 + 3 C 2 + ... + (n + 1) C n
ì N ( N - 1) ü n +1 n +1
LHS = (1 + N ) í1 + N + + ... + 1ý
î 1 ×2 þ = S r × nC r - 1 =rS= 1
r =1
( r - 1 + 1) × n C r -1
N N N
= (1 + N ) {1 + C 1 + C 2 + ... + C N } n +1

= (1 + N ) 2 N = n × 2n - 1 = RHS = S
r =1
( r - 1) × n C r -1 + nC r -1

II. Aliter n +1 n +1
n
= S n × n - 1C r - 2 + rS= 1 n
Cr
LHS = C 1 + 2 C 2 + 3 C 3 + ... + n C n = S r × nC r
r =1
r =1
-1

é n n n -1 ù
n
é n ù êQ C r = . Cr - 2ú
= S r × × n - 1C r
n n n -1 -1
-1 êëQ C r = r × Cr - 1ú ë r -1 û
r =1 r û
n -1 n -1 n -1 n -1
n = n (0 + C0 + C1 + C 2 + ... + Cn - 1)
=n S n - 1C r -1
r =1 + (n C 0 + n C 1 + n C 2 + ... + n C n )
n -1 n -1
=n( C0 + C 1 + n -1C 2 + ... + n -1
C n - 1] = n × 2n - 1 + 2n = (n + 2) × 2n - 1 = RHS
n -1
= n ×2 = RHS
460 Textbook of Algebra

y Example 60. If (1 + x )n = C 0 + C 1 x + C 2 x 2
y Example 61. If (1 + x )n = C 0 + C 1 x + C 2 x 2

+ ... + C n x n , prove that + ... + C n x n , prove that + 3 2 × C 3 + ... + n 2 × C n


C 0 + 3C 1 + 5C 2 + ... + (2n + 1) C n = (n + 1) 2n . 1 2 × C 1 + 2 2 × C 2 = n (n + 1) × 2n -2 .
Sol. Here, last term of C 0 + 3C 1 + 5C 2 + ... + (2n + 1) C n is Sol. Here, last term of 12 × C 1 + 22 × C 2 + 32 × C 3 + ... + n 2 × C n is
(2n + 1) C n i.e., (2n + 1) and last term with positive sign. n 2 × C n i.e., n 2 . Linear factors of n 2 are n and n; [start
Then, 2n + 1 = n × 2 + 1 always with greater factor] and last term with positive
sign.
or n ) 2n + 1(2
and n = n × 1 + 0 or n ) n (1
- 2n
1 -n
Here, q = 2 and r = 1 0
The given series is Here, q = 1 and r = 0
2 Then, the given series is
(1 + x )n = C 0 + C 1x + C 2 x + ... + C n x n
(1 + x )n = C 0 + C 1x + C 2 x 2 + C 3 x 3 + ... + C n x n
Now, replacing x by x 2 , we get
On differentiating both sides w.r.t. x, we get
(1 + x 2 )n = C 0 + C 1x 2
+ C2 x 4
+ ... + C n x 2n
nx (1 + x )n - 1 = C 1 + 2C 2 x + 3C 3 x 2
+ ... + n C n x n - 1 …(i)
On multiplying both sides by x 1, we get
and in last term, numerical is n C n i.e., n and power of
2n +1
x (1 + x 2 )n = C 0 x + C 1x 3 + C 2 x 5 + ... + C n x (1 + x ) is n - 1.
On differentiating both sides w.r.t. x, we get Then, n = (n - 1) × 1 + 1 or n - 1) n (1
x × n (1 + x 2 )n - 1 × 2x + (1 + x 2 )n × 1 = C 0 + 3C 1 x 2
+ 5C 2 x 4 n -1
2n -+
+ ... + (2n + 1) C n x
1
Putting x = 1, we get
Here, q = 1 and r = 1
n × 2n - 1 × 2 + 2n = C 0 + 3C 1 + 5C 2 + ... + (2n + 1) C n Now, multiplying both sides by x in Eq. (i), then
or C 0 + 3C 1 + 5C 2 + ... + (2n + 1) C n = (n + 1) 2n nx (1 + x )n -1 = C 1 x + 2C 2 x 2 + 3C 3 x 3 + ... + n C n x n
I. Aliter Differentiating on both sides w.r.t. x, we get
LHS = C 0 + 3 C 1 + 5 C 2 + ... + (2n + 1) C n n { x × (n - 1) ( 1 + x )n - 2 + (1 + x )n - 1 × 1}
= C 0 + (1 + 2) C 1 + (1 + 4 ) C 2 + ... + (1 + 2n ) C n = C 1 × 1 + 2 2 C 2 x + 32 C 3 x 2
+ ... + n 2 C n x n -1
= (C 0 + C 1 + C 2 + ... + C n ) + 2 (C 1 + 2C 2 + ... + n C n ) Putting x =1, we get
= 2n + 2 × n × 2n - 1 = 2n + n × 2n [from Illusration 58] n {1 × (n - 1) × 2n - 2 + 2n -1 } = 12 × C 1 + 22 × C 2 + 32 × C 3
n
= (n + 1) 2 = RHS + ... + n 2 × C n
II. Aliter or 12 × C 1 + 22 × C 2 + 32 × C 3 + ... + n 2 × C n = n (n + 1) 2n - 2
LHS = C 0 + 3 C 1 + 5 C 2 + ... + (2n + 1) C n
n n n
Aliter
= S
r =0
(2r + 1) C r =n
S
r =0
n
2r × C r + S
r =0
n
Cr LHS = 12 × C 1 + 22 × C 2 + 32 × C 3 + ... + n 2 × C n
n n
n n
= S r 2 . n C r = S r 2 × × n - 1C r - 1
n
=2
r =0
S r × n Cr + S
r =0
n
Cr r =1 r =1 r
é n n n -1 ù
n
é
n
ù êëQ C r = r × Cr - 1ú
û
= 2 S r × × n -1C r -1 + S n C r êQ n C r = × n - 1C r
n n
- 1ú
r =0 r r =0 ë r û n n
n n =n S r × n - 1C r - 1 = n rS= 1 {(r - 1) + 1} × n -1C r - 1
= 2n S
r =0
n -1
Cr -1 +
r =0
S n
Cr
r =1

n n

= 2n (0 + n -1
C0 + n -1
C1 + n -1
C 2 + ... + n -1
Cn - 1) =n S (r - 1) × n - 1C r - 1 + n rS= 1 n -1C r - 1
r =1

+ ( C 0 + C 1 + C 2 + ... + n C n )
n n n n n

= 2n × 2n - 1 + 2n = (n + 1) × 2n = RHS
=n S (n - 1) × n - 2 C r - 2 + n rS= 1 n - 1C r - 1
r =1
Chap 06 Binomial Theorem 461

n n = n (n - 1) { N C 0 + N C 1 + ... + N C N }
= n ( n - 1) S n - 2 C r - 2 + n rS= 1 n - 1C r -1
r =1 = n (n - 1) 2 N = n (n - 1) 2n - 2 = RHS
n-2 n-2 n-2
= n ( n - 1) ( 0 + C0 + C1 + C2 II. Aliter
+ ... + n-2
C n - 2 ) + n (n - 1 C 0 + n -1
C1 LHS = (1 × 2) C 2 + (2 × 3) C 3 + ... + {(n - 1) × n } C n
n

+ n -1
C 2 + ... + n -1
C n -1 ) = S
r =2
( r - 1) × r × n C r

= n (n - 1) × 2 n - 2 + n × 2 n - 1 = n (n + 1) 2n - 2 = RHS n
= S ( r - 1) × r × n × n - 1 × n - 2C r -2
n 2 r =2 r ( r - 1)
y Example 62. If (1 + x ) = C 0 + C 1 x + C 2 x
n
+ ... + C n x n , prove that (1 × 2) C 2 + (2 × 3) = ( n - 1) n S n - 2 C r -2
r =2
C 3 + K + {(n - 1) × n} C n = n (n - 1) 2n - 2 . n-2 n-2 n-2 n-2
= ( n - 1) n ( C0 + C1 + C 2 + ... + Cn - 2 )
Sol. Here, last term of n-2
= ( n - 1) n × 2 = RHS
(1 × 2) C 2 + (2 × 3) C 3 + ... + {(n - 1) × n } C n is (n - 1)n C n
i.e. (n - 1) n
y Example 63. If
[start with greater factor here greater factor is n] and last
(1 + x )n = C 0 + C 1 x + C 2 x 2 + C 3 x 3 + ... + C n x n , prove
term with positive sign, then n = n × 1 + 0
or n ) n (1 that C 0 - 2C 1 + 3C 2 - 4C 3 + ... + ( - 1)n (n + 1) C n = 0.
-n Sol. Numerical value of last term of
0 C 0 - 2C 1 + 3C 2 - 4C 3 + ... + ( - 1)n (n + 1) C n is
Here, q = 1 and r = 0 (n + 1) C n i.e., (n + 1), then
The given series is n + 1 = n × 1 + 1 or n ) n + 1 (1
(1 + x )n = C 0 + C 1 x + C 2 x 2
+ C3 x 3
+ ... + C n x n -n
Differentiating on both sides w.r.t. x , we get 1
n (1 + x )n -1 = 0 + C 1 + 2C 2 x + 3C 3 x 2 + ... + n C n x n -1 Here, q = 1 and r = 1
The given series is
Again, differentiating on both sides w.r.t. x, we get
(1 + x )n = C 0 + C 1x + C 2 x 2 + C 3 x 3
+... + C n x n
n (n - 1) (1 + x )n - 2 = 0 + 0 + (1 × 2) C 2 + (2 × 3) C 3 x
On multiplying both sides by x, we get
+ ... + {(n - 1) × n } C n x n - 2
x (1 + x )n = C 0 x + C 1x 2
+ C2 x 3
+ C3 x 4
+ ... + C n x n + 1
Putting x = 1, we get
On differentiating both sides w.r.t. x, we get
n (n - 1) (1 + 1)n - 2 = (1 × 2) C 2 + (2 × 3) C 3
x × n (1 + x )n -1 + (1 + x )n × 1 = C 0 + 2C 1x + 3C 2 x 2
+ ... + {(n - 1) n } × C n
+ 4 C 3 x 3 + ... + (n + 1) C n x n
or (1 × 2) C 2 + (2 × 3) C 3 + ... + {(n - 1) n } × C n = n (n - 1) 2n - 2
Putting x = - 1, we get
I. Aliter 0 = C 0 - 2C 1 + 3C 2 - 4C 3 + ... + ( - 1)n (n + 1) C n
LHS = (1 × 2)C 2 + (2 × 3)C 3 + (3 × 4)C 4
or C 0 - 2C 1 + 3C 2 - 4C 3 + ... + ( - 1)n (n + 1) C n = 0
+ ... + {(n - 1) n } × C n
n ( n - 1) n ( n - 1) ( n - 2) I. Aliter
= (1 × 2) + (2 × 3)
1 ×2 1 ×2 ×3 LHS = C 0 - 2C 1 + 3C 2 - 4C 3 + ... + ( -1)n (n + 1) C n
n ( n - 1) ( n - 2) ( n - 3) = C 0 - (C 1 + C 1 ) + (C 2 + 2C 2 ) - ( C 3 + 3C 3 )
+ (3 × 4 )
1 ×2 ×3 × 4 + ... + ( -1)n {C n + n C n }
+ ... + (n - 1) n × 1
= {C 0 - C 1 + C 2 - C 3 + ... + ( - 1)n C n }
ì ( n - 2) ( n - 2) ( n - 3) ü
= n (n - 1) í1 + + + ... + 1ý + { - C 1 + 2C 2 - 3C 3 + ... + ( - 1)n n C n }
î 1 1 × 2 þ
ì n ( n - 1) n (n - 1)(n - 2)ü
Now, in bracket, let n - 2 = N , then ï -n + 2 × -3 ï
= (1 - 1)n + í 1 ×2 1 ×2 ×3 ý
ì N N ( N - 1) ü ï + ... + ( -1)n × n ï
= n (n - 1) í1 + + + ... + 1ý î þ
î 1 2! þ
462 Textbook of Algebra

ì ( n - 1) ( n - 2) ü In bracket, put n - 1 = N , then


= 0 + n í - 1 + ( n - 1) - + ... + ( - 1)n ý
î 1 ×2 þ ì N N ( N - 1) ü
LHS = n í1 - + - ... + ( - 1) N ý
ì ( n - 1) ( n - 2) ü î 1 1 ×2 þ
= 0 - n í1 - (n - 1) + - ... + ( - 1)n -1ý
î 1 × 2 þ = n { N C 0 - N C 1 + N C 2 - ... + ( - 1)N N
CN }
Let in bracket, put n - 1 = N , we get N
= n (1 - 1) = 0 = RHS
ì N ( N - 1) ü
LHS = 0 - n í1 - N + - ... + ( - 1) N ý II. Aliter
î 1 × 2 þ
LHS = C 1 - 2C 2 + 3C 3 - ... + ( -1)n - 1 × n C n
N N N N N
= 0 - n { C 0 - C 1 + C 2 - ... + ( - 1) CN } n

= 0 - n ( 1 - 1) N
= 0 - 0 = 0 = RHS = S ( - 1)r - 1 × r × nC r
r =1
n
II. Aliter é n ù
LHS = C 0 - 2 C 1 + 3 C 2 - 4 C 3 + ... + ( - 1) (n + 1) C n n = S ( - 1)r - 1 × n × n - 1C r - 1 n n -1
êëQ C r = r × Cr - 1ú
r =1 û
n n n
= S ( -1)r (r + 1) n C r = r S= 0 ( - 1)r [r × n C r + n Cr ] =n S ( - 1)r - 1 × n - 1C r - 1
r =1
r =0
n
é n ù = n (1 - 1)n - 1 = 0 = RHS
= S ( -1)r [n ×n -1 C r -1 +n C r ]
r =0
n n -1
êëQ C r = r × C r -1 úû
y Example 65. If (1 + x ) n = C 0 + C 1 x + C 2 x 2
n n
=n S
r =0
( - 1)r × n - 1 C r -1 + S
r =0
( - 1)r × n C r + C 3 x 3 +... + C n x n
, prove that
n n C 0 - 3 C 1 + 5 C 2 - ... + ( -1)n (2n + 1) C n = 0.
= -n
r =0
S ( - 1) r -1
× n -1
Cr -1 + S
r =0
r
( - 1) × C r n
Sol. The numerical value of last term of
= - n (1 - 1)n - 1 + (1 - 1)n = 0 + 0 = 0 = RHS C 0 - 3C 1 + 5C 2 - ... + ( - 1)n (2n + 1) C n is (2n + 1) C n
i.e. (2n + 1)
y Example 64. If (1 + x )n = C 0 + C 1 x + C 2 x 2
and 2n + 1 = 2 × n + 1 or n ) 2n + 1 ( 2
+ C 3 x 3 +... + C n x n , prove that - 2n
C 1 - 2 C 2 + 3 C 3 - ... + ( - 1)n - 1 n C n = 0. 1
Here, q = 2 and r = 1
Sol. Numerical value of last term of
The given series is
C 1 - 2C 2 + 3C 3 - ... + ( - 1)n -1 n C n is nC n i.e., n, then 2 3
(1 + x )n = C 0 + C 1x + C 2 x + C3 x + ... + C n x n now,
and n = n × 1 + 0 or n ) n (1
replacing x by x 2 , then we get
-n
(1 + x 2 )n = C 0 + C 1x 2
+ C 2x 4
+ ... + C n x 2n
1
Here, q = 1 and r = 0 On multiplying both sides by x , we get
The given series is x (1 + x 2 )n = C 0 x + C 1x 3 + C 2 x 5 + ... + C n x 2n + 1
2 3
(1 + x )n = C 0 + C 1x + C 2 x + C3 x + ... + C n x n On differentiating both sides w.r.t. x , we get
On differentiating both sides w.r.t. x, we get x × n (1 + x 2 )n -1 2x + (1 + x 2 )n × 1 = C 0 + 3C 1x 2

n (1 + x )n -1 = 0 + C 1 + 2C 2 x + 3C 3 x 2
+ ... + n C n x n -1 + 5C 2 x 4 + ... + (2n + 1) C n x 2n

Putting x = - 1, we get Putting x = i in both sides, we get


n -1
0 = C 1 - 2C 2 + 3C 3 - ... + ( - 1) n Cn 0 + 0 = C 0 - 3C 1 + 5C 2 - ... + (2n + 1) ( -1)n C n
or C 1 - 2C 2 + 3C 3 - ... + ( - 1)n -1 n C n = 0 or C 0 - 3C 1 + 5C 2 - ... + ( - 1)n (2n + 1) C n = 0
I. Aliter I. Aliter
LHS = C 1 - 2C 2 + 3C 3 - ... + ( -1)n - 1 n × C n LHS = C 0 - 3C 1 + 5C 2 - ... + ( -1)n (2n + 1) C n
n ( n - 1) n ( n - 1) ( n - 2) = C 0 - (1 + 2) C 1 + (1 + 4 ) C 2 - ... + ( -1)n (1 + 2n ) C n
= n - 2× +3 - ... + ( - 1)n -1 × n × 1
1 ×2 1 ×2 ×3
= (C 0 - C 1 + C 2 - ... + ( - 1)n C n ) - 2 (C 1 - 2C 2
ì ( n - 1) ( n - 1) ( n - 2) ü
= n í1 - + - ... + ( - 1)n -1ý + ... + ( - 1)n - 1 n × C n )
î 1 1 × 2 þ
Chap 06 Binomial Theorem 463

1
= (1 - 1)n - 2 × 0 [from Example 64] é C x 2
C2 x 3
Cn x n + 1 ù
Þ = êC 0 x + 1 + + ... + ú
= 0 = RHS ë 2 3 n +1 û0
II. Aliter
2 n +1 - 1 C C C
n
LHS = C 0 - 3C 1 + 5C 2 - ... + ( -1) (2n + 1) C n Þ = C 0 + 1 + 2 + ... + n
n +1 2 3 n +1
n n
= S ( - 1)r (2r + 1) n C r
r =1
= S ( - 1)r [2r × n C r
r =1
+ nC r ] or C 0 +
C1 C 2
+
C
+ ... + n =
2n + 1 - 1
2 3 n +1 n +1
n n
= 2 S n × n - 1C r -1 + S ( - 1)r × n C r I. Aliter
r =1 r =1
C1 C 2 C
= 2n (1 - 1)n - 1 + (1 - 1)n = 0 + 0 = 0 = RHS LHS = C 0 + + + ... + n
2 3 n +1
n n ( n - 1) 1
=1+ + + ... +
1 ×2 1 ×2 ×3 n +1
Use of Integration 1 é (n + 1)n (n + 1) n (n - 1) ù
= êë(n + 1) + 1 × 2 + + ... + 1ú
This method is applied only when the numericals occur as n +1 1 ×2 ×3 û
the denominator of the binomial coefficient. Put n + 1 = N , then
1 é N ( N - 1) N ( N - 1) ( N - 2) ù
LHS = êN + + + ... + 1ú
Solution Process N ë 2 ! 3 ! û
If (1 + x ) n = C 0 + C 1 x + C 2 x 2 + C 3 x 3 + ... + C n x n , =
1 N
[ C 1 + N C 2 + N C 3 + ... + N C N ]
then integrate both sides between the suitable limits N
which gives the required series. 1 2N - 1 2n + 1 - 1
= [(1 + 1)N - 1] = = = RHS
1. If the sum contains C 0 , C 1 , C 2 , . . . , C n are all positive N N n +1
signs, then integrate between limits 0 to 1. II. Aliter
n
2. If the sum contains alternate signs (i.e., +, –), then
+ ... + n = S
C1 C 2 C Cr
LHS = C 0 + +
integrate between limits –1 to 0. 2 3 n +1 r=0r +1
3. If the sum contains odd coefficients (i.e., n n n n +1
Cr + 1 é n + 1C r +1 n C r ù
C 0 , C 2 , C 4 , . . .), then integrate between –1 to +1. = S Cr
= S êQ = ú
r =0 ( r + 1) r = 0 ( n + 1) êë n +1 r + 1 úû
4. If the sum contains even coefficients (i.e.,
n
C 1 , C 3 , C 5 , . . .), then subtracting (2) from (1) and then =
1
S n +1
Cr +1
dividing by 2. ( n + 1) r = 0
5. If in denominator of binomial coefficient product of 1 n +1 n +1 n +1
= ( C1 + C2 + C3
two numericals, then integrate two times first times ( n + 1)
taken limits between 0 to x and second times take n +1
+ ... + Cn + 1)
suitable limits.
1 2n + 1 - 1
y Example 66. If (1 + x ) = C 0 + C 1 x n = (2n + 1 - 1) = = RHS
n +1 n +1
+ C 2 x 2+ ... + C n x n , prove that
y Example 67. If (1 + x )n = C 0 + C 1 x + C 2 x 2
C C C 2 n +1 - 1
C 0 + 1 + 2 + ... + n = . + C 3 x 3+ ... + C n x n , prove that
2 3 n+1 n+1 C C C 1
Sol. Q(1 + x )n = C 0 + C 1x + C 2 x 2 + ... + C n x n …(i)
C 0 - 1 + 2 - ... + ( - 1)n n = .
2 3 n+ 1 n+1
Integrating both sides of Eq. (i) within limits 0 to 1, then we get Sol. Q(1 + x )n = C 0 + C 1x + C 2 x 2 + ... + C n x n …(i)
1 1
ò0 ò0
n 2 n
(1 + x ) dx = (C 0 + C 1x + C 2 x + ... + C n x ) dx Integrating on both sides of Eq. (i) within limits - 1 to 0,
1 then we get
é (1 + x )n + 1 ù 0 0
ò-1(1 + x ) ò- 1(C 0 + C 1x + C 2 x
n 2
ê ú dx = + ... + C n x n ) dx
ë n + 1 û0
464 Textbook of Algebra

0 0 1
é (1 + x )n ù é C1 x 2
C2 x 3 C x n +1 ù = ( n + 1C 1 - n + 1C 2 + n + 1C 3 - ... + ( - 1)n × n + 1C n + 1 )
Þê ú = êC 0 x + + +...+ n ú ( n + 1)
ë n + 1 û -1 ë 2 3 n + 1 û- 1
=
1
{ n + 1C 0 - ( n + 1C 0 - n + 1C 1 + n + 1C 2 - n + 1C 3
( n + 1)
1-0 æ C C C ö + ... + ( - 1)n + 1 n + 1C n + 1 )}
Þ = 0 - ç - C 0 + 1 - 2 + ... + ( - 1)n +1 n ÷
n +1 è 2 3 n + 1ø 1 1 1
= [ 1 - (1 - 1)n + 1 ] = [1 - 0] = = RHS
1 C C Cn ( n + 1) ( n + 1) n +1
Þ = C 0 - 1 + 2 - ... + ( - 1)n + 2
n +1 2 3 n +1
y Example 68. If (1 + x )n = C 0 + C 1 x + C 2 x 2
1 C C Cn
Þ = C 0 - 1 + 2 - ... + ( - 1)n + C 3 x 3 +... + C n x n , prove that
n +1 2 3 n +1
[Q ( - 1)n + 2 = ( - 1)n ( - 1)2 = ( - 1)n ] C0 C2 C4 2n
+ + + ... = .
C1 C 2 C 1 1 3 5 n+1
Hence, C 0 - + - ... + ( - 1)n n = 2 3
2 3 n +1 n + 1 Sol. Q (1 + x )n = C 0 + C 1x + C 2 x + C3 x
I. Aliter + C 4 x 4 +... + C n x n …(i)
C1 C 2 C
LHS = C 0 - + - ... + ( - 1)n n Integrating on both sides of Eq. (i) within limits - 1 to 1,
2 3 n +1 then we get
n n ( n - 1) 1 1 1 1
=1- + - ... + ( - 1)n = ò- 1 (1 + x ) ò-1(C 0 + C 1x + C 2 x
n 2 3 4
dx = + C3 x + C4 x
2 1 ×2 ×3 n + 1 ( n + 1)
( n + 1) n ( n + 1) n ( n - 1) + ... + C n x n ) dx
é ù
êë (n + 1) - 1 × 2 + - ... + ( - 1)n ú 1 1
1 ×2 ×3 û ò-1(C 0 + C 2 x + C 4 x 4 + ...)dx + ò (C 1x + C 3 x
2 3
= + ...)dx
-1
Put n + 1 = N , we get 1
= 2 ò (C 0 + C 2 x 2
+ C 4 x 4 + ...) dx + 0
é N ( N - 1) N ( N - 1) ( N - 2) ù 0
1 N - +
= ê 1 ×2 1 ×2 ×3 ú [by property of definite integral]
Nê N -1 ú
[since, second integral contains odd function]
ë - ... + ( - 1) û
1 1
1 N -1
é (1 + x )n + 1 ù éæ C2 x 3
C4 x 5 öù
= [ C 1 - N C 2 + N C 3 - ... + ( - 1)N ] ê ú = 2 ê çC 0 x + + + ...÷ ú
N ë n + 1 û -1 ëê è 3 5 ø úû 0
1 N
2n + 1
= - [ - N C 1 + N C 2 - N C 3 + ... + ( - 1) N C N ] æ C C ö
N Þ = 2 çC 0 + 2 + 4 + ...÷
n +1 è 3 5 ø
1 N C2 C4 2n
= - [ C 0 - N C1 + N C 2 - N C 3 or C 0 + + +K=
N 3 5 n +1
N
+ ... + ( - 1)N C N - NC 0 ] I. Aliter
1 1 1 C2 C4
=- [(1 - 1)N _ N C 0 ] = - [0 - 1] = LHS = C 0 + + + ...
N N N 3 5
n ( n - 1) n ( n - 1) ( n - 2) ( n - 3)
=
1
= RHS =1+ + + ...
n +1 1 ×2 ×3 1 ×2 ×3 × 4 ×5
1 ì n + 1 (n + 1)n (n - 1)
II. Aliter = í +
n
( n + 1) î 1 1 ×2 ×3
( - 1)r × C r
- ... + ( - 1)n n = S
C1 C 2 C
LHS = C 0 - + (n + 1)n (n - 1) (n - 2) (n - 3) ü
2 3 n +1 r = 0 r + 1 + + ...ý
n
1 ×2 ×3 × 4 ×5 þ
r
× nC r
= S ( - 1) =
1
{ n + 1C1 + n +1
C3 + n +1
C 5 + ...}
r =0 r +1
n +1
n +1 é n +1
Cr ù
n n
Cr C r +1
= S ( - 1)r ×
+1
êQ = ú =
1
[sum of even binomial coefficients of (1 + x )n + 1]
r =0 ( n + 1) êë n + 1 r + 1 úû
( n + 1)
n
=
1
S ( - 1)r × n + 1C r +1 =
2n + 1 - 1
=
2n
= RHS
( n + 1) r = 0 n +1 n +1
Chap 06 Binomial Theorem 465

C0 C2 C4 Put n + 1 = N , then
II. Aliter LHS = + + + ...
1 3 5 1 é N ( N - 1) N ( N - 1) ( N - 2) ( N - 3)
LHS = +
Case I If n is odd say n = 2m + 1, " m ÎW , then N êë 2! 4!
m 2 m +1 m 2m + 2
C 2r + 1 N ( N - 1)( N - 2) ( N - 3) ( N - 4 ) ( N - 5) ù
LHS = S C 2r
= S + + ...ú
r =0 2r + 1 r =0 (2m + 1) 6! û
é 2 m +1C 2r 2m+ 2
+1ù
C 2r 1 N
êQ = ú = [ C 2 + N C 4 + N C 6 + ... ]
2r + 1 2m +1 N
êë úû
1
1 2n = [ ( N C 0 + N C 2 + N C 4 + N C 6 + ...) - N C 0 ]
= ×22 m + 2 -1 = = RHS N
(2m + 1) n +1
1 -1 2n - 1
[Qn = 2m + 1] = [2 N - 1] = = RHS
N n +1
Case II If n is even say n = 2m, " m Î N , then
II. Aliter
m 2m m 2 m +1
C 2r C 2r
LHS = S = S +1
LHS =
C1 C 3 C 5
+ + + ...
r =0 2r + 1 r =0 (2m + 1) 2 4 6
é 2 m +1
C 2r 2m
C 2r ù Case I If n is odd say n = 2m + 1 , " m ÎW , then
+1
êQ = ú m 2 m +1 m 2m+2
2m + 1 2r + 1 úû C2r C 2r
êë LHS = S +1
= S +2
r =0 2r + 2 r =0 (2m + 2)
22 m + 1 - 1 2n
= = = RHS [Qn = 2m ] é 2m+2
C 2r +2
2 m +1
C 2r +1ù
2m + 1 n +1 êQ = ú
êë 2m + 2 2r + 2 úû
y Example 69. If (1 + x )n = C 0 + C 1 x + C 2 x 2+ C 3 x 3
1 2 m +2 2 m +2 2 m +2
= ( C2 + C 4 + ... + C2 m + 2)
C C C 2n - 1 ( 2m + 2)
+... + C n x n , prove that 1 + 3 + 5 + ... = .
2 4 6 n+1 1 2n - 1
= × ( 22 m + 2 - 1 - 2 m +1
C0) = [Q2m + 1 = n ]
Sol. We know that, from Examples (66) and (67) ( 2m + 2) n +1
C1 C 2 C 3 C 4 C 5 2n + 1 - 1 = RHS
C0 + + + + + + ... = …(i)
2 3 4 5 6 n +1 Case II If n is even say n = 2m, " m Î N , then
m -1 2m m -1 2 m +1
C1 C 2 C 3 C 4 C 5 1 C 2r C 2r
and C 0 -
2
+
3
-
4
+
5
-
6
+ ... =
n +1
…(ii) LHS = S +1
= S +2
r =0 (2r + 2) r =0 (2m + 1)
On subtracting Eq. (ii) from Eq. (i), we get é 2 m +1
+1ù
2m
C 2r +2 C 2r
n +1 êQ = ú
æC C C ö 2 -2 2m + 1 2r + 2
2 ç 1 + 3 + 5 + ...÷ = êë úû
è2 4 6 ø n +1 m -1
On dividing each sides by 2, we get =
1
S 2 m + 1C 2 r + 2
(2m + 1) r = 0
C1 C 3 C 5 2n - 1 1
+ + + ... = = ( 2 m +1C 2 + 2 m +1C 4 + 2 m +1
C6
2 4 6 n +1 (2m + 1)
2 m +1
C1 C 3 C 5 + ... + C2 n )
I. Aliter LHS = + + + ...
2 4 6 1
= × ( 22 m + 1 - 1 - 2m +1
C0)
n n ( n - 1) ( n - 2) ( 2m + 1)
= +
1 ×2 1 ×2 ×3 × 4 2n - 1
= = RHS [Qn = 2m ]
n ( n - 1) ( n - 2) ( n - 3) ( n - 4 ) n +1
+ + ...
1 ×2 ×3 × 4 ×5 ×6
1 é ( n + 1) n ( n + 1) n ( n - 1) ( n - 2) y Example 70. If (1 + x )n = C 0 + C 1 x
= +
n + 1 êë 1 × 2 1 ×2 ×3 × 4 + C 2 x 2+ ... + C n x n , prove that
n +1
( n + 1) n ( n - 1) ( n - 2) ( n - 3) ( n - 4 ) ù C 1 33 C 2 3 4 C 3 3n +1 C n 4 - 1
+ + ...ú 3C 0 + 3 2 + + + ... + = .
1 ×2 ×3 × 4 ×5 ×6 û 2 3 4 n+1 n+1
466 Textbook of Algebra

2 3 n
Sol. Q (1 + x )n = C 0 + C 1x + C 2 x + C3 x +... + C n x n …(i)
=
1
S n +1
Cr +1 × 3r +1

Integrating on both sides of Eq. (i) within limits 0 to 3, we get ( n + 1) r = 0


3 3 1 n +1 n +1 n +1
ò0 (1 + x ) dx = ò0 ( C 0 + C 1x + C 2 x = C1 × 3 + C 2 × 32 + C 3 × 33
2
n
+ C 3 x 3 +...+ C n x n )dx (
( n + 1)
3 n +1
é (1 + x )n + 1 ù é C1 x 2
C2 x 3
C3 x 4
+ ... + C n + 1 × 3n +1 )
Þê ú = êC 0 x + + + +
ë n + 1 û0 ë 2 3 4 1
= [ (1 + 3)n + 1 - n +1
C0]
n +1ù
3 ( n + 1)
C x
K+ n ú
n + 1 û0 4n + 1 - 1
= = RHS
n +1
4n + 1 - 1 32 C 1 33 C 2 34 C 3 3n + 1 C n
Þ = 3C 0 + + + + ... +
n +1 2 3 4 n +1 y Example 71. If (1 + x )n = C 0 + C 1 x + C 2 x 2+... + C n x n ,
Hence, 22 23 24 2n + 2 C n
32 C 1 33 C 2 34 C 3 3n + 1 C n 4n + 1 - 1 show that C0 + C1 + C 2 + ... +
3C 0 + + + + ... + = 1× 2 2× 3 3×4 (n + 1) (n + 2)
2 3 4 n +1 n +1 n +2
3 - 2n - 5
I. Aliter = .
(n + 1) (n + 2)
32 C 1 33 C 2 34 C 3 3n + 1 C n
LHS = 3C 0 + + + + ... + Sol. Given,
2 3 4 n +1
2 3
(1 + x )n = C 0 + C 1 x + C 2 x + C3 x +... + C n x n
32 × n 33 × n (n - 1) 34 × n (n - 1) (n - 2)
= 3 ×1 + + + ...(i)
2 1 ×2 ×3 1 ×2 ×3 × 4
3n +1 Integrating both sides of Eq. (i) within limits 0 to x , we get
+ ... +
n +1 x x
ò0 (1 + x ) ò0 ( C 0 + C 1x + C 2 x
n 2
dx = + ... + C n x n ) dx
1 é 2 3
3 ( n + 1) n 3 ( n + 1) n ( n - 1)
= ê3 × ( n + 1) + + é (1 + x )n + 1 ù
x
( n + 1) ë 1 ×2 1 ×2 ×3 Þ ê ú
34 ( n + 1) n ( n - 1) ( n - 2) ù ë n + 1 û0
+ + ... + 3n + 1 ú x
1 ×2 ×3 × 4 û é Cx 2
C x 3
C x n +1ù
= êC 0 x + 1 + 2 +K+ n ú
Put n + 1 = N , then ë 2 3 n + 1 û0
1 é 32 N ( N - 1) 33 N ( N - 1) ( N - 2)
LHS = ê3N + + (1 + x )n + 1 - 1 Cx 2
C2 x 3
Cn x n + 1
N ë 2! 3! Þ = C 0x + 1 + + ... +
( n + 1) 2 3 n +1
34 N ( N - 1) ( N - 2) ( N - 3) ù …(ii)
+ + ... + 3N ú
4! û Again, integrating both sides of Eq. (ii) within limits 0 to 2,
1 N we get
= [ C 1 (3) + N C 2 (3)2 + N C 3 (3)3 + ... + N C N (3)N ]
N 2( 1 + x )n + 1 - 1
=
1 N
[ C 0 + NC 1 ( 3) + N C 2 ( 3) 2 + N C 3 ( 3) 3
ò0 ( n + 1)
dx
N
+ ... + N C N (3) N - N C 0 ] 2æ C 1x 2
C2 x 3
Cn x n + 1 ö
= ò0 çC 0 x +
è
+ + ... +
n +1 ø
÷ dx
1 4 N - 1 4n + 1 - 1 2 3
= { ( 1 + 3) N - 1 } = = = RHS
N N n +1 2
1 æ (1 + x )n + 2 öù é C x 2 C 1x 3 C 2 x 4
II. Aliter Þ ç - x ÷ú = ê 0 + +
( n + 1) è n + 2 ø ûú 0 ë 1 × 2 2 ×3 3× 4
C 1 33 C 2 34 C 3 3n + 1 C n
LHS = 3C 0 + 32 + + + ... + 2
2 3 4 n +1 Cn x n + 2 ù
+K+ ú
n r +1 n +1 n +1 ( n + 1) ( n + 2) û 0
× nC r 3r × Cr
= S3 = S
+1
r =0 ( r + 1) r =0 ( n + 1) 1 ì 3n + 2 1 ü 22 23 24
Þ í -2- ý= C0 + C1 + C2
é n +1
Cr +1 Cr ù
n ( n + 1) î n + 2 n + 2þ 1 × 2 2 ×3 3× 4
êQ = ú
êë n +1 r + 1 úû 2n + 2 C n
+ ... +
( n + 1) ( n + 2)
Chap 06 Binomial Theorem 467

22 23 24 2n + 2 C n
Hence, C0 + C1 + C 2 + ... + When Each Term in Summation Contains
1 ×2 2 ×3 3× 4 ( n + 1) ( n + 2)
3 n+2
- 2n - 5
the Product of Two Binomial Coefficients
=
( n + 1) ( n + 2) or Square of Binomial Coefficients
I. Aliter Solution Process
22 23 24 2n + 2 C n
LHS = C0 + C1 + C2 + K + 1. If difference of the lower suffixes of binomial
1 ×2 2 ×3 3× 4 ( n + 1) ( n + 2) coefficients in each term is same.
22 23 24 n ( n - 1) 2n + 2 × 1 i.e. n
C 0 n C 2 + n C 1 × n C 3 + n C 2 × n C 4 + ...
= ( 1) + ×n + + ... +
1 ×2 2 ×3 3× 4 1 ×2 (n + 1) (n + 2)
Here, 2 - 0 = 3 - 1 = 4 - 2 = .... = 2
1 ì ( n + 2) ( n + 1) 2 ( n + 2) ( n + 1) n 3
= í 2 + 2 Case I If each term of series is positive, then
( n + 1) ( n + 2) î 1 ×2 1 ×2 ×3
(1 + x ) n = C 0 + C 1 x + C 2 x 2 + ... + C n x n …(i)
( n + 2) ( n + 1) n ( n - 1) 4 ü
+ 2 + ... + 2n + 2 ý
1 ×2 ×3 × 4 þ Interchanging 1 and x , we get
Put n + 2 = N , then we get ( x + 1) n = C 0 x n + C 1 x n - 1 + C 2 x n -2
+... + C n …(ii)
1 ì N ( N - 1) 2 N ( N - 1) ( N - 2) 3 Then, multiplying Eqs. (i) and (ii) and equate the
= í 2 + 2
N ( N - 1) î 1 × 2 1 ×2 ×3 coefficients of suitable power of x on both sides.
N ( N - 1) ( N - 2) ( N - 3) 4 ü 1
+ 2 + ... + 2N ý Replacing x by in Eq. (i), then we get
1 ×2 ×3 × 4 þ x
1 n
= { N C 2 ( 2) 2 + N C 3 ( 2) 3 + N C 4 ( 2) 4 æ 1ö C1 C2 C
N ( N - 1) ç1 + ÷ = C 0 + + + ... + n …(iii)
+ ... + N C N (2)N ] è xø x x 2
xn
1 Then, multiplying Eqs. (i) and (iii) and equate the
= { N C 0 + N C 1(2) + N C 2 (2)2 + N C 3 (2)3
N ( N - 1) coefficients of suitable power of x on both sides.
+ N C 4 (2)4 + ... + N C N (2)N - N C 0 - N C 1 (2)]
y Example 72. If (1 + x )n = C 0 + C 1 x + C 2 x 2
1
= {(1 + 2)N - 1 - 2N } + C 3 x 3 +... + C n x n , prove that
N ( N - 1)
3n + 2 - 1 - 2 (n + 2) 3n + 2 - 2n - 5 C 0C r + C 1C r +1 + C 2 C r + 2 + ... + C n - r C n
= = = RHS 2n !
( n + 2) ( n + 1) ( n + 1) ( n + 2) = .
(n - r )! (n + r )!
II. Aliter
22 23 24 2 n + 2× C n Sol. Here, differences of lower suffixes of binomial coefficients
LHS = ×C 0 + ×C1 + × C 2 + ... + in each term is r .
1 ×2 2 ×3 3× 4 ( n + 1) ( n + 2)
n +1
i.e., r - 0 = r + 1 - 1 = r + 2 - 2 = .... = n - (n - r ) = r
2r + 1 n
= S × Cr -1
Given,
r =1 r ( r + 1) (1 + x )n = C 0 + C 1x + C 2 x 2 + ... + C n - r x n - r + ... + C n x n
n +1 r +1
× n + 2 Cr é n+2 n
Cr - 1 ù
= S2 +1
êQ
Cr + 1
= ú Now,
…(i)
r =1 ( n + 1) ( n + 2) êë ( n + 1 ) ( n + 2 ) r ( r + 1) úû
n +1 ( x + 1)n = C 0 x n + C 1 x n - 1 + C 2 x n - 2 + ... + C r x n - r
=
1
S n + 2 C r + 1 × 2r + 1 + Cr +1x
n - r -1
+ Cr x n -r -2
+ ... + C n …(ii)
( n + 1) ( n + 2) r = 1 +2

1 On multiplying Eqs. (i) and (ii), we get


= [ n + 2 C 2 × 22 + n + 2 C 3 × 23
( n + 1) ( n + 2) (1 + x )2n = (C 0 + C 1x + C 2 x 2
+ ... + C n - r x n - r + ...
+ ... + n + 2 C n + 2 × 2n + 2 ]
1 + C n x n ) ´ (C 0 x n + C 1 x n - 1
= [ (1 + 2)n + 2 - n+2
C0 - n+2
C 1 × 21 ]
( n + 1) ( n + 2) + C 2 x n - 2 + ... + C r x n - r + C r +1x
n - r -1

(3 n + 2 - 2n - 5) + Cr +2 x n -r -2
+ ... + C n ) …(iii)
= = RHS
( n + 1) ( n + 2)
468 Textbook of Algebra

Now, coefficient of x n - r on LHS of Eq. (iii) = 2n


C n -r Corollary II For r = 1,
2n !
=
2n ! C 0C 1 + C 1C 2 + C 2 C 3 + ... + C n -1 C n =
(n - r ) ! (n + r ) ! ( n - 1) ! ( n + 1) !
Corollary III For r =2,
and coefficient of x n - r on RHS of Eq. (iii)
2n !
= C 0C r + C 1C r + 1 + C 2 C r + 2 + ... + C n - r C n C 0C 2 + C 1C 3 + C 2 C 4 + ... + C n - 2 C n =
( n - 2) ! ( n + 2) !
But Eq. (iii) is an identity, therefore coefficient of x n - r in
RHS = coefficient of x n - r in LHS. y Example 73. If (1 + x )n = C 0 + C 1 x
Þ C 0C r + C 1C r +1 + C 2 C r + 2 + ... + C n - r C n + C 2 x 2 +... + C n x n , prove that
=
2n ! 2n ! 1 × 3 × 5 ...(2n - 1) n
(n - r )! (n + r ) ! C 02 + C12 + C 22 = = ×2 .
n!n! n!
Aliter
Sol. Given, (1 + x )n = C 0 + C 1x + C 2 x 2 +... + C n x n …(i)
Given,
n n n -1 n-2
(1 + x )n = C 0 + C 1 x + C 2 x 2
+ ... + C r x r + C r r +1 Now, ( x + 1) = C 0 x + C 1x + C 2x + ... + C n …(ii)
+1x
+2 On multiplying Eqs. (i) and (ii), we get
+ Cr +2 xr + ... + C n - r x r + ... + C n x n …(i)
(1 + x )2n = ( C 0 + C 1x + C 2 x 2 +... + C n x n )
n
æ 1ö C C C Cr + 1 Cr + 2
Now, ç1 + ÷ = C 0 + 1 + 22 + ... + rr + r + 1 + r + 2 ´ (C 0 x n + C 1 x n - 1 + C 2 x n - 2 + ... + C n ) …(iii)
è x ø x x x x x
Cn - r Cn Now, coefficient of x n in RHS
+ ... + + ... + …(ii) 2 2 2 2
xn - r xn = C 0 + C 1 + C 2 + ... + C n
On multiplying Eqs. (i) and (ii), we get 2n 2n !
And coefficient of x n in LHS = Cn =
(1 + x ) 2n n !n !
2 r +1
= (C 0 + C 1 x + C 2 x + ... + C r x r + C r +1x
xn 1 × 2 × 3 × 4 × 5...× (2n - 1) 2n 1 × 3 × 5... (2n - 1) 2n n !
= =
+2
+ Cr +2 xr + .. + C n - r x n - r + .. + C n x n ) n !n ! n !n !

æ C C C Cr + 1 Cr + 2 But Eq. (iii) is an identity, therefore coefficient of x n in RHS


´ çC 0 + 1 + 22 + ... + rr + r + 1 + r + 2 = coefficient of x n in LHS.
è x x x x x
Cn ö 2 2 2 2 2n !
+ ... +
Cn - r
+ ... + Þ C 0 + C 1 + C 2 + ... + C n =
÷ …(iii) n !n !
xn - r xn ø
1 × 3 × 5 ×...× (2n - 1) n
1 = ×2
Now, coefficient of in RHS n!
xr Aliter
= (C 0C r + C 1C r + 1 + C 2 C r + 2 + ... + C n - r C n ) Given, (1 + x )n = C 0 + C 1 x + C 2 x 2
+ ... + C n x n …(i)
1
\ Coefficient of r in LHS = Coefficient of x n - r in æ 1ö C C
n
C
x Now, ç1 + ÷ = C 0 + 1 + 22 + ... + nn …(ii)
2n 2n 2n ! è xø x x x
(1 + x ) = C n - r =
( n - r ) ! (n + r ) ! On multiplying Eqs. (i) and (ii), we get
1 (1 + x )2n
But Eq. (iii) is an identity, therefore coefficient of in n
= ( C 0 + C 1x + C 2 x 2 + ... + C n x n )
xr x
æ C C C ö
1 ´ çC 0 + 1 + 22 + ... + nn ÷ …(iii)
RHS = coefficient of in LHS. è x x x ø
xr
Now, constant term in RHS = C 02 + C 12 + C 22 + ... + C n2
Þ C 0C r + C 1C r + C 2 C r + 2 + ... + C n -r C n
+1
2n ! (1 + x )2n
= Constant term in LHS = Constant term in
(n - r )! (n + r ) ! xn
2n !
Corollary I For r = 0, = Coefficient of x n in (1 + x )2n = 2n
Cn =
n !n !
2 2 2 2 2n !
C 0 + C 1 + C 2 + ... + C n = n ! 2n [1 × 3 × 5... (2n -1)] 2n [1 × 3 × 5... (2n - 1)]
(n !)2 = =
n !n ! n!
Chap 06 Binomial Theorem 469

2n 2n 2n
But Eq. (iii) is an identity, therefore the constant term in æ 1ö 2n C1 2n C 2 C 2n
RHS = constant term in LHS. and ç1 - ÷ = C0 - + - ... + …(ii)
è xø x x2 x 2n
2 2 2 2 2n ! {1 × 3 × 5... (2n - 1)} n
Þ C 0 + C 1 + C 2 + ... + C n = = 2 On multiplying Eqs. (i) and (ii), we get
n !n ! n!
( x 2 - 1) 2 n 2n 2n 2n
Case II If terms of the series alternately positive and 2n
=( C0 + C 1x + C 2 x 2 + ... + 2n
C 2n x 2n
)
x
negative, then 2n 2n 2n
2n C1 C2 C 2n
n
(1 - x ) = C 0 - C 1 x + C 2 x 2
- ... + ( -1) C n x n n
…(i) ´( C0 - + 2
- ... + ) …(iii)
x x x 2n
n -1 n -2
and ( x + 1) n = C 0 x n + C 1 x + C2 x + ... + C n …(ii) Now, constant term in RHS
Then, multiplying Eqs. (i) and (ii) and equate the = ( 2 n C 0 )2 - ( 2 n C 1 )2 + ( 2 n C 2 )2 - ... + ( 2 n C 2n )2
coefficient of suitable power of x on both sides. ( x 2 - 1) 2 n
Constant term in LHS = Constant term in
Or x 2n
1
Replacing x by in Eq. (i), we get = Coefficient of x 2n
in ( x - 1)2 2n
x
n = Coefficient of x 2 n in (1 - x 2 )2 n
æ 1ö C1 C2 C
ç1 - ÷ = C 0 - + - ... + ( - 1) n n ...(iii) 2n 2n
è x ø x x 2
xn = C n ( - 1)n = ( - 1)n × Cn

Then, multiplying Eqs. (i) and (iii) and equate the But Eq. (iii) is an identity, therefore the constant term in
RHS = constant term in LHS.
coefficient of suitable power of x on both sides. 2n
Þ( C 0 )2 - ( 2n
C 1 )2 + ( 2n
C 2 )2 - ... + ( 2n
C 2 n )2
y Example 74. Prove that = ( - 1)n × 2n
Cn
( 2n C 0 ) 2 - ( 2n C1 ) 2 +( 2n C 2 ) 2 - ... + ( 2n C 2n ) 2 = (- 1)n × 2n
Cn .
y Example 75. If (1 + x )n = C 0 + C 1 x
Sol. Since, (1 - x )2 n = 2n
C0 - 2n
C1 x + 2n
C2 x 2

+ C 2 x 2 +... + C n x n , prove that


- ... + ( - 1)2 n × 2n
C 2 n x2 n
C 0 2 - C 1 2 + C 2 2 - ... + ( -1)n × C n2 = 0 or
or (1 - x )2 n = 2n
C0 - 2n
C1 x + 2n
C 2 x 2 - ... +
C 2n x 2n 2n
n!
…(i) ( - 1)n / 2 × , according as n is odd or even.
and ( x + 1)2 n = 2n
C 0 x 2n + 2 nC 1 x 2 n - 1 + 2 nC 2 x 2 n - 2 (n / 2)! (n / 2)!
+ ... + 2 n C 2 n …(ii) Also, evaluate C 0 2 + C 1 2 + C 2 2 - ... + ( - 1)n × C n 2 for n
On multiplying Eqs. (i) and (ii), we get = 10 and n = 11
( x 2 - 1)2 n = ( 2 n C 0 - 2 n C 1 x + 2 n C 2 x 2 - ... + 2 n C 2n x 2 n ) Sol. Since, (1 - x )n = C 0 - C 1x + C 2 x 2 -... + ( - 1)n C n x n …(i)
´ ( 2 n C 0 x 2 n + 2 n C 1 x 2 n - 1 + 2 n C 2 x 2 n - 2 + ... + 2 n C 2 n ) and ( x + 1)n = C 0 x n + C 1 x n - 1 + C 2 x n - 2 +... + C n …(ii)

Now, coefficient of x 2 n in RHS …(iii) On multiplying Eqs. (i) and (ii), we get
2n 2 2n 2 2n
= ( C 0 ) - ( C 1 ) + ( C 2 ) - ... + ( C 2 n ) 2 2n 2 (1 - x 2 )n = {C 0 - C 1x + C 2 x 2
- ... + ( -1)n C n x n }
´ ( C 0 x n + C 1 x n - 1 + C 2 x n - 2 +... + C n ) …(iii)
Now, LHS can also be written as (1 - x 2 )2 n .
\ General term in LHS, Tr = 2n
C r ( - x 2 )r Now, coefficient of x n in RHS
+1
2n = C 02 - C 12 + C 22 - ... + ( - 1)n C n2
Putting r = n , we get Tn + 1 = ( - 1)n × Cn x 2n
General term in LHS = Tr +1 =
n
C r ( - x 2 ) r = n C r ( -1)r x 2r
Þ Coefficient of x 2 n in LHS = ( - 1)n × 2n
Cn
Putting 2r = n , we get r = n / 2
But Eq. (iii) is an identity, therefore coefficient of x 2 n in
\ T (n / 2 ) + 1 = n C n / 2 ( - 1)n / 2 x n
RHS = coefficient of x 2 n in LHS
\ Coefficient of x n in LHS = n C n / 2 ( - 1)n / 2
2n 2 2n 2 2n 2 2n 2
Þ ( C 0 ) - ( C 1 ) + ( C 2 ) - ... + ( C 2 n ) n!
= ( - 1)n / 2 ×
2n
= ( - 1)n × Cn ( n / 2) ! ( n / 2) !
Aliter ì0, if n is odd
ï éQ æ odd ö ! = ¥ ù
Since, (1 + x )2 n = =í ê çè 2 ÷ø
2n 2n 2n
C0 + C 1x + C 2x 2 n ! ú
( - 1)n / 2 , if n is even ë û
+ ... + 2n
C 2 n x 2 n …(i) ï ( n / 2) ! ( n / 2) !
î
470 Textbook of Algebra

But Eq. (iii) is an identity, therefore coefficient of x n in RHS 2. If sum of the lower suffixes of binomial
= coefficient of x n in LHS. coefficients in each term is same.
Þ C 02 - C 12 + C 22 - ... + ( - 1)n C n2 i.e., C 0 C n + C 1 C n - 1 + C 2 C n - 2 + ... + C n C 0
ì0 , if n is odd Here, 0 + n = 1 + (n - 1) = 2 + (n - 2 ) = ... = n + 0 = n
ï
=í n! Case I If each term of series is positive, then
( - 1)n / 2 , if n is even
ï ( / 2 ) ! ( n / 2) ! 2
î n (1 + x ) n = C 0 + C 1 x + C 2 x +... + C n x n
…(i)
Now, for n = 10, 2
10 !
and (1 + x ) n = C 0 + C 1 x + C 2 x +... + C n x n
…(ii)
C 02 - C 12 + C 22 - ... + C 10
2
= ( - 1)10 / 2 = - 252
5!5! Then, multiplying Eqs. (i) and (ii) and equate the
[Q10 is even] coefficient of suitable power of x on both sides.
and from n = 11, y Example 76. Prove that
m +n
C 02 - C 12 + C 22 - ... - C 11
2
=0 [Q11 is odd] Cr = m Cr + m Cr -1 n C1 + m
C r -2 n C 2 + ... + n C r
Aliter if r < m , r < n and m, n, r are positive integers.
n 2 n
Since, (1 + x ) = C 0 + C 1x + C 2 x +... + C n x …(i) Sol. Here, sum of lower suffixes of binomial coefficients in
each term is r .
1
Replacing x by - , then we get i.e. r = r - 1 + 1 = r - 2 + 2 = ... = r = r
x
Since,
n
æ 1ö C1 C 2 C
+ 2 - ... + ( - 1)n nn (1 + x )m = mC 0 + mC 1 x +... + mC r - 2 x r - 2 + mC r - 1 x r -1
ç1 - ÷ = C 0 - …(ii)
è x ø x x x m r m
+ C r x + ... + C m x …(i) m

On multiplying Eqs. (i) and (ii), we get and (1 + x ) = C 0 + C 1 x + C 2 x 2 + ... + n C r x r


n n n n
2 n
( x - 1) 2 + ... + n C n x n ...(ii)
n
= (C 0 + C 1 x + C 2 x + ... + C n x n ) ´
x On multiplying Eqs. (i) and (ii), we get
æ C1 C 2 n Cn ö
(1 + x )m + n = ( mC 0 + mC 1x + ... + mC r - 2 x r - 2 + mC r - 1x r - 1
çC 0 - + 2 - ... + ( - 1) n ÷ …(iii)
è x x x ø + mC r x r + ... + mC m x m ) ´ (n C 0 + n C 1x + n C 2 x 2
Now, constant term in RHS + ... + n C r x r + ... + n C n x n ) …(iii)
r
= C 10 - C 12 + C 22 - ... + ( -1)n C n2 Now, coefficient of x in RHS
= mC r × n C 0 +m C r - 1 × n C 1 +m C r - 2 × n C 2 + ... + mC 0 × n C r
\Constant term in LHS
= mC r + mC r -1 × n C 1 +m C r -2 × n C 2 + ... + n C r
( x 2 - 1)n
= Constant term in Coefficient of x r in LHS = m +n
Cr
xn
2
But Eq. (iii) is an identity, therefore coefficient of x r in LHS
= Coefficient of x n in ( x - 1)n = coefficient of x r in RHS.
= Coefficient of x n in n C n / 2 ( x 2 )n - (n / 2 ) ( - 1)n / 2 Þ m +n
C r = mC r + mC r -1×
n
C 1 + mC r -2 × n C 2 + ... + n C r
= ( - 1)n / 2 × n C n / 2 Case II If terms of the series alternately positive and
n! negative, then
= ( - 1)n / 2 ×
( n / 2) ! ( n / 2) ! (1 - x ) n = C 0 - C 1 x + C 2 x 2 - ... + ( - 1) n C n x n …(i)
ì 0, if n is odd
and (1 + x ) n = C 0 + C 1 x + C 2 x 2 + ... + C n x n …(ii)
ï
=í n!
( -1)n / 2 × , if n is even Then, multiplying Eqs. (i) and (ii) and equate the
ï ( n / 2 ) ! ( n / 2) !
î
coefficient of suitable power of x on both sides.
But Eq. (iii) is an identity, therefore the constant term in
RHS = constant term in LHS. y Example 77. If (1 + x )n = C 0 + C 1 x
Þ C 02 - C 12 + C 22 - ... + ( -1)n C n2 + C 2 x 2 +... + C n x n , prove that
ì0, if n is odd C 0 C n - C 1 C n - 1 + C 2 C n - 2 - ... + ( - 1)n C n C 0 = 0 or
ï
=í n ! n!
( -1)n / 2 × , if n is even ( - 1)n / 2 , according as n is odd or even.
ï ( n / 2) ! ( n / 2) !
î (n / 2)! (n / 2)!
Chap 06 Binomial Theorem 471

Sol. Given, (1 + x )n = C 0 + C 1x + C 2 x 2 +... + C n - 2 x n - 2 On multiplying Eqs. (i) and (ii), then we get
n (1 + x )2n - 1 = ( C 1 + 2C 2 x + 3C 3 x 2 +... + n C n x n -1 )
+ Cn - 1 x n - 1 + Cn x n …(i)
and (1 - x )n = C 0 - C 1x + C 2 x 2 - ... + ( - 1)n C n x n …(ii) ´ (C 0 x n + C 1 x n - 1 + C 2 x n - 2 + C 3 x n - 3 +... + C n ) …(iii)

On multiplying Eqs. (i) and (ii), we get Now, coefficient of x n - 1 on RHS


(1 - x 2 )n = ( C 0 + C 1x + C 2 x 2 +... + C n - 2 x n - 2 = C 12 + 2C 22 + 3C 32 + ... + n C n2
+ C n - 1 x n - 1 + C n x n ) ´ (C 2 - C 1x + C 2 x 2
- and coefficient of x n - 1 on LHS
... + ( - 1)n C n x n ) ...(iii) (2n - 1) !
= n × 2n - 1C n - 1 = n ×
( n - 1) ! n !
Now, coefficient of x n in RHS
( 2n - 1) ! ( 2n - 1) !
= C 0C n - C 1 C n - 1 + C 2 C n - 2 - K + ( - 1)n C n C 0 = =
(n - 1) ! (n - 1) ! {(n - 1) !)2 }
Now, general term in LHS,
But Eq. (iii) is an identity, therefore the coefficient of x n - 1
Tr + 1 = n C r ( - x 2 )r = ( - 1)r × n C r x 2 r
in RHS = coefficient of x n -1 in LHS.
Putting 2r = n , we get
2 2 2 2 (2n - 1) !
r = n /2 Þ C 1 + 2C 2 + 3C 3 + ... + nC n =
{(n - 1) !}
Now, Tn / 2 + 1 = ( -1)n / 2 × n C n / 2 x n
\ Coefficient of x n in LHS = ( -1)n / 2 . n C n / 2 y Example 79. If (1 + x )n = C 0 + C 1 x + C 2 x 2 +...+C n x n ,
n! C2 C2 C2 (2n + 1)!
= ( - 1)n / 2 × prove that C 0 2 + 1 + 2 + ... + n = .
( n / 2) ! ( n / 2) ! 2 3 n + 1 {(n + 1)!} 2
ì0 if n is odd
Sol. Given, (1 + x )n = C 0 + C 1 x + C 2 x 2 +... + C n x n
ï
=í n !
( -1)n / 2 × , if n is even Integrating both sides w.r.t. x within limits 0 to x, then we
ï ( n / 2) ! ( n / 2) !
î get
But Eq. (iii) is an identity, therefore the coefficient of x n in x x
ò0 (1 + x ) ò0 ( C 0 + C 1 x + C 2 x
n 2
dx = + ... + C n x n ) dx
RHS = coefficient of x n in LHS.
Þ C 0C n - C 1C n - 1 + C 2 C n - 2 - ... + ( - 1)n C n C 0 (1 + x )n + 1 - 1 C x2 C x3 C xn + 1
= C 0x + 1 + 2 + ... + n …(i)
(1 + n ) 2 3 n +1
ì0, if n is odd
ï and ( x + 1)n = C 0 x n + C 1 x n - 1 + C 2 x n - 2 + ... + C n …(ii)
=í n!
( -1)n / 2 × , if n is even
ï ( n / 2 ) ! ( n / 2) !
î
Multiplying Eqs. (i) and (ii), we get
3. If each term is the product of two binomial
1
coefficient divided or multiplied by an integer, { (1 + x )2n + 1 - (1 + x )n }
then integrating or differentiating by preceeding ( n + 1)
method. Then, multiplying two series and æ C x2 C x3 C xn + 1 ö
= çC 0 x + 1 + 2 + ... + n ÷
equate the coefficient of suitable power of x on è 2 3 n +1 ø
both sides.

y Example 78. If (1 + x )n = C 0 + C 1 x + C 2 x 2 ´ (C 0 x n + C 1 x n - 1 + C 2 x n - 2 + ... + C n ) …(iii)

+ C 3 x 3 +... + C n x n , prove that Now, coefficient of x n - 1 in RHS of Eq. (iii)


2 2 2
(2n - 1)! = C0 +
2 C1 C C
+ 2 + ... + n
C 12 + 2 C 22 + 3 C 32 + ... + nC n2 = . 2 3 n +1
((n - 1)!) 2
2 3 and coefficient of x n + 1 in LHS of Eq. (iii)
Sol. Given, (1 + x )n = C 0 + C 1x + C 2 x + C3 x +... + C n x n
1
Differentiating both sides w.r.t. x, we get = { 2n + 1C n + 1 - 0}
( n + 1)
n (1 + x )n - 1 = 0 + C 1 + 2C 2 x + 3C 3 x 2 +... + n C n x n -1 1 ( 2n + 1) !
= ×
Þ n (1 + x )n - 1 = C 1 + 2C 2 x + 3C 3 x 2
+... + n C n x n -1 ...(i) ( n + 1) ( n + 1) ! n !
and ( x + 1)n = C 0 x n + C 1x n - 1 + C 2 x n - 2 + C 3 x n - 3 ( 2n + 1) ! ( 2n + 1) !
= =
+... + C n …(ii) (n + 1) ! (n + 1) ! {(n + 1) !} 2
472 Textbook of Algebra

But Eq. (iii) is an identity, therefore coefficient of x n + 1 in [C 0 (1 + x )2n - C 1 (1 + x )2n -1 + C 2 (1 + x )2n - 2


RHS of Eq. (iii) = coefficient of x n + 1 in LHS of Eq. (iii). - C 3 (1 + x )2n - 3 + ... + ( - 1)n C n × (1 + x )n ]
C 12 C 22 C2 (2n + 1) ! = Coefficient of x n in
Þ C 02 + + + ... + n =
2 3 n + 1 {(n + 1) !} 2 (1 + x )n [C 0 (1 + x )n - C 1 (1 + x )n - 1 + C 2 (1 + x )n - 2
- C 3 (1 + x )n - 3 + ... + ( -1)n C n × 1]

Binomial Inside Binomial = Coefficient of x n in (1 + x )n [ ((1 + x ) - 1)n ]


= Coefficient of x n in (1 + x )n × x n d
The upper suffices of binomial coefficients are different = Constant term in (1 + x )n = 1 = RHS
but lower suffices are same.
n
y Example 80. Evaluate S n +r Cn .
r =0 Sum of the Series
n
Case I When i and j are independent.
Sol. S n + rCn = n
Cn + n +1
Cn + n+2
C n + ... + 2n
Cn
r =0 In this summation, three types of terms occur, when
= Coefficient of x n in i < j , i = j and i > j,
n ì ö üï
ï æ
n n n
[ (1 + x )n + (1 + x )n + 1 + (1 + x )n + 2 + ... + ... + (1 + x )2n ] i.e., S S a i a j = S ía i ç S a j ÷ ý
i=0 j =0 i=0 ç ÷
é (1 + x )n [(1 + x )n +1 - 1] ù ïî è j = 0 ø ïþ
= Coefficient of x n in ê ú 2 2
ë (1 + x ) - 1 û n n æ n ö æ n ö
= S a i S a j = ç S a i ÷ or ç S a j ÷
= Coefficient of x n + 1 in [ (1 + x )2n + 1 - (1 + x )n ] i=0 j =0 çi = 0 ÷ çj =0 ÷
è ø è ø
2n + 1 2n + 1 2
= Cn + 1 - 0 = Cn n n æ n ö
Corollary I S S n C i n C j = ç S n C i ÷
i=0 j =0 ç ÷
y Example 81. If (1 + x )n = C 0 + C 1 x èi = 0 ø
+ C 2 x 2+ ... + C n x n , prove that = (2 n ) 2 = 2 2n
C 0 × 2n C n - C 1 × 2n -2 C n + C 2 × 2n - 4 C n - ... = 2n
y Example 83. If (1 + x )n = C 0 + C 1 x
2n 2n - 2 2n - 4
Sol. LHS = C 0 × Cn - C1 × Cn + C 2 × C n - ...
+ C 2 x 2 +... + C n x n , find the values of the following.
n n
= Coefficient of x n in (i) S S (C i + C j )
i =0 j =0
[ C 0 (1 + x )2n - C 1 (1 + x )2n - 2 + C 2 (1 + x )2n - 4 - ... ]
n n
= Coefficient of x n in (ii) S S (i + j ) C i C j
i =0 j =0
[C 0 (1 + x )2 ] n - C 1 [(1 + x )2 ] n - 1 + C 2 [ (1 + x )2 ] n - 2 - ...]
n n n n n n
= Coefficient of x n in [[ (1 + x )2 - 1]n ] Sol. (i) S S (C i + C j ) = i S= 0 j S= 0 C i + i S= 0 j S= 0
i =0 j =0
Cj
n 2 n
= Coefficient of x in (2x + x ) n æ n ö n æ n ö
= Constant term in (2 + x ) = 2 = RHS n n = S ç S C i ÷÷ + i S= 0 çç j S= 0 C j ÷÷
j = 0 çi = 0
è ø è ø
n 2
y Example 82. If (1 + x ) = C 0 + C 1 x + C 2 x n n

3 n
= S (2n ) + S (2n ) = (n + 1) × 2n + (n + 1) × 2n
+ C 3 x +... + C n x , prove that j =0 i =0

= 2 (n + 1) 2n = (n + 1) 2n +1
C 0 × 2n C n - C 1 × 2n -1C n + C 2 × 2n - 2
Cn - C 3 × 2n - 3
C n + ... +
n n n n n n
( - 1)n C n × n C n = 1 (ii) S S (i + j )C i C j
i =0 j =0
= S S i Ci C j
i =0 j =0
+ S S j Ci C j
i =0 j =0
2n -1 2n - 2
Sol. LHS = C 0 × 2n
Cn - C1 × Cn + C 2 × C n - C 3 × 2n - 3 C n n æ n ö n æ n ö
+ ... + ( - 1)n C n × nC n = S i C i çç j S= 0 C j ÷÷ + j S= 0 j C j ççi S= 0 C i ÷÷
i =0
= Coefficient of x n in è ø è ø
Chap 06 Binomial Theorem 473

=
n
S i C i (2 n ) + j S= 0 j C j (2 n )
n
(vii) SS (i × j ) C i C j
i =0 0 £i < j £n
n n æ n n ö
= 2n S i n C i + 2n j S= 0 j n C j çS S C i ÷÷ - S S C i
i =0 çi = 0 j =0
è ø i = j
n n Sol. (i) SS Ci =
2
= 2n S i × × n - 1C i - 1 + 2 n S
n n 0 £i < j £n
j × × n - 1C j - 1
i =0 i j =0 j n n

n n ( n + 1) S C i - i S= 0 C i
= n × 2n S n - 1C i - 1 + n × 2 n j S= 0 n - 1C j - 1
i =0
=
i =0
2
= n × 2n - 1
n -1
= n × 2 n × 2n -1 + n × 2 n × 2 n - 1
(ii) SS j Ci = S
r =0
n
Cr
2n - 1 2n 0 £i < j £n
= n × 2× 2 = n ×2
{(r + 1) + (r + 2) + (r + 3) + ... + n }
Case II When i and j are dependent. n -1
( n - r ) ( n + r + 1)
In this summation, when i < j is equal to the sum of the = S n
Cr ×
terms when i > j, if a i and a j are symmetrical. So, in this r =0 2
case n -1
n n =
1
S n C r (n 2 - r 2 + n - r )
S S ai a j
i=0 j =0
= SS ai a j + S S ai a j 2r=0
0 £i < j £n i=j n -1 n -1 n -1
(n + n ) S n C r - S r × n C r - S r 2 × nC r
1 2 1 1
+ SS ai a j =
2 r =0 2r=0 2 r=0
0 £ j <i £n
1 1
=2 SS ai a j + S S ai a j = (n 2 + n ) (2n - 1) - × n × (2n - 1 - 1)
2 2
0 £i < j £n i=j
1
n n - × n [(n - 1) (2n - 2 - 1) + 2n - 1 - 1]
S S ai a j - S S ai a j
i=0 j =0
2
Þ SS ai a j =
i=j = n ( 3n + 1) × 2 n-3

0 £i < j £n 2
When a i and a j are not symmetrical, we find the sum by Remark
Here, j and Ci are not symmetrical.
listing all the terms.
Corollary I
n n (iii) Here, i ¹ j i.e., i > j or i < j
S S n C i n C j - SS n
Ci × nC j But C i and C j are symmetrical.
SS
0 £i < j £n
n
Ci nC j =
i=0 j =0 i=j
\ S S Ci C j =2 SS Ci C j
2 i ¹ j 0 £i < j £n
n
(2 n ) 2 - S (n Ci )2
2 2n - 2n C n 2n !
æ 22n - 2n C n ö
=2ç
i=0 ÷ [from corollary I]
= = = 2 2n - 1 - è 2 ø
2 2 2 (n !) 2
= 2 2n - 2n
Cn
y Example 84. If (1 + x )n = C 0 + C 1 x (iv) SS Ci C j = SS Ci C j + S S Ci C j
+ C 2 x +... + C n x n , find the values of the following.
2 0 £i £ j £n 0 £i < j £n i = j

1 2n
(i) SS Ci (ii) SS j Ci =
2
(2 - 2n
Cn ) + 2n
C n [from corollary I]
0 £i < j £n 0 £i < j £n
1
= (22n + 2n
(iii) S S Ci C j (iv) SS Ci C j 2
Cn )
i¹j 0 £i £ j £n
(v) SS (C i ± C j )2 = SS (C i2 + C 2j ± 2 C i C j )
(v) SS (C i ± C j ) 2 0 £i < j £n 0 £i < j £n

0 £i < j £n = SS (C i2 + C 2j ) ± 2 SS Ci C j
0 £i < j £n 0 £i < j £n
(vi) SS (i + j ) C i C j
Q SS ( C i2 + C 2j )
0 £i < j £n
0 £i < j £n
474 Textbook of Algebra

n n
S S (C i2 + C 2j ) - 2 i S= 0 C i2
n
Let P= SS (i + j ) C i C j ...(i)
0 £i < j £n
i =0 j =0
=
2 Replacing i by n - i and j by n - j in Eq. (i), then we
n æ n n ö get
S ç S C i 2 + j S= 0 C 2j ÷÷ - 2 × 2nC n
i =0 ç j =0 P= SS (n - i + n - j ) C n - i C n - j
è ø 0 £i < j £n
=
2 [Q sum of binomial expansion does not
n
change if we replace r byn - r ]
S ( ( n + 1) C i +
2 2n
Cn ) - 2 × Cn 2n

=
i =0
P= SS (2n - i - j ) C i C j
2 0 £i < j £n
n n
[Q n C r = n C n - r ] …(ii)
( n + 1) S C i 2 + 2nC n i S= 0 1 - 2 × 2nC n
i =0
= On adding Eqs. (i) and (ii), we get
2
(n + 1) × 2n C n + 2n C n × (n + 1) - 2 × 2n
Cn 2P = 2n SS Ci C j
= 0 £i < j £n
2
= n × 2n C n or P =n SS Ci C j =
n 2n
(2 - 2n
Cn )
\ SS (C i ± C j ) = n × 2 2n
C n ± (2 2n
- 2n
Cn )
0 £i < j £n 2
0 £i < j £n [from corollary I]
[from corollary 1] (vii) SS (i × j ) C i C j = SS (i n C i ) ( j × n C j )
= ( n m 1) 2n
C n ± 22n ; SS (i + j )C i C j
0 £i < j £n 0 £i < j £n

0 £i < j £n = n2 SS n -1
Ci - 1 n -1
Cj -1
0 £i < j £n
Remark
SS ( Ci + Cj ) = n × 2n
= n2 ê
é 22 (n - 1) - 2n - 2 C n -1 ù
0£ i < j £ n ú [from corollary I]
êë 2 úû
(vi) SS (i + j )C i C j
æ 1 ö
0 £i < j £n = n 2 ç2 2n - 3 - × 2n - 2 C n - 1 ÷
è 2 ø
Chap 06 Binomial Theorem 475

#L Exercise for Session 4


1. The coefficient of a4 b 8 c 9d 9 in the expansion of (abc + abd + acd + bcd )10 is
10 !
(a) 10 ! (b) (c) 2520 (d) None of these
4 !8 !9 !9 !
2. If (1 + 2x + 3x 2 )10 = a 0 + a1x + a 2 x 2 + ... + a 20 x 20, then a1 equals
(a) 210 (b) 20 (c) 10 (d) None of these

3. If (1 + x + x 2 + x 3 )5 = a 0 + a1x + a 2x 2
+ ... + a15 x 15, then a10 equals
(a) 99 (b) 100 (c) 101 (d) 110
4. Coefficient of x 15 3
in (1 + x + x + x ) is 4 n

5 5 5 3
(a) S nC5 - r × nC3r
r =0
(b) S n C5r
r =0
(c) S n C2r
r =0
(d) S n C3 - r × n C5r
r =0
n
æ 1ö
5. The number of terms in the expansion of ç x 2 + 1 + 2 ÷ , n Î N is
è x ø
n +2 n +3 2 n +1 3n + 1
(a) C2 (b) C2 (c) C2 n (d) C3 n

6. If (1+ x ) 10
= a 0 + a1x + a 2 x + ...+ a10 x , then (a 0 - a 2 + a4 - a 6 + a 8 - a10 ) + (a1 - a 3 + a 5 - a 7 + a 9 )2 is equal to
2 10 2

(a) 2 9 (b) 3 9 (c) 2 10 (d) 3 10

7. If (1+ x )n = C0 + C1x + C2 x 2 + C3 x 3
+ ...+ Cn x n, n being even the value of
C0 + (C0 + C1 ) + (C0 + C1 + C2 ) + ... + (C0 + C1 + C2 + ... + Cn - 1 ) is equal to
(a) n × 2n (b) n × 2n - 1 (c) n × 2n - 2 (d) n × 2n - 3
C0 C C C Cn
8. The value of - 1 + 2 - 3 + ... + ( - 1)n is
1× 3 2 × 3 3 × 3 4 × 3 (n + 1) × 3
3 n+1 1
(a) (b) (c) (d) None of these
n+ 1 3 3 (n + 1)
æ50ö æ50ö æ50ö æ50ö æ50ö æ50ö æn ö
9. The value of ç ÷ ç ÷ + ç ÷ ç ÷ + ... + ç ÷ ç ÷ , where nCr = ç ÷ , is
è 0ø è 1ø è 1ø è 2ø è49ø è50ø è rø
2
æ 100ö æ 100ö æ 50ö æ 50ö
(a) ç ÷ (b) ç ÷ (c) ç ÷ (d) ç ÷
è 50 ø è 51 ø è 25ø è 25ø

10. If Cr stands for 4Cr , then C0 C4 - C1 C3 + C2 C2 - C3 C1 + C4 C0 is equal to


(a) C1 (b) C2 (c) C3 (d) C4
n
11. The sum
r =0
S (r + 1) ( nCr )2 is equal to

(n + 2) (2n - 1) ! (n + 2) (2n + 1) ! (n + 2) (2n + 1) ! (n + 2) (2n - 1) !


(a) (b) (c) (d)
n ! (n - 1) ! n ! (n - 1) ! n ! (n + 1) ! n ! (n + 1) !
n æ r -1 n r ö
12. S ç S Cr Cp 2p ÷ is equal to
r =1 ç
èp = 0
÷
ø
(a) 4n - 3n + 1 (b) 4n - 3n - 1 (c) 4n - 3n + 2 (d) 4n - 3n

æ 10 10 ö æ 10 10
C ö
13. çç S Cr ÷÷ çç S ( - 1)m mm ÷÷ is equal to
èr = 0 ø è m = 0 2 ø
(a) 1 (b) 2 5 (c) 2 10 (d) 2 20

14. The value of SSSS 2 is equal to


0£i < j <k<l £n
n+1 n+ 2
(a) 2 (n + 1) 3 (b) 2 × C4 (c) 2 (n + 1)4 (d) 2 × C3
Shortcuts and Important Results to Remember
n
1 (r + 1th
) term from end in the expansion of r +1 æ xö
( x + y )n = (r + 1th 13 If the coefficients of x r , x in the expansion of ç a + ÷
) term from beginning in the expansion of è bø
( y + x )n . are equal, then n = (r + 1) (ab + 1) - 1, where n, r Î N and
2 If n Cr - 1, nCr , nCr + 1 are in AP, then (n - 2 r )2 = n + 2 or a, b are constants.
n
æ b ö
1
r = (n ± (n + 2 )) for r = 2, n = 7 and for r = 5, n = 7, 14. 14 Coefficient of x m in the expansion of ç ax p + q ÷
2 è x ø
np - m
= Coefficient of Tr + 1, where r = , where p, q, n Î N
3 Four consecutive binomial coefficients can never be p+q
in AP. and a, b are constants.
4 Three consecutive binomial coefficients can never be in 15 The term independent of x in the expansion of
n
GP or HP. æ p b ö np
ç ax + q ÷ is Tr + 1, where r = , where n , p, q Î N
5 If a, b, c , d are four consecutive coefficients in the è x ø p +q
a b c
expansion of (1 + x )n , then , , are in AP. and a, b are constants.
a+ b b+c c +d
16 Sum of the coefficients in the expansion of (ax + by )n is
a c æ b ö
(i) + =2 ç ÷ (a + b)n , where n Î N and a, b are constants.
a+ b c +d èb + cø
2 17 If (1 + x )n = C0 + C1 x + C2 x 2 + ... + Cn x n and p + q = 1, then
æ b ö ac
(ii) ç ÷ > n
èb + cø (a + b) (c + d ) (i) S r × Cr × pr × q n - r
r =0
= np
2n
6 If greatest term in (1 + x ) has the greatest coefficient, n

then
n
<x<
n+1
.
(ii) S r 2 × Cr × pr × q n - r
r =0
= n 2 p2 + npq
n+1 n
18 If (1 + x )n = C0 + C1 x + C2 x 2
+ ... + Cn x n, then
7 (a) The coefficient of x n - 1 in the expansion of
2n + 1 - 1
n n
( x - 1) ( x - 2 ) ( x - 3) ... ( x - n ) = - (1 + 2 + 3 + ... + n ) (i) S r × Cr = n × 2 n -1 (ii) S Cr
=
n (n + 1) r =0 r =0 r +1 n+1
=- = - n + 1C2 n n
2
(iii) S r 2 × Cr = n (n + 1) 2 n - 2 (iv) S (-1)r × r × Cr =0
(b) The coefficient of x n - 1 in the expansion of r =0 r =0
n r
( x + 1) ( x + 2 ) ( x + 3) K ( x + n )
n (n + 1) n + 1 (v) S (-1) Cr =
1
= (1 + 2 + 3 + K + n ) = = C2 r =0 r +1 n+1
2 n
8 The number of terms in the expansion of (vi) S (-1)r Cr
r =0
= 1+
1 1
+ +K+
1
r 2 3 n
ìn + 2
ïï 2 , if n is even n
n
( x + a) + ( x - a) = ín
(vii) S (- 1)r × r 2 × Cr =0
ï n + 1 , if n is odd r =0
ïî 2 n

9 The number of terms in the expansion of (viii) S (- 1)r × (a - r ) (b - r ) Cr


r =0
= 0, " n > 3
ìn n
ïï 2 , if n is even
n
( x + a) - ( x - a) = ín (ix) S (- 1)r (a - r ) (b - r ) (c - r ) Cr
r =0
= 0, " n > 3
ï n + 1 , if n is odd n
ïî 2
(x) S (- 1)r (a - r )3 Cr
r =0
= 0, " n > 3
10 The number of terms in the expansion of multinomial
n k
( x1 + x2 + x3 + ... + xm )n , when x1, x2 , x3 , ... , xm Î C and
(xi) S r(r - 1) (r - 2 )... (r - k + 1) Cr x r -k = d k (1 + x )n
n Î N, is n + m - 1Cm - 1. r =0 dx
n
d2
11 The number of terms in the expansion of for k = 2 , S r (r - 1) Cr = 2 [(1 + x )n ]x = 1 = n(n - 1) 2 n - 2
n r =0 dx
æ p b ö
ç ax + + c ÷ , where n, p Î N and a, b, c are n
è xp ø and for k = 3; S r(r - 1) (r - 2 ) (-1)r - 3Cr
r =0
constants, is 2n + 1.
d3
12 If the coefficients of pth and qth terms in the expansion of = [(1 + x )n ]x = - 1 = 0
(1 + x )n are equal, then p + q = n + 2, where p, q, n Î N. dx 3
Chap 06 Binomial Theorem 477

JEE Type Solved Examples :


Single Option Correct Type Questions
n This section contains 10 multiple choice examples. sinmq = m C 1(cos q )m -1 × sin q -m C 3 (cos q )m - 3 × sin 3 q +...
Each example has four choices (a), (b), (c) and (d) out of …(ii)
which ONLY ONE is correct. On adding Eqs. (i) and (ii), we get
æ 2n + 1ö æ 2n + 1ö æ 2n + 1ö cosmq + sin mq =m C 0 (cos q )m +m C 1(cos q )m -1 × sin q
l Ex. 1 If ç ÷ +ç ÷ +ç ÷ + ... = 170 , then n
è 0 ø è 3 ø è 6 ø -m C 2 (cos q )m - 2 sin 2 q - mC 3 (cos q )m - 3 sin 3 q
equals
(a) 2 (b) 4 (c) 6 (d) 8 æ pö
+m C 4 (cos q )m - 4 sin 4 q +...sin çmq + ÷
Sol. (b) Q (1 + x )2n +1 = 2n +1C 0 + 2n +1C 1x + 2n +1 C 2 x 2 + 2n +1 C 3 x 3 è 4ø

+ 2n +1C 4 x 4 + 2n +1 C 5 x 5 + 2n +1 C 6 x 6 + ... ì m C 0 +m C 1 tan q - mC 2 tan 2 q -m C 3 tan 3 q ü


= (cos q )m í ý
m 4 m 5
Putting x = 1, w, w2 (where w is cube root of unity) and î + C 4 tan q + C 5 tan q - ... þ
adding, we get p æ ( m + 1) p ö 1
Putting q = , 2 sin ç ÷ = m/2
22n +1 + (1 + w) 2n +1+ (1 + w2 )2n +1 = 3( 2n +1C 0 4 è 4 ø 2
+ 2n +1C 3 + 2n +1 C 6 + ... ì(m C 0 +m C 1 -m C 2 -m C 3 ) + (m C 4 +m C 5 -m C 6 -m C 7 )ü
( 2n +1) í ý
Þ 22n +1 - w2 - w2n +1 = 3( 2n +1C 0 + 2n +1 C 3 m m m m
î +...+( C m - 3 + C m - 2 - C m -1 - C m ) þ
+ 2n +1C 6 + ...) [Q1 + w + w2 = 0] m m m m m m m
Q( C 0 + C 1 - C 2 - C 3 ) + ( C 4 + C 5 - C 6 - C 7 ) m

2n +1 1 +... = 0 [given]
Þ C 0 +1+ 2n C 3 + 2n +1 C 6 + ... =
3 æ ( m + 1) p ö ( m + 1) p
\ sin ç ÷ =0 Þ = kp
(22n +1 - w2 (2n +1) - w2n +1 ) è 4 ø 4
æ2n + 1ö æ2n + 1ö æ2n + 1ö 1
Þ ç ÷+ç ÷+ç ÷ + ... = or m = 4k - 1,"k Î I
è 0 ø è 3 ø è 6 ø 3
(22n +1 - w2 (2n +1) - w2n +1 ) l Ex. 3 If coefficient of x n in the expansion of (1 + x ) 101
1 (1 - x + x 2 ) 100 is non-zero, then n cannot be of the form
Þ 170 = (22n +1 - w2 (2n +1) - w2n +1 )
3 (a) 3 l + 1 (b) 3l (c) 3 l + 2 (d) 4 l + 1
1
For n = 4, 170 = (512 - 1 - 1) = 170 [Qw3 = 1] Sol. (c) Q(1 + x )101(1 - x + x 2 )100 = (1 + x )((1 + x )(1 - x + x 2 ))100
3
= (1 + x )(1 + x 3 )100
Hence, n=4
= (1 + x )(1 +100 C 1x 3 +100 C 2 x 6 +100 C 3 x 9 +...+...+100 C 10 x 300 )
l Ex. 2 ( m C 0 +m C 1 -m C 2 -m C 3 ) Clearly, in this expression x 3 will present if n = 3l or
n = 3l + 1. So, n cannot be of the form 3l + 2.
+ ( m C 4 +m C 5 -m C 6 -m C 7 ) + ... = 0
if and only if for some positive integer k, m is equal to m æ10 ö æ 20 ö
p
lEx. 4 The sum å ç ÷ ç ÷ , (where = 0, if p < q) is
(a) 4k (b) 4k + 1 (c) 4k - 1 (d) 4k + 2 i =0è i ø è m - i ø q
maximum when m is
Sol. (c) If q Î R and i = -1 , then (cos q + i sin q )n (a) 5 (b) 10 (c) 15 (d) 20
= C 0 (cos q ) +m C 1(cos q )m -1(i sin q )
m m
m æ10ö æ 20 ö m 10 20
+m C 2 (cos q )m - 2 (i sin q )2 +...+ mC m (i sin q )m Sol. (c) å ç ÷ç ÷ = å C i C m -i
i =0 è i ø èm - i ø i = 0
(cos mq + i sin mq ) = [ mC 0 (cos q )m -m C 2 (cos q )m - 2 × sin 2 q 10 10 10 10
= C 0 × 20C m + C 1 ×20 C m -1 + C 2 ×20 C m - 2 +...+ C m ×20C 0
m m-4 4 m m -1
+ C 4 (cos q ) sin q - ...] + i [ C 1(cos q )
= Coefficient of x m in the expansion of product
m m-3 3
×sin q - C 3 (cos q ) sin q +...] (1 + x )10 (1 + x )20
[using Demoivre’s theorem] = Coefficient of x m in the expansion of (1 + x )30 = 30
Cm
Comparing real and imaginary parts, we get 30
To get maximum value of the given sum, C m should be
cosmq = m C 0 (cos q )m -m C 2 (cos q )m - 2 sin 2 q
30
+m C 4 (cos q )m - 4 sin 4 q -... …(i) maximum. Which is so, when m = = 15
2
478 Textbook of Algebra

n -1
l Ex. 5 If C r = (k 2 - 3 ) × n C r + 1 then k belongs to On subtracting Eq. (ii) from Eq. (i), then we get
(a) ( -¥, - 2] (b) [ 2, ¥ ) ì æ x ö æ x ö2 ü
ï1 + ç ÷ +ç ÷ ï
(c) [ - 3 , 3 ] (d) ( 3 , 2] S ï è1 + x ø è1 + x ø ï
= (1 + x )100 í ý
Sol. (d) Q n -1 2
C r = ( k - 3) × C r + 1 n (1 + x ) ï æ x ö
100
æ x ö ï
101
+...+ ç ÷ - 101 ç ÷
n -1 ïî è1 + x ø è 1 + x ø ïþ
Cr r +1
Þ k2 -3 = = …(i)
n
C r +1 n ì æ æ x ö101 ö ü
ï 1 × çç1 - ç ÷ ÷÷ ï
Þ 0 £ r £ n -1 è1 + x ø ø 101
100 ï è æ x ö ï
= (1 + x ) í - 101ç ÷ ý
Þ 1 £ r +1 £ n è1 + x ø
ï 1 - æç
x ö
÷ ï
1 r +1 ï è1 + x ø ï
Þ £ £1 î þ
n n
1 \ S = (1 + x )102 - x 101(1 + x ) - 101x 101
Þ £ ( k 2 - 3) £ 1
n and coefficient of x 50 in S = 102 C 50 .
1
Þ 3+ £ k2 £ 4 or 3 < k 2 £ 4 [here, n ³ 2]
n l Ex. 8 The largest integer l such that 2 l divides
n
\ k Î [ -2, 3 ) È ( 3, 2] 3 2 - 1, n Î N is
(a) n - 1 (b) n (c) n + 1 (d) n + 2
6 2n 2n
æ 1 ö æ b ö Sol. (d) Q3 - 1 = ( 4 - 1) -1
l Ex. 6 If ç x + + 1÷ = a 0 + ç a 1 x + 1 ÷
è ø è xø 2n 2n 2n -1 2n n n
-2
x = (4 - C1 × 4 + C 2 × 42 -...- 2 C 2n -1 × 4 + 1) - 1
æ b ö æ b ö n n
-1 2n (2n - 1) 2n - 2
+ç a 2 x 2 + 2 ÷ +...+ç a 6 x 6 + 6 ÷ , = 4 2 - 2n × 4 2 + ×4 -...-2n × 4
è x 2ø è x6 ø n+1 n+1
2
= 2n + 2 (22 -n - 2
- 22 - 4 + ... - 1) = 2n + 2 (Integer)
the value of a 0 is
n
(a) 121 (b) 131 Hence, 32 - 1 is divisible by 2n + 2 × l = n + 2
(c) 141 (d) 151
6 6 6 r l Ex. 9 The last term in the binomial expansion of
æ 1 ö æ 1ö
Sol. (c) Q ç x + + 1÷ = å C r ç x + ÷ for constant term r n log 3 8
è x ø r =0 è xø æ3 1 ö æ 1 ö
must be even integer. ç 2 - ÷ is ç 3 ÷ , the 5th term from beginning is
è 2ø è3 9 ø
\ a 0 = 6 C 0 + 6C 2 ´ 2 C 1 + 6C 4 ´ 4 C 2 + 6C 6 ´ 6C 3
(a) 10 C 6 (b) 2 10C 4
= 1 + 30 + 90 + 20 = 141 1
(c) × 10C 4 (d) None of the above
50 2
l Ex. 7 The coefficient of x in the series n
æ 1 ö
101 Sol. (a) Since, last term in the expansion of ç 3 2 - ÷
å rx r -1 (1 + x ) 101-r is è 2ø
r =1 log 3 8 n log 3 8
(a) 100 C 50 (b) 101C 50 æ 1 ö æ 1 ö æ 1 ö
=ç 3 ÷ Þ n Cn × ç - ÷ = ç 3 ÷
è3× 9 ø è 2ø è3× 9 ø
(c) 102 C 50 (d) 103 C 50
n/2 log 3 8
101 æ1ö æ 1 ö 3
Sol. (c) Let S = å rx r -1(1 + x )101-r Þ ( -1)n × ç ÷ = ç 5/3 ÷ = (3-5 / 3 )log 3 2
r =1 è2ø è3 ø
5 5
= (1 + x )100 + 2x (1 + x )99 + 3x 2 (1 + x )98 +...+101x 100 - ´ 3 ´ log 3 2 -5 æ1ö
=3 3 = 3-5 log 3 2 = 3log 3 2 = 2-5 = ç ÷
ìï è2ø
æ x ö üï
2 100
æ x ö æ x ö
S = (1 + x )100 í1 + 2 ç ÷ + 3ç ÷ +...+101 ç ÷ ý
ïî è1 + x ø è1 + x ø è1 + x ø ï æ1ö
n/2
æ1ö
5
þ Þ ( -1)n × ç ÷ = ç ÷ \ n = 10
…(i) è2ø è2ø
ìæ x ö æ x ö
2 ü æ 1 ö
4
ïç ÷ + 2ç ÷ ï Now, 5th term from beginning = 10 C 4 ( 3 2 )6 ç - ÷
Sx 100 ï è 1 + x ø è1 + x ø ï è 2ø
\ = (1 + x ) í ý …(ii)
(1 + x ) ï æ x ö
3
æ x ö ï
101
1
+3 +...+101 ç = 10 C 4 × 22 × = 10 C 4 = 10 C 6
ïî çè 1 + x ÷ø ÷
è 1 + x ø ïþ 2 2
Chap 06 Binomial Theorem 479

n n
l Ex. 10 If f ( x ) = å {r 2 ( n C r -n C r -1 ) + ( 2r + 1) n C r } = å ((r + 1)2 × n C r - r 2 ×n C r -1 )
r =1 r =1
l
and f (30 ) = 30( 2 ) , then the value of l is = ( n + 1) 2 × n C n - 12 × n C 0
(a) 3 (b) 4 (c) 5 (d) 6 = (n + 1)2 - 1 = (n 2 + 2n )
n
Sol. (c) Here, f ( x ) = å {r 2 (n C r -n C r -1 ) + (2r + 1)n C r } \ f (30) = (30)2 + 2(30) = 960
r =1
n = 30 ´ 32 = 30(2)5 = 30(2)l [given]
2 n 2 n
= å (r + 2r + 1) C r - r × C r -1
r =1 Hence, l =5

JEE Type Solved Examples :


More than One Correct Option Type Questions
n This section contains 5 multiple choice examples. Each æpö æ 5p ö
\ S 5 ç ÷ = 24 × sin ç ÷ = 16,
example has four choices (a), (b), (c) and (d) out of which è2ø è 2 ø
more than one may be correct.
æ pö æ 7p ö
n S 7 ç - ÷ = 26 × sin ç - ÷ = 26 ´ -1 ´ -1 = 64
æ 1ö è 2ø è 2 ø
l Ex. 11 Let a n = ç1 + ÷ . Then for each n Î N
è nø S 50 ( p ) = 249 × sin(50p ) = 0
(a) an ³ 2 (b) an < 3 (c) an < 4 (d) an < 2 and S 51( - p ) = 250 × sin( -51p ) = 0
Sol. (a, b, c)
æ 1ö
n
æ1ö n æ1ö
2 l Ex. 13 If a + b = k , when a , b > 0 and
Q an = ç1 + ÷ = n C 0 +n C 1 × ç ÷ + å n C r ç ÷
è nø èn ø r =2 èn ø n
2 S(k , n ) = å r 2 ( n C r )a r × b n -r , then
n
n æ1ö r =0
= 2 + å Cr ç ÷
r =2 èn ø , ) = 3(3a 2 + ab )
(a) S (13 (b) S ( 2, 4 ) = 16( 4a 2 + ab )
\ an ³ 2 for all n Î N (c) S (3,5) = 25(5a 2 + ab ) (d) S ( 4,6 ) = 36(6a 2 + ab )
n
æ 1ö Sol. (a, b)
Also, lim ç1 + ÷ = e = 2.7182 ...
n ®¥ è nø n
Q S(k ,n ) = å r 2 × (n C r )ar × bn -r
\ an < e r =0
n æn ö æ a ör
Finally, 2 £ an < e = bn å r 2 × ç r × n -1C r -1 ÷ × ç ÷
r =0 ç ÷ èb ø
n è ø
l Ex. 12 Let Sn ( x ) = å n C k sin(kx ) cos((n - k ) x ) then n
æa ö
r
k =0 = nbn å ((r - 1) + 1)n -1C r -1 × ç ÷
r =0 èb ø
æpö æ -p ö
(a) S 5 ç ÷ = 16 (b) S7 ç ÷ = 64 n
æa ö
r
è 2ø è 2 ø = nbn å ((n - 1) × n - 2C r - 2 +n -1 C r -1 ) ç ÷
r =0 èb ø
(c) S 50 ( p ) = 0 (d) S 51( - p ) = -250 2 n r -2
æa ö æa ö
Sol. (a, b, c) = nbn × (n - 1) × ç ÷ å n - 2C r - 2 ç ÷
n
èb ø r =0 èb ø
r -1
Q Sn ( x ) = å n C k sin(kx )cos((n - k )x ) …(i) æ ö n
a æa ö
k =0 +nbn × ç ÷ å n -1C r - ç ÷
èb ør =0 èb ø
Replace k by n - k in Eq. (i), then 2 n-2 n -1
æ a ö æ a ö æ a ö æ aö
n = nbn × (n - 1)ç ÷ ç1 + ÷ + nbn × ç ÷. ç1 + ÷
Sn ( x ) = å n C n -k sin((n - k )x )cos(kx ) èb ø è b ø èb ø è b ø
k =0
n = n (n - 1)a 2k n - 2 + nak n -1
n
or Sn ( x ) = å C k sin((n - k )x )cos(kx ) …(ii) = n 2a 2k n - 2 + nak n - 2 (k - a ) = n 2a 2k n - 2 + nabk n - 2
k =0

On adding Eqs. (i) and (ii), we get \S(1, 3) = 9a 2 + 3ab = 3(3a 2 + ab ) [Q a + b = k ]


n S(2, 4 ) = 16( 4a 2 + ab )
2Sn ( x ) = å n C k ×sin(nx ) = 2n × sin(nx )
k =0 S(3, 5) = 135(5a 2 + ab )
Þ Sn ( x ) = 2n -1 × sin(nx ) S( 4, 6) = 1536(6a 2 + ab )
480 Textbook of Algebra

l Ex. 14 The value of x, for which the ninth term in the Þ l = 2, - 2 / 5


10 Þ x = 102 , 10-2 / 5 [from Eq. (i)]
ì 1 ü
ï 10 2 log10 x ï
expansion of í +x×x ý is 450 is equal to
5 log10 x
ïî ( x ) ïþ
l Ex. 15 For a positive integer n, if the expansion of
2 -2 /5 æ 5 4ö
(a) 10 (b) 10 (c) 10 (d) 10 ç 2 + x ÷ has a term independent of x, then n can be
l èx ø
Sol. (b, d) Let log10 x = l Þ x = 10 …(i)
(a) 18 (b) 27 (c) 36 (d) 45
Given, 2 T 9 = 450 n
æ ö æ 5 ö
ç 10 ÷ Sol. (a, b, c, d) Let (r + 1)th term of ç + x 4 ÷ be independent
10 l 1/ 2 8 èx2 ø
Þ C8 ×ç ÷ × (10 × 10 ) = 450 n -r
l2 æ 5 ö
ç 52 ÷ of x. We have, Tr + 1 = n C r ç 2 ÷ ( x 4 )r = n C r × 5n -r × x 6r - 2n
è 10 ø
èx ø
10 10 l
Þ C2 × × 108 × 104 = 450 For this term to be independent of x,
5 l2
10
Þ
2
108 l + 4 - 5 l = 1 = 100 6r - 2n = 0 or n = 3r
Þ 8l + 4 - 5l2 = 0 For r = 6, 9, 12, 15,
Þ 5l2 - 8l - 4 = 0 n = 18,27,36, 45.

JEE Type Solved Examples :


Passage Based Questions
2n 4n r
This section contains 2 solved passages based upon each æ 1 ö æ1ö
S ar
n
1
of the passage 3 multiple choice examples have to be ç1 + + 2 ÷ = ç ÷
è x x ø r =0 èx ø
answered. Each of these examples has four choices (a), (b),
4n
(c) and (d) out of which ONLY ONE is correct.
Þ (1 + x + x 2 )2n = S ar
r =0
x 4n - r …(ii)
Passage I (Ex. Nos. 16 to 18) 4n 4n
2n From Eqs. (i) and (ii), we get S ar x r = S ar x 4n - r
Consider (1 + x + x 2 ) n = S
r=0
a r x r , where a 0 , a1 , r =0 r =0
4n - r
Equating the coefficient of x on both sides, we get
a 2 , ... , a 2n are real numbers and n is a positive integer.
n -1 a 4n - r = ar for 0 £ r £ 4n
16. The value of S a 2r
r =0
is Hence, ar = a 4n - r
Putting x = 1 in Eq. (i), then
9n - 2a 2n - 1 9n - 2a 2n + 1 4n
(a) (b)
4 4 S ar
r =0
= 32 n = 9n …(iii)
9n + 2a 2n - 1 9n + 2a 2n + 1 4n
(c)
4
(d)
4 Putting x = - 1 in Eq. (i), then S
r =0
( - 1)r ar = 1 …(iv)
n
17. The value of S a 2r - 1 is 16. (b) On adding Eqs. (iii) and (iv), we get
r =1 2 (a 0 + a 2 + a 4 + ... + a 2n - 2 + a 2n + ... + a 4n ) = 9n + 1
9n - 1 9n - 1 9n + 1 9n + 1 Þ2[2 (a 0 + a 2 + a 4 + ... + a 2n - 2 ) + a 2n ) = 9n + 1
(a) (b) (c) (d)
2 4 2 4 [Q ar = a 4n - r ]
18. The value of a 2 is 9n - 2a 2n + 1
\ a 0 + a 2 + a 4 + ... + a 2n - 2 =
4n + 1 3n + 1 4
(a) C 2 (b) C2
n -1 n
2n + 1
(d) n + 1C 2 9 - 2a 2n + 1
(c) C2 Þ S
r=0
a 2r =
4
Sol.
4n
17. (b) On subtracting Eq. (iv) from Eq. (iii), we get
We have, (1 + x + x 2 )2n = S
r =0
ar x r …(i)
2 (a1 + a 3 + a 5 + ... + a 2n - 1 + a 2n + 1 + ... + a 4n -1 ) = 9n - 1
1
Replacing x by in Eq. (i), we get Þ 2 [2 ( a1 + a 3 + a 5 + ... + a 2n - 1 ] = 9n - 1 [Q ar = a 4n -r ]
x
Chap 06 Binomial Theorem 481

9n - 1 21. The value of K + G is


\ a1 + a 3 + a 5 + ... + a 2n - 1 =
4 (a) 2 S - 2 (b) 2 S - 1
n
9n - 1 (c) 2 S + 1 (d) 2 S + 2
Þ S a 2r - 1
r =1
=
4 Sol.
18. (c)Qa 2 = Coefficient of x 2 in (1 + x + x 2 ) 2n 30 30 + r 30 30 + r
C r (2r - 1) Cr æ 30 - r + 1 ö
QS= S = S ç1 - ÷
\ (1 + x + x 2 ) 2n = S 2n !
a + b + g = 2n a ! b ! g !
(1 ) a ( x )b ( x 2 ) g r =1 30
C r (30 + r ) r =1 30
Cr è 30 + r ø
30 30 + r 30 + r
= S 2n !
x b + 2g = S éê Cr
-
Cr
.
(30 - r + 1) ù
ú
a + b + g = 2n a ! b ! g ! r =1
ë
30
Cr 30
Cr (30 + r ) û
For a 2, b + 2 g = 2 é (30 + r ) 29 + r ù
ê 30 + r C r
30 × Cr -1
Possible values of a, b, g are (2n - 2, 2, 0 ) and (2n - 1, 0, 1 ). (31 - r ) ú
= S ê 30 - r ×
2n ! 2n ! r =1 Cr 30
Cr 30 + r ú
\ a2 = + ê ú
(2n - 2) ! 2 ! 0 ! (2n -1) ! 0 ! 1 ! ë û
é 30 + r C r 29 + r
-1ù é n - r + 1ù
2n 2n 2n + 1 30 n
= C2 + C1 = Cr
C2
= Sê
r =1 30
- 30 ú êQ n
Cr
= ú
êë Cr Cr -1 úû êë Cr - 1 r úû
Passage II
æ 31 - r 30 30 ö
(Ex. Nos. 19 to 21) For n = 30 ç × Cr = Cr - 1÷
è r ø
30 30 + r 30
C r (2r - 1)
Let S= S 30
,K = S ( 30 C r ) 2 =
30 + 30
C 30
-
29 + 1
C0
= 60
C 30 - 1
r =1 C r (30 + r ) r=0 30
C 30 30
C0
60
and G= S (- 1) r ( 60 C r ) 2
r=0 K=
30
S ( 30C r )2 = 60C 30 and G = r S= 0 ( - 1)r ( 60 C r )2
60

r =0

19. The value of (G - S) is = ( 60 C 0 )2 - ( 60 C 1 )2 + ( 60 C 2 )2 - ... + ( 60 C 60 )2 = 60C 30


(a) 0 (b) 1 (c) 2 30 (d) 260 [Qn = 60 is even ]
60
19.(b) G - S = C 30 - ( 60 C 30 - 1) = 1
20. The value of (SK - SG ) is
(a) 0 (b) 1 20. (a) SK - SG = S ( K - G ) = S (G - G ) = 0 [Q K = G ]
(c) 230 (d) 260
21. (d) K + G = 2 × 60C 30 = 2 (S + 1) = 2 S + 2

JEE Type Solved Examples :


Single Integer Answer Type Questions
n
This section contains 2 examples. The answer to each Sol. (5) Here, ar = nC r
example is a single digit integer ranging from 0 to 9 n
ar Cr
(both inclusive). \ br = 1 + =1+ n
100
ar -1 Cr -1
l Ex. 22 The digit at unit’s place in 2 9 is n - r + 1 ( n + 1)
=1+ =
Sol. (2) Q 9100 = (2 × 4 + 1)100 = 4n + 1 [say] r r
n n
[where n is positive integer] ( n + 1)
\ 2 9 100
=2 4n +1
=2 4n
×2 = (16) × 2 n Þ P br = rP
r =1 =1 r
The digit at unit’s place in (16)n = 6. (n + 1) (n + 1) (n + 1) (n + 1) (n + 1)n
= × × ... =
\ The digit at unit’s place in (16)n × 2 = 2 1 2 3 n n!
n (101)100
=
l Ex. 23 If (1 + x ) = S a x , b = 1 +n ar r [given]
r r 100 !
r =0 ar - 1
n
(101) 100 n
and P br =
n \ n = 100 Þ =5
, then the value of is 20
r =1 100 ! 20
482 Textbook of Algebra

JEE Type Solved Examples :


Matching Type Questions
n This section contains 2 examples. Examples 24 and 25
= S 6!
(1)a (21/ 2 )b (31/ 3 )g
have three statements (A, B and C) given in Column I and a+b + g = 6 a !b! g !
four statements (p, q, r and s) in Column II. Any given
statement in Column I can have correct matching with = S 6!
2b / 2 × 3g / 3
one or more statement(s) given in Column II.
a+b + g = 6 a !b! g !
Values of (a , b, g ) for rational terms are (0, 0, 6) ,
l Ex. 24 (1, 2, 3), (3, 0, 3), (0, 6, 0), (2, 4, 0), ( 4, 2, 0), (6, 0, 0).
Column I Column II \ Number of rational terms = 7 i.e., m = 7
(A) If m and n are the numbers of rational (p) n - m = 6 and (1 + 3 2 + 5 3 )15 = (1 + 21/ 3 + 31/ 5 )15
terms in the expansions of ( 2 + 31 / 5 )10
and ( 3 + 51 / 8 ) 256 respectively, then = S 15 !
(1)a (21/ 3 )b (31/ 5 )g
a + b + g = 15 a !b! g !
(B) If m and n are the numbers of irrational (q) m + n = 20
terms in the expansions of (21 / 2 + 31 / 5 ) 40 = S 15 !
2b / 3 × 3g / 5
a + b + g = 15 a !b! g !
and (51 /10 + 21 / 6 )100 respectively, then
of (a , b, g ) for rational terms are
(C) If m and n are the numbers of rational (r) n - m = 31
terms in the expansions of (1 + 2 + 31 / 3 ) 6 (5, 0, 10), (2, 3, 10), (10, 0, 5), (7, 3, 5), ( 4, 6, 5), (1, 9, 5),
and (1 + 3 2 + 5 3 )15 respectively, then
(15, 0, 0), (12, 3, 0), (9, 6, 0), (6, 9, 0), (3, 12, 0), (15, 0, 0).
\ Number of rational terms = 13 i.e. n = 13
(s) m + n = 35
Hence, m + n = 20 and n - m = 6
(t) n - m = 39 n
Sol. (A) ® (r, s); (B) ® (t); (C) ® (p, q) l Ex. 25 If (1 + x ) n = S C r x r , match the following.
r =0
(A)Q ( 2 + 31/ 5 )10 = (21/ 2 + 31/ 5 )10
Column I Column II
10 - r r n
\ Tr +1 = 10C r × 2 2 ×3 5 (A) If S = S
r=0
l Cr and values of S are (p) a = b + c
For rational terms, r = 0, 10 [Q0 £ r £ 10] a, b, c for l = 1, r , r 2 respectively,
\ Number of rational terms = 2 then
n
(B) If S = r S
i.e., m = 2 and ( 3 + 51/ 8 )256 = (31/ 2 + 51/ 8 )256 ( - 1 )r l Cr and values of
=0
(q) a + b = c + 2
256 - R
256 S are a,b, c for l = 1, r , r 2
\ TR + 1 = CR ×3 2 × 5R / 8
respectively, then
For rational terms, r = 0, 8, 16, 24 , 32, ... , 256 [Q0 £ r £ 256] n
l Cr
\ Number of rational terms = 1 + 32 = 33 (C) IfS = S and values ofS are (r) a 3 + b 3 + c 3 = 3abc
r=0 (r + 1 )
i.e., n = 33 Þ m + n = 35 (s) and n - m = 31 a, b, c for l =1,r , r 2 respectively, then
40 - r
(B) Tr +1 in (21/ 3 +31/ 5 )40 = 40
Cr ×2 3 × 3r / 5 (s) bc - a + (c - a )b = 1

For rational terms, r = 10 , 25, 40 [Q0 £ r £ 40] (t) a + c = 4b


Q Number of rational terms = 3 Sol. (A) ® (p, q); (B) ® (p, r, t); (C) ®(s, t)
\ Number of irrational terms n

= Total terms - Number of rational terms


(A) For l = 1, a = S Cr
r =0
= 2n
n n
= 41 - 3 = 38 i.e. m = 38
100 - R For l = r , b = S r Cr
r =0
= S r × n × n - 1C r - 1
r =0 r
and T R +1 in (51/10 + 21/ 6 )100 = 100
CR × 5 10 × 2R / 6 n

rational terms, R = 0, 30, 60, 90 [Q0 £ R £ 100]


=n S
r =0
n -1
Cr -1 = n×2 n -1

n n
Q Number of rational terms = 4 and for l = r 2 , c = S r 2 Cr
r =0
= S r 2 × n × n - 1C r - 1
r =0
\ Number of irrational terms = 101 - 4 = 97 r
n n
i.e. n = 97 Þ m + n = 100 , n - m = 97 - 38 = 39
=n S r × n - 1C r -1 =n S r × n - 1C r - 1
(C)Q (1 + 2 + 31/ 3 )6 = (1 + 21/ 2 + 31/ 3 )6 r =0 r =1
Chap 06 Binomial Theorem 483

é n ù n n
= n ê S {(r - 1) + 1}n - 1C r - 1 ú
r =1
=n S ( - 1)r (r - 1) n - 1C r - 1 + n r S= 0 ( - 1)r × n - 1C r - 1
r =0
êë úû
=0+0 =0
é n n ù
= n ê S (r - 1) × n - 1C r - 1 + S n - 1C r - 1 ú \a = b = c = 0 Þ a = b + c
êë r =1 r =1
úû Þ a 3 + b 3 + c 3 = 3abc Þ a + c = 4b
n n
é ù
S æç n + 1 ö÷ × nC r
n Cr
(C) For l = 1, a = S
( n - 1) n - 2 1
= n ê S ( r - 1) × C r - 2 + 2n - 1 ú r = 0 ( r + 1)
=
r =1 ( r - 1) ( n + 1) r =0 èr + 1ø
êë úû
n
é n
= n ê(n - 1) S n - 2 C r - 2 + 2n - 1 ú
ù =
1
S n +1
Cr +1 =
1
(2n + 1 - 1)
r =1
( n + 1) r = 0 n +1
êë úû
n-2 n -1 2n + 1 - 1
= n [ ( n - 1) × 2 +2 ] = n (n + 1) 2n - 2 =
n +1
For n = 1, a = 2, b = 1, c = 1 üa = b + c n n
ý r × Cr æ 1 ö
and for n = 2, a = 4, b = 4, c = 6þ a + b = c + 2 For l = r , b = S = S ç1 - ÷ Cr
( r + 1) r = 0 è
r =0 r + 1ø
n
(B) For l = 1, a = S ( - 1)r × C r
r =0
=0 æ 2n + 1 - 1 ö (n - 1) 2n + 1
= 2n - ç ÷ =
For l = r , è n +1 ø n +1
n 2 n
×Cr æ 1 ö
n
b = S ( - 1) × r × C r = S ( - 1) × r ×
r n
n
r n -1
Cr
For l = r 2 , c = Sr = S ç( r - 1) + ÷ Cr
r =0 r =0
-1 r =0 (r +1) r = 0 è r + 1ø
r
n n n
= S r ×Cr - S Cr + S
n Cr
=n S ( - 1)
r =1
r
× n -1
Cr -1 = n ( 1 - 1) n -1
=0 r =0 r =0 r +1 r =0
n 2n + 1 - 1
and for l = r 2 , c = S ( - 1)r × r 2 × C r = n × 2n - 1 - 2n +
r =0 n +1
2 n -1
n ( n - n + 2) 2 -1
= S ( - 1)r × r 2 × n × n - 1C r - 1
r =0
=
( n + 1)
r
n 3 1 1 ü
=n S
r =0
( - 1) × r × n - 1C r
r
-1
For n = 1, a = ,b = ,c =
2 2 2 ï a + c = 4b
ý
n 7 5 7
=n S ( - 1)r { (r - 1) + 1} n - 1C r and for n = 2, a = ,b = ,c = ;ï bc - a + (c - a )b = 1
r =0
-1 3 3 3þ

JEE Type Solved Examples :


Statement I and II Type Questions
n
Directions Example numbers 26 and 27 are + (87 + 7C 1 × 86 + 7C 2 × 85 + ... + 7C 6 × 8 + 1)
Assertion-Reason type examples. Each of these examples
contains two statements: = 89 - 9 × 88 + 87 × ( 9 C 2 + 1) + 86 ( - 9C 3 + 7 )
Statement-1 (Assertion) and Statement-2 (Reason) + 85 ( 9C 4 + 7C 2 ) + ... + 8 ( 9 C 8 + 7C 6 )
Each of these examples also has four alternative choices, = 64 l [l is an integer]
only one of which is the correct answer. You have to select
\ 7 9 + 9 7 is divisible by 16.
the correct choice as given below.
(a) Statement-1 is true, Statement-2 is true; Statement-2 \ Statement-1 is true. Statement-2 is false.
is a correct explanation for Statement-1
(b) Statement-1 is true, Statement-2 is true; Statement-2
l Ex. 27. Statement-1 Number of distinct terms in the
is not a correct explanation for Statement-1 sum of expansion (1 + ax ) 10 + (1 - ax ) 10 is 22.
(c) Statement-1 is true, Statement-2 is false Statement-2 Number of terms in the expansion of (1 + x ) n
(d) Statement-1 is false, Statement-2 is true
is n +1, " n Î N.
l Ex. 26 Statement-1 (7 9 + 9 7 )is divisible by 16 Sol. (d) Q (1 + ax )10 + (1 - ax )10 = 2 {1 +
C 2 (ax )2 10

Statement-2 ( x y + y x ) is divisible by ( x + y ), " x , y . + C 4 (ax ) + C 6 (ax ) + C 8 (ax )8 + 10C 10 (ax )10 }


10 4 10 6 10

Sol. (c) 7 9 + 9 7 = ( 8 - 1)9 + ( 8 + 1)7 \ Number of distinct terms = 6


9 9 8 9 7 9
= ( 8 - C 1 × 8 + C 2 × 8 - C 3 × 8 + ... + C 8 × 8 - 1) 6 9 Þ Statement-1 is false but Statement-2 is obviously true.
484 Textbook of Algebra

Subjective Type Examples


And C k is the coefficient of x k y n - k x n - k y k
3
l Ex. 28 Find the coefficient independent of x in the
9
æ3 1 ö i.e., x n y n (r = s = t = k )
expansion of (1 + x + 2 x 3 ) ç x 2 - ÷ .
è2 3x ø Hence, the coefficient of x n y n in (1 + x )n (y + 1)n ( x + y )n
9
æ3 1 ö n
Sol. (r + 1) th term in the expansion of ç x 2 -
è2
÷
3x ø
3 3
= C 0 + C 1 + C 2 + ... + C n =
3 3
S Cr 3
r =0
9 -r r
æ3 ö æ 1 ö n+4
= 9C r ç x 2 ÷ ç- ÷
i.e., Tr +1
è2 ø è 3x ø l Ex. 30 Let (1 + x 2 ) 2 (1 + x ) n = S a k x k . If a 1, a 2 and a 3
k =0
9 -r r
æ3ö æ 1ö are in AP, find n.
= 9C r ç ÷ × x 18 - 2r × ç - ÷ × x -r
è2ø è 3ø Sol. We have,
9 -r
æ3ö æ 1ö
r
(1 + x 2 )2 (1 + x )n = (1 + 2x 2 + x 4 )
= 9C r ç ÷ × ç - ÷ × x 18 - 3r
è2ø è 3ø ´ (n C 0 + n C 1x + n C 2 x 2 + n C 3 x 3
+ ...)
2 3
Hence, general term in the expansion of (1 + x + 2x ) 3 = a 0 + a1x + a 2 x + a 3 x + ... [say]
9 9 -r r 2 3
æ3 2 1 ö 9 æ3ö æ 1ö 18 - 3r Now, comparing the coefficients of x , x and x , we get
ç x - ÷ = Cr ç ÷ ç- ÷ × x
è2 3x ø è2ø è 3ø a1 = n C 1, a 2 = 2 × n C 0 + n C 2 , a 3 = 2 × n C 1 + n C 3 …(i)
9 -r r
æ3ö æ 1ö 19 - 3r In a1, n ³ 1, in a 2 , n ³ 2 and in a 3 , n ³ 3
+ 9C r ç ÷ ç- ÷ × x
è2ø è 3ø \ n ³3 …(ii)
9 -r r From Eq. (i),
æ3ö æ 1ö 21 - 3r
+ 2 9C r ç ÷ ç- ÷ × x n ( n - 1) n 2 - n + 4
è2ø è 3ø a1 = n , a 2 = 2 + =
1 ×2 2
For independent term, putting 18 - 3r = 0, 19 - 3r = 0,
21 - 3r = 0 respectively, we get n (n - 1) (n - 2) n 3 - 3n 2 + 14n
and a 3 = 2n + =
r = 6, r = 19 / 3 [impossible] r = 7, second term do not given 1 ×2 ×3 6
the independent term. Since, a1, a 2 , a 3 are in AP.
Hence, coefficient independent of x Therefore, 2a 2 = a1 + a 3
3 6 2 7
æ3ö æ 1ö æ3ö æ 1ö n 3 - 3n 2 + 14n
= 9C 6 × ç ÷ × ç - ÷ + 0 + 2 × 9C 7 × ç ÷ ç - ÷ Þ n2 - n + 4 = n +
è2ø è 3ø è2ø è 3ø 6
27 9 1 7 2 17 Þ n 3 - 9n 2 + 26n - 24 = 0
= 9C 3 × - 2 ×9 C 2 × × = - =
.
8729 4 2187 18 27 54 or ( n - 2) ( n - 3) ( n - 4 ) = 0
\ n = 2, 3, 4
l Ex. 29 If (1 + x ) n = C 0 + C 1 x + C 2 x 2 + ... + C n x n , Hence, n = 3, 4 [from Eq. (ii)]
n
show that S C r 3 is equal to the coefficient of x n y n in the
r =0
l Ex. 31 If (1 - x ) = 3 n
n
S a r x r (1 - x ) 3n -2 r , find a r , where
r =0
expansion of {(1 + x ) (1 + y ) ( x + y )} n . n ÎN.
n
S ar x r (1 - x )3n - 2 r
n
n
Sol. (1 + x ) (y + 1) ( x + y ) = n n
S
r =0
Cr x r Sol. We have, (1 - x 3 )n =
r =0
n n n r
(1 - x )3n
S C sy n - s t S= 0 C t x n - ty t ...(i) Þ (1 - x )n (1 + x + x 2 )n = S ar × x
s=0 r =0 (1 - x )2 r
Since, C 0 is the coefficient of x 0y n - 0 x n - 0y 0
3 n
(1 - x )n (1 + x + x 2 )n ar × x r
Þ 3n
= S
i.e., x n y n ( r = s = t = 0) (1 - x ) r =0 (1 - x )2 r
n
Now, C 1 is the coefficient of x 1y n -1x n - 1y
3 n
é1 + x + x 2 ù xr
Þ ê 2 ú
= S ar ×
i.e., x n y n ( r = s = t = 1) ë (1 - x ) û r =0 (1 - x )2 r
Chap 06 Binomial Theorem 485

n n r n n
é ( 1 - x ) 2 + 3x ù é x ù Cr Cr +1
Þ ê 2
= S
ú r = 0 ar ê 2ú
Then, n
Cr
= n
Cr
ë (1 - x ) û ë ( 1 - x ) û -1

n r n -r +1 n -r é n
Cr n - r + 1ù
é æ x öù n
é x ù Þ = êQ n = ú
Þ ê1 + 3 ç 2 ÷ú
= S ar ê 2ú
…(i) r r +1 êë Cr - 1 r úû
ë è (1 - x ) ø û r = 0
ë (1 - x ) û
Þ ( n - r + 1) ( r + 1) = r ( n - r )
x Þ nr + n - r 2 - r + r + 1 = nr - r 2
Let A=
(1 - x )2 Þ n +1=0
n Þ n = -1
Then, Eq. (i) becomes (1 + 3A )n = S ar Ar
r =0 which is not possible, since n is a positive integer.
(ii) Suppose that rth, (r + 1)th and (r +2)th coefficients of
On comparing the coefficient of A r , we get
n
(1 + x )n are in HP,
C r × 3r = ar
i.e. n C r - 1,
n
C r , n C r + 1 are in HP.
Hence, ar = n C r × 3r 2 1 1
Then, n
=n +n
Cr Cr - 1 Cr + 1
l Ex. 32 If a 0 , a 1 , a 2 , ..., a 2n are the coefficients in the n n
Cr Cr
expansion of (1 + x + x 2 ) n in ascending powers of x, show Þ 2= n
+ n
Cr -1 Cr +1
that a 02 - a 12 - a 22 - ... + a 22n = a n . é n
Cr n - r +1ù
Sol. We have, (1 + x + x 2 )n = a 0 + a1x + a 2 x 2 + a 2n x 2n
...(i) êQ n = ú
êë Cr - 1 r úû
æ 1ö n - r +1 r + 1
Replacing x by ç - ÷ in Eq. (i), we get Þ 2= +
è xø r n -r
æ 1 1 ö a1 a 2
n
a 2n Þ 2r (n - r ) = (n - r + 1) (n - r ) + r (r + 1)
ç1 - + 2 ÷ = a 0 - + 2 - ... + 2n ...(ii) Þ 2nr - 2r 2 = n 2 - nr - nr + r 2 + n - r + r 2 + r
è x x ø x x x
Þ n 2 - 4nr + 4r 2 + n = 0 Þ (n - 2r )2 + n = 0
On multiplying Eqs. (i) and (ii), we get
n
which is not possible, as (n - 2r )2 ³ 0 and n is a
æ 1 1 ö positive integer.
(1 + x + x 2 )n ´ ç1 - + 2 ÷ = (a 0 + a1x + a 2 x 2
è x x ø n n
æ a a a ö
+ ... + a 2n x 2n ) ´ ça 0 - 1 + 22 - ... + 22nn ÷
l Ex. 34 Evaluate S S
i = 0 j =1
n
C j × j C i , i £ j.
è x x x ø n n

Þ
(1 + x 2 + x 4 )n
= (a 0 + a1x + a 2 x 2 + ... + a 2n x 2n ) Sol. We have, S S nC j × j C i
i = 0 j =1
x 2n
æ a a a ö = n C 1 ( 1C 0 + 1C 1 ) + n C 2 ( 2 C 0 + 2C 1 + 2C 2 )
´ ça 0 - 1 + 22 - ... + 22nn ÷ ...(iii)
è x x x ø + n C 3 ( 3C 0 + 3C 1 + 3C 2 + 3C 3 )
Constant term in RHS = a 02 - a12 + a 22 - ... + a 22n + n C 4 ( 4C 0 + 4C 1 + 4C 2 + 4C 3 + 4C 4 )
(1 + x 2 + x 4 ) n + ... + n C n (n C 0 + n C 1 + n C 2 + ... + n C n )
Now, constant term in 2n
= Coefficient of x 2n
x = n C 1(2) + n C 2 (2)2 + n C 3 (2)3 + ... + n C n (2)n = (1 + 2)n - 1
in (1 + x 2 + x 4 )n = an [replacing x by x 2 in Eq. (i)]
= 3n - 1
But Eq. (iii) is an identity, therefore, the constant term in
RHS = constant term in LHS.
l Ex. 35 Find the remainder, when 27 40 is divided by 12.
a 02 - a12 + a 22 - ... + a 22n = an
Sol. We have, 27 40 = (33 )40 = 3120 = 3 × ( 3)119 = 3 × ( 4 - 1)119
= 3 ( 4n - 1), where n is some integer
l Ex. 33 Show that no three consecutive binomial coeffi-
= 12n - 3 = 12n - 12 + 9 = 12 (n - 1) + 9
cients can be in (i) GP and (ii) HP.
= 12m + 9, where m is some integer.
Sol. (i) Suppose that the r th, (r + 1)th and (r + 2)th
coefficients of (1 + x )n are in GP. 27 40 9
\ =m +
n n 12 12
i.e., Cr - 1, C r , nC r +1 are in GP.
Hence, the remainder is 9.
486 Textbook of Algebra

l Ex. 36 Show that [( 3 + 1) 2n ] + 1 is divisible by 2 n + 1 , difference (26081690 - 41690 ) is also divisible by 7, since it is
" n Î N, where [ × ] denotes the greatest integer function. divisible by 2608 - 4 = 2604 = 7 ´ 372.
Sol. Let x = ( 3 + 1)2n = [ x ] + f ...(i) As to sum 32608 + 41690 , it can be rewritten as
where, 0£ f <1 3× (33 )869 + 4 × ( 4 3 )563
and ( 3 - 1)2n = f ¢ ...(ii) = 3 (28 - 1)869 + 4 (63 + 1)563
where, 0 < f ¢< 1 = 3 ( 7m - 1) + 4 ( 7n + 1)
On adding Eqs. (i) and (ii), we get [where, m and n are some positive integers]
[ x ] + f + f ¢ = ( 3 + 1)2n + ( 3 - 1)2n where p is some positive integer.
= ( 4 + 2 3 )n + ( 4 - 2 3 )n Hence, the remainder is 1.

= 2n { (2 + 3 )n + (2 - 3 )n } l Ex. 39 If C 0 , C 1 , C 2 , ..., C n are the binomial coefficients


n n n n n-2
= 2 × 2 { C 0 ( 2) + C 2 ( 2) in the expansion of (1 + x ) n , prove that
( 3 )2 + n C 4 (2)n - 4 ( 3 )4 + ...} (C 0 + 2C 1 + C 2 ) (C 1 + 2C 2 + C 3 )... (C n - 1 + 2C n + C n + 1 )
\ [ x ] + f + f ¢ = 2n + 1 k , where k is an integer. ...(iii) n
( n + 2) n
Hence, ( f + f ¢ ) is an integer.
= P (C r - 1 + C r ) .
( n + 1)! r =1
i.e., f + f ¢=1 [ Q0 < ( f + f ¢ ) < 2] Sol. LHS = (C 0 + 2C 1 + C 2 ) (C 1 + 2C 2 + C 3 ) ...
From Eq. (iii), we get (C n - 1 + 2C n + C n + 1)
n +1 n
[x ] + 1 = 2 k
= P ( nC r - 1 + 2 nC r
r =1
+ nC r + 1)
2n n +1
Þ [( 3 + 1) ] + 1 = 2 k [from Eq. (i)]
n
which shows that [( 3 + 1)2n ] + 1 divisible by 2n +1, " n Î N . = P {( nC r - 1 + nC r ) + (n C r
r =1
+ nC r + 1 )}

n
l Ex. 37 Find the number of rational terms and also find
the sum of rational terms in ( 2 + 3 3 + 6 5 ) 10 .
= P ( n + 1C r
r =1
+ n +1
Cr + 1) [by Pascal’s rule]

Sol. We have, ( 2 + 3 3 + 6 5 )10 = (21 / 2 + 31 / 3 + 51 / 6 )10 n n


æn + 2ö n +1 é n ù
= P ( n + 2C r + 1 ) = rP ç
=1 è
÷
ø
C r êQ C r =
n n -1
. Cr - 1ú
= S 10 !
2a / 2 × 3b / 3 × 5g / 6
r =1 r +1 ë r û
a + b + g = 10 a !b! g ! n n n

For rational terms, = P æç n + 2 ö÷ ( nC r - 1 + nC r ) = rP æn + 2ö


ç ÷ P (C r - 1 + C r )
r =1 èr + 1ø =1 r =1 èr + 1ø
a = 0, 2, 4, 6, 8, 10, b = 0, 3, 6, 9, g = 0, 6
n
Since, 0 £ a , b, g £ 10. ( n + 2) ( n + 2) ( n + 2) ( n + 2)
= × × ... P (C r -1 + Cr )
\ Possible triplets are ( 4, 0, 6), ( 4, 6, 0), (10,0,0). 2 3 4 ( n + 1) r = 1
There exists three rational terms. n
(n + 2)n
\ Required sum
= P (C r -1 + C r ) = RHS
( n + 1) ! r = 1
10 ! 10 ! 10 !
= 22 × 5 + 22 × 32 + 25
4 !0!6! 4 !6!0! 10 ! 0 ! 0 ! 2n 2n
= 4200 + 7560 + 32 = 11792 l Ex. 40 If S a r ( x - 2 ) r = rS= 0 br ( x - 3) r
r =0
and a k =1,
2n + 1
l Ex. 38 Find the remainder, when (1690 2608 + 2608 1690 ) is " k ³ n, show that bn = C n + 1.
2n 2n
divided by 7.
Sol. Q S ar ( x - 2)r = S br ( x - 3)r
Sol. We have, 16902608 + 26081690 = (16902608 - 32608 ) r =0 r =0

+ (26081690 - 41690 ) + (3 2608 + 41690 ) Let y =x - 3 Þy + 1 = x - 2


So, the given expression reduces to
The number (16902608 - 32608 ) is divisible by 2n 2n
1690 - 3 = 1687 = 7 ´ 241 which is divisible by 7, the S ar (1 + y )r
r =0
= S br y r
r =0
Chap 06 Binomial Theorem 487

Þ a 0 + a1 (1 + y ) + a 2 (1 + y )2 + ... + a 2n (1 + y )2n l Ex. 42 If (1 + x ) n = C 0 + C 1 x + C 2 x 2


= b 0 + b1y + ... + b 2n y 2n + C 3 x 3 + ... + C n x n , show that
C C C 1 1 1
Using ak = 1, " k ³ n, we get C 1 - 2 + 3 - ... ( - 1) n - 1 n = 1 + + + ... + .
2 3 n 2 3 n
a 0 + a1 (1 + y ) + a 2 (1 + y )2 +... + an - 1 (1 + y )n - 1
Sol. We know that,
n n +1 2n
+ (1 + y ) + (1 + y ) + ... + (1 + y ) (1 - x ) n = C 0 - C1 x + C 2 x 2 -... + ( -1) n C n x n
= b 0 + b1y + ... + bn y n + ... + b 2n y 2n or C 0 - (1 - x )n = C 1x - C 2 x 2 + C 3 x 3
- ... + ( - 1)n - 1 C n x n
n
On comparing the coefficient of y on both sides, we get
n
C n + n + 1C n + n+2
C n + ... + 2n C n = bn Þ 1 - (1 - x )n = C 1x - C 2 x 2 + C 3 x 3 -... + ( - 1)n - 1 C n x n
n +1
Þ C n + 1 + n + 1C n + n+2
C n + ... + 2n C n = bn Dividing in each side by x, then
1 - (1 - x )n
[Q n C r + n C r -1 = n +1
Cr ] = C 1 - C 2 x + C 3 x 2 - ... + ( - 1)n - 1 C n x n - 1
x
Þ n + 2 C n + 1 + n + 2C n + ... + 2n
C n = bn On integrating within limits 0 to 1, we have
1 1 - (1 - x )n 1
[adding first two terms] ò0 x
dx = ò (C 1 - C 2 x + C 3 x 2 -
0
If we combine terms on LHS finally, we get ... + ( - 1)n - 1 C n x n - 1 ) dx
2n + 1
C n +1 = bn 1
é C x 2 C 3x 3 xn ù
= êC 1x - 2 + - ... + ( - 1)n - 1 C n ú
ë 2 3 n û0
l Ex. 41 (i) If n is an odd natural number, prove that
n 1 - (1 - x )n
1 C C ( - 1)n - 1
( - 1) r ò0 dx = C 1 - 2 + 3 - ... +
S
r =0 nC
= 0. x 2 3 n
Cn
r Putting 1 - x = t in integral,
(ii) If n is an even natural number, find the value of Þ dx = - dt
n
( - 1) r
S
r = 0 nC
. when x ® 1, t ® 0 and when x ® 0, t ® 1
1 (1 - t n ) C C C
( - dt ) = C 1 - 2 + 3 -... + ( - 1)n - 1 n
r
n +1 \ ò
0 (1 - t ) 2 3 n
2
n
( - 1) r é
( - 1) r
( - 1)n - r ù
S S
n
1 (1 - t ) C2 C3 n - 1 Cn
= +
Sol. (i) We have,
r =0 n Cr r =0
ê n
êë C r
n ú
C n - r úû
Þ ò 0 (1 - t ) dt = C 1 - 2 + 3 - ... + ( - 1) n
n +1 n +1 1 C C
Þ ò (1 + t + t 2 + ... + t n - 1 ) dt = C 1 - 2 + 3
2
é 1 ( - 1) n ù 2
r é 1 1 ù 0 2 3
=
r =0
S ( - 1)r ê n +n ú = S ( - 1) ê n
r =0
-n ú
-... + ( - 1)n - 1
Cn
êë C r Cn - r úû ë Cr Cr û
n
1
=0 n n
[Qn is odd and C r = C n - r ] é t2 t3 tn ù C C
Þ êt + + + ... + ú = C 1 - 2 + 3
(ii) We have, ë 2 3 n û0 2 3
C
- ... + ( - 1)n - 1 n
n
-1
n r 2
é ( - 1)r ( - 1)n - r ù ( - 1)n / 2
S ( - 1)
n
= S ê n + n ú+ n 1 1 1 C2 C3 n - 1 Cn
r =0 n Cr r =0
êë C r C n - r ûú Cn / 2 Þ 1 + + + ... + = C 1 - + -... + ( - 1)
n
2 3 n 2 3 n
-1
C C C
Hence, C 1 - 2 + 3 - ... + ( - 1)n - 1 n
2
é 1 ( - 1)n ù ( - 1)n / 2
= S ( - 1)r
r =0
ên + n ú+ n 2 3 n
ë Cr Cr û Cn / 2 1 1 1
n = 1 + + + ... +
-1 2 3 n
2
é 1 1 ù ( - 1)n / 2
= S ( - 1)r ên + n ú+ n
r =0
ë Cr Cr û Cn / 2 l Ex. 43 If (1 + x ) n = C 0 + C 1 x + C 2 x 2 + C 3 x 3
én - 1 ù + ... + C n x n , find the sum of the seriesd
ê2 2 ú ( - 1)n / 2
= ê S ( - 1) × n ú + n
r C 0 C1 C 2 C3
- + - + ... + ( - 1) n
Cn
.
r =0 Cr ú Cn / 2
ê 2 6 10 14 4n + 2
ë û
488 Textbook of Algebra

n -1
1 ö n æç ö
C 0 C1 C 2 C 3 Cn æ1 n
Sol. Let S = - + - + ... + ( - 1)n n -r
2 6 10 14 4n + 2 \ S=
n
SS ç + ÷= S + S
r ÷
2 0 £i < j £n è C i C j ø 2 çèr = 0 C r r = 1 Cr ÷
ø
1 æC C C C Cn ö
= ç 0 - 1 + 2 - 3 + ... + ( - 1)n ÷ …(i) æ n n -r r ö n æ n ö n2
n n n
çS S ÷= ç S S
2è 1 3 5 7 2n + 1 ø n 1
= + ÷=
çr = 0 C r = 0 Cr ÷ ç ÷
Consider, (1 - x 2 )n = C 0 - C 1 x 2 + C 2 x 4 - C 3 x 6
2 è r ø 2 èr = 0 C r ø 2 r =0 Cr

+ ... + ( - 1)n C n x 2n l Ex. 45 If (1 + x ) n = C 0 + C 1 x + C 2 x 2 + C 3 x 3


1 1
Þ ò0 (1 - x
2 n
) dx = ò0 ( C 0 - C 1x
2
+ C2 x4 - C3 x6 + ... + C n x n , show that
+ ... + ( - 1)n C n x 2n ) dx C r 3 r +4
n
S
1 é C x 3
C2 x 5
C 3x 7 r=0 ( r + 1) ( r + 2) ( r + 3) ( r + 4)
ò0 (1 - x
2 n
Þ ) dx = êC 0 x - 1 + -
ë 3 5 7 æ n+4 3 ö
- S
1 n+4
= ç 4 C t 3 t ÷.
( n + 1) ( n + 2) ( n + 3) ( n + 4) çè ÷
2n + 1 ù
Cn x t=0
ø
+ ... + ( -1)n
ú
2n + 1 û
C r × 3r + 4
n
1 C C C
Þ ò (1 - x 2 )n dx = C 0 - 1 + 2 - 3 +... + ( - 1)n
Cn Sol. LHS = S
r =0 ( r + 1) ( r + 2) ( r + 3) ( r + 4 )
0 3 5 7 2n + 1
n r +4
Cr ×3
= S
r = 0 ( r + 1) ( r + 2) ( r + 3) ( r + 4 )
From Eq. (i), 4!
1 1 1 4!
ò0 (1 - x ) dx = 2S or S = 2 ò0 (1 - x ) dx
2 n 2 n
r +4
3r + 4
n n

Put x = sinq i.e., dx = cosq d q


= S Cr ×3 = S n!
×
r =0 r +4 C4 × 4 ! r =0 r !(n - r )! (r + 4 )! × 4 !
1 p/ 2
S = ò cos 2n +1 q d q 4 !r !
2 0 n
n ! × 3r + 4
By using Walli’s formula, = S
r =0 (n - r ) !× (n + 4 ) !
1 2n (2n - 2) (2n - 4 ) ... 4 × 2
S= × ×1 n
n !× 3r + 4 ( n + 1) ( n + 2) ( n + 3) ( n + 4 )
2 (2n + 1) (2n - 1) (2n - 3) ... 3 × 1 = S .
r =0 ( n - r ) ! × ( r + 4 ) ! ( n + 1) ( n + 2) ( n + 3) ( n + 4 )
1 {2n (2n - 2) (2n - 4 ) ... 4 × 2)}
2
= × n
(n + 4 ) ! 3r + 4
2 (2n + 1) ! = S
r =0 ( n - r ) ! ( r + 4 ) ! ( n + 1) ( n + 2) ( n + 3) ( n + 4 )
1 ( 2n n !)2 (n !)2
= × = 22n - 1 é n (n + 4 ) ! × 3r + 4 ù
êS
1
2 (2n + 1) ! (2n + 1) ! = ú
(n + 1) (n + 2) (n + 3) (n + 4 ) ëêr = 0 (n - r ) ! × (r + 4 ) ! úû
n
l Ex. 44 If (1 + x ) n = S Cr
r =0
x r , then prove that =
1 é n n+4
êS
ù
C r + 4 3r + 4 ú
(n + 1) (n + 2) (n + 3) (n + 4 ) êër = 0 úû
æ i j ö n2
n
SS ç + ÷= S 1
. ïì
n + 4
ïü
íS
1 n+4
0 £ i < j £ n è Ci Cj ø 2 r =0 Cr = C t 3t ý
( n + 1) ( n + 2) ( n + 3) ( n + 4 ) ï t = 4 ïþ
î
æ i j ö
Sol. Let S = SS ç + ÷ ...(i) [put r + 4 = t ]
0 £i < j £n è i
C C j ø
=
1
( n + 1) ( n + 2) ( n + 3) ( n + 4 )
Replacing i by n - i and j by n - j , we get
ìïn + 4 n + 4 3 üï
íS C t 3t - S n + 4C t 3t ý
æn -i n-jö æn - i n - jö
S= SS ç + ÷ = SS ç + ÷
0 £ i < j £ n è Cn - i Cn - j ø 0 £ i < j £ n è Ci Cj ø ïî t = 0 t=0 ïþ
ïì ïü
3
[QC r = C n - r ] ...(ii)
- S n + 4C t 3t ý
1 n+4
= í(1 + 3)
On adding Eqs. (i) and (ii), we get (n + 1) (n + 2) (n + 3) (n + 4 ) ïî t = 0 ïþ
æ1 1 ö ìï n + 4 üï
SS
3
- S n + 4C t 3t ý
2S = n ç + ÷ 1
= í4
0 £i < j £n è i
C C j ø (n + 1) (n + 2) (n + 3) (n + 4 ) ïî t=0 ïþ
= RHS
Chap 06 Binomial Theorem 489

9 9 l Ex. 48 Prove that


l Ex. 46 Prove that S xk divides S x kkkk . n 1 ì n-1 np ü
k =0 k =0 C 3 + n C 7 + n C 11 + ... = í2 - 2 n / 2 sin ý
9 2î 4þ
Sol. Let S1 = S x kkkk
k =0
= x 0 + x 1111 + x 2222
+ ... + x 9999
Sol. In given series difference in lower suffices is 4.
9 i.e., 7 - 3 = 11 - 7 = ... = 4
and S2 = S xk
k =0
= x0 + x 1 + x 2
+ ... + x 9
Now, (1)1/ 4 = (cos 0 + i sin 0)1/ 4
9 9 = (cos 2r p + i sin 2r p)1/ 4
Now, S1 - S 2 = S ( x kkkk - xk ) = S x k ( x 10 ) kkk - 1) rp rp
k =0 k =0 = cos + i sin , where r = 0, 1, 2, 3
9 9 2 2
= [ ( x 10 )kkk - 1] S x k = l k S= 0 x k
k =0
Four roots of unity = 1, i , - 1, - i = 1, a , a 2 , a 3 [say]
n
Þ S1 - S 2 = l S 2 Þ S1 = (1 + l ) S 2
9 9
and (1 + x )n = S nC r x r
r =0
Hence, S x kkkk is divisible by k S= 0 x k .
k =0
n
Putting x = 1, a , a 2 , a 3 , we get 2n = S n C r ...(i)
r =0
k
S ( -3) r - 1 × 3n C 2r - 1 = 0, where k = 3n
n
l Ex. 47 Prove that
r =1 2
(1 + a )n = S nC r a r
r =0
...(ii)
and n is an even positive integer. n
Sol. Given, n is an even positive integer. (1 + a 2 )n = S nC r a 2 r
r =0
...(iii)
Let n = 2m ;\ k = 3m, m Î N
n

LHS =
k
S ( - 3) r - 1 3n
C 2r -1 =
3m
S ( - 3) r - 1 6m
C 2r -1
and (1 + a 3 )n = S nC r a 3r
r =0
...(iv)
r =1 r =1
6m 6m On multiplying Eq. (i) by 1, Eq. (ii) by a, Eq. (iii) by a 2 and
= C1 - 3 × C 3 + 32 × 6mC 5
Eq. (iv) by a 3 and adding, we get
- ... + ( - 3)3m - 1 6mC 6m - 1 ...(i)
Þ 2n + a (1 + a )n + a 2 (1 + a 2 )n + a 3 (1 + a 3 )n
Consider (1 + i 3 )6m = 6m
C0 + 6m
C 1 (i 3 ) + 6m
C 2 (i 3 )2 n
+ 6m
C 3 (i 3 )3 + 6m
C 4 (i 3 )4 + 6m
C 5 (i 3 )5 = S nC r (1 + a r + 1 + a 2r + 2 + a 3r + 3 )
r =0
...(v)
+ ... + 6mC 6m - 1 (i 3 )6m - 1 + 6m
C 6m (i 3 )6m …(ii) For r = 3, 7, 11, ... RHS of Eq. (v)
6m
ì æ - 1 - i 3 öü = n C 3 (1 + a 4 + a 8 + a 12 ) + n C 7 (1 + a 8 + a 16 + a 24 )
Now, (1 + i 3 )6m = í( - 2) ç ÷ý = ( - 2 w2 )6m
î è 2 øþ + n C 11 (1 + a 12 + a 24 + a 36 ) + ...
= 26m , where w2 is cube root of unity. = 4 ( n C 3 + n C 7 + n C 11 + ...) [Qa 4 = 1]
Then, Eq. (ii) can be written as and LHS of Eq. (v)
6m 6m 6m 6m 2
2 = { C0 - C2 ×3 + C4 ×3 = 2n + i (1 + i )n + i 2 (1 + i 2 )n + i 3 (1 + i 3 )n
- ... + ( - 3)3m × 6m
C 6m } + i 3 { 6m C 1 - 6m
C3 ×3 = 2n + i (1 + i )n + 0 - i (1 - i )n
+ 6m
C 5 × 32 - ... + ( - 3)3m - 1 × 6m
C 6m - 1 } = 2n + i {(1 + i )n - (1 - i )n }
On comparing the imaginary part on both sides, we get é é æ 1 i öù ù
n

3 ( 6m C 1 - 3 × 6m
C 3 + 32 × 6mC 5 Since, ê(1 + i )n = ê 2 ç + ÷ú ú
êë ë è 2 2 øû ú
û
- ... + ( - 3)3m - 1 × 6m
C 6m - 1 ) = 0 n
n p ì p pü
6m 6m 2 6m = 2n + i 2n / 2 × 2i sin = 2n / 2 í cos + i sin ý
or C1 - 3 × C3 + 3 × C5 4 î 4 4þ
- ... + ( - 3)3m - 1 × 6m
C 6m - 1 = 0 n n /2 np n /2 ì np np ü
= 2 - 2 × 2 sin =2 í cos + i sin ý
3m 4 î 4 4þ
Þ å( - 3)r - 1 × 6mC 2r - 1 = 0 æ np ö
r =1 Hence, 4 ( n C 3 + n C 7 + n C 11 + ...) = 2 ç2n - 1 - 2n / 2 sin ÷
k è 4 ø
or å ( - 3)r - 1 × 3nC 2r - 1 = 0, where n = 2m and k = 3m Þ n 1æ
C 3 + n C 7 + n C 11 + ... = ç2n - 1 - 2n / 2 sin
np ö
÷
r =1 2è 4 ø
490 Textbook of Algebra

n -1 n +1 = (9 + 4 5 )n (9 - 4 5 )n = 1n = 1
l Ex. 49 Evaluate S S nCi
i = 0 j =1+i
n +1
C j.
\ ( I - 1) f + f 2
= I - 1 = 2N - 1 - 1 = 2N - 2
n -1 n +1 [from Eq. (v)]
Sol. Let P = S S
i = 0 j =1+i
n
Ci n +1
Cj = An even integer

n+1 n+1 n+1 l Ex. 51 If Pr is the coefficient of x r in the expansion of


= S n
C0 n +1
Cj + S n
C1 n +1
Cj + S n
C2 n +1
Cj 2 2 2
j =1 j =2 j =3
n+1 æ2 xö æ x ö æ x ö
(1 + x ) ç1 + ÷ ç1 + 2 ÷ ç1 + 3 ÷ ... , prove that
+ ... + S
j =n
n
Cn - 1 n +1
Cj è 2 ø è 2 ø è 2 ø
n+1 n+1 n+1 22 1072
Pr = (Pr - 1 + Pr - 2 ) and P4 = .
= nC 0 S n + 1C j + nC1 j S= 2 n + 1C j + nC 2 j S= 3 n + 1C j
j =1
r
( 2 - 1) 315
n+1
S n + 1C j
2 2 2
+ ... + n C n - 1 æ xö æ xö æ xö
j =n Sol. Let (1 + x )2 ç1 + ÷ ç1 + 2 ÷ ç1 + 3 ÷ ...,
è 2ø è 2 ø è 2 ø
= n C 0 (n + 1C 1 + n + 1C 2 + n + 1C 3 + ... + n + 1C n + 1 ) = 1 + P1 x + P2 x 2 + P3 x 3 + P4 x 4 +
+ n C 1 ( n + 1C 2 + n + 1C 3 + n + 1C 4 + ... + n + 1C n + 1 )
... + Pr - 1 x r - 1 + Pr x r + ... …(i)
+ n C 2 (n + 1C 3 + n + 1C 4 + n + 1C 5 + ... + n + 1C n + 1 )
x
+ ... + n C n - 1 (n + 1C n + n +1
Cn + 1) Replacing x by , we get
2
2 2 2 2
= n +1
C 1 × nC 0 + n +1
C 2 (n C 0 + n C 1 ) æ xö æ xö æ xö æ xö
ç 1 + ÷ ç 1 + ÷ ç 1 + ÷ ç1 + 4 ÷ ...
+ n + 1C 3 (n C 0 + n C 1 + n C 2 ) è 2ø è 22 ø è 23 ø è 2 ø
+ ... + n +1
C n + 1 (n C 0 + n C 1 + n C 2 + ... + n C n - 1 ) é æx ö æx ö
2
æx ö
3 ù
= ê1 + P1 ç ÷ + P2 ç ÷ + P3 ç ÷ + ...ú
= (n C 0 + n C 1 ) × n C 0 + (n C 1 + n C 2 ) (n C 0 + n C 1 ) êë è2ø è2ø è2ø úû
+ (n C 2 + n C 3 ) (n C 0 + n C 1 + n C 2 ) On multiplying both sides by (1 + x )2 , we get
2 2 2
+ ...+(n C n + n C n - 1 ) (n C 0 + n C 1 + n C 2 + ... + n C n - 1 ) + n æ xö æ xö æ xö
(1 + x )2 ç1 + ÷ ç1 + 2 ÷ ç1 + 3 ÷ ...
= (n C 0 )2 + (n C 1 )2 + (n C 2 )2 + ... + (n C n - 1 )2 è 2ø è 2 ø è 2 ø
é æx ö æx ö
2
æx ö
3 ù
+ 2 {n C 0 × n C 1 +n C 0 × n C 2 +n C 0 × n C 3 = (1 + x )2 ê1 + P1 ç ÷ + P2 ç ÷ + P3 ç ÷ + ...ú ...(ii)
êë è2ø è2ø è2ø úû
+ ... + n C 0 × n C n - 1 + ... + n C n - 2 n C n - 1 } + 2n - 1 + n
From Eqs. (i) and (ii), we get
= ( n C 0 + n C 1 + n C 2 + ... + n C n - 1 )2 + 2n - 1 + n 1 + P1 x + P2 x 2 + P3 x 3 + P4 x 4 +... + Pr - 1 x r - 1 + Pr x r + ...
= (2n - 1)2 + 2n - 1 + n = 22n - 2n + n é æx ö æx ö
2
æx ö
3 ù
= (1 + x )2 ê1 + P1 ç ÷ + P2 ç ÷ + P3 ç ÷ + ...ú
êë è2ø è2ø è2ø úû
l Ex. 50 If ( 9 + 4 5 ) n = I + f , n and I being positive inte- r
On equating coefficient of x , we get
gers and f is a proper fraction, show that (I - 1) f + f 2 is an æ1ö æ 1 ö æ 1 ö
Pr = Pr ç r ÷ + 2 Pr - 1 ç r - 1 ÷ + Pr - 2 ç r - 2 ÷
è2 ø è2 ø è2 ø
even integer.
Sol. (9 + 4 5 )n = I + f
æ 1ö 1
...(i) Þ Pr ç1 - r ÷ = r - 2 ( Pr -1 + Pr - 2)
è 2 ø 2
0£ f <1 ...(ii)
22
Let f ¢ = ( 9 - 4 5 )n ...(iii) Þ Pr = ( Pr -1 + Pr - 2)
(2r - 1)
and 0< f ¢<1 ...(iv) 1 1
From Eqs. (i) and (iii), we get Now, P0 = 1, P1 = 2 + 1 + + + ... = 4
2 22
I + f + f ¢ = ( 9 + 4 5 )n + ( 9 - 4 5 )n 22 ( P1 + P0 ) 20
= 2 {9n + n C 2 9n - 2 ( 4 5 )2 + ...} P2 = = ,
22 - 1 3
= 2N , where N is a positive integer. 22 ( P2 + P1 ) 128
and from Eqs. (ii) and (iii), we get 0 < f + f ¢ < 2 P3 = =
23 - 1 21
Since, f + f ¢ is an integer.
\ f + f ¢=1 æ 128 20 ö
4ç + ÷
I + 1 = 2N Þ 1 = 2N - 1 22 ( P3 + P2 ) è 21 3 ø 1072
Now, ...(v) and P4 = = =
Q n
( I + f ) (1 - f ) = (9 + 4 5 ) f ¢ 24 - 1 15 315
#L Binomial Theorem Exercise 1 :
Single Option Correct Type Questions
n -1 n
This section contains 30 multiple choice questions.
S
n
Cr
Each question has four choices (a), (b), (c) and (d) out of 9. is equal to
r=0 n Cr + nCr + 1
which ONLY ONE is correct
n
é 1 ù n n+1
3r 7r
1. If S ( - 1)r
r=0
n
C r ê r + 2 r + 3r + K upto m terms ú
(a)
2
(b)
2
ë2 2 2 û n (n + 1 ) n (n - 1 )
(c) (d)
æ 1 ö 2 2 (n + 1 )
= f (n ) ç1 - mn ÷,
è 2 ø 100
10. The largest term in the expansion of æç + ö÷
b b
3 is
è2 2ø
ò-3 f ( x
3 3
ln x ) d ( x ln x ) is equal to
100
(a) - 6 (b) - 3 æb ö
(a) b100 (b) ç ÷
(c) 3 (d) Cannot be determined è2ø
100
100 æb ö
2. The coefficient of (a × b × c 8 × d 9 × e × f ) in the expansion
3 6 (c) C 50 ç ÷ (d) 100C 50b100
è2ø
of (a + b + c - d - e - f ) 31 is 6
(a) 123210 (b) 23110 æ æç 1 ö÷ ö
ç ÷
(c) 3110 (d) None of these 11. If the fourth term of ç x è 1 + log x ø + 12 x ÷ is equal to
ç ÷
3. The sum of rational terms in ( 2 + 3 + 6 5 )10 , is è ø
(a) 12632 (b) 1260 200 and x > 1, x is equal to
(c) 126 (d) None of these 10
(a) 10 2 (b) 10 (c) 10 4 (d)
2
4. If (1 + x - 3x 2 ) 2145 = a 0 + a1 x + a 2 x 2 + ... , then m
a 0 - a1 + a 2 - a 3 + ... ends with 12. The coefficient of x in
m m +1
(a) 1 (b) 3 (1 + x ) + (1 + x ) + ... + (1 + x )n , m £ n, is
(c) 7 (d) 9 n+1 n -1
(a) Cm + 1 (b) Cm - 1
n
æ q p7 ö (c) Cm n n
(d) Cm + 1
5. In the expansion of ç + 10 3 ÷ , there is a term
ç p q ÷ø
è
13. The number of values of ' r ' satisfying the equation
similar to pq, then that term is equal to 39
C 3r - 1 - 39
Cr2 = 39
Cr2 - 1 - 39
C 3r is
(a) 45pq (b) 120 pq
(c) 210 pq (d) 252 pq (a) 1 (b) 2
(c) 3 (d) 4
6. Let (5 + 2 6 )n = I + f , where n, I Î N and 0 < f < 1, then 20 20
2
14. The sum S = C2 + 2× C 3 + 3 × 20 C 4 + ... + 19 × 20
C 20 is
the value of f - f + I × f - I , is
equal to
(a) a natural number (b) a negative integer (a) 1 + 5 × 2 20 (b) 1 + 2 21
(c) a prime number (d) an irrational number (c) 1 + 9 × 2 20 (d) 2 20
1 1
7. If x + = 1 and p = x 4000 + 4000 and q is the digit at 15. The remainder, if 1 + 2 + 2 2 + 2 3 + ... + 21999 is divided by
x x
n 5, is
unit place in the number 2 2 + 1, n Î N and n > 1, then
(a) 0 (b) 1
p + q , is (c) 2 (d) 3
(a) 8 (b) 6 1
(c) 7 (d) None of these 16. Coefficient of in the expansion of (1 + x )n (1 + 1 / x )n is
n
x
8. If the number of terms in æç x + 1 + ö÷ (n Î I + ) is 401,
1 n! 2n !
(a) (b)
è xø (n - 1 )! (n + 1 )! (n - 1 )! (n + 1 )!
then n is greater than n! 2n !
(a) 201 (b) 200 (c) (d)
(2n - 1 )! (2n + 1 )! (2n - 1 )! (2n + 1 )!
(c) 199 (d) None of these
492 Textbook of Algebra

4 2008 2009 2010


17. The last two digits of the number 19 9 is 25. If (3 + x +x ) = a 0 + a1 x + a 2 x 2
(a) 19 (b) 29 + ... + an x n , the value of
(c) 39 (d) 81 a a a a
n a 0 - 1 - 2 + a 3 - 4 - 5 + a 6 - ... is
æ a ö 2 2 2 2
18. If the second term in the expansion of çç13 a + ÷÷ is
n è a -1 ø (a) 1 (b) 2 2010
5/ 2 C3 (c) 5 2010 (d) 3 2010
14a , the value of n
is
C2
(a) 4 (b) 3 26. The total number of terms which depend on the value of
n
(c) 12 (d) 6 æ 1 ö
x in the expansion of ç x 2 - 2 + 2 ÷ is
19. If 6 83 + 8 83 is divided by 49, the remainder is è x ø
(a) 0 (b) 14 (a) 2n + 1 (b) 2n
(c) 35 (d) 42 (c) n + 1 (d) n
10
20. The sum of all the rational terms in the expansion of 27. The coefficient of x in the expansion of
1/ 4 1/ 3 12
(3 +4 ) is 2
(1 + x - x ) , is 3 8

(a) 91 (b) 251 (a) 420 (b) 476


(c) 273 (d) 283
(c) 532 (d) 588
21. Last four digits of the number N = 7 100 - 3100 is 28. The number of real negative terms in the binomial
(a) 2000 (b) 4000
expansion of (1 + ix ) 4n - 2 , n Î N , n > 0, i = - 1, is
(c) 6000 (d) 8000
(a) n (b) n + 1
22. If 5 99 is divided by 13, the remainder is (c) n - 1 (d) 2n
(a) 2 (b) 4 (c) 6 (d) 8 n n
æn ö æm ö
ì3 2003 ü 29. S S ç ÷ ç ÷ is equal to
23. The value of í ý, where { . } denotes the fractional p =1m = p èm ø è p ø
î 28 þ n
(a) 3 (b) 2n
part function is
(c) 3n + 2n (d) 3n - 2n
(a) 17/28 (b) 19/28
(c) 23/28 (d) 5/28 30. The largest real value of x, such that
20 4
æ 54 - r ö æ x r ö 8
24. The value of S r (20 - r )( 20 C r ) 2 is equal to Sç ÷ ç ÷ = is
r=0 r=0 è(4 - r ) !ø è r ! ø 3
(a) 400 37C 20 (b) 400 40C19 (a) 2 2 - 5 (b) 2 2 + 5
(c) 400 C19 38 38
(d) 400 C 20 (c) - 2 2 - 5 (d) - 2 2 + 5

#L Binomial Theorem Exercise 2 :


More than One Correct Option Type Questions
n
This section contains 15 multiple choice questions. 33. If n is a positive integer and (3 3 + 5) 2 n + 1 = a + b,
Each question has four choices (a), (b), (c) and (d) out of where a is an integer and 0 < b < 1, then
which MORE THAN ONE may be correct.
(a) a is an even integer
31. If in the expansion of (1 + x )m (1 - x )n , the coefficients (b) ( a + b ) 2 is divisible by 2 2n + 1
of x and x 2 are 3 and - 6 respectively, the values of m (c) the integer just below (3 3 + 5 ) 2n + 1 divisible by 3
and n are
(d) a is divisible by 10
(a) 3 (b) 6 (c) 9 (d) 12
32. If the coefficients of rth, (r + 1)th and (r + 2)th terms in 34. If (8 + 3 7 )n = P + F, where P is an integer and F is a
proper fraction, then
the expansion of (1 + x )14 are in AP, then r is /are
(a) P is an odd integer (b) P is an even integer
(a) 5 (b) 9
(c) F ( P + F ) = 1 (d) (1 - F ) ( P + F ) = 1
(c) 10 (d) 12
Chap 06 Binomial Theorem 493

n
35. The value of x for which the 6th term in the expansion
of
40. If f (n ) = S æç 30 ö æ 20 ö
÷ç ÷, then
i=0 è30 - i ø è30 - i ø
7
ì ü (a) maximum value of f (n ) is 50
C 25
ï log 2 x -1
(9 + 7) 1 ï (b) f ( 0 ) + f (1 ) + f (2 ) + ... + f (50 ) = 2 50
í2 + ý is 84, is
æ 1ö x -1
ï ç ÷
è5 ø
log 2 (3 + 1) ï
(c) f (n ) is always divisible by 50
î 2 þ
(a) 4 (b) 3 (d) f 2( 0 ) + f 2(1 ) + f 2(2 ) + ... + f 2(50 ) = 100C 50
(c) 2 (d) 1 41. Number of values of r satisfying the equation
69 69 69 69
36. Consider the binomial expansion of C 3r - 1 - Cr2 = Cr 2 - 1 - C 3r is
n
æ 1 ö (a) 1 (b) 2
ç x + 4 ÷ , n Î N , where the terms of the expansion
è 2× x ø (c) 3 (d) 7
8
42. If the middle term of æç x + ö
are written in decreasing powers of x. If the coefficients 1 630
sin - 1 x ÷ is equal to ,
of the first three terms form an arithmetic progression, è x ø 16
then the statement(s) which hold good is /are the values of x is/are
(a) Total number of terms in the expansion of the binomial p p
is 8 (a) - (b) -
3 6
(b) Number of terms in the expansion with integral power of p p
x is 3 (c) (d)
6 3
(c) There is no term in the expansion which is independent of
x
43. If b 2 < ac , the sum of the coefficients in the expansion of
(d) Fourth and fifth are the middle terms of the expansion (aa 2 x 2 + 2bax + c )n , (a, b, c , a ÎR, n ÎN ), is
37. Let (1 + x 2 ) 2 (1 + x )n = a 0 + a1 x + a 2 x 2 + ... , if (a) + ve, if a > 0 (b) + ve, if c > 0
(c) - ve, if a < 0, n is odd (d) + ve, if c < 0, n is even
a1 , a 2 and a 3 are in AP, the value of n is n
æ 1 ö
(a) 2 (b) 3 44. In the expansion of ç x 2 + 1 + ÷ , n Î N , then
(c) 4 (d) 7 è x2 ø
20
æ 17 ö (a) number of terms = 2n + 1
38. 10th term of ç3 - + 3 2÷ is (b) term independent of x = 2n - 1
è 4 ø
(c) coefficient of x 2n - 2 = n
(a) an irrational number (b) a rational number
(c) a positive integer (d) a negative integer (d) coefficient of x 2 = n
20
45. The coefficient of the (r + 1)th term of æç x + ö÷ , when
39. If (1 + x )n = C 0 + C 1 x + C 2 x 2 + C 3 x 3 + ... + C n x n , 1
then è xø
C 0 - (C 0 + C 1 ) + (C 0 + C 1 + C 2 ) expanded in the descending powers of x, is equal to the
10
- (C 0 + C 1 + C 2 + C 3 ) + ... + ( - 1)n - 1 æ 1 ö
coefficient of the 6th term of ç x 2 + 2 + 2 ÷ when
è x ø
(C 0 + C 1 + C 2 + ... + C n - 1 ), when n is even integer is
expanded in ascending powers of x . The value of r is
(a) a positive value (b) a negative value
(a) 5 (b) 6
(c) divisible by 2n - 1 (d) divisible by 2n (c) 14 (d) 15
494 Textbook of Algebra

#L Binomial Theorem Exercise 3 :


Passage Based Questions
n This section contains 7 passages. Based upon each of 52. If a is the remainder when 5 40 is divided by 11 and b is
the passage 3 multiple choice questions have to be the remainder when 2 2011 is divided by 17, the value of
answered. Each of these questions has four choices (a), a + b is
(b), (c) and (d) out of which ONLY ONE is correct. (a) 7 (b) 8
Passage I (Q. Nos. 46 to 48) (c) 9 (d) 10
2n 53. If 19 93 - 13 99 is divided by 162, the remainder is
Consider (1 + x + x ) = 2 n
S
r =0
r
ar x , where a0 , a1 , a2 , ... , a2 n are (a) 8 (b) 4
real numbers and n is a positive integer. (c) 1 (d) 0
n -1
46. The value of S ar is
54. If 13 99 is divided by 81, the remainder is
r=0 (a) 13 (b) 23
n
- 3 - an 3 - an n
an - 3n n
3 + an (c) 39 (d) 55
(a) (b) (c) (d)
2 2 2 2
n /2 - 1
Passage IV (Q. Nos. 55 to 57)
47. If n is even, the value of S
r=0
a 2 r is Consider the binomial expansion R = (1 + 2x )n = I + f , where I
is the integral part of R and f is the fractional part of R, n Î N .
3n - 1 + an 3n - 1 - an Also, the sum of coefficients of R is 2187.
(a) (b)
2 4 1
3n + 1 + an 3n + 1 - 2an 55. The value of (n + Rf ) for x = is
(c) (d) 2
2 4
n +1 (a) 7 (b) 8 (c) 9 (d) 10
2
56. If ith term is the greatest term for x = 1 / 3 , then i equals
48. If n is odd, the value of S a 2 r - 1 is
r =1 (a) 4 (b) 5 (c) 6 (d) 7
3n - 1 + 2an 3n - 1 + 2an
(a) (b) 57. If kth term is having greatest coefficient, the sum of all
2 4
possible values of k, is
3n + 1 + 2an 3n + 1 - 2an
(c) (d) (a) 7 (b) 9
2 4 (c) 11 (d) 13

Passage II (Q. Nos. 49 to 51) Passage V (Q. Nos. 58 to 60)


2 20 2 40
If (1 + x + 2x ) = a0 + a1 x + a2 x + ... + a40 x . If ( x + a1 ) ( x + a2 ) ( x + a3 ) ... ( x + an ) = xn + S 1 xn - 1

49. The value of a 0 + a 2 + a 4 + ... + a 38 is + S 2 xn - 2 + ... + S n


n
(a) 2 19 (2 19 - 1 ) (b) 2 20 (219 - 1 ) where, S 1 = S ai , S 2 = S S
i=1
ai a j , S 3 = ååå ai a j ak
1£i< j £n 1 £ i< j < k £ n
(c) 2 19 (2 20 - 1 ) (d) 2 20 (2 20 - 1 )
and so on.
50. The value of a1 + a 3 + a 5 + ... + a 37 is
58. If (1 + x )n = C 0 + C 1 x + C 2 x 2 + ... + C n x n , the
(a) 2 19 (2 19 - 20 ) (b) 2 19 (2 20 - 21 )
coefficient of x n in the expansion of
(c) 219 (219 - 21 ) (d) 219 (219 - 19 )
( x + C 0 ) ( x + 3C 1 ) ( x + 5C 2 ) ... ( x + (2n + 1) C n ), is
a 39
51. The value of , is (a) n × 2n (b) (n + 1 ) × 2n
a 40
(c) n × 2n + 1 (d) n × 2n +1
(a) 2 20 (b) (c) 10 (d) 1
59. If (1 + x )n = C 0 + C 1 x + C 2 x 2 + ... + C n x n , the
Passage III (Q. Nos. 52 to 54)
coefficient of x n - 1 in the expansion of
Suppose, m divided by n, then quotient q and remainder r
( x + C 0 ) ( x + C 1 ) ( x + C 2 ) ... ( x + C n ) is
i.e. n )m(q 1 1
- (a) 2 2n - 1 - 2n
Cn - 1 (b) 2 2n - 1 - 2n
Cn
2 2
r 1 1
(c) 2 2n - 1 - 2n + 1
Cn (d) 2 2n - 1 - 2n + 1
Cn - 1
or m = nq + r , " m, n , q , r Î I and n ¹ 0 2 2
Chap 06 Binomial Theorem 495

60. Coefficient of x 7 in the expansion of Passage VII (Q. Nos. 64 to 66)


(1 + x ) 2 (3 + x ) 3 (5 + x ) 4 is n

(a) 112 (b) 224 (c) 342 (d) 416 Let us consider the binomial expression (1 + x )n = S ar xr ,
r =0

Passage VI (Q. Nos. 61 to 63) where a4 , a5 and a6 are in AP, ( n < 10 ). Consider another
binomial expression of A = ( 3 2 + 4 3 )13 n , the expression of A
Let us consider the binomial expression
n contains some rational terms Ta 1 , Ta 2 , Ta 3 , ... , Ta m
æ 3ö
m
æ 5x x- 2 ö ( a1 < a2 < a3 < ... < am ).
A = ç x2 + ÷ and B = ç + ÷
è xø è 2 2 ø n

Sum of coefficients of expansion of B is 6561. The difference of


64. The value of S ai is
i =1
the coefficient of third term to the second term in the expansion (a) 63 (b) 127
of A is equal to 117. (c) 255 (d) 511
61. The value of m is 65. The value of am is
(a) 4 (b) 5 (c) 6 (d) 7
(a) 87 (b) 88
62. If n m is divided by 7, the remainder is (c) 89 (d) 90
(a) 1 (b) 2 (c) 3 (d) 5
63. The ratio of the coefficient of second term from the 66. The common difference of the arithmetic progression
beginning and the end in the expansion of B, is a1 , a 2 , a 3 , ..., am is
(a) 125 (b) 625 (a) 6 (b) 8
(c) 3125 (d) 15625 (c) 10 (d) 12

#L Binomial Theorem Exercise 4 :


Single Integer Answer Type Questions
n
This section contains 10 questions. The answer to 50 99 × 98
73. The value of 99 - 99 × 98 50 + (97 ) 50 - ... + 99 is
each question is a single digit integer, ranging from 1×2
0 to 9 (both inclusive).
74. If the greatest term in the expansion of (1 + x ) 2n has the
67. For integer n > 1, the digit at unit’s place in the number
æ 10 11 ö
100
greatest coefficient if and only if x Î ç , ÷ and the
S
r=0
n
r ! + 2 2 is è 11 10 ø
m
3n 3n æ 1ö n
fourth term in the expansion of ç lx + ÷ is ,
68. If (1 + x + x 2 + x 3 )n = S ar x r and S ar = k and if è xø 4
3n r=0 r=0
lnk the value of ml is
S
r=0
r ar = , the value of l is
2 75. If the value of
69. The number of rational terms in the expansion of
20 (n + 2) × n C 0 × 2n + 1 - (n + 1) × n C 1 × 2n + n × n C 2 × 2n - 1 - ...
æ3 1 ö
ç 4 + 4 ÷ is is equal to k (n + 1), the value of k is
è 6ø
76. If (1 + x + x 2 + ... + x 9 ) 4 ( x + x 2
+x 3
+ ... + x 9 )
70. If 2 2006 + 2006 is divided by 7, the remainder is 45

71. The last two digits of the natural number 19 94


is ab, = S ar x r
r =1
and the value of a 2 + a 6 + a10 + ... + a 42 is l,
the value of b - 3a is the sum of all digits of l is
é n C r + 4 × n C r +1 + 6 × n C r + 2 ù
ê ú
êë + 4 × n C r + 3 + n C r + 4 úû n + l
72. If = ,
é n C r + 3 × n C r +1 + 3 × n C r + 2 ù r +l
ê ú
êë + n C r + 3 úû
the value of l is
496 Textbook of Algebra

#L Binomial Theorem Exercise 5 :


Matching Type Questions
n This section contains 5 questions. Questions 77, 78 and 79 have three statements (A, B and C) given in Column I
and five statements (p, q, r, s and t) in Column II and questions 80 and 81 have four statements (A, B, C and D)
given in Column I and four statements (p, q, r and s) in Column II. Any given statement in Column I can have
correct matching with one or more statement(s) given in Column II.

77. (B) The sum of binomial coefficients of (q) 258


Column I Column II rational terms in the expansion of
(A) If l and m are the unit’s place (1 + 3 )62 is divisible by
(p) l + m2 = 1
2
digit in mn and nm respectively, æ 1 1ö
31
(r) 259
where m and n are the number of (C) If ç x + + x 2 + 2 ÷ = a0 x - 62
è x x ø
rational and irrational terms in the
expansion of (71/ 3 + 111/ 9 )6561 + a1 x - 61 + a2 x - 60 + ... + a124 x62 ,
respectively, then then a1 + a3 + a5 + ... + a123 is
divisible by
(B) If l and m are the unit’s place (q) lm + m l = 1
(s) 260
digit in mn and nm respectively,
where m and n are the number of (t) 261
terms with integral coefficients
and number of terms with 80.
non-integral coefficients in the Column I Column II
expansion of (71/ 3 + 51/ 2 × x )600
(A) If 11n + 21n is divisible by 16, then n can be (p) 4
respectively, then
(C) If l and m are the unit’s place (r) l+m=4
(B) The remainder, when 337 is divided by 80, (q) 5
is less than
digit in mn and nm respectively,
where m and n are the number of (C) In the expansion of (1 + x )29 coefficient of (r) 6
rational and irrational terms in the (r + 1) th term is equal to that of (r + k ) th
expansion of ( 2 + 3 3 + 6 7 )10 term, then the value of k cannot be
respectively, then (D) If the ratio of 2nd and 3rd terms in the (s) 7
(s) l l l l ... ¥ = m expansion of (a + b)n is equal to ratio of
3rd and 4th terms in the expansion of
(t) l + m = lm (a + b)n + 3, then n is less than

78.
Column I Column II 81.
æ 18 ö æ 18 ö æ 18ö æ 20ö Column I Column II
(A) If ç ÷ + 2ç ÷ + ç ÷ ³ ç ÷, (p) 5
è r - 2ø è r - 1ø è r ø è 13 ø (A) If number of dissimilar terms in the (p) a+ b+ c=3
then the values of r is /are expansion of (x + 2 y + 3z)n (n Î N )
(B) The digit in the unit’s place of the (q) 6 is an2 + bn + c, then
number 183 ! + 3183 is less than (B) If number of dissimilar terms in the (q) a+ b+ c=4
(C) If the 4th term in the expansion of 7 expansion of (x + y + z)2n + 1
n (r) - (x + y - z)2n + 1 (n Î N ) is
æ 1ö
ç ax + ÷ is 5 / 2, then na is less than an2 + bn + c, then
è xø
(s) 8 (C) If number of dissimilar terms in the (r) a + b = 2c
expansion of (x - y + z)n
(t) 9
+ (x + y - z)n (n Îis even natural
79. number) is an2 + bn + c, then
Column I Column II (D) If number of dissimilar terms in the (s) b + c = 8a
Sn
(p) æ x2 + 1 + x4 ö
(A) The sum of binomial coefficients of 257 expansion of ç ÷
terms containing power of x more è x2 ø
than x 30 in (1 + x )61 is divisible by (n Î N ) is an2 + bn + c, then
Chap 06 Binomial Theorem 497

#L Binomial Theorem Exercise 6 :


Statement I and II Type Questions
n Directions (Q. Nos. 71 to 82) are Assertion-Reason 84. Statement-1 In the expansion of (1 + x )n , if coefficient
type questions. Each of these questions contains two of 31st and 32nd terms are equal, then n = 61.
statements:
Statement-1 (Assertion) and Statement-2 (Reason) Statement-2 Middle term in the expansion of (1 + x )n ,
Each of these questions also has four alternative has greatest coefficient.
choices, only one of which is the correct answer. You 85. Statement-1 The number of terms in the expansion of
have to select the correct choice as given below. n
æ 1 ö
(a) Statement-1 is true, Statement-2 is true; Statement-2 ç x + + 1÷ is (2n + 1).
is a correct explanation for Statement-1 è x ø
(b) Statement-1 is true, Statement-2 is true; Statement-2 Statement-2 The number of terms in the expansion of
is not a correct explanation for Statement-1 ( x 1 + x 2 + x 3 + ... + x m )n is n + m - 1 C m - 1 .
(c) Statement1 is true, Statement-2 is false
(d) Statement-1 is false, Statement-2 is true 86. Statement-1 4 101 when divided by 101 leaves the
remainder 4.
82. Statement-1 Greatest coefficient in the expansion of
Statement-2 (n p - n ) when divided by ‘p’ leaves
(1 + 3x ) 6 is 6 C 3 × 3 3 .
remainder zero when n ³ 2,n Î N and p is a prime number.
Statement-2 Greatest coefficient in the expansion of 87. Statement-1 11 25 + 12 25 when divided by 23 leaves the
(1 + x ) 2n is the middle term. remainder zero.
83. Statement-1 The term independent of x in the Statement-2 a n + b n is always divisible by
25
æ 1 ö (a + b ), " n Î N .
expansion of ç x 2 + 2 + 2÷ is 50 C 25 .
è x ø 88. Statement-1 The maximum value of the term
Statement-2 In a binomial expansion middle term is independent of x in the expansion of (ax 1/ 6 + bx 1/ 3 ) 9 is 84.
independent of x. Statement-2 a 2 + b = 2.

#L Binomial Theorem Exercise 7 :


Subjective Type Questions
n
In this section, there are 24 subjective questions. é 1 ù
n

89. Determine the value of x in the expression of 94. Find n in the binomial ê 3 2 + ú , if the ratio of 7th
êë 3
3 úû
( x + x log10 x ) 5 , if the third term in the expansion is
term from beginning to 7th term from the end is 1 / 6.
1000000.
95. If S n = n C 0 n C 1 + n C 1n C 2 + ... + n C n - 1n C n and if
90. Find the value of
18 3 + 7 3 + 3 × 18 × 7 × 25 Sn + 1 15
. = , find n.
(3 6 + 6 × 243 × 2 +15 × 81 × 4 +20 × 27 × 8 + 15 × 9 × 16+6 × 3 × 32 + 64) Sn 4

æ a +1 a -1 ö
10 96. If ( 1 + x )n = C 0 + C 1 x + C 2 x 2 + .... + C n x n ,
91. Simplify ç 2 / 3 - ÷ into a binomial C1 C C C n(n + 1)
èa - a + 1 a - a 1/ 2 ø
1/ 3 prove that + 2 2 + 3 3 + ... + n n =
C0 C1 C2 Cn - 1 2
and determine the terms independent of a.
21
92. Show that there will be a term independent of x in the é æ a ö æ b öù
97. Which term in the expansion of ê 3 ç ÷ + ç3 ÷ ú
expansion of ( x a + x - b )n only, if an is a multiple of è
êë bø è a ø úû
( a + b ). contains a and b to one and same power.
93. If a, b and c are the three consecutive coefficients in the 98. Find the coefficient of x r in the expansion of
expansion of a power of ( 1 + x ), prove that the index of ( x + 3 )n - 1 + ( x + 3 )n - 2 ( x + 2)
2ac + b ( a + c ) + ( x + 3 )n - 3 ( x + 2) 2 + ... + ( x + 2)n - 1 .
the power is .
b 2 - ac
498 Textbook of Algebra

99. Prove that, if p is a prime number greater than 2, 107. Let a 0 , a1 , a 2 ,... are the coefficients in the expansion of
the difference [( 2 + 5 ) p ] - 2 p + 1 is divisible by p, ( 1 + x + x 2 )n arranged order of x . Find the value of
where [. ] denotes greatest integer. ar - n C 1 ar - 1 + n C r ar - 2 - ... + ( - 1 )r n C r a 0 , where r
100. If ((x)) represents the least integer greater than x, prove is not divisible by 3.
that ( ( {( 3 + 1) 2 n } ) ), n Î N is divisible by 2n + 1 . 108. If for z as real or complex.
101. Solve the equation ( 1 + z 2 + z 4 ) 8 = C 0 + C 1z 2 + C 2 z 4 +... + C 16 z 32 ,
11 10 11 8 11 6 11 4
C 1x - C 3x + C 5x - C 7x prove that
(i) C 0 - C 1 + C 2 - C 3 + ... + C 16 = 1
+ 11C 9 x 2
- 11C 11 = 0.
(ii) C 0 + C 3 + C 6 + C 9 + C 12 + C 15
200 200
+ ( C 2 + C 5 + C 8 + C 11 + C 14 ) w
102. If g ( x ) = S ar ×xr and f ( x ) = S b r x 4 , b r = 1 for
r=0 r = 10 + ( C 1 + C 4 + C 7 + C 10 + C 13 + C 16 ) w2 = 0,
r ³ 100 and g ( x ) = f (1 + x ), show that the greatest
where w is a cube root of unity.
coefficient in the expansion of ( 1 + x ) 201 is a 100 .
109. Let f ( x ) = a 0 + a1 x + a 2 x 2 + ... + a 2 n x 2 n and
103. If (1 + x )n = C 0 + C 1 x + C 2 x 2 + ... + C n x n , find the g ( x ) = b 0 + b1 x + b 2 x 2 + ... + bn - 1 x n - 1
value of + x n + x n + 1 + ... + x 2n
.
S S ( i + j )(C i
0 £i < j £n
+ C j + C i C j ). If f ( x ) = g ( x + 1), find an in terms of n.
110. If a 0 , a1 , a 2 ,... are the coefficients in the expansion of
104. Evaluate SS
0 £ i ¹ j £ 10
21
C i × 21 C j .
(1 + x + x 2 )n in ascending powers of x, prove that
105. Find the coefficients of x 4 in the expansions of (i) a 0 a1 - a1a 2 + a 2 a 3 - ... = 0
(i) (1 + x + x + x ) . 2 3 11 (ii) a 0 a 2 - a1a 3 + a 2 a 4 - ... + a 2 n - 2 a 2 n = an + 1
(ii) (2 - x + 3 x 2 ) 6. (iii) if E 1 = a 0 + a 3 + a 6 + ...; E 2 = a1 + a 4 + a 7 +... and
E 3 = a 2 + a 5 + a 8 + ..., then E 1 = E 2 = E . 3 = 3n - 1
106. Prove the identity
1 1 111. Prove that (n - 1) 2 C 1 + (n - 3) 2 C 3 + (n - 5) 2 C 5
+
2n + 1
Cr 2n + 1
Cr + 1 + .... = n(n + 1) 2n - 3 , where C r stands for n C r .
2n + 2 1 C 0 C1 C 2 Cn
= × 2n , 112. Show that - + - ... + ( -1)n
2n + 1 Cr 1 4 7 3n + 1
r = 2n - 1
( - 1 )r - 1 r 3n × n !
use it to prove S =
n
. =
1 × 4 × 7...(3n + 1)
, where C r stands for n C r .
r =1 2n
Cr n +1

#L Binomial Theorem Exercise 8 :


Questions Asked in Previous 13 Year’s Exams
n
This section contains questions asked in IIT-JEE, 114. If the coefficients of p th, ( p + 1)th and ( p + 2)th terms in
AIEEE, JEE Main & JEE Advanced from year 2005 expansion of (1 + x )n are in AP, then [AIEEE 2005, 3M]
to year 2017.
(a) n 2 - 2np + 4 p 2 = 0
æ30ö æ30ö æ30ö æ30ö æ30ö
113. The value of ç ÷ ç ÷ - ç ÷ ç ÷ + ç ÷ (b) n 2 - n ( 4 p + 1 ) + 4 p 2 - 2 = 0
è 0 ø è10ø è 1 ø è11ø è 2ø
æ30ö æ30ö æ30ö (c) n 2 - n( 4 p + 1 ) + 4 p 2 = 0
ç ÷ + ... + ç ÷ ç ÷ is
è12ø è20ø è30ø [IIT JEE 2005, 3M]
(d) None of the above
60 30 60 40
(a) C 20 (b) C10 (c) C 30 (d) C 30
Chap 06 Binomial Theorem 499

11 10 10
115. If the coefficient of x 7 in æçax 2 +

è
÷ is equal to the 122. Let S 1 = S j( j - 1) × 10 C j , S2 = S j × 10 C j and
bx ø j =1 j =1
11 10
æ 1 ö
coefficient of x - 7 in çax - 2 ÷ , then ab is equal to
è
S3 = S j 2 × 10 C j
bx ø [AIEEE 2005, 3M] j =1 [IIT-JEE 2010]
(a) 1 (b) 1 / 2 Statement-1 S 3 = 55 ´ 2 9
(c) 2 (d) 3
Statement-2 S 1 = 90 ´ 2 8 and S 2 = 10 ´ 2 8
116. For natural numbers m and n, if
(a) Statement-1 is true, Statement-2 is true; Statement-2 is a
(1 - y )m (1 + y )n = 1 + a1y + a 2 y 2 + ... and a1 = a 2 = 10, correct explanation for Statement-1
then (m, n ) is [AIEEE 2006, 3M] (b) Statement-1 is true, Statement-2 is true; Statement-2 is
(a) (20, 45 ) (b) (35, 20 ) not a correct explanation for Statement-1
(c) ( 45, 35 ) (d) (35, 45 ) (c) Statement-1 is true, Statement-2 is false
(d) Statement-1 is false, Statement-2 is true
117. In the binomial expansion of (a - b )n , n ³ 5, the sum of
a 123. The coefficient of x 7 in the expansion of
5th and 6th terms is zero, equals (1 - x - x 2 + x 3 ) 6 , is [AIEEE 2011, 4M]
b [AIEEE 2007, 3M]
5 6 (a) - 132 (b) - 144
(a) (b) (c) 132 (d) 144
n-4 n -5
n -5 n-4 124. If n is a positive integer, then ( 3 + 1) 2n - ( 3 - 1) 2n is
(c) (d)
6 5 [AIEEE 2012, 4M]
(a) an odd positive integer
118. The sum of the series (b) an even positive integer
20
C 0 - 20 C 1 + 20 C 2 - 20 C 3 +...+ 20
C 10 is [AIEEE 2007, 3M] (c) a rational number other than positive integer
1 (d) an irrational number
(a) - 20C10 (b) 20
C10
2 125. The term independent of x in the expansion of
20
(c) 0 (d) C10 10
æ x +1 x -1 ö
n ç - ÷ is
119. Statement-1 S (r + 1) n C r
r=0
= (n + 2) × 2n - 1 è x 2 / 3 - x 1 / 3 + 1 x - x 1/ 2 ø [JEE Main, 2013, 4M]
n (a) 120 (b) 210
Statement-2 S (r + 1) n C r
r=0
xr (c) 310 (d) 4

n n -1
126. The coefficients of three consecutive terms of (1 + x )n + 5
= (1 + x ) + nx (1 + x ) . [AIEEE 2007] are in the ratio 5 : 10 : 14, the value of n is
(a) Statement-1 is true, Statement-2 is true; Statement-2 is a [JEE Advanced 2013M]
correct explanation for Statement-1
127. If the coefficients of x 3 and x 4 in the expansion of
(b) Statement-1 is true, Statement-2 is true; Statement-2 is
(1 + ax + bx 2 ) (1 - 2x )18 in powers of x are both zero,
not a correct explanation for Statement-1
then (a, b ) is equal to [JEE Main 2014, 3M]
(c) Statement-1 is true, Statement-2 is false
æ 272 ö æ 272 ö
(d) Statement-1 is false, Statement-2 is true (a) ç14, ÷ (b) ç16, ÷
è 3 ø è 3 ø
120. The remainder left out when 8 2 n - (62) 2 n +1 is divided by æ 251 ö æ 251 ö
(c) ç14, ÷ (d) ç16, ÷
è 3 ø è 3 ø
9, is [AIEEE 2009, 4M]
11
(a) 8 (b) 0 (c) 2 (d) 7 128. Coefficient of x in the expansion of
121. For r = 0, 1, 2, ..., 10, let A r , Br and C r denote respectively, (1 + x ) (1 + x ) (1 + x 4 )12 is [JEE Advanced 2014, 3M]
2 4 3 7

the coefficients of x r in the expansion of (a) 1051 (b) 1106 (c) 1113 (d) 1120
10
129. The sum of coefficients of integral powers of x in the
(1 + x )10 , (1 + x ) 20 and (1 + x ) 30 , S A r ( B 10 Br
r =1
- C 10 A r )
binomial expansion of (1 - 2 x ) 50 , is [JEE Main 2015, 4M]
is equal to [IIT-JEE 2010, 5M] 1 50 1 50
(a) (2 + 1 ) (b) (3 + 1 )
(a) B 10 - C 10 (b) A10 ( B 10 - C10A10 ) 2 2
(c) 0 (d) C10 - B 10 1 1
(c) (3 50 ) (d) (3 50 - 1 )
2 2
500 Textbook of Algebra

130. The coefficients of x 9 in the expansion of 132. Let m be the smallest positive integer such that the
(1 + x )(1 + x 2 )(1 + x 3 )...(1 + x 100 ) is coefficient of x 2 in the expansion of
[JEE Advanced 2015, 4M] (1 + x ) 2 + (1 + x ) 3 +......+(1 + x ) 49 + (1 + mx ) 50 is
æ 2 4 ö
n (3n + 1) 51 C 3 for some positive integer n. Then the value
131. If the number of terms in the expansion of ç1 - + ÷ , of n is
è x x2 ø [JEE Advanced 2016, 3M]

x ¹ 0 is 28, then the sum of the coefficients of all the terms 133. The value of
in this expansion, is [JEE Main 2016, 4M] ( 21 C 1 - 10
C 1 ) + ( 21 C 2 - 10
C 2 ) + ( 21 C 3 - 10
C3 )+
(a) 243 (b) 729 (c) 64 (d) 2187 21
( C4 - 10 21
C 4 )+......+( C 10 - 10
C 10 ) is
[JEE Advanced 2017, 4M]
(a) 2 20 - 210 (b) 2 21 - 211
(c) 2 21 - 210 (d) 2 20 - 2 9

Answers
Exercise for Session 1 46. (b) 47. (d) 48. (b) 49. (c) 50. (b) 51. (c)
1. (c) 2. (a) 3. (c) 4. (c) 5. (b) 6. (b) 52. (c) 53. (d) 54. (d) 55. (b) 56. (a) 57. (b)
7. (c) 8. (d) 58. (b) 59. (b) 60. (d) 61. (c) 62. (a) 63. (d)
64. (b) 65. (c) 66. (d)
67. (0) 68. (3) 69. (3) 70. (8) 71. (6) 72. (4)
Exercise for Session 2
73. (0) 74. (3) 75. (4) 76. (9)
1. (b) 2. (c) 3. (d) 4. (b) 5. (c) 6. (c)
77. (A) ® (q, r); (B) ® (p, q, t); (C) ® (s)
7. (c) 8. (b) 9. (a) 10. (d)
78. (A) ® (r, s, t); (B) ® (s, t); (C) ®(p, q, r, s, t)
Exercise for Session 3 79. (A) ® (p, q, r, s); (B) ® (p, q, r, s, t); (C) ® (p, q, r, s, t)
1. (a) 2. (c) 3. (d) 4. (b) 5. (c) 6. (c) 80. (A) ® (q, s); (B) ® (p, q, r, s); (C) ® (q, s); (D) ® (r, s)
7. (a) 8. (c) 9. (a) 10. (c) 81. (A) ® (p, r); (B) ® (q); (C) ® (s); (D) ® (p, r)
82. (d) 83. (c) 84. (b) 85. (b) 86. (d) 87. (c)
Exercise for Session 4 88. (a)
1. (c) 2. (b) 3. (c) 4. (a) 5. (a) 6. (c) 89. x = 10 or 10 -5 / 2 90. 1 91. 210 94. 9 95. 4,2
7. (b) 8. (c) 9. (b) 10. (b) 11. (a) 12. (d)
13. (a) 14. (b) 98. nC r (3 n - r - 2 n - r ) 101. x = cot ç
æ r p ö , r = ± 1, ± 2,... , ± 5
97. 10 ÷
è 11 ø
Chapter Exercises ì 2n ! ü 1é 42! ù
103. n 2 × 2 n + ní2 2n - 1 - 104. ê 2 40 -
1. (d) 2. (d) 3. (a) 4. (b) 5. (d) 6. (b) 2ý ú
î 2(n!) þ 2ë 2(21!)2 û
7. (b) 8. (c) 9. (a) 10. (c) 11. (b) 12. (a)
13. (b) 14. (c) 15. (a) 16. (b) 17. (a) 18. (a) 2n + 1
105. (i) 990 (ii) 3660 107. 0 109. Cn + 1
19. (c) 20. (d) 21. (d) 22. (d) 23. (b) 24. (d)
113. (b) 114. (b) 115. (a) 116. (d) 117. (d) 118. (b)
25. (b) 26. (b) 27. (b) 28. (a) 29. (d) 30. (a)
31. (c,d) 32. (a,b) 33. (a,d) 34. (a,d) 35. (c,d) 36. (b,c) 119. (a) 120. (c) 121. (d) 122. (b) 123. (b) 124. (d)
37. (b,c) 38. (a,d) 39. (b,c) 40. (a,b,d) 41. (c,d) 42. (a,d) 125. (b) 126. (6) 127. (b) 128. (c) 129. (b) 130. (8)
43. (a,b,c,d) 44. (a,c) 45. (a,d) 131. (b) 132. (5) 133. (a)
5. We have,

Solutions
n -r r n -r r - n 7r
æ qö æ p7 ö n -
3r
+
Tr + 1 n
= Cr ç ÷ ç10 ÷ = Cr (q ) 2 10 ´ (p ) 2 10
è pø ç q3 ÷
è ø
5n - 8r 12r - 5n
= nCr × q 10 ×p 10

For coefficient of pq, we put


n é æ 1 ör æ3ö
r
æ7ö
r ù 5n - 8r 12r - 5n
1. Q å ( - 1)r nCr ê ç ÷ + ç ÷ + ç ÷ + Lupto m termsú = 1, =1
è ø è 4ø è8ø
r=0
ëê 2 úû 10 10
n n n Þ 5n - 8r - 10 = 0, 12r - 5n - 10 = 0
æ 1ö æ 3ö æ 7ö
= ç1 - ÷ + ç1 - ÷ + ç1 - ÷ + L upto m terms Þ r = 5, n = 10
è 2ø è 4ø è 8ø
1 1 1 \ T6 = 10C 5 pq =252 pq
= n + 2n + 3n + Lupto m terms
2 2 2 6. We have,
1 é (5 + 2 6 )n = (5 + 24 )n
æ ö ù
m
1
ê 1 - ç ÷ ú
2n êë è 2n ø ú æ 1 ö æ 1 ö Now, let I + f = (5 + 24 )n …(i)
= û=
ç n ÷ ç1 - mn ÷
æ 1ö è 2 -1 ø è 2 ø 0 £ f <1 …(ii)
ç1 - n ÷
è 2 ø and f ¢ = (5 - 24 )n …(iii)

\ f (n ) =
1 0 < f ¢ <1 …(iv)
2n - 1 On adding Eqs. (i) and (iii), we get
\ ò-33 f ( x 3 ln x ) × d ( x 3 ln x ) I + f + f ¢ = 2k (even integer)
1 Þ f ¢ + f ¢ =1
= ò3 × (3 x 2 ln x + x 2 )dx Þ f ¢ =1 - f
x 3 ln x
- 3 (2 -1) 2
\ f - f + If - I = f ( f - 1 ) + I ( f - 1 )
Since, ln x cannot be defined for x < 0.
= ( f -1) (I + f )
\ Above integral cannot be calculated.
= - (1 - f ) ( I + f ) = - f ¢( I + f )
2. Coefficient of (a 3 × b 6 × c 8 × d 9 × e × f ) in given expansion
= - (5 - 24 )n (5 + 24 )n
31 !
= ( - 1 ) 9 × ( - 1 )1 × ( - 1 )1 × = - (25 - 24 )n = - 1
3 !6 !8 !9 !1 !1 !
3. General term of given expression = a negative integer
1
=
10 !
2 a /2 × 3 b /3 × 5 g /6 …(i) 7. Given, x + =1 Þ x 2 - x + 1 = 0
a! b! g ! x
2
Þ ( x + w) ( x + w ) = 0
a, b, g satisfying two following property
0 £ a, b, g £ 10; a + b + g = 10 Þ x = - w, - w2
\ a = 0, 2, 4, 6, 8, 10; b = 0, 3, 6; g = 0, 6 1 1
\ p = ( - w) 4000 + = w4000 + 4000
Hence, possible pairs of ( a, b, g ) = ( 4, 6, 0 ); ( 4, 0, 6 ); (10, 0, 0 ) ( - w) 4000 w
\There exists three rational terms. 1 w2 + 1 w
=w+ = = - = -1
So, sum of rational terms w w w
10 ! 2 2 10 ! 2 1 10 ! 5 2
x = - w , p = -1
= ×2 ×3 + 2 ×5 + 2 =12632 Also, for
4 !6 ! 4 !6 ! 10 !
For n >1, 2n = 4k, k Î N
4. We have, n
(1 + x - 3 x 2 ) 2145 = a 0 + a1x + a 2x 2 + L \ 2 2 = 2 4k = (16 )k = last digit number is 6
n
On putting x = - 1, we get Now, q = unit digit at unit place in the number (2 2 + 1 )
a 0 - a1 + a 2 - K = ( - 3 ) 2145 = 6 +1 = 7
But we know that, \ p + q = -1 + 7 = 6
31 = 3, 3 2 = 9, 3 3 = 27, 3 4 = 81 æ 1ö
n
(1 + x + x 2 )n
8. Now, ç x + 1 + ÷ =
\ a 0 - a1 + a 2 + K = [( - 3 ) 4 ]536 ( - 3 )1 è x ø xn
\ End digit of ( - 3 ) 2145 Since, (1 + x + x 2 )n is of the form
= End digit of [( - 3 ) 4 ]536 ´ End digit of ( -3 )1 a 0 + a1x + a 2x 2 + L + a 2n x 2n which contains 2n + 1 terms.
= 1 ´3 =3 \ 2n + 1 = 401 Þ 2n = 400 Þ n =200
which is greater than 199.
502 Textbook of Algebra

n -1 Cr n n -1 n
Cr 14. We have, (1 + x ) 20 = 20C 0 + 20C1 x + 20C 2 x 2 + L + 20C 20 x 20
9. We have, å n n
= å n+1
r=0 Cr + Cr + 1 r = 0 Cr + 1 On dividing by x, we get
n -1Cr n
r +1 n -1 (1 + x ) 20 20C 0 20 20 20
= å = å = + C1 + C 2x + C 3x 2 + L + 20
C 20 x 19
n+1n
r=0 r = 0 n +1 x x
Cr
r +1 On differentiating w.r.t. x, we get
1 n(n + 1 ) n 20 (1 + x )19 × x - (1 + x ) 20 - 20C 0
= [1 + 2 + L + n ] = = = +0+ 20
C2
n+1 2 (n + 1 ) 2 x2 x2
10. Here, n =100, so the total number of terms is 101. + 2 ×20 C 3x + L + 19 ×20 C 20 x18
\ Largest term = Middle term = 51th term On putting x =1, we get
50 50 100
æb ö æb ö æb ö 1
= 100
C 50 ç ÷ ç ÷ = 100
C 50 ç ÷ 20 (2 )19 - (2 ) 20 = - + 20
C 2 + 2 ×20 C 3+ L + 19 ×20 C 20
è2ø è2ø è2ø 1
3 20
æ æç 1 ö÷ ö \ C 2 + 2 ×20 C 3 + L + 19 ×20 C 20 = 1 + 10 × 2 20 - 2 20 = 1 + 9 × 2 20
ç ç ÷÷
11. We have, T4 = C 3 ç x è 1 + log x ø ÷ ( x 1/12 ) 3 =200
6
[given] 1 (2 2000 - 1 )
ç ÷ 15. We have, S = = 2 2000 - 1 = (2 2 )1000 - 1
è ø 2 -1
3
2(1 + log x ) = (5 - 1 )1000 - 1
Þ 20 ( x ) x 1 /4 = 200
3
+
1 = (51000 - 1000C1 × 5 999 + 1000
C 2 × 5 998 L
2(1 + log x ) 4
Þ x = 10 1000
+ C 998 × 5 2 - 1000C 999 × 5 + 1 ) - 1
On taking logarithm on base 10, we get
= 5 (5 999 - 1000C1 × 5 998 + 1000
C 2 × 5 997- L - 1000C 999 )
é 3 1ù
ê + ú log x = 1 \Remainder is 0.
ë 2 (1 + log x ) 4 û n
æ 1ö (1 + x ) 2n
(6 + 1 + log x ) log x 16. Now, (1 + x )n ç1 + ÷ =
Þ =1 è xø xn
4 (1 + log x )
n
(log x ) 2 + 3 log x - 4 = 0 æ 1ö
Þ \Coefficient of x -1 in (1 + x )n ç1 + ÷
è xø
Þ (log x + 4 ) (log x - 1 ) = 0
(2n )!
Þ log x = - 4, 1 = Coefficient of xn -1 in (1 + x ) 2n = 2nCn - 1 =
(n - 1 )! (n + 1 )!
\ x = 10 -4 , 10
4 4
But x >1 17. Q 19 9 = (20 - 1) 9 = (20 - 1) 6521 = - 1 + (6521 ) ´ 20 + multiple
\ x =10 of 100
m m+1 = - 1 + 20 + multiple of 100
12. Q (1 + x ) + (1 + x ) + L + (1 + x )n
= 19 + multiple of 100
(1 + x )m {(1 + x )n - m + 1 - 1 } (1 + x )n + 1 - (1 + x )m 4
= = \Last two digits of the number 19 9 is 19.
(1 + x ) - 1 x
1
æ a ö
\ Coefficient of xm in 18. T2 = nC1 (13 a )n - 1 ç ÷ = 14 a
5/ 2
[given]
(1 + x )m + (1 + x )m + 1 + L + (1 + x )n è a -1 ø
n -1
(1 + x )n + 1 - (1 + x )m
1
1+
or coefficient of xm in Þ n(a ) 13 a 2 = 14 a 5/2
x
n -1
or coefficient of xm + 1 in (1 + x )n + 1 - (1 + x )m Þ na 13 a 3/2 = 14 a 5/2
= n + 1Cm + 1 - 0 = n + 1Cm + 1 When we put n =14, then it satisfies the above equation
n
C 3 14 C 3 14 - 3 + 1
13. We have, 39
C 3r - 1 + 39
C 3r = 39Cr 2 + 39
Cr 2 \ = = =4
-1
n
C 2 14 C 2 3
Þ 40
C 3r = Cr 2 40
19. 6 83 + 8 83 = (7 - 1) 83 + (7 + 1) 83

Þ 3 r = r 2 or 40 - 3r = r 2 = 2 (7 83 + 83
C 2 × 7 81 + 83
C 4 × 7 79+ K + 83 C 807 3 + 83 C 827 )
83
= 2 { 49m + C 82 × 7 }
Þ r = 0, 3 or r 2 + 3r - 40 = 0
where, m is an integer
Þ (r + 8 ) (r - 5 ) = 0 Þ r = 0, 3, 5, - 8
= 98m + 2 ×83 C1 × 7 = 98m + 2 × 83 × 7
But r = 0, - 8 do not satisfy the given equation
\ r = 3, 5 = 98m + 2 (77 + 6 ) × 7 = 49 (2m + 22 ) + 84
Chp 06 Binomial Theorem 503

20
= 49 (2m + 22 ) + 49 + 35 24. å r (20 - r ) ´ ( 20Cr ) 2
= 49 (2m + 23 ) + 35 = 49n + 35 r=0
20 20
where n is an integer.
= å r ´ 20Cr (20 - r ) ´ 20C 20 -r = å 20 ×19 Cr - 1 ´ 20 ´ 19C19 -r
Hence, the remainder is 35. r=0 r=0

20. In the expansion of (31/4 + 2 2/3 )12, the general term is = 400 å
20
19
Cr - 1 ´ C19 - r19
r 2r r=0
3-
Tr + 1 = 12Cr (31/4 )12 - r (2 2/3 )r = 12Cr 3 4 23 = 400 ´ Coefficient of x18 in (1 + x )19 (1 + x )19
Now, 0 £ r £ 12 = 400 ´ 38 C18 = 400 ´ 38C 20
\ r = 0, 12
25. Given, (3 + x 2008 + x 2009 ) 2010 = a 0 + a1x + a 2x 2 + L + an xn
Rational terms are T0 + 1 and T12 + 1
On putting x = w and w2 respectively, we get
Now, T1 = 12C 03 32 0 = 27
(3 + w2008 + w2009 ) 2010 = a 0 + a1w + a 2 w2 + a 3 w3 + L
Þ T13 = 12C123 0 × 2 8 = 256
or (3 + w + w2 ) 2010 = a 0 + a1 w + a 2 w2 + a 3 w3 + L
\ Required sum = T1 + T13
= 27 + 256 = 283 or 2 2010 = a 0 + a1 w + a 2 w2
21. N = 7100 - 3100 = (7 2 ) 50 - (3 2 ) 50 + a 3 w3 + a 4 w4 + a 5 w5 + a 6 w6 + L …(i)
= (50 - 1 ) 50 - (10 - 1 ) 50 and [3 + ( w2 ) 2008 + ( w2 ) 2009 ]2010
= [(50 ) 50 - 50C1 (50 ) 49 + 50
C 2(50 ) 48 - 50C 3 = a 0 + a1 w2 + a 2 w4 + a 3w6 + a 4 w8 + a 5 w10 + a 6 w12 + L
(50 ) 47 + L + 50
C 48 (50 ) 2 - 50C 49(50 ) + 1 ] or (3 + w2 + w) 2010
- [10 50 - 50C1 × 10 49 + 50C 2(10 ) 48 - 50C 3(10 ) 47 = a 0 + a1w2 + a 2 w4 + a 3 w6 + a 4 w8+ a 5 w10 + a 6 w12 + L
+L+ 50
C 48 (10 ) 2 - 50C 49(10 ) + 1 ] Þ2 2010 = a 0 + a1w2 + a 2w4 + a 3w6 + a 4 w8+ a 5 w10 + a 6 w12 …(ii)
= [10 4 m - 50C 47(50 ) 3 + 50C 48(50 ) 2 - 50C 49 (50 ) + 1 ] On adding Eqs. (i) and (ii), we get
2 ´ 2 2010 = 2a 0 + a1 ( w + w2 ) + a 2( w2 + w4 )
- [10 4 n - 50C 47(10 ) 3 + 50
C 48(10 ) 2 - 50C 49(10 ) + 1 ]
+ a 3 ( w3 + w6 ) + a 4 ( w4 + w8 )
when m and n are integers.
+ a 5( w5 + w10 ) + a 6( w6 + w12 ) + L
= 10 4 p - 50 C 3[(50 ) 3 - (10 ) 3 ] + 50 C 2[(50 ) 2
= 2a 0 - a1 - a 2 + 2a 3 - a 4 - a 5 + 2a 6 - L
- (10 ) 2 ] - 50C1[(50 ) - (10 )]
2010 1 1 1 1
When p is an integer. Þ 2 = a 0 - a1 - a 2 + a 3 - a 4 - a 5 + a 6 - L
2 2 2 2
= 10 4 p - 124 ´ 196 ´ 10 5 + 294 ´ 10 4 - 2000 = 10 4 q - 2000 n
æ 1 ö ( x 4 - 2 x 2 + 1 )n ( x 2 - 1 ) 2n
26. Now, ç x 2 - 2 + 2÷
= =
When q is an integer. è x ø x 2n x 2n
= 10 4 q - 10 4 + 10 4 - 2000 = 10 4 (q - 1 ) + 8000 \Total number of terms that are dependent of x is equal to
\ Last four digits = 0000 + 8000 = 8000 number of terms in the expansion of ( x 2 - 1 ) 2n that have
22. Let P = 5 99 = 5 ´ 5 98 = 5 (25) 49 = 5 (26 - 1) 49 degree of x different from 2n, which is given by
(2n + 1 ) - 1 = 2n.
= 5[ 49C 0(26 ) 49 - 49C1(26 ) 48 + 49
C 2(26 ) 47
27. Given expansion can be rewritten as [1 + x 2(1 - x )]8
- L + 49C 48(26 ) - 49C 49 × 1 ]
= 8C 0 + 8C1x 2(1 - x ) + 8C 2 x 4 (1 - x ) 2
= 5 ´ 26k - 5, when k is an integer.
P 5 8 + 8C 3x 6(1 - x ) 3 + 8C 4 x 8(1 - x ) 4 + 8C 5x10(1 - x ) 5 + L
\ = 10k - = 10k - 1 +
13 13 13 There are only two terms, which we get the coefficient of x10.
Hence, the remainder is 8. \Coefficient of x10 = 8C 4 [Coefficient of x 2 in (1 - x ) 4 ]
3 2003 3 2 ´ 3 2001 9 9 + 8C 5 [Coefficient of x 0 in (1 - x ) 5]
23. Now, = = (3 3 ) 667 = (28 - 1 ) 667
28 28 28 28
9 = 8C 4 ( 4 C 2 ) + 8C 5 (1 )
= {(28 ) 667 - 667C1(28 ) 666 + 667C 2(28 ) 665 - L + 667C 666(28 ) - 1 }
28 = ( 8C 4 )( 4 C 2 ) + 8C 3 = (70 ) (6 ) + 56 = 476
9 28. (1 + ix ) 4n - 2 = 4n - 2
C0 + 4n - 2
C1(ix ) + 4n - 2
C 2(ix ) 2
= 9k - , where k is an integer.
28 + L + 4n - 2C 4n - 2 (ix ) 4n - 2
19
= (9k - 1 ) + Here, we see that Ist negative term is T3 and the next term is T7
98
and the last negative term is T4n - 1.
ì 3 2003 ü ì 19ü 19
or í ý = í(9k - 1 ) + ý = Now, 3, 7, L, 4n - 1
î 28 þ î 28þ 28 It is an AP.
504 Textbook of Algebra

Q l = a + ( N - 1 )d r 14 - (r + 1 ) + 1
Þ 2= +
\ 4n - 1 = 3 + ( N - 1 ) 4 14 - r + 1 r +1
Þ n -1 = N -1 Þ N = n r 14 - r
æ n ö æm ö n! m! Þ 2= +
29. Q ç ÷ç ÷ = ´ 15 - r r +1
èm ø è p ø m !(n - m )! p !(m - p )!
Þ 2 (15 - r ) (r + 1 ) = r (r + 1 ) + (15 - r ) (14 - r )
n! æn ö æ n - p ö
= =ç ÷ç ÷ Þ - 2 r 2 + 28 r + 30 = 2 r 2 - 28 r + 210
(n - m )! p !(m - p )! è p ø èm - p ø
n æn ö æ n - p ö
Þ 4 r 2 - 56 r + 180 = 0 Þ r 2 - 14 r + 45 = 0
n n æ n ö æm ö n
\ å å ç ÷ ç ÷ = å å ç ÷ç ÷
p =1 m=p èm ø è p ø p =1 m=p è p ø èm - p ø Þ (r - 9 ) (r - 5 ) = 0
n æn ö n æ n - p ö Þ r = 5, 9
= å ç ÷ å ç ÷ 33. (3 3 + 5) 2n + 1
= ( 27 + 5 ) 2n + 1
p =1 è p ø m=p èm - p ø

n æ n ö n -p æn - p ö Now, leta + b = ( 27 + 5 ) 2n + 1 …(i)


= å ç ÷ å ç ÷ [where, t = m - p]
p =1 è p ø t = 0 è t ø 0 <b <1 …(ii)
n æn ö and let b ¢ = ( 27 - 5 ) 2n + 1 …(iii)
= 2n å ç ÷ 2n - p
p =1 è p ø 0 < b¢ <1 …(iv)
n æn ö 1 éæ 1ö
n ù n n On subtracting Eq. (iii) from Eq. (i), we get
= 2n å ç ÷ p = 2n ê ç1 + ÷ -1ú =3 -2
p =1 è p ø 2 êë è 2 ø
ûú a + b - b ¢ = ( 27 + 5 ) 2n + 1 - ( 27 - 5 ) 2n + 1 …(v)
4 5 4 - r æ xr ö 8 Þ a + 0 = 2p (even integer), " p Î N
30. Given, å ç ÷=
r = 0 ( 4 - r )! è r ! ø 3 Þ a = 2p = even integer
4
(5 + x ) 8 Also, from Eq. (v), we get
Þ =
4! 3 a = ( 27 + 5 ) 2n + 1 - ( 27 - 5 ) 2n + 1 divisible by
Þ (5 + x ) 4 = 64 = (2 2 ) 4 Þ 5 + x = ± 2 2 ( 27 + 5 ) - ( 27 - 5 ), i. e. divisible by 10.
\ x = 2 2 - 5 or x = - 2 2 - 5 34. We have, (8 + 3 7 )n = (8 + 63 )n
Hence, largest real value of x is 2 2 - 5. Now, let P + F = (8 + 63 )n …(i)
31. We have,
0 < F <1 …(ii)
Coefficient of x in (1 + x )m (1 - x )n = mC1 - nC1
and let F ¢ = (8 - 63 )n …(iii)
and coefficient of x 2 in (1 + x )m (1 - x )n = mC 2 - mC1 nC1 + nC 2
m
0 < F ¢ <1 …(iv)
According to the question, C1 - nC1 = 3 On adding Eqs. (i) and (iii), we get
Þ m -n =3 …(i) P + F + F ¢ = (8 + 63 )n + (8 - 63 )n …(v)
m
and C 2 - mC1 nC1 + nC 2 = - 6
Þ P + 1 = 2 p (even integer), " p Î N
m(m - 1 ) n(n - 1 )
Þ - mn + = -6 Þ P = 2 p - 1 = odd integer
2 2
\ F ¢ =1 - F
Þ (m - n ) 2 - (m + n ) = - 12
\ (1 - F ) ( P + F ) = F ¢( P + F ) = (8 - 63 )n (8 + 63 )n
Þ 9 - (m + n ) = - 12 [from Eq. (i)]
= (64 - 63 )n = 1n = 1
or m + n = 21 …(ii)
From Eqs. (i) and (ii), we get
35. We have, 6th term in the expansion of
7
ì 1 ü
m = 12 and n = 9 ï log 2 ( 9 x - 1 + 7) x -1
+ 1) ï
í2 + 2 (1/5) log 2 (3
ý
32. Coefficient of rth, (r + 1) th and (r + 2) th terms in (1 + x )14 are ïî ïþ
14
Cr - 1, 14Cr 7
ì 1 ü
and 14 Cr + 1, respectively. or í (9 x - 1 + 7 ) + x - 1 is T6 = T5 + 1
1/ 5 ý
î (3 + 1 ) þ
Now, according to the question, 2 (14 Cr ) = 14Cr - 1 + 14
Cr + 1 5
ì 1 ü
On dividing both sides by 14
Cr , we get = 7C 5 ( 9 x - 1 + 7 ) 2í x - 1 1/ 5 ý
14 14 î (3 + 1) þ
Cr - 1 Cr + 1
2= + (9 x - 1 + 7 ) (9 x - 1 + 7 )
14
Cr 14
Cr = 7C 2 × x -1
= 21 × x - 1 = 84 [given]
(3 + 1) (3 + 1)
Chp 06 Binomial Theorem 505

Þ (9 x - 1 + 7 ) = 4 (3 x - 1 + 1 ) Þ 2 (n C 2 + 2 ) = nC1 + (n C 3 + 2 nC1 )
Let 3 x - 1 = l, then é n(n - 1 ) ù n(n - 1 )(n - 2 )
Þ êë2 2 úû + 4 = 3n + 6
l2 - 4 l + 3 = 0
Þ n 3 - 9n 2 + 26n - 24 = 0
or (l -3) (l -1) = 0
Þ (n - 2 )(n 2 - 7n + 12 ) = 0
\ l = 3, 1
Þ (n - 2 )(n - 3 )(n - 4 ) = 0
Þ 3 x - 1 = 31, 3 0
Þ n = 2, 3, 4
or x - 1 = 1, 0 or x =2, 1
n n
Hence, n = 3, 4 (n ¹ 2,Q n C 3 is not defined)
æ 1 ö æ 1/ 2 1 ö
36. ç x + ÷ or ç x + x -1/ 4 ÷ 17 1 1
+ 3 2 = (9 + 8 + 12 2 ) = (3 + 2 2 ) 2
è 2× 4 x ø è 2 ø 38. We have,
4 4 4
n 2n - 3 2 n-3
æ1ö æ1ö 17 1 æ3 ö
= nC 0 × x 2 + nC1 × ç ÷ × x 4 + nC 2 × ç ÷ ×x 2 +L Þ 3- + 3 2 = 3 - (3 + 2 2 ) = ç - 2 ÷
è2ø è2ø 4 2 è2 ø
According to the question, 20
æ 17 ö
2 \10th term in ç3 - + 3 2 ÷ is
n æ1ö æ1ö è 4 ø
C 0, nC1 ç ÷, nC 2 ç ÷ are in AP.
è2ø è2ø 20 - 9
æ3ö
æ1ö
2 T9 + 1 = 20C 9 ç ÷ (- 2 )9
\ n
C1 = nC 0 + nC 2 ç ÷ è2ø
è2ø 9
-11 +
n(n - 1 ) = 20C 9( - 1 ) 9311 × 2 2
Þ n =1 +
4 ×2 -
13

Þ n 2 - 9n + 8 = 0 = - 20C 93112 2

Þ (n - 8 )(n - 1 ) = 0 which is a negative and an irrational number.


\ n = 8, n ¹ 1 39. We have, C 0 - (C 0 + C1 ) + (C 0 + C1 + C 2 ) - (C 0
option (a) Number of terms = 8 + 1 = 9 + C1 + C 2 + C 3 ) + L ( - 1 )n - 1 (C 0 + C1 + L + Cn - 1 )
r r r
4- æ1ö - For even integer, take n = 2m, we get
option (b) Now, Tr + 1 = 8Cr × x 2 ×ç ÷ ×x 4
è2ø = C 0 - (C 0 + C1 ) + (C 0 + C1 + C 2 )
Q 0 £r £8 - (C 0 + C1 + C 2 + C 3 ) + L - (C 0 + C1+ L + C 2m - 1 )
For integral powers of x, r = 0, 4, 8 = - (C1 + C 3 + C 5 + L + C 2m - 1 )
\Number of terms in the expansion with integral power of x = - (C1 + C 3 + C 5 + L + Cn - 1 ) [Q n = 2m ]
is 3.
= - 2n - 1
option (c) From option (b),
4-
3r r 40. We have,
æ1ö
Tr + 1 = 8Cr × x 4 ×ç ÷ n æ 30 ö æ 20 ö n æ 30 ö æ 20 ö
è2ø f (n ) = å ç ÷ç ÷= å ç ÷ç
50
÷ = Cn
i = 0 è 30 - i ø è n - i ø i = 0 è i ø è n - i ø
For independent of x,
3r \ f (n ) is greatest, when n =25
4-
=0
4 \Maximum value of f (n ) is 50C 25.
16 Also, f ( 0 ) + f (1 ) + L + f (50 )
r= ÏW
3 = 50C 0 + 50
C1 + 50
C2 + L + 50
C 50 =2 50
\No terms in the given expansion which is independent of x. 50
Also, Cn is not divisible by 50 for any n as 50 is not a prime
option (d) Middle term is number.
4
æ1ö 50
T5 = 8C 4 × x × ç ÷ å ( f (n )) 2 = ( 50C 0 ) 2 + ( 50C1 ) 2 + L + ( 50C 50 ) 2 = 100 C 50
è2ø n=0
69 69
i.e. only one middle term. 41. C 3r - 1 + C 3r = 69Cr 2 -1
+ 69
Cr 2
37. We have, 70
Þ C 3r = 70Cr 2
Coefficient of x, x 2 and x 3 in (1 + x 2 ) 2 (1 + x )n
Þ r 2 = 3r or 70 - 3r = r 2
i.e., values of a1, a 2 and a 3 in (1 + 2 x 2 + x 4 ) (1 + x )n
Þ r = 0, 3 or r 3 + 3r - 70 = 0
Þ a1 = nC1, a 2 = nC 2 + 2 and a 3 = nC 3 + 2 nC1
Þ r = 0, 3 or (r + 10 )(r - 7 ) = 0
According to the question,
Þ r = 0, 3, 7, - 10
2a 2 = a1 + a 3
506 Textbook of Algebra

But r = 0, - 10 do not satisfies the given equation. On differentiating both sides w.r.t. x, we get
Hence, two values of r satisfies, n (1 + x + x 2 )n × (1 + 2 x ) = l 2n - 1 + ¼ + 2nl 0 x 2n - 1
i. e. r = 3, 7
æ8 ö On putting x = 0, we get l 2n - 1 = n
42. Here, n is even, so middle term is ç + 1÷ th, i. e. 5th term.
è2 ø Hence, coefficient of x 2n - 2 = n
4
æ sin -1 x ö 630 æ 1 ö
n
\ T5 = 8C 4 ( x ) 4 ç ÷ = [given] and coefficient of x 2 in ç x 2 + 1 + 2 ÷
è x ø 16 è x ø

Þ 70 (sin -1 x )4 =
630
Þ (sin -1 x ) 4 =
9 = Coefficient of x 2n + 2 in (1 + x 2 + x 4 )n
10 16 = Coefficient of xn + 1 in (1 + x + x 2 )n
-1 3 3
Þ (sin x ) 2 = Þ sin -1 x = ± dn + 1
4 2 = (1 + x + x 2 )n ¹ n
p p dxn + 1
é -1 æ p p öù
Þ x=- , êQ sin x Î çè - 2 , 2 ÷ø ú 20
3 3 ë û æ 1ö
45. Now, ç x + ÷ = 20C 0 x 20 + 20
C1x18 + 20
C 2 x16
43. Sum of coefficients = (aa 2 + 2ba + c )n è xø
20
Let f ( a ) = aa 2 + 2ba + c + C 3 x14 + ¼

Now, D = 4b 2 - 4ac = 4 (b 2 - ac ) < 0 + 20


C9 x 2 + 20
C10 + C11x -2 + ¼ +
20
C 20 x - 20
20

\ f ( a ) < 0 or f ( a ) > 0, " a Î R Tr + 1 = 20Cr × x 20 - 2r …(i)


If a > 0, then f ( a ) > 0 10 20
æ 1 ö æ1 ö
Þ(aa 2 + 2ba + c )n > 0 and ç x 2 + 2 + 2 ÷ = ç + x÷
è x ø èx ø
If c > 0, i. e. f ( 0 ) > 0 Þ f ( a ) > 0
= 20C 0 x - 20 + C1x - 18 +
20
C 2x - 16
20
Þ (aa 2 + 2ba + c )n > 0
20 20
+ ¼+ C10 + C11x 2 + 20
C12 x 4
If a < 0, then f ( a ) < 0
20
Þ (aa 2 + 2ba + c )n < 0, if n is odd + ¼+ C 20 x 20
If c < 0, then f ( 0 ) < 0 Þ f ( a ) < 0 \ T6 = T5 + 1 = 20C 5 x - 10 …(ii)
Þ (aa 2 + 2ba + c )n > 0, if n is even. 20 20
According to the question, Cr = C 5
n
æ 1 ö (1 + x 2 + x 4 )n \ r = 5 or 20 = r + 5 Þ Sr = 5, 15
44. Q ç x 2 + 1 + 2 ÷ =
è x ø x 2n
a 0 + a1x 2 + a 2x 4 + ¼ + a 2n x 4n
n
Sol. (Q. Nos. 46 to 48)
= 2n
x 2n 46. We have, (1 + x + x 2 )n = å ar xr …(i)
r=0
\Number of terms = 2n + 1
n 1
æ 1 ö On replacing x by , we get
Term independent of x = an = Constant term in ç x 2 + 1 + 2 ÷ x
è x ø n 2n r
æ 1 1 ö æ1ö
= Coefficient of x 2n in (1 + x 2 + x 4 )n ç1 + + 2 ÷ = å ar ç ÷
è x x ø r=0 èxø
= Coefficient of xn in (1 + x + x 2 )n 2n
n Þ (1 + x + x 2 )n = S ar x 2n - r …(ii)
d
= (1 + x + x 2 )n ¹ 2n - 1 r=0
dxn From Eqs. (i) and (ii), we get
n
æ 1 ö 2n 2n
Coefficient of x 2n - 2 in ç x 2 + 1 + 2 ÷ S ar xr = S ar x 2n - r
è x ø r=0 r=0

= Coefficient of x 4n - 2 in (1 + x 2 + x 4 )n Equating the coefficient of x 2n - r on both sides, we get


= Coefficient of x 2n - 1 in (1 + x + x 2 )n a 2n - r = ar …(iii)
0 £ r £ 2n
Now, let (1 + x + x 2 )n = l 0 + l1x + l 2 x 2 + K
On putting r = 0, 1, 2, 3, ¼, n - 1, n, we get
+ l 2n - 1x 2l - 1 + l 2n x 2n a 2n = a 0
1 a 2n - 1 = a1
On replacing x by , we get
x a 2n - 2 = a 2
n a 2n - 3 = a 3
æ 1 1 ö l1 l 2 l 2n - 1 l
ç1 + + 2 ÷ = l0 + + 2 + ¼ + 2n - 1 + 22nn M M M
è x x ø x x x x
an + 1 = an - 1, an = an
or (1 + x + x 2 )n = l 2n + l 2n - 1 x + ¼ + l1x 2n - 1 + l 0 x 2n
Chp 06 Binomial Theorem 507

Then, a 0 + a1 + a 2 + ¼ + an - 1 On differentiating both sides of Eq. (v) w.r.t. x and put x = 0,


= an + 1 + an + 2 + ¼ + a 2n …(iv) we get
and on putting x = 1 in Eq. (i), we get a 39 = 20 (2 )19 …(vii)
2n
å ar = 3n 49. a 0 + a 2 + a 4 + ¼ + a 38 + a 40 = 219 (2 20 + 1) [from Eq. (iii)]
r=0 19 20
Þ a 0 + a 2 + a 4 + ¼ + a 38 = 2 (2 + 1 ) - a 40
Þ(a 0 + a1 + a 2 + ...+ an -1 ) + an + (an + 1 + an + 2 + ...+ a 2n ) = 3n 19 20 20
= 2 (2 + 1) - 2 [from Eq. (vi)]
From Eq.(iv), we get 19 20
= 2 (2 - 1)
2 (a 0 + a1 + a 2 +¼+ an - 1 ) = 3n - an
n -1
50. a1 + a 3 + a 5 + ¼ + a 37 + a 39 = 219 (2 20 - 1) [from Eq. (iv)]
(3n - an )
or å ar = Þ a1 + a 3 + a 5 + ¼ + a 37 = 2 (2 19 20
- 1 ) - a 39
r=0 2
19 20
47. On putting x = 1 and x = - 1 in Eq. (i), we get = 2 (2 - 1 ) - 20 (2 )19
[from Eq. (vii)]
3n = a 0 + a1 + a 2 + ¼ + a 2n …(v)
= 219 (2 20 - 21 )
1 = a 0 - a1 + a 2 - a 3 + ¼ + a 2n …(vi)
On adding and subtracting Eqs. (v) and (vi), we get a 39 20 (2 )19
51. From Eqs. (vi) and (vii), we get = = 10
3n + 1 a 40 2 20
= (a 0 + a 2 + ¼ + a 2n ) …(vii)
2
n
3 -1
n Sol. (Q. Nos. 52 to 54)
= (a1 + a 3 + a 5 + ¼ + a 2n - 1 ) …(viii)
2 52. 5 40 = (5 2 ) 20 = (22 + 3) 20 = 22l + 3 20, l Î N
Also, ar = a 2n - r Also, 3 20 = (3 2 )10 = (11 - 2 )10 = 11m + 210, m Î N
Put r = 0, 2, 4, 6, ¼,n - 1 Now, 210 = 1024 = 11 ´ 93 + 1
a 0 = a 2 n , a 2 = a 2 n - 2, a 4 = a 2 n - 4 , ¼
\ Remainder, a = 1
a n -1 = an + 1
Also, 2 2011 = 2 3(2 4 ) 502 = 2 3(17 - 1 ) 502
From Eq. (vii), we get
3n + 1 = 8 [(17 ) 502 - 502
C1(17 ) 501 + ¼ - 502
C 501(17 ) + 1 ]
= 2 (a 0 + a 2 + ¼ + an - 2 ) + an
2 = 8 (17 l + 1 ), l Î N = 8 ´ 17 l + 8
n
3 + 1 - 2 an \Remainder, b =8
= a 0 + a 2 + ¼ + an - 1
4 Hence, a + b =1 + 8 =9
48. From Eq. (viii), we get 53. 19 93 - 13 99 = (odd number) - (odd number) = even number
3n - 1 \ 19 93 - 13 99 is divisible by 2.
= 2 (a1 + a 3 + ¼ + an ) - an
2
Now, 19 93 - 13 99 = (18 + 1 ) 93 - (12 + 1 ) 99
3n - 1 + 2an
= (a1 + a 3 + ¼ + an ) = [(18 ) 93 + 93
C1(18 ) 92 + 93
C 2(18 ) 91¼ + ¼ + 93
C 92 (18 ) + 1 ]
4
99 99 98 99 97 99
- [(12 ) + C1 (12 ) + C 2 (12 ) + ¼ + C 98 (12 ) + 1 ]
n
Sol. (Q. Nos. 49 to 51) 2 93 2 99
= (18 ) l + C1 ´ 18 - (12 ) m - C1(12 )
Given, (1 + x + 2 x 2 ) 20 = a 0 + a1x + a 2x 2 + ¼ + a 40 x 40
When l and m are integers
On putting x = 1 and x = - 1 respectively, we get
= (18 ) 2 l - (12 ) 2m + 486
a 0 + a1 + a 2 + ¼ + a 40 = 4 20 = 2 40 …(i)
20 = 81 ´ 4 l - 12 2 ( 99C 2 + 12 × 99C 3 ) + 81 p + 486
and a 0 - a1 + a 2 - ¼ + a 40 = 2 …(ii)
From Eqs. (i) and (ii), we get = 81 (integer), where p is an integer.
a 0 + a 2 + a 4 + ¼ + a 38 + a 40 = 219 (2 20 + 1 ) …(iii) But 2 and 81 are co-prime.

and a1 + a 3 + a 5 + ¼ + a 37 + a 39 = 219 (2 20 - 1 ) …(iv) \ 19 93 - 13 99 is divisible by 162.


1 54. 13 99 = (12 + 1) 99 = (12) 99 + 99C1(12) 98 + 99C 2(12) 97 + ¼ +
Also, replacing x by in given expression, we get
x 99
C 97 (12 ) 2 + 99 C 98(12 ) + 1
20
æ 1 2 ö a1 a 2 a a a
= {(12 ) 99 + 99
C1(12 ) 98 + 99
C 2(12 ) 97
ç1 + + 2 ÷ = a0 + + 2 + ¼ + 38 + 39 + 40
è x x ø x x x 38 x 39 x 40 99
+ ¼+ C 97 (12 ) 2 } + 99
C1 (12 ) + 1
Þ (2 + x + x 2 ) 20 = a 0 x 40 + a1x 39
= 81 l + 99 ´ 12 + 1, where l is an integer
+ ¼ + a 38x 2 + a 39 x + a 40 …(v)
= 81 l + 81 ´ 14 + 55
On putting x = 0, we get a 40 = 2 20 …(vi)
\ Remainder = 55
508 Textbook of Algebra

n Sol. (Q. Nos. 55 to 57) = x 9 + (1 + 1 + 3 + 3 + 3 + 5 + 5 + 5 + 5 ) x 8


n
Here, (1 + 2 ) = 2187 + (1 × 1 + 1 × 3 + 1 × 3 + 1 × 3 + 1 × 5 + ¼ + 1 × 5 + 3 × 3
n
3 = 2187 = 3 Þn = 7 7 + 3 × 3 + 3 × 5 + ¼ + 5 × 5) x 7 + ¼
1 \Coefficient of x 7 = (1 + 1 + 3 + 3 + 3 + 5 + 5 + 5 + 5 ) 2
At x= , R = ( 2 + 1)7 = I + f
2 - (1 2 + 1 2 + 3 2 + 3 2 + 3 2 + 5 2 + 5 2 + 5 2 + 5 2 )
7
55. Let f ¢ = ( 2 - 1) , 0 < f ¢ < 1 2
\ Rf ¢ = ( 2 + 1 ) 7 ( 2 - 1 ) 7 = (1 ) 7 = 1 (31 ) 2 - (129 ) 961 - 129
= = = 416
\ (n + Rf ) = 7 + 1 = 8 2 2
(n + 1 ) ( x ) Sol. (Q. Nos. 61 to 83)
56. Here, m = n

1+x Since, sum of coefficient of B is 6561.


æ 1ö 2 æ5 1ö
n
(7 + 1 ) ç2 ´ ÷ 8´ \ ç + ÷ = 6561
è 3ø 3 = 16 = 3.2 è2 2ø
= =-
1 5 5
1+2´ Þ 3n = 6561 Þ 3n = 3 8
3 3
\ n =8
T[m ] + 1 = T3 + 1 = T4
61. Coefficient (T3 - T2 ) = 117
57. Here, n = 7
7
m
C 23 2 - mC131 = 117
C 7 -1 7
\Greatest coefficient = or C 3 Þ m =6
2 m 6 6
7 62. n = 8 = (1 + 7) = (1 + 7k )
C 7+1
and , i. e. 7C 4 Hence, remainder is 1.
2 8
Sum of values of k = (3 + 1 ) + ( 4 + 1 ) = 9 æ 5x x - 2 ö 7
æ5ö æ1ö
Coefficient of T2 in ç + ÷ 8
C1 ç ÷ ç ÷
è2 2 ø è2ø è2ø
n
Sol. (Q. Nos. 58 to 60) 63. 8
= 7
= 5 6 = 15625
æx -2
58. ( x + C 0 ) ( x + 3 × C1 ) ( x + 5 × C 2 ) + ¼ { x + (2n + 1) × Cn } 5x ö 8 æ1ö æ5ö
C1 ç ÷ ç ÷
Coefficient of T2 in ç + ÷ è2ø è2ø
è 2 2 ø
= x n + 1 + x n {C 0 + 3 × C1 + 5 × C 2 + ¼ + (2n + 1 ) × Cn }
\Coefficient of xn = C 0 + 3 × C1 + 5 × C 2 + ¼ + (2n + 1 ) × Cn n
Sol. (Q. Nos. 64 to 66)
= (C 0 + C1 + C 2 + ¼ + Cn ) Qa 4 , a 5, a 6 i.e., n C 4 , nC 5, nC 6 are in AP, then
+ 2 {C1 + 2 × C 2 + ¼ n × Cn }
2 ×n C 5 = nC 4 + nC 6
n ì n (n - 1 ) ü
= 2 + 2 ín + 2 × + ¼ + ný C4 n n
C 5 n -6 + 1
î 2 þ Þ 2= + n 6= +
C5 n
C5 n - 5 + 1 6
ì (n - 1 ) (n - 2 ) ü
= 2n + 2n í1 + (n - 1 ) + + ¼ + 1ý 5 n -5
î 1 ×2 þ Þ 2= +
n-4 6
n n -1 n -1 n -1 n -1
= 2 + 2n { C0 + C1 + C2 + ¼ + Cn - 1 }
Þ 12n - 48 = 30 + n 2 - 9n + 20
n n -1 n n n
= 2 + 2n (1 + 1 ) = 2 + n × 2 = (n + 1 ) 2 Þ n 2 - 21n + 98 = 0 Þ n = 7,14
59. ( x + C 0 ) ( x + C1 ) ( x + C 2 ) + ¼ + ( x + Cn ) Hence, n =7 [Q n < 10 ]
n
æ n ö æ ö Also, A = ( 3 2 + 4 3 )13n = (21/3 + 31/4 ) 91
= x n + 1 + ç S Cr ÷ xn + ç å å CiC j ÷ xn - 1 + ¼
èr = 0 ø è0 £i j £n ø \ Tr + 1 = 91Cr (21/3 ) 91 - r × (3 1/4 )r
\Coefficient of xn - 1 in åå CiC j 91- r
0 £i £ j £n = 91Cr × 2 3 × 3r /4 …(i)
1 ìï æ n öüï 1
2 n 7
ö æ n 64. å ai = å ai = a1 + a 2 + a 3 + ¼ + a 7
= í ç å Cr ÷ - ç å Cr2 ÷ý = {2 2n - 2nCn }
2 ïî èr = 0 ø èr = 0 øï 2
þ
i= 1 i =1

= 7C1 + 7C 2 + 7C 3 + ¼ + 7C 7 = 2 7 - 1 = 127
2n - 1 1 2n
=2 - × Cn
2 65. From Eq. (i), we get
60. ( x + 1) ( x + 3) ( x + 5) 4
2 3 0 £ r £ 91
For rational terms, r = 4, 16, 28, 40, 52, 64, 76, 88
= ( x + 1 ) ( x + 1 ) ( x + 3 ) ( x + 3 ) ( x + 3 )( x + 5 ) ( x + 5 )
(x + 5) (x + 5) Rational terms are T5, T17, T29, T41, T53, T65, T77, T89
\ am = 89
Chp 06 Binomial Theorem 509

4
66. Also, 5, 17, 29, 41 53,…, 89 71. We have, 19 9 = (20 - 1) 6561
are in AP with common difference 12.
= (20 ) 6561 - 6561
C1 (20 ) 6560 + 6561
C 2(20 ) 6559
2n
67. The unit digit of 2 is always 6 for n > 1. 6561
-¼- C 6559 (20 ) 2 + 6561
C 6560(20 ) 2 - 1
100
Now, å r ! = 0 ! + 1 ! + 2 ! + 3 ! + 4 ! + 10 (k ); k Î N = 1000 k - 6561
C 2( 400 ) + 6561
C1(20 ) - 1
r=0
where, k is an integer.
= 1 + 1 + 2 + 6 + 24 + 10 k = 34 + 10 k
100 n
= 1000 p + 6561 ´ 20 - 1 = 1000 p + 131220 - 1
\ Unit digit of å r ! + 2 2 where, p is an integer.
r=0
100
= 1000 p + 131219
n
= Unit place of å r ! + Unit place of 2 2 \ ab = 19
r=0
i.e. a = 1, b = 9
= 4 + 0 + 6 = 10, its unit place is 0.
3n
Hence, b - 3a = 9 - 3 = 6
68. Given, åar xr = (1 + x + x 2 + x 3 )n 72. n Cr + 4 ×n Cr + 1 + 6 ×n Cr + 2 + 4 ×n Cr + 3 + nCr + 4
r =0
= (n Cr + nCr + 1 ) + 3 (n Cr + 1 + nCr + 2 )
It is clear that ar is the coefficient of xr in the expansion of
(1 + x + x 2 + x 3 )n . + 3 (n Cr + 2 + nCr + 3 ) + (n Cr + 3 + nCr + 4 )
n+1
1 = Cr + 1 + 3 ×n + 1 Cr + 2 + 3 ×n + 1 Cr + 3 + n + 1 Cr + 4
On replacing x by in the given equation , we get
x = (n + 1Cr + 1 + n+1
Cr + 2 ) + 2 (n + 1Cr + 2
r
3n
æ1ö (1 + x + x 2 + x 3 )n + n+1
Cr + 3 ) + (n + 1Cr + 3 + n+1
Cr + 4 )
å ar ç ÷ =
r=0 è x ø x 3n n+ 2 n+ 2 n+ 2
= Cr + 2 + 2 × Cr + 3 + Cr + 4
Here, ar represents the coefficient of 3 3n - r in (1 + x + x 2 + x 3 )n . n+ 2 n+ 2 n+ 2 n+ 2
=( Cr + 2 + Cr + 3 ) + ( Cr + 3 + Cr + 4 )
Thus, ar = a 3n - r …(i) n+ 3 n+ 3
3n 3n = Cr + 3 + Cr + 4
Let I = å r ´ ar = å (3n - r ) a 3n - r
n + 4n+ 3
r=0 r=0
= n + 4Cr + 4 = Cr + 3
[replacing r by 3n - r ] r+ 4
3n
Similarly, n Cr + 3 ×n Cr + 1 + 3 ×n Cr + 2 + nCr + 3 = n + 3Cr + 3
= å (3n - r ) ar [from Eq. (i)]
r=0 n+4 n+l
3n 3n \ = Þ l=4
= 3n å - å r ar r+4 r+l
r=0 r=0 99 × 98
3nk 73. 99 50 - 99 × 98 50 + (97 ) 50 - ¼ + 99
Þ 2 I = 3nk Þ I = \ l =3 1 ×2
2 = 99 50 - 99C1 (98 ) 50 + 99C 2 (97 ) 50 - ¼ + 99
C 98 × 1
20 - r
99 50 99 50 99
69. We have, Tr + 1 = Cr 20
×4 3 ×6 - r /4 = C 0(99 ) - C1 (99 - 1 ) + C 2 (99 - 2 ) 50 - ¼
40 - 2r 99
+ C 98 (99 - 98 ) 50 - 99C 99 (99 - 99 ) 50
= 20Cr ×2 3 × 2 - r /4 × 3 - r /4
160 - 11r
= (99 ) 50 { 99C 0 - 99C1 + 99
C2 - ¼ + 99
C 98 - 99C 99 }
= 20Cr × 3 - r /4
× 2 12 + 50
C1 (99 ) 49 { 99C1 - 2 ×99 C 2 + 3 ×99 C 3 - ¼}
r 160 - 11r =0+0=0
For rational terms, and must be integers and
4 12 74. Given,
0 £ r £ 20. Q Greatest term in the expansion of (1 + x ) 2n has the greatest
r coefficient.
\ is an integer.
4 \ Tn + 1 = 2nCn xn (greatest term)
Þ r = 0, 4, 8, 12, 16, 20
160 - 11r \ Tn < Tn + 1 > Tn + 2
Clearly, for r = 8, 16 and 20 is also an integer. Þ 2n
Cn - 1 × xn - 1 < 2nCn × xn > 2nCn + 1 × xn + 1
12
\The number of rational terms is 3. 2n
Cn - 1 1 2n
Cn + 1
Þ × < 1 > ×x
70. We have, 2 2006 = 2 2 (2 3 ) 668 2n
Cn x 2n
Cn
= 4 (1 + 7 ) 668 = 4 (1 + 7k ) = 4 + 28k n 1 n
Þ × <1 > x
\ 2 2006 + 2006 = 4 + 28k + 7 ´ (286 ) + 4 n+1 x n+1
n n+1
Hence, remainder is 8. Þ x> and x <
n+1 n
510 Textbook of Algebra

æ n n + 1ö For rational term, r should be a multiple of 9, i.e.,


i.e., x Îç , ÷ r = 0, 9, 18,¼, 6561
èn + 1 n ø
Total rational terms, m = 730
æ 10 11 ö
Given, x Îç , ÷ and irrational terms, n = 6562 - 730 = 5832
è 11 10 ø
Let l = unit digit of (730 ) 5832 = 0
\ n = 10
m 3 and m = unit digit of (5832 ) 7300.
æ 1ö æ1ö n
Also, T4 in ç lx + ÷ = mC 3( lx )m - 3 ç ÷ = [given] = unit digit of (2 5 )145 × 2 5
è xø èxø 4
n = (2 ) (2 ) = 4
m
C 3 × lm - 3 × xm - 6 = [given]
4 \ lm + m l = ( 0 ) 4 + ( 4 ) 0 = 1
10 and l+m=0+4=4
= [Qn = 10]
4 (B) General term, tr + 1 = 600Cr (71/3 ) 600 - r ( x51/2 )r
5
= 600 - r
2 = 600
Cr (7 ) 3 5r /2 x r
Put m - 6 = 0, we get
For rational term, r should be multiple of 6.
m =6
i. e. r = 0, 6, ¼, 600
6 5
\ C 3 × l3 = \Total rational terms, m = 101
2
3
and total irrational terms,
5 1 1 æ1ö n = 601 - 101 = 500
Þ l3 = ´ = =ç ÷
2 20 8 è 2 ø Let l = unit digit of (m )n
1
\ l= = unit digit of (101 ) 500 = 1
2
1 and m = unit digit of (500 )101 = 0
Hence, ml = 6 ´ =3
2 \ l + m 2 = (1 ) 2 + ( 0 ) 2 = 1
2

75. We know that, ( x - 1)n = nC 0xn - nC1xn - 1 + nC 2xn - 2 - ¼ lm + m l = (1 ) 0 + ( 0 )1 = 1


\ x 2( x - 1 )n = nC 0xn + 2 - nC1xn + 1 + nC 2 xn - ¼ and l + m = 1 + 0 = 1 = lm
On differentiating w.r.t. x, we get 10 !
(C) ( 2 + 3 3 + 6 7 )10 = S ( 2 ) a ( 3 3 )b ( 6 7 ) g
2 x ( x - 1 )n + x 2 × n( x - 1 )n - 1 a !b ! g !
10 !
= (n + 2 ) nC 0 xn + 1 - (n + 1 ) nC1xn + n ×n C 2 xn - 1 - ¼ =S 2 a /2× 3 b /3× 3 g /6
a !b ! g !
On putting x = 2, we get Q a + b + g = 10
(n + 2 ) nC 0 2n + 1 - (n + 1 ) ×n C1 × 2n + n × nC 2 2n - 1 - ¼ For rational terms, a = 0, 2, 4, 6, 8, 10, b = 0, 3, 6, 9 and
= 4 + 4n = 4 (1 + n ) g = 0, 6
Possible triplets are ( 4, 6, 0 ), (10, 0, 0 ), ( 4, 0, 6 ).
\ k=4
\Total rational terms, m = 3
76. On putting x = 1, - 1, i , - i in the given expression, we get
Total irrational term, n = 10 + 2C 2 - 3 = 63
10 4 ´ 9 = a1 + a 2 + a 3 + a 4 + ¼ + a 45 …(i)
Let l = unit’s place digit of 3 63 = (3 4 )15 × 3 3 = 1 ´ 27 = 7
0 = - a1 + a 2 - a 3 + a 4 + ¼ - a 45 …(ii)
and m = unit’s place digit of 63 3 = unit digit of 3 3 = 7
(1 + i ) 4 × i = a1i - a 2 - a 3i + a 4 + ¼
Þ - i (2i ) 2 = - a1i + a 2 + a 3i - a 4 - ¼ Now, l l l l l¼¥ =m
Þ 4i = - a1i + a 2 + a 3i - a 4 - ¼ …(iii) ì1 1 1 ü 1/ 2
í + + + ¼ý
and - 4i = a1i + a 2 - a 3i - a 4 - ¼ …(iv) Þ lî 24 8 þ
= m l1 - 1/2 = m \ l = m = 7
On adding Eqs. (i), (ii), (iii) and (iv), we get æ 18 ö æ 18 ö æ18 ö æ20 ö
4 (a 2 + a 6 + a10 + a14 + ¼ + a 42 ) = 9 ´ 10 4 78. (A) Given, ç ÷+2ç ÷+ç ÷³ç ÷
èr - 2ø èr - 1 ø è r ø è13 ø
or a 2 + a 6 + a10 + ¼ + a 42 = 22500 = l [given] æ 18 ö æ 18 ö æ 18 ö æ18 ö æ20 ö
\Required sum = 2 + 2 + 5 + 0 + 0 = 9 Þ ç ÷+ç ÷+ç ÷+ç ÷³ç ÷
èr - 2 ø èr - 1 ø èr - 1 ø è r ø è13 ø
77. (A) General term, Tr + 1 = 6561Cr (71/3 ) 6561 - r (111/9 )r
æ 19 ö æ19 ö æ20 ö
æ 6561 - r ö (r /9) Þ ç ÷+ç ÷³ç ÷
= 6561
Cr 7 ç ÷ 11 èr - 1 ø è r ø è13 ø
è 3 ø
Chp 06 Binomial Theorem 511

æ20 ö æ20 ö T2 T ¢3
Þ ç ÷ ³ ç ÷ Þ 7 £ r £ 13 (D) Given, =
èr ø è7ø T3 T ¢4
\ r = 7, 8, 9, 10, 11, 12, 13
n
C1(a )n - 1 (b )1 n+ 3
C 2 (a )n + 1(b ) 2
Þ =
(B) The unit digit of 183! is 0.
n
C 2 (a )n - 2 (b ) 2 n+ 3
C 3(a )n (b ) 3
Now, 3183 = (3 4 ) 45(3 ) 3 n
C1 a n+ 3
C2 a
183 45
Þ n
´ = n+ 3
×
Unit digit of 3 = Unit digit of (81 ) ´ Unit digit of 27 C2 b C3 b
=1 ´7 =7 C 2 n + 3C 3
n
Þ =
\Unit digit of 183! + 3183 = 0 + 7 = 7 C1 n + 3C 2
n

which is less than 8 and 9. n -2+ 1 n + 3 -3 + 1


Þ =
æ1ö 5
3 2 3
(C) T4 = nC 3 (ax )n - 3 ç ÷ = Þ 3n - 3 = 2n + 2
èxø 2
5 1 \ n =5
Þ n C 3 a n - 3x n - 6 = Þ n = 6 and a =
2 3 81. (A) Number of dissimilar terms in the expansion of
1 ( x + 2y + 3z )n (n Î N )
\ na = 6 ´ = 2
3 1 3
= n + 3 - 1C 3 - 1 = n + 2C 2 = n 2 + n + 1
79. (A) The coefficient of power of x more than x 30 in (1 + x ) 61 is 2 2
61 61 61 1 3
C 31 + C 32 + ¼ + C 61 \ a = ,b = ,c =1
61
2 2
We know that, (1 + 1 ) = C 0 + 61C1 +
61 61
C2 + ¼ + 61
C 61 1 3
61 61 61 61 Hence, a + b + c = + + 1 = 3
Þ 2 = 2 ( C 31 + C 32 + ¼ + C 61 ) 2 2
Þ 61
C 31 + 61
C 32 + ¼ + 61
C 61 = 2 60 1 3
and a + b = + = 2 = 2c
2 2
Hence, 2 60 is divisible by 2 57, 2 58, 2 59, 2 60.
(B) We have,
(B) General term is, Tr + 1 = 62Cr ( 3 )r = 62Cr 3r /2
( x + y + z ) 2n + 1 = {( x + y ) + z } 2n + 1
For rational term = r should be multiple of 2.
= ( x + y ) 2n + 1 + 2n + 1
C1( x + y ) 2n z
i.e. r = 0, 2, 4, 6,¼, 62
2n + 1
\ Required sum = T1 + T3 + ¼ + T63 + C 2 ( x + y ) 2n - 1z 2 + 2n + 1
C 3 ( x + y ) 2n - 2z 3 + ¼
= 62C 0 + 62C 2 + ¼ + 62C 62 = 2 62 - 1 = 2 61 2n + 1
C 2n ( x + y ) z 2n + 2n + 1
C 2n + 1z 2n + 1
61 57 58 59 60 61
Hence, 2 is divisible by 2 , 2 , 2 , 2 , 2 .
and ( x + y - z ) 2n + 1 = {( x + y ) - z } 2n + 1
(C) Put x = 1 and x = - 1 in given expression, then we get
= ( x + y ) 2n + 1 - 2n + 1C1( x + y ) 2n z + 2n + 1C 2( x + y ) 2n -1z 2
4 31 = a 0 + a1 + a 2 + a 3 + ¼ + a124 …(i)
- 2n + 1C 3( x + y ) 2n - 2z 3 + ...+ 2n + 1C 2n ( x + y )z 2n - 2n + 1C 2n + 1z 2n+1
and 0 = a 0 - a1 + a 2 - a 3 + ¼ + a124 …(ii)
On subtracting Eq. (ii) from Eq. (i), we get \( x + y + z ) 2n + 1 - ( x + y - z ) 2n + 1
2 62 = 2 (a1 + a 3 + ¼ + a123 ) = 2 { 2n + 1C1 ( x + y ) 2n z + 2n + 1
C 3 ( x + y ) 2n - 2z 3 + ...+ z 2n + 1 }
Þ 2 61 = (a1 + a s + ¼ + a123 ) \The number of dissimilar terms in the expansion of
\ a1 + a 3 + ¼ + a123 is divisible by 2 57, 2 58, 2 59, 2 60, 2 61. ( x + y + z ) 2n + 1 - ( x + y - z ) 2n + 1
80. (A) (11)n + (21)n = (16 - 5)n + (16 + 5)n = (2n + 1 ) + (2n - 1 ) + (2n - 3 ) + ¼ 5 + 3 + 1
n n
= 2 [ C 0 (16 ) + C 2 (16 ) n n-2
(5 ) 2 (n + 1 )
= (2n + 1 + 1 ) = (n + 1 ) 2
n-4 2
n
+ C 4 (16 ) (5 ) + C 6(16 )n - 6(5 ) 6 + ¼ ]
4 n

= n 2 + 2n + 1
Hence, given expression is divisible by 16, if
n = 1, 3, 5, 7 \ a = 1, b = 2, c = 1
(B) 3 37 4 9 9
= (3 ) × 3 = (81 ) × 3 = 3 (80 + 1 ) ] 9 Hence, a + b + c =1 + 2 + 1 = 4

= 3 [ 9C1 (80 ) 8 + 9C 2 (80 ) 7 + ¼ + 1 ] (C) We have, ( x - y + z )n = { x - (y - z )}n


= xn - nC1 xn - 1(y - z ) + nC 2 xn - 2 (y - z ) 2
\Remainder of 3 37 is 3.
+ nC 3 xn - 3 (y - z ) 3
(C) Tr + 1 = Tr + k Þ 29Cr = 29Cr + k - 1
Þ 29 - r = r + k - 1 Þ 30 = 2 r + k [Q r £ 29 ] + ¼- nCn - 1x (y - z )n - 1 + nCn (y - z )n
For even values of k, i.e., k = 0, 2, 4, 6, 8, ¼, 28, and ( x + y - z )n = ( x + y - z )n
512 Textbook of Algebra

= xn + n C1 xn -1(y - z ) + n C 2 xn - 2(y - z ) 2 + n C 3 xn - 3(y - z ) 3 \Number of terms = 1 + n + n = 2n + 1


n
+ ...+ Cn -1 x (y - z ) n -1
+ (y - z ) n \Both the statements are correct but Statement-2 is not the
correct explanation of Statement-1.
\ ( x - y + z )n + ( x + y - z )n
86. 4101 - 4 = 4 ( 4100 - 1) = 4 (16 50 - 1)
= 2 [ xn + nC 2 xn - 2 (y - z ) 2
= 4 (16 25 + 1 ) (16 25 - 1 )
+ nC 4 xn - 4 (y - z )n + ¼ + (y - z )n ]
= 4 (divisible by 16 + 1) (divisible by 16 - 1)
\The number of dissilmilar terms in the expansion of = divisible by 102
( x - y + z )n + ( x + y - z )n = 1 + 3 + 5 + ¼ + (n + 1 ) \4101 - 4 is divisible by 102.
(n + 2 )
or if 4101 is divisible by 102, then remainder is 4.
(n + 2 ) 2 1 2
= 2 (1 + n + 1 ) = = (n + n + 1 ) \Statement-1 is false but Statement-2 is obviously true.
2 4 4
1 87. Q( xn + an ) is always divisible by ( x + a ) when n is odd natural
\ a = , b = 1, c = 1 number. Therefore, (11 25 + 12 25 ) is divisible by 11 + 12 = 23.
4
Hence, b + c = 1 + 1 = 2 = 8a \Statement-1 is always true but Statement-2 is false.
ån for n even natural number.
æx + 1 + x ö
2 4
(D)Q ç ÷ 9 -r r
-
è x2 ø 88. Tr + 1 = 9Cr (ax1/6 ) 9 -r (bx - 1/3 )r = 9Cr × a 9 - r × br × x 6 3

a 0 + a 2 x 2 + a 4 x 4 + ¼ + a 2n (n + 1) x 2n (n + 1) 9 -r 5
= For independent of x, put - =0
xn (n + 1) 6 3
1 Þ 9 - r - 2r = 0
\Number of terms = × 2n (n + 1 ) + 1 = n 2 + n + 1
2 Þ r =3
\ a = 1, b = 1, c = 1 \ T3 + 1 = 9C 3 × a 6b 3 = 84a 6b 3
Hence, a + b + c = 1 + 1 + 1 = 3
Now using A M ³ GM
and a + b = 1 + 1 = 2 = 2c
a2 + b 2
82. Statement-2 is obviously correct. Þ ³ (a 2b )1/2 Þ ³ (a 2b )1/2 [Q a 2 + b = 2 ]
2 2
Now, we have (1 + 3 x ) 6 = 6C 0 + 6C1(3 x ) + 6C 2(3 x ) 2
\ a 2b £ 1 Þ(a 2b ) 3 £ 1 3 Þ84a 6b 3 £ 84
6 3 6 4 6 5 6 6
+ C 3 (3 x ) + C 4 (3 x ) + C 5 (3 x ) + C 6 (3 x )
\ T4 £ 84
\Greatest coefficient in (1 + 3 x ) 6 is 6C 6 3 6. Hence, both statements are true and Statement-2 is the correct
So, Statement-1 is wrong. explanation of Statement-1.
æ 1 ö
25
æ 1ö
50 89. We have, 10000 = T3 = T2 + 1 = 5C 2 x ( 5 - 2) ( x log10 x ) 2
83. We have, ç x 2 + + 2÷ = çx + ÷
è x2 ø è xø Þ 100000 = x 3 × x 2 log10 x = x 3 + 2 log10 x
r 5
æ1ö Þ 3 + 2 log10 x = log x 100000 = 5 log x 10 =
\ Tr + 1 = 50Cr × C 50 - r× ç ÷ = 50Cr x 50- 2r
èxø log10 x
For independent of x, we put Þ2 (log10 x ) 2 + 3 log10 x - 5 = 0
50 - 2r = 0 Þ r = 25 Put log10 x = y , we get
\ T25 + 1 = 50C 25 2 5
2y + 3y - 5 = 0 Þ y = - or 1
n 2
But in binomial expansion of ( x + a ) , middle terms is 5
independent of x, iff x × a = 1. \ log10 x = - or 1
2
84. We have, Þ x = 10 or 10 - 5/2
Coefficient of 31st term in (1 + x )n = Coefficient of 32nd term
18 3 + 7 3 + 3 × 18 × 7 × 25
in (1 + x )n 90. We have,
é3 6 + 6 × 243 × 2 + 15 × 81 × 4 + 20 × 27 × 8 ù
Þ Coefficient of T30 + 1 = Coefficient of T31 + 1 ê ú
n ë + 15 × 9 × 16 + 6 × 3 × 32 + 64 û
Þ C 30 = nC 31 Þn = 30 + 31 = 61
(18 + 7 ) 3 (25 ) 3 (25 ) 3
Hence, both statements are correct but Statement-2 is not the = = = =1
(3 + 2 ) 6 (5 ) 6 (25 ) 3
correct Explanation of Statement-1.
10
æ 1 ö æ
n
1ö æ 1ö
2 æ a+1 a -1 ö
85. We have, ç x + + 1÷ = 1 + nC1 ç x + ÷ + nC 2 ç x + ÷ 91. We have, ç - ÷
è x ø è x ø è xø è a 2/ 3 - a 1/ 3 + 1 a - a 1/ 2 ø
n 10
æ 1ö é (a1/3 ) 3 + 1 3 (a1/2 ) 2 - 1 2 ù
+ ¼ + nCn ç x + ÷ = ê 2/ 3 - 1/ 2 1/ 2 ú
è xø 1/ 3
ë a - a + 1 a (a - 1 ) û
Chp 06 Binomial Theorem 513

10
æ a 1/ 2 + 1 ö According to the question, we have
= ç(a1/3 + 1 ) - ÷ = (a1/3 - a - 1/2 )10 n-6
è a 1/ 2 ø n
C6 × 2 3 × 3- 2 1
n - 12 n - 12
1
10 1/ 3 10 - r - 1/ 2 r 6 -n
= Þ2 3 ×3 3 =
Now, Tr + 1 = Cr (a ) (- a ) ...(i) 6 6
n
10 - r - r
C6 × 4 ×3 3
× n - 12
10
= Cr a 3 2 ( - 1 )r n - 12
Þ (6 ) 3 = 6- 1 Þ = - 1, n = 9
It will be independent of a, if 2
10 - r r 95. We know that, (1 + x )n ( x + 1)n
- = 0 Þ20 - 2 r - 3 r = 0
3 2 = [n C 0 + nC1 x + nC 2 x 2 + ¼+ n Cn xn ]
Þ r =4
´ [n C 0 xn + nC1 xn - 1 + nC 2 xn - 2 + ¼ + nCn ]
Putting r = 4 in Eq. (i), we get
Equating coefficient of xn + 1 on both sides, we get
T5 = 10C 4 ( - 1 ) 4 = 10C 4 = 210
2n
Cn + 1 = [n C 0nC1 + n C1nC 2 + ¼ + nCn - 1nCn ]
92. The general term in ( xa + x - b )n is
\ Sn = 2nCn + 1
Tr + 1 = nCr ( xa )n -r ( x - b )r = nCr xa (n - r ) - br = nCr xan - (a + b ) r
2n + 2
Sn + 1 15 Cn + 2 15
For independent of x, we must have an - (a + b ) = r = 0 But = Þ =
2n
an Sn 4 Cn + 1 4
Þ r= Þ an = (a + b ) r , r Î N
a+b (2n + 2 )
2n + 1
Cn + 1 15
Þ × =
Þ an is multiple of (a + b ). (n + 2 ) 2n
Cn + 1 4
93. Let n be the index of power in (1 + x ). Then, n Cr = a …(i)
2 (n + 1 ) 2n + 1
Cn 15
n Þ × =
Cr + 1 = b …(ii) (n + 2 ) 2n
Cn - 1 4
n
and Cr + 2 = c …(iii) 2 (n + 1 ) 2n + 1 15
Þ × =
From Eqs. (i) and (ii), we get (n + 2 ) n 4
n
Cr a Þ 8 (2n 2 + 3n + 1 ) = 15n 2 + 30n
n
=
Cr + 1 b Þ n 2 - 6n + 8 = 0
r +1 a an - b \ n = 4, 2
Þ = Þ r= …(iv)
n -r b a+b C1 C2 C3 C
96. + 2× + 3× + ¼+ n× n
From Eqs. (ii) and (iii), we get C0 C1 C2 Cn - 1
n
Cr + 1 b n n (n - 1 ) n (n - 1 ) (n - 2 ) 2! 1
= = + 2× + 3× ´ + ¼+ n×
n
Cr + 2 c 1 2n 3! n (n - 1 ) n
r +2 b bn - b - 2c n (n + 1 )
Þ = Þr= …(v) = n + (n - 1 ) + (n - 2 ) + ¼ + 1
n -r -1 c b+c 2
21
From Eqs. (iv) and (v), we get é æ a ö æ b öù - 1/ 2 1/ 3
97. We have, ê 3 ç ÷ + ç 3 ÷ ú = [ab ] + (ba - 1/3 )1/2 ]21
bn - b - 2c an - b
= ë è bø è a øû
b+c a+b Let Tr + 1 contain a and b to one and the same power.
Þ (b 2 - ac ) n = 2ac + b (a + c ) 21 - r

2ac + b (a + c ) \ Tr + 1 = 21Cr (ab - 1/2 ) 3 (ba - 1/3 )r /2


Þ n= 21 - r r r 21 - r
(b 2 - ac ) - -
n
= 21Cr × a 3 6 ×b 2 6

é 1 ù 21 - r r r 21 - r 21 - r 2r
94. Given expansion is ê 3 2 +
3 ú
. - = - \ =
ë 3û Q
3 6 2 6 2 3
\7th term from the beginning Þ 63 - 3r = 4r Þ 63 = 7r Þ r = 9
n-6 n-6
\Required term = r + 1 = 10
= nC 6 (2 ) 3 × (3 -1/3 ) 6 = nC 6 2 3 × 3- 2
n
98. Given series is a GP.
é 1 ù é æ x + 2ö ù
n
\Again, 7th term from end in ê 3 2 + 3 ú
ë 3û ê1 - ç ÷ ú
ê è x + 3ø ú n n
n
\S = ( x + 3 )n - 1 ë û = ( x + 3 )n [( x + 3 ) - ( x + 2 ) ]
é 1 ù x+2
= 7th term from beginning in ê 3 + 3 3 ú 1- ( x + 3 )n
ë 3 û x+3
6 -n
= nC 6(3 -1/3 )n - 6(2 ) 2 = nC 63 3 ×4 = ( x + 3 )n - ( x + 2 )n = (3 + x )n - (2 + x )n
514 Textbook of Algebra

\Coefficient of xr in S = nCr 3n - r - nCr 2n - r = - Re {1 + ix }11 }

= nCr (3n - r - 2n - r ) Let x = cot q = - Re{1 - i cot q}’’


éì sin q + i cos qü11 ù ì (i )11 (cos q - i sin q)ü
11

99. Let (2 + 5 ) p or ( 5 + 2) p = I + f …(i) = - Re êí ý ú = - Re í ý


êëî sin q þ úû î sin q þ
0 £ f <1 …(ii)
ì - i (cos 11 q - i sin 11 q) ü sin 11 q
p = - Re í ý= =0 [given]
( 5 - 2) = f ¢ …(iii) î sin11 q þ sin11 q
and 0 < f ¢<1 …(iv) q¹0
\ sin 11q = rp
On subtracting Eq. (iii) from Eq. (i), we get
rp
or q = , r = ± 1, ± 2, ± 3, ± 4, ± 5
I + f - f ¢ = ( 5 + 2)p - ( 5 - 2)p 11
æ rp ö
= 2 { p C1( 5 ) p - 1 × 21 + pC 3( 5 ) p - 3 × 2 3+ ¼+ pC p ( 5 ) 02 p } \ x = cot q = cot ç ÷
è 11 ø
[Q integer value of f - f ¢ = 0] r = ± 1, ± 2,¼, ± 5
\ [( 5 + 2 ) p ] + 0 - 2 p + 1 102. Since, g( x ) = f (1 + x )
= 2 { p C1 ( 5 ) p - 1 × 2 + pC 3 ( 5 ) p - 3 × 2 3 +¼+ p C p - 2( 5 ) 2 × 2 p - 2 } 200 200
g( x ) = å ar xr and f ( x ) = å br xr
r=0 r=0
= pl = p (integer) 200 200
\ [( 5 + 2 ) p ] - 2 p + 1 is divisible by P. \ å ar xr = å br (1 + x )r
r=0 r=0

100. Let ( 3 + 1 ) 2n = I + f , 0 £ f < 1 Now, a 0 + a1 x + a 2 x 2 + ¼ + a 200 x 200


and f ¢ = ( 3 - 1 ) 2n = b 0 + b1 (1 + x ) + b 2 (1 + x ) 2+ ¼ + b100 (1 + x )100
+ b101 (1 + x )101 + ¼ + b 200 (1 + x ) 200
\I + f + f ¢ = ( 3 + 1 ) 2n + ( 3 - 1 ) 2n
Þ a 0 + a1 x + a 2 x 2 + ¼ + a 200 x 200
= [( 3 + 1 ) 2 ]n + [( 3 - 1 ) 2 ]n
= b 0 + b1 (1 + x ) + b 2 (1 + x ) 2 + ¼ + (1 + x )100
= ( 4 + 2 3 )n + ( 4 - 2 3 )n
+ (1 + x )101 + ¼ + (1 + x ) 200 [Q b100 = b101 = ¼= b 200 = 1 ]
= 2n (2 + 3 )n + 2n (2 - 3 )n
Equating the coefficient of x100, we get
= 2n × 2 {n C 02n + nC 2(2n - 2 ) (3 )
a100 = 100C100 + 101
C100 + 102
C100 + ¼+ 200
C100
n n-4 2
+ C 4 (2 ) (3 ) + ¼} = 101
C101 + 101
C100 + 102
C100 + ¼ + 200
C100
n+1
\ I + 1 =2 , k ÎI [Q f + f ¢ = 1 ] = 102
C101 + 102
C100 + ¼ + 200
C100
or [({( 3 + 1 ) 2n })] = 2n + 1k M M M M
200 200 201
= C101 + C100 = C101 …(i)
Hence, [({( 3 + 1 ) 2n })], n Î N is divisible by 2n + 1.
201
Again, greatest coefficient in the expansion of (1 + x )
101. 11C1 × x10 - 11C 3 × x 8 + 11C 5 × x 6- 11C 7x 4 + 11C 9 x 2 - 11C11 = 0
= Coefficient of middle term
Now, (1 + ix )11 = 11C 0 + 11C1ix + 11C 2 (ix ) 2 = Coefficient of T101 or coefficient of T102
201 201 201
+ 11C 3 (ix ) 3 + 11C 4 (ix ) 4 + 11C 5 (ix ) 5 = C100 or C101 = C100 …(ii)
From Eqs. (i) and (ii), we get
+ 11C 6 (ix ) 6 + 11C 7 (ix ) 7 + ¼ + 11C11 (ix )11
The greatest coefficient in the expansion of (1 + x ) 201.
Þ (1 + ix )11 = (11C 0 _ 11C 2 x 2 + 11C 4 x 4 -11 C 6 x 6
103. P = åå (i + j ) (Ci + C j + CiC j ) …(i)
+ 11C 8 x 8 + 11C10 x10 ) 0 £i < j £n

+ i (11C1 x - 11C 3 x 3 + 11C 5 x 5 - 11C 7 x 7+ 11C 9 x 9 - 11C11 x11 ) Replacing i by n - i and j by n - j in Eq. (i), we get
P = å å (n - i + n - j ) (Cn - i + Cn - j + Cn - i Cn - j )
Comparing real part on both sides, we get 0 £i < j £n
11 11 11 2 11 4 11 6
(1 + ix ) = ( C11 - C 9 x + C 7 x - C 5 x = åå (2n - i - j ) (Ci + C j + CiC j ) [Q nCr = nCn - r ] …(ii)
0 £i < j £n
+ 11C 3 x 8 - 11C1 x10 ) [Q n Cr = nCn - r ]
On adding Eqs. (i) and (ii), then we get
Þ Re {(1 + ix )11 } = - (11C1 x10 - 11C 3 x 8 + 11C 5 x 6
2 P = 2n åå (Ci + C j + CiC j )
- 11C 7 x 4 + 11C 9 x 2 - 11C11 ) 0 £i < j £n

Þ 11
C1x10 - 11C 3 x 8 + 11C 5 x 6- 11C 7 x 4 + 11C 9 x 2 - 11C11 \ P =n åå (Ci + C j ) + n åå CiC j
0 £i < j £n 0 £i < j £n
Chp 06 Binomial Theorem 515

= n × n (C 0 + C1 + C 2 +¼ + Cn ) +
n 2n 2n
(2 - Cn ) æ 2n + 1 ö é 1 1 1
=ç ÷ê - +
2 è 2n + 2 ø ë 2n + 1C1 2n + 1
C2 2n + 1
C3
2 n
2n - 1
n
= n ×2 + n ×2 - ×2n Cn 1 (2n - 1 ) ù
2 - 2n + 1
+ ¼+ 2n + 1 ú
C4 C 2n û
2 n ì 2n - 1 2n ! ü
= n × 2 + n í2 - ý æ 2n + 1 ö é 1 1 1
î 2 (n !) 2 þ =ç ÷ - +
21
è 2n + 2 ø êë 2n + 1C1 2n + 1
C2 2n + 1
C3
104. åå Ci ×21 C j
0 £ i ¹ j £ 10
1 1 2n ù
1 é 10 10 21 21 ù 10 21 2 - 2n + 1
+ ¼- 2n + 1
+ 2n + 1 ú
= êi å å Ci C j ú - å ( Ci ) C4 C 2n C 2n û
2 ë =0 j=0 û i=0
1 é 10 21 21 - r ù 1 10 21 2 æ 2n + 1 ö é æ 1 1 ö
= =ç ÷ êç - ÷
êi å Ci 2 ú - 2 iå ( Ci ) è 2n + 2 ø ë è 2n + 1C1 2n + 1
C 2n ø
2 ë =0 û =0

2 20 × 2 20 42C 21 1 é 40 ( 42 )! ù æ 1 1 ö 2n ù
= - = ê2 - ú - çç 2n + 1 - ÷ + ¼+ ú
2 2 ´2 2 ë 2 (21 !) 2 û è C2 2n + 1
C 2n - 1 ÷ø 2n + 1
C 2n úû
105. (i) We have, (1 + x + x 2 + x 3 )11 = (1 + x )11 (1 + x 2)11 æ 2n + 1 ö é 2n ù æ 2n + 1 ö æ 2n ö
=ç ÷ 0+ ú=ç ÷ç ÷
11 11 2 11
= (1 + C1 x + C 2 x + C 3 x + C 4 x + ¼) 3 11 4
è 2n + 2 ø êë 2n + 1
C 2n û è 2n + 2 ø è 2n + 1 ø
´ (1 + 11C1 x 2 + 11C 2 x 4 + ¼) 2n n
= =
\Coefficient of x 4 = 11C 2 + 11C 2 ´ 11C1 + 11C 4 2n + 2 n + 1
= 55 + 605 + 330 = 990 107. Given, (1 + x + x 2 )n = a 0 + a1 x + a 2 x 2 + ¼ + ar - 2 xr - 2
(ii) [(2 - x ) + 3 x 2 ]6 + ar -1 xr - 1 + ar xr + ¼ + a 2n x 2n …(i)
= 6 C 0(2 - x ) 6 + 6C1 (2 - x ) 5 (3 x ) 2+ 6C 2 (2 - x ) 4 (3 x 2 ) 2 + ¼ n n n n 2
and (1 - x ) = C 0 - C1x + C 2 x
= 6C 0 [ 6C 4 (2 ) 2 ] + 6C1 ´ 3 [ 5C 2(2 ) 3 ] + 6C 2 ´ 9 [ 4 C 0(2 ) 4 ] - ¼+ ( - 1 )r nCr xr + ¼ + ( - 1 )n nCn …(ii)
[equating coefficient of x 4 ] On multiplying Eqs. (i) and (ii) and equating coefficient of xr
= 60 + 1440 + 2160 = 3660 on both sides, we get
1 1 Coefficient of xr in (1 - x 3 )n
106. LHS = 2n + 1 + 2n + 1
Cr Cr + 1 = nC 0 ar - nC1ar - 1+ nC 2ar - 2 - ¼ + ( - 1 )r nCrar
(2n + 1 - r )! r ! (2n - r )! (r + 1 )! Since, r is not a multiple of 3, therefore the expression (1 - x 3 )n
= +
(2n + 1 )! (2n + 1 )! does not contain xr in any term.
é (2n + 1 - r ) + r + 1 ù \ Coefficient of xr in (1 - x 3 )n = 0
= (2n - r )! (r )! ê ú
ë (2n + 1 )! û Hence, ar - nC1 ar - 1 + nC 2 ar - 2 - ¼ + ( - 1 )r nCr a 0 = 0
(2n - r )! (r )!(2n + 2 ) 2n + 2 1
= = × 108. Given, (1 + z 2 + z 4 ) 8 = C 0 + C1 z 2+ C 2 z 4 + ¼ + C16 z 32
(2n + 1 ) (2n )! 2n + 1 2n Cr
(i) Put z = i , we get
( - 1 )r - 1 × r 2n - 1
2n - 1 æ 2n + 1 ö
Now, å = å ( - 1 )r - 1 ç ÷ (1 - 1 + 1 ) 8 = C 0 - C1 + C 2 - C 3 + ¼ + C16
r =1
2n
Cr r =1 è 2n + 2 ø
C 0 - C1 + C 2 - C 3 + ¼ + C16 = 1
é 1 1 ù
ê 2n + 1 + 2n + 1 ú ´r (ii) Put z = w, we get
êë Cr Cr + 1 úû (1 + w2 + w4 ) 8 = C 0+ C1 w2 + C 2 w4 + C 3 w6+ ¼ + C16 w32
æ 2n + 1 ö 2n - 1 r -1
æ 1 1 ö Þ(1 + w2 + w) 8 = C 0+ C1 w2 + C 2 w + C 3 + C 4 w2
=ç ÷ å (- 1) çç 2n + 1 + 2n + 1 ÷ ´r
è 2n + 2 ø r = 1 è Cr Cr + 1 ÷ø + C 5 w + ¼ + C16 w2
æ 2n + 1 ö é æ 1 1 ö Þ 0 = (C 0 + C 3 + C 6 + ¼ + C15 )
=ç ÷ êç + 2n + 1 ÷
è 2n + 2 ø ë è 2n + 1C1 C2 ø + (C 2+ C 5 + ¼ + C14 ) w + (C1 + C 4 + ¼ + C16 ) w2

æ 1 1 ö æ 1 1 ö 109. Given, f ( x ) = g( x + 1)
-2 ç 2n + 1 + 2n + 1 ÷ + 3 ç 2n + 1 + ÷
è C2 C3 ø è C3 2n + 1
C4 ø \ a 0 x 0+ a1 x + a 2 x 2 + ¼ + a 2n x 2n
æ 1 1 ö = b0 + b1 ( x + 1 ) + b2 ( x + 1 ) 2 + ¼ + bn - 1 ( x + 1 )n - 1
- 4 ç 2n + 1 + 2n + 1 ÷
è C4 C5 ø + ( x + 1 )n + ( x + 1 )n + 1 + ( x + 1 )n + 2 + ¼ + ( x + 1 ) 2n
æ 1 1 ö Equating coefficient of xn on both sides, we get
+ ¼ + (2n - 1 ) çç 2n + 1 + ÷
è C 2n - 1 2n + 1
C 2n ÷ø an = nCn + n + 1Cn + ¼ + 2nCn
516 Textbook of Algebra

= n + 1Cn + 1 + n+1
Cn + ¼ + 2n
Cn [Q nCn = n + 1Cn + 1 ] Þ a 2 + a 5 + a 8 + ¼= 3n - 1
= n + 2Cn + 1 + ¼ + 2n
Cn = 2nCn + 1 + 2n
Cn Hence, a 0+ a 3 + a 6+ ¼ = a1 + a 4 + a 7 + ¼
n n
[Q Cr + Cr - 1 = n+1
Cr ] = a 2 + a 5 + a 8 + ... = 3n - 1

= 2n + 1
Cn + 1 111. LHS = (n - 1) 2 C1 + (n - 3) 2 C 3 + (n -5) 2 C 5 + ¼

110. Let (1 + x + x 2 )n = a 0 + a1 x + a 2 x 2 +¼+ a 2n x 2n = n 2 (C1 + C 3 + C 5 + ¼) - 2n (C1 + 3 C 3 + 5 C 5 + ¼)


...(i) + (1 2C1 + 3 2C 3 + 5 2C 5 + ¼)
æ 1ö = n (2 2 n -1
) ( - 2n )
Replacing x by ç - ÷ , we get
è xø
æn n -1 n n -1 n ö
n ç × C0 + × 3 × C 2+ × 5 ×n - 1 C 4 + ¼÷
æ 1 1 ö a1 a 2 a 2n è1 3 5 ø
ç1 - + 2 ÷ = a 0 - + 2 - ¼ + 2n
è x x ø x x x æ n n n ö
+ ç1 2 × ×n - 1 C 0 + 3 2 × ×n -1 C 2 + 5 2 × ×n -1 C 4 + ¼÷
Þ (1 - x + x 2 )n = a 0 x 2n - a1 x 2n - 1+ a 2 x 2n - 2 - ¼ + a 2n x 2n è 1 3 5 ø
…(ii) = n 2 × 2n - 1 - 2n 2(n -1C 0+ n -1
C2 + n -1
C 4 + ¼)
(i) Multiplying Eqs. (i) and (ii) and equating the coefficient of n -1
+ n (1 × C 0 + 3 ×n - 1 C 2 + 5 ×n - 1 C 4 + ¼)
x 2n + 1, then we get
Coefficient of x 2n + 1 in (1 + x 2 + x 4 )n = n 2 × 2n - 1 - 2n 2 × 2n - 2

= a 0a1 - a1a 2 + a 2a 3 - ¼ + n[n - 1C 0 + (2 + 1 ) ×n - 1 C 2 + ( 4 + 1 ) × n - 1C 4 + ¼]


In RHS, put x 2 = y , we get = n 2 × 2n - 2(2 - 2 ) + n[(n -1C 0 + n -1C 2 + n -1C 4 + ... )]
a 0a1 - a1a 2 + a 2a 3 - ¼ = 0 (only even powers contains) + (2 ×n - 1 C 2 + 4 ×n - 1 C 4 + ¼)]
(ii) Multiplying Eqs. (i) and (ii) and equating the coefficient of = 0 + n [2n - 2 + (n - 1 ) {n - 2C1 + n-2
C 3 + ¼}]
x 2n + 2 n-2 n-3 n-3
= n [2 + (n - 1 ) × 2 ] = n ×2 (2 + n - 1 )
Coefficient of x 2n + 2 in (1 + x 2+ x 4 )n
= n (n + 1 ) 2 n - 3 = RHS
= Coefficient of y n + 1 in (1 + y + y 2 )n
112. (1 - x 3 )n = C 0 - C1x 3 + C 2 x 6 - C 3 x 9 + ¼ + ( - 1)n Cn x 3n
= a 0a 2 -a1a 3 + a 2a 4 - ¼ + a 2n - 2 × a 2n
1
[Q put x 2 = y ] Then, ò (1 - x 3 )n dx
0
= an + 1 1
(iii) Put x = 1, w and w2 in Eq. (i), we get = ò (C 0 - C1 x 3+ C 2 x 6 - ¼ + ( - 1 )n Cn ( x 3n ) dx
0
n 1
3 = a 0 + a1+ a 2+ a 3 + ¼ + a 2n …(iii) é C x C x 4 C2 x 7 C x 3n + 1 ù
=ê 0 - 1 + - ¼ + ( - 1 )n n ú
Þ(1 + w + w2 )n = a 0 + a1 w + a 2 w2 + a 3 w3+ ¼ + a 2n w2n ë 1 4 7 3n + 1 û 0

Þ 0 = a 0 + a1 w + a 2 w2 + a 3+ ¼ …(iv) C C C Cn
= 0 - 1 + 2 - ¼+ ( - 1 )n
1 4 7 3n + 1
and (1 + w2 + w4 )n = a 0 + a1 w2 + a 2 w4 + a 3 w6+ ¼ 1
Let I n = ò (1 - x 3 )n × 1 dx
2
Þ 0 = a 0 + a1 w + a 2 w + a 3 + ¼ …(v) 0
1
= [(1 - x 3 )n × x ]10 - ò n (1 - x 3)n - 1 × ( - 3 x 2) × x dx
on adding Eqs. (iii), (iv) and (v), we get 0
1 3 n -1
3n = 3a 0 + a1 (1 + w + w2 ) = 0 - 3n ò0 (1 - x ) (1 - x 3 - 1 ) dx
+ a 2 (1 + w2 + w) + 3a 3 + ¼ +
= - 3n ( I n - I n -1 )
Þ 3n = 3 (a 0 + a 3 + a 6+ ¼) Þ a 0 + a 3 + a 6 + ¼ = 3n - 1 3n 3 (n - 1 )
Þ In = I n - 1; I n - 1 = In - 2
(3n + 1 ) (3n -2 )
On multiplying Eqs. (iv) and (v) by w2 and w, respectively
and then adding Eqs. (iii), (iv) and (v), we get 3 (n - 2 )
In - 2 = In - 3
3n = a 0 (1 + w2 + w) + a1 (1 + w3 + w3 ) (3n - 5 )
M M M
+ a 2 (1 + w4 + w3 ) + a 3 (1 + w + w2 ) + ¼
3 ×3 3 ×2 3 ×1 3 ×1
= 3 (a1 + a 4 + ¼) I3 = I 2; I 2 = I 1; I 1 = I0 = (1 )
10 7 4 3
Þa1 + a 4 + ¼ = 3n - 1
(3 × 1 ) (3 × 2 ) (3 × 3 ) ¼ (3 × n ) 3n × n !
\ In = ×1 =
Again, multiplying Eq. (iv) by w and Eq. (v) by w2, respectively 1 × 4 × 7 × 10 ¼ (3n + 1 ) 1 × 4 × 7 × 10 ¼ (3n + 1 )
and then adding Eqs. (iii), (iv) and (v), we get
C 0 C1 C 2 Cn 3n × n !
3n = 3 (a 2+ a 5 + a 8+ ¼) Hence, - + - ¼ + ( - 1 )n =
1 4 7 3n + 1 1 × 4 × 7 ¼ (3n + 1 )
Chp 06 Binomial Theorem 517

113. (1 - x ) 30 = 30C 0 x 0 - 30C1 x1 + 30C 2 x 2 + ¼ …(i) n


C 5 (a )n - 5 ( - b ) 5 æn - 5 + 1ö a
Þ = -1 Þ ç ÷=
(x + 1) 30 30
= C0 x 30
+ 30
C1 x 29 30
+ C2 x 28 n
C 4 (a )n - 4 ( - b ) 4 è 5 ø b
+ ¼ + 30C10 x 20 + ¼ + 30
C 30 x 0 …(ii) a n-4
\ =
On multiplying Eqs. (i) and (ii) and equating the coefficient of b 5
x 20 on both sides, we get 118. 20
C 0 - 20C1 + 20
C 2 - 20C 3 + ¼
required sum = coefficient of x 20 in (1 - x 2 ) 30 = 30C10 20
C9 + 20
C10 - 20C11 + 20
C12 - ¼ + 20
C 20 = 0
114. Coefficients of p th, ( p + 1) th and ( p + 2) th terms in expansion Þ 2 { C 0 - C1 + C 2 - C 3 + ...- C 9 } + 20C10 = 0
20 20 20 20 20
n n n n
of (1 + x ) are C p - 1, C p , C p + 1.
Þ 2 { 20C 0 - 20C1 + 20
C 2 - 20C 3 + ¼ + 20
C9 + 20
C10 } = 20C10
n n n
Then, 2 × C p = C p - 1 + C p + 1 1
n n
Þ 20
C 0 - 20C1 + 20
C 2 - 20C 3 + ¼ - 20C 9 + 20
C10 = 20
C10
Cp - 1 Cp + 1 2
2= n
+ n
n n n
Cp Cp 119. å (r + 1) nCr = å r ×n Cr + å nCr
r=0 r=0 r=0
p n-p n n
2= + n
= å r × ×n - 1 Cr - 1 + å nCr
n-p+1 p+1 r=0 r r=0
n n
Þ 2 (n - p + 1 ) ( p + 1 ) = p ( p + 1 ) + (n - p ) (n - p +1 ) =n å n -1
Cr - 1 + å nCr
Þ n 2 - n (4p + 1) + 4p 2 - 2 = 0 r=0 r=0

11 = n × 2n - 1 + 2n = (n + 2 ) 2n - 1
æ 1ö
115. In the expansion of çax 2 + ÷ , the general term is Statement-1 is true.
è bx ø n n n
11 2 11 - r
r
æ 1 ö 11 11 - r 1 22 - 3r and å (r + 1 ) nCr x r = å r ×n Cr × x r + å nCr × x r
Tr + 1 = Cr (ax ) ç ÷ = Cra x r=0 r=0 r=0
è bx ø br n n
n
For x 7, we must have 22 - 3r = 7 Þr = 5 and then the = å r × ×n - 1 Cr - 1 × x r + å nCr × x r
r=0 r r=0
1 a6 n n
coefficient of x 7 = 11C 5 × a11 - 5 5 = 11C 5 5 =n å n -1
Cr - 1 × x r + å nCr × x r
b b r=0 r=0
11
æ 1 ö = nx (1 + x )n - 1 + (1 + x )n
Similarly, in the expansion of çax - 2 ÷ , the general term is
è bx ø
11 - r
On substituting x = 1, then we get
11 r a 11 - 3r n
Tr + 1 = Cr ( - 1 ) ×x å (r + 1 ) nCr = n × 2n - 1 + 2n = (n + 2 ) 2n - 1
br r=0
For x - 7 we must have, 11 - 3r = - 7
Hence, Statement-2 is also true and it is a correct explanation
a 5 11 a 5 for Statement-1.
Þ r = 6 and then coefficient of x - 7 is 11C 6 = C5 6
b6 b 120. 8 2n - (62) 2n + 1 = (64)n - (62) 2n + 1
a 6 11 a 5
11 = (63 + 1 )n - (63 - 1 ) 2n + 1
As given, = C 5 6 Þab = 1
C5
b5 b
= (1 + 63 )n + (1 - 63 ) 2n + 1
116. Q (1 - y )m (1 + y )n = (1 - mC1 y + mC 2 y 2¼)
= {1 + nC1 × 63 + nC 2(63 ) 2 + ¼ + nCn (63 )n }
n
´ (1 + C1 y + C 2 y + ¼)n 2
+ {1 - 2n + 1C1 (63 ) + 2n + 1C 2 (63 ) 2 ¼ - 2n + 1C 2n + 1 (62 ) 2n + 1}
= 1 + (n - m ) y + (m C 2 + nC 2 - mn ) + ¼ = 2 + 63 {n C1 + nC 2 × 63 + ¼ + nCn (63 ) n - 1 - 2n + 1
C1
a1 = n - m = 10 2n + 1 2n + 1
Then [given]…(i) + C 2 × 63 ¼ - C 2n + 1(63 ) 2n }
m
and C 2 + nC 2 - mn = a 2 = 10 (given) \ Remainder is 2.
m (m - 1 ) n (n - 1 ) 121. Q A r = 10C r, B r = 20C r and C r = 30C r
Þ + - mn = 10
2 2 10 10
Þ m 2+ n 2 - m - n - 2mn = 20 \ å Ar ( B 10B r - C 10A r ) = å 10
Cr ( 20C10 ×20 Cr - 30C10 ×10 Cr )
r =1 r =1
2
Þ (n - m ) - (m + n ) = 20 [Q n - m = 10 ] 10 10

or 100 - (m - n = 20 = 20C10 å (10Cr ) ( 20Cr ) - 30C10 å (10Cr ) 2


r =1 r =1
\ m + n = 80 …(ii)
= 20 C10 ( 30C10 - 1 ) - 30C10 ( 20C10 - 1 ) = C10 - B10
On solving Eqs. (i) and (ii), we get
n 10
n = 45, m = 35 122. Use n Cr = × n -1Cr -1, then S1 = å C (C - 1) × 10CC
Hence, (m, n ) = (35, 45 ) r C =1

T6 10 10.9 8 10
117. Q T5 + T6 = 0 Þ = -1 = å C (C - 1 ) × × CC - 2 = 90 å 8CC - 2 = 90 ´ 2 8
T5 C =1 C (C - 1 ) C =1
518 Textbook of Algebra

10 10 10
10 9
× CC -1 = 10 å 9CC -1 = 10 ´ 2 9
S 2 = å C × 10CC = å C × and coefficient of x 4 = 18C 4 × 2 4 - 18C 3 × 2 3 × a + 18C 2 × 2 2 × b = 0
C =1 C =1 C C =1
Þ 32a - 3b = 240 …(ii)
10 10
2 10 2 10 9 On solving Eqs. (i) and (ii), we get
andS 3 = å C × CC = å C × × CC -1
C =1 C =1 C 272
10
a = 16, b =
3
= 10 å ((C - 1 ) + 1 ) × 9 CC -1
C =1 æ 272 ö
\ (a, b ) = ç16, ÷
10 è 3 ø
= 10 å (9 × 8 CC - 2 + 9 CC -1 ) = 10 (9.2 8 + 2 9 ) = 55 ´ 29
C =1 128. Q(1 + x 2) 4 (1 + x 3) 7 (1 + x 4 )12
Both statements are true but Statement-2 is not correct = (1 + 4C1 x 2 + 4C 2x 4 + 4C 3x 6 + 4C 4 x 8 )
Explanation for Statement-1.
´ (1 + 7C1 x 3+ 7 C 2 x 6+ 7 C 3x 9 + ¼) ´ (1 + 12C1x 4 + 12C 2x 8 + ¼)
123. Here, (1 - x - x 2 + x 3) 6 = (1 - x ) 6 (1 - x 2 ) 6 = (1 - x 2 ) 6(1 - x ) 6
Required coefficient
= (1 - 6C1 x 2 + 6C 2 x 4 - 6C 3 x 6 + ¼)
= 12C 2 ×7 C1 × 1 + 12C1 × 7C1 × 4C 2 + 7C1 × 4 C 4 + 7C 3 × 4C1
´ (1 - 6C1 x + 6C 2 x 2 - 6C 3 x 3
= 462 + 504 + 7 + 140 = 1113
+ 6C 4 x 4 - 6C 5 x 5 + 6C 6 x 6 )
129. Q Tr + 1 = 50Cr ( - 2 x )r = 50Cr ( - 2)r × xr /2
\ Coefficient of x 7 in (1 - x - x 2 + x 3 ) 6
= 6C1 ´ 6C 5 + ( 6C 2 ´ ( - 6C 3 ) + {( - 6C 3 ) ´ ( - 6C1 )} For integral powers of x, r = 0, 2, 4, 6, ¼, 50
= 36 - 300 + 120 = - 144 \ Required sum = 50C 0 + 2 2 ×50 C 2 + 2 4 ×50 C 4 + ¼ + 2 50 ×50 C 50
124. ( 3 + 1) 2n - ( 3 - 1) 2n = 2 { 2n C1( 3 ) 2n - 1 + 2nC 3 1 1
[(1+ 2 ) 50+ (1 - 2 ) 50 ] = (3 50 + 1 )
=
( 3 ) 2n - 3 + ¼ + 2n
C 2n - 1( 3 )} 2 2
= 2 3 { 2n C1( 3 ) 2n - 2 + C 3( 3 ) 2n - 4 + ¼ +
2n 2n
C 2n - 1 } 130. In the Expansion of (1 + x )(1 + x 2 )(1 + x 3 )...(1 + x100 ).
2n n -1 2n n-2 2n x 9 can be found in the following ways
= 2 3 { C1 (3 ) + C 3(3 ) + ... C 2n -1 }
x 9, x 1 + 8, x 2 + 7, x 3 + 6, x 4 + 5, x 1 + 2 + 6, x 1 + 3 + 5, x 2 + 3 + 4
= 3 ´ even integer
There are 8 cases
125. Q x + 1 = ( x 1/3) 3 + 1 3 = ( x 1/3 + 1) ( x 2/3 - x 1/3 + 1)
The coenfficient of x 9 in each cases is 1
x+1 \ Required coefficient = 8
\ 2/ 3
= x 1/ 3 + 1
x - x 1/ 3 + 1 131. Total number of terms = n + 2C 2 = 28
and x - 1 = ( x 1/ 2) 2 - 1 2 = ( x 1/ 2 + 1 ) ( x 1/ 2 - 1 ) Þ (n + 2 )(n + 1 ) = 56 = (6 + 2 )(6 + 1 )
x -1 (x - 1) x 1/ 2 + 1 \ n =6
Now, 1/ 2
= 1/ 2 1/ 2 = = 1 + x - 1/2,
x-x x (x - 1) x 1/ 2 Sum of coefficients = (1 - 2 + 4 )n = 3 6 = 729
10 [Note In the solution it is considered that different terms in
æ x+1 x -1 ö
then ç 2/3 - ÷ = ( x 1/3 - x - 1/2 )10 the expansion having same powers are not merged, as such it
è x - x + 1 x - x 1/ 2 ø
1/ 3
should be a bonus question]
\ Tr + 1 = 10Cr ( x 1/3)10 - r ( - x - 1/2 )r 132. Coefficient of x 2 in the expansion
10 - r r = 2C 2 + 3C 2 + 4C 2 + 5C 2 + ...+ 49C 2 + 50C 2 × m 2
For independent of x, - = 0 Þr = 4
3 2 = 3C 3 + 3C 2 + 4C 2 + 5C 2 + ...+ 49
C2 + 50
C2 × m2
10 10 × 9 × 8 × 7
\ T4 + 1 = C 4 = = 210 = 4C 3 + 4C 2 + 5C 2 + ...+ 49
C2 + 50
C2 × m2
1 ×2 ×3 × 4
126. Since, n + 5Cr : n + 5Cr - 1 = 2 Þ3 r = n + 6 …(i) = 50C 3 + 50
C 2 × m 2 (Applying again and again Pascal’s rule)

7 = ( 50C 3 + 50
C2 ) + 50
C 2(m 2 - 1 )
n+ 5
and Cr + 1 : n + 5Cr =
Þ12r = 5n + 18 …(ii)
5 = 51C 3 + 50
C 2(m 2 - 1 ) = (3n + 1 ) 51C 3 (given)
From Eqs. (i) and (ii), we get or 50
C 2(m 2 - 1 ) = 3n × 51C 3
4(n + 6 ) = 5n + 18 Þn = 6
m 2 - 1 51 m2 - 1
127. (1 + ax + bx 2 ) (1 - 2x )18 or = = 17 or =n
3n 3 51
= (1 + ax + bx 2 ) [1 - 18C1(2 x ) + 18C 2 (2 x ) 2 for m = 16, n = 5
- 18C 3(2 x ) 3 + 18C 4 (2 x ) 4 - ¼] 133. ( 21C1 + 21C 2 + 21C 3 + 21C 4 +....+ 21C10 )
According to the question, Coefficient of x 3 - (10C1 + 10C 2 + 10C 3 + ...+ 10C10 )
18 18 2 18 1
= - C 3 × 8 + a × C 2 × 2 - b × C1 × 2 = 0 = (2 21 - 2 ) - (210 - 1 ) = (2 20 - 1 ) - (210 - 1 )
544 2
Þ17 a - b = …(i)
3 = 2 20 - 210
CHAPTER

07
Determinants
Learning Part
Session 1
● Definition of Determinants

● Expansion of Determinant

● Sarrus Rule for Expansion

● Window Rule for Expansion

Session 2
● Minors and Cofactors

● Use of Determinants in Coordinate Geometry

● Properties of Determinants

Session 3
● Examples on Largest Value of a Third Order Determinant

● Multiplication of Two Determinants of the Same Order

● System of Linear Equations

● Cramer’s Rule

● Nature of Solutions of System of Linear Equations

● System of Homogeneous Linear Equations

Session 4
● Differentiation of Determinant

● Integration of a Determinant

● Walli’s Formula

● Use of S in Determinant

Practice Part
● JEE Type Examples
● Chapter Exercises

Arihant on Your Mobile !


Exercises with the #L
symbol can be practised on your mobile. See inside cover page to activate for free.
Session 1
Definition of Determinants, Expansion of Determinant,
Sarrus Rule for Expansion, Window Rule for Expansion
Determinants were invented independently by Gabriel x y
=
Cramer, whose now well-known rule for solving linear b 2c 3 - b 3c 2 c 2a 3 - c 3a 2
system was published in 1750, although not in present day
z
notation. The now-standard “Vertical line notation”, i.e. “| = =k [say]
|” was given in 1841 by Arthur Cayley. The working a 2b 3 - a 3b 2
knowledge of determinants is a basic necessity for a Þ x = k (b 2 c 3 - b 3 c 2 ), y = k (c 2 a 3 - c 3 a 2 )
student. Determinants have wide applications in and z = k (a 2 b 3 - a 3 b 2 )
Engineering, Science, Economics, Social science, etc.
On putting these values of x , y and z in Eq. (i), we get
a 1 (b 2 c 3 - b 3 c 2 ) + b 1 (c 2 a 3 - c 3 a 2 ) + c 1 (a 2 b 3 - a 3 b 2 ) = 0
Definition of Determinants or a 1 (b 2 c 3 - b 3 c 2 ) - b 1 (c 3 a 2 - c 2 a 3 )
Consider the system of two homogeneous linear equations + c 1 (a 2 b 3 - a 3 b 2 ) = 0 …(iv)
a 1 x + b 1y = 0 …(i) b2 c2 c2 a2 a2 b2
or a1 - b1 + c1 = 0 …(v)
a 2 x + b 2y = 0 …(ii) b3 c3 c3 a3 a3 b3
in the two variables x and y. From these equations, we a1 b1 c1
obtain
Usually this is written as a 2 b2 c2 = 0
a1 y a a a
- = =- 2 Þ 1 = 2 a3 b3 c3
b1 x b2 b1 b2
a1 b1 c1
Þ a 1b 2 - a 2b 1 = 0
Here, the expression a 2 b 2 c 2 consisting of three rows
a1 b1
The result a 1 b 2 - a 2 b 1 is represented by a3 b3 c3
a2 b2
and three columns, is called determinant of order three.
which is known as determinant of order two. The quantities
The quantities a 1 , b 1 , c 1 , a 2 , b 2 , c 2 , a 3 , b 3 and c 3 are called
a 1 , b 1 , a 2 and b 2 are called constituents or elements of the
constituents or elements of the determinant.
determinant and a 1 b 2 - a 2 b 1 is called its value.
The horizontal lines are called rows and vertical lines are Remark
called columns. Here, this determinant consists two rows 1. A determinant is generally denoted by D or D.
and two columns. 2. A determinant of the nth order consists of n rows and n
columns and its expansion contains n! terms.
For example, The value of the determinant
3. A determinant of nth order consists of n rows and n columns.
2 3 \ Number of constituents in determinant = n2
= 2 ´ ( - 5 ) - 3 ´ 4 = - 10 - 12 = - 22
4 -5 4. In a determinant the horizontal lines counting from top to
bottom 1st, 2nd, 3rd, … respectively, known as rows and
Now, let us consider the system of three homogeneous linear denoted by R1, R2, R3, K and vertical lines from left to right 1st,
equations 2nd, 3rd, K respectively, known as columns and denoted by
a 1 x + b 1y + c 1z = 0 …(i) C1, C2, C3, L .
5. Shape of every determinant is square.
a 2 x + b 2y + c 2 z = 0 …(ii)
6. Sign system for order 2, order 3, order 4, … are given by
a 3 x + b 3y + c 3z = 0 …(iii) + - + -
+ - +
On solving Eqs. (ii) and (iii) for x , y and z by + - - + - +
, - + - , ,L
cross-multiplication, we get - + + - + -
+ - +
- + - +
Chap 07 Determinants 521

Expansion of Determinant D2 = 4
3
2
4
-1
-9
1
2
2
-1
+6
1
3
2
4
(i) Expansion of two order = 4 ( - 3 - 8) - 9 ( - 1 - 4 ) + 6( 4 - 6)
a1 b1 a1 b1 = - 44 + 45 - 12
= - = a1b 2 - b1a 2
a2 b2 b2 a2 = 1 - 12
= - 11
5 -4 5 -4
For example, = - and expanding the determinant along second column
-3 2 2 -3
3 9 1 4 1 4
= 10 - 12 = - 2 D3 = - 2 +4 - ( - 1)
2 6 2 6 3 9
(ii) Expansion of third order = - 2 (18 - 18) + 4 (6 - 8) + 1 (9 - 12)
=0-8-3
(a) With respect to first row.
= - 11
a1 × × × b1 × × × c 1 Hence, D1 = D 2 = D 3
b c2
a2 b2 c 2 = a1 2
b3 c3 1 sin q 1
a3 b3 c3
y Example 2. If D = - sin q 1 sin q ,
a2 c2 a2 b2
-b 1 + c1 -1 - sin q 1
a3 c3 a3 b3
prove that 2 £ D £ 4.
= a 1 (b 2 c 3 - b 3 c 2 ) - b 1 (a 2 c 3 - a 3 c 2 )
1 sin q 1
+ c 1 (a 2 b 3 - a 3 b 2 ) Sol. Given, D = - sin q 1 sin q
(b) With respect to second column.
-1 - sin q 1
a1 b1 c 1
Expanding along first row, we get
M
a2 c2 1 sin q - sin q sin q
a2 b2 c 2 = - b1 D =1 - sin q
a3 c3 - sin q 1 -1 1
M
- sin q 1
a3 b3 c3 +1
a1 c1 a1 c1 -1 - sin q
+ b2 - b3
a3 c3 a2 c2 = (1 + sin 2 q ) - sin q ( - sin q + sin q ) + (sin 2 q + 1)
= - b 1 (a 2 c 3 - a 3 c 2 ) + b 2 (a 1 c 3 - a 3 c 1 ) = 2(1 + sin 2 q )
- b 3 (a 1 c 2 - a 2 c 1 ) Again,0 £ sin 2 q £ 1
Remark Þ 1 £ (1 + sin 2 q ) £ 1 + 1
A determinant can be expanded along any of its row or column.
Þ 2 £ 2 (1 + sin 2 q ) £ 4
Value of the determinant remains same in any of the cases.
\ 2£ D £ 4
y Example 1. Find the value of the determinant
1 2 4
3 4 9 Sarrus Rule for Expansion
2 -1 6 Sarrus gave a rule for a determinant of order 3.
Sol. Expanding the determinant along the first row Rule Write down the three rows of the D and rewrite the
4 9 3 9 3 4 first two rows. The three diagonals sloping down to the
D1 = 1 -2 +4 right given the three terms and the three diagonals
-1 6 2 6 2 -1
sloping down to the left also given the three terms.
= 1 (24 + 9 ) - 2 (18 - 18) + 4 ( - 3 - 8)
= 33 - 0 - 44 a1 b1 c1
= - 11 If D = a2 b2 c2
and expanding the determinant along third column a3 b3 c3
522 Textbook of Algebra

Rule a1 b1 c1
Window Rule for Expansion
a2 b2 c2
Window rule valid only for third order determinant.
a3 b3 c3 a1 b1 c 1
a3 b 2 c1 b1 c1 a1 b2 c3 Let D = a2 b2 c 2
a1
a1 b 3 c2 a2 b3 c1 a3 b3 c3
a2 b2 c2
a2 b 1 c2 a3 b1 c2 In this method, rewrite first two elements of second row
Sum = N Sum = P and third row, then
\ D=P -N
a1 b1 c 1
3 2 5
Rule a2 b2 c 2 a2 b2
y Example 3. Expand 9 -1 4 by Sarrus rule.
a3 b3 c3 a3 b3
2 3 -5
Now, taking positive sign with a 1 , b 1 and c 1 .
3 2 5
Sol. Let D = 9 -1 4 D = a 1 (b 2 c 3 - b 3 c 2 ) + b 1 (c 2 a 3 - c 3 a 2 )
+ c 1 (a 2 b 3 - a 3 b 2 )
2 3 -5
Rule 3 2 5 1 2 3
9
y Example 5. Expand 4 6 2 by window rule.
–1 4
5 9 4
2 3 –5
1 2 3
–10 3 2 5 15 Sol. Let D = 4 6 2
36 –9 4 135 5 9 4
9
–90 16 1 2 3
N=–64 P=166
Rule: 4 6 2 4 6
\ D = P - N = 166 - ( - 64 ) = 230 5 9 4 5 9
\ D = 1(24 - 18) + 2 (10 - 16) + 3 (36 - 30)
y Example 4. If a, b , c ÎR , find the number of real
= 6 - 12 + 18 = 12
x c -b
roots of the equation -c x a =0 y Example 6. Find the value of the determinant
b -a x -1 2 1
x c -b 3+ 2 2 2+ 2 2 1 .
Sol. Let D = -c x a 3-2 2 2-2 2 1
b -a x -1 2 1
Rule x c –b Sol. Let D = 3 + 2 2 2 + 2 2 1 and let 2 2 = l,
–c x a
3-2 2 2-2 2 1
-1 2 1
b –a x
then D = 3+ l 2+ l 1
–b2x
x c –b x3 3 -l 2- l 1
–a2x – abc -1 2 1
–c x a
– c2x abc Rule 3 + l 2 + l 1 3 + l 2 + l
N =– x (a2 + b2 + c2) P = x3
3- l 2- l 1 3- l 2- l
\ D=P-N Now, D = - 1(2 + l - 2 + l ) + 2(3 - l - 3 - l )
= x 3 + x (a 2 + b 2 + c 2 ) = 0 [given] + 1 [(3 + l ) (2 - l ) - (3 - l ) (2 + l )]
\ x = 0 or x = - (a 2 + b 2 + c 2 )
2
= - 2l - 4 l + ( -2 l ) = -8l = - 16 2
Þ x = 0 or x = ± i (a 2 + b 2 + c 2 ), where i = -1
[Q l = 2 2 ]
Hence, number of real roots is one.
Chap 07 Determinants 523

#L Exercise for Session 1


1 4 20
1 Sum of real roots of the equation 1 -2 5 = 0 is
1 2x 5x 2
(a) -2 (b) -1 (c) 0 (d) 1
6i -3i 1
2 If 4 3i -1 = x + iy , i = -1, then
20 3 i
(a) x = 3, y = 1 (b) x = 1, y = 3
(c) x = 0, y = 3 (d) x = 0, y = 0
l2 + 3l l - 1 l + 3
3 If pl + ql + rl + sl + t = l2 + 1 2 - l l - 3 , then t is equal to
4 3 2

l2 - 3 l + 4 3l
(a) 7 (b) 14
(c) 21 (d) 28
6 7 x 2 - 13
4 If one root of the equation 2 x - 13 22
= 0 is x = 2, the sum of all other five roots is
2
x - 13 3 7
(a) 2 15 (b) -2
(c) 20 + 15 - 2 (d) None of these
tan A 1 1
5 If A, B and C are the angles of a non-right angled DABC, the value of 1 tan B 1 is
1 1 tan C
(a) 0 (b) 1 (c) 2 (d) 3
1 3 cos q 1
6 If D = sin q 1 3 cos q , the maximum value of D is
1 sin q 1
(a) -10 (b) - 10 (c) 10 (d) 10
a 1 1
7 If the value of the determinant 1 b 1 is positive, then (a, b , c > 0)
1 1 c
(a) abc > 1 (b) abc > - 8 (c) abc < - 8 (d) abc > - 2
Session 2
Minors and Cofactors, Use of Determinants in
Coordinate Geometry, Properties of Determinants

Minors and Cofactors where


a 22 a 23 +1
a 11 a 12 a 13 × × × a 1n M 11 = , C 11 = ( -1) 1 M 11 = M 11
a 32 a 33
a 21 a 21 a 23 × × × a 2n
D = a 31 × × × a 3n a 21 a 23
Let a 32 a 33 M 12 = , C 12 = ( -1) 1 + 2 M 12 = - M 12
××× ××× ××× ××× ××× a 31 a 33
a n1 a n2 an3 ××× a 21 a 22
ann
M 13 = , C 13 = ( -1) 1 + 3 M 13 = M 13
be a determinant of order n, n ³ 2, then the determinant of a 31 a 32
order n - 1 obtained from the determinant D after deleting a 12 a 13
the ith row and jth column is called the minor of the M 21 = , C 21 = ( -1) 2 + 1 M 21 = - M 21
a 32 a 33
element a ij and it is usually denoted by M ij , where
i = 1, 2, 3, K, n and j = 1, 2, 3,K, n. a 11 a 13
M 22 = , C 22 = ( -1) 2 + 2 M 22 = M 22
If M ij is the minor of the element a ij in the determinant D, a 31 a 33
then ( -1) i + j M ij is called the cofactor of the element a 11 a 12
M 23 = , C 23 = ( -1) 2 + 3 M 23 = - M 23
a ij . It is usually denoted by C ij . a 31 a 32
Thus, C ij = ( -1) i + j M ij
a 12 a 13
M 31 = , C 31 = ( -1) 3 + 1 M 31 = M 31
ì M ij , if i + j is an even integer a 22 a 23

î-M ij , if i + j is an odd integer
a 11 a 13
M 32 = , C 32 = ( -1) 3 + 2 M 32 = - M 32
a 11 a 12 a 21 a 23
(i) Let D = , then
a 21 a 22
a 11 a 12
M 33 = , C 33 = ( -1) 3 + 3 M 33 = M 33
M 11 = |a 22 | = a 22 , M 12 = |a 21 | = a 21 , a 21 a 22
M 21 = |a 12 | = a 12 , M 22 = |a 11 | = a 11 and
C 11 = ( -1) 1 + 1 M 11 = a 22 , Important Results for Cofactors
1. The sum of products of the elements of any row or column
C 12 = ( -1) 1 + 2 M 12 = - a 21 , with their corresponding cofactors is equal to the value of the
determinant.
C 21 = ( -1) 2 + 1 M 21 = - a 12 i.e., D = a11 C11 + a12 C12 + a13 C13 = a11C11 + a21C21 + a31C31
and C 22 = ( -1) 2 + 2 M 22 = a 11 = a21C21 + a22C22 + a23C23 = a12C12 + a22C22 + a32C32
= a31C31 + a32C32 + a33C33 = a13C13 + a23C23 + a33C33
a 11 a 12 a 13
Now, value of n order determinant
(ii) Let D = a 21 a 22 a 23 a11 a12 a13 × × × × × × a1n
a 31 a 32 a 33 a21 a22 a23 × × × × × × a2n
a a a ××× ××× a3n
Determinants of minors and cofactors are D = 31 32 33
××× ××× ××× ××× ××× ×××
M 11 M 12 M 13 C 11 C 12 C 13 ××× ××× ××× ××× ××× ×××
M c
D = M 21 M 22 M 23 , D = C 21 C 22 C 23 an1 an2 an3 × × × × × × ann

M 31 M 32 M 33 C 31 a 32 a 33 = a11C11 + a12C12 + a13C13 + × × × + a1n C1n


(when expanded along first row)
Chap 07 Determinants 525

2. The sum of the product of element of any row (or column) 2 4


with corresponding cofactors of another row (or column) is M 32 = [delete 3rd row and 2nd column]
7 -5
equal to zero.
i.e., a11C21 + a12C22 + a13C23 = 0, = -10 - 28 = - 38
a11C13 + a21C23 + a31C33 = 0 , etc. \ C 32 = ( -1)3 + 2 M 32 = - M 32 = 38
3. If the value of a n order determinant is D, then the value of the
determinant formed by the cofactors of corresponding 2 3
and M 33 = = 4 - 21 = - 17
elements of the given determinant is given by 7 2
Dc = Dn - 1 [delete 3rd row and 3rd column]
i.e., in case of second order determinant \ C 33 = ( -1)3 + 3 M 33 = M 33 = - 17
Dc = D Hence, determinants of minors and cofactors are
and third order determinant Dc = D2.
1 61 -23 1 -61 -23
13 -26 -26 and -13 -26 26 , respectively.
y Example 7. Find the determinants of minors and
-23 -38 -17 -23 38 -17
2 3 4
cofactors of the determinant 7 2 -5 .
Goyal’s Method for Cofactors
8 -1 3 (Direct Method)
2 -5 This method applied only for third order determinant.
Sol. Here, M11 = =6-5=1
-1 3
[delete 1st row and 1st column] a1 a2 a 3
\ C 11 = ( -1)1 + 1 M11 = M11 = 1 Method If D = b1 b2 b 3
7 -5 c1 c2 c3
M12 = = 21 + 40 = 61
8 3 Step I Write down the three rows of the D and rewrite first
[delete 1st row and 2nd column] two rows.
\ C 12 = ( -1)1 + 2 M12 = - M12 = - 61 a1 a2 a 3
7 2 b1 b2 b3
M13 = = -7 - 16 = -23
8 -1 i.e. c1 c2 c3
[delete 1st row and 3rd column] a1 a2 a3
1+ 3
\ C 13 = ( -1) M13 = M13 = - 23 b1 b2 b3
3 4 Step II Alter Step I, rewrite first two columns.
M 21 = [delete 2nd row and 1st column]
-1 3
a1 a2 a 3 a1 a2
= 9 + 4 = 13 b1 b2 b3 b1 b2
\ C 21 = ( -1)2 + 1 M 21 = - M 21 = - 13 i.e., c1 c2 c3 c1 c2
2 4 a1 a2 a3 a1 a2
M 22 = [delete 2nd row and 2nd column]
8 3
b1 b2 b3 b1 b2
= 6 - 32 = - 26
Step III After step II, deleting first row and first column,
\ C 22 = ( -1)2 + 2 M 22 = M 22 = - 26 then we get all cofactors i.e.
2 3 b2 b 3 b1 b2
M 23 = [delete 2nd row and 3rd column]
8 -1 c2 c3 c1 c2
= - 2 - 24 = - 26 a2 a3 a1 a2
\ C 23 = ( -1)2 + 3 M 23 = - M 23 = 26 b2 b3 b1 b2
3 4
M 31 = [delete 3rd row and 1st column] b 2 c 3 - b 3c 2 b 3c 1 - b 1c 3 b 1c 2 - b 2 c 1
2 -5 c
or D = c 2 a 3 - c 3 a 2 c 3a 1 - c 1a 3 c 1a 2 - c 2 a 1
= - 15 - 8 = - 23
a 2 b 3 - a 3b 2 a 3b 1 - a 1b 3 a 1b 2 - a 2 b 1
\ C 31 = ( -1)3 + 1 M 31 = M 31 = - 23
526 Textbook of Algebra

y Example 8. Find the determinant of cofactors of the x1 y1 1


1 2 3 x2 y2 1 =0
determinant -4 3 6 by Direct Method. x3 y3 1
2 -7 9 (iii) If a r x + b r y + c r = 0; r = 1, 2, 3 are the sides of a
1 2 3 triangle, then the area of the triangle is given by
2
Sol. Let D = -4 3 6 a1 b1 c1
2 -7 9 1
D= a2 b2 c2
|2 C 1 C 2 C 3 |
Step I Write down the three rows of the D and rewrite first a3 b3 c3
two rows.
1 2 3 where C 1 , C 2 and C 3 are the cofactors of the elements
c 1 , c 2 and c 3 respectively, in the determinant
-4 3 6
i.e., 2 -7 9
a1 b1 c 1
1 2 3 a2 b2 c2
-4 3 6 a3 b3 c3
Step II After step I, rewrite first two columns (iv) Equation of straight line passing through two points
1 2 3 1 2 ( x 1 , y 1 ) and ( x 2 , y 2 ) is
-4 3 6 -4 3 x y 1
i.e., 2 -7 9 2 -7 x1 y1 1 = 0
1 2 3 1 2 x2 y2 1
-4 3 6 -4 3
(v) If three lines a r x + b r y + c r = 0; r = 1, 2, 3 are
Step III After Step II, deleting first row and first column, concurrent, then
then we get all cofactors i.e., a1 b1 c 1
3 6 -4 3 a2 b2 c2 = 0
69 48 22
-7 9 2 -7 a3 b3 c3
or Dc = -39 3 11
2 3 1 2
3 -18 11 (vi) If ax 2 + 2hxy + by 2 + 2 gx + 2 fy + c = 0 represents a
3 6 -4 3
pair of straight lines, then
y Example 9. If the value of a third order determinant a h g
is 11, find the value of the square of the determinant h b f =0
formed by the cofactors.
g f c
Sol. Here, n = 3 and D = 11
\ ( Dc )2 = ( D2 )2 = D4 = 114 = 14641 (vii) Equation of circle through three non-collinear points
( x 1 , y 1 ), ( x 2 , y 2 ) and ( x 3 , y 3 ) is given by
x2 +y2 x y 1
Use of Determinants in x 12 + y 12 x1 y1 1
=0
Coordinate Geometry x 22 + y 22 x2 y2 1
x 23 + y 23 x3 y3 1
(i) Area of triangle whose vertices are ( x 1 , y 1 ), ( x 2 , y 2 )
and ( x 3 , y 3 ) is given by
Some Useful Operations
x1 y1 1 (i) The interchange of ith row and jth row is denoted by
1
D = x2 y2 1 R i « R j . (In case of column C i « C j )
2
x3 y3 1 (ii) The addition of m times the elements of jth row to the
corresponding elements of ith row is denoted by
(ii) If points ( x 1 , y 1 ), ( x 2 , y 2 ) and ( x 3 , y 3 ) and collinear, R i ® R i + mR j .
then (In case of column C i ® C i + mC j )
Chap 07 Determinants 527

(iii) The addition of m times the elements of jth row and n a1 b1 c1 a2 b2 c2


times the elements of k th row to the corresponding Hence, a2 b2 c 2 = - a1 b1 c1
elements of ith row is denoted by
a3 b3 c3 a3 b3 c3
R i ® R i + mR j + nR k .
(In case of column C i ® C i + mC j + nC k ) Remark
If any row (or column) of a determinant D be passed over m rows
(or columns), then the resulting determinant = ( -1) m D.
Properties of Determinants
We shall establish certain properties of a determinant of Property III If two rows (or columns) of a
the third order but reader should note that these are determinant are identical, then the value of the
capable of application to a determinant of any order. determinant is zero.
Property I The value of a determinant remains a1 b1 c1 a1 b1 c1
unaltered when rows are changed into corresponding Proof Let D = a 2 b2 c 2 = - a2 b2 c2 = - D
columns and columns are changed into a1 b1 c1 a1 b1 c1
corresponding rows.
a1 b1 c 1 [by R 1 « R 3 ]
Proof Let D = a 2 b 2 c 2 Þ 2D = 0
a3 b3 c3 \ D =0
Expanding the determinant along the first row, then Property IV If the elements of any row (or any
D = a 1 (b 2 c 3 - b 3 c 2 ) - b 1 (a 2 c 3 - a 3 c 2 ) +c 1 (a 2 b 3 - a 3 b 2 ) column) of a determinant be each multiplied by the
same factor k, then the value of the determinant is
= a 1 (b 2 c 3 - b 3 c 2 ) - a 2 (b 1 c 3 - b 3 c 1 ) + a 3 (b 1 c 2 - b 2 c 1 ) multiplied by k.
a1 a2 a3 a1 b1 c 1
= b1 b2 b3 Proof Let D = a 2 b 2 c 2 = a 1 C 11 + b 1 C 12 + c 1 C 13
c1 c2 c3 a3 b3 c3
ka 1 kb 1 kc 1
= D ¢, where D ¢ be the value of the determinant when
rows of determinant D are changed into corresponding Then, a 2 b2 c 2 = ka 1 C 11 + kb 1 C 12 + kc 1 C 13
columns. a3 b3 c3
Property II If any two rows (or two columns) of a (where C 11 , C 12 and C 13 are the cofactors of a1 , b1 and c 1 in D)
determinant are interchanged, then the sign of
determinant is changed and the numerical value
remains unaltered. = k (a 1 C 11 + b 1 C 12 + c 1 C 13 ) = k D
a1 b1 c 1 Property V If every element of some column (or
row) is the sum of two items, then the determinant is
Proof Let D = a 2 b 2 c 2 equal to the sum of two determinants; one
a3 b3 c3 containing one the first term in place of each sum,
Expanding the determinant along the first row, then the other only the second term. The remaining
D = a 1 (b 2 c 3 - b 3 c 2 ) - b 1 (a 2 c 3 - a 3 c 2 ) elements of both determinants are the same as in the
given determinant i.e.,
+ c 1 (a 2 b 3 - a 3 b 2 )
= - a 2 (b 1 c 3 - b 3 c 1 ) + b 2 (a 1 c 3 - a 3 c 1 ) a1 + x b1 c 1 a1 b1 c 1 x b1 c 1
- c 2 (a 1 b 3 - a 3 b 1 ) a2 + y b2 c 2 = a2 b2 c 2 + y b2 c 2
= - [a 2 (b 1 c 3 - b 3 c 1 ) - b 2 (a 1 c 3 - a 3 c 1 ) a3 + z b3 c3 a3 b3 c3 z b3 c3
+ c 2 (a 1 b 3 - a 3 b 1 )]
a1 + x b1 c1
a2 b2 c2
Proof Let D = a2 + y b2 c2
= - a1 b1 c1 [by R 1 « R 2 ]
a3 +z b3 c3
a3 b3 c3
Expanding the determinant along first column, then
528 Textbook of Algebra

b2 c2 b c1 Property VII If each element on one side or other


D = (a 1 + x ) - (a 2 + y ) 1
b3 c3 b3 c3 side or both side of the principal diagonal of
determinant is zero, then the value of the
b1 c1
+ (a 3 + z ) determinant is the product of the diagonal element.
b2 c2
a 0 0 a i h a 0 0
b2 c2 b1 c1 b1 c1 i.e., f b 0 = 0 b g = 0 b 0 = abc
= a1 - a2 +a3
b3 c3 b3 c3 b2 c2 e d c 0 0 c 0 0 c
b2 c2 b1 c1 b1 c1
+x -y +z a L 0L 0
b3 c3 b3 c3 b2 c2
Proof Let D = f b 0
a1 b1 c1 x b1 c1 e d c
= a2 b2 c2 + y b2 c2
b 0
a3 b3 c3 z b3 c3 Expanding along R 1 , we get a = a (bc ) = abc
d c
Remark Property VIII If determinant D becomes zero on
a1 + b1 + c1 d1 + e1 f1 a1 d1 f1 a1 e1 f1
putting x = a , then we say that ( x - a ) is a factor of D.
1. a2 + b2 + c2 d2 + e2 f2 = a2 d2 f2 + a2 e2 f2
a3 + b3 + c3 d3 + e3 f3 a3 d3 f3 a3 e3 f3 x 5 2
2
b1 d1 f1 b1 e1 f1 c1 d1 f1 c1 e1 f1 i.e., if D= x 9 4
+ b2 d2 f2 + b2 e2 f2 + c2 d2 f2 + c2 e2 f2 x3 16 8
b3 d3 f3 b3 e3 f3 c3 d3 f3 c3 e3 f3
at x = 2, D = 0 [because C 1 and C 3 are identical at
2. If each element of first row of a determinant consists of
algebraic sum of p elements, second row consists of
x = 2]
algebraic sum of q elements, third row consists of algebraic and at x = 0, D = 0 [because all elements of C 1 are zero]
sum of r elements and so on.
Hence, ( x - 0 ) and ( x - 2 ) are the factors of D.
Then, given determinant is equivalent to the sum of
p ´ q ´ r ´ × × × other determinants in each of which the
elements consists of single term.
Remark
1. It should be noted that while applying operations on
Property VI The value of the determinant does not determinant that atleast one row (or column) must remain
change, if the elements of any row (or column) are unchanged.
increased or diminished by equimultiples of the 2. Maximum number of operations at a time = order - 1
corresponding elements of any other row (or 3. It should be noted that, if the row (or column) which is
column) of the determinant. changed by multiplied a non-zero number, then the
determinant will be divided by that number.
a 1 + mb 1 + nc 1 b 1 c 1 a1 b1 c 1
i.e., a 2 + mb 2 + nc 2 b 2 c 2 = a 2 b 2 c 2 Examples on Properties
a 3 + mb 3 + nc 3 b3 c3 a3 b3 c3 13 16 19
a 1 + mb 1 + nc 1 b1 c1 a1 b1 c1 y Example 10. Evaluate 14 17 20 .
Proof a 2 + mb 2 + nc 2 b2 c 2 = a2 b2 c2 15 18 21
a 3 + mb 3 + nc 3 b3 c3 a3 b3 c3
13 16 19
b1 b1 c1 c1 b1 c1 Sol. Let D = 14 17 20
+ m b2 b2 c2 + n c2 b2 c2 15 18 21
b3 b3 c3 c3 b3 c3 Applying R 2 ® R 2 - R1 and R 3 ® R 3 - R1, then
a1 b1 c1 a1 b1 c1 13 16 19 13 16 19
= a2 b2 c 2 + m × 0 + n × 0 = a2 b2 c2 D= 1 1 1 =2 1 1 1 =0
a3 b3 c3 a3 b3 c3 2 2 2 1 1 1
[QR 2 and R 3 are identical]
Chap 07 Determinants 529

a b g b m f y Example 13. Without expanding as far as possible,


y Example 11. Prove that q f y = a l q. prove that
1 1 1
l m n g n y
x y z = ( x - y ) ( y - z ) (z - x ) ( x + y + z ).
a b g a q l 3 3
x y z3
Sol. LHS = q f y = b f m
1 1 1
l m n g y n
Sol. Let D= x y z
[interchanging rows and columns]
a l q x 3 y3 z3
= ( - 1) b m f [C 2 « C 3 ] for x = y , D = 0 [QC 1 and C 2 are identical]
g n y Hence, ( x - y ) is a factor of D . Similarly, (y - z ) and
b m f (z - x ) are factors of D . But D is a homogeneous expression
of the 4 th degree in x , y and z.
= ( - 1) 2 a l q [R1 « R 2 ]
There must be one more factor of the 1st degree in x ,y and
g n y z say k ( x + y + z ), where k is a constant.
b m f Let D = k ( x - y ) (y - z ) (z - x ) ( x + y + z )
= a l q = RHS On putting x = 0, y = 1 and z = 2, then
g n y 1 1 1
0 1 2 = k ( 0 - 1) ( 1 - 2) ( 2 - 0) ( 0 + 1 + 2)
y Example 12. Use the properties of determinant and 0 1 8
without expanding, prove that
b +c q+r y +z a p x Þ 1 ×(8 - 2) = k ( -1) ( -1) (2) (3) \ k = 1
Þ D = ( x - y ) (y - z ) (z - x ) ( x + y + z ) = RHS
c+a r +p z + x =2b q y.
a+b p + q x + y c r z y Example 14. Solve for x,
4x 6x + 2 8 x + 1
b+c q +r y +z
Sol. Let LHS = D = c + a r + p z + x 6x + 2 9x + 3 12x = 0.
a+b p +q x+y 8 x + 1 12x 16 x + 2
Applying R1 ® R1 + R 2 + R 3 , then 3
Sol. Applying C 2 ® C 2 - C 1 and C 3 ® C 3 - 2C 1
2( a + b + c ) 2( p + q + r ) 2( x + y + z ) 2
4x 2 1
D= c +a r +p z+x
Then, 6x + 2 0 -4 = 0
a+b p +q x +y
8x + 1 - ( 3 / 2) 0
a+b+c p +q +r x +y +z 3ö
æ
=2 c +a r +p z+x ÷ + ( 8x + 1) ( - 8 - 0) = 0
Þ 4 x ( 0 - 6) - ( 6x + 2) ç0 +

è
a+b p +q x +y 11
Þ - 97 x - 11 = 0 Þ x = -
Applying R 2 ® R 2 - R1 and R 3 ® R 3 - R1, then 97
a+b+c p +q +r x +y +z
y Example 15. Prove that
D =2 -b -q -y
a 2 + 1 ab ac
-c -r -z
ab b + 1 bc = 1 + a 2 + b 2 + c 2 .
2
Applying R1 ® R1 + R 2 + R 3 , then
ac bc c2 +1
a p x a p x
D = 2 -b -q -y = 2 ( - 1 ) ( - 1 ) b q y a2 + 1 ab ac
-c -r -z c r z Sol. Let LHS = D = ab 2
b +1 bc
a p x ac bc c2 + 1
=2b q y = RHS On taking common a, b and c from R1 , R 2 and R 3
c r z respectively, then
530 Textbook of Algebra

a2 + 1 a a2 1 1 a a2
b c
a = b b 2 1 + abc 1 b b 2
b2 + 1 c c2 1 1 c c2
D = abc a c
b
2
c +1 1 a2 a 1 a a2
a b
c = ( -1) 1 b b + abc 1 b b 2
2
[by C 1 « C 3 ]
Now, multiplying in C 1, C 2 and C 3 by a, b and c respectively, 1 c2 c 1 c c2
then
1 a a2 1 a a2
a2 + 1 b2 c2
= ( -1)2 1 b b 2 + abc 1 b b 2 [by C 2 « C 3 ]
D= a2 b2 + 1 c2
1 c c2 1 c c2
a2 b2 c2 +1
Applying C 1 ® C 1 + C 2 + C 3 , then 1 a a2
1 + a2 + b2 + c 2 b2 c2 = 1 b b 2 (1 + abc )
D = 1 + a2 + b2 + c 2 b2 + 1 c2 1 c c2
2 2 2 2 2
1+a +b +c b c +1 Applying R 2 ® R 2 - R1 and R 3 ® R 3 - R1, then

1 b 2
c 2 1 a a2
= (1 + a 2 + b 2 + c 2 ) 1 b 2 + 1 c2 D = 0 b - a b - a 2 (1 + abc )
2

1 b2 c2 + 1 0 c - a c 2 - a2

Applying R 2 ® R 2 - R1 and R 3 ® R 3 - R1, then 1 a a2


1 b2 c2 = (b - a ) (c - a ) (1 + abc ) 0 1 b + a
O 0 1 c +a
D = (1 + a 2 + b 2 + c 2 ) 0 1 0 Applying R 3 ® R 3 - R 2 , then
O 1 a a2
0 0 1 O
2 2 2 2
= (1 + a + b + c ) × 1 × 1 × 1 = (1 + a + b + c ) = RHS 2 2 = (b - a ) (c - a ) (1 + abc ) 0 1 b+a
O
y Example 16. If a, b and c are all different and if 0 0 c -b
a a2 1+ a3 = (b - a ) (c - b ) (c - a ) (1 + abc )
b b2 1 + b 3 = 0, prove that abc = - 1. = (a - b ) (b - c ) (c - a ) (1 + abc )
c c2 1+ c 3 But given that, D =0
\ (a - b ) (b - c ) (c - a ) (1 + abc ) = 0
a a2 1 + a3 a a2 1 a a2 a3
Þ 1 + abc = 0
Sol. Let D = b b 2 1 + b 3 = b b 2 1 + b b 2 b 3
[since a, b and c are different, so a ¹ b, b ¹ c , c ¹ a ]
c c2 1 + c3 c c2 1 c c2 c3
Hence, abc = - 1
Chap 07 Determinants 531

#L Exercise for Session 2


0 -2 1
1 If l and m are the cofactors of 3 and -2 respectively, in the determinant 3 -1 2 , the value of l + m is
4 5 6
(a) 5 (b) 7 (c) 9 (d) 11
a b c
2 If a, b and c are distinct and D = b c a , then the square of the determinant of its cofactors is divisible by
c a b
(a) (a 2 + b 2 + c 2 )2 (b) (ab + bc + ca )2 (c) (a + b + c )2 (d) (a + b + c )4

3 An equilateral triangle has each of its sides of length 4 cm. If ( x r , y r ) (r = 1, 2, 3) are its vertices, the value of
2
x1 y1 1
x2 y2 1 is
x3 y3 1
(a) 192 (b) 768 (c) 1024 (d) 128
4 If the lines ax + y + 1 = 0, x + by + 1 = 0 and x + y + c = 0 (a, b and c being distinct and different from 1) are
a b c
concurrent, the value of + + is
a -1 b -1 c -1
(a) 0 (b) 1 (c) 2 (d) 3
pa qb rc
5 If p + q + r = 0 = a + b + c, the value of the determinant qc ra pb is
rb pc qa
(a) 0 (b) pa + qb + rc (c) 1 (d) None of the above
a 2 + 2n + 1 + 2p b 2 + 2n + 2 + 3q c2 + p
6 If p, q and r are in AP, the value of determinant 2n + p 2n + 1 + q 2q is
2
a +2 +p n
b + 2n + 1 + 2q
2
c2 - r
(a) 1 (b) 0 (c) a 2b 2c 2 - 2n (d) (a 2 + b 2 + c 2 ) - 2n q

7 Let {D1, D2, D3, ..., Dn} be the set of third order determinants that can be made with the distinct non-zero real
numbers a1, a 2,..., a 9. Then,
n n
(a) å Di =1 (b) å Di =0 (c) Di = D j , " i , j (d) None of these
i =1 i =1

x 3 6 2 x 7 4 5 x
8 If 3 6 x = x 7 2 = 5 x 4 = 0, then x is equal to
6 x 3 7 2 x x 4 5
(a) 0 (b) -9 (c) 3 (d) None of these
a-x c b
9 If a + b + c = 0, the one root of c b -x a = 0 is
b a c-x
(a) 1 (b) 2 (c) a 2 + b 2 + c 2 (d) 0
2 2 2
1+ a x (1 + b ) x (1 + c ) x
10 If a + b + c = - 2 and f ( x ) = (1 + a 2 )x
2 2 2
1 + b 2x (1 + c 2 ) x , the f ( x ) is a polynomial of degree
(1 + a 2 ) x (1 + b 2 ) x 1 + c 2x
(a) 0 (b) 1 (c) 2 (d) 3
a2 d 2 x
11 If a, b , c, d , e and f are in GP, the value of b 2 e 2 y , is
c2 f 2 z
(a) depends on x and y (b) depends on x and z (c) depends on y and z (d) independent of x, y and z
Session 3
Examples on Largest Value of a Third Order Determinant,
Multiplication of Two Determinants of the Same Order, System
of Linear Equations, Cramer’s Rule, Nature of Solutions of System
of Linear Equations, System of Homogeneous Linear Equations

Examples on Largest Value cannot equal 6. The following determinant satisfies the

of a Third Order Determinant given conditions and equals the largest value
-1 1 1
y Example 17. Find the largest value of a third order 1 -1 1 = - 1( 1 - 1) - 1( - 1 - 1) + 1( 1 + 1) = 4
determinant whose elements are 0 or 1. 1 1 -1
a1 b1 c 1
y Example 19. Show that the value of a third order
Sol. Let D = a 2 b 2 c 2
determinant whose all elements are 1 or -1 is an even
a3 b3 c 3
number.
D = a1 (b 2 c 3 - b 3c 2 ) - b1 (a 2 c 3 - a 3c 2 ) + c 1 (a 2 b 3 - a 3b 2 ) a1 b1 c 1
Sol. Let D = a 2 b 2 c 2
= (a1 b 2 c 3 + a 2 b 3c 1 + a 3b1c 2 ) - (b1c 3a 2 + b 2 c 1a 3 + b 3c 2 a1 )
a3 b3 c 3
Since, each element of D is either 0 or 1, therefore the value a2 a
of the D cannot exceed 3. But to attain this value, each Applying R 2 ® R 2 - R1 and R 3 ® R 3 - 3 R1, then
expression with a positive sign must equal 1, while those a1 a1
with a negative sign must be 0. However, if a1 L b1 L c1
a1 b 2 c 3 = a 2 b 3 c 1 = a 3 b1 c 2 = 1, every element of the M
determinant must be 1, making its value zero. Thus, noting a2 a2
that 0 b2 - b1 c2 - c1
D= a1 a1
0 1 1 M
1 0 1 =2 a3 a3
0 b3 - b1 c3 - c1
1 1 0 a1 a1
The largest value of D is 2. Expanding along C 1, we get
ìæ a öæ a ö æ a öæ a öü
y Example 18. Find the largest value of a third order D = a1í çb 2 - 2 b1 ÷ çc 3 - 3 c 1 ÷ - çb 3 - 3 b1 ÷ çc 2 - 2 c 1 ÷ý …(i)
determinant, whose elements are 1 or -1. îè a1 ø è a1 ø è a1 ø è a1 øþ

a1 b1 c1 Since, a1, a 2 , a 3 , b1, b 2 , b 3 , c 1, c 2 , c 3 are 1 or -1.


Sol. Let D = a 2 b 2 c 2 a2 a a a
\ b2 , b1, c 3 , 3 c 1, b 3 , 3 b1, c 2 , 2 c 1 are 1 or -1
a3 b3 c 3 a1 a1 a1 a1
\ D = a1 (b 2 c 3 - b 3c 2 ) - b1 (a 2 c 3 - a 3c 2 ) + c 1 (a 2 b 3 - a 3b 2 ) a2 a a a
Þ b2 - b 1, c 3 - 3 c 1, b 3 - 3 b 1, c 2 - 2 c 1 are 2, -2 or 0.
= (a1b 2 c 3 + a 2 b 3c 1 + a 3b1c 2 ) - (b1c 3a 2 + b 2 c 1a 3 + b 3c 2 a1 ) a1 a1 a1 a1
Since, each element of D is either 1 or -1, therefore the æ a2 ö æ a3 ö
value of the D cannot exceed 6. But it can be 6 only if \ çb 2 - b1 ÷ çc 3 - c 1 ÷
è a1 ø è a1 ø
a1b 2 c 3 = a 2 b 3c 1 = a 3b1c 2 = 1 …(i)
æ a3 ö æ a2 ö
and b1c 3a 2 = b 2 c 1a 3 = b 3c 2 a1 = - 1 …(ii) and çb 3 - b1 ÷ çc 2 - c 1 ÷ are 4, - 4
è a1 ø è a1 ø
In the first case, the product of the nine elements of the
determinant equals 1, while it is -1 in the second case, so or 0 = an even number
the two cannot occur simultaneously i.e., the determinant From Eq. (i), D = an even number (a1 = 1 or -1)
Chap 07 Determinants 533

2 -2
Multiplication of Two =
9
13 -10 -5
Determinants of the Same Order -14 -3 18
Let the two determinants of third order be Applying C 1 ® C 1 + C 3 and C 2 ® C 2 + C 3 , then
a1 b1 c 1 a 1 b1 g1 7 0 -2
D 1 = a 2 b 2 c 2 and D 2 = a 2 b2 g2 D = 8 -15 -5
a3 b3 c3 a 3 b3 g3 4 15 18
Let D be their product. Applying R 2 ® R 2 + R 3 , then
7 0 -2
Method of Multiplication (Row by Row) M
Take the first row of D 1 and the first row of D 2 i.e., D = 12 0 13
a 1 , b 1 , c 1 and a 1 , b1 , g 1 multiplying the corresponding M
elements and add. The result is a 1 a 1 + b 1 b1 + c 1 g 1 is the 4 L 15 L 18
first element of first row of D.
Expanding along C 2 , we get
Now, similar product first row of D 1 and second row of D 2
7 -2
gives a 1 a 2 + b 1 b2 + c 1 g 2 is the second element of first row - 15 = - 15 (91 + 24 ) = - 15 ´ 115 = - 1725
of D and the product of first row of D 1 and third row of D 2 12 13
gives a 1 a 3 + b 1 b 3 + c 1 g 3 is the third element of first row
of D. The second row and third row of D is obtained by y Example 21. If a x 12 + by 12
multiplying second row and third row of D 1 with 1st, 2nd, + cz 12 = ax 22 + by 22 + cz 22 = ax 32 + by 32 + cz 32 = d ,
3rd row of D 2 in the above manner.
a x 2 x 3 + by 2 y 3 + cz 2 z 3
a1 b1 c 1 a 1 b1 g 1 = a x 3 x 1 + by 3 y 1 + cz 3z 1 = ax 1 x 2 + by 1 y 2 + cz 1z 2 = f,
Hence, D = D 1 ´ D 2 = a 2 b 2 c 2 ´ a 2 b2 g 2 then prove that
a 3 b 3 c 3 a 3 b3 g 3 x1 y 1 z 1 1/ 2
ì (d + 2 f ) ü
a 1 a 1 + b 1 b1 + c 1 g 1 a 1 a 2 + b 1 b2 + c 1 g 2 x 2 y 2 z 2 = (d - f ) í ý
î abc þ
= a 2 a 1 + b 2 b1 + c 2 g 1 a 2 a 2 + b 2 b2 + c 2 g 2 x3 y 3 z3
a 3 a 1 + b 3 b1 + c 3 g 1 a 3 a 2 + b 3 b2 + c 3 g 2 x 1 y 1 z1
a 1a 3 + b 1b 3 + c 1 g 3 Sol. Let LHS = D = x 2 y 2 z 2
x 3 y3 z3
a2 a 3 + b2 b 3 + c 2 g 3
a 3a 3 + b 3b 3 + c 3 g 3 x 1 y 1 z1 x 1 y 1 z1
2
\ D = D ´ D = x 2 y2 z2 ´ x 2 y2 z2
Multiplication can also be performed row by column or
x 3 y3 z3 x 3 y3 z3
column by row or column by column as required in the
problem. x 1 y 1 z1 ax 1 by1 cz1
1
= x 2 y 2 z 2 ´ ax 2 by 2 cz 2
1 2 3 -2 1 3 abc
x 3 y3 z3 ax 3 by 3 cz 3
y Example 20. Evaluate -2 3 2 ´ 3 -2 1.
ax 12 + by12 + cz12 ax 1x 2 + by1y 2 + cz1z 2
3 4 -4 2 1 -2 1
= ax 1x 2 + by1y 2 + cz1z 2 ax 22 + by 22 + cz 22
Using the concept of multiplication of determinants. abc
ax 3 x 1 + by 3y1 + cz 3z1 ax 2 x 3 + by 2 y 3 + cz 2 z 3
1 2 3 -2 1 3
Sol. Let D = -2 3 2 ´ 3 -2 1 ax 3 x 1 + by 3y1 + cz 3z1
3 4 -4 2 1 -2 ax 2 x 3 + by 2 y 3 + cz 2 z 3 [multiplying row by row]
ax 32 + by 32 + cz 32
On multiplying row by row, we get
d f f
-2 + 2 + 9 3 - 4 + 3 2+2-6 1
= f d f [given]
D= 4 + 3 + 6 -6 - 6 + 2 -4 + 3 - 4 abc
f f d
-6 + 4 - 12 9 - 8 - 4 6+ 4 +8
534 Textbook of Algebra

Applying C 1 ® C 1 + C 2 + C 3 , then y Example 22. Show that


d + 2f f f 1 f f a 2 + x 2 ab - cx ac + bx x c -b
2
1 (d + 2 f )
= d + 2f d f = 1 d f 2 2
abc abc ab + cx b + x bc - ax = -c x a .
d + 2f f d 1 f d
ac - bx bc + ax c 2 + x 2 b -a x
Applying R 2 ® R 2 - R1 and R 3 ® R 3 - R1, then
1 f f x c -b
O Sol. Let D = -c x a
(d + 2 f ) (d + 2 f ) b -a x
= 0 d-f 0 = (d - f )2
abc abc
O Cofactors of 1st row of D are x 2 + a 2 , cx + ab, ac - bx ,
0 0 d-f cofactors of 2nd row of D are ab - cx , x 2 + b 2 , ax + bc and
1/2 cofactors of 3rd row of D are ac + bx , bc - ax , x 2 + c 2 .
ìd + 2 f ü
\ D = (d - f ) í ý = RHS Hence, the determinant of the cofactors of D is
î abc þ
a2 + x 2 ab + cx ac - bx
An Important Property c
D = ab - cx 2
b +x 2
bc + ax
If A 1 , B 1 and C 1 ,K are respectively the cofactors of the ac + bx bc - ax c2 + x2
elements a 1 , b 1 and c 1 ,K of the determinant.
Interchanging rows into columns, we get
a1 b1 c1 A1 B 1 C1 a2 + x 2 ab - cx ac + bx x c -b
2
2
D = a2 b2 c 2 , D ¹ 0 , then A 2 B2 C2 = D Dc = ab + cx b2 + x 2 bc - ax = -c x a [Q Dc = D2 ]
a3 b3 c3 A3 B3 C3 ac - bx bc + ax 2
c +x 2
b -a x
Proof Consider
a1 b1 c 1 A1 B1 C1 y Example 23. Prove the following by multiplication of
determinants and power cofactor formula
a2 b2 c 2 ´ A 2 B2 C2 2
a3 b3 c3 A B3 C3
0 c b b2 +c2 ab ac
3
c 0 a = ab c + a2
2
bc
a 1 A 1 + b 1 B 1 + c 1C 1 a 1 A 2 + b 1 B 2 + c 1C 2
b a 0 ac bc a +b2
2
= a 2 A 1 + b 2 B 1 + c 2C 1 a 2 A 2 + b 2 B 2 + c 2C 2
a 3 A 1 + b 3 B 1 + c 3C 1 a 3 A 2 + b 3 B 2 + c 3C 2 -a 2 ab ac
2
a1 A 3 + b 1 B 3 + c 1C 3 = ab -b bc = 4a 2b 2c 2
a2 A 3 + b 2 B 3 + c 2C 3 ac bc -c 2
a3A 3 + b 3B 3 + c 3C 3 0 c b
[multiplying row by row] Sol. Let D = c 0 a . Expanding along R1, then
b a 0
D 0 0 æ as a i A j + b i B j + c i C j ö
3 ç ÷ D = 0 - c (0 - ab ) + b (ac - 0) = 2abc
= 0 D 0 =D ç ìD, i = j ÷ 2

0 0 D ç = í 0, i ¹ j ÷ 0 c b
è î ø \ c 0 a = D2 = (2abc )2 = 4a 2b 2c 2 …(i)
A1 B1 C1 A1 B1 C1 b a 0
3
Þ D A2 B2 C 2 = D or A2 B2 C 2 = D2 0 c b
2
0 c b 0 c b
A3 B3 C3 A3 B3 C3 Also, c 0 a = c 0 a ´ c 0 a
[Q D ¹ 0 ] b a 0 b a 0 b a 0
c
Note Let D ¹ 0 and D denotes the determinant formed by the b2 + c 2 ab ac
cofactors of D and n is order of determinant, then = ab c +a2 2
bc …(ii)
Dc = Dn - 1
ac bc a2 + b2
This is known as power cofactor formula.
[multiplying row by row]
Chap 07 Determinants 535

-a 2 ab ac y Example 24. Prove that


and D = ab -b 2
c
bc = D3 - 1 = D2 a1 a 1 + b 1 b 1 a1 a 2 + b 1 b 2 a1 a 3 + b 1 b 3
ac bc -c 2 a 2 a 1 + b 2 b 1 a 2a 2 + b 2 b 2 a 2 a 3 + b 2 b 3 = 0.
0 c b
2 a 3a 1 + b 3b 1 a 3a 2 + b 3b 2 a 3a 3 + b 3b 3
= c 0 a …(iii) a1a 1 + b1b1 a1a 2 + b1b 2 a1a 3 + b1b 3
b a 0 Sol. LHS = a 2 a 1 + b 2 b1 a 2a 2 + b 2b 2 a 2 a 3 + b 2 b 3
a 3a 1 + b 3b1 a 3a 2 + b 3b 2 a 3a 3 + b 3b 3
From Eqs. (i), (ii) and (iii), we get
0 c b
2
b2 + c 2 ab ac a1 b1 0 a 1 b1 0
c 0 a = ab 2
c +a 2
bc = a2 b2 0 ´ a 2 b 2 0 [row by row]
b a 0 ac bc a + b2
2 a3 b3 0 a 3 b 3 0
= 0 ´ 0 = 0 = RHS
-a 2 ab ac
= ab -b 2 bc = 4a 2b 2c 2 y Example 25. Prove that
ac bc -c 2
2 a+b+ g + d
a+b+ g + d 2 (a + b ) ( g + d )
Express a Determinant Into ab + gd ab ( g + d ) + gd (a + b )
Product of Two Determinants ab + gd
a 1 a 1 + b 1 b1 a 1 a 2 + b 1 b2 ab (g + d) + gd (a + b) = 0.
Consider the determinant
a 2 a 1 + b 2 b1 a 2 a 2 + b 2 b2 2 abgd
a 1 a 1 + b 1 b1 a 1 a 2 + b 1 b2 2 a +b + g + d
Let D = Sol. LHS = a + b + g + d 2( a + b ) ( g + d )
a 2 a 1 + b 2 b1 a 2 a 2 + b 2 b2
ab + gd ab ( g + d ) + gd (a + b )
By the property of determinant, D can be written as
ab + gd
a a a 1 a 2 + b 1 b2 b b a 1 a 2 + b 1 b2
D= 1 1 + 1 1 ab(g + d) + gd(a + b)
a 2 a 1 a 2 a 2 + b 2 b2 b 2 b1 a 2 a 2 + b 2 b2
2abgd
a 1a 1 a 1a 2 a 1a 1 b 1 b2 b 1 b1 a 1a 2
= + + 1 1 0 1 1 0
a2 a 1 a2 a 2 a2 a 1 b 2 b2 b 2 b1 a2 a 2 = a +b g + d 0 ´ g + d a + b 0 [row by row]
b 1 b1 b 1 b2 ab gd 0 gd ab 0
+
b 2 b1 b 2 b2 = 0 ´ 0 = 0 = RHS
a1 a1 a b1 b a1
= a 1a 2 + a 1 b2 1 + b1 a 2 1 y Example 26. Prove that
a2 a2 a2 b2 b2 a2
cos ( A - P ) cos ( A - Q ) cos ( A - R )
b1 b1 cos (B - P ) cos (B - Q ) cos (B - R ) = 0.
+ b1 b2
b2 b2
cos (C - P ) cos (C - Q ) cos (C - R )
a1 b1 a b1
= 0 + a 1 b2 - b1 a 2 1 +0 cos( A - P ) cos( A - Q ) cos( A - R )
a2 b2 a2 b2 Sol. LHS = cos( B - P ) cos ( B - Q ) cos( B - R )
a1 b1 cos(C - P ) cos(C - Q ) cos(C - R )
= (a 1 b2 - a 2 b1 )
a2 b2 cos A sin A 0 cos P sin P 0
a1 b1 a1 b1 = cos B sin B 0 ´ cos Q sin Q 0 [row by row]
= ´
a2 b2 a2 b2 cos C sin C 0 cos R sin R 0
a 1 a 1 + b 1 b1 a 1 a 2 + b 1 b2 a b1 a1 b1 = 0 ´ 0 = 0 = RHS
\ = 1 ´
a 2 a 1 + b 2 b1 a 2 a 2 + b 2 b2 a 2 b2 a2 b2
536 Textbook of Algebra

y Example 27. If a , b and g are real numbers, 1 + 2ax + a 2 x 2 1 + 2bx + b 2 x 2 1 + 2cx + c 2 x 2


without expanding at any stage, prove that = 1 + 2ay + a 2y 2 1 + 2by + b 2y 2 1 + 2cy + c 2y 2
1 cos ( b - a ) cos ( g - a ) 1 + 2az + a 2z 2 1 + 2bz + b 2z 2 1 + 2cz + c 2z 2

cos (a - b ) 1 cos ( g - b ) = 0. [multiplying row by row]


2 2
cos (a - g ) cos (b - g ) 1 (1 + ax ) (1 + bx ) (1 + cx )2
2 2
= (1 + ay ) (1 + by ) (1 + cy )2 = RHS
1 cos(b - a ) cos( g - a )
(1 + az )2 (1 + bz )2 (1 + cz )2
Sol. LHS = cos(a - b ) 1 cos( g - b )
cos(a - g ) cos(b - g ) 1
cos(a - a ) cos( b - a ) cos( g - a )
= cos(a - b ) cos( b - b ) cos( g - b )
System of Linear Equations
(i) Consistent equations Definite and unique solution
cos(a - g ) cos( b - g ) cos( g - g )
[Intersecting lines]
cos a sin a 0 cos a sin a 0 A system of (linear) equations is said to be consistent,
= cos b sin b 0 ´ cos b sin b 0 if it has atleast one solution.
cos g sin g 0 cos g sin g 0 x + y = 2ü
= 0 ´ 0 = 0 = RHS
For example, System of equations ý is
x - y = 6þ
y Example 28. If a, b , c , x , y , z ÎR, prove that consistent because it has a solution x = 4, y = - 2.
Here, two lines intersect at one point.
(a - x ) 2 (b - x ) 2 (c - x ) 2 i.e., intersecting lines.
(a - y ) 2 (b - y ) 2 (c - y ) 2 (ii) Inconsistent equations No solution [Parallel lines]
(a - z ) 2 (b - z ) 2 (c - z ) 2 A system of (linear) equations is said to be
inconsistent, if it has no solution.
(1 + ax ) 2 (1 + bx ) 2 (1 + cx ) 2
Let a 1 x + b 1 y + c 1 = 0 and a 2 x + b 2 y + c 2 = 0, then
= (1 + ay ) 2 (1 + by ) 2 (1 + cy ) 2 . a1 b1 c 1
= ¹
(1 + az ) 2 (1 + bz ) 2 (1 + cz ) 2 a2 b2 c 2
(a - x )2 (b - x )2 (c - x )2 Þ Given equations are inconsistent.
x +y =2 ü
Sol. LHS = (a - y )2 (b - y )2 (c - y )2 For example, System of equations ý is
(a - z )2 (b - z )2 (c - z )2 2 x + 2y = 5þ
inconsistent because it has no solution i.e., there is no
a 2 - 2ax + x 2 b 2 - 2bx + x 2 c 2 - 2cx + x 2 value of x and y which satisfy both the equations.
= a 2 - 2ay + y 2 b 2 - 2by + y 2 c 2 - 2cy + y 2 Here, the two lines are parallel.
a 2 - 2az + z 2 b 2 - 2bz + z 2 c 2 - 2cz + z 2 (iii) Dependent equations Infinite solutions
[Identical lines]
1 2x x2 a2 -a 1
2 2 A system of (linear) equations is said to be dependent,
= 1 2y y ´ b -b 1 [row by row]
if it has infinite solutions.
1 2z z2 c2 -c 1
Let a 1 x + b 1 y + c 1 = 0 and a 2 x + b 2 y + c 2 = 0, then
1 2x x2 1 a a2 a1 b1 c 1
= = Þ Given equations are dependent.
= 1 2y y2 ´ ( - 1) ( - 1) 1 b b 2 a2 b2 c 2
1 2z z2 1 c c2 x + 2y = 3 ü
For example, System of equations ý is
[C 1 « C 3 and taking ( -1) common from second 2 x + 4y = 6þ
determinant] dependent because it has infinite solutions i.e., there
1 2x x2 1 a a2 are infinite values of x and y satisfy both the
= 1 2y y2 ´ 1 b b2 equations. Here, the two lines are identical.
1 2z z2 1 c c2
Chap 07 Determinants 537

Cramer’s Rule Thus, x=


D1
D
D D
, y = 2 , z = 3 , where D ¹ 0
D D
…(iv)
System of linear equations in two variables The rule given in Eq. (iv) to find the values of x , y and z is
Let us consider a system of equations be called the CRAMER’S RULE.
a 1 x + b 1 y + c 1 = 0ü a1 b1 Remark
ý where ¹
a 2 x + b 2y + c 2 = 0 þ a2 b2 1. D i is obtained by replacing elements of ith columns by
On solving by cross-multiplication, we get d1, d2, d3, where i = 1, 2, 3.
x y 1 2. Cramer’s rule can be used only when D ¹ 0.
= =
(b 1 c 2 - b 2 c 1 ) (c 1 a 2 - c 2 a 1 ) (a 1 b 2 - a 2 b 1 )

or
x
=
y
=
1 Nature of Solution of System of
b1
b2
c1
c2
c1
c2
a1
a2
a1
a2
b1
b2
Linear Equations
Let us consider a system of linear equations be
b1 c1 c1 a1 a 1 x + b 1y + c 1z = d 1
b2 c2 c2 a2 a 2 x + b 2y + c 2z = d 2
or x= ,y =
a1 b1 a1 b1 a 3 x + b 3y + c 3z = d 3
a2 b2 a2 b2 Now, there are two cases arise:
Case I If D ¹ 0
D D D
System of Linear Equations In this case, x = 1 , y = 2 , z = 3
D D D
in Three Variables Then, system will have unique finite solutions and so
Let us consider a system of linear equations be equations are consistent.
a 1 x + b 1y + c 1z = d 1 …(i) Case II If D = 0
a2 x + b2 y + c 2 z = d 2 …(ii) (a) When atleast one of D 1 , D 2 , D 3 be non-zero
a 3 x + b 3y + c 3z = d 3 …(iii) (i) Let D 1 ¹ 0, then from D 1 = xD will not be satisfied
a1 b1 c 1 d 1 b1 c 1 for any value of x because D = 0 and D 1 ¹ 0 and
Here, D = a 2 b 2 c 2 , D1 = d 2 b 2 c 2 , hence no value of x is possible.
a3 b3 c3 d3 b3 c3 (ii) Let D 2 ¹ 0, then from D 2 = yD will not be satisfied
for any value of y because D = 0 and D 2 ¹ 0 and
a1 d1 c1 a1 b1 d1 hence no value of y is possible.
D2 = a 2 d2 c 2 and D 3 = a 2 b2 d2 (iii) Let D 3 ¹ 0, then from D 3 = zD will not be satisfied
a3 d3 c3 a3 b3 d3 for any value of z because D = 0 and D 3 ¹ 0 and
If D ¹ 0, then hence no value of z is possible.
d1 b1 c1 a 1 x + b 1y + c 1z b 1 c1 Thus, if D = 0 and any of D 1 , D 2 , D 3 is non-zero.
D1 = d 2 b2 c 2 = a 2 x + b 2y + c 2z b 2 c2 Then, the system has no solution i.e., equations
are inconsistent.
d3 b3 c3 a 3 x + b 3y + c 3z b 3 c3
(b) When D 1 = D 2 = D 3 = 0
Applying C 1 ® C 1 - yC 2 - zC 3 , then D 1 = xDü
a1 x b1 c 1 a1 b1 c1 ï
In this case, D 2 = yDý will be true for all values of x , y
D1 = a 2 x b 2 c 2 = x a 2 b 2 c 2 = xD
D 3 = zDïþ
a3x b3 c 3 a3 b3 c3
and z.
D
\ x = 1 , where D ¹ 0 But, since a 1 x + b 1 y + c 1 z = d 1 , therefore only two of
D x , y and z will be independent and third will be
Similarly, D 2 = yD and D 3 = zD dependent on the other two.
D D Thus, the system will have infinite number of
\ y = 2 and z = 3
D D solutions i.e., equations are consistent.
538 Textbook of Algebra

Remark Expanding along R 3 , then


1. If D ¹ 0, the system will have unique finite solution and so 9 2
equations are consistent. D 1 = ( - 1) = - (108 - 104 ) = - 4
52 12
2. If D = 0 and atleast one of D1, D 2, D 3 be non-zero, then the
system has no solution i.e., equations are inconsistent. 1 9 1
3. If D = D1 = D 2 = D 3 = 0, the equations will have infinite Þ D 2 = 2 52 7
number of solutions i.e. equations are consistent.
2 0 -1
y Example 29. Solve the following system of Applying C 1 ® C 1 + 2C 3 , then
equations by Cramer’s rule. 3 9 1
x + y = 4 and 3x - 2y = 9 M
1 1
Sol. Here, D = = -2 - 3 = -5 ¹ 0 D 2 = 16 52 7
3 -2
M
4 1
D1 = = - 8 - 9 = - 17 0 L 0 L -1
9 -2
Expanding along R 3 , then
1 4
and D 2 = = 9 - 12 = - 3 1 1 9
3 9 3 9
D 2 = ( - 1) = - (156 - 144 ) = - 12 and D 3 = 2 5 52
Then, by Cramer’s rule 16 52
2 1 0
D -17 17 D -3 3
x= 1= = and y = 2 = =
D -5 5 D -5 5 Applying C 1 ® C 1 - 2C 2 , then
17 3 -1 1 9
\ x = ,y =
5 5 M
D 3 = -8 5 52
y Example 30. Solve the following system of
equations by Cramer’s rule. M
0 L 1 L 0
x+ y +z =9
2x + 5y + 7z = 52 Expanding along R 3 , then
-1 9
2x + y - z = 0 D 3 = ( - 1)
1 1 1 -8 52
Sol. Here, D = 2 5 7 = - ( -52 + 72) = - 20
2 1 -1 Then, by Cramer’s rule
Applying C 2 ® C 2 - C 1 and C 3 ® C 3 - C 1, then D -4 -12
x= 1= = 1, y = =3
D -4 -4
1 L 0 L 0
D -20
M and z= 3 = =5
D -4
= 2 3 5
\ x = 1, y = 3, z = 5
M
2 -1 -3 y Example 31. For what values of p and q, the
Expanding along R1, then system of equations
9 1 1 x +y +z =6
3 5
D =1 = - 9 + 5 = - 4 ¹ 0, D1 = 52 5 7
-1 -3 x + 2y + 3z = 10
0 1 -1
x + 2y + pz = q has
Applying C 2 ® C 2 + C 3 , then (i) unique solution?
9 2 1
(ii) an infinitely many solutions?
M
(iii) no solution?
D1 = 52 12 7 Sol. Given equations are
M x +y +z = 6 Þ x + 2y + 3z = 10
0 L 0 L -1 x + 2y + pz = q
Chap 07 Determinants 539

1 1 1 6 1 1 y Example 32. Find the value of l, if the following


\ D= 1 2 3 = ( p - 3) Þ D 1 = 10 2 3 equations are consistent
1 2 p q 2 p x + y - 3= 0
= 6(2p - 6) - 1(10p - 3q ) + (20 - 2q ) (1 + l ) x + (2 + l ) y - 8 = 0
x - (1 + l )y + (2 + l ) = 0
= 2p + q - 16
Sol. The given equations in two unknowns are consistent, then
1 6 1
1 1 -3
D 2 = 1 10 3
(1 + l ) (2 + l ) -8 = 0
1 q p
1 - (1 + l ) (2 + l )
= 1(10p - 3q ) - 6( p - 3) + 1(q - 10) = 4 p - 2q + 8
1 1 6 Applying C 2 ® C 2 - C 1 and C 3 ® C 3 + 3C 1, then
and D 3 = 1 2 10 1 L 0 L 0
1 2 q M
(1 + l ) 1 ( 3l - 5) = 0
= 1(2q - 20) - 1(q - 10) + 6(2 - 2) = q - 10
M
(i) For unique solution, A ¹ 0 Þ p ¹ 3, q Î R
1 - (2 + l ) (5 + l )
(ii) For infinitely many solutions, D = D1 = D 2 = D 3 = 0
\ p = 3, q = 10 Expanding along R1, then
1 3l - 5
(iii) For no solution, D = 0 and atleast one of D1, D 2 , D 3 is 1× =0
non-zero is p = 3 and q ¹ 10. - (2 + l ) (5 + l )
Þ ( 5 + l ) + ( 2 + l ) ( 3l - 5) = 0
Þ 3l2 + 2l - 5 = 0 or (3l + 5) ( l - 1) = 0
Condition for Consistency of Three \ l = 1, -5 / 3
Linear Equations in Two Unknowns
Let us consider a system of linear equations in x and y
a 1 x + b 1y + c 1 = 0 …(i)
System of Homogeneous
a 2 x + b 2y + c 2 = 0 …(ii) Linear Equations
a 3 x + b 3y + c 3 = 0 …(iii) Let us consider a system of homogeneous linear equations
will be consistent, the values of x and y obtained from any in three unknown x , y and z be
two equations satisfy the third equation. a 1 x + b 1y + c 1z = 0 …(i)
On solving Eqs. (ii) and (iii) by Cramer’s rule, we have a2 x + b2 y + c 2 z = 0 …(ii)
x y 1 a 3 x + b 3y + c 3z = 0 …(iii)
= = a1 b1 c 1
b2 c 2 c 2 a2 a2 b2
Here, D = a2 b2 c 2
b3 c3 c3 a3 a3 b3
a3 b3 c3
These values of x and y will satisfy Eq. (i), then
Case I If D ¹ 0, then x = 0, y = 0, z = 0 is the only solution of
b c2 c a2 a b2
a1 2 + b1 2 + c1 2 =0 above system. This solution is called a Trivial solution.
b3 c3 c3 a3 a3 b3
Case II If D = 0, atleast one of x , y and z is non-zero. This
b2 c2 a2 c2 a2 b2 solution is called a Non-trivial solution.
Þ a1 - b1 + c1 =0
b3 c3 a3 c3 a3 b3 Explanation From Eqs. (ii) and (iii), we get
x y z
a1 b1 c1 = =
(b 2 c 3 - b 3 c 2 ) (c 2 a 3 - c 3 a 2 ) (a 2 b 3 - a 3 b 2 )
\ a2 b2 c2 = 0
x y z
a3 b3 c3 or = = = k [say] ( ¹ 0 )
b2 c 2 c 2 a2 a2 b2
which is the required condition.
b3 c3 c3 a3 a3 b3
Remark b2 c2 c a2 a b2
For consistency of three linear equations in two knowns, the \ x =k ,y =k 2 and z = k 2
number of solution is one. b3 c3 c3 a3 a3 b3
540 Textbook of Algebra

On putting these values of x , y and z in Eq. (i), we get Expanding along R 2 , we get
l - 1 5l + 1
ì b c2 ü ì c 2 a2 ü ì a2 b2 ü ( l - 3) =0
a 1 ík 2 ý + b 1 ík ý + c 1 ík ý =0 3- l l -3
î b3 c3 þ î c3 a3 þ î a3 b3 þ Þ ( l - 3) [( l - 1) ( l - 3) - (3 - l ) (5l + 1)] = 0
b c2 a c2 a b2 Þ ( l - 3) 2 × 6l = 0
Þ a1 2 - b1 2 + c1 2 = 0 [Qk ¹ 0 ]
b3 c3 a3 c3 a3 b3 \ l =0, 3
a1 b1 c 1 Here, smallest value of l is 0.
or a 2 b 2 c 2 = 0 or D = 0 \ The first two equations can be written as x - y = 0 and
x + 2y - 3z = 0.
a3 b3 c3 Using Cramer’s rule, we get
x y z
This is the condition for system have Non-trivial solution. = =
-1 0 0 1 1 -1
Remark 2 -3 -3 1 1 2
1. If D ¹ 0, the given system of equations has only zero solution x y z x y z
for all its variables, then the given equations are said to have Þ = = Þ = =
Trivial solution. 3 3 3 1 1 1
\ x :y :z = 1:1:1
2. If D = 0, the given system of equations has no solution or
infinite solutions for all its variables, then the given
equations are said to have Non-trivial solution. y Example 34. Given, x = cy + bz , y = az + cx and
z = bx + ay , where x , y and z are not all zero, prove
y Example 33. Find all values of l for which the that a 2 + b 2 + c 2 + 2abc = 1.
equations Sol. The given equation can be rewritten as
( l - 1) x + ( 3l + 1) y + 2lz = 0 x - cy - bz = 0
( l - 1) x + (4 l - 2)y + ( l + 3) z = 0 - cx + y - az = 0
-bx - ay + z = 0
2x + ( 3l + 1) y + 3 ( l - 1) z = 0 Since, x, y and z are not all zero, the system will have
non-trivial solution, if
possess non-trivial solution and find the ratios x : y : z ,
1 -c -b
where l has the smallest of these values.
-c 1 -a = 0
Sol. The given system of linear equations has non-trivial solu-
tion, then we must have -b -a 1
l - 1 3l + 1 2l Applying C 2 ® C 2 + cC 1 and C 3 ® C 3 + bC 1, then
l - 1 4l - 2 l + 3 = 0 1 L 0 L 0
2 3l + 1 3( l - 1) M
-c 1 - c2 - a - bc = 0
Applying R 2 ® R 2 - R1 and R 3 ® R 3 - R1, then
M
l - 1 3l + 1 2l
-b - a - bc 1 - b2
0 l -3 -l + 3 = 0
3- l 0 l -3 Expanding along R1, we get
Applying C 3 ® C 3 + C 2 , then 1 - c2 - a - bc
1 =0
l -1 3l + 1 5l + 1 - a - bc 1 - b2
M Þ (1 - c 2 ) (1 - b 2 ) - (a + bc )2 = 0
0 L l -3 L 0 =0 Þ 1 - b 2 - c 2 + b 2c 2 - a 2 - b 2c 2 - 2abc = 0
M Þ a 2 + b 2 + c 2 + 2abc = 1
3- l 0 l -3
Chap 07 Determinants 541

#L Exercise for Session 3


1. Number of second order determinants which have maximum values whose each entry is either -1or 1 is equal to
(a) 2 (b) 4 (c) 6 (d) 8
2. Minimum value of a second order determinant whose each entry is either 1 or 2 is equal to
(a) 0 (b) -1 (c) -2 (d) -3
l1 m1 n1
3. If l i2 + m i2 + n i2 = 1, (i = 1, 2, 3) and l il j + m i m j + n i n j = 0, (i ¹ j ; i , j = 1, 2, 3) and D = l 2 m 2 n 2 , then
l 3 m 3 n3
(a) |D | = 3 (b) |D | = 2 (c) |D | = 1 (d) |D | = 0
a11 a12 a13
4. Let D 0 = a 21 a 22 a 23 and D1 denotes the determinant formed by the cofactors of elements of D 0 and D 2
a 31 a 32 a 33
denote the determinant formed by the cofactors of D1 and so on. D n denotes the determinant formed by the
cofactors of D n - 1, the determinant value of D n is
n 2
(a) D20n (b) D20 (c) Dn0 (d) D20

1 x x2 x3 -1 0 x - x4
5. If x x2 1 = 3, then the value of 0 x - x 4 x 3 - 1 , is
x2 1 x x - x4 x3 -1 0
(a) 6 (b) 9 (c) 18 (d) 27
2 2 2 2
(a1 - b 1) (a1 - b 2 ) (a1 - b 3) (a1 - b4 )
(a 2 - b 1)2 (a 2 - b 2 )2 (a 2 - b 3)2 (a 2 - b4 )2
6. The value of the determinant , is
(a 3 - b 1)2 (a 3 - b 2 )2 (a 3 - b 3 )2 (a 3 - b4 )2
(a4 - b 1)2 (a4 - b 2 )2 (a4 - b 3)2 (a4 - b4 )2
(a) depends on ai ,i = 1, 2, 3, 4 (b) depends on b i, i = 1, 2, 3, 4 (c) depends on c i , i = 1, 2, 3, 4 (d) 0
1 + x1 1 + x1x 1 + x1x 2
7. Value of 1 + x 2 1 + x 2 x 1 + x 2 x 2 depends upon
1+ x3 1 + x 3x 1 + x 3x 2
(a) only x (b) only x1 (c) only x 2 (d) None of these

8. If the system of linear equations x + y + z = 6, x + 2y + 3z = 14 and 2x + 5y + lz = m ( l, m ÎR ) has a unique


solution, then
(a) l ¹ 8 (b) l = 8 and m ¹ 36 (c) l = 8 and m = 36 (d) None of these
9. The system of equations ax - y - z = a - 1, x - ay - z = a - 1, x - y - az = a - 1
has no solution, if a is
(a) either -2 or 1 (b) -2 (c) 1 (d) not (-2)

10. The system of equations x + 2y - 4z = 3, 2x - 3y + 2z = 5 and x - 12y + 16z = 1has


(a) inconsistent solution (b) unique solution (c) infinitely many solutions (d) None of these

11. If c < 1and the system of equations x + y - 1 = 0, 2x - y - c = 0 and -bx + 3by - c = 0 is consistent, then the
possible real values of b are
(a) b Îæç -3,

(b) b Îæç - , 4ö÷ (c) b Îæç - , 3ö÷
3 3
÷ (d) None of these
è 4ø è 2 ø è 4 ø
12. The equations x + 2y = 3, y - 2x = 1and 7x - 6y + a = 0 are consistent for
(a) a = 7 (b) a = 1 (c) a = 11 (d) None of these
13. Values of k for which the system of equations x + ky + 3z = 0, kx + 2y + 2z = 0 and 2x + 3y + 4z = 0 possesses
non-trivial solution
(a) ìí2, üý (b) ìí-2, üý (c) ìí2, - üý (d) ìí-2, - üý
5 5 5 5
î 4þ î 4þ î 9þ î 4þ
Session 4
Differentiation of Determinant, Integration of a
Determinant, Walli’s Formula, Use of Σ in Determinant

Differentiation of Determinant Corollary II For n = 3, D( x ) = [C 1 C 2 C 3 ], then


Let D( x ) be a determinant of order n. If we write D ¢( x ) = [C 1¢ C 2 C 3 ] + [C 1 C 2¢ C 3 ] + [C 1 C 2 C 3¢ ]
D ( x ) = [C 1 C 2 C 3 KC n ], where C 1 , C 2 , C 3 ,K, C n denotes é R1 ù é R ¢1 ù é R 1 ù é R 1 ù
1st, 2nd, 3rd, K, nth columns respectively, then
Also, if D( x ) = R 2 , then D ¢( x ) = ê R 2 ú + ê R ¢2 ú + ê R 2 ú
ê ú
D¢ ( x ) = [C 1¢ C 2 C 3 × × × C n ] + [C 1 C ¢ 2 C 3 × × × C n ] ê ú ê ú ê ú ê ú
êë R 3 úû êë R 3 úû êë R 3 úû êë R ¢3 úû
+[C 1 C 2 C 3¢ × × × C n ] + × × × +[C 1 C 2 C 3 × × × C n¢ ]
a1 (x ) a2 (x ) a 3 (x )
= å [C 1 ¢ C 2 C 3 × × × C n ]
For example, Let D( x ) = b 1 ( x ) b 2 ( x ) b 3 ( x ) , then
where C i¢ denotes the column which contains the
derivative of all the functions in the ith column C i . Also, if c 1 (x ) c 2 (x ) c 3 (x )
é R1 ù a 1¢ ( x ) a 2¢ ( x ) a ¢3 ( x )
êR ú D ¢( x ) = b 1 ( x ) b 2 ( x ) b 3 ( x )
ê 2ú
D( x ) = ê R 3 ú c 1 ( x ) c 2 ( x ) c 3( x )
ê ú
ê M ú a1 (x ) a2 (x ) a 3 (x ) a1 (x ) a2 (x ) a 3 (x )
êë R n úû + b 1¢ ( x ) b 2¢ ( x ) b 3¢ ( x ) + b 1 ( x ) b 2 ( x ) b 3 ( x )
where R 1 , R 2 , R 3 ,K, R n denote 1st, 2nd, 3rd, K, nth rows c 1 (x ) c 2 (x ) c 3 (x ) c 1¢ ( x ) c 2¢ ( x ) c 3¢ ( x )
respectively, then
é R 1¢ ù é R 1 ù é R 1 ù [derivative according to rowwise]
é R1 ù é R 1¢ ù
ê ú ê ú ê ú ê ú êR ú
ê R 2 ú ê R 2¢ ú ê R 2 ú R
ê 2ú ê 2ú Remark
D ¢( x ) = ê R 3 ú + ê R 3 ú + ê R 3¢ ú + × × × + ê R 3 ú = å ê R 3 ú 1. In a third order determinant, if two rows (columns) consist
ê ú ê ú ê ú ê ú ê ú functions of x and third row (column) is constant, let
ê M ú ê M ú ê M ú ê M ú ê M ú a1( x ) a2( x ) a3( x )
ê R n úû êë R n úû ê R n ú ëê R n¢ úû êë R n ú
ë ë û û D( x ) = b1( x ) b2( x ) b 3( x ) , then
c1 c2 c3
where R ¢i denotes the row which contains the derivative of
a1¢( x ) a2¢( x ) a3¢( x ) a1( x ) a2( x ) a3( x )
all the functions in the ith row R i .
D ¢( x ) = b1( x ) b2( x ) b 3( x ) + b1¢( x ) b2¢( x ) b¢3( x )
Corollary I For n = 2, c1 c2 c3 c1 c2 c3
D( x ) = [C 1 C 2 ] , then D ¢( x ) = [C 1¢ C 2 ] + [C 1 C 2¢ ] 2. In a third order determinant, if only one row (column)
consists functions of x and other rows (columns) are
éR ù éR ¢ ù é R ù
Also, if D( x ) = ê 1 ú , then D ¢( x ) = ê 1 ú + ê 1 ú constant, let
ëR2 û ë R 2 û ë R ¢2 û a1( x ) a2( x ) a3( x ) a1¢( x ) a2¢( x ) a3¢( x )
D( x ) = b1 b2 b 3 , then D ¢( x ) = b1 b2 b3
a1 (x ) b1 (x )
For example, Let D( x ) = , then c1 c2 c3 c1 c2 c3
a2 (x ) b2 (x ) and in general
dn dn dn
a 1¢ ( x ) b 1¢ ( x )
a (x ) b1 (x ) dx n
{a1( x )}
dx n
{a2( x )}
dx n
{a3( x )}
D ¢( x ) = + 1 d n
{D( x )} = b1 b2 b3
a 2 ( x ) b 2 ( x ) a 2¢ ( x ) b 2¢ ( x ) dx n
c1 c2 c3
[derivative according to rowwise]
Chap 07 Determinants 543

Important Derivatives y Example 37. Let a be a repeated root of a


quadratic equation f ( x ) = 0 and A( x ), B( x ) and C ( x ) be
(Committed to Memory) polynomials of degree 3, 4, and 5 respectively, show
If a and b are constants and n Î N , then A( x ) B( x ) C ( x )
d ny that A(a ) B(a ) C (a ) is divisible by f ( x ), where
1. if y = (ax + b ) n , then = n !an
n
dx A ¢(a ) B ¢(a ) C ¢(a )
d ny æ np ö prime ( ¢ ) denotes the derivatives.
2. if y = sin (ax + b ), then = sin ç + ax + b ÷ × a n
dx n è 2 ø Sol. Since, a is a repeated root of the quadratic equation
d ny æ np ö f ( x ) = 0, then f ( x ) can be written as f ( x ) = a ( x - a )2 ,
3. if y = cos (ax + b ), then = cos ç + ax + b ÷ × a n where a is some non-zero constant.
dx n è 2 ø
A ( x ) B( x ) C ( x )
sin x cos x sin x Let g ( x ) = A (a ) B(a ) C (a )
y Example 35. If f ( x ) = cos x - sin x cos x , A ¢(a ) B ¢(a ) C ¢(a )
x 1 1 g ( x ) is divisible by f ( x ), if it is divisible by ( x - a )2 i.e.,
f ¢( 0 ) 2
find the value of 2 + { f ¢(1)} . g(a ) = 0 and g ¢ (a ) = 0. As A ( x ), B( x ) and C ( x ) are
polynomials of degree 3, 4 and 5, respectively.
cos x - sin x cos x sin x cos x sin x
\ Degree of g ( x ) ³ 2
Sol. f ¢( x ) = cos x - sin x cos x + - sin x - cos x - sin x
A (a ) B(a ) C (a )
x 1 1 x 1 1
Now, g (a ) = A (a ) B(a ) C (a ) = 0
sin x cos x sin x
A ¢(a ) B ¢ (a ) C ¢(a )
+ cos x - sin x cos x [derivative according to rowwise]
1 0 0 [Q R1 and R 2 are identical]
cos x sin x A ¢( x ) B ¢ ( x ) C ¢( x )
=0+0+1 = cos 2 x + sin 2 x = 1
- sin x cos x Also, g ¢ ( x ) = A (a ) B(a ) C (a )
\ f ¢( x ) = 1 Þ f ¢(0) = 1 and f ¢(1) = 1 A ¢(a ) B ¢ (a ) C ¢(a )
f ¢( 0 )
Þ 2 + { f ¢(1)} 2 = 21 + 12 = 3 A ¢(a ) B ¢ (a ) C ¢(a )
\ g ¢ (a ) = A (a ) B(a ) C (a ) = 0
cos x sin x cos x
A ¢(a ) B ¢(a ) C ¢(a )
y Example 36. Let f ( x ) = cos 2x sin 2x 2 cos 2x ,
[Q R1 and R 3 are identical]
cos 3x sin 3x 3 cos 3x
This implies that f ( x ) divides g ( x ).
æpö
then find the value of f ¢ ç ÷.
è2ø y Example 38. Find the coefficient of x in the
cos x sin x cos x determinant
Sol. Given, f ( x ) = cos 2x sin 2x 2 cos 2x (1 + x )a 1b1 (1 + x )a 1b2 (1 + x )a 1b3
cos 3x sin 3x 3 cos 3x (1 + x )a 2 b1 (1 + x )a 2 b2 (1 + x )a 2 b3
- sin x sin x cos x
(1 + x )a 3 b1 (1 + x )a 3 b2 (1 + x )a 3 b3
\ f ¢( x ) = -2sin 2x sin 2x 2 cos 2x
-3sin 3x sin 3x 3cos3x Sol. We know that, if f ( x ) be a polynomial, then coefficient of
cos x cos x cos x cos x sin x - sin x 1 n
x n in f ( x ) = f (0).
+ cos 2x 2 cos 2x 2 cos 2x + cos 2x sin 2x -4 sin 2x n!
cos 3x 3 cos 3x 3 cos 3x cos 3x sin 3x -9 sin 3x (1 + x )a1b1 (1 + x )a1b2 (1 + x )a1b3
[derivative according to columnwise] Let f ( x ) = (1 + x )a2 b1 (1 + x )a2 b2 (1 + x )a2 b3
-1 1 0 0 1 -1 (1 + x )a3 b1 (1 + x )a3 b2 (1 + x )a3 b3
æpö
Þ f ¢ç ÷ = 0 0 -2 + 0 + -1 0 0 a1b1(1 + x )a1b1 - 1 a1b 2 (1 + x )a1b2 - 1 a1b 3 (1 + x )a1b3 - 1
è2ø
3 -1 0 0 -1 9 \ f ¢( x ) = (1 + x )a2 b1 (1 + x )a2 b2 (1 + x )a2 b3
[QC 2 = C 3 in second determinant] (1 + x )a3 b1 (1 + x )a3 b2 (1 + x )a3 b3
= 2( 1 - 3) + 1( 9 - 1) = - 4 + 8 = 4
544 Textbook of Algebra

(1 + x )a1b1
a b2
(1 + x ) 1 (1 + x )a1 b3 ì a q l ü
ï ï
+ a 2 b1 (1 + x )a2 b1 - 1 a 2b 2 (1 + x )a2 b2 - 1 a 2 b 3 (1 + x )a2 b3 - 1 ísum of all cofactors in D (0), where D (0) = b f m ý
ï g y n ïþ
(1 + x )a3 b1 (1 + x )a3 b2 (1 + x )a3 b3 î
\ D ¢¢( x ) = 0 [QS is constant]
(1 + x )a1b1 (1 + x )a1 b2 (1 + x )a1 b3
Since, D ¢( x ) = S
+ (1 + x )a2 b1 (1 + x )a2 b2 (1 +x )a2 b3 On integrating D( x ) = Sx + C
a 3 b1 - 1 a 3 b2 - 1
a 3b1 (1 + x ) a 3b 2 (1 + x ) a 3b 3 (1 + x )a3 b3 - 1 \ D(0) = 0 + C
a1b1 a1b 2 a1b 3 1 1 1 Hence, D( x ) = Sx + D(0)
\ f ¢(0) = 1 1 1 + a 2 b1 a 2 b 2 a 3 b 3 xn sin x cos x
1 1 1 1 1 1 æ np ö æ np ö
1 1 1 y Example 40. If f ( x ) = n ! sin ç ÷ cos ç ÷ ,
è 2 ø è 2 ø
+ 1 1 1 2
p p p3
a 3b1 a 3b 2 a 3b 3 n
= 0 + 0 + 0= 0 d
then find the value of { f ( x )} at x = 0, n Î I.
f ¢(0) dx n
\ Coefficient of x in f ( x ) = =0
1! dn dn dn
n
(x n) n
(sin x ) (cos x )
Aliter dx dx dx n
n
d æn p ö æn p ö
(1 + x )a1b1 (1 + x )a1b2 (1 + x )a1b3 Sol. { f ( x )} = n! sin ç ÷ cos ç ÷
dx n è 2 ø è 2 ø
Let (1 + x )a2 b1 (1 + x )a2 b2 (1 + x )a2 b3 = A + Bx + Cx 2 + ×××
p p2 p3
(1 + x )a3 b1 (1 + x )a3 b2 (1 + x )a3 b3

On differentiating both sides w.r.t. x and then put x = 0 in æn p ö æn p ö


n ! sin ç + x÷ cos ç + x÷
both sides, we get è 2 ø è 2 ø
a1b1 a1b2 a1b3 1 1 1 1 1 1 æn p ö æn p ö
= n! sin ç ÷ cos ç ÷
B= 1 1 1 + a 2 b1 a 2 b2 a 2 b3 + 1 1 1 è 2 ø è 2 ø
1 1 1 1 1 1 a 3b1 a 3b2 a 3b3 p p2 p3

= 0 + 0 + 0= 0
æn p ö æn p ö
Hence, coefficient of x in given determinant is 0. n ! sin ç ÷ cos ç ÷
è 2 ø è 2 ø
a+ x q+ x l+ x dn æn p ö æn p ö
\ { f ( x )} at( x = 0) = n ! sin ç ÷ cos ç ÷
dx n è 2 ø è 2 ø
y Example 39. If D( x ) = b + x f + x m + x ,
p p2 p3
g+x y+x g+x
show that D ¢¢( x ) = 0 and D( x ) = D(0) + Sx , where S =0 [Q R1 and R 2 are identical]
denotes the sum of all the cofactors of all elements in
D(0) and dash denotes the derivative.
1 q+x l+x a+x 1 l+x
Integration of a Determinant
Sol. We have, D ¢( x ) = 1 f+x m+x + b+x 1 m+x
f ( x ) g ( x ) h( x )
1 y+x n+x g +x 1 n+x Let D( x ) = p q r
a+x q+x 1 l m n
+ b + x f+ x 1 where p, q, r , l, m and n are constants, then
g+x y+x 1 b b b

Applying C 2 ® C 2 - xC 1 and C 3 ® C 3 - xC 1 in first, b


òa f ( x )dx òa g ( x )dx òa h( x ) dx
C 1 ® C 1 - xC 2 and C 3 ® C 3 - xC 2 in second and òa D( x ) dx = p q r
C 1 ® C 1 - xC 3 and C 2 ® C 2 - xC 3 in third, then l m n
1 q l a 1 l a q 1
Remark
D ¢( x ) = 1 f m + b 1 m + b f 1
If in a determinant, the elements of more than one columns or
1 y n g 1 n g y 1 rows are functions of x , then the integration can be done only
after evaluation or expansion of the determinant.
Chap 07 Determinants 545

a b c
Important Integrals (Committed to Memory) y Example 41. If D( x ) = 6 4 3 , then
p /2 sinn x p x x 2
x3
1. (i) ò0 n n
sin x + cos x
dx =
4
1
find ò D( x ) dx.
0
p /2 cos n x a b c
=ò dx , "n Î R 1
0 n
sin x + cos x n
Sol. ò D( x ) dx = 6 4 3
0 1 1 1 3
p /2 tan x n
p p /2 dx ò dx
x ò0 x 2dx ò dx
x
ò0 dx = = ò , "n Î R 0 0
(ii)
n 4 0 1 + tann x
1 + tan x
a b c a b c
p /2 dx p p /2 cot n x
(iii) ò0 1 + cot n x
= =ò
4 0 1 + cot n x
dx , "n Î R = 6
1
4
1
3
1
= 6 4
1 1
3
1
éx2 ù éx 3 ù éx4 ù
p /2 p /2 p ê ú ê ú ê ú 2 3 4
2. (i) ò0 ln sin x dx = ò
0
ln cos x dx = - ln 2
2
ë 2 û0 ë 3 û0 ë 4 û0
p æ 1ö a b c
or ln ç ÷ 0
2 è2ø
1 0 0
Applying R 2 ® R 2 - 12R 3 , then ò D( x )dx = =0
p /2 p /2
0 1 1 1
(ii) ò0 ln tan x dx = ò
0
ln cot x dx = 0 2 3 4
p /2 p /2 p y Example 42. If
(iii) ò0 ln sec x dx = ò
0
ln cosec x dx =
2
ln 2
sin x
sin 5 x ln sin x
sin x + cos x
n n
Walli’s Formula f (x ) = n åk
k =1
Pk
k =1
,
p /2
(An easy way to evaluate ò sinm x cos n x dx, where 8 p æ 1ö p
0 ln ç ÷
p /2 15 2 è2ø 4
m, n ÎW ) We have, ò0 sinm x cos n x dx
then find the value of
p /2
ò0 f ( x ) d x.
{(m - 1)(m - 3 ) K2 or 1} {(n - 1) (n - 3 ) K2 or 1}
= p/ 2
{(m + n ) (m + n - 2 ) (m + n - 4 ) K2 or 1} Sol. ò0 f ( x ) dx
p/ 2 p/ 2 p/ 2 sin x
where, p is p / 2, ifm and n are both even, otherwise p = 1. ò0 sin 5 x dx ò0 lnsin x dx ò0
The last factor in each of three products is either 1 or 2. In sin x + cos x
n n
case any ofm or n is 1, we simply write 1 as the only factor to = n åk Pk ,
replace its product. If any ofm or n is zero provided, we put 1 k =1 k =1
as the only factor in its product and we regard 0 as even. 8 p æ1ö p
ln ç ÷
For example, 15 2 è2ø 4
p /2 [5 × 3 × 1][3 × 1] p 3 p p p
1. ò sin 6 x cos 4 x dx = × = 4 2
× - ln 2
0 [10 × 8 × 6 × 4 × 2 ] 2 512 5 3 2 4
p /2 [5 × 3 × 1][2 ] 2 n n
2. ò sin 6 x cos 3 x dx =
0 [9 × 7 × 5 × 3 × 1]
×1=
63
= n åk Pk [by Walli’s formula]
k =1 k =1
p /2 [ 4 × 2 ][ 6 × 4 × 2 ] 1 8 p æ1ö p
3. ò sin5 x cos 7 x dx = ×1= ln ç ÷
0 [12 × 10 × 8 × 6 × 4 × 2 ] 120 15 2 è2ø 4
p /2 [ 7 × 5 × 3 × 1 ] p 35 p p æ1ö p
4. ò sin 8 x dx = × = 8
ln ç ÷
0 [8 × 6 × 4 × 2 ] 2 256 15 2 è2ø 4
p /2 [6 × 4 × 2 ] 16 n n
5. ò cos 7 x dx = ×1= åk
0 [7 × 5 × 3 × 1] 35
= n
k =1
Pk
k =1
=0 [since R1 and R 3 are identical]

p /2 [9 × 7 × 5 × 3 × 1][1] 1 8 p æ1ö p
6. ò0 sin10 x cos x dx =
[11 × 9 × 7 × 5 × 3 × 1]
×1=
11 15
ln ç ÷
2 è2ø 4
546 Textbook of Algebra

y Example 43. Let f ( x )


sec x cos x sec 2 x + cot x cosec x
Important Summation
= cos 2 x cos 2 x cosec 2 x , then find (Committed to Memory)
2 2
n
n(n + 1)
1 cos x cos x 1. S r = S n = 1 + 2 + 3 +¼ + n =
p /2 r =1 2
the value of ò0 f ( x ) dx. n
n (n + 1) (2n + 1)
2
2. S r 2 = S n 2 = 12 + 2 2 + 3 2 +¼ + n 2 =
Sol. Applying C 2 ® C 2 - cos x C 1,then r =1 6
n
2 3. S r 3 = Sn 3 = 1 3 + 2 3 + 3 3 + ¼ + n 3
sec x 0 sec x + cot x cosec x r =1
2
f ( x ) = cos 2 x cos 2 x - cos 4 x cosec2 x é n(n +1) ù
= = (å n)2
1 0 cos 2 x êë 2 úû
[expanding along C 2 ] n
4. S a = S a = a + a + a +¼ + a = an
sec x sec 2 x + cot x cosec x r =1 1442443
= (cos 2 x - cos 4 x ) n times
1 cos 2 x
n
= (cos 2 x - cos 4 x ) (cos x - sec 2 x - cot x cosec x ) 5. S ( l - 1) lr - 1 = ln - 1, " l ¹ 1 and l > 1
r =1
2 æ 2 1 cos x ö
= cos x ( 1 - cos x ) ç cos x - - ÷ (n - 1) b ü
è cos x sin 2 x ø
2 ì æ nb ö
sin ía + ý sin ç ÷
æ 1 cos x ö
n
î 2 þ è 2 ø
= cos 2 x sin 2 x ç cos x - - ÷ 6. å sin [a + (r - 1) b] =
è cos x sin 2 x ø
2 r =1 æbö
sin ç ÷
= cos 3 x sin 2 x - sin 2 x - cos 3 x è2ø
= - cos 3 x (1 - sin 2 x ) - sin 2 x Particular For a = b = q.
5
f ( x ) = - cos x - sin x 2
ìæ n + 1 ö ü æ nq ö
sin íç ÷ qý × sin ç ÷
p/ 2 p/ 2 p/ 2 n è 2 ø þ è 2 ø
\ ò0 f ( x ) dx = - ò0 cos 5 x dx - ò0 sin 2 x dx S sin r q = î
æ 4 2 ö æ1 p ö æ8 pö
r =1 æqö
= - ç × × 1÷ - ç × ÷ = - ç + ÷ sin ç ÷
è 5 3 ø è2 2 ø è 15 4 ø è2ø
[ by Walli’s formula] ì (n - 1) ü æ nb ö
cos ía + bý sin ç ÷
n
î 2 þ è 2 ø
7. S cos {a + (r - 1) b} =
Use of S in Determinant r =1 æbö
sin ç ÷
è2ø
f (r ) g (r ) h(r )
Particular For a = b = q.
If D(r ) = a b c
ìæ n + 1 ö ü æ nq ö
a1 b1 c1 cos íç ÷ qý sin ç ÷
n
îè 2 ø þ è 2 ø
å cos r q =
where a, b, c , a 1 , b 1 and c 1 are constants, independent of r, r =1 æqö
then sin ç ÷
è2ø
n n n
å f (r ) å g(r ) å h(r ) n
n r =1 r =1 r =1 8. å { f (r + 1) - f (r )} = f (n + 1) - f (1)
r =1
å D(r ) = a b c
r =1 n æ
1 n
1 1 ö
a1 b1 c1 Particular å = å ç - ÷
r = 1 r (r + 1) r =1 èr r + 1ø
1 1 n
= - =
Remark 1 n +1 n +1
If in a determinant, the elements of more than one columns or
n
rows are function of r, then the å can be done only after n
evaluation or expansion of the determinant. 9. å C r = 2n
r =1
Chap 07 Determinants 547

Remark n2 - 1 2n n +1
n
Capital pie P is not direct applicable in determint i.e., å Dr = n2 - 1 2n n +1 =0
n n n r =0

n
P f ( r ) rP= 1 g( r ) rP= 1 h( r )
r =1
cos 2 (n 2 ) cos 2 n cos 2 (n + 1)
P
r =1
D( r ) ¹ a b c [since R1 and R 2 are identical]
a1 b1 c1
y Example 45. Let n be a positive integer and
Explanation P
n
D( r ) = D( 1) ´ D( 2) ´ ¼ ´ D( n)
r2 +r r +1 r -2
r =1 2
D r = 2r + 3r - 1 3r 3r - 3 and
f ( 1) g ( 1) h( 1) f ( 2) g ( 2) h( 2) f ( n) g ( n) h( n)
= a b c ´ a b c ´¼´ a b c r 2 + 2r + 3 2r - 1 2r - 1
a1 b1 c1 a1 b1 c1 a1 b1 c1 n
n n n
å D r = an 2 + bn + c , find the value of a + b + c .
r =1
P f ( r ) rP= 1 g ( r ) rP= 1 h( r )
r =1
¹ a b c r 2 + 1r
r +1 r -2
a1 b1 c1 Sol. We have, Dr = 2r 2 + 3r - 1 3r 3r - 3
2
r + 2r + 3 2r - 1 2r - 1

y Example 44. Let n be a positive integer and Applying R 2 ® R 2 - ( R1 + R 3 ), then


2r - 1 n
Cr 1 r2 +r r +1 r -2
2 n
Dr = n - 1 2 n+1 , prove that M
2 2 2 2 Dr = -4 ¼ 0 ¼ 0
cos n cos n cos (n + 1)
n
M
å D r = 0. r 2 + 2r + 3 2r - 1 2r - 1
r =0
2r - 1 n
Cr 1 Expanding along R 2 , we get
Sol. We have, Dr = n2 - 1 2n n +1 r +1 r -2
=4
2
cos (n ) cos n 2 2 2
cos (n + 1) 2r - 1 2r - 1

n n n = 4 [(r + 1) (2r - 1) - (r - 2) (2r - 1)]


n
å (2r - 1) å Cr å 1
r =0 r =0 r =0 = 24r - 12
n
2 n
\ å Dr = n -1 2 n +1 n n n
r =0 Now, å Dr = 24 å r - 12 å 1
cos 2 (n 2 ) cos n 2
cos 2 (n + 1) r =1 r =1 r =1

n ( n + 1)
n n n = 24 - 12n = 12n (n + 1 - 1)
Now, å (2r - 1) = 2 å r - å 1 2
r =0 r =0 r =0
= 12n 2 = an 2 + bn + c [given]
= 2 ( 0 + 1 + 2 + 3 + ¼ + n ) - ( 1 + 1 + 1 + ¼ + 1)
144 42444 3 For n = 1, we have
(n + 1) times
2n (n + 1)
= 2
- ( n + 1) = ( n + 1) ( n - 1) = n - 1 a + b + c = 12
2
548 Textbook of Algebra

#L Exercise for Session 4


x x2 x3
1. If f ( x ) = 1 2 3 , f ¢(1) is equal to
0 1 x
(a) - 1 (b) 0 (c) 1 (d) 2
sec x x2 x
f (x )
2. Let f ( x ) = 2 sin x x 3 2x 2 , lim is equal to
x ® 0 x4
2
tan 3x x x
(a) 0 (b) - 1 (c) 2 (d) 3
x 2 x
3. Let x 2 x 6 = Ax 4 + Bx 3
+ Cx 2
+ Dx + E, the value of 5A + 4B + 3C + 2D + E is equal to
x x 6
(a) - 16 (b) - 11 (c) 0 (d) 16
3
x sin x cos x
d3
4. Let f ( x ) = 6 -1 0 , where p is a constant. Then {f ( x )} at x = 0 is
dx 3
p p2 p3
(a) p (b) p + p 2 (c) p + p 3 (d) independent of p
y y1 y 2
d ny
5. If y = sin mx , the value of the determinant y 3 y4 y 5 , where y n = , is
dx n
y6 y7 y8
2 3
(a) m (b) m (c) m 9 (d) None of these
2
2 cos x sin 2x - sin x
p/ 2
6. Let f ( x ) = sin 2x 2 sin2 x cos x , the value of ò { f ( x ) + f ¢( x )} dx, is
0
sin x - cos x 0
p 3p
(a) (b) p (c) (d) 2p
2 2
x2 æxö
cos x e 2x cos 2 ç ÷
è2ø
p/ 2
7. If f ( x ) = x2 sec x sin x + x 3 , the value of ò ( x 2 + 1) [ f ( x ) + f ¢( x )] dx , is
- p/ 2
1 2 x + tan x

(a) - 1 (b) 0 (c) 1 (d) 2


2 4 1
sin x + cos x ln cos x 2
1 + (tan x )
2 p/ 2
8. If f ( x ) = p p p4 , the value of ò f ( x ) dx is
7 1 1 0
- ln 2
16 2 4

(a) 2 (b) - 1 (c) 0 (d) None of these


1 n n
n
9. If D k = 2k n2 + n + 1 n2 + n and å D k = 56, then n is equal to
k =1
2k - 1 n2 n2 + n + 1
(a) 4 (b) 6 (c) 8 (d) None of these
n-2 n-2 n-2
Cr - 2 Cr - 1 Cr
n
10. The value of å ( - 2)r -3 1 1 (n > 2) is
r =2
2 -1 0
n n
(a) 2n - 1 + (- 1) (b) 2n + 1 + (- 1) (c) 2n - 3 + (- 1)n (d) None of these
Shortcuts and Important Results to Remember
1 Symmetric Determinant The elements situated at equal 4 (i) If D = 0, then Dc = 0, where Dc denotes the
distance from the diagonal are equal both in magnitude determinant of cofactors of elements of D.
a h g (ii) If D ¹ 0, then Dc = Dn - 1, where n is order of D.
and sign. i.e. h b f = abc + 2 fgh - af 2 - bg 2 - ch 2
a11 a12 a13
g f c
(iii) Let D = a21 a22 a23
2 Skew-symmetric Determinant All the diagonal elements a31 a32 a33
are zero and the elements situated at equal distance from
the diagonal are equal in magnitude but opposite in sign. The sum of products of the elements of any row or
The value of skew-symmetric determinant of even order is column with the corresponding cofactors is equal to
always a perfect square and that of odd order is always the value of determinant, i.e.
0 c -b
0 a 2
a11C11 + a12C12 + a13C13 = a21C21 + a22C22 + a23C23
zero i.e. = a and - c 0 a =0 = a31C31 + a32C32 + a33C33 = D
-a 0
b -a 0
and sum of products of the elements of any row or
3 Circulant Determinant The elements of the rows (or column with the cofactors of the corresponding
columns) are in cyclic order. i.e., elements of any other row or column is zero, i.e.,
1 a a2 a11C21 + a12C22 + a13C23 = a11C31 + a12 C32 + a13C33
(i) 1 b b2 = (a - b) (b - c ) (c - a) =0
1 c c2 5 A homogeneous system of equations is never consistent.
a b c 1 1 1
6 Conjugate of a Determinant If ai, bi and c i Î C (i = 1, 2, 3)
(ii) a2 b2 c 2 = a2 b2 c 2
bc ca ab a3 b3 c 3 a1 b1 c1 a1 b1 c1
and D = a2 b2 c 2 , then D = a2 b2 c 2
= (a - b) (b - c ) (c - a) (ab + bc + ca)
a3 b3 c 3 a3 b3 c 3
a bc abc a a2 a3
(iii) b ca abc = b b2 b3 = abc (a - b)(b - c )(c - a) (i) If D is purely real, then D = D
c ab abc c c2 c3 (ii) If D is purely imaginary, then D = - D

1 1 1 7 (i) If x1, x2 , x3 , ¼ are in AP or a x1 , a x2 , a x3 , ¼ are in GP,


(iv) a b c = (a - b) (b - c ) (c - a) (a + b + c ) x1 x2 x3
a3 b3 c 3 then xn + 1 xn + 2 xn + 3 = 0
x2 n + 1 x2 n + 2 x2 n + 3
a b c
(v) b c a = - (a3 + b3 + c 3 - 3abc ) (ii) If a1, a2 , a3 , ¼ are in GP and ai > 0, i = 1, 2, 3, …,
c a b
log an log an + 1 log a n + 2
Remark then log an + 3 log an + 4 log an + 5 = 0
These results direct applicable in lengthy questions as behaviour
log an + 6 log an + 7 log an + 8
of standard results.
550 Textbook of Algebra

JEE Type Solved Examples :


Single Option Correct Type Questions
n This section contains 10 multiple choice examples. Sol. (b) We observe that,
Each example has four choices (a), (b), (c) and (d) out of 0 -b -c
which ONLY ONE is correct.
D= b 0 -a
l Ex. 1 If ( x 1 - x 2 ) 2 + (y 1 - y 2 ) 2 = a 2 , c a 0
( x 2 - x 3 ) 2 + (y 2 - y 3 ) 2 = b 2 and 0 b c
2
x1 y 1 1 Þ D = -b 0 a
( x 3 - x 1 ) 2 + (y 3 - y 1 ) 2 = c 2 and k x 2 y 2 1 -c -a 0
x 3 y3 1 [interchanging rows and columns]
= (a + b + c ) (b + c - a ) (c + a - b ) (a + b - c ), the value of k 0 -b -c
is
=- b 0 -a
(a) 1 (b) 2 (c) 4 (d) 8
c a 0
Sol. (c) Consider the triangle with vertices B( x 1, y1 ), C ( x 2 , y 2 )
and A ( x 3 , y 3 ) and [taking (– 1) common from each row]
AB = c , BC = a and CA = b Þ D=-D
(x3 , y3 ) A
Hence, D is purely imaginary.

c l Ex. 3 The equation


b
(1 + x ) 2 (1 - x ) 2 - (2 + x 2 )
(x1 , y1 ) B
a
C (x2 , y2 ) 2 x +1 3x 1 - 5x
x +1 2x 2 - 3x
x1 y1 1
1
\ Area of DABC = x2 y2 1 …(i) (1 + x ) 2 2 x +1 x + 1
2
x3 y3 1 + (1 - x ) 2 3x 2 x = 0 has
Also, area of DABC = s (s - a )(s - b )(s - c ), 1 - 2x 3x - 2 2 x - 3
where 2s = a + b + c …(ii) (a) no real solution
From Eqs. (i) and (ii), we get (b) 4 real solutions
x 1 y1 1 (c) two real and two non-real solutions
1 (d) infinite number of solutions, real or non-real
x 2 y 2 1 = s (s - a )(s - b )(s - c )
2
x 3 y3 1
Sol. (d) Interchanging rows and columns in first determinant,
On squaring and simplifying, we get then
2
x1 y1 1 (1 + x )2 2x + 1 x +1
4 x2 y2 1 = (a + b + c ) (b + c - a ) (1 - x )2 3x 2x
x3 y3 1 - ( 2 + x 2 ) 1 - 5x 2 - 3x
(c + a - b ) (a + b - c )
Hence, the value of k is 4. ( 1 + x ) 2 2x + 1 x + 1
l Ex. 2 If a , b and c are complex numbers, the determinant + (1 - x )2 3x 2x = 0
1 - 2x 3x - 2 2x - 3
0 -b -c
D= b 0 - a is (1 + x )2 2x + 1 x +1
c a 0 Þ (1 - x )2 3x 2x =0
(a) a non-zero real number (b) purely imaginary -( 1 + x ) 2 - 2x - 1 - x - 1
(c) 0 (d) None of these
Chap 07 Determinants 551

Applying R 3 ® R 3 + R 1, then (1 + x ) a (1 + 2 x ) b 1
2
(1 + x ) 2x + 1 x + 1 l Ex. 6 If f ( x ) = 1 (1 + x ) a
(1 + 2 x ) b ; a , b
2
Þ (1 - x ) 3x 2x =0
(1 + 2 x ) b 1 (1 + x ) a
0 0 0
being positive integers, then
Þ 0=0 (a) constant term in f ( x ) is 4
which is true for all values of x . (b) coefficient of x in f ( x ) is 0
Hence, given equation has infinite number of solutions, real (c) constant term in f ( x ) is (a - b )
or non-real. (d) constant term in f ( x ) is (a + b )
(1 + x )a (1 + 2x )b 1
l Ex. 4 If X , Y and Z are positive numbers such that Y and a
Sol. (b) Let 1 (1 + x ) (1 + 2x )b
Z have respectively 1 and 0 at their unit’s place and b
X 4 1 ( 1 + 2x ) 1 (1 + x )a

D= Y 0 1 = A + Bx + Cx 2 + L
Z 1 0 Put x = 0, then A = 0
If ( D +1) is divisible by 10, then X has at its unit’s place On differentiating both sides w.r.t. x and then put x = 0
(a) 0 (b) 1 a 2b 0 1 1 1 1 1 1
(c) 2 (d) 3 1 1 1 + 0 a 2b + 1 1 1 = B
Sol. (c) Let X = 10x + l, Y = 10y + 1 and Z =10z , where 1 1 1 1 1 1 2b 0 a
x , y , z Î N , then
X 4 1 10x + l 4 1 \ 0+0+0= B
D = Y 0 1 = 10y + 1 0 1 Þ B=0
Z 1 0 10z 1 0 Hence, constant term in f ( x ) is zero and coefficient of x in
f ( x ) is 0.
10x 4 1 l 4 1 2
= 10y 0 1 + 1 0 1 l Ex. 7 If f j = å a ij x i , j = 1, 2, 3 and f j¢ and f j¢¢are denoted
i=0
10z 1 0 0 1 0
f1 f 2 f3
x 4 1 df j d2 fj
by and respectively, then g ( x ) = f 1¢ f 2¢ f 3¢ is
= 10 y 0 1 + (1 - l ) dx dx 2
f 1¢¢ f 2¢¢ f 3¢¢
z 1 0
(a) a constant (b) a linear in x
Þ D + 1 = 10k + ( 2 - l ), (c) a quadratic in x (d) a cubic in x
x 4 1 f1¢ f 2¢ f 3¢ f3 f1f1 f 2 f 3
f2
where k = y 0 1 . Sol. (a) Q g ¢( x ) = f1¢ f 2¢ f 3¢ + f1¢¢ f 2¢¢ f 3¢¢ + f1¢ f 2¢ f 3¢
z 1 0 f1¢¢ f 2¢¢ f 3¢¢ f1¢¢ f 2¢¢ f 3¢¢ f1¢¢¢ f 2¢¢¢ f 3¢¢¢
It is given that ( D +1) is divisible by 10. Therefore, 2 - l = 0 =0+0+0 [Q f j is a quadratic function]
i.e., l = 2
\ g ( x ) = c = constant
\ X = 10x + 2
Þ 2 is at unit’s place of X . (a -1) n 6
2 2
l Ex. 8 Let Da = (a -1) 2n 4n - 2 ,
lEx. 5 The number of distinct values of a 2 ´ 2 determinant
(a -1) 3 3n 3 3n 2 - 3n
whose entries are from the set { -1, 0,1}, is
n
(a) 3 (b) 4 (c) 5 (d) 6 the value of å D a is
Sol. (c) Possible values are - 2, - 1, 0,1, 2 a =1

1 0 1 -1 0 1 (n - 1)n
i.e., = 1, = 0, = - 1, (a) 0 (b)
-1 1 0 0 1 -1 2
(n -1)n 2 (n - 1)n ( 2n - 1)
1 -1 -1 1 (c) (d)
= 2, = -2 2 3
1 1 1 1
552 Textbook of Algebra

n Sol. (a) Applying C 3 ® C 3 + C 2 - C 1, then


å ( a - 1) n 6
a =1 1 cos x 0
n n
2 2
Sol. (a) å Da = å ( a - 1) 2n 4n - 2 M
a =1 a =1 D( x ) = 1 + sin x cos x 0
n
å (a - 1)3 3n 3 3n 2 - 3n M
a =1
sin x K sin x K 1
(n - 1)n 1
n 6 = 1 (cos x - cos x - cos x sin x ) = - sin 2x
2 2
(n - 1)n (2n - 1)
= 2n 2 4n - 2 p/ 2 1 p / 2
6
2 2
\ ò0 D( x )dx = - ò
2 0
sin 2x dx
( n - 1) n
3n 3 3n 2 - 3n 1 1 1
4 = [cos 2x ]p0 / 2 = ( - 1 - 1) = -
4 4 2

1 1 6 l Ex. 10 Number of values of a for which the system of


(n - 1)n 2
2n - 1
= 2n 4n - 2 equations a 2 x + ( 2 - a )y = 4 + a 2 and ax + ( 2a - 1)y = a 5 - 2
23
(n - 1)n possess no solution, is
3n 2 3n 2 - 3n
2 (a) 0 (b) 1
(n - 1)n (c) 2 (d) infinite
[taking and n common from C 1 and C 2 ]
2 a2 2-a
Sol. (c) Q D = = a 2 ( 2a - 1) - a(2 - a )
Applying C 3 ® C 3 - 6C 1, then a 2a - 1
= 2a(a + 1)(a - 1)
1 1 0
n (n - 1)n 2 2n - 1 For no solution, D = 0
å Da = 2n 0 =0 \ a = -1, 0,1
a =1 2 3
(n - 1)n 4 + a2 2 -a
3n 2 0 Þ D1 =
2 a -25
2a - 1
1 cos x 1 - cos x Values of D1 at a = -1, 0, 1 are -6, 0, 6 respectively and
l Ex. 9 If D ( x ) = 1 + sin x cos x 1 + sin x - cos x , then a2 4 +a2
D2 =
sin x sin x 1 a a5 -2
p /2 Values of D 2 at a = -1, 0, 1 are 2, 0, - 6, respectively.
ò0 D( x ) dx is equal to
For no solution,
1 1 1 D = 0 and atleast one of D1, D 2 is non-zero.
(a) - (b) 0 (c) (d)
2 4 2
\ a = - 1,1

JEE Type Solved Examples :


More than One Correct Option Type Questions
n
This section contains 5 multiple choice examples. Each Sol. (a,c,d) Applying C 2 ® C 2 - C 1 - 2C 3 , then
example has four choices (a), (b), (c) and (d) out of which
a2 - (b 2 + c 2 ) bc a 2 b 2 + c 2 bc
more than one may be correct. 2
b - (c + a ) ca = - b 2 c 2 + a 2 ca
2 2

a2 a 2 - (b - c ) 2 bc c2 - (a 2 + b 2 ) ab c 2 a 2 + b 2 ab
l Ex. 11 The determinant b 2 b 2 - (c - a ) 2 ca is
Applying C 2 ® C 2 + C 1, then
c2 c 2 - (a - b ) 2 ab
divisible by a 2 a 2 + b 2 + c 2 bc
(a) a + b + c (b) (a + b ) (b + c ) (c + a ) = - b 2 a 2 + b 2 + c 2 ca
2 2 2
(c) a + b + c (d) (a - b ) (b - c ) (c - a ) c 2 a 2 + b 2 + c 2 ab
Chap 07 Determinants 553

Applying R 2 ® R 2 - R 1 and R 3 ® R 3 - R 1, then p p


Þ 4 q = (2n - 1)
Þ q = (2n - 1)
2 2 2 2 2 8
a K a + b +c K bc
p 3p
M For n = 0, 2, then q = - , and A ÎR
8 8
= - b2 - a2 0 c (a - b )
M l Ex. 13 The digits A, B, C are such that the three digit
2 2
c -a 0 - b(c - a ) numbers A88, 6B 8, 86 C are divisible by 72, the determinant
- (a + b )(a - b ) c (a - b ) A 6 8
= (a 2 + b 2 + c 2 ) 8 B 6 is divisible by
(c + a )(c - a ) - b(c - a )
8 8 C
- (a + b ) c
= (a - b )(c - a )(a 2 + b 2 + c 2 ) (a) 72 (b) 144 (c) 288 (d) 216
c +a -b
Applying C 1 ® C 1 - C 2 , then
Sol. (a, b, c)
- (a + b + c ) c
= (a - b ) (c - a ) (a 2 + b 2 + c 2 ) Q A 88, 6B 8, 86C are divisible by 72.
(a + b + c ) -b
\ A 88, 6B 8, 86C are also divisible by 9.
= (a - b ) (b - c ) (c - a ) (a + b + c ) (a 2 + b 2 + c 2 ) Þ A + 8 + 8, 6 + B + 8, 8 + 6 + C
are divisible by 9, then A = 2, B = 4, C = 4
p p
l Ex. 12 The value of q lying between - and and A 6 8 2 6 8
4 2
Let D= 8 B 6 = 8 4 6
p
0£A£ and satisfying the equation 8 8 C 8 8 4
2
1 + sin 2 A cos 2 A 2 sin 4 q = 2(16 - 48) - 6(32 - 48) + 8(64 - 32) =288
2
sin A 1 + cos A 2
2 sin 4 q = 0, are Hence, D is divisible by 72, 144 and 288.

sin 2 A 2
cos A 1 + 2 sin 4 q l Ex. 14 If p, q, r and s are in AP and
p p 3p p + sin x q + sin x p - r + sin x
(a) A = ,q = - (b) A = =q
4 8 8 f ( x ) = q + sin x r + sin x - 1 + sin x
p p p 3p
(c) A = , q = - (d) A = , q = r + sin x s + sin x s - q + sin x
5 8 6 8
1
Sol. (a, b, c, d) such that ò f ( x ) dx = - 2, the common difference of the AP
0
1 + sin 2 A cos 2 A 2 sin 4 q can be
2
Q sin A 1 + cos 2 A 2sin 4 q = 0 (a) -1 (b) 1/2 (c) 1 (d) 2
sin 2 A 2
cos A 1 + 2sin 4 q Sol. (a, c)
p + sin x q + sin x p - r + sin x
Applying R 2 ® R 2 - R 1 and R 3 ® R 3 - R 1, then 1
Q f (x ) = 2q + 2 sin x 2r + 2 sin x - 2 + 2 sin x
1 + sin 2 A cos 2 A 2 sin 4 q 2
r + sin x s + sin x s - q + sin x
-1 1 0 =0
-1 0 1 Applying R 2 ® R 2 - ( R 1 + R 3 ), then
p + sin x q + sin x p - r + sin x
Applying C 1 ® C 1 + C 2 , then
M
2 cos 2 A 2 sin 4 q 1
f (x ) = 0 K 0 K -2
M 2
M
0 1 0 =0
K K r + sin x s + sin x s - q +sin x
M
[Q2q = p + r , 2r = q + s and p + s = q + r ]
-1 0 1
( - 2) p + sin x q + sin x
Þ 1( 2 + 2 sin 4 q ) = 0 =-
2 r + sin x s + sin x
\ sin 4 q = - 1
554 Textbook of Algebra

Applying C 2 ® C 2 - C 1, then Sol. (a, b, c, d )


p + sin x D a2 -b
f (x ) = Here, D = = - a 2b 2 + b 2 = -(a 2 - 1)b 2
p + 2D + sin x D b - b2
[where D = common difference] If D = 0, then a 2 = 1, b = 0
= D [ p + sin x - p - 2D - sin x ] = - 2D 2 a2 - b - b
Now, D1 =
1 2 + 4b - b 2
and ò0 f ( x )dx = - 4
1-b -b
1 For a 2 = 1, D1 = = b ( b + 1) ( b + 2)
ò0 ( - 2 D
2
Þ )dx = - 4 Þ - 2 D 2 = - 2 2 + 4b - b 2
\ D 2 =1 Þ D = ± 1 a2 a2 - b
and D2 =
b 2 + 4b
l Ex. 15 If the system of equations a 2 x - by = a 2 - b and 1 1-b
For a 2 = 1, D 2 = = ( b + 1) ( b + 2)
bx - b 2 y = 2 + 4b possess an infinite number of solutions, b 2 + 4b
the possible values of a and b are For infinite number of solutions, D = D1 = D 2 = 0
(a) a = 1, b = - 1 (b) a = 1, b = - 2
\ a 2 = 1, b = - 1, - 2 Þ a = ± 1, b = - 1, b = - 2
(c) a = - 1, b = - 1 (d) a = - 1, b = - 2

JEE Type Solved Examples :


Passage Based Questions
n
This section contains 2 solved passages based upon each 8 a 6
of the passage 3 multiple choice examples have to be D1 = 5 2 b = 8(6 - b ) - a (15 - 4b ) + 6(5 - 8)
answered. Each of these examples has four choices (a), (b), 4 1 3
(c) and (d) out of which ONLY ONE is correct.
= 4ab - 15a - 8b + 30 = (a - 2) ( 4b - 15)
Passage I 2 8 6
(Ex. Nos. 16 to 18)
D2 = 1 5 b = 0 [\ R1 = 2R 3 ]
Consider the system of equations 1 4 3
2 x + ay + 6z = 8; x + 2y + bz = 5
2 a 8
x + y + 3z = 4
D 3 = 1 2 5 = 2(8 - 5) - a( 4 - 5) + 8(1 - 2)
16. The system has unique solution, if 1 1 4
(a) a = 2, b = 3 (b) a = 2, b ¹ 3
(c) a ¹ 2, b = 3 (d) a ¹ 2, b ¹ 3 = 6 +a - 8 = a - 2

17. The system has infinite solutions, if 16. (d) The system has unique solution, if
(a) a = 2, b Î R (b) a = 3, b Î R D ¹0
(c) a Î R , b = 2 (d) a Î R , b = 3 Þ ( a - 2) ( b - 3) ¹ 0
18. The system has no solution, if Þ a ¹ 2, b ¹ 3
(a) a = 2, b = 3 (b) a = 2, b ¹ 3 17. (a) The system has infinite solution, if
(c) a ¹ 2, b = 3 (d) a ¹ 2, b ¹ 3 D = D1 = D 2 = D 3 = 0
2 a 6 Þ a -2 = 0
Sol. D = 1 2 b = 2(6 - b ) - a (3 - b ) + 6(1 - 2) or a = 2, b Î R
1 1 3 18. (c) The system has no solution, if
= ab - 3a - 2b + 6 = (a - 2)(b - 3) D = 0 and atleast one of D1, D 2 and D 3 is non-zero.
Þ a ¹ 2, b = 3
Chap 07 Determinants 555

Passage II Taking x , x 2 and x 3 common from R 1, R 2 and R 3 , then


(Ex. Nos. 19 to 20) 1 ( x - 1) ( x - 1)( x - 2)
x2 x3 f (x ) = x × x 2 × x 3
1 ( x 2 - 1) ( x 2 - 1)( x 2 - 2)
Let C i , C i and C i ( i = 1, 2, 3) be Binomial coefficients,
x

where x Î N 1 ( x 3 - 1) ( x 3 - 1)( x 3 - 2)
x x x
C1 C2 C3 1 1 x -2
x2 x2 x2
and f ( x ) =12 C1 C2 C 3 , then 6
= x ( x - 1) 2
1 x +1 ( x + 1)( x 2 - 2)
x3 x 3C x3
C1 2 C3 1 x + x + 1 ( x + x + 1)( x 3 - 2)
2 2

19. f ( x ) is a polynomial of degree Applying R 1 ® R 1 - R 2 and R 3 ® R 3 - R 2 , then

(a) 6 (b) 10 0 -x x (3 - x - x 2 )
(c) 14 (d) 18 M
f ( x ) = x 6 ( x - 1)2 1 L x + 1 L ( x + 1)( x 2 - 2)
20. If f ( x ) = ( x -1) m x n ( x +1) p , where m, n, p Î N , then the M
value of å mn is 0 x2 x 2 ( x 2 + x 3 - 3)
(a) 32 (b) 43 Expanding along C 1, then
(c) 44 (d) 56 - x x (3 - x - x 2 )
Sol.
= - x 6 ( x - 1) 2
x 2 x 2 ( x 2 + x 3 - 3)
x x x
C1 C2 C3
2 2
x2 - 1 3 -x - x2
Q f ( x ) =12 x
C1 x
C2 C3 = - x 9 ( x - 1) 2
x3 x3 x3
1 x 2 + x 3 -3
C1 C2 C3
= - x 9 ( x - 1) 2 ( - x 2 - x 3 + 3 - 3 + x + x 2 )
x ( x - 1) x ( x - 1)( x - 2)
x = x 10 ( x - 1)2 ( x 2 - 1) = x 10 ( x - 1)3 ( x + 1)
2 6
x 2( x 2 - 1) x 2 ( x 2 - 1)( x 2 - 2) 19. (c) f ( x ) is a polynomial of degree 14.
= 12 x 2
2 6
20. (b) Here, m = 3, n = 10 and p =1
x 3 ( x 3 - 1) x 3 ( x 3 - 1)( x 3 - 2)
x3
2 6 \ å mn = mn + np + pm = 30 + 10 + 3 = 43

JEE Type Solved Examples :


Single Integer Answer Type Questions
n
This section contains 2 examples. The answer to each l Ex. 22 If P , Q and R are the angles of a triangle, the value
example is a single digit integer ranging from 0 to 9 tan P 1 1
(both inclusive).
of 1 tan Q 1 is
r r -1 1 1 tan R
l Ex. 21 If D r = , where r is a natural number,
r -1 r tan P 1 1
1024
Sol. (2) 1 tan Q 1 = tan P (tan Q tan R - 1)
the value of 10 å D r is
r =1 1 1 tan R
Sol. (4) Q D r = r 2 - ( r - 1) 2 - 1(tan R - 1) + 1(1 - tan Q )
1024 = tan P tan Q tan R - (tan P + tan Q + tan R ) + 2
\ å D r = (1024 )2 - (1 - 1)2 = (1024 )2 = 220 = 0 +2
r =1

1024
[Q In DPQR, tan P + tan Q + tan R = tan P tan Q tan R ]
Þ 10 å D r = 22 = 4 =2
r =1
556 Textbook of Algebra

JEE Type Solved Examples :


Matching Type Questions
n This section contains 2 eamples. Example 23 have four l Ex. 24 Suppose a , b and c are distinct and x , y and z are
statements (A, B, C and D) given in Column I and four
statement (p, q, r and s) in Column II and example 24
connected by the system of equations x + ay + a 2 z = a 3 ,
have three statements (A, B and C) given in Column I x + by + b 2 z = b 3 and x + cy + c 2 z = c 3 .
and four statements (p, q, r and s) in Column II. Any
given statement in Column I can have correct matching Column I Column II
with one or more statements(s) given in Column II.
(A) For x = 1, y = 2 and z =3, (a + b + c ) - (ab + bc + ca ) is (p) 3
l Ex. 23 Let f ( x ) denotes the determinant divisible by

x2 2x 1 + x 2 (B) For x = 4, y = 3 and z = 2,(ab + bc + ca )abc is (q) 6


f ( x ) = x 2 +1 x +1 1 . divisible by

x -1 x -1 (C) For x = 6, y = 4 and z =2, (abc )a + b + c is divisible by (r) 9

On expansion f ( x ) is seen to be a 4th degree polynomial (s) 12


given by f ( x ) = a 0 x 4 + a 1 x 3 + a 2 x 2 + a 3 x + a 4 .
Sol. (A) ® (p, r) (B) ® ( p, r); (C) ® ( p, q, r, s)
Using differentiation of determinant or otherwise match the
entries in Column I with one or more entries of the elements a¹b¹c
of Column II. 1 a a2
Column I Column II D = 1 b b 2 = (a - b ) (b - c )(c - a )
(A) a 02 + a1 is divisible by (p) 2 1 c c2
(B) a 22 + a 4 is divisible by (q) 3 a3 a a2 a a2 a3 1 a a2
3 2 2 3
(C) a 02 + a 2 is divisible by (r) 4 D1 = b b b = b b b =abc 1 b b 2
3 2 2
c c c c c c3 1 c c2
(D) a 42 + a 32 + a12 is divisible by (s) 5
= abc (a - b ) (b - c ) (c - a )
Sol. (A) ® (p, s); (B) ® (p, r); (C) ® (p, q); (D) ® (q)
1 a3 a2 1 a2 a3
2 2
x 2x 1+ x
D 2 = 1 b3 b2 = - 1 b2 b3
Q f (x ) = x 2 + 1 x + 1 1
1 c3 c2 1 c2 c 3
x -1 x -1
= - (a - b ) (b - c ) (c - a ) (ab + bc + ca )
Applying C 1 ® C 1 - C 3 , then
-1 2x 1 +x2 1 a a3
f (x ) = x 2 x +1 1 and D 3 = 1 b b 3 = (a - b ) (b - c ) (c - a ) (a + b + c )
1 -1 x -1 1 c c3

Expanding along R1, then By Cramer’s rule, we get


f ( x ) = - ( x 2 - 1 + 1) - 2x ( x 3 - x 2 - 1) D
x = 1 = abc
+ (1 + x 2 )( - x 2 - x - 1) D
= - 3x 4 + x 3 - 3x 2 + x - 1 …(i) D2 D
y= = - (ab + bc + ca ), z = 3 = a + b + c
According to the question, we get D D
f ( x ) = a 0 x 4 + a1x 3 + a 2 x 2 + a 3 x + a 4 …(ii) (A) (a + b + c )- (ab + bc + ca ) = z y = 32 = 9, which is divisible
From Eqs. (i) and (ii), we get by 3 and 9.
a 0 = - 3,a1 = 1, a 2 = - 3, a 3 = 1, a 4 = - 1 (B) (ab + bc + ca )abc = ( - y )x = ( - 3)4 = 81, which is
(A) a 02 + a1 = ( - 3)2 + 1 = 9 + 1 = 10 = 2 ´ 5 divisible by 3 and 9.
(B) a 22 + a 4 = ( - 3)2 - 1 = 9 - 1 = 8 = 2 ´ 4 (C) (abc )a + b + c = x 3 = 62 = 36, which is divisible by 3, 6, 9
(C) a 02 + a 2 = ( - 3)2 - 3 = 9 - 3 = 6 = 2 ´ 3 and 12.
(D) a 42 + a 32 + a12 = ( - 1)2 + (1)2 + (1)2 = 1 + 1 + 1 = 3
Chap 07 Determinants 557

JEE Type Solved Examples :


Statement I and II Type Questions
n +1
n Directions Example numbers 25 and 26 are
Sol. (d) Q Õ Dr = D 2 × D 3 × D 4 K Dn + 1
Assertion-Reason type examples. Each of these examples r =2
contains two statements: 1 n! 6 2 n! 6
Statement-1 (Assertion) and Statement-2 (Reason)
= 1 (n !)2 4n - 2 ´ 4 ( n ! ) 2 4n - 2
Each of these examples also has four alternative choices,
1 (n !)3 3n 3 - 2n 8 (n ! ) 3 3n 2 - 2n
only one of which is the correct answer. You have to select
the correct choice as given below. n n! 6
(a) Statement-1 is true, Statement-2 is true; Statement-2 ´ K ´ n 2 (n ! )2 4n - 2 ¹ 0
is a correct explanation for Statement-1 n 3 (n ! )3 (3n 2 - 2n )
(b) Statement-1 is true, Statement-2 is true; Statement-2 \ Statement-1 is false and Statement-2 is true.
is not a correct explanation for Statement-1
(c) Statement-1 is true, Statement-2 is false l Ex. 26 Consider the determinant
(d) Statement-1 is false, Statement-2 is true
0 x 2 - a x3 - b
2
f (x ) = x + a 0 x 2 +c .
l Ex. 25 Statement-1 Let
x 4 + b x -c 0
(r -1) n! 6 n +1
Statement-1 f ( x ) = 0 has one root x = 0.
D r = (r -1) 2 (n ! ) 2 4n - 2 , then Õ D r = 0.
r =2 Statement-2 The value of skew-symmetric determinant of
(r -1) 3 (n ! ) 3 3n 2 - 2n
odd order is always zero.
n +1 Sol. (a) For x = 0, the determinant reduces to the determinant
Statement-2 Õ D r = D 2 × D 3 × D 4 K D n + 1 of a skew-symmetric of odd order which is always zero.
r =2 Hence, x = 0 is the solution of given equation f ( x ) = 0.

Subjective Type Examples


n
In this section, there are 20 subjective solved examples. 1 +a 1 1
Sol. Let D= 1 1+b 1
l Ex. 27 A determinant of second order is made with the 1 1 1+c
elements 0 and 1. Find the number of determinants with
non-negative values. Since, the answer contain abc, then taking a, b and c
common from R1, R 2 and R 3 respectively, then
Sol. The number of determinants that can be made with 0 and 1
1 1 1
= 2 ´ 2 ´ 2 ´ 2 = 16 +1
a a a
and there are only three determinants of second order with 1 1 1
negative values D = abc +1
b b b
1 1 0 1 0 1 1 1 1
i.e., , , +1
1 0 1 1 1 0 c c c
Therefore, number of determinants with non-negative æ 1 1 1ö
But answer also contains ç1 + + + ÷ ,
values = 16 - 3 = 13 è a b cø

1 +a 1 1 then applying R1 ® R1 + R 2 + R 3
\ D =abc
l Ex. 28 Prove that 1 1+b 1
1 1 1 1 1 1 1 1 1
1 1 1+c 1+ + + 1+ + + 1+ + +
a b c a b c a b c
æ 1 1 1ö 1 1 1
= abc ç1 + + + ÷ , hence find the value of the +1
è a b cø b b b
1 1 1
determinant, if a , b and c are the roots of the equation +1
px 3 - qx 2 + rx - s = 0. c c c
558 Textbook of Algebra

æ 1 1 1ö Applying C 1 ® C 1 + C 3 , then
Taking ç1 + + + ÷ common from R 1, then
è a b cø p p 1
= log R q q 1 = 0 [since C 1 and C 2 are identical]
1 1 1
r r 1
æ 1 1 1ö 1 1
+1
1
D = abc ç1 + + + ÷
è a b cø b b b = RHS
1 1 1
+1
c c c l Ex. 30 Prove that
Applying C 2 ® C 2 - C 1, then - 2a a + b a + c
1 0 1 b + a - 2b b + c = 4 (b + c ) (c + a ) (a + b ).
æ 1 1 1ö 1 1 c + a c + b - 2c
D = abc ç1 + + + ÷ 1
è a b cø b b
1 1 - 2a a+b a+c
0 +1
c c Sol. Let D = b + a - 2b b +c
Expanding along C 2 , then c +a c + b - 2c
1 1
æ 1 1 1ö
D = abc ç1 + + + ÷ 1 1 On putting a + b = 0, b = - a
è a b cø +1
c c - 2a 0 a +c
æ 1 1 1ö D= c -a
Hence,D = abc ç1 + + + ÷ Then, 0 2a
è a b cø
c + a c - a - 2c
æ 1 1 1ö
2nd Part D = abc ç1 + + + ÷ = abc + bc + ca + ab Expanding along R1, then
è a b cø
s r æs + r ö D = - 2a { - 4ac - (c - a )2 } - 0 + (a + c ) {0 - 2a (c + a )}
= + =ç ÷
p p è p ø = 2a(c + a )2 - 2a(c + a )2 = 0
Hence, (a + b ) is a factor of D, similarly (b + c ) and (c + a )
are the factors of D.
l Ex. 29 If a , b and c are positive and are the pth, qth and
On expansion of determinant we can see that each term of
rth terms, respectively of a GP. Show without expanding that the determinant is a homogeneous expression in a, b and c
log a p 1 of degree 3 and also RHS is a homogeneous expression of
degree 3.
log b q 1 = 0.
Let D = k (a + b ) (b + c ) (c + a )
log c r 1 - 2a a + b a + c
Sol. Let A be the first term and R be the common ratio of GP, or b + a - 2b b + c = k (a + b ) (b + c ) (c + a )
then c +a c + b - 2c
a = p th term = AR p - 1
On putting a = 0, b = 1 and c =2, we get
b = q th term = AR q - 1 0 1 2
-1
c = r th term = ARr 1 - 2 3 = k ( 0 + 1) ( 1 + 2) ( 2 + 0)
\ log a = log A + ( p - 1) log R, 2 3 -4
log b = log A + (q - 1)log R and
log c = log A + (r - 1) log R Þ 0 - 1( - 4 - 6) + 2(3 + 4 ) = 6k
log a p 1 Þ 24 = 6k
\ k =4
\ LHS = log b q 1
- 2a a + b a + c
log c r 1
Hence, b + a - 2b b + c = 4(a + b ) (b + c ) (c + a )
log A + ( p - 1) log R p 1 c + a c + b - 2c
= log A + (q - 1)log R q 1
log A + (r - 1) log R r 1 l Ex. 31 If bc + qr = ca + rp = ab + pq = -1,
Applying C 1 ® C 1 - (log A )C 3 , then ap bp cr
( p - 1) log R p 1 ( p - 1) p 1 show that a b c = 0.
= (q - 1) log R q 1 = log R (q - 1) q 1 p q r
(r - 1) log R r 1 ( r - 1) r 1
Chap 07 Determinants 559

Sol. Given equations can be rewritten as and å a 5 = a 5 + b 5 = - 32 w10 - 32 w5


bc + qr + 1 = 0 …(i) = - 32( w + w2 ) = - 32 ( - 1) = 32
ca + rp + 1 = 0 …(ii)
åa åa 2 åa 3 2 - 4 - 16
ab + pq + 1 = 0 …(iii)
Let D = åa 2 åa 3 åa 4 = - 4 - 16 - 16
On multiplying Eqs. (i), (ii) and (iii) by ap , bq and cr
respectively, we get åa 3 åa 4 åa 5 - 16 - 16 32
(abc )p + ( pqr )a + ap = 0 1 -2 -8 1 -2 - 8
(abc )q + ( pqr )b + bq = 0 = 2( - 4 ) ( - 16) 1 4 4 = 128 1 4 4
(abc )r + ( pqr )c + cr = 0 1 1 -2 1 1 -2
These equations are consistent, given equations three but
abc and pqr are two. Applying R 2 ® R 2 - R 1 and R 3 ® R 3 - R 1, then
p a ap 1 -2 - 8
Hence, q b bq = 0 D = 128 0 6 12
r c cr 0 3 6

p q r Expanding along C 1, we get


Þ a b c =0 6 12
D = 128 × 1 × = 128(36 - 36) = 0
ap bq cr 3 6
[interchanging rows into columns]
ap bq cr
l Ex. 33 If a 2 + b 2 + c 2 =1, prove that
Þ ( - 1) a b c =0 [ R1 « R 3 ] a 2 + (b 2 + c 2 ) cos f ab (1 - cos f)
p q r ba (1 - cos f) b 2 + (c 2 + a 2 ) cos f
ap bq cr ca (1 - cos f) cb (1 - cos f)
Hence, a b c =0 ac (1 - cos f)
p q r bc (1 - cos f)
c + (a 2 + b 2 ) cos f
2
l Ex. 32 If a and b are the roots of the equations
is independent of a , b and c.
åa åa 2 åa3 Sol. Let
2
x - 2 x + 4 = 0, find the value of å a 2 åa3 åa 4 . a 2 + (b 2 + c 2 )cos f ab(1 - cos f )
åa3 åa 4
åa 5
D= ba(1 - cos f ) b 2 + (c 2 + a 2 )cos f
ca(1 - cos f ) cb (1 - cos f )
Sol. Given, x 2 - 2x + 4 = 0
ac (1 - cos f )
\ x =1 ± i 3
bc (1 - cos f )
\ a =1 + i 3
c 2 + (a 2 + b 2 )cos f
and b =1 - i 3
On multiplying C 1, C 2 and C 3 by a, b and c respectively and
taking a, b and c common from R 1, R 2 and R 3 respectively,
æ -1 - i 3 ö æ -1 + i 3 ö we get
Þ a = - 2ç ÷ and b = - 2ç ÷
è 2 ø è 2 ø a 2 + (b 2 + c 2 ) cos f b 2 (1 - cos f )
abc 2
a = -2 w2 and b = - 2 w, where w is the cube root of unity. D= a (1 - cos f ) b + (c 2 + a 2 )cos f
2
abc
a 2 (1 - cos f ) b 2 (1 - cos f )
å a = a + b = - 2( w + w)2 = - 2 ( - 1) = 2
c 2 (1 - cos f )
å a 2 = a 2 + b 2 = 4 w4 + 4 w2 = 4( w + w)2 = 4 ( - 1) = - 4
3 3 3 6 3
c 2 (1 - cos f )
å a = a + b = - 8 w - 8 w = - 8 - 8 = - 16
c 2 + (a 2 + b 2 )cos f
å a 4 = a 4 + b 4 = 16 w8 + 16 w4 = 16( w2 + w)
Applying C 1 ® C 1 + C 2 + C 3 , then
= 16( - 1) = - 16
560 Textbook of Algebra

a2 + b2 +c 2 b 2 1 - cos f ) c 2 (1 - cos f ) = xa 3 + ( x 2 + a 2 x + ax + a 3 - x 2 )
D = a 2 + b 2 + c 2 b 2 + ( c 2 + a 2 ) cos f c 2(1 - cos f ) = xa 3 + a 2 x + ax + a 3 = a 3 + x (a 3 + a 2 + a )
2 2 2 2
a + b +c b (1 - cos f ) c + (a 2 + b 2 )cos f
2
x × a(a 3 - 1) é x ( a 3 - 1) ù
=a3 + = a3 ê1 + 2 ú = RHS
2 2 2
Taking a + b + c common from C 1, then ( a - 1) ë a ( a - 1) û
D = (a 2 + b 2 + c 2 )
bc - a 2 ca - b 2 ab - c 2
1 b 2 (1 - cos f ) c 2 (1 - cos f )
1 b 2 + (c 2 + a 2 )cos f c 2 (1 - cos f )
l Ex. 35 (i) Prove that ca - b 2 ab - c 2 bc - a 2
1 2
b (1 - cos f ) c + (a 2 + b 2 ) cos f
2 ab - c 2 bc - a 2 ca - b 2

Applying R 1 ® R 2 - R 1 and R 3 ® R 3 - R 1, then a2 b2 b2


D =1 = b2 a2 b2 ,
1 b 2 (1 - cos f ) c 2 (1 - cos f ) b2 b2 a2
2 2 2
0 (a + b + c ) cos f 0
where a 2 = a 2 + b 2 + c 2 and b 2 = ab + bc + ca.
0 0 (a + b + c 2 ) cos f
2 2

bc - a 2 ca - b 2 ab - c 2
2 2 2 2 2
= (a + b + c ) cos f (ii) Prove that ca - b 2 ab - c 2 bc - a 2 is divisible
[by property, since all elements zero below leading ab - c 2 bc - a 2 ca - b 2
diagonal]
= 12 × cos f = cos 2 f [Qa 2 + b 2 + c 2 = 1] by (a + b + c ) 2 . Find the quotient.
which is independent of a, b and c. bc - a 2 ca - b 2 ab - c 2
(iii) Prove that ca - b 2 ab - c 2 bc - a 2
l Ex. 34 If a ¹ 0 and a ¹1, show that ab - c 2 bc - a 2 ca - b 2
x +1 x x
é x (a 3 - 1) ù a2 c2 2ac - b 2
x x +a x = a 3 ê1 + ú.
2 ë a 2 (a - 1) û = 2ab - c 2 b2 a2 .
x x x +a
b2 2bc - a 2 c2
Sol. Let
x +1 x x x +1 x x 2bc - a 2 c2 b2
LHS = D = x x +a x = x + 0 x +a x (iv) Prove that c2 2ca - b 2 a2
x x x +a2 x +0 x x +a2 b2 a2 2ab - c 2
x x x 1 x x = (a 3 + b 3 + c 3
- 3abc ) 2 .
= x x +a x + 0 x +a x
a b c
2
x x x +a 0 x x +a2
Sol. (i) Let D = b c a
c a b
\ Determinant of cofactors of D is
Applying R 2 ® R 2 - R1 and R 3 ® R 3 - R 1 in first
determinant, then bc - a 2 ca - b 2 ab - c 2
x x 1 xx x Dc = ca - b 2 ab - c 2 bc - a 2 = D3 - 1 = D2
D= 0 a 0 + 0 x +a x ab - c 2 bc - a 2 ca - b 2
0 0 a2 0 x x +a2 a b c
2

Expanding first determinant by property, since all elements = b c a ...(i)


below leading diagonal are zero and expanding second c a b
determinant along C 1, then
x +a x a b c a b c
D = x ×a ×a2 + 1 × = b c a ´ b c a
x x +a2
c a b c a b
= xa 3 + {( x + a ) ( x + a 2 ) - x 2 }
Chap 07 Determinants 561

a 2 + b 2 + c 2 ab + bc + ca ab + bc + ca 2bc - a 2 c2 b2
= ab + bc + ca a 2 + b 2 + c 2 ab + bc + ca [row by row] (iv) LHS = c2 2ca - b 2 a2
ab + bc + ca ab + bc + ca a 2 + b 2 + c 2 b 2
a 2
2ab - c 2

a 2 b2 b2 a b c -a c b
= b2 a 2 b2 = b c a ´ - b a c [row by row]
b2 b2 a2 c a b -c b a
2
bc - a 2 ca - b 2 ab - c 2 a2 b2 b2 a b c a b c a b c
Hence, ca - b 2 ab - c 2 bc - a 2 = b 2 a2 b2 = b c a ´ b c a = b c a
2 2 2 2 2
ab - c bc - a ca - b b b a2 c a b c a b c a b

(ii) From Eq. (i), we get = (a + b + c )2 (a 2 + b 2 + c 2 - ab - bc - ca )2 [from Eq. (ii)]


2 2 2 2
bc - a ca - b ab - c a b c 3 3 3
= [ - (a + b + c - 3abc )] 2

ca - b 2 ab - c 2 2
bc - a = b c a = (a 3 + b 3 + c 3
- 3abc )2 = RHS
2
ab - c bc - a 2 ca - b 2 c a b
= (a 3 + b 3 + c 3 - 3abc )2
l Ex. 36 Let a and b be the roots of the equation
= (a + b + c )2 (a 2 + b 2 + c 2 - ab - bc - ca )2 ax 2 + bx + c = 0. Let Sn = a n + bn for n ³1. Evaluate the
bc - a 2 ca - b 2 ab - c 2 3 1 + S1 1 + S 2
Therefore, ca - b 2 ab - c 2 bc - a 2 is divisible by determinant 1 + S1 1 + S 2 1 + S3 .
ab - c 2 bc - a 2 ca - b 2 1 + S 2 1 + S3 1 + S 4
(a + b + c )2 . Sol. Since, a and b are the roots of the equation
Hence, the quotient is (a 2 + b 2 + c 2
- ab - bc - ca )2 . ax 2 + bx + c = 0.
(iii) From Eq. (i), we get b c D
2
\ a + b = - , ab = and a -b =
bc - a 2 ca - b 2 ab - c 2 a b c a a a
ca - b 2 ab - c 2 bc - a 2 = b c a 3 1 + S1 1 + S2
ab - c 2 bc - a 2 ca - b 2 c a b Let D = 1 + S1 1 + S 2 1 + S3
2 1 + S2 1 + S3 1 + S4
a b c a b c a b c
Let D = b c a = b c a ´ b c a 3 1+a +b 1 + a 2 + b2
c a b c a b c a b = 1 + a + b 1 + a 2 + b2 1 + a 3 + b3
a b c a -c b 1 + a 2 + b2 1 + a 3 + b3 1 + a 4 + b4
= b c a ´ b -a c 1 1 1 1 1 1
c a b c -b a = 1 a b ´ 1 a b = D1 ´ D1 [say]
2 2 2 2
a 2
c 2
2ac - b 2 1 a b 1 a b
2
= 2ab - c b2 a2 [row by row] \ D= D21 …(i)
b2 2bc - a 2 c2 1 1 1
bc - a 2
ca - b 2
ab - c 2 \ D1 = 1 a b
Hence, ca - b 2 ab - c 2 bc - a 2 1 a 2 b2
ab - c 2 bc - a 2 ca - b 2 Applying C 2 ® C 2 - C 1 and C 3 ® C 3 - C 1, then
1 0 0
a2 c2 2ac - b 2
2 2 D1 = 1 a - 1 b - 1
= 2ab - c b a2
1 a 2 -1 b2 -1
b2 2bc - a 2 c2
562 Textbook of Algebra

Expanding along R1, then -a cos C cos B


1
a -1 b -1 1 1 = a cos C -1 cos A
D1 = 2 2 = ( a - 1) ( b - 1) a
a -1 b - 1 a +1 b +1 a cos B cos A -1
= {ab - (a + b ) + 1} ( b - a ) Applying C 1 ® C 1 + bC 2 + cC 3 , then
\ D = D21 = [ab - (a + b ) + 1]2 ( b - a )2 0 cos C cos B
2 1
2 2
æ c b ö D (a + b + c ) (b - 4ac ) = 0 -1 cos A = 0 = RHS
= ç + + 1÷ × 2 = a
èa a ø a a4 0 cos A -1

l Ex. 37 If A, B and C are the angles of a triangle, show l Ex. 38 Without expanding at any stage, evaluate the
that value of the determinant
sin 2 A sin C sin B 2 tan A cot B + cot A tan B
(i) sin C sin 2 B sin A = 0. tan B cot A + cot B tan A 2
sin B sin A sin 2C tan C cot A + cot C tan A tan C cot B + cot C tan B

- 1 + cos B cos C + cos B cos B tan A cot C + cot A tan C


(ii) cos C + cos A -1 + cos A cos A = 0. tan B cot C + cot B tan C .
-1 + cos B - 1 + cos A -1 2

sin 2A sin C sin B Sol. The given determinant can be written as the product of
two determinants
Sol. (i) LHS = sin C sin 2B sin A
tan A cot A 0 cot A tan A 0
sin B sin A sin 2C
tan B cot B 0 ´ cot B tan B 0 = 0 ´ 0 = 0
2ka cos A kc kb tan C cot C 0 cot C tan C 0
= kc 2kb cos B ka [from sine rule]
kb ka 2kc cos C l Ex. 39 Suppose that digit numbers A 28, 3B 9 and 62C,
2a cos A c b where A, B and C are integers between 0 and 9 are divisible
=k 3
c 2b cos B a A 3 6
b a 2c cos C by a fixed integer k, prove that the determinant 8 9 C
a cos A + a cos A a cos B + b cos A 2 B 2
= k 3 a cos B + b cos A b cos B + b cos B is also divisible by k.
a cos C + c cos A c cos B + b cos C
a cos C + c cos A Sol. Given, A 28, 3B 9 and 62C are divisible by k, then

b cos C + c cos B A 28 = 100A + 20 + 8 = n1k …(i)


c cos C + c cos C 3B 9 = 300 + 10B + 9 = n 2 k …(ii)
and 62C = 600 + 20 + C = n 3k …(iii)
cos A a 0 a cos A 0
where n1, n 2 , n 3 Î I (integers).
= k3 cos B b 0 ´ b cos B 0 = 0 ´ 0 = 0 = RHS
A 3 6
cos C c 0 c cos C 0
Let D= 8 9 C
- 1 + cos B cos C + cos B cos B
2 B 2
(ii) LHS = cos C + cos A - 1 + cos A cos A
- 1 + cos B - 1 + cos A -1 Applying R 2 ® R 2 + 10R 3 + 100 R 1, then
A 3 6
Applying C 1 ® C 1 - C 3 and C 2 ® C 2 - C 3 , then
A = 100A + 20 + 8 300 + 10B + 9 600 + 20 + C
-1 cos C cos B
2 B 2
= cos C -1 cos A
cos B cos A -1
Chap 07 Determinants 563

A 3 6 x + c1 x +a x +a
= n1k n 2 k n 3k [using Eqs. (i), (ii) and (iii)] Sol. Since, f ( x ) = x + b x + c2 x +a
2 B 2 x +b x +b x + c3
A 3 6 Applying C 2 ® C 2 - C 1 and C 3 ® C 3 - C 2 , then
= k n1 n 2 n 3 x + c1 a - c1 0
2 B 2 f (x ) = x + b c2 - b a - c2
Hence, D is divisible by k. x +b 0 c3 - b

sin x sin ( x + h ) sin ( x + 2h ) 1 a - c1 0 c1 a - c1 0


l Ex. 40 If D = sin ( x + 2h ) sin x sin ( x + h ) , f (x ) = x 1 c 2 - b a - c 2 + b c2 - b a -c2
1 0 c3 - b b 0 c3 - b
sin ( x + h ) sin ( x + 2h ) sin x
æ Dö So, f ( x ) is linear.
find lim ç ÷ .
h ® 0è h 2 ø Let f ( x ) = Px + Q
Then, f ( - a ) = - aP + Q , f ( - b ) = - bP + Q
Sol. Let a = sin x , b = sin ( x + h ) and c = sin ( x + 2h )
a b c a b c f ( 0) = 0 × P + Q = Q
bf ( - a ) - af ( - b )
D= c a b = b c a = (a 3 + b 3 + c 3 - 3abc ) = …(ii)
(b - a )
b c a c a b
From Eq. (i), we get
1
= (a + b + c ) [(a - b )2 + (b - c )2 + (c - a )2 ] c1 - a 0 0
2
æ hö h f ( - a) = b - a c2 - a 0
Now, a - b = sin x - sin ( x + h ) = - 2 cos ç x + ÷ sin
è 2ø 2 b-a b - a c3 - a
æ 3h ö h = (c 1 - a ) (c 2 - a ) (c 3 - a )
b - c = sin ( x + h ) - sin ( x + 2h ) = - 2 cos ç x + ÷ sin
è 2ø 2 Similarly, f ( - b ) = (c 1 - b ) (c 2 - b ) (c 3 - b )
and c - a = sin ( x + 2h ) - sin x = 2 cos ( x + h ) sin h
D 1 and g ( x ) = (c 1 - x ) (c 2 - x )(c 3 - x )
\ = (a + b + c ) g (a ) = f ( - a )
h2 2
éæa - b ö2 æb - c ö2 æc - a ö2 ù \ g (b ) = f ( - b )
êç ÷ +ç ÷ +ç ÷ ú Now, from Eq. (ii), we get
è h ø è h ø è h ø ú
ëê û bg (a ) - ag (b )
1 f ( 0) =
= [sin x + sin ( x + h ) + sin ( x + 2h )] ´ (b - a )
2
éæ æ hö hö
2
ê ç - 2 cos ç x + ÷ sin ÷ l Ex. 42 If f ( x ) is a polynomial of degree < 3, prove that
êç è 2ø 2÷
êç h ÷ 1 a f (a ) /( x - a ) 1 a a2
ê çè ÷
ø
êë 1 b f (b ) /( x - b ) ¸ 1 b b 2
2
æ æ 3h ö hö 1 c f (c ) /( x - c ) 1 c c2
ç - 2 cos ç x + ÷ sin ÷
2 ÷ æ 2 cos ( x + h ) sin h ö ù
2
è 2ø
+ç +ç ÷ ú f (x )
ç h ÷ è h ø ú = .
ç ÷ û ( x - a )( x - b )( x - c )
è ø
D 1 f (x ) A B C
\ lim = (3 sin x ) (cos 2 x + cos 2 x + 4 cos 2 x ) Sol. = + + [let]
h ® 0 h2 2 ( x - a) ( x - b) ( x - c ) ( x - a) ( x - b) ( x - c )
\ = 9 sin x cos 2 x …(i)
On comparing the various powers of x , we get
x + c1 x +a x +a ì f (a ) f (a )
l Ex. 41 If f ( x ) = x + b x +c2 x + a , show that ï A = (a - b ) (a - c ) = - (a - b ) (c - a )
ï
x +b x +b x + c3 ï f (b ) f (b )
Þ íB = =-
ï ( b - a ) ( b - c ) ( a - b ) (b - c )
bg (a ) - ag (b )
f ( x ) is linear in x. Hence, deduce that f (0 ) = , ïC = f ( c )
=-
f (c )
(b - a ) ïî (c - a ) (c - b ) (b - c ) (c - a )
where g ( x ) = (c 1 - x ) (c 2 - x ) (c 3 - x ).
564 Textbook of Algebra

Now, from Eq. (i), we get l Ex. 44 Let S be the sum of all possible determinants of
f (x ) order 2 having 0, 1, 2 and 3 as their elements. Find the
( x - a) ( x - b) ( x - c ) common root a of the equations
f (a ) f (b ) f (c ) x 2 + ax + [m + 1] = 0,
(c - b ) - (c - a ) + (b - a )
( x - a) (x - b) (x - c ) x 2 + bx + [m + 4 ] = 0
=
(a - b ) (b - c ) (c - a ) and x 2 - cx + [m +15 ] = 0,
1 a f (a )/( x - a ) such that a > S, where a + b + c = 0 and
1 b f (b )/( x - b ) 1 2n r
m = lim å
n ® ¥ n r =1
1 c f (c )/( x - c ) (n + r 2 )
2
=
1 a a2 and [.] denotes the greatest integer function.
1 b b2 Sol. Let a be a common root of the given equations, then
1 c c2 a 2 + aa + [m + 1] = 0
Þ a 2 + aa + [m ] + 1 = 0 …(i)
l Ex. 43 If f (a , b ) =
f (b ) - f (a ) 2
and a + ba + [m + 4 ] = 0
b -a
Þ a 2 + ba + [m ] + 4 = 0 …(ii)
f (b, c ) - f (a , b )
f (a , b, c ) = , prove that 2
(c - a ) and a - c a + [m + 15] = 0

f (a ) f (b ) f (c ) 1 1 1 Þ a 2 - c a + [m ] + 15 = 0 …(iii)
f (a , b, c ) = 1 1 1 ¸ a b c . On adding Eqs. (i) and (ii) and subtracting Eq. (iii), we get
a b c a2 b2 c2 a 2 + (a + b + c ) a + [m ] - 10 = 0

f (b, c ) - f (a, b ) a 2 + 0 + [m ] - 10 = 0 [Qa + b + c = 0]


Sol. LHS = f (a,b, c ) =
(c - a ) Þ a 2 + [m ] - 10 = 0 …(iv)
f (c ) - f (b ) f (b ) - f (a ) 1 2n
r
- Also, m = lim å
(c - b ) (b - a ) n ®¥ n = n +r2
2
= r 1
(c - a ) 2n
1 r /n 2 x
=
(b - a ) { f (c ) - f (b )} - (c - b ) { f (b ) - f (a )} = lim å
n ® ¥ r = 1n
×
1 + (r /n ) 2
= ò0 (1 + x 2 )
dx
(b - a ) (c - b ) (c - a )
2
( f (a ) × (c - b ) - f (b ) × (c - a ) + f (c ) × (b - a )) [ (1 + x 2 )]0 = 5 - 1
=
(a - b ) (b - c ) (c - a ) Now, [m ] = [ 5 - 1] = 1
1 1 1 1 1 1 From Eq. (iv), we get
f (a ) - f (b ) + f (c )
b c a c a b a 2 + 1 - 10 = 0 Þ a 2 = 9
=
1 1 1 \ a = ±3
a b c Now, number of determinants of order 2 having
a2 b2 c 2 0, 1, 2, 3 = 4 ! = 24
f (a ) f (b ) f (c ) a1 a 2
1 1 1 Let D1 = be one such determinant and their exists
a3 a4
a b c another determinant.
=
1 1 1 a3 a4
Let D 2 = [obtained on interchanging R1 and R 2 ]
a b c a1 a 2
a2 b2 c 2
such that D1 + D 2 = 0
f (a ) f (b ) f (c ) 1 1 1 QS = Sum of all the 24 determinants = 0
= 1 1 1 ¸ a b c = RHS Since, a >S Þ a >0
a b c a2 b2 c 2 \ a =3
Chap 07 Determinants 565

l Ex. 45 If a 1 , a 2 , a 3 and b1 , b 2 , b 3 Î R and are such that which is true.


a i b j ¹1 for 1 £ i, j £ 3, Case II If a1 > a 2 > a 3 and b1 > b 2 > b 3
\ a1 - a 2 > 0, a 2 - a 3 > 0
1 - a 13 b13 1 - a 13 b 23 1 - a 13 b33
and b1 - b 2 > 0, b 2 - b 3 > 0
1 - a 1b1 1 - a 1b 2 1 - a 1b3
a1 - a 3 > 0 Þ a 3 - a1 < 0
1 - a 32 b13 1 - a 32 b 23 1 - a 32 b33
> 0 provided either and b1 - b 3 > 0 Þ b 3 - b1 < 0
1 - a 2 b1 1 - a 2b 2 1 - a 2 b3
Hence, (a1 - a 2 ) (a 2 - a 3 ) (a 3 - a1 ) < 0
1 - a 33 b13 1 - a 33 b 23 1 - a 33 b33 and (b1 - b 2 ) (b 2 - b 3 ) (b 3 - b1 ) < 0
1 - a 3 b1 1 - a 3b 2 1 - a 3 b3 \ (a1 - a 2 ) (a 2 - a 3 ) (a 3 - a1 ) (b1 - b 2 )
a 1 < a 2 < a 3 and b1 < b 2 < b 3 or a 1 > a 2 > a 3 and (b 2 - b 3 ) (b 3 - b1 ) > 0
b 1 > b 2 > b 3. which is true.

x 3 - y 3 ( x - y ) ( x 2 + xy + y 2 ) l Ex. 46 Show that a six-digit number abcdef is divisible by


Sol. Since, = = x 2 + xy +y 2
x -y (x - y ) 11, if and only if ab + cd + ef is divisible by 11. Hence or
otherwise, find one set of values of two-digit numbers x , y
Hence, the given determinant becomes
x 23 42
1 + a1b1 + a12 b12 1 + a1b 2 + a12 b 22 1 + a1 b 3 + a12 b 32
and z, so that the value of the determinant 13 37 y is
1 + a 2 b1 + a 22 b12 1 + a 2 b 2 + a 22 b 22 1 + a 2 b 3 + a 22 b 32 > 0
1 + a 3b1 + a 32 b12 1 + a 3b 2 + a 32 b 22 1 + a 3 b 3 + a 32 b 32
19 z 34
divisible by 99 (without expanding the determinant).
1 a1 a12 1 b1 a12 Sol. Since, abcdef = ab 0000 + cd 00 + ef
Þ 1 a 2 a 22 ´ 1 b 2 b 22 >0 = (9999 + 1) ab + (99 + 1) cd + ef
1 a 3 a 32 1 b 3 b 32 = 9999 ab + 99 cd + ab + cd + ef
Þ (a1 - a 2 )(a 2 - a 3 ) (a 3 - a1 ) (b1 - b 2 ) Given, abcdef is divisible by 11, if and only if ab + cd + ef is
(b 2 - b 3 )(b 3 - b1 ) > 0 divisible by 11. Now, let x = ab, y = cd and z = ef .
[each being a two-digit number]
ì 1 a a2 ü
ï ï x 23 42 ab 23 42
2
íQ 1 b b = (a - b ) (b - c ) (c - a )ý Again, let D = 13 37 y = 13 37 cd
ï 1 c c2 ï
î þ 19 z 34 19 ef 34
Case I If a1 < a 2 < a 3 and b1 < b 2 < b 3 , then Applying R 1 ® R1 + 100 R 2 + 10000 R 3 , we get
(a1 - a 2 ) < 0, (a 2 - a 3 ) < 0 1913ab ef 3723 34cd 42
and (b1 - b 2 ) < 0, (b 2 - b 3 ) < 0 D= 13 37 cd
(a1 - a 3 ) < 0
19 ef 34
and (b1 - b 3 ) < 0
\ (a 3 - a1 ) > 0 Now, 1913ab is divisible by 11, if and only if
\ (b 3 - b1 ) > 0 19 + 13 + ab = 32 + ab is divisible by 11 Þ ab = 01, 12, 23, ¼
Then, (a1 - a 2 ) (a 2 - a 3 ) (a 3 - a1 ) > 0 Again, 1913ab is divisible by 9, if and only if
and (b1 - b 2 ) (b 2 - b 3 ) (b 3 - b1 ) > 0 1 + 9 + 1 + 3 + a + b = 14 + a + b is divisibe by 9.
\ (a1 - a 2 ) (a 2 - a 3 ) (a 3 - a1 ) (b1 - b 2 ) The above two conditions are satisfied for a = 6, b = 7. Thus,
x = 67. Similarly, y = 23 and z = 39.
(b 2 - b 3 ) (b 3 - b1 ) > 0
#L Determinants Exercise 1 :
Single Option Correct Type Questions
n This section contains 30 multiple choice questions. 7. If f ( x ) = a + bx + cx 2 and a , b and g are the roots of the
Each question has four choices (a), (b), (c) and (d) out of a b c
which ONLY ONE is correct 3
equation x = 1, then b c a is equal to
3 1 + f (1) 1 + f (2)
c a b
1. If f (n ) = a n + b n and 1 + f (1) 1 + f (2) 1 + f (3)
(a) f ( a ) + f ( b ) + f ( g )
1 + f (2) 1 + f (3) 1 + f ( 4 ) (b) f ( a ) f ( b ) + f ( b ) f ( g ) + f ( g ) f ( a )
= k (1 - a ) 2 (1 - b ) 2 (a - b ) 2 , k 2 d is equal to (c) f ( a ) f ( b ) f ( g )
(a) 1 (b) - 1 (d) - f ( a ) f ( b ) f ( g )
(c) ab (d) abg cos 2x sin 2 x cos 4 x
x +a x +b x +a -c 8. When the determinant sin x 2
cos 2x cos 2 x is
2. Let D( x ) = x + b x + c x -1 and cos 4 x 2
cos x cos 2x
x +c x +d x -b +d
expanded in powers of sin x , the constant term in that
2
ò0 D( x ) dx = - 16, where a, b, c and d are in AP, then the expression is
(a) 1 (b) 0 (c) - 1 (d) 2
common difference of the AP is equal to
(a) ± 1 (b) ± 2 (c) ± 3 (d) ± 4 9. If [ ] denotes the greatest integer less than or equal to
the real number under consideration and
1+ x2
x x 3
- 1 £ x < 0, 0 £ y < 1, 1 £ z < 2, the value of the
3. If D( x ) = loge (1 + x 2 ) e x sin x , then
[x ] + 1 [y ] [z ]
cos x tan x sin 2 x
determinant [x ] [y ] + 1 [z ] is
(a) D( x ) is divisible by x (b) D( x ) = 0
[x ] [y ] [z ] + 1
(c) D ¢( x ) = 0 (d) None of these
(a) [ x ] (b) [y ]
4. If a, b and c are sides of a triangle and (c) [z ] (d) None of these
a2 b2 c2 y2 - xy x2
2 2 2
(a + 1) (b + 1) (c + 1) = 0, then 10. The determinant a b c is equal to
(a - 1) 2 (b - 1) 2 (c - 1) 2 a¢ b¢ c¢
(a) DABC is an equilateral triangle bx + ay cx + by a ¢ x + b ¢y bx + cy
(b) DABC is a right angled isosceles triangle (a) (b)
b ¢ x + a ¢y c ¢ x + b ¢y ax + by b ¢ x + c ¢y
(c) DABC is an isosceles triangle
(d) None of the above bx + cy ax + by ax + by bx + cy
(c) (d)
b ¢ x + c ¢y a ¢ x + b ¢y a ¢ x + b ¢y b ¢x + c ¢y
a x x x
x b x x 11. If A, B and C are angles of a triangle, the value of
5. If = f ( x ) - xf ¢( x ), then f ( x ) is equal to
x x g x e 2i A e - iC e - iB
x x x d e - iC e2i B
e - iA is (where i = - 1)
(a) ( x - a ) ( x - b ) ( x - g ) ( x - d) e - iB e - i BA e 2i C
(b) ( x + a ) ( x + b ) ( x + g ) ( x + d)
(c) 2( x - a ) ( x - b ) ( x - g ) ( x - d) (a) 1 (b) - 1 (c) - 2 (d) - 4
(d) None of the above n n+2 2n
x x x
a
a b -c c +b 12. If 1 x a = 0, " x Î R, where n Î N ,
n+5 a+6 2n + 5
6. If a + c b c - a = 0, the line ax + by + c = 0 x x x
a -b a +b c the value of a is
passes through the fixed point which is (a) n (b) n - 1
(a) (1, 2 ) (b) (1, 1 ) (c) ( - 2, 1 ) (d) (1, 0 ) (c) n + 1 (d) None of these
Chap 07 Determinants 567

13. If x , y and z are the integers in AP lying between 1 and 19. If f ( x ), g ( x ) and h( x ) are polynomials of degree 4 and
9 and x 51, y 41 and z 31 are three digits number, the f ( x ) g ( x ) h( x )
5 4 3 a b c = mx 4 + nx 3
+ rx 2 + sx + t be an
value of x 51 y 41 z 31 is p q r
x y z identity in x, then
(a) x + y + z f ¢¢¢(0) - f ¢¢(0) g ¢¢¢(0) - g ¢¢(0) h ¢¢¢(0) - h ¢¢(0)
(b) x - y + z a b c
(c) 0
(d) None of the above
p q r
is equal to
14. If a1 b1 c 1 , a 2 b 2 c 2 and a 3 b 3 c 3 are three digit even
(a) 2 (3n + r ) (b) 3 (2n - r )
c1 a1 b1 (c) 3(2n + r ) (d) 2(3n - r )
natural numbers and D = c 2 a2 b 2 , then D is cos ( x + a ) cos ( x + b ) cos ( x + g )
c 3 a3 b3 20. If f ( x ) = sin ( x + a ) sin ( x + b ) sin ( x + g ) , then
(a) divisible by 2 but not necessarily by 4 sin ( b - g ) sin ( g - a ) sin (a - b )
(b) divisible by 4 but not necessarily by 8
f (q ) - 2 f ( f) + f ( y ) is equal to
(c) divisible by 8
(a) 0 (b) a - b
(d) None of the above
(c) a + b + g (d) a + b - g
15. If a, b and c are sides of DABC such that 1 1 1
c b cos B + cb a cos A + ba + cg
21. If a b c = (a - b ) (b - c ) (c - a ) (a + b + c ), where
a c cos B + ab b cos A + ca + ag = 0
a3 b3 c3
b a cos B + bb c cos A + aa + bg
a, b and c are all different, then the determinant
æ + pö 1 1 1
ç where a , b, g Î R and ÐA, ÐB, ÐC ¹ ÷, DABC is
è 2ø ( x - a) 2
( x - b) 2
(x - c )2 vanishes
(a) an isosceles (b) an equilateral ( x - b) ( x - c ) ( x - c ) ( x - a) ( x - a) ( x - b)
(c) can’t say (d) None of these
when
16. If x 1 , x 2 and y 1 , y 2 are the roots of the equations (a) a + b + c = 0 (b) x =
1
(a + b + c )
2 2 3
3x - 18x + 9 = 0 and y - 4y + 2 = 0, the value of the
1
x 1x 2 y 1y 2 1 (c) x = (a + b + c ) (d) x = a + b + c
2
determinant x 1 + x 2 y1 + y 2 2 is
22. Leta, b, c Î R such that no two of them are equal and satisfy
sin ( p x 1 x 2 ) cos ( p / 2y 1y 2 ) 1
2a b c
(a) 0 (b) 1
(c) 2 (d) None of these
b c 2a = 0, the equation24ax 2 + 4bx + c = 0 has
10 10 11 c 2a b
C4 C5 Cm
11 11 12 é 1ù
17. If the value of D = C6 C7 C m + 2 is equal to (a) atleast one root in ê 0, ú
12 12 13 ë 2û
C8 C9 Cm + 4
é 1 1ù
(b) atleast one root in ê - , ú
zero, then m is equal to ë 2 2û
(a) 6 (b) 4 (c) atleast one root in [- 1, 0]
(c) 5 (d) None of these
(d) atleast two roots in [ 0, 2 ]
18. The value of the determinant
1 sin (a - b ) q cos (a - b ) q 23. The number of positive integral solution of the equation
3
a sin aq cos aq is independent of x +1 x 2y x 2z
2 3
a2 sin (a - b ) q cos (a - b ) q xy y +1 y 2z = 11 is
2 2 3
(a) a (b) b xz yz z +1
(c) q (d) a (a) 0 (b) 3 (c) 6 (d) 12
568 Textbook of Algebra

24. If f ( x ) = ax 2 + bx + c , a, b, c Î R and equation 27. Given, f ( x ) = log10 x and g ( x ) = e pix .


f ( x ) - x = 0 has imaginary roots a , b g and d be the roots f (x )× g(x ) ( f ( x )) g (x ) 1
2 2 2 g (x 2 )
2 a d If f( x ) = f ( x ) × g ( x ) ( f ( x )) 0 , the value of
of f ( f ( x )) - x = 0, then b 0 a is 3 3 3 g (x 3 )
f ( x ) × g ( x ) ( f ( x )) 1
g b 1 f (10), is
(a) 0 (b) purely real (a) 1 (b) 2 (c) 0 (d) None of these
(c) purely imaginary (d) None of these 28. The value of the determinant
1 (a 2 x - a - 2 x ) 2 (a 2 x + a - 2 x ) 2
25. If the system of equations 2x - y + z = 0, x - 2y + z = 0,
tx - y + 2z = 0 has infinitely many solutions and f ( x ) be 1 ( b 2x - b - 2x ) 2 ( b 2 x + b - 2 x ) 2 , is
a continuous function, such that f (5 + x ) + f ( x ) = 2, 1 ( g 2x - g - 2x ) 2 ( g 2x + g - 2x ) 2
- 2t
then ò f ( x ) dx is equal to (a) 0 (b) ( abg ) 2x (c) ( abg ) - 2x (d) None of these
0
(a) 0 (b) - 2t (c) 5 (d) t 29. If a, b and c are non-zero real numbers and if the
2 4 2 8 equations (a - 1)x = y + z, (b - 1) y = z + x ,
26. If (1 + ax + bx ) = a 0 + a1 x + a 2 x + ¼ + a8 x ,
(c - 1) z = x + y has a non-trivial solution, then
where a, b, a 0 , a1 , ¼ , a 8 Î R such that a 0 + a1 + a 2 ¹ 0 ab + bc + ca equals to
a 0 a1 a 2 (a) a + b + c (b) abc (c) 1 (d) None of these
and a1 a 2 a 0 = 0, then 30. The set of equations lx - y + (cos q ) z = 0,
a2 a0 a1 3x + y + 2z = 0, (cos q ) x + y + 2z = 0, 0 £ q £ 2p, has non-
3 5 1 5 trivial solution(s)
(a) a = , b = (b) a = , b = (a) for no value of l and q
4 8 4 32
(b) for all value of l and q
2
(c) a = 1, b = (d) None of these (c) for all values of l and only two values of q
3
(d) for only one value of l and all values of q

#L Determinants Exercise 2 :
More than One Correct Option Type Questions
n
This section contains 15 multiple choice questions. (a) 0 (b) independent of n
Each question has four choices (a), (b), (c) and (d) out of (c) independent of q (d) independent of x, y and z
which MORE THAN ONE may be correct. a b aa + b
a +x2 2
ab ac 34. The determinant b c ba + c is equal to
31. The determinant D = ab 2
b +x 2
bc is aa + b ba + c 0
ac bc c2 + x2 zero, if
divisible by (a) a, b and c are in AP
(a) x (b) x 2 (c) x 3 (d) x 4 (b) a, b, c, are in GP
32. The value of the determinant (c) a, b and c are in HP
6 2i 3+ 6 (d) (x - a) is a factor of ax 2 + 2bx + c

12 3 + 8i 3 2 + 6 i , is (where i = - 1) 2 cos x 1 0
18 2 + 12 i 27 + 2i 35. Let f ( x ) = 1 2 cos x 1 , then
(a) complex (b) real (c) irrational (d) rational 0 1 2 cos x
æpö
(a) f ç ÷ = - 1
è3ø
1
2k - 1 sin k q æpö
k (k + 1 ) (b) f ¢ç ÷ = 3
n è3ø
33. If Dk = x y z , then å D k p
k =1 (c) ò f ( x ) dx = 0
æn + 1ö n
sin ç ÷ q sin q 0
n è 2 ø 2 p
2n - 1 (d) ò f ( x ) dx = 0
n+1 q -p
sin
2
is equal to
Chap 07 Determinants 569

x 2 - 5x + 3
2x - 5 3 4x - 4 ( x - 2) 2 x 3

2
36. If D( x ) = 3x + x + 4 6x + 1 9 41. If D( x ) = 8x - 4 2 (x - 2 2)2 ( x + 1) 3 , then
7 x 2 - 6x + 9 14 x - 6 21 12x - 4 3 ( x - 2 3 ) 2 ( x - 1) 3
3
= ax + bx 2 + cx + d , then (a) term independent of x in D( x ) = 16(5 - 2 - 3 )
(a) a = 0 (b) b = 0 (c) c = 0 (d) d = 47 (b) coefficient of x in D( x ) = 48(1 + 2 - 3 )
37. If a, b and c are the sides of a triangle and A, B and C are (c) coefficient of x in D( x ) = 16 (5 + 2 - 3 )
the angles opposite to a, b and c respectively, then (d) coefficient of x in D( x ) is divisible by 16
a2 b sin A C sin A 42. If
D = b sin A 1 cos A is independent of 3 3x 3x 2
+ 2a 2
C sin A cos A 1 f (x ) = 3x 3x 2 + 2a 2 3x 3 + 6a 2 x ,
2 3 3
(a) a (b) b (c) c (d) A, B, C 3x + 2a 3x + 6a x 3x + 12a 2 x 2 + 2a 4
2 4

a 0a2 then
38. Let f (a, b ) = 1 (2a + b ) (a + b ) 2 , then (a) f ¢( x ) = 0
0 1 (2a + 3b ) (b) y = f ( x ) is a straight line parallel to X-axis
2
(a) (a + b ) is a factor of f (a, b ) (c) ò f ( x ) dx = 32 a 4
0
(b) (a + 2b ) is a factor of f (a, b )
(c) (2a + b ) is a factor of f (a,b ) (d) None of the above
(d) a is a factor of f (a, b ) 43. If a > b > c and the system of equations ax + by + cz = 0,
2
sec x 1 1 bx + cy + az = 0, cx + ay + bz = 0 has a non-trivial
39. If f ( x ) = cos x 2
cos 2 x 2
cosec x , then solution, then both the roots of the quadratic equation
2 at 2 + bt + c , are
1 cos x cot 2 x
(a) real
p /4 1
(a) ò f ( x ) dx = (3 p + 8 ) (b) of opposite sign
- p /4 16 (c) positive
æpö (d) complex
(b) f ¢ç ÷ = 0
è2ø
44. The values of l and b for which the equations
(c) maximum value of f ( x ) is 1
x + y + z = 3, x + 3y + 2z = 6 and x + ly + 3z = b have
(d) minimum value of f ( x ) is 0
(a) a unique solution, if l ¹ 5, b Î R
a a + x2 a + x2 + x4 (b) no solution, if l ¹ 5, b = 9
40. If 2a 3a + 2x 2
4a + 3x 2 + 2x 4 (c) infinite many solution, l = 5, b = 9
(d) None of the above
3a 6a + 3x 2 10a + 6x 2 + 3x 4
45. Let l and a be real. Let S denote the set of all values of l
= a 0 + a1 x + a 2 x 2 + a 3 x 3 + a 4 x 4
+ a5 x 5 for which the system of linear equations
6 7
+ a 6 x + a 7 x and l x + (sin a ) y + (cos a ) z = 0
f ( x ) = + a 0 x 2 + a 3 x + a 6 , then x + (cos a ) y + (sin a ) z = 0
(a) f ( x ) ³ 0, " x Î R if a > 0 - x + (sin a ) y - (cos a ) z = 0
(b) f ( x ) = 0, only if a = 0 has a non-trivial solution, then S contains
(c) f ( x ) = 0, has two equal roots (a) ( - 1, 1 ) (b) [ - 2, - 1 ]
(d) f ( x ) = 0, has more than two root if a = 0 (c) [1, 2 ] (d) ( - 2, 2 )
570 Textbook of Algebra

#L Determinants Exercise 3 :
Passage Based Questions
n This section contains 7 passages. Based upon each of the 51. If a 3 + b 3 + c 3 - 3abc = -3 and A = bc - a 2 , B = ca - b 2
passage 3 multiple choice questions have to be and C = ab - c 2 , then the value of aA + bB + cC , is
answered. Each of these questions has four choices (a), (b), (a) -3 (b) 3 (c) -9 (d) 9
(c) and (d) out of which ONLY ONE is correct.
Passage III
Passage I (Q. Nos. 52 to 54)
(Q. Nos. 46 to 48)
If a, b, g are the roots of x 3 + 2x 2 - x - 3 = 0
Consider the system of equations a b g
x + y + z = 5; x + 2 y + 3 z = 9; x + 3 y + lz = m
The system is called smart, brilliant, good and lazy
52. The value of g a b is equal to
according as it has solution, unique solution, infinitely b g a
many solutions and no solution, respectively. (a) 14 (b) -2 (c) 10 (d) 14
46. The system is smart, if 53. If the absolute value of the expression
(a) l ¹ 5 or l = 5 and m = 13 (b) l ¹ 5 and m = 13 a -1 b -1 g -1 m
+ + can be expressed as , where m and
(c) l ¹ 5 and m ¹ 13 (d) l ¹ 5 or l = 5 and m ¹ 13 a +2 b +2 g +2 n
47. The system is good, if m n2
(a) l ¹ 5 or l = 5 and m = 13 (b) l = 5 and m = 13 n are co-prime, the value of , is
m -n m +n
(c) l = 5 and m ¹ 13
(d) l ¹ 5, m is any real number (a) 17 (b) 27 (c) 37 (d) 47
2 2 2
48. The system is lazy, if 54. If a = a + b + g , b = ab + bg + ga , the value of
(a) l ¹ 5 or l = 5 and m = 13 (b) l = 5 and m = 13 a b b
(c) l = 5 and m ¹ 13 (d) l ¹ 5 or l = 5 and m ¹ 13 b a b , is
Passage II b b a
(Q. Nos. 49 to 51) (a) 14 (b) 49 (c) 98 (d) 196

a11 a12 a13 Passage IV


i+ j
If D = a 21 a 22 a 23 and C ij = ( -1) M ij , where M ij (Q. Nos. 55 to 57)
a 31 a 32 a 33 Suppose f ( x ) is a function satisfying the following
is a determinant obtained by deleting ith row and conditions:
C11 C12 C13 (i) f (0) = 2, f (1) = 1
5
jth column, then C 21 C 22 C 23 = D2 . (ii) f ( x )has a minimum value at x =
2
C 31 C 32 C 33 2ax 2ax - 1 2ax + b + 1
1 x x2 x 3-1 0 x -x4 (iii) For all x, f ¢ ( x ) = b b +1 -1 .
49. If x x2 1 = 5 and D = 0 x -x4 x 3-1 , 2 ( ax + b) 2ax + 2b + 1 2ax + b
x2 1 x x -x4 x 3-1 0 55. The value of f (2) + f (3) is
2
then sum of digits of D , is (a) 1 (b)
3
(c) 2 (d)
5
(a) 7 (b) 8 (c) 13 (d) 11 2 2
50. Suppose a, b, c Î R, a + b + c > 0, A = bc - a 2 , B = ca - b 2 56. The number of solutions of the equation f ( x ) + 1 = 0 is
A B C (a) 0 (b) 1 (c) 2 (d) infinite
and C = ab - c 2 and B C A = 49, then the value of
57. Range of f ( x ) is
C A B
a 3 + b 3 + c 3 - 3abc , is æ 7ù é3 ö é7 ö æ 3ù
(a) ç -¥, ú (b) ê , ¥ ÷ (c) ê , ¥ ÷ (d) ç -¥, ú
è 16 û ë4 ø ë 16 ø è 4û
(a) -7 (b) 7 (c) -2401 (d) 2401
Chap 07 Determinants 571

Passage V 62. If a, b, c are in GP, then


(Q. Nos. 58 to 60) (a) r 3 = p 3q (b) p 3 = r 3q (c) p 3 = q 3r (d) q 3 = p 3r
x ex - 1 ( x - 1)3 63. If D = 27 and a 2 + b 2 + c 2 = 2 , then the value of å a 2 b, is
If x - ln x cos ( x - 1) ( x - 1)2 = a0 + a1 ( x - 1) + a2 ( x - 1)2 + ... (a) 3(2 2 - p ) (b) 3(2 2 - r )
tan x sin 2 x cos 2 x (c) 3(2 2 - q ) (d) 3(2 2 - p - q )

58. The value of cos -1 (a1 ), is


Passage VII
p p
(a) 0 (b) (c) (d) p (Q. Nos. 64 to 66)
4 2
2
59. The value of lim (sin x ) is x a +n ab ac
x ®a0 2
If Dn = ab b +n bc , n Î N and the equation
(a) 1 (b) e (c) e - 1 (d) None of these
2
ac bc c +n
60. The equation whose roots are a 0 and a1 , is
(a) x 2 - x = 0 (b) x 2 - 2 x = 0 x 3 - lx 2 + 11x - 6 = 0 has roots a, b, c and a, b, c are in AP.
(c) x 2 - 3 x = 0 (d) None of these 7
64. The value of å D n is
Passage VI r =1
3
(Q. Nos. 61 to 63) (a) (12 ) (b) (14 ) 3 (c) (26 ) 3 (d) (28 ) 3
2 2
-bc b + bc c + bc D2n
2 65. The value of is
Let D = a + ac -ac c 2 + ac and the equation Dn
a 2 + ab b 2 + ab -ab (a) < 8 (b) = 8
(c) > 8 (d) None of these
x - px + qx - r = 0 has roots a, b, c, where a, b, c Ï R + .
3 2
30
æ 27 D r - D3r ö
61. The value of D is 66. The value of å ç ÷ is
è r =1 27r 2 ø
(a) £ 9r 3 (b) ³ 27r 2 (c) £ 27r 2 (d) ³ 81r 3
(a) 130 (b) 190 (c) 280 (d) 340

#L Determinants Exercise 4 :
Single Integer Answer Type Questions
n
This section contains 10 questions. The answer to each 1 cos a cos b 0 cos a cos b
question is a single digit integer, ranging from 70. If cos a 1 cos g = cos a 0 cos g ,
0 to 9 (both inclusive).
cos b cos g 1 cos b cos g 0
32 + k 42 32 + 3 + k
cos 2 a + cos 2 b + cos 2 g is equal to
67. If 4 2 + k 52 4 2 + 4 + k = 0,
52 + k 62 52 + 5 + k (b + c ) 2 a2 a2
71. Let f (a, b, c ) = b2 (c + a ) 2 b2 , the
k k k
the value of 2 2 2 ¼ ¥ is c 2
c 2
(a + b ) 2
68. Let a , b and g are three distinct roots of greatest integer n Î N such that (a + b + c )n ¢ divides
x -1 -6 2 -1 f (a, b, c ) is
æ 1 1 1ö
-6 x -2 -4 = 0, the value of ç + + ÷ is 72. If 0 £ q £ p and the system of equations
èa b g ø
2 -4 x -6 x = (sin q ) y + (cos q ) z
x -1 3
x e ( x - 1) y = z + (cos q ) x
n
69. If x - ln x cos ( x - 1) ( x - 1) 2 = å ar ( x - 1)r , z = (sin q ) x + y
r=0
tan x sin 2 x cos 2 x 8q
has a non-trivial solution, then is equal to
the value of (2 a0
+3 ) a1 a1 +1
is p
572 Textbook of Algebra

1 1 1 1 1+a 1 1
1 2 3 4 75. If a ¹ 0, b ¹ 0, c ¹ 0 and 1 + b 1 + 2b 1 = 0,
73. The value of the determinant is
1 3 6 10 1 + c 1 + c 1 + 3c
1 4 10 20 the value of | a - 1 + b - 1 + c - 1 | is equal to
74. If a, b, c and d are the roots of the equation 76. If the system of equations
x 4 + 2x 3
+ 4 x 2 + 8x + 16 = 0, the value of the ax + hy + g = 0; hx + by + f = 0
1+a 1 1 1 and ax 2 + 2 hxy + by 2 + 2gx + 2 fy + c + l = 0 has a
1 1+b 1 1 unique solution and
determinant is
1 1 1+c 1 abc + 2 fgh - af 2
- bg 2 - ch 2
= 8, the value of l is
1 1 1 1+d h 2 - ab

#L Determinants Exercise 5 :
Matching Type Questions
n This section contains 5 questions. Questions 77 to 81 have three statements (A, B and C) given in Column I and four
statements (p, q, r and s) in Column II. Any given statement in Column I can have correct matching with one or more
statement(s) given in Column II.

77. Column I Column II


2 2 2
(A) If a,b, c are three complex numbers such that a + b + c = 0 and (p) 2
b2 + c2 ab ac
ab c2 + a2 bc = la 2b 2c 2, then l is divisible by
ac bc a2 + b2
a a +b a +b +c
(B) (q) 3
If a,b, c Î R and 2a 5a + 2b 7a + 5b + 2c = -1024, then a is divisible by
3a 7a + 3b 9a + 7b + 3c
x - 1 2x 2 - 5 x 3 - 1 4
(C) (r)
Let D( x ) = 2 x 2 + 5 2 x + 2 x 3 + 3 and ax + b be the remainder, when D( x ) is
x3 -1 x + 1 3x 2 - 2
divided by x 2 - 1, then 4a + 2b is divisible by
(s) 5
(t) 6

78. Column I Column II


(A) Let f1( x ) = x + a1, f 2( x ) = x 2 + b1x + b2, x1 = 2, x 2 = 3 and x 3 = 5 and (p) Even number
1 1 1
D = f1( x1 ) f1( x 2 ) f1( x 3 ) , then D is
f 2( x1 ) f 2( x 2 ) f 2( x 3 )
1 b1 a1
(B) If | a1 - b1| = 6 and f ( x ) = 1 b1 2a1 - x , then the minimum value of f ( x ) is (q) Prime number
1 2b1 - x a1
x - 2 (x - 1)2 x3
If coefficient of x in f ( x ) = x - 1 x2 ( x + 1 ) 3 is l, then | l | is (r) Odd number
(C)
x (x + 1) (x + 2)3
2

(s) Composite number


(t) Perfect number
Chap 07 Determinants 573

79. Column I Column II


2
x + 3x x - 1 x +3
(A) If x 2 + 1 2 + 3 x x - 3 = ax 4 + bx 3 + cx 2 + dx + e, then e + a is divisible by (p) 2
x2 -3 x + 4 3x

x -1 5x 7
(B) If x - 1 x - 1 8 = ax 3 + bx 2 + cx + d , then (e + a - 3 ) is divisible by
2 (q) 3
2x 3x 0

x 3 + 4x x + 3 x -2
(r) 5
(C) If x -2 5x x - 1 = ax 5 + bx 4 + cx 3 + dx 2 + ex + f , then ( f + e ) is
x -3 x + 2 4x
divisible by
(s) 6
(t) 7

80. Column I Column II


2 2 2
(A) If a + b + c = 1 and (p) independent of a
a 2 + (b 2 + c 2 )d ab(1 - d ) ca(1 - d )
D= ab(1 - d ) b + (c 2 + a 2 )d
2
bc(1 - d ) , then D is
ca(1 - d ) bc(1 - d ) c 2 + (a + b 2 )d

1 1 -(a + b )
c c c2
(B) -(b + c ) 1 1 (q)
If D = , then D is independent of b
a2 a a
-bd (b + c ) (ad + 2bd + cd ) -(a + b )bd
a 2c ac ac 2
sin a cosa sin(a + d ) (r)
(C)
If D = sin b cosb sin(b + d ) , then D is independent of c
sin c cosc sin(c + d )

(s) independent of d
(t) zero

81. Column I Column II


(A) If n be the number of distinct values of 2 ´ 2 determinant whose entries are from (p) 2
the set { -1, 0, 1 }, then (n - 1 ) 2 is divisible by

(B) If n be the number of 2 ´ 2 determinants with non-negative values whose entries (q) 3
from the set { 0, 1 }, then (n - 1 ) is divisible by

(C) If n be the number of 2 ´ 2 determinants with negative values whose entries from (r) 4
the set { -1, 1 }, then n(n + 1 ) is divisible by

(s) 5
(t) 6
574 Textbook of Algebra

#L Determinants Exercise 6 :
Statement I and II Type Questions
n Directions (Q. Nos. 82 to 87) are Assertion-Reason type Statement-2 The value of skew-symmetric determinant
questions. Each of these questions contains two of odd order equals zero.
statements:
Statement-1 (Assertion) and Statement-2 (Reason)
(1 + x )11 (1 + x )12 (1 + x )13
Each of these questions also has four alternative choices, 85. Statement-1 f ( x ) = (1 + x ) 21 (1 + x ) 22 (1 + x ) 23 ,
only one of which is the correct answer. You have to select (1 + x ) 31 (1 + x ) 32 (1 + x ) 33
the correct choice as given below.
the coefficient of x in f ( x ) = 0
(a) Statement-1 is true, Statement-2 is true; Statement-2
is a correct explanation for Statement-1 Statement-2 If P( x ) = a 0 + a 1 x + a 2 x 2 + a 3 x 3
(b) Statement-1 is true, Statement-2 is true; Statement-2 + ¼ + an x n , then a1 = P ¢ (0), where dash denotes the
is not a correct explanation for Statement-1
(c) Statement1 is true, Statement-2 is false
differential coefficient.
(d) Statement-1 is false, Statement-2 is true 86. Statement-1 If system of equations 2x + 3y = a
r r +1 n
and bx + 4y = 5 has infinite solution,
82. Statement-1 If D(r ) = then å D(r ) = - 3n
r +3 r + 4 r =1 15 8
then a = , b =
f 1 (r ) f 2 (r ) 4 5
Statement-2 If D(r ) =
f 3 (r ) f 4 (r ) Statement-2 Straight lines a 1 x + b 1y + c 1 = 0
n n
å f 1 (r ) å f 2 (r ) and a 2 x + b 2 y + c 2 = 0 are parallel,
n
r =1 r =1 a b c
then å D(r ) = n n if 1 = 1 ¹ 1
r =1
å f 3 (r ) å f 4 (r ) a2 b2 c 2
r =1 r =1
1 2 3
83. Consider the determinant 87. Statement-1 The value of the determinant 4 5 6 ¹ 0
a 1 + b1 x 2 a 1x 2 + b 1 c1
2 2
7 8 0
D = a2 + b2 x a2 x + b2 c 2 = 0,
a3 + b 3 x 2 a3 x 2 + b3 c3 Statement-2 Neither of two rows or columns of
1 2 3
where ai , bi , c i Î R (i = 1, 2, 3) and x Î R. 4 5 6 is identical.
Statement-1 The value of x satisfying D = 0 are 7 8 0
x = 1, - 1
a 1 b1 c 1
88. Statement-1 The digits A, B and C re such that
Statement-2 If a 2 b 2 c 2 = 0, then D = 0.
the three digit numbers A88, 6B8, 86C are divisible
a3 b3 c3 A 6 8
84. Statement-1 The value of determinant by 72, then the determinant 8 B 6 is divisible
æ pö æ pö 8 8 C
sin p cos ç x + ÷ tan ç x - ÷
è 4 ø è 4ø by 288.
æ pö æpö æx ö
sin ç x - ÷ - cos ç ÷ ln ç ÷ is zero. Statement-2 A = B = ?
è 4ø è2ø èy ø
æp ö æy ö
cot ç + x ÷ ln ç ÷ tan p
è4 ø èx ø
Chap 07 Determinants 575

#L Determinants Exercise 7 :
Subjective Type Questions
n In this section, there are 20 subjective questions. 98. Show that the determinant D( x ) is given by D( x ) =
b +c c b sin ( x + a ) cos ( x + a ) a + x sin a
89. Prove that c c +a a = 4abc . sin ( x + b ) cos ( x + b ) b + x sin b . isindependent of x.
b a a +b sin ( x + g ) cos ( x + g ) c + x sin g
a -b -c 2a 2a x x x
C1 C2 C3
90. Prove that 2b b - c -a 2b = (a + b + c ) 3 . y y y
99. Evaluate C1 C2 C3 .
2c 2c c -a - b z z z
C1 C2 C3
13 + 3 2 5 5
100. (i) Find maximum value of
91. Find the value of determinant 15 + 26 5 10 . 1 + sin 2 x cos 2 x 4 sin 2x
3 + 65 15 5 f (x ) = sin 2 x 1 + cos 2 x 4 sin 2x .
bc ca ab
sin 2 x 2
cos x 1 + 4 sin 2x
92. Find the value of the determinant p q r , where
(ii) Let A, B and C be the angles of a triangle, such that
1 1 1
A ³ B ³ C.
a, b and c respectively are the pth,q th and rth terms of a
harmonic progression. Find the minimum value of D, where
93. Without expanding the determinant at any stage, prove sin 2 A sin A cos A cos 2 A
3 D = sin 2 B sin B cos B cos 2 B .
-5 3 + 5i - 4i
2 sin 2 C sin C cos C cos 2 C
that 3 - 5i 8 4 + 5i has a purely real value.
3 x 2 - 4 x + 6 2x 2 + 4 x + 10 3x 2 - 2x + 16
+ 4i 4 - 5i 9
2 101. If f ( x ) = x -2 2x + 2 3x - 1 ,
ah + bg g ab + ch
1 2 3
94. Prove without expanding that bf + ba f hb + bc = a 3 x 2 sin x
af + bc c bg + fc then find the value of ò f ( x )dx .
ah + bg a h -3
1+x6
bf + ba h b . 102. If Y = sX and Z = tX all the variables beings functions of
af + bc g f X Y Z
s1 t 1
95. If A, B and C are the angles of a triangle and x, then prove that X 1 Z1 = X 3 Y1 ,
s2 t2
X 2 Y2 Z 2
1 1 1
where suffixes denote the order of differentiation with
1 + sin A 1 + sin B 1 + sin C = 0, respect to x .
2 2 2
sin A + sin A sin B + sin B sin C + sin C 103. If f , g and h are differentiable functions of x and
then prove that DABC must be isosceles. f g h
bg bg ¢ + b ¢g b ¢g ¢ D = (x f )¢ ( xg ) ¢ ( xh ) ¢ , then prove that
96. Prove that g a ga ¢ + g ¢a g ¢a ¢ ( x 2 f ) ¢¢ ( x 2 g ) ¢¢ ( x 2 h ) ¢¢
ab ab ¢ + a ¢b a ¢b ¢ f g h
= (ab ¢ - a ¢b ) (bg ¢ - b ¢g ) ( g a ¢ - g ¢a ). D¢ = f¢ g¢ h¢ .
u ( x 3 f ¢¢ ) ¢ ( x 3 g ¢¢ ) ¢ ( x 3 h ¢¢ ) ¢
97. If y = , where u and v are functions of x, show that
v 104. If | a1 | > | a 2 | + | a 3 |,| b 2 | > | b 1 | + | b 3 | and
2
u v 0
d y a1 a 2 a3
v3 = u¢ v v .
dx 2 | c 3 | > | c 1 | + | c 2 |, then show that b1 b 2 b 3 ¹ 0.
u ¢¢ v ¢¢ 2v ¢
c1 c 2 c3
576 Textbook of Algebra

(a - a 1 ) - 2 (a - a 1 ) - 1 a1-1 107. Eliminates


-2 -1
105. Show that (a - a 2 ) (a - a 2 ) a 2-1 = (i) a, b and c
(a - a 3 ) - 2 (a - a 3 ) -1 a 3-1 (ii) x , y, z from the equations
a 2 P (a i - a j ) by cz cz ax
± . Write out the terms of the product in -a + + = 0, - b + + =0
P ai P(a - ai ) 2 z y x z
the numerator and give the resulting expression its ax by
correct sign. and -c + + = 0.
y x
106. Show that in general there are three values of t for
which the following system of equations has a 108. If x , z and y are not all zero and if
non-trivial solution (a - t )x + by + cz = 0,
ax + by + cz = 0, bx + cy + az = 0
bx + (c - t )y + az = 0 and cx + ay + (b - t )z = 0.
and cx + ay + bz = 0, then
Express the product of these values of t in the form of a prove that x : y : z = 1 : 1 : 1 or 1 : w : w2 or 1 : w2 : w.
determinant.

#L Determinants Exercise 8 :
Questions Asked in Previous 13 Year’s Exam
n
This section contains questions asked in IIT-JEE, 113. Consider the system of equations
AIEEE, JEE Main & JEE Advanced from year 2005 to x - 2y + 3z = -1
year 2017.
- x + y - 2z = k
109. If a 2 + b 2 + c 2 = -2 and x - 3y + 4z = 1
1+a2x (1 + b 2 )x (1 + c 2 )x
Statement-1 The system of equations has no solutions
f ( x ) = (1 + a 2 )x 1 + b 2 x (1 + c 2 )x , then f ( x ) is a
for k ¹ 3. [IIT-JEE 2008, 3M]
(1 + a 2 )x (1 + b 2 )x 1 + c 2 x and 1 3 -1
polynomial of degree [AIEEE 2005, 3M] Statement-2 The determinant -1 -2 k ¹ 0, for k ¹ 3.
(a) 3 (b) 2 (c) 1 (d) 0 1 4 1
110. The system of equations (a) Statement-1 is true, Statement-2 is true and Statement-2
ax + y + z = a - 1, is a correct explanation for Statement-1.
x + ay + z = a - 1 (b) Statement-1 is true, Statement-2 is true and Statement-2 is
and x + y + az = a - 1 not a correct explanation for Statement-1.
has no solution, if a is [AIEEE 2005, 3M] (c) Statement-1 is true, Statement-2 is false.
(a) not -2 (b) 1 (d) Statement-1 is false, Statement-2 is true.
(c) -2 (d) Either -2 or 1
114. Let a, b, c be any real numbers. Suppose that there are
111. If a1 , a 2 , a 3 ,..., an ,... are in GP, then the determinant
real numbers x, y, z not all zero such that
log an log an + 1 log an + 2 x = cy + bz, y = az + cx and z = bx + ay. Then,
D = log an + 3 log an + 4 log an + 5 is equal to a 2 + b 2 + c 2 + 2abc is equal to [AIEEE 2008, 3M]
log an + 6 log an + 7 log an + 8 (a) -1 (b) 0 (c) 1 (d) 2
[AIEEE 2005, 3M]
(a) 1 (b) 0 (c) 4 (d) 2 115. Let a, b, c be such that b(a + c ) ¹ 0. If
1 1 1 a a +1 a -1 a +1 b +1 c -1
112. If D = 1 1 + x 1 for x ¹ 0, y ¹ 0, then D is -b b + 1 b - 1 + a - 1 b -1 c + 1 = 0,
1 1 1+y [AIEEE 2007, 3M] c c -1 c +1 n +2 n +1
( -1) a (-1) b ( -1)n c
(a) divisible by neither x nor y then the value of n is [AIEEE 2009, 4M]
(b) divisible by both x and y (a) any integer (b) zero
(c) divisible by x but not y (c) an even integer (d) any odd integer
(d) divisible by y but not x
Chap 07 Determinants 577

1 tan q 1 122. Which of the following values of a satisfy the equation


116. If f (q ) = - tan q 1 tan q , then the set (1 + a ) 2 (1 + 2 a ) 2 (1 + 3 a ) 2
-1 - tan q 1 (2 + a ) 2 (2 + 2 a ) 2 (2 + 3 a ) 2 = -648 a ?
ì pü (3 + a ) 2 (3 + 2 a ) 2 (3 + 3 a ) 2
í f (q ): 0 £ q < ý is
î 2þ [IIT-JEE 2011, 2M]
[JEE Advanced 2015, 4M]
(a) ( -¥, - 1 ) È (1, ¥ ) (b) [2, ¥ ) (a) -4 (b) 9
(c) ( -¥, 0 ] È [2, ¥ ) (d) ( -¥, - 1 ] È [1, ¥ ) (c) -9 (d) 4
117. The number of values of k for which the linear equations 123. The system of linear equations
4 x + ky + 2z = 0 x + ly - z = 0
kx + 4y + z = 0 lx - y - z = 0
2x + 2y + z = 0 x + y - lz = 0
Possess a non-zero solution is [AIEEE 2011,4M] has a non-trivial solution for [JEE Main 2016, 4M]
(a) zero (b) 3 (c) 2 (d) 1 (a) exactly one-value of l
118. If the trivial solution is the only solution of the system (b) exactly two values of l
of equations (c) exactly three values of l
x - ky + z = 0 (d) infinitely many values of l.

kx + 3y - kz = 0 124. The total number of distinct x Î R for which


3x + y - z = 0 x x2 1+ x 3
2
Then, the set of values of k is 2x 4x 1 + 8x 3 = 10 is
(a) {2, - 3 } (b) R - {2, - 3 }
3x 9x 2 1 + 27 x 3
(c) R - {2 } (d) R - { -3 } [AIEEE 2011, 4M] [JEE Advanced 2016, 3M]

119. The number of values of k for which the system of 125. Let a, l, m ÎR. Consider the system of linear equations
equations (k + 1)x + 8y = 4k ; kx + (k + 3)y = 3k - 1
ax + 2y = l
has no solution, is
(a) 1 (b) 2 3x - 2y = m
(c) 3 (d) infinite [JEE Main 2013, 4M] Which of the following statement(s) is (are) correct?
n n [JEE Advanced 2016, 4M]
120. If a , b ¹ 0 and f (n ) = a + b and
(a) If a = -3, then the system has infinitely many solutions
3 1 + f (1) 1 + f (2) for all values of l and m
1 + f (1) 1 + f (2) 1 + f (3) = k(1 - a ) 2 (1 - b ) 2 (a - b ) 2 , (b) If a ¹ -3, then the system has a unique solution for all
1 + f (2) 1 + f (3) 1 + f ( 4 ) values of l and m
(c) If l + m = 0, then the system has infinitely many solutions
then k is equal to [JEE Main 2014, 4M]
for a = -3
(a) 1 (b) -1
(d) If l + m ¹ 0, then the system has no solution for a = -3
(c) ab (d) 1 / ab
121. The set of all values of l for which the system of linear 126. If S is the set of distinct values of ‘b’ for which the
following system of linear equations
equations
2x 1 - 2x 2 + x 3 = lx 1 x +y +z = 1
2x 1 - 3x 2 + 2x 3 = lx 2 x + ay + z = 1
ax + by + z = 0
- x 1 + 2x 2 = lx 3
has a non-trivial solution [JEE Main 2015, 4M] has no solution, then S is [JEE Main 2017, 4M]
(a) contains two elements (a) an infinite set
(b) contains more than two elements (b) a finite set containing two or more elements
(c) is an empty set (c) a singleton
(d) is a singleton (d) an empty set
Answers
Exercise for Session 1 52. (c) 53. (c) 54. (d) 55. (a) 56. (a) 57. (c)
1. (d) 2. (d) 3. (c) 4. (b) 5. (c) 6. (d) 58. (c) 59. (a) 60. (d) 61. (b) 62. (d) 63. (b)
7. (b) 64. (b) 65. (a) 66. (c) 67. (2) 68. (9) 69. (2)
70. (1) 71. (3) 72. (6) 73. (1) 74. (8) 75. (3)
Exercise for Session 2
76.(8) 77. (A) ® (p,r); (B) ® (p,r); (C) ® (p,q,s,t)
1. (c) 2. (d) 3. (a) 4. (c) 5. (a) 6. (b)
78. (A) ® (p,s,t); (B) ® (r,t); (C) ® (p,q)
7. (b) 8. (b) 9. (d) 10. (c) 11. (d)
79. (A) ® (r); (B) ® (r,t); (C) ® (p,q,s)
Exercise for Session 3 80. (A) ® (p,q,r); (B) ® (p,q,r,s,t); (C) ® (p,q,r,s,t)
1. (b) 2. (c) 3. (c) 4. (b) 5. (b) 6. (d) 81. (A) ® (p,r); (B) ® (p,q,r,t); (C) ® (p,r,s)
7. (d) 8. (a) 9. (b) 10. (c) 11. (c) 12. (a) 82. (c) 83. (b) 84. (a) 85. (a) 86. (b) 87. (b)
13. (a) 14. (a) 88. (c) 91. 15 2 - 25 3
1
Exercise for Session 4 92. 0 99. xyz (x - y)( y - z )( z - x)
12
1. (c) 2. (b) 3. (b) 4. (d) 5. (d) 6. (b)
100. (i) 6 (ii) 0 101. 0
7. (b) 8. (c) 9. (d) 10. (a)
2
105. - a (a1 - a2 )(a2 - a3 )(a3 - a1 )
Chapter Exercises a b c
1. (a) 2. (b) 3. (a) 4. (c) 5. (a) 6. (b) 106. b c a
7. (d) 8. (c) 9. (c) 10. (d) 11. (d) 12. (c)
c a b
13. (c) 14. (a) 15. (b) 16. (a) 17. (c) 18. (a)
19. (d) 20. (a) 21. (b) 22. (a) 23. (b) 24. (b) yz zx xy
107. (i) + + +1= 0 (ii) a3 + b3 + c3 = 5 abc
25. (b) 26. (b) 27. (c) 28. (a) 29. (b) 30. (a) x2 y2 z2
31. (a, b, c, d) 32. (b, d) 33. (a, b, c, d) 34.(b, d)
109. (b) 110. (c) 111. (b) 112. (b) 113. (a) 114. (c)
35. (a, c, d) 36. (a, b, c) 37. (a, b, c, d) 38. (a, b, d)
39. (a, b, c, d) 40. (a, c, d) 41. (a, b) 42. (a, b) 115. (d) 116. (b) 117. (c) 118. (b) 119. (a) 120. (a)
43. (a, b) 44. (a, c) 45. (a, b, c) 46. (a) 121. (a) 122. (b, c) 123. (c) 124. (2) 125. (b,c,d)
47. (b) 48. (c) 49. (c) 50. (b) 51. (b) 126. (c)
Applying R2 → R2 − R 1, then

Solutions
1. Q f (n ) = α n + βn Also,
2
∆( x ) =
x+a x+a+D
2D 2D
= 2D (x + a − x − a − D ) = − 2D 2

∫0 ∆(x ) dx = − 16
3 1 + f (1 ) 1 + f (2 ) ⇒ − 2 D 2 (2 ) = − 16
Let ∆ = 1 + f (1 ) 1 + f (2 ) 1 + f (3 ) ∴ D 2 = 4 or D = ± 2
1 + f (2 ) 1 + f (3 ) 1 + f ( 4 )
x 1+ x2 x3
3 1 + α + β 1 + α 2 + β2
3. Let ∆( x ) = loge (1 + x ) 2
e x
sin x
= 1+α+β 1 + α 2 + β2 1 + α 3 + β3
cos x tan x sin 2 x
1 + α 2 + β2 1 + α 3 + β3 1 + α 4 + β4
2 = a + bx + cx 2 +…
1 1 1 1 1 1 1 1 1
On putting x = 0, we get
= 1 α β × 1 α β = 1 α β
0 1 0
1 α 2 β2 1 α 2 β2 1 α 2 β2
0 1 0 =a
Applying C 2 → C 2 − C1 and C 3 → C 3 − C1, then
2 1 0 0
1 L 0 L 0
∴ 0 =a
M
or a = 0, then
∆= 1 α −1 β −1
∆( x ) = bx + cx 2 +…
M
Hence, ∆( x ) is divisible by x .
1 α2 − 1 β2 − 1
a2 b2 c2
Expanding along R 1, we get
2 2 4. Given, (a + 1) 2
(b + 1 ) (c + 1 ) 2 = 0
2
α −1 β −1 1 1
∆= = (α − 1 ) 2 ( β − 1 ) 2 (a − 1 ) 2 (b − 1 ) 2 (c − 1 ) 2
α 2 − 1 β2 − 1 α+1 β+1
= (α − 1 ) 2( β − 1 ) 2 ( β − α ) 2 = (1 − α ) 2 (1 − β ) 2 (α − β ) 2 a2 b2 c2
= k(1 − α ) 2 (1 − β ) 2 (α − β ) 2 [given] ⇒ a 2 + 2a + 1 b 2 + 2b + 1 c 2 + 2c + 1 = 0
∴ k =1 a 2 − 2a + 1 b 2 − 2b + 1 c 2 − 2c + 1

2. Qa, b, c and d are in AP. Let D be the common difference,then Applying R2 → R2 − R 3, then
b = a + D, c = a + 2 D, d = a + 3 D …(i) a2 b2 c2
x+a x+b x + a −c 4a 4b 4c =0
and ∆( x ) = x + b x+c x −1 a 2 − 2a + 1 b 2 − 2b + 1 c 2 − 2c + 1
x+c x+d x −b + d 1
Applying R 3 → R 3 − R 1 + R2 , then
On putting the values of b, c and d from Eq.(i) in ∆( x ) , then 2
x+a x + a + D x − 2D a2 b2 c2
∆( x ) = x + a + D x + a + 2 D x − 1 4 a b c =0
x + a + 2D x + a + 3D x + 2D 1 1 1
1 1 1 1
Applying R2 → R2 − ( R 1 + R 3 ), then
2 ⇒ − a b c =0 [Q R1 → R 3]
x+a x+a+D x − 2D
a2 b2 c2
M
1 1 1
∆( x ) = 0 L 0 L −1
⇒ a b c =0
M 2 2
a b c2
x + a + 2D x + a + 3D x + 2D
⇒ (a − b ) (b − c ) (c − a ) = 0
Expanding along R2 , then ∴ a − b = 0 or b − c = 0 or c − a = 0
x+a x+a+D ⇒ a = b or b = c or c = a
∆( x ) =
x + a + 2D x + a + 3D Hence, ∆ABC is an isosceles triangle.
580 Textbook of Algebra

α x x x a b c
x β x x 7. Q b c a = − (a 3 + b 3 + c 3 − 3abc )
5. Let ∆ =
x x γ x c a b
x x x δ = − (a + b + c ) (a + bω + cω 2 ) (a + bω 2 + cω )
Applying C 2 → C 2 − C1, C 3 → C 3 − C1 and C 4 → C 4 − C1, then [where ω is cube roots of unity]
α x −α x −α x −α = − f (α ) f ( β ) f ( γ ) [Qα = 1, β = ω, γ = ω 2 ]
x β−x 0 0
∆= cos 2 x sin 2 x cos 4 x
x 0 γ−x 0
8. Let ∆ = sin 2 x cos 2 x cos2 x
x 0 0 δ−x
cos 4 x cos2 x cos 2 x
Expanding along first column, then
∆ = α (β − x ) (γ − x ) (δ − x ) − x (x − α ) (γ − x ) (δ − x ) 1 − 2 sin 2 x sin 2 x 1 − 8 sin 2 x (1 − sin 2 x )
+ x (δ − x ) (x − α ) (x − β) − x (x − α ) (β − x ) (γ − x ) = sin 2 x 1 − 2 sin 2 x 1 − sin 2 x
= ( x − α )( x − β ) ( x − γ ) ( x − δ ) − x [( x − α )( x − γ ) ( x − δ ) 1 − 8 sin x (1 − sin x ) 1 − sin x
2 2 2
1 − 2 sin 2 x
+ ( x − β )( x − γ ) ( x − δ ) 1 0 1
+ ( x − α )( x − β ) ( x − δ ) + ( x − α ) ( x − β ) ( x − γ )] [given] The required constant term is 0 1 1
= f ( x ) − x f ′( x ) 1 1 1
∴ f (x ) = (x − α ) (x − β) (x − γ ) (x − δ ) Applying C 3 → C 3 − C1, then
a b −c c + b 1 L 0 L 0
6. Given, a+c b c −a = 0 M
a −b a + b c 0 1 1 = 1( 0 − 1 ) = − 1
a b −c c + b
2 M
1 2
⇒ a + ac b c −a = 0 1 1 0
a 2
a − ab a + b c 9. Q − 1 ≤ x < 0 ⇒ [x ] = − 1
Applying C1 → C1 + bC 2 + cC 3, then 0 ≤y <1 ⇒ [y ] = 0
a + b + c b −c c + b
2 2 2 1 ≤z <2 ⇒ [z ] = 1
1 2 0 0 1
⇒ a + b2 + c2 b c −a = 0 [x ] + 1 [y ] [z ] M
a 2
a +b +c a+b
2 2
c Let ∆ = [ x ] [y ] + 1 [z ] = − 1 L 1 L 1
Applying R2 → R2 − R1 and R 3 → R 3 − R 1, then [x ] [y ] [z ] + 1 M
a2 + b2 + c2 L b − c L c −b −1 0 2
0 1
1
M Expanding along C 2 , then ∆ = = 1 = [z ]
⇒ 0 c −b −a = 0 −1 2
a
M y2 − xy x2 xy 2 − xy x 2y
1
0 a+c −b 10. Let ∆ = a b c = ax b cy
xy
Expanding along C1, then a′ b′ c′ a′ x b′ c ′y
(a 2 + b 2 + c 2 ) c −b −a Applying C1 → C1 + y C 2 and C 3 → C 3 + xC 2 , then
⇒ =0
a a+c −b 0 L − xy L 0
(a 2 + b 2 + c 2 ) M
⇒ [( − bc + (b + a ) (a + c )] = 0 1
a ∆= ax + by b bx + cy
xy
(a 2 + b 2 + c 2 ) ( − bc + ab + bc + a 2 + ac )
⇒ =0
M
a a ′ x + b ′y b′ b ′ x + c ′y
⇒ (a 2 + b 2 + c 2 ) (a + b + c ) = 0 Expanding along R 1, then
Q a2 + b2 + c2 ≠ 0 1 ax + by bx + cy
= ⋅ xy ⋅
∴ a+b+c=0 xy a ′ x + b ′y b ′ x + c ′y
Therefore, line ax + by + c = 0 passes through the fixed point ax + by bx + cy
(1, 1). =
a ′ x + b ′y b ′ x + c ′y
Chap 07 Determinants 581

11. Q In a triangle A + B + C = π and e π = cos π + i sin π = − 1 14. As a1 b1 c1, a 2 b2 c 2 and a 3b 3c 3 are even natural numbers each of
e i (B + C ) = e i (π −A) = e i π⋅ e i A = − e − i A c1, c 2, c 3 is divisible by 2.
⇒ e − i (B + C )
= −e iA Let Ci = 2 λ i for i = 1, 2, 3 and λ i ∈ N , then
Similarly, e − i ( A + B ) = − e iC and e − i (C + A ) = − e iB 2 λ 1 a1 b1 λ 1 a1 b1
Taking e i A , e i B , e iC common from R 1, R 2 and R 3 respectively, ∆ = 2 λ 2 a 2 b2 = 2 λ 2 a 2 b2 = 2m
we get 2 λ 3 a 3 b3 λ 3 a3 b 3
e iA e − i (A + C ) e − i (A + B ) where m is some natural number. Thus, ∆ is divisible by 2.
∆ = e i A⋅ e iB⋅ e iC e − i ( B + C ) e iB e − i (A + B ) That ∆ may not be divisible by 4 can be seen by taking the
− i (B + C ) − i (A + C ) three numbers as 112, 122 and 134.
e e e iC
2 1 1
e iA − e iB − e iC ∆ = 2 1 2 = 2(3 − 2 ) − 1(6 − 8 ) + 1(2 − 4 ) = 2

=e = −e iA
e iB
− e iC 4 1 3
−e iA
−e iB
e iC which is divisible by 2 but not by 4.
iA
Taking e , e , e iB iC
common from C1, C 2 and C 3 respectively, c b cos B + c β a cos A + b α + c γ
we get 15. Let ∆ = a c cos B + a β b cos A + c α + a γ
1 −1 −1 b a cos B + b β c cos A + a α + b γ
∆ = ( − 1 ) e i A ⋅ e iB ⋅ e iC − 1 1 −1 Applying C 2 → C 2 − βC1 and C 3 → C 3 − γ C1, then
−1 −1 1 c b cos B a cos A + bα
1 −1 −1 ∆ = a c cos B b cos A + cα
= (− 1) e i π − 1 1 −1 b a cos B c cos A + aα
−1 −1 1 Applying C 3 → C 3 − α sec B C 2 , then
1 −1 −1 1 −1 −1 c b cos B a cos A c b a
= (− 1) (− 1) − 1 1 −1 = −1 1 −1 ∆ = a c cos B b cos A = cos A cos B a c b
−1 −1 1 −1 −1 1 b a cos B c cos A b a c
a b c
Applying C 2 → C 2 + C1 and C 3 → C 3 + C1, then
Applying C1 ↔ C 3, then ∆ = − cos A cos B b c a
1 0 0
c a b
∆ = −1 0 − 2 = 1 ( 0 − 4) = − 4 1
−1 −2 0 = − cos A cos B (a + b + c ) ⋅ [(a − b ) 2 + (b − c ) 2 + (c − a ) 2 ]
2
12. Taking x 5 common from R 3, then Given, cos A ≠ 0, cos B ≠ 0 and a + b + c ≠ 0
Q ∆=0
xn xn+ 2 x 2n
∴ (a − b ) 2 + (b − c ) 2 + (c − a ) 2 = 0
x 5
1 xa a = 0, ∀ x ∈ R
which is independent, when a − b = 0, b − c = 0 and c − a = 0
xn xa + 1 x 2n i.e., a =b =c
⇒ a + 1 =n + 2 ⇒ a =n + 1 Hence, ∆ABC is an equilateral.
13. Since, x, y and z are in AP. 16. Here, x1 + x 2 = 6, x1x 2 = 3 …(i)

∴ 2y = x + z …(i) and y1 + y 2 = 4, y1y 2 = 2 
5 4 3
Let ∆ = x 51 y 41 z 31 x1x 2 y1y 2 1
x y z Let ∆ = x1 + x 2 y1 + y 2 2
π 
5 4 3 sin ( π x1x 2 ) cos  y1y 2  1
2 
= 100 x + 50 + 1 100y + 40 + 1 100z + 30 + 1
x y z 3 2 1
1 = 6 4 2 [from Eq. (i)]
Applying R2 → R2 − ( R 1 + R 3), then  π
2 sin 3 π cos   1
 4
5 0 3
= 100 x + 50 + 1 0 100z + 30 + 1 [from Eq. (i)] 3 2 1
x 0 z Applying R2 → R2 − 2 R 1, then ∆ = 0 0 0 =0
1
=0 [Q all elements of C 2 are zeroes] 0 1
2
582 Textbook of Algebra

10 10 11
C4 C5 Cm Now, subtracting Eq. (iv) from Eq. (v), we get
17. Q ∆ = 11
C6 11
C7 12
Cm + 2 f ′′′( 0 ) − f ′′ ( 0 ) g ′′′ ( 0 ) − g ′′ ( 0 ) h ′′′( 0 ) − h ′′ ( 0 )
12
C8 12
C9 13
Cm + 4 a b c
p q r
Applying C 2 → C 2 + C1 and use Pascal’s rule
( nCr + nCr − 1 = n + 1Cr ), then = 6n − 2r = 2 (3n − r )
cos ( x + α ) cos ( x + β ) cos ( x + γ )
10 11 11
C4 C5 Cm 20. Q f ( x ) = sin ( x + α ) sin ( x + β) sin ( x + γ )
∆= 11
C6 12
C7 12
Cm + 2 = 0 [given] sin ( β − γ ) sin ( γ − α ) sin (α − β )
12 13 13
C8 C9 Cm + 4
On differentiating both sides w.r.t. x , then
∴ m =5 − sin ( x + α ) − sin( x + β ) − sin ( x + γ )
1 sin (α − β ) θ cos (α − β ) θ f ′ (x ) = sin ( x + α ) sin ( x + β ) sin ( x + γ )
18. Let ∆ = a sin αθ cos αθ sin ( β − γ ) sin ( γ − α ) sin (α − β )
a sin (α − β ) θ cos (α − β ) θ
2
cos ( x + α ) cos ( x + β ) cos ( x + γ )
Applying R 1 → R 1 − R 3, then + cos ( x + α ) cos ( x + β ) cos ( x + γ )
sin ( β − γ ) sin ( γ − α ) sin ( α − β )
1 − a2 L 0 L 0
M sin( x + α ) sin ( x + β ) sin ( x + γ )
∆= a sin αθ cos αθ = − sin ( x + α ) sin ( x + β ) sin ( x + γ )
M sin ( β − γ ) sin ( γ − α ) sin ( α − β )
a2 sin(α − β ) θ cos (α − β ) θ cos ( x + α ) cos ( x + β ) cos ( x + γ )
+ cos ( x + α ) cos ( x + β ) cos ( x + γ )
Expanding along R 1, then
sin ( β − γ ) sin ( γ − α ) sin ( α − β )
sin αθ cos αθ
∆ = (1 − a ) 2
=0+0 [Q R 1 and R 2 are identical]
sin (α − β ) θ cos (α − β ) θ
=0
= (1 − a 2 ) [sin αθ ⋅ cos (α − β ) θ − cos αθ ⋅ sin (α − β ) θ ] ∴ f (x ) = c [constant]
Now, f (θ ) − 2 f ( φ ) + f ( ψ ) = c − 2c + c = 0
= (1 − a 2 ) sin (αθ − αθ + βθ ) = (1 − a 2 ) sin βθ
1 1 1
f ( x ) g ( x ) h( x ) 21. Let ∆ = a b c
19. Let F ( x ) = a b c = mx 4 + nx 3 + rx 2 + sx + t a3 b3 c3 1 1 1
p q r a b c
…(i) Taking a, b, c common from C1, C 2, C 3, then = abc 1 1 1
On differentiating twice and thrice of Eq. (i) w.r.t.x , then a2 b2 c2
f ′′ ( x ) g ′′ ( x ) h ′′ ( x ) On multiplying in R 1 by abc , then
F ′′ ( x ) = a b c bc ca ab 1 1 1
p q r ∆ = 1 1 1 = − bc ca ab [R1 ↔ R2 ]
= 12 mx 2 + 6nx + 2r …(ii) a2 b2 c2 a2 b2 c2

f ′′′ ( x ) g ′′′ ( x ) h ′′′ ( x ) 1 1 1


F ′′′ ( x ) = a b c = 24 mx + 6n …(iii) = a2 b2 c2 [R2 ↔ R 3]
p q r bc ca ab

On putting x = 0 in Eqs. (ii) and (iii), we get = (a − b ) (b − c ) (c − a ) (a + b + c )


1 1 1
f ′′ ( 0 ) g ′′ ( 0 ) h ′′ ( 0 )
Now, D = (x − a )2 (x − b )2 (x − c )2
a b c = 2r …(iv)
(x − b ) (x − c ) (x − c ) (x − a ) (x − a ) (x − b )
p q r
= (b − a ) (c − b ) (a − c ) ( 3 x − a − b − c )
f ′′ ( 0 ) g ′′ ( 0 ) h ′′ ( 0 ) Now, given that a, b and c are all different, then D = 0
and a b c = 6n …(v) ∴ 3x −a − b − c = 0
p q r 1
⇒ x = (a + b + c )
3
Chap 07 Determinants 583

22. Given, determinant Similarly, f [ f ( x )] − x < 0, ∀ x ∈ R


2a (bc − 4a 2 ) − b (b 2 − 2ac ) + c ( 2ab − c 2 ) = 0 Thus, roots of the equation f [ f ( x )] − x = 0 are imaginary
⇒ − [(2a ) 3 + b 3 + c 3 − 3 ⋅ 2a ⋅ b ⋅ c ] = 0 2 α δ
1 Let z= β 0 α
⇒ ( 2a + b + c ) [( 2a − b ) 2 + (b − c ) 2 + (c − 2a ) 2 ] = 0
2 γ β 1
⇒ 2a + b + c = 0 …(i) [Qb ≠ c ]
2 α δ 2 β γ 2 α δ
Let f ( x ) = 8ax 3 + 2bx 2 + c x + d
Then, z = β 0 α = α 0 β = β 0 α =z
 1 b c 2a + b + c
∴ f ( 0 ) = d and for   = a + + + d = +d γ β 1 δ α 1 γ β 1
 2 2 2 2
0 Hence, z is purely real.
= + d =d [from Eq. (i)]
2 25. For infinitely many solutions
 1 ∆ = ∆1 = ∆ 2 = ∆ 3 = 0
⇒ f (0) = f  
 2 2 −1 1
So, f ( x ) satisfies Rolle’s theorem and hence f ′ ( x ) = 0 has ∆ = 0 ⇒ 1 −2 1 = 0 ⇒ t =5
 1 t −1 2
atleast one root in 0, .
 2 
For t = 5, ∆1 = ∆ 2 = ∆ 3 = 0
x3+1 x 2y x 2z − 2t − 10 −5 − 10
23. Given, xy 2 y 3 + 1 y 2z = 11
Now, ∫0 f ( x ) dx = ∫
0
f ( x ) dx = ∫
0
f ( x ) dx + ∫− 5 f ( x ) dx

xz 2 yz 2 z3 + 1 =∫
− 10 − 10
−5
f ( x + 5 ) dx + ∫− 5 f ( x ) dx
Taking x, y , z common from C1, C 2, C 3 respectively, then − 10
x3+ 1 =∫ [ f ( x + 5 ) + f ( x )] dx
2 2 −5
x x
x − 10
y3 +1 =∫ 2dx = 2 ( − 10 + 5 )
−5
⇒ xyz y 2
y 2
= 11
y = − 10 = − 2t
2 2 z3 + 1 26. On putting x = 0, we get a 0 = 1
z z
z
On differentiating both sides w.r.t. x and putting x = 0 , we get
On multiplying R 1 by x , R2 by y and R3 by z, we get
a 1 = 4a
x3+ 1 x3 x3
On differentiating again w.r.t. x and putting x = 0, we get
⇒ y 3
y +1
3
y3 = 11
2a 2 = 12a 2 + 8b
z3 z3 z3+ 1
or a 2 = 6a 2 + 4b
Applying R 1 → R 1 + R2 + R 3, then
a1 a1 a 2
x 3 + y 3 + z 3 + 1 x3 + y 3 + z 3 + 1 x 3 + y 3 + z 3 + 1
Also, given a 0 a 2 a 0 = 0
y3 y3+1 y3 = 11
a 2 a 0 a1
z 3
z 3
z +1
3
⇒ − (a 30 + a 13 + a 32 − 3a 0 a1 a 2 ) = 0
Applying C 2 → C 2 − C1 and C 3 → C 3 − C1, then
1
x3+y3+z3+1 0 0 ⇒ (a 0 + a 1 + a 2 ) [(a 0 − a 1) 2 + (a 1 − a 2 ) 2
2
y3
O
0 = 11 + (a 2 − a 0) 2 ] = 0
1
z3 0 O Q a 0 + a1 + a 2 ≠ 0
1
∴ (a 0 − a 1) + (a 1 − a 2 ) + (a 2 − a 0) 2 = 0
2 2
⇒ x 3 + y 3 + z 3 + 1 = 11
⇒ a 0 − a 1 = 0, a 1 − a 2 = 0, a 2 − a 0 = 0
⇒ x 3 + y 3 + z 3 = 10
∴ a 0 = a1 = a 2
Therefore, the ordered triplets are (2, 1, 1 ), (1, 2, 1 ) and (1, 1, 2 ).
⇒ 1 = 4a = 6a 2 + 4b
24. Q f ( x ) − x = 0 has imaginary roots.
1 5
Then, f ( x ) − x > 0 or f ( x ) − x, 0, ∀ x ∈ R ⇒ a= and b =
4 32
for f ( x ) − x > 0, ∀ x ∈ R,
27. Q f ( x ) = log10 x and g ( x ) = e πi x
then f [ f ( x )] − f ( x ) > 0, ∀ x ∈ R
On adding, we get ∴ f (10 ) = log10 10 = 1
f [ f ( x )] − x > 0, ∀ x ∈ R and g(10 ) = e 10πi = ( − 1 )10 = 1
584 Textbook of Algebra

f (10 2 ) = log10 10 2 = 2 ⇒ (cos θ − 3 ) (2 + cos θ ) = 0


and g (10 ) = e2 100πi
= (− 1) 100
=1 cosθ = 3,−2, where −2 is neglected.
λ −1 cos θ
f (10 ) = log10 10 = 3
3 3
Hence, 3 1 2 > 0 only trivial solution is possible.
and g (10 3 ) = e 1000πi = ( − 1 )1000 = 1 cos θ 1 2
f (x ) ⋅ g (x ) [ f ( x )]g ( x ) 1
2 a2 + x2 ab ac
Given, φ( x ) = f ( x 2 ) ⋅ g ( x 2 ) [ f ( x 2 )]g ( x )
0 31. Q ∆ = ab b2 + x2 bc
3
f ( x 3 ) ⋅ g ( x 3 ) [ f ( x 3 )]g ( x )
1 ac bc c2 + x2
f (10 ) ⋅ g(10 ) [ f ( 10 )] g (10)
1 Taking a, b, c common from R 1, R 2 , R 3 respectively, then
g (102 )
∴ φ (10 ) = f (10 ) ⋅ g(10 ) [ f (10 )]
2 2 2
0 a2 + x2
3 b c
f (10 3 ) ⋅ g(10 3 ) [ f (10 3 )]g (10 )
1 a
b +x
2 2
1 1 1 ∆ = abc a c
b
= 2 2 0 =0 c 2 + x2
a b
3 3 1 c
1 ( α − α − 2x ) 2 ( α 2x + α − 2x ) 2
2x
On multiplying in C1, C 2, C 3 by a, b, c respectively, then
28. Let ∆ = 1 ( β 2x − β − 2x ) 2 ( β 2x + β − 2x ) 2 a2 + x2 b2 c2
1 ( γ 2x − γ − 2x ) 2 ( γ 2x + γ − 2x ) 2 ∆= a 2
b +x
2 2
c2
1 (α 2 x − α − 2 x )2 4 a2 b2 c2 + x2
Applying C 3 → C 3 − C 2, then ∆ = 1 ( β 2 x − β − 2 x ) 2 4 =0 Now, applying C1 → C1 + C 2 + C 3, then
1 (γ 2 x − γ − 2 x )2 4 x2 + a2 + b2 + c2 b2 c2
29. The given equations can be written as ∆= x +a +b +c b +x
2 2 2 2 2 2
c2
(a − 1 ) x − y − z = 0, x2 + a2 + b2 + c2 b2 c2+ x2
− x + (b − 1 ) y − z = 0 Applying R 2 → R 2 − R 1 and R 3 → R 3 − R 1, then
and − x − y + (c − 1 ) z = 0
x2 + a2 + b2 + c2 b2 c2
For non-trivial solution
a −1 −1 −1 ∆= 0 O 2
x 0
−1 b −1 −1 = 0 0 0 O x2
−1 −1 c −1 = x 4 (x 2 + a 2 + b 2 + c 2 )
Applying C1 → C1 − C 3 and C 2 → C 2 − C 3, then 6 2i 3+ 6
a 0 −1 32. Let ∆ = 12 3+ 8i 3 2+ 6i
0 b −1 = 0 18 2 + 12 i 27 + 2i
−c −c c −1
Applying R 2 → R 2 − 2 R1 and R 3 → R 3 − 3 R 1, then
Expanding along R 1, then
⇒ a (bc − b − c ) − 0 − 1 ( 0 + bc ) = 0 6 L 2i L 3+ 6
⇒ ab + bc + ca = abc M
λ −1 cos θ ∆= 0 3 −2 3 + 6i
30. For non-trivial solution 3 1 2 =0 M
cos θ 1 2 0 2 −3 2 + 2i
Applying R 3 → R 3 − R 2 , then Expanding along C1, we get
λ −1 cos θ 3 − 2 3 + 6i
= 6
M 2 − 3 2 + 2i
3 1 2 =0
= 6 [ − 3 6 + 2i 3 + 2 6 − 2i 3 ]
M
cos θ − 3 L 0 L 0 = 6 (− 6 ) = − 6 [real and rational]
n
Expanding along R 3, then 33. ∑ 2k − 1 = 1 + 2 + 22 + ... + 2n = 2n − 1
⇒ (cos θ − 3 ) ( − 2 − cos θ ) = 0 k =1
Chap 07 Determinants 585

n
1 n
1 1  i.e. a, b and c are in GP and ( x − α ) is a factor of
∑ k (k + 1) = ∑  k − k + 1 ax 2 + 2bx + c = 0.
k =1 k =1
1 1 n 2 cos x 1 0
= − =
1 n+1 n+1 35. Q f ( x ) = 1 2 cos x 1
0 1 2 cos x
 n + 1  nθ 
sin   θ sin  
n
 2   2 = 2 cos x ( 4 cos x − 1 ) − 1 ( 2 cos x − 0 ) + 0
2
and ∑ sin kθ =
 θ  = 2 cos x ( 4 cos2 x − 1 − 1 )
k =1 sin  
 2
= 4 cos x ( 2 cos2 x − 1 )
= 4 cos x cos 2 x
1
2k − 1 sin kθ = 2 (cos 3 x + cos x )
k (k + 1 )
x y z Option (a)
Dk =  n + 1  nθ   π  3π π  1
Given,
sin   θ sin   f   = 2  cos + cos  = 2  − 1 +  = − 1
n  2   2  3  3 3   2
2n − 1
n+1  θ Option (b)
sin  
 2 f ′( x ) = 2 ( − 3 sin 3 x − sin x )
 π  π  3
∴ f ′   = 2  − 3 sin π − sin  = 2  0 −  =− 3
n
 3  3   2 
n n
∑2 k −1 1
∑ k (k + 1) ∑ sin kθ
k =1 k =1
k =1 Option (c)
π
n π π sin 3 x 
∴ ∑ Dk = x y
 n + 1
z
 nθ 
∫0 f (x ) dx = 2 ∫0 (cos3x + cos x ) dx = 2  3
+ sin x

0
k =1 n sin   θ sin  
2n − 1  2   2 = 2 [( 0 + 0 ) − ( 0 + 0 )] = 0
n+1
 θ Option (d)
sin  
 2 π π π
∫0 f (x ) dx = 2 ∫− π (cos3x + cos x ) dx = 4 ∫0 (cos 3x + cos x ) dx
 n + 1  nθ 
sin   θ sin   =0 [from option (c)]
n  2   2
2n − 1 x 2 − 5x + 3 2x − 5 3
n+1  θ
sin   36. Q ∆ ( x ) = 3x 3 + x + 4 6x + 1
 2 9
= x y z =0 7 x 2 − 6 x + 9 14 x − 6 21
 n + 1  nθ 
sin   θ sin   Applying R2 → R2 − 3 R1 and R3 → R 3 − 7 R1, then
n  2   2
2n − 1
n+1  θ x 2 − 5x + 3 K 2x − 5 K 3
sin  
 2 M
a b aα + b = 16 x − 5 16 0
34. We have, b c bα + c = 0 M
aα + b bα + c 0 29 x − 12 29 0
Applying C 3 → C 3 − αC1 − C 2 , then 16 x − 5 16
Expanding along C 3, we get = 3
a b 0 29 x − 12 29
M Applying C1 → C1 − x C 2, then
b c 0 =0
− 5 16
M ∆ (x ) = 3 = 3 ( − 145 + 192 ) = 3 × 47
− 12 29
aa + b L ba + c L −(aa 2 + 2ba + c )
Expanding along C 3 , we get = 141 = ax 3 + bx 2 + cx + d [given]
− (aα 2 + 2bα + c ) (ac − b 2 ) = 0 ∴ a = 0, b = 0, c = 0, d = 141
⇒ (aα + 2bα + c ) (b − ac ) = 0
2 2
a2 b sin A c sin A
⇒ b 2 − ac = 0 and aα 2 + 2bα + c = 0 37. Q ∆ = b sin A 1 cos A
c sin A cos A 1
586 Textbook of Algebra

Taking common a from each R 1 and C1, then sec 2 x 0 1


b sin A c sin A M
1
a a 1 sin B sin C f ( x ) = cos2 x K cos2 x − cos4 x K cosec2 x
b sin A
∆= 1 cos A = sin B 1 cos A M
a
c sin A sin C cos A 1 1 0 cot 2x
cos A 1
a Expanding along C 2, then
[ by sine rule] sec 2 x 1
Applying C 2 → C 2 − sin B C1 and C 3 → C 3 − sin C C1, then f ( x ) = sin 2 x cos2 x
1 cot 2 x
1 L 0 L 0
= sin 2 x cos2 x ( cosec2 x − 1 )
M
= sin 2 x cos2 x cot 2 x = cos4 x
∆ = sin B 1 − sin 2 B cos A − sin B sin C
M option (a)
π /4 π /4 π /4
sin C cos A − sin B sin C 1 − sin C
2
∫− π /4 f (x ) dx = ∫− π /4 cos4 x dx = 2 ∫
0
cos4 x dx
2
Expanding along R 1, then π /4  1 + cos 2 x 
=2 ∫   dx
cos [ π − ( B + C )] 0  2 
cos2 B
∆= − sin B sin C 1 π /2  1 + cos x 
2

cos [ π − ( B + C )] − sin B sin C cos2 C =2 ×


2 ∫0 
 2
 dx

[Q A + B + C = π ] 1 π /2

4 ∫0
= (1 + 2 cos x + cos2 x ) dx
cos2 B − cos ( B + C ) − sin B sin C
=
− cos ( B + C ) − sin B sin C cos2 C 1 π /2 1 π /2 1 π /2
= ∫ 1 ⋅ dx + ∫ cos x dx + ∫ cos2 x dx
4 0 2 0 4 0
cos2 B − cos B cosC 1 π  1 1 1 π
=  − 0 + (sin x)0 + ⋅ ⋅
= π /2
− cos B cos C cos2 C 4 2  2 4 2 2
= cos2 B cos2 C − cos2 B cos2 C = 0 π 1 π 1 1
= + (1 − 0 ) + = (2π + 8 + π ) = (3π + 8)
8 2 16 16 16
a a2 0
option (b)
38. Q f (a, b ) = 1 ( 2a + b ) (a + b ) 2
Q f ′( x ) = 4 cos3 x ⋅ ( − sin x )
0 1 ( 2a + 3b )
π
∴ f′  = 0
Applying C 2 → C 2 − aC1, then  2
a K 0 K 0 option (c) and (d)
M Q 0 ≤ cos4 x ≤ 1
f (a, b ) = 1 (a + b ) (a + b ) 2 Q Maximum value of f ( x ) is 1.
M and minimum value of f ( x ) is 0.
0 1 (2a + 3b ) a a + x2 a + x2 + x4
40. Let ∆ = 2a 3a + 2 x 2 4a + 3 x 2 + 2 x 4
Expanding along R 1, then
3a 6a + 3 x 2 10a + 6 x 2 + 3 x 4
(a + b ) (a + b ) 2
f (a, b ) = a Applying R2 → R2 − 2 R 1 and R 3 → R3 − 3 R 1,then
1 ( 2a + 3b )
a a + x2 a + x2 + x4
1 (a + b ) ∆= 0 a 2a + x 2
= a (a + b )
1 ( 2a + 3b ) 0 3a 7a + 3 x 2
= a (a + b ) ( 2a + 3b − a − b ) Applying R 3 → R 3 − 3 R 2, then
= a (a + b ) (a + 2b ) a a + x2 a + x2 + x4
O
sec 2 x 1 1 ∆= 0 a 2a + x 2
39. Q f ( x ) = cos2 x cos2 x cosec2 x 0 0 O a
1 cos2 x cot 2 x
= a 3 = a 0 + a1 x + a 2 x 2 + a 3x 3 + a 4 x 4 + a 5 x 5
Applying C 2 → C 2 − cos x C1, then 2
+ a 6 x 6 + a 7 x 7 [given]
Chap 07 Determinants 587

∴ a 0 = a 3, a1 = 0, a 2 = 0, a 3 = 0, a 4 = 0, a 5 = 0, a 6 = 0, a 7 = 0 Applying C 3 → C 3 − x C 2 , then
and f ( x ) = a 0 x 2 + a 3 x + a 6 = a 3x 2 3 0 2a 2
= 3x 2a 2 2a 2x
option (a) f ( x ) ≥ 0 ⇒ a 3x 2 ≥ 0
3 x 2 + 2a 2 4a 2x 2a 2x 2 + 2a 4
If a 3 > 0, then x 2 ≥ 0
3 0 1
∴ a > 0, x ∈ R
option (b) If a = 0, then f ( x ) = 0 = 4a 4 3x 1 x
and If x = 0 , then f ( x ) = 0 3 x 2 + 2a 2 2 x x2 + a2
∴ Aliter (b) is fail Applying C1 → C1 − 3 C 3, then
option (c) f ( x ) = 0 0 0 1
⇒ a x = 0 or x = 0
3 2 2 M
∴ x = 0, 0 f ( x ) = 4a 4 0 1 x
option (d) For a = 0, f ( x ) = 0 is an identity, then it has more M
than two roots. − a 2 L 2x L x 2 + a 2
4x − 4 (x − 2)2 x3 Expanding along C1, we get
41. Let ∆ ( x ) = 8x − 4 2 ( x − 2 2 ) ( x + 1) 3
2
= 4a 4 [ − a 2 ( 0 − 1 )] = 4a 6
12 x − 4 3 ( x − 2 3 ) 2 ( x − 1 ) 3
∴ f ′ (x ) = 0
= a 0 + a1x + a 2 x 2 + ... …(i) i.e. y = f ( x ) is a straight line parallel to X -axis.
On putting x = 0 in Eq. (i), then 43. Qa > b > c and given equations are
−4 4 0 ax + by + cz = 0,
−4 2 8 1 = a0 bx + cy + az = 0
− 4 3 12 −1 and cx + ay + bz = 0
For non-trivial solution
or a 0 = − 4 ( − 8 − 12 ) − 4 ( 4 2 + 4 3 )
a b c
= 16 ( 5 − 2 − 3 ) = term independent of x in ∆. b c a =0
Also, on differentiating Eq. (i) w.r.t. x and then put x = 0, we c a b
get
⇒ 3abc − (a 3 + b 3 + c 3 ) = 0
4 −4 −40 4 0
∴ a+b+c=0
−4 2 8 1 + 8 −4 2 3
If α and β be the roots of at 2 + bt + c = 0
−4 3 12 −1 −4 3 12 −1
b c
−4 4 0 ∴ α + β = − and αβ =
a a
+−4 2 8 1 = a1 and D = b 2 − 4ac = ( − a − c ) 2 − 4ac = (a − c ) 2 > 0
12 −4 3 3
For opposite sign | α − β | > 0
∴ a1 = 4 ( − 8 − 12 ) + 4 ( 4 2 + 4 3 ) ⇒ (α − β ) 2 > 0 ⇒ (α + β ) 2 − 4 αβ > 0
− 4 ( 4 2 − 36 ) − 4 ( − 8 + 12 3 ) b 2 4c
⇒ − > 0 ⇒ ( − a − c ) 2 − 4ac > 0
a2 a
− 4 ( 24 + 4 3 ) − 4 ( − 12 2 − 12 )
⇒ (a − c ) 2 > 0, true
= 48 + 48 2 − 48 3 = 48 (1 + 2 − 3 )
Hence, the roots are real and have opposite sign.
= Coefficient of x in ∆( x ) 1 1 1
3 3x 3 x 2 + 2a 2 44. Here, ∆ = 1 3 2 = 1( 9 − 2λ ) − 1 ( 3 − 2) + 1 ( λ − 3)
42. Q f ( x ) = 3x 3 x 2 + 2a 2 3 x 3 + 6a 2x 1 λ 3
3 x + 2a
2 2
3 x 3 + 6a 2x 3 x 4 + 12a 2x 2 + 2a 4 = − (λ − 5)
Applying C 3 → C 3 − x C 2 and C 2 → C 2 − x C1, then 3 1 1
∆1 = 6 3 2 = 3( 9 − 2 λ ) − 1 (18 − 2b ) + 1 ( 6 λ − 3b )
3 0 2a 2
b λ 3
f (x ) = 3x 2a 2 4a 2x
3 x + 2a
2 2
4a 2x 6a 2x 2 + 2a 4 = − (b − 9 )
588 Textbook of Algebra

1 3 1 47. The system is good, if


∆2 = 1 6 2 = 1 ∆ = ∆1 = ∆ 2 = ∆ 3 = 0
1 b 3 ⇒ λ = 5 and µ = 13
(18 − 2b ) − 3 (3 − 2 ) + 1 (b − 6 ) = − (b − 9 ) 48. The system is lazy, if
1 1 3 ∆ = 0 and atleast one of ∆1, ∆ 2, ∆ 3 ≠ 0
and ∆ 3 = 1 3 6 = 1 ⇒ λ = 5 and µ ≠ 13
1 λ b Sol. (Q. Nos. 49 to 51)
2
bc − a 2 ca − b 2 ab − c 2 a b c
(3b − 6 λ ) − 1 (b − 6 ) + 3 ( λ − 3 ) = (2b − 3 λ − 3 )
Aliter (a) for unique solution ∆ ≠ 0 Q ca − b ab − c bc − a = b c a
2 2 2
...(i)
i.e. λ ≠ 5 , b ∈R ab − c 2 bc − a 2 ca − b 2 c a b
Aliter (b) for no solution For a = 1, b = x and c = x 2
D = 0 and atleast one of ∆1, ∆ 2, ∆ 3 is non-zero 2
∴ λ =5,b ≠9 x3 −1 0 x − x4 1 x x2
Aliter (c) For infinite many solution 0 x −x 4
x −1 = x x2
3
1
∆ = ∆1 = ∆ 2 = ∆ 3 = 0 x − x4 x −1
3
0 x2 1 x
∴ λ = 5, b = 9 ∴ ∆ = 5 2 = 25
45. For non-trivial solutions 49. Q ∆2 = (25) 2 = 625
λ sin α cosα Sum of digits of ∆2 = 6 + 2 + 5 = 13
1 cosα sin α =0 50. From Eq. (i), we get
−1 sin α − cosα A B C a b c
2

Expanding along C1, we get B C A = b c a


⇒ λ ( − cos2 α − sin 2 α ) − 1 ( − sin α cosα − sin α cosα ) C A B c a b
− 1 (sin 2 α − cos2 α ) = 0 a b c
2

⇒ − λ + sin 2α + cos2α = 0 ⇒ 49 = b c a
c a b
⇒ λ = (sin 2α + cos2α )
a b c
Q − 2 ≤ sin 2α + cos2α ≤ 2
⇒ q b c a = ±7
∴ − 2≤λ≤ 2 c a b
⇒ S = [ − 2, 2 ] ⇒ −(a + b + c 3 − 3abc ) = ±7
3 3

Sol. (Q. Nos. 46 to 48) ⇒ a 3 + b 3 + c 3 − 3abc = m 7


1 1 1 ∴ a 3 + b 3 + c 3 − 3abc = 7 [Qa + b + c > 0]
∆= 1 2 3 = ( λ − 5 ), a b c
1 3 λ 51. Q aA + bB + cC = b c a = −(a 3 + b 3 + c 3 − 3abc )
5 1 1 c a b
∆1 = 9 2 3 = ( λ + µ − 18 ), = −( −3 ) = 3
µ 3 λ Sol. (Q. Nos. 52 to 54)
Q α + β + γ = −2, αβ + βγ + γα = −1 and α β γ = 3
1 5 1
α β γ α β γ
∆2 = 1 9 3 = ( 4 λ − 2µ + 6 )
52. γ α β =− β γ α = α 3 + β3 + γ 3 − 3 α β γ
1 µ λ
β γ α γ α β
1 1 5
= (α + β + γ )(α 2 + β 2 + γ 2 − αβ − βγ − γα )
and ∆ 3 = 1 2 9 = ( µ − 13 )
1 3 µ = (α + β + γ )[(α + β + γ ) 2 − 3(αβ + βγ + γα )]
= ( −2 )( 4 − 9 ) = 10
46. The system is smart, if
α −1 2x + 1
∆ ≠0 ⇒ λ ≠5 53. Let x = ⇒α =
α +2 1−x
or ∆ = ∆1 = ∆ 2 = ∆ 3 = 0
Q α is a root of x 3 + 2 x 2 − x − 3 = 0
⇒ λ = 5 and µ = 13
⇒ α 3 + 2α 2 − α − 3 = 0
Chap 07 Determinants 589

 2x + 1
3
 2x + 1  2x + 1
2 4 10   9 15 
⇒   + 2  −  −3 = 0 55. Q f (2) + f (3) =  − + 2 +  − + 2 = 1
 1−x   1−x   1−x  4 4  4 4 

⇒ x 3 + 6 x 2 + 21 x − 1 = 0 ...(i) x 2 5x
56. Q f ( x ) + 1 = 0 ⇒ − +3 = 0
α −1 β −1 γ −1 4 4
Hence, , and are the roots of Eq. (i), then 25 23
α +2 β+2 γ+2 Q D = −3 = − < 0
α −1 β −1 γ −1 16 16
+ + = −6 ∴ Number of solutions = 0
α +2 β+2 γ+2
 25 
α −1 β −1 γ −1 6 m −  − 2
∴ + + = = D  16  7
α +2 β+1 γ+2 1 n 57. Minimum value of f ( x ) = − =− =
4a 1 16
⇒ m = 6 and n = 1, 7 
,∞
16 
Hence, range of f ( x ) is
m n2 6 1
then = = 42 − 5 = 37
m −n m + n 5 7 Sol. (Q. Nos. 58 to 60)
a b b a β γ
2 Put x = 1 on both sides, we get
1 1 0
54. Q b a b = β γ α = (α + β + γ − 3 αβγ ) 3 3 3 2
1 1 0 = a0 ⇒ 0 = a0
b b a γ α β
tan 1 sin 2 1 cos2 1
= (α + β + γ ) 2[(α + β + γ ) 2 − 3(αβ + βγ + γα )]2
we observe that
= ( −2 ) 2[( −2 ) 2 + 3 ]2 = 4 × 49 = 196 a1 = f ′(1 )
Sol. (Q. Nos. 55 to 57) x e x −1 (x − 1)3
2ax 2ax − 1 2ax + b + 1 where f ( x ) = x − ln x cos( x − 1 ) ( x − 1 ) 2
Q f ′( x ) = b b +1 −1 tan x sin 2 x cos2 x
2(ax + b ) 2ax + 2b + 1 2ax + b
1 e x −1 3( x − 1 ) 2
Applying C 2 → C 2 − C1 and C 3 → C 3 − C1, then
f ′( x ) = x − ln x cos( x − 1 ) ( x − 1 ) 2
2ax −1 b +1
tan x sin 2 x cos2 x
f ′( x ) = b 1 −1 −b
2ax + 2b 1 −b x e x −1 (x − 1)3
1
Applying R3 → R3 − R1 , then + 1− − sin( x − 1 ) 2( x − 1 )
x
2ax −1 b +1 tan x sin 2 x cos2 x
f ′( x ) = b 1 −1 − b
x e x −1 (x − 1)3
2b 2 −2b − 1
+ x − ln x cos( x − 1 ) ( x − 1 ) 2
Applying R3 → R3 − 2 R2 , then
sec 2 x sin 2 x − sin 2 x
2ax −1 b +1
M 1 1 0 1 1 0
f ′( x ) = b 1 −1 − b ⇒ f ′(1 ) = 1 1 0 + 0 0 0
M tan 1 sin 2 1 cos2 1 tan 1 sin 2 1 cos2 1
0 L 0 L 1 1 1 0
⇒ f ′( x ) = (2ax + b ) + 1 1 0
∴ f ( x ) = ax 2 + bx + c sec 2 1 sin 2 − sin 2

f (0) = 2 ⇒ c = 2 ...(i) = 0+ 0+ 0 = 0
and f ( 1 ) = 1 ⇒ a + b + 2 = 1 ⇒ a + b = −1 ...(ii) ∴ a1 = 0
π
Also,
 5
f ′   = 0 ⇒ 5a + b = 0 ...(iii) 58. cos−1(a1 ) = cos−1( 0) =
 2 2
From Eqs. (ii) and (iii), we get
59. Let P = lim(sin x ) x = lim(sin x ) x
x→a 0 x→ 0
1 5
a = and b = − ∴ ln p = lim x ln sin x
4 4 x→0
[form (0 × ∞)]
x 2 5x lnsin x cot x
∴ f (x ) = − +2 = lim = lim [ by L′ Hospital’s Rule]
4 4 x→ 0 Yx x→ 0 −Yx 2
590 Textbook of Algebra

= − lim
x2
= −1 × 0 = 0
∴ ∑ a 2b = a 2b + a 2c + b 2a + b 2c + c 2a + c 2b
x→ 0 tan x
= (a + b + c )(ab + bc + ca ) − 3abc
∴ P = e0 = 1 = 3 p − 3r
60. Required Equation is = 6 2 − 3r
( x − a 0 )( x − a1 ) = 0
[Q (a + b + c) 2 = a 2 + b 2 + c 2 + 2(ab + bc + ca )]
⇒ ( x − 0 )( x − 0 ) = 0
= 3(2 2 − r ) [∴p 2 = 8 ⇒ p = 2 2 ]
⇒ x2 = 0
Sol. (Q. Nos. 61 to 63) Sol. (Q. Nos. 64 to 66)
Multiplying R1, R2, R3 by a,b, c respectively and then taking a,b, c Taking a,b, c common from R1, R2, R3 respectively and then
common from C1, C 2, C 3, multiplying by a, b, c is C1, C 2, C 3 respectively, we get
we get a2 + n b2 c2
−bc ab + ac ac + ab ∆n = a 2 b2 + n c2
∆ = ab + bc −ac bc + ab a2 b2 c2 + n
ac + bc bc + ac −ab Applying C1 → C1 + C 2 + C 3, then
Applying C 2 → C 2 − C1 and C 3 → C 3 − C1 and then taking n + a2 + b2 + c2 b2 c2
(ab + bc + ca ) from C 2 and C 3, we get
∆n = n + a 2 + b 2 + c 2 b 2 + n c2
−bc 1 1
n+a +b +c
2 2 2
b 2
c +n
2
∆ = (ab + bc + ca ) ab + bc − 1
2
0
ac + bc 0 −1 Applying R2 → R2 − R1 and R3 → R3 − R1, then

n + a2 + b2 + c2 b2 c2
Applying R1 → R1 + R2 + R3, we get O
ab + bc + ca L 0 L 0 ∆n = 0 n 0
M O
= (ab + bc + ca ) 2 ab + bc −1 0 0 0 n
M
∆n = n + n (a + b + c )
3 2 2 2 2
...(i)
ac + bc 0 −1
Also, a + b + c = λ
−1 0
= (ab + bc + ca ) 3 = (ab + bc + ca ) 3 3b = λ [Qa,b, c are in AP ]
0 −1 λ
∴ b=
Also, a, b and c are the roots of 3
x 3 − px 2 + qx − r = 0 Also, b is root of x − λx + 11 x − 6 = 0
3 2

Q a + b + c = p, ab + bc + ca = q, abc = r ⇒ b 3 − λb 2 + 11b − 6 = 0
⇒ ∆ = q3 ...(i) λ3 λ3 11 λ
⇒ − + −6 = 0
61. Q AM ≥ GM 27 9 3
 ab + bc + ca  ⇒ 2 λ3 − 99 λ + 162 = 0
⇒   ≥ (ab ⋅ bc ⋅ ca )
1/3
 3  Q λ =6
q Then, equation becomes x − 6 x + 11 x − 6 = 0
3 2
⇒ ≥ (r 2 )1 / 3 ⇒ q 3 ≥ 27r 2
3 ∴ x = 1, 2, 3
or ∆ ≥ 27r2 Let a = 1, b = 2 and c = 3
[from Eq. (i)]
From Eq. (i), we get
62. Q a,b and c are in GP.
∆n = n 3 + 14n 2
∴ mb 2 = ac ⇒ b 3 = abc = r ⇒ b = r 1/3
and b is a root of x 3 − px 2 + qx − r = 0
n
n(n + 1 )(3n 2 + 59n + 28 )
∴ ∑ ∆n = 12
⇒ b 3 − pb 2 + qb − r = 0 n =1

⇒ r − pr 2 /3
+ qr 1 / 3 − r = 0
7
7 ⋅ 8(147 + 59 ⋅ 7 + 28 )
64. ∑ ∆r = 12
= (14 ) 3
⇒ p r =q r
3 2 3
r =1

∴ q =pr
3 3 ∆ 2n 8(n + 7 )
65. = <8
63. Q ∆ = 27 ⇒q = 27 3 ∆n (n + 14 )
∆ 2n
∴ q =3 ∴ <8
∆n
or ab + bc + ca = 3 and a 2 + b 2 + c 2 = 2
Chap 07 Determinants 591

66. Q ∆ r = r 3 + 14r 2 x e x −1 (x − 1)3


27 ∆r − ∆ 3r 28 69. We have, x − ln x cos ( x − 1 ) ( x − 1 ) 2
∴ =
27r 2 3 tan x sin 2 x cos2 x
 27 ∆r − ∆ 3r  28
30
⇒ ∑  27r 2 
 = × 30 = 280
3
= a 0 + a1 ( x − 1 ) + a 2 ( x − 1 ) 2 + ... + an ( x − 1 )n …(i)
r =1
On putting x = 1 in Eq. (i) , we get
32 + k 42 32 + 3 + k 1 1 0
67. We have, 42 + k 52 42 + 4 + k = 0 1 1 0 = a 0 + 0 + 0 + ...
52 + k 62 52 + 5 + k tan 1 sin 2 1 cos2 1
Applying C 3 → C 3 − C1, then ∴ a0 = 0 [Q R1 and R2 an identical]
On differentiating Eq. (i) both sides w.r.t.x, then
3 +k 4
2 2
3
42 + k 52 4 =0 1 e x −1 3 (x − 1)2
52 + k 62 5 x − ln x cos ( x − 1 ) ( x − 1 ) 2
tan x sin 2 x cos2 x
Applying R2 → R2 − R 1 and R 3 → R 3 − R 1, then
x e x −1 (x − 1)3
9 + k 16 3  1
9 1 =0 + 1 −  − sin( x − 1 ) 2( x − 1 )
7  x
16 20 2 tan x sin 2 x cos2 x
x −1
⇒ (9 + k ) (18 − 20 ) − 16 (14 − 16 ) + 3 (140 − 144 ) = 0 x e (x − 1)3
⇒ − 18 − 2k + 32 − 12 = 0 ⇒ 2k = 2 + x − ln x cos ( x − 1 ) ( x − 1 ) 2
sec 2 x sin 2 x − sin 2 x
∴ k =1
1 1 1
+ + + ... + ∞
= 0 + a1 + 2a 2 ( x − 1 ) + 3a 3 ( x − 1 ) 2 + ... + nan ( x − 1 )n − 1
Now, 2k
2k
2k ... ∞ = (2k ) 2 4 8 Now, on putting x = 1, we get
1
1 1 0 1 1 0
2
1−
1 1 1 0 + 0 0 0
= (2k ) 2 = 2k = 2 1 = 2 tan 1 sin 2 1 cos2 1 tan 1 sin 2 1 cos2 1
x −1 − 6 2 1 1 0
68. We have, −6 x −2 − 4 = 0 + 1 1 0
2 − 4 x −6 sec 2 1 sin 2 − sin 2
Applying C 2 → C 2 + 3 C 3, then = a 1 + 0 + 0 + ... + 0
x −1 0 2 ∴a1 = 0 + 0 + 0 = 0
a1 + 1
− 6 x − 14 −4 =0 Hence, (2a 0 + 3a 1 ) = ( 2 0 + 3 0 ) 0 + 1 = (1 + 1 )1 = 21 = 2
2 3 x − 22 x − 6 1 cos α cos β 0 cos α cos β
Expanding along R 1, then 70. Given, cos α 1 cos γ = cos α 0 cos γ
( x − 1 ) {( x − 14 ) ( x − 6 ) + 4 (3 x − 22 )} − 0 + 2 cos β cos γ 1 cos β cos γ 0
{ − 18 x + 132 − 2 x + 2 } = 0 ⇒ 1 (1 − cos γ ) − cos α (cos α − cos β cos γ )
2

⇒ ( x − 1 ) ( x − 8 x − 4 ) + 2 ( − 20 x + 160 ) = 0
2
+ cos β (cos γ cos α − cos β )
⇒ x − 9 x − 36 x + 324 = 0
3 2 = 0 − cosα ( 0 − cosβ cos γ ) + cosβ (cos γ cosα − 0 )
⇒ 1 − cos2 α − cos2 β − cos2 γ
⇒ (x − 9) (x − 6) (x + 6) = 0
∴ x = 9 or 6 or − 6 + 2 cos α cos β cos γ = 2 cos α cos β cos γ
Now, let α = 9, β = 6 , γ = − 6 ⇒ 1 − cos α − cos2 β − cos2 γ = 0
2

1 1 1 1 1 1 1 Hence, cos2 α + cos2 β + cos2 γ = 1


∴ + + = + − =
α β γ 9 6 6 9
−1
(b + c ) 2 a2 a2
 1 1 1 71. Q f (a, b, c ) = b 2
(c + a ) 2
b2
∴  + +  =9
 α β γ c2 c2 (a + b ) 2
592 Textbook of Algebra

Applying C 2 → C 2 − C1 and C 3 → C 3 − C1, then 3π


⇒ θ= [Qθ ∈[ 0, π ] ]
4
(b + c ) 2 a 2 − (b + c ) 2 a 2 − (b + c ) 2

f (a, b, c ) = b2 (c + a ) 2 − b 2 0 Hence, =6
π
c2 0 (a + b ) 2 − c 2 1 1 1 1
(b + c ) 2 (a + b + c ) (a − b − c ) (a + b + c ) (a − b − c ) 1 2 3 4
73. Let ∆=
= b2 (c + a + b ) (c + a − b ) 0 1 3 6 10
c 2
0 (a + b + c ) (a + b − c ) 1 4 10 20
Applying R2 → R2 − R 1, R3 → R3 − R 1 and R4 → R4 − R 1, then
(b + c ) 2 a − b − c a − b − c
= (a + b + c ) 2 b2 c + a −b 0 1 L 1 L 1 L 1
c2 0 a + b −c M
0 1 2 3
Applying R 1 → R 1 − ( R 2 + R 3), then
2bc − 2c − 2b ∆= M
f (a, b, c ) = (a + b + c ) 2 b 2 c + a − b 0 0 2 5 9
c2 0 a + b −c M
0 3 9 19
1 1
Applying C 2 → C 2 + C1 and C 3 → C 3 + C1, then
b c 1 2 3
2bc L 0 L 0 Expanding along C1, then ∆ = 2 5 9
M 3 9 19
b2 1 2 3
f (a, b, c ) = (a + b + c ) 2 b
2
c+a
c
M Applying R2 → R2 − 2 R1 and R3 → R 3 − R 1, then = 0 1 3
c2 0 3 10
c2 a+b
b 1 3
Expanding along C1, we get ∆ = 1 = 10 − 9 = 1
Expanding along R 1, then 3 10
f (a, b, c ) = (a + b + c ) 2 [2bc {(c + a ) (a + b ) − bc }] 1+a 1 1 1
= (a + b + c ) { 2bc (ac + bc + a + ab − bc )}
2 2 1 1+b 1 1
74. Let ∆ =
1 1 1+c 1
= 2bc (a + b + c ) a (a + b + c )
2

1 1 1 1+d
= 2abc (a + b + c ) 3
Taking a, b, c, d common from R 1, R 2 , R 3 and R 4 respectively,
We get, greatest integer n ∈ N such that (a + b + c )n divides
then
f (a, b, c ) is 3. 1
1 1 1
72. The system of equations has a non-trivial solution, then 1+
a a a a
1 − sin θ − cos θ 1 1 1 1
1+
− cos θ 1 −1 =0 ∆ = abcd b b b b
− sin θ − 1 1 1 1 1
1 1+
c c c c
Applying C 3 → C 3 + C 2, then 1 1 1 1
1+
1 L − sin θ L − sin θ − cos θ d d d d
M Applying R1 → R 1 + R 2 + R 3 + R 4 and taking
− cos θ 1 0 =0  1 1 1 1
1 + + + +  common, we get
M  a b c d
− sin θ −1 0  1 1 1 1
∆ = abcd 1 + + + + 
Expanding along C 3, then  a b c d
1 1 1 1
( − sin θ − cos θ ) (cos θ + sin θ ) = 0
1 1 1 1
⇒ (sin θ + cos θ ) 2 = 0 1+
b b b b
⇒ sin θ + cos θ = 0 1 1
1+
1 1
⇒ sin θ = − cos θ c c c c
1 1 1 1
∴ tan θ = − 1 1+
d d d d
Chap 07 Determinants 593

Applying C 2 → C 2 − C1, C 3 → C 3 − C1 and C 4 → C 4 − C1, then ⇒ x ⋅ 0 + y ⋅ 0 + g x + fy + c + λ = 0 [from Eqs. (i) and (ii)]
 1 1 1 1 ⇒ gx + fy + c + λ = 0 …(iv)
∆ = abcd 1 + + + + 
 a b c d
According to the question Eqs. (i), (ii) and (iii) has unique
1 0 0 0
1 O solution. So, Eqs. (i), (ii) and (iv) has unique solution,
1 0 0
b O a h g
1 then h b f =0
0 1 0
c
1
O g f c+λ
0 0 1
d ⇒ a (bc + bλ − f 2 ) − h (ch + hλ − fg ) + g (hf − bg )
 1 1 1 1
= abcd 1 + + + +  1 ⋅ 1 ⋅ 1 ⇒ (abc + 2 fgh − af 2 − bg 2 − ch 2 ) = λ(h 2 − ab )
 a b c d
= abcd + (bcd + acd + abd + abc ) = σ 4 + σ 3 abc + 2 fgh − af 2 − bg 2 − ch 2
or =λ
16  8 h 2 − ab
= + −  = 8
1  1
According to the question, λ = 8
1+a 1 1
77. (A) → ( p, r); (B) → ( p, r); (C) → ( p, q, s, t)
75. Given, 1 + b 1 + 2b 1 =0
(A) Using a 2 + b 2 + c 2 = 0, we get
1+c 1 + c 1 + 3c
Taking a, b, c common from R 1, R 2 and R 3 respectively, then b2 + c2 ab ac −a 2 ab ac
1 1 1 ∆= ab c +a
2 2
bc = ab −b 2 bc
1+
a a a ac bc a + b2
2
ac bc −c 2
1 1 1
abc 1 + 2+ =0 −a a a
b b b
1+1 1+
1 1 = abc b −b b
3+
c c c c c −c
 1 1 1
Applying R 1 → R 1 + R2 + R3 and taking 3 + + + 
 a b c [taking a, b, c common from C1, C 2, C 3 respectively]
common, we get Applying C 2 → C 2 + C1 and C 3 → C 3 + C1, then
1 1 1 −a ... 0 ... 0
 1 1 1 1 1 1
abc 3 + + +  1 + 2+ =0 M
 a b c b b b ∆ = abc b 0 2b
1 1
1+ 1+ 3+1 M
c c c
Applying C 2 → C 2 − C1 and C 3 → C 3 − C1, then c 2c 0
1 L 0 L 0 = (abc ) ( −a ) ( −4 bc ) = 4a b 2 c 2 2

M
∴ λ=4
 1 1 1 1 + 1 1 −1 = 0
abc 3 + + +  a a +b a +b +c
 a b c 2
M (B) Let ∆ = 2a 5a + 2b 7a + 5b + 2c
1
1+ 0 2 3a 7a + 3b 9a + 7b + 3c
c
Expanding along R 1, we get Applying R2 → R2 − 2 R1 and R3 → R3 − 3 R1, then
 1 1 1 a L a +b L a +b +c
2 abc 3 + + +  = 0
 a b c M
Q a ≠ 0, b ≠ 0, c ≠ 0 ∆= 0 3a 5a + 3b
1 1 1
∴ + + = − 3 or | a − 1 + b − 1 + c − 1| = 3 M
a b c 0 4a 6a + 4b
76. Given equations 3a 5a + 3b
ax + hy + g = 0, …(i) =a
4a 6a + 4b
hx + by + f = 0 …(ii)
= a(18a 2 + 12ab − 20a 2 − 12ab )
and ax + 2hxy + by + 2 g x + 2 fy + c + λ = 0
2 2
…(iii)
= −2a 3 = −1024 [given]
Eq. (iii), can be written as ⇒ a = 512 = 8 3
3

x (ax + hy + g ) + y (hx + by + f ) + gx + fy + c + λ = 0 ∴ a =8
594 Textbook of Algebra

x − 1 2x 2 − 5 x 3 − 1 D (a + b ) 2 − 4a1b1
Minimum value of f ( x ) = − =− 1 1
(C) Let ∆( x ) = 2 x 2 + 5 2 x + 2 x 3 + 3 ...(i) 4a 4( −1 )
x3 −1 x + 1 3x 2 − 2 (a1 − b1 ) 2 36
= = =9
4 4
According to the question,
(C)Q f ( x ) is a polynomial of degree atmost 6 in x .
∆( x ) = ( x 2 − 1 ) P ( x ) + ax + b
If f ( x ) = a 0 + a1x + a 2 x 2 + a 3x 3 + a 4 x 4 + a 5x 5 + a 6x 6
∴ ∆(1 ) = a + b and ∆( −1 ) = −a + b ...(ii)
From Eq. (i), we get ⇒ λ = a1 = f ′( 0 )
0 L 3 L 0 1 1 0 −2 −2 0 −2 1 0
M = 1 0 1 + −1 0 1 + −1 0 3
∆(1 ) = 7 4 4 = 3(7 − 0 ) = 21 1 1 8 0 2 8 0 1 12
M = −8 − 12 + 18 = −2
0 2 1 ∴ |λ|= 2
79. (A) → (r); (B) → (r, t); (C) → ( p, q, s)
−2 L −3 L 2
x 2 + 3x x −1 x +3
M
(A) Let f ( x ) = x 2 + 1 2 + 3x x −3
and ∆( −1 ) = 7 0 2 = 3(7 + 4 ) = 33
x2 −3 x+4 3x
M
−2 0 1 f ( x ) = ax 4 + bx 3 + cx 2 + dx + e ...(i)
0 −1 3
From Eq. (ii), a + b = 21 and −a + b = 33,
∴ e = f ( 0 ) = 1 2 −3 = 0 + 1 ( 0 − 9 ) + 3 ( 4 + 6 ) = 21
we get a = −6, b = 27
−3 4 0
∴ 4a + 2b = −24 + 54 = 30
Dividing both sides of Eq. (i) by x 4 i.e., C1 by x 2, C 2 by x
78. (A) → ( p, s, t); (B) → (r, t); (C) → ( p, q)
and C 3 by x and then taking lim , we get
1 1 1 1 1 1 x→ ∞
1 1 1
(A)Q ∆ = f1( x1 ) f1( x 2 ) f1( x 3 ) = f1(2 ) f1(3 ) f1(5 )
a = 1 3 1 = 1 (8 ) − 1 (2 ) + 1 ( −2 ) = 4
f 2( x1 ) f 2( x 2 ) f 2( x 3 ) f 2(2 ) f 2(3 ) f 2(5 )
1 1 3
1 1 1
Hence, e + a = 25
= 2 + a1 3 + a1 5 + a1
x −1 5x 7
4 + 2b1 + b2 9 + 3b1 + b2 25 + 5b1 + b2
(B) Let f ( x ) = x 2 − 1 x − 1 8 = ax 3 + bx 2 + cx + d ...(i)
Applying C 2 → C 2 − C1 and C 3 → C 3 − C1, then 2x 3x 0
1 L 0 L 0
M 1 0 7 −1 5 7 −1 0 0
∆= 2 + a1 1 3 ∴ c = f ′( 0 ) = 0 −1 8 + −1 1 8 + −1 −1 0
M 2 0 0 0 3 0 0 0 0
4 + 2b1 + b2 5 + b1 21 + 3b = 2( 0 + 7 ) − 3 ( −8 + 7 ) + 0 = 17
1 3 Dividing both sides of Eq. (i) by x 3 i.e., c1 by x 2, c 2 by x and
=
5 + b1 21 + 3b1 taking lim , we get
x→∞
= 21 + 3b1 − 15 − 3b1 = 6 0 5 7
1 b1 a1 M
(B)Q f ( x ) = 1 b1 2a1 − x a = 1 L 1 L 8 = −1 ( 0 − 21 ) = 21
1 2b1 − x a1 M
Applying R2 → R2 − R1 and R3 → R3 − R1, then 0 3 0
1 L b1 L a1 Hence, c + a − 3 = 35
M
x 3 + 4x x + 3 x −2
f (x ) = 0 0 a1 − x
(C) Let g( x ) = x − 2 5x x −1
M
x −3 x +2 4x
0 b1 − x 0
= −(a1 − x )(b1 − x ) = − x 2 + (a1 + b1 ) x − a1 b1 = ax 5 + bx 4 + cx 3 + dx 2 + ex + f
Chap 07 Determinants 595

0 3 −2 1 0 1 −1 0 1 1 −1
i.e., = 1, = 0, = −1, = 2,
∴ f = g( 0 ) = −2 0 −1 = 0 − 3 ( 0 − 3 ) − 2 ( −4 − 0 ) = 17 −1 1 0 0 1 −1 1 1
−3 2 0 −1 1
= −2 ∴ n = 5 ⇒ (n − 1 ) 2 = 16
4 3 −2 0 1 −2 0 3 1 1 1
and e = g ′( 0 ) = 1 0 −1 + −2 5 −1 + −2 0 1 (B) There are only three determinants of second order with
1 2 0 −3 1 0 −3 2 4 negative value
0 1 1 1 0 1
= 1 − 23 + 11 = −11 , ,
Hence, f + e = 17 − 11 = 6 1 0 1 0 1 1

80. (A) → ( p, q, r); (B) → ( p, q, r, s, t); (C) → ( p, q, r, s, t) Number of possible determinants with elements 0 and 1
are 2 4 = 16. Therefore, number of determinants with
(A) Taking common a, b, c from R1, R2 and R3 respectively and non-negative values is 13.
then multiplying in C1, C 2 and C 3 by a, b, c respectively, then ∴ n = 13
a + (b 2 + c 2 )d b 2(1 − d ) c 2(1 − d ) ⇒ (n − 1 ) = 12
∆= a (1 − d )
2
b + (c + a )d
2 2 2
c 2(1 − d ) (C) There are only four determinants of second order with
negative value
a 2(1 − d ) b 2(1 − d ) c 2 + (a 2 + b 2 )d
−1 1 1 1 −1 −1 1 −1
, , ,
Applying C1 → C1 + C 2 + C 3, then 1 1 1 −1 −1 1 −1 −1
1 b 2(1 − d ) c 2(1 − d ) ∴ n = 4 ⇒ n(n + 1 ) = 20
∆ = 1 b 2 + (c 2 + a 2 )d c 2(1 − d ) 82. Statement-1
1 b 2(1 − d ) c + (a 2 + b 2 )d
2 r r +1
∆ (r ) = = r (r + 4 ) − (r + 1 ) (r + 3 )
r +3 r +4
[Q a 2 + b 2 + c 2 = 1 ]
= (r 2 + 4r ) − (r 2 + 4r + 3 ) = − 3
Applying R2 → R2 − R1 and R3 → R3 − R1, then n n
1 b 2(1 − d ) c 2(1 − d ) ∴ Σ ∆(r ) = rΣ=1(− 3)
r =1
O
= ( − 3 ) + ( − 3 ) + ( − 3 ) + ... + ( − 3 ) = − 3n
∆= 0 d 0 =d2 1444442444443
n times
O ⇒ Statement-1 is true.
0 0 d Statement-2
[Q a 2 + b 2 + c 2 = 1 ] f (r ) f 2(r )
∆ (r ) = 1 = f1(r ) f 4 (r ) − f 2(r ) f 3(r )
(B) Multiplying C1 by a, C 2 by b and C 3 by c, then f 3(r ) f 4 (r )
a b (a + b )
− n n

1
c
(b + c )
c
b c
c ∴ ∑ ∆ (r ) = ∑ [ f1(r ) f 4 (r ) − f 2(r ) f 3(r )]
r =1 r =1
∆= −
abc a a a n n


bd (b + c ) bd (a + 2b + c )

(a + b )bd = ∑ [ f1(r ) f 4 (r )] − ∑ [ f 2 (r ) f 3(r )] …(i)
ac ac ac r =1 r =1
n n
Applying C1 → C1 + C 2 + C 3, then
∑ f1(r ) ∑ f 2(r )
b (a + b ) r =1 r =1
0 − and n n
c
1 b c
c
∑ f 3(r ) ∑ f 4 (r )
∆= 0 =0 r =1 r =1
abc a a
bd (a + 2b + c ) (a + b )bd  n   n   n   n 
0 − =  ∑ f1(r )  ∑ f 4 (r ) −  ∑ f 2(r )  ∑ f 3(r ) …(ii)
ac ac        
r = 1  r = 1  r = 1  r = 1 
(C) Applying C 3 → C 3 − cosd C1 − sin d C 2 , then n n

sin a cosa 0 n ∑ f1(r ) ∑ f 2(r )


∑ ∆ (r ) ≠
r =1 r =1
∆ = sin b cosb 0 =0 From Eqs. (i) and (ii), we get n n
sin c cosc 0
r =1
∑ f 3(r ) ∑ f 4 (r )
r =1 r =1
81. (A) → ( p, r); (B) → ( p, q, r, t); (C) → ( p, r, s)
∴ Statement-2 is false.
(A) Possible values are −2 , −1, 0, 1, 2 and numbering
Hence, Statement-1 is true and Statement-2 is false.
determinant = 3 4 = 81
596 Textbook of Algebra

2 3
a1 + b1x 2 a1x 2 + b1 c1 86. Here, ∆= = 8 − 3b,
83. Q ∆ = a + b x 2 a x 2 + b c b 4
2 2 2 2 2
a 3
a 3 + b 3x 2 a 3x 2 + b 3 c3 ∆1 = = 4a − 15
2
5 4
a1 b1 c1 1 x 0
…(i) 2 a
= a 2 b2 c 2 × x 2 1 0 and ∆2 = = 10 − ab
b 5
a 3 b3 c 3 0 0 1
For infinite solutions, ∆ = ∆1 = ∆ 2 = 0
Statement-1 If ∆ = 0, then
15 8
a1 b1 c1 1 x2 0 We get, a = and b =
4 3
a 2 b2 c 2 × x 2 1 0 = 0
∴ Statement-1 is true and if lines a1x + b1y + c1 = 0
a 3 b3 c 3 0 0 1
and a 2 x + b2 y + c 2 = 0 are parallel, then
1 x2 0 a 1 b 1 c1
= ≠
M a 2 b 2 c2
⇒ x2 1 0 = 0 ⇒ 1 − x = 0 or x = 1
4 4
∴ Statement-2 is true, but in Statement-1
M
[Q x ≠ − 1]
2
2 3 a
0 L 0 L 1 = =
b 4 5
Statement-1 is true 3 3 3
a1 b1 c1 ⇒ = =
4 4 4
Now, if a 2 b2 c 2 = 0, then [ both equation are identical]
a 3 b3 c 3 ∴ Statement-2 is not a correct explanation for Statement-1.
∆=0 [from Eq. (i)]
1 2 3
Statement-2 is also true.
87. Q 4 5 6 = 1 ( 0 − 48) − 2 ( 0 − 42) + 3 (32 − 35)
Hence, both the statements are true but Statement-2 is not a
correct explanation of Statement-1. 7 8 0
84. Statement-2 is always true for Statement-1 = − 48 + 84 − 9
 π π π  π  = 84 − 57 = 27 ≠ 0
cos  x +  = cos  −  − x  = sin  − x
 4 2 4  4  ∴ Statement-1 is true.
 π Also, in given determinant neither two rows or columns are
= − sin  x − 
 4 identical, Statement-2 is true, Statement-2 is not a correct
explanation for Statement-1.
π  π π  π 
cot  + x = cot  −  − x  = tan  − x 88. Q A88, 6B 8, 86 C are divisible by 72, then A88 = 72 λ, 6B 8 = 72 µ
4  2 4  4 
 π  and 86 C = 72 υ, where λ , µ, υ ∈ N .
= − tan  x − 
 4 A 6 8
y   x 8 B 6
Also, ln   = − ln   Q
 x y  8 8 C
Therefore, determinant given in Statement-1 is skew-symmetric Applying R 3 → R 3 + 10 R 2 + 100 R 1, then
and hence its value is zero. Hence, both statements are true and
Statement-2 is a correct explanation of Statement-1. A 6 8
(1 + x )
11
(1 + x ) 12
(1 + x ) 13 8 B 6
85. (1 + x ) 21 (1 + x ) 22 (1 + x ) 23 = A0 + A1x + A2 x 2 + ... [ let] 100 A + 80 + 8 600 + 10 B + 8 800 + 60 + c
(1 + x ) 31 (1 + x ) 32 (1 + x ) 33 A 6 8 A 6 8
On differentiating both sides w.r.t.x and then put x = 0, we get = 8 B 6 = 72 8 B 6 …(i)
11 12 13 1 1 1 1 1 1 72 λ 72 µ 72 υ λ µ υ
1 1 1 + 21 22 23 + 1 1 1 = 0 + A1 + 0 + 0 + ... Now, A88 is also divisible by 9, then
1 1 1 1 1 1 31 32 33 A + 8 + 8 = A + 16 is divisible by 9
∴ A =2
⇒ 0 + 0 + 0 = A1 ∴ A1 = 0
and 6 B 8 is also divisible by 9, then 6 + B + 8 = B + 14 is
∴ Coefficient of x in f ( x ) = 0
divisible by 9
Both statements are true, Statement-2 is a correct explanation
∴ B=4
of Statement-1.
Chap 07 Determinants 597

From Eq. (i), we get 13 + 3 2 5 5


2 6 8 1 3 4 91. Let ∆ = 15 + 26 5 10
= 72 8 2 6 = 288 4 1 3 = 288 [integer] 3 + 65 15 5
λ µ υ λ µ υ
3 2 5 5 13 2 5 5
Statement-1 is true and Statement-2 is false. = 15 5 10 + 26 5 10
b+c c b
3 15 5 65 15 5
89. Let ∆= c c +a a
b a a+b Taking common from 1st determinant 3, 5 and 5 from
Applying R 1 → R 1 − ( R 2 + R 3), then C1, C 2 and C 3 respectively and taking common from 2nd
0 − 2a − 2a determinant 13, 5 and 5 from C1, C 3 and C 3 respectively,
∴ ∆ = c c +a a we get 1 2 1
b a a+b = 3× 5× 5 5 5 2 + 13 × 5 × 5
Taking ( −2a ) common from R1, then 3 3 5 1 2 1
0 1 1
2 5 2
∆ = ( − 2a ) c c + a a
1 2 1 5 3 5
b a a+b
= 3 ×5 5 5 2 +0 [QC1 and C 3 are identical]
Applying C 2 → C 2 − C 3, then
0 0 1 3 3 5
∴ ∆ = ( −2a ) c c a 1 2 1
b −b a + b =5 3 5 5 2
3 3 5
Expanding along R 1, we get
c c Applying C 2 → C 2 − C1,
∆ = ( − 2a ) ⋅ 1 ⋅ = ( − 2a ) ( − 2bc ) 1 1 1
b −b
then ∆ =5 3 5 0 2
Hence, ∆ = 4abc
3 0 5
a − b −c 2a 2a
90. Let ∆ = 2b b − c −a 2b Expanding along C 2, then
c −a − b 5 2
2c 2c ∆ = 5 3 ⋅ (−1) = − 5 3 (5 − 6 )
3 5
Since, the answer is (a + b + c ) 3, we shall try to get (a + b + c ).
= − 25 3 + 15 2
Applying R1 → R 1 + R2 + R3, then
a + b + c a + b + c a + b +c = 15 2 − 25 3
∆= 2b b − c −a 2b 92. Given that, a, b and c are p th, q th and r th terms of HP ⇒ ,
1 1
2c 2c c −a − b a b
1
Taking (a + b + c ) common from R 1, we get and are p th, q th and r th terms of an AP. Let A and D are the
c
1 1 1 first term and common difference of AP, then
∆ = (a + b + c ) 2b b − c − a 2b 1
= A + ( p −1)D …(i)
2c 2c c −a − b a
1
Applying C1 → C 2 − C1 and C 3 → C 3 − C1 = A + (q − 1 ) D …(ii)
b
1 0 0 1
∴ ∆ = (a + b + c ) 2b − a − b − c 0 = A + (r − 1 ) D …(iii)
c
2c 0 − c −a − b Now, given determinant is
[by property, since all elements above 1 1 1
bc ca ab
leading diagonal are zero] a b c
= (a + b + c ) ⋅ 1 ⋅ ( − a − b − c ) ⋅ ( − c − a − b ) ∆ = p q r = abc p q r
Hence, ∆ = (a + b + c ) 3 1 1 1 1 1 1
598 Textbook of Algebra

1 1 1 Expanding along R1, then


On substituting the values of , and from Eqs. (i), (ii) and
a b c sin B − sin A
∆=
(iii) in ∆, then (sin B − sin A ) (sin B + sin A + 1 )
A + ( p − 1 ) D A + ( q − 1 ) D A + (r − 1 ) D sin C − sin A
∆ = abc p q r (sin C − sin A ) (sin C + sin A + 1 )
1 1 1 = (sin B − sin A )(sin C − sin A )
Applying R 1 → R 1 − ( A − D ) R 3 − DR 2 , then 1 1
sin B + sin A + 1 sin C + sin A + 1
0 0 0
∆ = abc p q r = 0 = (sin B − sin A ) (sin C − sin A ) (sin C − sin B )
1 1 1 But, given ∆ = 0
3 ∴ (sin B − sin A ) (sin C − sin A ) (sin C − sin B ) = 0
−5 3 + 5i − 4i
2 ∴ sin B − sin A = 0 or sin C − sin A = 0
93. Let z = 3 − 5i 8 4 + 5i or sin C − sin B = 0
3
+ 4i 4 − 5i 9 ⇒ sin B = sin A or sin C = sin A or sin C = sin B
2 B = A or C = A or C = B
3 In all the three cases, we will have an isosceles triangle.
−5 3 − 5i + 4i
2 βγ βγ ′ + β′ γ β′ γ′
Then, z = 3 + 5i 8 4 − 5i [i.e., conjugate of z]
96. Let ∆ = γ ′ α γ ′ α ′ + γ′ α γ′ α ′
3
− 4i 4 + 5i 9 αβ αβ ′ + α ′β α ′β′
2
3 Taking β′ γ ′ , γ ′ α ′ and α ′β′ common from R1, R2 and R3
−5 3 + 5i − 4i
2 respectively, then
= 3 − 5i 8 4 + 5i β γ β γ
3 + 1
+ 4i 4 − 5i 9 β′ γ ′ β′ γ ′
2 γ α γ α
[interchanging rows into columns] ∆ = ( β′ γ ′ ) ( γ ′ α ′ ) (α ′ β′ ) + 1
γ ′ α′ γ ′ α′
⇒ z =z α β α β
+ 1
Hence, z is purely real. α ′ β′ α ′ β′
ah + bg g ab + ch Applying R2 → R2 − R 1 and R3 → R3 − R 1
94. LHS = bf + ba f hb + bc
β γ β γ
af + bc c bg + fc + 1
β′ γ ′ β′ γ ′
ah + bg g a ah + bg g h
γ α β α β
= b bf + ba f h + c bf + ba f b Then, ∆ = (α ′β′ γ ′ ) 2  −   −  0

γ ′ α ′ β′   α ′ β′ 
af + bc c g af + bc c f
β α γ α γ
In second determinant, applying C1 → C1 − bC 2 − aC 3, then  −   −  0
β′  α ′ γ ′   α′ γ ′ 
ah + bg bg a 0 g h
= bf + ba bf h + c 0 f b β γ β γ
+ 1
β′ γ ′ β′ γ ′
af + bc bc g 0 c f
α β  α γ γ
In first determinant, applying C 2 → C 2 − C1, then = (α ′β′ γ ′ ) 2  −   −  1 0
 α ′ β′   α ′ γ ′  γ ′
ah + bg − ah a ah + bg a h β
= bf + ba − ba h + 0 = a bf + ba h b = RHS 1 0
β′
af + bc − af q af + bc g f Expanding along C 3, then
1 1 1
α β  α γ  γ β
95. Let ∆ = 1 + sin A 1 + sin B 1 + sin C ∆ = (α ′β′ γ ′ ) 2  −   −   − 
 α ′ β′   α ′ γ ′   γ ′ β′ 
sin A + sin A sin B + sin B sin C + sin 2 C
2 2

Applying C 2 → C 2 − C1and C 3 → C 3 − C1, then α β  β γ  γ α


= (α ′β′ γ ′ ) 2  −   −   − 
1 0  α ′ β′   β′ γ ′   γ ′ α ′ 
∆= 1 + sin A sin B − sin A
(αβ′ − α ′β ) (βγ′ − β′ γ ) ( γα ′ − γ ′ α )
sin A + sin 2 A (sin B − sin A ) (sin B + sin A + 1) = (α ′β′ γ ′ ) 2
(α ′β′ γ ′ ) 2
0
sin C − sin A Hence, ∆ = (αβ′ − α ′ β ) ( βγ ′ − β′ γ ) ( γα ′ − γ ′ α )
(sin C − sin A ) (sin C + sin A + 1 )
Chap 07 Determinants 599

97. Since, y =
u sin ( x + α ) cos ( x + α ) sin α
v = sin ( x + β ) cos ( x + β ) sin β
du dv
v −u sin ( x + γ ) cos ( x + γ ) sin γ

dy dx dx vu′ − uv ′
= =
dx v 2
v2 Applying C1 → C1 − (cos x ) C 3 and C 2 → C 2 + (sin x ) C 3, we get
dy sin x cosα cos x cosα sin α
⇒ v2 = vu′ − uv ′ …(i)
dx ∆′( x ) = sin x cos β cos x cosβ sin β
On differentiating both sides w.r.t. x, we get sin x cos γ cos x cos γ sin γ
d 2y dy cos α cos α sin α
v2 2 + ⋅ 2vv ′ = (vu ′′ + u′ v ′ ) − (uv ′′ + v′ u′ )
dx dx = sin x ⋅ cos x cos β cos β sin β
d 2y dy cos γ cos γ sin γ
⇒ v2 2
+ 2vv ′ = vu ′′ − uv ′′
dx dx = sin x ⋅ cos x × 0 [QC1 and C 2 are identical]
On multiplying both sides by v, then =0
d 2y  dy  Thus, ∆( x ) is independent of x.
v3 + 2v ′ v 2  = v 2u ′′ − uvv ′′ x (x − 1) x (x − 1) (x − 2)
dx 2  dx  x
x
C1 x
C2 C3 x
1 ⋅2 1 ⋅2 ⋅3
d 2y y (y − 1 ) y (y − 1 ) (y − 2 )
⇒ v3 + 2v ′ (vu ′ − uv ′ ) = v 2u ′′ − uvv ′′ [from Eq. (i)] 99. Let ∆ = y
C1 y
C2 yC3 = y
dx 2 z z 1 ⋅2 1 ⋅2 ⋅3
C1 C2 z C3 z (z − 1 ) z (z − 1 ) (z − 2 )
2 z
3d
y
⇒ v = 2uv 2 − uvv′′ − 2vu′ v′+ v 2u ′′ …(ii) 1 ⋅2 1 ⋅2 ⋅3
dx 2
1 x − 1 x − 3x + 2
2
v v 0 xyz
= 1 y − 1 y 2 − 3y + 2
and v′ v′ v = u (2v′ − vu ′′ ) − v(2u′ v ′− u ′′ v )
2
12
1 z − 1 z 2 − 3z + 2
v ′′ v ′′ 2v ′′
Applying C 2 → C 2 + C1, then
= 2uv ′ 2 − 4vv ′′ − 2vu′ v ′ + v 2 u ′′ …(iii)
1 x x 2 − 3x + 2
From Eqs. (ii) and (iii), we get xyz
∆= 1 y y 2 − 3y + 2
u v 0 12
2
3d y
1 z z 2 − 3z + 2
v = u′ v′ v
dx 2 Applying C 3 → C 3 + 3C 2 − 2C1, then
u′ v ′′ 2v ′
1 x x2
98. Here, we have to prove that ∆( x ) is independent of x . So, it is xyz 1
sufficient to prove that ∆′( x ) = 0 ∆= 1 y y2 = xyz ( x − y ) (y − z ) (z − x )
12 2 12
sin ( x + α ) cos ( x + α ) a + x sin α 1 z z
Now, ∆( x ) = sin ( x + β ) cos ( x + β ) b + x sin β 1 + sin 2 x cos2 x 4 sin 2 x
sin ( x + γ ) cos ( x + γ) c + x sin γ 100. (i) Q f (x ) = 2
sin x 1 + cos x 2
4 sin 2 x
On differentiating w.r.t. x, we get sin 2 x cos2 x 1 + 4 sin 2 x
cos ( x + α ) cos ( x + α ) a + x sin α Applying R2 → R2 − R1 and R3 → R3 − R1, then
∆′( x ) = cos ( x + β ) cos ( x + β ) b + x sin β 1 + sin 2 x cos2 x 4 sin 2 x
cos ( x + γ ) cos ( x + γ ) c + x sin γ f (x ) = −1 1 0
sin ( x + α ) − sin ( x + α ) a + x sin α −1 0 1
+ sin ( x + β ) − sin ( x + β ) b + x sin β Applying C 2 → C 2 + C1, then
sin ( x + γ ) − sin ( x + γ ) c + x sin γ 1 + sin 2 x 2 4 sin 2 x
sin ( x + α ) cos ( x + α ) sin α M
+ sin ( x + β ) cos ( x + β ) sin β f (x ) = −1 … 0 … 0
sin ( x + γ ) cos ( x + γ ) sin γ M
−1 −1 1
sin ( x + α ) cos ( x + α ) sin α
= 0 − 0 + sin ( x + β ) cos ( x + β ) sin β Expanding along R2 , then
2 4 sin 2 x
sin ( x + γ ) cos ( x + γ ) sin γ f (x ) = = 2 + 4 sin 2 x
−1 1
600 Textbook of Algebra

∴ Maximum value of Z1 = tX 1 + Xt1 …(iii)


f ( x ) = 2 + 4 (1 ) = 6 and Z 2 = tX 2 + Xt 2 + 2 X 1t1 …(iv)
sin 2 A sin A cos A cos2 A X Y Z
(ii) Q ∆ = sin 2 B sin B cos B cos2 B LHS = X 1 Y1 Z1
sin 2 C sin C cos C cos2 C X 2 Y2 Z2
tan 2 A tan A 1 X sX tX
= cos A cos B cos C tan 2 B tan B 1
2 2 2
= X1 sX 1 + Xs1 tX 1 + Xt1
tan 2 C tan C 1 X 2 sX 2 + Xs 2 + 2 X 1s1 tX 2 + Xt 2 + 2 X 1t1
1 tan A tan 2 A [using Eqs. (i), (ii), (iii) and (iv)]
= − cos2 A cos2 B cos2 C 1 tan B tan 2 B Applying C 2 → C 2 − sC1 and C 3 → C 3 − tC1, then
1 tan C tan 2 C
X 0 0
= − cos2 A cos2 B cos2 C (tan A − tan B ) = X1 Xs 1 Xt 1
(tan B − tan C ) (tan C − tan A ) X2 Xs 2 + 2 X 1s 1 Xt 2 + 2 X 1t1
= − sin ( A − B ) sin( B − C ) sin (C − A ) Expanding w.r.t. R1, then
= sin ( A − B ) sin ( B − C ) sin ( A − C ) ≥ 0 [Q A ≥ B ≥ C ]
s1 t1
∴ ∆≥0 = X2
Xs 2 + 2 X 1s 1 Xt 2 + 2 X 1t1
Hence, minimum value of ∆ is 0.
Applying R2 → R2 − 2 X 1R1, then
x 2 − 4 x + 6 2 x 2 + 4 x + 10 3 x 2 − 2 x + 16
s1 t1 s1 t1
101. Let f ( x ) = x −2 2x + 2 3x − 1 = X2 = X3 = RHS
Xs 2 Xt 2 s 2 t2
1 2 3
On differentiating w.r.t. x , we get 103. Given determinant may be expressed as
f g
2x − 4 4x + 4 6x − 2
∆= xf ′ + f xg′ + g
f ′ (x ) = x − 2 2x + 2 3x − 1
( x 2 f ′′ + 4 xf ′ + 2 f ) ( x 2g ′′ + 4 x g ′ + 2 g )
1 2 3 h
x 2 − 4 x + 6 2 x 2 + 4 x + 10 3 x 2 − 2 x + 16 xh′ + h
+ 1 2 3 ( x h′′ + 4 xh′ + 2h )
2

1 2 3 Now, applying R3 → R3 − 4 R2 + 2 R 1, then


f g h
x 2 − 4 x + 6 2 x 2 + 4 x + 10 3 x 2 − 2 x + 16
∆ = xf ′+ f xg ′ + g xh′ + h
+ x −2 2x + 2 3x − 1
x 2 f ′′ x 2g ′′ x 2h ′′
0 0 0
f g h
f ′ ( x ) = 0, ∀ x ∈ R and f ( x ) = Constant Applying R2 → R2 − R1, then ∆ = xf ′ xg ′ xh′
6 10 16 x 2 f ′′ x 2g ′′ x 2h ′′
As, f (0) = − 2 2 − 1 = 2∴ f (x ) = 2 f g h
1 2 3 ⇒ ∆ =x f′ g′ h′
3x 2 sin x 3 x 2 sin x x f ′′ x g ′′
2 2
x 2h ′′
Now, I =∫ f ( x ) dx = 2 ∫− 3 1 + x 6 dx
− 3 1 + x6 f g h
x 2 sin x ⇒ ∆ = f′ g′ h′
Let g( x ) =
1 + x6 x 3 f ′′ x 3g ′′ x 3h ′′
− x 2 sin x f′ g h′ f g h
∴ g( − x ) = = − g( x )
1 + x6 ∴ ∆′ = f ′ g′ h′ + f ′′ g ′′ h ′′
Hence, g is an odd function. x 3 f ′′ x 3g ′′ x 3h ′′ x 3 f ′′ x 3g ′′ x 3h ′′
∴ I =0 f g h
102. Since, Y = X and Z = tX + f′ g′ h′
( x 3 f ′′ )′ ( x 3g′′ )′ ( x 3h ′′ )′
Y1 = sX 1 + Xs1 …(i)
Y2 = sX 2 + Xs 2 + 2 X 1s1 …(ii)
Chap 07 Determinants 601

f g h ∆ = 0 is a cubic equation in t.
=0+0+ f′ g′ h′ So, it has in general three solutions t1, t 2 and t 3.
( x 3 f ′′ )′ ( x 3g ′′ )′ ( x 3h ′′ )′ Let ∆ = a 0t 3 + a1t 2 + a 2 t + a 3
f g h Clearly, a 0 = Coefficient of t 3= −1,
a a
Hence, ∆′ = f′ g′ h′ so t1t 2 t 3 = − 3 = − 3 = a 3 = Constant term in the expansion
a0 −1
( x 3 f ′′ ) ′ ( x 3g′′ ) ′ ( x 3h ′′ ) ′ of ∆. i.e. ∆ (at t = 0)
a b c
104. Let the given determinant be equal to zero. Then, there exist
x, y and z not all zero, such that ∴ t1 t 2 t 3 = a 3 = b c a
a1x + a 2 y + a 3z = 0, b1x + b2 y + b3z = 0 c a b
and c1x + c 2 y + c 3z = 0 107. (i) Eliminating a, b and c from given equations, we obtain
Assume that, | x | ≥ | y | ≥ | z | and x ≠ 0. Then, from y z
a1x = ( − a 2 y ) + ( −a 3z ) −1
z y
∴ | a1x | = | − a 2 y − a 3z | ≤ | a 2 y | + | a 3z | z x
⇒ | a1| | x | ≤ | a 2 | | y | + | a 3 | | z | −1 =0
x z
But x ≠ 0 i.e. | a1 | ≤ | a 2 | + | a 3 | x y
−1
Similarly, | b2 | ≤ | b1 | + | b3 | y z
| c 3 | ≤ | c1 | + | c 2 | Applying R2 → R2 − R1 and R3 → R3 − R1, then
which is contradiction. Hence, the assumption that the y z
determinant is zero must be wrong. −1
z y
(a − a1 ) − 2 (a − a1 ) − 1 a1− 1 z y x z
0 − − =0
105. LHS = (a − a 2 ) − 2 (a − a 2 ) − 1 a 2− 1 x z z y
(a − a 3 ) − 2 (a − a 3 ) − 1 a 3− 1 0
x y y z
− −
y z x y
1 (a − a1 ) a1− 1 (a − a1 ) 2
Expanding along C1, then
= (a − a1 ) −2 −2
(a − a 2 ) (a − a 3 ) −2
1 (a − a 2 ) a 2− 1 (a − a 2 ) 2
 z y  y z   x y   x z 
1 (a − a 3 ) a 3− 1 (a − a 3 ) 2 − −   −  + −   −  =0
 x x  x y  y z   z y 
Applying R2 → R2 − R 1 and R3 → − R3 − R 1, then yz zx xy
⇒ + + +1=0
x2 y 2 z 2
1 a1− 1 (a − a1 ) 2
(a − a1 ) (ii) To eliminate x, y and z.
(a − a1a 2 ) (a1 − a 2 )
2 y z x
LHS =
1
0 (a1 − a 2 ) Let α = , β = and γ = in the given equations,
Π (a − ai ) 2
a1a 2 z x
c
y
(a 2 − a1a 3 ) (a1 − a 3 ) bα + = a, …(i)
0 (a1 − a 3 ) α
a1a 3 a
Expanding w.r.t. 1st column, then cβ + = b …(ii)
β
(a − a1a 2 ) (a1 − a 2 )
2
b
(a1 − a 2 ) and aγ + = c …(iii)
LHS =
1 a1a 2 γ
Π (a − ai )2 (a1 − a 3 )
(a 2 − a1a 3 ) (a1 − a 3 ) Also, αβγ = 1
a1a 3 From Eqs. (i), (ii) and (iii), we get
a 2 − a1a 2  c  a  b
1 bα +  cβ +  aγ +  = abc
(a − a 2 ) (a1 − a 3 ) a1a 2  α  
β  γ
= 1
Π (a − ai )2 1 a 2 − a1a 3
βγ αγ αβ
a1a 3 ⇒ 2abc + ac 2 + a2b + b 2c
α β γ
(a1 − a 2 ) (a1 − a 3 ) a 2 (a 2 − a 3 ) − a Π (ai − a j )
2
= = γ β α
a1a 2 a 3 Π (a − ai ) 2
Π ai Π (a − ai )2 + a 2c + bc 2 + ab 2 = abc
αβ γα βγ
Numerator = − a 2 (a1 − a 2 ) (a 2 − a 3 ) (a 3 − a1 ) 1 1 [Qαβγ = 1]
⇒ ac 2 2 + a 2b 2
The resulting expression has negative sign. α β
106. The given system of equation will have a non-trivial solution 2 1
in the determinant of coefficients. + b c 2 + a 2c γ 2 + bc 2 β 2 + ab 2 α 2 = − abc
γ
a −t b c
 c2   a2   b2 
∴ ∆= b c −t a ⇒a  2 + b 2 α 2  + b  2 + β 2c 2  + c  2 + a 2 γ 2  = − abc
α  β  γ 
c a b −t
…(iv)
602 Textbook of Algebra

On squaring Eqs. (i), (ii) and (iii), we get Applying R2 → R2 − R1 and R3 → R3 − R1, then
c2 a2
b 2 α 2 + 2 = a 2 − 2bc, c 2 β 2 + 2 = b 2 − 2ca and 1 (1 + b 2 ) x (1 + c 2 ) x
α β
b2 = 0 1−x 0 = (1 − x 2 )
a γ + 2 = c 2 − 2ab
2 2
γ 0 0 1−x
On putting these values in Eq. (iv), we get Hence, degree of f ( x ) = 2
a (a 2 − 2bc ) + b (b 2 − 2ca ) + c (c 2 − 2ab ) = − abc
a 3 + b 3 + c 3 = 5abc 110. For no solution or infinitely many solutions
a b c α 1 1
108. Here, ∆ = b c a . According to the question, x, y and z not 1 α 1 =0
c a b 1 1 α
all zero. Hence, the given system of equations has non-trivial Applying C1 → C1 + C 2 + C 3, then
solution. α +2 1 1
∆=0
a b c α +2 α 1 = 0
b c a =0 α+2 1 α
c a b Applying R1 → R2 − R1 and R3 → R3 − R1, then

1
(a + b + c ) [(a − b ) + (b − c ) + (c − a ) 2 ] = 0
2 2 α +2 1 1
2 0 α −1 0 = 0 ⇒ (α − 1 ) 2 (α + 2 ) = 0
∴ a+b+c=0
0 0 α −1
or (a − b ) 2 + (b − c ) 2 + (c − a ) 2 = 0
∴ α = 1, − 2
Case I If a + b + c = 0
From first two equations, For α = 1, clearly there an infinitely many solutions and when
we put α = −2 in given system of equations and adding them
ax + by − (a + b )z = 0 together LHS ≠ RHS. i.e., no solution.
bx − (a + b ) y + ax = 0
111. Qa1, a 2, a 3,... are in GP.
[by cross-multiplication law]
∴ Using an = a1 r n −1, we get the given determinant, as
x y z
∴ = = log(a1r n −1 ) log(a1r n ) log(a1r n + 1 )
ab − (a + b ) 2 − b (a + b ) − a 2 − a (a + b ) − b 2 n+ 2
log(a1r ) log(a1r n + 3 ) log(a1r n + 4 )
x y z
⇒ = = log(a1r n + 5 ) log(a1r n + 6 ) log(a1r n + 7 )
− (a + ab + b ) − (a + ab + b ) − (a + ab + b 2 )
2 2 2 2 2

∴ x :y :z =1:1:1 Applying C 2 → C 2 − C1 and C 3 → C 3 − C1 and


Case II If (a − b ) + (b − c ) + (c − a ) 2 = 0
2 2
 m
using log m − log n = log   , we get
It is possible only, when n
a − b = 0, b − c = 0 and c − a = 0 log(a1r n −1 ) log r 2 log r
Then, a =b =c
log(a1r n + 2 ) log r 2 log r = 0
In this case all the three equations reduce in the forms
x+y +z =0 …(i) log(a1r n + 5 ) log r 2 log r
Then, Eq. (i) will be satisfied, if [QC 2 and C 3 are proportional]
x = k, y = k ω, z = k ω 2 112. Applying R2 → R2 − R1 and R3 → R3 − R1, then
or x = k , y = k ω 2, z = k ω 1 1 1
where ω is the cube root of unity. D= 0 x 0 = xy
Then, x : y : z = 1 : ω : ω 2 or 1 : ω 2 : ω 0 0 y
1 −2 3
Hence, combined both cases, we get
113. Q D = −1 1 −2 = 0
x :y :z =1:1:1
1 −3 4
or 1 : ω : ω2
−1 −2 3
or 1 : ω2 : ω and D1 = k 1 − 2 = (3 − k ) = 0, if k = 3
109. Applying C1 → C1 + C 2 + C 3, then 1 −3 4
1 (1 + b ) x (1 + c ) x
2 2 1 −1 3
f ( x ) = 1 1 + b 2x (1 + c 2 ) x [Q a 2 + b 2 + c 2 + 2 = 0 ] D2 = − 1 k − 2 = (k − 3 ) = 0, if k = 3
1 (1 + b 2 ) x 1 + c 2x 1 −3 4
Chap 07 Determinants 603

−1 −2 3 ⇒1( −3 + k ) + k( − k + 3k ) + 1(k − 9 ) ≠ 0
D3 = k 1 − 2 = (k = 3 ) = 0, if k = 3 ⇒ 2k 2 + 2k − 12 ≠ 0
1 −3 4 ⇒ k2 + k −6 ≠ 0
∴ System of equations has no solution for k ≠ 3. ⇒ (k + 3 )(k − 2 ) ≠ 0
114. The system of equations ⇒ k ≠ 2, − 3
or k ∈ R − {2, − 3 }
x − cy − bz = 0, − cx + y − az = 0 and − bx − ay + z = 0
have non-trivial solution, if k +1 8
119. ∆ = = (k + 1 )(k + 3 ) − 8k = k 2 − 4k + 3
1 −c −b k k =3
−c 1 −a = 0 ∴ ∆ = (k − 1 )(k − 3 )
−b −a 1 4k 8
∆1 = = 4k 2 + 12k − 24k + 8 = 4k 2 − 12k + 8
⇒ 1 + 2 ( − a )( − b )( − c ) − a 2 − b 2 − c 2 = 0 3k − 1 k + 3

or a 2 + b 2 + c 2 + 2abc = 1 ∆1 = 4(k − 1 )(k − 2 )


a a + 1 a −1 a + 1 b + 1 c −1 k +1 4k
and ∆ 2 = = (k + 1 )(3k − 1 ) − 4k 2 = −k 2 + 2k + 1
115. − b b + 1 b − 1 + ( −1) a − 1 b − 1 c + 1 = 0
n k 3k − 1
c c −1 c + 1 a −b c ∴ ∆ 2 = −(k − 1 ) 2

a a + 1 a −1 a + 1 a −1 a As given no solutions
⇒ − b b + 1 b − 1 + ( −1 )n b + 1 b − 1 −b = 0 ⇒ ∆1 and ∆ 2 ≠ 0
but ∆=0
c c −1 c + 1 c −1 c + 1 c
k =3
[by property] 3 1 + f (1 ) 1 + f (2 )
a a + 1 a −1 a a + 1 a −1 120. Q 1 + f (1) 1 + f (2) 1 + f (3)
⇒ − b b + 1 b − 1 + ( −1 )n + 2 − b b + 1 b − 1 = 0 1 + f (2 ) 1 + f (3 ) 1 + f ( 4 )
c c −1 c + 1 c c −1 c + 1
1+1+1 1+α +β 1 + α 2 + β2
116. Applying R1 → R1 + R3, then = 1+α +β 1 + α 2 + β2 1 + α 3 + β3
θ L 0 L 2 1 + α 2 + β2 1 + α 3 + β3 1 + α 4 + β4
M 2
1 1 1 1 1 11 1 1
f (θ ) = − tan θ 1 tan θ
= 1 α β × 1 α β = 1 α β
M
1 α 2 β2 1 α2 β2 1 α 2 β2
−1 − tan θ 1
= {(1 − α )(1 − β )(α − β )} 2
= 2 (1 + tan θ ) = 2 sec θ ≥ 2
2 2
= (1 − α ) 2(1 − β ) 2(α − β ) 2
∴f (θ ) ∈ [2, ∞ )
So, k = 1.
117. Non-zero solution means non-trivial solution.
121. The given system can be written as
For non-trivial solution of the given system of linear equations
4 k 2 (2 − λ ) x1 − 2 x 2 + x 3 = 0
k 4 1 =0 2 x1 − (3 + λ ) x 2 + 2 x 3 = 0
2 2 1 − x1 + 2 x 2 − λx 3 = 0
⇒ 4( 4 − 2 ) − k(k − 2 ) + (2k − 8 ) = 0 For non-trivial solutions, ∆ = 0
⇒ − k 2 + 6k − 8 = 0 2−λ −2 1
⇒ k − 6k + 8 = 0
2
2 −(3 + λ ) 2 =0
⇒ (k − 2 )(k − 4 ) = 0
−1 2 −λ
∴ k = 2, 4
Clearly, there exist values of k. ⇒ (2 − λ )( λ2 + 3 λ − 4 ) + 2( −2 λ + 2 ) + 1( 4 − 3 − λ ) = 0
1 −k 1 ⇒ λ3 + λ2 − 5 λ + 3 = 0
118. For trivial solution k 3 −k ≠ 0 ⇒ λ = 1, 1, − 3
3 1 −1 Hence, λ has two values.
604 Textbook of Algebra

122. Applying R2 → R2 − R1 and R3 → R3 − R1, then ⇒ x3 =


5
or x 3 = −1
(1 + α)2 (1 + 2α ) 2 (1 + 3α)2 6
1/ 3
2α + 3 4α + 3 6α + 3 = − 648 α  5
∴ x=  , −1
 6
4α + 8 8α + 8 12α + 8
i.e. Two distinct values of x.
Applying R3 → R3 − 2 R2 , then
a 2
(1 + α)2 (1 + 2α ) 2 (1 + 3α)2 125. ∆= = −2a − 6,
3 −2
2α + 3 4α + 3 6α + 3 = − 648 α
λ 2
2 2 2 ∆1= = − 2( λ + µ )
µ −2
1
Applying C 2 → C 2 − (C1 + C 3 ) , then a λ
2 or ∆2 = = αµ − 3 λ
3 µ
(1 + α) 2 ...
−α 2 ...
(1 + 3α)2
M System has unique solution for ∆ ≠ 0
2α + 3 4α + 3 6α + 3 = − 648 α
M ∴ a ≠ −3 for all values λ and µ
2 0 2 System has infinitely many solution for
⇒ α 2( 4α + 6 − 12α − 6 ) = −648α ∆ = ∆1 = ∆ 2 = 0
∴ a = −3, λ + µ = 0, aµ − 3 λ = 0
⇒ − 8α 3 = −648α
and system has no solution
⇒ α 3 − 81α = 0 ∆ = 0 ⇒ a = −3
∴ α = 0, 9, − 9 and λ +µ ≠ 0
123. For non-trivial solution 1 1 1
 1 λ −1 126. Q ∆ = 1 a 1 = 1 (a − b) − 1(1 − a ) + 1(b − a 2 ) = − (a − 1) 2
λ −1 −1 = 0  
  a b 1 
 1 1 − λ 
1 1 1
⇒ 1( λ + 1 ) − λ ( − λ2 + 1 ) − 1( λ + 1 ) = 0 ∆1 = 1 a 1  = 1(a − b ) − 1(1 ) + 1(b ) = (a − 1 )
 
⇒ λ(λ2 −1) = 0 0 b 1 
⇒ λ = 0, ±1 1 1 1
1 1 ∆ 2 = 1 1 1  = 1 (1 ) − 1(1 − a ) + 1( 0 − a ) = 0
3
1+ x
   
124. x 3 2 4 1 + 8 x 3  = 10 a 0 1 
3 9 1 + 27 x 3  1 1 1
 
1 1 1  1 1 1  and ∆ 3 = 1 a 1  = 1 ( −b ) − 1( −a ) + 1(b − a 2 ) = − a(a − 1 )
 
⇒ x 3 2 4 1  + x 6 2 4 8  = 10 a b 0 
   
3 9 1  3 9 27  For a = 1, ∆ = ∆1 = ∆ 2 = ∆ 3 = 0
Applying C 2 → C 2 − C1 and C 3 → C 3 − C1, then and for b = 1 only
x +y +z =1
1 0 0 1 0 0 
x +y +z =1
x 3 2 2 − 1  + x 6 2 2 6  = 10
    x +y +z = 0
3 6 − 2  3 6 24 
i.e. no solution (Q RHS are not equal)
⇒ 2 x 3 + 12 x 6 = 10 or 6 x 6 + x 3 − 5 = 0 Hence, for no solution b = 1 only.
or (6 x 3 − 5 )( x 3 + 1 ) = 0
CHAPTER

08
Matrices
Learning Part
Session 1
● Definition

● Types of Matrices

● Difference Between a Matrix and a Determinant

● Equal Matrices

● Operations of Matrices

● Various Kinds of Matrices

Session 2
● Transpose of a Matrix

● Symmetric Matrix

● Orthogonal Matrix

● Complex Conjugate (or Conjugate) of a Matrix

● Hermitian Matrix

● Unitary Matrix

● Determinant of a Matrix

● Singular and Non-Singular Matrices

Session 3
● Adjoint of a Matrix ● Inverse of a Matrix

● Elementary Row Operations ● Equivalent Matrices

● Matrix Polynomial ● Use of Mathematical Induction

Session 4
● Solutions of Linear Simultaneous Equations Using Matrix Method

Practice Part
● JEE Type Examples
● Chapter Exercises

Arihant on Your Mobile !


Exercises with the #L
symbol can be practised on your mobile. See inside cover page to activate for free.
606 Textbook of Algebra

J. J. Sylvester was the first to use the word ‘‘Matrix” in 1850 and later on in 1858 Arthur Cayley developed the theory of
matrices in a systematic way. ‘Matrices’ is a powerful tool in mathematics and its study is becoming important day by
day due to its wide applications in almost every branch of science. This mathematical tool is not only used in certain
branches of sciences but also in genetics, economics, sociology, modern psychology and industrial management.

Session 1
Definition, Types of Matrices, Difference Between
a Matrix and a Determinant, Equal Matrices, Operations
of Matrices, Various Kinds of Matrices

Definition y Example 1. If a matrix has 12 elements, what are the


A set of mn numbers (real or complex) arranged in the possible orders it can have? What will be the possible
form of a rectangular array having m rows and n columns orders if it has 7 elements?
is called a matrix of order m × n or an m × n matrix (which Sol. We know that, if a matrix is of order m × n , it has mn
is read as m by n matrix). elements. Thus, to find all possible orders of a matrix with
12 elements, we will find all ordered pairs of natural
An m × n matrix is usually written as numbers, whose product is 12.
 a 11 a 12 a 13 ... a 1n  Thus, all possible ordered pairs are (1, 12), (12, 1), (2, 6), (6, 2),
a a 22 a 23 ... a 2n  (3, 4 ), ( 4, 3).
 21 
 a 31 a 32 a 33 ... a 3n  Hence, possible orders are 1 × 12, 12 × 1, 2 × 6, 6 × 2, 3 × 4
 ... and 4 × 3.
... ... ... ... 
  If the matrix has 7 elements, then the possible orders will be
 ... ... ... ... ...  1 × 7 and 7 × 1.
a 
 m1 a m 2 a m 3 ... a mn  y Example 2. Construct a 2 × 3 matrix A = [aij ], whose
In a compact form the above matrix is represented by[a ij ], elements are given by
i = 1, 2, 3, ..., m, j = 1, 2, 3, ..., n or simply by [a ij ]m × n , where (i +2 j ) 2 1
the symbols a ij represent any numbers (a ij lies in the ith (i) aij = . (ii) aij = 2i − 3 j .
row (from top) and jth column (from left)). 2 2
i − j , i ≥ j
Notations A matrix is denoted by capital letter such as A, B, (iii) aij =  .
C, …, X ,Y, Z. i + j , i < j
i 
Note (iv) aij =   ,
1. A matrix may be represented by the symbols [ aij ], ( aij ), aij j
or by a single capital letter A. (say) where [.] denotes the greatest integer function.
A = [ aij ] m × n or ( aij ) m × n or aij  2i 
Generally, the first system is adopted. (v) aij =   ,
2. The numbers a11, a12, ..., etc., of rectangular array are called 3 j 
the elements or entries of the matrix. where {.} denotes the fractional part function.
3. A matrix is essentially an arrangement of elements and has no  3i + 4 j 
(vi) aij =  ,
value.  2 
4. The plural of ‘matrix’ is ‘matrices’. where (.) denotes the least integer function.
Chap 08 Matrices 607

a a a13   2i 
Sol. We have, A =  11 12  (v) Since, aij =   , therefore [Q0 ≤ { x } < 1]
 21 22
a a a 23  2 × 3 3 j 
(i + 2 j )2  2 2  2  1 1
(i) Since, aij = , therefore a11 =   = , a12 =   =   = ,
2  
3 3  6  3 3
( 1 + 2) 2 9 (1 + 4 )2 25  2 2  4  1 1
a11 = = , a12 = = , a13 =   = , a 21 =   = 1 +  = ,
2 2 2 2 9  9  3  3 3
(1 + 6)2 49 ( 2 + 2) 2
a13 = = , a 21 = = 8,  4   2 2  4 4
2 2 2 a 22 =   =   = and a 23 =   =
 6   3 3 9 9
(2 + 4 )2 ( 2 + 6) 2
a 22 = = 18 and a 23 = = 32 2 1 2
2 2 
Hence, the required matrix is A = 3 3 9 .
9 25 49  1 2 4
Hence, the required matrix isA =  2 2 2 .  
 8 18  3 3 9
 32  3i + 4 j 
1 (vi) Since, aij =   , therefore [Q( x ) ≥ x ]
(ii) Since, aij = 2i − 3 j , therefore  2 
2
3 + 4 7
a11 =
1
2−3 =
1
−1 = ,
1 a11 =   =   = (3.5) = 4,
 2   2
2 2 2
1 1 4  3 + 8  11
a12 = 2−6 = − 4 = = 2, a12 =   =   = (5.5) = 6,
2 2 2  2   2

a13 =
1
2−9 =
1
−7 = ,
7  3 + 12  15
a13 =   =   = (7.5) = 8,
2 2 2  2   2
1 1 1
a 21 = 2−3 = −1 = ,  6 + 4   10
2 2 2 a 21 =   =   = (5) = 5,
 2   2
1 1 4
a 22 = 2−6 = − 4 = =2  6 + 8  14 
2 2 2 a 22 =   =   = (7 ) = 7
 2  2
1 1 5
and a 23 = 4 −9 = −5 =
2 2 2  6 + 12  18
and a 23 =   =   = (9 ) = 9
1 7  2   2
2 2 2 
Hence, the required matrix is A =  . Hence, the required matrix is
1 5 4 6 8
 2  A=
2 2 
5 7 9
i − j , i ≥ j
(iii) Since, aij =  , therefore
i + j , i < j
a11 = 1 − 1 = 0, a12 = 1 + 2 = 3, a13 = 1 + 3 = 4,
a 21 = 2 − 1 = 1, a 22 = 2 − 2 = 0 and a 23 = 2 + 3 = 5
Types of Matrices
Hence, the required matrix is 1. Row Matrix or Row Vector
0 3 4 A matrix is said to be row matrix or row vector, if it
A= 
1 0 5 contains only one row, i.e. a matrix A = [a ij ]m × n is said to
i  be row matrix, ifm = 1.
(iv) Since, aij =   , therefore [Q[ x ] ≤ x ]
j For example,
1 1 (i) A = [a 11 a 12 a 13 ... a 1n ]1 × n
a11 =   = [1] = 1, a12 =   = [0 . 5] = 0,
1 2 (ii) B = [3 5 − 7 9 ]1 × 4
1 2
a13 =   = [0.33] = 0, a 21 =   = [2] = 2 are called row matrices.
3 1
 
2 2
and a 22 =   = [1] = 1 and a 23 =   = [0.67] = 0
2. Column Matrix or Column Vector
2 3 A matrix is said to be column matrix or column vector, if
1 0 0 it contains only one column, i.e., a matrix A = [a ij ]m × n is
Hence, the required matrix is A =   said to be column matrix, if n = 1. For example,
2 1 0
608 Textbook of Algebra

 a 11  7 Thus, A = diag (2), B = diag (− 1, 2) and C = diag (3, 5, 7).


a  0
 21    Remark
(i) A =  a 31  (ii) B = − 8  (i) No element of principal diagonal in a diagonal matrix is zero.
 M  2 (ii) Minimum number of zero in a diagonal matrix is given by
    n( n − 1), where n is order of matrix.
a mn  m × 1  1  5 × 1
are called column matrices. 6. Scalar Matrix
A diagonal matrix is said to be a scalar matrix, if its
3. Rectangular Matrix diagonal elements are equal. Thus, A = [a ij ]n × n is called
A matrix is said to be rectangular matrix, if the number of scalar matrix, if
rows and the number of columns are not equal i.e., a 0, if i ≠ j
matrix A = [a ij ]m × n is called a rectangular matrix, iff a ij =  , where k is scalar.
m ≠ n. For example, k, if i = j
1 3 4 5 2 − 3 For example,
5 0 0 

(i) A = 2 0 − 3 8  (ii) B =  3 0 2 0 
    (i) [7] (ii)   (iii) 0 5 0 
0 2   
7 4 2 5  3 × 4  4 8  3 × 2
0 0 5 
are called rectangular matrices. are scalar matrices of order 1, 2 and 3, respectively. They
can be written as diag (7), diag (2, 2) and diag (5, 5, 5),
4. Square Matrix respectively.
A matrix is said to be a square matrix, if the number of
rows and the number of columns are equal i.e., a matrix 7. Unit or Identity Matrix
A = [a ij ]m × n is called a square matrix, iff m = n. A diagonal matrix is said to be an identity matrix, if its
For example, diagonal elements are equal to 1.
a 11 a 12 a 13  Thus, A = [a ij ]n × n is called unit or identity matrix, if
a b 
(i) A = a 21 a 22 a 23  (ii) B =   0, if i ≠ j
  c d  2 × 2 a ij = 
a 31 a 32 a 33  3 × 3 1, if i = j
are called square matrices. A unit matrix of order n is denoted by I n or I. For example,
1 0 0 
1 0 
(iii) I 3 = 0 1 0 
Remark
(i) I 1 =[1] (ii) I 2 =  
If A= [ aij ] is a square matrix of order n, then elements (entries)
0 1   
a11, a22, a33, ...,ann are said to constitute the diagonal of the matrix A. 0 0 1
The line along which the diagonal elements lie is called principal
1 4 0 are identity matrices of order 1, 2 and 3, respectively.
or leading diagonal. Thus, if A = 8 3 − 2 , then the elements 8. Singleton Matrix
 
9 2 5
A matrix is said to be singleton matrix, if it has only one
of the diagonal of Aare 1, 3, 5.
element i.e. a matrix A = [a ij ]m × n is said to be singleton
5. Diagonal Matrix matrix, if m = n = 1.
For example, [3], [k ], [− 2] are singleton matrices.
A square matrix is said to be a diagonal matrix, if all its
non-diagonal elements are zero. Thus, A = [a ij ]n × n is 9. Triangular Matrix
called a diagonal matrix, if a ij = 0, when i ≠ j. A square matrix is called a triangular matrix, if its each
For example, element above or below the principal diagonal is zero. It is
3 0 0 
− 1 0
(iii) C = 0 5 0 
of two types:
(i) A =[2 ] (ii) B =  
 0 2    (a) Upper Triangular Matrix A square matrix in
0 0 7  which all elements below the principal diagonal are
are diagonal matrices of order 1, 2 and 3, respectively. A zero is called an upper triangular matrix i.e., a matrix
diagonal matrix of order n having d 1 , d 2 , d 3 ,..., d n as A = [a ij ]n × n is said to be an upper triangular matrix,
diagonal elements may be denoted by diag (d 1 , d 2 , d 3 , .., d n ). if a ij = 0, when i > j.
Chap 08 Matrices 609

For example,  2 3 4
 0 −1 7
3 −2 4 1  
0 For example, A =  3 5 4
2
is a vertical matrix.
2 −3
(i)    2 7 9 
0 0 7 5  
0 8  4 × 4  − 1 2 − 5  5 × 3
 0 0
[Q number of rows (5) > number of columns (3)]

a 11 a 12 a 13 a 14 a 15  12. Null Matrix or Zero Matrix


 0 a 22 a 23 a 24 a 25  A matrix is said to be null matrix or zero matrix, if all
  elements are zero i.e., a matrix A = [a ij ]m × n is said to be a
(ii)  0 0 a 33 a 34 a 35 
 0 zero or null matrix, iff a ij = 0, ∀ i, j . It is denoted by O.
0 0 a 44 a 45 
  For example,
 0 0 0 0 a 55  5 × 5 0 0 0 
0 0 0 
(i) O 2 × 3 =   (ii) O 3 × 3 = 0 0 0 
0 0 0   
0 0 0 
are upper triangular matrices. are called the null matrices.
(b) Lower Triangular Matrix A square matrix in
which all elements above the principal diagonal are 13. Sub-Matrix
zero is called a lower triangular matrix i.e., a matrix A matrix which is obtained from a given matrix by
A = [a ij ]n × n is said to be a lower triangular matrix, if deleting any number of rows and number of columns is
a ij = 0, when i < j. For example, called a sub-matrix of the given matrix.
8 9 5 
 3 4 2 3 4  .
10 0 0 0 For example,   is a sub-matrix of
7 0 0  8  − 2 5   
9 0 0 3 −2 5 
(i) 5 4 0  (ii)  
  5 6 7 0
2 3 4  3×3 1
14. Trace of a Matrix
 2 3 4  4×4 The sum of all diagonal elements of a square matrix
are lower triangular matrices. A = [a ij ]n × n (say) is called the trace of a matrix A and is
denoted by Tr ( A ).
Note n

Minimum number of zeroes in a triangular matrix is given by Thus, Tr ( A ) = ∑ a ii


n( n − 1) i =1
, where n is order of matrix.
2 2 − 7 9
For example, If A = 0 3 2 , then
10. Horizontal Matrix  
8 9 4 
A matrix is said to be horizontal matrix, if the number of
rows is less than the number of columns i.e., a matrix Tr ( A ) = 2 + 3 + 4 = 9
A =[a ij ]m × n is said to horizontal matrix, iff m < n. Properties of Trace of a Matrix
2 3 4 5 Let A = [a ij ]n × n , B = [b ij ]n × n and k is a scalar, then
For example, A = 8 9 7 −2  is a horizontal (i) Tr (kA ) = k ⋅ Tr ( A )
 
2 −2 −3 4  3 × 4 (ii) Tr ( A ± B ) = Tr ( A ) ± Tr ( B )
matrix. [Q number of rows (3) < number of columns (4)] (iii) Tr ( AB ) = Tr ( BA )
(iv) Tr ( A ) = Tr ( A′ )
11. Vertical Matrix (v) Tr ( I n ) = n
A matrix is said to be vertical matrix, if the number of (vi) Tr ( AB ) ≠ Tr ( A ) Tr ( B )
rows is greater than the number of columns i.e., a matrix (vii) Tr ( A ) = Tr (C A C − 1 ),
A =[a ij ]m × n is said to vertical matrix, iff m > n. where C is a non-singular square matrix of order n.
610 Textbook of Algebra

15. Determinant of Square Matrix Thus, if A =[a ij ]m × n , B = [b ij ]p × q , then A = B, iff


Let A = [a ij ]n × n be a matrix. The determinant formed by (i) m = p, n = q (ii) a ij = b ij , ∀ i, j
the elements of A is said to be the determinant of matrix  − 1 2 4
A.This is denoted by A . For example, If A =   and
 3 0 52 × 3
For example,
a b c 
3 4 5 3 4 5 B=  are equal matrices, then
  d e f  2 × 3
If A = 6 7 8 , then A = 6 7 8 = − 39.
  a = −1, b = 2, c = 4d = 3, e = 0, f = 5
2 − 3 5  2 −3 5
 x + 3 2y + x   − x − 1 0 
Remark y Example 3. If  = 2w 
, then
1. If A1, A2, A3, ..., An are square matrices of the same order,  z − 1 4w − 8   3
then A1 A2 A3 ... An = A1 A2 A3 ... An . find the value of x + y + z + w .
2. If k is a scalar and Ais a square matrix of order n, then Sol. As the given matrices are equal so their corresponding
kA = k n A elements are equal.
x + 3 = − x − 1 ⇒ 2x = − 4 …(i)
16. Comparable Matrices ∴ x = −2
Two matrices A = [a ij ]m × n and B =[b ij ]p × q are said to be 2y + x = 0
comparable, if m = p and n = q. ⇒ 2y − 2 = 0 [from Eq. (i)]
For example, ⇒ y =1 …(ii)
a b c  p q r  z −1=3
The matrices   and   are comparable ⇒ z=4 …(iii)
d e f  s t u  4w − 8 = 2w
1 2 2 4 6  ⇒ 2w = 8
but the matrices   and   are not comparable.
4 8 5 3 1 ∴ w =4 …(iv)
Hence, x + y + z + w = − 2 + 1 + 4 + 4 = 1 + 8 = 9

Difference Between a Matrix y Example 4. If 


2 α + 1 3 β  α + 3 β 2 + 2
= ,
β 2 − 5 β  0
and a Determinant  0
find the equation whose roots are α and β.
−6 

(i) A matrix cannot be reduced to a number but


determinant can be reduced to a number. Sol. The given matrices will be equal, iff
(ii) The number of rows may or may not be equal to the 2α + 1 = α + 3 ⇒ α = 2
number of columns in matrices but in determinant the 3 β = β2 + 2 ⇒ β2 − 3 β + 2 = 0
number of rows is equal to the number of columns. ∴ β = 1, 2 and β 2 − 5 β = − 6 …(i)
(iii) On interchanging the rows and columns, a different
⇒ β − 5β + 6 = 0
2
matrix is formed but in determinant it does not
change the value. ∴ β = 2, 3 …(ii)
(iv) A square matrix A such that A ≠ 0, is called a From Eqs. (i) and (ii), we get β = 2
non-singular matrix. If A = 0, then the matrix A is ⇒ α = 2, β = 2
called a singular matrix. ∴ Required equation is x 2 − (2 + 2)x + 2 ⋅ 2 = 0
(v) Matrices represented by [ ], ( ), but determinant is ⇒ x 2 − 4x + 4 = 0
represented by .

Equal Matrices Operations of Matrices


Two matrices are said to be equal, if
(i) they are of the same order i.e., if they have same
Addition of Matrices
number of rows and columns. Let A, B be two matrices, each of order m × n. Then, their
(ii) the elements in the corresponding positions of the sum A + B is a matrix of order m × n and is obtained by
two matrices are equal. adding the corresponding elements of A and B.
Chap 08 Matrices 611

Thus, if A = [a ij ]m × n and B = [b ij ]m × n , then Properties of Matrix Addition


A + B = [a ij + b ij ]m × n , ∀ i, j
Property 1 Addition of matrices is commutative,
 1 3 5  0 3 i.e. A+B=B+A
y Example 5. Given, A = − 2 0  2 , B = −2
 0 where A and B are any two m × n matrices, i.e. matrices of
    the same order.
 0 4 − 3  0 − 4 
Property 2 Addition of matrices is associative
 4 1 − 2 
 (A + B ) + C = A + (B + C )
1. Find (whichever defined)
i.e.
and C = 3 2
  where A, B and C are any three matrices of the same order
 2 − 1 7  m × n (say).
(i) A + B. (ii) A + C. Property 3 Existence of additive identity
Sol. (i) Given, A is a matrix of the type 3 × 3 i.e. A +O = A =O + A
and B is a matrix of the type 3 × 2. where A be any m × n matrix and O be the m × n null
Since, A and B are not of the same type. matrix. The null matrix O is the identity element for
∴ Sum A + B is not defined. matrix addition.
` (ii) As A and C are two matrices of the same type, Property 4 Existence of additive inverse
therefore the sum A + C is defined.
If A be any m × n matrix, then there exists another m × n
 1 3 5 4 1 − 2 matrix B, such that A + B = O = B + A

∴ A + C = −2 0 
2 + 3 2 1
    where O is the m × n null matrix.
 0 4 − 3  2 − 1 7 Here, the matrix B is called the additive inverse of the
1 + 4 3 + 1 5 − 2  5 4 3 matrix A or the negative of A.
= −2 + 3 0 + 2 2 + 1  = 1 2 3 Property 5 Cancellation laws
   
 0 + 2 4 − 1 −3 + 7 2 3 4 If A, B and C are matrices of the same order m × n (say),
then A + B = A +C ⇒B =C [left cancellation law]
y Example 6. If a, b ;b , c and c , a are the roots of and B + A = C + A ⇒ B = C [right cancellation law]
x 2 − 4 x + 3 = 0, x 2 − 8 x + 15 = 0 and x 2 − 6 x + 5 = 0,

respectively. Compute  2
a 2 + c 2 a 2 + b 2 

Scalar Multiplication
b + c a2 + c 2 
2
Let A =[a ij ]m × n be a matrix and k be any number called a
 2ac −2ab  scalar. Then, the matrix obtained by multiplying every
+ . element of A by k is called the scalar multiple of A by k
 −2bc −2ac 
and is denoted by kA.
Sol. Q x 2 − 4x + 3 = 0
Thus, kA = [ka ij ]m × n
⇒ ( x − 1) ( x − 3) = 0 ∴ x = 1, 3
x 2 − 8x + 15 = 0 Properties of Scalar Multiplication
⇒ ( x − 3) ( x − 5) = 0 ∴ x = 3, 5 If A = [a ij ]m × n , B = [b ij ]m × n are two matrices and k, l are
and x 2 − 6x + 5 = 0 scalars , then
⇒ ( x − 5) ( x − 1) = 0 ∴ x = 5, 1 (i) k ( A + B ) = kA + kB (ii) (k + l ) A = kA + lA
It is clear that a = 1, b = 3 and c = 5 (iii) (kl )A = k (lA ) = l(kA )
a 2 + c 2 a 2 + b 2   2ac −2ab (iv) ( −k ) A = − (k A ) = k ( − A )
Now,  2 2
+ −2bc
b + c a + c 
2 2
 −2ac  (v) 1A = A, ( −1) A = − A
a 2 + c 2 + 2ac a 2 + b 2 − 2ab (a + c )2 (a − b )2 
= 2 =  y Example 7. Determine the matrix A,
b + c − 2bc a 2 + c 2 − 2ac  (b − c )2 (a − c )2 
2
 1 2 3 5 4 1 
(1 + 5)2 (1 − 3)2  36 4 
= = when A = 4 −1 −2 −3 + 2  3 2 4 .
 
2     
(3 − 5) (1 − 5)   4 16
2
 4 2 6   3 8 2 
612 Textbook of Algebra

 4 8 12  10 8 2 3 − 1 −1 − 2 2 + 3
Sol. A = −4 −8 −12  +  6 4 8 = 4 − 5 2 − 0 5 − 2
     
 16 8 24   6 16 4 2 − 1 0 + 1 3 − 1
 4 + 10 8 + 8 12 + 2  14 16 14   2 −3 5  2 −3 5
2C = −1 2 3 ⇒ C = −1 2 3
1
= −4 + 6 −8 + 4 −12 + 8 =  2 −4 − 4 ∴
      2  
 16 + 6 8 + 16 24 + 4  22 24 28   1 1 2  1 1 2
 1 −3 / 2 5 / 2
0 2   0 3a  
= −1 / 2 3 / 2
y Example 8. If A =   and k A =  , then 
1

3 − 4  2b 24   1 / 2 1 / 2 1 
find the value of b − a − k.
0 2  0 2k  y Example 10. Solve the following equations for X and
Sol. We have, A=  ⇒ k A = 3k
3 − 4   −4k   3 −3 0 4 1 5 
Y . 2X − Y =   ,2Y + X =  
 0 3a 3 3 2   −1 4 −4 
But kA =  
2b 24 3 −3 0
Sol. Given, 2X − Y = 
0 2k   0 3a 3 3 2
∴ 3k =
 −4k  2b 24 On multiplying both sides by 2, we get
⇒ 2k = 3a, 3k = 2b, − 4k = 24 3 −3 0 6 −6 0
4 X − 2Y = 2   ; 4 X − 2Y =   …(i)
⇒ k = − 6, a = − 4, b = − 9 3 3 2 6 6 4
Hence, b − a − k = − 9 − ( − 4 ) − ( − 6) = − 9 + 4 + 6 = 1  4 1 5
also given X + 2Y =   …(ii)
− 1 4 − 4
Subtraction of Matrices Adding Eqs. (i) and (ii), we get
Let A, B be two matrices, each of order m × n. Then, their 6 −6 0  4 1 5
5X =  +
subtraction A − B is a matrix of order m × n and is 6 6 4 −1 4 −4
obtained by subtracting the corresponding elements of A 6 + 4 −6 + 1 0 + 5  10 −5 5
and B. Thus, if A = [a ij ]m × n and B = [b ij ]m × n , = =
6 − 1 6 + 4 4 − 4  5 10 0
then A − B = [a ij − b ij ]m × n , ∀ i, j
1 10 −5 5 2 −1 1
2 3  a b  ∴ X =  ⇒ X =
5  5 10 0 1 2 0

For example, If A = 4 5 and B = c d ,
 
    Putting the value of X in Eq. (ii), we get
6 7  e f  2 −1 1  4 1 5
1 2 0 + 2Y = −1 4 −4
2 3  a b  2 − a 3 − b     
then A − B =  4 5  − c d  =  4 − c 5 − d  ⇒
 4 1
2Y = 
5 2 −1 1

     
6 7  e f  6 − e 7 − f  −1 4 −4 1 2 0
4 −2 1 + 1 5 − 1   2 2 4
 1 2 −3  = =
−1 − 1 4 − 2 −4 − 0 −2 2 −4
y Example 9. Given, A =  5 0 2  and  1 1 2
  ∴ Y =
 1 −1 1  −1 1 −2
 3 −1 2 2 −1 1  1 1 2
Hence, X = and Y = 
B = 4 2 5. Find the matrix C such that A + 2 C = B.
 1 −2
1 2 0 − 1
 
2 0 3
Remark
Sol. Given, A + 2C = B If two matrices Aand B are of the same order, then only their
3 −1 2 1 2 −3  addition and subtraction is possible and these matrices are said
2C = B − A = 4 2 5 − 5 0 2 
to be conformable for addition or subtraction. On the other
    hand, if the matrices Aand B are of different orders, then their
2 0 3 1 −1 1  addition and subtraction is not possible and these matrices are
called non-conformable for addition and subtraction.
Chap 08 Matrices 613

Multiplication For convenience of multiplication we write columns in


horizontal rectangles.
Conformable for Multiplication 0 1 2 0 1 2
If A and B be two matrices which are said to be 1 –1 2 –2 0 –1
conformable for the product AB . If the number of columns 1 2 3 1 2 3
in A (called the pre-factor) is equal to the number of rows 1 –1 2 –2 0 –1
in B (called the post-factor) otherwise non-conformable 2 3 4 2 3 4
for multiplication. Thus, 1 –1 2 –2 0 –1

(i) AB is defined, if number of columns in A = number of  0 × 1 + 1 × ( −1) + 2 × 2 0 × ( − 2) + 1 × 0 + 2 × ( −1)


rows in B. = 1 × 1 + 2 × ( − 1) + 3 × 2 1 × ( −2) + 2 × 0 + 3 × ( −1) 
(ii) BA is defined, if number of columns in B = number of  
2 × 1 + 3 × ( −1) + 4 × 2 2 × ( −2) + 3 × 0 + 4 × ( −1) 3 × 2
rows in A.
0 − 1 + 4 0+0−2  3 −2
Multiplication of Matrices = 1 − 2 + 6 −2 + 0 − 3  = 5 −5
   
Let A = [a ij ]m × n and B = [b ij ]n × p be two matrices, then 2 − 3 + 8 −4 + 0 − 4 3 × 2 7 −8 3 × 2
the product AB is defined as the matrix C = [C ij ]m × p ,
n Since, the number of columns of B is 2 and the number of
where C ij = ∑ a ij b jk , 1 ≤ i ≤ m, 1 ≤ k ≤ p rows of A is 3, BA is not defined (Q2 ≠ 3).
j =1

= a i1 b 1k + a i2 b 2k + a i 3 b 3k + ... + a in b nk Remark
Verification for the product to be correct .
i.e., (i, k ) th entry of the product AB is the sum of the
From above example
product of the corresponding elements of the ith row of A
 0 1 2   1 − 2  3 − 2
(pre-factor) and kth column of B (post-factor).  1 2 3 × − 1 0 =  5 − 5
Note      
 2 3 4   2 − 1  7 − 8 
 A = Pre-factor
In the product AB,  Sum 3 6 9 15 −15
 B = Post-factor
1 = 3 × 1 + 6 × ( −1) + 9 × 2
0 1 2   1 −2
−1 = 3 − 6 + 18
y Example 11. If A = 1 2 3 and B = −1 0  ,
  Now, 3 6 9
    2 = 15
2 3 4   2 −1 
obtain the product AB and explain why BA is not defined? −2 = 3 ( −2) + 6 × 0 + 9 × ( − 1)
and 369 0 = −6 + 0 −9
Sol. Here, the number of columns in A = 3 = the number of
−1 = − 15
rows in B. Therefore, the product AB is defined.
C1 C 2
 0 − tan (α / 2)
0 1 2 R1  1 −2 y Example 12. If A =   and I is
AB = 1 2 3 R 2 × −1 0  tan (α / 2) 0 
   
2 3 4 R 3  2 −1
a 2 × 2 unit matrix, prove that
cos α − sin α 
R1, R 2 , R 3 are rows of A and C 1, C 2 are columns of B. I + A = (I − A )  .
 R1C 1 R1C 2   sin α cos α 
∴ AB = R 2 C 1 R 2 C 2  1 0
Sol. Since, I = 

and given A = 
0 − tan(α / 2)
   
 R 3C 1 R 3C 2  3 × 2 0 1 tan (α / 2) 0 
1 –2  1 − tan (α /2 )
∴ I +A=  …(i)
0 1 2 –1 0 1 2 0 tan(α / 2) 1 
2 –1
cos α − sin α
1 –2 RHS = ( I − A ) 
1 2 3 –1 1 2 3 0  sin α cos α 
2 –1
 1 tan (α / 2) cos α − sin α
1 –2 =   sin α
2 3 4 –1 2 3 4 0 − tan(α / 2) 1   cos α
2 –1 3 ×2
614 Textbook of Algebra

 1 tan (α /2 ) Property 5 If product of two matrices is a zero matrix, it


= 
 − tan( α / 2 ) 1  is not necessary that one of the matrices is a zero matrix.
1 − tan 2 (α / 2) −2 tan (α / 2)  For example,
  1 1  −1 1   1 ⋅ ( −1) + 1 . 1 1 ⋅ 1 + 1 ⋅ ( −1) 
1 + tan (α / 2) 1 + tan 2 (α / 2)
2
(i)  × = 
 2 tan (α / 2) 1 − tan (α / 2)  2 2   1 −1 2 ⋅ ( −1) + 2 . 1 2 ⋅ 1 + 2 ⋅ ( −1) 
2

1 + tan 2 (α / 2) 1 + tan 2 (α / 2) 
 0 0 
Let tan (α /2 ) = λ , then = =O
1 − λ 2 −2 λ  0 0 
 
 1 λ  1 + λ 2 1 + λ 2  0 0  0 0  0 ⋅ 0 + 0 ⋅ 0 0 ⋅ 0 + 0 ⋅ 1 0 0 
RHS =   (ii)  × = =  =O
− λ 1   2 λ 1 − λ2  1 0  0 1 1 ⋅ 0 + 0 ⋅ 0 1 ⋅ 0 + 0 ⋅ 1 0 0 
1 + λ 2 1 + λ 2 
  None of the matrices on the LHS is a null matrix
 1 − λ 2 + 2λ 2 − 2λ + λ ( 1 − λ 2 )  whereas their product is a null matrix.
 
1 + λ2 1 + λ2
=  Note If A and B are two non-zero matrices such that AB = 0, then
 −λ (1 − λ 2 ) + 2λ 2λ + 1 − λ 
2 2
A and B are called the divisors of zero. Also, if
 1 + λ2 1 + λ2  AB = 0 ⇒ |AB| = 0 ⇒| A|| B| = 0
 
⇒ |A| = 0 or| B| = 0 but not the converse.
 1 + λ2 − λ (1 + λ 2 )
  −λ
Property 6 Multiplication of a matrix A by a null matrix
1 + λ2 1 + λ2   1
= = conformable with A for multiplication.
 λ (1 + λ 2 ) 1 + λ  λ
2
1 
 1 + λ2 3 4 
1 + λ 2  0 0 0 
 For example, If A = 5 6  and O =   ,
 1 − tan(α / 2)   0 0 0  2 × 3
=  [Q λ = tan (α / 2)] 7 8  3 × 2
tan(α / 2) 1 
0 0 0 
=I +A [from Eq. (i)]
= LHS then AO = 0 0 0  , which is a 3 × 3 null matrix.
 
0 0 0  3 × 3
Pre-multiplication and Property 7 Multiplication of a matrix by itself
Post-multiplication of Matrices The product of A A A ... m times = A m and ( A m ) n = A mn
The matrix AB is the matrix B pre-multiplied by A and the Note
matrix BA is the matrix B post-multiplied by A. 1. If I be unit matrix, then I 2 = I 3 = K = I m = I ( m ∈ I+ )

Properties of Multiplication of Matrices 2. If Aand B are two matrices of the same order, then
(i) ( A + B) 2 = A2 + AB + BA + B2
Property 1 Multiplication of matrices is not commutative
(ii) ( A − B) 2 = A2 − AB − BA + B2
i.e. AB ≠ BA (iii) ( A − B)( A + B) = A2 + AB − BA + B2
Note (iv) ( A + B)( A − B) = A2 − AB + BA − B2
1. If AB = − BA, then Aand B are said to anti-commute. (v) A ( − B) = ( − A)( B) = − AB
1 0  3 0 3 0
2. If A =   and B = 0 4 , then AB = BA = 0 8 .  1 2 2 1 
0 2      y Example 13. If A =   ,B =  and
Observe that multiplication of diagonal matrices of same order −2 3 2 3
will be commutative. −3 1 
Property 2 Matrix multiplication associative if C= , verify that ( AB ) C = A(BC )
 2 0
conformability assumed.
and A(B + C ) = AB + AC .
i.e. A( BC ) = ( AB ) C
 1 2 2 1
Property 3 Matrix multiplication is distributive with Sol. We have, AB =  × 
−2 3 2 3
respect to addition. i.e. A( B + C ) = AB + AC , whenever
both sides of equality are defined.  1 ⋅2 + 2 ⋅2 1 ⋅ 1 + 2 ⋅ 3  6 7
= = 
Property 4 If A is an m × n matrix, then I m A = A = A I n . ( −2) ⋅ 2 + 3 ⋅ 2 ( −2) ⋅ 1 + 3 ⋅ 3 2 7
Chap 08 Matrices 615

2 1 −3 1 2 ⋅ ( −3) + 1 ⋅ 2 2 ⋅ 1 + 1 ⋅ 0 0 0 1 
BC =  × =
2 3  2 0 2 ⋅ ( −3) + 3 ⋅ 2 2 ⋅ 1 + 3 ⋅ 0
  = p q r 
 
− 6 + 2 2 + 0 −4 2 pr p + qr q + r 2 
= =  0 2
− 6 + 6 2 + 0   0 0 1   0 1 0
 1 2 −3 1  1 ⋅ ( −3) + 2 ⋅ 2 1 ⋅1 + 2 ⋅0  ∴ A = A ⋅A = p
3 2
q r ×  0 0 1
AC =  × =    
   pr p + qr q + r 
2
p q r 
−2 3  2 0 ( −2) ⋅ ( −3) + 3 ⋅ 2 ( −2) ⋅ 1 + 3 ⋅ 0
−3 + 4 1 + 0  1 1  p q r 
= =  
−2 + 0 12 −2 
 = p + qr q +r 2
 6+6 
pr

…(i)
pq + r p pr + q 2 + qr 2 p + 2qr + r 
2 3
2 1 −3 1 2 − 3 1 + 1 −1 2
B+C =  + = =
2 3  2 0 2 + 2 3 + 0  4 3 1 0 0  0 1 0
6 7 −3 1 −18 + 14 6 + 0 and pI + qA + rA 2 = p 0 1 0 + q  0 0 1
Now, ( AB )C =     
× =  0 0 1 p q r 
2 7  2 0  − 6 + 14 2 + 0
−4 6 0 0 1 
=  …(i)
 8 2 +rp q r 
 
 1 2 −4 2 pr p + qr q + r 2 
A ( BC ) =  × 
−2 3  0 2 p 0 0  0 q 0  0 0 r 
−4 + 0 2 + 4  −4 6 = 0 p 0 +  0 0 q+  pr qr r2 
= =  …(ii)      2 
 8 + 0 −4 + 6   8 2  0 0 p  pq q 2 qr  pr pr + qr 2 qr + r 3 
Thus, from Eqs. (i) and (ii), we get, (AB)C = A(BC)  p +0+0 0+q +0 0+0+r 
 1 2 −1 2 −1 + 8 2 + 6  
= 0 + 0 + pr p + 0 + qr 0+q +r 
2
Now, A ( B + C ) =  × =    
−2 3  4 3  2 + 12 −4 + 9  + + + + pr + qr 2 p + qr + qr + r 
2 2 3
0 pq pr 0 q
 7 8  p q r 
=  …(iii)
14 5 =  pr p + qr q +r2  …(ii)
 
pq + pr q 2 + pr + qr 2 p + 2qr + r 3 
2
6 7  1 1   6 + 1 7 + 1
and AB + AC =  + =  
2 7 12 −2 2 + 12 7 − 2 Thus, from Eqs. (i) and (ii), we get A 3 = pI + qA + rA 2
 7 8
=  …(iv) y Example 15. Find x, so that
14 5  1 3 2  1 
Thus, from Eqs. (iii) and (iv), we get [1 x 1] 0 5 1   1  = O.
A ( B + C ) = AB + AC   
0 3 2  x 
0 1 0 1 3 2  1 
y Example 14. If A = 0 0 1 , show that Sol. We have, [1 x 1] 0 5 1  1  = O
    
0 3 2 x 
p q r 
1
A 3 = pI + qA + rA 2 .
⇒ [1 5x + 6 x + 4 ]  1  = O
Sol. We have, A 2 = A ⋅ A  
x 
 0 1 0  0 1 0
⇒ [1 + 5x + 6 + x 2 + 4 x ] = O
=  0 0 1 ×  0 0 1
    or x 2 + 9x + 7 = 0
p q r  p q r 
− 9 ± (81 − 28) − 9 ± 53
∴ x= ⇒ x=
2 2
616 Textbook of Algebra

Various Kinds of Matrices  1


y Example 17. Show that  5 2
1 3
6  is nilpotent
 
Idempotent Matrix −2 −1 −3 
matrix of order 3.
A square matrix A is called idempotent provided it
 1 1 3
satisfies the relation A 2 = A .
Sol. Let A =  5 2 6
Note  
An = A, ∀ n ≥ 2, n ∈ N. −2 −1 −3 

 1 1 3  1 1 3
y Example 16. Show that the matrix
∴A = A⋅A =  5
2
2 6 ×  5 2 6
 2 −2 −4     
−2 −1 −3  −2 −1 − 3 
A = − 1 3 4  is idempotent.
   1+5−6 1+2−3 3+6−9 
 1 −2 − 3  = 5 + 10 − 12 5 + 4 − 6 15 + 12 − 18

 
 2 −2 −4   2 −2 −4   − 2 − 5 + 6 − 2 − 2 + 3 − 6 − 6 + 9 
Sol. A = A ⋅ A = − 1
2
3 4  × − 1 3 4  0 0
    0
 1 −2 −3   1 −2 − 3  = 3 3 9
 
2 ⋅ 2 + ( − 2) ⋅( − 1) + ( − 4 ) ⋅1 −1 −1 −3 
=  ( − 1) ⋅ 2 + 3 ⋅ ( − 1) + 4 ⋅ 1
  0 00  1 1 3
 1.2 + ( − 2) ⋅ ( − 1) + ( − 3) ⋅ 1 ∴ A3 = A2 ⋅ A =  3 39× 5 2 6
2 ⋅ ( − 2) + ( − 2) ⋅ 3 + ( − 4 ) ⋅ ( − 2)    
−1 −1 −3  −2 −1 −3 
( − 1) ⋅ ( − 2) + 3 ⋅ 3 + 4 ⋅ ( − 2)
1 ⋅ ( − 2) + ( − 2) ⋅ 3 + ( − 3) ⋅ ( − 2)  0+0+0 0+0+0 0+0+0  0 0 0
2 ⋅ ( −4 ) + ( −2) ⋅ 4 + ( − 4 ) ⋅ ( −3) = 3 + 15 − 18 3 + 6 − 9 9 + 18 − 27 = 0 0 0 = O
   
( − 1) ⋅ ( − 4 ) + 3 ⋅ 4 + 4 ⋅ ( − 3)   − 1 − 5 + 6 − 1 − 2 + 3 − 3 − 6 + 9  0 0 0

1 ⋅ ( −4 ) + ( −2) ⋅ 4 + ( − 3) ⋅ ( −3) ∴ A 3 = O i.e., A k = O

 2 −2 −4  Here, k =3
= −1 3 4 = A Hence, the matrix A is nilpotent of order 3.
 
 1 −2 −3 
Involutory Matrix
Hence, the matrix A is idempotent.
A square matrix A is called involutory provided it satisfies
the relation A 2 = I , where I is identity matrix.
Periodic Matrix Note A = A− 1 for an involutory matrix.

A square matrix A is called periodic, if A k + 1 = A , where y Example 18. Show that the matrix
k is a positive integer. If k is the least positive integer
 − 5 −8 0
for which A k + 1 = A , then k is said to be period of A. 
A= 3 5 0  is involutory.
For k = 1, we get A 2 = A and we called it to be  
 1 2 −1 
idempotent matrix.
Note − 5 −8 0  −5 −8 0
Period of an idempotent matrix is 1. Sol. A = A ⋅ A =  3
2
5 0 ×  3 5 0
   
 1 2 − 1   1 2 − 1 
Nilpotent Matrix
A square matrix A is called nilpotent matrix of order m  25 − 24 + 0 40 − 40 + 0 0 + 0 + 0 1 0 0
provided it satisfies the relation A k = O and A k − 1 ≠ O , = − 15 + 15 + 0 − 24 + 25 + 0 0 + 0 + 0 =
 0 1 0 = I
   
where k is positive integer and O is null matrix and k is the  − 5 + 6 − 1 − 8 + 10 − 2 0 + 0 + 1 0 0 1
order of the nilpotent matrix A. Hence, the given matrix A is involutory.
Chap 08 Matrices 617

#L Exercise for Session 1


α 2 
1. If A =   and A = 125, α is equal to
3

2 α 
(a) ± 2 (b) ± 3
(c) ± 5 (d) 0
1 − 1 a 1
2. If A =   , B = b and ( A + B )2 = A2 + B 2, the value of a + b is
2 − 1  −1 
(a) 4 (b) 5
(c) 6 (d) 7
1 2
3. If A =   and A − λA − I 2 = O, then λ is equal to
2

2 3
(a) − 4 (b) − 2
(c) 2 (d) 4
0 α  a b 
4. Let A =   and ( A + I )50 − 50A =   , the value of a + b + c + d , is
0 0  c d 
(a) 1 (b) 2
(c) 4 (d) None of these
 cos θ sin θ 
5. If A =   , then A = I is true for
2

 − sin θ cos θ 
π
(a) θ= 0 (b) θ =
4
π
(c) θ = (d) None of these
2
α β 
6. If  is to be the square root of two rowed unit matrix, then α, β and γ should satisfy the relation
γ − α 

(a) 1 − α 2 + βγ = 0 (b) α 2 + βγ − 1 = 0

(c) 1 + α 2 + βγ = 0 (d) 1 − α 2 − βγ = 0

 1 0
7. If A =  100
 , then A is equal to
1 / 2 1
 1 0  1 0
(a)   (b)  
 25 0  50 1
 1 0
(c)  (d) None of these
 (1/ 2)
100
1
1 1 1 2 1 3 1 n  1 378
8. If the product of n matrices   0 1 0 1 ... 0 1 is equal to the matrix 0 , the value of n is equal
0 1      1 
to
(a) 26 (b) 27
(c) 377 (d) 378
9. If A and B are two matrices such that AB = B and BA = A, then A2 + B 2 is equal to
(a) 2AB (b) 2BA
(c) A + B (d) AB
Session 2
Transpose of a Matrix, Symmetric Matrix, Orthogonal
Matrix, Complex Conjugate (or Conjugate) of a Matrix,
Hermitian Matrix, Unitary Matrix, Determinant of a Matrix,
Singular and Non-Singular Matrices,

Transpose of a Matrix Sol. We have, A T + A = I 2


Let A =[a ij ]m × n be any given matrix, then the matrix  cos θ sin θ  cos θ − sin θ 1 0
⇒   + = 
obtained by interchanging the rows and columns of A is − sin θ cos θ  sin θ cos θ  0 1
called the transpose of A. Transpose of the matrix A is
2 cos θ 0  1 0
denoted by A′ or A T or A t . In other words, if ⇒  0 =
A =[a ij ]m × n , then A ′ = [a ji ]n × m .  2 cos θ 0 1

For example, 1  π π
⇒ cos θ = = cos   ⇒ θ = 2nπ ± , n ∈ I
 2 3 4 5 2 3 3
If A = −2 −1 4 8  ,

 
 7 5 3 1 3 × 4 Symmetric Matrix
2 −2 7  A square matrix A =[a ij ]n × n is said to be symmetric, if
3 −1 5 
A ′ = A i.e., a ij = a ji , ∀ i, j.
then A′ =  
4 4 3 For example,
 5 8 1 a h g  a h g 
 4 × 3
If A = h b f , then A ′ = h b f 
 
Properties of Transpose Matrices    
 g f c   g f c 
If A′ and B ′ denote the transpose of A and B respectively,
then Here, A is symmetric matrix as A ′ = A.
(i) (A ′ ) ′ = A Note
(ii) (A ± B )′ = A ′ ± B ′; A and B are conformable for 1. Maximum number of distinct entries in any symmetric matrix
n( n + 1)
matrix addition. of order n is .
2
(iii) (kA )′ = kA ′; k is a scalar. 2. For any square matrix Awith real number entries, then A + A′
(iv) (AB )′ = B ′ A ′; A and B are conformable for matrix is a symmetric matrix.
product AB. Proof ( A + A′ ) ′ = A′ + ( A′ ) ′ = A′ + A = A + A′
In general, (A1 A 2 A 3 ... An −1 An ) ′ = A ′n A ′n − 1 ...
A ′ 3 A ′ 2 A ′ 1 (reversal law for transpose).
Skew-Symmetric Matrix
Remark A square matrix A = [a ij ]n × n is said to be skew-symmetric
I ′ = I, where I is an identity matrix. matrix, if A ′ = − A ,i.e. a i j = − a ji ,∀ i, j. (the pair of
conjugate elements are additive inverse of each other)
cos θ − sin θ  Now, if we put i = j, we have a ii = − a ii .
y Example 19. If A =  , find the values
 sin θ cos θ  Therefore, 2a ii = 0 or a ii = 0, ∀ i ’s.
of θ satisfying the equation A T + A = I 2 . This means that all the diagonal elements of a
skew-symmetric matrix are zero, but not the converse.
Chap 08 Matrices 619

For example, 3 2  0 3  3 5
Now, P + Q =  + = =A
 0 h g 2 2 −3 0 −1 2
If A = −h 0 f , then Hence, A is represented as the sum of a symmetric and a
  skew-symmetric matrix.
−g − f 0 
 0 −h −g  0 h g
A ′ = h 0 − f = −  −h
  0 f=−A
Properties of Symmetric and

 g f

0 

 −g −f

0 
Skew-Symmetric Matrices
(i) If A be a square matrix, then AA′ and A ′ A are
Here, A is skew-symmetric matrix as A ′ = − A . symmetric matrices.
Note (ii) All positive integral powers of a symmetric matrix are
1. Trace of a skew-symmetric matrix is always 0. symmetric, because
2. For any square matrix Awith real number entries, then A − A′ ( An ) ′ = ( A′ )n
is a skew-symmetric matrix. (iii) All positive odd integral powers of a skew-symmetric
Proof ( A − A′ ) ′ = A′ − ( A′ ) ′ = A′ − A = −( A − A′ ) matrix are skew-symmetric and positive even integral
3. Every square matrix can be uniquely expressed as the sum of powers of a skew-symmetric matrix are symmetric,
a symmetric and a skew-symmetric matrix. because ( An ) ′ = ( A′ )n
i.e. If Ais a square matrix, then we can write
1 1 (iv) If A be a symmetric matrix and B be a square matrix
A = ( A + A′ ) + ( A − A′ )
2 2 of order that of A,then −A, kA, A ′ , A −1 , A n and B ′ AB
are also symmetric matrices, where n ∈ N and k is a
y Example 20. The square matrix A = [aij ]m ×m given scalar.
by aij = (i − j )n , show that A is symmetric and (v) If A be a skew-symmetric matrix, then
(a) A 2n is a symmetric matrix for n ∈ N .
skew-symmetric matrices according as n is even or
(b) A 2n + 1 is a skew-symmetric matrix for n ∈ N .
odd, respectively.
(c) kA is a skew-symmetric matrix, where k is scalar.
Sol. Qaij = (i − j )n = ( −1)n ( j − i )n
(d) B ′ AB is also skew-symmetric matrix, where B is
a ji , n is even integer a square matrix of order that of A.
= ( −1)n a ji = 
(vi) If A and B are two symmetric matrices, then
 −a ji ,n is odd integer
(a) A ± B, AB + BA are symmetric matrices.
Hence, A is symmetric if n is even and skew-symmetric if
n is odd integer. (b) AB − BA is a skew-symmetric matrix
(c) AB is a symmetric matrix, iff AB = BA
y Example 21. Express A as the sum of a symmetric (where A and B are square matrices of same order)
 3 5 (vii) If A and B are two skew-symmetric matrices, then
and a skew-symmetric matrix, where A =  .
−1 2  (a) A ± B, AB − BA are skew-symmetric matrices.
Sol. We have, (b) AB + BA is a symmetric matrix.
 3 5 3 −1 (where A and B are square matrices of same order)
A=  , then A ′ =  
−1 2 5 2  (viii) If A be a skew-symmetric matrix and C is a column
1 1 6 4 3 2 matrix, then C ′AC is a zero matrix, where C ′AC is
Let P= (A + A′ ) =  = =P′
2 4 4 2 2
conformable.
2
1
Thus, P = ( A + A ′ ) is a symmetric matrix.
2 Orthogonal Matrix
1 1  0 6  0 3
Also, let Q = ( A − A ′ ) =  = A square matrix A is said to be orthogonal matrix, iff
2 2 −6 0 −3 0 AA ′ = I , where I is an identity matrix.
0 −3   0 3
Then, Q ′ =   = − −3 0 = − Q
Note
3 0    1. If AA′ = I, then A−1 = A.
1
Thus, Q = ( A − A ′ ) is a skew-symmetric matrix. 2. If Aand B are orthogonal, then AB is also orthogonal.
2 3. If Ais orthogonal, then A−1 and A′ are also orthogonal.
620 Textbook of Algebra

 0 2β γ  Sol. Since, AA ′ = 9 I 3
y Example 22. If α β −γ  is orthogonal, then find 1 2 2  1 2 a 1 0 0
  2 1 −2  2 1
⇒ 2 = 9 0 1 0
α −β γ      
a 2 b  2 −2 b 0 0 1
the value of 2 α 2 + 6 β 2 + 3γ 2 .
 9 0 a + 2b + 4  9 0 0
0 2β γ 0 α α
⇒  2a − 2b + 2 = 0 9 0
Sol. Let A = α −γ  , then A ′ = 2 β β −β
0 9
β    
    a + 2b + 4 2a − 2b + 2 a + b + 4
2 2
0 0 9
α −β γ   γ − γ γ 
Equating the corresponding elements, we get
Since, A is orthogonal.
a + 2b + 4 = 0 …(i)
∴ AA ′ = I
2a − 2b + 2 = 0 …(ii)
 0 2β γ   0 α α  1 0 0
⇒ α β −γ  2β β −β = 0 1 0 and a2 + b2 + 4 = 9 …(iii)
    
α −β γ   γ − γ γ  0 0 1 From Eqs. (i) and (ii), we get
a = − 2 and b = − 1
 4β2 + γ 2 2β 2 − γ 2 − 2β 2 + γ 2  1 0 0 Hence, | a | + | b | = | −2| + | − 1| = 2 + 1 = 3
 2  0 1 0
⇒  2β − γ 2 α 2 + β 2 + γ 2 α 2 − β 2 − γ 2  =
 
− 2β 2 + γ 2 α 2 − β 2 − γ 2 α 2 + β 2 + γ 2  0 0 1
 
Equating the corresponding elements, we get Complex Conjugate
4β2 + γ 2 = 1 …(i) (or Conjugate) of a Matrix
2β 2 − γ 2 = 0 …(ii)
If a matrix A is having complex numbers as its elements,
and α2 + β2 + γ 2 = 1 …(iii) the matrix obtained from A by replacing each element of A
From Eqs. (i) and (ii), we get by its conjugate (a ± ib = a m ib, where i = −1 ) is called
1 1
β 2 = and γ 2 = the conjugate of matrix A and is denoted by A .
6 3
2 + 5 i 3 −i 7 
From Eq. (iii),
For example, If A =  −2 i 6 + i 7 − 5 i , where i = −1,
1 1 1
α 2 = 1− β2 − γ 2 = 1 −
− =  
6 3 2  1 − i 3 6 i 
1 1 1
Hence, 2α 2 + 6β 2 + 3γ 2 = 2 × + 6 × + 3 × = 3 2 − 5 i 3 +i 7 
2 6 3
then A =  2i 6 − i 7 + 5i
Aliter  
 1 + i 3 −6 i 
The rows of matrix A are unit orthogonal vectors
Note
→ →
R1⋅ R 2 = 0 ⇒ 2β 2 − γ 2 = 0 ⇒ 2β 2 = γ 2 …(i) If all elements of A are real, then A = A.
→ →
R2 ⋅ R3 = 0 ⇒ α 2 − β 2 − γ 2 = 0 ⇒ β 2 + γ 2 = α 2
→ →
…(ii)
Properties of Complex Conjugate
and R 3 ⋅ R 3 = 1 ⇒ α + β + γ = 1
2 2 2
…(iii) of a Matrix
From Eqs. (i), (ii) and (iii), we get If A and B are two matrices of same order, then
1 1
α 2 = , β 2 = and γ 2 =
1 =
2 6 3 (i) ( A ) = A
∴ 2α 2 + 6β 2 + 3γ 2 = 3 (ii) ( A + B ) = A + B, where A and B being conformable for
1 2 2  addition.
y Example 23. If A = 2 1 −2  is a matrix (iii) (kA ) = k A , where k is real.
 
a 2 b  (iv) ( AB ) = A B, where A and B being conformable for
satisfying AA ′ = 9I 3 , find the value of | a | + |b |. multiplication.
Chap 08 Matrices 621

Conjugate Transpose of a Matrix Note


For any square matrix Awith complex number entries, then
The conjugate of the transpose of a matrix A is called the A + Aθ is a Hermitian matrix.
conjugate transpose of A and is denoted by A θ i.e. Proof ( A + Aθ ) θ = Aθ + ( Aθ ) θ = Aθ + A= A + Aθ
A θ = Conjugate of A ′ = ( A ′ )
For example, Skew-Hermitian Matrix
2 + 4 i 3 5 − 9i A square matrix A = [a ij ]n × n is said to be skew-hermitian
 3 i ,
If A=

4 5 + 2i
 matrix. If A θ = − A , i.e. a ij = − a ij , ∀ i, j. If we put j = i, we
 2 −5 4 − i  have a ii = − a ii ⇒ a ii + a ii = 0 ⇒ a ii is purely imaginary
where i = −1, for all i ’s. This means that all the diagonal elements of a
2 − 4 i 4 2  skew-hermitian matrix must be purely imaginary or zero.
θ
then A = ( A ′ ) =  3 5 − 2i −5  For example,
 
5 + 9 i −3 i 4 + i   2i −2 − 3 i −2 + i 
If 
A = 2 − 3i −i 3i  , where i = −1,
 
Properties of Transpose Conjugate Matrix  2 + i 3i 0 
If A and B are two matrices of same order, then  2i 2 − 3i 2 + i
(i) ( A ) ′ = ( A ′ ) θ θ
(ii) ( A ) = A then 
A ′ = −2 − 3 i −i 3i 
 
θ θ θ
(iii) ( A + B ) = A + B , where A and B being  −2 + i 3i 0 
conformable for addition.  −2 i 2 + 3i 2 − i
θ  −3 i 
(iv) (kA ) θ = k A θ , where k is real. ∴ A = ( A ′ ) = −2 + 3 i

i

θ θ θ
(v) ( AB ) = B A , where A and B being conformable for  −2 − i −3 i 0 
multiplication  2i −2 − 3 i −2 + i 

= − 2 − 3i −i 3i  = − A
 
Hermitian Matrix  2 + i 3i 0 
A square matrix A = [a ij ]n ×n is said to be hermitian, if Hence, A is skew-hermitian matrix.
A θ = A i.e., a ij = a ji , ∀ i, j. If we put j = i, we have a ii = a ii Note
⇒ a ii is purely real for all i ’s. 1. For any square matrix Awith complex number entries, then
A − Aθ is a skew-hermitian matrix.
This means that all the diagonal elements of a hermitian
Proof ( A − Aθ ) θ = ( Aθ ) − ( Aθ ) θ = Aθ − A = − ( A − Aθ )
matrix must be purely real.
2. Every square matrix (with complex elements) can be uniquely
For example, expressed as the sum of a hermitian and a skew-hermitian
 α λ + iµ θ + i φ  matrix i.e.

If 
A = λ − iµ β x + iy 
If Ais a square matrix, then we can write
  1 1
A = ( A + Aθ ) + ( A − Aθ )
 θ − iφ x − iy γ  2 2

where α, β, γ, λ, µ, θ, φ, x , y ∈ R and i = −1, then y Example 24. Express A as the sum of a hermitian
 α λ − iµ θ − iφ  and a skew-hermitian matrix, where

A ′ = λ + iµ β x − iy 
2 + 3 i 7 
  A=  , i = −1.
 θ + iφ x + iy γ   1 − i 2i 
2 + 3i 7  2 − 3i 1 + i 
 α λ + iµ θ + iφ  Sol. We have,A =   , then A θ = ( A ′ ) = 
 1 − i 2i   7 −2i 
∴ θ 
A = ( A ′ ) = λ − iµ β x + iy  = A
   i
 θ − iφ x − iy γ  1 1  4 8 + i  2 4+ 
θ 2 θ
Let P = ( A + A ) =  = =P
2 8 − i 0  4 − i
Here, A is hermitian matrix as A θ = A . 2
 0 
 2 
622 Textbook of Algebra

Thus, P =
1
( A + A θ ) is a hermitian matrix. 1  1 1+ i 
y Example 25. Verify that the matrix is
3 1 − i −1 
2
1 1  6i 6 − i
Also, let Q = ( A − A θ ) =  unitary, where i = −1.
2 2 −6 − i 4 i 
1  1 1 + i
 i  i Sol. Let A = , then
 3i 3−   − 3i −3 +  3 1 − i −1 
= 2 = − 2 = − Qθ
  1  1 1 + i
−3 −
i
2i  3 +
i
− 2i  A θ = ( A ′) =
 2   2  3 1 − i −1 

Thus, Q =
1
( A − A θ ) is a skew-hermitian matrix. 1  1 1 + i 1  1 1 + i
∴ AA θ =   × 1 − i −1 
2 3 1 − i − 1  3  
 i  i
3 0 1 0
 2 4 +   3i 3−  1
2 + 2 =0 3 = 0 1 = I
Now, P + Q =      3   
i i
4 − 0  −3 − 2i 
 2   2  Hence, A is unitary matrix.
2 + 3i 7 
=  =A
 1 − i 2i  Determinant of a Matrix
Hence, A is represented as the sum of a hermitian and a Let A be a square matrix, then the determinant formed by
skew-hermitian matrix. the elements of A without changing their respective
positions is called the determinant of A and is denoted by
Properties of Hermitian and det A or | A |.
Skew-Hermitian Matrices a 1 a 2 a 3 
 
a1 a2 a 3
(i) If A be a square matrix, then AA θ and A θ A are i.e., If A = b 1 b 2 b 3 , then | A | = b 1 b 2 b 3 .
 
hermitian matrices. c 1 c 2 c 3  c1 c2 c 3
(ii) If A is a hermitian matrix, then
(a) iA is skew-hermitian matrix, where i = −1. Properties of the Determinant
(b) iff A is hermitian matrix. of a Matrix
(c) kA is hermitian matrix, where k ∈ R.
If A and B are square matrices of same order, then
(iii) If A is a skew-hermitian matrix, then
(i) | A | exists ⇔ A is a square matrix.
(a) iA is hermitian matrix, where i = −1.
(ii) | A ′ | = | A |
(b) iff A is skew-hermitian matrix.
(c) kA is skew-hermitian matrix, where k ∈ R. (iii) | AB | = | A || B | and | AB | = | BA |
(iv) If A and B are hermitian matrices of same order, then (iv) If A is orthogonal matrix, then | A | = ± 1
(a) k 1 A + k 2 B is also hermitian, where k 1 , k 2 ∈ R. (v) If A is skew-symmetric matrix of odd order, then
(b) AB is also hermitian, if AB = BA . |A| = 0
(c) AB + BA is a hermitian matrix. (vi) If A is skew-symmetric matrix of even order, then | A |
(d) AB − BA is a skew-hermitian matrix. is a perfect square.
(v) If A and B are skew-hermitian matrices of same order, (vii) | kA | = k n | A |, where n is order of A and k is scalar.
then k 1 A + k 2 B is also skew-hermitian matrix.
(viii)| A n | = | A |n , where n ∈ N
(ix) If A = diag(a 1 , a 2 , a 3 , K, a n ), then
Unitary Matrix | A | = a1 ⋅ a2 ⋅ a 3 Kan
A square matrix A is said to be unitary matrix iff AA θ = I ,
y Example 26. If A , B and C are square matrices of
where I is an identity matrix.
order n and det( A ) = 2 , det(B ) = 3 and det(C ) = 5, then
Note find the value of 10 det ( A 3B 2 C −1 ).
1. If AAθ = I, then A−1 = Aθ
Sol. Given, | A | = 2 , | B | = 3 and | C | = 5.
2. If Aand B are unitary, then AB is also unitary.
3. If Ais unitary, then A−1 and A′ are also unitary. Now, 10det( A 3 B 2 C −1 ) = 10 × | A 3 B 2 C −1|
Chap 08 Matrices 623

= 10 × | A 3 | × | B 2 | × | C −1 | = 10 × | A | 3 × | B | 2 × | C | −1 1 + 2ω 2017 + ω 2018 ω 2018



10 × | A | 3 × | B 2 | 10 × 2 3 × 32 Sol. Let A =  1 1 + ω 2018 + 2ω 2017
= = = 144
|C | 5  ω 2017
ω 2018

1 
a b c  ω 2017 

y Example 27. If A = b c a , abc = 1, A T A = I, then 2 + ω 2017 + 2ω 2018 
 
c a b  Q ω 3 = 1 ⇒ ω 2017 = ω

find the value of a 3 + b 3 + c 3 . and ω 2018 = ω 2 , then

Sol. Q AT A = I 1 + 2ω + ω 2 ω2 1 
 
⇒ | A T A | = | I | ⇒ | A T || A | = 1 A= 1 1 + ω + 2ω
2
ω 
 ω ω2 2 + ω + 2ω 2 
⇒ | A || A | = 1 [Q| A T | = | A | ]  
⇒ | A| = ± 1 ω ω 2 1 
 
a b c = 1 ω ω  [Q1 + ω + ω 2 = 0]
⇒ b c a = ±1 ω ω 2 − ω 
 
c a b
ω ω2 1 ω ω 1
⇒ 3abc − (a + b 3 + c 3 ) = ± 1
3
Now, | A | = 1 ω ω =ω 1 1 ω =0
or 3 − (a 3 + b 3 + c 3 ) = ± 1 [Qabc = 1] ω ω2 −ω ω ω −ω
or a 3 + b 3 + c 3 = 3 ± 1 = 2or 4 [QC 1 = C 2 ]
Thus, | A | = 0.
Hence, A is singular matrix.
Singular and Non-Singular y Example 29. Find the real values of x for which the
Matrices x + 1

3 5 
A square matrix A is said to be a singular, if | A | = 0 and a matrix 1 x+3 5  is non-singular.
 
square matrix A is said to be non-singular, if | A | ≠ 0.  1 3 x + 5
For example,
x + 1 3 5 
 1 2 3 Sol. Let A =  1 x +3 5 
(i) A =  −1 0 2  is a singular matrix.  
x + 5
   1 3
 2 4 6  x +1 3 5
Since, | A | = 0. ∴ | A| = 1 x +3 5
2 3  1 3 x +5
(ii) A =   is a non-singular matrix.
4 5 Applying C 1 → C 1 + C 2 + C 3 , then

Since, | A | = 10 − 12 = − 2 ≠ 0 x +9 3 5
| A| = x + 9 x +3 5
y Example 28. If ω ≠ 1 is a complex cube root of unity, x +9 3 x +5
then prove that Applying R 2 → R 2 − R1 and R 3 → R 3 − R1, then
1 + 2 ω 2017 + ω 2018 ω 2018 x +9 3 5

1 + ω 2018 + 2 ω 2017
O
 1 | A| = 0 x 0 = x 2( x + 9)
 ω 2017 ω 2018 0 0 O x
1  Q A is non-singular.
ω 2017  is singular. ∴ | A| ≠ 0 ⇒ x 2 ( x + 9 ) ≠ 0
 ∴ x ≠ 0, − 9
2 + ω 2017 + 2 ω 2018 
Hence, x ∈ R − {0, − 9 }.
624 Textbook of Algebra

#L Exercise for Session 2


 4 x + 2
1 If A =   is symmetric, then x is equal to
2x − 3 x + 1
(a) 2 (b) 3
(c) 4 (d) 5
2 If A and B are symmetric matrices, then ABA is
(a) symmetric matrix (b) skew-symmetric matrix
(c) diagonal matrix (d) scalar matrix
3 If A and B are symmetric matrices of the same order and P = AB + BA and Q = AB − BA, then (PQ )′ is equal to
(a) PQ (b)QP
(c) − QP (d) None of these
4 If A is a skew-symmetric matrix and n is odd positive integer, then An is
(a) a skew-symmetric matrix (b) a symmetric matrix
(c) a diagonal matrix (d) None of these
5 If A is symmetric as well as skew-symmetric matrix, then A is
(a) diagonal matrix (b) null matrix
(c) triangular matrix (d) None of these
6 If A is square matrix order 3, then |( A − A′ )2015| is
(a) |A| (b) |A′ |
(c) 0 (d) None of these
7 The maximum number of different elements required to form a symmetric matrix of order 6 is
(a) 15 (b) 17
(c) 19 (d) 21
8 If A and B are square matrices of order 3 × 3 such that A is an orthogonal matrix and B is a skew-symmetric
matrix, then which of the following statement is true?
(a) |AB | = 1 (b) |AB | = 0
(c) |AB | = − 1 (d) None of these
 i 1− 2i 
9 The matrix A =  , where i = −1, is
− 1 − 2 i 0 
(a) symmetric (b) skew-symmetric
(c) hermitian (d) skew-hermitian
10 If A and B are square matrices of same order such that A* = A and B * = B, where A*denotes the conjugate
transpose of A, then ( AB − BA) * is equal to
(a) null matrix (b) AB − BA
(c) BA − AB (d) None of these
1 1 i
11 If matrix A = − i ,i = −1 is unitary matrix, a is equal to
2  a 
(a) 2 (b) − 1
(c) 0 (d) 1
12 If A is a 3 × 3 matrix and det (3A) = k {det( A)} , k is equal to
(a) 9 (b) 6
(c) 1 (d) 27
13 If A and B are square matrices of order 3 such that | A| = − 1, | B | = 3, then |3AB | is equal to
(a) − 9 (b) − 81
(c) − 27 (d) 81
Chap 08 Matrices 625

14 If A is a square matrix such that A2 = A, then det ( A) is equal to


(a) 0 or 1 (b) − 2 or 2
(c) − 3 or 3 (d) None of these
15 If I is a unit matrix of order 10, the determinant of I is equal to
(a) 10 (b) 1
1
(c) (d) 9
10

2− i 3−i 
16 If Ai =  − i 
2−i 
, then ∑ det( Ai ) is equal to
3 i =1
3 5
(a) (b)
4 24
5 7
(c) (d)
4 144
3 − x 2 2 
17 The number of values of x for which the matrix A =  2 4−x 1  is singular, is
 
 − 2 −4 − 1 − x 
(a) 0 (b) 1
(c) 2 (d) 3
 3 − 1+ x 2 
18 The number of values of x in the closed interval [− 4, − 1], the matrix  3 −1 x + 2 is singular, is
 
x + 3 −1 2 
(a) 0 (b) 1
(c) 2 (d) 3
− x x 2
19 The values of x for which the given matrix  2 x − x  will be non-singular are
 
 x −2 − x 
(a) − 2 ≤ x ≤ 2 (b) for all x other than 2 and − 2
(c) x ≥ 2 (d) x ≤ − 2
Session 3
Adjoint of a Matrix, Inverse of a Matrix (Reciprocal Matrix),
Elementary Row Operations (Transformations), Equivalent
Matrices, Matrix Polynomial, Use of Mathematical Induction,

Adjoint of a Matrix Rule to Write Cofactors of


LetA =[a ij ] be a square matrix of order n and let C ij be an Element aij
cofactor of a ij in A . Then, the transpose of the matrix of
Cross the row and column intersection at the element a ij
cofactors of elements of A is called the adjoint of A and is
and the determinant which is left be denoted by D, then
denoted by adj (A).
 D , if i + j = even integer
Thus, adj ( A ) = [C ij ] ′ Cofactors of a ij = 
⇒ ( adj A ) ij = C ji = Cofactor of a ji in A − D , if i + j = odd integer
a 11 a 12 a 13  3 1 4
i.e. if A = a 21 a 22 a 23 , then y Example 30. Find the cofactor of a 23 
in 0 2 − 1 .
   
a 31 a 32 a 33   1 − 3 5 
C 11 C 12 C 13  ′ C 11 C 21 C 31  3 1 4

adj A = C 21 C 22 C 23  = C 12 C 22 C 32 
 
M
Sol. Let A = 0 L 2 L −1 
   
C 31 C 32 C 33  C 13 C 23 C 33  1 −3 
M
5
where C ij denotes the cofactor of a ij in A . ∴ Cofactor of a 23 = − D [Q2 + 3 = odd]
a22 a 23 3 1
where D =
Here, C 11 = = a 22 a 33 − a 23 a 32 , 1 −3
a 32 a 33 [after crossing the 2nd row and 3rd column]
a 21 a 23 = − 9 − 1 = − 10
C 12 = − = a 31 a 23 − a 33 a 21 , Hence, cofactor of a 23 = − ( − 10) = 10
a 31 a 33
a 21 a 22 Note
C 13 = = a 21 a 32 − a 31 a 22 , The adjoint of a square matrix of order 2 is obtained by
a 31 a 32 interchanging the diagonal elements and changing signs of
a 12 a 13 off-diagonal elements.
C 21 = − = a 13 a 32 − a 12 a 33 , If A= 
 a b
a 32 a 33 , then
c d 
a 11 a 13  d −b 
C 22 = = a 11 a 33 − a 31 a 13 , (adj A) = 
− c a 
a 31 a 33
a 11 a 12 y Example 31. Find the adjoint of the matrix
C 23 = − = a 12 a 31 − a 11 a 32 ,
a 31 a 32  1 2 3
C 31 =
a 12 a 13
= a 12 a 23 − a 22 a 13 ,
A = 0 5 0.
 
a 22 a 23 2 4 3
a 11 a 13 Sol. If C be the matrix of cofactors of the element in | A |, then
C 32 = − = a 13 a 21 − a 11 a 23
a 21 a 23 C 11 C 12 C 13 
a 11 a 12 C = C 21 C 22 C 23 
and C 33 = = a 11 a 22 − a 21 a 12  
a 21 a 22 C 31 C 32 C 33 
Chap 08 Matrices 627

 0 5 

5
4
0
3

0
2
0
3 2 4 
Properties of Adjoint Matrix
   12 0 −10 
Property 1 If A be a square matrix order n, then
= −
2 3 1 3 1 2   6 −3
− = 0
 4 3 2 3 2 4    A ( adj A ) = ( adj A )A = | A | I n
  − 15 0 5 
 2 3 1 3 1 2  i.e., the product of a matrix and its adjoint is commutative.

 5 0 0 0 0 5 
Deductions of Property 1
 12 6 −15 
⇒ adjA = C ′ =  0 0 
Deduction 1 If A be a square singular matrix of order n,
−3
  then A( adj A ) = ( adj A ) A = O [null matrix]
−10 0 5 
Since, for singular matrix, | A | = 0.

Maha Shortcut for Adjoint Deduction 2 If A be a square non-singular matrix of order


n, then | adj A | = | A |n − 1
(Goyal’s Method) Since, for non-singular matrix, | A | ≠ 0.
This method applied only for third order square matrix.
ProofQ A (adj A ) = | A | I n
1 2 3 
Taking determinant on both sides, then
Method : Let A = 0 5 0 
  | A ( adj A )| = || A | I n |
2 4 3  ⇒ | A || adj A | = | A |n | I n | = | A |n [Q| I n | = 1]
Step I Write down the three rows of A and rewrite first two
⇒ | adj A | = | A |n − 1 [Q| A | ≠ 0]
rows.
i.e. Note m
1 2 3 In general|adj (adj (adj K (adj A)))| = | A|( n − 1)
144444 42444444 3
0 5 0 adj repeat m times

2 4 3 Property 2 If A and B are square matrices of order n, then


1 2 3 adj ( AB ) = ( adj B )( adj A )
0 5 0 Property 3 If A is a square matrix of order n, then
Step II After Step I, rewrite first two columns. ( adj A ) ′ = adj A ′
i.e. Property 4 If A be a square non-singular matrix of order
1 2 3 1 2
n, then adj ( adj A ) = | A |n − 2 A
0 5 0 0 5
2 4 3 2 4 Proof Q A (adj A ) = | A | I n …(i)
1 2 3 1 2 Replace A by adj A , then
0 5 0 0 5 (adj A )(adj (adj A )) = | adj A | I n
Step III After Step II, deleting first row and first column, then = | A |n − 1 I n [Q| adj A | = | A |n − 1 ]
we get all elements of adj A i.e., = I n | A |n − 1
1 L 2 L 3 L 1 L 2
Pre-multiplying both sides by matrix A, then
M
A( adj A )( adj ( adj A )) = AI n | A |n − 1 = A | A |n − 1
0 5 0 0 5
M ⇒ 15 0 –10 ⇒ | A | I n ( adj( adj A )) = A | A |n − 1
2 4 3 2 4
first column of adj A ⇒ ( adj ( adj A )) = A | A |n − 2
M ⇒ 6 –3 0 or adj ( adj A ) = | A |n − 2 A
second column of adj A
1 2 3 1 2 Property 5 If A be a square non-singular matrix of order
M ⇒ –15 0 5 n, then
2
| adj ( adj A )| = | A | (n − 1 )
third column of adj A
0 5 0 0 5
 15 6 −15  ProofQ adj (adj A ) = | A |n − 2 A
or adj A =  0 −3 0
  Taking determinant on both sides, then
− 10 0 5  | adj (adj A )| = || A |n − 2 A |
628 Textbook of Algebra

= | A |n (n − 2 ) | A | [Q| kA | = k n | A | ] (i) | A || adj A | = | A || A | 2 [Qn = 3]


2
− 2n + 1 2 = | A | 3 = 4 3 = 64
= | A |n = | A | (n − 1 ) 3

Note (ii) | adj ( adj ( adj A ))| = | A | (3 − 1) = | A | 8 = 4 8 = 216


In general,|adj (adj (adj ... (adj A)))| = | A|( n − 1)
m
(iii) | adj (3A )| = | 32 adj A | = (32 )3 | adj A |
144444 42444444 3
adj m times = 36 | A | 2 = 729 × 4 2 = 11664
Property 6 If A be a square matrix of order n and k is a (iv) adj( adj A ) = | A | 2 A = 16A
scalar, then
adj (kA ) = k n − 1 ⋅ ( adj A ) 3 − 3 4 
Proof Q A( adj A ) = | A | I n …(i) y Example 33. If A = 2 − 3 4  and B is the adjoint
 
Replace A by kA , then 0 − 1 1 
kA(adj (kA )) = | kA | I n = k n | A | I n of A, find the value of | AB + 2I |, where I is the identity
matrix of order 3.
⇒ A(adj (kA )) = k n − 1 | A | I n
3 − 3 4
= k n − 1 A(adj A ) [from Eq. (i)] Sol. Q A = 2 − 3 4
 
n −1
Hence, adj (kA ) = k ( adj A ) 0 − 1 1
Property 7 If A be a square matrix of order n and m ∈ N , 3 −3 4
then ( adj A ) = ( adj A )
m m ∴ | A| = 2 − 3 4
0 −1 1
Property 8 If A and B be two square matrices of order n
= 3( − 3 + 4 ) + 3(2 − 0) + 4( − 2 − 0) = 1 ≠ 0
such that B is the adjoint of A and k is a scalar, then
∴ | AB + 2I | = (| A | + 2)3 [by property 8]
| AB + kI n | = (| A | + k ) n
= (1 + 2)3 = 33 = 27
ProofQ B = adj A
∴ AB = A(adj A ) = | A | I n Inverse of a Matrix
LHS = | AB + kI n | = || A | I n + kI n | = |(| A | + k ) I n |
(Reciprocal Matrix)
= (| A | + k ) n | I n | = (| A | + k ) n = RHS
A square matrix A (non-singular) of order n is said to be
Property 9 Adjoint of a diagonal matrix is a diagonal invertible, if there exists a square matrix B of the same
matrix. order such that AB = I n = BA,
a 0 0  bc 0 0  then B is called the inverse (reciprocal) of A and is denoted
i.e. If A = 0 b 0 , then adj A =  0 ca 0 
 
by A −1 . Thus, A −1 = B ⇔ AB = I n = BA
   
0 0 c   0 0 ab  We have, A (adj A ) = | A | I n
Note ⇒ A A (adj A ) = A −1 I n | A |
−1
adj ( In ) = In .
⇒ I n (adj A ) = A −1 | A | I n
−1 1 1
adj A
y Example 32. If A =  1 −1 1 , find the values of ∴ A −1 = ; provided | A | ≠ 0
  |A |
 1 1 −1  Note The necessary and sufficient condition for a square matrix A
to be invertible is that| A| ≠ 0.

(i) | A| | adj A| (ii) | adj ( adj ( adj A))|


(iii) | adj (3A)| (iv) adj (adj A ) Properties of Inverse of a Matrix
−1 1 1
Sol. Q A=1 −1 1
Property 1 (Uniqueness of inverse) Every invertible
  matrix possesses a unique inverse.
 11 − 1
Proof Let A be an invertible matrix of order n × n. Let B
∴ | A | = ( − 1)(1 − 1) − (1)( − 1 − 1) + (1)(1 + 1) = 4 ≠ 0
and C be two inverses of A. Then,
⇒ A is non-singular.
AB = BA = I n …(i)
Chap 08 Matrices 629

and AC = CA = I n …(ii) Proof We have,


Now, AB = I n ( A k ) −1 = ( A × A × A × K × A ) − 1
1444 424444 3
⇒ C ( AB ) = CI n [ pre-multiplying by C ] repeat k times
⇒ (CA ) B = CI n [by associativity] = A × A × A × K × A −1
−1 −1 −1

⇒ I n B = CI n [QCA = I n by Eq. (ii)] 14444 4244444 3


repeat k times
⇒ B =C [ by reversal law for inverse]
Hence, an invertible matrix possesses a unique inverse. = (A −1 k
) =A −k

Property 2 (Reversal law) If A and B are invertible


matrices of order n × n, then AB is invertible and Property 5 Let A be an invertible matrix of order n, then
( AB ) −1 = B −1 A −1 . ( A −1 ) −1 = A .

Proof It is given that A and B are invertible matrices. Proof We have, A −1 A = I n


| A | ≠ 0 and | B | ≠ 0 ⇒ | A || B | ≠ 0 ∴ Inverse of A −1 = A ⇒ ( A −1 ) −1 = A
⇒ | AB | ≠ 0 Note
Hence, AB is an invertible matrix. I n−1 = I n as I n−1 I n = I n
Now, ( AB )( B −1 A −1 ) = A( BB −1 ) A −1 [by associativity] Property 6 Let A be an invertible matrix of order n, then
= ( A I n ) A −1 [Q BB −1
= In] | A −1 | =
1
.
|A|
= A A −1 [Q A I n = A]
Proof Q A is invertible, then | A | ≠ 0.
= In [Q A A −1 = I n ]
Now, A A −1 = I n = A −1 A
−1
Also, ( B A −1 )( AB ) = B −1
( A −1 A ) B [by associativity]
−1 −1
⇒ | A A −1 | = | I n |
=B (I n B ) [Q A A = I n ]
−1
⇒ | A || A −1 | = 1
=B B [Q I n B = B ]
[Q| AB | = | A || B | and | I n | = 1]
−1
= In [Q B B = In] 1
−1
⇒ | A −1 | = [Q | A | ≠ 0 ]
Thus, ( AB )( B A −1 ) = I n = ( B −1
A −1 )( AB ) |A|
Hence, ( AB ) −1 = B −1
A −1 Property 7 Inverse of a non-singular diagonal matrix is a
Note diagonal matrix.
If A, B, C, …, Y, Z are invertible matrices, then a 0 0 
( ABC K YZ ) −1 = Z −1Y −1 K C −1B−1 A−1 [reversal law] i.e. If A = 0 b 0  and | A | ≠ 0, then
 
Property 3 Let A be an invertible matrix of order n, then 0 0 c 
A′ is also invertible and ( A ′ ) −1 = ( A −1 ) ′. 1 
a 0 0 
Proof Q A is invertible matrix  1 
∴ | A | ≠ 0 ⇒ | A′ | ≠ 0 [Q| A | = | A ′ |] A −1 =  0 0
−1  b 
Hence, A is also invertible. 0 0 1
Now, AA −1 = I n = A −1 A  c 
⇒ ( AA −1 ) ′ = ( I n ) ′ = ( A −1 A ) ′ Note
The inverse of a non-singular square matrix Aof order 2 is
⇒ ( A −1 ) ′ A ′ = I n = A ′ ( A −1 ) ′ obtained by interchanging the diagonal elements and changing
signs of off-diagonal elements and dividing by| A. |
[by reversal law for transpose] For example,
−1 −1
⇒ (A′ ) = (A )′ [by definition of inverse]  a b
If A=   and| A| = ( ad − bc ) ≠ 0, then
Property 4 Let A be an invertible matrix of order n and c d 
1  d −b 
k ∈ N , then A−1 =
( ad − bc ) − c a 
( A k ) −1 = ( A −1 ) k = A −k
630 Textbook of Algebra

y Example 34. Compute the inverse of the matrix y Example 36. Matrices A and B satisfy AB = B −1 ,
0 1 2   2 −2 
where B =  , find the value of λ for which
A =  1 2 3. − 1 0 
 
 3 1 1  λA − 2B −1 + I = O, without finding B −1 .
0 1 2 Sol. Q AB = B −1 or AB 2 = I
Sol. We have, A = 1 2 3 Now, λ A − 2B −1 + I = O
 
3 1 1 ⇒ λ AB − 2B −1B + IB = O [post-multiplying by
0 1 2 B]
Then, | A | = 1 2 3 = 0 ⋅ (2 − 3) − 1(1 − 9 ) + 2(1 − 6) ⇒ λ AB − 2I + B = O
3 1 1 ⇒ λ AB 2 − 2IB + B 2 = O
= −2≠0 [again post-multiplying by B]
−1
∴ A exists. ⇒ λ AB 2 − 2B + B 2 = O
Now, cofactors along R1 = − 1, 8, − 5 ⇒ λI − 2B + B 2 = O [Q AB 2 = I ]
cofactors along R 2 = 1, − 6, 3
1 0  2 −2  2 −2   2 −2  0 0
cofactors along R 3 = − 1, 2, − 1 ⇒ λ  − 2 + =
0 1 −1 0  −1 0  −1 0  0 0
Let C is a matrix of cofactors of the elements in | A |
λ 0  4 − 4  6 − 4  0 0
− 1 8 −5  ⇒ 0 − + =
 λ  − 2 0  −2 2  0 0
∴ C = 1 −6 3 
 
− 1 2 − 1  λ + 2 0  0 0
⇒  0 =
 λ + 2 0 0
−1 1 −1 
∴ adj A = C ′ =  8 −6 2 ⇒ λ +2=0
  ∴ λ = −2
− 5 3 − 1 
−1 1 −1 y Example 37. If A, B and C are three non-singular
1
2
−1 adj A
Hence, A = =− 8 −6 square matrices of order 3 satisfying the equation
| A| 2 
A 2 = A −1 and let B = A 8 and C = A 2 , find the value of
− 5 3 − 1 
 1 −1 1
det (B − C ).
 2 2 2 Sol. Q B = A 8 = ( A 2 ) 4 = ( A −1 ) 4 [Q A −1 = A 2 ]
= − 4 3 −1 
  = ( A 4 ) −1 = ( A 2 ⋅ 2 ) − 1
 5 −3 1
 2 2 2 = (( A 2 )2 )−1 = (( A 2 )−1 )2
= (( A −1 )−1 )2 = A 2 = C
y Example 35. If A and B are symmetric non-singular
matrices of same order, AB = BA and A −1B −1 exist, So, B =C ⇒B −C =0
∴ det ( B − C ) = 0
prove that A −1B −1 is symmetric.
Sol. Q A ′ = A, B ′ = B and | A | ≠ 0, | B | ′ ≠ 0
∴ (A B −1 −1
) ′ = ( B − 1 ) ′ ( A −1 ) ′ Elementary Row Operations
[ by reversal law of transpose]
(Transformations)
= ( B ′ )−1( A ′ )−1 [by property 3]
The following three types of operations (transformations)
= B −1A −1 [Q A ′ = A and B ′ = B ]
on the rows of a given matrix are known as elementary
= ( AB )−1 [ by reversal law of inverse] row operation (transformations).
−1
= ( BA ) [Q AB = BA ] (i) The interchange of ith and jth rows is denoted by
R i ↔ R j or R ij .
= A −1B −1 [by reversal law of inverse]
−1 −1
(ii) The multiplication of the ith row by a constant
Hence, A B is symmetric.
k (k ≠ 0 ) is denoted by R i → kR i or R i (k )
Chap 08 Matrices 631

(iii) The addition of the ith row to the elements of the jth 1 0 9 
row multiplied by constant k (k ≠ 0 ) is denoted by A ~ 0 1 −2 
 
R i → R i + kR j or R ij (k ). 0 0 1 

Note Applying R1 → R1 − 9 R 3 and R 2 → R 2 + 2R 3 , we get


Similarly, we can define the three column operations,

You might also like